You are on page 1of 756

EBD_7504

• Corporate Office : 45, 2nd Floor, Maharishi Dayanand Marg, Corner Market,
Malviya Nagar, New Delhi-110017
Tel. : 011-49842349 / 49842350

Typeset by Disha DTP Team

DISHA PUBLICATION
ALL RIGHTS RESERVED

© Copyright Publisher
No part of this publication may be reproduced in any form without prior permission of the publisher. The author and the
publisher do not take any legal responsibility for any errors or misrepresentations that might have crept in. We have tried
and made our best efforts to provide accurate up-to-date information in this book.

For further information about books from DISHA,


Log on to www.dishapublication.com or www.aiets.co.in or email to info@aiets.co.in
CONTENTS
PHYSICS – Chapter-wise Tests
1. Physical World, Units & Measurements P-1 – P-4
2. Motion in a Straight Line P-5 – P-8
3. Motion in a Plane P-9 – P-12
4. Laws of Motion P-13 – P-16
5. Work, Energy and Power P-17 – P-20
6. System of Particles and Rotational Motion P-21 – P-24
7. Gravitation P-25 – P-28
8. Mechanical Properties of Solids P-29 – P-32
9. Mechanical Properties of Fluids P-33 – P-36
10. Thermal Properties of Matter P-37 – P-40
11. Thermodynamics P-41 – P-44
12. Kinetic Theory P-45 – P-48
13. Oscillations P-49 – P-52
14. Waves P-53 – P-56
15. Electric Charges and Fields P-57 – P-60
16. Electrostatic Potential & Capacitance P-61 – P-64
17. Current Electricity P-65 – P-68
18. Moving Charges and Magnetism P-69 – P-72
19. Magnetism and Matter P-73 – P-76
20. Electromagnetic Induction P-77 – P-80
21. Alternating Current P-81 – P-84
22. Electromagnetic Waves P-85 – P-88
23. Ray Optics and Optical Instruments P-89 – P-92
24. Wave Optics P-93 – P-96
25. Dual Nature of Radiation and Matter P-97 – P-100
26. Atoms P-101 – P-104
27. Nuclei P-105 – P-108
28. Semiconductor Electronics: Materials, Devices and Simple Circuits P-109 – P-112
29. Communication System P-113 – P-116

CHEMISTRY – Chapter-wise Tests


30. Some Basic Concepts of Chemistry C-1 – C-4
31. Structure of Atom C-5 – C-8
32. Classification of Elements and Periodicity in Properties C-9 – C-12
33. Chemical Bonding and Molecular Structure C-13 – C-16
34. States of Matter C-17 – C-20
35. Thermodynamics C-21 – C-24
36. Equilibrium C-25 – C-28
EBD_7504
37. Redox Reaction C-29 – C-32
38. Hydrogen C-33 – C-36
39. The s – Block Elements C-37 – C-40
40. The p – Block Elements (Group 13 & 14) C-41 – C-44
41. Organic Chemistry : Some Basic Principles & Techniques C-45 – C-48
42. Hydrocarbons C-49 – C-52
43. Environmental Chemistry C-53 – C-56
44. The Solid State C-57 – C-60
45. Solutions C-61 – C-64
46. Electrochemistry C-65 – C-68
47. Chemical Kinetics C-69 – C-72
48. Surface Chemistry C-73 – C-76
49. General Principles and Processes of Isolation of Elements C-77 – C-80
50. The p – Block Elements (Group 15, 16, 17 and 18) C-81 – C-84
51. The d – and f – Block Elements C-85 – C-88
52. Coordination Compounds C-89 – C-92
53. Haloalkanes and Haloarenes C-93 – C-96
54. Alcohols, Phenols and Ethers C-97 – C-100
55. Aldehydes, Ketones and Carboxylic acids C-101 – C-104
56. Amines C-105 – C-108
57. Biomolecules C-109 – C-112
58. Polymers C-113 – C-116
59. Chemistry in Everyday Life C-117 – C-120

MATHEMATICS – Chapter-wise Tests


60. Sets M-1 – M-4
61. Relations and Functions M-5 – M-8
62. Trigonometric Functions M-9 – M-12
63. Principle of Mathematical Induction M-13 – M-16
64. Complex Numbers And Quadratic Equations M-17 – M-20
65. Linear Inequalities M-21 – M-24
66. Permutations and Combinations M-25 – M-28
67. Binomial Theorem M-29 – M-32
68. Sequences and Series M-33 – M-36
69. Straight Lines and Pair of Straight Lines M-37 – M-40
70. Conic Sections M-41 – M-44
71. Limits and Derivatives M-45 – M-48
72. Mathematical Reasoning M-49 – M-52
73. Statistics M-53 – M-56
74. Probability M-57 – M-60
75. Relations and Functions M-61 – M-64
76. Inverse Trigonometric Functions M-65 – M-68
77. Matrices M-69 – M-72
78. Determinants M-73 – M-76
79. Continuity and Differentiability M-77 – M-80

[ iv ]
80. Applications of Derivatives M-81 – M-84
81. Integrals M-85 – M-88
82. Applications of Integrals M-89 – M-92
83. Differential Equations M-93 – M-96
84. Vector Algebra M-97 – M-100
85. Three Dimensional Geometry M-101 – M-104
86. Probability M-105 – M-108
87. Properties of Triangles M-109 – M-112

PHYSICS – Subject-wise Tests


88. Full Syllabus Test – 1 PT-1 – PT-4
89. Full Syllabus Test – 2 PT-5 – PT-8
90. Full Syllabus Test – 3 PT-9 – PT-12
91. Full Syllabus Test – 4 PT-13 – PT-16

CHEMISTRY – Subject-wise Tests


92. Full Syllabus Test – 1 CT-1 – CT-4
93. Full Syllabus Test – 2 CT-5 – CT-8
94. Full Syllabus Test – 3 CT-9 – CT-12
95. Full Syllabus Test – 4 CT-13 – CT-16

MATHEMATICS – Subject-wise Tests


96. Full Syllabus Test – 1 MT-1 – MT-4
97. Full Syllabus Test – 2 MT-5 – MT-8
98. Full Syllabus Test – 3 MT-9 – MT-12
99. Full Syllabus Test – 4 MT-13 – MT-16

FULL TESTS
100. Full Test – 1 FT-1 – FT-12
101. Full Test – 2 FT-13 – FT-24

HINTS & SOLUTIONS


PHYSICS – Chapter-wise Tests S-P-1 – S-P-96
CHEMISTRY – Chapter-wise Tests S-C-1 – S-C-64
MATHEMATICS – Chapter-wise Tests S-M-1 – S-M-112
PHYSICS – Subject-wise Tests S-PT-1 – S-PT-11
CHEMISTRY – Subject-wise Tests S-CT-1 – S-CT-8
MATHEMATICS – Subject-wise Tests S-MT-1 – S-MT-17
FULL TESTS S-FT-1 – S-FT-20
EBD_7504
101 SP EED TESTS
Mark s Scored =
S uccess Gap =
Max. Cut-off Qualifying Correct Ans wers × 1 -
Speed Tes t Time Qualifying Marks -
Mark s Marks Mark s (0.25 × Incorrect
Mark s Scored
Ans wers )
1 60 120 45 60
2 60 120 45 60
3 60 120 45 60
4 60 120 40 50
5 60 120 45 60
6 60 120 40 50
7 60 120 45 60
8 60 120 45 60
9 60 120 40 50
10 60 120 45 60
11 60 120 40 50
12 60 120 45 60
13 60 120 40 50
14 60 120 45 60
15 60 120 45 60
16 60 120 45 60
17 60 120 40 50
18 60 120 40 50
19 60 120 45 60
20 60 120 45 60
21 60 120 45 60
22 60 120 45 60
23 60 120 40 50
24 60 120 40 50
25 60 120 40 50
26 60 120 45 60
27 60 120 45 60
28 60 120 45 60
29 60 120 45 60
30 60 120 37 52
31 60 120 37 51
32 60 120 38 55
33 60 120 35 50
34 60 120 38 53
35 60 120 36 52
101 SP EED TESTS
Mark s Scored =
S uccess Gap =
Max. Cut-off Qualifying Correct Ans wers × 1 -
Speed Tes t Time Qualifying Marks -
Mark s Marks Mark s (0 .25 × Incorrect
Mark s Scored
Ans wers )
36 60 120 35 50
37 60 120 40 60
38 60 120 40 60
39 60 120 37 56
40 60 120 38 52
41 60 120 36 53
42 60 120 35 50
43 60 120 40 60
44 60 120 35 50
45 60 120 35 50
46 60 120 37 53
47 60 120 38 56
48 60 120 40 60
49 60 120 38 56
50 60 120 35 51
51 60 120 38 52
52 60 120 36 54
53 60 120 37 54
54 60 120 36 52
55 60 120 35 51
56 60 120 36 52
57 60 120 37 56
58 60 120 38 57
59 60 120 37 55
60 60 120 40 55
61 60 120 38 50
62 60 120 40 58
63 60 120 40 55
64 60 120 37 55
65 60 120 42 60
66 60 120 36 48
67 60 120 38 55
68 60 120 38 55
69 60 120 38 55
70 60 120 37 50
EBD_7504
101 SP EED TESTS
Marks S cored =
Succes s Gap =
Max. Cut-off Qualifying Correct Ans wers × 1 -
Speed Tes t Time Qualifying Marks -
Mark s Mark s Mark s (0.25 × Incorrect
Mark s Scored
Ans wers )
71 60 120 38 50
72 60 120 40 58
73 60 120 38 50
74 60 120 40 55
75 60 120 35 50
76 60 120 38 55
77 60 120 40 55
78 60 120 36 53
79 60 120 41 60
80 60 120 38 53
81 60 120 36 53
82 60 120 35 50
83 60 120 38 55
84 60 120 35 48
85 60 120 35 52
86 60 120 37 54
87 60 120 40 58
88 60 120 45 60
89 60 120 45 60
90 60 120 45 60
91 60 120 45 60
92 60 120 45 60
93 60 120 45 60
94 60 120 45 60
95 60 120 45 60
96 60 120 45 60
97 60 120 45 60
98 60 120 45 60
99 60 120 45 60
100 60 120 125 165
101 60 120 125 165
PHYSICS Speed
Physical World, Units & Measurements TEST
No. of Questions
30
Maximum Marks
120
Time
1 Hour
1
Chapter-wise

GENERAL INSTRUCTIONS
• This test contains 30 MCQ's. For each question only one option is correct. Darken the correct circle/ bubble in the
Response Grid provided on each page.
• You have to evaluate your Response Grids yourself with the help of solutions provided at the end of this book.
• Each correct answer will get you 4 marks and 1 mark shall be deduced for each incorrect answer. No mark will be given/
deducted if no bubble is filled. Keep a timer in front of you and stop immediately at the end of 60 min.
• The sheet follows a particular syllabus. Do not attempt the sheet before you have completed your preparation for that
syllabus.
• After completing the sheet check your answers with the solution booklet and complete the Result Grid. Finally spend time
to analyse your performance and revise the areas which emerge out as weak in your evaluation.

1. A and B have different dimensions. Then which of the 3. Young’s modulus of a material has the same unit as
following relation will be meaningful ? (a) pressure
é Aù (b) strain
(a) êBú (b) [A – B] (c) compressibility
ë û
(d) force
(c) [A + B] (d) [eA/B]
2. N divisions on the main scale of a vernier calliper coincide 4. The time period of a body under S.H.M. is represented by:
with (N + 1) divisions of the vernier scale. If each division of T = Pa Db Sc where P is pressure, D is density and S is
main scale is ‘a’ units, then the least count of the instrument surface tension, then values of a, b and c are
is 3 1
(a) - , ,1 (b) -1, - 2, 3
a 2 2
(a) a (b)
N
1 3 1 1
N a (c) ,- ,- (d) 1, 2,
(c) ´a (d) 2 2 2 3
N +1 N +1

RESPONSE GRID 1. 2. 3. 4.

Space for Rough Work


EBD_7504
P-2 NTA JEE Main

æ a ö (a) angle (b) length


5. In the eqn. ç P + 2 ÷ (V - b) = constant, the unit of a is (c) mass (d) time
è V ø 12. The current voltage relation of a diode is given by I =
(a) dyne cm5 (b) dyne cm4 (e1000V/T – 1)mA, where the applied voltage V is in volts
(c) dyne/cm 3 (d) dyne cm2 and the temperature T is in degree kelvin. If a student makes
6. The mass and volume of a body are found to be 5.00 ± 0.05 an error measuring ±0.01 V while measuring the current of 5
kg and 1.00 ± 0.05 m3 respectively. Then the maximum mA at 300 K, what will be the error in the value of current in
possible percentage error in its density is mA?
(a) 6% (b) 3% (a) 0.2 mA (b) 0.02 mA
(c) 10% (d) 5% (c) 0.5 mA (d) 0.05 mA
7. The density of material in CGS system of units is 4g/cm3. In 13. The unit of impulse is the same as that of
a system of units in which unit of length is 10 cm and unit of (a) energy (b) power
mass is 100 g, the value of density of material will be (c) momentum (d) velocity
(a) 0.4 unit (b) 40 unit 14. A student measured the length of a rod as 3.50 cm. Which
(c) 400 unit (d) 0.04 unit instrument did he use to measure it?
8. Of the following quantities, which one has dimensions (a) A meter scale
different from the remaining three? (b) A vernier calliper where the 10 divisions in vernier scale
(a) Energy per unit volume matches with 9 divisions in main scale and main scale
(b) Force per unit area has 10 divisions in 1 cm
(c) Product of voltage and charge per unit volume (c) A screw gauge having 100 divisions in the circular scale
(d) Angular momentum and pitch as 1 mm
9. The percentage error in measuring M, L and T are 1%, 1.5% (d) A screw gauge having 50 divisions in the circular scale
and 3% respectively. Then the percentage error in measuring and pitch as 1 mm
the physical quantity with dimensions ML–1 T–1 is 15. Weber is the unit of
(a) 1% (b) 3.5% (a) magnetic susceptibility
(c) 3% (d) 5.5% (b) intensity of magnetisation
10. The unit of permittivity of free space, eo is (c) magnetic flux
(a) coulomb2/(newton-metre)2 (d) magnetic permeability
(b) coulomb/newton-metre 16. An object is moving through the liquid. The viscous
(c) newton-meter2/coulomb2 damping force acting on it is proportional to the velocity.
(d) coulomb2/newton-metre2 Then dimensions of constant of proportionality are
11. If E, m, J and G represent energy, mass, angular momentum (a) [ML–1T–1] (b) [MLT–1]
and gravitational constant respectively, then the
dimensional formula of EJ2/m5G2 is same as that of the (c) [M0LT–1] (d) [ML0T–1]

5. 6. 7. 8. 9.
RESPONSE 10. 11. 12. 13. 14.
GRID
15. 16.

Space for Rough Work


Physics P-3

17. Two full turns of the circular scale of a screw gauge cover a 23. In an experiment four quantities a, b, c and d are measured
distance of 1mm on its main scale. The total number of with percentage error 1%, 2%, 3% and 4% respectively.
divisions on the circular scale is 50. Further, it is found that
a 3b 2
the screw gauge has a zero error of –0.03 mm. While Quantity P is calculated as P = % error in P is
measuring the diameter of a thin wire, a student notes the cd
main scale reading of 3 mm and the number of circular scale (a) 10% (b) 7%
divisions in line with the main scale as 35. The diameter of (c) 4% (d) 14%
the wire is
24. The density of a solid ball is to be determined in an
(a) 3.32 mm (b) 3.73 mm experiment. The diameter of the ball is measured with a screw
(c) 3.67 mm (d) 3.38 mm gauge, whose pitch is 0.5 mm and there are 50 divisions on
18. The SI unit of electric flux is the circular scale. The reading on the main scale is
(a) Cm–2 (b) coulomb 2.5 mm and that on the circular scale is 20 divisions. If the
(c) ampere (d) volt metre measured mass of the ball has a relative error of 2%, the
19. If Q denote the charge on the plate of a capacitor of relative percentage error in the density is

Q2 (a) 0.9% (b) 2.4%


capacitance C then the dimensional formula for is (c) 3.1% (d) 4.2%
C
(a) [L2M2T] (b) [LMT2] DV
2 –2 25. A quantity X is given by e 0 L where Î0 is the
(c) [L MT ] (d) [L2M2T2] Dt
20. The respective number of significant figures for the numbers permittivity of the free space, L is a length, DV is a potential
23.023, 0.0003 and 2.1 × 10–3 are difference and Dt is a time interval. The dimensional formula
(a) 5, 1, 2 (b) 5, 1, 5 for X is the same as that of
(c) 5, 5, 2 (d) 4, 4, 2 (a) resistance (b) charge
21. The dimensions of mobility are (c) voltage (d) current
(a) M–2 T 2 A (b) M–1 T 2 A 26. ln a simple pendulum experiment, the maximum percentage
–2
(c) M T A 3 (d) M–1 T 3 A error in the measurement of length is 2% and that in the
22. The physical quantities not having same dimensions are observation of the time-period is 3%. Then the maximum
(a) torque and work percentage error in determination of the acceleration due to
(b) momentum and Planck’s constant gravity g is
(c) stress and Young’s modulus (a) 5% (b) 6%
(d) speed and (m0e0)–1/2 (c) 1% (d) 8%

RESPONSE 17. 18. 19. 20. 21.


GRID 22. 23. 24. 25. 26.

Space for Rough Work


EBD_7504
P-4 NTA JEE Main

29. In an experiment the angles are required to be measured


1 e2 using an instrument, 29 divisions of the main scale exactly
27. The dimensions of are
Îo hc coincide with the 30 divisions of the vernier scale. If the
smallest division of the main scale is half- a degree (= 0.5°),
(a) M–1 L–3 T4 A2 (b) ML3 T–4 A–2 then the least count of the instrument is :
(c) M0 L0 T0 A0 (d) M–1 L–3 T2 A (a) half minute (b) one degree
28. If the capacitance of a nanocapacitor is measured in terms (c) half degree (d) one minute
of a unit ‘u’ made by combining the electric charge ‘e’, 30. A physical quantity of the dimensions of length that can be
Bohr radius ‘a0’, Planck’s constant ‘h’ and speed of light ‘c’ e2
then formed out of c, G and is [c is velocity of light, G is
4pe0
universal constant of gravitation and e is charge]
e2 h u=
hc 1/ 2
(a) u= (b) 2 é e2 ù 1 é e2 ù
1/ 2
a0 e a0 c2 ê G ú ê ú
(a) (b)
ëê 4pe 0 ûú c 2 êë G4 pe0 úû
e2 c e2 a 0 1/ 2
(c) u= (d) u= 1 e2 1 é e2 ù
ha 0 hc (c) G (d) ê G ú
c 4pe0 c 2 êë 4pe 0 úû

RESPONSE GRID 27. 28. 29. 30.

PHYSICS CHAPTERWISE SPEED TEST-1


Total Questions 30 Total Marks 120
Attempted Correct
Incorrect Net Score
Cut-off Score 45 Qualifying Score 60
Success Gap = Net Score – Qualifying Score
Net Score = (Correct × 4) – (Incorrect × 1)
Space for Rough Work
PHYSICS Speed
Motion in a Straight Line TEST
No. of Questions
30
Maximum Marks
120
Time
1 Hour
2
Chapter-wise

GENERAL INSTRUCTIONS
• This test contains 30 MCQ's. For each question only one option is correct. Darken the correct circle/ bubble in the
Response Grid provided on each page.
• You have to evaluate your Response Grids yourself with the help of solutions provided at the end of this book.
• Each correct answer will get you 4 marks and 1 mark shall be deduced for each incorrect answer. No mark will be given/
deducted if no bubble is filled. Keep a timer in front of you and stop immediately at the end of 60 min.
• The sheet follows a particular syllabus. Do not attempt the sheet before you have completed your preparation for that
syllabus.
• After completing the sheet check your answers with the solution booklet and complete the Result Grid. Finally spend time
to analyse your performance and revise the areas which emerge out as weak in your evaluation.

1. The co-ordinates of a moving particle at any time ‘t’are given


by x = at3 and y = bt3. The speed of the particle at time ‘t’ æ u 2 + v2 ö
is given by (c) uv (d) ç ÷
è 2 ø
(a) 3t a 2 + b2 (b) 3t 2 a 2 + b2 3. A particle starts moving rectilinearly at time t = 0 such that
its velocity v changes with time t according to the equation
(c) t 2 a 2 + b2 (d) a 2 + b2 v = t2 – t where t is in seconds and v is in m/s. Find the time
2. A goods train accelerating uniformly on a straight railway interval for which the particle retards.
track, approaches an electric pole standing on the side of
1 1
track. Its engine passes the pole with velocity u and the (a) <t <1 (b) >t >1
guard’s room passes with velocity v. The middle wagon of 2 2
the train passes the pole with a velocity 1 1 3
(c) <t <1 (d) <t<
4 2 4
u+v 1 2
(a) (b) u + v2
2 2

RESPONSE GRID 1. 2. 3.

Space for Rough Work


EBD_7504
P-6 NTA JEE Main

4. A ball is dropped on a floor and bounces back to a height 7. A metro train starts from rest and in five seconds achieves a
somewhat less than the original height. Which of the curves speed 108 km/h. After that it moves with constant velocity
depicts its motion correctly? and comes to rest after travelling 45m with uniform
retardation. If total distance travelled is 395 m, find total
y y time of travelling.
(a) 12.2 s (b) 15.3 s
(a) (b) (c) 9 s (d) 17.2 s
8. The deceleration experienced by a moving motor boat after
t t its engine is cut off, is given by dv/dt = – kv3 where k is a
constant. If v0 is the magnitude of the velocity at cut-off,
y y the magnitude of the velocity at a time t after the cut-off is
(c) (d) v0
(a) (b) v0 e –kt
(2v 0 2 kt + 1)
t t
(c) v0 / 2 (d) v 0
5. A thief is running away on a straight road on a jeep moving
with a speed of 9 m/s. A police man chases him on a motor 9. A ball is dropped from the top of a tower of height 100 m and
cycle moving at a speed of 10 m/s. If the instantaneous at the same time another ball is projected vertically upwards
separation of jeep from the motor cycle is 100 m, how long from ground with a velocity 25 ms–1. Then the distance
will it take for the police man to catch the thief? from the top of the tower, at which the two balls meet is
(a) 1 second (b) 19 second (a) 68.4 m (b) 48.4 m
(c) 90 second (d) 100 second (c) 18.4 m (d) 78.4 m
6. The displacement x of a particle varies with time according 10. A person moves 30 m north and then 20 m towards east and
a
to the relation x = (1 - e - bt ). Then select the false finally 30 2 m in south-west direction. The displacement
b of the person from the origin will be
alternative.
(a) 10 m along north (b) 10 m along south
1
(a) At t = , the displacement of the particle is nearly (c) 10 m along west (d) zero
b
2æa ö 11. The velocity of a particle is v = v0 + gt + ft2. If its position is
ç ÷ x = 0 at t = 0, then its displacement after unit time (t = 1) is
3èbø
(a) v0 + g /2 + f (b) v0 + 2g + 3f
(b) The velocity and acceleration of the particle at t = 0 are (c) v0 + g /2 + f/3 (d) v0 + g + f
a and –ab respectively 12. A car, moving with a speed of 50 km/hr, can be stopped by
a brakes after at least 6 m. If the same car is moving at a speed
(c) The particle cannot go beyond x =
b of 100 km/hr, the minimum stopping distance is
(d) The particle will not come back to its starting point at (a) 12 m (b) 18 m
t®¥ (c) 24 m (d) 6 m

RESPONSE 4. 5. 6. 7. 8.
GRID 9. 10. 11. 12.

Space for Rough Work


Physics P-7

13. The water drops fall at regular intervals from a tap 5 m above 19. Two cars are moving in the same direction with the same
the ground. The third drop is leaving the tap at an instant speed 30 km/hr. They are separated by a distance of 5 km.
when the first drop touches the ground. How far above the The speed of a car moving in the opposite direction if it
ground is the second drop at that instant? (Take g = 10 m/s2) meets these two cars at an interval of 4 minutes, will be
(a) 1.25 m (b) 2.50 m (a) 40 km/hr (b) 45km/hr
(c) 3.75 m (d) 5.00 m (c) 30 km/hr (d) 15 km/hr
14. Two trains each of 50 m long are moving parallel towards 20. A particle moves along a straight line OX. At a time t (in
each other at speeds 10 m/s and 15 m/s respectively. After seconds) the distance x (in metres) of the particle from O is
given by x = 40 + 12t – t3. How long would the particle travel
what time will they pass each other?
before coming to rest?
2 (a) 40 m (b) 56 m
(a) 5 sec (b) 4 sec
3 (c) 16 m (d) 24 m
(c) 2 sec (d) 6 sec 21. A body moving with a uniform acceleration crosses a
15. The displacement ‘x’ (in meter) of a particle of mass ‘m’ (in distance of 65 m in the 5 th second and 105 m in 9th second.
kg) moving in one dimension under the action of a force, is How far will it go in 20 s?
related to time ‘t’ (in sec) by t = x + 3 . The displacement (a) 2040 m (b) 240 m
(c) 2400 m (d) 2004 m
of the particle when its velocity is zero, will be
22. A particle moving along x-axis has acceleration f, at time t,
(a) 2 m (b) 4 m
(c) zero (d) 6 m
given by f = f 0 æç 1 - ö÷ , where f0 and T are constants. The
t
16. A particle moves in a straight line with a constant acceleration. è Tø
It changes its velocity from 10 ms–1 to 20 ms–1 while passing particle at t = 0 has zero velocity. In the time interval between
through a distance 135 m in t second. The value of t is: t = 0 and the instant when f = 0, the particle’s velocity (vx) is
(a) 10 s (b) 1.8 s 1 2
(c) 12 s (d) 9 s (a) fT (b) f0T2
2 0
17. An automobile travelling with a speed of 60 km/h, can brake
to stop within a distance of 20m. If the car is going twice as 1
(c) fT (d) f0T
fast i.e., 120 km/h, the stopping distance will be 2 0
(a) 60 m (b) 40 m 23. A body is thrown vertically up with a velocity u. It passes
(c) 20 m (d) 80 m three points A, B and C in its upward journey with velocities
18. A stone is dropped into a well in which the level of water is u u u
, and respectively. The ratio of AB and BC is
h below the top of the well. If v is velocity of sound, the time 2 3 4
T after which the splash is heard is given by (a) 20 : 7 (b) 2
(c) 10 : 7 (d) 1
æ 2hö h
(a) T = 2h/v (b) T= ç ÷ + 24. A boat takes 2 hours to travel 8 km and back in still water
è g ø v lake. With water velocity of 4 km h–1, the time taken for
going upstream of 8 km and coming back is
æ 2h ö h æ h ö 2h
(c) T= ç ÷+ (d) T = çç ÷÷ + (a) 160 minutes (b) 80 minutes
è v ø g è 2g ø v (c) 100 minutes (d) 120 minutes

13. 14. 15. 16. 17.


RESPONSE 18. 19. 20. 21. 22.
GRID
23. 24.
Space for Rough Work
EBD_7504
P-8 NTA JEE Main

25. A particle starts its motion from rest under the action of a (a) t = 21 sec. (b) t = 2 5 sec
constant force. If the distance covered in first 10 seconds (c) t = 20 sec. (d) None of these
is S1 and that covered in the first 20 seconds is S2, then: 28. A stone falls freely under gravity. It covers distances h 1, h2
(a) S2 = 3S1 (b) S2 = 4S1 and h3 in the first 5 seconds, the next 5 seconds and the
(c) S2 = S1 (d) S2 = 2S1 next 5 seconds respectively. The relation between h1, h2
26. A body starts from rest and travels a distance x with uniform and h3 is
acceleration, then it travels a distance 2x with uniform speed, h2 h3
finally it travels a distance 3x with uniform retardation and (a) h1 = = (b) h2 = 3h 1 and h3 = 3h2
3 5
comes to rest. If the complete motion of the particle is along
(c) h1 = h2 = h3 (d) h1 = 2h 2 = 3h 3
a straight line, then the ratio of its average velocity to
maximum velocity is 29. A train of 150 metre length is going towards north direction
at a speed of 10 m/s . A parrot flies at the speed of 5 m/s
(a) 2/5 (b) 3/5
towards south direction parallel to the railway track. The
(c) 4/5 (d) 6/7 time taken by the parrot to cross the train is
27. Initially car A is 10.5 m ahead of car B. Both start moving at (a) 12 sec (b) 8 sec
time t = 0 in the same direction along a straight line. The (c) 15 sec (d) 10 sec
velocity time graph of two cars is shown in figure. The time 30. A car, starting from rest, accelerates at the rate f through a
when the car B will catch the car A, will be distance S, then continues at constant speed for time t and
V f
car B then decelerates at the rate to come to rest. If the total
2
distance traversed is 15 S , then
10m/s car A 1
(a) S = ft 2 (b) S = f t
6
1 2 1 2
45° (c) S = ft (d) S = ft
t 4 72

RESPONSE 25. 26. 27. 28. 29.


GRID 30.

PHYSICS CHAPTERWISE SPEED TEST-2


Total Questions 30 Total Marks 120
Attempted Correct
Incorrect Net Score
Cut-off Score 45 Qualifying Score 60
Success Gap = Net Score – Qualifying Score
Space for Rough Work
Net Score = (Correct × 4) – (Incorrect × 1)
Space for Rough Work
PHYSICS Speed
Motion in a Plane TEST
No. of Questions
30
Maximum Marks
120
Time
1 Hour
3
Chapter-wise

GENERAL INSTRUCTIONS
• This test contains 30 MCQ's. For each question only one option is correct. Darken the correct circle/ bubble in the
Response Grid provided on each page.
• You have to evaluate your Response Grids yourself with the help of solutions provided at the end of this book.
• Each correct answer will get you 4 marks and 1 mark shall be deduced for each incorrect answer. No mark will be given/
deducted if no bubble is filled. Keep a timer in front of you and stop immediately at the end of 60 min.
• The sheet follows a particular syllabus. Do not attempt the sheet before you have completed your preparation for that
syllabus.
• After completing the sheet check your answers with the solution booklet and complete the Result Grid. Finally spend time
to analyse your performance and revise the areas which emerge out as weak in your evaluation.

r r r r r r (a) 15 s (b) 10.98 s


1. If | A ´ B |= 3A.B then the value of | A ´ B | is
(c) 5.49 s (d) 2.745 s
(a) (A2 + B2 + 3AB)1/ 2 (b) (A2 + B2 + AB)1/ 2 r
4. For a particle in uniform circular motion, the acceleration a
1/ 2 at a point P(R,q) on the circle of radius R is (Here q is measured
æ 2 2 ABö from the x-axis)
(c) çè A + B + ÷ø (d) A + B
3
n2 n2
2. A projectile is given an initial velocity of (iˆ + 2 ˆj ) m/s, where (a) - cos q iˆ + sin q ˆj
R R
iˆ is along the ground and ĵ is along the vertical. If g = 10
m/s2 , the equation of its trajectory is : n2 n2
(b) - sin q iˆ + cos q ˆj
(a) y = x – 5x2 (b) y = 2x – 5x2 R R
(c) 4y = 2x – 5x 2 (d) 4y = 2x – 25x2
n2 n2
3. A projectile is thrown in the upward direction making an (c) - cos q iˆ - sin q ˆj
angle of 60° with the horizontal direction with a velocity of R R
147 m s–1. Then the time after which its inclination with the
n2 ˆ n2 ˆ
horizontal is 45°, is (d) i+ j
R R

RESPONSE GRID 1. 2. 3. 4.

Space for Rough Work


EBD_7504
P-10 NTA JEE Main
r r r ur r r
5. Six vectors, a, b, c, d , e and f have the magnitudes and r r wt wt
9. If vectors A = cos wtiˆ + sinwtjˆ and B = cos ˆi + sin ˆj
directions indicated in the figure. Which of the following 2 2
statements is true? are functions of time, then the value of t at which they are
r r r b orthogonal to each other is :
(a) b + c = f
r r r a c p p
(b) d + c = f (a) t = (b) t =
r r r 2w w
(c) d + e = f d e p
f
r r r (c) t = 0 (d) t =
(d) b + e = f 4w
6. Two particles start simultaneously from the same point and 10. A bus is moving on a straight road towards north with a
move along two straight lines, one with uniform velocity v uniform speed of 50 km/hour turns through 90°. If the speed
and other with a uniform acceleration a. If a is the angle remains unchanged after turning, the increase in the velocity
between the lines of motion of two particles then the least of bus in the turning process is
value of relative velocity will be at time given by (a) 70.7 km/hour along south-west direction
v v (b) 70.7 km/hour along north-west direction.
(a) sin a (b) cos a (c) 50 km/hour along west
a a
(d) zero
v v
(c) tan a (d) cot a 11. The velocity of projection of oblique projectile is
a a
7. Initial velocity with which a body is projected is 10 m/sec (6î + 8ˆj) m s -1 . The horizontal range of the projectile is
and angle of projection is 60°. Find the range R (a) 4.9 m (b) 9.6 m
y
15 3m (c) 19.6 m (d) 14 m
(a)
2 12. A point P moves in counter-clockwise direction on a circular
40 path as shown in the figure. The movement of 'P' is such
(b) m that it sweeps out a length s = t3 + 5, where s is in metres and
3
(c) 5 3m t is in seconds. The radius of the path is 20 m. The
R
30° acceleration of 'P' when t = 2 s is nearly
20 x
(d) m y
3
8. A particle moves in a circle of radius 4 cm clockwise at
constant speed 2 cm/s. If x̂ and ŷ are unit acceleration B
vectors along X and Y-axis respectively (in cm/s2), the P(x,y)
acceleration of the particle at the instant half way between P
and Q is given by m
y 20
(a) -4(xˆ + y) ˆ P
x
(b) 4(xˆ + y)ˆ O A
O x
(c) -(xˆ + y)
ˆ / 2 Q (a) 13 m/s2 (b) 12 m/s2
(d) (xˆ - y)
ˆ /4 (c) 7.2 ms2 (d) 14 m/s2

RESPONSE 5. 6. 7. 8. 9.
GRID 10. 11. 12.
Space for Rough Work
Physics P-11
® ® r
13. The resultant of two vectors A and B is perpendicular to (c) Velocity is perpendicular to r and acceleration is
®
the vector A and its magnitude is equal to half the directed towards the origin r
® ® ® (d) Velocity is perpendicular to r and acceleration is
magnitude of vector B . The angle between A and B is directed away from the origin
(a) 120° (b) 150° 18. A ship A is moving Westwards with a speed of 10 km h–1
(c) 135° (d) 180° and a ship B 100 km South of A, is moving Northwards with
14. A man running along a straight road with uniform velocity a speed of 10 km h–1. The time after which the distance
r between them becomes shortest, is :
u = u ˆi feels that the rain is falling vertically down along – ĵ .
(a) 5 h (b) 5 2h
If he doubles his speed, he finds that the rain is coming at
an angle q with the vertical. The velocity of the rain with (c) 10 2 h (d) 0 h
respect to the ground is 19. A projectile is fired at an angle of 45° with the horizontal.
Elevation angle of the projectile at its highest point as seen
u ˆ from the point of projection is
(a) ui – uj (b) ui - j
tan q
-1 æ 1 ö
(a) 60° (b) tan çè ÷ø
(c) 2uiˆ + u cot qˆj (d) ui + u sin qˆj 2
15. Two projectiles A and B thrown with speeds in the ratio æ 3 ö
(c) tan -1 ç (d) 45°
1 : 2 acquired the same heights. If A is thrown at an angle è 2 ÷ø
r
of 45° with the horizontal, the angle of projection of B will be 20. The position vector of a particle R as a function of time is
r
(a) 0° (b) 60° given by R = 4sin(2 pt)iˆ + 4cos(2 pt) ˆj
(c) 30° (d) 45°
16. A stone tied to the end of a string of 1 m long is whirled in a where R is in meter, t in seconds and î and ĵ denote unit
horizontal circle with a constant speed. If the stone makes vectors along x-and y-directions, respectively.
22 revolutions in 44 seconds, what is the magnitude and Which one of the following statements is wrong for the
direction of acceleration of the stone? motion of particle?
(a) p2 m s–2 and direction along the radius towards the
centre v2
(a) Magnitude of acceleration vector is , where v is
(b) p2 m s–2 and direction along the radius away from the R
centre the velocity of particle
(c) p2 m s–2 and direction along the tangent to the circle (b) Magnitude of the velocity of particle is 8 meter/second
(d) p2/4 m s–2 and direction along the radius towards the (c) Path of the particle is a circle of radius 4 meter.
centre r
17. A particle moves so that its position vector is given by (d) Acceleration vector is along - R
r uur uur uur uur uur uur
r = cos wtxˆ +sin wtyˆ . Where w is a constant. Which of the 21. The vectors A and B are such that | A + B |=| A - B |
following is true ?
r The angle between the two vectors is
(a) Velocity and acceleration both are perpendicular to r (a) 60° (b) 75°
r
(b) Velocity and acceleration both are parallel to r (c) 45° (d) 90°

RESPONSE 13. 14. 15. 16. 17.


GRID 18. 19. 20. 21.

Space for Rough Work


EBD_7504
P-12 NTA JEE Main

22. Passengers in the jet transport (a) 3 (b) 4


A flying east at a speed of 800 (c) 21 (d) 1
kmh –1 observe a second jet 27. A stone projected with a velocity u at an angle q with the
plane B that passes under the horizontal reaches maximum height H1. When it is projected
transport in horizontal flight.
æp ö
Although the nose of B is with velocity u at an angle çè - q÷ø with the horizontal, it
2
pointed in the 45° north east
reaches maximum height H2. The relation between the
direction, plane B appears to the passengers in A to be
horizontal range R of the projectile, heights H1 and H2 is
moving away from the transport at the 60° angle as shown.
The true velocity of B is (a) R = 4 H1H 2 (b) R = 4(H1 – H2)
(a) 586 kmh–1 (b) 600 kmh–1
(c) 717 kmh –1 (d) 400 kmh–1 H12
(c) R = 4 (H1 + H2) (d) R=
23. An artillary piece which consistently shoots its shells with H 22
the same muzzle speed has a maximum range R. To hit a 28. A water fountain on the ground sprinkles water all around
target which is R/2 from the gun and on the same level, the it. If the speed of water coming out of the fountain is v, the
elevation angle of the gun should be total area around the fountain that gets wet is :
(a) 15° (b) 45°
(c) 30° (d) 60° v4 p v4 v2 v2
(a) p 2 (b) (c) p (d) p
24. A car runs at a constant speed on a circular track of radius g 2 g2 g 2
g
100 m, taking 62.8 seconds in every circular loop. The average 29. The vector sum of two forces is perpendicular to their vector
velocity and average speed for each circular loop differences. In that case, the forces
respectively, is (a) cannot be predicted
(a) 0, 10 m/s (b) 10 m/s, 10 m/s (b) are equal to each other
(c) 10 m/s, 0 (d) 0, 0 (c) are equal to each other in magnitude
25. A vector of magnitude b is rotated through angle q. What is (d) are not equal to each other in magnitude
the change in magnitude of the vector? 30. A particle crossing the origin of co-ordinates at time t = 0,
q q moves in the xy-plane with a constant acceleration a in the
(a) 2b sin (b) 2b cos y-direction. If its equation of motion is y = bx2 (b is a
2 2
constant), its velocity component in the x-direction is
(c) 2b sin q (d) 2b cos q
26. A boat which has a speed of 5 km/hr in still water crosses a 2b a a b
(a) (b) (c) (d)
river of width 1 km along the shortest possible path in 15 a 2b b a
minutes. The velocity of the river water in km/hr is

RESPONSE 22. 23. 24. 25. 26.


GRID 27. 28. 29. 30.

PHYSICS CHAPTERWISE SPEED TEST-3


Total Questions 30 Total Marks 120
Attempted Correct
Incorrect Net Score
Space for Rough Work
Cut-off Score 45 Qualifying Score 60
Success Gap = Net Score – Qualifying Score
Net Score = (Correct × 4) – (Incorrect × 1)
Space for Rough Work
PHYSICS Speed
Laws of Motion TEST
No. of Questions
30
Maximum Marks
120 1 Hour
Time 4
Chapter-wise

GENERAL INSTRUCTIONS
• This test contains 30 MCQ's. For each question only one option is correct. Darken the correct circle/ bubble in the
Response Grid provided on each page.
• You have to evaluate your Response Grids yourself with the help of solutions provided at the end of this book.
• Each correct answer will get you 4 marks and 1 mark shall be deduced for each incorrect answer. No mark will be given/
deducted if no bubble is filled. Keep a timer in front of you and stop immediately at the end of 60 min.
• The sheet follows a particular syllabus. Do not attempt the sheet before you have completed your preparation for that
syllabus.
• After completing the sheet check your answers with the solution booklet and complete the Result Grid. Finally spend time
to analyse your performance and revise the areas which emerge out as weak in your evaluation.

1. A player stops a football weighing 0.5 kg which comes flying 3. A mass ‘m’ is supported by a massless string wound around
towards him with a velocity of 10m/s. If the impact lasts for a uniform hollow cylinder of mass m and radius R. If the
1/50th sec. and the ball bounces back with a velocity of 15 string does not slip on the cylinder, with what acceleration
m/s, then the average force involved is will the mass release?
(a) 250 N (b) 1250 N 2g
(c) 500 N (d) 625 N (a)
3
2. A 5000 kg rocket is set for vertical firing. The exhaust speed R
is 800 m/s. To give an initial upward acceleration of 20 m/s2, g
(b) m
the amount of gas ejected per second to supply the needed 2
thrust will be (Take g = 10 m/s2) 5g
(a) 127.5 kg/s (b) 137.5 kg/s (c)
6
m
(c) 155.5 kg/s (d) 187.5 kg/s (d) g

RESPONSE GRID 1. 2. 3.

Space for Rough Work


EBD_7504
P-14 NTA JEE Main

4. A 40 kg slab rests on a frictionless floor as shown in 9. A car having a mass of 1000 kg is moving at a speed of 30
the figure. A 10 kg block rests on the top of the slab. The metres/sec. Brakes are applied to bring the car to rest. If the
static coefficient of friction between the block and slab frictional force between the tyres and the road surface is
is 0.60 while the coefficient of kinetic friction is 0.40. The 5000 newtons, the car will come to rest in
10 kg block is acted upon by a horizontal force of 100 N. If (a) 5 seconds (b) 10 seconds
g = 9.8 m/s2, the resultaing acceleration of the slab will be (c) 12 seconds (d) 6 seconds
(a) 0.98 m/s2 100 N A
10 kg 10. Block A of mass m and block B of
(b) 1.47 m/s2 B
40 kg B mass 2m are placed on a fixed A
(c) 1.52 m/s2 m 2m
2 triangular wedge by means of a
(d) 6.1 m/s
massless, inextensible string and a 45° 45°
5. The pulleys and strings shown in the figure are smooth and
frictionless pulley as shown in figure.
of negligible mass. For the system to remain in equilibrium,
The wedge is inclined at 45° to the horizontal on both the
the angle q should be
sides. If the coefficient of friction between the block A and
(a) 0o the wedge is 2/3 and that between the block B and the wedge
(b) 30o q is 1/3 and both the blocks A and B are released from rest, the
acceleration of A will be
(c) 45o Ö2m (a) –1 ms–2 (b) 1.2 ms–2 (c) 0.2 ms–2 (d) zero
(d) 60o 11. The rate of mass of the gas emitted from rear of a rocket is
m m initially 0.1 kg/sec. If the speed of the gas relative to the
6. A satellite in a force free space sweeps stationary rocket is 50 m/sec and mass of the rocket is 2 kg, then the
interplanetary dust at a rate (dM/dt) = av. The acceleration acceleration of the rocket in m/sec2 is
of satellite is (a) 5 (b) 5.2 (c) 2.5 (d) 25
-2av 2 -a v 2 -a v 2 12. A block of mass m is resting on a smooth horizontal surface.
(a) (b) (c) (d) – av2
M M 2M One end of a uniform rope of mass (m/3) is fixed to the block,
7. A monkey is decending from the branch of a tree with which is pulled in the horizontal direction by applying a
constant acceleration. If the breaking strength is 75% of the force F at the other end. The tension in the middle of the
weight of the monkey, the minimum acceleration with which rope is
monkey can slide down without breaking the branch is 8 1 1 7
(a) F (b) F (c) F (d) F
3g g g 7 7 8 8
(a) g (b) (c) (d) 13. A body of mass M is kept on a rough horizontal surface
4 4 2
8. A plank with a box on it at one end (friction coefficient µ). A person is trying to pull the body
is gradually raised about the other by applying a horizontal force but the body is not moving.
end. As the angle of inclination with The force by the surface on the body is F, then
mg (a) F = Mg
the horizontal reaches 30º the box q
starts to slip and slides 4.0 m down (b) F = mMg
the plank in 4.0s. The coefficients of static and kinetic friction
between the box and the plank will be, respectively :
(c) Mg £ F £ Mg 1 + µ 2
(a) 0.6 and 0.5 (b) 0.5 and 0.6
(c) 0.4 and 0.3 (d) 0.6 and 0.6 (d) Mg ³ F ³ Mg 1 + µ 2

RESPONSE 4. 5. 6. 7. 8.
GRID 9. 10. 11. 12. 13.
Space for Rough Work
Physics P-15

(d) F a - x
14. Which one of the following motions on a smooth plane F x 2 2
surface does not involve force? (c)
2m a 2m x
(a) Accelerated motion in a straight line 19. Two blocks are connected over a
(b) Retarded motion in a straight line massless pulley as shown in fig.
(c) Motion with constant momentum along a straight line The mass of block A is 10 kg and A
(d) Motion along a straight line with varying velocity the coefficient of kinetic friction is
15. A block A of mass m1 rests on a horizontal table. A light 0.2. Block A slides down the incline 30º B
string connected to it passes over a frictionless pulley at at constant speed. The mass of
the edge of table and from its other end another block B of block B in kg is
mass m2 is suspended. The coefficient of kinetic friction (a) 3.5 (b) 3.3 (c) 3.0 (d) 2.5
between the block and the table is µk. When the block A is 20. A light spring balance hangs from the hook of the other
sliding on the table, the tension in the string is light spring balance and a block of mass M kg hangs from
(m 2 – m k m1 ) g m1m 2 (1 + m k ) g the former one. Then the true statement about the scale
(a) (b) reading is
(m1 + m 2 ) (m1 + m 2 )
(a) both the scales read M kg each
m1m2 (1 – mk )g (m 2 + m k m1 )g (b) the scale of the lower one reads M kg and of the upper
(c) (m1 + m2 ) (d) (m1 + m 2 ) one zero
16. The upper half of an inclined plane with inclination f is perfectly (c) the reading of the two scales can be anything but the
smooth while the lower half is rough. A body starting from rest sum of the reading will be M kg
at the top will again come to rest at the bottom if the coefficient (d) both the scales read M/2 kg each
of friction for the lower half is given by 21. Tension in the cable supporting an elevator, is equal to the
(a) 2 cos f (b) 2 sin f (c) tan f (d) 2 tan f weight of the elevator. From this, we can conclude that the
17. A particle describes a horizontal circle in a conical funnel elevator is going up or down with a
whose inner surface is smooth with speed of 0.5 m/s. What (a) uniform velocity (b) uniform acceleration
is the height of the plane of circle from vertex of the funnel? (c) variable acceleration (d) either (b) or (c)
(a) 0.25 cm (b) 2 cm (c) 4 cm (d) 2.5 cm 22. A particle tied to a string describes a vertical circular motion
18. Two particles of mass m each are of radius r continually. If it has a velocity 3 gr at the
tied at the ends of a light string of highest point, then the ratio of the respective tensions in
length 2a. The whole system is F the string holding it at the highest and lowest points is
kept on a frictionless horizontal (a) 4 : 3 (b) 5 : 4 (c) 1 : 4 (d) 3 : 2
surface with the string held tight
23. A block of mass m is connected to another block of mass M
so that each mass is at a distance
by a spring (massless) of spring constant k. The block are
'a' from the centre P (as shown in m P
m
kept on a smooth horizontal plane. Initially the blocks are at
the figure).
a a rest and the spring is unstretched. Then a constant force F
Now, the mid-point of the string is pulled vertically upwards starts acting on the block of mass M to pull it. Find the force
with a small but constant force F. As a result, the particles move of the block of mass m.
towards each other on the surface. The magnitude of MF mF
acceleration, when the separation between them becomes 2x, is (a) (b)
(m + M ) M
F a F x ( M + m ) F mF
(a) (b) (c) (d)
2m a2 - x2 2m a 2 - x 2 m ( m +M)

RESPONSE 14. 15. 16. 17. 18.


GRID 19. 20. 21. 22. 23.
Space for Rough Work
EBD_7504
P-16 NTA JEE Main

24. A block of mass m is placed on a surface with a vertical 28. A bullet is fired from a gun. The force on the bullet is given by
x 3 F = 600 – 2 × 105 t where, F is in newton and t in second. The
cross section given by y = . If the coefficient of friction force on the bullet becomes zero as soon as it leaves the
6
is 0.5, the maximum height above the ground at which the barrel. What is the average impulse imparted to the bullet?
block can be placed without slipping is: (a) 1.8 N-s (b) zero (c) 9 N-s (d) 0.9 N-s
1 2 1 1 29. A block of 7 kg is placed on a rough horizontal surface and
(a) m (b) m (c) m (d) m
6 3 3 2 is pulled through a variable force F (in N) = 5t, where t is time
25. A ball of mass 10 g moving perpendicular to the plane of the in second, at an angle of 37° with the horizontal as shown in
wall strikes it and rebounds in the same line with the same figure. The coefficient of static friction of the block with the
velocity. If the impulse experienced by the wall is 0.54 Ns, surface is one. If the force starts acting at t = 0s, the time at
the velocity of the ball is which the block starts to slide is t0 sec. Find the value of
(a) 27 ms–1 (b) 3.7 ms–1 (c) 54 ms–1 (d) 37 ms–1 4
t0/2 in sec. (g = 10 m/s2 and cos 37° = )
26. A block is kept on a inclined plane of inclination q of length l. 5
The velocity of particle at the bottom of inclined is (the F = 5t
coefficient of friction is m)
(a) [2gl(m cos q - sin q)]1 / 2 (b) 2gl(sin q - m cos q) 37°
7 kg
(c) 2gl(sin q + m cos q) (d) 2gl(cos q + m sin q)
/////////////////////////////////////////////////////////
27. Three forces start acting simultaneously C (a) 3 (b) 4 (c) 5 (d) 6
r
on a particle moving with velocity, v . 30. A stationary body of mass 3 kg explodes into three equal
These forces are represented in magnitude pieces. Two of the pieces fly off in two mutually
and direction by the three sides of a perpendicular directions, one with a velocity of 3iˆ ms - 1
triangle ABC. The particle will now move
with velocity A B and the other with a velocity of 4ˆj ms - 1. If the explosion
r occurs in 10–4 s, the average force acting on the third piece
(a) less than v
r in newton is
(b) greater than v
ˆ ´ 10 - 4
(a) (3iˆ + 4j) ˆ ´ 10 - 4
(b) (3iˆ - 4j)
(c) |v| in the direction of the largest force BC
(d) vr , remaining unchanged (c) (3iˆ - 4ˆj) ´ 10 4 (d) - (3iˆ + 4j)
ˆ ´ 104

RESPONSE 24. 25. 26. 27. 28.


GRID 29. 30.

PHYSICS CHAPTERWISE SPEED TEST-4


Total Questions 30 Total Marks 120
Attempted Correct
Incorrect Net Score
Cut-off Score 40 Qualifying Score
Space for Rough Work
50
Success Gap = Net Score – Qualifying Score
Net Score = (Correct × 4) – (Incorrect × 1)
Space for Rough Work
PHYSICS Speed
Work, Energy and Power TEST
No. of Questions
30
Maximum Marks
120
Time
1 Hour
5
Chapter-wise

GENERAL INSTRUCTIONS
• This test contains 30 MCQ's. For each question only one option is correct. Darken the correct circle/ bubble in the
Response Grid provided on each page.
• You have to evaluate your Response Grids yourself with the help of solutions provided at the end of this book.
• Each correct answer will get you 4 marks and 1 mark shall be deduced for each incorrect answer. No mark will be given/
deducted if no bubble is filled. Keep a timer in front of you and stop immediately at the end of 60 min.
• The sheet follows a particular syllabus. Do not attempt the sheet before you have completed your preparation for that
syllabus.
• After completing the sheet check your answers with the solution booklet and complete the Result Grid. Finally spend time
to analyse your performance and revise the areas which emerge out as weak in your evaluation.

1. A spring of spring constant 5 × 10 3 N/m is stretched initially 4. A ball is thrown vertically downwards from a height of 20 m
by 5cm from the unstretched position. Then the work with an initial velocity v0. It collides with the ground and
required to stretch it further by another 5 cm is loses 50% of its energy in collision and rebounds to the
(a) 12.50 Nm (b) 18.75 Nm same height. The initial velocity v0 is : (Take g = 10 ms–2)
(c) 25.00 Nm (d) 6.25 Nm (a) 20 ms–1 (b) 28 ms–1
2. A particle of mass 10 g moves along a circle of radius 6.4 cm (c) 10 ms –1 (d) 14 ms–1
with a constant tangential acceleration. What is the 5. When a rubber-band is stretched by a distance x, it exerts
magnitude of this acceleration if the kinetic energy of the restoring force of magnitude F = ax + bx 2 where a and b are
particle becomes equal to 8 × 10–4 J by the end of the second constants. The work done in stretching the unstretched
revolution after the beginning of the motion ? rubber-band by L is:
(a) 0.1 m/s2 (b) 0.15 m/s2

3.
(c) 0.18 m/s 2 (d) 0.2 m/s2
A body is moved along a straight line by a machine
(a) aL2 + bL3 (b)
1
2
(aL2 + bL3 )
delivering a constant power. The distance moved by the
body in time ‘t’ is proportional to aL2 bL3 1 æ aL2 bL3 ö
(c) + (d) ç + ÷
(a) t 3/4 (b) t 3/2 (c) t 1/4 (d) t 1/2 2 3 2 çè 2 3 ÷ø

RESPONSE GRID 1. 2. 3. 4. 5.

Space for Rough Work


EBD_7504
P-18 NTA JEE Main

6. A particle is acted by a force F = kx, where k is a +ve constant. (b) WP > WQ ; WQ > WP
Its potential energy at x = 0 is zero. Which curve correctly (c) WP < WQ ; WQ < WP
represents the variation of potential energy of the block
(d) WP = WQ ; WP > WQ
with respect to x
U 10. A body is allowed to fall freely under gravity from a height
U
of 10m. If it looses 25% of its energy due to impact with the
ground, then the maximum height it rises after one impact is
(a) x (b) x
(a) 2.5m (b) 5.0m (c) 7.5m (d) 8.2m
11. Water falls from a height of 60 m at the rate of 15 kg/s to
operate a turbine. The losses due to frictional force are 10%
U U of energy. How much power is generated by the turbine?( g
= 10 m/s2)
(c) x (d) x (a) 8.1 kW (b) 10.2 kW
(c) 12.3 kW (d) 7.0 kW
12. A glass marble dropped from a certain height above the
7. A particle of mass m is driven by a machine that delivers a horizontal surface reaches the surface in time t and then
constant power of k watts. If the particle starts from rest the continues to bounce up and down. The time in which the
force on the particle at time t is marble finally comes to rest is
(a) en t (b) e2 t
(a) mk t –1/2 (b) 2mk t –1/2
é1+ e ù é1 - e ù
(c) tê (d) tê ú
(c)
1
mk t –1/2 (d)
mk –1/2
t ë1 - e úû ë1 + e û
2 2
13. A block C of mass m is moving with velocity v0 and collides
8. A moving body with a mass m1 and velocity u strikes a elastically with block A of mass m and connected to another
stationary body of mass m2. The masses m1 and m2 should block B of mass 2m through spring constant k. What is k if
be in the ratio m1/m2 so as to decrease the velocity of the x0 is compression of spring when velocity of A and B is
first body to 2u/3 and giving a velocity of v to m 2 assuming same?
a perfectly elastic impact. Then the ratio m1/m2 is
C v0 A B `
(a) 5 (b) 1 / 5 (c) 1 / 25 (d) 25
9. Two similar springs P and Q have spring constants K P and
KQ, such that KP > KQ. They are stretched, first by the same mv02 mv02
(a) (b)
amount (case a,) then by the same force (case b). The work x 02 2x 02
done by the springs WP and WQ are related as, in case (a)
and case (b), respectively 3 mv02 2 mv02
(c) 2 x 02 (d) 3 x 02
(a) WP = WQ ; WP = WQ

RESPONSE 6. 7. 8. 9. 10.
GRID 11. 12. 13.
Space for Rough Work
Physics P-19

14. In the figure shown, a particle of mass m is released from the The total mechanical energy of the particle is 2 J. Then, the
position A on a smooth track. When the particle reaches at maximum speed (in m/s) is
B, then normal reaction on it by the track is
3
A (a) (b)
B 2
2
3h h 1
(c) (d) 2
2
19. A car of mass m starts from rest and accelerates so that the
2 m 2g instantaneous power delivered to the car has a constant
(a) mg (b) 2mg (c) mg (d)
3 h magnitude P0. The instantaneous velocity of this car is
15. A body of mass 1 kg begins to move under the action of a proportional to :
r (a) t 2P 0 (b) t 1/2
time dependent force F=(2tiˆ+3t 2 ˆj) N, where î and ĵ are
unit vectors alogn x and y axis. What power will be developed t
(c) t –1/2 (d)
by the force at the time t? m
(a) (2t2 + 3t3)W (b) (2t2 + 4t4)W 20. A block of mass m rests on a rough horizontal surface
3 4
(c) (2t + 3t ) W (d) (2t3 + 3t5)W (Coefficient of friction is µ). When a bullet of mass m/2
16. A bullet of mass 20 g and moving with 600 m/s collides with strikes horizontally, and get embedded in it, the block moves
a block of mass 4 kg hanging with the string. What is the a distance d before coming to rest. The initial velocity of the
velocity of bullet when it comes out of block, if block rises bullet is k 2mgd , then the value of k is
to height 0.2 m after collision? m/2
(a) 200 m/s (b) 150 m/s (c) 400 m/s (d) 300 m/s m
17. A body of mass m kg is ascending on a smooth inclined /////////////////////////////////////////////////////////////
d
æ 1ö (a) 2 (b) 3
plane of inclination q ç sin q = ÷ with constant acceleration
è xø (c) 4 (d) 5
of a m/s 2. The final velocity of the body is
21. A force acts on a 30 gm particle in such a way that the
v m/s. The work done by the body during this motion is
position of the particle as a function of time is given by x =
(Initial velocity of the body = 0)
3t – 4t2 + t3, where x is in metres and t is in seconds. The
1 mv2 æ g ö work done during the first 4 seconds is
(a) mv2 (g + xa) (b) ç + a÷
2 2 è2 ø (a) 576mJ (b) 450mJ
2mv 2 x mv 2 (c) 490mJ (d) 530mJ
(c) ( a + gx ) (d) ( g + xa ) 22. A steel ball of mass 5g is thrown downward with velocity 10
a 2ax
18. The potential energy of a 1 kg particle free to move along m/s from height 19.5 m. It penetrates sand by 50 cm. The
change in mechanical energy will be (g = 10 m/s2)
æ x4 x2 ö (a) 1J (b) 1.25 J
the x-axis is given by V( x) = ç - ÷ J.
ç 4 2 ÷ (c) 1.5 J (d) 1.75 J
è ø

RESPONSE 14. 15. 16. 17. 18.


GRID 19. 20. 21. 22.
Space for Rough Work
EBD_7504
P-20 NTA JEE Main

23. A 10 H.P. motor pumps out water from a well of depth 20 m 27. An engine pumps water through a hose pipe. Water passes
and fills a water tank of volume 22380 litres at a height of through the pipe and leaves it with a velocity of 2 m/s. The
10 m from the ground. The running time of the motor to fill mass per unit length of water in the pipe is 100 kg/m. What
the empty water tank is (g = 10ms–2) is the power of the engine?
(a) 5 minutes (b) 10 minutes (a) 400 W (b) 200 W
(c) 15 minutes (d) 20 minutes (c) 100 W (d) 800 W
24. A 3 kg ball strikes a heavyrigid wall with a speed 28. A body of mass 50kg is projected vertically upwards with
60º velocity of 100 m/sec. After 5 seconds this body breaks into
of 10 m/s at an angle of 60º. It gets reflected with
the same speed and angle as shown here. If the two pieces of 20 kg and 30 kg. If 20 kg piece travels upwards
60º with 150 m/sec, then the velocity of other block will be
ball is in contact with the wall for 0.20s, what is
the average force exerted on the ball bythe wall? (a) 15 m/sec downwards (b) 15 m/sec upwards
(a) 150N (b) 300N (c) 150 3N (d) zero (c) 51 m/sec downwards (d) 51 m/sec upwards
29. The K.E. acquired by a mass m in travelling a certain distance
25. A 2 kg block slides on a horizontal floor with a speed of 4m/s.
d, starting form rest, under the action of a constant force is
It strikes a uncompressed spring, and compresses it till the
directly proportional to
block is motionless. The kinetic friction force is 15N and spring
(a) m (b) m
constant is 10,000 N/m. The spring compresses by 1
(a) 8.5 cm (b) 5.5 cm (c) 2.5 cm (d) 11.0 cm (c) (d) independent of m
26. A stone is tied to a string of length l and is whirled in a m
30. A vertical spring with force constant k is fixed on a table. A
vertical circle with the other end of the string as the centre. ball of mass m at a height h above the free upper end of the
At a certain instant of time, the stone is at its lowest position spring falls vertically on the spring so that the spring is
and has a speed u. The magnitude of the change in velocity compressed by a distance d. The net work done in the
as it reaches a position where the string is horizontal process is
(g being acceleration due to gravity) is 1 1
(a) mg(h + d) - kd 2 (b) mg(h - d) - kd 2
(a) 2g l (b) 2 ( u 2 - g l) 2 2
1 2 1
u - u2 - 2g l (c) mg(h - d) + kd (d) mg(h + d) + kd2
(c) u2 - g l (d) 2 2

RESPONSE 23. 24. 25. 26. 27.


GRID 28. 29. 30.

PHYSICS CHAPTERWISE SPEED TEST-5


Total Questions 30 Total Marks 120
Attempted Correct
Incorrect Net Score
Cut-off Score 45 Qualifying
Space for Rough Work Score 60
Success Gap = Net Score – Qualifying Score
Net Score = (Correct × 4) – (Incorrect × 1)
Space for Rough Work
PHYSICS Speed
System of Particles and Rotational Motion TEST
No. of Questions
30
Maximum Marks
120
Time
1 Hour
6
Chapter-wise

GENERAL INSTRUCTIONS
• This test contains 30 MCQ's. For each question only one option is correct. Darken the correct circle/ bubble in the
Response Grid provided on each page.
• You have to evaluate your Response Grids yourself with the help of solutions provided at the end of this book.
• Each correct answer will get you 4 marks and 1 mark shall be deduced for each incorrect answer. No mark will be given/
deducted if no bubble is filled. Keep a timer in front of you and stop immediately at the end of 60 min.
• The sheet follows a particular syllabus. Do not attempt the sheet before you have completed your preparation for that
syllabus.
• After completing the sheet check your answers with the solution booklet and complete the Result Grid. Finally spend time
to analyse your performance and revise the areas which emerge out as weak in your evaluation.

1. A hollow sphere is held suspended. Sand 3. A solid sphere is rolling on a


is now poured into it in stages. surface as shown in figure,
The centre of mass of the sphere with with a translational velocity v
m s–1. If it is to climb the h
the sand
inclined surface continuing to
(a) rises continuously roll without slipping, then v
(b) remains unchanged in the process SAND minimum velocity for this to
(c) first rises and then falls to the happen is
original position
(d) first falls and then rises to the 7 7 10
(a) 2gh (b) gh (c) gh (d) gh
original position 5 2 7
2. From a solid sphere of mass M and radius R a cube of 4. A loop of radius r and mass m rotating with an angular velocity
maximum possible volume is cut. Moment of inertia of cube w0 is placed on a rough horizontal surface. The initial velocity
about an axis passing through its center and perpendicular of the centre of the hoop is zero.What will be the velocity of
to one of its faces is : the centre of the hoop when it ceases to slip?
4MR 2 4MR 2 MR 2 MR 2 rw0 rw0 rw0
(a) (b) (c) (d) (a) (b) (c) (d) rw0
9 3p 3 3p 32 2p 16 2p 4 3 2

RESPONSE GRID 1. 2. 3. 4.
Space for Rough Work
EBD_7504
P-22 NTA JEE Main

5. A thin rod of length ‘L’ is lying along the x-axis with its ends 9. A ball rolls without slipping. The radius of gyration of the
at x = 0 and x = L. Its linear density (mass/length) varies with ball about an axis passing through its centre of mass is K. If
n radius of the ball be R, then the fraction of total energy
æ xö
x as k ç ÷ , where n can be zero or any positive number. If associated with its rotational energy will be
è Lø
the position xCM of the centre of mass of the rod is plotted K2 K2 R2 K2 + R 2
against ‘n’, which of the following graphs best approximates (a) (b) 2 (c) 2 (d)
the dependence of xCM on n? R2 K2 + R K2 + R R2
xCM xCM 10. Two masses m1 and m2 are connected by a massless spring
L of spring constant k and unstretched length l. The masses
L L are placed on a frictionless straight channel, which are
(a) (b) consider our x-axis. They are initially at x = 0 and x = l
2 2
n n respectively. At t = 0, a velocity v0 is suddenly imparted to
O O
xCM xCM the first particle. At a later time t, the centre of mass of the
L L two masses is at :
L L m2 l
(c) 2
(d) 2 (a) x = m + m
n n 1 2
O O
6. A ring of mass M and radius R is rotating about its axis with m1l m2 v0t
angular velocity w. Two identical bodies each of mass m are (b) x = m + m + m + m
1 2 1 2
now gently attached at the two ends of a diameter of the
ring. Because of this, the kinetic energy loss will be : m2 l m2v0t m2 l m1v0 t
(c) x = + (d) x = m + m + m + m
m ( M + 2 m) 2 2 Mm m1 + m1 m1 + m2
(a) w R (b) w2 R2 1 2 1 2
M ( M + m) 11. From a circular ring of mass M and radius R, an arc
Mm ( M + m) M 2 2 corresponding to a 90° sector is removed. The moment of
(c) w2 R 2 (d) ( M + 2 m) w R
( M + 2m) inertia of the ramaining part of the ring about an axis passing
7. A wooden cube is placed on a rough horizontal table, a through the centre of the ring and perpendicular to the plane
force is applied to the cube. Gradually the force is increased. of the ring is k times MR2. Then the value of k is
Whether the cube slides before toppling or topples before (a) 3/4 (b) 7/8 (c) 1/4 (d) 1
sliding is independent of : 12. A mass m moves in a circle on a v0
(a) the position of point of application of the force smooth horizontal plane with
(b) the length of the edge of the cube
velocity v0 at a radius R0. The
(c) mass of the cube
(d) Coefficient of friction between the cube and the table mass is attached to string which m

8. Acertain bicycle can go up a in F 1


passes through a smooth hole in
Cha the plane as shown.
gentle incline with constant speed
when the frictional force of R2 Road The tension in the string is increased gradually and finally
ground pushing the rear wheel is R1 R
F2 = 4 N. With what force F1 must m moves in a circle of radius 0 . The final value of the
4N 2
the chain pull on the sprocket F2 = kinetic energy is
wheel if R1=5 cm and R2 = 30 cm? Horizontal
1 1
35 (a) mv02 (b) 2mv02 (c) mv02 (d) mv20
(a) 4 N (b) 24 N (c) 140 N (d) N 4 2
4
RESPONSE 5. 6. 7. 8. 9.
GRID 10. 11. 12.
Space for Rough Work
Physics P-23

13. A uniform rod of length l is free to rotate in a vertical plane 18. Fig. shows a disc rolling on a horizontal plane with linear
about a fixed horizontal axis through O. The rod begins velocity v. Its linear velocity is v and angular velocity is w.
rotating from rest from its unstable equilibrium position. Which of the following gives the velocity of the particle P
A
When it has turned through an angle q, its angular velocity on the rim of the disc ? P
w

(a) v (1 + cos q) r
w is given as O
(b) v (1 – cos q) q
v
6g 6g q q O M
sin q sin (c) v (1 + sin q)
(a) (b)
l l 2 (d) v (1 – sin q)
B N
6g q 6g 19. A solid sphere of mass M and
(c) cos (d) cos q radius R is pulled horizontally on
l 2 l
a sufficiently rough surface as
14. A rod PQ of length L revolves in a horizontal plane about the
axis YY´. The angular velocity of the rod is w. If A is the area shown in the figure.
of cross-section of the rod and r be its density, its rotational Choose the correct alternative.
kinetic energy is (a) The acceleration of the centre of mass is F/M
1 3 2 1 3 2 2 F
(a) AL rw (b) AL rw (b) The acceleration of the centre of mass is
3 2 3M
1 1 (c) The friction force on the sphere acts forward
(c) AL3rw 2 (d) AL3rw 2 (d) The magnitude of the friction force is F/3
24 18
15. A solid sphere of mass 2 kg rolls on a smooth horizontal 20. Point masses 1, 2, 3 and 4 kg are lying at the point (0, 0, 0),
surface at 10 m/s. It then rolls up a smooth inclined plane of (2, 0, 0), (0, 3, 0) and (–2, –2, 0) respectively. The moment of
inertia of this system about x-axis will be
inclination 30° with the horizontal. The height attained by
the sphere before it stops is (a) 43 kgm2 (b) 34 kgm2 (c) 27 kgm2 (d) 72 kgm2
(a) 700 cm (b) 701 cm (c) 7.1 m (d) 70 m 21. A circular disc of radius R rolls without slipping along the
horizontal surface with constant velocity v0. We consider a
16. A pulley is in the form of a disc of mass M and radius R. In
point A on the surface of the disc, then the acceleration of
following figure two masses M1 and M2 are connected by a the point A is
light inextensible string which passes over the pulley. (a) constant in magnitude as well as direction.
Assuming that the string does not slip /////////////////////////// (b) constant in direction
over the pulley, the angular momentum (c) constant in magnitude
of system at the instant shown, about (d) cannot say
O
axis of rotation of pulley is 22. A hollow smooth uniform sphere A of mass m rolls without
é 1 ù sliding on a smooth horizontal surface. It collides head on
êë M 2 + M1 + k M úû vR then elastically with another stationary smooth solid sphere B of
the same mass m and same radius. The ratio of kinetic energy
M1 M2 v
find the value of k. of B to that of A just after the collision is
(a) 1 (b) 2 (c) 4 (d) 5 A B
17. A couple produces (a) 1 : 1
(a) purely linear motion (b) 2 : 3 v0
(b) purely rotational motion
(c) linear and rotational motion (c) 3 : 2
(d) no motion (d) None of these

RESPONSE 13. 14. 15. 16. 17.


GRID 18. 19. 20. 21. 22.
Space for Rough Work
EBD_7504
P-24 NTA JEE Main

23. The moment of inertia of a body about a given axis is (c) angular velocity and moment of inertia about the axis
1.2 kg m2. Initially, the body is at rest. In order to produce a of rotation
rotational kinetic energy of 1500 joule, an angular (d) total angular momentum and moment of inertia about
acceleration of 25 radian/sec2 must be applied about that the axis of rotation
axis for a duration of 27. A tennis ball (treated as hollow spherical shell) starting from
(a) 4 seconds (b) 2 seconds O rolls down a hill. At point A the ball becomes air borne
(c) 8 seconds (d) 10 seconds leaving at an angle of 30° with the horizontal. The ball strikes
24. A pulley of radius 2 m is rotated about its axis by a force the ground at B. What is the value of the distance AB ?
F = (20t – 5t2) newton (where t is measured in seconds) applied (Moment of inertia of a O
tangentially. If the moment of inertia of the pulley about its spherical shell of mass m
axis of rotation is 10 kg-m2 the number of rotations made by and radius R about its
the pulley before its direction of motion is reversed, is:
2 2.0 m
(a) more than 3 but less than 6 diameter = mR2 )
(b) more than 6 but less than 9 3
(a) 1.87 m (b) 2.08 m 30°
(c) more than 9 (d) less than 3 A B
(c) 1.57 m (d) 1.77 m 0.2 m
25. A uniform square plate has a small piece Q of an irregular
shape removed and glued to the centre of the plate 28. Two identical discs of mass m and radius r are
leaving a hole behind. Then the moment of inertia about arranged as shown in the figure. If a is the //////////////////
y y
the z-axis angular acceleration of the lower disc and acm
is acceleration of centre of mass of the lower
(a) increases disc, then relation between a cm ,
Hole
Q a and r is
(b) decreases x x
(a) acm = a/r (b) acm = 2ar
(c) remains same
(c) acm = a r (d) None of these
(d) changed in unpredicted manner. 29. A gymnast takes turns with her arms and legs stretched.
26. An equilateral triangle ABC formed When she pulls her arms and legs in
A (a) the angular velocity decreases
from a uniform wire has two small
identical beads initially located at A. g (b) the moment of inertia decreases
The triangle is set rotating about the (c) the angular velocity stays constant
(d) the angular momentum increases
vertical axis AO. Then the beads are
B C 30. The moment of inertia of a uniform semicircular wire of mass
released from rest simultaneously and O m and radius r, about an axis passing through its centre of
allowed to slide down, one along AB and the other along
æ kö
AC as shown. Neglecting frictional effects, the quantities mass and perpendicular to its plane is mr 2 ç1 - 2 ÷ . Find
that are conserved as the beads slide down, are è p ø
(a) angular velocity and total energy (kinetic and potential) the value of k.
(b) total angular momentum and total energy (a) 2 (b) 3 (c) 4 (d) 5

RESPONSE 23. 24. 25. 26. 27.


GRID 28. 29. 30.

PHYSICS CHAPTERWISE SPEED TEST-6


Total Questions 30 Total Marks 120
Attempted Correct
Incorrect Net
Space Score
for Rough Work

Cut-off Score 40 Qualifying Score 50


Success Gap = Net Score – Qualifying Score
Net Score = (Correct × 4) – (Incorrect × 1)
Space for Rough Work
PHYSICS Speed
Gravitation TEST
No. of Questions
30
Maximum Marks
120
Time
1 Hour
7
Chapter-wise

GENERAL INSTRUCTIONS
• This test contains 30 MCQ's. For each question only one option is correct. Darken the correct circle/ bubble in the
Response Grid provided on each page.
• You have to evaluate your Response Grids yourself with the help of solutions provided at the end of this book.
• Each correct answer will get you 4 marks and 1 mark shall be deduced for each incorrect answer. No mark will be given/
deducted if no bubble is filled. Keep a timer in front of you and stop immediately at the end of 60 min.
• The sheet follows a particular syllabus. Do not attempt the sheet before you have completed your preparation for that
syllabus.
• After completing the sheet check your answers with the solution booklet and complete the Result Grid. Finally spend time
to analyse your performance and revise the areas which emerge out as weak in your evaluation.

1. A solid sphere of uniform m1 m2


density and radius R applies a v1 v2
A
gravitational force of attraction
equal to F1 on a particle placed d
at A, distance 2R from the centre What is their speed when their separation is ‘d’?
of the sphere. R R (Speed of m1 is v1 and that of m2 is v2)
A spherical cavity of radius R/2 is now made in the sphere (a) v1 = v2
as shown in the figure. The sphere with cavity now applies
a gravitational force F2 on the same particle placed at A. 2G 2G
(b) v1 = m 2 v 2 = m1
The ratio F2/F1 will be d ( m1 + m 2 ) , d ( m1 + m 2 )
(a) 1/2 (b) 3
2G 2G
(c) 7 (d) 1/9 (c) v1 = m1 v 2 = m2
d ( m1 + m 2 ) , d ( m1 + m 2 )
2. Two hypothetical planets of masses m1 and m2 are at rest
when they are infinite distance apart. Because of the
gravitational force they move towards each other along the 2G 2G
(d) v1 = m 2 , v2 = m 2
line joining their centres. m1 m2

RESPONSE GRID 1. 2.
Space for Rough Work
EBD_7504
P-26 NTA JEE Main

3. The gravitational force of attraction between a uniform 8. If the radius of the earth were to shrink by 1%, with its mass
sphere of mass M and a uniform rod of length l and mass m remaining the same, the acceleration due to gravity on the
oriented as shown is earth’s surface would
(a) decrease by 1% (b) decrease by 2%
M m (c) increase by 1% (d) increase by 2%
9. A uniform ring of mass m and radius r is placed r

r l directly above a uniform sphere of mass M


and of equal radius. The centre of the ring is 3r 2r
GMm GM
(a) (b) (c) Mmr2 + l (d) (r2 + l) mM directly above the centre of the sphere at a
r(r + l ) r2 distance r 3 as shown in the figure.
4. A satellite of mass m revolves around the earth of radius R The gravitational force exerted by the sphere
at a height ‘x’ from its surface. If g is the acceleration due to on the ring will be
gravity on the surface of the earth, the orbital speed of the
satellite is GMm GMm GMm GMm
(a) (b) (c) 3 (d)
8r 2
4r 2
8r 2 8r 3 3
gR 2 gR æ gR 2 ö 1/ 2
(a) (b) (c) gx (d) çç ÷
÷
10. The period of moon's rotation around the earth is nearly 29
R+x R-x è R + x ø days. If moon's mass were 2 fold its present value and all
5. A body is projected up with a velocity equal to 3/4th of the other things remain unchanged, the period of moon's rotation
escape velocity from the surface of the earth. The height it would be nearly
reaches from the centre of the earth is (Radius of the earth = R) (a) 29 2 days (b) 29 / 2 days
10R 16R 9R 10R
(a) (b) (c) (d) (c) 29 × 2 days (d) 29 days
9 7 8 3 11. The mean radius of earth is R, its angular speed on its own
6. A Planet is revolving around the sun. axis is w and the acceleration due to gravity at earth's surface
B is g. What will be the radius of the orbit of a geostationary
satellite ?
A C (a) (R2g / w2)1/3 (b) (Rg / w2)1/3
S
(c) (R2w2 / g)1/3 (d) (R2g / w)1/3
12. Which graph correctly presents the variation of acceleration
D
due to gravity with the distance from the centre of the earth
Which of the following is correct option? (radius of the earth =RE)?
(a) The time taken in travelling DAB is less than that for
BCD
(b) The time taken in travelling DAB is greater than that
g g
for BCD
(c) The time taken in travelling CDA is less than that for (a) (b)
ABC
(d) The time taken in travelling CDA is greater than that
for ABC RE r RE r
7. In a region of only gravitational field B 3
C
of mass 'M' a particle is shifted from A M
to B via three different paths in the 2 g g
figure. The work done in different paths 1 (c) (d)
are W1, W2, W3 respectively then
A
(a) W1 = W2 = W3 (b) W1 > W2 > W3
r r
(c) W1 = W2 > W3 (d) W1 < W2 < W3

RESPONSE 3. 4. 5. 6. 7.
GRID 8. 9. 10. 11. 12.
Space for Rough Work
Physics P-27

13. Consider Earth to be a homogeneous sphere. Scientist A 18. The change in the value of ‘g’ at a height ‘h’ above the
goes deep down in a mine and scientist B goes high up in a surface of the earth is the same as at a depth ‘d’ below the
balloon. The gravitational field measured by surface of earth. When both ‘d’ and ‘h’ are much smaller
(a) A goes on decreasing and that by B goes on increasing than the radius of earth, then which one of the following is
(b) B goes on decreasing and that by A goes on increasing correct ?
(c) each decreases at the same rate 3h h
(d) each decreases at different rates (a) d = (b) d = (c) d = h (d) d =2 h
2 2
14. A ball is dropped from a satellite revolving round the earth 19. Two identical geostationary satellites are moving with equal
at a height of 120 km. The ball will speeds in the same orbit but their sense of rotation brings
(a) continue to move with same speed along a straight them on a collision course. The debris will
line tangentially to the satellite at that time. (a) fall down
(b) continue to move with the same speed along the (b) move up
original orbit of satellite. (c) begin to move from east to west in the same orbit
(c) fall down to earth gradually (d) begin to move from west to east in the same orbit
(d) go far away in space 20. A tunnel is dug along a chord of the earth at a perpendicular
15. The gravitational field, due to the 'left over part' of a uniform distance R/2 from the earth’s centre. The wall of the tunnel
sphere (from which a part as shown, has been 'removed may be assumed to be frictionless. A particle is released
out'), at a very far off point, P, located as shown, would be from one end of the tunnel. The acceleration of the particle
(nearly) : varies with x (distance of the particle from the centre)
according to
Removed Mass of complete acceleration acceleration
Part sphere = M
P
R R
(a) (b)
x x x
x = R/2 x=R
x = R/3 x = R/2
5 GM 8 GM 7 GM 6 GM acceleration acceleration
(a) (b) (c) 2
(d)
6 x 2 9 x 2 8 x 7 x2
16. A body starts from rest from a point distance R0 from the
centre of the earth. The velocity acquired by the body when (c) (d)
it reaches the surface of the earth will be (R represents radius
of the earth). x = R/2 x=R
x
x = R/2 x=R
x

æ1 1 ö æ 1 1ö 21. A diametrical tunnel is dug across the Earth. A ball is dropped


(a) 2 G M çç - ÷ (b) 2 G M çç - ÷÷ into the tunnel from one side. The velocity of the ball when
R R ÷ R R
è 0ø è 0 ø
it reaches the centre of the Earth is .... (Given : gravitational
æ1 1 ö æ1 1 ö 3 GM
(c) G M çç - ÷ (d) 2 G M çç - ÷
÷ potential at the centre of Earth = – )
R R ÷ R R
è 0ø è 0ø 2 R
17. A satellite of mass M is moving in a circle of radius R under (a) R (b) gR (c) 2.5gR (d) 7.1gR
a centripetal force given by (–k/R2), where k is a constant.
Then 22. A satellite revolves around the earth of radius R in a circular
k orbit of radius 3R. The percentage increase in energy
(a) The kinetic energy of the particle is R required to lift it to an orbit of radius 5R is
12
(a) 10 % (b) 20 % (c) 30 % (d) 40 %
(b) The total energy of the particle is æç - k ö÷ 23. If the earth is treated as a sphere of radius R and mass M;
è 2R ø
æ kö its angular momentum about the axis of its rotation with
(c) The kinetic energy of the particle is ç - ÷ period T, is
è Rø
æ k ö pMR 3 MR 2 p 2 pMR 2 4 pMR 2
(d) The potential energy of the particle is ç ÷ (a) (b) (c) (d)
è 2R ø T T 5T 5T

13. 14. 15. 16. 17.


RESPONSE
18. 19. 20. 21. 22.
GRID 23.
Space for Rough Work
EBD_7504
P-28 NTA JEE Main

24. A (nonrotating) star 28. If suddenly the gravitational force of attraction between the
collapses onto itself from an earth and a satellite revolving around it becomes zero, then
initial radius Ri with its mass a the satellite will
remaining unchan ged. g (a) continue to move in its orbit with same speed
Which curve in figure best d (b) move tangentially to the original orbit with same speed
gives the gravitational b (c) become stationary in its orbit
acceleration a g on the a (d) move towards the earth
surface of the star as a 29. Suppose, the acceleration due to gravity at the Earth's
function of the radius of the surface is 10 m s–2 and at the surface of Mars it is
star during the collapse R
Ri 4.0 m s–2. A 60 kg pasenger goes from the Earth to the Mars
(a) a (b) b (c) c (d) d in a spaceship moving with a constant velocity. Neglect all
25. Which one of the following plots represents the variation other objects in the sky. Which part of figure best represents
of gravitational field on a particle with distance r due to a the weight (net gravitational force) of the passenger as a
thin spherical shell of radius R ? (r is measured from the function of time?
centre of the spherical shell)
Weight
F
F
600 N A

(a) (b) B
O r O r 200 N
R R
C
F F
D t0 time

(c) (d) (a) A (b) B (c) C (d) D


O
R
r O
R
r 30. Four equal masses (each of mass M) are placed at the corners
26. A uniform spherical shell gradually shrinks maintaining of a square of side a. The escape velocity of a body from the
its shape. The gravitational potential at the centre centre O of the square is
(a) increases (b) decreases
(c) remains constant (d) cannot say 2GM 8 2 GM
(a) 4 (b)
27. A satellite is launched into a circular orbit of radius R around a a
the earth. A second satellite is launched into an orbit of
radius 1.01 R. The period of second satellite is larger than 4GM 4 2 GM
the first one by approximately (c) (d)
(a) 0.5% (b) 1.0% (c) 1.5% (d) 3.0% a a

RESPONSE 24. 25. 26. 27. 28.


GRID 29. 30.

PHYSICS CHAPTERWISE SPEED TEST-7


Total Questions 30 Total Marks 120
Attempted Correct
Incorrect Net Score
Cut-off Score 45 Qualifying Score 60
Success Gap = Net Score – Qualifying Score
Net Score = (Correct × 4) – (Incorrect × 1)
Space for Rough Work
PHYSICS Speed
Mechanical Properties of Solids TEST
No. of Questions
30
Maximum Marks
120
Time
1 Hour
8
Chapter-wise

GENERAL INSTRUCTIONS
• This test contains 30 MCQ's. For each question only one option is correct. Darken the correct circle/ bubble in the
Response Grid provided on each page.
• You have to evaluate your Response Grids yourself with the help of solutions provided at the end of this book.
• Each correct answer will get you 4 marks and 1 mark shall be deduced for each incorrect answer. No mark will be given/
deducted if no bubble is filled. Keep a timer in front of you and stop immediately at the end of 60 min.
• The sheet follows a particular syllabus. Do not attempt the sheet before you have completed your preparation for that
syllabus.
• After completing the sheet check your answers with the solution booklet and complete the Result Grid. Finally spend time
to analyse your performance and revise the areas which emerge out as weak in your evaluation.

Load D
1. Two wires A and B are of the same material. Their lengths (a) OA C
are in the ratio 1 : 2 and the diameter are in the ratio 2 : 1. If B
they are pulled by the same force, then increase in length (b) OC
A
will be in the ratio
(c) OD
(a) 2 : 1 (b) 1 : 4 (c) 1 : 8 (d) 8 : 1
2. For a constant hydraulic stress on an object, the fractional (d) OB O Elongation

æ DV ö 4. A metal wire of length L1 and area of cross-section A is


change in the object volume ç ÷ and its bulk modulus attached to a rigid support. Another metal wire of length L2
è V ø and of the same cross-sectional area is attached to the free
(B) are related as end of the first wire. A body of mass M is then suspended
DV DV 1 from the free end of the second wire. If Y1 and Y2 are the
(a) µB (b) µ Young’s moduli of the wires respectively, the effective force
V V B
constant of the system of two wires is
DV D V
(c) µ B2 (d) µ B -2 (Y1Y2 ) A (Y1Y2 ) A
V V (a) (b)
2(Y1 L2 + Y2 L1 ) ( L1 L2 )1/2
3. The load versus elongation graphs for four wires of same
length and made of the same material are shown in the figure. (Y1Y2 ) A (Y1Y2 )1/2 A
The thinnest wire is represented by the line (c) (d)
Y1 L2 + Y2 L1 ( L2 L1 )1/2

RESPONSE GRID 1. 2. 3. 4.
Space for Rough Work
EBD_7504
P-30 NTA JEE Main

5. Choose the wrong statement.


(a) The bulk modulus for solids is much larger than for
liquids.
(b) Gases are least compressible. (c) (d)
(c) The incompressibility of the solids is due to the tight
coupling between neighbouring atoms.
(d) The reciprocal of the bulk modulus is called 10. In materials like aluminium and copper, the correct order of
compressibility. magnitude of various elastic modului is:
6. K is the force constant of a spring. The work done in (a) Young’s modulus < shear modulus < bulk modulus.
increasing its extension from l1 to l2 will be (b) Bulk modulus < shear modulus < Young’s modulus
K (c) Shear modulus < Young’s modulus < bulk modulus.
(a) K(l2 – l1) (b) (l2 + l1 )
2 (d) Bulk modulus < Young’s modulus < shear modulus.
K 2 2 11. An elastic string of unstretched length L and force constant
(c) K (l22 - l12 ) (d) (l2 - l1 )
2 k is stretched by a small length x. It is further stretched by
7. A steel wire of length l and cross sectional area A is stretched another small length y. The work done in the second
by 1 cm under a given load. When the same load is applied stretching is :
to another steel wire of double its length and half of its
1 2 1
cross section area, the amount of stretching (extension) is (a) ky (b) k ( x2 + y 2 )
2 2
(a) 0.5 cm (b) 2 cm (c) 4 cm (d) 1.5 cm
8. A cube at temperature 0ºC is compressed equally from all 1 1
(c) k ( x + y)2 (d) ky (2 x + y )
sides by an external pressure P. By what amount should its 2 2
temperature be raised to bring it back to the size it had before 12. Steel ruptures when a shear of 3.5 × 108 N m–2 is applied.
the external pressure was applied. The bulk modulus of the The force needed to punch a 1 cm diameter hole in a steel
material of the cube is B and the coefficient of linear sheet 0.3 cm thick is nearly:
expansion is a. (a) 1.4 × 104 N (b) 2.7 × 104 N
(a) P/B a (b) P/3 B a (c) 3 p a/B (d) 3 B/P (c) 3.3 × 104 N (d) 1.1 × 104 N
9. Stress Vs strain for the elastic tissue of the aorta, the large 13. Copper of fixed volume ‘V; is drawn into wire of length ‘l’.
tube (vessel) carrying blood from the heart, will be : [stress When this wire is subjected to a constant force ‘F’, the
is proportional to square of the strain for the elastic tissue extension produced in the wire is ‘Dl’. Which of the following
of the aorta] graphs is a straight line?
1
(a) Dl versus (b) Dl versus l2
l
(a) (b) 1
(c) Dl versus 2 (d) Dl versus l
l

RESPONSE 5. 6. 7. 8. 9.
GRID 10. 11. 12. 13.

Space for Rough Work


Physics P-31

14. If the ratio of lengths, radii and Young's moduli of steel and II.Rubber does not return to its original length after it is
brass wires in the figure are a, b and c respectively, then the stretched
corresponding ratio of increase in their lengths is : III. The rubber band will get heated if it is stretched and
3c released
(a) Steel
2ab2 Which of these can be deduced from the graph:
(a) III only (b) II and III (c) I and III (d) I only
2a 2 c M
(b) 18. When a 4 kg mass is hung vertically on a light spring that
b Brass obeys Hooke’s law, the spring stretches by 2 cms. The work
3a required to be done by an external agent in stretching this
(c) 2M spring by 5 cm will be (g = 9.8 m/sec2)
2b2c
(a) 4.900 joule (b) 2.450 joule
2ac (c) 0.495 joule (d) 0.245 joule
(d)
b2 19. A circular tube of mean radius 8 cm and thickness 0.04 cm is
15. The Poisson’s ratio of a material is 0.5. If a force is applied to a melted up and recast into a solid rod of the same length. The
wire of this material, there is a decrease in the cross-sectional ratio of the torsional rigidities of the circular tube and the
area by 4%. The percentage increase in the length is: solid rod is
(a) 1% (b) 2% (c) 2.5% (d) 4% (8.02) 4 - (7.98) 4 (8.02) 2 - ( 7.98) 2
16. The end of a uniform wire of length L and of weight W is (a) (b)
(0.8) 4 (0.8) 2
attached rigidly to a point in the roof and a weight W1 is
(0.8) 2 (0.8) 2
suspended from its lower end. If s is the area of cross-section (c) (d)
3L (8.02) 4 - (7.98) 4 (8.02) 3 - (7.98) 2
of the wire, the stress in the wire at a height from its 20. A mild steel wire of length 2L and cross-sectional area A is
4
lower end is : stretched, well within elastic limit, horizontally between two
pillars. A mass m is suspended from the mid point of the
W
W1 W1 + wire. Strain in the wire is
(a) (b) 4
s s 2L
3W x
W1 + W1 + W
(c) 4 (d)
s m
s
17. The diagram shows a force- x2 x x2 x2
extension graph for a rubber (a) (b) (c) (d)
2L L L 2L
Extension

band. Consider the following 21. Two wires A and B of same material and of equal length with
statements : the radii in the ratio 1 : 2 are subjected to identical loads. If
I. It will be easier to compress the length of A increases by 8 mm, then the increase in
this rubber than expand it length of B is
Force (a) 2 mm (b) 4 mm (c) 8 mm (d) 16 mm

RESPONSE 14. 15. 16. 17. 18.


GRID 19. 20. 21.
Space for Rough Work
EBD_7504
P-32 NTA JEE Main

22. Select the correct statement(s) from the following. 26. A beam of metal supported at the two edges is loaded at the
I. Modulus of rigidity for a liquid is zero centre. The depression at the centre is proportional to
II. Young's modulus of a material decreases with rise in
temperature d
III. Poisson's ratio is unitless
(a) I only (b) II only (c) I and II (d) I, II and III (a) Y 2 (b) Y (c) 1/Y (d) 1/Y 2
23. The upper end of a wire of diameter 12mm and length 1m is 27. The length of an elastic string is a metre when the
clamped and its other end is twisted through an angle of longitudinal tension is 4 N and b metre when the longitudinal
30°. The angle of shear is tension is 5 N. The length of the string in metre when the
(a) 18° (b) 0.18° (c) 36° (d) 0.36° longitudinal tension is 9 N is
24. A structural steel rod has a radius of 10 mm and length of 1
1.0 m. A 100 kN force stretches it along its length. Young’s (a) a – b (b) 5b – 4a (c) 2b – a (d) 4a – 3b
4
modulus of structural steel is 2 × 1011 Nm–2. The percentage 28. A thick rope of density r and length L is hung from a rigid
strain is about support. The Young’s modulus of the material of rope is Y.
(a) 0.16% (b) 0.32% (c) 0.08% (d) 0.24% The increase in length of the rope due to its own weight is
25. The graph given is a stress- (a) (1/4) r g L2/Y (b) (1/2) r g L2/Y
strain curve for (c) r g L /Y2 (d) r g L/Y
(a) elastic objects 1.0
29. A metal rod of Young's modulus 2 × 1010 N m–2 undergoes
Stress (N/ m2)

(b) plastics an elastic strain of 0.06%. The energy per unit volume stored
(c) elastomers 0.5 in J m–3 is
(d) None of these (a) 3600 (b) 7200 (c) 10800 (d) 14400
30. For the same cross-sectional area and for a given load, the
0 ratio of depressions for the beam of a square cross-section
0.5 1.0 and circular cross-section is
Strain (a) 3 : p (b) p : 3 (c) 1 : p (d) p : 1

RESPONSE 22. 23. 24. 25. 26.


GRID 27. 28. 29. 30.

PHYSICS CHAPTERWISE SPEED TEST-8


Total Questions 30 Total Marks 120
Attempted Correct
Incorrect Net Score
Cut-off Score 45 Qualifying Score 60
Success Gap = Net Score – Qualifying Score
Net Score = (Correct × 4) – (Incorrect × 1)
Space for Rough Work

Space for Rough Work


PHYSICS Speed
Mechanical Properties of Fluids TEST
No. of Questions
30
Maximum Marks
120 1 Hour
Time 9
Chapter-wise

GENERAL INSTRUCTIONS
• This test contains 30 MCQ's. For each question only one option is correct. Darken the correct circle/ bubble in the
Response Grid provided on each page.
• You have to evaluate your Response Grids yourself with the help of solutions provided at the end of this book.
• Each correct answer will get you 4 marks and 1 mark shall be deduced for each incorrect answer. No mark will be given/
deducted if no bubble is filled. Keep a timer in front of you and stop immediately at the end of 60 min.
• The sheet follows a particular syllabus. Do not attempt the sheet before you have completed your preparation for that
syllabus.
• After completing the sheet check your answers with the solution booklet and complete the Result Grid. Finally spend time
to analyse your performance and revise the areas which emerge out as weak in your evaluation.

1. The diagram below shows three soap bubbles A, B and C the tube). Mass of liquid in the capillary tube is m. If radius
prepared by blowing the capillary tube fitted with stop cocks of the capillary tube is increased by 50%, then mass of liquid
S, S1, S2 and S3. When all the three stop cocks S1, S2 and S3 that will rise in the tube, is
are opened and S closed, then
2 3 9
(a) m (b) m (c) m (d) m
C 3 2 4
S1 S S
S3 2 3. A lead shot of 1 mm diameter falls through a long column of
A B glycerine. The variation of its velocity v with distance
covered is represented by
(a) B will start collapsing with the increasing volume of A
and C v v
(b) C will start collapsing with the increasing volume of A (a) (b)
and B Distance Distance
covered covered
(c) volumes of A, B and C will become equal at equilibrium
(d) C and A will both start collapsing with the increasing
v v
volume of B
2. A capillary tube of radius r is immersed vertically in a liquid (c) (d)
such that liquid rises in it to height h (less than the length of Distance Distance
covered covered

RESPONSE GRID 1. 2. 3.

Space for Rough Work


EBD_7504
P-34 NTA JEE Main

4. Two mercury drops (each of radius ‘r’) merge to form bigger 9. A tank is filled with water upto a height H. Water is allowed
drop. The surface energy of the bigger drop, if T is the to come out of a hole P in one of the walls at a depth h below
surface tension, is : the surface of water (see fig.) Express the horizontal distance
X in terms of H and h.
(a) 4pr 2T (b) 2pr 2T
(a) X = h (H - h ) h
(c) 28 / 3 pr 2T (d) 25/ 3 pr 2T P
h
5. Wax is coated on the inner wall of a capillary tube and the (b) X= (H - h ) H
tube is then dipped in water. Then, compared to the unwaxed 2
capillary, the angle of contact q and the height h upto which (c) X = 2 h (H - h)
water rises change. These changes are : x
(a) q increases and h also increases (d) X = 4 h ( H - h )
(b) q decreases and h also decreases 10. Two capillary of length L and 2L and of radius R and 2R are
(c) q increases and h decreases connected in series. The net rate of flow of fluid through
(d) q decreases and h increases them will be (given rate to the flow through single capillary,
6. There is a circular tube in a vertical plane. Two liquids which
do not mix and of densities d1 and d2 are filled in the tube. pPR 4
X= )
Each liquid subtends 90º angle at centre. Radius joining 8hL
d1 8 9 5 7
their interface makes an angle a with vertical. Ratio (a) X (b) X (c) X (d) X
d2 9 8 7 5
11. A thin liquid film formed between a U-shaped wire and a
is: light slider supports a weight of 1.5 × 10–2 N (see figure).
1 + sin a The length of the slider is 30 cm and its weight negligible.
(a) The surface tension of the liquid film is
1 - sin a
1 + cos a a d2 (a) 0.0125 Nm–1 (b) 0.1 Nm–1 Film
(b)
1 - cos a
1 + tan a
(c) d1 (c) 0.05 Nm–1 (d) 0.025 Nm–1
1 - tan a W
1 + sin a 12. A liquid does not wet the sides of a solid, if the angle of
(d) contact is
1 - cos a
7. An isolated and charged spherical soap bubble has a radius (a) zero (b) obtuse (more than 90°)
r and the pressure inside is atmospheric. T is the surface (c) acute (less than 90°) (d) 90° (right angle)
13. If the terminal speed of a sphere of gold (density = 19.5 kg/m3)
tension of soap solution. If charge on drop is X pr 2rTe 0 is 0.2 m/s in a viscous liquid (density = 1.5 kg/m3), find the
then find the value of X. terminal speed of a sphere of silver (density = 10.5 kg/m3) of
(a) 8 (b) 9 (c) 7 (d) 2 the same size in the same liquid
8. A thread is tied slightly loose to a wire frame as in figure and (a) 0.4 m/s (b) 0.133 m/s
the frame is dipped into a soap solution and taken out. The (c) 0.1 m/s (d) 0.2 m/s
frame is completely covered with the film. When the portion 14. Select the correct statements from the following.
A is punctured with a pin, the thread (a) Bunsen burner and sprayers work on Bernoulli's
(a) becomes concave towards A Frame principle
A
(b) becomes convex towards A (b) Blood flow in arteries is explained by Bernoulli's
(c) remains in the initial position B principle
Thread
(d) either (a) or (b) depending (c) A siphon works on account of atmospheric pressure.
on the size of A w.r. t. B (d) All are correct

4. 5. 6. 7. 8.
RESPONSE
9. 10. 11. 12. 13.
GRID
14.
Space for Rough Work
Physics P-35

15. A ball of radius r and density r falls freely under r2 g 2r 2 g


(c) (r - s) (d) (r - s)
gravity through a distance h before entering 9h 9h
water. Velocity of ball does not change even on h 20. A square hole of side length l is made at a
entering water. If viscosity of water is h the value depth of h and a circular hole of radius r is made h
4h
of h is given by at a depth of 4h from the surface of water in a A v 1

2 2 æ 1- r ö 2 2 æ r -1 ö water tank kept on a horizontal surface. If l <<


(a) r ç ÷g (b) r ç
81 è h ø
÷g
h, r << h and the rate of water flow from the B v
9 è h ø
2

2 2 holes is the same, then r is equal to


2 4 æ r -1 ö 2 4 æ r -1 ö
(c) r ç ÷ g (d) r ç ÷ g l l l l
81 è h ø 9 è h ø
(a) (b) (c) (d)
16. The relative velocity of two parallel layers of water is 2p 3p 3p 2p
8 cm/sec. If the perpendicular distance between the layers 21. A spherical solid ball of volume V is made of a material of
is 0.1 cm, then velocity gradient will be density r1. It is falling through a liquid of density
(a) 80/sec (b) 60 /sec r1 (r2< r1). Assume that the liquid applies a viscous force on
(c) 50/sec (d) 40/sec the ball that is proportional to the square of its speed
17. A vessel with water is placed on a
v, i.e., Fviscous = –kv2 (k > 0). The terminal speed of the ball
weighing pan and it reads 600 g. Now
a ball of mass 40 g and density 0.80 g is
cm–3 is sunk into the water with a pin Vg (r1 – r2 ) Vgr1
of negligible volume, as shown in (a) (b)
Weighing pan k k
figure keeping it sunk. The weighting
pan will show a reading Vg r1 Vg (r1 – r2 )
(a) 600 g (b) 550 g (c) 650 g (d) 632 g (c) k (d)
k
18. A glass tube of uniform internal radius 22. Two liquids of densities d1 and d2 are flowing in identical
(r) has a valve separating the two capillary tubes uder the same pressure difference. If t1 and
identical ends. Initially, the valve is in 2 1 t2 are time taken for the flow of equal quantities (mass) of
a tightly closed position. End 1 has a liquids, then the ratio of coefficient of viscosity of liquids
hemispherical soap bubble of radius r. must be
End 2 has sub-hemispherical soap bubble (figure). Just after
opening the valve d1 t 1 t d2 t2 d1t 1
(a) (b) 1 (c) (d)
(a) air from end 1 flows towards end 2. No change in the d2t2 t2 d1 t 1 d 2t 2
volume of the soap bubbles 23. Let T1 be surface tension between T
(b) air from end 1 flows towards end 2. Volume of the soap solid and air, T2 be the surface liquid air
bubble at end 1 decreases tension between solid and liquid and
q
(c) no change occurs T be the surface tension between T1 Solid T2
(d) air from end 2 flows towards end 1. Volume of the soap liquid and air. Then in equilibrium,
bubble at end 1 increases for a drop of liquid on a clean glass
19. What is the velocity v of a metallic ball of radius r falling in plate, the correct relation is (q is
a tank of liquid at the instant when its acceleration is one - angle of contact)
half that of a freely falling body ? (The densities of metal T T
and of liquid are r and s respectively, and the viscosity of (a) cos q = (b) cos q =
T1 + T2 T1 - T2
the liquid is h). T1 + T2 T1 - T2
r2g r2 g
(c) cos q = (d) cos q =
(a) (r - 2s) (b) (2r - s ) T T
9h 9h

RESPONSE 15. 16. 17. 18. 19.


GRID 20. 21. 22. 23.

Space for Rough Work


EBD_7504
P-36 NTA JEE Main

24. Figure shows a weigh-bridge, with a beaker P with water on 27. On heating water, bubbles being formed
one pan and a balancing weight R on the other. A solid ball at the bottom of the vessel detach and
Q is hanging with a thread outside water. It has volume 40 rise. Take the bubbles to be spheres of
cm3 and weighs 80 g. If this solid is lowered to sink fully in radius R and making a circular contact
water, but not touching the beaker anywhere, the balancing of radius r with the bottom of the vessel.
weight R' will be If r << R and the surface tension of water
R
is T, value of r just before bubbles
Q
detach is: (density of water is rw)
2r

2 2rw g 2 rw g
P
R (a) R (b) R
3T 6T
rw g 2 3r w g
(c) R2 (d) R
(a) same as R (b) 40 g less than R T T
28. Consider an iceberg floating in sea water. The density of
(c) 40 g more than R (d) 80 g more than R
25. A tank has a small hole at its botom of area of cross-section sea water is 1.03 g/cc and that of ice is 0.92 g/cc. The fraction
a. Liquid is being poured in the tank at the rate Vm3/s, of total volume of iceberg above the level of sea water is
the maximum level of liquid in the container will be (Area of near by
tank = A) (a) 1.8% (b) 3% (c) 8% (d) 11%
29. The lift of an air plane is based on
V V2 V2 V (a) Torricelli's theorem
(a) (b) (c) (d)
gaA 2gAa gAa 2gaA (b) Bernoulli's theorem
26. A jar is filled with two non-mixing liquids 1 and r1
(c) Law of gravitation
2 having densities r1 and, r2 respectively. A (d) conservation of linear momentum
r3
solid ball, made of a material of density r3 , is 30. The cylindrical tube of a spray pump has radius, R, one end
dropped in the jar. It comes to equilibrium in of which has n fine holes, each of radius r. If the speed of the
liquid in the tube is V, the speed of the ejection of the liquid
the position shown in the figure.Which of the
through the holes is :
following is true for r1, r2and r3?
(a) r3 < r1 < r2 (b) r1 > r3 > r2 VR 2 VR 2 V 2R VR 2
(a) (b) (c) (d)
(c) r1 < r2 < r3 (d) r1 < r3 < r2 nr 2 n3r 2 nr n2r2

24. 25. 26. 27. 28.


RESPONSE GRID
29. 30.

PHYSICS CHAPTERWISE SPEED TEST-9


Total Questions 30 Total Marks 120
Attempted Correct
Incorrect Net Score
Cut-off Score 40 Qualifying Score 50
Success Gap = Net Score – Qualifying Score
Net Score = (Correct × 4) – (Incorrect × 1)
Space for Rough Work
PHYSICS Speed
Thermal Properties of Matter TEST
No. of Questions
30
Maximum Marks
120
Time
1 Hour
10
Chapter-wise

GENERAL INSTRUCTIONS
• This test contains 30 MCQ's. For each question only one option is correct. Darken the correct circle/ bubble in the
Response Grid provided on each page.
• You have to evaluate your Response Grids yourself with the help of solutions provided at the end of this book.
• Each correct answer will get you 4 marks and 1 mark shall be deduced for each incorrect answer. No mark will be given/
deducted if no bubble is filled. Keep a timer in front of you and stop immediately at the end of 60 min.
• The sheet follows a particular syllabus. Do not attempt the sheet before you have completed your preparation for that
syllabus.
• After completing the sheet check your answers with the solution booklet and complete the Result Grid. Finally spend time
to analyse your performance and revise the areas which emerge out as weak in your evaluation.

1. A cylindrical metal rod is shaped into a ring with a small gap 3. The sprinkling of water slightly reduces the temperature
as shown. On heating the system : of a closed room because
(a) x decreases, r and d increase (a) temperature of water is less than that of the room
(b) x and r increase, d decreases (b) specific heat of water is high
(c) x, r and d all increase (c) water has large latent heat of vaporisation
(d) x and r decrease, d remains constant (d) water is a bad conductor of heat
2. The total radiant energy per unit area, normal to the direction 4. Which of the given graphs proves Newton’s law of cooling?
of incidence, received at a distance R from the centre of a
star of radius r, whose outer surface radiates as a black
log (T – T0 )
log(T – T0)

body at a temperature T K is given by: (s is Stefan's


(a) (b)
constant)
s r 2T 4 s r 2T 4 Time Time

(a) (b)
R2 4p r 2
log (T – T0)

s r 4T 4 4 p s r 2T 4 (c) (d) None of these


(c) 4 (d) 2
r R Time

RESPONSE GRID 1. 2. 3. 4.
Space for Rough Work
EBD_7504
P-38 NTA JEE Main

5. If lm denotes the wavelength at which the radiative (a) the temperature difference across A is 15°C
emission from a black body at a temperature T K is maximum. (b) the temperature difference across A is 5°C
Then (c) the temperature difference across A is 10°C
(a) lm µ T–1 (d) the rate of transfer of heat through A is more than that
through B
(b) lm µ T4 10. The filament of an evacuated light bulb has a length 10 cm,
(c) lm is independent of T diameter 0.2 mm and emissivity 0.2. Calculate the power it
(d) lm µ T radiates at 2000 K. (s = 5.67 × 10–8 W/m2 K4)
6. Three rods of same dimensions are arranged as shown in (a) 21.5 W (b) 15.5 W (c) 8.9 W (d) 11.4 W
figure, have thermal conductivities K1, K2 and K3 . The 11. A rectangular block is heated from 0ºC to 100ºC. The
points P and Q are maintained at different temeratures for percentage increase in its length is 0.10%. What will be the
the heat to flow at the same rate along PRQ and PQ. Then percentage increase in its volume?
which of the following option is correct? (a) 0.03% (b) 0.10% (c) 0.30% (d) 0.5%
R
12. The figure shows a system of two
1 concentric spheres of radii r1 and r2 are
(a) K3 = ( K1 + K 2 )
2 kept at temperatures T 1 and T 2 , r1
(b) K3 = K1 + K2 respectively. The radial rate of flow of T1
K 1 K2 heat in a substance between the two
K1K 2 concentric spheres is proportional to r2 T2
(c) K 3 =
K1 + K2 (r2 - r1 )
æ ö
(d) K3 = –2(K1 + K2) P Q (a) ln ç r2 ÷ (b)
K è 1ø
r (r1 r2 )
7. A metallic rod l cm long, A square cm in3 cross-section is
heated through tºC. If Young’s modulus of elasticity of the r1 r2
(c) ( r2 - r1 ) (d)
metal is E and the mean coefficient of linear expansion is a per (r2 - r1 )
degree celsius, then the compressional force required to
prevent the rod from expanding along its length is 13. A block of steel heated to 100ºC is left

Temperature
C
in a room to cool. Which of the curves
(a) E A a t (b) E A a t/(1 + at) B
shown in fig., represents the correct
(c) E A a t/(1 – a t) (d) E l a t behaviour?
A

8. Which of the following statements is/are false about


mode of heat transfer? (a) A (b) B Time

(a) In radiation, heat is transfered from one medium to (c) C (d) None of these
another without affecting the intervening medium 14. Which of the following will expand the most for same rise
(b) Radiation and convection are possible in vaccum in temperature?
while conduction requires material medium. (a) Aluminium (b) Glass
(c) Conduction is possible in solids while convection (c) Wood (d) All will expand same
15. The plots of intensity versus T3
occurs in liquids and gases.
wavelength for three black bodies T2
(d) All are correct at temperatures T 1 , T 2 and T 3 I T
1
9. A wall has two layers A and B made of different materials. respectively are as shown. Their
The thickness of both layers is the same. The thermal temperature are such that
conductivity of A and B are KA and KB such that KA = 3KB. l
(a) T1 > T2 > T3 (b) T1 > T3 > T2
The temperature across the wall is 20°C. In thermal
equilibrium (c) T2 > T3 > T1 (d) T3 > T2 > T1

RESPONSE 5. 6. 7. 8. 9.
GRID 10. 11. 12. 13. 14.
15.
Space for Rough Work
Physics P-39

16. When the temperature of a rod increases from t to t + Dt, its 20. Two rods of same length and
moment of inertia increases from I to I + DI. If a be the transfer a given amount of heat
coefficient of linear expansion of the rod, then the value of 12 second, when they are
DI joined as shown in figure (i).
is
I But when they are joined as
aDt Dt shwon in figure (ii), then they
(a) 2aDt (b) aDT (c) (d)
2 a will transfer same heat in same
17. Two rods, one of aluminum and the other made of steel, conditions in
having initial length l1 and l2 are connected together to (a) 24 s (b) 13 s (c) 15 s (d) 48 s
form a single rod of length l1 + l2. The coefficients of linear 21. A long metallic bar is carrying heat from one of its ends to
expansion for aluminum and steel are aa and as and
respectively. If the length of each rod increases by the same the other end under steady–state. The variation of
amount when their temperature are raised by t0C, then find temperature q along the length x of the bar from its hot end
the ratio l1/(l1 + l2) is best described by which of the following figures?
(a) as/aa (b) aa/as q q
(c) a s/(aa + as) (d) aa/(aa + as)
18. In a vertical U-tube containing a liquid, (a) (b)
the two arms are maintained at t1 x x
different temperatures t1 and t2. The t2
liquid columns in the two arms have l1 q q
heights l 1 and l 2 respectively. The l
2
coefficient of volume expansion of the
liquid is equal to (c) (d)
l1 – l2 l1 – l2
(a) (b) x x
l2 t1 – l1t2 l1t1 – l2 t2
22. One end of a thermally insulated rod is kept at a temperature
l1 + l2 l1 + l2
(c) (d) T1 and the other at T2. The rod is composed of two sections
l2 t1 + l1t2 l1t1 + l2 t2 of length l1 and l2 and thermal conductivities K1 and K2
19. The top of an insulated respectively. The temperature at the interface of the two
cylindrical container is covered sections is
by a disc having emissivity 0.6 Oil out
T1 l1 l2 T2
and conductivity 0.167 WK–
1m–1 and thickness 1 cm. The
Oil in
temperature is maintained by K1 K2
circulating oil as shown in figure. Find the radiation loss to
( K1l1T1 + K 2 l2T2 ) ( K 2l2T1 + K1l1T2 )
the surrounding in Jm–2 s–1 if temperature of the upper (a) (b)
surface of the disc is 27°C and temperature of the ( K1l1 + K 2 l2 ) ( K1l1 + K 2 l2 )
surrounding is 27°C. ( K 2l1T1 + K1l2T2 ) ( K1l2T1 + K 2l1T2 )
(a) 595 Jm–2s–1 (b) 545 Jm–2s–1 (c) (d)
( K 2 l1 + K1l2 ) ( K1l2 + K 2 l1 )
(c) 495 Jm–2s–1 (d) None of these

RESPONSE 16. 17. 18. 19. 20.


GRID 21. 22.

Space for Rough Work


EBD_7504
P-40 NTA JEE Main

23. Two spheres of different materials one with double the radius 28. A student takes 50gm wax (specific heat = 0.6 kcal/kg°C)
and one-fourth wall thickness of the other are filled with ice. and heats it till it boils. The graph between temperature and
If the time taken for complete melting of ice in the larger time is as follows. Heat supplied to the wax per minute and
sphere is 25 minute and for smaller one is 16 minute, the boiling point are respectively
ratio of thermal conductivities of the materials of larger

Temperature (°C)
250
spheres to that of smaller sphere is 200
(a) 4 : 5 (b) 5 : 4 (c) 25 : 8 (d) 8 : 25 150
24. If a, b and g are coefficient of linear, area and volume 100
expansion respectively, then 50
(a) g = 3a (b) a = 3g (c) b = 3a (d) g = 3b 0
1 2 3 4 567 8
25. Steam at 100°C is passed into 20 g of water at 10°C. When Time (Minute)
water acquires a temperature of 80°C, the mass of water (a) 500 cal, 50°C (b) 1000 cal, 100°C
present will be: (c) 1500 cal, 200°C (d) 1000 cal, 200°C
[Takespecific heat of water = 1 cal g– 1 °C– 1 and latent heat 29. Consider two identical iron
of steam = 540 cal g– 1] spheres , one which lie on a
(a) 24 g (b) 31.5 g (c) 42.5 g (d) 22.5 g thermally insulating plate,
26. In a room where the temperature is 30°C, a body cools from while the other hangs from
61°C to 59°C in 4 minutes. The time (in minutes) taken by the an insulatory thread. Equal
body to cool from 51°C to 49°C will be : amount of heat is supplied
(a) 8 (b) 5 (c) 6 (d) 4
27. Two rods of same length and area of cross-section A 1 and to the two spheres, then
A2 have their ends at the same temperature. If K1 and K2 (a) temperature of A will be greater than B
are their thermal conductivities, c1 and c2 are their specific (b) temperature of B will be greater than A
(c) their temperature will be equal
heats and d1 and d2 are their densities, then the rate of flow
of heat is the same in both the rods if (d) can’t be predicted
30. Two rods of the same length and diameter having thermal
A1 - k1 A1 k c d conductivities K1 and K2 are joined in parallel. The equivalent
(a) = (b) = 1 1 1
A2 k2 A 2 k 2 c2 d2 thermal conductivity of the combination is
A1 k 2 c1 d1 A1 k 2 K1 K 2 K + K2
(c) = (d) = (a) K + K (b) K1 + K2 (c) 1 (d) K1 K 2
A 2 c 2 d 2 k1 A 2 k1 1 2 2

RESPONSE 23. 24. 25. 26. 27.


GRID 28. 29. 30.

PHYSICS CHAPTERWISE SPEED TEST-10


Total Questions 30 Total Marks 120
Attempted Correct
Incorrect Net Score
Cut-off Score 45 Qualifying Score 60
Success Gap = Net Score –Space
Qualifying
for Rough Work
Score
Net Score = (Correct × 4) – (Incorrect × 1)
Space for Rough Work
PHYSICS Speed
Thermodynamics TEST
No. of Questions
30
Maximum Marks
120
Time
1 Hour
11
Chapter-wise

GENERAL INSTRUCTIONS
• This test contains 30 MCQ's. For each question only one option is correct. Darken the correct circle/ bubble in the
Response Grid provided on each page.
• You have to evaluate your Response Grids yourself with the help of solutions provided at the end of this book.
• Each correct answer will get you 4 marks and 1 mark shall be deduced for each incorrect answer. No mark will be given/
deducted if no bubble is filled. Keep a timer in front of you and stop immediately at the end of 60 min.
• The sheet follows a particular syllabus. Do not attempt the sheet before you have completed your preparation for that
syllabus.
• After completing the sheet check your answers with the solution booklet and complete the Result Grid. Finally spend time
to analyse your performance and revise the areas which emerge out as weak in your evaluation.

1. The relation between U, P and V for an ideal gas in an (a) 4200 J (b) 5000 J
adiabatic process is given by relation U = a + bP V. Find the
(c) 9000 J (d) 9800 J
value of adiabatic exponent (g) of this gas
3. An insulated container of gas has two chambers separated by
b +1 b +1
(a) (b) an insulating partition. One of the chambers has volume V1
b a and contains ideal gas at pressure P1 and temperature T1.
a +1 a The other chamber has volume V2 and contains ideal gas at
(c) (d)
b a+b pressure P2 and temperature T2. If the partition is removed
2. Carbon monoxide is carried around P without doing any work on the gas, the final equilibrium
a closed cycle abc in which bc is an P2 b temperature of the gas in the container will be
isothermal process as shown in the
figure. The gas absorbs 7000 J of heat 1 1 + P2V2 )
T1T2 ( PV 1 1T1 + P2V2T2
PV
a c (a) (b)
as its temperture increases from 300 P1
1 1T2 + P2V2T1
PV 1 1 + P2V2
PV
K to 1000 K in going from a to b. The O V
quantity of heat rejected by the gas V1 V2
1 1T2 + P2V2T1
PV 1 1 + P2V2 )
T1T2 ( PV
during the process ca is (c) (d)
1 1 + P2V2
PV 1 1T1 + P2V2T2
PV

RESPONSE GRID 1. 2. 3.
Space for Rough Work
EBD_7504
P-42 NTA JEE Main

4. The efficiency of an ideal gas with adiabatic exponent ‘g’ for P


the shown cyclic process would be
A
( 2 ln 2 - 1) 2P0
(a) v
g / (g - 1)
2V0 C P0 B
(1 - 2 ln 2)
(b) g /( g - 1)
V0 2V0 V
( 2 ln 2 + 1) V0 A
(c) B 9P0 V0 9P0 V0
g /( g - 1) (a) (b)
T0 2T0 T 2nR nR
( 2 ln 2 - 1)
(d) 9P0 V0 3P0 V0
g /( g + 1) (c) (d)
5. A refrigerator works between 4°C and 30°C. It is required to 4nR 2nR
remove 600 calories of heat every second in order to keep 10. Consider a spherical shell of radius R at temperature T. The
the temperature of the refrigerated space constant. The black body radiation inside it can be considered as an ideal gas
U
power required is: (Take 1 cal = 4.2 joule) of photons with internal energy per unit volume u = µ T4
(a) 2.365 W (b) 23.65 W (c) 236.5 W (d) 2365 W V
1æUö
6. A system goes from A to B via two processes I and II as and pressure p = ç ÷ . If the shell now undergoes an
shown in figure. If DU1 and DU2 are the changes in internal 3è V ø
energies in the processes I and II respectively, then the adiabatic expansion the relation between T and R is :
relation between DU1 and DU2 is 1 1
p (a) T µ (b) T µ 3
(a) DU1 = DU2 II R R
(b) DU2 < DU1 A B (c) T µ e–R (d) T µ e–3R
I 11. The specific heat capacity of a metal at low temperature (T)
(c) DU2 > DU1
3
(d) can not be determined v æ T ö
is given as C (kJK –1kg –1 ) = 32 ç ÷ . A 100 g vessel of
7. A mass of diatomic gas (g = 1.4) at a pressure of 2 è 400 ø
atmospheres is compressed adiabatically so that its this metal is to be cooled from 20 K to 4 K by a special
temperature rises from 27°C to 927°C. The pressure of the refrigerator operating at room temperature (27°C). The
gas in final state is amount of work required to cool in vessel is
(a) 28 atm (b) 68.7 atm (c) 256 atm (d) 8 atm (a) equal to 0.002 kJ
8. A diatomic ideal gas is used in a Carnot engine as the working (b) greater than 0.148 kJ
substance. If during the adiabatic expansion part of the cycle (c) between 0.148 kJ and 0.028 kJ
the volume of the gas increases from V to 32 V, the efficiency (d) less than 0.028 kJ
of the engine is 1
(a) 0.5 (b) 0.75 (c) 0.99 (d) 0.25 12. A diatomic ideal gas is compressed adiabatically to of
32
9. 'n' moles of an ideal gas undergoes a process A ® B as
its initial volume. If the initial temperature of the gas is Ti (in
shown in the figure. The maximum temperature of the gas
Kelvin) and the final temperature is aTi, the value of a is
during the process will be :
(a) 8 (b) 4 (c) 3 (d) 5

RESPONSE 4. 5. 6. 7. 8.
GRID 9. 10. 11. 12.
Space for Rough Work
Physics P-43

13. In a Carnot engine, the temperature of reservoir is 927°C 18. The temperature-entropy diagram of a reversible engine
and that of sink is 27°C. If the work done by the engine cycle is given in the figure. Its efficiency is
when it transfers heat from reservoir to sink is 12.6 × 106J,
1
the quantity of heat absorbed by the engine from the (a) T
reservoir is 4
2T0
(a) 16.8 × 106 J (b) 4 × 106 J 1
(c) 7.6 × 106 J (d) 4.2 × 106 J (b)
2
14. 5.6 litre of helium gas at STP is adiabatically compressed to 2 T0
0.7 litre. Taking the initial temperature to be T1, the work (c)
3
done in the process is
1 S
9 3 15 9 (d) S0 2S0
(a) RT1 (b) RT1 (c) RT1 (d) RT1 3
8 2 8 2 19. If the ratio of specific heat of a gas at constant pressure to
15. Four curves A, B, C and D are drawn in the figure for a given that at constant volume is g, the change in internal energy
amount of a gas. The curves which represent adiabatic and of a mass of gas, when the volume changes from V to 2V at
isothermal changes are constant pressure P, is
R
(a) (b) PV
P B ( g - 1)
C
A
D
PV g PV
(c) (d)
( g - 1) ( g - 1)
20. A Carnot engine whose efficiency is 40%, receives heat at
V
500K. If the efficiency is to be 50%, the source temperature
(a) C and D respectively (b) D and C respectively
for the same exhaust temperature is
(c) A and B respectively (d) B and A respectively (a) 900 K (b) 600 K (c) 700 K (d) 800 K
16. A reversible engine converts one-sixth of the heat input 21. Calculate the work done when 1 mole of a perfect gas is
into work. When the temperature of the sink is reduced by compressed adiabatically. The initial pressure and volume
62ºC, the efficiency of the engine is doubled. The of the gas are 105 N/m2 and 6 litre respectively. The final
temperatures of the source and sink are volume of the gas is 2 litres. Molar specific heat of the gas at
(a) 99ºC, 37ºC (b) 80ºC, 37ºC constant volume is 3R/2. [Given (3)5/3 = 6.19]
(c) 95ºC, 37ºC (d) 90ºC, 37ºC (a) –957 J (b) +957 J (c) – 805 J (d) + 805 J
17. When the state of a gas adiabatically changed from an 22. 1 gm of water at a pressure of 1.01 × 105 Pa is converted into
equilibrium state A to another equilibrium state B an amount steam without any change of temperature. The volume of 1
of work done on the stystem is 35 J. If the gas is taken from g of steam is 1671 cc and the latent heat of evaporation is
state A to B via process in which the net heat absorbed by 540 cal. The change in internal energy due to evaporation of
the system is 12 cal, then the net work done by the system 1 gm of water is
is (1 cal = 4.19 J) (a) » 167 cal (b) 500 cal
(a) 13.2 J (b) 15.4 J (c) 12.6 J (d) 16.8 J (c) 540 cal (d) 581 cal

RESPONSE 13. 14. 15. 16. 17.


GRID 18. 19. 20. 21. 22.

Space for Rough Work


EBD_7504
P-44 NTA JEE Main

23. A diatomic gas initally at 18ºC is compressed adiabatically (a) the source is increased by 50°C
to one eighth of its original volume. The temperature after (b) the sink is decreased by 50°C
compression will be (c) source is increased by 25°C and that of sink is
(a) 18ºC (b) 887ºC (c) 327ºC (d) 395.5°C decreased by 25°C
24. An ideal gas goes from state A to (d) both source and sink are decreased by 25°C each.
state B via three different processes 28. Helium gas goes through a cycle
as indicated in the P-V diagram : P A 1 B C
2 ABCDA (consisting of two isochoric 2p0
If Q1, Q2, Q3 indicate the heat a and isobaric lines) as shown in figure.
absorbed by the gas along the 3 B
Efficiency of this cycle is nearly p0 D
three processes and DU1, DU2, DU3 A
(Assume the gas to be close to ideal
indicate the change in internal
energy along the three processes V gas) V 2V 0 0
respectively, then
(a) 15.4% (b) 9.1% (c) 10.5% (d) 12.5%
(a) Q1 > Q2 > Q3 and DU1 = DU2 = DU3
29. The volume of an ideal gas is 1 litre and its pressure is equal
(b) Q3 > Q2 > Q1 and DU1= DU2 = DU3 to 72 cm of mercury column. The volume of gas is made
(c) Q1 = Q2 = Q3 and DU1 > DU2 > DU3 900 cm3 by compressing it isothermally. The stress of the
(d) Q3 > Q2 > Q1 and DU1> DU2 > DU3 gas will be
(a) 8 cm of Hg (b) 7 cm of Hg
1
25. A Carnot engine, having an efficiency of h = as heat (c) 6 cm of Hg (d) 4 cm of Hg
10 30. A gas is enclosed in a cylinder with a movable frictionless
engine, is used as a refrigerator. If the work done on the
system is 10 J, the amount of energy absorbed from the piston. Its initial thermodynamic state at pressure Pi = 105
reservoir at lower temperature is Pa and volume Vi = 10–3 m3 changes to a final state at Pf = (1/
(a) 99 J (b) 90 J (c) 1 J (d) 100 J 32) × 105 Pa and Vf = 8 × 10–3 m3 in an adiabatic quasi-static
26. In a thermodynamic process, fixed mass of a gas is changed process, such that P3 V5 = constant. Consider another
in such a manner that the gas release 20 J of heat and 8 J of thermodynamic process that brings the system from the same
work was done on the gas. If the initial internal energy of initial state to the same final state in two steps: an isobaric
the gas was 30 J, the final internal energy will be expansion at Pi followed by an isochoric (isovolumetric)
(a) 2 joule (b) 18 joule (c) 42 joule (d) 58 joule
process at volume Vf . The amount of heat supplied to the
27. A Carnot engine is working between 127°C and 27°C. The
increase in efficiency will be maximum when the temperature system in the two-step process is approximately
of (a) 112 J (b) 294 J (c) 588 J (d) 813 J

RESPONSE 23. 24. 25. 26. 27.


GRID 28. 29. 30.

PHYSICS CHAPTERWISE SPEED TEST-11


Total Questions 30 Total Marks 120
Attempted Correct
Incorrect Net Score
Space for Rough Work

Cut-off Score 40 Qualifying Score 50


Success Gap = Net Score – Qualifying Score
Net Score = (Correct × 4) – (Incorrect × 1)
Space for Rough Work
PHYSICS Speed
Kinetic Theory TEST
No. of Questions
30
Maximum Marks
120
Time
1 Hour
12
Chapter-wise

GENERAL INSTRUCTIONS
• This test contains 30 MCQ's. For each question only one option is correct. Darken the correct circle/ bubble in the
Response Grid provided on each page.
• You have to evaluate your Response Grids yourself with the help of solutions provided at the end of this book.
• Each correct answer will get you 4 marks and 1 mark shall be deduced for each incorrect answer. No mark will be given/
deducted if no bubble is filled. Keep a timer in front of you and stop immediately at the end of 60 min.
• The sheet follows a particular syllabus. Do not attempt the sheet before you have completed your preparation for that
syllabus.
• After completing the sheet check your answers with the solution booklet and complete the Result Grid. Finally spend time
to analyse your performance and revise the areas which emerge out as weak in your evaluation.

1. Air is pumped into an automobile tube upto a pressure of 3. A gaseous mixture consists of 16 g of helium and 16 g of
200 kPa in the morning when the air temperature is 22°C.
Cp
During the day, temperature rises to 42°C and the tube oxygen. The ratio of the mixture is
expands by 2%. The pressure of the air in the tube at this Cv
temperature, will be approximately
(a) 1.62 (b) 1.59
(a) 212 kPa (b) 209 kPa
(c) 1.54 (d) 1.4
(c) 206 kPa (d) 200 kPa
4. Two containers A and B are partly filled with water and
2. 4.0 g of a gas occupies 22.4 litres at NTP. The specific heat closed. The volume of A is twice that of B and it contains
capacity of the gas at constant volume is 5.0JK–1. If the
half the amount of water in B. If both are at the same
speed of any quantity x in this gas at NTP is 952 ms–1, then
temperature, the water vapour in the containers will have
the heat capacity at constant pressure is (Take gas constant
pressure in the ratio of
R = 8.3 JK–1 mol–1)
(a) 1 : 2 (b) 1 : 1
(a) 7.5 JK–1 mol–1 (b) 7.0 JK–1 mol–1
(c) 8.5 JK–1 mol–1 (d) 8.0 JK–1 mol–1 (c) 2 : 1 (d) 4 : 1

RESPONSE GRID 1. 2. 3. 4.

Space for Rough Work


EBD_7504
P-46 NTA JEE Main

5. The figure shows the volume V versus temperature T graphs 8. Figure shows a parabolic graph between T and 1/V for a
for a certain mass of a perfect gas at two constant pressures mixture of a gases undergoing an adiabatic process. What
of P1 and P2. What inference can you draw from the is the ratio of Vrms of molecules and speed of sound in mixture?
graphs? V P2 T
(a) 3/ 2
(a) P1 > P2 P1
(b) P1 < P2 q2
(b) 2 2T0
(c) P1 = P2 q1 T T0
(d) No inference can be drawn due to insufficient
information. (c) 2/3 1/V
1/V0 4/V0
6. Consider an ideal gas confined in an isolated closed chamber.
As the gas undergoes an adiabatic expansion, the average (d) 3
time of collision between molecules increases as Vq, where
9. 1 mole of a monatomic and 2 mole of a diatomic gas are
æ Cp ö mixed. The resulting gas is taken through a process in which
V is the volume of the gas. The value of q is : ç g = ÷ molar heat capacity was found 3R. Polytropic constant in
è Cv ø
g +1 g -1 the process is
(a) (b) (a) –1/5 (b) 1/5 (c) 2/5 (d) –2/5
2 2
10. A thermally insulated vessel contains an ideal gas of
3g + 5 3g - 5 molecular mass M and ratio of specific heats g. It is moving
(c) (d)
6 6 with speed v and it suddenly brought to rest. Assuming no
7. Work done by a system under isothermal change from a heat is lost to the surroundings, its temperature increases
volume V1 to V2 for a gas which obeys Vander Waal's by
æ an 2 ö ( g –1) gMv 2
equation (V - bn) ç P + ÷ = nRT is (a) Mv 2 K (b) K
ç V ÷ 2gR 2R
è ø
( g – 1) (g –1)
æ V - nb ö (c) Mv 2 K (d) Mv 2 K
2 æ V1 - V2 ö 2R 2( g + 1) R
(a) nRT log e ç 2 ÷ + an ç ÷
è V1 - nb ø è V1V2 ø 11. The temperature at which proton in hydrogen gas would
æ V - nb ö
have enough energy to overcome a barrier of 4.14 × 10–14 J is
2 æ V1 - V2 ö
(b) nRT log10 ç 2 ÷ + an ç ÷ (Boltzmann constant = 1.38 × 10–23 JK–1)
è V1 - nb ø è V1V2 ø (a) 2 × 109 K (b) 109 K (c) 6 × 109 K (d) 3 × 109 K
æ V - nb ö 2 æ V1 - V2 ö 12. If 2 moles of an ideal monatomic gas at temperature T0 is
(c) nRT loge ç 2 ÷ + bn ç ÷ mixed with 4 moles of another ideal monatomic gas at
V
è 1 - nb ø è V1V2 ø
temperature 2T0, then the temperature of the mixture is
æ V - nb ö 2 æ V1V2 ö 5 3 4 5
(d) nRT log e ç 1 ÷ + an ç ÷ (a) T0 (b) T0 (c) T0 (d) T0
è V2 - nb ø è V1 – V2 ø 3 2 3 4

RESPONSE 5. 6. 7. 8. 9.
GRID 10. 11. 12.
Space for Rough Work
Physics P-47

13. The root mean square velocity of hydrogen molecules at 19. The temperature of the mixture of one mole of helium and
300 K is 1930 metre/sec. Then the r.m.s velocity of oxygen one mole of hydrogen is increased from 0°C to 100°C at
molecules at 1200 K will be constant pressure. The amount of heat delivered will be
(a) 482.5 metre/sec (b) 965 metre/sec (a) 600 cal (b) 1200 cal
(c) 1930 metre/sec (d) 3860 metre/sec (c) 1800 cal (d) 3600 cal
14. For a gas, difference between two specific heats is 5000 J/ 20. The P-V diagram of a diatomic gas is a straight line passing
mole°C. If the ratio of specific heats is 1.6, the two specific through origin. The molar heat capacity of the gas in the
heats in J/mole-°C are process will be
(a) CP = 1.33 × 104, CV = 2.66 × 104
4R
(b) CP = 13.3 × 104, CV = 8.33 × 103 (a) 4 R (b) 2.5 R (c) 3 R (d)
3
(c) CP = 1.33 × 104, CV = 8.33 × 103
21. Three perfect gases at absolute temperatures T1, T2 and T3
(d) CP = 2.6 × 104, CV = 8.33 × 104 are mixed. The masses of molecules are m1, m2 and m3 and
15. A graph is plotted with PV/T on y-axis and mass of the gas the number of molecules are n1, n2 and n3 respectively.
along x-axis for different gases. The graph is Assuming no loss of energy, the final temperature of the
(a) a straight line parallel to x-axis for all the gases mixture is :
(b) a straight line passing through origin with a slope
n1T1 + n2T2 + n3T3 n1T12 + n2T22 + n3T32
having a constant value for all the gases (a) (b)
n1 + n2 + n3 n1T1 + n2T2 + n3T3
(c) a straight line passing through origin with a slope
having different values for different gases
(d) a straight line parallel to y-axis for all the gases
n12T12 + n22T22 + n32T32 (T1 + T2 + T3 )
(c) (d)
n1T1 + n2T2 + n3T3 3
16. A gas mixture consists of 2 moles of oxygen and 4 moles of
Argon at temperature T. Neglecting all vibrational moles, 22. A sample of an ideal gas occupies a volume V at a pressure
the total internal energy of the system is P and absolute temperature T. The mass of each molecule is
(a) 4 RT (b) 15 RT (c) 9 RT (d) 11RT m. The equation for density is
17. One mole of a gas occupies 22.4 lit at N.T.P. Calculate the (a) m k T (b) P/k T
difference between two molar specific heats of the gas. (c) P/(k T V) (d) P m/k T
J = 4200 J/kcal. 23. N molecules, each of mass m, of gas A and 2 N molecules, each
(a) 1.979 k cal/kmol K (b) 2.378 k cal/kmol K of mass 2 m, of gas B are contained in the same vessel which
(c) 4.569 kcal/kmol K (d) 3.028 k cal/ kmol K is maintained at a temperature T. The mean square velocity of
18. If the intermolecular forces vanish away, the volume molecules of B type is denoted by V2 and the mean square
occupied by the molecules contained in 4.5 g water at V
standard temperature and pressure will be velocity of A type is denoted by V1, then 1 is
V2
(a) 5.6 litre (b) 4.5 litre
(c) 11.2 litre (d) 6.5 litre (a) 2 (b) 1 (c) 1/3 (d) 2/3

13. 14. 15. 16. 17.


RESPONSE
18. 19. 20. 21. 22.
GRID
23.
Space for Rough Work
EBD_7504
P-48 NTA JEE Main

24. A vessel has 6g of hydrogen at pressure P and temperature of 1 litre capacity, without any change in temperature. The
500K. A small hole is made in it so that hydrogen leaks out. total pressure would be
How much hydrogen leaks out if the final pressure is P/2 (a) 1.5 atm (b) 0.5 atm (c) 2.0 atm (d) 1.0 atm
and temperature falls to 300 K ? 28. If the potential energy of a gas molecule is
(a) 2g (b) 3g (c) 4g (d) 1g U = M/r6 – N/r12, M and N being positive constants. Then
25. The work of 146 kJ is performed in order to compress one the potential energy at equilibrium must be
kilomole of gas adiabatically and in this process the (a) zero (b) M2/4N (c) N2/4M (d) MN2/4
temperature of the gas increases by 7°C. The gas is (R = 8.3 29. Consider a gas with density r and c as the root mean
J mol–1 K–1) square velocity of its molecules contained in a volume. If
(a) diatomic the system moves as whole with velocity v, then the pressure
(b) triatomic exerted by the gas is
(c) a mixture of monatomic and diatomic 1 2 1
(d) monatomic (a) rc (b) r( c + v ) 2
3 3
26. The molar heat capacities of a mixture of two gases at constant
1 1
volume is 13R/6. The ratio of number of moles of the first gas (c) r(c – v) 2 (d) r(c –2 – v ) 2
to the second is 1 : 2. The respective gases may be 3 3
(a) O2 and N2 (b) He and Ne 30. At 10° C the value of the density of a fixed mass of an ideal
(c) He and N2 (d) N2 and He gas divided by its pressure is x. At 110°C this ratio is:
27. One litre of oxygen at a pressure of 1 atm, and 2 litres of 383 10 283
(a) x (b) x (c) x (d) x
nitrogen at a pressure of 0.5 atm are introduced in the vessel 283 110 383

RESPONSE 24. 25. 26. 27. 28.


GRID 29. 30.

PHYSICS CHAPTERWISE SPEED TEST-12


Total Questions 30 Total Marks 120
Attempted Correct
Incorrect Net Score
Cut-off Score 45 Qualifying Score 60
Success Gap = Net Score – Qualifying Score
Net Score = (Correct × 4) – (Incorrect × 1)
Space for Rough Work
PHYSICS Speed
Oscillations TEST
No. of Questions
30
Maximum Marks
120
Time
1 Hour
13
Chapter-wise

GENERAL INSTRUCTIONS
• This test contains 30 MCQ's. For each question only one option is correct. Darken the correct circle/ bubble in the
Response Grid provided on each page.
• You have to evaluate your Response Grids yourself with the help of solutions provided at the end of this book.
• Each correct answer will get you 4 marks and 1 mark shall be deduced for each incorrect answer. No mark will be given/
deducted if no bubble is filled. Keep a timer in front of you and stop immediately at the end of 60 min.
• The sheet follows a particular syllabus. Do not attempt the sheet before you have completed your preparation for that
syllabus.
• After completing the sheet check your answers with the solution booklet and complete the Result Grid. Finally spend time
to analyse your performance and revise the areas which emerge out as weak in your evaluation.

1. If x, v and a denote the displacement, the velocity and the It is connected by both springs as shown in fig. then time
acceleration of a particle executing simple harmonic motion period is t0. The correct relation is
of time period T, then, which of the following does not change
with time? (a) t 02 = t12 + t 22
k1 k2
(a) aT/x (b) aT + 2pv
(b) t 0-2 = t 1-2 + t 2-2
(c) aT/v (d) a2T2 + 4p2v2
2. A particle of mass m executes simple harmonic motion with (c) t 0-1 = t1-1 + t -2 1 m
amplitude a and frequency n. The average kinetic energy
during its motion from the position of equilibrium to the end (d) t 0 = t1 + t 2
is rd
æ 1ö
(a) 2p2ma2n2 (b) p2ma2n2 4. The amplitude of a damped oscillator becomes ç ÷ in 2
è 3ø
1 2 2
1
(c) ma n (d) 4p2ma2n2 seconds. If its amplitude after 6 seconds is times the
4 n
original amplitude, the value of n is
3. A mass is suspended separately by two different springs in
successive order, then time periods is t1 and t2 respectively. (a) 32 (b) 33 (c) 33 (d) 23

RESPONSE GRID 1. 2. 3. 4.
Space for Rough Work
EBD_7504
P-50 NTA JEE Main

5. A mass M, attached to a horizontal spring, executes S.H.M. m m


with amplitude A1. When the mass M passes through its (c) 2 2 (d) 2 2
w0 -w (w 0
+w )
mean position then a smaller mass m is placed over it and
both of them move together with amplitude A2. The ratio of 9. A point mass oscillates along the x-axis according to the law
x = x0 cos(wt – p/4). If the acceleration of the particle is
æ A1 ö written as a = A cos(wt – d), then
çè A ÷ø
is:
2
(a) A = x0w2, d = 3p/4 (b) A = x0, d = –p/4
1 1
M +m æ M ö 2 (c) æ M + m ö 2 (d) M (c) A = x0w2, d = p/4 (d) A = x0w2, d = –p/4
(a) (b) çè ÷ø çè ÷ø
M M +m M M +m 10. The particle executing simple harmonic motion has a kinetic
energy K0 cos2 wt. The maximum values of the potential
6. A uniform rod of length L and mass M energy and the total energy are respectively
is pivoted at the centre. Its two ends are (a) K0/2 and K0 (b) K0 and 2K0
attached to two springs of equal spring (c) K0 and K0 (d) 0 and 2K0
constants k. The springs are fixed to rigid 11. A simple pendulum attached to the ceiling of a stationary
supports as shown in the figure, and the rod is free to oscillate lift has a time period T. The distance y covered by the lift
in the horizontal plane. The rod is gently pushed through a moving upwards varies with time t as y = t2 where y is in
small angle q in one direction and released. The frequency metres and t in seconds. If g = 10 m/s2, the time period of
of oscillation is pendulum will be
1 2k 1 k 1 6k 1 24k 4 5 5 6
(a) (b) (c) (d) (a) T (b) T (c) T (d) T
2p M 2p M 2p M 2p M 5 6 4 5
7. An ideal gas enclosed in a vertical cylindrical container 12. The amplitude of a damped oscillator decreases to 0.9 times
supports a freely moving piston of mass M. The piston and its original magnitude in 5s. In another 10s it will decrease to
the cylinder have equal cross sectional area A. When the a times its original magnitude, where a equals
piston is in equilibrium, the volume of the gas is V0 and its (a) 0.7 (b) 0.81 (c) 0.729 (d) 0.6
pressure is P0. The piston is slightly displaced from the 13. A particle moves with simple harmonic motion in a straight
equilibrium position and released. Assuming that the system line. In first ts, after starting from rest it travels a distance a,
and in next t s it travels 2a, in same direction, then:
is completely isolated from its surrounding, the piston (a) amplitude of motion is 3a
executes a simple harmonic motion with frequency (b) time period of oscillations is 8t
1 AgP0 1 V0 MP0 (c) amplitude of motion is 4a
(a) (b) (d) time period of oscillations is 6t
m
2p V0 M 2p A 2 g
14. A mass m fall on spring of spring constant k h
1 A 2 gP0 1 MV0 and negligible mass from a height h.
(c) (d) Assuming it sticks to the pan and executes
2p MV0 2p AgP0
simple harmonic motion, the maximum height k
8. A particle of mass m is attached to a spring (of spring upto which the pan will rise is
constant k) and has a natural angular frequency w0. An
external force F(t) proportional to cos wt(w ¹ w0) is applied mg mg é 2kh ù
to the oscillator. The displacement of the oscillator will be (a) (b) ê 1+ - 1ú
k k ë mg û
proportional to
1 1 mg é 2kh ù mg é kh ù
(a) 2 2 (b) 2 2 (c) ê 1+ + 1ú (d) ê 1+ - 1ú
m (w 0 + w ) m (w 0 - w ) k ë mg û k ë mg û

RESPONSE 5. 6. 7. 8. 9.
GRID 10. 11. 12. 13. 14.
Space for Rough Work
Physics P-51

15. A 1 kg mass is attached to a 19. In the figure shown, the spring is light and

///////////////
spring of force constant 600 3m/s
1 kg has a force constant k. The pulley is light and
0.5kg
N/m and rests on a smooth //////////////////////////////////////////////////// smooth and the string is light. The suspended
horizontal surface with other end of the spring tied to wall block has a mass m. On giving a slight
as shown in figure. A second mass of 0.5 kg slides along the displacement vartically to the block in the
surface towards the first at 3m/s. If the masses make a downward direction from its equilibrium
perfectly inelastic collision, then find amplitude and time position the block executes S.H.M. on being
period of oscillation of combined mass. released with time period T. Then m
p p
(a) 5cm, s (b) 5cm, s m m
10 5 (a) T = 2p (b) T = 2p
k 2k
2p p
(c) 4cm, s (d) 4cm, s
5 3 2m m
16. A pendulum with time period of 1s is losing energy due (c) T = 2p (d) T = 4p
k k
to damping. At certain time its energy is 45 J. If after
completing 15 oscillations, its energy has become 15 J, its 20. A pendulum made of a uniform wire of cross sectional area
damping constant (in s–1) is : A has time period T. When an additional mass M is added to
its bob, the time period changes to TM. If the Young's
1 1 1
(a) (b) ln3 (c) 2 (d) ln3 1
2 30 15 modulus of the material of the wire is Y then is equal to:
Y
17. A rod of length l is in motion such that its ends A and B are (g = gravitational acceleration)
moving along x-axis and y-axis respectively. It is given that
é æT é æ T ö2 ù A
dq ö ù A
2
= 2 rad/sec always. P is a fixed point on the rod. Let M (a) ê1 - ç M ÷ ú (b) ê1 - ç ÷ ú
dt ë è T ø û Mg ëê è TM ø úû Mg
be the projection of P on x-axis. For the time interval in
p éæ T ö2 ù A é T 2 ù Mg
which q changes from 0 to , the correct statement is
2 (c) êç M ÷ - 1ú (d) êæç M ö÷ - 1ú
(a) The acceleration of M is always directed towards right ëè T ø û Mg ëè T ø û A
(b) M executes SHM 21. A coin is placed on a horizontal platform which undergoes
(c) M moves with constant speed vertical simple harmonic motion of angular frequency w. The
(d) M moves with constant acceleration amplitude of oscillation is gradually increased. The coin will
18. A point particle of mass 0.1 kg is executing S.H.M. of leave contact with the platform for the first time
amplitude 0.1 m. When the particle passes through the mean (a) at the mean position of the platform
position, its kinetic energy is 8 × 10–3 joule. Obtain the
equation of motion of this particle, if the initial phase of g
(b) for an amplitude of
oscillation is 45º. w2
æ pö æ pö g2
(a) y = 0.1sin ç ±4t + ÷ (b) y = 0.2sin ç ±4t + ÷
è 4 ø è 4ø (c) for an amplitude of
w2
æ pö æ pö
(c) y = 0.1sin çè ±2t + ÷ø (d) y = 0.2sin ç ±2t + ÷ (d) at the highest position of the platform
4 è 4ø

RESPONSE 15. 16. 17. 18. 19.


GRID 20. 21.
Space for Rough Work
EBD_7504
P-52 NTA JEE Main

22. For a simple pendulum, a graph is plotted between its kinetic 27. A body executes simple harmonic motion under the action
energy (KE) and potential energy (PE) against its 4
displacement d. Which one of the following represents these of a force F1 with a time period s. If the force is changed
correctly? (graphs are schematic and not drawn to scale) 5
E E 3
KE to F2, it executes S.H.M. with time period s. If both the
PE 5
forces F1 and F2 act simultaneously in the same direction
(a) d (b) KE on the body, its time period in second is
d
12 7 24 5
PE (a) (b) (c) (d)
E E PE
25 5 25 7
KE 28. A block connected to a spring oscillates vertically. A damping
PE
forceFd, acts on the block by the surrounding medium. Given
(c) d (d) KE
d as Fd = –bV, b is a positive constant which depends on :
(a) viscosity of the medium
23. The bob of a simple pendulum executes simple harmonic (b) size of the block
motion in water with a period t, while the period of oscillation (c) shape of the block
of the bob is t0 in air. Neglecting frictional force of water and
given that the density of the bob is (4/3) × 1000 kg/m3. The (d) All of these
relationship between t and t0 is 29. A uniform pole of length l = 2
(a) t = 2t0 (b) t = t0/2 (c) t = t0 (d) t = 4t0 L is laid on smooth horizontal
table as shown in figure. The M
24. Two springs, of force constants k1 and k2 are connected to x =0
a mass m as shown. The frequency of oscillation of the mass of pole is M and it is O
mass is f. If both k1 and k2 are made four times their original connected to a frictionless axis
values, the frequency of oscillation becomes at O.A spring with force 2L
m constant k is connected to the other end. The pole is
k1 k2
displaced by a small angle q0 from equilibrium position and
released such that it performs small oscillations. Then
(a) 2 f (b) f /2 (c) f /4 (d) 4 f
M k
25. Starting from the origin a body oscillates simple harmonically (a) w0 = (b) w0 =
with a period of 2 s. After what time will its kinetic energy be 3k 3M
75% of the total energy? 3k k
1 1 1 1 (c) w0 = (d) w 0 =
(a) s (b) s (c) s (d) s M 2M
6 4 3 12 30. If a simple pendulum of length l has maximum angular
26. In S.H.M., the ratio of kinetic energy at mean position to the displacement q, then the maximum K.E. of bob of mass m is
potential energy when the displacement is half of the
amplitude is 1
(a) ml / g (b) mg / 2l
4 2 4 1 2
(a) (b) (c) (d) (c) mgl (1 – cos q) (d) mgl sin q/2
1 3 3 2
RESPONSE 22. 23. 24. 25. 26.
GRID 27. 28. 29. 30.

PHYSICS CHAPTERWISE SPEED TEST-13


Total Questions 30 Total Marks 120
Attempted Correct
Space for Rough Work

Incorrect Net Score


Cut-off Score 40 Qualifying Score 50
Success Gap = Net Score – Qualifying Score
Net Score = (Correct × 4) – (Incorrect × 1)
Space for Rough Work
PHYSICS Speed
Waves TEST
No. of Questions
30
Maximum Marks
120
Time
1 Hour
14
Chapter-wise

GENERAL INSTRUCTIONS
• This test contains 30 MCQ's. For each question only one option is correct. Darken the correct circle/ bubble in the
Response Grid provided on each page.
• You have to evaluate your Response Grids yourself with the help of solutions provided at the end of this book.
• Each correct answer will get you 4 marks and 1 mark shall be deduced for each incorrect answer. No mark will be given/
deducted if no bubble is filled. Keep a timer in front of you and stop immediately at the end of 60 min.
• The sheet follows a particular syllabus. Do not attempt the sheet before you have completed your preparation for that
syllabus.
• After completing the sheet check your answers with the solution booklet and complete the Result Grid. Finally spend time
to analyse your performance and revise the areas which emerge out as weak in your evaluation.

1. When a wave travel in a medium, the particle displacement 3. An engine approaches a hill with a constant speed. When it
is given by the equation y = a sin 2p (bt– cx) where a, b and is at a distance of 0.9 km, it blows a whistle whose echo is
c are constants. The maximum particle velocity will be twice
heard by the driver after 5 seconds. If the speed of sound in
the wave velocity if
air is 330 m/s, then the speed of the engine is :
1 1
(a) c= (b) c = pa (c) b = ac (d) b = (a) 32 m/s (b) 27.5 m/s
pa ac
2. A thick uniform rope of length L is hanging ///////////////// (c) 60 m/s (d) 30 m/s
from a rigid support. A transverse wave 4. A sonometer wire of length 1.5 m is made of steel. The tension
of wavelength l0 is set up at the middle of in it produces an elastic strain of 1%. What is the fundamental
rope as shown in figure. The wavelength L
of the wave as it reaches to the topmost frequency of steel if density and elasticity of steel are 7.7 ×
point is 103 kg/m3 and 2.2 × 1011 N/m2 respectively?
l0 (a) 188.5 Hz (b) 178.2 Hz
(a) 2l0 (b) 2l 0 (c) (d) l0 (c) 200.5 Hz (d) 770 Hz
2

RESPONSE GRID 1. 2. 3. 4.

Space for Rough Work


EBD_7504
P-54 NTA JEE Main

5. The fundamental frequency of a sonometer wire of length l so that one half of its volume is submerged. The new
is f0. A bridge is now introduced at a distance of Dl from the fundamental frequency in Hz is
centre of the wire (Dl << l). The number of beats heard if 1/ 2 1/ 2
both sides of the bridges are set into vibration in their æ 2r - 1 ö æ 2r ö
(a) 300çç ÷÷ (b) 300çç ÷÷
fundamental modes are – è 2r ø è 2r - 1 ø
8f 0 Dl f0 Dl 2f0 Dl 4f0 Dl
(a) (b) (c) (d) æ 2r ö æ 2r - 1 ö
l l l l (c) 300çç ÷÷ (d) 300çç ÷÷
è 2r - 1 ø è 2r ø
6. A source of sound S emitting
waves of frequency 100 Hz and 11. The transverse displacement y (x, t) of a wave on a string is
an observor O are located at given by y( x, t ) = e
(
- ax2 + bt 2 + 2 abxt ).
some distance from each other.
The source is moving with a 60° This represents a:
–1 O
speed of 19.4 ms at an angle S b
of 60° with the source observer (a) wave moving in –x direction with speed
a
line as shown in the figure. The observor is at rest. The (b) standing wave of frequency b
apparent frequency observed by the observer is (velocity
of sound in air is 330 ms–1) 1
(c) standing wave of frequency
(a) 103 Hz (b) 106 Hz (c) 97 Hz (d) 100 Hz b
7. Two identical piano wires kept under the same tension T a
have a fundamental frequency of 600 Hz. The fractional (d) wave moving in + x direction with speed
b
increase in the tension of one of the wires which will lead to 12. In the figure shown the wave speed is v. The velocity of car
occurrence of 6 beats/s when both the wires oscillate is v0. The beat frequency for the observer will be
together would be
(a) 0.02 (b) 0.03 (c) 0.04 (d) 0.01
8. Two sound sources emitting sound each of wavelength l
are fixed at a given distance apart. A listener moves with a
velocity u along the line joining the two sources. The number 2 f 0 vv0 2 f0v2
of beats heard by him per second is (a) (b)
v 2 + v0 2 v 2 - v0 2
u 2u u u
(a) (b) (c) (d) 2 f 0 vv0 f 0 vv0
2l l l 3l (c) v 2 - v 2 (d) v 2 - v 2
9. A star, which is emitting radiation at a wavelength of 0 0

5000 Å, is approaching the earth with a velocity of 1.50 × 106 13. A source of sound A emitting waves of frequency 1800 Hz
m/s. The change in wavelength of the radiation as received is falling towards ground with a terminal speed v. The
on the earth is observer B on the ground directly beneath the source
(a) 0.25 Å (b) 2.5 Å (c) 25 Å (d) 250 Å receives waves of frequency 2150 Hz. The source A receives
10. An object of specific gravity r is hung from a thin steel waves, reflected from ground of frequency nearly: (Speed
wire. The fundamental frequency for transverse standing of sound = 343 m/s)
waves in the wire is 300 Hz. The object is immersed in water (a) 2150 Hz (b) 2500 Hz (c) 1800 Hz (d) 2400 Hz

RESPONSE 5. 6. 7. 8. 9.
GRID 10. 11. 12. 13.
Space for Rough Work
Physics P-55

14. A pipe of length 85 cm is closed from one end. Find the 19. Two points are located at a distance of 10 m and 15 m from
number of possible natural oscillations of air column in the the source of oscillation. The period of oscillation is 0.05
pipe whose frequencies lie below 1250 Hz. The velocity of sec and the velocity of the wave is 300 m/sec. What is the
sound in air is 340 m/s. phase difference between the oscillations of two points?
(a) 12 (b) 8 (c) 6 (d) 4 p 2p p
15. The equation of a wave on a string of linear mass density (a) (b) (c) p (d)
3 3 6
0.04 kg m–1 is given by
20. A uniform rope of length L and mass m1 hangs vertically
é æ t x öù from a rigid support. A block of mass m 2 is attached to the
y =0.02(m) sin ê2p ç - ÷ú .
ë è 0.04(s ) 0.50(m) ø û free end of the rope. A transverse pulse of wavelength l1 is
produced at the lower end of the rope. The wavelength of
The tension in the string is
the pulse when it reaches the top of the rope is l2 the ratio
(a) 4.0 N (b) 12.5 N (c) 0.5 N (d) 6.25 N
l2/l1 is
16. A sound absorber attenuates the sound level by 20 dB. The
intensity decreases by a factor of m1 m1 + m 2
(a) 100 (b) 1000 (c) 10000 (d) 10 (a) m2 (b) m2
17. A massless rod of length L is suspended by two identical
strings AB and CD of equal length. A block of mass m is m2 m1 + m 2
suspended from point O such that BO is equal to ‘x’. Further (c) m1 (d) m1
it is observed that the frequency of 1st harmonic in AB is
21. If n1, n2 and n3 are the fundamental frequencies of three
equal to 2nd harmonic frequency in CD. ‘x’ is
segments into which a string is divided, then the original
L A C fundamental frequency n of the string is given by
(a)
5 (a) n = n1 + n2 + n3
4L 1 1 1 1
(b) (b) = + +
5 n n1 n2 n3
3L O
(c) B D 1 1 1 1
4 x L
(c) = + +
n n1 n2 n3
m
L
(d) (d) n = n1 + n2 + n3
4
18. An organ pipe P1, closed at one end vibrating in its first 22. The vibrations of a string of length 60 cm fixed at both the
harmonic and another pipe P2, open at both ends vibrating æ 4p x ö
in its third harmonic, are in resonance with a given tuning ends are represented by the equation y = 2 sin ç cos
è 15 ÷ø
fork. The ratio of the lengths of P1 and P2 is :
(96pt) where x and y are in cm. The maximum number of
8 1 1 1 loops that can be formed in it is
(a) (b) (c) (d)
3 6 2 3 (a) 4 (b) 16 (c) 5 (d) 15

RESPONSE 14. 15. 16. 17. 18.


GRID 19. 20. 21. 22.

Space for Rough Work


EBD_7504
P-56 NTA JEE Main

23. The frequency of whistle of an engine appears to be (4/5) th 27. Each of the two strings of length 51.6 cm and 49.1 cm are
of initial frequency when it crosses a stationary observer. If tensioned separately by 20 N force. Mass per unit length
the velocity of sound is 330 m/s, then the speed of engine of both the strings is same and equal to 1 g/m. When both
will be the strings vibrate simultaneously the number of beats is
(a) 30 m/s (b) 36.6 m/s (c) 40 m/s (d) 330 m/s (a) 7 (b) 8 (c) 3 (d) 5
28. The fundamental frequency of a closed organ pipe of length
24. A sonometer wire supports a 4 kg load and vibrates in
20 cm is equal to the second overtone of an organ pipe open
fundamental mode with a tuning fork of frequency 416 Hz.
at both the ends. The length of organ pipe open at both the
The length of the wire between the bridges is now doubled. ends is
In order to maintain fundamental mode, the load should be (a) 100 cm (b) 120 cm (c) 140 cm (d) 80 cm
changed to 29. In a standing wave experiment, a 1.2 kg horizontal rope is
(a) 1 kg (b) 2 kg (c) 4 kg (d) 16 kg fixed in place at its two ends (x = 0 and x = 2.0 m) and made
25. The wavelength of two waves are 50 and 51 cm respectively. to oscillate up and down in the fundamental mode, at
If the temperature of the room is 20°C then what will be the frequency 5.0 Hz. At t = 0, the point at x = 1.0 m has zero
number of beats produced per second by these waves, when displacement and is moving upward in the positive direction
the speed of sound at 0°C is 332 m/s? of y axis with a transverse velocity 3.14 m/s.
(a) 24 (b) 14 (c) 10 (d) 25 Tension in the rope is
26. In a resonance tube with tuning fork of frequency 512 Hz, (a) 60 N (b) 100 N (c) 120 N (d) 240 N
first resonance occurs at water level equal to 30.3 cm and 30. The equation of a travelling wave is y = 60 cos (180 t – 6x)
second resonance occurs at 63.7 cm. The maximum possible where y is in mm, t in second and x in metres. The ratio of
error in the speed of sound is maximum particle velocity to velocity of wave propagation is
(a) 51.2 cm/s (b) 102.4 cm/s (a) 3.6 × 10–2 (b) 3.6 × 10–4
(c) 204.8 cm/s (d) 153.6 cm/s (c) 3.6 × 10–6 (d) 3.6 × 10–11

RESPONSE 23. 24. 25. 26. 27.


GRID 28. 29. 30.

PHYSICS CHAPTERWISE SPEED TEST-14


Total Questions 30 Total Marks 120
Attempted Correct
Incorrect Net Score
Cut-off Score 45 Qualifying Score 60
Success Gap = Net Score – Qualifying Score
Net Score = (Correct × 4) – (Incorrect × 1)
Space for Rough Work
PHYSICS Speed
Electric Charges and Fields TEST
No. of Questions
30
Maximum Marks
120
Time
1 Hour
15
Chapter-wise

GENERAL INSTRUCTIONS
• This test contains 30 MCQ's. For each question only one option is correct. Darken the correct circle/ bubble in the
Response Grid provided on each page.
• You have to evaluate your Response Grids yourself with the help of solutions provided at the end of this book.
• Each correct answer will get you 4 marks and 1 mark shall be deduced for each incorrect answer. No mark will be given/
deducted if no bubble is filled. Keep a timer in front of you and stop immediately at the end of 60 min.
• The sheet follows a particular syllabus. Do not attempt the sheet before you have completed your preparation for that
syllabus.
• After completing the sheet check your answers with the solution booklet and complete the Result Grid. Finally spend time
to analyse your performance and revise the areas which emerge out as weak in your evaluation.

1. A solid conducting sphere of radius a (a) 70.7% (b) 29.3%


has a net positive charge 2Q. A (c) 9.7% (d) 14.6%
conducting spherical shell of inner radius
b and outer radius c is concentric with 3. In the figure, the net electric flux
r r
the solid sphere and has a net charge – a through the area A is f = E × A Q
Q.The surface charge density on the b
inner and outer surfaces of the spherical when the system is in air. On
c
shell will be respectively immersing the system in water the A

net electric flux through the area


2Q Q Q Q
(a) - 2
,
2 (b) - , (a) becomes zero (b) remains same
4 pb 4 pc 4 pb 4 pc 2
2
(c) increases (d) decreases
Q Q
(c) 0, (d) ,0 4. An electric dipole is placed in a uniform electric field. The
4pc 2 4pc 2 dipole will experience
2. The surface charge density of a thin charged disc of radius (a) a force that will displace it in the direction of the field
R is s. The value of the electric field at the centre of the disc (b) a force that will displace it in a direction opposite to the
s field.
is . With respect to the field at the centre, the electric
2 Î0 (c) a torque which will rotate it without displacement
field along the axis at a distance R from the centre of the disc (d) a torque which will rotate it and a force that will displace it
reduces by

RESPONSE GRID 1. 2. 3. 4.
Space for Rough Work
EBD_7504
P-58 NTA JEE Main

5. Figure shows two charges of equal magnitude separated by 9. A charge q is placed at the centre of the q
a distance 2a. As we move away from the charge situated at open end of a cylindrical vessel. The flux
x = 0 to the charge situated at x = 2a, which of the following of the electric field through the surface of
graphs shows the correct behaviour of electric field ? the vessel is
+q +q (a) zero (b) q/eo (c) q/2eo (d) 2q/eo
x=a 10. A short electric dipole of –q O +q
x=0 x = 2a r a
(a) y (b) y dipole moment p is placed C
® ®
E E at a distance r from the
centre of a solid metallic
O
a
x O a 2a
x sphere of radius a (<< r) as
2a
shown in the figure. The r
electric field intensity at the centre of sphere C due to induced
charge on the sphere is
y y
(c) ® (d) ® 1 2p
E
E (a) zero (b) along CO
4pe0 r 3
O x
a 2a 1 2p 1 p
O
a 2a
x
(c) along OC (d) along CO
4pe 0 r 3 4pe0 r 3
11. A semi-circular arc of radius ‘a’ is charged uniformly and the
6. An uniform electric field E exists along positive x-axis. The charge per unit length is l. The electric field at the centre of
work done in moving a charge 0.5 C through a distance 2 m this arc is
along a direction making an angle 60° with x-axis is 10 J.
Then the magnitude of electric field is l l l l2
(a) 2pe a (b) 2 pe a 2 (c) 2 (d)
4p e 0 a 2pe0 a
(a) 5 Vm–1 (b) 2 Vm–1 (c) 5 Vm –1 (d) 20 Vm–1 0 0

7. An electric dipole is placed along the x-axis at the origin O. 12. A hollow cylinder has a charge q coulomb within it. If f is
A point P is at a distance of 20 cm from this origin such that the electric flux in units of voltmeter associated with the
OP makes an angle p/3 with the x-axis. If the electric field at curved surface B, the flux linked with the plane surface A in
P makes an angle q with the x-axis, the value of q would be units of voltmeter will be B
p p æ 3ö q
(a)
3
(b)
3
+ tan -1 ç ÷
ç 2 ÷ (a) 2e (b) f C A
è ø 0 3
æ 3ö
(c)
2p
(d) tan -1 çç ÷ q 1æ q ö
÷ (c) e - f (d) 2 ç e - f÷
3 è 2 ø 0 è 0 ø
8. A spherically symmetric charge distribution is characterised 13. A point Q lies on the perpendicular bisector of an electrical
by a charge density having the following variations:
dipole of dipole moment p. If the distance of Q from the
æ rö dipole is r (much larger than the size of the dipole), then the
r ( r ) = ro ç1 - ÷ for r < R r(r) = 0 for r ³ R electric field at Q is proportional to
è Rø
Where r is the distance from the centre of the charge (a) p–1 and r–2 (b) p and r –2 (c) p2 and r –3 (d) p and r –3
distribution ro is a constant. The electric field at an internal 14. A solid sphere of radius R has a charge Q distributed in its
point (r < R) is: volume with a charge density r = k r a, where k and a are
constants and r is the distance from its centre.
ro æ r r 2 ö ro æ r r2 ö
(a) ç - ÷ (b) ç - ÷
4e o è 3 4R ø eo è 3 4R ø If the electric field at r = R is 1 times that at r = R, find the
2 8
ro æ r r 2 ö ro æ r r 2 ö value of a
- -
(c) 3e o çè 3 4R ÷ø (d) 12e o çè 3 4R ÷ø (a) 2 (b) 3 (c) 5 (d) 6

RESPONSE 5. 6. 7. 8. 9.
GRID 10. 11. 12. 13. 14.
Space for Rough Work
Physics P-59

15. The spatial distribution of electric field due to charges (A, B) is 19. An oil drop of radius r and density r is held stationary in a
shown in figure. Which one of the following statements is uniform vertically upwards electric field 'E'. If r0 (< r) is the
correct ? density of air and e is quanta of charge, then the drop has–
4pr3 (r - r0 ) g
(a) excess electrons
3eE
A B
4 p r 2 (r - r0 ) g
(b) excess electrons
eE

(a) A is +ve and B –ve, |A| > |B| 4pr3 (r - r0 ) g


(c) deficiency of electrons
(b) A is –ve and B +ve, |A| = |B| 3eE
(c) Both are +ve but A > B
(d) Both are –ve but A > B 4 pr 2 (r - r0 ) g
(d) deficiency of electrons
16. A small sphere carrying a charge ‘q’ is hanging in between eE
two parallel plates by a string of length L. Time period of r
pendulum is T0. When parallel plates are charged, the time ( )
20. A surface has the area vector A = 2i$ + 3 $j m 2 . The flux of an
period changes to T. The ratio T/T0 is equal to
r V
electric field through it if the field is E = 4$i :
L m
m
(a) 8 V-m (b) 12 V-m (c) 20 V-m (d) zero
21. Two small similar metal spheres A and B having charges 4q
and – 4q, when placed at a certain distance apart, exert an
1/ 2
æ g
3/ 2
ö electric force F on each other. When another identical
æ qE ö
(a) ç g + m ÷ (b) ç qE ÷ uncharged sphere C, first touched with A then with B and
ç ÷ çg+ ÷ then removed to infinity, the force of interaction between A
è g ø è m ø and B for the same separation will be
1/ 2 (a) F/2 (b) F/8 (c) F/16 (d) F/32
æ g ö
(c) ç qE ÷ (d) None of these 22. A square surface of side L meter in the
çg+ ÷ plane of the paper is placed in a uniform E

è m ø electric field E (volt/m) acting along the


r q
17. In a medium of dielectric constant K, the electric field is E . If same plane at an angle q with the
Î0 is permittivity of the free space, the electric displacement horizontal side of the square as shown in
vector is Figure. The electric flux linked to the
r r r surface, in units of volt. m, is
KE E Î0 E r
(a) (b) (c) (d) K Î0 E (a) EL2 (b) EL2 cos q
(c) EL2 sin q (d) zero
Î0 K Î 0 K Y
ur ur
-q
18. Three charges –q1 , +q2 and –q3 are 3
23. Two point dipoles of dipole moment p1 and p 2 are at a
placed as shown in the figure. The x - ur ur
component of the force on –q 1 is
a distance x from each other and p1 || p 2 . The force between
b
proportional to the dipoles is :
-q 1
+q X 2

q2 q3 q2 q3 1 4 p1 p2 1 3 p1 p2
(a) 2 - 2 cos q (b) + 2 sin q (a) (b)
b a b2 4pe0 x 4 4pe0 x3
a
q q q2 q3 1 6 p1 p2 1 8 p1 p2
(c) 2 + 3 cos q (d) - 2 sin q (c) (d)
b 2
a 2 b2 a 4pe0 x 4 4pe0 x 4

RESPONSE 15. 16. 17. 18. 19.


GRID 20. 21. 22. 23.
Space for Rough Work
EBD_7504
P-60 NTA JEE Main

24. In a uniformly charged sphere of total charge Q and radius 27. Which of the following is a wrong statement?
R, the electric field E is plotted as function of distance from (a) The charge of an isolated system is conserved
the centre. The graph which would correspond to the above (b) It is not possible to create or destroy charged particles
will be:
(c) It is possible to create or destroy charged particles
(d) It is not possible to create or destroy net charge
E E 28. A square surface of side L metres is in
(a) (b) the plane of the paper. A uniform
uur E
electric field E (volt /m), also in the
r r
plane of the paper, is
E E
limited only to the lower half of the square surface (see figure).
The electric flux in SI units associated with the surface is
(c) (d) (a) EL2/2 (b) zero (c) EL2 (d) EL2 / (2e0)
29. A thin semi-circular ring of radius r has a
r j
r positive charge q distributed uniformly
25. A particle of mass m and charge q is placed at rest in a ur
uniform electric field E and then released. The kinetic energy over it. The net electric field E at the centre
attained by the particle after moving a distance y is O is i
(a) qEy2 (b) qE2 y (c) qEy (d) q2 Ey q q O
ˆj ˆ
26. A positive charge +Q is fixed at a B (a) (b) - 2 2 j
point A. Another positively u
4p 2 e 0 r 2 4p e 0 r
charged particle of mass m and q ˆ q ˆj
charge +q is projected from a point
d
(c) - 2 2 j (d)
2p e 0 r 2p e 0 r 2
2
B with velocity u as shown in the 30. Two positive ions, each carrying a charge q, are separated
figure. The point B is at the large A C
by a distance d. If F is the force of repulsion between the
distance from A and at distance d +Q
ions, the number of electrons missing from each ion will be (e
from the line AC. The initial velocity is parallel to the line AC.
The point C is at very large distance from A. The minimum being the charge of an electron)
distance (in meter) of +q from +Q during the motion is
4pe 0 Fd 2 4pe0 Fe 2
d (1 + A) . Find the value of A. [Take Qq = 4pe0 mu2d and (a) (b)
e2 d2
d = ( 2 - 1) meter.]
(a) 3 (b) 2 (c) 4 (d) 5 4pe0 Fd 2 4pe 0 Fd 2
(c) (d)
e2 q2

RESPONSE 24. 25. 26. 27. 28.


GRID 29. 30.

PHYSICS CHAPTERWISE SPEED TEST-15


Total Questions 30 Total Marks 120
Attempted Correct
Incorrect Net Score
Cut-off Score 45 Qualifying Score 60
Success Gap = Net Score – Qualifying Score
Net Score = (Correct × 4) – (Incorrect × 1)
Space for Rough Work

Space for Rough Work


PHYSICS Speed
Electrostatic Potential & Capacitance TEST
No. of Questions
30
Maximum Marks
120
Time
1 Hour
16
Chapter-wise

GENERAL INSTRUCTIONS
• This test contains 30 MCQ's. For each question only one option is correct. Darken the correct circle/ bubble in the
Response Grid provided on each page.
• You have to evaluate your Response Grids yourself with the help of solutions provided at the end of this book.
• Each correct answer will get you 4 marks and 1 mark shall be deduced for each incorrect answer. No mark will be given/
deducted if no bubble is filled. Keep a timer in front of you and stop immediately at the end of 60 min.
• The sheet follows a particular syllabus. Do not attempt the sheet before you have completed your preparation for that
syllabus.
• After completing the sheet check your answers with the solution booklet and complete the Result Grid. Finally spend time
to analyse your performance and revise the areas which emerge out as weak in your evaluation.

1. If n drops, each charged to a potential V, coalesce to form a


single drop. The potential of the big drop will be
V V d K (c)
(a) 2/3 (b) 1/ 3 (c) Vn1/3 (d) Vn2/3
n n
Cc
2. The electric potential V(x) in a region around the origin is
given by V(x) = 4x2 volts. The electric charge enclosed in a (a) both Cb, Cc > Ca (b) Cc > Ca while Cb > Ca
cube of 1 m side with its centre at the origin is (in coulomb) (c) both Cb, Cc < Ca (d) Ca = Cb = Cc
(a) 8e0 (b) – 4e0 (c) 0 (d) – 8e0 4. The gap between the plates of a parallel plate capacitor of
3. The capacitance of a parallel plate capacitor is Ca (Fig. a). A area A and distance between plates d, is filled with a dielectric
dielectric of dielectric constant K is inserted as shown in fig whose permittivity varies linearly from Î1 at one plate to Î2
(b) and (c). If Cb and Cc denote the capacitances in fig (b) at the other. The capacitance of capacitor is:
and (c), then (a) Î0(Î1 + Î2)A/d
d/2 (b) Î0(Î2 + Î1)A/2d
d
K (c) Î0A/[dln(Î2/Î1)]
(a)
Ca Cb
(b) (d) Î0(Î2 – Î1)A/[dln(Î2/Î1)]

RESPONSE GRID 1. 2. 3. 4.
Space for Rough Work
EBD_7504
P-62 NTA JEE Main

5. Figure (i) shows two capacitors connected in series and 9. A, B and C are three points
A
connected by a battery. The graph (ii) shows the variation in a uniform electric field. B ®
The electric potential is E
of potential as one moves from left to right on the branch
AB containing the capacitors. Then (a) maximum at B C
(b) maximum at C
C1 C2
V (c) same at all the three points A, B and C
A B
(d) maximum at A
10. A unit charge moves on an equipotential surface from a
point A to point B, then
E (a) VA – VB = + ve (b) VA – VB = 0
(i) (ii )
(c) VA – VB = – ve (d) it is stationary
(a) C1 = C2 (b) C1 < C2 11. Identify the false statement.
(c) C1 > C2 (a) Inside a charged or neutral conductor, electrostatic
(d) C1 and C2 cannot be compared field is zero
6. In a hollow spherical shell, potential (V) changes with respect (b) The electrostatic field at the surface of the charged
conductor must be tangential to the surface at any
to distance (s) from centre as point
(a) (b) (c) There is no net charge at any point inside the
conductor
(d) Electrostatic potential is constant throughout the
V V
volume of the conductor
12. A, B, C, D, E, F are conduct- A B C D E F
ing plates each of area A and
S S
any two consecutive plates
(c) (d) separated by a distance d.
The net energy stored in the
V system after the switch S is
V
closed is 180V
S

3e0 A 2 5e 0 A 2
S S (a) V (b) V
2d 12d
7. What is the effective capacitance between points X and Y?
e0 A 2 e0 A 2
C1=6m F (c) V (d) V
2d d
C3 =6m F C5=20m F C2=6m F 13. A parallel plate capacitor is made of two plates
X Y of length l, width w and separated by distance
A C B D
C4=6m F d. A dielectric slab (dielectric constant K) that x
fits exactly between the plates is held near
(a) 24 mF (b) 18 mF (c) 12 mF (d) 6 mF the edge of the plates. It is pulled into the l
8. Charges +q and –q are placed at points
A and B respectively which are a ¶U
R capacitor by a force F = - where U is the d
distance 2L apart, C is the midpoint ¶x
between A and B. The work done in energy of the capacitor when dielectric is inside the
moving a charge +Q along the capacitor up to distance x (See figure). If the charge on the
semicircle CRD is capacitor is Q then the force on the dielectric when it is near
A C B D the edge is:
qQ qQ Q 2d Q 2w
(a) 2pe0 L (b) 6pe 0 L (a) K (b) ( K - 1)
2wl2 eo 2dl 2e 0
qQ qQ Q2w
- Q 2d
(c) 6pe 0 L (d) 4pe0 L (c) ( K - 1) (d)
2dl 2 e o
K
2wl 2 e o

RESPONSE 5. 6. 7. 8. 9.
GRID 10. 11. 12. 13.
Space for Rough Work
Physics P-63

14. Two concentric, thin metallic spheres of radii R1 and R2 19. Two thin wire rings each having a radius R are placed at a
(R1 > R2) bear charges Q1 and Q2 respectively. Then the distance d apart with their axes coinciding. The charges on
the two rings are +q and –q. The potential difference
potential at distance r between R 1 and R 2 will be between the centres of the two rings is
æ 1 ö q é1 1 ù
çè k = 4pe ÷ø (a) ê - ú
0 2p Î0 êë R R 2 + d 2 úû
æ Q + Q2 ö æQ Q ö qR
(a) kç 1 ÷ø (b) kç 1 + 2 ÷ (b)
è r è r R2 ø 4p Î0 d 2
æQ Q ö æQ Q ö q é1 1 ù
(c) kç 2 + 1÷ (d) kç 1 + 2 ÷ (c) ê - ú
è r R1 ø è R1 R 2 ø 4p Î0 êë R R 2 + d 2 úû
15. An alpha particle is accelerated through a potential difference (d) zero
of 106 volt. Its kinetic energy will be 20. Two spherical conductors A and B of radii a b B
(a) 1 MeV (b) 2 MeV (c) 4 MeV (d) 8 MeV and b (b>a) are placed concentrically in air. A
16. The space between the plates of a parallel plate capacitor is The two are connected by a copper wire as a
filled with a ‘dielectric’ whose ‘dielectric constant’ varies shown in figure. Then the equivalent
with distance as per the relation: capacitance of the system is
K(x) = Ko + lx (l = a constant)
The capacitance C, of the capacitor, would be related to its ab
(a) 4pe 0 (b) 4pe0(a + b)
vacuum capacitance Co for the relation : b-a
(c) 4pe0b (d) 4pe0a
ld l
(a) C = Co (b) C = Co 21. 1 2
ln (1 + K o ld ) d.l n (1 + K o ld )
ld l
(c) C = Co (d) C = Co
ln (1 + ld / K o ) d.ln (1 + K o / ld ) V
2m F 8m F
17. In the circuit given below, the charge in mC, on the capacitor
having 5 mF is
2mF A capacitor of 2mF is charged as shown in the diagram.
3mF When the switch S is turned to position 2, the percentage of
e d
its stored energy dissipated is :
f
5mF
c
(a) 0% (b) 20% (c) 75% (d) 80%
4mF
22. A non-conducting disc of radius ‘a’ and uniform surface charge
a + b density s is placed on the ground, with its axis vertical. A
6V particle of mass m and positive charge q is dropped, along the
(a) 4.5 (b) 9 (c) 7 (d) 15 axis of the disc, from a height H with zero initial velocity. The
18. For the circuit shown in figure, which of the following particle has q/m = 4pe0g/s. Electrostatic potential at H is
statements is true? s
(a) e éë(a + H ) - H ùû
2 2 1/2
V1= 30V V2 = 20V
S1 S2 S3 0
s é 2
(b) (a + H 2 )1/2 + H ùû
C1 = 2pF C2= 3pF
e0 ë
s é 2
(a) With S1 closed V1 = 15V, V2 = 20V (c) (a + H 2 )1/2 - H ùû
2e 0 ë
(b) With S3 closed V1 = V2 = 25 V
s é 2
(c) With S1 and S2 closed V1 = V2 = 0 (d) (a + H 2 )1/2 + H ùû
(d) With S1 and S3 closed, V1 = 30 V, V2 = 20 V 2e 0 ë

RESPONSE 14. 15. 16. 17. 18.


GRID 19. 20. 21. 22.
Space for Rough Work
EBD_7504
P-64 NTA JEE Main

23. Which of the following figure shows the correct


equipotential surfaces of a system of two positive charges? C1
C2

(a) + + (b) + + A B
C3
C4

(c) + + (d) + + (a) 3 mF (b) 6 mF (c) 4.5 mF (c) 2.5 mF


28. A parallel plate air capacitor is charged to a potential
2 A difference of V volts. After disconnecting the charging
24. The capacitance of the capacitor of plate A1 battery the distance between the plates of the capacitor is
areas A1 and A2 (A1 < A2) at a distance d, increased using an insulating handle. As a result the
as shown in figure is potential difference between the plates
Î0 (A1 + A 2 ) Î0 A 2 (a) does not change (b) becomes zero
(a) (b) (c) increases (d) decreases
2d d
+Q
Î 29. Figure shows three circular arcs,
0 A1
(c) Î0 A1A 2 (d) each of radius R and total charge as
d d d 45°
indicated. The net electric potential
25. Two identical metal plates are given positive charges Q1 –2Q
30°

and Q2 ( < Q1) respectively. If they are now brought close at the centre of curvature is
R
together to form a parallel plate capacitor with capacitance Q Q
+3Q
C, the potential difference between them is (a) 2pe R (b) 4pe R
0 0
Q1 + Q 2 Q1 + Q 2
(a) (b) 2Q Q
2C C (c) (d)
Q1 – Q 2 Q1 – Q 2 pe0 R pe0 R
(c) (d) 30. A fully charged capacitor has a capacitance ‘C’. It
C 2C
26. A positively charged particle is released from rest in an uniform is discharged through a small coil of resistance wire
electric field. The electric potential energy of the charge embedded in a thermally insulated block of specific heat
capacity ‘s’ and mass ‘m’. If the temperature of the block is
(a) remains a constant because the electric field is uniform raised by ‘DT’, the potential difference ‘V’ across the
(b) increases because the charge moves along the electric field capacitance is
(c) decreases because the charge moves along the electric
field mC DT 2mC DT
(a) (b)
(d) decreases because the charge moves opposite to the s s
electric field 2ms DT ms DT
27. In a given network the equivalent capacitance between A (c) (d)
and B is [C1 = C4 = 1 mF, C2 = C3 = 2mF] C C

RESPONSE 23. 24. 25. 26. 27.


GRID 28. 29. 30.

PHYSICS CHAPTERWISE SPEED TEST-16


Total Questions 30 Total Marks 120
Attempted Correct
Incorrect Net Score
Cut-off Score 45 Qualifying Score 60
Success Gap = Net Score Space
– Qualifying
for Rough Work
Score
Net Score = (Correct × 4) – (Incorrect × 1)
Space for Rough Work
PHYSICS Speed
Current Electricity TEST
No. of Questions
30
Maximum Marks
120
Time
1 Hour
17
Chapter-wise

GENERAL INSTRUCTIONS
• This test contains 30 MCQ's. For each question only one option is correct. Darken the correct circle/ bubble in the
Response Grid provided on each page.
• You have to evaluate your Response Grids yourself with the help of solutions provided at the end of this book.
• Each correct answer will get you 4 marks and 1 mark shall be deduced for each incorrect answer. No mark will be given/
deducted if no bubble is filled. Keep a timer in front of you and stop immediately at the end of 60 min.
• The sheet follows a particular syllabus. Do not attempt the sheet before you have completed your preparation for that
syllabus.
• After completing the sheet check your answers with the solution booklet and complete the Result Grid. Finally spend time
to analyse your performance and revise the areas which emerge out as weak in your evaluation.

1. Variation of current passing through a conductor as the (a) 1 W


voltage applied across its ends is varied as shown in the
adjoining diagram. If the resistance (R) is determined at the
(b) 2 W
points A, B, C and D, we will find that

(a) RC = RD V (c) 3 W
C D
(b) RB > RA B (d) 4 W
A
(c) RC > RB 3. The masses of the three wires of copper are in the ratio of
1 : 3 : 5 and their lengths are in the ratio of 5 : 3 : 1. The ratio
(d) RA > RB i of their electrical resistance is
2. Twelve resistors each of resistance 16 W are connected in (a) 1 : 3 : 5 (b) 5 : 3 : 1
the circuit as shown. The net resistance between AB is
(c) 1 : 25 : 125 (d) 125 : 25 : 1

RESPONSE GRID 1. 2. 3.

Space for Rough Work


EBD_7504
P-66 NTA JEE Main

4. A cylindrical solid of length L and radius a is having varying (c) When the coil is divided into four equal parts and all
resistivity given by r = r0x, where r0 is a positive constant the four parts are connected to the battery in parallel
and x is measured from left end of solid. The cell shown in (d) When only half the coil is connected to the battery
the figure is having emf V and negligible internal resistance. 9. There is an infinite wire grid with cells in the form of
The electric field as a function of x is best described by equilateral triangles. The resistance of each wire between
r = r 0x neighbouring joint connections is R0. The net resistance of
the whole grid between the points A and B as shown is

x
A B
V
2V 2V R0 R0
(a) x (b) x R0
2 r0 L2 (a) R0 (b) (c) (d)
L 2 3 4
V 10. A potentiometer circuit is set up as shown. The potential
(c) x (d) None of these gradient, across the potentiometer wire, is k volt/cm and the
L2 ammeter, present in the circuit, reads 1.0 A when two way
5. All batteries are having emf 10 volt and internal resistance key is switched off. The balance points, when the key
negligible. All resistors are in ohms. Calculate the current in between the terminals (i) 1 and 2 (ii) 1 and 3, is plugged in,
the right most 2W resistor. are found to be at lengths l1 cm and l2 cm respectively. The
2 magnitudes, of the resistors R and X, in ohms, are then,
10 equal, respectively, to + –
()
2 2 10 2 2
(a) k (l2 – l1) and k l2
10 A 1 B
10 (b) k l1 and k (l2 – l1) G
2
3
2 10 (c) k (l2 – l1) and k l1
– R X
A
25 25 12 6 (d) k l1 and k l2 +
(a) A (b) A (c) A (d) A ()
12 6 25 25
6. n equal resistors are first connected in series and then 11. If voltage across a bulb rated 220 Volt-100 Watt drops by
connected in parallel. What is the ratio of the maximum to 2.5% of its rated value, the percentage of the rated value by
the minimum resistance ? which the power would decrease is :
(a) n (b) 1/n2 (c) n2 (d) 1/ n (a) 20% (b) 2.5% (c) 5% (d) 10%
7. A battery is charged at a potential of 15V for 8 hours when 12. The resistance of the coil of an ammeter is R. The shunt
the current flowing is 10A. The battery on discharge required to increase its range n-fold should have a resistance
supplies a current of 5A for 15 hours. The mean terminal
voltage during discharge is 14V. The “watt-hour” efficiency R R R
(a) (b) (c) (d) nR
of the battery is n n -1 n +1
(a) 87.5% (b) 82.5% (c) 80% (d) 90% 13. A conducting wire of cross-sectional area 1 cm 2 has
8. You are given a resistance coil and a battery. In which of 3 × 1023 charge carriers per m3. If wire carries a current of 24
the following cases is largest amount of heat generated ? mA, then drift velocity of carriers is
(a) When the coil is connected to the battery directly (a) 5 × 10–2 m/s (b) 0.5 m/s
(b) When the coil is divided into two equal parts and both (c) 5 × 10–3 m/s (d) 5 × 10–6 m/s
the parts are connected to the battery in parallel

RESPONSE 4. 5. 6. 7. 8.
GRID 9. 10. 11. 12. 13.
Space for Rough Work
Physics P-67

14. The figure shows a meter-bridge circuit, X = 12W and (c) the resistance will be halved and the specific resistance
R = 18W. The jockey J is at the null point. If R is made 8W, will remain unchanged
through the jockey J have to be moved by 4 × A cm to obtain (d) the resistance will be halved and the specific resistance
null point again then find the value of A. will be doubled
+ – 20 The length of a wire of a potentiometer is 100 cm, and the e.
m.f. of its standard cell is E volt. It is employed to measure
X R
the e.m.f. of a battery whose internal resistance is 0.5W. If
the balance point is obtained at l = 30 cm from the positive
end, the e.m.f. of the battery is
A J B
30 E 30 E
(a) (b) 100 - 0.5
(a) 2 (b) 5 (c) 8 (d) 6 100.5 ( )
15. In the given circuit diagram the current through the battery 30 ( E - 0.5i ) 30 E
and the charge on the capacitor respectively in steady state (c) (d)
100 100
are 6V I
where i is the current in the potentiometer wire.
(a) 1 A and 3 µC 1W 21. If current flowing in a conductor changes by 1% then power
consumed will change by
(b) 17 A and 0 µC 2W (a) 10% (b) 2% (c) 1% (d) 100%
22. The total current supplied to the circuit by the battery is
6 12 3W
(c) A and µC (a) 4 A
7 7 6V
2W
6W
(b) 2 A 3W

(d) 11A and 3 µC 0.5 µF4W 1.5W


16. The length of a given cylindrical wire is increased by 100%. (c) 1 A
Due to the consequent decrease in diameter the change in (d) 6 A
the resistance of the wire will be 23. The e.m.f. developed in a thermo-couple is given by
(a) 200% (b) 100% (c) 50% (d) 300%
17. Drift velocity Vd varies with the intensity of electric field as 1
E = a T + b T2
per the relation 2
where T is the temperature of hot junction, cold junction
1 being at 0ºC. The thermo electric power of the couple is
(a) Vd µ E (b) Vd µ
E
b
(c) Vd = constant (d) Vd µ E2 (a) a+ T (b) a +bT
18. Product of conductivity and resistivity of a metallic 2
conductor depends on a T 2 b T3
(a) Area of cross-section (b) Temperature (c) + (d) a / 2 b
2 6
(c) Pressure (d) None of these 24. To get maximum current in a resistance of 3 ohms, one can
19. The electric resistance of a certain wire of iron is R. If its use n rows of m cells (connected in series) connected in
length and radius are both doubled, then parallel. If the total number of cells is 24 and the internal
(a) the resistance and the specific resistance, will both resistance of a cell is 0.5 ohms then
remain unchanged
(a) m = 12, n = 2 (b) m = 8, n = 3
(b) the resistance will be doubled and the specific
(c) m = 2, n = 12 (d) m = 6, n = 4
resistance will be halved

14. 15. 16. 17. 18.


RESPONSE
19. 20. 21. 22. 23.
GRID
24.
Space for Rough Work
EBD_7504
P-68 NTA JEE Main

25. See the electric circuit shown in the figure. (a) Bulb 1 will glow more first and then its brightness will
Which of the following R become less than bulb 2
equations is a correct equation (b) Bulb 1
for it? (c) Bulb 2
i1 e 1 r1
(a) e2 – i2 r2 – e1 – i1 r1 = 0 (d) Both glow equally
(b) - e2 – (i1 + i2) R+ i2 r2 = 0 28. A car battery has e.m.f. 12 volt and internal resistance 5 × 10–2
(c) e1 – (i1 + i2) R + i1 r1 = 0 i2 ohm. If it draws 60 amp current, the terminal voltage of the
r e battery will be
(d) e1 – (i1 + i2) R– i1 r1 = 0 2 2

(a) 15 volt (b) 3 volt (c) 5 volt (d) 9 volt\


26. In a neon gas discharge tube Ne+ ions moving through a
29. Two rods are joined end to end, as shown. Both have a
cross-section of the tube each second to the right is 2.9 ×
cross-sectional area of 0.01 cm2. Each is 1 meter long. One
1018, while 1.2 × 1018 electrons move towards left in the same
rod is of copper with a resistivity of 1.7 × 10–6 ohm-centimeter,
time; the electronic charge being 1.6 × 10–19 C, the net electric
the other is of iron with a resistivity of 10 –5 ohm-centimeter.
current is
(a) 0.27 A to the right (b) 0.66 A to the right How much voltage is required to produce a current of 1
(c) 0.66 A to the left (d) zero ampere in the rods? V

27. A 6.0 volt battery is connected to two light bulbs as shown (a) 0.117 V
in figure. Light bulb 1 has resistance 3 ohm while light bulb (b) 0.00145 V
2 has resistance 6 ohm. Battery has negligible internal (c) 0.0145 V
resistance. Which bulb will glow brighter? (d) 1.7 × 10–6 V Cu Fe
Bulb 1 30. Two sources of equal emf are connected to an external
resistance R. The internal resistance of the two sources are
R1and R2 (R1 > R1). If the potential difference across the
Bulb 2 source having internal resistance R 2 is zero, then
(a) R = R2 - R1
(b) R = R2 ´ ( R1 + R2 ) /( R2 - R1 )
+ – (c) R = R1R2 /( R2 - R1 )
(d) R = R1R2 /( R1 - R2 )
6.0 V

RESPONSE 25. 26. 27. 28. 29.


GRID 30.

PHYSICS CHAPTERWISE SPEED TEST-17


Total Questions 30 Total Marks 120
Attempted Correct
Incorrect Net Score
Cut-off Score 40 Qualifying Score 50
Success Gap = Net Score – Qualifying Score
Net Score = (Correct × 4) – (Incorrect × 1)
Space for Rough Work
PHYSICS Speed
Moving Charges and Magnetism TEST
No. of Questions
30
Maximum Marks
120
Time
1 Hour
18
Chapter-wise

GENERAL INSTRUCTIONS
• This test contains 30 MCQ's. For each question only one option is correct. Darken the correct circle/ bubble in the
Response Grid provided on each page.
• You have to evaluate your Response Grids yourself with the help of solutions provided at the end of this book.
• Each correct answer will get you 4 marks and 1 mark shall be deduced for each incorrect answer. No mark will be given/
deducted if no bubble is filled. Keep a timer in front of you and stop immediately at the end of 60 min.
• The sheet follows a particular syllabus. Do not attempt the sheet before you have completed your preparation for that
syllabus.
• After completing the sheet check your answers with the solution booklet and complete the Result Grid. Finally spend time
to analyse your performance and revise the areas which emerge out as weak in your evaluation.

1. A 10 eV electron is circulating in a plane at right angles to a 3. Two long parallel wires carry currents i1 andn i2 such that
uniform field at magnetic induction 10–4 Wb/m2 (= 1.0 i1 > i2. When the currents are in the same direction, the
gauss). The orbital radius of the electron is magnetic field at a point midway between the wires is
(a) 12 cm (b) 16 cm 6 × 10–6 T. If the direction of i2 is reversed, the field becomes
(c) 11 cm (d) 18 cm i
3 × 10–5 T. The ratio of 1 is
2. An insulating rod of length l carries a charge q distributed i2
uniformly on it. The rod is pivoted at its mid point and is 1 2 3
rotated at a frequency f about a fixed axis perpendicular to (a) (b) 2 (c) (d)
2 3 2
rod and passing through the pivot. The magnetic moment 4. A moving coil galvanometer has 150 equal divisions. Its
1 current sensitivity is 10-divisions per milliampere and voltage
of the rod system is pqfl 2 . Find the value of a. sensitivity is 2 divisions per millivolt. In order that each
2a
division reads 1 volt, the resistance in ohms needed to be
(a) 6 (b) 4 connected in series with the coil will be
(c) 5 (d) 8 (a) 105 (b) 103 (c) 9995 (d) 99995

RESPONSE GRID 1. 2. 3. 4.

Space for Rough Work


EBD_7504
P-70 NTA JEE Main

5. A wire carrying current I has the shape as shown in adjoining 10. In the adjoining figure, two very long, parallel wires A and B
figure. Linear parts of the wire are very long and parallel to carry currents of 10 ampere and 20 ampere respectively, and
X-axis while semicircular portion of radius R is lying in Y-Z are at a distance 20 cm apart. If a third wire C (length 15 cm)
plane. Magnetic field at point O is : having a current of 10 ampere is placed between them, then
Z
ur how much force will act on C ? The direction of current in all
(a) B = –
m0 I $
4p R
( m i ´ 2k$
) the three wires is same. A C B

ur (a) 3 × 10–5 N (left)


(b) B = – 0
m I $
4p R
( p i + 2k$ ) R
I
(b) 3 × 10–5 N (right)
ur m 0 I
Y (c) 6 × 10–5 N (left)
( )
O
(c) B = p $i – 2k$ (d) 6 × 10–5 N (right) 20 cm
4p R
ur m 0 I I 11. Two tangent galvanometers A and B have coils of radii 8 cm
(d) B =
4p R
( p $i + 2k$ ) X
and 16 cm respectively and having resistance of 8W each.
They are connected in parallel with a cell of emf 4V and
6. A closely wound solenoid of 2000 turns and area of cross- negligible internal resistance. The deflections produced in
section 1.5 × 10–4 m2 carries a current of 2.0 A. It suspended the tangent galvanometers A and B are 30° and 60°
through its centre and perpendicular to its length, allowing respectively. If A has 2 turns, then B must have
it to turn in a horizontal plane in a uniform magnetic field 5 × (a) 18 turns (b) 12 turns
10–2 tesla making an angle of 30° with the axis of the (c) 6 turns (d) 2 turns
solenoid. The torque on the solenoid will be: 12. A 2 µC charge moving around a circle with a frequency of
(a) 3 × 10–2 N-m (b) 3 × 10–3 N-m 6.25 × 1012 Hz produces a magnetic field 6.28 tesla at the
(c) 1.5 × 10 N-m–3 (d) 1.5 × 10–2 N-m centre of the circle. The radius of the circle is
7. An alternating electric field, of frequency v, is applied across (a) 2.25 m (b) 0.25 m (c) 13.0 m (d) 1.25 m
the dees (radius = R) of a cyclotron that is being used to 13. A current loop ABCD is held fixed on the plane of the paper
accelerate protons (mass = m). The operating magnetic field as shown in the figure. The arcs BC (radius = b) and DA
(B) used in the cyclotron and the kinetic energy (K) of the (radius = a) of the loop are joined by two straight wires AB
proton beam, produced by it, are given by : and CD. A steady current I is flowing in the loop. Angle
(a) B = mn and K = 2mp2n2R2
made by AB and CD at the origin O is 30°. Another straight
e thin wire with steady current I1 flowing out of the plane of
the paper is kept at the origin.
(b) B = pmn and K = m2pnR2
2
The magnitude of the magnetic field (B) due to the loop
e
ABCD at the origin (O) is :
(c) B = 2pm n and K = 2mp2n2R2 B
e m o I (b - a )
(a) a A
mn 24ab
(d) B = and K = m2pnR2
e mo I é b - a ù
8. A galvanometer of 50 ohm resistance has 25 divisions. A (b) I1 30° I
current of 4 × 10–4 ampere gives a deflection of one per 4p êë ab úû O
division. To convert this galvanometer into a voltmeter mo I
having a range of 25 volts, it should be connected with a [2(b - a) + p / 3(a + b)] D
(c)
resistance of 4p b C
(a) 2450 W in series (b) 2500 W in series. (d) zero
(c) 245 W in series. (d) 2550 W in series. 14. A galvanometer of resistance, G is shunted by a resistance
9. A long straight wire of radius a carries a steady current i. S ohm. To keep the main current in the circuit unchanged,
The current is uniformly distributed across its cross section. the resistance to be put in series with the galvanometer is
The ratio of the magnetic field at a/2 and 2a is S2 SG G2 G
(a) 1/2 (b) 1/4 (c) 4 (d) 1 (a) (b) (c) (d)
(S + G) (S + G) (S + G) (S + G)

RESPONSE 5. 6. 7. 8. 9.
GRID 10. 11. 12. 13. 14.
Space for Rough Work
Physics P-71

15. A closed loop PQRS carrying a current is placed in a uniform


é æ q ö 1/2 ù é æ q ö 1/2 ù
magnetic field. Q sin -1 ê Bd ç cos -1 ê Bd ç
(a) ÷ ú (b) ÷ ú
If the magnetic forces on segments PS, êë è 2mV ø úû êë è 2mV ø úû
SR, and RQ are F1 , F2 and F3 respectively
and are in the plane of the paper and along P é æ q ö 1/2 ù
the directions shown, the force on the (c) tan -1 ê Bd ç ú (d) zero
F3 è 2mV ÷ø ú
segment QP is F1 ëê û
(a) F3 – F1– F2 20. A long insulated copper wire is closely wound as a spiral of
‘N’ turns. The spiral has inner radius ‘a’ and outer radius
(b) (F3 – F1 ) 2
+ F22 S R ‘b’. The spiral lies in the XY plane and a steady current ‘I’
F2 flows through the wire. The Z-component of the magnetic
(c) (F3 – F1 )2 – F22 (d) F3 – F1+F2 field at the centre of the spiral is
16. A beam of electrons is moving with constant velocity in a m0 NI æbö
region having simultaneous perpendicular electric and (a) ln ç ÷
2(b - a) èaø
magnetic fields of strength 20 Vm–1 and 0.5 T respectively
at right angles to the direction of motion of the electrons.
m0 NI æb+ aö
Then the velocity of electrons must be (b) ln çè ÷
2(b - a) b-aø
(a) 8 m/s (b) 20 m/s (c) 40 m/s (d) 1 m / s
40 m0 NI æbö
17. A current I flows in an infinitely long wire with cross section (c) ln ç ÷
in the form of a semi-circular ring of radius R. The magnitude 2b èaø
of the magnetic induction along its axis is:
m0 NI æb+ aö
m0 I m0 I m0 I m0 I (d) ln çè ÷
(a) (b) (c) (d) 2b b-aø
2p 2 R 2p R 4p R p2 R
18. A very long straight wire carries a current I. At the instant 21. A particle of charge q and mass m moves in a circular orbit
r of radius r with angular speed w . The ratio of the magnitude
when a charge + Q at point P has velocity v , as shown, of its magnetic moment to that of its angular momentum
the force on the charge is depends on
Y (a) w and q (b) w , q and m
(c) q and m (d) w and m
22. A thin flexible wire of length L is connected to two adjacent
fixed points and carries a current I in the clockwise direction,
as shown in the figure. When the system is put in a uniform
O X
magnetic field of strength B going into the plane of the
paper, the wire takes the shape of a circle. The tension in the
wire is

(a) along OY (b) opposite to OY


(c) along OX (d) opposite to OX
19. A particle of mass m and charge q, accelerated by potential
difference V enters a region of uniform transverse magnetic
field B. If d is the thickness of region of magnetic field, then IBL IBL IBL
the deviation of the particle from initial direction when it (a) IBL (b) (c) (d)
p 2p 4p
leaves the field is

RESPONSE 15. 16. 17. 18. 19.


GRID 20. 21. 22.
Space for Rough Work
EBD_7504
P-72 NTA JEE Main

23. A battery is connected between two points A and B on the Given e = 1.60 × 10–19 C, m = 1.67 × 10–27 kg.
circumference of a uniform conducting ring of radius r and 1 MeV = 1.6 × 10–13 J)
resistance R. One of the arcs AB of the ring subtends an (a) 3.421 MeV (b) 4.421 MeV
angle q at the centre.The value of the magnetic induction at (c) 5.421 MeV (d) 7.421 MeV
the centre due to the current in the ring is 27. The deflection in a galvanometer falls from 50 division to 20
(a) proportional to 2 (180° – q) when a 12 ohm shunt is applied. The galvanometer resistance
(b) inversely proportional to r is
(a) 18 ohm (b) 36 ohm (c) 24 ohm (d) 30 ohm
(c) zero, only if q = 180°
28. When a long wire carrying a steady current is bent into a
(d) zero for all values of q circular coil of one turn, the magnetic induction at its centre
24. In a mass spectrometer used for measuring the masses of is B. When the same wire carrying the same current is bent
ions, the ions are initially accelerated by an electric potential to form a circular coil of n turns of a smaller radius, the
V and then made to describe semicircular path of radius R magnetic induction at the centre will be
using a magnetic field B. If V and B are kept constant, the (a) B/n (b) nB (c) B/n2 (d) n2B
29. A charged particle moves through a magnetic field
æ charge on the ion ö
ratio ç ÷ will be proportional to perpendicular to its direction. Then
è mass of the ion ø (a) kinetic energy changes but the momentum is constant
(a) 1/R2 (b) R2 (c) R (d) 1/R (b) the momentum changes but the kinetic energy is
25. A uniformly charged ring of radius 10 cm rotates at a constant
frequency of 104 rps about its axis. The ratio of energy (c) both momentum and kinetic energy of the particle are
density of electric field to the energy density of the magnetic not constant
field at a point on the axis at distance 20 cm from the centre (d) both momentum and kinetic energy of the particle are
is 9.1 × 10a . Find the value of a. constant
30. A charged particle is released from rest in a region of steady
(a) 7 (b) 9 (c) 8 (d) 6
and uniform electric and magnetic fields which are parallel
26. A cyclotron's oscillator frequency is 10 MHz. If the radius to each other. The particle will move in a
of its 'dees' is 60 cm, what is the kinetic energy of the proton (a) straight line (b) circle
beam produced by the accelerator ? (c) helix (d) cycloid

RESPONSE 23. 24. 25. 26. 27.


GRID 28. 29. 30.

PHYSICS CHAPTERWISE SPEED TEST-18


Total Questions 30 Total Marks 120
Attempted Correct
Incorrect Net Score
Cut-off Score 40 Qualifying Score 50
Success Gap = Net Score – Qualifying Score
Net Score = (Correct × 4) – (Incorrect × 1)
Space for Rough Work
PHYSICS Speed
Magnetism and Matter TEST
No. of Questions
30
Maximum Marks
120
Time
1 Hour
19
Chapter-wise

GENERAL INSTRUCTIONS
• This test contains 30 MCQ's. For each question only one option is correct. Darken the correct circle/ bubble in the
Response Grid provided on each page.
• You have to evaluate your Response Grids yourself with the help of solutions provided at the end of this book.
• Each correct answer will get you 4 marks and 1 mark shall be deduced for each incorrect answer. No mark will be given/
deducted if no bubble is filled. Keep a timer in front of you and stop immediately at the end of 60 min.
• The sheet follows a particular syllabus. Do not attempt the sheet before you have completed your preparation for that
syllabus.
• After completing the sheet check your answers with the solution booklet and complete the Result Grid. Finally spend time
to analyse your performance and revise the areas which emerge out as weak in your evaluation.

1. Two identical magnetic dipoles of magnetic moments 3. Relative permittivity and permeability of a material e r and
1.0 A-m2 each, placed at a separation of 2 m with their axis m r , respectively. Which of the following values of these
perpendicular to each other. The resultant magnetic field at quantities are allowed for a diamagnetic material?
point midway between the dipole is
(a) 5 × 10–7 T (b) 5 × 10–7 T (a) e r = 0.5, m r = 1.5 (b) e r = 1.5, mr = 0.5
–7
(c) 10 T (d) 2 × 10–7 T
(c) e r = 0.5, m r = 0.5 (d) e r = 1.5, mr = 1.5
2. Two identical thin bar magnets each of length l and pole
strength m are placed at right angles to each other, with 4. If the period of oscillation of freely suspended bar magnet
north pole of one touching south pole of the other, then the in earth’s horizontal field H is 4 sec. When another magnet
magnetic moment of the system is N1 is brought near it, the period of oscillation is reduced to 2s.
(a) 1 ml The magnetic field of second bar magnet is
(b) 2 ml (a) 4 H (b) 3 H
2
(c) 2 ml (c) 2 H (d) 3H
(d) ml/2 S1
N2 S2

RESPONSE GRID 1. 2. 3. 4.
Space for Rough Work
EBD_7504
P-74 NTA JEE Main

5. A coil in the shape of an equilateral triangle of side l is 10. A bar magnet 8 cms long is placed in the magnetic merdian
suspended between the pole pieces of a permanent magnet with the N-pole pointing towards geographical north. Two
®
such that B is in the plane of the coil. If due to a current i in netural points separated by a distance of 6 cms are obtained
the triangle a torque t acts on it, the side l of the triangle is on the equatorial axis of the magnet. If horizontal component
1 1 of earth’s field = 3.2 × 10–5 T, then pole strength of magnet
2 æ t ö2 æ t ö2 is
(a) ç ÷ (b) 2çç ÷÷
3 è B.i ø è 3B.i ø (a) 5 ab-amp × cm (b) 10 ab-amp × cm
t
(c) 2.5 ab-amp × cm (d) 20 ab-amp × cm
2 æ t ö 1
(c) ç ÷ (d) 11. If the susceptibility of dia, para and ferro magnetic materials
3 è B.i ø 3 B.i
are cd, cp, cf respectively, then
6. A compass needle whose magnetic moment is 60 Am2, is
(a) cd < cp < cf (b) cd < cf < cp
directed towards geographical north at any place
experiencing moment of force of 1.2 × 10–3 Nm. At that place (c) cf < cd < cp (d) cf < cp < cd
the horizontal component of earth field is 40 micro W/m2. 12. The figure shows the various positions (labelled by
What is the value of dip angle at that place? subscripts) of small magnetised needles P and Q. The arrows
show the direction of their magnetic moment. Which
(a) 30° (b) 60° (c) 45° (d) 15°
configuration corresponds to the lowest potential energy
7. The materials suitable for making electromagnets should
among all the configurations shown ?
have Q4
(a) high retentivity and low coercivity
(a) PQ3
(b) low retentivity and low coercivity
(c) high retentivity and high coercivity (b) PQ4 Q5 P
(d) low retentivity and high coercivity Q1 Q2
8. The length of a magnet is large compared to its width and (c) PQ5
breadth. The time period of its oscillation in a vibration
magnetometer is 2s. The magnet is cut along its length into Q6
(d) PQ6
three equal parts and these parts are then placed on each
other with their like poles together. The time period of this 13. A magnetic needle is kept in a non-uniform magnetic field. It
combination will be experiences
(a) neither a force nor a torque
2 2
(a) 2 3s (b) s (c) 2 s (d) s (b) a torque but not a force
3 3 (c) a force but not a torque
9. A magnetic dipole is under the influence of two magnetic
(d) a force and a torque
fields The angle between the field directions is 60° and one
of the fields has a magnitude of 1.2 × 10–2 T. If the dipole 14. The angle of dip at a place is 37° and the vertical component
comes to stable equilibrium at an angle of 15° with this field, of the earth’s magnetic field is 6 × 10–5T. The earth’s
what is the magnitude of other field ? magnetic field at this place is (tan 37° = 3/4)
(a) 4.4 × 10–3 tesla (b) 5.2 × 10–3 tesla (a) 7 × 10–5 T (b) 6 × 10–5 T
(c) 3.4 × 10–3 tesla (d) 7.8 × 10–3 tesla (c) 5 × 10–5 T (d) 10–4 T

RESPONSE 5. 6. 7. 8. 9.
GRID 10. 11. 12. 13. 14.
Space for Rough Work
Physics P-75

15. Needles N1, N2 and N3 are made of a ferromagnetic, a (b) For vacuum medium its value is one.
paramagnetic and a diamagnetic substance respectively. A (c) For ferromagnetic materials mr > > 1
magnet when brought close to them will (d) For paramagnetic materials mr < 1.
(a) attract N1 and N2 strongly but repel N3 20. A bar magnet of moment of inertia 9 × l0–5 kg m2 placed in a
(b) attract N1 strongly, N2 weakly and repel N3 weakly vibration magnetometer and oscillating in a uniform
(c) attract N1 strongly, but repel N2 and N3 weakly magnetic field l6p2 × l0–5T makes 20 oscillations in 15 s. The
(d) attract all three of them magnetic moment of the bar magnet is
16. A vibration magnetometer consists of two identical bar (a) 3 Am2 (b) 2 Am2 (c) 5 Am2 (d) 4 Am2
magnets placed one over the other such that they are 21. Two short bar magnets P and Q are arranged such that their
perpendicular and bisect each other. The time period of centres are on the X-axis and are separated by a large
oscillation in a horizontal magnetic field is 25/4 seconds. distance. The magnetic axes of P and Q are along X and Y
One of the magnets is removed and if the other magnet axes respectively. At a point R, midway between their centres,
oscillates in the same field, then the time period in seconds if B is the magnitude of induction due to Q, the magnitude
is of total induction at R due to the both magnets is
(a) 21/4 (b) 21/2 (c) 2 (d) 23/4
17. Two identical bar magnets are fixed with their centres at a 5
(a) 3B (b) 5B (c) B (d) B
distance d apart. A stationary charge Q is placed at P in 2
between the gap of the two magnets at a distance D from 22. A dip circle is so set that its needle moves freely in the
the centre O as shown in the Figure magnetic meridian. In this position, the angle of dip is 40º.
P Now the dip circle is rotated so that the plane in which the
D needle moves makes an angle of 30º with the magnetic
meridian. In this position, the needle will dip by an angle
(a) 40º (b) 30º
O (c) more than 40º (d) less than 40º
S N N S
23. A watch glass containing some powdered substance is
placed between the pole pieces of a magnet. Deep concavity
d
The force on the charge Q is is observed at the centre. The substance in the watch glass
is
(a) directed perpendicular to the plane of paper
(b) zero (a) iron (b) chromium
(c) directed along OP (c) carbon (d) wood
(d) directed along PO 24. Imagine rolling a sheet of paper into a cylinder and placing
a bar magnet near its end as shown in figure. What can you
18. The magnetic field of earth at the equator is approximately 4 r r r
× 10–5 T. The radius of earth is 6.4 × 10 6 m. Then the dipole say about the sign of B.dA for every area dA on the surface?
moment of the earth will be nearly of the order of:
(a) 1023 A m2 (b) 1020 A m2
(c) 1016 A m2 (d) 1010 A m2 (a) Positive
19. Which of the following is not correct about relative magnetic (b) Negative
permeability (mr)? (c) No sign
(a) It is a dimensionsless pure ratio. (d) Can be positive or negative

RESPONSE 15. 16. 17. 18. 19.


GRID 20. 21. 22. 23. 24.

Space for Rough Work


EBD_7504
P-76 NTA JEE Main

25. A magnetic needle lying parallel to a magnetic field requires 28. The coercivity of a small magnet where the ferromagnet
W units of work to turn it through 60°. The torque required gets demagnetized is 3 × 103 Am–1. The current required to
to keep the needle in this position will be be passed in a solenoid of length 10 cm and number of turns
100, so that the magnet gets demagnetized when inside the
W solenoid, is:
(a) 2W (b) W (c) (d) 3W
2 (a) 30 mA (b) 60 mA (c) 3 A (d) 6 A
26. The earth’s magnetic field lines resemble that of a dipole at 29. A thin rectangular magnet suspended freely has a period of
oscillation equal to T. Now it is broken into two equal halves
the centre of the earth. If the magnetic moment of this dipole
(each having half of the original length) and one piece is
is close to 8 × 1022 Am2, the value of earth’s magnetic field made to oscillate freely in the same field. If its period of
near the equator is close to (radius of the earth = 6.4 × 106 m)
T'
(a) 0.6 Gauss (b) 1.2 Gauss oscillation is T ' , the ratio is
T
(c) 1.8 Gauss (d) 0.32 Gauss
27. Two short bar magnets of length 1 cm each have magnetic 1 1 1
(a) (b) (c) 2 (d)
moments 1.20 Am2 and 1.00 Am2 respectively. They are 2 2 2 4
placed on a horizontal table parallel to each other with their 30. Which of the following statements is incorrect about
N poles pointing towards the South. They have a common hysteresis ?
magnetic equator and are separated by a distance of (a) This effect is common to all ferromagnetic substances
20.0 cm. The value of the resultand horizontal magnetic (b) The hysteresis loop area is proportional to the thermal
induction at the mid-point O of the line joining their centres energy developed per unit volume of the material
is close to (Horizontal component of earth’s magnetic
(c) The hysteresis loop area is independent of the thermal
induction is 3.6× 10–5 Wb/m2) energy developed per unit volume of the material
(a) 3.6 × 10–5 Wb/m2 (b) 2.56 × 10–4 Wb/m2 (d) The shape of the hysteresis loop is characteristic of
(c) 3.50 × 10–4 Wb/m2 (d) 5.80 × 10–4 Wb/m2 the material

RESPONSE 25. 26. 27. 28. 29.


GRID 30.

PHYSICS CHAPTERWISE SPEED TEST-19


Total Questions 30 Total Marks 120
Attempted Correct
Incorrect Net Score
Cut-off Score 45 Qualifying Score 60
Success Gap = Net Score – Qualifying Score
Net Score = (Correct × 4) – (Incorrect × 1)
Space for Rough Work
PHYSICS Speed
Electromagnetic Induction TEST
No. of Questions
30
Maximum Marks
120
Time
1 Hour
20
Chapter-wise

GENERAL INSTRUCTIONS
• This test contains 30 MCQ's. For each question only one option is correct. Darken the correct circle/ bubble in the
Response Grid provided on each page.
• You have to evaluate your Response Grids yourself with the help of solutions provided at the end of this book.
• Each correct answer will get you 4 marks and 1 mark shall be deduced for each incorrect answer. No mark will be given/
deducted if no bubble is filled. Keep a timer in front of you and stop immediately at the end of 60 min.
• The sheet follows a particular syllabus. Do not attempt the sheet before you have completed your preparation for that
syllabus.
• After completing the sheet check your answers with the solution booklet and complete the Result Grid. Finally spend time
to analyse your performance and revise the areas which emerge out as weak in your evaluation.

1. A metal rod of length l cuts across a uniform magnetic field using the similar wire but the direction of winding was
B with a velocity v. If the resistance of the circuit of which reversed in each layer; coil 3 with inductance L3 was wound
the rod forms a part is r, then the force required to move the using a superconducting wire. The self inductance of the
rod is coils L1, L2, L3 are
B2 l 2 v Blv B2 lv B2 l 2 v 2 (a) L1 = L2= L3 (b) L1 = L2; L3 = 0
(a) (b) (c) (d) (c) L1= L3; L2= 0 (d) L1 > L2 > L3
r r r r
2. The self inductance of a long solenoid cannot be increased 4. A rectangular, a square, a circular and an elliptical loop, all
by in the (x – y) plane, are movingr out of a uniform magnetic
(a) increasing its area of cross section field with a constant velocity, V = v iˆ . The magnetic field
(b) increasing its length is directed along the negative z axis direction. The induced
(c) changing the medium with greater permeability emf, during the passage of these loops, out of the field
(d) increasing the current through it region, will not remain constant for
3. Three solenoid coils of same dimension, same number of (a) the circular and the elliptical loops.
turns and same number of layers of windings are taken. Coil (b) only the elliptical loop.
1 with inductance L1 was wound using a Mn wire of (c) any of the four loops.
resistance 11 W/m, coil 2 with inductance L2 was wound (d) the rectangular, circular and elliptical loops.

RESPONSE GRID 1. 2. 3. 4.
Space for Rough Work
EBD_7504
P-78 NTA JEE Main

5. A metal ring is held horizontally and bar magnet is dropped 2 Bwl2 3Bwl 2
through the ring with its length along the axis of the ring. (a) (b)
The acceleration of the falling magnet 2 2
(a) is equal to g 4 Bwl2 5 Bwl 2
(c) (d)
(b) is less than g 2 2
(c) is more than g 11. Figure shows a conducting rod of negligible resistance that
(d) depends on the diameter of ring and length of magnet can slide on smooth U-shaped rail made of wire of resistance
1W/m. Position of the conducting rod at t = 0 is shown. A
6. Which of the following figure correctly depicts the Lenz’s time t dependent magnetic field B = 2t tesla is switched on at
law. The arrows show the movement of the labelled pole of t = 0.
a bar magnet into a closed circular loop and the arrows on
the circle show the direction of the induced current
Conducting
l = 20cm. rod

(a) (b) l = 40cm.


N N At t = 0, when the magnetic field is switched on, the
conducting rod is moved to the left at constant speed 5cm/
s by some external means. The rod moves perpendicular to
the rails. At t = 2s, induced emf has magnitude
(c) (d) (a) 0.12V (b) 0.08V (c) 0.04V (d) 0.02 V
S S
12. A square loop of side a is rotating about its diagonal rwith
7. The magnetic flux (in weber) linked with a coil of resistance angular velocity w in a perpendicular magnetic field B . It
10 W is varying with respect to time t as f = 4t2 + 2t + 1. Then has 10 turns. The emf induced is
the current in the coil at time t = 1 second is (a) B a2 sin wt
(a) 0.5 A (b) 2 A (c) 1.5 A (d) 1 A (b) B a2 cos wt × × × ×B

8. Two coils are placed close to each other. The mutual (c) 5 2 B a2
inductance of the pair of coils depends upon (d) 10 B a2 w sin wt a
(a) the rates at which currents are changing in the two 13. A conducting wire frame is placed in a magnetic field which
coils is directed into the paper. The magnetic field is increasing at
(b) relative position and orientation of the two coils a constant rate. The directions of induced current in wires
(c) the materials of the wires of the coils AB and CD are
(d) the currents in the two coils (a) B to A and D to C
(b) A to B and C to D
9. The armature of a dc motor has 20W resistance. It draws a (c) A to B and D to C
current of 1.5 A when run by a 220 V dc supply. The value (d) B to A and C to D
of the back emf induced in it is
14. The pointer of a dead-beat galvanometer gives a steady
(a) 150 V (b) 170 V (c) 180 V (d) 190 V deflection because
10. A metallic rod of length ‘l’ is tied to a string of length 2l and (a) eddy currents are produced in the conducting frame
made to rotate with angular speed w on a horizontal table over which the coil is wound.
with one end of the string fixed. If there is a vertical magnetic (b) its magnet is very strong.
field ‘B’ in the region, the e.m.f. induced across the ends of (c) its pointer is very light.
the rod is (d) its frame is made of ebonite.

RESPONSE 5. 6. 7. 8. 9.
GRID 10. 11. 12. 13. 14.
Space for Rough Work
Physics P-79

15. A square metal loop of side 10 cm and resistance 1 W is 20. In fig, CODF is a semicircular x x x x x
moved with a constant velocity partly inside a uniform loop of a conducting wire w
Cx x x x x
magnetic field of 2 Wbm–2, directed into the paper, as shown of resistance R and radius r. B
in the figure. The loop is connected to a network of five It is placed in a uniform x x x x x
resistors each of value 3W. If a steady current of 1 mA magnetic field B, which is F O
flows in the loop, then the speed of the loop is x x x x x
directed into the page r
v (perpendicular to the plane x x x x x
D
Ä Ä Ä Ä Ä
of the loop). x x x x x
Ä Ä Ä Ä Ä
The loop is rotated with a constant angular speed w about
Ä Ä Ä Ä Ä

Ä Ä Ä Ä Ä
an axis passing through the centre O, and perpendicular to
Ä Ä Ä Ä Ä
the page. Then the induced current in the wire loop is
(a) zero (b) Br2 w/R
(a) 0.5 cms–1 (b) 1 cms–1 (c) 2 cms–1 (d) 4 cms–1 2
(c) Br w/2R (d) Bpr2 w/R
16. The figure shows certain wire segments c d 21. The magnetic flux through a circuit of resistance R changes
joined together to form a coplanar loop. a b by an amount Df in a time Dt. Then the total quantity of
The loop is placed in a perpendicular electric charge Q that passes any point in the circuit during
magnetic field in the direction going into
the time Dt is represented by
the plane of the figure.
Df 1 Df
The magnitude of the field increases with time. I1 and I2 are (a) Q = R . (b) Q = .
the currents in the segments ab and cd. Then, Dt R Dt
(a) I1 > I2 (b) I1 < I2 Df Df
(c) Q = (d) Q =
(c) I1 is in the direction ba and I2 is in the direction cd R Dt
(d) I1 is in the direction ab and I2 is in the direction dc 22. A uniform magnetic field of induction B is confined to a
cylindrical region of radius R. The magnetic field is increasing
17. The mutual inductance of a pair of coils, each of N turns, is
M henry. If a current of I ampere in one of the coils is dB
brought to zero in t second, the emf induced per turn in the at a constant rate of tesla/second). An electron of
dt
other coil, in volt, will be charge e, placed at the point P on the periphery of the field
MI NMI MN MI experiences an acceleration.
(a) (b) (c) (d)
t t It Nt
1 eR dB
18. A thin semicircular conducting ring of (a) towards left
radius R is falling with its plane vertical × B × × × × 2 m dt
in a horizontal magnetic induction B. × × × × × 1 eR dB B
At the position MNQ, the speed of the (b) towards right R
× × ×V × × 2 m dt
ring is V and the potential difference
× × × × ×
developed across the ring is M Q eR dB
(a) Zero (c) towards left P
m dt
(b) B vpR2 / 2 and M is at higher potential (d) zero
(c) pRBV and Q is at higher potential 23. As a result of change in the magnetic flux linked to the
(d) 2RBV and Q is at higher potential closed loop shown in the fig, an e.m.f. V volt is induced in
19. A boat is moving due east in a region where the earth's the loop. The work done (joule) in taking a charge Q
magnetic field is 5.0 × 10–5 NA–1 m–1 due north and horizontal. coulomb once along the loop is
The boat carries a vertical aerial 2 m long. If the speed of the
boat is 1.50 ms–1, the magnitude of the induced emf in the (a) QV (b) 2QV
wire of aerial is: (c) QV/2 (d) Zero
(a) 0.75 mV (b) 0.50 mV (c) 0.15 mV (d) 1mV

RESPONSE 15. 16. 17. 18. 19.


GRID 20. 21. 22. 23.
Space for Rough Work
EBD_7504
P-80 NTA JEE Main

24. In an inductor of self-inductance L = 2 mH, current changes 27. A thin circular ring of area A is held perpendicular to a
with time according to relation i = t2e–t. At what time emf is uniform magnetic field of induction B. A small cut is made in
zero? the ring and a galvanometer is connected across the ends
(a) 4s (b) 3s (c) 2s (d) 1s such that the total resistance of the circuit is R. When the
25. Two identical circular loops of metal wire are lying on a ring is suddenly squeezed to zero area, the charge flowing
table without touching each other. Loop–A carries a curent through the galvanometer is
which increases with time. In response, the loop–B BR AB B2 A
(a) (b) (c) ABR (d)
(a) remains stationary A R R2
(b) is attracted by the loop-A 28. A magnet is moved towards a coil (i) quickly (ii) slowly,
then the induced e.m.f. is
(c) is repelled by the loop-A
(a) larger in case (i)
(d) rotates about its CM, with CM fixed (b) smaller in case (i)
(CM is the centre of mass) (c) equal to both the cases
26. In the figure shown a square loop PQRS of side ‘a’ and (d) larger or smaller depending upon the radius of the coil
resistance ‘1 unit’ is placed near an infinitely long wire 29. A coil having n turns and resistance R W is connected with
carrying a constant current I. The sides PQ and RS are parallel a galvanometer of resistance 4R W. This combination is
to the wire. The wire and the loop are in the same plane. The moved in time t seconds from a magnetic field W1 weber to
loop is rotated by 180° about an axis parallel to the long wire W2 weber. The induced current in the circuit is
and passing through the mid points of the side QR and PS. ( W1 - W2 ) n ( W2 - W1 )
The total amount of charge which passes through any point (a) - (b) -
Rnt 5 Rt
of the loop during rotation is
( W2 - W1 ) n ( W2 - W1 )
m 0 Ia (c) - (d) -
(a) ln 2 Q R 5 Rnt Rt
2p
I a 30. Two coaxial solenoids are made by winding thin insulated
m 0 Ia wire over a pipe of cross-sectional area A = 10 cm2 and
(b) ln 2
p a P S length = 20 cm. If one of the solenoid has 300 turns and the
other 400 turns, their mutual inductance is
m 0 Ia 2 a
(c) (m0 = 4p × 10 –7 Tm A–1)
2p (a) 2.4p × 10–5 H (b) 4.8p × 10–4 H
(d) cannot be found because time of rotation not given (c) 4.8p × 10 H–5 (d) 2.4p × 10–4 H

RESPONSE 24. 25. 26. 27. 28.


GRID 29. 30.

PHYSICS CHAPTERWISE SPEED TEST-20


Total Questions 30 Total Marks 120
Attempted Correct
Incorrect Net Score
Cut-off Score 45 Qualifying Score 60
Success Gap = Net Score – Qualifying Score
Net Score = (Correct × 4) – (Incorrect × 1)
Space for Rough Work
PHYSICS Speed
Alternating Current TEST
No. of Questions
30
Maximum Marks
120
Time
1 Hour
21
Chapter-wise

GENERAL INSTRUCTIONS
• This test contains 30 MCQ's. For each question only one option is correct. Darken the correct circle/ bubble in the
Response Grid provided on each page.
• You have to evaluate your Response Grids yourself with the help of solutions provided at the end of this book.
• Each correct answer will get you 4 marks and 1 mark shall be deduced for each incorrect answer. No mark will be given/
deducted if no bubble is filled. Keep a timer in front of you and stop immediately at the end of 60 min.
• The sheet follows a particular syllabus. Do not attempt the sheet before you have completed your preparation for that
syllabus.
• After completing the sheet check your answers with the solution booklet and complete the Result Grid. Finally spend time
to analyse your performance and revise the areas which emerge out as weak in your evaluation.

1. A coil of inductance 300 mH and resistance 2W is connected C R


to a source of voltage 2V. The current reaches half of its 100V 60V 30V
steady state value in
(a) 0.1 s (b) 0.05 s (c) 0.3 s (d) 0.15 s
2. A series R-C circuit is connected to an alternating voltage
source. Consider two situations:
V = V0 sin w 0t (w 0 = w 1)
(A) When capacitor is air filled.
The value of w2 is
(B) When capacitor is mica filled.
Current through resistor is i and voltage across capacitor is 3 5 5 3
(a) w1 (b) w1 (c) w1 (d) w1
V then : 5 3 3 5
(a) Va > Vb (b) ia > ib (c) Va = Vb (d) Va < Vb 4. A bulb is rated at 100 V, 100 W, it can be treated as a resistor.
3. Consider the RLC circuit shown below connected to an AC Find out the inductance of an inductor (called choke coil)
source of constant peak voltage V0 and variable frequency that should be connected in series with the bulb to operate
w0. The value of L is 20 mH. For a certain value w0 = w1, rms the bulb at its rated power with the help of an ac source of
200 V and 50 Hz.
voltage across L, C, R are shown in the diagram. At w0 = w2,
it is found that rms voltage across resistance is 50V. Then p 2 3
(a) H (b) 100 H (c) H (d) H
3 p p
RESPONSE GRID 1. 2. 3. 4.
Space for Rough Work
EBD_7504
P-82 NTA JEE Main

5. An inductor of inductance L = 400 mH 10 second. For series combination the time for needed for
and resistors of resistance R1 = 2W E reducing the voltage of the fully charged series combination
and R2 = 2W are connected to a battery L by half is
of emf 12 V as shown in the figure. R 1 (a) 10 second (b) 5 second
The internal resistance of the battery (c) 2.5 second (d) 20 second
is negligible. The switch S is closed at R 2 11. In an oscillating LC circuit the maximum charge on the
t = 0. The potential drop across L as a capacitor is Q. The charge on the capacitor when the energy
function of time is `
S
is stored equally between the electric and magnetic field is
12 -3t
(a) e V (b) 6(1 – e–t/0.2)V Q Q Q
t (a) (b) (c) (d) Q
(c) 12e–5t V (d) 6e–5t V 2 3 2
6. An ac source of angular frequency w is fed across a resistor 12. The voltage time (V-t) graph +V0
r and a capacitor C in series. The current registered is I. If for triangular wave having
T/2
now the frequency of source is changed to w/3 (but peak value V0 is as shown in 0
T/4 T
t
maintaining the same voltage), the current in the circuit is figure.The rms value of V in
found to be halved. The ratio of reactance to resistance at time interval from t = 0 to T/4 is –V0
the original frequency w is
V0
then find the value of x.
3 2 1 4 x
(a) (b) (c) (d)
5 5 5 5
7. An ideal coil of 10H is connected in series with a resistance (a) 5 (b) 4 (c) 7 (d) 3
of 5W and a battery of 5V. 2second after the connection is 13. In the circuit shown below, the key K is closed at t = 0. The
made, the current flowing in ampere in the circuit is current through the battery is
(a) (1 – e–1) (b) (1 – e) (c) e (d) e–1 V K
8. In a series LCR circuit R = 200W and the voltage and the
frequency of the main supply is 220V and 50 Hz respectively.
L R1
On taking out the capacitance from the circuit the current
lags behind the voltage by 30°. On taking out the inductor
from the circuit the current leads the voltage by 30°. The R2
power dissipated in the LCR circuit is
(a) 305 W (b) 210 W (c) Zero W (d) 242 W VR1R2 V
9. A fully charged capacitor C with initial charge q0 is connected (a) at t = 0 and R at t = ¥
to a coil of self inductance L at t = 0. The time at which the R12 + R22 2
energy is stored equally between the electric and the V V ( R1 + R2 )
magnetic fields is: (b) at t = 0 and at t = ¥
p R2 R1 R2
(a) LC (b) 2p LC (c) LC (d) p LC
4 V VR1R2
10. Combination of two identical capacitors, a resistor R and a (c) at t = 0 and at t = ¥
dc voltage source of voltage 6V is used in an experiment on R2 R12 + R22
a (C-R) circuit. It is found that for a parallel combination of V ( R1 + R2 ) V
the capacitor the time in which the voltage of the fully (d) at t = 0 and at t = ¥
charged combination reduces to half its original voltage is R1 R2 R2

RESPONSE 5. 6. 7. 8. 9.
GRID 10. 11. 12. 13.
Space for Rough Work
Physics P-83

14. The tuning circuit of a radio receiver has a resistance of 18. A coil has resistance 30 ohm and inductive reactance 20
50 W , an inductor of 10 mH and a variable capacitor. A ohm at 50 Hz frequency. If an ac source, of 200 volt, 100 Hz,
1 MHz radio wave produces a potential difference of is connected across the coil, the current in the coil will be
0.1 mV. The values of the capacitor to produce resonance is 20
(a) 4.0 A (b) 8.0 A (c) A (d) 2.0 A
(Take p2 = 10) 13
(a) 2.5 pF (b) 5.0 pF (c) 25 pF (d) 50 pF 19. Which one of the following curves represents the variation
15. The instantaneous values of alternating current and voltages of impedance (Z) with frequency f in series LCR circuit?
in a circuit are given as Z
Z
1
i= sin(100pt ) amper (a) (b)
2
1
e= sin(100 pt + p / 3) Volt
2 f
f
The average power in Watts consumed in the circuit is :
Z Z
1 3 1 1
(a) (b) (c) (d) (c) (d)
4 4 2 8
16. An inductor (L = 100 mH), a resistor (R = 100 W) and a
battery (E = 100 V) are initially connected in series as shown
in the figure. After a long time the battery is disconnected f f
after short circuiting the points A and B. The current in the 20. The primary and secondary coil of a transformer have 50
circuit 1 ms after the short circuit is and 1500 turns respectively. If the magnetic flux f linked
L with the primary coil is given by f = f0 + 4t, where f is in
webers, t is time in seconds and f0 is a constant, the output
voltage across the secondary coil is
R (a) 120 volts (b) 220 volts
(c) 30 volts (d) 90 volts
A B 21. The primary winding of a transformer has 100 turns and its
secondary winding has 200 turns. The primary is connected
E
(a) 1/eA (b) eA (c) 0.1 A (d) 1 A to an A.C. supply of 120 V and the current flowing in it is 10
A. The voltage and the current in the secondary are
17. In an alternating current circuit in which an inductance and (a) 240 V, 5 A (b) 240 V, 10 A
capacitance are joined in series, current is found to be (c) 60 V, 20 A (d) 120 V, 20 A
maximum when the value of inductance is 0.5 henry and the 3
value of capacitance is 8µF. The angular frequency of applied 22. The current in a LR circuit builds up to th of its steady
4
alternating voltage will be state value in 4s. The time constant of this circuit is
(a) 5000 rad/sec (b) 4000 rad/sec 1 2 3 4
(c) 2 × 105 rad/sec (d) 500 rad/sec (a) s (b) s (c) s (d) s
ln2 ln2 ln2 ln2

RESPONSE 14. 15. 16. 17. 18.


GRID 19. 20. 21. 22.
Space for Rough Work
EBD_7504
P-84 NTA JEE Main

23. A coil of resistance 50 W is connected across a 5.0 V 28. The core of any transformer is laminated so as to
battery. 0.1 s after the battery is connected, the current in (a) reduce the energy loss due to eddy currents
the coil is 60 mA. Calculate the inductance of the coil. (b) make it light weight
(a) 5.5 H (b) 1.5 H (c) 2.5 H (d) 9.5 H (c) make it robust and strong
24. The inductance between A and D is (d) increase the secondary voltage
29. In an LCR circuit as shown below both switches are open
initially. Now switch S1 is closed, S2 kept open. (q is charge
A 3H 3H 3H D on the capacitor and t = RC is Capacitive time constant).
Which of the following statement is correct?
(a) 3.66 H (b) 9 H (c) 0.66 H (d) 1 H V
25. An LCR series circuit is connected to a source of alternating
current. At resonance, the applied voltage and the current R
flowing through the circuit will have a phase difference of S1
p p
(a) p (b) (c) (d) 0 C S2
2 4
26. In an electrical circuit R, L, C and an a.c. voltage source are
L
all connected in series. When L is removed from the circuit,
the phase difference between the voltage the current in the (a) Work done by the battery is half of the energy
circuit is p/3. If instead, C is removed from the circuit, the dissipated in the resistor
phase difference is again p/3. The power factor of the circuit (b) At, t = t, q = CV/2
is : (c) At, t = 2t, q = CV (1 – e–2)
(a) 1/2 (b) 1/ 2 (c) 1 (d) 3/2 (d) At, t = 2 t, q = CV (1 – e–1)
27. A 100 mF capacitor in series with a 40W resistance is 30. An AC generator of 220 V having internal resistance r = 10W
connected to a 110 V, 60 Hz supply. and external resistance R = 100W. What is the power
What is the maximum current in the circuit? developed in the external circuit?
(a) 3.24 A (b) 4.25 A (c) 2.25 A (d) 5.20 A (a) 484 W (b) 400 W (c) 441 W (d) 369 W

RESPONSE 23. 24. 25. 26. 27.


GRID 28. 29. 30.

PHYSICS CHAPTERWISE SPEED TEST-21


Total Questions 30 Total Marks 120
Attempted Correct
Incorrect Net Score
Cut-off Score 45 Qualifying
Space for Rough Work Score 60
Success Gap = Net Score – Qualifying Score
Net Score = (Correct × 4) – (Incorrect × 1)
Space for Rough Work
PHYSICS Speed
Electromagnetic Waves TEST
No. of Questions
30
Maximum Marks
120
Time
1 Hour
22
Chapter-wise

GENERAL INSTRUCTIONS
• This test contains 30 MCQ's. For each question only one option is correct. Darken the correct circle/ bubble in the
Response Grid provided on each page.
• You have to evaluate your Response Grids yourself with the help of solutions provided at the end of this book.
• Each correct answer will get you 4 marks and 1 mark shall be deduced for each incorrect answer. No mark will be given/
deducted if no bubble is filled. Keep a timer in front of you and stop immediately at the end of 60 min.
• The sheet follows a particular syllabus. Do not attempt the sheet before you have completed your preparation for that
syllabus.
• After completing the sheet check your answers with the solution booklet and complete the Result Grid. Finally spend time
to analyse your performance and revise the areas which emerge out as weak in your evaluation.

1. An electromagnetic wave in vacuum has the electric and 3. The figure here gives the electric field of an electromagnetic
r r wave at a certain point and a certain instant. The wave is
magnetic field E and B , which are always perpendicular to
r transporting energy in the negative z-direction. The direction
each other. The direction of polarization is given by X and of the magnetic field of the wave at that point and instant is
r
that of wave propagation by k . Then
r r r r r r r r r r
(a) X || B and k || B ´ E (b) X || E and k || E ´ B
r r r r r r r r r r
(c) X || B and k || E ´ B (d) X || E and k || B ´ E
2. If E and B represent elctric and magnetic field vectors of the
electromagnetic wave, the direction of propagation of
electromagnetic wave is along
r r
(a) E (b) B (a) + ve x-direction (b) –ve x-direction
r r r r (c) +ve z-direction (d) –ve y-direction
(c) B ´ E (d) E ´ B

RESPONSE GRID 1. 2. 3.

Space for Rough Work


EBD_7504
P-86 NTA JEE Main

4. A plane electromagnetic wave is incident on a material (a) A


surface. If the wave delivers momentum p and energy E,
then (b) B

(a) p = 0, E = 0 (b) p ¹ 0, E ¹ 0 (c) C

(c) p ¹ 0, E = 0 (d) p = 0, E ¹ 0 (d) D


5. Intensity of electromagnetic wave will be 10. The electric field part of an electromagnetic wave in a
medium is represented by Ex=0;
(a) I = cm 0 B20 / 2 (b) I = ce 0 B20 / 2
N éæ rad ö æ –2 rad ö ù ;
E y = 2.5 cos êç 2π ×106 ÷ t – ç π ×10 ÷x
C ëè mø è s ø úû
(c) I= B02 / cm 0 (d) I= E 02 / 2ce 0
6. The electric field of an electromagnetic wave travelling through Ez = 0. The wave is :
vaccum is given by the equation E = E0 sin (kx – wt). The (a) moving along x direction with frequency 106 Hz and
quantity that is independent of wavelength is wave length 100 m.
(b) moving along x direction with frequency 106 Hz and
k
(a) kw (b) (c) k2 w (d) w wave length 200 m.
w
(c) moving along – x direction with frequency 106 Hz and
7. When an electromagnetic waves enter the ionised layer of wave length 200 m.
ionosphere, the motion of electron cloud produces a space
current and the electric field has its own capacitative (d) moving along y direction with frequency 2p × 106 Hz
displacement current, then and wave length 200 m.
(a) the space current is in phase of displacement current 11. A lamp emits monochromatic green light uniformly in all
directions. The lamp is 3% efficient in converting electrical
(b) the space current lags behind the displacement current
power to electromagnetic waves and consumes 100 W of
by a phase 180°.
power. The amplitude of the electric field associated with
(c) the space current lags behind the displacement current the electromagnetic radiation at a distance of 5 m from the
by a phase 90°. lamp will be nearly:
(d) the space current leads the displacement current by a
(a) 1.34 V/m (b) 2.68 V/m (c) 4.02 V/m (d) 5.36 V/m
phase 90°.
12. During the propagation of electromagnetic waves in a
8. The rms value of the electric field of the light coming from
medium:
the Sun is 720 N/C. The average total energy density of the
electromagnetic wave is (a) Electric energy density is double of the magnetic
(a) 4.58 × 10–6 J/m3 (b) 6.37 × 10–9 J/m3 energy density.
(c) 81.35 × 10–12 J/m3 (d) 3.3 × 10–3 J/m3 (b) Electric energy density is half of the magnetic energy
density.
9. The figure shows graphs of the electric field magnitude E
versus time t for four uniform electric fields, all contained (c) Electric energy density is equal to the magnetic energy
within identical circular regions. Which of them is according density.
to the magnitudes of the magnetic field? (d) Both electric and magnetic energy densities are zero.

RESPONSE 4. 5. 6. 7. 8.
GRID 9. 10. 11. 12.
Space for Rough Work
Physics P-87

13. A plane electromagnetic wave travels in free space along 18. Out of the following options which one can be used to
x-axis. At a particular point in space, the electric field along produce a propagating electromagnetic wave ?
y-axis is 9.3 V m–1. The magnetic induction (B) along z-axis
(a) A charge moving at constant velocity
is
(a) 3.1 × 10–8 T (b) 3 × 10–5 T (b) A stationary charge
(c) 3 × 10–6 T (d) 9.3 × 10–6 T (c) A chargeless particle
14. The speed of electromagnetic wave in vacuum depends (d) An accelerating charge
upon the source of radiation. It 19. The pressure exerted by an electromagnetic wave of
(a) increases as we move from g-rays to radio waves intensity I (watts/m2) on a nonreflecting surface is [c is the
(b) decreases as we move from g-rays to radio waves velocity of light]
(c) is same for all of them (a) Ic (b) Ic2 (c) I/c (d) I/c2
(d) None of these 20. Which of the following electromagnetic waves has minimum
15. The energy of electromagnetic wave in vacuum is given by frequency ?
the relation (a) Microwaves (b) Audible waves
2 2
E B 1 1 (c) Ultrasonic wave (d) Radiowaves
(a) + (b) e0E 2 + µ0B2
2e 0 2µ 0 2 2 21. In an electromagnetic wave
2 2 2 (a) power is transmitted along the magnetic field
E +B 1 B
(c) (d) e0E 2 +
c 2 2µ0 (b) power is transmitted along the electric field
16. If microwaves, X rays, infrared, gamma rays, ultra-violet, (c) power is equally transferred along the electric and
radio waves and visible parts of the electromagnetic magnetic fields
spectrum are denoted by M, X, I, G, U, R and V then which
(d) power is transmitted in a direction perpendicular to
of the following is the arrangement in ascending order of
both the fields
wavelength ?
(a) R, M, I, V, U, X and G (b) M, R, V, X, U, G and I 22. A point source of electromagnetic radiation has an average
power output of 1500 W. The maximum value of electric field
(c) G, X, U, V, I, M and R (d) I, M, R, U, V, X and G
at a distance of 3m from this sources in Vm–1 is
17. The ratio of amplitude of magnetic field to the amplitude of
electric field for an electromagnetic wave propagating in 500 250
vacuum is equal to : (a) 500 (b) 100 (c) (d)
3 3
(a) the speed of light in vacuum
23. All components of the electromagnetic spectrum in vacuum
(b) reciprocal of speed of light in vacuum
have the same
(c) the ratio of magnetic permeability to the electric
susceptibility of vacuum (a) energy (b) velocity
(d) unity (c) wavelength (d) frequency

13. 14. 15. 16. 17.


RESPONSE 18. 19. 20. 21. 22.
GRID
23.
Space for Rough Work
EBD_7504
P-88 NTA JEE Main

24. Which of the following statement is false for the properties 27. An electromagnetic wave going through vacuum is
of electromagnetic waves? described by E = E0sin(kx – wt); B = B0 sin (kx – wt). Which
of the following equations is true
(a) Both electric and magnetic field vectors attain the
maxima and minima at the same place and same time. (a) E0k = B0w (b) E0w = B0k
(c) E0B0 = wk (d) None of these
(b) The energy in electromagnetic wave is divided equally
between electric and magnetic vectors 28. If c is the speed of electromagnetic waves in vacuum, its
speed in a medium of dielectric constant K and relative
(c) Both electric and magnetic field vectors are parallel to permeability µr is
each other and perpendicular to the direction of
propagation of wave 1
(a) v= (b) v = c m rK
(d) These waves do not require any material medium for mr K
propagation.
c K
25. A radiation of energy ‘E’ falls normally on a perfectly v= v=
(c) mr K (d) mr C
reflecting surface. The momentum transferred to the surface
is (C = Velocity of light) 29. A plane electromagnetic wave in a non-magnetic dielectric
ur ur
medium is given by E = E 0 (4 ´ 10 -7 x - 50t ) with distance
2E 2E E E
(a) (b) (c) (d) being in meter and time in seconds. The dielectric constant
C C2 C2 C of the medium is :
26. If vs, vx and vm are the speed of soft gamma rays, X-rays (a) 2.4 (b) 5.8 (c) 8.2 (d) 4.8
and microwaves respectively in vacuum, then 30. 15
Frequency of a wave is 6 × 10 Hz. The wave is
(a) vs > vx > vm (b) vs < vx < vm (a) radiowave (b) microwave
(c) vs > vx < vm (d) vs = vx = vm (c) x-ray (d) None of these

RESPONSE 24. 25. 26. 27. 28.


GRID 29. 30.

PHYSICS CHAPTERWISE SPEED TEST-22


Total Questions 30 Total Marks 120
Attempted Correct
Incorrect Net Score
Cut-off Score 45 Qualifying Score 60
Success Gap = Net Score – Qualifying Score
Net Score = (Correct × 4) – (Incorrect × 1)
Space for Rough Work
PHYSICS Speed
Ray Optics and Optical Instruments TEST
No. of Questions
30
Maximum Marks
120
Time
1 Hour
23
Chapter-wise

GENERAL INSTRUCTIONS
• This test contains 30 MCQ's. For each question only one option is correct. Darken the correct circle/ bubble in the
Response Grid provided on each page.
• You have to evaluate your Response Grids yourself with the help of solutions provided at the end of this book.
• Each correct answer will get you 4 marks and 1 mark shall be deduced for each incorrect answer. No mark will be given/
deducted if no bubble is filled. Keep a timer in front of you and stop immediately at the end of 60 min.
• The sheet follows a particular syllabus. Do not attempt the sheet before you have completed your preparation for that
syllabus.
• After completing the sheet check your answers with the solution booklet and complete the Result Grid. Finally spend time
to analyse your performance and revise the areas which emerge out as weak in your evaluation.

1. A car is fitted with a convex side-view mirror of focal length 3. The position of final image formed by the given lens
20 cm. A second car 2.8 m behind the first car is overtaking combination from the third lens will be at a distance of
the first car at a relative speed of 15 m/s. The speed of the (f1 = + 10 cm, f2 = – 10 cm and f3 = + 30 cm).
image of the second car as seen in the mirror of the first one
is :
1
(a) m/s (b) 10 m/s
15
1
(c) 15 m/s (d) m/s
10
30 cm 5 cm 10 cm
2. A fish looking up through the water sees the outside world (a) 15 cm (b) infinity (c) 45 cm (d) 30 cm
contained in a circular horizon. If the refractive index of
A
4 4. If the refractive index of the material of a prism is cot
and
water is and the fish is 12 cm below the surface, the 2
3
the angle of prism is A, then angle of minimum deviation is
radius of this circle in cm is
p p
(a) 36 5 (b) 4 5 (c) 36 7 (d) 36/ 7 (a) p - 2A (b) p - A (c) - 2A (d) -A
2 2

RESPONSE GRID 1. 2. 3. 4.
Space for Rough Work
EBD_7504
P-90 NTA JEE Main

5. A telescope has an objective of focal length 100 cm and an æ 2 ö


eyepiece of focal length 5 cm. What is the magnifying power (a) sin -1 ( 3/2 ) (b) sin -1 ç
è 3ø
÷
of the telescope when the final image is formed at the least
æ 1 ö
distance of distinct vision ?
(c) sin -1 ç ÷ (d) sin–1 (1/2)
(a) 20 (b) 24 (c) 28 (d) 32 è 3ø
6. In the figure shown, L is a converging lens of focal length 9. A ray of light passes through an equilateral prism such that
10 cm and M is a concave mirror of radius of curvature the angle of incidence is equal to the angle of emergence and
20cm. A point object O is placed in front of the lens at a 3
distance 15 cm. AB and CD are optical axes of the lens and the latter is equal to th of angle of prism. The angle of
mirror respectively. The distance of the final image formed 4
deviation is
by this system from the optical centre of the lens is x 26 cm. (a) 25° (b) 30° (c) 45° (d) 35°
Find the value of x in cm. 10. Aperture of the human eye is 2 mm. Assuming the mean
[The distance between CD and AB is 1 cm] wavelength of light to be 5000 Å, the angular resolution
M
L limit of the eye is nearly
C D (a) 2 minute (b) 1 minute
O 1cm B
A (c) 0.5 minute (d) 1.5 minute
15cm 11. When a plane mirror is placed horizontally on a level ground
at a distance of 60 m from the foot of a tower, the top of the
tower and its image in the mirror subtend an angle of 90° at
45cm
the eye. The height of the tower will be
(a) 2 (b) 3 (c) 6 (d) 4
7. An object at 2.4 m in front of a lens forms a sharp image on (a) 30 m (b) 60 m (c) 90 m (d) 120 m
12. A light ray falls on a rectangular glass slab as 45º
a film 12 cm behind the lens. A glass plate 1 cm thick, of
shown. The index of refraction of the glass, if
refractive index 1.50 is interposed between lens and film total internal reflection is to occur at the
with its plane faces parallel to film. At what distance vertical face, is
(from lens) should object shifted to be in sharp focus of
film? (a) 3/ 2 (b)
( 3 + 1) Glass

(a) 7.2 m (b) 2.4 m (c) 3.2 m (d) 5.6 m 2


8. A transparent solid cylindrical rod has a refractive index of
(c)
( 2 +1 ) (d) 5 / 2
2 2
. It is surrounded by air. A light ray is incident at the mid-
3 13. A lens made of glass whose index of refraction is 1.60 has a
point of one end of the rod as shown in the figure. focal length of + 20 cm in air. Its focal length in water, whose
refractive index is 1.33, will be
(a) three times longer than in air
q (b) two times longer than in air
(c) same as in air
The incident angle q for which the light ray grazes along the (d) None of these
wall of the rod is :
RESPONSE 5. 6. 7. 8. 9.
GRID 10. 11. 12. 13.
Space for Rough Work
Physics P-91

14. For a prism kept in air it is found that for an angle of incidence 19. A ray PQ incident on the refracting A
60°, the angle of Prism A, angle of deviation d and angle of face BA is refracted in the prism
BAC as shown in the figure and 60° R
emergence ‘e’ become equal. Then the refractive index of Q
the prism is emerges from the other refracting S
(a) 1.73 (b) 1.15 (c) 1.5 (d) 1.33 face AC as RS such that AQ = AR.
15. A person can see clearly only upto a distance of 30 cm. He If the angle of prism A = 60° and the P B C
wants to read a book placed at a distance of 50 cm from his refractive index of the material of prism is 3 , then the
eyes. What is the power of the lens of his spectacles ? angle of deviation of the ray is
(a) –1.0 D (b) –1.33 D (c) –1.67 D (d) –2.0 D (a) 60° (b) 45°
16. An object is placed at a distance of 40 cm in front of a (c) 30° (d) None of these
concave mirror of focal length 20 cm. The image produced is 20. A rectangular glass slab ABCD A
(a) real, inverted and smaller in size D
of refractive index n1 is immersed
(b) real, inverted and of same size n1 n2
in water of refractive index n2(n1 amax
(c) real and erect > n2). A ray of light is incident at B C
(d) virtual and inverted the surface AB of the slab as
17. An observer can see through a pin- shown. The maximum value of the angle of incidence amax
hole the top end of a thin rod of such that the ray comes out only from the other surface CD
height h, placed as shown in the is given by
figure. The beaker height is 3h and én æ æ n ööù
its radius h. When the beaker is 3h (a) sin-1 ê 1 cos ç sin -1 ç 2 ÷ ÷ ú
filled with a liquid up to a height ëê n2 è è n1 ø ø ûú
2h, he can see the lower end of the h
rod. Then the refractive index of the é æ æ 1 öö ù
2h (b) sin-1 ên1 cos ç sin-1 ç ÷ ÷ ú
liquid is è è n2 ø ø ûú
ëê
5 5 3 3 æn ö
(a)
2
(b)
2
(c)
2
(d)
2 (c) sin -1 ç 1 ÷
è n2 ø
18. Two plano-concave lenses (1 and 2) of glass of refractive
index 1.5 have radii of curvature 25 cm and 20 cm. They are æn ö
(d) sin -1 ç 2 ÷
placed in contact with their curved surface towards each è n1 ø
other and the space between them is filled with liquid of 21. A microscope is focussed on a mark on a piece of paper and
refractive index 4/3. Then the combination is then a slab of glass of thickness 3 cm and refractive index
(a) convex lens of focal length 70 cm 1.5 is placed over the mark. How should the microscope be
(b) concave lens of focal length 70 cm moved to get the mark in focus again ?
(c) concave lens of focal length 66.6 cm (a) 4.5 cm downward (b) 1 cm downward
(d) convex lens of focal length 66.6 cm (c) 2 cm upward (d) 1 cm upward

RESPONSE 14. 15. 16. 17. 18.


GRID 19. 20. 21.
Space for Rough Work
EBD_7504
P-92 NTA JEE Main

22. A container is filled with Radius of


water (m = 1.33) upto a curvature =20cm
heigh t of 33.25 cm. A Air
15 cm
concave mirror is placed 15 t=20cm
µ=3/2
cm above the water level
and the image of an object
m=1.33 25 cm Paper
placed at th e bottom is 33.25 cm
(a) 10 cm (b) 15 cm
formed 25 cm below the
water level. Focal length of O
I
(c) 50 cm (d) None of these
26. A thin prism of angle 15º made of glass of refractive index µ1
the mirror is
= 1.5 is combined with another prism of glass of refractive
(a) 15 cm (b) 20 cm (c) –18.31 cm (d) 10 cm index µ2 = 1.75. The combination of the prism produces
23. The focal lengths of the objective and the eye piece of a dispersion without deviation. The angle of the second prism
compound microscope are 2.0 cm and 3.0 cm, respectively. should be
The distance between the objective and the eye piece is (a) 7° (b) 10° (c) 12° (d) 5°
15.0 cm. The final image formed by the eye piece is at infinity. 27. A telescope consists of two thin lenses of focal lengths, 0.3
The two lenses are thin. The distance in cm of the object m and 3 cm respectively. It is focused on moon which
and the image produced by the objective, measured from subtends an angle of 0.5° at the objective. Then the angle
the objective lens, are respectively subtended at the eye by the final image will be
(a) 2.4 and 12.0 (b) 2.4 and 15.0 (a) 5° (b) 0.25° (c) 0.5° (d) 0.35°
(c) 2.0 and 12.0 (d) 2.0 and 3.0 28. A bi-convex lens made of glass (refractive index 1.5) is put
24. A rectangular block of glass is placed on a mark made on the in a liquid of refractive index 1.7. Its focal length will
surface of the table and it is viewed from the vertical position (a) decrease and change sign
of eye. If refractive index of glass be m and its thickness d, (b) increase and change sign
then the mark will appear to be raised up by (c) decrease and remain of the same sign
(d) increase and remain of the same sign
(m + 1)d (m - 1)d 29. To get three images of a single object, one should have two
(a) (b)
m m plane mirrors at an angle of
(m + 1) (a) 60º (b) 90º (c) 120º (d) 30º
(m - 1) m
(c) (d) 30. A thin convergent glass lens (mg = 1.5) has a power of
md d + 5.0 D. When this lens is immersed in a liquid of refractive
25. A planoconcave lens is placed on a paper on which a flower index m, it acts as a divergent lens of focal length 100 cm.
is drawn. How far above its actual position does the flower The value of m must be
appear to be? (a) 4/3 (b) 5/3 (c) 5/4 (d) 6/5

RESPONSE 22. 23. 24. 25. 26.


GRID 27. 28. 29. 30.

PHYSICS CHAPTERWISE SPEED TEST-23


Total Questions 30 Total Marks 120
Attempted Correct
Space for Rough Work

Incorrect Net Score


Cut-off Score 40 Qualifying Score 50
Success Gap = Net Score – Qualifying Score
Net Score = (Correct × 4) – (Incorrect × 1)
Space for Rough Work
PHYSICS Speed
Wave Optics TEST
No. of Questions
30
Maximum Marks
120
Time
1 Hour
24
Chapter-wise

GENERAL INSTRUCTIONS
• This test contains 30 MCQ's. For each question only one option is correct. Darken the correct circle/ bubble in the
Response Grid provided on each page.
• You have to evaluate your Response Grids yourself with the help of solutions provided at the end of this book.
• Each correct answer will get you 4 marks and 1 mark shall be deduced for each incorrect answer. No mark will be given/
deducted if no bubble is filled. Keep a timer in front of you and stop immediately at the end of 60 min.
• The sheet follows a particular syllabus. Do not attempt the sheet before you have completed your preparation for that
syllabus.
• After completing the sheet check your answers with the solution booklet and complete the Result Grid. Finally spend time
to analyse your performance and revise the areas which emerge out as weak in your evaluation.

1. A YDSE is conducted in water (µ1) as shown in figure. A 3. In a Fresnel biprism experiment, the two positions of lens
glass plate of thickness t and refractive index µ2 is placed in give separation between the slits as 16 cm and 9 cm
the path of S2. The optical path difference at O is respectively. What is the actual distance of separation?
(a) (m 2 - 1)t (a) 12.5 cm (b) 12 cm (c) 13 cm (d) 14 cm
S water µ
4. Which of the following diagrams represent the variation of
1
(b) (m1 - 1)t
1

S
O
electric field vector with time for a circularly polarised
æ m2 ö
(c) ç - 1÷ t S2
µ2
t light ?
m
è 1 ø
Screen (a) (b)
(d) (m2 – m1 )t
|E |
2. The condition for obtaining secondary maxima in the |E |

diffraction pattern due to single slit is t


(a) t
l (b)
(a) a sin q = nl (b) a sin q = ( 2n - 1) (c) (d)
2 |E |
|E |
nl
(c) a sin q = ( 2n - 1) l (d) a sin q = t t
2 (c)
(d)

RESPONSE GRID 1. 2. 3. 4.
Space for Rough Work
EBD_7504
P-94 NTA JEE Main

5. Two sources of light of wavelengths 2500 Å and 3500 Å are A B


used in Young’s double slit expt. simultaneously. Which
orders of fringes of two wavelength patterns coincide?
(a) 3rd order of 1st source and 5th of the 2nd 33° 33°
(b) 7th order of 1st and 5th order of 2nd Glass slab
(c) 5th order of 1st and 3rd order of 2nd
(d) 5th order of 1st and 7th order of 2nd (a) the intensity remains unchanged
6. The intensity at the maximum in a Young's double slit (b) the intensity is reduced to zero and remains at zero
experiment is I 0. Distance between two slits is d = 5l, where (c) the intensity gradually reduces to zero and then again
l is the wavelength of light used in the experiment. What will increase
be the intensity in front of one of the slits on the screen (d) the intensity increases continuously
placed at a distance D = 10 d ? 11. Two identical light waves, propagating in the same direction,
have a phase difference d. After they superpose, the intensity
I0 3 I0 of the resulting wave will be proportional to
(a) I0 (b) (c) I0 (d)
4 4 2 (a) cos d (b) cos (d/2)
7. The relation between the polarising angle and the critical (c) cos2 (d/2) (d) cos2 d
angle is 12. In Young’s double slit experiment, the separation between
(a) ip = tan–1 (cosec qc) the slits is halved and the distance between the slits and
(b) ip = tan–1 (sin qc) screen is doubled. The fringe width will
(c) ip = cot–1 (sin qc) (a) be halved (b) be doubled
(d) ip = cot–1 (cosec qc) (c) be quadrupled (d) remain unchanged
13. In Young's double slit experiment shown in figure S1 and S2
8. In YSDE, both slits are covered by transparent slab. Upper
are coherent sources and S is the screen having a hole at a
slit is covered by slab of R.I. 1.5 and thickness t and lower is
point 1.0mm away from the central line.
4 White light (400 to 700nm) is sent through the slits. Which
covered by R.I. and thickness 2t, then central maxima
3 wavelength passing through the hole has strong intensity?
y
(a) shifts in +ve y-axis direction S1 Centre of
screen
0.5mm
(b) shifts in –ve y-axis direction x S2 Screen
1.0mm
(c) remains at same position hole
50cm S
(d) may shift in upward
or downward depending upon wavelength of light (a) 400 nm (b) 700 nm (c) 500 nm (d) 667 nm
9. In a single slit diffraction experiment, the width of the slit is 14. According to Huygens, medium through which light
made double its original width. Then the central maxima of waves travel is
the diffraction pattern will become (a) vacuum only (b) luminiferous ether
(a) narrower and fainter (b) narrower and brighter (c) liquid only (d) solid only
(c) broader and fainter (d) broader and brighter 15. In the Young’s double-slit experiment, the intensity of light
at a point on the screen where the path difference is l is K,
10. A beam of light is incident on a glass slab (m = 1.54) in a
(l being the wave length of light used). The intensity at a
direction as shown in the figure. The reflected light is
point where the path difference is l/4, will be:
analysed by a polaroid prism. On rotating the polaroid, (tan
(a) K (b) K/4 (c) K/2 (d) Zero
57° = 1.54)

5. 6. 7. 8. 9.
RESPONSE
10. 11. 12. 13. 14.
GRID
15.
Space for Rough Work
Physics P-95

16. Figure shows two coherent sources S1 and S2 vibrating in


p
same phase. AB is an irregular wire lying at a far distance (a) radian (b) p radian
2
l -3
from the sources S1 and S2. Let = 10 and ÐBOA = 0.12°. p p
d (c) radian (d) radian
How many bright spots will be seen on the wire, including 8 4
points A and B? 22. The adjacent figure shows Fraunhoffer’s diffraction due to
A
a single slit. If first minimum is obtained in the direction
S1 shown, then the path difference between rays 1 and 3 is
d O
S2 1
B
2
(a) 5 (b) 4 (c) 3 (d) 7 3
17. A thin film of soap solution (µs = 1.4) lies on the top of a
glass plate (µg = 1.5). When visible light is incident almost
normal to the plate, two adjacent reflection maxima are
observed at two wavelengths 420 and 630 nm. The minimum (a) 0 (b) l / 4 (c) l / 2 (d) l
thickness of the soap solution is 23. If yellow light emitted by sodium lamp in Young’s double
(a) 420 nm (b) 450 nm slit experiment is replaced by a monochromatic blue light of
(c) 630 nm (d) 1260 nm the same intensity
18. In the phenomena of diffraction of light, when blue light is (a) fringe width will decrease
used in the experiment in spite of red light, then (b) finge width will increase
(a) fringes will become narrower (c) fringe width will remain unchanged
(b) fringes will become broader (d) fringes will become less intense
(c) no change in fringe width 24. In the figure shown if a parallel
(d) None of these beam of white light is incident
19. Huygen’s concept of secondary wave on the plane of the slits then
d 2d/3
(a) allows us to find the focal length of a thick lens the distance of the nearest O
(b) is a geometrical method to find a wavefront white spot on the screen from
(c) is used to determine the velocity of light O is d/A. Find the value of A. D
(d) is used to explain polarisation (assume d << D, l << d]
20. Two coherent point sources S1 and S2 are separated by a (a) 3 (b) 5 (c) 6 (d) 4
small distance d as shown. The fringes obtained on the 25. The figure shows the interference pattern obtained in a
vertical screen will be : double-slit experiment using light of wavelength 600nm. 1,
(a) points 2, 3, 4 and 5 are marked on five fringes.
d
Central bright
(b) straight bands S1 S2 screen fringe
D
(c) concentric circles
(d) semicircles
21. At the first minimum adjacent to the central maximum of a
single-slit diffraction pattern, the phase difference between 1 2 3 4 5
the Huygen's wavelet from the edge of the slit and the The third order bright fringe is
wavelet from the midpoint of the slit is : (a) 2 (b) 3 (c) 4 (d) 5

RESPONSE 16. 17. 18. 19. 20.


GRID 21. 22. 23. 24. 25.
Space for Rough Work
EBD_7504
P-96 NTA JEE Main

26. The Young’s double slit experiment is performed with blue


l2
and with green light of wavelengths 4360Å and 5460Å (b) a= and b min = 4lL
respectively. If x is the distance of 4th maxima from the central L
one, then l2 æ 2l2 ö
(a) x (blue) = x (green) (b) x (blue) > x (green) (c) a= and b min = ç
ç L ÷
÷
L è ø
x (blue) 5460
(c) x (blue) < x (green) (d) = æ 2l2 ö
x (green) 4360
(d) a = ll and b min = ç ÷
ç L ÷
27. Two polaroids are placed in the path of unpolarized beam of è ø
intensity I0 such that no light is emitted from the second 29. The wavefronts of a light wave travelling in vacuum are
polaroid. If a third polarioid whose polarization axis makes given by x + y + z = c. The angle made by the direction of
an angle q with the polarization axis of first polaroids, is propagation of light with the X-axis is
placed between these polaroids then the intensity of light
emerging from the last polaroid will be (a) 0º (b) 45º (c) 90º (d) cos –1 (1/ 3)
30. A point source is emitting light of wavelength 6000 Å is
æ I0 ö 2 æ I0 ö 2
(a) ç ÷ sin 2q (b) ç ÷ sin 2q placed at a very small height h above a flat reflecting surface
è 8ø è 4ø MN as shown in the figure.
p Screen
æ I0 ö
(c) ç ÷ cos 4 q (d) I 0 cos 4 q
è 2ø D
S
28. The box of a pin hole camera, of length L, has a hole of h
////////////////////////////////////////////
radius a. It is assumed that when the hole is illuminated by M N

a parallel beam of light of wavelength l the spread of the The intensity of the reflected light is 36% of the incident
spot (obtained on the opposite wall of the camera) is the intensity. Interference fringes are observed on a screen
sum of its geometrical spread and the spread due to placed parallel to the reflecting surface at a very large distance
diffraction. The spot would then have its minimum size (say D from it.
bmin) when : The shape of the interference fringes on the screen is
(a) circle (b) ellipse
(a) a = lL and bmin = 4lL (c) parabola (d) straight line

RESPONSE GRID 26. 27. 28. 29. 30.

PHYSICS CHAPTERWISE SPEED TEST-24


Total Questions 30 Total Marks 120
Attempted Correct
Incorrect Net Score
Cut-off Score 40 Qualifying Score 50
Success Gap = Net Score – Qualifying Score
Net Score = (Correct × 4) – (Incorrect × 1)
Space for Rough Work
PHYSICS Speed
Dual Nature of Radiation and Matter TEST
No. of Questions
30
Maximum Marks
120
Time
1 Hour
25
Chapter-wise

GENERAL INSTRUCTIONS
• This test contains 30 MCQ's. For each question only one option is correct. Darken the correct circle/ bubble in the
Response Grid provided on each page.
• You have to evaluate your Response Grids yourself with the help of solutions provided at the end of this book.
• Each correct answer will get you 4 marks and 1 mark shall be deduced for each incorrect answer. No mark will be given/
deducted if no bubble is filled. Keep a timer in front of you and stop immediately at the end of 60 min.
• The sheet follows a particular syllabus. Do not attempt the sheet before you have completed your preparation for that
syllabus.
• After completing the sheet check your answers with the solution booklet and complete the Result Grid. Finally spend time
to analyse your performance and revise the areas which emerge out as weak in your evaluation.

1. Photoelectric effect experiments are performed using three 2. Two identical photocathodes receive light of frequencies f1
different metal plates p, q and r having work functions and f2. If the velocites of the photo electrons (of mass m )
fp = 2.0 eV, fq = 2.5 eV and fr = 3.0 eV, respectively. A light coming out are respectively v1 and v2, then
beam containing wavelengths of 550 nm, 450 nm and 350
nm with equal intensities illuminates each of the plates. The 2 2 2h é 2h
1/ 2
ù
correct I-V graph for the experiment is [Take hc = 1240 eV (a) v1 - v2 = ( f - f ) (b) v1 + v2 = ê ( f1 + f 2 )ú
m 1 2 ëm û
nm]
I I 1/ 2
(a) (b) 2 2 2h é 2h ù
p
(c) v1 + v2 = ( f + f 2 ) (d) v1 - v2 = ê ( f1 - f2 )ú
q p m 1 ë m û
r q
r
V V 3. An X-ray tube is operated at 15 kV. Calculate the upper limit
I
I of the speed of the electrons striking the target.
r
(c) (d) (a) 7.26 × 107 m/s (b) 7.62 × 107 m/s
q
p
r q
p
(c) 7.62 × 107 cm/s (d) 7.26 × 109 m/s
V V

RESPONSE GRID 1. 2. 3.

Space for Rough Work


EBD_7504
P-98 NTA JEE Main

4. The anode voltage of a photocell is kept fixed. The (a) the energy of the incident photon is 4.7 eV
wavelength l of the light falling on the cathode is gradually (b) the energy of the incident photon is 2.3 eV
changed. The plate current I of the photocell varies as (c) if higher frequency photon be used, the photoelectric
follows current will rise
(a) (b) (d) when the energy of photon is 3.5 eV, the photoelectric
current will be maximum
I I 10. Two insulating plates are both uniformly charged in
such a way that the potential difference between them is
O l O l
V2 – V1 = 20 V. (i.e., plate 2 is at a higher potential). The
(c) (d) plates are separated by d = 0.1 m and can be treated as
infinitely large. An electron is released from rest on the inner
I I surface of plate 1. What is its speed when it hits plate 2?
(e = 1.6 × 10–19 C, me = 9.11 × 10–31 kg)
O l
O l
5. In a photoelectric experiment, with light of wavelength l, Y
the fastest electron has speed v. If the exciting wavelength
is changed to 5l/4, the speed of the fastest emitted electron 0.1 m
will become
X
5 5
(a) v (b) v
4 3
1 2
5 5 (a) 2.65 × 106 m/s (b) 7.02 × 1012 m/s
(c) less than v (d) greater than v 6
(c) 1.87 × 10 m/s (d) 32 × 10–19 m/s
3 3
6. A 200 W sodium street lamp emits yellow light of wavelength 11. Monochromatic radiation emitted when electron on
0.6 µm. Assuming it to be 25% efficient in converting hydrogen atom jumps from first excited to the ground state
electrical energy to light, the number of photons of yellow irradiates a photosensitive material. The stopping potential
light it emits per second is is measured to be 3.57 V. The threshold frequency of the
(a) 1.5 × 1020 (b) 6 × 1018 materials is :
(c) 62 × 10 20 (d) 3 × 1019 (a) 4 × 1015 Hz (b) 5 × 1015 Hz
7. The wavelength le of an electron and lp of a photon are of (c) 1.6 × 10 Hz15 (d) 2.5 × 1015 Hz
same energy E are related by 12. When the energy of the incident radiation is incredased by
(a) l p µ l e (b) l p µ l e 20%, the kinetic energy of the photoelectrons emitted from
a metal surface increased from 0.5 eV to 0.8 eV. The work
1
(c) lp µ (d) l p µ l e2 function of the metal is :
le (a) 0.65 eV (b) 1.0 eV (c) 1.3 eV (d) 1.5 eV
8. The cathode of a photoelectric cell is changed such that the 13. The maximum velocity of an electron emitted by light of
work function changes from W1 to W2 (W2 > W1). If the wavelength l incident on the surface of a metal of work-
current before and after changes are I1 and I2, all other function f is
conditions remaining unchanged, then (assuming hn > W2)
(a) I1 = I2 (b) I1 < I2 2( hc + lf ) 2( hc + lf )
(a) (b)
(c) I1 > I2 (d) I1 < I2 < 2 I1 ml ml
9. In photoelectric effect the work function of a metal is 3.5 eV.
2( hc - lf ) 2(hl - f)
The emitted electrons can be stopped by applying a potential (c) (d)
of –1.2 V. Then ml m

RESPONSE 4. 5. 6. 7. 8.
GRID 9. 10. 11. 12. 13.

Space for Rough Work


Physics P-99

14. Which one of the following graphs represents the variation 19. Two radiations of photons energies 1 eV and 2.5 eV,
of maximum kinetic energy (EK) of the emitted electrons successively illuminate a photosensitive metallic surface of
with frequency u in photoelectric effect correctly ? work function 0.5 eV. The ratio of the maximum speeds of
(a) EK (b) EK the emitted electrons is :
(a) 1 : 4 (b) 1 : 2 (c) 1 : 1 (d) 1 : 5
20. Photoelectric emission is observed from a metallic surface
for frequencies v1 and v2 of the incident light rays (v1 > v2).
If the maximum values of kinetic energy of the photoelectrons
u u emitted in the two cases are in the ratio of 1 : k, then the
threshold frequency of the metallic surface is
(c) (d) EK
EK v1 - v 2 kv1 - v 2
(a) (b)
k -1 k -1
kv2 - v1 v 2 - v1
(c) (d)
u u k -1 k
u0
21. In an electron gun, the potential difference between the
15. If E1, E2, E3 are the respective kinetic energies of an electron,
filament and plate is 3000 V. What will be the velocity of
an alpha-particle and a proton, each having the same
electron emitting from the gun?
de-Broglie wavelength, then
(a) 3 × 108 m/s (b) 3.18 × 107 m/s
(a) E1 > E3 > E2 (b) E2 > E3 > E1 7
(c) 3.52 × 10 m/s (d) 3.26 × 107 m/s
(c) E1 > E2 > E3 (d) E1 = E2 = E3 22. A 5 watt source emits monochromatic light of wavelength
16. In photoelectric effect, stopping potential for a light of 5000 Å. When placed 0.5 m away, it liberates photoelectrons
frequency n 1 is V1. If light is replaced by another having a from a photosensitive metallic surface. When the source is
frequency n 2 then its stopping potential will be moved to a distance of 1.0 m, the number of photoelectrons
h h liberated will be reduced by a factor of
(a) V1 - ( n 2 - n1 ) (b) V1 + ( n 2 + n1 ) (a) 8 (b) 16 (c) 2 (d) 4
e e
23. X-rays are produced in X-ray tube operating at a given
h h
(c) V1 + ( n 2 - 2n1 ) (d) V1 + ( n 2 - n1 ) accelerating voltage. The wavelength of the continuous
e e X-rays has values from
17. A certain metallic surface is illuminated with monochromatic (a) 0 to ¥
light of wavelength l. The stopping potential for photo- (b) lmin to ¥, where lmin > 0
electric current for this light is 3V0. If the same surface is
(c) 0 to lmax, where lmax < ¥
illuminated with light of wavelength 2l, the stopping
potential is V0. The threshold wavelength for this surface (d) lmin to lmax, where 0 < lmin < lmax < ¥
for photo-electric effect is 24. In the photoeletric effect, electrons are emitted
(a) at a rate that is proportional to the amplitude of the
l l incident radiation
(a) 4 l (b) (c) (d) 6 l
4 6 (b) with a maximum velocity proportional to the frequency
18. The ratio of the respective de Broglie wavelengths of the incident radiation
associated with electrons accelerated from rest with the (c) at a rate that is independent of the emitter
voltages 100 V, 200 V and 300 V is (d) only if the frequency of the incident radiations is above
1 1 1 1 a certain threshold value
(a) 1 : 2 : 3 (b) 1 : 4 : 9 (c) 1: : (d) 1: :
2 3 2 3

14. 15. 16. 17. 18.


RESPONSE
19. 20. 21. 22. 23.
GRID
24.
Space for Rough Work
EBD_7504
P-100 NTA JEE Main

25. In an experiment on photoelectric effect, a student plots radiations. Which one of the following is a correct
stopping potential V0 against reciprocal of the wavelength statement?
photocurrent
l of the incident light for two different metals A and B. These
are shown in the figure.
2
V0 Metal A 3
Metal B 1

Retarding potential Anode potential


(a) Curves (1) and (2) represent incident radiations of
1/l same frequency but of different intensities.
Looking at the graphs, you can most appropriately say that: (b) Curves (2) and (3) represent incident radiations of
(a) Work function of metal B is greater than that of metal A different frequencies and different intensities.
(b) For light of certain wavelength falling on both metal, (c) Curves (2) and (3) represent incident radiations of
same frequency having same intensity.
maximum kinetic energy of electrons emitted from A
(d) Curves (1) and (2) represent incident radiations of
will be greater than those emitted from B. different frequencies and different intensities.
(c) Work function of metal A is greater than that of metal B
29. The de-Broglie wavelength of neutron in thermal
(d) Students data is not correct equilibrium at temperature T is
26. Light of wavelength 500 nm is incident on a metal with work
30.8 3.08
function 2.28 eV. The de Broglie wavelength of the emitted (a) Å (b) Å
electron is: T T
(a) < 2.8 × 10-9 m (b) ³ 2.8 × 10-9 m 0.308 0.0308
(c) £ 2.8 × 10-12 m (d) < 2.8 × 10-10 m (c) Å (d) Å
27. An electron of mass m and charge e initially at rest gets T T
accelerated by a constant electric field E. The rate of change 30. The stopping potential (V 0 ) versus V0
2
of de-Broglie wavelength of this electron at time t ignoring frequency (v) plot of a substance is
relativistic effects is shown in figure, the threshold 1
wavelength is
-h -eht - mh -h (a) 5 × 1014m
(a) 2 (b) (c) 2 (d) e E (b) 6000 Å 4 5 6 7 8
e Et E e Et v × 1014 Hz
28. The figure shows a plot of photo current versus anode (c) 5000 Å
potential for a photo sensitive surface for three different (d) Cannot be estimated from given data

RESPONSE 25. 26. 27. 28. 29.


GRID 30.

PHYSICS CHAPTERWISE SPEED TEST-25


Total Questions 30 Total Marks 120
Attempted Correct
Incorrect Net Score
Cut-off Score 40 Qualifying Score 50
Success Gap = Net Score – Qualifying Score
Net Score = (Correct × 4)Work
Space for Rough – (Incorrect × 1)
Space for Rough Work
PHYSICS Speed
Atoms TEST
No. of Questions
30
Maximum Marks
120
Time
1 Hour
26
Chapter-wise

GENERAL INSTRUCTIONS
• This test contains 30 MCQ's. For each question only one option is correct. Darken the correct circle/ bubble in the
Response Grid provided on each page.
• You have to evaluate your Response Grids yourself with the help of solutions provided at the end of this book.
• Each correct answer will get you 4 marks and 1 mark shall be deduced for each incorrect answer. No mark will be given/
deducted if no bubble is filled. Keep a timer in front of you and stop immediately at the end of 60 min.
• The sheet follows a particular syllabus. Do not attempt the sheet before you have completed your preparation for that
syllabus.
• After completing the sheet check your answers with the solution booklet and complete the Result Grid. Finally spend time
to analyse your performance and revise the areas which emerge out as weak in your evaluation.

1. In the Bohr’s model of hydrogen-like atom the force between


æ e n2 h ö æ e ö nh
the nucleus and the electron is modified as (a) ç ÷ (b) ç ÷
ç 2m 2p ÷ è m ø 2p
è ø
e2 æ 1 b ö
F= ç + ÷ , where b is a constant. For this atom,
4pe0 è r 2 r 3 ø 2
æ e ö nh æ e ön h
the radius of the nth orbit in terms of the Bohr radius (c) ç ÷ (d) ç ÷
è 2m ø 2p è m ø 2p
æ e h2 ö
ç a0 = 0 ÷ is: 3. In Rutherford scattering experiment, the number of
ç mp e 2 ÷ø a-particles scattered at 60° is 5 × 106. The number of
è
(a) rn = a0n – b (b) rn = a0n2 + b a-particles scattered at 120° will be
(c) rn = a0n2 – b (d) rn = a0n + b
3
2. In the Bohr model an electron moves in a circular orbit around (a) 15 × 106 (b) × 106
5
the proton. Considering the orbiting electron to be a circular
current loop, the magnetic moment of the hydrogen atom, 5
(c) × 106 (d) None of these
when the electron is in nth excited state, is : 9

RESPONSE GRID 1. 2. 3.
Space for Rough Work
EBD_7504
P-102 NTA JEE Main

4. Electrons in a certain energy level n = n1, can emit 3 spectral (b) 3 lines in the Lyman series
lines. When they are in another energy level, n = n 2. They (c) 1 line in the Lyman series and 2 lines in the Balmar
can emit 6 spectral lines. The orbital speed of the electrons series
in the two orbits are in the ratio of (d) 3 lines in the Balmer series
(a) 4 : 3 (b) 3 : 4 (c) 2 : 1 (d) 1 : 2 9. A Hydrogen atom and a Li++ ion are both in the second
5. Energy of an electron in an excited hydrogen atom is excited state. If lH and lLi are their respective electronic
–3.4 eV. Its angular momentum will be angular momenta, and EH and ELi their respective energies,
(a) 3.72 × 10–34Js (b) 2.10 × 10–34Js then
(c) 1.51 × 10–34 Js (d) 4.20 × 10–34Js (a) lH > lLi and |EH| > |ELi| (b) lH = lLi and |EH| < |ELi|
6. The radiation corresponding to 3 ® 2 transition of hydrogen (c) lH = lLi and |EH| > |ELi| (d) lH < lLi and |EH| < |ELi|
atom falls on a metal surface to produce photoelectrons. 10. The electric potential between a proton and an electron is
These electrons are made to enter a magnetic field of 3 × r
10–4 T. If the radius of the largest circular path followed by given by V = V0 ln , where r0 is a constant. Assuming
r0
these electrons is 10.0 mm, the work function of the metal is
close to: Bohr’s model to be applicable, write variation of rn with n, n
(a) 1.8 eV (b) 1.1 eV (c) 0.8 eV (d) 1.6 eV being the principal quantum number?
7. th
If in hydrogen atom, radius of n Bohr orbit is r n, frequency (a) rn µ n (b) rn µ 1/n
of revolution of electron in n th orbit is fn, choose the correct (c) rn µ n2 (d) rn µ 1/n2
option. 11. The largest wavelength in the ultraviolet region of the
rn
hydrogen spectrum is 122 nm. The smallest wavelength in
ær ö the infrared region of the hydrogen spectrum (to the nearest
lo g ç n ÷
è r1 ø
integer) is
(a) (b)
(a) 802 nm (b) 823 nm (c) 1882 nm (d) 1648 nm
O
12. An electron in the hydrogen atom jumps from excited state
O n log n
n to the ground state. The wavelength so emitted illuminates
æf ö a photosensitive material having work function 2.75 eV. If
log ç n ÷
è f1 ø the stopping potential of the photoelectron is 10 V, the value
(c) (d) Both (a) and (b) of n is
(a) 3 (b) 4 (c) 5 (d) 2
O log n 13. Ionization potential of hydrogen atom is 13.6eV. Hydrogen
8. A 12.5 eV electron beam is used to bombard gaseous atoms in the ground state are excited by monochromatic
hydrogen at room temperature. It will emit : radiation of photon energy 12.1 eV. According to Bohr’s
(a) 2 lines in the Lyman series and 1 line in the Balmar theory, the spectral lines emitted by hydrogen will be
series (a) three (b) four (c) one (d) fwo

RESPONSE 4. 5. 6. 7. 8.
GRID 9. 10. 11. 12. 13.
Space for Rough Work
Physics P-103

14. Electron in hydrogen atom first jumps from third excited 19. If u1 is the frequency of the series limit of Lyman series,
state to second excited state and then from second excited u2 is the frequency of the first line of Lyman series and
to the first excited state. The ratio of the wavelength l1 : l2 u3 is the frequency of the series limit of the Balmer series
emitted in the two cases is then
(a) 7/5 (b) 27/20 (c) 27/5 (d) 20/7 (a) u1 - u2 = u3 (b) u1 = u2 - u3
15. The diagram shows the energy levels for an electron in a 1 1 1 1 1 1
(c) = + (d) u = u + u
certain atom. Which transition shown represents the u2 u1 u3 1 2 3
emission of a photon with the most energy? 20. Hydrogen (1H1), Deuterium (1H2), singly ionised Helium
n=4 (2He4)+ and doubly ionised lithium (3Li6)++ all have one
n =3
electron around the nucleus. Consider an electron transition
n=2 from n = 2 to n = 1. If the wavelengths of emitted radiation
are l1, l2, l3 and l4 respectively then approximately which
one of the following is correct?
n =1
I II III IV (a) 4l1 = 2l2 = 2l3 = l4 (b) l1 = 2l2 = 2l3 = l4
(a) IV (b) III (c) II (d) I (c) l1 = l2 = 4l3 = 9l4 (d) l1 = 2l2 = 3l3 = 4l4
16. Consider 3rd orbit of He+ (Helium), using non-relativistic 21. When an a-particle of mass 'm' moving with velocity 'v'
approach, the speed of electron in this orbit will be [given bombards on a heavy nucleus of charge 'Ze', its distance of
K = 9 × 109 constant, Z = 2 and h (Plank's Constant) closest approach from the nucleus depends on m as :
= 6.6 × 10–34 J s]
(a) 1.46 × 106 m/s (b) 0.73 × 106 m/s 1 1
(a) (b)
8
(c) 3.0 × 10 m/s (d) 2.92 × 106 m/s m m
17. In Hydrogen spectrum, the wavelength of Ha line is 656 nm, 1
whereas in the spectrum of a distant galaxy, Ha line (c) (d) m
m2
wavelength is 706 nm. Estimated speed of the galaxy with
respect to earth is 22. If the atom 100Fm257 follows the Bohr model and the radius
(a) 2 × 108 m/s (b) 2 × 107m/s of 100Fm257 is n times the Bohr radius, then find n.
6
(c) 2 × 10 m/s (d) 2 × 105 m/s (a) 100 (b) 200 (c) 4 (d) 1/4
18. An electron of a stationary hydrogen atom passes from the 23. The ionisation potential of H-atom is 13.6 V. When it is excited
fifth energy level to the ground level. The velocity that the from ground state by monochromatic radiations of 970.6 Å,
atom acquired as a result of photon emission will be : (m is the the number of emission lines will be (according to Bohr’s
mass of the electron, R, Rydberg constant and h Planck’s theory)
constant) (a) 10 (b) 8 (c) 6 (d) 4
24hR 25hR 24. The energy of He+ in the ground state is – 54.4 eV, then the
(a) (b)
25m 24m energy of Li++ in the first excited state will be
25m 24m (a) – 30.6 eV (b) 27.2 eV
(c) (d)
24hR 25hR (c) – 13.6 eV (d) – 27.2 eV

RESPONSE 14. 15. 16. 17. 18.


GRID 19. 20. 21. 22. 23.
24.
Space for Rough Work
EBD_7504
P-104 NTA JEE Main

25. One of the lines in the emission spectrum of Li2+ has the 28. Suppose an electron is attracted towards the origin by a
same wavelength as that of the 2nd line of Balmer series in k
hydrogen spectrum. The electronic transition corresponding force where ‘k’ is a constant and ‘r’ is the distance of the
r
to this line is n = 12 ® n = x. Find the value of x. electron from the origin. By applying Bohr model to this
(a) 8 (b) 6 (c) 7 (c) 5 system, the radius of the nth orbital of the electron is found
26. Which of the plots shown in the figure represents speed to be ‘rn’ and the kinetic energy of the electron to be ‘Tn’.
(vn) of the electron in a hydrogen atom as a function of the Then which of the following is true?
principal quantum number (n)?
1 2
A (a) Tn µ , rn µ n (b) Tn independent of n, rn µ n
C n 2

1 1
vn (c) Tn µ , rn µ n (d) Tn µ , rn µ n 2
D n n
29. An energy of 24.6 eV is required to remove one of the
electrons from a neutral helium atom. The energy in (eV)
B required to remove both the electrons from a neutral helium
o 1 2 3 4 n atom is
(a) 38.2 (b) 49.2 (c) 51.8 (d) 79.0
(a) B (b) D (c) C (d) A
30. The electron in a hydrogen atom makes a transition from an
27. A diatomic molecule is made of two masses m1 and m2 which
excited state to the ground state. Which of the following
are separated by a distance r. If we calculate its rotational
statements is true?
energy by applying Bohr's rule of angular momentum
(a) Its kinetic energy increases and its potential and total
quantization, its energy will be given by : (n is an integer)
energies decreases.
(m1 + m2 )2 n2 h2 n2 h 2 (b) Its kinetic energy decreases, potential energy increases
(a) (b)
2m12 m22 r 2 2(m1 + m2 )r 2 and its total energy remains the same.
(c) Its kinetic and total energies decrease and its potential
2n 2 h 2 (m1 + m2 )n2 h 2 energy increases.
(c) (d)
(m1 + m2 )r 2 2m1m2 r 2 (d) Its kinetic, potential and total energies decrease.

RESPONSE 25. 26. 27. 28. 29.


GRID 30.

PHYSICS CHAPTERWISE SPEED TEST-26


Total Questions 30 Total Marks 120
Attempted Correct
Incorrect Net Score
Cut-off Score 45 Qualifying Score
Space for Rough Work
60
Success Gap = Net Score – Qualifying Score
Net Score = (Correct × 4) – (Incorrect × 1)
Space for Rough Work
PHYSICS Speed
Nuclei TEST
No. of Questions
30
Maximum Marks
120
Time
1 Hour
27
Chapter-wise

GENERAL INSTRUCTIONS
• This test contains 30 MCQ's. For each question only one option is correct. Darken the correct circle/ bubble in the
Response Grid provided on each page.
• You have to evaluate your Response Grids yourself with the help of solutions provided at the end of this book.
• Each correct answer will get you 4 marks and 1 mark shall be deduced for each incorrect answer. No mark will be given/
deducted if no bubble is filled. Keep a timer in front of you and stop immediately at the end of 60 min.
• The sheet follows a particular syllabus. Do not attempt the sheet before you have completed your preparation for that
syllabus.
• After completing the sheet check your answers with the solution booklet and complete the Result Grid. Finally spend time
to analyse your performance and revise the areas which emerge out as weak in your evaluation.

1. Binding energy per nucleon versus mass number curve for 3. Radioactive element decays to form a stable nuclide. The
nuclei is shown in the figure. W, X, Y and Z are four nuclei rate of decay of reactant is correctly depicted by
indicated on the curve. The process that would release N N
energy is
(a) Y ® 2Z (a) (b)
(b) W ® X + Z t t
(c) W ® 2Y N N
(d) X ® Y + Z
2. The decay constants of a radioactive substance for a and b (c) (d)
emission are la and lb respectively. If the substance emits t t
a and b simultaneously, then the average half life of the 4. A nucleus of mass M + Dm is at rest and decays into two
material will be M
2Ta Tb daughter nuclei of equal mass each. Speed of light is c.
2
(a) T + T (b) Ta + Tb The speed of daughter nuclei is
a b
Ta Tb Dm Dm Dm
( )
1 2 Dm
Ta + Tb (a) c (b) c (c) c (d) c
(c) Ta + Tb (d) M + Dm M M M + Dm
2

RESPONSE GRID 1. 2. 3. 4.
Space for Rough Work
EBD_7504
P-106 NTA JEE Main

5. The radioactivity of a sample is R1 at a time T1 and R2 at a (a) M (A, Z) = ZMp + (A – Z) Mn –BE/c2


time T2. If the half-life of the specimen is T, the number of (b) M (A, Z) = ZMp+ ( A–Z) Mn + BE
atoms that have disintegrated in the time (T1 – T2 ) is
proportional to (c) M (A, Z) = ZMp + (A – Z) Mn – BE
(a) (R1T1 – R2T2) (b) (R1 – R2) (d) M (A, Z) = ZMp + (A – Z)Mn + BE/c2
(c) (R1 – R2)/T (d) (R1 – R2) T 11. How does the binding energy per nucleon vary with the
6. Which of the following Statements is correct?
increase in the number of nucleons?
(a) The rate of radioactive decay cannot be controlled but
that of nuclear fission can be controlled (a) Increases continuously with mass number
(b) Nuclear forces are short range, attractive and charge (b) Decreases continuously with mass number
dependent (c) First decreases and then increases with increase in
(c) Nuclei of atoms having same number of neutrons are mass number
known as isobars (d) First increases and then decreases with increase in
(d) Wavelength of matter waves is given by deBroglie mass number
formula but that of photons is not given by the same 12. A radioactive nucleus undergoes a series of decay according
formula to the scheme
7. A nucleus disintegrates into two nuclear parts which have
a b a g
their velocities in the ratio 2 : 1. Ratio of their nuclear sizes A ¾¾
® A1 ¾¾
® A 2 ¾¾
® A 3 ¾¾
® A4
will be If the mass number and atomic number of ‘A’ are 180 and 72
(a) 21/3 : 1 (b) 1 : 31/2 (c) 31/2 : 1 (d) 1 : 21/3
respectively, then what are these numbers for A4
8. The electrons cannot exist inside the nucleus because
(a) de-Broglie wavelength associated with electron in (a) 172 and 69 (b) 174 and 70
(c) 176 and 69 (d) 176 and 70
b -decay is much less than the size of nucleus
13. If a neutron breaks into a proton and an electron then the
(b) de-Broglie wavelength associated with electron in energy released during this process is : (mass of neutron =
b -decay is much greater than the size of nucleus 1.6725 × 10–27 kg, mass of proton = 1.6725 × 10–27 kg, mass
(c) de-Broglie wavelength associated with electron in of electron = 9 × 10–31 kg)
b -decay is equal to the size of nucleus (a) 0.51 MeV (b) 7.10 MeV
(d) negative charge cannot exist in the nucleus (c) 6.30 MeV (d) 5.4 MeV
9. The energy spectrum of b-particles [Number N(E) as a 14. If radius of the 27
nucleus is taken to be RAl, then the
function of b-energy E] emitted from a radioactive source is 12 Al
125
(a) (b) radius of 53 Te nucleus is nearly:
N(E) N(E) 5 3
(a) R Al (b) R Al
E
E
3 5
E0
E0 1/3 1/3
(c) æ 13 ö (d) æç ö÷ R Al
53
(c) (d) ç ÷ R Al
N(E) N(E)
è 53 ø è 13 ø
15. Binding energy per nucleon plot against the mass number
E E for stable nuclei is shown in the figure. Which curve is
E0 E0 correct?
10. If M (A; Z), Mp and Mn denote the masses of the nucleus (a) A
A (b) B
Z X, proton and neutron respectively in units of u ( 1u =
931.5 MeV/c2) and BE represents its bonding energy in (c) C
MeV, then (d) D

5. 6. 7. 8. 9.
RESPONSE
10. 11. 12. 13. 14.
GRID
15.
Space for Rough Work
Physics P-107

16. An element A decays into element C by a two step processes 22. If 200 MeV energy is released in the fission of a single U 235
A ® B + 2He4 and B
A A®C B + 2e–. Then nucleus, the number of fissions required per second to
(a) A and C are isotopes (b) A and C are isobars produce 1 kilowatt power shall be (Given 1eV = 1.6 × 10–19 J)
(c) A and B are isotopes (d) A and B are isobars (a) 3.125 × 1013 (b) 3.125 × 1014
17. At time t = 0, N1 nuclei of decay constant l1 and N2 nuclei (c) 3.125 × 10 15 (d) 3.125 × 1016
of decay constant l2 are mixed. The decay rate of mixture is 23. In a radioactive material the activity at time t1 is R1 and at a
-(l1 + l 2 ) t later time t2, it is R2. If the decay constant of the material is
(a) - N1 N 2 e
l, then
æN ö
(b) - çç 1 ÷÷e -(l1 + l 2 ) t (a) R1 = R2 e l (t1 -t2 ) (b) R1 = R2 e(t2 / t1 )
è N2 ø
t t
(c) - ( N1l1e -l1 + N1l 2 e -l 2 ) (c) R1 = R2 (d) R1 = R2 e -l (t1 -t2 )
24. In the options given below, if E denotes the rest mass energy
(d) - N1l1N 2 l 2 e - l1 +l 2
( )t
of a nucleus and n a neutron, the correct option is
18. A radioactive nucleus of mass number A, initially at rest,
emits an a-particle with a speed v. The recoil speed of æ 236 ö æ 137 ö æ 97 ö
(a) Eç U÷ > Eç I ÷ + E ç Y ÷ + 2 E (n )
daughter nucleus is è 92 ø è 53 ø è 39 ø
2v 2v 4v 4v
(a) (b) (c) (d) æ 236 ö æ 137 ö æ 97 ö
A-4 A+4 A-4 A+4 (b) Eç U÷ < Eç I ÷ + E ç Y ÷ + 2 E (n )
19. Consider a radioactive material of half-life 1.0 minute. If one è 92 ø è 53 ø è 39 ø
of the nuclei decays now, the next one will decay
(a) after 1 minute æ 236 ö æ140 ö æ 94 ö
(c) Eç U÷ < Eç Ba ÷ + E ç Kr ÷ + 2 E (n)
1 è 92 ø è 56 ø è 36 ø
(b) after log 2 minute
e æ 236 ö æ 140 ö æ 94 ö
1 (d) Eç U÷ = Eç Ba ÷ + E ç Kr ÷ + 2 E (n)
(c) after minute, where N is the number of nuclei present è 92 ø è 56 ø è 36 ø
N
at that moment 25. If the nuclear force between two protons, two neutrons and
(d) after any time between proton and neutron is denoted by Fpp, Fnn and Fpn
20. A nucleus with atomic number Z= 92 emits the following in respectively, then
a sequence: (a) Fpp » Fnn » Fpn (b) Fpp ¹ Fnn and Fpp = Fnn
a, b - , b - a, a, a, a, a, b - , b - , a, b + , b + , a
(c) Fpp = Fnn = Fpn (d) Fpp ¹ Fnn ¹ Fpn
Then Z of the resulting nucleus is
26. In any fission process, the ratio of
(a) 76 (b) 78 (c) 82 (d) 74
mass of fission products
TA is
21. The ratio of half-life times of two elements A and B is . mass of parent nucleus
TB
(a) equal to 1
lA
The ratio of respective decay constant , is (b) greater than 1
lB
(a) TB / TA (b) TA / TB (c) less than 1
TA + TB TA - TB (d) depends on the mass of the parent nucleus
(c) (d)
TA TA

RESPONSE 16. 17. 18. 19. 20.


GRID 21. 22. 23. 24. 25.
26.
Space for Rough Work
EBD_7504
P-108 NTA JEE Main

27. 28. Which one is correct about fission?


(a) Approx. 0.1% mass converts into energy
(b) Most of energy of fission is in the form of heat
(c) In a fission of U 23 5 about 200 eV energy is
released
(d) On an average, one neutron is released per fission
of U235
29. A neutron travelling with a velocity v and kinetic
energy E has a perfectly elastic head-on collision with
a nucleus of an atom of mass number A at rest. The
fraction of total energy retained by the neutron is
In the above plot of binding energy per nucleon E b, against
approximately
the nuclear mass M; A, B, C, D, E, F correspond to different
(a) [(A – 1)/(A + 1)]2 (b) [(A + 1)/(A – 1)]2
nuclei. Consider four reactions : 2
(c) [(A – 1)/A] (d) [(A + 1)/A]2
(i) A + B ® C + e (ii) C ® A + B + e
30. The correct relation between t av = average life and
(iii) D + E ® F + e (iv) F® D + E + e,
t 1/2 = half life for a radioactive nuclei.
where e is the energy released? In which reactions is e
positive? 1
(a) t av = t 1/2 (b) t av = t
(a) (i) and (iii) (b) (ii) and (iv) 2 1/2
(c) (ii) and (iii) (d) (i) and (iv) (c) 0.693 t av = t 1/2 (d) t av = 0.693 t 1/2

RESPONSE GRID 27. 28. 29. 30.

PHYSICS CHAPTERWISE SPEED TEST-27


Total Questions 30 Total Marks 120
Attempted Correct
Incorrect Net Score
Cut-off Score 45 Qualifying Score 60
Success Gap = Net Score – Qualifying Score
Net Score = (Correct × 4) – (Incorrect × 1)
Space for Rough Work
PHYSICS Speed
Semiconductor Electronics: Materials, TEST
Devices and Simple Circuits
No. of Questions
30
Maximum Marks
120
Time
1 Hour
28
Chapter-wise

GENERAL INSTRUCTIONS
• This test contains 30 MCQ's. For each question only one option is correct. Darken the correct circle/ bubble in the
Response Grid provided on each page.
• You have to evaluate your Response Grids yourself with the help of solutions provided at the end of this book.
• Each correct answer will get you 4 marks and 1 mark shall be deduced for each incorrect answer. No mark will be given/
deducted if no bubble is filled. Keep a timer in front of you and stop immediately at the end of 60 min.
• The sheet follows a particular syllabus. Do not attempt the sheet before you have completed your preparation for that
syllabus.
• After completing the sheet check your answers with the solution booklet and complete the Result Grid. Finally spend time
to analyse your performance and revise the areas which emerge out as weak in your evaluation.

1. A pure semiconductor has equal electron and hole (a) OR gate (b) XOR gate
concentration of 1016 m–3. Doping by indium increases (c) AND gate (d) NAND gate
number of hole concentration n h to 5 × 1022 m–3. Then, the
value of number of electron concentration n e in the doped 4. Which of the following statements is incorrect?
semiconductor is (a) The resistance of intrinsic semiconductors decrease
(a) 106/m3 (b) 1022/m3 with increase of temperature
(c) 2 × 106/m3 (d) 2 × 109/m3
2. A change of 8.0 mA in the emitter current bring a change of (b) Doping pure Si with trivalent impurities give p-type
7.9 mA in the collector current. The values of parameters a semiconductors
and b are respectively (c) The majority carriers in n-type semiconductors are holes
(a) 0.99, 90 (b) 0.96,79 (c) 0.97,99 (d) 0.99,79 (d) A p-n junction can act as a semiconductor diode
3. The following circut diagram represents
5. In a npn transistor 1010 electrons enter the emitter in
A 10–6 s. 4% of the electrons are lost in the base. The current
transfer ratio will be
Y
(a) 0.98 (b) 0.97
B (c) 0.96 (d) 0.94

RESPONSE GRID 1. 2. 3. 4. 5.
Space for Rough Work
EBD_7504
P-110 NTA JEE Main

6. The correct truth table for system of four NAND gates as 8. In common emitter amplifier, the current gain is 62. The
shown in figure is : collector resistance and input resistance are 5 kW an 500W
respectively. If the input voltage is 0.01 V, the output voltage
A
is
(a) 0.62 V (b) 6.2 V (c) 62 V (d) 620 V
9. The circuit diagram shows a logic combination with the
Y states of outputs X, Y and Z given for inputs P, Q, R and S
all at state 1. When inputs P and R change to state 0 with
inputs Q and S still at 1, the states of outputs X, Y and Z
B change to
P(1)
A B Y A B Y X(1)
Q(1)
0 0 0 0 0 0
Z(0)
0 1 1 0 1 0
(a) (b) R(1)
1 0 1 1 0 1 Y(1)
S(1)
1 1 0 1 1 1
(a) 1, 0, 0 (b) 1, 1, 1 (c) 0, 1, 0 (d) 0, 0, 1
A B Y A B Y 10. A sinusoidal voltage of amplitude 25 volt and frequency
0 0 1 0 0 1 50Hz is applied to a half wave rectifier using P-n junction
diode. No filter is used and the load resistance is 1000W.
0 1 1 0 1 0 The forward resistance Rf of ideal diode is 10W. The
(c) (d)
1 0 0 1 0 1 percentage efficiency of rectifier is
1 1 0 1 1 1 (a) 40% (b) 20% (c) 30% (d) 15%
7. A Zener diode is connected to a battery and a load as show 11. The intrinsic conductivity of germanium at 27° is 2.13 mho
below: m–1 and mobilities of electrons and holes are 0.38 and 0.18
m2V–1s–1 respectively. The density of charge carriers is
4 kW A IL (a) 2.37 × 1019 m–3 (b) 3.28 × 1019 m–3
(c) 7.83 × 1019 m–3 (d) 8.47 × 1019 m–3
IZ 12. The circuit has two oppositively connected ideal diodes in
I
10 V = VZ RL = 2kW parallel. The current flowing in the circuit is
60 V
4W

B D1 D2
The currents, I, IZ and IL are respectively.
12V
(a) 15 mA, 5 mA, 10 mA 3W 2W
(b) 15 mA, 7.5 mA, 7.5 mA
(c) 12.5 mA, 5 mA, 7.5 mA
(d) 12.5 mA, 7.5 mA, 5 mA (a) 1.71 A (b) 2.00 A (c) 2.31 A (d) 1.33 A

RESPONSE 6. 7. 8. 9. 10.
GRID 11. 12.
Space for Rough Work
Physics P-111

13. A working transistor with its three legs marked P, Q and R is 19. Which of the junction diodes shown below are forward
tested using a multimeter. No conduction is found between biased?
P and Q. By connecting the common (negative) terminal of –10 V +10 V
the multimeter to R and the other (positive) terminal to P or
Q, some resistance is seen on the multimeter. Which of the R R
following is true for the transistor?
(a) It is an npn transistor with R as base (a) (b)
(b) It is a pnp transistor with R as base
(c) It is a pnp transistor with R as emitter –5 V +5 V
(d) It is an npn transistor with R as collector R R
14. The diagram of a logic circuit is given below. The output F –10 V –5 V
of the circuit is represented by
(c) (d)
W
X
F
W 20. For LED’s to emit light in visible region of electromagnetic
Y
light, it should have energy band gap in the range of:
(a) 0.1 eV to 0.4 eV (b) 0.5 eV to 0.8 eV
(a) W . (X + Y) (b) W . (X . Y)
(c) W + (X . Y) (d) W + (X + Y) (c) 0.9 eV to 1.6 eV (d) 1.7 eV to 3.0 eV
15. The concentration of hole-electron pairs in pure silicon at 21. In a CE transistor amplifier, the audio signal voltage across
T = 300 K is 7 × 1015 per cubic meter. Antimony is doped the collector resistance of 2kW is 2V. If the base resistance
into silicon in a proportion of 1 atom in 107 Si atoms. is 1kW and the current amplification of the transistor is 100,
Assuming half of the impurity atoms contribute electron in the input signal voltage is :
the conduction band, calculate the factor by which the (a) 0.1 V (b) 1.0 V (c) 1 mV (d) 10 mV
number of charge carriers increases due to doping. The
22. The current gain in the common emitter mode of a transistor
number of silicon atoms per cubic meter is 5 × 1028
(a) 2.8 × 105 (b) 3.1 × 102 is 10. The input impedance is 20kW and load of resistance is
(c) 4.2 × 10 5 (d) 1.8 × 105 100kW. The power gain is
16. For a transistor amplifier in common emitter configuration (a) 300 (b) 500 (c) 200 (d) 100
for load impedance of 1kW (hfe = 50 and h0e = 25) the current 23. In the circuit given below, A and B represent two inputs and
gain is C represents the output.
(a) – 24.8 (b) – 15.7 (c) – 5.2 (d) – 48.78
A
17. A PN-junction has a thickness of the order of
(a) 1 cm (b) 1mm (c) 10–6 m (d) 10–12 cm C
18. In a common emitter (CE) amplifier having a voltage gain G,
the transistor used has transconductance 0.03 mho and B
current gain 25. If the above transistor is replaced with
another one with transconductance 0.02 mho and current
gain 20, the voltage gain will be The circuit represents
1 5 2 (a) NOR gate (b) AND gate
(a) 1.5 G (b) G (c) G (d) G (c) NAND gate (d) OR gate
3 4 3

RESPONSE 13. 14. 15. 16. 17.


GRID 18. 19. 20. 21. 22.
23.
Space for Rough Work
EBD_7504
P-112 NTA JEE Main

24. If the lattice constant of this semiconductor is decreased, 27. The following configuration of gate is equivalent to
then which of the following is correct? A OR
conduction B
Ec
band width Y
band gap Eg AND
valence NAND
band width Ev
(a) NAND gate (b) XOR gate
(a) All Ec, Eg, Ev increase (c) OR gate (d) NOR gate
(b) Ec and Ev increase, but Eg decreases 28. A piece of copper and another of germanium are cooled
(c) Ec and Ev decrease, but Eg increases from room temperature to 77K. The resistance of
(a) copper increases and germanium decreases
(d) All Ec, Eg, Ev decrease (b) each of them decreases
25. The ratio of electron and hole currents in a semiconductor (c) each of them increases
is 7/4 and the ratio of drift velocities of electrons and holes (d) copper decreases and germanium increases
is 5/4, then the ratio of concentrations of electrons and holes 29. Figure shows a circuit in which three identical diodes are
will be used. Each diode has forward resistance of 20 W and infinite
(a) 5/7 (b) 7/5 (c) 25/49 (d) 49/25 backward resistance. Resistors R1 = R2 = R3 = 50 W. Battery
26. The I-V characteristic of a P-N junction diode is shown voltage is 6 V. The current through R3 is :
below. The approximate dynamic resistance of the p-n
junction when a forward bias voltage of 2 volt is applied is D1 R1
I (mA)
D2 D3 R2
800
+ –
6V R3
400
(a) 50 mA (b) 100 mA (c) 60 mA (d) 25 mA
30. The current gain for a transistor working as common-base
2 2.1 V (volt) amplifier is 0.96. If the emitter current is 7.2 mA, then the
base current is
(a) 1 W (b) 0.25 W (c) 0.5 W (d) 5 W (a) 0.29 mA (b) 0.35 mA (c) 0.39 mA (d) 0.43 mA

RESPONSE 24. 25. 26. 27. 28.


GRID 29. 30.

PHYSICS CHAPTERWISE SPEED TEST-28


Total Questions 30 Total Marks 120
Attempted Correct
Incorrect Net Score
Cut-off Score 45 Qualifying Score 60
Space for Rough Work
Success Gap = Net Score – Qualifying Score
Net Score = (Correct × 4) – (Incorrect × 1)
Space for Rough Work
PHYSICS Speed
Communication System TEST
No. of Questions
30
Maximum Marks
120
Time
1 Hour
29
Chapter-wise

GENERAL INSTRUCTIONS
• This test contains 30 MCQ's. For each question only one option is correct. Darken the correct circle/ bubble in the
Response Grid provided on each page.
• You have to evaluate your Response Grids yourself with the help of solutions provided at the end of this book.
• Each correct answer will get you 4 marks and 1 mark shall be deduced for each incorrect answer. No mark will be given/
deducted if no bubble is filled. Keep a timer in front of you and stop immediately at the end of 60 min.
• The sheet follows a particular syllabus. Do not attempt the sheet before you have completed your preparation for that
syllabus.
• After completing the sheet check your answers with the solution booklet and complete the Result Grid. Finally spend time
to analyse your performance and revise the areas which emerge out as weak in your evaluation.

1. If the heights of transmitting and the receiving antennas are


each equal to h, the maximum line-of-sight distance between (a) w c and wc2 + w m
2 (b) wc, wc + wm and wc – wm
them is (R is the radius of earth)
(c) wc and wm (d) wc and wc wm
(a) 2Rh (b) 4Rh
3. A 1000 kHz carrier wave is modulated by an audio signal of
(c) 6Rh (d) 8Rh frequency range 100 – 5000 Hz. Then the width of the channel
2. Given the electric field of a complete amplitude modulated in kHz is
wave as (a) 5 (b) 10 (c) 20 (d) 40
æ ö 4. A device with input x(t) and output y(t) is characterized by:
ˆ c 1 + Em cos wm t cos wc t .
®
E = iE çè ÷ y(t) = x2 (t). An FM signal with frequency deviation of 90
Ec ø kHz and modulating signal bandwidth of 5 kHz is applid to
Where the subscript c stands for the carrier wave and m for this device. The bandwidth of the output signal is
the modulating signal. The frequencies present in the (a) 370 kHz (b) 190 kHz
modulated wave are (c) 380 kHz (d) 95 kHz

RESPONSE GRID 1. 2. 3. 4.
Space for Rough Work
EBD_7504
P-114 NTA JEE Main

5. An amplitude modulated voltage is expressed as 10. The frequency deviation in a FM transmission is 18.75 KHz.
e = 10 (1 + 0.8 cos 2000 pt) cos 3 × 106 pt volt. The peak If it broadcasts in 88-108 MHz band, then the percentage
voltage of carrier wave is modulation is
(a) 5 V (b) 8 V (c) 10 V (d) 100 V (a) 10% (b) 25%
6. The power radiated from antenna of length l is proportional (c) 50% (d) 75%
to 11. An AM- signal is given as
2 3
xAM (t) = 100 [p(t) + 0.5g(t)] cos wct in interval 0 £ t £ 1. One
ælö ælö ælö l set of possible values of the modulating signal and
(a) çl÷ (b) çl÷ (c) çl÷ (d)
è ø è ø è ø l modulation index would be
(a) t, 0.5 (b) t, 1.0
7. The mobile telephones operate typically in the range of
(c) t, 1.5 (d) t2, 2.0
(a) 1–100 MHz (b) 100–200 MHz
(c) 1000–2000 MHz (d) 800–950 MHz 12. Degree of modulation
8. Depth of modulation (ma) in terms of Emax and Emin is (a) can take any value
(a) ma = Emax + Emin/Emin (b) should be less than 100%
(b) ma = Emax – Emin/Emax (c) should exceed 100%
(c) ma = Emax – Emin/Emax + Emin (d) exactly 100%
(d) ma = Emax + Emin/Emax – Emin 13. What is the modulation index of an over modulated wave
9. Television signals on earth cannot be received at distances (a) 1 (b) Zero (c) < 1 (d) > 1
greater than 100 km from the transmission station. The reason 14. In earth’s atmosphere, for F2-layer, the virtual height and
behind this is that critical frequency in night time are
(a) the receiver antenna is unable to detect the signal at a (a) 210 km and 5 MHz (b) 250 km and 6 MHz
disance greater than 100 km (c) 280 km and 7 MHz (d) 350 km and 6 MHz
(b) the TV programme consists of both audio and video 15. The maximum peak to peak voltage of an AM wire is 24 mV
signals and the minimum peak to peak voltage is 8 mV. The
modulation factor is
(c) the TV signals are less powerful than radio signals (a) 10% (b) 20%
(d) the surface of earth is curved like a sphere (c) 25% (d) 50%

5. 6. 7. 8. 9.
RESPONSE
10. 11. 12. 13. 14.
GRID
15.

Space for Rough Work


Physics P-115

16. For good demodulation of AM signal of carrier frequency f, 21. A radio station has two channels. One is AM at 1020 kHz
the value of RC should be and the other FM at 89.5 MHz. For good results you will use
1 1 (a) longer antenna for the AM channel and shorter for the
(a) RC = (b) RC < FM
f f
(b) shorter antenna for the AM channel and longer for the
1 1 FM
(c) RC ³ (d) RC >>
f f (c) same length antenna will work for both
17. Array gain of an antenna is (d) information given is not enough
(a) directly proportional to power radiated by isotropic 22. For sky wave propagation of a 10 MHz signal, what should
antenna be the minimum electron density in ionosphere
(b) invesely proportional to power radiated by isotropic (a) ~ 1.2 × 1012 m–3 (b) ~ 106 m–3
antenna (c) ~ 10 m 14 –3 (d) ~ 1022 m–3
(c) directly proportional to power radiated by practical 23. A broadcast radio transmitter radiates 12 kW when
antenna percentage of modulation is 50%, then the unmodulated
(d) inversely proportional to square of power radiated by carrier power is
practical antenna. (a) 5.67 kW (b) 7.15 kW
18. An audio signal represented as 25 sin 2p(2000 t) amplitude (c) 9.6 kW (d) 12 kW
modulated by a carrier wave : 60 sin 2p(100, 000)t. The 24. Which of the following statement is NOT correct?
modulation index of the modulated signal is (a) Ground wave signals are more stable than the sky wave
(a) 25% (b) 41.6 % (c) 50 % (d) 75 % signals.
19. A radio transmitter transmits at 830 kHz. At a certain (b) The critical frequency of an ionospheric layer is the
distance from the transmitter magnetic field has amplitude highest frequency that will be reflected back by the
4.82 × 10–11T. The electric field and the wavelength are layer when it is vertically incident.
respectively (c) Electromagnetic waves of frequencies higher than
(a) 0.014 N/C, 36 m (b) 0.14 N/C, 36 m about 30 MHz cannot penetrate the ionosphere.
(d) Sky wave signals in the broadcast frequency range are
(c) 0.14 N/C, 360 m (d) 0.014 N/C, 360 m
stronger at night than in the day time.
20. A carrier frequency of 1.5 MHz and peak value of 50 V is
25. If a carrier wave c(t) = A sin wct is amplitude modulated by a
amplitude modulated with a signal frequency of 10 kHz
modulator signal m(t) = A sin wmt then the equation of
producing 50% modulation. The lower and upper side-band
modulated signal [Cm(t)] and its modulation index are
frequencies in kHz are
respectively
(a) 1490, 1510 (b) 1510, 1490
(a) Cm (t) = A (1 + sin wm t) sin wc t and 2
1 1 1 1
(c) , (d) , (b) Cm (t) = A (1 + sin wm t) sin wm t and 1
1490 1510 1510 1490
(c) Cm (t) = A (1 + sin wm t) sin wc t and 1
(d) Cm (t) = A (1 + sin wc t) sin wm t and 2

RESPONSE 16. 17. 18. 19. 20.


GRID 21. 22. 23. 24. 25.

Space for Rough Work


EBD_7504
P-116 NTA JEE Main

26. Modulation is the process of superposing xAM (t) = 10 (1 + 0.5 sin 2pfmt) cos 2pfct
(a) low frequency audio signal on high frequency waves The average side-band power for the AM signal given above
(b) low frequency radio signal on low frequency audio is
waves (a) 25 (b) 12.5
(c) high frequency audio signal on low frequency radio (c) 6.25 (d) 3.125
waves 29. When radio waves passes through ionosphere, phase
(d) low frequency audio signal on low frequency radio difference between space current and capacitive
waves displacement current is
27. Calculate the power developed by an amplitude modulated (a) 0 rad (b) (3p /2) rad
wave in a load resistance of 100 W, if the peak voltage of (c) (p / 2) rad (d) p rad
carrier wave is 100 V and modulation index is 0.4. 30. An antenna has a radiation resistance of 68W, a load
(a) 50 watt (b) 54 watt resistance of 10W, and power gain of 16. The directive gain
(c) 104 watt (d) 4 watt of the antenna is
28. Consider the following amplitude modulated (AM) signal, (a) 15 (b) 16.02
where fm < B (c) 17.08 (d) 18.35

RESPONSE GRID 26. 27. 28. 29. 30.

PHYSICS CHAPTERWISE SPEED TEST-29


Total Questions 30 Total Marks 120
Attempted Correct
Incorrect Net Score
Cut-off Score 45 Qualifying Score 60
Success Gap = Net Score – Qualifying Score
Net Score = (Correct × 4) – (Incorrect × 1)
Space for Rough Work
CHEMISTRY Speed
Some Basic Concepts of Chemistry TEST
No. of Questions
30
Maximum Marks
120
Time
1 Hour
30
Chapter-wise

GENERAL INSTRUCTIONS
• This test contains 30 MCQ's. For each question only one option is correct. Darken the correct circle/ bubble in the
Response Grid provided on each page.
• You have to evaluate your Response Grids yourself with the help of solutions provided at the end of this book.
• Each correct answer will get you 4 marks and 1 mark shall be deduced for each incorrect answer. No mark will be given/
deducted if no bubble is filled. Keep a timer in front of you and stop immediately at the end of 60 min.
• The sheet follows a particular syllabus. Do not attempt the sheet before you have completed your preparation for that
syllabus.
• After completing the sheet check your answers with the solution booklet and complete the Result Grid. Finally spend time
to analyse your performance and revise the areas which emerge out as weak in your evaluation.

1. In compound A, 1.00g of nitrogen unites with 0.57g of (a) 2.88 × 10–3 (b) 28.8 × 10–3
oxygen. In compound B, 2.00g of nitrogen combines (c) 288 × 10–3 (d) 28.8 × 103
with 2.24g of oxygen. In compound C, 3.00g of nitrogen 3. What volume of hydrogen gas, at 273 K and 1 atm. pressure
combines with 5.11g of oxygen. These results obey the will be consumed in obtaining 21.6 g of elemental boron
following law (atomic mass = 10.8) from the reduction of boron trichloride
(a) law of constant proportion by hydrogen ?
(b) law of multiple proportion (a) 67.2 L (b) 44.8 L
(c) law of reciprocal proportion (c) 22.4 L (d) 89.6 L
(d) Dalton’s law of partial pressure 4. Number of g of oxygen in 32.2 g Na2SO4.10 H2O is
2. 1021 molecules are removed from 200 mg of CO2. The moles (a) 20.8 (b) 2.24
of CO2 left are : (c) 22.4 (d) 2.08

RESPONSE GRID 1. 2. 3. 4.

Space for Rough Work


EBD_7504
C-2 NTA JEE Main

5. 6.02 × 1020 molecules of urea are present in 100 ml of its (a) 1.45 (b) 1.64
solution. The concentration of urea solution is (c) 1.88 (d) 1.22
(a) 0.02 M (b) 0.01 M 11. A gas occupies a volume of 300 cc at 27°C and 620 mm
(c) 0.001 M (d) 0.1 M pressure. The volume of gas at 47°C and 640 mm pressure
(Avogadro constant, NA = 6.02 × 1023 mol–1) is:
6. If we consider that 1/6, in place of 1/12, mass of carbon atom (a) 260 cc (b) 310 cc
is taken to be the relative atomic mass unit, the mass of one
(c) 390 cc (d) 450 cc
mole of the substance will
12. Haemoglobin contains 0.33% of iron by weight. The
(a) be a function of the molecular mass of the substance
molecular weight of haemoglobin is approximately 67200.
(b) remain unchanged
The number of iron atoms (at. wt. of Fe = 56) present in one
(c) increase two fold molecule of haemoglobin is
(d) decrease twice
(a) 6 (b) 1
7. The weight of NaCl decomposed by 4.9g of H2SO4, if 6 g of
(c) 2 (d) 4
sodium hydrogen sulphate and 1.825 g of HCl, were
produced in the reaction is: 13. The volume of 20 volume H2O2 required to get 5 litres of O2
at STP is
(a) 6.921 g (b) 4.65 g
(c) 2.925 g (d) 1.4 g (a) 250 ml (b) 125 ml
8. Which one of the following is the lightest? (c) 100 ml (d) 50 ml.
(a) 0.2 mole of hydrogen gas 14. In the reaction,
(b) 6.023 × 1022 molecules of nitrogen 2Al(s) + 6HCl(aq) ® 2Al3+ (aq) + 6Cl - (aq) + 3H 2 (g)
(c) 0.1 g of silver
(a) 11.2 L H2(g) at STP is produced for every mole HCl(aq)
(d) 0.1 mole of oxygen gas
consumed
9. How many moles of magnesium phosphate, Mg 3(PO4)2 will
(b) 6 L HCl(aq) is consumed for every 3 L H2(g) produced
contain 0.25 mole of oxygen atoms?
(a) 1.25 × 10–2 (b) 2.5 × 10–2 (c) 33.6 L H2(g) is produced regardless of temperature and
pressure for every mole Al that reacts
(c) 0.02 (d) 3.125 × 10–2
10. The density (in g mL–1) of a 3.60 M sulphuric acid solution (d) 67.2 H2(g) at STP is produced for every mole Al that
that is 29% H2SO4 (molar mass = 98 g mol–1) by mass will be reacts.

RESPONSE 5. 6. 7. 8. 9.
GRID 10. 11. 12. 13. 14.
Space for Rough Work
Chemistry C-3

15. The concentrated sulphuric acid that is peddled commercial On complete reaction with excess HCl, one kilogram of the
is 95% H2SO4 by weight. If the density of this commercial washing soda will evolve:
acid is 1.834 g cm–3, the molarity of this solution is (a) 9 mol of CO2 (b) 16 mol of CO2
(a) 17.8 M (b) 12.0 M (c) 17 mol of CO2 (d) 18 mol of CO2
(c) 10.5 M (d) 15.7 M
21. Arrange the numbers in increasing no. of significant figures.
16. What is the mass of precipitate formed when 50 mL of 16.9%
0.002600, 2.6000, 2.6, 0.260
solution of AgNO3 is mixed with 50 mL of 5.8% NaCl solution ?
(a) 2.6 < 0.260 < 0.002600 < 2.6000
(Ag = 107.8, N = 14, O = 16, Na = 23, Cl = 35.5)
(a) 28 g (b) 3.5 g (b) 2.6000 < 2.6 < 0.002600 < 0.260

(c) 7 g (d) 14 g (c) 0.260 < 2.6 < 0.002600 < 2.6000
17. Number of valence electrons in 4.2 gram of N3– ion is (d) 0.002600 < 0.260 < 2.6 < 2.6000
(a) 4.2 NA (b) 0.1 NA 22. Dissolving 120 g of a compound (mol. wt. 60) in 1000 g of
(c) 1.6 NA (d) 3.2 NA water gave a solution of density 1.12 g/mL. The molarity of
the solution is:
18. A transition metal M forms a volatile chloride which has a
vapour density of 94.8. If it contains 74.75% of chlorine the (a) 1.00 M (b) 2.00 M
formula of the metal chloride will be (c) 2.50 M (d) 4.00 M
(a) MCl3 (b) MCl2 23. A gaseous compound of nitrogen and hydrogen contains
(c) MCl4 (d) MCl5 12.5% (by mass) of hydrogen. The density of the compound
19. A gaseous hydrocarbon gives upon combustion 0.72 g of relative to hydrogen is 16. The molecular formula of the
water and 3.08 g. of CO2. The empirical formula of the compound is:
hydrocarbon is : (a) NH2 (b) N3H
(a) C2H4 (b) C3H4 (c) NH3 (d) N2H4
(c) C6H5 (d) C7H8 24. The amount of BaSO4 formed upon mixing 100 mL of 20.8%
20. Following is the composition of a washing soda sample : BaCl2 solution with 50 mL of 9.8% H2SO4 solution with
Substance Molecular Wt. Mass percent 50 mL of 9.8% H2SO4 solution will be:
Na2CO3 106.0 84.8 (Ba = 137, Cl = 35.5, S = 32, H = 1 and O = 16)
NaHCO3 84.0 8.4 (a) 23.3 g (b) 11.65 g
NaCl 58.5 6.8 (c) 30.6 g (d) 33.2 g

RESPONSE 15. 16. 17. 18. 19.


GRID 20. 21. 22. 23. 24.
Space for Rough Work
EBD_7504
C-4 NTA JEE Main

25. 2 g of a mixture of CO and CO2 on reaction with excess I2O5 (a) 2.8 kg (b) 6.4 kg
produced 2.54 g of I 2 . What will be the mass % of CO 2 in (c) 9.6 kg (d) 96 kg
the original mixture ? 29. A gas mixture of 3 litres of propane (C3 H8 ) and butane
(a) 35 (b) 70 (C 4 H10 ) on complete combustion at 25° C produced
(c) 30 (d) 60 10 litre CO 2 . Find out the composition of gas mixture
26. 7.5 grams of a gas occupy 5.6 litres of volume at STP. The (Propane : Butane)
gas is (a) 2 : 1 (b) 1 : 2
(a) N2O (b) NO (c) 1.5 : 1.5 (d) 0.5 : 2.5
30. An organic compound contains 49.3% carbon, 6.84%
(c) CO (d) CO2
hydrogen and its vapour density is 73. Molecular formula
27. Number of moles of KMnO4 required to oxidize one mole of
of the compound is :
Fe(C2O4) in acidic medium is
(a) 0.167 (b) 0.6 (a) C 3H 5O 2 (b) C 4 H10 O 2
(c) 0.2 (d) 0.4 (c) C 6 H10 O 4 (d) C 3 H10 O 2
28. What is the weight of oxygen required for the complete
combustion of 2.8 kg of ethylene ?

RESPONSE 25. 26. 27. 28. 29.


GRID 30.

CHEMISTRY CHAPTERWISE SPEED TEST-30


Total Questions 30 Total Marks 120
Attempted Correct
Incorrect Net Score
Cut-off Score 37 Qualifying Score 52
Success Gap = Net Score – Qualifying Score
Net Score = (Correct × 4) – (Incorrect × 1)
Space for Rough Work
CHEMISTRY Speed
Structure of Atom TEST
No. of Questions
30
Maximum Marks
120
Time
1 Hour
31
Chapter-wise

GENERAL INSTRUCTIONS
• This test contains 30 MCQ's. For each question only one option is correct. Darken the correct circle/ bubble in the
Response Grid provided on each page.
• You have to evaluate your Response Grids yourself with the help of solutions provided at the end of this book.
• Each correct answer will get you 4 marks and 1 mark shall be deduced for each incorrect answer. No mark will be given/
deducted if no bubble is filled. Keep a timer in front of you and stop immediately at the end of 60 min.
• The sheet follows a particular syllabus. Do not attempt the sheet before you have completed your preparation for that
syllabus.
• After completing the sheet check your answers with the solution booklet and complete the Result Grid. Finally spend time
to analyse your performance and revise the areas which emerge out as weak in your evaluation.

1. Among the following groupings which represents the 3. The de-Broglie wavelength of an electron in the ground
collection of isoelectronic species? state of hydrogen atom is : [K.E. = 13.6 eV; 1eV =
1.602 ´ 10 -19 J]
(a) NO + , C 22 - , O -2 , CO (b) N 2 , C 22 - , CO, NO
(a) 33.28 nm (b) 3.328 nm
(c) CO, NO + , CN - , C 22 - (d) NO, CN - , N 2 , O -2 (c) 0.3328 nm (d) 0.0332 nm
2. The compound in which cation is isoelectronic with anion 4. The frequency of light emitted for the transition n = 4 to
is : n = 2 of the He+ is equal to the transition in H atom
corresponding to which of the following ?
(a) NaCl (b) CsF
(a) n = 2 to n = 1 (b) n = 3 to n = 2
(c) NaI (d) K2S
(c) n = 4 to n = 3 (d) n = 3 to n = 1

RESPONSE GRID 1. 2. 3. 4.

Space for Rough Work


EBD_7504
C-6 NTA JEE Main

5. The first emission line in the atomic spectrum of hydrogen 11. Energy levels, A, B, C, of a certain atom correspond to
in the Balmer series appears at increasing values of energy i.e., EA < EB < EC. If l1, l2, l3
9R 7R are the wave lengths of radiations corresponding to the
(a) cm -1 (b) cm -1 transition from C to B, B to A and C to A respectively, which
400 144
of the following statements is correct ?
3R 5R
(c) cm -1 (d) cm -1 C
4 36 l1
6. In hydrogen atomic spectrum, a series limit is found at B
12186.3 cm–1. Then it belong to
(a) Lyman series (b) Balmer series l2 l3
(c) Paschen series (d) Brackett series
A
7. Two fast moving particles X and Y are associated with
l1l 2
de Broglie wavelengths 1 nm and 4 nm respectively. If mass (a) l3 = l1 + l2 (b) l3 = l + l
of X in nine times the mass of Y, the ratio of kinetic energies 1 2

of X and Y would be (c) l1 + l2 + l3 = 0 (d) l32 = l12 + l 22


(a) 3 : 1 (b) 9 : 1 12. If uncertainty in position and momentum are equal, then
(c) 5 : 12 (d) 16 : 9 uncertainty in velocity is :
8. The ratio of magnetic moments of Fe(III) and Co(II) is 1 h h
(a) 7 : 3 (b) 3 : 7 (a) (b)
2m p 2p
(c) 7 : 3 (d) 3 : 7 1 h h
(c) (d)
9. The values of Planck's constant is 6.63 × 10–34 Js. The m p p
velocity of light is 3.0 × 108 m s–1. Which value is closest to 13. The electrons, identified by quantum numbers n and l (i) n
the wavelength in nanometres of a quantum of light with = 4, l = 1 (ii) n = 4, l = 0 (iii) n = 3, l = 2 (iv) n = 3,
frequency of 8 × 1015 s–1? l = 1 can be placed in order of increasing energy, from the
(a) 5 × 10–18 (b) 4 × 101 lowest to highest, as
(c) 3 × 107 (d) 2 × 10–25 (a) (iv) < (ii) < (iii) < (i) (b) (ii) < (iv) < (i) < (iii)
10. Li and a proton are accelerated by the same potential, their (c) (i) < (iii) < (ii) < (iv) (d) (iii) < (i) < (iv) < (ii)
de Broglie wavelengths lLi and lp have the ratio (assume 14. Ionisation energy of He is 19.6 × 10–18 J atom–1. The energy
+

mLi = 9mp) of the first stationary state (n = 1) of Li2+ is


(a) 1 : 2 (b) 1 : 4 (a) 4.41 × 10–16 J atom–1 (b) –4.41 × 10–17 J atom–1
(c) –2.2 × 10–15 J atom–1 (d) 8.82 × 10–17 J atom–1
(c) 1 : 1 (d) 1 : 3 3

RESPONSE 5. 6. 7. 8. 9.
GRID 10. 11. 12. 13. 14.
Space for Rough Work
Chemistry C-7

15. The kinetic energy of an electron in the second Bohr orbit (a) BO 33 - , CO 32 - , NO 3-
of a hydrogen atom is [ a0 is Bohr radius] :
h2 h2 (b) SO32 - , CO32 - , NO3-
(a) (b)
4p 2 ma02 16p 2 ma02 (c) CN - , N 2 , C 22 -
h2 h2
(c) (d) (d) PO 34 - , SO 24 - , ClO -4
2
32p 2
ma02 64p ma02
20. The ionization enthalpy of hydrogen atom is
æ Z2 ö 1.312 × 106 J mol–1. The energy required to excite the electron
16. Energy of an electron is given by E = – 2.178 × 10-18 J ç ÷.
è n2 ø in the atom from n = 1 to n = 2 is
(a) 8.51 × 105 J mol–1 (b) 6.56 × 105 J mol–1
Wavelength of light required to excite an electron in an
5
(c) 7.56 × 10 J mol –1 (d) 9.84 × 105 J mol–1
hydrogen atom from level n = 1 to n = 2 will be :
(h = 6.62 × 10 –34 Js and c = 3.0 × 108 ms–1) 21. The limiting line in Balmer series will have a frequency of
(Rydberg constant, R¥ = 3.29 × 1015 cycles/s)
(a) 1.214 × 10–7 m (b) 2.816 × 10.–7 m
(a) 8.22 × 1014 s–1 (b) 3.29 × 1015 s–1
(c) 6.500 × 10–7 m (d) 8.500 × 10–7 m
(c) 3.65 × 1014 s–1 (d) 5.26 × 1013 s–1
17. If the nitrogen atom had electronic configuration 1s7 it would
22. The energy required to break one mole of Cl – Cl bonds in
have energy lower than that of the normal ground state
Cl2 is 242 kJ mol–1. The longest wavelength of light capable
configuration 1s2 2s2 2p3 because the electrons would be
of breaking a single Cl – Cl bond is
closer to the nucleus. Yet 1s7 is not observed. It violates
(c = 3 × 108 ms–1 and NA = 6.02 × 1023 mol–1).
(a) Heisenberg’s uncertainty principle
(a) 594 nm (b) 640 nm
(b) Hund’s rule
(c) 700 nm (d) 494 nm
(c) Pauli exclusion principle
23. The de Broglie wavelength of a car of mass 1000 kg and
(d) Bohr postulate of stationary orbits velocity 36 km/hr is :
18. In a hydrogen atom, if energy of an electron in ground state (a) 6.626 × l0–34 m (b) 6.626 × 10–38 m
is 13.6. ev, then that in the 2nd excited state is (c) 6.626 × 10–31 m (d) 6.626 × 10– 30 m
(a) 1.51 eV (b) 3.4 eV 24. If the radius of first orbit of H atom is a0, the de-Broglie
(c) 6.04 eV (d) 13.6 eV. wavelength of an electron in the third orbit is
19. Of the following sets which one does NOT contain (a) 4pa0 (b) 8pa0
isoelectronic species? (c) 6pa0 (d) 2pa0

RESPONSE 15. 16. 17. 18. 19.


GRID 20. 21. 22. 23. 24.
Space for Rough Work
EBD_7504
C-8 NTA JEE Main

25. Which of the following radial distribution graphs correspond 28. If lo and l be threshold wavelength and wavelength of
to l = 2 for the H atom ? incident light, the velocity of photoelectron ejected from
the metal surface is:
(a) (b) 2h 2hc
(a)
m
( lo - l ) (b)
m
( lo - l )
2hc æ lo - l ö 2h æ 1 1 ö
(c) (d) -
m çè ll o ÷ø m çè l o l ÷ø
(c) (d) 29. If m and e are the mass and charge of the revolving electron
in the orbit of radius r for hydrogen atom, the total energy
of the revolving electron will be:
1 e2 e2
(a) (b) -
26. If the kinetic energy of an electron is increased four times, 2 r r
the wavelength of the de-Broglie wave associated with it
me2 1 e2
would become (c) (d) -
r 2 r
(a) one fourth (b) half
30. The dissociation energy of H2 is 430.53 KJ mol–1 . If
(c) four times (d) two times
hydrogen is dissociated by illumination with radiation of
27. The correct set of four quantum numbers for the valence
wavelength 253.7 nm the fraction of the radiant energy which
electrons of rubidium atom (Z = 37) is:
will be converted into kinetic energy is given by
1 1
(a) 5, 0, 0, + (b) 5,1, 0, + (a) 100% (b) 8.76%
2 2
(c) 2.22% (d) 1.22%
1 1
(c) 5,1,1, + (d) 5, 0,1, +
2 2

RESPONSE 25. 26. 27. 28. 29.


GRID 30.

CHEMISTRY CHAPTERWISE SPEED TEST-31


Total Questions 30 Total Marks 120
Attempted Correct
Incorrect Net Score
Cut-off Score 37 Qualifying
Space for Rough Work Score 51
Success Gap = Net Score – Qualifying Score
Net Score = (Correct × 4) – (Incorrect × 1)
Space for Rough Work
CHEMISTRY Speed
Classification of Elements and TEST
32
Periodicity in Properties
No. of Questions Maximum Marks Time
30 120 1 Hour Chapter-wise

GENERAL INSTRUCTIONS
• This test contains 30 MCQ's. For each question only one option is correct. Darken the correct circle/ bubble in the
Response Grid provided on each page.
• You have to evaluate your Response Grids yourself with the help of solutions provided at the end of this book.
• Each correct answer will get you 4 marks and 1 mark shall be deduced for each incorrect answer. No mark will be given/
deducted if no bubble is filled. Keep a timer in front of you and stop immediately at the end of 60 min.
• The sheet follows a particular syllabus. Do not attempt the sheet before you have completed your preparation for that
syllabus.
• After completing the sheet check your answers with the solution booklet and complete the Result Grid. Finally spend time
to analyse your performance and revise the areas which emerge out as weak in your evaluation.

1. The correct sequence which shows decreasing order of the (a) 219pm, 72pm (b) 75pm, 72pm
ionic radii of the elements is (c) 147pm, 72pm (d) 147pm, 144pm
(a) Al3+ > Mg 2+ > Na + > F- > O 2 -
3. Arrange the following in increasing order of ionic radii?
(b) Na + > Mg 2+ > Al3+ > O2- > F- C4–,N3–,F–,O2–
(c) Na + > F- > Mg 2 + > O 2 - > Al3+
(a) C4– < N3– < O2– < F–
(d) O2- > F- > Na + > Mg 2+ > Al3+
(b) N3– < C4– < O2– < F–
2. The van der Waal and covalent radii of fluorine atom
respectively from the following figure are. (c) F–< O2– < N3– < C4–
294 pm (d) O2– < F– < N3– < C4–
4. Which is not the correct order for the stated property.
(a) Ba > Sr > Mg ; atomic radius
F F F F (b) F > O > N ; first ionization enthalpy
(c) Cl > F > I ; electron affinity
(d) O > Se > Te ; electronegativity
144 pm 144 pm

RESPONSE GRID 1. 2. 3. 4.
Space for Rough Work
EBD_7504
C-10 NTA JEE Main

5. In which of the following arrangements, the order is NOT 10. The radii of F, F–, O and O2– are in the order
according to the property indicated against it? (a) O 2 - > F - > F > O
(a) Li < Na < K < Rb :
Increasing metallic radius (b) F - > O 2 - > F > O
(b) I < Br < F < Cl : (c) O 2 - > O > F - > F
Increasing electron gain enthalpy (d) O 2 - > F - > O > F
(with negative sign) 11. Which of the following has the maximum number of unpaired
(c) B < C < N < O electrons?
Increasing first ionization enthalpy (a) Mg2+
(d) Al3+ < Mg 2+ < Na + < F - (b) Ti3+
Increasing ionic size (c) V3+
6. The symbol and IUPAC name for the element with atomic (d) Fe2+
number 120, respectively are 12. The incorrect statement among the following is
(a) Ubn and unbinilium (a) The first ionization potential of Al is less than the first
(b) Ubn and unbiunium ionization potential of Mg
(c) Ubn and unnibium (b) The second ionization potential of Mg is greater than
(d) Ubn and unnilium the second ionization potential of Na
7. Sequence of acidic character is (c) The first ionization potential of Na is less than the first
(a) N2O5 > SO2 > CO > CO2 ionization potential of Mg
(b) N2O5 > SO2 > CO2 > CO (d) The third ionization potential of Mg is greater than the
(c) SO2 > CO2 > CO > N2O5 third ionization potential of Al.
13. According to the Periodic Law of elements, the variation in
(d) SO2 > N2O5 > CO > CO2
properties of elements is related to their
8. The correct order of ionization energy for carbon, nitrogen
(a) nuclear masses
and oxygen atoms is: (b) atomic numbers
(a) C > N > O (b) C > N < O (c) nuclear neutron-proton number ratios
(c) C < N > O (d) C < N < O (d) atomic masses
9. Which of the following order is wrong? 14. Identify the correct order of the size of the following:
(a) NH3 < PH3 < AsH3 – Acidic (a) Ca2+ < K+ < Ar < Cl– < S2–
(b) Li < Be < B < C – First IP (b) Ar < Ca2+ < K+ < Cl– < S2–
(c) Al2O3 < MgO < Na2O < K2O – Basic (c) Ca2+ < Ar < K+ < Cl– < S2–
(d) Li+ < Na+ < K+ < Cs+ – Ionic radius (d) Ca2+ < K+ < Ar < S2– < Cl–

RESPONSE 5. 6. 7. 8. 9.
GRID 10. 11. 12. 13. 14.

Space for Rough Work


Chemistry C-11

15. Following statements regarding the periodic trends of 19. Which one of the following has largest ionic radius?
chemical reactivity of the alkali metals and the halogens are
given. Which of these statements gives the correct picture? (a) Li+ (b) O 22 -
(a) Chemical reactivity increases with increase in atomic (d) F–
number down the group in both the alkali metals and
(c) B3+
halogens 20. Which one of the following arrangements represents the
(b) In alkali metals the reactivity increases but in the correct order of least negative to most negative electron gain
halogens it decreases with increase in atomic number enthalpy for C, Ca, Al, F and O?
down the group (a) Ca < Al < C < O < F
(c) The reactivity decreases in the alkali metals but (b) Al < Ca < O < C < F
increases in the halogens with increase in atomic (c) Al < O < C < Ca < F
number down the group (d) C < F < O < Al < Ca
(d) In both the alkali metals and the halogens the chemical 21. Which of the following elements represents highly
reactivity decreases with increase in atomic number electropositive as well as highly electronegative character in
down the group its period?
16. In which of the following arrangements, the sequence is not (a) Hydrogen (b) Nitrogen
strictly according to the property written against it? (c) Fluorine (d) None of these
(a) HF < HCl < HBr , HI : increasing acid strength 22. Which one of the following ions has the highest value of
(b) NH3 < PH3 < AsH3 <SbH3 : increasing basic strength ionic radius ?
(c) B < C < O < N : increasing first ionization enthalpy (a) O2– (b) B3+
(c) Li + (d) F–
(d) CO2 < SiO2 < SnO2 < PbO2 : increasing oxidising power
17. Which of the following order is wrong? 23. Among Al2O3, SiO2, P2O3 and SO2 the correct order of acid
(a) NH3 < PH3 < AsH3 — Acidic strength is
(b) Li < Be < B < C — IE1 (a) Al2O3 < SiO2< SO2 < P2O3
(b) SiO2< SO2 < Al2O3 < P2O3
(c) Al2 O3 < MgO < Na 2 O < K 2 O — Basic (c) SO2< P2O3 < SiO2 < Al2O3
(d) Li + < Na + < K + < Cs + — Ionic radius (d) Al2O3 < SiO2< P2O3 < SO2
18. The correct order of electron gain enthalpy with negative 24. Which of the following arrangements represents the
sign of F, Cl, Br and I, having atomic number 9, 17, 35 and 53 increasing order (smallest to largest) of ionic radii of the
respectively, is : given species O2–, S2–, N3–, P3–?
(a) F > Cl > Br > I (b) Cl > F > Br > I (a) O2– < N3– < S2– < P3– (b) O2– < P3– < N3– < S2–
(c) Br > Cl > I > F (d) I > Br > Cl > F (c) N3 < O2– < P3– < S2– (d) N3– < S2– < O2– < P3–

RESPONSE 15. 16. 17. 18. 19.


GRID 20. 21. 22. 23. 24.

Space for Rough Work


EBD_7504
C-12 NTA JEE Main

25. Which of the following oxides is amphoteric in character? Element D iH 1 D iH 2 D eg H


(a) SnO2 (b) SiO 2 I 520 7300 – 60
(c) CO2 (d) CaO II 419 3051 – 48
26. The formation of the oxide ion O2–(g), from oxygen atom III 1681 3374 – 328
requires first an exothermic and then an endothermic step as IV 1008 1846 – 295
shown below : V 2372 5251 + 48
O(g) + e– ® O–(g); Df H = –141 kJ mol–1 The most reactive metal and the least reactive non-metal of
these are respectively
O– (g) + e– ® O2– (g); Df H = +780 kJ mol–1 (a) I and V (b) V and II
Thus process of formation of O2– in gas phase is unfavourable (c) II and V (d) IV and V
even though O2– is isoelectronic with neon. It is due to the 29. Consider the following statements:
fact that (i) Atomic radii decreases across a row of the periodic
table when we move from left to right.
(a) Electron repulsion outweighs the stability gained by
(ii) Atomic radii increases down the column as we move
achieving noble gas configuration from top to bottom.
(b) O– ion has comparatively smaller size than oxygen atom (iii) Although the order of elements is based on atomic
(c) Oxygen is more electronegative numbers, vertical families share similar chemical
(d) Addition of electron in oxygen results in larger size of properties.
the ion. Which of the statement(s) given above is/are correct?
27. Which of the following statements is wrong ? (a) (i) and (ii) (b) (i) and (iii)
(a) van der Waal’s radius of iodine is more than its covalent (c) (ii) and (iii) (d) (i), (ii) and (iii)
radius 30. Match Column-I with Column-II and select the correct
(b) All isoelectronic ions belong to same period of the answer by the given codes.
periodic table Columnn-I Column-II
(c) I.E.1 of N is higher than that of O while I.E.2 of O is (Atoms) (Properties)
higher than that of N (A) He (p) High electronegative
(d) The electron gain enthalpy of N is almost zero while (B) F (q) Most electropositive
(C) Rb (r) Strongest reducing agent
that of P is 74.3 kJ mol –1
(D) Li (s) Highest ionisation energy
28. The first (DiH1) and second (DiH2) ionization enthalpies (a) A – (s), B – (q), C – (r), D – (p)
(in kJ mol–1 ) and the electron gain enthalpy (D egH) (b) A – (p), B – (s), C – (q), D – (r)
(in kJ mol–1) of the elements I, II, III, IV and V are given (c) A – (s), B – (p), C – (r), D – (q)
below (d) A – (s), B – (p), C – (q), D – (r)

RESPONSE 25. 26. 27. 28. 29.


GRID 30.

CHEMISTRY CHAPTERW ISE SPEED TEST-32


Total Questions 30 Total M arks 120
Attempted Correct
Incorrect Net
Space Score
for Rough Work

Cut-off Score 38 Qualifying Score 55


Success Gap = N et Score – Qualifying Score
Net Score = (Correct × 4) – (Incorrect × 1)
Space for Rough Work
CHEMISTRY Speed
Chemical Bonding and Molecular Structure TEST
No. of Questions
30
Maximum Marks
120
Time
1 Hour
33
Chapter-wise

GENERAL INSTRUCTIONS
• This test contains 30 MCQ's. For each question only one option is correct. Darken the correct circle/ bubble in the
Response Grid provided on each page.
• You have to evaluate your Response Grids yourself with the help of solutions provided at the end of this book.
• Each correct answer will get you 4 marks and 1 mark shall be deduced for each incorrect answer. No mark will be given/
deducted if no bubble is filled. Keep a timer in front of you and stop immediately at the end of 60 min.
• The sheet follows a particular syllabus. Do not attempt the sheet before you have completed your preparation for that
syllabus.
• After completing the sheet check your answers with the solution booklet and complete the Result Grid. Finally spend time
to analyse your performance and revise the areas which emerge out as weak in your evaluation.

1. Which of the following does not contain coordinate bond ? H CH3 Cl CH3
| | | |
(a) BH 4– (b) NH +4 (c) C = C (d) C = C
+
| | | |
(c) CO 23 – (d) H 3O CH3 H CH3 Cl
2. Which of the following has the highest dipole moment? 3. Of the following hydrides which one has the lowest boiling
CH3 H point ?
H | | (a) AsH 3 (b) SbH 3 (c) PH 3 (d) NH 3
(a) C=O (b) C = C
| | 4. In compounds of type ECl3, where E = B, P, As or Bi, the
H CH 3 H angles Cl - E- Cl for different E are in the order.
(a) B > P = As = Bi (b) B > P > As > Bi
(c) B < P = As = Bi (d) B < P < As < Bi

RESPONSE GRID 1. 2. 3. 4.

Space for Rough Work


EBD_7504
C-14 NTA JEE Main

5. The electronegativity difference between N and F is greater 9. Which of the two ions from the list given below have the
than that between N and H yet the dipole moment of geometry that is explained by the same hybridization of
NH3 (1.5 D) is larger than that of NF3 (0.2D). This is because orbitals, NO2–, NO3–, NH2–, NH4+, SCN– ?
(a) in NH3 the atomic dipole and bond dipole are in the (a) NO2– and NO3– (b) NH2– and NO3–
same direction whereas in NF3 these are in opposite – –
(c) SCN and NH2 (d) NO2– and NH2–
directions
10. The values of electronegativity of atoms A and B are 1.20
(b) in NH3 as well as NF3 the atomic dipole and bond
and 4.0 respectively. The percentage of ionic character of
dipole are in opposite directions
A - B bond is
(c) in NH3 the atomic dipole and bond dipole are in the
(a) 50% (b) 72.24%
opposite directions whereas in NF3 these are in the
(c) 55.3% (d) 43%
same direction
11. 3-
In PO 4 ion the formal charge on the oxygen atom of P –O
(d) in NH3 as well as in NF3 the atomic dipole and bond
bond is
dipole are in the same direction
(a) + 1 (b) – 1
6. KF combines with HF to form KHF2. The compound
(c) – 0.75 (d) + 0.75
contains the species
12. Which molecule/ion out of the following does not contain
(a) K+, F– and H+ (b) K+, F– and HF unpaired electrons?
(c) K+, and [HF2]– (b) [KHF]+and F2 (a) N +2 (b) O2
7. On changing N2 to N2+, the dissociation energy of N–N (c) O 2 2 - (d) B2
bond ..... and on changing O2 to O+2 the dissociation energy 13. Which of the following pairs of ions are isoelectronic and
of O–O bond.... isostructural ?
(a) increases, decreases
(a) ClO3– , CO32– (b) SO32– , NO3–
(b) decreases, increases
(c) decreases in both cases (c) ClO3– , SO32– (d) CO32– , SO32–
(d) increases in both cases 14. Which of the following is correct increasing order of lone
8. According to MO theory which of the following lists ranks pair of electrons on the central atom?
the nitrogen species in terms of increasing bond order? (a) IF7 < IF5 < ClF3 < XeF2
N 2– –
N 2 < N 22– < N 2– (b) IF7 < XeF2 < ClF2 < IF5
(a) 2 < N2 < N2 (b)
(c) IF7 < ClF3 < XeF2 < IF5
(c) N 2– < N 2–
2 < N2 (d) N 2– < N 2 < N 22– (d) IF7 < XeF2 < IF5 < ClF3

RESPONSE 5. 6. 7. 8. 9.
GRID 10. 11. 12. 13. 14.
Space for Rough Work
Chemistry C-15

15. Consider the molecules CH4, NH3 and H2O. Which of the
(c) NH 3 changes to NH +4
given statements is false?
(a) The H–C–H bond angle in CH4, the H–N–H bond (d) in all cases
angle in NH3, and the H–O–H bond angle in H2O are 20. The states of hybridization of boron and oxygen atoms in
all greater than 90° boric acid (H3BO3) are respectively
(b) The H–O–H bond angle in H2 O is larger than the H– (a) sp3 and sp2 (b) sp2 and sp 3
C–H bond angle in CH4. 2
(c) sp and sp 2 (d) sp3 and sp3
(c) The H–O–H bond angle in H2O is smaller than the H– 21. Among the following ions, the p π – d π overlap could be
N–H bond angle in NH3. present in
(d) The H–C–H bond angle in CH4 is larger than the
H–N–H bond angle in NH3.
(a) NO 3– (b) PO 34–
16. Predict the correct order among the following : (c) CO 23 – (d) NO 2–
(a) lone pair- lone pair > lone pair - bond pair > bond pair - 22. The decreasing values of bond angles from NH3 (106º) to
bond pair SbH3 (101º) down group-15 of the periodic table is due to
(b) lone pair - lone pair > bond pair - bond pair > lone pair
(a) decreasing lp-bp repulsion
- bond pair
(c) bond pair - bond pair > lone pair - bond pair > lone pair (b) decreasing electronegativity
- lone pair (c) increasing bp-bp repulsion
(d) lone pair - bond pair > bond pair - bond pair > lone pair (d) increasing p-orbital character in sp3
- lone pair 23. Which one of the following pairs of species have the same
17. In which of the following species the interatomic bond angle bond order?
is 109° 28’? (a) CN– and NO+ (b) CN– and CN+
(a) NH3, BF4– (b) NH4+, BF3 –
(c) O 2 and CN – (d) NO+ and CN+
(c) NH3, BF4 (d) NH2–, BF3.
24. N2 and O2 are converted into monocations, N +2 and
18. In XeF2, XeF4 and XeF6, the number of lone pairs on Xe are
respectively O +2 respectively. Which of the following statements is
(a) 2, 3, 1 (b) 1, 2, 3 wrong?
(c) 4, 1, 2 (d) 3, 2, 1
(a) In N +2 , N - N bond weakens
19. Hybridisation of the underline atom changes in:
(b) In O +2 , the O - O bond order increases
(a) AlH3 changes to AlH -4
(c) In O +2 , paramagnetism decreases
(b) H 2 O changes to H3O+
(d) N +2 becomes diamagnetic

RESPONSE 15. 16. 17. 18. 19.


GRID 20. 21. 22. 23. 24.
Space for Rough Work
EBD_7504
C-16 NTA JEE Main

25. Th e bond dissociation energy of B – F in BF3 is 27. Amongst LiCl, RbCl, BeCl2 and MgCl2 the compounds with
646 kJ mol–1 whereas that of C – F in CF4 is 515 kJ mol–1. The the greatest and the least ionic character, respectively are:
correct reason for higher B – F bond dissociation energy as (a) LiCl and RbCl (b) RbCl and BeCl2
compared to that of C – F is (c) MgCl2 and BeCl2 (d) RbCl and MgCl2
(a) stronger s bond between B and F in BF3 as compared 28. Bond order normally gives idea of stability of a molecular
to that between C and F in CF4. species. All the molecules viz. H2, Li2 and B2 have the same
(b) significant pp – pp interaction between B and F in BF 3 bond order yet they are not equally stable. Their stability
whereas there is no possibility of such interaction order is
between C and F in CF4. (a) H2 > B2 > Li2 (b) Li2 > H2 > B2
(c) lower degree of pp – pp interaction between B and F in (c) Li2 > B2 > H2 (d) B2 > H2 > Li2
BF3 than that between C and F in CF4. 29. The shape of IF6– is :
(d) smaller size of B– atom as compared to that of C– atom. (a) Trigonally distorted octahedron
26. Ortho-Nitrophenol is less soluble in water than p- and m- (b) Pyramidal
Nitrophenols because :
(c) Octahedral
(a) o-Nitrophenol is more volatile steam than those of m-
(d) Square antiprism
and p-isomers.
30. Which one of the following molecules is paramagnetic?
(b) o-Nitrophenol shows intramolecular H-bonding
(c) o-Nitrophenol shows intermolecular H-bonding (a) N2 (b) NO
(d) Melting point of o-Nitrophenol is lower than those of (c) CO (d) O3
m- and p-isomers.

RESPONSE 25. 26. 27. 28. 29.


GRID 30.

CHEMISTRY CHAPTERWISE SPEED TEST-33


Total Questions 30 Total Marks 120
Attempted Correct
Incorrect Net Score
Cut-off Score 35 Qualifying Score 50
Success Gap = Net Score – Qualifying Score
Net Score = (Correct × 4) – (Incorrect × 1)
Space for Rough Work
Space for Rough Work
CHEMISTRY Speed
States of Matter TEST
No. of Questions
30
Maximum Marks
120
Time
1 Hour
34
Chapter-wise

GENERAL INSTRUCTIONS
• This test contains 30 MCQ's. For each question only one option is correct. Darken the correct circle/ bubble in the
Response Grid provided on each page.
• You have to evaluate your Response Grids yourself with the help of solutions provided at the end of this book.
• Each correct answer will get you 4 marks and 1 mark shall be deduced for each incorrect answer. No mark will be given/
deducted if no bubble is filled. Keep a timer in front of you and stop immediately at the end of 60 min.
• The sheet follows a particular syllabus. Do not attempt the sheet before you have completed your preparation for that
syllabus.
• After completing the sheet check your answers with the solution booklet and complete the Result Grid. Finally spend time
to analyse your performance and revise the areas which emerge out as weak in your evaluation.

1. Which of the following volume (V) - temperature (T) plots V(L) V(L)
represents the behaviour of one mole of an ideal gas at one (30.6 L
atmospheric pressure ? (22.4 L 373 K) (22.4 L
V(L) 273K) 273K)
V(L)
(36.8 L (14.2 L
373 K) (26.8 L 373 K)
(22.4 L 373 K)
(22.4 L
273K) 273K)
T(K) T(K)
(c) (d)

T(K) T(K)
(a) (b)

RESPONSE GRID 1.

Space for Rough Work


EBD_7504
C-18 NTA JEE Main

2. The volume-temperature graphs of a given mass of an ideal (a) 0.492 atm (b) 49.2 atm
gas at constant pressure are shown below. (c) 4.52 atm (d) 0.0492 atm
8. A bottle of dry ammonia and a bottle of dry hydrogen
p2 p
3 chloride connected through a long tube are opened
V p1
simultaneously at both ends the white ammonium chloride
ring first formed will be
(a) at the centre of the tube.
(b) near the hydrogen chloride bottle.
O 273 T(K) (c) near the ammonia bottle.
(d) throughout the length of the tube.
What is the correct order of pressures ?
9. The r.m.s velocity of CO2 at temperature T (in kelvin) is
(a) p1 > p3 > p2 (b) p1 > p2 > p3
x cms–1. At what temperature (in kelvin) the r.m.s. velocity
(c) p2 > p3 > p1 (d) p2 > p1 > p3
of nitrous oxide would be 4x cms–1 ?
3. The inversion temperature Ti (K) of hydrogen is (given van
(a) 16 T (b) 2 T
der Waal’s constants a and b are 0.244 atm L2 mol–2 and
(c) 4 T (d) 32 T
0.027 L mol–1 respectively)
10. A mixture of two gases A and B in the mole ratio 2 : 3 is kept
(a) 440 (b) 220 (c) 110 (d) 330
in a 2 litre vessel. A second 3L vessel has the same two gases
4. The temperature of the gas is raised from 27°C to 927°C, the in the mole ratio 3 : 5. Both gas mixtures have the same tem-
root mean square velocity is perature and same total pressure. They are allowed to inter-
(a) 927 / 27 time the earlier value mix and the final temperature and the total pressure are the
(b) same as before same as the initial values, the final volume being 5 litres.
(c) halved Given that the molar masses are MA and MB. What is the
(d) doubled mean molar mass of the final mixture?
5. Equal masses of methane and oxygen are mixed in an empty 5M A + 8M B 77 M A + 123M B
(a) (b)
container at 25°C. The fraction of the total pressure exerted 13 200
by oxygen is 123M A + 77 M B 123M A + 77 M B
(c) (d)
(a) 1/2 (b) 2/3 250 150
11. At constant volume and temperature conditions, the rate of
1 273 diffusion DA and DB of gases A and B having densities rA
(c) ´ (d) 1/3.
3 298 and rB are related by the expression.
6. The ratio between the root mean square speed of H2 at 50 K
12 12
and that of O2 at 800 K is, é r ù é r ù
(a) 4 (b) 2
(a) DA = ê DB A ú (b) DA = ê DB B ú
ë rB û ë rA û
(c) 1 (d) 1/4 12 12
ær ö ær ö
7. Calculate the total pressure in a 10.0 L cylinder which contains (c) DA = D B A (d) DA = D B ç B ÷
çè r ÷ø è rA ø
0.4g helium, 1.6 g oxygen and 1.4 g nitrogen at 27°C. B

RESPONSE 2. 3. 4. 5. 6.
GRID 7. 8. 9. 10. 11.
Space for Rough Work
Chemistry C-19

12. For one mole of a van der Waals gas when b = 0 and 17. Which one of the following statements is NOT true about
T = 300 K, the PV vs, 1/V plot is shown below. The value of the the effect of an increase in temperature on the distribution
van der Waals constant a (atm. liter2 mol–2) is : of molecular speeds in a gas?
(a) The area under the distribution curve remains the same
24.6
as under the lower temperature
PV(litre atm mol )
–1

23.1 (b) The distribution becomes broader


21.6 (c) The fraction of the molecules with the most probable
20.1 speed increases
(d) The most probable speed increases
18. According to the kinetic theory of gases, in an ideal gas,
between two successive collisions a gas molecule travels
(a) in a wavy path
(b) in a straight line path
0 2.0 3.0 (c) with an accelerated velocity
–1
1/V(mol litre ) (d) in a circular path
(a) 1.0 (b) 4.5 (c) 1.5 (d) 3.0 19. For 1 mol of an ideal gas at a constant temperature T, the
13. As the temperature is raised from 20ºC to 40ºC, the average plot of (log P) against (log V) is a (P : Pressure, V : Volume)
kinetic energy of neon atoms changes by a factor of which (a) Straight line parallel to x-axis.
of the following ? (b) Straight line with a negative slope.
(c) Curve starting at origin.
(a) 313 / 293 (b) (313 / 293)
(d) Straight line passing through origin.
(c) 1 / 2 (d) 2 20. When CO2(g) is passed over red hot coke it partially gets
14. Maximum deviation from ideal gas is expected from : reduced to CO(g). Upon passing 0.5 L of CO2(g) over red
(a) N2(g) (b) CH4(g) hot coke, the total volume of the gases increased to 700 mL.
(c) NH3 (g) (d) H2(g) The composition of the gaseous mixture at STP is
15. Equal masses of H2,O2 and methane have been taken in a (a) CO2 = 300 mL; CO = 400 mL
container of volume V at temperature 27°C in identical (b) CO2 = 0.0 mL; CO = 700 mL
conditions. The ratio of the volumes of gases H2 : O2 : (c) CO2 = 200 mL; CO = 500 mL
methane would be : (d) CO2 = 350 mL; CO = 350 mL
(a) 8 : 16 : 1 (b) 16 : 8 : 1 21. For gaseous state, if most probable speed is denoted by C*,
(c) 16 : 1 : 2 (d) 8 : 1 : 2
average speed by C and mean square speed by C, then for
16. Equal moles of hydrogen and oxygen gases are placed in a
a large number of molecules the ratios of these speeds are :
container with a pin-hole through which both can escape.
What fraction of the oxygen escapes in the time required for (a) C* : C : C = 1.225 : 1.128 : 1
one-half of the hydrogen to escape ? (b) C* : C : C = 1.128 : 1.225 : 1
(a) 1/8 (b) 1/4 (c) C* : C : C = 1 : 1.128 : 1.225
(c) 3/8 (d) 1/2 (d) C* : C : C = 1 : 1.225 : 1.128

RESPONSE 12. 13. 14. 15. 16.


GRID 17. 18. 19. 20. 21.
Space for Rough Work
EBD_7504
C-20 NTA JEE Main

22. Sulphur dioxide and oxygen were allowed to diffuse through 27. 56 g of nitrogen and 96 g of oxygen are mixed isothermally
a porous partition. 20 dm3 of SO2 diffuses through the and at a total pressure of 10 atm. The partial pressures of
porous partition in 60 seconds. The volume of O2 in dm3 oxygen and nitrogen (in atm) are respectively
which diffuses under the similar condition in 30 seconds (a) 4, 6 (b) 5, 5
will be (atomic mass of sulphur = 32 u): (c) 2, 8 (d) 6, 4
(a) 7.09 (b) 14.1 28. Two vessels containing gases A and B are interconected as
(c) 10.0 (d) 28.2 shown in the figure. The stopper is opened, the gases are
23. By how many folds the temperature of a gas would increase allowed to mix homogeneously. The partial pressures of A
when the root mean square velocity of the gas molecules in and B in the mixture will be, respectively
a container of fixed volume is increased from 5 × 104 cm/s to Gas A Gas B
10 × 104 cm/s ?
(a) Two (b) Three 12 L 8L
8 atm 5 atm
(c) Six (d) Four
24. The temperature at which oxygen molecules have the same (a) 8 and 5 atom (b) 9.6 and 4 atm
root mean square speed as helium atoms have at 300 K is: (c) 4.8 and 2 atm (d) 6.4 and 4 atm
Atomic masses: He = 4 u, O = 16 u)
29. A neon-dioxygen mixture contains 70.6 g O2 and 167.5 g
(a) 300 K (b) 600 K
(c) 1200 K (d) 2400 K neon. If pressure of the mixture of gases in the cylinder is 25
25. If Z is a compressibility factor, van der Waals equation at bar. What is the partial pressure of O2 and Ne in the mixture
low pressure can be written as: respectively ?
RT a (a) 5.25 bar, 10 bar (b) 19.75 bar, 5.25 bar
(a) Z = 1 + (b) Z = 1 - (c) 19.75 bar, 10 bar (d) 5.75 bar, 19.75 bar
Pb VRT
Pb Pb 30. Let the most probable velocity of hydrogen molecules at a
(c) Z = 1 - (d) Z = 1 + temperature of t° C be V0. When the temperature is raised
RT RT
26. The initial volume of a gas cylinder is 750.0 mL. If the to (2t + 273)°C the new rms velocity is (suppose all the
molecules dissociate into atoms at latter temperature)
pressure of gas inside the cylinder changes from 840.0 mm
Hg to 360.0 mm Hg, the final volume the gas will be: (a) 2 3V0 (b) 6V0
(a) 1.750 L (b) 3.60 L
æ 273 ö 2
(c) 4.032 L (d) 7.50 L (c) 3ç 2 + ÷V0 (d) V0
è t ø 3

RESPONSE 22. 23. 24. 25. 26.


GRID 27. 28. 29. 30.

CHEMISTRY CHAPTERWISE SPEED TEST-34


Total Questions 30 Total Marks 120
Attempted Correct
Space for Rough Work
Incorrect Net Score
Cut-off Score 38 Qualifying Score 53
Success Gap = Net Score – Qualifying Score
Net Score = (Correct × 4) – (Incorrect × 1)
Space for Rough Work
CHEMISTRY Speed
Thermodynamics TEST
No. of Questions
30
Maximum Marks
120
Time
1 Hour
35
Chapter-wise

GENERAL INSTRUCTIONS
• This test contains 30 MCQ's. For each question only one option is correct. Darken the correct circle/ bubble in the
Response Grid provided on each page.
• You have to evaluate your Response Grids yourself with the help of solutions provided at the end of this book.
• Each correct answer will get you 4 marks and 1 mark shall be deduced for each incorrect answer. No mark will be given/
deducted if no bubble is filled. Keep a timer in front of you and stop immediately at the end of 60 min.
• The sheet follows a particular syllabus. Do not attempt the sheet before you have completed your preparation for that
syllabus.
• After completing the sheet check your answers with the solution booklet and complete the Result Grid. Finally spend time
to analyse your performance and revise the areas which emerge out as weak in your evaluation.

1. Assuming that water vapour is an ideal gas, the internal 3. The maximum work which a system can perform at constant
energy change (DU) when 1 mol of water is vapourised at temperature and pressure equals
1 bar pressure and 100°C, (given : molar enthalpy of (a) DH – DE
vapourisation of water at 1 bar and 373 K = 41 kJ mol –1 and (b) (– DG)
R = 8.3 J mol–1 K–1) will be (c) q – DE
(a) 41.00 kJ mol–1 (b) 4.100 kJ mol–1 (d) P.DV – DG
(c) 3.7904 kJ mol –1 (d) 37.904 kJ mol–1 4. The entropy change in the fusion of one mole of a
2. In a closed insulated container, a liquid is stirred with a solid melting at 27ºC (Latent heat of fusion, 2930 J mol–1)
paddle to increase the temperature, which of the following is :
is true? (a) 9.77 J K–1 mol–1
(b) 10.73 J K–1 mol–1
(a) DE = W ¹ 0, q = 0 (b) DE = W = q ¹ 0 (c) 2930 J K–1 mol–1
(c) DE = 0, W = q ¹ 0 (d) W = 0, DE = q ¹ 0 (d) 108.5 J K–1 mol–1

RESPONSE GRID 1. 2. 3. 4.

Space for Rough Work


EBD_7504
C-22 NTA JEE Main

5. The enthalpy of vapourisation of water from the following H 2O(l ) ® H + ( aq ) + OH – ( aq ); DH = 57.32kJ


two equations is.
1
1 H2 (g )+ O2 (g ) ¾¾ ® H2O(l); ΔH= –286.20kJ
H 2 (g) + O 2 (g) ¾¾ ® H 2O (l ), DH = - 286 kJ 2
2 The value of enthalpy of formation of OH– ion at 25° C is:
1 (a) –228.88 kJ (b) +228.88 kJ
H 2 (g) + O 2 (g) , ® H2 O (g) , DH = - 245.5 kJ
2 (c) –343.52 kJ (d) –22.88 kJ
(a) 6.02 kJ (b) 40.5 kJ 10. For a particular reversible reaction at temperature T, DH and
(c) 62.3 kJ (d) 1.25 kJ DS were found to be both +ve. If Te is the temperature at
6. The standard enthalpy of formation of NH3 is – 46.0 kJ mol–1. equilibrium, the reaction would be spontaneous when
If the enthalpy of formation of H2 from its atoms is (a) Te > T (b) T > Te
– 436 kJ mol–1 and that of N2 is – 712 kJ mol–1, the average (c) Te is 5 times T (d) T = Te
bond enthalpy of N – H bond in NH3 is 11. Consider the following spontaneous reaction
(a) – 964 kJ mol–1 (b) + 352 kJ mol–1 2C8 H18 ( g ) + 25 O 2 ( g ) ® 16CO 2 ( g ) + 18H 2O ( g )
(c) + 1056 kJ mol –1 (d) – 1102 kJ mol–1
The sign of DH, DS, DG would be respectively
7. Calculate enthalpy change for the change
(a) +, –, + (b) –, + , –
8S( g ) ¾¾ ↑ S8 ( g ), given that (c) +, +, – (d) –, –, –
® 2H( g ) + 2S( g ), DH = 239.0 k cal mol-1 ;
H 2S2 ( g ) ¾¾ 12. The favourable conditions for a spontaneous reaction are
-1
H 2S( g ) ¾¾
® 2H( g ) + S( g ), DH = 175.0 k cal mol (a ) TDS > DH, DH = + ve, DS = + ve
(a) + 512.0 k cal (b) – 512.0 k cal (b) T DS > DH, DH = +ve, DS = - ve
(c) 508.0 k cal (d) – 508.0 k cal
(c) T DS = DH, DH = -ve, DS = - ve
8. From the following bond energies:
(d) T DS = DH, DH = +ve, DS = + ve
H – H bond energy: 431.37 kJ mol–1
C = C bond energy: 606.10 kJ mol–1 13. The enthalpies of the following reactions are shown below.
C – C bond energy: 336.49 kJ mol–1 1 1
H2(g) + O2(g) ® OH(g) ; DH = 42.09 kJ mol–1
C – H bond energy: 410.50 kJ mol–1 2 2
Enthalpy for the reaction, H2(g) ® 2H(g) ; DH = 435.89 kJ mol–1
O2(g) ® 2O(g) ; DH = 495.05 kJ mol–1
H H H H Calculate the O –– H bond energy for the hydroxyl radical.
| | | |
C = C+ H - H ¾¾
® H - C- C - H (a) 223.18 kJ mol–1 (b) 423.38 kJ mol–1
| | | | (c) 513.28 kJ mol–1 (d) 113.38 kJ mol–1
H H H H 14. When 5 litres of a gas mixture of methane and propane is
perfectly combusted at 0°C and 1 atmosphere, 16 litre
will be: of oxygen at the same temperature and pressure is
(a) – 243.6 kJ mol–1 (b) –120.0 kJ mol–1 consumed. The amount of heat released from
(c) 553.0 kJ mol–1 (d) 1523.6 kJ mol–1 this combustion in kJ (DHcomb (CH4) = 890 kJ mol –1,
9. On the basis of the following thermochemical data : DHcomb (C3H8) = 2220 kJ mol–1) is
+ (a) 32 (b) 38
[Δ f H°(H (aq) = 0)]
(c) 317 (d) 477

RESPONSE 5. 6. 7. 8. 9.
GRID 10. 11. 12. 13. 14.

Space for Rough Work


Chemistry C-23

15. The heat of combustion of carbon to CO 2 is (a) – 250 kJ (b) + 125 kJ


–393.5 kJ/mol. The heat released upon formation of 35.2 g of (c) – 125 kJ (d) + 250 kJ
CO2 from carbon and oxygen gas is 21. Standard entropy of X2 , Y2 and X Y3 are 60, 40 and
(a) –315 kJ (b) +315kJ 50 J K–1 mol–1, respectively. For the reaction,
(c) –630 kJ (d) –3.15 kJ 1 3
16. Using the data provided, calculate the multiple bond energy X 2 + Y2 ® XY3 , DH = –30kJ , to be at equilibrium, the
2 2
(kJ mol–1) of a C º C bond in C2H2. That energy is (take the temperature will be
bond energy of a C – H bond as 350 kJ mol–1) (a) 1250 K (b) 500 K
® HC º CH(g);
2C(s) + H2(g) ¾¾ DH = 225 kJ mol–1 (c) 750 K (d) 1000 K
2C(s) ¾¾ ® 2C(g) ; DH = 1410 kJ mol–1 22. The enthalpy changes for the following processes are listed
below :
H2(g) ¾¾ ® 2H(g) ; DH = 330 kJ mol–1
Cl2(g) ® 2Cl(g), 242.3 kJ mol–1
(a) 1165 (b) 837
I2(g) ® 2I(g), 151.0 kJ mol–1
(c) 865 (d) 815
ICl(g) ® I(g) + Cl(g), 211.3 kJ mol–1
17. For a given reaction, DH = 35.5 kJ mol-1 and DS = 83.6 JK-1
I2(s) ® I2(g), 62.76 kJ mol–1
mol-1. The reaction is spontaneous at : (Assume that DH and
Given that the standard states for iodine and chlorine are
DS do not vary with tempearature)
I2(s) and Cl2(g), the standard enthalpy of formation for ICl(g)
(a) T > 425 K (b) All temperatures
is :
(c) T > 298 K (d) T < 425 K
(a) +16.8 kJ mol–1
18. If an endothermic reaction is non-spontaneous at freezing
(b) +244.8 kJ mol–1
point of water and becomes feasible at its boiling point,
then (c) –14.6 kJ mol–1
(a) DH is –ve, DS is +ve (d) –16.8 kJ mol–1
(b) DH and DS both are +ve 23. Bond enthalpies of H 2 , X 2 and HX are in the ratio 2 : 1 : 2.
(c) DH and DS both are –ve If enthalpy of formation of HX is – 50 kJ mol –1, the bond
(d) DH is +ve, DS is -ve enthalpy of X 2 is
19. The standard enthalpies of formation of CO2(g), H2O(l) and (a) 100 kJ mol–1 (b) 300 kJ mol–1
glucose(s) at 25°C are –400 kJ/mol, –300 kJ/mol and –1300 (c) 200 kJ mol–1 (d) 400 kJ mol–1
kJ/mol, respectively. The standard enthalpy of combustion 24. Given that:
per gram of glucose at 25°C is
(i) D f H° of N 2O is 82 kJ mol –1
(a) +2900 kJ (b) –2900 kJ
(c) –16.11 kJ (d) +16.11 kJ (ii) Bond energies of N º N, N = N, O = O and N = O are
20. If at 298 K the bond energies of C — H, C — C, C = C and H 946, 418, 498 and 607 kJ mol–1 respectively,
— H bonds are respectively 414, 347, 615 and 435 kJ mol –1, The resonance energy of N2O is :
the value of enthalpy change for the reaction (a) –88kJ (b) –66kJ
H 2 C = CH 2 (g) + H 2 (g) ® H 3C — CH 3 (g) at 298 K (c) –62kJ (d) –44kJ
will be

RESPONSE 15. 16. 17. 18. 19.


GRID 20. 21. 22. 23. 24.

Space for Rough Work


EBD_7504
C-24 NTA JEE Main

25. Given that bond energies of H – H and Cl – Cl are 430 kJ mol– 1 29. Read the following statements carefully and choose the
and 240 kJ mol–1 respectively and DHf for HCl is – 90 kJ correct answer
mol– 1, bond enthalpy of HCl is (i) Expansion of a gas in vacuum (pex = 0) is called free
(a) 380 kJ mol–1 (b) 425 kJ mol–1 expansion.
(c) 245 kJ mol –1 (d) 290 kJ mol–1 (ii) Work is done during free expansion of an ideal gas
26. Under isothermal condition for one mole of ideal gas what whether the process is reversible or irreversible
is the ratio of work done under reversible to irreversible (iii) No work is done during free expansion of an ideal gas
process, initially held at 20 atm undergoes expansion from whether the process is reversible or irreversible
1L to 2L, at 298K, under external pressure of 10 atm?
(iv) No work is done during free expansion of an ideal gas
(a) 1.7 (b) 2.0
when the process is reversible
(c) 1.4 (d) 1.0
27. Temperature of 5 moles of a gas is decreased by 2K at constant (a) Only statement (iii) is correct
pressure. Indicate the correct statement (b) Statements (i) and (iii) are correct
(a) Work done by gas is = 5 R (c) Statements (ii) and (iv) are correct
(b) Work done over the gas is = 10 R (d) Statements (i) and (iv) are correct
(c) Work done by the gas = 10 R 30. The (S°) of the following substances are:
(d) Work done = 0 CH4 (g) 186.2 JK–1 mol–1
O2 (g) 205.2 JK–1 mol–1
28. For complete combustion of ethanol,
CO2 (g) 213.6 JK–1 mol–1
C2 H5 OH ( l ) + 3O2 ( g ) ¾¾
® 2CO2 ( g ) + 3H 2 O ( l ) , H2O (g) 69.9.JK–1 mol–1
the amount of heat produced as measured in bomb The entropy change (DS°) for the reaction
calorimeter, is 1364.47 kJ mol–1 at 25ºC. Assuming ideality CH4(g) + 2O2(g) ® CO2(g) + 2H2O(l) is:
the enthalpy of combustion, DcH, for the reaction will be: (a) – 312.5 J K–1 mol–1
(R = 8.314 kJ mol–1) (b) – 242.8 J K–1 mol–1
(a) -1366.95 kJ mol-1 (b) -1361.95 kJ mol-1 (c) – 108.1 J K–1 mol–1
(c) -1460.95 kJ mol-1 (d) -1350.50 kJ mol-1 (d) – 37.6 J K–1 mol–1

RESPONSE 25. 26. 27. 28. 29.


GRID 30.

CHEMISTRY CHAPTERWISE SPEED TEST-35


Total Questions 30 Total Marks 120
Attempted Correct
Incorrect Net Score
Cut-off Score 36 Qualifying Score 52
Success Gap = Net Score – Qualifying Score
Net Score = (Correct × 4) – (Incorrect × 1)
CHEMISTRY Speed
Equilibrium TEST
No. of Questions
30
Maximum Marks
120
Time
1 Hour
36
Chapter-wise

GENERAL INSTRUCTIONS
• This test contains 30 MCQ's. For each question only one option is correct. Darken the correct circle/ bubble in the
Response Grid provided on each page.
• You have to evaluate your Response Grids yourself with the help of solutions provided at the end of this book.
• Each correct answer will get you 4 marks and 1 mark shall be deduced for each incorrect answer. No mark will be given/
deducted if no bubble is filled. Keep a timer in front of you and stop immediately at the end of 60 min.
• The sheet follows a particular syllabus. Do not attempt the sheet before you have completed your preparation for that
syllabus.
• After completing the sheet check your answers with the solution booklet and complete the Result Grid. Finally spend time
to analyse your performance and revise the areas which emerge out as weak in your evaluation.

1. Kc for the reaction (a) 1 : 10 (b) 4 : 5


N2 (g) + O2 (g) ƒ 2NO (g) (c) 10 : 1 (d) 5 : 4
at 300 K is 4.0 × 10–6. Kp for the above reaction will be 4. The exothermic formation of ClF3 is represented by the
(R = 2 cal mol–1 K–1) equation :
(a) 2.4 × 10–3 (b) 4 × 10–6 Cl2 (g) + 3F2 (g) ƒ 2ClF3(g) ; Δ H = – 329 kJ
–6
(c) 4 × 10 (RT) 2 (d) 16 × 10–12
2. At 100°C the Kw of water is 55 times its value at 25°C. What Which of the following will increase the quantity of ClF3 in
an equilibrium mixture of Cl2, F2 and ClF3?
will be the pH of neutral solution? (log 55 = 1.74)
(a) 6.13 (b) 7.00 (a) Adding F2
(c) 7.87 (d) 5.13 (b) Increasing the volume of the container
3. The dissociation constant of a weak acid is 1 × 10 – 4. In order
to prepare a buffer solution with a pH = 5 the [Salt]/[Acid] (c) Removing Cl2
ratio should be (d) Increasing the temperature

RESPONSE GRID 1. 2. 3. 4.

Space for Rough Work


EBD_7504
C-26 NTA JEE Main

5. The Ksp of Ag2CrO4, AgCl, AgBr and AgI are respectively, 9. The equilibrium constant of the following are :
1.1 × 10–12, 1.8 × 10–10, 5.0 × 10–13, 8.3 × 10–17. Which N2 + 3H2 ƒ 2NH3 K1
one of the following salts will precipitate last if AgNO3 N2 + O2 ƒ 2NO K2
solution is added to the solution containing equal moles of 1
NaCl, NaBr, NaI and Na2CrO4? H 2 + O 2 ® H 2O K3
2
(a) AgCl (b) AgBr The equilibrium constant (K) of the reaction :
(c) Ag2CrO4 (d) AgI
5 K
ˆˆˆ†
6. MY and NY3, two nearly insoluble salts, have the same Ksp 2NH3 + O 2 ‡ˆˆˆ 2NO + 3H2O, will be;
values of 6.2 × 10–13 at room temperature. Which statement 2
would be true in regard to MY and NY3 ? (a) K 2 K33 / K1 (b) K2K3/K1
(a) The molar solubilities of MY and NY3 in water are
identical. (c) K32 K3 / K1 (d) K1K 33 / K 2
(b) The molar solubility of MY in water is less than that of 10. The %yield of ammonia as a function of time in the reaction
NY3 N2(g) + 3H2(g) ƒ 2NH3(g), DH < 0 at (P, T1) is given below
(c) The salts MY and NY3 are more soluble in 0.5 M KY
than in pure water.
(d) The addition of the salt of KY to solution of MY and

% y i e ld
NY3 will have no effect on their solubilities.
7. The following reaction is performed at 298 K.
2NO(g) + O2(g) ƒ 2NO2(g) time
The standard free energy of formation of NO(g) is 86.6 kj/mol If this reaction is conducted at (P, T2), with T2 > T1, the
at 298 K. What is the standard free energy of formation of %yield of ammonia as a function of time is represented by
NO2(g) at 298 K? (Kp = 1.6 × 1012)
T2 T1
ln (1.6 ´ 1012 )
(a) 86600 – % y ie ld
T1 T2

% y i e ld
R (298)
(b) 0.5[2 × 86,600 – R(298) ln(1.6 × 1012)]
(c) R(298) ln(1.6 × 1012) – 86600 (a) (b)
(d) 86600 + R(298) ln(1.6 × l012)
time time
8. Consider the following liquid - vapour equilibrium.
Liquid ƒ Vapour T2
T1
Which of the following relations is correct ?
T1
dlnG DH v
% y ie ld

dlnP DH v

% y ie ld
(a) 2
= 2 (b) = T2
dT RT dT RT (c) (d)
dlnP -DH v dlnP -DH v
(c) = (d) =
dT 2 T2 dT RT 2 time time

RESPONSE 5. 6. 7. 8. 9.
GRID 10.
Space for Rough Work
Chemistry C-27

11. Concentration of the Ag+ ions in a saturated solution of (a) 6.3 atm (b) 6.93 atm
Ag2 C2O4 is 2.2 × 10–4 mol L–1. Solubility product of (c) 0.63 atm (d) 0.693 atm
Ag2C2O4 is :-
16. The first and second dissociation constants of an acid H2 A
(a) 2.66 × 10–12 (b) 4.5 × 10–11
are 1.0 × 10–5 and 5.0 × 10–10 respectively. The overall
(c) 5.3 × 10–12 (d) 2.42 × 10–8 dissociation constant of the acid will be
12. For the following reaction in gaseous phase (a) 0.2 × 105 (b) 5.0 × 10–5
1
CO( g ) + O2 ( g ) ® CO2 ( g ), K p / K c is (c) 5.0 × 1015 (d) 5.0 × 10–15.
2
(a) (RT)1/2 (b) (RT)–1/2 17. 2.5 ml of (2/5) M weak monoacidic base (Kb = 1 × 10–12 at
25°) is titrated with (2/15) M HCl in water at 25°C. The
(c) (RT) (d) (RT)–1
concentration of H+ at equivalence point is (Kw = 1 × 10–14
13. The degree of dissociation of PCl5 (a) obeying the at 25°C)
equilibrium PCl 5 ƒ PCl 3 + Cl 2 is related to the equilibrium (a) 3.7 × 10–14 M (b) 3.2 × 10–7 M
pressure by (c) 3.2 × 10–2 M (d) 2.7 × 10–2 M
18. Calculate the pH of a solution obtained by diluting 1 mL of
1 1
(a) a µ (b) a µ 0.10 M weak monoacidic base to 100 mL at constant
P4 P temperature if Kb of the base is 1 × 10–5 ?
1 (a) 8 (b) 9
(c) a µ 2 (d) a µ P
P (c) 10 (d) 11
14. A 20 litre container at 400 K contains CO2(g) at pressure 0.4 19. In a saturated solution of the sparingly soluble strong
atm and an excess of SrO (neglect the volume of solid SrO). electrolyte AgIO3 (molecular mass = 283) the equilibrium
The volume of the container is now decreased by moving
ˆˆ† Ag + ( aq ) + IO3- ( aq ) . If the
which sets is AgIO3(s) ‡ˆˆ
the movable piston fitted in the container. The maximum
volume of the container, when pressure of CO 2 attains its solubility product constant Ksp of AgIO3 at a given
maximum value, will be :- temperature is 1.0 × 10–8, what is the mass of AgIO3
(Given that : SrCO3 (s) ƒ SrO (s) + CO2(g), Kp = 1.6 atm) contained in 100 ml of its saturated solution?
(a) 10 litre (b) 4 litre (a) 1.0 × 10– 4 g (b) 28.3 × 10–2 g
(c) 2 litre (d) 5 litre (c) 2.83 × 10–3 g (d) 1.0 × 10–7 g.
15. For the reaction C(s) + CO2(g) ® 2CO(g), Kp = 63 atm at 1000 20. Species acting as both Bronsted acid and base is
K. If at equilibrium : Pco = 10 Pco2, then the total pressure (a) (HSO4)–1 (b) Na2CO3
of the gases at equilibrium is (c) NH3 (d) OH–1.

RESPONSE 11. 12. 13. 14. 15.


GRID 16. 17. 18. 19. 20.
Space for Rough Work
EBD_7504
C-28 NTA JEE Main

21. Let the solubility of an aqueous solution of Mg(OH)2 be x When K p and K c are compared at 184°C, it is found that
then its Ksp is
(a) 4x3 (b) 108x5 (a) Whether Kp is greater than, less than or equal to Kc
(c) 27x 4 (d) 9x. depends upon the total gas pressure
22. The Ksp for Cr(OH)3 is 1.6 × 10–30. The solubility of this (b) Kp = Kc
compound in water is : (c) Kp is less than Kc
(d) Kp is greater than Kc
(a) 4
1.6 ´ 10 -30 (b) 4
1.6 ´ 10 -30 / 27
27. Zirconium phosphate [Zr 3(PO4)4] dissociates into three
(c) 1.6 ´ 10-30/ 27 (d) 1.6 ´ 10 -30 zirconium cations of charge + 4 and four phosphate anions
23. Which equilibrium can be described as an acid-base reaction of charge – 3. If molar solubility of zirconium phosphate is
using the Lewis acid-base definition but not using the denoted by S and its solubility product by Ksp then which
Bronsted-Lowry definition? of the following relationship between S and Ksp is correct?
(a) 2NH3 + H2SO4 ƒ 2NH4+ + SO42– (a) S = {Ksp/ (6912)1/7} (b) S = {Ksp/ 144}1/7
(b) NH3 + CH3COOH ƒ NH4+ + CH3COO– (c) S = {Ksp/ 6912}1/7 (d) S = {Ksp/ 6912}7
(c) H2O + CH3COOH ƒ H3O+ + CH3COO– 28. The pKa of a weak acid, HA, is 4.80. The pKb of a weak base,
(d) [Cu(H2O)4]2– + 4 NH3 ƒ [Cu(NH3)4]2+ + 4H2O BOH, is 4.78. The pH of an aqueous solution of the
24. Which one of the following statements is not true? correspondng salt, BA, will be
(a) pH + pOH = 14 for all aqueous solutions (a) 9.58 (b) 4.79
(b) The pH of 1 × 10–8 M HCl is 8 (c) 7.01 (d) 9.22
(c) 96,500 coulombs of electricity when passed through a 29. Which one of the following arrangements represents the
CuSO4 solution deposits 1 gram equivalent of copper correct order of solubilities of sparingly soluble salts
at the cathode Hg2Cl2, Cr 2(SO4)3, BaSO4 and CrCl3 respectively ?
(d) The conjugate base of H 2 PO -4 is HPO 24- (a) BaSO4 > Hg 2 Cl2 > Cr2 (SO 4 )3 > CrCl3
25. How many litres of water must be added to 1 litre an aqueous (b) BaSO4 > Hg 2 Cl2 > CrCl3 > Cr2 (SO4 )3
solution of HCl with a pH of 1 to create an aqueous solution
with pH of 2 ? (c) BaSO4 > CrCl3 > Hg 2Cl2 > Cr2 (SO4 )3
(a) 0.1 L (b) 0.9 L (c) 2.0 L (d) 9.0 L (d) Hg 2Cl 2 > BaSO4 > CrCl3 > Cr2 (SO4 )3
26. For the reaction :
30. Calculate the pH of 0.5 M aqueous solution of NaCN, the
ˆˆ† 2NO(g) + O 2 ( g ) ,
2NO2(g) ‡ˆˆ pKb of CN - is 4.70
(K c = 1.8 ´ 10- 6 at 184°C) (R = 0.0831 kJ/ (mol. K)) (a) 4.70 (b) 11.5
(c) 7 (d) 6.5

21. 22. 23. 24. 25.


RESPONSE GRID
26. 27. 28.
Space for Rough Work
29. 30.

CHEMISTRY CHAPTERWISE SPEED TEST-36


Total Questions 30 Total Marks 120
Attempted Correct
Incorrect Net Score
Cut-off Score 35 Qualifying Score 50
Success Gap = Net Score – Qualifying Score
CHEMISTRY Speed
Redox Reaction TEST
No. of Questions
30
Maximum Marks
120
Time
1 Hour
37
Chapter-wise

GENERAL INSTRUCTIONS
• This test contains 30 MCQ's. For each question only one option is correct. Darken the correct circle/ bubble in the
Response Grid provided on each page.
• You have to evaluate your Response Grids yourself with the help of solutions provided at the end of this book.
• Each correct answer will get you 4 marks and 1 mark shall be deduced for each incorrect answer. No mark will be given/
deducted if no bubble is filled. Keep a timer in front of you and stop immediately at the end of 60 min.
• The sheet follows a particular syllabus. Do not attempt the sheet before you have completed your preparation for that
syllabus.
• After completing the sheet check your answers with the solution booklet and complete the Result Grid. Finally spend time
to analyse your performance and revise the areas which emerge out as weak in your evaluation.

1. In which of the following pairs, there is greatest difference in 3. Several blocks of magnesium are fixed to the bottom of a
the oxidation number of the underlined elements ? ship to
(a) NO 2 and N 2 O 4 (b) P 2 O 5 and P 4 O10 (a) make the ship lighter
(b) prevent action of water and salt
(c) N 2 O and N O (d) SO2 and SO3 (c) prevent puncturing by under-sea rocks
2. Which of the following is a redox reaction? (d) keep away the sharks
(a) NaCl + KNO3 ® NaNO3 + KCl 4. When KMnO4 reacts with acidified FeSO4
(a) FeSO4 is oxidised and KMnO4 is reduced
(b) CaC2O4 + 2HCl ® CaCl2 + H2C2O4
(b) only KMnO4 is oxidised
(c) Mg(OH)2 + 2NH4Cl ® MgCl2 + 2NH4OH
(c) only FeSO4 is oxidised
(d) Zn + 2AgCN ® 2Ag + Zn(CN)2. (d) None of these

RESPONSE GRID 1. 2. 3. 4.

Space for Rough Work


EBD_7504
C-30 NTA JEE Main

5. Which of the following chemical reactions depict the The values of X, Y and Z in the above redox reaction are
oxidizing beahviour of H2SO4? respectively :
(a) NaCl + H 2 SO 4 ¾¾® NaHSO 4 + HCl (a) 2, 1, 2 (b) 2, 1, 3
(b) (c) 3, 1, 6 (d) 3, 1, 4
2PCl5 + H 2SO 4 ¾¾
® 2POCl3 + 2HCl + SO 2 Cl2
10. Oxidation state of sulphur in anions SO32 - , S2 O42 - and
(c) 2 HI + H 2SO 4 ¾
¾® I 2 + SO 2 + 2 H 2 O S2 O62- increases in the orders :
(d) Ca(OH) 2 + H 2SO 4 ¾¾
® CaSO 4 + 2H 2 O (a) S2 O62 - < S2 O24 - < SO32 -
6. Which of the following statements are correct concerning (b) SO26 - < S2 O24 - < S2 O62 -
redox properties? (c) S2 O42 - < SO32 - < S2 O62 -
(i) A metal M for which E° for the half life reaction (d) S2 O42 - < S2 O26 - < SO32 -
Mn+ + ne– ‡ˆˆ ˆˆ† M is very negative will be a good 11. Amongst the following, identify the species with an atom in
reducing agent. + 6 oxidation state:
(ii) The oxidizing power of the halogens decreases from (a) [MnO4]– (b) [Cr(CN)6]3–
chlorine to iodine. (c) Cr2O3 (d) CrO2Cl2
(iii) The reducing power of hydrogen halides increases from 12. Which one of the following cannot function as an oxidising
hydrogen chloride to hydrogen iodide agent ?
(a) (i), (ii) and (iii) (b) (i) and (ii) (a) I– (b) S(s)
(c) (i) only (d) (ii) and (iii) (c) NO3- (aq) (d) Cr2 O72-
7. In the following balanced reaction, 13. A compound of Xe and F is found to have 53.5% of Xe.
X MnO4- + Y C2 O24 - + Z H + What is oxidation number of Xe in this compound ?
Z (a) –4 (b) 0
X Mn 2+ + 2Y CO 2 + H 2 O
2 (c) +4 (d) +6
values of X, Y and Z respectively are 14. Copper becomes green when exposed to moist air for a long
(a) 2, 5, 16 (b) 8, 2, 5 period. This is due to:
(c) 5, 2, 16 (d) 5, 8, 4
(a) the formation of a layer of cupric oxide on the surface
8. Arrange the following in the order of their decreasing
of copper.
electrode potentials : Mg, K, Ba and Ca
(a) K, Ca, Ba, Mg (b) Ba, Ca, K, Mg (b) the formation of a layer of basic carbonate of copper
(c) Ca, Mg, K, Ba (d) Mg, Ca, Ba, K on the surface of copper.
9. Given : (c) the formation of a layer of cupric hydroxide on the
XNa2HAsO3 + YNaBrO3 + ZHCl ® surface of copper.
NaBr + H3AsO4 + NaCl (d) the formation of basic copper sulphate layer on the
surface of the metal.

RESPONSE 5. 6. 7. 8. 9.
GRID 10. 11. 12. 13. 14.
Space for Rough Work
Chemistry C-31

15. In the standardization of Na 2S 2O3 using K2Cr 2O7 by (a) oxygen is more – ve in SO2
iodometry, the equivalent weight of K2Cr2O7 is (b) hydrogen in H2S is more + ve than oxygen
(a) (molecular weight)/2 (c) S in SO2 has one oxidation state
(b) (molecular weight)/6 (d) S in H2S has – 2 oxidation state
(c) (molecular weight)/3 20. Which of the following cannot act as reducing agent?
(d) same as molecular weight (a) NO2 (b) SO2
16. Consider the reaction: (c) CO2 (d) ClO2
21. C2H6 (g) + nO2(g) ® CO2(g) + H2O(l)
H 2SO3 (aq) + Sn 4+ (aq) + H 2 O(l) In this equation, the ratio of the coefficients of CO2 and
® Sn 2 + (aq) + HSO4- (aq) + 3H + (aq) H2O is
(a) 1 : 1 (b) 2 : 3
Which of the following statements is correct? (c) 3 : 2 (d) 1 : 3
(a) Sn 4+ is the oxidizing agent because it undergoes 22. Which substance serves as reducing agent in the following
oxidation reaction ?
(b) Sn 4+ is the reducing agent because it undergoes 14H+ + Cr2 O 27 - + 3Ni ¾
¾® 2Cr3+ + 7H2O + 3Ni2+
oxidation
(a) H2O (b) Ni
(c) H2SO3 is the reducing agent because it undergoes
oxidation (c) H+ (d) Cr2 O27-
(d) H2SO3 is the reducing agent because it undergoes 23. Which of the following reactions depict the oxidising
reduction behaviour of H2SO4 :
17. The species that undergoes disproportionation in an alkaline (a) 2PCl5 + H 2SO4 ¾¾
® 2POCl3 + 2HCl + SO2Cl 2
medium are
(a) Cl2 (b) MnO24 - (b) 2NaOH + H 2SO 4 ¾¾
® Na 2SO 4 + H 2 O
(c) NO2 (d) All of these (c) NaCl + H 2 SO4 ¾¾
® NaHSO 4 + HCl
18. How many electrons are involved in the following redox
reaction? (d) 2HI + H 2 SO 4 ¾¾
® I 2 + SO 2 + 2H 2 O
24. Which one of the following reactions involves
Cr2 O72 - + Fe2 + + C 2O 24- ® Cr3+ + Fe3+ + CO2 (Unbalanced)
disproportionation?
(a) 3 (b) 4 (a) 2H2SO4 + Cu ® CuSO4 + 2H2O + SO2
(c) 6 (d) 5 (b) As2O3 + 3H2S ® As2S3 + 3H2O
19. H2S acts only as a reducing agent while SO 2 can act both as (c) 2KOH + Cl2 ® KCl + KOCl + H2O
a reducing and oxidizing agent because (d) Ca3P2 + 6H2O ® 3Ca(OH)2 + 2PH3

RESPONSE 15. 16. 17. 18. 19.


GRID 20. 21. 22. 23. 24.
Space for Rough Work
EBD_7504
C-32 NTA JEE Main

25. Point out the correct statement of the following about (c) Thiosulphate undergoes oxidation by bromine and
Na2S4O6. reduction by iodine in these reactions
(a) Average oxidation number of S atom is +2 (d) Bromine undergoes oxidation and iodine undergoes
(b) Oxidation number of two S atoms is zero each and that reduction in these reactions
of other two is +5 each 28. Standard reduction potentials of the half reactions are given
(c) Oxidation number of two S atoms is +1 each and that of below :
other two is +4 each F2(g) + 2e– ® 2F– (aq); E° = + 2.85 V
(d) None of these Cl2(g) + 2e– ® 2Cl–(aq); E° = + 1.36 V
26. The pair of compounds in which both the metals are in the Br2(l) + 2e– ® 2Br–(aq); E° = + 1.06 V
highest possible oxidation state is I2(s) + 2e– ® 2I–(aq); E° = + 0.53 V
(a) [Fe(CN ) 6 ]4 - , [Co(CN ) 6 ]3- The strongest oxidising and reducing agents respectively
are :
(b) CrO2 Cl 2 , MnO-4
(a) F2 and I– (b) Br2 and Cl–
(c) Cl2 and Br – (d) Cl2 and I2
(c) TiO 3 , MnO 2
29. A mixture of potassium chlorate, oxalic acid and sulphuric
(d) [Co(CN) 6 ]3- , MnO3 acid is heated. During the reaction which element undergoes
27. Thiosulphate reacts differently with iodine and bromine in maximum change in the oxidation number ?
the reactions given below. (a) S (b) H
(c) Cl (d) C
2S2 O32 - + I 2 ® S4 O62 - + 2I - 30. Zn gives H2 gas with H2SO4 and HCl but not with HNO3
S2 O32 - + 2Br 2 + 5H 2O ® 2SO24 - + 2Br - + 10H + because
(a) Zn acts as an oxidising agent when it reacts with HNO3
Which of the following statements justifies the above dual (b) HNO3 is weaker acid than H2SO4 and HCl
behaviour of thiosulphate? (c) In electrochemical series, Zn is above hydrogen
(a) Bromine is a stronger oxidant than iodine
(b) Bromine is a weaker oxidant than iodine (d) NO3- is reduced in preference to hydronium ion

RESPONSE 25. 26. 27. 28. 29.


GRID 30.

CHEMISTRY CHAPTERWISE SPEED TEST-37


Total Questions 30 Total Marks 120
Attempted Correct
Incorrect Net Score
Cut-off Score 40 Qualifying Score 60
Success Gap = Net Score –Space
Qualifying
for Rough Work
Score
Net Score = (Correct × 4) – (Incorrect × 1)
Space for Rough Work
CHEMISTRY Speed
Hydrogen TEST
No. of Questions
30
Maximum Marks
120
Time
1 Hour
38
Chapter-wise

GENERAL INSTRUCTIONS
• This test contains 30 MCQ's. For each question only one option is correct. Darken the correct circle/ bubble in the
Response Grid provided on each page.
• You have to evaluate your Response Grids yourself with the help of solutions provided at the end of this book.
• Each correct answer will get you 4 marks and 1 mark shall be deduced for each incorrect answer. No mark will be given/
deducted if no bubble is filled. Keep a timer in front of you and stop immediately at the end of 60 min.
• The sheet follows a particular syllabus. Do not attempt the sheet before you have completed your preparation for that
syllabus.
• After completing the sheet check your answers with the solution booklet and complete the Result Grid. Finally spend time
to analyse your performance and revise the areas which emerge out as weak in your evaluation.

1. Which of the following will not displace hydrogen 3. The adsorption of hydrogen by metals is called
(a) Ba (a) Dehydrogenation
(b) Pb (b) Hydrogenation
(c) Hg (c) Occlusion
(d) Sn (d) Adsorption
2. Which statement is correct for hydrogen ? 4. The low density of ice compared to water is due to
(a) It has a very high ionisation potential (a) hydrogen bonding interactions
(b) It is always collected at cathode (b) dipole – dipole interactions
(c) It can exists both in +1 as well as –1 oxidation state (c) dipole – induced dipole interactions
(d) It has same electronegativity as halogens (d) induced dipole – induced dipole interactions

RESPONSE GRID 1. 2. 3. 4.

Space for Rough Work


EBD_7504
C-34 NTA JEE Main

5. Which of the following terms is not correct for hydrogen ?


C n H 2n + 2 + nH 2 O ( g ) ¾¾¾¾
1270 K
(c) ® nCO + ( 2n + 1) H 2
(a) Its molecule is diatomic Ni

(b) It exists both as H+ and H– in different chemical


CO ( g ) + 2H 2 ( g ) ¾¾¾¾
Cobalt
compounds (d) ® CH3OH ( l)
Catalyst
(c) It is the only species which has no neutrons in the 10. Elements of which of the following group(s) of periodic table
nucleus do not form hydrides?
(d) Heavy water is unstable because hydrogen is (a) Groups 7, 8, 9 (b) Group 13
substituted by its isotope deuterium
(c) Groups 15, 16, 17 (d) Group 14
6. Hydrogen can be fused to form helium at
11. Which one of the following pairs of substances will not
(a) High temperature and high pressure produce hydrogen when reacted together?
(b) High temperature and low pressure (a) Copper and conc. nitric acid
(c) Low temperature and high pressure (b) Ethanol and metallic sodium
(d) Low temperature and low pressure (c) Magnesium and steam
7. The hydride ion, H–, is a stronger base than the hydroxide (d) Phenol and metallic sodium
ion, OH–. Which one of the following reactions will occur if
12. Which of the following is not a use of dihydrogen ?
sodium hydride (NaH) is dissolved in water?
(a) It used in fuel cells for generating electrical energy.
(a) H - (aq ) + H 2 O(l) ® H3O - (aq )
(b) Atomic hydrogen and oxy-hydrogen torches are
(b) H - ( aq ) + H 2 O( l ) ® OH - ( aq ) + H 2 ( g ) used for cutting and welding purposes.
(c) H - ( aq ) + H 2O(l ) ® OH - ( aq ) + 2 H + ( aq ) + 2e (c) It used in the synthesis of hydroquinone and tartaric
acid.
(d) H - (aq ) + H 2O(l) ® No reaction
(d) Both (b) and (c)
8. Blackened oil painting can be restored into original form by
13. Calculate the normality of 10 volume H2O2 ?
the action of
(a) 1.7 N (b) 12 N
(a) Chlorine (b) BaO2
(c) 30.3 N (d) 0.0303 N
(c) H2O2 (d) MnO2
14. At its melting point ice is lighter than water because
9. Which of the following reactions is an example of use of
water gas in the synthesis of other compounds? (a) H2O molecules are more closely packed in solid state
(b) ice crystals have hollow hexagonal arrangement of H2O
CH 4 ( g ) + H 2 O ( g ) ¾¾¾¾
® CO ( g ) + H 2 ( g )
1270 K
(a) Ni
molecules.
(c) on melting of ice the H2O molecule shrinks in size
CO ( g ) + H 2 O ( g ) ¾¾¾¾
® CO 2 ( g ) + H 2 ( g )
673 K
(b) Catalyst
(d) ice froms mostly heavy water on first melting.

RESPONSE 5. 6. 7. 8. 9.
GRID 10. 11. 12. 13. 14.
Space for Rough Work
Chemistry C-35

15. Identify x and y in following reaction: 20. Hydrogen molecules differs from chlorine molecule in the
following respect
2HSO-4 (aq) ¾¾¾¾¾
electrolysis hydrolysis
® x ¾¾¾¾¾ ®
(a) Hydrogen molecule is non-polar but chlorine molecule
y + 2H+(aq) + H2O2(aq)
is polar
(a) x = H2SO4 (aq), y = 2HSO -4 (aq) (b) Hydrogen molecule is polar while chlorine molecule is
(b) x = HO3SOOSO3H(aq), y = 2HSO -4 (aq) non-polar
(c) x = HO3SOOSO3H (aq), y = H2SO4(aq) (c) Hydrogen molecule can form intermolecular hydrogen
bonds but chlorine molecule does not
(d) x = H2SO4(aq) , y = HO3SOOSO3H(aq)
(d) Hydrogen molecule cannot participate in coordination
16. Which of the following species is diamagnetic in nature?
bond formation but chlorine molecule can
(a) H -2 (b) H +2 21. Which of the following chemicals is not present in clear
(c) H2 (d) He +2 hard water?
(a) MgCO3 (b) MgSO4
17. The number of moles of H2 in 0.224 litre of hydrogen gas
(c) Mg(HCO3)2 (d) CaCl2
at STP (273 K, 1 atm) is :
(a) 0.1 22. True peroxide is
(b) 0.01 (a) BaO 2 (b) MnO2
(c) 0.001
(c) PbO 2 (d) NO2
(d) 1
23. Very pure hydrogen (99.9) can be made by which of the
18. Hydrogen is not obtained when Zn reacts with
following processes ?
(a) cold water
(a) Reaction of methane with steam
(b) dil H2SO4
(b) Mixing natural hydrocarbons of high molecular weight
(c) dil. HCl
(c) Electrolysis of water
(d) 20% NaOH
(d) Reaction of salts like hydrides with water
19. Which of the following statements in relation to the
hydrogen atom is correct ? 24. An inorganic compound gives off O2 when heated, turns an
acidic solution of KI violet and reduces acidified
(a) 3s, 3p and 3d orbitals all have the same energy
KMnO4. The compound is
(b) 3s and 3p orbitals are of lower energy than 3d orbital
(a) SO3 (b) KNO3
(c) 3p orbital is lower in energy than 3d orbital
(d) 3s orbital is lower in energy than 3p orbital (c) H2O2 (d) All of these

RESPONSE 15. 16. 17. 18. 19.


GRID 20. 21. 22. 23. 24.
Space for Rough Work
EBD_7504
C-36 NTA JEE Main

25. Hydrogen peroxide acts both as an oxidising and as a 28. From the following statements regarding H2O2, choose the
reducing agent depending upon the nature of the reacting incorrect statement :
species. In which of the following cases H2O2 acts as a (a) It has to be stored in plastic or wax lined glass bottles in
reducing agent in acid medium? dark
(a) MnO-4 (b) Cr2 O72- (b) It has to be kept away from dust
(c) It can act only as an oxidizing agent
(c) SO32- (d) KI
(d) It decomposes on exposure to light
26. Choose the correct statement : 29. When zeolite (hydrated sodium aluminium silicate) is treated
The reason for use of polyphosphates as water softening with hard water the sodium ions are exchanged with
agents is, that
(a) H+ ions (b) Ca2+ ions
(a) they form soluble complexes with anionic species
(b) they precipitate out cationic species (c) SO 4 2 - ions (d) OH– ions
(c) they precipitate out anionic species 30. Identify x and y in following reaction. What is the mixture
(d) they form soluble complexes with cationic species of x and y called?
27. Which statement is wrong? CH4(g) + H2O(g) ¾¾¾
1270 K
®x + y
Ni
(a) Ordinary hydrogen is an equilibrium mixture of ortho
and para hydrogen (a) x = CO2, y = H2O, water gas
(b) In ortho hydrogen spin of two nuclei is in same direction (b) x = CO, y = H2O, syn gas
(c) Ortho and para forms do not resemble in their chemical (c) x = CO, y = H2, water gas
properties (d) x = CO2, y = H2, syn gas
(d) In para hydrogen spin of two nuclei is in opposite
direction.

RESPONSE 25. 26. 27. 28. 29.


GRID 30.

CHEMISTRY CHAPTERWISE SPEED TEST-38


Total Questions 30 Total Marks 120
Attempted Correct
Incorrect Net
Space Score
for Rough Work

Cut-off Score 40 Qualifying Score 60


Success Gap = Net Score – Qualifying Score
Net Score = (Correct × 4) – (Incorrect × 1)
Space for Rough Work
CHEMISTRY Speed
The s-Block Elements TEST
No. of Questions
30
Maximum Marks
120
Time
1 Hour
39
Chapter-wise

GENERAL INSTRUCTIONS
• This test contains 30 MCQ's. For each question only one option is correct. Darken the correct circle/ bubble in the
Response Grid provided on each page.
• You have to evaluate your Response Grids yourself with the help of solutions provided at the end of this book.
• Each correct answer will get you 4 marks and 1 mark shall be deduced for each incorrect answer. No mark will be given/
deducted if no bubble is filled. Keep a timer in front of you and stop immediately at the end of 60 min.
• The sheet follows a particular syllabus. Do not attempt the sheet before you have completed your preparation for that
syllabus.
• After completing the sheet check your answers with the solution booklet and complete the Result Grid. Finally spend time
to analyse your performance and revise the areas which emerge out as weak in your evaluation.

1. Alkali metals are generally extracted by 3. Strongest bond is in between


(a) reduction methods (a) CsF
(b) double decomposition methods (b) NaCl
(c) displacement methods (c) Both (a) and (b)
(d) electrolytic methods (d) None of above
2. Which of the following represents a correct sequence of 4. A metal salt solution forms a yellow precipitate with
reducing power of the following elements? potassium chromate in acetic acid, a white precipitate with
(a) Li > Cs > Rb dil H2SO4, but gives no precipitate with NaCl. The metal
(b) Rb > Cs > Li salt solution will consist of
(c) Cs > Li > Rb (a) PbCO3 (b) BaCO3
(d) Li > Rb > Cs (c) MgCO3 (d) CaCO3

RESPONSE GRID 1. 2. 3. 4.

Space for Rough Work


EBD_7504
C-38 NTA JEE Main

5. Which of the following has lowest thermal stability ? 11. Which of the following statements about Na2O2 is not
(a) Li2CO3 (b) Na 2CO3 correct?
(a) It is diamagnetic in nature
(c) K 2CO3 (d) Rb2CO3 (b) It is derivative of H2O2
6. The first (IE1) and second (IE2) ionisation energies (kJ/mol)
(c) Na2O2 oxidises Cr3+ to CrO24 - in acid medium.
of a few elements designated by Roman numerals are given
below. Which of these would be an alkali metal? (d) It is the super oxide of sodium
IE1 IE2
(a) I 2372 5251 12. All of the following substances react with water. The pair
(b) II 520 7300 that gives the same gaseous product is
(c) III 900 1760 (a) K and KO2
(d) IV 16803 380 (b) Na and Na2O2
7. The solubilities of carbonates decrease down the magnesium (c) Ca and CaH2
group due to a decrease in (d) Ba and BaO2
(a) hydration energies of cations 13. Which is not correctly matched?
(b) inter-ionic attraction (1) Basic strength Cs2O < Rb2O < K2O < Na2O < Li2O of
(c) entropy of solution formation oxides
(d) lattice energies of solids (2) Stability of Na2O2 < K2O2 < Rb2O2 < Cs2O2 perox-
8. KO2 (potassium super oxide) is used in oxygen cylinders in ides
space and submarines because it (3) Stability of LiHCO3 < NaHCO3 < KHCO3 bicarbonates
(a) absorbs CO2 and increases O2 content < RbHCO3 < CsHCO3
(b) eliminates moisture (4) Melting point NaF < NaCl < NaBr < NaI
(c) absorbs CO2 (a) 1 and 4
(d) produces ozone. (b) 1 and 3
9. Which one of the following salts does not impart colour to (c) 1 and 2
the flame ? (d) 2 and 3
(a) KI (b) LiCl 14. If NaOH is added to an aqueous solution of Zn 2 + ions, a
(c) CaCl2 (d) MgCl2 white precipitate appears and on adding excess NaOH, the
10. Amongst LiCl, RbCl, BeCl 2 and MgCl2 the compounds precipitate dissolves. In this solution zinc exists in the :
with the greatest and least ionic character respectively are : (a) both in cationic and anionic parts
(b) there is no zinc left in the solution
(a) LiCl and RbCl (b) MgCl2 and BeCl 2
(c) cationic part
(c) RbCl and BeCl 2 (d) RbCl and MgCl2 (d) anionic part.

RESPONSE 5. 6. 7. 8. 9.
GRID 10. 11. 12. 13. 14.
Space for Rough Work
Chemistry C-39

15. The compound A on heating gives a colourless gas and a (c) KHCO3 < NaHCO3 < CaCO3
residue that is dissolved in water to obtain B. Excess of CO 2 (d) CaCO3 < NaHCO3 < KHCO3
is bubbled through aqueous solution of B, C is formed which 21. For a good quality cement what should be the ratio of
is recovered in the solid form. Solid C on gentle heating following :
gives back A. The compound is I. Silica to alumina
(a) CaSO4.2H2O (b) CaCO3 II. CaO to the total of oxides of SiO2, Al2O3 and
(c) Na2CO3 (d) K2CO3 Fe2O3
16. In Castner-Kellner cell for production of sodium hydroxide: (a) I = 2.5 to 4
(a) Brine is electrolyzed with Pt electrodes II = Greater than 2
(b) Brine is electrolyzed using graphite electrodes (b) I = Nearly 4
II = Less than 2
(c) Molten sodium chloride is electrolysed
(c) I = 2.5
(d) Sodium amalgam is formed at mercury cathode
II = Closer to 2
17. A metal X on heating in nitrogen gas gives Y. Y on treatment
(d) I = 2.5 to 4
with H2O gives a colourless gas which when passed through
II = Closer to 2
CuSO4 solution gives a blue colour. Y is 22. Which one of the following does not react with water even
(a) Mg(NO3)2 (b) Mg3N2 under red hot condition?
(c) NH3 (d) MgO (a) Na (b) Be
18. The metals A and B form oxide but B also forms nitride when (c) Ca (d) K
both burn in air. The A and B are 23. Which of the following are found in biological fluids
(a) Cs, K (b) Mg, Ca Na+, Mg2+, Ca2+, K+, Sr2+, Li+ and Ba2+
(c) Li, Na (d) K, Mg (a) Mg2+, Ca2+, and Sr2+
19. The melting point of lithium (181°C) is just double the melting (b) Na2 +and K+
point of sodium (98°C) because (c) Na+, K+, Mg2+and Ca2+
(a) down the group, the hydration energy decreases (d) Sr+, Li and Ba2+
(b) down the group, the ionization energy decreases 24. Which of the following statements is not correct for alkali
(c) down the group the cohesive energy decreases metals?
(d) None of these (a) Alkali metals are the most electropositive metals.
20. Which of the following are arranged in increasing order of (b) Alkali metals exist in free state in nature.
solubilities? (c) These metals have the largest size in a particular
(a) CaCO3 < KHCO3 < NaHCO3 period of the periodic table.
(b) NaHCO3 < KHCO3 < CaCO3 (d) Both (b) and (c)

RESPONSE 15. 16. 17. 18. 19.


GRID 20. 21. 22. 23. 24.
Space for Rough Work
EBD_7504
C-40 NTA JEE Main

25. Which one of the following alkaline earth metal sulphates 28. Magnesium wire burns in the atmosphere of CO2 because
has its hydration enthalpy greater than its lattice enthalpy ? (a) Magnesium acts as an oxidising agent
(a) BaSO4 (b) SrSO4 (b) Magnesium has 2 electrons in the outermost orbit.
(c) CaSO4 (d) BeSO4 (c) Magnesium acts as a reducing agent and removes
26. The metallic sodium disolves in liquid ammonia to form a oxygen from CO2
deep blue coloured solution. The deep blue colour is due to (d) None of these
formation of: 29. The first ionisation potential of Na is 5.1 eV. The value of
(a) solvated electron, e(NH3 )-x electron gain enthalpy of Na+ will be
(b) solvated atomic sodium, Na(NH3)y (a) – 2.55 eV (b) – 5.1 eV
(c) (Na+ + Na–) (c) – 10.2 eV (d) + 2.55 eV
(d) NaNH2 + H2 - +
30. Stability of the species Li2, Li 2 and Li 2 increases in the
27. A firework gives out crimson coloured light. It contains a
salt of order of :
(a) Ca (b) Na (a) Li 2 < Li 2+ < Li 2- (b) Li -2 < Li +2 < Li 2
(c) Sr (d) Ba (c) Li 2 < Li 2- < Li 2+ (d) Li -2 < Li 2 < Li 2+

RESPONSE 25. 26. 27. 28. 29.


GRID 30.

CHEMISTRY CHAPTERWISE SPEED TEST-39


Total Questions 30 Total Marks 120
Attempted Correct
Incorrect Net Score
Cut-off Score 37 Qualifying Score 56
Success Gap = Net Score – Qualifying Score
Net Score = (Correct × 4) – (Incorrect × 1)
Space for Rough Work
CHEMISTRY Speed
The p-Block Elements (Group 13 & 14) TEST
No. of Questions
30
Maximum Marks
120
Time
1 Hour
40
Chapter-wise

GENERAL INSTRUCTIONS
• This test contains 30 MCQ's. For each question only one option is correct. Darken the correct circle/ bubble in the
Response Grid provided on each page.
• You have to evaluate your Response Grids yourself with the help of solutions provided at the end of this book.
• Each correct answer will get you 4 marks and 1 mark shall be deduced for each incorrect answer. No mark will be given/
deducted if no bubble is filled. Keep a timer in front of you and stop immediately at the end of 60 min.
• The sheet follows a particular syllabus. Do not attempt the sheet before you have completed your preparation for that
syllabus.
• After completing the sheet check your answers with the solution booklet and complete the Result Grid. Finally spend time
to analyse your performance and revise the areas which emerge out as weak in your evaluation.

1. Boric acid is polymeric due to (c) It sublimes at 180ºC


(a) its acidic nature (d) It is a strong Lewis base
(b) the presence of hydrogen bonds 3. The approximate percentage of silica in cement is:
(c) its monobasic nature (a) 5 – 10% (b) 15 – 20%
(d) its geometry (c) 20 – 25% (d) 25 – 30%
2. Which of the following statements about anhydrous 4. An aqueous solution of potash alum gives :
aluminium chloride is correct? (a) two types of ions (b) only one type of ion
(a) It exists as AlCl3 molecules (c) four types of ions (d) three types of ions
(b) It is not easily hydrolysed

RESPONSE GRID 1. 2. 3. 4.

Space for Rough Work


EBD_7504
C-42 NTA JEE Main

5. The I.E1 among the group 13 member follows as (a) large size of silicon
(a) B > Al < Ga < Tl (b) B > Al > Ga > Tl (b) more electropositive nature of silicon
(c) B > Ga > Al > Tl (d) B > Ga < Al < Tl (c) availability of d-orbitals in silicon
6. Non-oxide ceramics can be (d) Both (a) and (b)
(a) B4C (b) SiC
12. Soldiers of Napolean army while at alps during freezing
(c) Si3N4 (d) All of these winter suffered a serious problem as regards to the tin
7. Be2C and Al4C3 are called – buttons of their uniforms. White metallic tin buttons got
(a) ethanides (b) methanides converted to grey powder. This transformation is related to
(c) carbonides (d) acetylides (a) a change in the partial pressure of oxygen in the air
8. Anhydrous AlCl3 cannot be obtained from which of the (b) a change in the crystalline structure of tin
following reactions ? (c) an interaction with nitrogen of the air at very low
(a) Heating AlCl3.6H2O temperatures
(b) By passing dry HCl over hot aluminium powder (d) an interaction with water vapours contained in the
(c) By passing dry Cl2 over hot aluminium powder humid air
(d) By passing dry Cl2 over a hot mixture of alumina and coke 13. On adding ammonium hydroxide solution to Al2(SO4)3 (aq) :
9. Aluminium is extracted from alumina (Al2O3 ) by electrolysis (a) A precipitate is formed which does not dissolve in excess
of a molten mixture of : of ammonium hydroxide
(a) Al2O3 + HF + NaAlF4 (b) A precipitate is formed which dissolves in excess of
(b) Al2O3 + CaF2 + NaAlF4 ammonia solution
(c) Al2O3 + Na3AlF6 + CaF2 (c) No precipitate is formed
(d) Al2O3 + KF + Na3AlF6 (d) None of these
10. Alum helps in purifying water by
14. Gas A is bubbled through slaked lime when a white
(a) forming Si complex with clay partiles precipitate is formed. On prolonged bubbling the precipitate
(b) sulphate part which combines with the dirt and removes it is dissolved. On heating the resultant solution, the white
(c) coagulaing the mud particles precipitate appears with evolution of gas B. The gases A
(d) making mud water soluble. and B respectively are

11. Carbon and silicon belong to group 14. The maximum (a) CO and CO (b) CO 2 and CO
coordination number of carbon in commonly occurring
compounds is 4, whereas that of silicon is 6. This is due to (c) CO and CO 2 (d) CO 2 and CO 2

RESPONSE 5. 6. 7. 8. 9.
GRID 10. 11. 12. 13. 14.
Space for Rough Work
Chemistry C-43

15. The straight chain polymer is formed by H


H H
(a) hydrolysis of CH3SiCl 3 followed by condensation (b) B B – Diborane
polymerisation H H
H
(b) hydrolysis of (CH3)4Si by addition polymerisation Cl Cl Cl
(c) hydrolysis of (CH3)2SiCl2 followed by condensation (c) Al Cl – Aluminium chloride
Cl Al Al
polymerisation
(d) hydrolysis of (CH3)3SiCl followed by condensation
polymerisation (d) Cl B – Cl – Boron trichloride
16. It is because of inability of ns2 electrons of the valence shell Cl
to participate in bonding that:-
(a) Sn2+ is oxidising while Pb4+ is reducing 21. PbF4, PbCl4 exist but PbBr4 and PbI4 do not exist because of
(b) Sn2+ and Pb2+ are both oxidising and reducing (a) large size of Br– and I–
(c) Sn4+ is reducing while Pb4+ is oxidising (b) strong oxidising character of Pb4+
(d) Sn2+ is reducing while Pb4+ is oxidising (c) strong reducing character of Pb4+
17. A group 14 element is oxidised to form corresponding oxide (d) low electronegativity of Br – and I–.
which is gaseous in nature, when dissolved in water pH of 22. In silicon dioxide
the water decreases further addition of group 2 hydroxides (a) there are double bonds between silicon and oxygen
leads to precipitation. This oxide can be atoms
(a) GeO2 (b) CO (b) silicon atom is bonded to two oxygen atoms
(c) CO2 (d) SnO2 (c) each silicon atom is surrounded by two oxygen atoms
18. In borax bead test which compound is formed? and each oxygen atom is bonded to two silicon atoms
(a) Ortho-borate (b) Meta-borate (d) each silicon atom is surrounded by four oxygen atoms
(c) Double oxide (d) Tetra-borate and each oxygen atom is bonded to two silicon atoms.
19. Orthoboric acid when heated to red hot gives 23. Which one of the following is the correct statement?
(a) metaboric acid (b) pyroboric acid (a) Boric acid is a protonic acid
(c) boron and water (d) boric anhydride (b) Beryllium exhibits coordination number of six
20. Which of the following compounds is not matched correctly (c) Chlorides of both beryllium and aluminium have bridged
with its structure? structures in solid phase
H (d) B2H6.2NH3 is known as ‘inorganic benzene’
24. Graphite is a soft solid lubricant extremely difficult to melt.
B The reason for this anomalous behaviour is that graphite
H–N N–H (a) is an allotropic form of diamond
(a) – Borazine
H–B B–H (b) has molecules of variable molecular masses like polymers
N (c) has carbon atoms arranged in large plates of rings of
strongly bound carbon atoms with weak interplate bonds
H (d) is a non-crystalline substance

RESPONSE 15. 16. 17. 18. 19.


GRID 20. 21. 22. 23. 24.
Space for Rough Work
EBD_7504
C-44 NTA JEE Main

25. The gas evolved on heating CaF 2 and SiO 2 with 28. The catenation tendency of C, Si and Ge is in the order
concentrated H2SO4, on hydrolysis gives a white gelatinous Ge < Si < C. The bond energies (in kJ mol– l) of C – C, Si – Si
precipitate. The precipitate is: and Ge –Ge bonds are respectively;
(a) hydrofluosilicic acid (b) silica gel (a) 348, 297, 260 (b) 297, 348, 260
(c) silicic acid (d) calciumfluorosilicate (c) 348, 260, 297 (d) 260, 297, 348
26. Glass is a 29. Example of a three-dimensional silicate is:
(a) super-cooled liquid (b) gel (a) Zeolites (b) Ultramarines
(c) polymeric mixture (d) micro-crystalline solid (c) Feldspars (d) Beryls
27. Identify the incorrect statement : 30. CO2 and N2 are non-supporters of combustion. However for
(a) In (Si3O9)6–, tetrahedral SiO4 units share two oxygen putting out fires CO2 is preferred over N2 because CO2
atoms.
(a) does not burn
(b) Trialkylchlorosilane on hydrolysis gives R3SiOH.
(b) forms non-combustible products with burnin g
(c) SiCl4 undergoes hydrolysis to give H4SiO4. substances
(d) (Si3O9)6– has cyclic structure. (c) is denser than nitrogen
(d) is a more reactive gas

RESPONSE 25. 26. 27. 28. 29.


GRID 30.

CHEMISTRY CHAPTERWISE SPEED TEST-40


Total Questions 30 Total Marks 120
Attempted Correct
Incorrect Net Score
Cut-off Score 38 Qualifying Score 52
Success Gap = Net Score – Qualifying Score
Net Score = (Correct × 4) – (Incorrect × 1)

Space for Rough Work


CHEMISTRY Speed
Organic Chemistry : Some Basic TEST
Principles & Techniques
No. of Questions
30
Maximum Marks
120
Time
1 Hour
41
Chapter-wise

GENERAL INSTRUCTIONS
• This test contains 30 MCQ's. For each question only one option is correct. Darken the correct circle/ bubble in the
Response Grid provided on each page.
• You have to evaluate your Response Grids yourself with the help of solutions provided at the end of this book.
• Each correct answer will get you 4 marks and 1 mark shall be deduced for each incorrect answer. No mark will be given/
deducted if no bubble is filled. Keep a timer in front of you and stop immediately at the end of 60 min.
• The sheet follows a particular syllabus. Do not attempt the sheet before you have completed your preparation for that
syllabus.
• After completing the sheet check your answers with the solution booklet and complete the Result Grid. Finally spend time
to analyse your performance and revise the areas which emerge out as weak in your evaluation.

1. In crystallisation process impurities which impart colour (c) 4- methyl -3- oxo -1- pentanol
to the solution are removed by which of the following ? (d) Hexanol -1 - one -3
(a) Repeated crystallisation
3. The prussian blue colour obtained in the Lassaigne’s test
(b) Activated charcoal
for nitrogen is due to the formation of
(c) Bleaching agent
(a) Iron (II) hexacyanoferrate (III)
(d) Both (a) and (b)
(b) Iron (III) hexacyanoferrate (II)
2. The IUPAC name of
(c) Iron (III) hexacyanoferrate (III)
CH3 O (d) Iron (II) hexacyanoferrate (II)
| || 4. Which one of the following orders is correct regarding the
CH 3- CH - C - CH 2 - CH 2OH is
–I effect of the substituents ?
(a) 1- hydroxy -4- methyl -3- pentanone (a) —NR2 <—OR >—F (b) —NR2 > —OR >—F
(b) 2- methyl -5- hydroxy -3- pentanone (c) —NR2 <—OR <—F (d) —NR2 >—OR <—F

RESPONSE GRID 1. 2. 3. 4.

Space for Rough Work


EBD_7504
C-46 NTA JEE Main

5. Aniline is purified by 10. The stability of the compounds


(a) steam distillation (b) simple distillation
(c) vacuum distillation (d) extraction with a solvent
6. Vinylcarbinol is
(a) HO–CH2–CH=CH2 (b) CH3C(OH)=CH2 (i) (ii) (iii) (iv)
(c) CH3–CH=CH–OH (d) CH3–C(CH2OH)= CH2 (a) (iv) > (iii) > (i) > (ii) (b) (i) > (iii) > (ii) > (iv)
7. Which of the following is true regarding hyperconjugation, (c) (ii) > (iii) > (i) > (iv) (d) (iv) > (i) > (iii) > (ii)
also known as no-bond resonance? 11. The reason for the loss of optical activity of lactic acid when
(i) Like inductive effect it involves donation of electrons OH group is changed by H is that :
through s bonds (a) Chiral centre of the molecule is destroyed
(ii) Hyperconjugation involves overlapping of filled (b) Molecule acquires asymmetry
orbitals with the empty p orbital of the carbocation (c) Due to change in configuration
(iii) Like resonance, it involves displacement of p or lone (d) Structural change occurs
pair of electrons to the carbon bearing positive charge 12. In Duma's method for estimation of nitrogen, 0.25 g of an
(iv) It involves delocalisation of s and p- electrons organic compound gave 40 mL of nitrogen collected at 300
(a) (ii) and (iv) (b) (ii) K temperature and 725 mm pressure. If the aqueous tension
(c) (iii) (d) none
at 300 K is 25 mm, the percentage of nitrogen in the
8. The accepted IUPAC name of the camphor is
compound is :
O (a) 18.20 (b) 16.76
(c) 15.76 (d) 17.36
13. Sodium nitroprusside, when added to an alkaline solution
(a) 1, 7, 7 – trimethyl bicyclo [2. 2. 1] heptan – 2 – one of sulphide ions, produces purple colour ion due to the
(b) 1, 7, 7, – trimethyl bicyclo [2. 1. 2] heptan – 2 – one formation of
(c) 1, 2, 2 – trimethyl bicyclo [2. 2. 1] heptan – 6 – one (a) Na[Fe(H2O)5 NOS] (b) Na2[Fe(H2O)5 NOS]
(d) None of these (c) Na3[Fe(CN)5NOS] (d) Na4[Fe(CN)5NOS]
9. Fractional distillation is used when 14. The most satisfactory method to separate mixture of sugars
(a) there is a large difference in the boiling point of liquids is
(b) there is a small difference in the boiling points of liq- (a) fractional crystallisation
uids (b) sublimation
(c) boiling points of liquids are same (c) chromatography
(d) liquids form a constant boiling mixture (d) benedict reagent

RESPONSE 5. 6. 7. 8. 9.
GRID 10. 11. 12. 13. 14.
Space for Rough Work
Chemistry C-47

15. Me (a) The staggered conformation of ethane is less stable


Me H H than eclipsed conformation, because staggered
conformation has torsional strain
(b) The eclipsed conformation of ethane is more stable
H than staggered conformation, because eclipsed
Me conformation has no torsional strain
Hydrogenation of the above compound in the presence of (c) The eclipsed conformation of ethane is more stable
poisoned palladium catalyst gives than staggered conformation even though the eclipsed
(a) optically active compound conformation has torsional strain
(b) an optically inactive compound (d) The staggered conformation of ethane is more stable
(c) a racemic mixture than eclipsed conformation, because staggered
conformation has no torsional strain.
(d) a diastereomeric mixture
20. The number of asymmetric C-atom created and number of
16. Base strength of : possible stereoisomers when benzil (PhCOCOPh) is reduced
– , (B) with LiAlH4.
(A) H 3CCH 2
H 2C = CH and (C) H - C º C
(a) 2, 3 (b) 2, 2 (c) 2, 4 (d) 3, 2
is in the order of : 21. The most suitable method of separation of 1 : 1 mixture of
(a) (B) > (A) > (C) (b) (C) > (B) > (A) ortho and para-nitrophenols is :
(c) (A) > (C) > (B) (d) (A) > (B) > (C) (a) Chromatography
(b) Crystallisation
17. Which of the following biphenyls is optically active ?
(c) Steam distillation
Br Br (d) Sublimation
O2N
22. In Kjeldahl’s method nitrogen present is estimated as
(a) (b) (a) N2 (b) NH3
I I (c) NO2 (d) None of these
I 23. Which of the following sequence of T and F is correct for
I CH3
given statements. Here T stands for correct and F stands for
false statement
(c) (d) (i) The more the number of contributing structures, the
CH3 more is the resonance energy.
I
18. Which of the following compounds does not show (ii) The resonance structures have different positions of
nuclei but same number of unpaired electrons
Lassaigne’s test for nitrogen ?
(iii) The energy of actual structure of the molecule (the
(a) Urea (b) Hydrazine
resonance hybrid) is lower than that of any of the
(c) Phenylhydrazine (d) Azobenzene
canonical structures.
19. The correct statement regarding the comparison of (a) TTT (b) TFT
staggered and eclipsed conformation of ethane, is (c) FFF (d) TFF

RESPONSE 15. 16. 17. 18. 19.


GRID 20. 21. 22. 23.
Space for Rough Work
EBD_7504
C-48 NTA JEE Main

24. Which of the following compounds undergoes nucleophilic 27. A similarity between optical and geometrical isomerism is
substitution reaction most easily ? that
Cl (a) each forms equal number of isomers for a given
Cl compound
(a) (b) (b) if in a compound one is present then so is the other
(c) both are included in stereoisomerism
NO2
(d) they have no similarity.
CH3
28. Arrange the carbanions,
Cl
Cl (CH3 )3 C , C Cl3 , (CH3 ) 2 C H , C6 H5 CH 2
(c) (d) in order of their decreasing stability :
(a) (CH3 ) 2 C H > C Cl3 > C6 H5 C H2 > (CH3 )3 C
OCH3 (b) C Cl3 > C6 H5 CH 2 > (CH3 )2 CH > (CH3 )3 C
25. The stability of carbanions in the following :
(c) (CH3 )3 C > (CH3 )2 CH > C6 H 5 CH 2 > C Cl3
(I) RC = C (II) (d) C6 H5 CH 2 > C Cl3 > (CH3 )3 C > (CH3 ) 2 CH
29. Chlorine in vinyl chloride is less reactive because
(III) R 2C = CH (IV) R 3C - CH 2
(a) sp2 - hybridised carbon has more acidic character than
is in the order of : sp3 - hybridised carbon
(a) (I) > (II) > (III) > (IV) (b) (II) > (III) > (IV) > (I) (b) C - Cl bond develops partial double bond character
(c) (IV) > (II) > (III) > (I) (d) (I) > (III) > (II) > (IV) (c) of resonance
26. The percentage of sulphur in an organic compound whose (d) All are correct
0.32 g produces 0.233 g of BaSO4 [At. wt. Ba = 137, S = 32] 30. A compound contains 38.8% C, 16% H and 45.2% N. The
is formula of compound would be :
(a) 1.0 (b) 10.0 (a) CH 3 NH 2 (b) CH 3CN
(c) 23.5 (d) 32.1
(c) C 2 H 5CN (d) C H2(NH2)2

RESPONSE 24. 25. 26. 27. 28.


GRID 26. 30.

CHEMISTRY CHAPTERWISE SPEED TEST-41


Total Questions 30 Total Marks 120
Attempted Correct
Incorrect Net Score
Cut-off Score 36 SpaceQualifying
for Rough Work Score 53
Success Gap = Net Score – Qualifying Score
Net Score = (Correct × 4) – (Incorrect × 1)
Space for Rough Work
CHEMISTRY Speed
Hydrocarbons TEST
No. of Questions
30
Maximum Marks
120
Time
1 Hour
42
Chapter-wise

GENERAL INSTRUCTIONS
• This test contains 30 MCQ's. For each question only one option is correct. Darken the correct circle/ bubble in the
Response Grid provided on each page.
• You have to evaluate your Response Grids yourself with the help of solutions provided at the end of this book.
• Each correct answer will get you 4 marks and 1 mark shall be deduced for each incorrect answer. No mark will be given/
deducted if no bubble is filled. Keep a timer in front of you and stop immediately at the end of 60 min.
• The sheet follows a particular syllabus. Do not attempt the sheet before you have completed your preparation for that
syllabus.
• After completing the sheet check your answers with the solution booklet and complete the Result Grid. Finally spend time
to analyse your performance and revise the areas which emerge out as weak in your evaluation.

1. In the given reaction 3. One mole of a symmetrical alkene on ozonolysis gives two
X moles of an aldehyde having a molecular mass of 44 u. The
CH3CH 2CH = CHCH3 ¾¾® alkene is
CH3CH 2COOH +CH3COOH (a) propene
The X is (b) 1-butene
(a) C2 H5ONa (c) 2-butene
(b) Conc. HCl +Anhy.ZnCl2 (d) ethene
(c) Anh. AlCl3 4. Benzene on reaction with ozone forms __________.
(d) KMnO4/OH– (a) 2 molecules of aldehyde and 1 molecule of ketone
2. Ethyl hydrogen sulphate is obtained by reaction of H2SO4 (b) 2 molecules of ketone and 1 molecule of aldehyde
on
(c) triozonide
(a) Ethylene (b) Ethane
(c) Ethyl chloride (d) Ethanol (d) hexaozonide

RESPONSE GRID 1. 2. 3. 4.

Space for Rough Work


EBD_7504
C-50 NTA JEE Main

5. When neo-pentyl bromide is subjected to Wurtz reaction, 10. 1, 2-Dibromocyclohexane on dehydrobromination gives :
the product formed is
(a) 2,2,4,4-tetramethylhexane (a) (b)
(b) 2,2,4,4-tetramethylpentane
(c) 2,2,5,5-tetramethylhexane (c) (d) None of these
(d) 2,2,3,3-tetramethylhexane
6. Which one of the following reactions is expected to readily CrO Cl
give a hydrocarbon product in good yields ? 11. C6H5CH3 ¾¾¾¾ 2 2
®Z
In the given sequence Z is:
(a) Electrolytic
RCOOK ¾¾ ¾¾¾
® (a) benzaldehyde
oxidation
(b) toluic acid
Br
(b) RCOO - Ag + ¾ ¾
¾2®
(c) phenyl acetic acid
CH 3 CH 3 ¾¾
¾
Cl
2® (d) benzoic acid
(c)
hu 12. Acid catalyzed hydration of alkenes except ethene leads to
C H OH the formation of
(d) (CH 3 ) 3 CCl ¾ ¾2 ¾
5 ¾®
(a) mixture of secondary and tertiary alcohols
7. What will be the product in the following reaction?
(b) mixture of primary and secondary alcohols
CH2
NBS (c) secondary or tertiary alcohol
(d) primary alcohol
CH3 13. Which of the following organic compounds has same
Br
hybridization as its combustion product (CO2)?
CH3
(a) (b) (a) Ethane
Br
(b) Ethyne
(c) Ethene
CH2Br CH3
(d) Ethanol
(c) (d) 14. In reaction sequence
Br CH2OH
Hypochlorous R
CH 2 = CH 2 ¾¾ ¾¾¾ ¾® M ¾¾® |
8. A hydrocarbon contains 10.5 g carbon and 1 g hydrogen. Its acid CH2OH
0.36 g has 1 L volume at 1 atm and 127°C, hydrocarbon is :
molecule 'M' and reagent 'R' respectively are
(a) C6H7 (b) C7H8
(c) C5H6 (d) None of these (a) CH3CH2Cl and NaOH
9. The conversion of 2, 3-dibromobutane to 2-butene with Zn (b) CH3CH2OH and H2SO4
and alcohol is (c) CH2Cl . CH2OH and aqueous NaHCO3
(a) redox reaction (b) a-elimination (d) CH2 — CH2 and heat
(c) b-elimination (d) Both (a) and (b)
O

RESPONSE 5. 6. 7. 8. 9.
GRID 10. 11. 12. 13. 14.
Space for Rough Work
Chemistry C-51

15. Predict the correct intermediate and product in the following


reaction : (c) (d)
H O, H SO
H3C - C º CH ¾¾¾¾¾¾
2 2 2 ® Intermediate ® product S
HgSO4 (A) 20. A group which deactivates th e benzene ring
(B)
towards electrophilic substitution but which directs the
(a)A : H3C – C = CH B : H3C – C = CH2
| | incoming group principally to the o- and p-positions is
OH SO4 (a) –NH2 (b) –Cl
(c) –NO2 (d) –C2H5
(b) A : H3C – C = CH3 B : H3C – C º CH 21. In the reaction
||
O (1)NaNH 2 / liq.NH3 (1)NaNH 2 / liq.NH 3
H - C º CH X (2)CH CH Br Y
(c) A : H3C – C = CH2 B : H3C – C = CH3 (2)CH3CH 2 Br 3 2
| ||
OH O X and Y are
(a) X = 1-Butyne ; Y = 3-Hexyne
(d) A : H3C – C = CH2 B : H3C - C - CH3 (b) X = 2-Butyne ; Y = 3-Hexyne
| ||
SO4 O (c) X = 2-Butyne ; Y = 2-Hexyne
(d) X = 1-Butyne ; Y = 2-Hexyne
16. The treatment of CH3MgX with CH 3C º C - H produces 22. The main product of the reaction is :
(a) CH 3 - CH = CH 2 (b) CH 3C º C - CH3
Na, NH 3 ( l )
C C–R ¾¾¾¾¾ ® Product
H H R 'OH
| |
(c) CH 3 - C = C- CH3 (d) CH4 C C–R CH2CH2R
(a) (b)
17. A single compound of the structure :
CH3 CH3
H H H R
OHC C C
C H C O C C C C
H2 H2 (c) (d) H
R
is obtainable from ozonolysis of which of the following cyclic
compounds ? 23. In the following reaction, the major product is
H 3C H3C CH3 CH3
(a) (b) CH2
H3C HC 1 equivalent HBr
2

CH3
H3C CH3
(c) (d) CH3
CH3 CH3
CH3 (a) H2C (b) H C
18. Which of the following reagents convert propene to 3
Br
Br
1-propanol?
(a) H2O, H2SO4 (b) Aqueous KOH CH3 CH3
(c) MgSO4, NaBH4/H2O (d) B2H6, H2O2, OH– (c) (d) Br
19. Which of the following chemical system is non aromatic? H2C Br H 3C
24. Which of these will not react with acetylene?
(a) (b) (a) NaOH (b) ammonical AgNO3
(c) Na (d) HCl

RESPONSE 15. 16. 17. 18. 19.


GRID 20. 21. 22. 23. 24.
Space for Rough Work
EBD_7504
C-52 NTA JEE Main

CH2 - CH = CH 2 CH2 – CH = CH 2
29. on mercuration-demercuration
25. ® X, X is :
+ HCl ¾¾ produces the major product:
CH2 – CH – CH3
CH2 - CH - CH3 CH2 - CH 2 - CH 2 Cl (a)
OH
|
Cl
(a) (b) CH2 – CH2 – CH2 – OH
(b)

Cl
CH2 – CH – CH3
|
CH - CH 2 - CH3 CH2 - CH = CH2 (c)
OH OH
Cl
(c) (d) CH2 – COOH
(d)
26. The major product of the following reaction
30. For the synthesis of the following compound :
CH CH MgCl
O CH 3
H3C - C º CH ¾¾¾¾¾¾
3 2 ®
ether H 3O +
CH 2OH
CHCH 3
HO CH 2 CH 3 || CH 3
Which method is best ?
(a) (b) CH 3

C º CH C CH 3 ¾¾¾¾¾
NBS, CCl (heat )
4 ¾¾® NaOH
¾¾ ¾
¾®
C Method I : benzoyl peroxide
HO CH 2 HO CH 3
CH 3
(c) (d)
CH 3
27. The gas liberated by the electrolysis of Dipotassium
Method II : Br / FeBr Mg/ ethane
i) H2C = O
succinate solution is: ¾¾2¾ ¾¾
3 ®¾¾¾¾¾®¾(¾ ¾¾¾ ¾®
(a) Ethane (b) Ethyne (ii) H3O+
(c) Ethene (d) Propene
CH3
28. 1, 3-Butadiene when treated with Br 2 gives
(a) 1, 4-dibromo-2-butene (b) 1, 3-dibromo-2-butene (a) Method I (b) Method II
(c) 3, 4-dibromo-1-butene (d) 2, 3-dibromo-2-butene (c) Both (I) and (II) (d) Neither (I) nor (II)
are equally good

RESPONSE 25. 26. 27. 28. 26.


GRID 30.

CHEMISTRY CHAPTERWISE SPEED TEST-42


Total Questions 30 Total M arks 120
Space for Rough Work
Attempted Correct
Incorrect Net Score
Cut-off Score 35 Qualifying Score 50
Success Gap = Net Score – Qualifying Score
)
CHEMISTRY Speed
Environmental Chemistry TEST
No. of Questions
30
Maximum Marks
120
Time
1 Hour
43
Chapter-wise

GENERAL INSTRUCTIONS
• This test contains 30 MCQ's. For each question only one option is correct. Darken the correct circle/ bubble in the
Response Grid provided on each page.
• You have to evaluate your Response Grids yourself with the help of solutions provided at the end of this book.
• Each correct answer will get you 4 marks and 1 mark shall be deduced for each incorrect answer. No mark will be given/
deducted if no bubble is filled. Keep a timer in front of you and stop immediately at the end of 60 min.
• The sheet follows a particular syllabus. Do not attempt the sheet before you have completed your preparation for that
syllabus.
• After completing the sheet check your answers with the solution booklet and complete the Result Grid. Finally spend time
to analyse your performance and revise the areas which emerge out as weak in your evaluation.

1. High concentration of which of the following in atmosphere while far from earth these have wavelength independent
leads to stiffness of flower buds which eventually fall off behaviour.
from plants? 3. What is the concentration of dissolved oxygen in cold
(a) NO2 (b) SO2 water ?
(c) CFC (d) Smog
(a) 5 ppm (b) 10 ppm
2. Green house gases
(c) 200, 000 ppm (d) 100 ppm
(a) allow shorter wavelength to enter earth's atmosphere
4. Soil is polluted by
while doesn't allow longer wavelength to leave the
earth's atmosphere. I. pesticides
(b) allow longer wavelength to enter earth atmosphere II. synthetic fertilizers
while doesn't allow shorter wavelength to leave the III. green manure
surface Choose the correct option.
(c) don't have wavelength specific character. (a) I and III (b) I and II
(d) show wavelength specific behaviour near the earth (c) II and III (d) I, II and III

RESPONSE GRID 1. 2. 3. 4.

Space for Rough Work


EBD_7504
C-54 NTA JEE Main

5. Water is often treated with chlorine to (iv) Mists, smoke and dust
(a) remove hardness (v) Smog
(b) increase oxygen content (a) (i), (iv) and (v) (b) (iii) only
(c) kill germs (c) (ii) only (d) (ii) and (v)
(d) remove suspended particles 11. Formation of photochemical smog takes place in
6. The substance which is a primary pollutant? (a) winter during day time
(a) H2SO4 (b) CO (b) summer during morning time
(c) PAN (d) Aldehydes (c) summer during day time
7. Which of the following is/are the hazardous pollutant(s) (d) winter during morning time
present in automobile exhaust gases? 12. B.O.D. test or biochemical oxygen demand test is made for
(i) N2 (ii) CO measuring
(iii) CH4 (iv) Oxides of nitrogen (a) air pollution (b) water pollution
(a) (ii) and (iii) (b) (i) and (ii) (c) noise pollution (d) soil pollution
(c) (ii) and (iv) (d) (i) and (iii) 13. The smog is essentially caused by the presence of
8. “Reducing potentially hazardous waste through smarter (a) Oxides of sulphur and nitrogen
production”. (b) O2 and N2
This represents a great step forward for (c) O2 and O3
(a) green revolution (b) green chemistry (d) O3 and N2
(c) industrial revolution (d) green biotechnology 14. Which of the following statements about the depletion of
9. When rain is accompanied by a thunderstorm, the collected ozone layer is correct?
rain water will have a pH value (a) The problem of ozone depletion is less serious at poles
(a) slightly lower than that of rain water without because NO2 solidifies and is not available for
thunderstorm consuming ClO· radicals.
(b) slightly higher than that when the thunderstorm is not (b) The problem of ozone depletion is more serious at poles
there because ice crystals in the clouds over poles act as
(c) uninfluenced by occurrence of thunderstorm catalyst for photochemical reactions involving the
(d) which depends upon the amount of dust in air decomposition of ozone of Cl· and ClO· radicals.
10. The irritant red haze in the traffic and congested places (c) Freons, chlorofluorocarbons, are inert. Chemically, they
is due to presence of which of the following ? do not react with ozone in stratosphere.
(i) Oxides of sulphur (d) Oxides of nitrogen also do not react with ozone in
(ii) Oxides of nitrogen stratosphere.
(iii) Carbon dioxide

RESPONSE 5. 6. 7. 8. 9.
GRID 10. 11. 12. 13. 14.
Space for Rough Work
Chemistry C-55

15. Ozone layer of stratosphere requires protection from 19. Phosphate fertilizers when added to water leads to
indiscriminate use of (a) increased growth of decomposers
(a) balloons (b) reduced algal growth
(b) pesticides (c) increased algal growth
(c) atomic explosions (d) nutrient enrichment (eutrophication)
(d) aerosols and high flying jets 20. Thermal pollution affects mainly –
16. Identify the incorrect statement from the following : (a) vegetation (b) aquatic creature
(a) Ozone absorbs the intense ultraviolet radiation of the (c) rocks (d) air
sun. 21. Roasting of sulphides give the gas X as a by product. This is
(b) Depletion of ozone layer is because of its chemical colorless gas with choking smell of burnt sulphur and caused
reactions with chlorofluoro alkanes. great damage to respiratory organs as a result of acid rain.
(c) Ozone absorbs infrared radiation.
Its aqueous solution is acidic, acts as a reducing agent and
(d) Oxides of nitrogen in the atmosphere can cause the
its acid has never been isolated. The gas X is :
depletion of ozone layer.
(a) SO2 (b) CO2
17. Which of the following statements about polar stratosphere
(c) SO3 (d) H2S
clouds (PSCs) is not correct?
22. Sewage water is purified by
(a) PSCs do not react with chlorine nitrate and HCl (a) aquatic plants (b) micro-oganisms
(b) Type I clouds are formed at about –77ºC and contain (c) light (d) fishes
solid HNO3 . 3H2O 23. Carbon monoxide (CO) is harmful to man because
(c) Type II clouds are formed at about –85ºC and contain (a) it forms carbolic acid
some ice (b) it generates excess CO2
(d) A tight whirlpool of wind called Polar Vortex is formed (c) it is carcinogenic
which surrounds Antarctica (d) it competes with O2 for haemoglobin
18. Which of the following practices will not come under green 24. Which one of the following statements is not true?
chemistry? (a) Dissolved oxygen (DO) in cold water can reach a
(a) If possible, making use of soap made of vegetable oils concentration upto 10 ppm.
instead of using synthetic detergents. (b) Clean water would have a BOD value of 5 ppm.
(b) Using H2O2 for bleaching purpose instead of using (c) Fluoride deficiency in drinking water is harmful. Soluble
chlorine based bleaching agents fluoride is often used to bring its concentration upto 1
(c) Using bicycles for travelling small distances instead ppm.
of using petrol/ diesel based vehicles (d) When the pH of rain water is higher than 6.5, it is called
(d) Using plastic cans for neatly storing substances acid rain.

RESPONSE 15. 16. 17. 18. 19.


GRID 20. 21. 22. 23. 24.
Space for Rough Work
EBD_7504
C-56 NTA JEE Main

25. Frequent occurrence of water blooms in a lake indicates 28. Which one of the following statement is not true ?
(a) nutrient deficiency (a) pH of drinking water should be between 5.5 – 9.5.
(b) oxygen deficiency (b) Concentration of DO below 6 ppm is good for the
(c) excessive nutrient availability growth of fish.
(d) absence of herbivores in the lake (c) Clean water would have a BOD value of less than 5 ppm.
(d) Oxides of sulphur, nitrogen and carbon are the most
26. Which of the following is not a common component of
widespread air pollutant.
photochemical smog?
29. A dental disease characterised by mottling of teeth is due to
(a) Ozone presence of a certain chemical element in drinking water.
(b) Acrolein Which is that element?
(c) Peroxyacetyl nitrate (a) Boron (b) Chlorine
(d) Chlorofluorocarbons (c) Fluorine (d) Mercury
27. Which of the following is a sink for CO ? 30. Minamata disease is due to pollution of
(a) Microorganism present in the soil (a) aresenic into the atmosphere
(b) Oceans (b) organic waste into drinking water
(c) Plants (c) oil spill in water
(d) Haemoglobin (d) industrial waste mercury into fishing water

RESPONSE 25. 26. 27. 28. 29.


GRID 30.

CHEMISTRY CHAPTERWISE SPEED TEST-43


Total Questions 30 Total Marks 120
Attempted Correct
Incorrect Net Score
Cut-off Score 40 Qualifying Score 60
Success Gap = Net Score – Qualifying Score
Net Score = (Correct × 4) – (Incorrect × 1)
Space for Rough Work
CHEMISTRY Speed
The Solid State TEST
No. of Questions
30
Maximum Marks
120
Time
1 Hour
44
Chapter-wise

GENERAL INSTRUCTIONS
• This test contains 30 MCQ's. For each question only one option is correct. Darken the correct circle/ bubble in the
Response Grid provided on each page.
• You have to evaluate your Response Grids yourself with the help of solutions provided at the end of this book.
• Each correct answer will get you 4 marks and 1 mark shall be deduced for each incorrect answer. No mark will be given/
deducted if no bubble is filled. Keep a timer in front of you and stop immediately at the end of 60 min.
• The sheet follows a particular syllabus. Do not attempt the sheet before you have completed your preparation for that
syllabus.
• After completing the sheet check your answers with the solution booklet and complete the Result Grid. Finally spend time
to analyse your performance and revise the areas which emerge out as weak in your evaluation.

1. If Germanium crystallises in the same way as diamond, then (a) primitive (b) face centered
which of the following statement is not correct? (c) body centered (d) end centered
(a) Every atom in the structure is tetrahedrally bonded to 3. Doping of AgCl crystals with CdCl 2 results in
4 atoms. (a) Frenkel defect
(b) Unit cell consists of 8 Ge atoms and co-ordination (b) Schottky defect
number is 4. (c) Substitutional cation vacancy
(c) All the octahedral voids are occupied. (d) Formation of F - centres
(d) All the octahedral voids and 50% tetrahedral voids 4. A metal crystallizes in 2 cubic phases fcc and bcc whose
remain unoccupied. unit cell lengths are 3.5 Å and 3.0Å respectively. The ratio
2. The cubic unit cell of a metal (molar mass = 63.55g mol–1) of their densities is
has an edge length of 362 pm. Its density is 8.92g cm–3. (a) 0.72 (b) 2.04
The type of unit cell is (c) 1.46 (d) 3.12

RESPONSE GRID 1. 2. 3. 4.

Space for Rough Work


EBD_7504
C-58 NTA JEE Main

5. Coordination numbers of Zn 2+ and S2– in the crystal (a) MnO2 (b) TiO2
structure of wurtzite are
(c) VO2 (d) CrO2
(a) 4, 4 (b) 6, 6
11. Which of the following solids is not an electrical conductor?
(c) 8, 4 (d) 8, 8
(a) Mg (s) (b) TiO (s)
6. The coordination number X (........) of each ion in KBr is
(c) I2 (s) (d) H2O (s)
changed to Y (..........) by.............
12. The r+ / r– ratio of ZnS is 0.402. Pick out the false statements
(a) X = 6 to Y = 8 applying higher temperature
of the following
(b) X = 8 to Y = 6 applying high pressure
(a) ZnS is 4 : 4 coordination compound.
(c) X = 6 to Y = 8 applying high pressure
(b) ZnS does not crystallize in rock salt type lattice because
(d) None of these r+ / r– is too small to avoid overlapping of S2– ions.
7. The radius of Li+ ion is 60pm and that of F- is 136 pm. (c) Zn2+ ion is too small to fit precisely into the octahedral
Structure of LiF and coordination number is voids of S2– ions.
(a) Like NaCl, C.No. = 6 (b) Like CsCl, C.No. = 8 (d) Zn 2+ ion is too large to fit into the octahedral voids of
S2– ions.
(c) Anti fluoride, C.No. = 8 (d) None of these
13. The unit cell dimensions of a cubic lattice (edges a, b, c and
8. The number of carbon atoms per unit cell of diamond unit cell
the angles between them, a, b and g) are
is :
(a) a = b = c, a = b = g = 900
(a) 8 (b) 6
(b) a = b ¹ c, a = b = g = 900
(c) 1 (d) 4
(c) a = b = c,a = g = 900, b ¹ 900
9. The pyknometric density of sodium chloride crystal is
2.165 × 103 kg m–3 while its X-ray density is 2.178 × 103 kg m–3. (d) a ¹ b ¹ c,a = b = 900, g ¹ 900
The fraction of unoccupied sites in sodium chloride crystal 14. The pure crystalline substance on being heated gradually
is first forms a turbid liquid at constant temperature and still at
(a) 5.96 × 10–3 (b) 5.96 × 104 higher temperature turbidity completely disappears. The
behaviour is a characteristic of substance forming.
(c) 5.96 × 10–2 (d) 5.96 × 10–1
(a) Allotropic crystals (b) Liquid crystals
10. Which of the folliowing metal oxides is anti-ferromagnetic in
nature? (c) Isomeric crystals (d) Isomorphous crystals.

RESPONSE 5. 6. 7. 8. 9.
GRID 10. 11. 12. 13. 14.
Space for Rough Work
Chemistry C-59

15. To get a n-type semiconductor from silicon, it should be (a) 0.414 (b) 0.155
doped with a substance with valency ............ . (c) > 0.731 (d) 0.731
(a) 2 (b) 1 21. The incorrect statement for the sulphur atom of ZnS is
(c) 3 (d) 5
(a) As S 2 - is larger than Zn 2 + only 4 rather than 6 or 8
16. The second order Bragg diffraction of X-rays with l = 1.0Å
from a set of parallel planes in a metal occurs at an angle of S 2 - can be packed around Zn 2 +
60°. The distance between the scattering planes in the crystal (b) Its structure is similar to diamond except that alternate
is atoms are Zn and S
(a) 0.575Å (b) 1.00Å 2-
(c) As S is larger than Zn 2 + only 6 rather than 8 or 4
(c) 2.00Å (d) 1.15Å
17. Which of the following oxides shows electrical properties sulphide ions can be placed around Zn 2 + ions
like metals ? (d) ZnS is a covalent compound
(a) SiO2 (b) MgO 22. AB; crystallizes in a body centred cubic lattice with edge
(c) SO2 (s) (d) CrO2 length ‘ a’ equal to 387 pm. The distance between two
18. Al (at. wt 27) crystallizes in the cubic system with a cell edge oppositely charged ions in the lattice is :
(a) 335 pm (b) 250 pm (c) 200 pm (d) 300 pm
of 4.05 Å . Its density is 2.7 g per cm3 . Determine the unit
cell type calculate the radius of the Al atom 23. How many unit cells are present in a cube-shaped ideal
(a) fcc, 2.432 Å (b) bcc, 2.432 Å crystal of NaCl of mass 1.00 g ?
(c) bcc, 1.432 Å (d) fcc, 1.432 Å [Atomic masses : Na = 23, Cl = 35.5]
19. The ratio of inner planar distancess of three types of planes (a) 5.14 × 1021 unit cells (b) 1.28 × 1021 unit cells
(d100 , d110 , d111 ) for simple cubic lattice are (c) 1.71 × 1021 unit cells (d) 2.57 × 1021 unit cells
24. The interionic distance for cesium chloride crystal will be
1 1
(a) 1 : 1 : 1 (b) : :1 a
2 3 3 2 (a) a (b)
2
(c) 3 : 2 :1 (d) 1 : 2 : 3 2a
3a
+ (c) (d)
r 2 3
20. For a cubic geometry the limiting is :
r–

RESPONSE 15. 16. 17. 18. 19.


GRID 20. 21. 22. 23. 24.
Space for Rough Work
EBD_7504
C-60 NTA JEE Main

25. For fcc if AB is just like the rock salt like structure then, A+ (a) Na+ ions are packed in octahedral holes between the
and B– are located at – planes of close-packed Cl– ions.
(a) A+ - Tetrahedral voids ; B– - Corner (b) Zn 2+ ions are packed in tetrahedral holes
(b) A+ - Corner and faces ; B– - Octahedral voids (c) Cs + ions are packed in a simple cubic array of
(c) A+ - Octahedral voids ; B– - Corner and faces Cl– ions
(d) All of these
(d) A+ - Corner and faces ; B– - Octahedral and tetrahedral
voids 29. What type of crystal defect is indicated in the diagram below?
26. Which of the following exists as covalent crystals in the solid Na + Cl - Na + Cl - Na + Cl -
state ? Cl– c Cl– Na+ c Na+
(a) Iodine (b) Silicon Na+Cl– c Cl– Na+ Cl–
(c) Sulphur (d) Phosphorus Cl– Na+ Cl–Na+ c Na+
27. The packing fraction for a body-centred cubic is (a) Interstitial defect
(a) 0.42 (b) 0.53 (b) Schottky defect
(c) 0.68 (d) 0.82 (c) Frenkel defect
28. Consider the radii 0.095 nm (Na+), 0.181 nm (Cl– ), (d) Frenkel & Schottky defects
0.074 nm (Zn 2+), 0.184 nm (S 2– ), 0.068 nm (Ti 4+), 30. What is the coordination number of sodium in Na2O?
0.140 nm (O2–), 0.169 nm (Cs+). Choose the correct option (a) 6 (b) 4
from among the following. (Use radius ratio rules) (c) 8 (d) 2

RESPONSE 25. 26. 27. 28. 29.


GRID 30.

CHEMISTRY CHAPTERWISE SPEED TEST-44


Total Questions 30 Total Marks 120
Attempted Correct
Incorrect Net Score
Cut-off Score 35 Qualifying Score 50
Success Gap = Net Score – Qualifying Score
Net Score = (Correct × 4) – (Incorrect × 1)
Space for Rough Work
CHEMISTRY Speed
Solutions TEST
No. of Questions
30
Maximum Marks
120
Time
1 Hour
45
Chapter-wise

GENERAL INSTRUCTIONS
• This test contains 30 MCQ's. For each question only one option is correct. Darken the correct circle/ bubble in the
Response Grid provided on each page.
• You have to evaluate your Response Grids yourself with the help of solutions provided at the end of this book.
• Each correct answer will get you 4 marks and 1 mark shall be deduced for each incorrect answer. No mark will be given/
deducted if no bubble is filled. Keep a timer in front of you and stop immediately at the end of 60 min.
• The sheet follows a particular syllabus. Do not attempt the sheet before you have completed your preparation for that
syllabus.
• After completing the sheet check your answers with the solution booklet and complete the Result Grid. Finally spend time
to analyse your performance and revise the areas which emerge out as weak in your evaluation.

1. The solubility of a solid in a liquid is significantly affected (d) DHsol < 0; solubility ­ ; temperature ­
by temperature changes.
2. The vapour pressure of a solution of the liquids A
ˆˆ† Solution.
Solute + Solvent ‡ˆˆ (p° = 80 mm Hg and x A = 0.4) and B (p°= 120 mm Hg and
The system being in a dynamic equilibrium must follow x B = 0.6) is found to be 100 mm Hg. It shows that the
Le-chatelier’s principle. Considering the Le-chatelier’s solution exhibits
principle which of the following is correct? (a) positive deviation from ideal behaviour
(a) DHsol > 0; solubility ­ ; temperature ¯ (b) negative deviation from ideal behaviour
(b) DHsol < 0; solubility ¯ ; temperature ­ (c) ideal behaviour

(c) DHsol > 0; solubility ¯ ; temperature ­ (d) positive deviation for lower conc. and negative for higher
conc.

RESPONSE GRID 1. 2.

Space for Rough Work


EBD_7504
C-62 NTA JEE Main

1 1 (a) 0.58 (b) 0.73


3. Plot of Vs ( x A mole fraction of A in liquid state (c) 0.83 (d) 0.92
xA yA
and yA in vapour state) is linear whose slope and intercept 8. A solution containing components A and B follows Raoult's
respectively are given law when
po - po (a) A – B attraction force is greater than A – A and B – B
(a) p oB / p oA , B o A (b) A – B attraction force is less than A – A and B – B
pB (c) A – B attraction force remains same as A–A and B –B
p - po
o
(d) Volume of solution is different from sum of volume of
(b) p oA - p oB , A o B solute and solvent
pB
9. Two 1-litre flask A and B are connected to each other by a
p - po
o
(c) p oB - p oA , B o A valve which is closed. Flask A has benzene in equilibrium
pB with its vapours at 30°C. The flask B, is evacuated, and the
p - po
o valve is opened. Which of the following is true. If temperature
(d) p oB / p oA , A o B is kept constant.’
pB
(a) Some of the benzene molecules would move to flask B
4. Coolent used in car radiator is aqueous solution of ethylene from flask A.
glycol. In order to prevent the solution from freezing at (b) Vapour pressure will be half the initial value.
– 0.3 °C. How much ethylene glycol must be added to 5 kg of (c) The vapour pressure remains unchanged
water ? (Kf = 1.86 K kg mol –1) (d) Some more of the liquid benzene in flask A would
(a) 50 kg (b) 50 g evaporate.
(c) 45 g (d) 40 g 10. For a solution of two liquids A and B it was proved that
5. A solution contains non-volatile solute of molecular mass
M2. Which of the following can be used to calculate the PS = x A (p °A - p °B ) + p °B . The resulting solution will be
molecular mass of solute in terms of osmotic pressure ? (a) Non -ideal (b) ideal
æm ö æ m ö RT (c) semi-ideal (d) None of these
(a) M 2 = ç 2 ÷VRT (b) M 2 = ç 2 ÷ 11. A 0.0020 m aqueous solution of an ionic compound
è p ø è V ø p
Co(NH3)5(NO2)Cl freezes at – 0.00732 °C. Number of moles
æ m2 ö æ m2 ö p of ions which 1 mol of ionic compound produces on being
(c) M 2 = ç ÷pRT (d) M 2 = ç ÷
è V ø è V ø RT dissolved in water will be (Kf = – 1.86°C/m)
6. Henry’s law constant of oxygen is 1.4 × 10–3 mol. lit–1. atm–1 (a) 3 (b) 4
at 298 K. How much of oxygen is dissolved in 100 ml at 298 (c) 1 (d) 2
K when the partial pressure of oxygen is 0.5 atm? 12. A solution of urea (mol. mass 56 g mol-1) boils at 100.18°C
(a) 1.4 g (b) 3.2 g at the atmospheric pressure. If K f and Kb for water are
(c) 22.4 mg (d) 2.24 mg 1.86 and 0.512 K kg mol-1 respectively, the above solution
7. What is the degree of dissociation of sodium chloride, if the will freeze at
molar mass determined by a cryoscopic method was found to (a) 0.654°C (b) - 0.654°C
be 31.80 g mol-1 [Atomic mass Na = 23 g mol –1 Cl = 35.5 g (c) 6.54°C (d) - 6.54°C
mol–1] ?

RESPONSE 3. 4. 5. 6. 7.
GRID 8. 9. 10. 11. 12.
Space for Rough Work
Chemistry C-63

13. In mixture A and B components show -ve deviation as


(a) D Vmix > 0 p2 III p2°
(b) D Hmix < 0 = p1 +

Vapour pressure
p Total II
(c) A – B interaction is weaker than A – A and B – B p1° p2
interaction
(d) A – B interaction is stronger than A – A and B – B p
1
interaction. I
14. All form ideal solution except
(a) C6H6 and C6H5 CH3 x1 = 1 Mole fraction x1 = 0
x2 = 0 x2 x2 = 1
(b) C2H6 and C2H5I
(c) C6H5Cl and C6H5Br In this figure, lines I and II pass through the point for which.
(d) C2H5 I and C2H5 OH. (a) x1 ¹ 1; x2 = 1 (b) x1 = x2 ¹ 1
15. A binary liquid solution is prepared by mixing n-heptane and (c) x1 = 1; x2 ¹ 1 (d) x1 = x2 = 1
ethanol. Which one of the following statements is correct 19. Which of the following modes of expressing concentration is
regarding the behaviour of the solution? independent of temperature ?
(a) The solution is non-ideal, showing – ve deviation from (a) Molarity (b) Molality
Raoult’s Law. (c) Formality (d) Normality
(b) The solution is non-ideal, showing + ve deviation from 20. How many grams of concentrated nitric acid solution should
Raoult’s Law. be used to prepare 250 mL of 2.0M HNO3 ? The concentrated
(c) n-heptane shows + ve deviation while ethanol shows acid is 70% HNO3
– ve deviation from Raoult’s Law. (a) 90.0 g conc. HNO3 (b) 70.0 g conc. HNO3
(d) The solution formed is an ideal solution. (c) 54.0 g conc. HNO3 (d) 45.0 g conc. HNO3
16. We have three aqueous solutions of NaCl labelled as 'A', 'B' 21. Which among the following will show maximum osmotic
and 'C' with concentrations 0.1M, 0.01M and 0.001M, pressure?
respectively. The value of van't Hoff factor for these (a) 1 M NaCl (b) 1 M MgCl2
solutions will be in the order ________. (c) 1 M (NH4)3PO4 (d) 1 M Na2SO4
(a) iA < iB < iC (b) iA > iB > iC 22. The boiling point of 0.2 mol kg–1 solution of X in water is
(c) iA = iB = iC (d) iA < iB > iC greater than equimolal solution of Y in water. Which one of
17. During depression of freezing point in a solution the the following statements is true in this case ?
following are in equilibrium (a) Molecular mass of X is greater than the molecular mass
(a) liquid solvent, solid solvent of Y.
(b) liquid solvent, solid solute (b) Molecular mass of X is less than the molecular mass of
(c) liquid solute, solid solute Y.
(d) liquid solute, solid solvent (c) Y is undergoing dissociation in water while X
18. A plot of p1 or p2 vs the mole fractions x1 and x2 is given undergoes no change.
as. (d) X is undergoing dissociation in water.

RESPONSE 13. 14. 15. 16. 17.


GRID 18. 19. 20. 21. 22.
Space for Rough Work
EBD_7504
C-64 NTA JEE Main

23. Which of the following 0.10 m aqueous solutions will have dissociates completely, molar concentration of sodium ions,
the lowest freezing point ? Na + and carbonate ions, CO32– are respectively (Molar
(a) Al2(SO4)3 (b) C6H12O6 mass of Na2CO3 = 106 g mol–1)
(c) KCl (d) C12 H22 O11 (a) 0.955 M and 1.910 M
24. If sodium sulphate is considered to be completely (b) 1.910 M and 0.955 M
dissociated into cations and anions in aqueous solution, (c) 1.90 M and 1.910 M
the change in freezing point of water (DTf), when 0.01 mol (d) 0.477 M and 0.477 M
of sodium sulphate is dissolved in 1 kg of water, is (Kf = 1.86 28. Azeotropic mixture of HCl and H2O has
K kg mol–1) (a) 48% HCl (b) 22.2% HCl
(a) 0.372 K (b) 0.0558 K (c) 36% HCl (d) 20.2% HCl
29. Freezing point of an aqueous solution is – 0.186°C. If
(c) 0.0744 K (d) 0.0186 K
the values of Kb and Kf of water are respectively 0.52 K kg
25. Which one of the following salts will have the same value of mol–1 and 1.86 K kg mol–1, then the elevation of boiling
van’t Hoff factor (i) as that of K4[Fe(CN)6]. point of the solution in K is
(a) Al2(SO4)3 (b) NaCl (a) 0.52 (b) 1.04
(c) Al(NO3)3 (d) Na2SO4. (c) 1.34 (d) 0.052
26. Mole fraction of the solute in a 1.00 molal aqueous solution 30. Which of the following statements, regarding the mole
is : fraction (x) of a component in solution, is incorrect?
(a) 0.1770 (b) 0.0177 (a) 0 < x < 1
(b) x < 1
(c) 0.0344 (d) 1.7700
(c) x is always non-negative
27. 25.3 g of sodium carbonate, Na 2 CO3 is dissolved in enough (d) –2 < x < 2
water to make 250 mL of solution. If sodium carbonate

RESPONSE 23. 24. 25. 26. 27.


GRID 28. 29. 30.

CHEMISTRY CHAPTERWISE SPEED TEST-45


Total Questions 30 Total Marks 120
Attempted Correct
Incorrect Net Score
Cut-off Score 35 Qualifying Score 50
Success Gap = Net Score – Qualifying Score
Space for Rough Work
Net Score = (Correct × 4) – (Incorrect × 1)
CHEMISTRY Speed
Electrochemistry TEST
No. of Questions
30
Maximum Marks
120
Time
1 Hour
46
Chapter-wise

GENERAL INSTRUCTIONS
• This test contains 30 MCQ's. For each question only one option is correct. Darken the correct circle/ bubble in the
Response Grid provided on each page.
• You have to evaluate your Response Grids yourself with the help of solutions provided at the end of this book.
• Each correct answer will get you 4 marks and 1 mark shall be deduced for each incorrect answer. No mark will be given/
deducted if no bubble is filled. Keep a timer in front of you and stop immediately at the end of 60 min.
• The sheet follows a particular syllabus. Do not attempt the sheet before you have completed your preparation for that
syllabus.
• After completing the sheet check your answers with the solution booklet and complete the Result Grid. Finally spend time
to analyse your performance and revise the areas which emerge out as weak in your evaluation.

1. Which of the following expressions correctly represents (a) 1.25 × 10–6 (b) 6.25 × 10–4
the equivalent conductance at infinite dilution of Al2(SO4)3, (c) 1.25 × 10 –4 (d) 1.25 × 10–5
3. Aqueous solution of which of the following compounds is
Given that L °Al3+ and L°SO2- are the equivalent the best conductor of electric current ?
4
conductances at infinite dilution of the respective ions? (a) Acetic acid, C2H4O2
(b) Hydrochloric acid, HCl
1 ° 1 2L° + 3L°
(a) L 3+ + L° 2- (b) Al3+ SO24-
(c) Ammonia, NH3
3 Al 2 SO4 (d) Fructose, C6H12O6
L° + L° æL° + L ° 2- ö ´ 6 4. The standard EMF of Daniell cell is 1.10 volt. The maximum
(c) Al3+ SO42 - (d) è Al3+ SO4 ø electrical work obtained from the Daniell cell is
M (a) 212.3 kJ
2. The equivalent conductance of solution of a weak (b) 175.4 kJ
32
(c) 106.15 kJ
monobasic acid is 8.0 mho cm2 and at infinite dilution is
(d) 53.07 kJ
400 mho cm2. The dissociation constant of this acid is:

RESPONSE GRID 1. 2. 3. 4.

Space for Rough Work


EBD_7504
C-66 NTA JEE Main

9. If the E°cell for a given reaction has a negative value, then


5. Which of the following reaction occurs at the cathode during which of the following gives the correct relationships for the
the charging of lead storage battery? values of DG° and Keq ?
(a) Pb 2 + + 2e - ¾ (a) DG° > 0 ; Keq > 1 (b) DG° < 0 ; Keq > 1
¾® Pb
(c) DG° < 0 ; Keq < 1 (d) DG° > 0 ; Keq < 1
(b) Pb 2+ + SO 24- ¾
¾® PbSO 4 10. Standard electrode potentials are : Fe +2 /Fe
[ Eº = –0.44]; Fe+3/Fe+2 Eº = + 0.77 ; If Fe+2, Fe+3 and Fe
¾® Pb 2 + + 2e -
(c) Pb ¾
blocks are kept together, then
¾® 2PbO 2 + 4H + + SO 24- + 2e -
(d) PbSO 4 + 2H 2 O ¾ (a) Fe+3 increases
6. Molar ionic conductivities of a two-bivalent electrolytes (b) Fe+3 decreases
Fe +2
x 2 + and y2 - are 57 and 73 respectively. The molar (c)
Fe +3
remains unchanged
conductivity of the solution formed by them will be (d) Fe+2 decreases
(a) 130 S cm2 mol–1 11. An electrolytic cell contains a solution of Ag2SO4 and has
(b) 65 S cm2 mol–1 platinum electrodes. A current is passed until 1.6 gm of O2
(c) 260 S cm2 mol–1 has been liberated at anode. The amount of silver deposited
(d) 187 S cm2 mol–1 at cathode would be
7. Kohlrausch’s law states that at : (a) 107.88 gm (b) 1.6 gm
(a) finite dilution, each ion makes definite contribution to (c) 0.8 gm (d) 21.60 gm
equivalent conductance of an electrolyte, whatever be 12. If f denotes reduction potential, then which is true?
the nature of the other ion of the electrolyte.
(a) E°cell = f right – f left (b) E°cell = f left + f right
(b) infinite dilution each ion makes definite contribution
to equivalent conductance of an electrolyte depending (c) E°cell = f left – f right (d) E°cell = – ( f left + f right).
on the nature of the other ion of the electrolyte. 13. In a fuel cell methanol is used as fuel and oxygen gas is used
(c) infinite dilution, each ion makes definite contribution as an oxidizer. The reaction is
to conductance of an electrolyte whatever be the nature CH 3OH(l ) + 3/2O 2 (g) ¾¾ ® CO2 (g) + 2H 2O(l )
of the other ion of the electrolyte.
At 298 K standard Gibb’s energies of formation for CH3OH(l),
(d) infinite dilution, each ion makes definite contriubtion
H2O(l) and and CO2 (g) are –166.2 –237.2 and –394.4 kJ mol–1
to equivalent conductance of an electrolyte, whatever
respectively. If standard enthalpy of combustion of methonal
be the nature of the other ion of the electrolyte.
is – 726 kJ mol–1, efficiency of the fuel cell will be:
8. Standard free energies of formation (in kJ/mol) at 298 K are
(a) 87% (b) 90%
– 237.2, – 394.4 and – 8.2 for H2O(l), CO2(g) and pentane (g),
(c) 97% (d) 80%
respectively. The value E°cell for the pentane-oxygen fuel
14. For the cell reaction,
cell is :
Cu2+ (C1, aq) + Zn(s) = Zn 2+ (C2, aq) + Cu(s) of an
(a) 1.968 V
electrochemical cell, the change in free energy, DG, at a given
(b) 2.0968 V
temperature is a function of
(c) 1.0968 V
(a) ln (C1) (b) ln (C2/C1)
(d) 0.0968 V
(c) ln (C2) (d) ln (C1 + C2)

RESPONSE 5. 6. 7. 8. 9.
GRID 10. 11. 12. 13. 14.

Space for Rough Work


Chemistry C-67

15. When electric current is passed through acidified water, (c) MnO-4 can be used in aqueous H2SO4
112 ml of hydrogen gas at STP collected at the cathode in
965 seconds. The current passed in amperes is (d) Cr2O 27 - can be used in aqueous H2SO4
(a) 1.0 (b) 0.5 (c) 0.1 (d) 2.0 20. A hypothetical electrochemical cell is shown below
16. The electrode potential E 2+ of a zinc electrode at
(Zn Zn) A|A + (xM)||B+ (yM)|B
25°C with an aqueous solution of 0.1 M ZnSO4 is The emf measured is +0.20 V. The cell reaction is
2.303RT (a) A+ + e–® A; B++ e– ® B
[ E° 2+ = – 0.76 V. Assume = 0.06 at 298 K]. (b) The cell reaction cannot be predicted
(Zn Zn) F
(c) A + B+ ® A+ + B
(a) + 0.73 (b) – 0.79
(d) A+ + B ® A + B+
(c) – 0.82 (d) – 0.70
17. A gas X at 1 atm is bubbled through a solution containing a 21. Conductance of 0.1 M KCl (conductivity = X Ohm-1cm -1 )
mixture of 1 M Y– and M Z– at 25°C. If the reduction potential
of Z > Y > X, then, filled in a conductivity cell is Y Ohm-1 . If the conductance
(a) Y will oxidize X and not Z
(b) Y will oxidize Z and not X of 0.1 M NaOH filled in the same cell is Z Ohm-1 , the molar
(c) Y will oxidize both X and Z conductance of NaOH will be
(d) Y will reduce both X and Z XZ XZ
(a) 10 3 XZ (b) 10 4 XZ (c) 10 (d) 0.1
18. For the electrochemical cell, M | M + || X - | X, Y Y Y Y
o – 22. On the basis of the following E° values, the strongest
E o M + / M = 0.44V and E (X/X ) = 0.33V.. oxidizing agent is :
From this data one can deduce that Fe(CN)6]4– ®[Fe(CN)6]3– + e– ; E° = – 0.35 V
(a) M+X®M++X– is the spontaneous reaction Fe2+ ® Fe3+ + e–; E° = – 0.77 V
(b) M++X–®M+X is the spontaneous reaction (a) [Fe(CN)6]4– (b) Fe2+
(c) Ecell = 0.77 V (c) Fe3+ (d) [Fe(CN)6]3–
(d) Ecell = –0.77 V 23. The mathematical expression for law of independent
19. Standard electrode potential data are useful for understanding migration of ions and Ostwald’s dilution law are given by
the suitability of an oxidant in a redox titration. Some half cell
reactions and their standard potentials are given below: (a) L = L om - BC½ (b) L o = F(U + + U - )
MnO -4 (aq.) + 8H + (aq.) + 5e - ® Mn 2 + (aq.) + 4H 2 O(l) Lo 1 Lm c
E° = 1.51 V (c) Lom = n +l + + n -l - (d) L = o + o 2
m L m K a (L m )
24. The variation of equivalent conductance of strong
Cr2 O 27 - (aq.) + 14H + (aq.) + 6e - ® 2Cr 3+ (aq.) + 7 H 2 O(l) electrolyte with (concentration)½ is represented by
E° = 1.38 V
Fe3+ (aq.) + e - ® Fe 2 + (aq.) E° = 0.77 V
Cl 2 (g) + 2e- ® 2Cl - (aq.) E° = 1.40 V (a) l (b) l
Identify the only incorrect statement regarding the c c
quantitative estimation of aqueous Fe(NO3)2
(a) MnO-4 can be used in aqueous HCl
(b) Cr2O 27 - can be used in aqueous HCl (c) l (d) l
c c

RESPONSE 15. 16. 17. 18. 19.


GRID 20. 21. 22. 23. 24.

Space for Rough Work


EBD_7504
C-68 NTA JEE Main

25. A device that converts energy of combustion of fuels like (c) generate heat
hydrogen and methane, directly into electrical energy is (d) remove adsorbed oxygen from electron surfaces
known as : 29. Consider the following relations for emf of a electrochemical
(a) Electrolytic cell (b) Dynamo cell:
(c) Ni-Cd cell (d) Fuel Cell (i) emf of cell = (Oxidation potential of anode) – (Reduction
26. In acidic medium MnO2 is an oxidant as potential of cathode)
(ii) emf of cell = (Oxidation potential of anode) + (Reduction
MnO 2 (s) + 4H + + 2e - ¾ ¾® Mn 2+ + 2H 2 O potential of cathode)
If the pH of solution is decreased by one unit, the electrode (iii) emf of cell = (Reduction potential of anode) + (Reduction
potential of cathode)
potential of the half cell Pt : MnO 2, Mn 2+ will change by (iv) emf of cell = (Oxidation potential of anode) – (Oxidation
(a) 0.236 V (b) – 0.236 V potential of cathode)
(c) – 0.118 V (d) 0.118 V Which of the above relations are correct?
27. Consider the following cell reaction: (a) (ii) and (iv)
+ 2+ (b) (iii) and (i)
2Fe( s) + O2 ( g) + 4H (aq) ® 2Fe (aq) + 2H2O(l);E° = 1.67V
2+ –3
At [Fe ] = 10 M, p(O2) = 0.1 atm and pH = 3, the cell (c) (i) and (ii)
potential at 25ºC is (d) (iii) and (iv)
(a) 1.47 V (b) 1.77 V 30. A hydrogen gas electrode is made by dipping platinum wire
(c) 1.87 V (d) 1.57 V in a solution of HCl of pH = 10 and by passing hydrogen
28. In a hydrogen-oxygen fuel cell, combustion of hydrogen gas around the platinum wire at one atm pressure. The
occurs to oxidation potential of electrode would be ?
(a) produce high purity water (a) 0.59 V (b) 0.118 V
(b) create potential difference between two electrodes (c) 1.18 V (d) 0.059 V

RESPONSE 25. 26. 27. 28. 29.


GRID 30.

CHEMISTRY CHAPTERWISE SPEED TEST-46


Total Questions 30 Total Marks 120
Attempted Correct
Incorrect Net Score
Cut-off Score 37 Qualifying Score 53
Success Gap = Net Score – Qualifying Score
Net Score = (Correct × 4) – (Incorrect × 1)
Space for Rough Work
CHEMISTRY Speed
Chemical Kinetics TEST
No. of Questions
30
Maximum Marks
120
Time
1 Hour
47
Chapter-wise

GENERAL INSTRUCTIONS
• This test contains 30 MCQ's. For each question only one option is correct. Darken the correct circle/ bubble in the
Response Grid provided on each page.
• You have to evaluate your Response Grids yourself with the help of solutions provided at the end of this book.
• Each correct answer will get you 4 marks and 1 mark shall be deduced for each incorrect answer. No mark will be given/
deducted if no bubble is filled. Keep a timer in front of you and stop immediately at the end of 60 min.
• The sheet follows a particular syllabus. Do not attempt the sheet before you have completed your preparation for that
syllabus.
• After completing the sheet check your answers with the solution booklet and complete the Result Grid. Finally spend time
to analyse your performance and revise the areas which emerge out as weak in your evaluation.

1. The temperature dependence of rate constant (k) of a 3. The half life period for catalytic decomposition of AB3 at 50
chemical reaction is written in terms of Arrhenius equation,
mm Hg is 4 hrs and at 100 mm Hg it is 2 hrs. The order of
k = A.e - Ea /RT Activation energy (Ea) of the reaction can be reaction is
calculated by plotting
(a) 1 (b) 3
1 1
(a) k vs. (b) log k vs (c) 2 (d) 0
log T T 4. For the first order reaction
1
(c) log k vs. log T (d) k vs. T C 2 H 4 O(g) ® CH 4 (g) + CO(g) ,the initial pressure of
2. The rate constant of the reaction A ® B is 0.6 × 10–3 mol L–1S–1. C2H4O(g) is 80 torr and total pressure at the end of 20
If the concentration of A is 5 M then concentration of B minutes is 120 torr. The time needed for 75% decomposition
after 20 minutes is : of C2H4O would be :
(a) 1.08 M (b) 3.60 M (a) 20 minutes (b) 40 minutes
(c) 0.36 M (d) 0.72 M (c) 80 minutes (d) 120 minutes

RESPONSE GRID 1. 2. 3. 4.

Space for Rough Work


EBD_7504
C-70 NTA JEE Main

5. When a biochemical reaction is carried out in laboratory in 10. Select the rate law that corresponds to data shown for the
the absence of enzyme then rate of reaction obtained is following reaction
10–6 times, then activation energy of reaction in the Presence A+B ¾ ¾® products.
of enzyme is Exp. [A] [B] Initial rate
6 1 0.012 0.035 0.1
(a) 2 0.024 0.070 0.8
RT
3 0.024 0.035 0.1
(b) Different from Ea obtained in laboratory
4 0.012 0.070 0.8
(c) P is required (a) rate = k [B]3
(d) Can't say anything (b) rate = k [B]4
6. In a reaction A ® Products, when start is made from (c) rate = k [A] [B]3
8.0 × 10–2 M of A, half-life is found to be 120 minute. For the (d) rate = k [A]2 [B]2
initial concentration 4.0 × 10–2 M, the half-life of the reaction 11. A radioactive element gets spilled over the floor of a room. Its
becomes 240 minute. The order of the reaction is : half-life period is 30 days. If the initial velocity is ten times the
(a) zero (b) one permissible value, after how many days will it be safe to enter
(c) two (d) 0.5 the room?
7. In a first-order reaction A ® B, if k is rate constant and (a) 100 days (b) 1000 days
inital concentration of the reactant A is 0.5 M, then the half-
(c) 300 days (d) 10 days
life is
12. The energies of activation for forward and reverse reactions
log 2
(a) log 2 (b) for A2 + B2 ƒ 2AB are 180 kJ mol–1 and 200 kJ mol–1
k k 0.5 respectively. The presence of a catalyst lowers the activation
ln 2 0.693 energy of both (forward and reverse) reactions by 100 kJ
(c) (d)
k 0.5k mol–1. The enthalpy change of the reaction (A2 + B2 ↑
8. The rate expression for the reaction A(g) + B(g)®C(g) is 2AB) in the presence of a catalyst will be (in kJ mol–1)
(a) 20 (b) 300
rate = kC 2A C½B . What changes in the initial concentrations (c) 120 (d) 280
of A and B will cause the rate of reaction to increase by a
13. Consider the reaction :
factor of eight?
(a) CA×2; CB×2 (b) CA×2; CB×4 Cl2 ( aq ) + H 2S( aq ) ® S(s ) + 2H + (aq) + 2Cl- (aq )
(c) CA×1; CB×4 (d) CA×4; CB×1 The rate equation for this reaction is
9. The reaction A(g) + 2B(g) ® C(g) + D(g) is an elementary
process. In an experiment involving this reaction, the initial rate = k[Cl2 ][H 2S]
partial pressures of A and B are pA = 0.60 atm and pB= 0.80 Which of these mechanisms is/are consistent with this rate
atm respectively. When pC= 0.20 atm, the rate of the reaction equation?
relative to the initial rate is
A. Cl 2 + H 2S ® H + + Cl - + Cl+ + HS- (slow)
(a) 1/6 (b) 1/12
(c) 1/36 (d) 1/18 Cl + + HS- ® H + + Cl- + S (fast)
RESPONSE 5. 6. 7. 8. 9.
GRID 10. 11. 12. 13.
Space for Rough Work
Chemistry C-71

B. H 2S ƒ H + + HS- (fast equilibrium) 18. Activation energy (Ea) and rate constants (k1 and k2) of a
chemical reaction at two different temperatures (T1 and T2)
Cl 2 + HS- ® 2Cl - + H + + S (Slow) are related by :
(a) B only (b) Both A and B
k2 E æ1 1ö E æ1 1ö
< , a çç , ÷÷÷
k2
(c) Neither A nor B (d) A only (a) ln
ç
(b) ln < , a çç , ÷÷÷
k1 R è T1 T2 ø÷ k1 R èç T2 T1 ø÷
14. The initial rates of reaction
3A + 2B + C ¾¾ ® Products, at different initial
E æ1 1ö k2 E æ1 1ö
< a çç , ÷÷÷
k2
concentrations are given below: (c) ln < , a çç ∗ ÷÷÷ (d) ln ç
Initial rate, [A]0, M [B]0, M [C]0, M k1 ç
R è 2T T ÷
1ø k1 R è 1T T ÷

Ms–1
19. Consider a reaction aG + bH ® Products. When
5.0 × 10–3 0.010 0.005 0.010
concentration of both the reactants G and H is doubled, the
5.0 × 10–3 0.010 0.005 0.015
rate increases by eight times. However, when concentration
1.0 × 10–2 0.010 0.010 0.010
of G is doubled keeping the concentration of H fixed, the
1.25 × 10–3 0.005 0.005 0.010
rate is doubled. The overall order of the reaction is
The order with respect to the reactants, A, B and C are
respectively (a) 0 (b) 1
(a) 3, 2, 0 (b) 3, 2, 1 (c) 2, 2, 0 (d) 2, 1, 0 (c) 2 (d) 3
15. In a reversible reaction the energy of activation of the 20. The rate law for a reaction between the substances A and B
forward reaction is 50 kcal. The energy of activation for the is given by
reverse reaction will be Rate = k [A]n [B]m
(a) < 50 kcal On doubling the concentration of A and halving the
(b) either greater than or less than 50 kcal concentration of B, the ratio of the new rate to the earlier rate
(c) 50 kcal
of the reaction will be as
(d) > 50 kcal
(a) (m + n) (b) (n – m)
16. Half life period of a first-order reaction is 1386 seconds. The
specific rate constant of the reaction is: 1
(a) 0.5 × 10–2 s–1 (b) 0.5 × 10–3 s–1 (c) 2(n – m) (d) (m + n )
2
(c) 5.0 × 10–2 s–1 (d) 5.0 × 10–3 s–1
21. The rate constants k1 and k2 for two different reactions are
17. t 1 can be taken as the time taken for the concentration of a
1016 . e–2000/T and 1015 . e–1000/T, respectively. The temperature
4
3 at which k1 = k2 is :
reactant to drop to of its initial value. If the rate constant
4 t
for a first order reaction is k, the 1 can be written as 2000 1000
4 (a) 1000 K (b) K (c) 2000 K (d) K
(a) 0.75/k (b) 0.69/k 2.303 2.303
(c) 0.29/k (d) 0.10/k
RESPONSE 14. 15. 16. 17. 18.
GRID 19. 20. 21.
Space for Rough Work
EBD_7504
C-72 NTA JEE Main

22. Match the columns 26. The given reaction


Column - I Column - II 2FeCl 3 + SnCl 2 ¾ ¾® 2FeCl 2 + SnCl 4
[R ] - [R]
(A) k = 0 (p) k = 2.303 log 2 is an example of
t t1 2 (a) first order reaction (b) second order reaction
2.303 [R ] (c) third order reaction (d) None of these
(B) k = log 0 (q) Rate constant for zero
t [R] 27. Consider the reaction
order reaction N2 (g) + 3H2 (g) ® 2 NH3 (g)
(C) Value of k for first (r) k = [R]0 d[ NH 3 ] d[H 2 ]
2t1 2 The equality relationship between and - is
dt dt
order reaction when d[ NH 3 ] 2 d[ H 2 ] d[ NH 3 ] 3 d[H 2 ]
t = t 1 and[R] =
[R]0 (a) + =- (b) + =-
dt 3 dt dt 2 dt
2
2 d[ NH 3 ] d[H 2 ] d[ NH 3 ] 1 d[ H 2 ]
(D) Value of k for zero (s) Rate constant for first (c) =- (d) =-
order reaction when order reaction dt dt dt 3 dt
28. Which of the following statements is incorrect?
[R]0
t = t1 2 and [R] = (a) Activation energy for the forward reaction equals to
2 activation energy for the reverse reaction
(a) A – (s), B – (q), C – (p), D – (r)
(b) For a reversible reaction, an increase in temperature
(b) A – (q), B – (s), C – (p), D – (r) increases the reaction rate for both the forward and the
(c) A – (q), B – (p), C – (s), D – (r) backward reaction
(d) A – (q), B – (s), C – (p), D – (t) (c) The larger the initial reactant concentration for a second
23. The slope in Arrhenius plot, is equal to: order reaction, the shorter is its half-life.
Ea Ea (d) When Dt is infinitesimally small, the average rate equals
(a) - (b)
2.303 R R the instantaneous rate
R 29. 3A ® B + C, it would be a zero order reaction when
(c) - (d) None of these
2.303 Ea (a) the rate of reaction is proportional to square of
concentration of A
24. For the reaction A + B ¾¾ ® products, it is observed that: (b) the rate of reaction remains same at any concentration
(1) On doubling the initial concentration of A only, the of A
rate of reaction is also doubled and (c) the rate remains unchanged at any concentration of B
(2) On doubling the initial concentrations of both A and B, and C
there is a change by a factor of 8 in the rate of the reaction. (d) the rate of reaction doubles if concentration of B is
The rate of this reaction is given by: increased to double
(a) rate = k [A] [B]2 (b) rate = k [A]2 [B]2 30. The decomposition of a substance follows first order kinetics.
(c) rate = k [A] [B] (d) rate = k [A]2 [B] Its concentration is reduced to 1/8th of its initial value in 24
25. For the reaction 2 N 2O 5 ® 4NO 2 + O 2 , rate and rate minutes. The rate constant of the decomposition process is
constant are 1.02 × 10–4 mol lit–1 sec–1 and 3.4×10–5 sec–1 0.692
(a) 1/24 min–1 (b) min -1
respectively then concentration of N 2O5 at that time will be 24
(a) 1.732M (b) 3M 2.303 æ1ö 2.303 æ8ö
(c) 3.4 × 105M (d) 1.02 × 10–4M (c) log ç ÷ min -1 (d) log ç ÷ min -1
24 è8ø 24 è1ø
RESPONSE 22. 23. 24. Work
Space for Rough 25. 26.
GRID 27. 28. 29. 30.

CHEMISTRY CHAPTERWISE SPEED TEST-47


Total Questions 30 Total Marks 120
Attempted Correct
Incorrect Net Score
Cut-off Score 38 Qualifying Score 56
Success Gap = Net Score – Qualifying Score
CHEMISTRY Speed
Surface Chemistry TEST
No. of Questions
30
Maximum Marks
120
Time
1 Hour
48
Chapter-wise

GENERAL INSTRUCTIONS
• This test contains 30 MCQ's. For each question only one option is correct. Darken the correct circle/ bubble in the
Response Grid provided on each page.
• You have to evaluate your Response Grids yourself with the help of solutions provided at the end of this book.
• Each correct answer will get you 4 marks and 1 mark shall be deduced for each incorrect answer. No mark will be given/
deducted if no bubble is filled. Keep a timer in front of you and stop immediately at the end of 60 min.
• The sheet follows a particular syllabus. Do not attempt the sheet before you have completed your preparation for that
syllabus.
• After completing the sheet check your answers with the solution booklet and complete the Result Grid. Finally spend time
to analyse your performance and revise the areas which emerge out as weak in your evaluation.

1. Which of the following electrolytes is least effective in (c) mixing equal volumes of equimolar solutions of AgNO3
coagulating ferric hydroxide solution? and KI
(a) KBr (b) K2SO4 (d) None of these
(c) K2CrO4 (d) K4 [Fe(CN)6] 4. Which of the following statements is not true about the oil-
2. Cod Liver oil is in-water type emulsion?
(a) fat dispersed in water (b) water dispersed in fat
(a) On addition of small amount of water, no separate layer
(c) water dispersed in oil (d) fat dispersed in fat.
of water appears
3. One desires to prepare a positively charged sol of silver
iodide. This can be achieved by (b) On addition of oil, separate layer of oil is formed
(a) adding small amount of AgNO3 solution to KI solution (c) Addition of an electrolyte causes the conductivity of
in slight excess the emulsion to increase
(b) adding small amount of KI solution to AgNO3 solution (d) Addition of small amount of oil soluble dye renders
in slight excess the entire emulsion coloured.

RESPONSE GRID 1. 2. 3. 4.

Space for Rough Work


EBD_7504
C-74 NTA JEE Main

5. Freundlich equation for adsorption of gases (in amount of x g) 11. Which of the following curves is in accordance with
on a solid (in amount of m g) at constant temperature can be Freundlich adsorption isotherm ?
expressed as (a) (b)

log x/m ®
x 1

log x/m ®
x 1
(a) log = log p + log k (b) log = log k + log p
m n m n

x x 1
(c) µ pn (d) = log p + log k p®
m m n p®
6. The size of colloidal particles is of the order (c) (d)
(a) > 0.1 m

log x/m ®
(b) 1mm to 0.2m

log x/m ®
(c) < 1 mm (d) > 10 mm
7. Which of the following constitutes irreversible colloidal
system in water as dispersion medium?
log p¢ ® log p ®
(a) Clay (b) Platinum
(c) Fe(OH)3 (d) All of these 12. A colloidal solution is subjected to an electrical field. The
8. Which of the following forms cationic micelles above certain particles move towards anode. The coagulation of same sol
concentration? is studied using NaCl, BaCl2 and AlCl3 solutions. Their
(a) Sodium dodecyl sulphate coagulating power should be
(b) Sodium acetate (a) NaCl > BaCl2 > AlCl3
(c) Urea (b) BaCl2 > AlCl3 > NaCl
(d) Cetyl trimethyl ammonium bromide (c) AlCl3 > BaCl3 > NaCl
9. Position of non-polar and polar part in micelle is (d) BaCl2 > NaCl > AlCl3
(a) polar at outer surface and non-polar at inner surface 13. Which of the following is most effective in causing the
(b) polar at inner surface and non-polar at outer surface coagulation of ferric hydroxide sol?
(c) distributed all over the surface (a) KCl (b) KNO3
(d) present in the surface only (c) K2SO4 (d) K3[Fe(CN)6]
10. The cause of Brownian movement is 14. Alum helps in purifying water by
(a) Heat changes in liquid state (a) forming Si complex with clay particles
(b) Convectional currents (b) sulphate part which combines with the dirt and removes
(c) The impact of molecules of the dispersion medium on it
the colloidal particles. (c) aluminium which coagulates the mud particles
(d) Attractive forces between the colloidal particles and (d) making mud water soluble
molecules of dispersion medium.

RESPONSE 5. 6. 7. 8. 9.
GRID 10. 11. 12. 13. 14.
Space for Rough Work
Chemistry C-75

15. Which is not correct regarding the adsorption of a gas on (b) Mixing the sols has no effect
surface of solid? (c) Sodium sulphate solution causes coagulation in both
(a) On increasing temperature, adsorption increases sols
continuously (d) Magnesium chloride solution coagulates, the gold sol
(b) Enthalpy and entropy changes are –ve more readily than the iron (III) hydroxide sol.
(c) Adsorption is more for some specific substances 21. Which of the following will show Tyndall effect?
(d) This phenomenon is reversible (a) Aqueous solution of soap below critical micelle
16. The heats of adsorption in physisorption lie in the range (in concentration.
kJ/mol) (b) Aqueous solution of soap above critical micelle
(a) 40 - 400 (b) 40 - 100 concentration.
(c) 10 - 40 (d) 1 - 10 (c) Aqueous solution of sodium chloride.
17. Example of intrinsic colloid is (d) Aqueous solution of sugar.
(a) glue (b) sulphur 22. If x is amount of adsorbate and m is amount of adsorbent,
(c) Fe (d) As 2 S3 which of the following relations is not related to adsorption
18. For adsorption of a gas on a solid, the plot of log x/m vs log process ?
P is linear with slope equal to (n being whole number) (a) x / m = f (p) at constant T.
(a) k (b) log k (b) x / m = f (T) at constant p.
1 (c) p = f (T) at constant (x / m).
(c) n (d)
n x
(d) = p ´T
19. Which of the following statements is incorrect? m
(a) Colloidal gold is used for intramuscular injection. 23. In Langmuir's model of adsorption of a gas on a solid surface
(b) Colloidal solution of latex is used in preparation of (a) the mass of gas striking a given area of surface is
rubber. proportional to the pressure of the gas
(c) Photographic films are prepared by coating an (b) the mass of gas striking a given area of surface is
emulsion of AgBr in gelatin over glass plate. independent of the pressure of the gas
(d) Tannin used in leather industry contains positively (c) the rate of dissociation of adsorbed molecules from
charged colloidal particles. the surface does not depend on the surface covered
(d) the adsorption at a single site on the surface may
20. The disperse phase in colloidal iron (III) hydroxide and
involve multiple molecules at the same time
colloidal gold is positively and negatively charged,
respectively. Which of the following statements is NOT 24. Which of the following gas molecules have maximum value
correct? of enthalpy of physisorption?
(a) C2H6 (b) Ne
(a) Coagulation in both sols can be brought about by
electrophoresis (c) H2O (d) H2

RESPONSE 15. 16. 17. 18. 19.


GRID 20. 21. 22. 23. 24.
Space for Rough Work
EBD_7504
C-76 NTA JEE Main

25. Which one of the following statements is not correct? 28. At high pressure, the entire surface gets covered by a
(a) The value of equilibrium constant is changed in the monomolecular layer of the gas follows
presence of a catalyst in the reaction at equilibrium (a) three-halved order (b) second-order
(b) Enzymes catalyse mainly bio-chemical reactions (c) first-order (d) zero-order
(c) Coenzymes increase the catalytic activity of enzyme 29. At the Critical Micelle Concentration (CMC) the surfactant
(d) Catalyst does not initiate any reaction molecules
26. Among the electrolytes Na2SO4, CaCl2, Al2(SO4)3 and (a) decompose
NH4Cl, the most effective coagulating agent for Sb2S3 sol (b) dissociate
is (c) associate
(a) Na2SO4 (b) CaCl2 (d) become completely soluble
(c) Al2(SO4)3 (d) NH4Cl 30. Peptization involves
27. Which of the following is not a favourable condition for (a) precipitation of colloidal particles
physical adsorption ? (b) disintegration of colloidal aggregates
(a) High pressure (c) evaporation of dispersion medium
(b) Negative DH (d) impact of molecules of the dispersion medium on the
(c) Higher critical temperature of adsorbate colloidal particles
(d) High temperature

RESPONSE 25. 26. 27. 28. 29.


GRID 30.

CHEMISTRY CHAPTERWISE SPEED TEST-48


Total Questions 30 Total Marks 120
Attempted Correct
Incorrect Net Score
Cut-off Score 40 Qualifying Score 60
Success Gap = Net Score – Qualifying Score
Net Score = (Correct × 4) – (Incorrect × 1)
Space for Rough Work
CHEMISTRY Speed
General Principles and Processes of TEST
Isolation of Elements
No. of Questions
30
Maximum Marks
120
Time
1 Hour
49
Chapter-wise

GENERAL INSTRUCTIONS
• This test contains 30 MCQ's. For each question only one option is correct. Darken the correct circle/ bubble in the
Response Grid provided on each page.
• You have to evaluate your Response Grids yourself with the help of solutions provided at the end of this book.
• Each correct answer will get you 4 marks and 1 mark shall be deduced for each incorrect answer. No mark will be given/
deducted if no bubble is filled. Keep a timer in front of you and stop immediately at the end of 60 min.
• The sheet follows a particular syllabus. Do not attempt the sheet before you have completed your preparation for that
syllabus.
• After completing the sheet check your answers with the solution booklet and complete the Result Grid. Finally spend time
to analyse your performance and revise the areas which emerge out as weak in your evaluation.

1. Identity x, y, z for the following metallurgical process. 2. Bronze is a mixture of


x
Metal sulphide ¾¾ y (a) Pb + Sn (b) Cu + Sn
® Metal oxide ¾¾ ® Impure metal
(c) Cu + Zn (d) Pb + Zn
z
¾¾ ® Pure metal. 3. Which reagent is used in Bayer’s process?
x, y and z are respectively (a) Na2CO3 (b) Carbon
(a) roasting, smelting, electrolysis (c) NaOH (d) Silica
(b) roasting, calcination, smelting 4. Thomas slag is
(c) calcination, auto-reduction, bassemerisation (a) Ca3(PO4)2
(d) none of the above is correct (b) CaSiO3
(c) Mixture of (a) and (b)
(d) FeSiO3

RESPONSE GRID 1. 2. 3. 4.

Space for Rough Work


EBD_7504
C-78 NTA JEE Main

5. Which of the following condition favours the reduction of 11. D G° vs T plot in the Ellingham’s diagram slopes downward
a metal oxide to metal? for the reaction
(a) DH = +ve, TDS = + ve at low temperature 1
(b) DH = +ve, TDS = – ve at any temperature (a) Mg + O 2 ® MgO
(c) DH = –ve, TDS = – ve at high temperature 2
(d) DH = –ve, TDS = + ve at any temperature 1
(b) 2Ag + O 2 ® Ag 2O
6. Which of the following fluxes is used to remove acidic 2
impurities in metallurgical process? 1
(c) C + O 2 ® CO
(a) Silica 2
(b) Lime stone 1
(d) CO + O 2 ® CO 2
(c) Sodium chloride 2
(d) Sodium carbonate 12. Which of the following elements is present as the impurity
7. A basic lining is given to a furnace by using to the maximum extent in the pig iron ?
(a) Calcined dalomite (a) Manganese (b) Carbon
(b) Lime stone (c) Silicon (d) Phosphorus
(c) hae matite
13. Among the following statements the incorrect one is
(d) silica
(a) Calamine and siderite are carbonates
8. Electromagnetic separation is used in the concentration of
(b) Argentite and cuprite are oxides
(a) copper pyrites (b) bauxite (c) Zinc blende and iron pyrites are sulphides
(c) cassiterite (d) cinnabar (d) Malachite and azurite are ores of copper.
9. Extraction of gold and silver involves leaching with CN– 14. Match list I with list II and select the correct answer using
ion. Silver is later recovered by the codes given below the lists:
(a) distillation List I List II
I. Cyanide process A. Ultrapure Ge
(b) zone refining
II. Floatation process B. Pine oil
(c) displacement with Zn III. Electrolytic reduction C. Extraction of Al
(d) liquation IV. Zone refining D. Extraction of Au
10. Which of the following reactions is an example for calcination Codes:
process ? (a) I-C, II-A, III-D, IV-B (b) I-D,II-B,III-C,IV-A
(a) 2Ag + 2HCl + (O ) ® 2AgCl + H 2 O (c) I-C,II-B,III-D, IV-A (d) I-D,II-A,III-C,IV-B
15. A coupled reaction is takes place as follow–
(b) 2Zn + O 2 ® 2 ZnO A + B ––––® C + D, D Gº = + x kj
(c) 2ZnS + 3O 2 ® 2ZnO + 2SO 2 D + E ––––® F D Gº = – y kj
for the spontaneity of reaction A + B + E –––® C+F,
(d) MgCO3 ® MgO + CO 2 which of the following is correct?
(a) 2x = y (b) x < y
(c) x > y (d) x = (y)× TDS

5. 6. 7. 8. 9.
RESPONSE 10. 11. 12. 13. 14.
GRID 15.

Space for Rough Work


Chemistry C-79

16. Aluminium is extracted from alumina (Al2O3 ) by electrolysis 20. In the cyanide extraction process of silver from argentite
of a molten mixture of ore, the oxidising and reducing agents used are
(a) Al2O3 + HF + NaAlF4 (a) O2 and CO respectively
(b) Al2O3 + CaF2 + NaAlF4 (b) O2 and Zn dust respectively
(c) Al2O3 + Na3AlF6 + CaF2 (c) HNO3 and Zn dust respectively
(d) Al2O3 + KF + Na3AlF6 (d) HNO3 and CO respectively
17. Which of the following statements, about the advantage of 21. The metal oxide which cannot be reduced to metal by carbon
roasting of sulphide ore before reduction is not true? is
(a) Fe2O3 (b) Al2O3
(a) The DG of of the sulphide is greater than those for CS2 (c) PbO (d) ZnO
and H2S. 22. Which of the following reaction takes place in blast furnace
during extraction of copper ?
(b) The DG of is negative for roasting of sulphide ore to oxide. (a) 2Cu 2S + 3O 2 ¾¾ ® 2Cu 2O + 2SO2
(c) Roasting of the sulphide to the oxide is thermodynamically (b) 2FeS + 3O 2 ¾¾ ® 2FeO + 2SO 2
feasible. (c) 2Cu 2O + Cu 2S ¾¾ ® 6Cu + SO2
(d) Carbon and hydrogen are suitable reducing agents for (d) All of these
reduction of metal sulphides. 23. Which process represents the change,
18. In the extraction of copper from its sulphide ore, the metal is Ti + 2I 2 ® TiI 4 ® Ti + 2I 2
finally obtained by the reduction of cuprous oxide with : (a) Cupellation
(a) Copper (I) sulphide (Cu2S) (b) Van Arkel
(b) Sulphur dioxide (SO2) (c) Polling
(c) Iron sulphide (FeS)
(d) Zone Refining
(d) Carbon monoxide (CO)
24. Among the following groups of oxides, the group containing
19. The method of zone refining of metals is based on the
oxides that cannot be reduced by carbon to give the
principle of
respective metals is
(a) Greater solubility of the impurity in the molten state
than in the solid (a) Cu 2 O, SnO 2 (b) Fe 2 O 3 , ZnO
(b) Greater mobility of the pure metal than that of the (c) CaO, K 2O (d) PbO, Fe3O 4
impurity
25. Which of the following pairs of metals is purified by van
(c) Higher melting point of the impurity than that of the
Arkel method ?
pure metal
(a) Ga and In (b) Zr and Ti
(d) Greater noble character of the solid metal than that of
the impurity (c) Ag and Au (d) Ni and Fe

RESPONSE 16. 17. 18. 19. 20.


GRID 21. 22. 23. 24. 25.

Space for Rough Work


EBD_7504
C-80 NTA JEE Main

26. Which of the following reaction(s) occur in temperature 29. Which of the following statements is correct ?
range 500 – 800 K in blast furnace. (a) Gangues are carefully chosen to combine with the slag
(i) Fe2O3 + CO ¾¾ ® 2FeO + CO2 present in the ore to produce easily fusible flux to carry
(ii) Fe3O4 + 4CO ¾¾ ® 3Fe + 4CO2 away the impurities
(iii) FeO + CO ¾¾ ® Fe + CO 2 (b) Slags are carefully chosen to combine with the flux
(iv) C + CO 2 ¾¾ ® 2CO present in the ore to produce easily fusible gangue to
carry away the impurities
(a) (i) and (ii) (b) (i), (ii) and (iii)
(c) Gangues are carefully chosen to combine with the flux
(c) (iii) and (iv) (d) (iv) only
present in the ore to produce easily fusible slag to
27. Which of the following pair is incorrectly matched ?
carry away the impurities
(a) Magnetite – Fe3O4 (b) Copper glance – Cu2S
(d) Fluxes are carefully chosen to combine with the gangue
(c) Calamine – ZnCO3 (d) Zincite – ZnS
present in the ore to produce easily fusible slag to
28. Before introducing FeO in blast furnace, it is converted to
carry away the impurities
Fe2O3 by roasting so that
30. The value of Df Gº for formation of Cr 2 O3 is – 540 kJmol–1 and
(a) it may not be removed as slag with silica that of Al2O3 is – 827 kJ mol –1. What is the value of DrG° for
(b) it may not evaporate in the furnace the reaction?
(c) presence of it may increase the m.pt. of charge 4 2 2 4
(d) None of these. Al(s) + Cr2 O 3 (s) ® Al 2 O3 (s) + Cr(s).
3 3 3 3
(a) – 574 kJmol–1 (b) –287kJ mol–1
(c) + 574 kJ mol–1 (d) +287kJ mol–1

RESPONSE GRID 26. 27. 28. 29. 30.

CHEMISTRY CHAPTERWISE SPEED TEST-49


Total Questions 30 Total Marks 120
Attempted Correct
Incorrect Net Score
Cut-off Score 38 Qualifying Score 56
Success Gap = Net Score – Qualifying Score
Net Score = (Correct × 4) – (Incorrect × 1)

Space for Rough Work


CHEMISTRY Speed
The p-Block Elements TEST
(Group 15, 16, 17 and 18)
No. of Questions
30
Maximum Marks
120
Time
1 Hour
50
Chapter-wise

GENERAL INSTRUCTIONS
• This test contains 30 MCQ's. For each question only one option is correct. Darken the correct circle/ bubble in the
Response Grid provided on each page.
• You have to evaluate your Response Grids yourself with the help of solutions provided at the end of this book.
• Each correct answer will get you 4 marks and 1 mark shall be deduced for each incorrect answer. No mark will be given/
deducted if no bubble is filled. Keep a timer in front of you and stop immediately at the end of 60 min.
• The sheet follows a particular syllabus. Do not attempt the sheet before you have completed your preparation for that
syllabus.
• After completing the sheet check your answers with the solution booklet and complete the Result Grid. Finally spend time
to analyse your performance and revise the areas which emerge out as weak in your evaluation.

1. The oxyacid of phosphorous in which phosphorous has 3. The deep blue colour produced on adding excess of ammonia
the lowest oxidation state is to copper sulphate is due to presence of
(a) hypophosphorous acid 2+
(a) Cu 2 + (b) Cu ( NH 3 ) 4
(b) orthophosphoric acid
(c) pyrophosphoric acid (c) Cu ( NH 3 ) 26 + (d) Cu ( NH 3 ) 22 +
(d) metaphosphoric acid
2. The decreasing values of bond angles from NH3 (106º) to 4. In a cyclotrimetaphosphoric acid molecule, how many single
SbH3 (101º) down group-15 of the periodic table is due to and double bonds are present?
(a) 3 double bonds; 9 single bonds
(a) decreasing lp-bp repulsion
(b) 6 double bonds; 6 single bonds
(b) decreasing electronegativity
(c) 3 double bonds; 12 single bonds
(c) increasing bp-bp repulsion
(d) Zero double bonds; 12 single bonds
(d) increasing p-orbital character in sp3

RESPONSE GRID 1. 2. 3. 4.

Space for Rough Work


EBD_7504
C-82 NTA JEE Main

5. In nitroprusside ion, the iron and NO exist as Fe++ and NO+ 9. The hybridization in ICl7 is
rather than Fe+++ and NO. These forms can be differentiated (a) sp3 d 3
by (b) d2 sp 3
(a) estimating the concentration of iron (c) sp3 d
(b) measuring the concentration of CN - (d) sp3
(c) measuring the solid state magnetic moment 10. The compound of sulphur that can be used as refrigerant is
(d) thermally decomposing the compound (a) SO2
6. The brown ring test for NO -2 and NO 3- is due to the (b) SO3
formation of complex ion with a formula (c) S2Cl2
(a) [Fe(H2O)6]2+ (d) H2SO4
(b) [Fe(NO)(CN)5]2+ 11. Which of the following is a saline oxide ?
(c) [Fe(H2O)5NO]2+ (a) Na2O2 (b) BaO2
(d) [Fe(H2O)(NO)5]2+
(c) Na2O (d) Fe2O3
7. Which compound is used in photography?
12. Which one of the following substances has the highest
(a) Na2SO5 proton affinity ?
(b) Na2S2O8 (a) H2S (b) NH3
(c) Na2S2O6 (c) PH3 (d) H2O
(d) Na2S2O3 13. Which one of the following statement regarding helium is
8. Concentrated hydrochloric acid when kept in open air incorrect ?
sometimes produces a cloud of white fumes. The explanation (a) It is used to pr oduce an d sustain powerful
for it is that superconducting magnets
(a) oxygen in air reacts with the emitted HCl gas to form a
(b) It is used as a cryogenic agent for carrying out
cloud of chlorine gas
experiments at low temperatures
(b) strong affinity of HCl gas for moisture in air results in
forming of droplets of liquid solution which appears like (c) It is used to fill gas balloons instead of hydrogen
a cloudy smoke. because it is lighter and non-inflammable
(c) due to strong affinity for water, concentrated (d) It is used in gas-cooled nuclear reactors
hydrochloric acid pulls moisture of air towards itself. 14. Caro’s acid is
This moisture forms droplets of water and hence the (a) H2SO3
cloud. (b) H3S2O5
(d) concentrated hydrochloric acid emits strongly smelling (c) H2SO5
HCl gas all the time. (d) H2S2O8

RESPONSE 5. 6. 7. 8. 9.
GRID 10. 11. 12. 13. 14.
Space for Rough Work
Chemistry C-83

15. The acid which forms two series of salts is 21. Yellow ammonium sulphide is
(a) H3PO4 (b) H3PO3 (a) ( NH 4 ) 2 S8
(c) H3BO3 (d) H 3 PO -2
(b) ( NH 4 ) 2 S
16. Which of the following species is not a pseudo halide
(c) ( NH 4 ) 2 S x
(a) CNO– (b) RCOO–
(c) OCN– (d) NNN– (d) ( NH 4 ) 2 S 4
17. Gaseous HCl is a poor conductor of electricity while its 22. The correct order of increasing bond angles in the following
aqueous solution is a good conductor this is because species are :
(a) H 2 O is a good conductor of electricity
(a) Cl 2 O < ClO 2 < ClO 2–
(b) a gas cannot conduct electricity but a liquid can
(c) HCl gas does not obey Ohm’s law, whereas the solution (b) ClO2 < Cl 2O < ClO 2–
does
(d) HCl ionises in aqueous solution (c) Cl 2O < ClO 2– < ClO2
18. The number of hydrogen atom(s) attached to phosphorus
atom in hypophosphorous acid is (d) ClO 2– < Cl 2O < ClO2
(a) three (b) one 23. Which one of the following reactions of xenon compounds
(c) two (d) zero is not feasible?
19. What products are expected from the disproportionation (a) 3Xe F4 + 6H 2 O ¾¾ ® 2 Xe + Xe O3 +12HF +1.5O 2
reaction of hypochlorous acid?
(b) 2Xe F2 + 2H 2 O ¾¾
® 2 Xe + 4HF + O 2
(a) HCl and Cl 2O
(b) HCl and HClO3 (c) Xe F6 + RbF ¾¾
® Rb[Xe F7 ]
(c) HClO3 and Cl2O (d) Xe O3 + 6HF ¾¾ ® Xe F6 + 3H 2 O
(d) HClO2 and HClO4 24. Trigonal bipyramidal geometry is shown by :
20. Shape of XeOF4 is (a) XeO3F2
(a) octahedral (b) XeO3F2
(b) square pyramidal (c) FXeOSO2F
(c) pyramidal (d) [XeF8]2–
(d) T-shaped

RESPONSE 15. 16. 17. 18. 19.


GRID 20. 21. 22. 23. 24.
Space for Rough Work
EBD_7504
C-84 NTA JEE Main

25. Which of the following is used to produce and sustain (a) HOClO < HOCl < HOClO3 < HOClO2
powerful superconducting magnets to form an essential part (b) HOClO2 < HOClO3 < HOClO < HOCl
of NMR spectrometer ? (c) HOClO3 < HOClO2 < HOClO < HOCl
(a) Ar (b) Ne (d) HOCl < HOClO < HOClO2 < HOClO3
(c) Rn (d) He 29. Shapes of certain interhalogen compounds are stated below.
26. Oxidation of thiosulphate by iodine gives Which one of them is not correctly stated?
(a) tetrathionate ion (b) sulphide ion (a) IF7 : pentagonal bipyramid
(c) sulphate ion (d) sulphite ion (b) BrF5 : trigonal bipyramid
(c) BrF3 : planar T-shaped
27. The crystals of ferrous sulphate on heating give :
(d) ICI3 : planar dimeric
(a) FeO + SO 2 + H 2 O 30. The nitrogen oxides that contain(s) N–N bond(s) is /are
(b) FeO + SO 3 + H 2SO 4 + H 2O (i) N2O (ii) N2O3
(iii) N2O4 (iv) N2O5
(c) Fe 2 O 3 + SO 2 + H 2SO 4 + H 2O (a) (i) , (ii) (b) (ii) , (iii) , (iv)
(c) (iii) , (iv) (d) (i), (ii) and (iii)
(d) Fe 2O 3 + H 2SO 4 + H 2 O
28. Which one of the following orders correctly represents the
increasing acid strengths of the given acids?

RESPONSE 25. 26. 27. 28. 29.


GRID 30.

CHEMISTRY CHAPTERWISE SPEED TEST-50


Total Questions 30 Total Marks 120
Attempted Correct
Incorrect Net Score
Cut-off Score 35 Qualifying Score 51
Success Gap = Net Score – Qualifying Score
Net Score = (Correct × 4) – (Incorrect × 1)
Space for Rough Work
CHEMISTRY Speed
The d- and f- Block Elements TEST
No. of Questions
30
Maximum Marks
120
Time
1 Hour
51
Chapter-wise

GENERAL INSTRUCTIONS
• This test contains 30 MCQ's. For each question only one option is correct. Darken the correct circle/ bubble in the
Response Grid provided on each page.
• You have to evaluate your Response Grids yourself with the help of solutions provided at the end of this book.
• Each correct answer will get you 4 marks and 1 mark shall be deduced for each incorrect answer. No mark will be given/
deducted if no bubble is filled. Keep a timer in front of you and stop immediately at the end of 60 min.
• The sheet follows a particular syllabus. Do not attempt the sheet before you have completed your preparation for that
syllabus.
• After completing the sheet check your answers with the solution booklet and complete the Result Grid. Finally spend time
to analyse your performance and revise the areas which emerge out as weak in your evaluation.

1. Of the following outer electronic configurations of atoms,


the highest oxidation state is achieved by which one of
3. (n - 1)d 10ns 2 is the general electronic configuration of
them ? (a) Fe, Co, Ni
(a) (n – 1)d 3 ns2 (b) Cu, Ag, Au
(b) (n – 1)d 5 ns1 (c) Zn, Cd, Hg
(c) (n – 1)d 8 ns2 (d) Se, Y, La
(d) (n – 1)d 5 ns2 4. Super conductors are derived from compounds of
2. In which of the following cases, the stability of two (a) p-Block elements
oxidation states is correctly represented (b) lanthanides
(a) Ti3+ > Ti4+ (b) Mn2+ > Mn3+ (c) actinides
2+
(c) Fe > Fe 3+ (d) Cu+ > Cu2+ (d) transition elements

RESPONSE GRID 1. 2. 3. 4.

Space for Rough Work


EBD_7504
C-86 NTA JEE Main

5. Which one of the elements with the following outer orbital (d) Copper does not give H2 on reaction with dil. H2SO4.
configurations may exhibit the largest number of oxidation 10. When KMnO4 acts as an oxidising agent and ultimately forms
states? [MnO4]–1, MnO2, Mn2O3, Mn+2 then the number of electrons
(a) 3d 54s1 (b) 3d 54s2 transferred in each case respectively is
(c) 3d 24s2 (d) 3d 34s2 (a) 4, 3, 1, 5 (b) 1, 5, 3, 7 (c) 1, 3, 4, 5 (d) 3, 5, 7, 1.
6. The catalytic activity of transition metals and their 11. For which of the following species d-d transition does not
compounds is mainly due to : account for its colour ?
(a) their magnetic behaviour
(b) their unfilled d-orbitals (a) 2-
Cr2 O 7 (b) CrO24 -
(c) their ability to adopt variable oxidation state
(d) their chemical reactivity (c) CrO2 Cl 2 (d) All
7. Arrange Ce+3, La+3, Pm+3 and Yb+3 in increasing order of their 12. The "spin-only" magnetic moment [in units of Bohr
ionic radii. magneton, (µB)] of Ni2+ in aqueous solution would be (At.
(a) Yb+3 < Pm+3 < Ce+3 < La+3 No. Ni = 28)
(b) Ce+3 < Yb+3 < Pm+3 < La+3 (a) 6 (b) 1.73
(c) Yb+3 < Pm+3 < La+3 < Ce+3 (c) 2.82 (d) 4.90
(d) Pm+3 < La+3 < Ce+3 < Yb+3.
13. Heating mixture of Cu2O and Cu2S will give
8. What would happen when a solution of potassium chromate
(a) Cu2SO3 (b) CuO + CuS
is treated with an excess of dilute nitric acid?
(c) Cu + SO3 (d) Cu + SO2
(a) Cr2O 72- and H2O are formed 14. In which of the following lanthanides oxidation state +2 is
(b) CrO 24- is reduced to +3 state of Cr most stable?
(a) Ce (b) Eu
(c) CrO 24- is oxidized to +7 state of Cr (c) Tb (d) Dy
(d) Cr3+ and Cr2 O 72- are formed 15. Which one of the following nitrates will leave behind a metal
on strong heating ?
9. On the basis of data given below,
(a) Copper nitrate (b) Manganese nitrate
Ed
-
= -0.37 , E d
-
= +1.57 (c) Silver nitrate (d) Ferric nitrate
Sc3 + /Sc2 + Mn 3+ /Mn 2+
16. The black compound formed during the reaction be t we en
Ed
-
2+ = -0.90 , d
-
E 2+ = 0.34 sodium thiosulphate and silver nitrate is
Cr /Cr Cu /Cu
Which of the following statements is incorrect? (a) silver thiosulphate ( Ag2S2O3 )
(a) Sc 3+ has good stability due of [Ar]3d 0 4s 0 (b) silver sulphide ( Ag 2S)
configuration.
(c) silver sulphate ( Ag 2SO 4 )
(b) Mn3+ is more stable than Mn2+.
(c) Cr2+ is reducing in nature. (d) silver sulphite ( Ag 2SO3 )

5. 6. 7. 8. 9.
RESPONSE 10. 11. 12. 13. 14.
GRID 15. 16.

Space for Rough Work


Chemistry C-87

17. Which of the following element is responsible for oxidation 22. AgCl is soluble in NH4OH solution. The solubility is due to
of water to O2 in biological process? the formation of
(a) Fe (b) Mn (a) AgOH (b) Ag 2O
(c) Cu (d) Mo
18. Which of the following arrangements does not represent +
(c) éë Ag ( NH3 )2 ùû (d) NH 4Cl
the correct order of the property stated against it ?
(a) V2+ < Cr2+ < Mn2+ < Fe2+ : paramagnetic behaviour 23. Consider the complexes given below and choose the correct
(b) Ni2+ < Co2+ < Fe2+ < Mn2+ : ionic size order of paramagnetic moments (spin only)
(c) Co3+ < Fe3+ < Cr3+ < Sc3+ : stability in aqueous solution I. [FeF6]3– ; II. [CrF6]3– ;
(d) Sc < Ti < Cr < Mn : number of oxidation states
III. [V(H2O)6]3+ ; IV. [Ti(H2O)6]3+
19. Which of the following factors may be regarded as the main
cause of lanthanoide contraction? (a) I > II > III > IV (b) I < II < III < IV
(a) Greater shielding of 5d electrons by 4f electrons (c) I > III > IV > II (d) III > I > IV > II
(b) Poorer shielding of 5d electrons by 4f electrons 24. The value of the ‘spin only’ magnetic moment for one of the
(c) Effective shielding of one of 4f electrons by another in following configurations is 2.82 BM. The correct one is
the subshell (a) d5 (in strong ligand field)
(d) Poor shielding of one of 4f electron by another in the (b) d3 (in weak as well as in strong fields)
subshell (c) d4 (in weak ligand fields)
20. Which of the following elements shows maximum number of (d) d4 (in strong ligand fields)
different oxidation states in its compounds? 25. In neutral or faintly alkaline medium, thiosulphate is
(a) Eu (b) Ld quantitatively oxidized by KMnO4 to
(c) Gd (d) Am (a) SO32– (b) SO42–
21. An explosion take place when conc. H2SO4 is added to (c) SO2 (d) SO52–
KMnO4. Which of the following is formed? 26. Parke’s process is used
(a) Mn 2O7 (b) MnO 2 (a) to extract Ag using NaCN
(b) to extract Cu using CuFeS2
(c) MnSO4 (d) M 2O3 (c) to extract Ag from argentiferrous lead
(d) to extract Ag by forming amalgam

RESPONSE 17. 18. 19. 20. 21.


GRID 22. 23. 24. 25. 26.

Space for Rough Work


EBD_7504
C-88 NTA JEE Main

27. A metal which is not affected by conc. H 2SO 4 , HNO3 (c) Copper hydroxide is an amphoteric substance
(d) In acidic solutions hydration protects copper ions
or alkalies forms an compound X This compound X can be
used to give a complex which finds its application for tonning 29. A red solid is insoluble in water. However it becomes soluble
if some KI is added to water. Heating the red solid in a test
in photography. The metal is
tube results in liberation of some violet coloured fumes and
(a) Au (b) Ag
droplets of a metal appear on the cooler parts of the test
(c) Hg (d) Cu
tube. The red solid is
28. Ammonia forms the complex ion [Cu(NH3)4]2+ with copper
(a) HgI2 (b) HgO
ions in alkaline solutions but not in acidic solutions. What
(c) Pb3O4 (d) (NH4)2Cr2O7
is the reason for it ?
30. Four successive members of the first series of the transition
(a) In acidic solutions protons coordinate with ammonia
metals are listed below. For which one of them the standard
molecules forming NH +4 ions and NH3 molecules are
not available
( º
)
potential E M 2 + / M value has a positive sign?

(b) In alkaline solutions insoluble Cu(OH)2 is precipitated (a) Co (Z = 27) (b) Ni (Z = 28)
which is soluble in excess of any alkali (c) Cu (Z = 29) (d) Fe (Z = 26)

RESPONSE
27. 28. 29. 30.
GRID

CHEMISTRY CHAPTERWISE SPEED TEST-51


Total Questions 30 Total Marks 120
Attempted Correct
Incorrect Net Score
Cut-off Score 38 Qualifying Score 52
Success Gap = Net Score – Qualifying Score
Net Score = (Correct × 4) – (Incorrect × 1)

Space for Rough Work


CHEMISTRY Speed
Coordination Compounds TEST
No. of Questions
30
Maximum Marks
120
Time
1 Hour
52
Chapter-wise

GENERAL INSTRUCTIONS
• This test contains 30 MCQ's. For each question only one option is correct. Darken the correct circle/ bubble in the
Response Grid provided on each page.
• You have to evaluate your Response Grids yourself with the help of solutions provided at the end of this book.
• Each correct answer will get you 4 marks and 1 mark shall be deduced for each incorrect answer. No mark will be given/
deducted if no bubble is filled. Keep a timer in front of you and stop immediately at the end of 60 min.
• The sheet follows a particular syllabus. Do not attempt the sheet before you have completed your preparation for that
syllabus.
• After completing the sheet check your answers with the solution booklet and complete the Result Grid. Finally spend time
to analyse your performance and revise the areas which emerge out as weak in your evaluation.

1. Which of the following is incorrect regarding 3. An example of double salt is


spectrochemical series? (a) Bleaching powder
(a) NH3 > H2O (b) F- > C2O 42- (b) K 4 [Fe(CN) 6 ]
(c) NCS– > SCN– (d) en >edta4– (c) Hypo
2. C63H88CoN14O14P is the formulae of the Cyanocobalamine, (d) Potash alum
(vitamin B12) it contain CN– and CN– is very poisonous, 4. Nickel (Z = 28) combines with a uninegative monodentate
ligand X– to form a paramagnetic complex [NiX4]2–. The
than why this compound does not prove to be fatal for
number of unpaired electron(s) in the nickel and geometry
us? (it inhibit the electron transport chain ?
of this complex ion are, respectively :
(a) CN– forms covalent bond
(a) one, square planar
(b) CN– is coordinating to the cobalt as the ligand (b) two, square planar
(c) CN– hydrolysis immediately (c) one, tetrahedral
(d) All of these (d) two, tetrahedral

RESPONSE GRID 1. 2. 3. 4.

Space for Rough Work


EBD_7504
C-90 NTA JEE Main

5. The crystal field splitting energy for octahedral (D0) and 10. Which of the following is organo-metallic compound ?
tetrahedral (Dt) complexes is related as (a) Ti(C2H4)4 (b) Ti(OC2H5)4
1 4 (c) Ti(OCOCH3)4 (d) Ti(OC6H5)4
(a) Dt = – D 0 (b) Dt = – D 0
2 9 11. Which is not p - bonded complex ?
3 2 (a) Zeise’s salt (b) Ferrocene
(c) Dt = – D 0 (d) D t = – D 0
5 5 (c) Dibenzene chromiun (d) Tetraethyl lead
6. A square planar complex is formed by hybridisation of which 12. A solution containing 2.675 g of CoCl3. 6 NH3 (molar mass
atomic orbitals? = 267.5 g mol–1) is passed through a cation exchanger. The
chloride ions obtained in solution were treated with excess
(a) s, px , py , dyz (b) s, p x , p y , d x 2 - y2
of AgNO3 to give 4.78 g of AgCl (molar mass = 143.5 g
(c) s, p x , p y , d (d) s, py , pz , dxy mol–1). The formula of the complex is
z2
(At. mass of Ag = 108 u)
7. Consider the following complex [Co(NH3)5CO3]ClO4. The
coordination number, oxidation number, number of d- (a) [Co(NH3 )6 ]Cl3 (b) [CoCl2 (NH3 )4 ]Cl
electrons and number of unpaired d-electrons on the metal (c) [CoCl3 (NH3 )3 ] (d) [CoCl(NH3 )5 ]Cl2
are respectively
13. The type of isomerism present in Pentaminenitrochromium
(a) 6, 3, 6, 0 (b) 7, 2, 7, 1 (III) chloride is
(c) 7, 1, 6, 4 (d) 6, 2, 7, 3 (a) optical (b) linkage
8. How many EDTA (ethylenediaminetetraacetic acid) (c) ionisation (d) polymerisation.
molecules are required to make an octahedral complex with 14. Red precipitate is obtained when ethanol solution of
a Ca2+ ion? dimethylglyoxime is added to ammoniacal Ni(II). Which of
(a) One (b) Two the following statements is not true ?
(c) Six (d) Three (a) Red complex has a square planar geometry.
9. Which of the following are inner orbital complex (i.e., (b) Complex has symmetrical H-bonding
involving d2sp3 hybridisation) and is paramagnetic in (c) Red complex has a tetrahedral geometry.
nature? (d) Dimethylglyoxime functions as bidentate ligand.
(a) [Mn(CN)6]3– , [Fe(CN)6]3–, [Co(C2O4)3]3–
OH
(b) [MnCl6]3–, [FeF6]3–, [CoF6]3– H3C C N
(c) [Mn(CN)6]3–, [Fe(CN)6]3– dimethylglyoxime =
H3C C N
(d) [MnCl6]3– , [Fe(CN)6]3–, [Co(C2O4)3]3– OH

RESPONSE 5. 6. 7. 8. 9.
GRID 10. 11. 12. 13. 14.
Space for Rough Work
Chemistry C-91

15. An excess of AgNO3 is added to 100 mL of a 0.01 M solution COOH


of dichlorotetraaquachromium (iii) chloride. The number of CH2
(d) HOOC–H2C H
moles of AgCl precipitated would be : N–CH–CH–N
(a) 0.002 (b) 0.003 H CH2–COOH
CH2
(c) 0.01 (d) 0.001
HOOC
16. Magnetic moment of Gd3+ ion (Z = 64) is
21. The structure of which of the following chloro species can
(a) 3.62 BM (b) 9.72 BM be explained on the basis of dsp2 hybridization ?
(c) 7.9 BM (d) 10.60 BM
17. Which of the following species represent the example of (a) PdCl24 - (b) FeCl 24 -
dsp2 - hybridisation ?
(a) [Fe(CN)6]3– (b) [Ni(CN)4]2–
(c) CoCl 24 - (d) NiCl 24 -
(c) [Ag(CN)2] – (d) [Co(CN)6]3– 22. The octahedral complex of a metal ion M3+ with four
18. Which one of the following has largest number of monodentate ligands L1, L2, L3 and L4 absorb wavelengths
isomers ? in the region of red, green, yellow and blue, respectively.
The increasing order of ligand strength of the four ligands
(a) [Ir(PR 3 ) 2 H(CO)]2 + (b) [Co( NH 3 )5 Cl]2+
is:
(c) [Ru ( NH 3 ) 4 Cl 2 ]+ (d) [Co(en ) 2 Cl 2 ]+ (a) L4 < L3 < L2 < L1 (b) L1 < L3 < L2 < L4
19. In Fe(CO)5, the Fe – C bond possesses (c) L3 < L2 < L4 < L1 (d) L1 < L2 < L4 < L3
(a) ionic character (b) s-character only
23. Which one of the following coordination compounds is
(c) p-character (d) both s and p characters used to inhibit the growth of tumours?
20. The correct structure of ethylenediaminetetraacetic acid (a) Trans-platin (b) EDTA complex of calcium
(EDTA) is (c) [(Ph3P)3RhCl] (d) Cis-platin
24. [NiCl2 {P(C2H5)2(C6H5)}2] exhibits temperature dependent
HOOC – H2C CH2– COOH
(a) N – CH = CH – N magnetic behaviour (paramagnetic/diamagnetic). The
HOOC – H2C CH2– COOH coordination geometries of Ni 2+ in the paramagnetic and
diamagnetic states are respectively
HOOC COOH
(b) N – CH2 – CH2 – N (a) tetrahedral and tetrahedral
HOOC COOH
(b) square planar and square planar
HOOC–H2C CH2–COOH (c) tetrahedral and square planar
(c) N – CH2 – CH2 – N
HOOC–H2C CH2–COOH (d) square planar and tetrahedral

RESPONSE 15. 16. 17. 18. 19.


GRID 20. 21. 22. 23. 24.
Space for Rough Work
EBD_7504
C-92 NTA JEE Main

25. Which will give a white precipitate with AgNO3 in aqueous hybridisation involved and the number of unpaired electrons
solution present in the complex are respectively:
(a) [Co(NH3 )5 Cl](NO2 )2 (b) [Pt ( NH 3 ) 6 ]Cl 4 (a) sp3, two (b) dsp2, zero
2
(c) dsp , one (d) sp3, zero
(c) [Pt (en )Cl 2 ] (d) [Cu ( NH 3 ) 4 ]SO 4 28. Which of the following species is not expected to be a
26. Which of the following name formula combinations is not ligand?
correct? (a) NO (b) NH +4
(c) NH 2 CH 2 CH 2 NH 2 (d) Both (a) and (b)
Formula
29. Which of the following organometallic compound is s and
Potassium p -bonded ?
(a) K2[Pt(CN)4]
tetracyanoplatinate (II)
(a) [Fe(h5 - C5 H 5 ) 2 ] (b) [PtCl3 (h2 - C 2 H 4 )]
Pentacyanomagnate (II)
(b) [Mn(CN)5]2–
ion (c) [Co (CO ) 5 NH 3 ]2 + (d) Al(CH 3 )3
Potassium diammine 30. The most stable complex among the following is
(c) K[Cr(NH3)2Cl4]
tetrachlorochromate (III)
(a) K3[Al(C2O4)3] (b) [Pt(en)2]Cl
(d) [Co(NH3)4(H2O)I]SO4 Magnetron value.
(c) [Ag(NH3)2]Cl (d) K2[Ni(EDTA)]
27. Nickel (Z = 28) combines with a uninegative monodentate
ligand to form a diamagnetic complex [NiL4]2– . The

RESPONSE 25. 26. 27. 28. 29.


GRID 30.

CHEMISTRY CHAPTERWISE SPEED TEST-52


Total Questions 30 Total Marks 120
Attempted Correct
Incorrect Net Score
Cut-off Score 36 Qualifying Score 54
Success Gap = Net Score – Qualifying Score
Net Score = (Correct × 4) – (Incorrect × 1)
Space for Rough Work
CHEMISTRY Speed
Haloalkanes and Haloarenes TEST
No. of Questions
30
Maximum Marks
120
Time
1 Hour
53
Chapter-wise

GENERAL INSTRUCTIONS
• This test contains 30 MCQ's. For each question only one option is correct. Darken the correct circle/ bubble in the
Response Grid provided on each page.
• You have to evaluate your Response Grids yourself with the help of solutions provided at the end of this book.
• Each correct answer will get you 4 marks and 1 mark shall be deduced for each incorrect answer. No mark will be given/
deducted if no bubble is filled. Keep a timer in front of you and stop immediately at the end of 60 min.
• The sheet follows a particular syllabus. Do not attempt the sheet before you have completed your preparation for that
syllabus.
• After completing the sheet check your answers with the solution booklet and complete the Result Grid. Finally spend time
to analyse your performance and revise the areas which emerge out as weak in your evaluation.

1. In the preparation of chlorobenzene from aniline, the most 3. Vapour density of an organic compound is 23. It contains
suitable reagent is 52.17% of carbon and 13% of hydrogen. The compound
(a) Chlorine in the presence of ultraviolet light gives iodoform test. The compound is :
(b) Chlorine in the presence of AlCl3 (a) ethanol
(c) Nitrous acid followed by heating with Cu2Cl2 (b) dimethyl ether
(d) HCl and Cu2Cl2 (c) acetone
2. Which reagent cannot be used to prepare an alkyl halide (d) methanol
from an alcohol ? 4. Reaction of trans 2-phenyl-1-bromocyclopentane on
(a) HCl + ZnCl2 reaction with alcoholic KOH produces
(a) 1-phenylcyclopentene
(b) NaCl
(b) 3-phenylcyclopentene
(c) PCl5
(c) 4-phenylcyclopentene
(d) SOCl2
(d) 2-phenylcyclopentene

RESPONSE GRID 1. 2. 3. 4.

Space for Rough Work


EBD_7504
C-94 NTA JEE Main

5. Which of the following is an example of SN2 reaction? 9. Which of the following will have a mesoisomer also?
(a) CH 3 Br + OH - ¾
¾® CH 3OH + Br - (a) 2, 3- Dichloropentane
(b) 2, 3-Dichlorobutane
(b) CH 3 - C H - CH 3 + OH - ¾
¾® CH 3 - C H - CH 3
| | (c) 2-Chlorobutane
Br OH
(d) 2-Hydroxypropanoic acid
(c) -H 2O
CH 3CH 2 OH ¾¾ ¾
¾® CH 2 = CH 2
10. The pesticide DDT slowly changes to
(d) (CH3 )3 C - Br + OH - ¾¾
®(CH3 )3COH + Br - (a) CCl3-CHO and chlorobenzene
6. The reaction of C 6 H 5 N +2 Cl - with CuCl gives (b) p, p'-Dichlorodiphenylethene
(c) p, p'-Dichlorodiphenyldichloroethane
(a) C6 H 5Cl (b) C 6 H 6 (d) p, p'-Dichlorodiphenyldichloroethene
11. o-Methoxybromobenzene is treated with sodamide and then
(c) C 6 H 5 - C 6 H 5 (d) C 6 H 4 Cl 2
with ammonia. The product formed is
7. On sulphonation of C6H5Cl (a) o-Methoxyaniline (b) Aniline
(a) m-Chlorobenzenesulphonic acid is formed (c) Methoxybenzene (d) m-Methoxyaniline
(b) Benzenesulphonic acid is formed 12. The starting substance for the preparation of iodoform is
(c) o-Chlorobenzenesulphonic acid is formed any one of the following, except
(d) o- and p-Chlorobenzenesulphonic acid is formed (a) CH 3CH (OH)CH 3 (b) CH 3CH 2 OH
8. The structure of the major product formed in the following
reaction (c) HCH 2OH (d) CH 3COCH 3
CH2Cl 13. Which of the following statements is wrong?
NaCN
DMF (a) Ethyl chloride on reduction with Zn-Cu couple and
is alcohol gives ethane.
I (b) The reaction of methyl magnesium bromide with
CH2Cl acetone gives butanol-2.
CH2CN
(a) (b) (c) Alkyl halides follow the following reactivity sequence
on reaction with alkenes.
CN I R – I > R – Br > R – Cl > R – I
(d) C2H4Cl2 may exist in two isomeric forms
CH2Cl
14. An organic compound A (C4 H9 Cl) on reaction with
CH2CN
(c) (d) CN Na/diethyl ether gives a hydrocarbon which on
monochlorination gives only one chloro derivative, then A
I is
CN
(a) tert-butyl chloride (b) sec-butyl chloride
(c) isobutyl chloride (d) n-butyl chloride

RESPONSE 5. 6. 7. 8. 9.
GRID 10. 11. 12. 13. 14.

Space for Rough Work


Chemistry C-95

15. If chloroform is left open in air in the presence of sunlight, it 21. Iodoform can be prepared from all except :
gives (a) Ethyl methyl ketone
(a) carbon tetrachloride (b) carbonyl chloride (b) Isopropyl alcohol
(c) mustard gas (d) lewisite (c) 3-Methyl 2-butanone
16. The reaction of toluene with Cl 2 in presence of FeCl3 gives (d) Isobutyl alcohol
22. The reagent(s) for the following conversion,
' X ' and reaction in presence of light gives ‘Y’. Thus, ‘X’ and ?
‘Y’ are : Br H H
Br
(a) X = Benzal chloride, Y = o – Chlorotoluene is/are
(b) X = m – Chlorotoluene, Y = p –Chlorotoluene (a) alcoholic KOH
(c) X = o –and p – Chlorotoluene, (b) alcoholic KOH followed by NaNH2
Y = Trichloromethylbenzene (c) aqueous KOH followed by NaNH2
(d) X = Benzyl chloride, Y = m – Chlorotoluene (d) Zn/CH3OH
17. CH 3 Br + Nu - ¾ ¾® CH 3 - Nu + Br - The decreasing 23. In a nucleophilic substitution reaction:
order of the rate of the above reaction with nucleophiles DMF
(Nu–) A to D is R – Br + Cl - ¾¾¾® R – Cl + Br - ,
[Nu– = (A) PhO–, (B) AcO–, (C) HO–, (D) CH3O–] which one of the following undergoes complete inversion
(a) A > B > C > D (b) B > D > C > A of configuration?
(c) D > C > A > B (d) D > C > B > A (a) C6H5CHC6H5Br (b) C6H5CH2Br
18. The Wurtz-Fittig reaction involves condensation of : (c) C6H5CHCH3Br (d) C6H5CCH3C6H5Br
(a) two molecules of aryl halides 24. Chlorobenzne reacts with trichloro acetaldehyde in the
(b) one molecule of each of aryl-halide and alkyl-halide. presence of H2SO4.
(c) one molecule of each of aryl-halide and phenol. O
(d) two molecules of aralkyl-halides. H2SO4
19. The major organic compound formed by the reaction of Cl + H – C – CCl3
1, 1, 1-trichloroethane with silver powder is: 2
(a) Acetylene (b) Ethene
(c) 2 - Butyne (d) 2 - Butene The major product formed is:
20. Which of the following pairs of compounds are enantiomers? Cl
CH3 CH3
HO H HO H (a) Cl C Cl
(a) and
H OH HO H
CH3 CH3 Cl
CH3 CH3
Cl
(b) H OH HO H (b) Cl C
and Cl
HO H H HO
CH3 CH3 CH2Cl
CH3 CH3
(c) H OH HO H (c) Cl CH Cl
and
HO H HO H
CH3 CH3 CCl3
CH3 CH3
H OH H OH (d) Cl CH Cl
(d) and
HO H H OH
CH3 CH3 Cl

RESPONSE 15. 16. 17. 18. 19.


GRID 20. 21. 22. 23. 24.

Space for Rough Work


EBD_7504
C-96 NTA JEE Main

25. Benzene reacts with n-propyl chloride in the presence of 28. What is the product of the following reaction ?
anhydrous AlCl3 to give
Cl
(a) 3 – Propyl – 1 – chlorobenzene
(b) n-Propylbenzene ( CH ) NLi
(c) No reaction ¾¾ ¾
3 ¾
2 ¾®
?
(CH 3 ) 2 NH
(d) Isopropylbenzene
26. Replacement of Cl of chlorobenzene to give phenol requires (a) N, N-dimethyl aniline
drastic conditions but chlorine of 2, 4-dinitrochlorobenzene (b) phenyl lithium (C 6 H 5 Li)
is readily replaced. This is because
(c) para chloro-N, N-dimethyl aniline
(a) NO2 makes the ring electron rich at ortho and para
positions (d) meta chloro-N, N-dimethyl aniline
(b) NO2 withdraws e – from meta-position 29. The major product formed when 1, 1, 1-trichloropropane is
(c) NO2 donates e– at m-position treated with aqueous potassium hydroxide is:
(a) Propyne (b) 1-Propanol
(d) NO2 withdraws e– from ortho/para-positions
(c) 2-Propanol (d) Propionic acid
27. Silver benzoate reacts with bromine to form
30. Consider the reactions :
COOBr (i) C H OH
(CH 3 ) 2 CH - CH 2 Br ¾¾2 ¾¾¾
5 ®
(CH 3 )2 CH - CH 2OC 2H 5 + HBr
(a) (b)
C H O-
(ii) 2 5
(CH 3 ) 2 CH - CH 2 Br ¾¾¾¾¾®
(CH 3 )2 CH - CH 2OC 2 H5 + Br -
COOAg
The mechanisms of reactions (i) and (ii) are respectively :
(a) SN1 and SN2 (b) SN1 and SN1
(c) (d) C 6 H 5Br (c) SN2 and SN2 (d) SN2 and SN1
Br

RESPONSE 25. 26. 27. 28. 29.


GRID 30.

CHEMISTRY CHAPTERWISE SPEED TEST-53


Total Questions 30 Total Marks 120
Attempted Correct
Incorrect Net Score
Cut-off Score 37 Qualifying Score 54
Success Gap = Net Score – Qualifying Score
Net Score = (Correct × 4) – (Incorrect × 1)
Space for Rough Work
CHEMISTRY Speed
Alcohols, Phenols and Ethers TEST
No. of Questions
30
Maximum Marks
120 1 Hour
Time 54
Chapter-wise

GENERAL INSTRUCTIONS
• This test contains 30 MCQ's. For each question only one option is correct. Darken the correct circle/ bubble in the
Response Grid provided on each page.
• You have to evaluate your Response Grids yourself with the help of solutions provided at the end of this book.
• Each correct answer will get you 4 marks and 1 mark shall be deduced for each incorrect answer. No mark will be given/
deducted if no bubble is filled. Keep a timer in front of you and stop immediately at the end of 60 min.
• The sheet follows a particular syllabus. Do not attempt the sheet before you have completed your preparation for that
syllabus.
• After completing the sheet check your answers with the solution booklet and complete the Result Grid. Finally spend time
to analyse your performance and revise the areas which emerge out as weak in your evaluation.

1. Lucas reagent is (c) CH 3 - CH = CH - OH


(a) Conc. HCl and anhydrous ZnCl2
(b) Conc. HNO3 and hydrous ZnCl2 (d) CH 3 - C(CH 2 OH) = CH 2
(c) Conc. HCl and hydrous ZnCl2 4. Among the following compounds which can be dehydrated
(d) Conc. HNO3 and anhydrous ZnCl2 very easily is
2. Diethyl ether reacts, inspite of its usual inert nature, with : CH3
|
(a) Dilute suphuric acid (a) CH3 CH 2 C CH 2 CH3
(b) Dilute sodium hydroxide |
OH
(c) Boron trifluoride
(d) Metallic sodium OH
|
3. Vinyl carbinol is (b) CH3CH 2 CH 2CHCH3
(a) HO - CH 2 - CH = CH 2
(c) CH 3 CH 2 CH 2 CH 2 CH 2 OH
(b) CH 3C(OH ) = CH 2
(d) CH3 CH 2 CHCH 2 CH 2 OH
|
CH3

RESPONSE GRID 1. 2. 3. 4.

Space for Rough Work


EBD_7504
C-98 NTA JEE Main

5. CH3CH2OH can be converted into CH3CHO by ________ 10. Which of the following cannot be made by using
(a) catalytic hydrogenation Williamson’s synthesis?
(b) treatment with LiAlH4 (a) Methoxybenzene
(c) treatment with pyridinium chlorochromate (b) Benzyl p-nitrophenyl ether
(d) treatment with KMnO4 (c) Methyl tertiary butyl ether
6. Epichlorohydrin is (d) Di-tert-butyl ether
(a) 3-Chloropropane 11. Rectified spirit is a mixture of
(b) 3-Chloropropan-1-ol (a) 95% ethyl alcohol + 5% water
(c) 3-Chloro-1, 2-epoxypropane (b) 94% ethyl alcohol + 4.53 water
(d) None of these (c) 94.4% ethyl alcohol + 5.43% water
7. 2-Phenylethanol may be prepared by the reaction of (d) 95.87% ethyl alcohol + 4.13% water
phenylmagnesium bromide with 12. Absolute alcohol (100% alcohol) is prepared by distilling
(a) HCHO rectified spirit over
(b) CH3CHO (a) Na
(c) CH3COCH3 (b) CaCl2
(c) Mg
O
(d) (d) Mg(OC2H5)2
13. Reagent used to convert allyl alcohol to acrolein is
8. HBr reacts with CH2 = CH – OCH3 under anhydrous (a) MnO2
conditions at room temperature to give (b) H2O2
(a) BrCH2 – CH2 – OCH3 (c) OsO4
(b) H3C – CHBr – OCH3 (d) KMnO4
(c) CH3CHO and CH3Br 14. In Williamson synthesis of mixed ether having a primary
(d) BrCH2CHO and CH3OH and a tertiary alkyl group if tertiary halide is used,
9. From amongst the following alcohols the one that would then :
react fastest with conc. HCl and anhydrous ZnCl2, is (a) Rate of reaction will be slow due to slow cleavage of
(a) 2-Butanol carbon-halogen bond.
(b) 2- Methylpropan-2-ol (b) Alkene will be the main product.
(c) 2-Methylpropanol (c) Simple ether will form instead of mixed ether.
(d) 1- Butanol (d) Expected mixed ether will be formed.

RESPONSE 5. 6. 7. 8. 9.
GRID 10. 11. 12. 13. 14.

Space for Rough Work


Chemistry C-99

15. Arrange the following in increasing order of their acidity? 20. Williamson’s synthesis is used to prepare
o–cresol(a), salicyclic acid(b), phenol(c) (a) acetone (b) diethyl ether
(a) c < a < b (b) b < c < a (c) P.V.C. (d) bakelite
(c) a < b < a (d) a < c < b 21. Aspirin is an acetylation product of
16. Zerevitinov’s determination of active hydrogen in a (a) p-Dihydroxybenzene
compound is based upon its reaction with (b) o-Hydroxybenzoic acid
(a) Na (b) CH3Mgl (c) o-Dihydroxybenzene
(c) Zn (d) Al (d) m-Hydroxybenzoic acid
17. When wine is put in air, it becomes sour due to
HNO Phenol
(a) bacteria 22. o - Xylene ¾¾ ¾
¾3 ® X ¾¾ ¾
¾® Y. The product Y is
H 2SO 4
(b) oxidation of C2H5OH to CH3COOH
(c) virus (a) Phthalic acid
(d) formic acid formation (b) Isophthalic acid
18. Osmium tetraoxide is a reagent used for (c) Phenolphthalein
(a) hydroxylation of acetylenes (d) o-Hydroxysulphonic acid
(b) hydroxylation of olefins to give cis-diols 23. The following reaction
(c) hydroxylation of olefins to form trans- diols OH
(d) hydroxylation of carbonyl compounds
Anhyd. OH
19. Sodium phenoxide when heated with CO2 under pressure at
+ HCl + HCN
125ºC yields a product which on acetylation produces C ZnCl2
CHO
ONa +
125°
+ CO 2 ¾¾¾¾
® B ¾¾¾® C
H is known as:
5 Atm Ac2O (a) Perkin reaction
The major product C would be (b) Gatterman-Koch Formylation
(c) Kolbe’s reaction
OCOCH3 OH
(d) Gattermann reaction
COOH COCH3 24. Denaturation of alcohol is the
(a) (b) (a) mixing of CuSO4 (a foul smelling solid) and pyridine
(to give the colour) to make the commercial alcohol
COCH3 unfit for drinking
(b) mixing of CuSO4 (to give the colour) and pyridine (a
OH OCOCH3 foul smelling solid) to make the commercial alcohol unfit
for drinking
COOCH3 (c) mixing of Cu(OAc)2 and ammonia to make the
(c) (d)
COOH commercial alcohol unfit for drinking
(d) mixing of Cu(OAc) 2 and pyridine to make the
commercial alcohol unfit for drinking

RESPONSE 15. 16. 17. 18. 19.


GRID 20. 21. 22. 23. 24.

Space for Rough Work


EBD_7504
C-100 NTA JEE Main

25. Formation of which compound given below from 27. Which one of the following substituents at para-
1 - butanol needs an oxidising agent? position is most effective in stabilizing the phenoxide
(a) CH 3CH 2 CH 2 CH 2 Br

O
(b) CH 3CH 2CH 2CH = O ion?

(c) (CH 3CH 2 CH 2 CH 2 ) 2 O (a) – CH3 (b) – OCH3


(d) CH 3 - CH 2CH = CH 2 (c) – COCH3 (d) – CH2OH
26. What is X in the following reaction ? 28. Williamson synthesis of ether is an example of:
(a) Nucleophilic addition
OH OCH3 (b) Electrophilic addition
O | |
X (c) Electrophilic substitution
CH 2 C - CH 3 ¾¾® H 2C - C - CH3
| | (d) Nucleophilic substitution
CH 3 CH3 29. Widespread deaths due to liquor poisoning occurs due to
(a) presence of carbonic acid in liquor
(a) CH 3OH, H 2SO 4 (b) presence of ethyl alcohol in liquor
+
(c) presence of methyl alcohol in liquor
(b) CH OH, CH O- Na (d) presence of lead compounds in liquor
3 3
30. Which of the following diols would cleave into two fragments
(c) H 2 O / H 2SO 4 followed by CH 3OH
with HIO 4
(d) CH 3 MgBr / ether followed by H 3 O +
(a) 1, 3-hexanediol (b) 2, 4-hexanediol
(c) 1, 6-hexanediol (d) 3, 4-hexanediol

RESPONSE 25. 26. 27. 28. 29.


GRID 30.

CHEMISTRY CHAPTERWISE SPEED TEST-54


Total Questions 30 Total Marks 120
Attempted Correct
Incorrect Net Score
Cut-off Score 36 Qualifying Score 52
Success Gap = Net Score – Qualifying Score
Net Score = (Correct × 4) – (Incorrect × 1)
Space for Rough Work
CHEMISTRY Speed
Aldehydes, Ketones TEST
and Carboxylic acids
No. of Questions
30
Maximum Marks
120 1 Hour
Time 55
Chapter-wise

GENERAL INSTRUCTIONS
• This test contains 30 MCQ's. For each question only one option is correct. Darken the correct circle/ bubble in the
Response Grid provided on each page.
• You have to evaluate your Response Grids yourself with the help of solutions provided at the end of this book.
• Each correct answer will get you 4 marks and 1 mark shall be deduced for each incorrect answer. No mark will be given/
deducted if no bubble is filled. Keep a timer in front of you and stop immediately at the end of 60 min.
• The sheet follows a particular syllabus. Do not attempt the sheet before you have completed your preparation for that
syllabus.
• After completing the sheet check your answers with the solution booklet and complete the Result Grid. Finally spend time
to analyse your performance and revise the areas which emerge out as weak in your evaluation.

1. IUPAC name of following will be (a) CH 3COCH 3 (b) CCl 3CH 2 CHO
CHO
(c) CH 3CH 2CH 2OH (d) CH 3CH 2CHO
CH3
3. Acetone oxime is obtained by reacting acetone with
(a) NH3 (b) NH2OH
(c) NH2Na (d) NH2.NH2
OH 4. Which alkene on ozonolysis gives CH3 CH2 CHO and
(a) 4-formyl 3-methyl 1-hydroxy benzene CH3CCH3
(b) 4-formyl 3-methyl phenol ||
(c) 4-hydroxy 2-methyl benzaldehyde O
CH3
(d) 4-hydroxy 2-methyl carbaldehyde (a) CH3CH2CH = C (b) CH3CH2CH = CHCH2CH3
CH3
2. In which of the following, the number of carbon atoms does
(c) CH3CH2CH = CH CH3 (d) CH 3 - C = CHCH 3
not remain same when carboxylic acid is obtained by |
oxidation CH3

RESPONSE GRID 1. 2. 3. 4.

Space for Rough Work


EBD_7504
C-102 NTA JEE Main

5. Predict the product in the given reaction. 10. In a set of the given reactions, acetic acid yielded a product C.
CHO CH 3COOH + PCl5 ¾¾
® A ¾¾¾®
6 6 B
C H
Anh.AlCl3
50 % KOH C 2 H5MgBr
¾¾¾¾¾¾ ®C
Ether
Cl Product C would be -

CH2OH CH2COO C2 H 5
(a) +
|
(a) CH 3 - C (OH)C 6 H 5 (b) CH 3 CH(OH)C 2 H 5
Cl Cl
(c) CH 3COC 6 H 5 (d) CH 3 CH(OH)C6 H 5
CH2OH OH
(b) + 11. An organic compound ‘A’ on treatment with NH3 gives ‘B’
which on heating gives ‘C’, ‘C’ when treated with Br2 in the
presence of KOH produces ethylamine. Compound ‘A’ is:
OH OH (a) CH3COOH (b) CH3 CH2 CH2 COOH

CH2OH COO (c) CH3 – CHCOOH (d) CH3CH2COOH
(c) +
CH3
Cl Cl 12. Match th e compounds given in List-I with
List-II and select the suitable option using the code given
CH2OH COO– below :
(d) + List I List-II
(A) Benzaldehyde (i) Phenolphthalein
OH OH (B) Phthalic anhydride (ii) Benzoin condensation
6. The carboxyl functional group (– COOH) is present in (C) Phenyl benzoate (iii) Oil of wintergreen
(a) picric acid (b) barbituric acid (D) Methyl salicylate (iv) Fries rearrangement
(c) ascorbic acid (d) aspirin Code :
7. Which one of the following is reduced with zinc and (A) (B) (C) (D)
hydrochloric acid to give the corresponding hydrocarbon? (a) (iv) (i) (iii) (ii)
(a) Acetamide (b) Acetic acid (b) (iv) (ii) (iii) (i)
(c) (ii) (iii) (iv) (i)
(c) Ethyl acetate (d) Butan-2-one (d) (ii) (i) (iv) (iii)
8. Carboxylic acid group does not give the usual addition and X
elimination reactions of aldehydes and ketones because 13. C6H5CH=CHCHO ¾¾® C6H5CH=CHCH2OH
In the above sequence X can be :
(a) O–H bond is more polar than C = O group (a) H2/Ni (b) NaBH4
(b) carboxylate ion gets ionised (c) K2Cr2O7/H+ (d) Both (a) and (b)
(c) carboxylate ion gets stabilised by resonance 14. Acetaldehyde reacts with semicarbazide and forms
(d) it exists as – COOH and there is no carbonyl group semicarbazone. Its structure is
9. Phenylmethyl ketone can be converted into ethylbenzene (a) CH3CH = NNHCON = CHCH3
in one step by which of the following reagents? (b) CH3CH = NNHCONH2
(a) LiAlH4 (b) Zn-Hg/HCl (c) CH 3 CH = N — N — CONH 2
|
(c) NaBH4 (d) CH3MgI OH
(d) CH3CH = N—CONHNH2

RESPONSE 5. 6. 7. 8. 9.
GRID 10. 11. 12. 13. 14.
Space for Rough Work
Chemistry C-103

15. Which one of the following can be oxidised to the corre- (a) Oxalic acid and H2
sponding carbonyl compund? (b) Sodium oxalate and H2
(a) 2-hydroxy-propane (c) CO2 and NaOH
(b) Ortho-nitro-phenol (d) Sodium oxalate
(c) Phenol 21. 5 - methyl -2 -hexanone can be synthesised from acetoacetic
(d) 2-methyl-2 hydroxy-propane ester and RX. Which of the following RX is used ?
16. Benzoic acid may be converted to ethyl benzoate by reaction (a) (CH3 ) 2CHBr
with :
(a) Sodium ethoxide (b) (CH3 )2 CHCH 2 Br
(b) Ethyl chloride (c) CH3CH 2CHBrCH3
(c) Dry HCl—C2H5OH
(d) (CH3 )2 CHCH 2CH 2Br
(d) Ethanol
17. Heating mixture of sodium benzoate and soda-lime gives 22. The following tetrahedral intermediate breaks down to
(a) benzene (b) methane OH
(c) sodium phenoxide (d) calcium benzoate |
18. Which of the following compounds is most reactive towards CH 3 - CH 2 - C - Cl
nucleophilic addition reactions? |
OCH 3
O
|| (a) propanal and HCl
(a) CH3 – C – H (b) methyl propanoate and HCl
O (c) propanoic acid and CH 3Cl
||
(b) CH3 – C– CH3 (d) propyl chloride and CH 3OH
O 23. The correct order of increasing acid strength of the
(c) C–H compounds
(A) CH3CO2H (B) MeOCH2CO2H
O
Me
(d) C–CH3 (C) CF3CO2H (D) CO2H is
Me
(a) D < A < B < C (b) A < D < B < C
19. The increasing order of the rate of HCN addition to
compound A – D is (c) B < D < A < C (d) D < A < C < B
24. The compound that neither forms semicarbazone nor oxime
(A) HCHO (B) CH3COCH3
is
(C) PhCOCH3 (D) PhCOPh (a) HCHO
(a) D < C < B < A (b) C < D < B < A (b) CH3COCH2Cl
(c) A < B < C < D (d) D < B < C < A (c) CH3CHO
20. Sodium formate on heating yields (d) CH3CONHCH3

RESPONSE 15. 16. 17. 18. 19.


GRID 20. 21. 22. 23. 24.
Space for Rough Work
EBD_7504
C-104 NTA JEE Main

25. Which of the following cannot reduce Fehling solution? (a) acetoacetic ester
(a) Formic acid (b) Acetic acid (b) methyl acetoacetate
(c) Formaldehyde (d) Acetaldehyde (c) ethyl propionate
26. Pinacolone is (d) ethyl butyrate
(a) 2, 3-Dimethyl-2, 3-butanediol 29. The reagent which can be used to distinguish acetophenone
(b) 3,3-Dimethyl-2-butanone from benzophenone is
(c) 1-Phenyl-2-propanone (a) 2,4- dinitrophenylhydrazine
(d) 1, 1-Diphenyl-1, 2-ethandiol (b) aqueous solution of NaHSO3
27. Conversion of acetaldehyde into ethyl acetate in presence (c) benedict reagent
of aluminium ethoxide is called (d) I2and Na2CO3
(a) Aldol condensation (b) Cope reaction 30. The compound formed when malonic ester is heated with
(c) Tischenko reaction (d) Benzoin condensation urea is
28. Self condensation of two moles of ethyl acetate in presence (a) Cinnamic acid (b) Butyric acid
of sodium ethoxide yields (c) Barbituric acid (d) Crotonic acid.

RESPONSE 25. 26. 27. 28. 29.


GRID 30.

CHEMISTRY CHAPTERWISE SPEED TEST-55


Total Questions 30 Total Marks 120
Attempted Correct
Incorrect Net Score
Cut-off Score 35 Qualifying Score 51
Success Gap = Net Score – Qualifying Score
Net Score = (Correct × 4) – (Incorrect × 1)

Space for Rough Work


CHEMISTRY Speed
Amines TEST
No. of Questions
30
Maximum Marks
120
Time
1 Hour
56
Chapter-wise

GENERAL INSTRUCTIONS
• This test contains 30 MCQ's. For each question only one option is correct. Darken the correct circle/ bubble in the
Response Grid provided on each page.
• You have to evaluate your Response Grids yourself with the help of solutions provided at the end of this book.
• Each correct answer will get you 4 marks and 1 mark shall be deduced for each incorrect answer. No mark will be given/
deducted if no bubble is filled. Keep a timer in front of you and stop immediately at the end of 60 min.
• The sheet follows a particular syllabus. Do not attempt the sheet before you have completed your preparation for that
syllabus.
• After completing the sheet check your answers with the solution booklet and complete the Result Grid. Finally spend time
to analyse your performance and revise the areas which emerge out as weak in your evaluation.

1. An organic amino compound reacts with aqueous nitrous (ii) In Gattermann reaction nucleophiles are introduced in
acid at low temperature to produce an oily nitrosoamine. benzene ring in the presence of copper powder and
The compound is HCl.
(a) CH3NH2 (b) CH3CH2NH2 (iii) The yield in Gattermann reaction is found to be better
(c) CH3CH2NHCH2CH3 (d) (CH3CH2)3N than Sandmayer reaction.
2. Nitrobenzene can be prepared from benzene by using a (a) (i) and (ii) (b) (i), (ii) and (iii)
mixture of conc. HNO3 and conc. H2SO4 in the mixture, (c) (ii) and (iii) (d) (i) and (iii)
nitric acid acts as a/an: 4. Consider the following sequence of reactions :
(a) acid (b) base
Compound[A] ¾¾¾¾
Reduction
® [B] ¾¾¾
HNO 2
® CH 3 CH 2 OH
(c) catalyst (d) reducing agent
3. Which of the following statements are correct ? The compound [A] is
(i) In Sandmeyer reaction nucleophiles like Cl–, Br– and (a) CH3CH2CN (b) CH3NO2
CN– are indroduced in benzene ring in the presence of (c) CH3NC (d) CH3CN
Cu+ ion

RESPONSE GRID 1. 2. 3. 4.

Space for Rough Work


EBD_7504
C-106 NTA JEE Main

5. The correct order of basicity of the following compounds NaCN Ni/ H2


11. CH3CH2Cl ¾¾ ¾
¾®X ¾¾¾ ¾®Y ¾acetic
¾¾¾¾¾
anhydride
¾® Z
NH 2 NH 2 NH 2 Z in the above reacting sequence is
(a) CH 3 CH 2 CH 2 NHCOCH 3
(b) CH 3 CH 2 CH 2 NH 2
A B C (c) CH 3CH 2 CH 2 CONHCH 3
NO2 (d) CH 3 CH 2 CH 2 CONHCOCH 3
(a) B > A > C (b) A > B > C 12. Match the compounds given in List - I with their
characteristic reactions given in List - II. Select the correct
(c) C > A > B (d) C > B > A option.
6. The consituent of the powerful explosive RDX is formed List - I (Compounds) List - II (Reactions)
during the nitration of (1) CH3CH2CH2CH2NH2 (i) alkaline hydrolysis
(a) toluene (b) phenol (2) CH3C º CH (ii) with KOH (alcohol) and CHCl3
(c) glycerol (d) urotropine produces bad smell
7. C 6 H 5C º N an d C 6 H 5 N ®
= C exhibit which type of (3) CH3CH2COOCH3 (iii) gives white ppt. with ammonical
AgNO3
isomerism?
(4) CH3CH(OH)CH3 (iv) with Lucas reagent cloudiness
(a) Position (b) Functional appears after 5 minutes
(c) Dextroisomerism (d) Position isomerism Options :
8. Which of the following r eagents will convert (1) (2) (3) (4)
p-methylbenzenediazonium chloride into p-cresol? (a) (iv) (ii) (iii) (i)
(a) Cu powder (b) H2O (b) (ii) (i) (iv) (iii)
(c) H3PO2 (d) C6H5OH (c) (iii) (ii) (i) (iv)
(d) (ii) (iii) (i) (iv)
9. Replacement of - N 2+ Cl - from benzene diazonium chloride 13. Nitrosoamines (R2N – N = O) are insoluble in water. On
by iodine can be done by using heating with conc. H2SO4, they give secondary amines. The
reaction is called
(a) HI (b) NaOI
(a) Liebermann nitroso reaction
(c) PI3 (d) KI (b) Etard reaction
10. Which of the following compounds is most basic? (c) Fries reaction
(d) Perkin reaction
(a) O2N NH2 (b) CH2NH2 14. All three amines 1°, 2°, 3° react with
1. H2O 2. R–X
(c) N – COCH3 (d) NH2 3. HCl 4. (CH3CO)2O
(a) 1, 2 (b) 4 only
H
(c) 1, 2, 4 (d) 1, 2, 3

RESPONSE 5. 6. 7. 8. 9.
GRID 10. 11. 12. 13. 14.

Space for Rough Work


Chemistry C-107

15. Which one of the following on reduction with lithium 20. The reduction of nitro compounds is most preferred in the
aluminium hydride yields a secondary amine? presence of
(a) Methyl isocyanide (a) Pd/H2 in ethanol (b) Sn + HCl
(b) Acetamide (c) finely divided Ni (d) iron scrap and HCl
(c) Methyl cyanide 21. Ethyl isocyanide on hydrolysis in acidic medium generates
(a) propanoic acid and ammonium salt
(d) Nitroethane (b) ethanoic acid and ammonium salt
16. Aniline reacts with phosgene and KOH to form (c) methylamine salt and ethanoic acid
OH O (d) ethylamine salt and methanoic acid
22. Acetamide is treated with the following reagents separately.
C Cl
(a) (b) Which one of these would yield methylamine?
(a) NaOH – Br2 (b) Sodalime
(c) Hot conc. H2SO4 (d) PCl5
NHCOCl NCO 23. Match the compounds in List I with their nature from List II,
(c) (d) as seen in aqueous medium
List I List II
17. When an iline reacts with oil of bitter almonds I. Acetamide A. Acidic
(C 6 H 5CHO ) condensation takes place and benzal II. Benzonitrile B. Basic
III. Triethylamine C. Neutral
derivative is formed. This is known as
IV. Phenol
(a) Million's base (b) Schiff's reagent
(a) I–C; II–C; III–B; IV–A (b) I–B; II–C; III–C; IV–A
(c) Schiff's base (d) Benedict's reagent
(c) I–C; II–B; III–B; IV–C (d) I–A; II–A; III–C; IV–B
18. Which reaction sequence would be best to prepare
3-chloroanilne from benzene ? 24. The correct order of basicities of the following compounds
is
(a) Chlorination, nitration, reduction
(b) Nitration, chlorination, reduction NH
(c) Nitration, reduction, chlorination CH3 C CH3 CH2 NH2
(d) Nitration, reduction, acetylation, chlorination, NH2
hydrolysis 1 2
19. A nitro alkane reacts with HONO to give insoluble product
in alkali which turns blue on treatment with an alkali. The O
nitro alkane is (CH3)2NH CH3 C NH2
(a) CH 3 CH 2 NO 2 (b) CH3 - CH - CH 2 NO 2
| 3 4
CH3
(a) 2 > 1 > 3 > 4 (b) 1 > 3 > 2 > 4
(c) (CH 3 ) 2 CHNO 2 (d) (CH 3 )3 CNO 2 (c) 3 > 1 > 2 > 4 (d) 1 > 2 > 3 > 4

RESPONSE 15. 16. 17. 18. 19.


GRID 20. 21. 22. 23. 24.

Space for Rough Work


EBD_7504
C-108 NTA JEE Main

25. Which of the following reactions will not give a primary (a) CH 3 CH 2 Br and NH 3
amine?
(a) CH 3 CONH 2 ¾Br / KOH
¾2¾ ¾ ¾® (b) CH 3CH 2 NH 2 and H 2 / Pt
(b) LiAlH
CH 3 CN ¾¾ ¾¾
4® (c) CH 3 CH = O and NH 3
(c) LiAlH 4
CH 3 NC ¾¾ ¾¾® (d) LiAlH4 followed by H 2 O and then CH 3 CH 2 Br
(d) CH 3CONH 2 ¾¾ ¾¾®LiAlH 4 29. The intermediate obtained in the following reaction
26 Indicate which nitrogen compound amongst the following O
would undergo Hoffmann reaction (i.e.., reaction with Br2 || NaN 3
and strong KOH) to furnish the primary amine (R – NH2) R - C - Cl ¾¾ ¾ ® RNH 2
D
(a) RCONHCH 3 (b) RCOONH 4
O
(c) RCONH2 (d) R – CO – NHOH || - +

:
: :
HgCl (a) R - C - N - N º N (b) R – N = C = O
27. In the reaction C 6 H 5 NH 2 + CS2 ¾¾ ¾
¾2®
the product
D
(c) Both (a) and (b) (d) None of these
obtained is
30. Which of the following gives propylamine upon hydrolysis?
(a) phenyl isocyanide (b) phenyl cyanide
(c) p-amino benzene (d) phenyl isothiocyanate (a) CH 3CH 2 C º N
sulphonic acid
(b) (CH 3CH 2 CH 2 ) 2 NH
28. Reagents capable of converting cyclohexanone to
N-ethyl cyclohexylamine is O
O NHCH 2 CH 3 ||
(c) CH3 - C - NH.CH 2 CH 2 CH3
?
¾¾¾® (d) CH 3CH 2 - CH = NH

RESPONSE 25. 26. 27. 28. 29.


GRID 30.

CHEMISTRY CHAPTERWISE SPEED TEST-56


Total Questions 30 Total Marks 120
Attempted Correct
Incorrect Net Score
Cut-off Score 36 Qualifying Score 52
Success Gap = Net Score – Qualifying Score
Net Score = (Correct × 4) – (Incorrect × 1)
Space for Rough Work
CHEMISTRY Speed
Biomolecules TEST
No. of Questions
30
Maximum Marks
120
Time
1 Hour
57
Chapter-wise

GENERAL INSTRUCTIONS
• This test contains 30 MCQ's. For each question only one option is correct. Darken the correct circle/ bubble in the
Response Grid provided on each page.
• You have to evaluate your Response Grids yourself with the help of solutions provided at the end of this book.
• Each correct answer will get you 4 marks and 1 mark shall be deduced for each incorrect answer. No mark will be given/
deducted if no bubble is filled. Keep a timer in front of you and stop immediately at the end of 60 min.
• The sheet follows a particular syllabus. Do not attempt the sheet before you have completed your preparation for that
syllabus.
• After completing the sheet check your answers with the solution booklet and complete the Result Grid. Finally spend time
to analyse your performance and revise the areas which emerge out as weak in your evaluation.

1. Three cyclic structures of monosaccharides are given below 2. Which of the following statements about vitamin B12 is
which of these are anomers incorrect ?
(a) It has a cobalt atom
HO H (b) It also occurs in plants
H OH HO H
HO H (c) It is also present in rain water
H OH H OH
O (d) It is needed for human body in very small amounts
O HO O HO H
HO H H
H OH 3. Proteins when heated with conc. HNO3 give a yellow colour.
H OH H OH This is
HO H
H H
H (a) Oxidizing test (b) Xanthoproteic test
CH2OH CH2OH CH2OH
(I) (II) (III) (c) Hoppe’s test (d) Acid base test
4. The change in the optical rotation (with time) of freshly
(a) I and II (b) II and III prepared solution of sugar is known as
(c) I and III (d) III is anomer of I and II (a) Specific rotation (b) Inversion
(c) Rotatory motion (d) Mutarotation

RESPONSE GRID 1. 2. 3. 4.
Space for Rough Work
EBD_7504
C-110 NTA JEE Main

5. Table sugar is a 10. The number of amino acids found in proteins that a human
(a) disaccharide of d-glucose and d-fructose body can synthesize is
(b) a monosaccharide (a) 20
(c) a disaccharide containing two glucose units (b) 10
(d) D-glucose (c) 5
6. The pair of compounds in which both the compounds give (d) 14
positive test with Tollen’s reagent is 11. Chemically amylose is a _________ with 200–1000
(a) Glucose and Sucrose a-D-(+)-glucose units held by ______ glycosidic linkage
(b) Fructose and Sucrose (a) long unbranched chain, C1– C6.
(c) Acetophenone and Hexanal (b) branched chain, C1 – C4.
(d) Glucose and Fructose (c) long unbranched chain, C1– C4.
7. Hydrolysis of sucrose is called (d) branched chain, C1– C6
(a) hydration 12. Which is wrongly matched?
(b) saponification (a) Insulin - steroid hormone
(c) esterification (b) Estrone - control the uterine cycle in women
(d) inversion (c) Oxytocin - contraction of uterus
8. Which is not a true statement? (d) Pot. metabisulphite - food preservative
(a) a- carbon of a-amino acid is asymmetric 13. If one strand of DNA has the sequence ATGCTTGA, the
sequence in the complimentary strand would be
(b) all proteins are found in L-form
(a) TACGAACT
(c) human body can synthesize all proteins they need
(b) TCCGAACT
(d) at pH=7 both amino acids and carboxylic groups exist
in the ionised form (c) TACGTACT
9. The best solvent for removing butter stain from cloth is (d) TACGTAGT
(a) CHCl3 14. In nucleic acids, the sequence is
(b) C2H5OH (a) phosphate - base - sugar
(c) C2H5OC2H5 (b) sugar- base-phosphate
(d) H2O (c) base- sugar - phosphate
(d) base- phosphate - sugar

RESPONSE 5. 6. 7. 8. 9.
GRID 10. 11. 12. 13. 14.

Space for Rough Work


Chemistry C-111

15. Which statement is incorrect about peptide bond? (a) a ketohexose


(a) C–N bond length in proteins is longer than usual C–N (b) an aldohexose
bond length (c) an a-furanose
(b) Spectroscopic analysis shows planar structure of (d) an a-pyranose
- C - NH - bond 20. A reagent suitable for the determination of N-terminal
|| residue of a peptide is
O
(a) p-Toluenesulphonyl chloride
(c) C–N bond length in proteins is smaller than usual C–N (b) 2, 4-Dinitrophenylhydrazine
bond length (c) Carboxypeptidase
(d) None of these (d) 2, 4-Dinitrofluorobenzene
16. An acidic amino acid among the following is 21. Amino acids generally exist in the form of Zwitter ions. This
(a) glycine (b) valine means they contain
(c) proline (d) leucine (a) basic—NH2 group and acidic —COOH group
+
17. In an amino acid, the carboxyl group ionises at pK a 1 =2.34 (b) the basic— NH3 group and acidic —COO– group
and ammonium ion at pK a 2 =9.60. The isoelectric point of (c) basic—NH2 and acidic —H+ group
+
the amino acid is at pH (d) basic–COO– group and acidic — N H 3 group
(a) 5.97 (b) 2.34 22. The two functional groups present in a typical carbohydrate
(c) 9.60 (d) 6.97 are:
18. For osazone formation, the effective structural unit necessary is (a) – CHO and – COOH
(a) CH 2OCH3 (b) CH 2OH (b) > C = O and – OH
| |
(c) – OH and – CHO
CO CO
| | (d) – OH and – COOH
23. Enzymes are essential as biocatalysts. They function in
(c) CH 2OH (d) CHO (a) aqueous medium, temp = 30–35ºc; pH=7
| |
(b) organic medium
CHOCH 3 CHOCH 3
| | (c) aqueous medium under extreme pH conditions
(d) None of these
19. The following carbohydrate is 24. a - Helix is found in
(a) DNA
(b) RNA
(c) lipid
(d) Carbohydrates

RESPONSE 15. 16. 17. 18. 19.


GRID 20. 21. 22. 23. 24.

Space for Rough Work


EBD_7504
C-112 NTA JEE Main

25. Lack of essential amino acids in the diet leads to the disease 28. Identify the correct statement regarding enzymes
called (a) enzymes are specific biological catalysts that cannot be
(a) Night blindness (b) Pernicious anaemia poisoned
(c) Kwashiorkor (d) Sickel cell anaemia (b) enzymes are normally heterogeneous catalysts that are
26. Which of the following structures represents thymine ? very specific in their action
(c) enzymes are specific biological catalysts that can
O NH2 normally function at very high temperatures (t~1000k)
HN N (d) enzymes are specific biological catalysts that possess
(a) (b) well-defined active sites
O N O N 29. In both DNA and RNA, heterocyclic base and phosphate
H H ester linkages are at –
NH2 OH (a) C5' and C1' respectively of the sugar molecule
N N CH3 (b) C1' and C5' respectively of the sugar molecule
(c) (d) (c) C '2 and C5' respectively of the sugar molecule
HO N HO N (d) C5' and C '2 respectively of the sugar molecule
30. Glucose molecule reacts with X number of molecules of
27. The segment of DNA which acts as the instrumental manual
phenylhydrazine to yield osazone. The value of X is
for the synthesis of the protein is:
(a) three (b) two
(a) ribose (b) gene
(c) one (d) four
(c) nucleoside (d) nucleotide

RESPONSE 25. 26. 27. 28. 29.


GRID 30.

CHEMISTRY CHAPTERWISE SPEED TEST-57


Total Questions 30 Total Marks 120
Attempted Correct
Incorrect Net Score
Cut-off Score 37 Qualifying Score 56
Success Gap = Net Score – Qualifying Score
Net Score = (Correct × 4) – (Incorrect × 1)
Space for Rough Work
CHEMISTRY Speed
Polymers TEST
No. of Questions
30
Maximum Marks
120
Time
1 Hour
58
Chapter-wise

GENERAL INSTRUCTIONS
• This test contains 30 MCQ's. For each question only one option is correct. Darken the correct circle/ bubble in the
Response Grid provided on each page.
• You have to evaluate your Response Grids yourself with the help of solutions provided at the end of this book.
• Each correct answer will get you 4 marks and 1 mark shall be deduced for each incorrect answer. No mark will be given/
deducted if no bubble is filled. Keep a timer in front of you and stop immediately at the end of 60 min.
• The sheet follows a particular syllabus. Do not attempt the sheet before you have completed your preparation for that
syllabus.
• After completing the sheet check your answers with the solution booklet and complete the Result Grid. Finally spend time
to analyse your performance and revise the areas which emerge out as weak in your evaluation.

1. Which of the following compound is used for preparation 2. Which one of the following is not an example of chain
of melamine formaldehyde polymer ? growth polymer?
(a) Neoprene (b) Buna-S
N (c) PMMA (d) Glyptal
N N
NH2 3. Urethane is
(a) (b) (a) H2N - C º N (b) H 2 N - C - OH
H2N NH2 ||
NH2 O
(c) HO - C º N (d) H 2 N - C - OC 2 H 5
H2N NH2 ||
N NH2 O
4. The turbidity of a polymer solution measures
N N (a) the light scattered by solution
(c) (d)
N (b) the light absorbed by a solution
NH2 H2N (c) the light transmitted by a solution
(d) None of these

RESPONSE GRID 1. 2. 3. 4.

Space for Rough Work


EBD_7504
C-114 NTA JEE Main

5. Which of the following polymers do not involve cross


æ ö
linkages? ç ÷
(a) Melmac (b) Bakelite (a) Neoprene; ç - CH 2 - C = CH - CH 2 - CH 2 -÷
ç | ÷
(c) Polythene (d) Vulcanised rubber è Cl øn
6. Which compound/set of compounds is used in the (b) Terylene;
manufacture of nylon 6?
(– OC COOCH2 – CH2 – O–)n
(a) CH = CH2
(c) Nylon 66; [ –NH(CH 2 ) 6 NH CO(CH 2 ) 4 CO - ]n
(b) HOOC(CH2)4COOH + NH2(CH2)6 NH2
(d) Teflon; ( -CF2 - CF2 -)n
CH3
| 12. Which one of the following sets forms the biodegradable
(c) CH 2 = CH - C = CH 2 polymer?
(a) CH2 = CH – CN and CH2 = CH – CH = CH2
O (b) H2N – CH2 – COOH and H2N–(CH2)5 – COOH
(c) HO – CH2 – CH2 – OH and
(d)
HOOC COOH
7. Ebonite is
(a) natural rubber (b) synthetic rubber
(c) highly vulcanized rubber (d) polypropene (d) CH = CH2 and CH2 = CH – CH = CH2

é CH3 ù 13. Which of the following catalyst is used in preparation of


ê | ú high density polythene ?
8. Monomer of ê - C - CH 2 - ú is
ê | ú (a) Peroxide catalyst
êë CH3 úû (b) Ziegler - Natta catalyst
n
(c) Wilkinson’s catalyst
(a) 2-Methylpropene (b) Styrene (d) Pd - catalyst
(c) Propylene (d) Ethene 14. Which is the monomer of Neoprene in the following ?
9. Caprolactam polymerises to give CH 2 = C ¾ CH = CH 2
(a)
(a) terylene (b) teflon ½
CH 3
(c) glyptal (d) Nylon-6
10. Polymer used in bullet proof glass is (b) CH 2 = C ¾ CH = CH 2
(a) PMMA (b) Lexan ½
Cl
(c) Nomex (d) Kevlar
(c) CH2 = CH ¾ C º CH
11. Structures of some common polymers are given. Which one
(d) CH2 = CH ¾ CH = CH2
is not correctly presented?

RESPONSE 5. 6. 7. 8. 9.
GRID 10. 11. 12. 13. 14.
Space for Rough Work
Chemistry C-115

15. Buna-N synthetic rubber is a copolymer of : 21. Which compound/set of compounds is used in the
(a) H2C = CH – CH = CH2 and H5C6 – CH = CH2 manufacture of nylon-6,6?
(b) H2C = CH – CN and H2C = CH – CH = CH2 (a) HOOC(CH2)4COOH + H2N(CH2)6NH2
(b) CH2 = CH–C(CH3) = CH2
(c) H2C = CH – CN and H 2 C = CH – C = CH 2 (c) CH2 = CH2
|
CH3
(d) HOOC COOH
Cl + HOCH2 – CH2 OH
|
(d) H 2 C = CH – C = CH 2 and H 2C = CH – CH = CH 2 22. Which of the following is not correctly matched?
16. The polymer containing strong intermolecular forces e.g.
hydrogen bonding, is é ù
(a) teflon (b) nylon 6, 6 (a) Neoprene; ê -CH 2 - C = CH - CH 2 - ú
ê | ú
(c) polystyrene (d) natural rubber ê ú
ë Cl ûn
17. Formation of polyethylene from calcium carbide takes place
as follows O
é ù
ê || ú
CaC 2 + 2H 2 O ¾
¾® Ca (OH ) 2 + C 2 H 2 (b) Nylon-66: ê - NH - (CH 2 ) 6 - NH - CO - (CH 2 ) 4 - C - O - ú
ê ú
êë úû n
C2 H2 + H2 ¾
¾® C 2 H 4
nC 2 H 4 ¾
¾® ( - CH 2 - CH 2 - ) n é O Où
ê || | |ú
The amount of polyethylene obtained from 64.1 kg of CaC2 is
(c) Terylene; ê- OCH 2 - CH 2 - O - C - Cú
(a) 7 kg (b) 14 kg ê ú
(c) 21 kg (d) 28 kg êë úû n
18. Ziegler-Natta catalyst is
é CH3 ù
(a) K[PtCl3 (C 2 H 4 )] (b) (Ph 3P) 3 RhCl ê | ú
(d) PMMA : –ê CH 2 - C - ú
(c) Al 2(C2 H5 )6 + TiCl 4 (d) Fe(C5 H 5 ) 2 ê | ú
19. Nylon 6,6 is a polyamide obtained by the reaction of êë COOCH3 úû
n
(a) COOH(CH2)4 COOH + NH2C6H4NH2
23. Plexiglas (PMMA) is a polymer of
(b) COOH(CH2)4 COOH + NH2 (CH2)6 NH2
(a) acrylic acid
(c) COOH (CH2)6 COOH + NH2 (CH2)4 NH2 (b) methyl acrylate
(d) COOHC6H4 COOH–(p) + NH2 (CH2)6 NH2 (c) methyl methacrylate
20. Which one of the following monomers gives the polymer (d) None of these
neoprene on polymerization? 24. Orlon is a polymer of
Cl (a) styrene
| (b) tetrafluoroethylene
(a) CCl 2 = CCl 2 (b) CH 2 = C - CH = CH 2
(c) vinyl chloride
(c) CF2 = CF2 (d) CH 2 = CHCl (d) acrylonitrile

RESPONSE 15. 16. 17. 18. 19.


GRID 20. 21. 22. 23. 24.
Space for Rough Work
EBD_7504
C-116 NTA JEE Main

25. Synthetic human hair wigs are made from a copolymer of 29. Which of the following structures represents neoprene
vinyl chloride and acrylonitrile and is called polymer?
(a) PVC (b) Polyacrylonitrile (a) –(CH 2 – C = CH – CH 2 –) n
(c) Cellulose (d) Dynel |
26. Melamine plastic crockery is a codensation polymer of Cl
(a) HCHO and melamine (b) HCHO and ethylene CN
(c) melamine and ethylene (d) None of these |
27. Match List-I (Monomer) with List II (Polymer) and select (b) –(CH 2 – CH –) n
the correct answer using the codes given below the lists:
List I List II Cl
I. Hexamethylenediamine A. Bakelite |
II. Phenol B. Dacron (c) –( CH 2 – CH –)n
III. Phthalic acid C. Glyptal
IV. Terephthalic acid D. Melamine (d) –( CH – CH 2 –)n
E. Nylon |
Codes: C6 H5
(a) I-E, II-A, III-B, IV-C (b) I-E, II-A, III-C, IV-B
(c) I-D, II-C, III-A, IV-B (d) I-D, II-C, III-A, IV-B 30. Which one of the following is used to make ‘non-stick’
28. Among cellulose, poly (vinyl chloride), nylon and natural cookware?
rubber, the polymer in which the intermolecular force of (a) PVC
attraction is weakest is (b) Polystyrene
(a) nylon (b) poly (vinyl chloride) (c) Poly (ethylene terephthalate)
(c) cellulose (d) natural Rubber (d) Polytetrafluoroethylene

RESPONSE 25. 26. 27. 28. 29.


GRID 30.

CHEMISTRY CHAPTERWISE SPEED TEST-58


Total Questions 30 Total Marks 120
Attempted Correct
Incorrect Net Score
Cut-off Score 38 Qualifying Score 57
Success Gap = Net Score – Qualifying Score
Net Score = (Correct × 4) – (Incorrect × 1)
Space for Rough Work
CHEMISTRY Speed
Chemistry in Everyday Life TEST
No. of Questions
30
Maximum Marks
120 1 Hour
Time 59
Chapter-wise

GENERAL INSTRUCTIONS
• This test contains 30 MCQ's. For each question only one option is correct. Darken the correct circle/ bubble in the
Response Grid provided on each page.
• You have to evaluate your Response Grids yourself with the help of solutions provided at the end of this book.
• Each correct answer will get you 4 marks and 1 mark shall be deduced for each incorrect answer. No mark will be given/
deducted if no bubble is filled. Keep a timer in front of you and stop immediately at the end of 60 min.
• The sheet follows a particular syllabus. Do not attempt the sheet before you have completed your preparation for that
syllabus.
• After completing the sheet check your answers with the solution booklet and complete the Result Grid. Finally spend time
to analyse your performance and revise the areas which emerge out as weak in your evaluation.

1. The drug H is used as CH2NH2 CH2CH2CH2NH2


N
(a) (b)
N CH2 OH OH
OH OH
CH2 NH2 CH3CH2NH2 CH2 CHCO2H
(a) Antacid (b) Analgesic
NH2
(c) Antimicrobial (d) Antiseptic (c) (d)
2. Parkinson's disease is linked to abnormalities in the levels of OH OH
dopamine in the body. The structure of dopamine is OH OH

RESPONSE GRID 1. 2.

Space for Rough Work


EBD_7504
C-118 NTA JEE Main

3. The correct structure of the drug paracetamol is 8. Which one of the following is employed as a tranquilizer
OH OH drug?
(a) Promethazine (b) Valium
(a) (b) (c) Naproxen (d) Mifepristone
9. Which of the following substances cannot be used as a
fixative in perfumes?
CONH2 NHCOCH3
(a) Benzoin
Cl Cl (b) Sandalwood
(c) Glyceryl dinitrate
(d) Glyceryl diacetate
(c) (d)
10. Detergents are prepared by the action of H2SO4 followed
CONH2 COCH3 by neutralization by starting with
4. Arsenic drugs are mainly used in the treatment of (a) chloesterol (b) lauryl alcohol
(a) Jaundice (b) Typhoid (b) cyclohexanol (d) p-nitrophenol
(c) Syphilis (d) Cholera 11. Which of the following is not correctly matched?
5. Antiseptic chloroxylenol is (i) Proteins that are – Receptors
(a) 4-chloro-3, 5-dimethylphenol
(b) 3-chloro-4, 5-dimethylphenol crucial to body’s communication
(c) 4-chloro-2, 5-dimethylphenol process.
(d) 5-chloro-3, 4-dimethylphenol (ii) Drugs that mimic – Antagonists
6. The following compound is used as the natural messenger by
O switching on the receptor.
O – C – CH3 (iii) Drugs that binds to – Agonists
the receptor site and inhibit
COOH its natural function.
(a) an anti-inflammatory compound (a) (ii) only (b) (iii) only
(b) analgesic (c) (i) and (iii) (d) (ii) and (iii)
(c) hypnotic 12. Commonly used antiseptic 'Dettol' is a mixture of
(d) antiseptic (a) o-chlorophenozylenol + terpeneol
7. Which of the following ceramics possesses good strength, (b) o-cresol + terpeneol
translucency and very low porosity (c) phenol + terpeneol
(a) whiteware (b) earthenware
(d) chloroxylenol + terpeneol
(c) stoneware (d) bricks and tiles
RESPONSE 3. 4. 5. 6. 7.
GRID 8. 9. 10. 11. 12.
Space for Rough Work
Chemistry C-119

13. Which one of the following can possibly be used as analgesic (a) X = Broad spectrum antibiotics.
without causing addiction and mood modification ? Y = Narrow spectrum antibiotics.
(a) Diazepam (b) Morphine (b) X = Broad spectrum antibiotics.
(c) N-Acetyl-para-aminophenol Y = Limited spectrum antibiotics.
(d) Tetrahydrocannabinol (c) X = Narrow spectrum antibiotics.
14. The indicator used in the titration of a strong acid and a Y = Limited spectrum antibiotics.
strong base is (d) X = Narrow spectrum antibiotics.
(a) phenolphthalein (b) methyl orange Y = Broad spectrum antibiotics.
(c) alizarin yellow (d) red litmus
20. Structurally biodegradable detergent should contain
15. Benzalkonium chloride is a
(a) cationic surfactant and antiseptic (a) normal alkyl chain (b) branched alkyl chain
(b) anionic surfactant and soluble in most of organic (c) phenyl side chain (d) cyclohexyl side chain
solvents 21. H1 – Receptor antagonists is a term associated with :
(c) cationic surfactant and insoluble in most of organic (a) Antiseptics (b) Antihistamins
solvents (c) Antacids (d) Analgesics
(d) cationic surfactant and antimalarial 22. Which of the following is not true ?
16. During the deepening of the colour of a dye, the absorption (a) Some disinfectants can be used as antiseptics
shifts towards red. This shifting is called
(b) Sulphadiazine is a synthetic antibacterial
(a) bathochromic shift (b) hypsochromic shift
(c) Aspirin is analgesic as well as antipyretic
(c) hyperchromic shift (d) auxochromic shift
17. Amoxillin is semi-synthetic modification of (d) Polystyrene is used to make non-stick cookware
(a) penicillin (b) streptomycin 23. Which of the followings forms the base of talcum powder?
(c) tetracycline (d) chloramphenicol (a) Zine stearate
18. Select the incorrect statement. (b) Sodium aluminium silicate
(a) Equanil is used to control depression and hypertension. (c) Magnesium hydrosilicate
(b) Mifepristone is a synthetic steroid used as “morning (d) Chalk
after pill”. 24. The structure given below is known as
(c) 0.2 percent solution of phenol is an antiseptic while its
O
1.0 percent solution is a disinfectant. P H H S CH3
(d) A drug which kills the organism in the body is called CH 2 - C - NH
bacteriostatic. CH3
N COOH
19. Antibiotics that are effective mainly against Gram-positive O H
or Gram-negative bacteria X. Antibiotics that are effective
(a) Penicillin F (b) Penicillin G
against a single organism or disease are Y
(c) Penicillin K (d) Ampicillin
What is X and Y ?

13. 14. 15. 16. 17.


RESPONSE 18. 19. 20. 21. 22.
GRID 23. 24.
Space for Rough Work
EBD_7504
C-120 NTA JEE Main

25. Most of the deodorants contain aluminium salts because (c) Disinfectants harm the living tissues.
they (d) A 0.2% solution of phenol is an antiseptic while 1%
(a) act as antiperspirants solution acts as a disinfectant.
(b) act as anti-bacterial agents 28. The substance which is not an artificial sweetener
(c) mask body odour (a) sucralose (b) alitame
(d) All the these (c) saccharin (d) sucrose
26. Chemical substances used by insects for communication 29. An antibiotic with a broad spectrum
are called (a) kills the antibodies
(a) pheromones (b) hormones (b) acts on a specific antigen
(c) enzymes (d) nucleoproteins (c) acts on different antigens
27. Antiseptics and disinfectants either kill or prevent growth (d) acts on both the antigens and antibodies
of microorganisms. Identify which of the following 30. SLV -3 uses ______ propellants
statements is not true: (a) solid (b) liquid
(a) Chlorine and iodine are used as strong disinfectants. (c) solid-liquid (d) biliquid
(b) Dilute solutions of Boric acid and Hydrogen Peroxide
are strong antiseptics.

RESPONSE 25. 26. 27. 28. 29.


GRID 30.

CHEMISTRY CHAPTERWISE SPEED TEST-59


Total Questions 30 Total Marks 120
Attempted Correct
Incorrect Net Score
Cut-off Score 37 Qualifying Score 55
Success Gap = Net Score – Qualifying Score
Net Score = (Correct × 4) – (Incorrect × 1)
Space for Rough Work
MATHEMATICS Speed
Sets TEST
No. of Questions
30
Maximum Marks
120
Time
1 Hour
60
Chapter-wise

GENERAL INSTRUCTIONS
• This test contains 30 MCQ's. For each question only one option is correct. Darken the correct circle/ bubble in the
Response Grid provided on each page.
• You have to evaluate your Response Grids yourself with the help of solutions provided at the end of this book.
• Each correct answer will get you 4 marks and 1 mark shall be deduced for each incorrect answer. No mark will be given/
deducted if no bubble is filled. Keep a timer in front of you and stop immediately at the end of 60 min.
• The sheet follows a particular syllabus. Do not attempt the sheet before you have completed your preparation for that
syllabus.
• After completing the sheet check your answers with the solution booklet and complete the Result Grid. Finally spend time
to analyse your performance and revise the areas which emerge out as weak in your evaluation.

1. Let A = {(1, 2), (3, 4), 5}, then which of the following is 3. One of the partitions of the set {1, 2, 5, x, y, 2 , 3 } is
incorrect?
(a) {3, 4} Ï A as (3, 4) is an element of A (a) {{1, 2, x}, {x, 5, y}, { 2 , 3 }}
(b) {5}, {(3, 4)} are subsets of A but not elements of A
(c) {1, 2}, {5} are subsets of A (b) {{1,2, 2 }, {x,y, 2 }, { 5, 2 , 3 }}
(d) {(1, 2), (3, 4), 5} is subset of A
2. A market research group conducted a survey of 1000 (c) {{1, 2}, {5, x}, { 2 , 3 }}
consumers and reported that 720 consumers liked product
(d) {{1, 2, 5}, {x, y}, { 2 , 3 }}
A and 450 consumers liked product B. What is the least
number that must have liked both products ? 4. Let A and B be two sets then (A È B) 'È (A 'Ç B) is equal
(a) 170 (b) 280 to
(c) 220 (d) None of these (a) A ¢ (b) A
(c) B ¢ (d) None of these

RESPONSE GRID 1. 2. 3. 4.

Space for Rough Work


EBD_7504
M-2 NTA JEE Main

5. Let A = {(n, 2n) : n Î N} and B = {(2n, 3n) : n Î N}. What is (a) 48, 1 (b) 34, 3
A Ç B equal to ? (c) 46, 4 (d) 42, 2
(a) {(n, 6n) : n Î N} (b) {(2n, 6n) : n Î N} 11. Let X and Y be two non-empty sets such that
(c) {(n, 3n) : n Î N} (d) f X Ç A = Y Ç A = f and X È A = Y È A for some non-empty
6. If aN = {ax : x Î N} and bN Ç cN = dN, where b, c Î N are set A. Then
(a) X is a proper subset of Y
relatively prime, then
(b) Y is a proper subset of X
(a) d = bc (b) c = bd
(c) X = Y
(c) b = cd (d) None of these
(d) X and Y are disjoint sets
7. In a class of 55 students, the number of students studying
12. Let A and B are two sets in a universal set U. Then which of
different subjects are 23 in Mathematics, 24 in Physics, 19 in
these is/are correct ?
Chemistry, 12 in Mathematics and Physics, 9 in Mathemat-
(a) A – B = A' – B'
ics and Chemistry, 7 in Physics and Chemistry and 4 in all
(b) A – (A – B) = A Ç B
the three subjects. The number of students who have taken
(c) A – B = A' Ç B'
exactly one subject is
(d) A È B = (A – B) È (B–A) È (A Ç B)
(a) 6 (b) 9
13. If A and B are non-empty sets such that A É B, then
(c) 7 (d) All of these (a) B' – A' = A – B (b) B' – A' = B – A
8. A set A has 3 elements and another set B has 6 elements.
(c) A' – B' = A – B (d) A' Ç B' = B – A
Then
14. In a town of 10,000 families, it was found that 40% families
(a) 3 £ n (A È B) £ 6 (b) 3 £ n (A È B) £ 9
buy newspaper A, 20% families buy newspaper B and 10%
(c) 6 £ n (A È B) £ 9 (d) 0 £ n (A È B) £ 9
families buy newspaper C. 5% families buy A and B, 3%
9. If A = {1, 2, 5} and B = {3, 4, 5, 9}, then A D B is equal to buy B and C and 4% buy A and C. If 2% families buy all the
(a) {1, 2, 5, 9} (b) {1, 2, 3, 4, 9} newspapers, then
(c) {1, 2, 3, 4, 5, 9} (d) None of these (a) 3,300 families buy A only
10. At a certain conference of 100 people, there are 29 Indian (b) 1,400 families buy B only.
women and 23 Indian men. Of these Indian people 4 are (c) 4000 families buy none of A, B and C
doctors and 24 are either men or doctors. There are no foreign (d) All are correct
doctors. How many foreigners and women doctors are
attending the conference?

RESPONSE 5. 6. 7. 8. 9.
GRID 10. 11. 12. 13. 14.
Space for Rough Work
Mathematics M-3

15. In a battle 70% of the combatants lost one eye, 80% an ear, (a) 13 (b) 24
75% an arm, 85% a leg, x % lost all the four limbs. The (c) 28 (d) 52
minimum value of x is 20. In a class of 60 students, 23 play Hockey 15 Play Basket-ball
(a) 10 (b) 12 and 20 play cricket. 7 play Hockey and Basket-ball, 5 play
cricket and Basket-ball, 4 play Hockey and Cricket and 15
(c) 15 (d) None of these students do not play any of these games. Then
16. Let n (U) = 700, n (A) = 200, n (B) = 300, n (A Ç B) = 100, then (a) 4 play Hockey, Basket-ball and Cricket
n (A' Ç B') is equal to (b) 20 play Hockey but not Cricket
(a) 400 (b) 600 (c) 1 plays Hockey and Cricket but not Basket-ball
(c) 300 (d) None of these (d) All above are correct
17. Statement-1 : If B = U – A, then n(B) = n(U) – n(A) where U 21. The set (A \ B) È (B \ A) is equal to
is universal set. (a) [ A \ ( A Ç B)] Ç [ B \ ( A Ç B)]
Statement-2 : For any three arbitrary set A, B, C we have if
C = A – B, then n(C) = n(A) – n(B). (b) ( A È B) \ ( A Ç B)
(a) Statement -1 is true, Statement-2 is true; Statement -2 is (c) A \ ( A Ç B)
a correct explanation for Statement-1.
(d) A Ç B \ A È B
(b) Statement -1 is true, Statement-2 is true; Statement -2
is not a correct explanation for Statement-1. 22. If A is the set of the divisors of the number 15, B is the set of
(c) Statement -1 is false, Statement-2 is true. prime numbers smaller than 10 and C is the set of even
(d) Statement -1 is true, Statement-2 is false. numbers smaller than 9, then (A È C) Ç B is the set
18. Each student in a class of 40, studies at least one of the (a) {1, 3, 5} (b) {1, 2, 3}
subjects English, Mathematics and Economics. 16 study (c) {2, 3, 5} (d) {2, 5}
English, 22 Economics and 26 Mathematics, 5 study English 23. Two finite sets have m and n elements. The number of subsets
of the first set is 112 more than that of the second set. The
and Economics, 14 Mathematics and Economics and 2 study
values of m and n are, respectively,
all the three subjects. The number of students who study
English and Mathematics but not Economics is (a) 4, 7 (b) 7, 4
(a) 7 (b) 5 (c) 4, 4 (d) 7, 7
(c) 10 (d) 4 24. The number of students who take both the subjects
19. In a class of 80 students numbered a to 80, all odd mathematics and chemistry is 30. This represents 10% of
numbered students opt of Cricket, students whose the enrolment in mathematics and 12% of the enrolment in
numbers are divisible by 5 opt for Football and those chemistry. How many students take at least one of these
whose numbers are divisible by 7 opt for Hockey. The two subjects?
number of students who do not opt any of the three (a) 520 (b) 490
games, is (c) 560 (d) 480

RESPONSE 15. 16. 17. 18. 19.


GRID 20. 21. 22. 23. 24.
Space for Rough Work
EBD_7504
M-4 NTA JEE Main

25. If n(A) = 1000, n(B) = 500 and if n(A Ç B) ³ 1 and 28. The value of (A È B È C) Ç (A Ç Bc Ç Cc)c Ç Cc, is
n(A È B) = p, then (a) B Ç Cc (b) Bc Ç Cc
(a) 500 £ p £ 1000 (b) 1001 £ p £ 1498 (c) B Ç C (d) A Ç B Ç C
(c) 1000 £ p £ 1498 (d) 1000 £ p £ 1499 29. In a town of 10,000 families it was found that 40% family buy
newspaper A, 20% buy newspaper B and 10% families buy
26. The number of elements in the set
newspaper C, 5% families buy A and B, 3% buy B and C and
{(a, b) : 2a2 + 3b2 = 35, a, b Î Z}, where Z is the set of all 4% buy A and C. If 2% families buy all the three newspa-
integers, is pers, then number of families which buy A only is
(a) 2 (b) 4 (a) 3100 (b) 3300
(c) 8 (d) 12 (c) 2900 (d) 1400
27. Let A, B, C be finite sets. Suppose that n (A) = 10, n (B) = 15, n 30. Statement-1 : If A È B = A È C and A Ç B = A Ç C, then
(C) = 20, n (AÇB) = 8 and n (BÇC) = 9. Then the possible B = C.
value of n (AÈBÈC) is Statement-2 : A È (B Ç C) = (A È B) Ç (A È C).
(a) 26 (a) Statement -1 is true, Statement-2 is true; Statement -2 is
(b) 27 a correct explanation for Statement-1.
(b) Statement -1 is true, Statement-2 is true; Statement -2
(c) 28
is not a correct explanation for Statement-1.
(d) Any of the three values 26, 27, 28 is possible
(c) Statement -1 is false, Statement-2 is true.
(d) Statement -1 is true, Statement-2 is false.

RESPONSE 25. 26. 27. 28. 29.


GRID 30.

MATHEMATICS CHAPTERWISE SPEED TEST-60


Total Questions 30 Total Marks 120
Attempted Correct
Incorrect Net Score
Cut-off Score 40 Qualifying Score 55
Success Gap = Net Score – Qualifying Score
Net Score = (Correct × 4) – (Incorrect × 1)
Space for Rough Work
MATHEMATICS Speed
Relations and Functions TEST
No. of Questions
30
Maximum Marks
120
Time
1 Hour
61
Chapter-wise

GENERAL INSTRUCTIONS
• This test contains 30 MCQ's. For each question only one option is correct. Darken the correct circle/ bubble in the
Response Grid provided on each page.
• You have to evaluate your Response Grids yourself with the help of solutions provided at the end of this book.
• Each correct answer will get you 4 marks and 1 mark shall be deduced for each incorrect answer. No mark will be given/
deducted if no bubble is filled. Keep a timer in front of you and stop immediately at the end of 60 min.
• The sheet follows a particular syllabus. Do not attempt the sheet before you have completed your preparation for that
syllabus.
• After completing the sheet check your answers with the solution booklet and complete the Result Grid. Finally spend time
to analyse your performance and revise the areas which emerge out as weak in your evaluation.

1. For the following relation


æ 1 ö = log x4, then f(e–x) is
If 3f(x) – f ç
R = {(0, 0), (0, 1), (1, 1), (2, 1), (2, 2), (2, 0), (1, 0), (0, 2), (0, 1)} 3. ÷
(a) domain = {0, 1} (b) range = {0, 1, 2} èxø
(c) both correct (d) None of these (a) 1+ x (b) 1/x
2. The domain of the function 2 x + 8 - x2 (c) x (d) – x
x 2 - 5x + 6 +
is 1
(a) [2, 3] (b) [–2, 4] 4. The domain of the function f ( x) = is
x -x
(c) [–2, 2] È [3, 4] (d) [–2, 1] È [2, 4]
(a) (0, ¥ ) (b) (– ¥ , 0)
(c) (– ¥ , ¥ ) – {0} (d) (– ¥ , ¥ )

RESPONSE GRID 1. 2. 3. 4.

Space for Rough Work


EBD_7504
M-6 NTA JEE Main

(a) R = {(0, 3), (0, –3), (3, 0), (–3, 0)}


5. f(x) = | x |2 -5 | x | +6 + 8 + 2 | x | - | x |2 is real for all x (b) Domain of R = {–3, 0, 3}
in (c) Range of R = {–3, 0, 3}
(a) [–4, –3] (b) [–3, –2] (d) None of these
(c) [–2, 2] (d) [3, 4] 11. Let f (x) = 1 + x 2 , then
x(x - p) x(x - q) (a) f (xy) = f (x) . f (y) (b) f (xy) > f (x) . f ( y)
6. f (x) = + , p ¹ q. What is the value of
q-p p -q (c) f (xy) < f (x) . f (y) (d) None of these
f (p) + f (q) ?
12. The domain of the function f (x) = x - 1 - x 2 is
(a) f (p – q) (b) f (p + q)
(c) f (p (p + q)) (d) f (q (p – q))
é 1 ù é 1 ù
7. A real valued function f (x) satisfies the functional equation (a) ê -1, - úÈê ,1ú
ë 2û ë 2 û
f (x – y) = f (x) f (y) – f (a – x) f (a + y)
where a is a given constant and f (0) = 1, f (2a – x) is equal to (b) [–1, 1]
(a) – f (x) (b) f (x)
æ 1ù é 1 ö
(c) f (a) + f (a – x) (d) f (– x) (c) çè -¥, - ú È ê , + ¥÷
ø
2û ë 2
8. Domain of definition of the function
3
f ( x) = + log10 ( x 3 - x) , is é 1 ù
4 - x2 (d) ê ,1ú
ë 2 û
(a) ( -1,0) È (1,2) È ( 2, ¥) (b) (a, 2)
(c) ( -1,0) È ( a,2) (d) (1,2) È (2, ¥ ) . 3 x x
13. Period of the function sin + cos5 is :
9. Let A = {1, 2, 3, 4, 5}; B = {2, 3, 6, 7}. Then the number of 2 5
elements in (A × B) Ç (B × A) is
(a) 2 p (b) 10 p
(a) 18 (b) 6 (c) 8 p (d) 5 p
(c) 4 (d) 0
14. If n(A) = 4, n(B) = 3, n(A × B × C) = 24, then n(C) =
10. A relation R is defined in the set Z of integers as follows
(a) 288 (b) 1
(x, y) Î R iff x2 + y2 = 9. Which of the following is false?
(c) 12 (d) 2

RESPONSE 5. 6. 7. 8. 9.
GRID 10. 11. 12. 13. 14.

Space for Rough Work


Mathematics M-7

15. If S = {1, 2, 3, 4, 5,} and R = {(x, y) : x + y < 6} then n (R) = (a) (1, 5); (2, 3); (3, 5) (b) (5, 1); (3, 2); (5, 3)
(a) 8 (b) 10 (c) (1, 5); (2, 3); (5, 3) (d) None of these
(c) 6 (d) 5
æ x2 + e ö

(
16. The function f ( x ) = log x + x 2 + 1 , is ) 21. If f(x) = ln ç 2 ÷ , then range of f(x) is
è x + 1ø

(a) neither an even nor an odd function (a) (0, 1) (b) (0, 1]
(b) an even function (c) [0, 1) (d) {0, 1}
(c) an odd function
æ 1+ xö
(d) a periodic function 22. The function f (x) = log ç satisfies the equation
è 1 - x ÷ø
x
17. Let f (x) = and ‘a’ be a real number. If x0 = a, (a) f (x + 2) – 2f (x + 1) + f (x) = 0
1- x
x1 = f (x0), x2 = f (x1), x3 = f (x2)....... If x2009 = 1, (b) f (x + 1) + f (x) = f (x (x + 1))
then the value of a is (c) f (x1) · f (x2) = f (x1 + x2)
2009
(a) 0 (b) æ x1 + x2 ö
2010 (d) f (x1) + f (x2) = f çè 1 + x x ÷ø
1 2
1 1
(c) (d) 23. If f : R ® R satisfies f ( x + y ) = f ( x) + f ( y ) , for all x,
2009 2010
18. The domain of the function n
æ æ 1 ö ö y Î R and f(1) = 7, then S f ( r ) is
f ( x ) = log 2 ç – log1/ 2 ç1 + 1/ 4 ÷ – 1÷ is r=1
è è x ø ø
(a) (0, 1) (b) (0, 1] 7 n (n + 1) 7n
(a) (b)
(c) [1, ¥) (d) (1, ¥) 2 2

1 7 (n + 1)
19. The domain of the function f (x) = is (c) (d) 7n + (n + 1) .
2
x2 - 3x + 2
24. If { } denotes the fractional part of x, the range of the function
(a) ( – ¥, 1) (b) ( – ¥, 1) È (2, ¥)
f (x) = { x}2 - 2{ x} is
(c) ( – ¥, 1] È [2, ¥) (d) (2, ¥)
20. If (1, 3), (2, 5) and (3, 3) are three elements of A × B and the (a) f (b) [0, 1/2]
total number of elements in A × B is 6, then the remaining (c) {0, 1/2} (d) {0}
elements of A × B are

RESPONSE 15. 16. 17. 18. 19.


GRID 20. 21. 22. 23. 24.

Space for Rough Work


EBD_7504
M-8 NTA JEE Main

(a) [3/2, ¥) (b) [1, 3/2]


x -1
25. If f (x) = , then f (2x) is equal to (c) (–¥, 1] (d) (1, 3/2)
x +1
28. If f(x + y) = f (x) + 2y2 + kxy and f(a) = 2, f(b) = 8, then f(x) is
f ( x) + 1 3 f ( x) + 1 of the form
(a) (b)
f ( x) + 3 f ( x) + 3 (a) 2x2 (b) 2x2 + 1
(c) 2x2 – 1 (d) x 2
f ( x) + 3 f ( x) + 3
(c) (d) 29. The relation R defined on the set A = {1, 2, 3, 4, 5} by
f ( x) + 1 3 f ( x) + 1 R= {(x, y) : | x2 – y2 | < 16} is given by
(a) {(1, 1), (2, 1), (3, 1), (4, 1), (2, 3)}
x2 - x + 1 (b) {(2, 2), (3, 2), (4, 2), (2, 4)}
26. The range of the function f (x) = 2 where x Î R, is
x + x +1 (c) {(3, 3), (3, 4), (5, 4), (4, 3), (3, 1)}
(a) (– ¥, 3] (b) (–¥, ¥) (d) None of these
30. Which of the following relation is NOT a function
é1 ù (a) f = {(x, x) | x Î R} (b) g = {(x, 3) | x Î R}
(c) [3, ¥) (d) êë 3 , 3úû
1
27. The domain of the function f (x) = exp( 5x - 3 - 2x 2 ) (c) h = { ( n, ) | n Î I} (d) t = {(n, n2) | n Î N}
n
is

RESPONSE 25. 26. 27. 28. 29.


GRID 30.

MATHEMATICS CHAPTERWISE SPEED TEST-61


Total Questions 30 Total Marks 120
Attempted Correct
Incorrect Net Score
Cut-off Score 38 Qualifying Score 50
Success Gap = Net Score – Qualifying Score
Net Score = (Correct × 4)Work– (Incorrect × 1)
Space for Rough

Space for Rough Work


MATHEMATICS Speed
Trigonometric Functions TEST
No. of Questions
30
Maximum Marks
120
Time
1 Hour
62
Chapter-wise

GENERAL INSTRUCTIONS
• This test contains 30 MCQ's. For each question only one option is correct. Darken the correct circle/ bubble in the
Response Grid provided on each page.
• You have to evaluate your Response Grids yourself with the help of solutions provided at the end of this book.
• Each correct answer will get you 4 marks and 1 mark shall be deduced for each incorrect answer. No mark will be given/
deducted if no bubble is filled. Keep a timer in front of you and stop immediately at the end of 60 min.
• The sheet follows a particular syllabus. Do not attempt the sheet before you have completed your preparation for that
syllabus.
• After completing the sheet check your answers with the solution booklet and complete the Result Grid. Finally spend time
to analyse your performance and revise the areas which emerge out as weak in your evaluation.

1. If y = cos2x + sec2x, then 3. If an angle q is divided into 2 parts A and B such that
A – B = k and A + B = q and tan A : tan B = k : 1, then the
(a) y£2 (b) y £1
value of sin k is :
(c) y³2 (d) 1 < y < 2 k +1 k
(a) sin q (b) sin q
k -1 k +1
sin q + sin 2q
2. Period of is k -1
cos q + cos 2q (c) sin q (d) None of these
k +1
(a) 2 p (b) p
4. If 2 y cos q = x sin q and 2 x sec q - y cosec q = 3, then
2p p
(c) (d) x2 + 4 y 2 =
3 3
(a) 4 (b) – 4
(c) ±4 (d) None of these

RESPONSE GRID 1. 2. 3. 4.

Space for Rough Work


EBD_7504
M-10 NTA JEE Main

5. The equation sin 4 x + cos4 x = a has a solution for (a) 6 sin A (b) 7 sin A
(a) all of values of a (b) a = –1 (c) 6 cos A (d) 7 cos A
1 1 11. General solution of the equation
(c) a= – (d) £ a £1
2 2 ( 3 –1)sin q + ( 3 + 1) cos q = 2 is
6. If for n ÎN, fn(q) = tan q/2 (1 + sec q) (1+sec 2q) (1 + sec p p p
(a) 2np ± + (b) np + (–1)n
4q) .... (1 + sec 2nq), then correct statement is 4 12 2
(a) f2 (p/16) = 1 (b) f3 (p/32) = 1 p p
(c) 2np ± – (d) None
(c) f4 (p/64) = 1 (d) All of these 4 12
cos6 x + 6cos 4 x + 15cos2 x + 10 12. The least positive non-integral solution of the equation
7. The expression is equal
cos5x + 5cos3x + 10cos x sin p( x 2 + x) = sin px 2 is
to (a) rational
(a) cos 2x (b) 2cos x
(c) cos 2 x (d) 1 + cos x. (b) irrational of the form p

æ pö sin(a + b + g ) p -1
8. If a , b, g Î çè 0, ÷ø , then is (c) irrational of the form , where p is an odd integer
2 sin a + sin b + sin g 4
(a) < 1 (b) > 1
p +1
(c) = 1 (d) None of these (d) irrational of the form , where p is an even integer
9. The value of 4
13. If A and B are positive acute angles satisfying
æ p öæ 3p ö æ 7p ö æ 9p ö
ç 1 + cos 10 ÷ ç1 + cos 10 ÷ ç 1 + cos 10 ÷ ç1 + cos 10 ÷ is 3 cos 2 A + 2 cos 2 B = 4 and
3 sin A 2 cos B
=
è øè øè øè ø ,
sin B cos A
1 1 Then the value of A + 2B is equal to :
(a) (b)
8 16 p p
(a) (b)
1 6 2
(c) (d) None of these
32 p p
(c) (d)
10. If sin A - 6 cos A = 7 cos A, then 3 4
14. The greatest and least value of sin x cos x are
cos A + 6 sin A is equal to 1 1
(a) 1 , – 1 (b) ,-
2 2
1 1
(c) ,- (d) 2, – 2
4 4

RESPONSE 5. 6. 7. 8. 9.
GRID 10. 11. 12. 13. 14.

Space for Rough Work


Mathematics M-11

15. If tan (cot x) = cot (tan x), then


cos2 a + cos 2 b + cos 2 g
2 4 20. If q is an angle given by cos q =
(a) sin 2 x = (b) sin x = sin 2 a + sin 2 b + sin 2 g
(2n + 1) p (2n + 1) p
where a, b, g are the equal angles made by a line with the
4 positive directions of the axes, then the measure of q is
(c) sin 2 x = (d) None of these
(2n + 1) p p p
(a) (b)
3 6
1æ x yö
16. sin q = ç + necessarily implies : p p
2è y x ÷ø (c) (d)
2 4
(a) x > y (b) x < y 21. sin 12° sin 24° sin 48° sin 84° =
(c) x = y (a) cos 20° cos 40° cos 60° cos 80°
(d) both x and y are purely imaginary (b) sin 20° sin 40° sin 60° sin 80°
n n 3
17. If pn = cos q + sin q , then pn - pn-2 = kpn-4 , where : (c)
15
(a) k = 1 (b) k = - sin 2 q cos 2 q (d) None of these
(c) k = sin 2 q (d) k = cos 2 q 22. If Sn = cos n q + sin n q then the value of 3S4 - 2S6 is given
18. If f (x) = cos (log x) then by
1 ì æ xö ü (a) 4 (b) 0
f (x)f (y) – í f ç ÷ + f ( xy)ý is equal to : (c) 1 (d) 7
2 è yø
î þ
(a) 0 (b) 1 23. The set of all x in (-p, p) satisfying | 4 sin x - 1 | < 5 is
(c) –1 (d) none of these given by
19. Statement-1 : The maximum and minimum values of the æ p 3p ö æ p ö
function (a) ç- , ÷ (b) ç - , p ÷
è 10 10 ø è 10 ø
1
f (x) = does not exist
6 sin x - 8 cos x + 5 æ 3p ö
(c) (-p, p) (d) ç - p, ÷
Statement-2 : The given function is an unbounded function. è 10 ø
(a) Statement - 1 is false, Statement-2 is true
sin x cos x
(b) Statement - 1 is true, Statement-2 is true ; Statement-2 24. Let f (x) = - then range of f(x) is
is a correct explanation for Statement-1 1 + tan 2 x 1 + cot 2 x
(c) Statement - 1 is true, Statement-2 is true ; Statement-2 (a) [–1, 0] (b) [0, 1]
is not a correct explanation for Statement-1
(d) Statement - 1 is true, Statement-2 is false (c) [–1, 1] (d) none of these

RESPONSE 15. 16. 17. 18. 19.


GRID 20. 21. 22. 23. 24.

Space for Rough Work


EBD_7504
M-12 NTA JEE Main

sin( x + y ) a + b tan x 28. If cos q + cos 2q + cos 3q = 0 , then the general value of q is :
25. If = , then is equal to (a) q = 2mp ± 2p / 3 (b) q = 2mp ± p / 4
sin( x - y ) a - b tan y
b a (c) q = mp + (- 1)n 2p / 3 (d) q = mp + (- 1)np / 3
(a) (b)
p p
The maximum value of sin æç x + ö÷ + cos æç x + ö÷ is in the
a b 29.
(c) ab (d) None of these è 6ø è 6ø
26. Statement-1: If a and b are two distinct solutions of the
æ pö
æ a + bö interval ç 0, ÷ if the value of x is
equation a cos x + b sin x = c, then tan çè ÷ is è 2ø
2 ø
independent of c. p p
(a) (b)
Statement-2: Solution of a cos x + b sin x = c is possible, if 6 12
– ( a 2 + b2 ) £ c £ ( a 2 + b2 ) p p
(c) (d)
(a) Statement -1 is true, Statement-2 is true; Statement -2 is 3 4
a correct explanation for Statement-1 30. If a, b, g , d are the smallest positive angles in ascending
(b) Statement -1 is true, Statement-2 is true; Statement -2 order of magnitude which have their sines equal to the
is not a correct explanation for Statement-1 positive number x, then the value of
(c) Statement -1 is false, Statement-2 is true
(d) Statement -1 is true, Statement-2 is false a b g d
4 sin + 3sin + 2 sin + sin is equal to
27. The value of tan2 q sec2q (cot2q – cos2q) is 2 2 2 2
(a) 0 (b) 1
(a) 2 1 – x (b) 2 1+ x
1
(c) –1 (d) (c) 2 x (d) None of these
2

RESPONSE 25. 26. 27. 28. 29.


GRID 30.

MATHEMATICS CHAPTERWISE SPEED TEST-62


Total Questions 30 Total Marks 120
Attempted Correct
Incorrect Net Score
Cut-off Score 40 Qualifying Score 58
Success Gap = Net Score – Qualifying Score
Net Score = (Correct × 4) – (Incorrect × 1)
Space for Rough Work
MATHEMATICS Speed
Principle of Mathematical Induction TEST
No. of Questions
30
Maximum Marks
120
Time
1 Hour
63
Chapter-wise

GENERAL INSTRUCTIONS
• This test contains 30 MCQ's. For each question only one option is correct. Darken the correct circle/ bubble in the
Response Grid provided on each page.
• You have to evaluate your Response Grids yourself with the help of solutions provided at the end of this book.
• Each correct answer will get you 4 marks and 1 mark shall be deduced for each incorrect answer. No mark will be given/
deducted if no bubble is filled. Keep a timer in front of you and stop immediately at the end of 60 min.
• The sheet follows a particular syllabus. Do not attempt the sheet before you have completed your preparation for that
syllabus.
• After completing the sheet check your answers with the solution booklet and complete the Result Grid. Finally spend time
to analyse your performance and revise the areas which emerge out as weak in your evaluation.

1. Let P(n) : “2n < (1 × 2 × 3 × ... × n)”. Then the smallest (a) 11 (b) 12
positive integer for which P(n) is true is (c) 13 (d) 9
(a) 1 (b) 2 4. A student was asked to prove a statement P(n) by induction.
(c) 3 (d) 4 He proved that P(k + 1) is true whenever P(k) is true for all
2. n n
If P(n) : “46 + 16 + k is divisible by 64 for n Î N” is true, k > 5 Î N and also that P (5) is true. On the basis of this he
then the least negative integral value of k is. could conclude that P(n) is true
(a) – 1 (b) 1 (a) for all n Î N
(c) 2 (d) – 2 (b) for all n > 5
3. Use principle of mathematical induction to find the value (c) for all n ³ 5
of k, where (102n – 1 + 1) is divisible by k. (d) for all n < 5

RESPONSE GRID 1. 2. 3. 4.

Space for Rough Work


EBD_7504
M-14 NTA JEE Main

5. Let T(k) be the statement 1 + 3 + 5 + ... + (2k – 1)= k2 +10


Which of the following is correct? 10. If an = 7 + 7 + 7 + ... ... having n radical signs then by
(a) T(1) is true
methods of mathematical induction which is true?
(b) T(k) is true Þ T(k + 1) is true
(a) an > 7 " n ³ 1 (b) an < 7 " n ³ 1
(c) T(n) is true for all n Î N
(c) an < 4 " n ³ 1 (d) an < 3 " n ³ 1
(d) All above are correct
11. For every positive integral value of n, 3n > n3 when
6. Let S (k ) = 1 + 3 + 5... + (2k - 1) = 3 + k 2 . Then which of the
(a) n>2 (b) n³3
following is true?
(c) n³4 (d) n<4
(a) Principle of mathematical induction can be used to
prove the formula
4n (2n)!
12. If < , then P(n) is true for
(b) S (k ) Þ S (k + 1) n + 1 (n!)2

(c) S (k ) Þ
/ S (k + 1) (a) n ³ 1 (b) n > 0
(d) S (1) is correct (c) n < 0 (d) n ³ 2

7. For natural number n, 2n (n - 1)! < nn , if 13. If n Î N , then x 2 n -1 + y 2 n -1 is divisible by

(a) n<2 (b) n>2 (a) x + y (b) x – y


(c) n³2 (d) Never (c) x2 + y2 (d) x2 + xy
14. For a positive integer n,
8. For all positive integral values of n, 32 n - 2 n + 1 is divisible
by 1 1 1 1
Let a(n) = 1 + + + +…+ n . Then
(a) 2 (b) 4 2 3 4 (2 ) - 1
(c) 8 (d) 12
(a) a(100) £ 100 (b) a(100) > 100
9. For every natural number n, n(n + 1) is always
(c) a(200) £ 100 (d) a(200) < 100
(a) Even (b) Odd
(c) Multiple of 3 (d) Multiple of 4

RESPONSE 5. 6. 7. 8. 9.
GRID 10. 11. 12. 13. 14.

Space for Rough Work


Mathematics M-15

15. 2n > n2 when n Î N such that n (n + 1) n (n + 3)


(a) (b)
(a) n > 2 (b) n > 3 4 (n + 2 ) (n + 3) 4 (n + 1) (n + 2)
(c) n < 5 (d) n ³ 5 n (n + 2)
16. For every natural number n, n(n2 – 1) is divisible by (c) (d) None of these
4 (n + 1) (n + 3)
(a) 4 (b) 6
22. For every positive integer n, 7n – 3n is divisible by
(c) 10 (d) None of these (a) 7 (b) 3
17. If 49n + 16n + l is divisible by 64 for all n Î N, then the least (c) 4 (d) 5
negative value of l is
(a) –2 (b) –1 n5 n3 7n
23. For all n Î N, the sum of + + is
5 3 15
(c) –3 (d) – 4
(a) a negative integer (b) a whole number
18. If n Î N and n is odd, then n (n2 – 1) is divisible by (c) a real number (d) a natural number
(a) 24 (b) 16
24. For n Î N, xn+1 + (x + 1)2n–1 is divisible by
(c) 32 (d) 19
(a) x (b) x + 1
19. For each n Î N , the correct statement is (c) x2 + x + 1 (d) x2 – x + 1
n 2 > 2n 25. If n is a positive integer, then 52n + 2 – 24n – 25 is
(a) 2n < n (b)
divisible by
(c) n 4 < 10n (d) 23n > 7 n + 1 (a) 574 (b) 575
(c) 674 (d) 576
20. P(n) : 2.7n + 3.5n – 5 is divisible by 26. For all n ³ 1,
(a) 24, " n Î N 1 1 1 1
+ + + ..... + =
(b) 21, " n Î N 1.2 2.3 3.4 n( n + 1)
(c) 35, " n Î N n 1
(a) (b)
(d) 50, " n Î N n +1 n +1
21. By mathematical induction,
1
(c) (d) None of these
1 1 1 n(n + 1)
+ + ..... + is equal to
1× 2 ×3 2 × 3× 4 n (n + 1)(n + 2)

RESPONSE 15. 16. 17. 18. 19.


GRID 20. 21. 22. 23. 24.
25. 26.

Space for Rough Work


EBD_7504
M-16 NTA JEE Main

27. By the principle of induction " n Î N, 32n when divided Statement-2 : For every natural number n ³ 2,
by 8, leaves remainder n (n + 1) < n + 1.
(a) 2 (b) 3
(c) 7 (d) 1 (a) Statement-1 is correct, Statement-2 is correct;
Statement-2 is a correct explanation for Statement-1.
1 (b) Statement-1 is correct, Statement-2 is correct;
28. Statement-1 : 1 + 2 + 3 + ..... + n < (2n + 1)2, n Î N.
8 Statement-2 is not a correct explanation for Statement-1
Statement-2 : n(n + 1) (n + 5) is a multiple of 3, n Î N. (c) Statement-1 is correct, Statement-2 is incorrect
(a) Only Statement-1 is true (d) Statement-1 is incorrect, Statement-2 is correct.
(b) Only Statement-2 is true 30. For all n Î N, 41n – 14n is a multiple of
(c) Both Statements are true (a) 26 (b) 27
(d) Both Statements are false (c) 25 (d) None of these
29. Statement-1 : For every natural number n ³ 2,
1 1 1
+ + ..... + > n
1 2 n

RESPONSE
27. 28. 29. 30.
GRID

MATHEMATICS CHAPTERWISE SPEED TEST-63


Total Questions 30 Total Marks 120
Attempted Correct
Incorrect Net Score
Cut-off Score 40 Qualifying Score 55
Success Gap = Net Score – Qualifying Score
Net Score = (Correct × 4) – (Incorrect × 1)
Space for Rough Work
MATHEMATICS Speed
Complex Numbers TEST
And Quadratic Equations
No. of Questions
30
Maximum Marks
120
Time
1 Hour
64
Chapter-wise

GENERAL INSTRUCTIONS
• This test contains 30 MCQ's. For each question only one option is correct. Darken the correct circle/ bubble in the
Response Grid provided on each page.
• You have to evaluate your Response Grids yourself with the help of solutions provided at the end of this book.
• Each correct answer will get you 4 marks and 1 mark shall be deduced for each incorrect answer. No mark will be given/
deducted if no bubble is filled. Keep a timer in front of you and stop immediately at the end of 60 min.
• The sheet follows a particular syllabus. Do not attempt the sheet before you have completed your preparation for that
syllabus.
• After completing the sheet check your answers with the solution booklet and complete the Result Grid. Finally spend time
to analyse your performance and revise the areas which emerge out as weak in your evaluation.

1. The smallest positive integer n for which


(1 + i)2n = (1 – i)2n is : 3. If a, b be the roots of the equation x 2 - px + q = 0 , then
(a) 1 (b) 2 the equation whose roots are
(c) 3 (d) 4
2. If a and b be the roots of x2 + px + q = 0, then æ a2 ö æ b2 ö
(wa + w 2b)(w 2 a + wb) a2ç - b ÷ and b 2 ç - a ÷ is
is equal to (w, w2 are complex cube ç b ÷ ç a ÷
a2 b2 è ø è ø
+
b a (a) qx 2 - p(p 2 - q)(p 2 - 4q )x - p 2 q 2 (p 2 - 4q) = 0
roots of unity) px 2 - q (p 2 - p)(p 2 - 4q )x + p 2 q 2 (p 2 - 4q) = 0
(b)
q (c) px 2 - qx + p = 0
(a) - (b) a b
p
p (d) None of these
(c) - (d) w
q

RESPONSE GRID 1. 2. 3.

Space for Rough Work


EBD_7504
M-18 NTA JEE Main

4. If a and b be the values of x in m2 (x2 – x) + 2mx + 3 = 0 9. If z = x + iy is a variable complex number such that
and m1 and m2 be two values of m for which a and b are
z -1 p
a b 4 arg = then :
connected by the relation + = . Then the value of z +1 4
b a 3 (a) x2 – y2 – 2x = 1 (b) x2 + y2 – 2x = 1
2 2
(c) x + y – 2y = 1 (d) x2 + y2 + 2x = 1
m12 m 22 10. Let a > 0, b > 0 and c > 0. Then both the roots of the equation
+ is
m2 m1 ax2 + bx + c = 0
(a) 6 (b) 68 (a) are real and negative
(b) have negative real parts
3 68
(c) (d) - (c) are rational numbers
68 3 (d) None of these
5. If z1 = 3 + i 3 and z2 = 3 + i , then in which quadrant 11. Let z lies on the circle centred at the origin. If area of the
triangle whose vertices are z, wz and z+wz, where w is the
æ z1 ö
ç ÷ lies? cube root of unity is 4 3 sq. unit. Then radius of the circle
è z2 ø is :
(a) I (b) II (a) 1 unit (b) 2 units
(c) III (d) IV (c) 4 units (d) None of these
6. The root of the equation 2(1 + i ) x 2 - 4 ( 2 - i ) x - 5 - 3i = 0 12. For a complex number z, the minimum value of
which has greater modulus is | z | + | z – 2 | is
(a) 1 (b) 2
3 - 5i 5 - 3i
(a) (b) (c) 3 (d) None of these
2 2 13. The complex number z satisfying the equations
3-i
(c) (d) None of these | z | -4 =| z - i | = | z + 5i |= 0 , is
2
(cos q + i sin q ) 4 (a) 3 -i (b) 2 3 , 2i
7. Value of is
(cos q - i sin q) 3 (c) ,2 3 ∗ 2i (d) 0
(a) cos 5q + i sin 5q (b) cos 7q + i sin 7q 14. If a , b, g and a, b, c are complex numbers such that
(c) cos 4q + i sin 4q (d) cos q + i sin q a b c
a b g
2 + + = 1 +i and + + = 0 , then the value of
3 z a b c a b g
8. Number of solutions of the equation, z 3 + = 0 , where
z
a2 b2 g2
z is a complex number and | z |= 3 is + + is equal to
a2 b2 c2
(a) 2 (b) 3
(a) –1 (b) 2i
(c) 6 (d) 4
(c) 0 (d) +1

4. 5. 6. 7. 8.
RESPONSE
9. 10. 11. 12. 13.
GRID
14.

Space for Rough Work


Mathematics M-19

x2 - 4 x + 3 x2 - 4 x + 3
20. If p, q, r are non-zero real numbers, the two equation,
(
15. If 7 - 4 3 ) (
+ 7+4 3 ) = 14, then the 2 2
2 a 2 x 2 - 2 abx + b 2 = 0 an d p x + 3pqx + q = 0
2

value of x is given by have :


(a) 2, 2 ± 2 (b) 2 ± 3 , 3 (a) no common root
(c) 3 ± 2 , 2 (d) None of these (b) one common root if 2a 2 + b 2 = p 2 + q 2
(c) two common roots if 3pq = 2ab
16. If a, b be the roots of ax 2 + bx + c = 0 and g, d those of (d) two common roots if 3qb = 2 ap
lx 2 + mx + n = 0 , then the equation whose roots are ag + b d 21. The centre of a regular hexagon is at the point
and a d +bg is z = i. If one of its vertices is at 2 + i, then the adjacent
(a) a 2 l 2 x 2 - ablmx + b 2 l n + acm 2 - 4 acl n = 0 vertices of 2 + i are at the points

alx2 - ablmx + (a + b - c)(l + m - n) = 0 (a) 1± 2i (b) i + 1 ± 3


(b)
2 2 2 2 2 2 2
(c) a l x + (a + b )(l + m ) x - (a + b - c)(l + m - n) = 0 (c) 2 + i(1 ± 3) (d) 1 + i(1 ± 3)
(d) None of these 22. If a, b, c are real numbers a ¹ 0. If a, is a root of a2x2 + bx
æ -1 + - 3 ö
100
æ -1- - 3 ö
100 + c = 0, b is a root of a2x2 – bx – c = 0 and 0 < a < b, then the
17. ç ÷ +ç ÷ is equal to equation a2x2 + 2bx + 2c = 0 has a g root that always satisfies:
ç 2 ÷ ç 2 ÷
è ø è ø a ∗b a ,b
(a) 2 (b) zero (a) g = (b) g =
2 2
(c) – 1 (d) 1
3x 2 + 9 x + 17 (c) g = a (d) a < g < b
18. If x is real, the maximum value of is 23. If the roots of the equation (x – a) (x – b) + (x – b)
3x 2 + 9 x + 7
(x – c) + (x – c) (x – a) = 0 are equal, then a2 + b2 + c2 =
1 (a) a + b + c (b) 2a + b + c
(a) (b) 41
4 (c) 3abc (d) ab + bc + ca
17 1 + b + ai
(c) 1 (d)
7 24. If a + ib = 1, then the simplified form of is
1 + b - ai
æ 1 3 ö æ 3 + 4i ö (a) b + ai (b) a + bi
19. çè + ÷ç ÷ is equal to :
1 - 2i 1 + i ø è 2 - 4i ø (c) (1 + b)2 + a2 (d) ai
1 9 1 9
(a) + i (b) - i
2 2 2 2
1 9 1 9
(c) - i (d) + i
4 4 4 4

RESPONSE 15. 16. 17. 18. 19.


GRID 20. 21. 22. 23. 24.

Space for Rough Work


EBD_7504
M-20 NTA JEE Main

25. Let a, b, c, p, q be real numbers. Suppose a, b are the roots 27. If a, b are the roots of the equation ax2 + bx + c = 0 such
that b < a < 0, then the quadratic equation whose roots are
1 |a|, |b|, is given by
of the equation x2 + 2px + q = 0 and a, are the roots of the
b (a) |a| x2 + |b| x + |c| = 0 (b) ax2 – |b| x + c = 0
2
(c) |a| x – |b| x + |c| = 0 (d) a|x|2 + b|x| + |c| = 0
equation x2 + 2bx + c = 0, where b2 Ï (–1, 0, 1)
Statement-1: (p2 – q)(b2 – ac) ³ 0 28. If z = 2 + i, then ( z – 1) ( z – 5) + ( z – 1) (z – 5) is equal to
Statement-2: b ¹ pa or c ¹ qa (a) 2 (b) 7
(a) Statement -1 is true, Statement-2 is true; Statement -2 is (c) –1 (d) –4
a correct explanation for Statement-1 29. If a, b are the roots of the equation 2x2 + 6x + b = 0, (b < 0)
(b) Statement -1 is true, Statement-2 is true; Statement -2
a b
is NOT a correct explanation for Statement-1 then + is less than :
b a
(c) Statement -1 is false, Statement-2 is true
(d) Statement -1 is true, Statement-2 is false (a) 1 (b) –1
26. If w is a non-real cube root of unity, then (c) 2 (d) – 2

1 + 2w + 3w2 2 + 3w + 3w2 (cos x + i sin x) (cos y + i sin y)


+ is equal to 30. A + iB form of is equal to :
(cot u + i )(1 + i tan v)
2 + 3w + w2 3 + 3w + 2w2
(a) – 2 w (b) 2 w (a) sin u cos v [cos (x + y – u – v) + i sin (x + y – u – v)]
(c) w (d) 0 (b) sin u cos v [cos (x + y + u + v) + i sin (x + y + u + v)]
(c) sin u cos v [cos (x + y + u + v) – i sin (x + y – u + v)]
(d) None of these

RESPONSE 25. 26. 27. 28. 29.


GRID 30.

MATHEMATICS CHAPTERWISE SPEED TEST-64


Total Questions 30 Total Marks 120
Attempted Correct
Incorrect Net Score
Cut-off Score 37 Qualifying Score 55
Success Gap = Net Score – Qualifying Score
Net Score = (Correct × 4) – (Incorrect × 1)
Space for Rough Work
MATHEMATICS Speed
Linear Inequalities TEST
No. of Questions
30
Maximum Marks
120
Time
1 Hour
65
Chapter-wise

GENERAL INSTRUCTIONS
• This test contains 30 MCQ's. For each question only one option is correct. Darken the correct circle/ bubble in the
Response Grid provided on each page.
• You have to evaluate your Response Grids yourself with the help of solutions provided at the end of this book.
• Each correct answer will get you 4 marks and 1 mark shall be deduced for each incorrect answer. No mark will be given/
deducted if no bubble is filled. Keep a timer in front of you and stop immediately at the end of 60 min.
• The sheet follows a particular syllabus. Do not attempt the sheet before you have completed your preparation for that
syllabus.
• After completing the sheet check your answers with the solution booklet and complete the Result Grid. Finally spend time
to analyse your performance and revise the areas which emerge out as weak in your evaluation.

3 é5 + 5 é5 + 5 2ù
1. The solution set of the inequality | x + 2 | - | x - 1 |< x -
2
is (b) ê 2 , ¥ ) (d) ê , - ú
ëê êë 2 3 úû
æ9 ö æ 3ö 3. |2x – 3| < |x + 5|, then x belongs to
(a) ç , ¥÷ (b) ç - ¥, ÷
è2 ø è 2ø (a) (–3, 5) (b) (5, 9)

æ 3ö æ 3ö æ 2 ö æ 2ö
(c) ç - 2, - ÷ (d) ç - 1, ÷ (c) çè - , 8÷ø (d) çè -8, ÷ø
è 2ø è 2ø 3 3
4. If x satisfies the inequalities x + 7 < 2x + 3 and
2. If (y 2 + 5y + 3)(x 2 + x + 1) < 2x for all x Î R , then y lies 2x + 4 < 5x + 3, then x lies in the interval
in the interval (a) (– ¥, 3) (b) (1, 3)
æ 5 – 5 5 ∗ 5 ö÷ (c) (4, ¥)
ç ÷÷ (d) (–¥, –1)
(a) (–⁄ , 2] (b) ççç 2 , 2 ø÷
è

RESPONSE GRID 1. 2. 3. 4.

Space for Rough Work


EBD_7504
M-22 NTA JEE Main

5. The shaded region shown in the figure is given by the 10. If –7 < x < 18 and 9 < y < 20, then the range of x + y is:
inequations
(a) [2, 38] (b) (2, 38]
Y
(c) [2, 38) (d) (2, 38)
(0, 14) (19, 14) 11. The region represented by 2x + 3y – 5 £ 0 and
4x – 3y + 2 £ 0, is
(a) Not in first quadrant
X (b) Bounded in first quadrant
(15, 0) (c) Unbounded in first quadrant
(a) 14x + 5y ³ 70, y £ 14 and x – y ³ 5 (d) None of these
(b) 14x + 5y £ 70, y £ 14 and x – y ³ 5 12. The set of real values of x satisfying | x - 1 |£ 3 and
(c) 14x + 5y ³ 70, y ³ 14 and x – y ³ 5 | x - 1 |³ 1 is
(d) 14x + 5y ³ 70, y £ 14 and x – y £ 5
(a) [2, 4] (b) (-¥, 2] È [4, + ¥)
x 2 + 6x + 9
6. If log1 / 2 < - log 2 ( x + 1), then x lies in the (c) [-2, 0] È [2, 4] (d) None of these
2(x + 1)
13. IQ of a person is given by the formula
interval
(a) (-1, - 1 + 2 2 ) (b) (1 - 2 2 , 2) MA
IQ = ´100
(c) (-1, ¥) (d) None of these CA
7. The solution set of the inequality where, MA is mental age and CA is chronological age.
If 80 £ IQ £ 140 for a group of 12 years children, then the
| 9 x - 3 x +1 - 15 |< 2.9 x - 3x is range of their mental age is
(a) (-¥, 1) (b) (1, ¥) (a) 9.8 £ MA £ 16.8 (b) 10 £ MA £ 16
(c) 9.6 £ MA £ 16.8 (d) 9.6 £ MA £ 16.6
(c) (-¥, 1] (d) None of these
8. The region represented by the inequation system x, y ³ 0, 14. Find the range of values of x that satisfy the following
y £ 6, x + y £ 3, is system of in-equations.
(a) Unbounded in first quadrant –17 £ 3x + 10 £ –2;
(b) Unbounded in first and second quadrants
(c) Bounded in first quadrant –22 £ 5x + 13 £ 3 and
(d) None of these –19 £ 2x – 9 £ –3
9. Find all the positive integer valued solutions (x, y, z) of (a) (–5, –4) (b) [–5, –4]
the systems of inequalities (c) [–5, –3) (d) (–5, –3)
ì 3 x + 2 y - z = 4, 15. The vertex of common graph of inequalities 2x + y ³ 2 and
ï x – y £ 3, is
í 2 x - y + 2 z = 6,
ï x + y + z < 7. æ 5 4ö
î (a) (0, 0) (b) çè , - ÷ø
3 3
Determine how many such triplets exist?
æ5 4ö æ 4 5ö
(a) 0 (b) 1 (c) çè , ÷ (d) çè - , ÷ø
3 3ø 3 3
(c) 3 (d) None of these

5. 6. 7. 8. 9.
RESPONSE 10. 11. 12. 13. 14.
GRID
15.
Space for Rough Work
Mathematics M-23

16. Ankur appeared in an examination which has 5 subjects, 22. A vertex of a feasible region by the linear constraints
out of five, in four subjects he got 90, 70, 75, 65 marks 3x + 4y £ 18, 2x + 3y ³ 3 and x, y ³ 0, is
respectively. The minimum & maximum marks he should (a) (0, 2) (b) (4.8, 0)
score in fifth subject so that the average mark is greater
(c) (0, 3) (d) None of these
than or equal to 70 and less than or equal to 75 is
(a) 55, 75 (b) 55, 70 23. The true statement for the graph of inequations 3x + 2y £ 6
(c) 50, 75 (d) 50, 70 and 6x + 4y ³ 20, is
17. Given that a, b are two integers such that the positive integer (a) Both graphs are disjoint
solutions of the system of inequalities 9x – a ³ 0, and 8x – b < 0 (b) Both do not contain origin
are 1, 2, 3. Find the number of the ordered pairs (a, b).
(a) 72 (b) 75 (c) Both contain point (1, 1)
(c) 81 (d) None of these (d) None of these
18. The number of pairs of consecutive odd natural numbers
both of which are larger than 10, such that their sum is less x+3 + x
24. Solve for x, >1
than 40, is x+2
(a) 4 (b) 6 (a) x Î (–5, –2) È (–1, ¥) (b) x Î (5, 2) È (–1, ¥)
(c) 3 (d) 8
(c) x Î (5, 2) (d) x Î (–1, ¥)
19. The set of real values of x satisfying | x - 1 | -1 | £ 1 is
25. A vertex of bounded region of inequalities x ³ 0, x + 2y ³ 0
(a) [-1, 3] (b) [0, 2] and 2x + y £ 4, is
(c) [–1, 1] (d) None of these (a) (1, 1) (b) (0, 1)
20. The cost and revenue functions of a product are given by
(c) (3, 0) (d) (0, 0)
C(x) =2x + 80 and R(x) = 5x + 20 respectively, where x is the
number of items produced by the manufacture. How many x+2
items the manufacturer must sell to realize some profit? 26. The set of real values of x for which log 0.2 £ 1 is
x
(a) more than 20 (b) more than or equal to 20
(c) more than 25 (d) None of these
æ 5ù é5 ö
21. A man wants to cut three lengths from a single piece of (a) ç - ¥, - ú È (0, ¥) (b) ê 2 , ¥÷
board of length 91 cm. The second length is to be 3 cm è 2û ë ø
longer than the shortest and the third length is to be twice
as long as the shortest. The possible length of the shortest (c) (-¥, - 2) È [0, ¥ ) (d) none of these
board, if the third piece is to be at least 5 cm longer than the 27. The pairs of consecutive even positive integers, both of
second, is
which are larger than 5 such that their sum is less than 23,
(a) less than 8 cm
are
(b) greater than or equal to 8 cm but less than or equal
to 22 cm (a) (4, 6), (6, 8), (8, 10), (10, 12)
(c) less than 22 cm (b) (6, 8), (8, 10), (10, 12)
(d) greater than 22 cm (c) (6, 8), (8, 10), (10, 12), (12, 14)
(d) (8, 10), (10, 12)

16. 17. 18. 19. 20.


RESPONSE 21. 22. 23. 24.
GRID
25. 26. 27.
Space for Rough Work
EBD_7504
M-24 NTA JEE Main

28. Shaded region is represented by 29. In which quadrant, the bounded region for inequations
Y x + y £ 1 and x – y £ 1 is situated
x + y = 20
(0,20) (a) I, II (b) I, III
(c) II, III (d) All the four quadrants
C(10,16) æ 20 40 ö
Bç , ÷ 30. Given that the solution set for x of the inequality
è 3 3ø
2x + 5y = 80 2m + x 4mx - 1 3
£ is x ³ , find the value of the
3 2 4
X parameter m.
A(20,0) (40,0)
7 9
(a) 2x + 5y ³ 80, x + y £ 20, x ³ 0, y £ 0 (a) (b)
10 10
(b) 2x + 5y ³ 80, x + y ³ 20, x ³ 0, y ³ 0
(c) 2x + 5y £ 80, x + y £ 20, x ³ 0, y ³ 0 9
(c) (d) None of these
(d) 2x + 5y £ 80, x + y £ 20, x £ 0, y £ 0 11

RESPONSE 28. 29. 30.


GRID

MATHEMATICS CHAPTERWISE SPEED TEST-65


Total Questions 30 Total Marks 120
Attempted Correct
Incorrect Net Score
Cut-off Score 42 Qualifying Score 60
Success Gap = Net Score – Qualifying Score
Net Score = (Correct × 4) – (Incorrect × 1)
Space for Rough Work
MATHEMATICS Speed
Permutations and Combinations TEST
No. of Questions
30
Maximum Marks
120
Time
1 Hour
66
Chapter-wise

GENERAL INSTRUCTIONS
• This test contains 30 MCQ's. For each question only one option is correct. Darken the correct circle/ bubble in the
Response Grid provided on each page.
• You have to evaluate your Response Grids yourself with the help of solutions provided at the end of this book.
• Each correct answer will get you 4 marks and 1 mark shall be deduced for each incorrect answer. No mark will be given/
deducted if no bubble is filled. Keep a timer in front of you and stop immediately at the end of 60 min.
• The sheet follows a particular syllabus. Do not attempt the sheet before you have completed your preparation for that
syllabus.
• After completing the sheet check your answers with the solution booklet and complete the Result Grid. Finally spend time
to analyse your performance and revise the areas which emerge out as weak in your evaluation.

1. If all permutations of the letters of the word AGAIN are 3. The letters of the word MODESTY are written in all possible
arranged as in dictionary, then fiftieth word is orders and these words are written out as in a dictionary
then the rank of the word MODESTY is
(a) NAAGI (b) NAGAI
(a) 5040 (b) 720
(c) NAAIG (d) NAIAG
(c) 1681 (d) 2520
2. A boat is to be manned by eight men of whom 2 can only
row on bow side and 3 can only row on stroke side, the 4. The number of ways in which a mixed doubles game in
number of ways in which the crew can be arranged is tennis can be arranged from 5 married couples, if no husband
and wife play in the same game, is
(a) 4360 (b) 5760
(a) 46 (b) 54
(c) 5930 (d) None of these
(c) 60 (d) None of these

RESPONSE GRID 1. 2. 3. 4.

Space for Rough Work


EBD_7504
M-26 NTA JEE Main

5. ABCD is a convex quadrilateral. 3, 4, 5 and 6 points are 2


(a) 3 p 2 (p – 1) + 1 (b) 3 p (p – 1)
marked on the sides AB, BC, CD and DA respectively . The
number of triangles with vertices on different sides is (c) p 2 (4p – 3) (d) None of these
(a) 270 (b) 320 11. In a 12 - storey house ten people enter a lift cabin. It is
(c) 282 (d) 342 known that they will left in groups of 2, 3 and 5 people at
6. Let Tn denote the number of triangles which can be formed different storeys. The number of ways they can do so if the
lift does not stop to the second storey is
by using the vertices of a regular polygon of n sides. If
(a) 78 (b) 112
Tn+1 – Tn = 28, then n equals (c) 720 (d) 132
(a) 4 (b) 5 12. Given that n is odd, the number of ways in which three
(c) 6 (d) 8 numbers in A.P. can be selected from 1, 2, 3, ..... n is
7. If a denotes the number of permutations of x + 2 things
taken all at a time, b the number of permutations of x things (n - 1) 2 (n + 1) 2
(a) (b)
taken 11 at a time and c the number of permutations of x – 11 2 4
things taken all at a time such that a = 182 bc, then the value
of x will be (n + 1) 2 (n - 1) 2
(c) (d)
(a) 12 (b) 10 2 4
(c) 8 (d) 6 13. A meeting is to be addressed by 5 speakers A, B, C, D, E. In
how many ways can the speakers be ordered, if B must not
8. Given five line segments of length 2, 3, 4, 5, 6 units. Then precede A (immediately or otherwise)?
the number of triangles that can be formed by joining these
(a) 120 (b) 24
lines is
(a) 5C3 – 3 (b) 5C3 – 1 (c) 60 (d) 54 × 4
14. A contest consists of predicting the results (win, draw or
(c) 5C3 (d) 5C3 – 2
n
defeat) of 10 football matches. Then the number of ways in
9. The value of 2 [1.3.5...(2n – 3) (2n – 1)] is which one entry contains at least 5 correct answers is :
(2n)! (2n)! 4
(a) (b) 5
n! 2n (a) 10
3 - å
r =1
10
Cr 2 r 10
(b) 3 – å 10 Cr 210-r
r=0
n!
(c) (d) None of these 5 10
(2n )! 10 10
(c) å Cr (d) å C r 3r
10. There are three coplanar parallel lines. If any p points are r =1 r=6
taken on each of the lines, the maximum number of triangles
with vertices at these points is

RESPONSE 5. 6. 7. 8. 9.
GRID 10. 11. 12. 13. 14.

Space for Rough Work


Mathematics M-27

15. Statement-1 : The maximum number of points of intersection (a) 10 (b) 9


of 8 circles of unequal radii is 56. (c) 13 (d) 17
Statement-2 : The maximum number of points into which 4 20. A teaparty is arranged for 16 people along two sides of a
circles of unequal radii and 4 non coincident straight lines large table with 8 chairs on each side. Four men want to sit
intersect, is 50. on one particular side and two on the other side. The number
of ways in which they can be seated is
(a) Statement-1 is true, Statement-2 is true, Statement-2 is
6! 8!10! 8! 8!10!
a correct explanation for Statement -1 (a) (b)
4! 6! 4! 6!
(b) Statement -1 is true, Statement -2 is true ; Statement-2
is NOT a correct explanation for Statement - 1 8! 8! 6!
(c) (d) None of these
(c) Statement -1 is false, Statement -2 is true 6! 4!
(d) Statement - 1 is true, Statement- 2 is false 21. Five balls of different colours are to be placed in three boxes
16. The number of squares that can be formed on a chessboard of different sizes. Each box can hold all five balls. Find the
is number of ways in which we can place the balls in the boxes
(a) 64 (b) 160 (order is not considered in the box) so that no box remains
(c) 224 (d) 204 empty.
17. The number of triangles whose vertices are at the vertices (a) 100 (b) 75
of an octagon but none of whose sides happen to come
from the sides of the octagon is (c) 150 (d) 200
(a) 24 (b) 52 22. Assuming the balls to be identical except for difference in
(c) 48 (d) 16 colours, the number of ways in which one or more balls can
18. Six persons A, B, C, D, E and F are to be seated at a circular be selected from 10 white, 9 green and 7 black balls is :
table. The number of ways, this can be done if A must have
(a) 880 (b) 629
either B or C on his right and B must have either C or D on
his right is (c) 630 (d) 879
(a) 36 (b) 12 23. If nCr–1 = 28, nCr = 56 and nCr+1 = 70, then the value of r is
(c) 24 (d) 18 equal to
(a) 1 (b) 2
19. If m1 and m2 satisfy the relation
(c) 3 (d) 4

( m+3 Pm ) ,
m+ 5 11 24. The number of numbers greater than a million that can be
Pm +1 = (m -1) formed with the digits 2, 3, 0, 3, 4, 2 and 3 is
2
(a) 360 (b) 340
then m1 + m2 is equal to (c) 370 (d) None of these

RESPONSE 15. 16. 17. 18. 19.


GRID 20. 21. 22. 23. 24.

Space for Rough Work


EBD_7504
M-28 NTA JEE Main

25. There are 4 letters and 4 directed envelopes. The number of 28. How many arrangements can be made out of the letters of
ways in which all the letters can be put in wrong envelope the word “ MOTHER” taken four at a time so that each
is arrangement contains the letter ´M´?
(a) 9 (b) 4 (a) 240 (b) 120
(c) 5 (d) 12 (c) 60 (d) 360
26. If 12Pr = 11P6 + 6. 11P5 then r is equal to: 29. If the letters of the word KRISNA are arranged in all
(a) 6 (b) 5 possible ways and these words are written out as in a
(c) 7 (d) None of these dictionary, then the rank of the word KRISNA is
27. The number of values of r satisfying the equation (a) 324 (b) 341
39 (c) 359 (d) None of these
C 3r -1 - 39 C = 39 C - 39 C 3r is
r2 r 2 -1 30. A committee of 4 persons is to be formed from 2 ladies, 2 old
(a) 1 (b) 2 men and 4 young men such that it includes at least 1 lady, at
(c) 3 (d) 4 least 1 old man and at most 2 young men. Then the total
number of ways in which this committee can be formed is :
(a) 40 (b) 41
(c) 16 (d) 32

RESPONSE 25. 26. 27. 28. 29.


GRID 30.

MATHEMATICS CHAPTERWISE SPEED TEST-66


Total Questions 30 Total Marks 120
Attempted Correct
Incorrect Net Score
Cut-off Score 36 Qualifying Score 48
Success Gap = Net Score – Qualifying Score
Net Score = (Correct × 4) – (Incorrect × 1)
Space for Rough Work
MATHEMATICS Speed
Binomial Theorem TEST
No. of Questions
30
Maximum Marks
120
Time
1 Hour
67
Chapter-wise

GENERAL INSTRUCTIONS
• This test contains 30 MCQ's. For each question only one option is correct. Darken the correct circle/ bubble in the
Response Grid provided on each page.
• You have to evaluate your Response Grids yourself with the help of solutions provided at the end of this book.
• Each correct answer will get you 4 marks and 1 mark shall be deduced for each incorrect answer. No mark will be given/
deducted if no bubble is filled. Keep a timer in front of you and stop immediately at the end of 60 min.
• The sheet follows a particular syllabus. Do not attempt the sheet before you have completed your preparation for that
syllabus.
• After completing the sheet check your answers with the solution booklet and complete the Result Grid. Finally spend time
to analyse your performance and revise the areas which emerge out as weak in your evaluation.

1. If x = 9950 + 10050 and y = (101)50 then 3. If Pn denotes the product of the binomial coefficients in the
(a) x = y (b) x < y Pn +1
expansion of (1 + x)n, then P equals
(c) x > y (d) None of these n

C0 C2 C4 C6 n +1 nn
2. + + + + ........ = (a) (b)
1 3 5 7 n! n!
(n + 1) n (n + 1) n +1
2n +1 2n +1 - 1 (c) (d)
(a) (b) (n + 1) ! (n + 1) !
n +1 n +1
6

2n 4. The value of 50
C4 + å 56 - r C3 is
(c) (d) None of these r =1
n +1 55 55
(a) C4 (b) C3
56 56
(c) C3 (d) C4

RESPONSE GRID 1. 2. 3. 4.

Space for Rough Work


EBD_7504
M-30 NTA JEE Main

5. If C0, C1, C2, ......., C15 are binomial coefficients in 10. Coefficient of x25 in expansion of expression
C1 C2 C3 C15 50
(1 + x)15, then C + 2 C + 3 C + ..... + 15 C = å
50
Cr (2 x - 3)r (2 - x )50 - r is
0 1 2 14 r =0
(a) 60 (b) 120 (a) 50 C (b) – 50C
25 30
(c) 64 (d) 124 50 C 50C
(c) 30 (d) – 25
6. The value of the term independent of x in the expansion of
4 11. If (1 + x)2n = a0 + a1x + a2x2 + ..... + a2nx2n, then
æ x 2ö
ç 1 + - ÷ , x ¹ 0 is equal to
è 2 xø 1
(a) a0 + a2 + a4 + .... = (a + a + a + a + ....)
(a) 1 (b) – 6 2 0 1 2 3
(c) –5 (d) 6 (b) an+1 < an
7. The coefficient of x5 in (1 + 2x + 3x2 + ....)–7/2 is (c) an-3 = an+3
(a) 15 (b) 21 (d) All of these
(c) 12 (d) 30 12. One value of a for which the coefficients of the middle terms
8. A set contains (2n + 1) elements. If the number of subsets in the expansion of (1 + ax)4 and (1 – ax)6 are equal,
of this set which contain at most n elements is 4096, then -3
the value of n is is . Other value of ‘a’ is
10
(a) 6 (b) 15
(a) 0 (b) 1 (c) 2 (d) 3
(c) 21 (d) None of these
13. Number of ways of selection of 8 letters from 24 letters of
3
9. If x is so small that x and higher powers of x may be which 8 are a, 8 are b and the rest unlike, is given by
3 (a) 27 (b) 8.28
3
æ 1 ö (c) 10.27 (d) None of these
(1 + x) 2 - ç 1 + x÷
è 2 ø
neglected, then may be approximated 14. The expression
1
(1 - x ) 2 6
æ 2 ö
as ( 2 x 2 + 1 + 2 x 2 - 1)6 + ç ÷ is a
ç 2 2 ÷
è 2 x + 1 + 2 x - 1 ø
3 3
(a) 1 - x2 (b) 3 x + x2
8 8 polynomial of degree:
3 2 x 3 2 (a) 5 (b) 6
(c) - x (d) - x
8 2 8 (c) 7 (d) 8

RESPONSE 5. 6. 7. 8. 9.
GRID 10. 11. 12. 13. 14.

Space for Rough Work


Mathematics M-31

15. The value of n


r
æ 30 ö æ 30 ö æ 30 ö æ 30 ö æ 30 ö æ 30 ö æ 30 ö æ 30 ö 20. If (1 + x)n = å Cr x , then the value of
r =0
ç ÷ ç ÷ - ç ÷ ç ÷ + ç ÷ ç ÷ ..... + ç ÷ ç ÷
0 10 1
è øè ø è øè ø è øè ø 11 2 12 è 20 ø è 30 ø
æ nö C 0 + (C0 + C1 ) + (C 0 + C1 + C 2 ) + ......
is where ç ÷ = n Cr
è rø + (C0 + C1 + C2 + ... + Cn ) is
æ 30 ö æ 30 ö
(a) çç ÷÷ (b) çç ÷÷
è 10 ø è 15 ø (a) n . 2n -1 (b) (n + 2) . 2n

æ 60 ö æ 31ö (c) 2n (d) (n + 2) . 2n – 1


(c) çç ÷÷ (d) çç ÷÷
è 30 ø è10 ø 1
16. If 'n' is positive integer and three consecutive coefficient in 21. Let a = 3 223 + 1 and for all n ³ 3, let f (n) = nC0.an–1 – nC1
the expansion of (1 + x)n are in the ratio 6 : 33 : 110, then n is an–2 + nC2an–3 – ........ + (–1)n–1.nCn–1a0. If the value of
equal to : f (2007) + f (2008) = 9k, where k Î N, then find k.
(a) 9 (b) 6
(c) 12 (d) 16 (a) 2187 (b) 1987
11 (c) 3232 (d) 4187
17. If the coefficient of x in é ax 2 + 1 ù
7
equals the
êë bx úû 22. Find the value of 4nC + 4nC + 4nC + ...... + 4nC
0 4 8 4n

-7 é æ 1 öù
11 (a) (-1)n 22n-1 + 24n-2 (b) (-1)n 22n-1
coefficient of x in ê ax - ç ú , then a and b satisfy
ë è bx 2 ÷ø û
(c) (-1)n 24n-1 (d) None of these
the relation
(a) a – b = 1 (b) a + b = 1 23. The sum of the series
a 20 20 20 20 20
(c) =1 (d) ab = 1 C0 - C1 + C2 - C3 + ..... -..... + C10 is
b
18. If 79 + 97 is divided by 64 then the remainder is (a) 0 (b) 20
C10
(a) 0 (b) 1
(c) 2 (d) 63 1 20
(c) - 20 C10 C10(d)
19. For natural numbers m, n if (1 - y )m (1 + y ) n 2
24. If 7103 is divided by 25, then the remainder is
= 1 + a1 y + a2 y 2 + ¼ and a1 = a2 = 10, then (m, n) is
(a) 20 (b) 16
(a) (20, 45) (b) (35, 20)
(c) (45, 35) (d) (35, 45) (c) 18 (d) 15

RESPONSE 15. 16. 17. 18. 19.


GRID 20. 21. 22. 23. 24.

Space for Rough Work


EBD_7504
M-32 NTA JEE Main

25. If x is very small in magnitude compared with a, then


27. The coefficien t of xn in the expansion of
1 1
æ a ö2
æ a ö2 (1 - 9x + 20x 2 ) - 1 is
ç ÷ +ç ÷ can be approximately equal to
èa+xø èa-xø (b) 5 n + 1 – 4 n + 1
(a) 5n – 4 n
1 x x
(a) 1+ (b) (c) 5n –1 – 4n –1 (d) None of these
2 a a
28. If T0 , T1 , T2 ....Tn represent the terms in the expansion of
3 x2 3 x2 (x + a)n, then (T0 –T2 + T4 – .......)2 + (T1 – T3 + T5 – .....)2 =
(c) 1 + (d) 2+
4 a2 4 a2
(a) ( x2 + a2 ) (b) ( x 2 + a 2 )n
26. If C0, C1, C2, ......, Cn be the coefficients in the expansion of
2 2 . C 0 2 3 . C1 2 n + 2.C n (c) ( x 2 + a 2 )1/ n (d) ( x 2 + a 2 )-1/ n
(1 + x)n, then + + ..... + is 29. n
The coefficient of x in the polynomial
1.2 2.3 (n + 1) (n + 2)
equal to ( x + n C 0 )( x + 3 . n C1 )( x + 5 . n C 2 ) .... (x+(2n + 1) nCn) is
(a) n . 2n (b) n . 2n + 1
3n + 1 - 2n - 5 3n + 2 - 2n - 5 n
(a) (b) (c) (n + 1) . 2 (d) n . 2n + 1
(n + 1) (n + 2) (n + 1) (n + 2) 30. In the expansion of (1 + x)18 , if the coefficients of (2r + 4)th
and (r – 2)th terms are equal, then the value of r is :
3 n + 2 + 2n - 5 (a) 12 (b) 10
(c) (d) None of these
(n + 1) (n + 2) (c) 8 (d) 6

RESPONSE 25. 26. 27. 28. 29.


GRID 30.

MATHEMATICS CHAPTERWISE SPEED TEST-67


Total Questions 30 Total Marks 120
Attempted Correct
Incorrect Net Score
Cut-off Score 38 Qualifying Score 55
Success Gap = Net Score – Qualifying Score
Net Score = (Correct × 4) – (Incorrect × 1)

Space for Rough Work


MATHEMATICS Speed
Sequences and Series TEST
No. of Questions
30
Maximum Marks
120
Time
1 Hour
68
Chapter-wise

GENERAL INSTRUCTIONS
• This test contains 30 MCQ's. For each question only one option is correct. Darken the correct circle/ bubble in the
Response Grid provided on each page.
• You have to evaluate your Response Grids yourself with the help of solutions provided at the end of this book.
• Each correct answer will get you 4 marks and 1 mark shall be deduced for each incorrect answer. No mark will be given/
deducted if no bubble is filled. Keep a timer in front of you and stop immediately at the end of 60 min.
• The sheet follows a particular syllabus. Do not attempt the sheet before you have completed your preparation for that
syllabus.
• After completing the sheet check your answers with the solution booklet and complete the Result Grid. Finally spend time
to analyse your performance and revise the areas which emerge out as weak in your evaluation.

1. If a 2 , b 2 , c 2 are in A.P. consider two statements n n +1


(a) (b)
n +1 n
1 1 1
(i) , , are in A.P.. n +1 2n + 1
b+c c+a a +b (c) (d)
n+2 n+2
a b c 3. If the A.M. between a and b is m times their H.M. then
(ii) , , are in A.P., then
b+c c+a a +b a: b=
(a) (i) and (ii) both correct
(b) (i) and (ii) both incorrect
(a) m + m -1 : m - m -1
(c) (i) correct (ii) incorrect (b) m - m - 1: m + 1 + m - 1
(d) (i) incorrect (ii) correct
2. An A.P., a G.P., and a H.P. have a and b for their first two (c) m + m +1: m - m + 1
2n + 2 2n + 2 (d) None of these
b -a
terms. Their (n + 2)th terms will be in G.P. if =
2n
ab(b - a 2n )

RESPONSE GRID 1. 2. 3.

Space for Rough Work


EBD_7504
M-34 NTA JEE Main

4. The sum of each of two sets of three terms in A.P. is 15. The
1.3 1.3.5
common difference of the first set is greater than that of the 9. The sum of the series 1 + + + ........¥ is
second by 1 and the ratio of the products of the terms in the 6 6.8
first set and that of the second set is 7 : 8. The ratio of the (a) 1 (b) 0
smallest terms in two sets of terms is (c) ¥ (d) 4
2 1
10. If S1, S2, S3, ...., Sn are the sum of infinite geometric series
3 11
(a) or (b) or whose first terms are 1, 2, 3, ..., n and whose comon ratios
4 12 3 2
1 1 1 1
2
or
3 are 2 , 3 , ,..., + respectively, then the value of
(c) (d) None of these 4 (n 1)
3 4
5. The 20th terms of the series 2 + 3 + 5 + 9 + 16 +.......is S12 + S 22 + S32 + ... + S22 n -1 is equal to
(a) 950 (b) 975
(c) 990 (d) 1010 1
(a) [ n ( 2 n + 1)( 4 n + 1) - 3]
6. Sum of the series 3
1 2 3 1
+ + + .... (b) [n (2n + 1)(4n + 1) + 3]
1 + 1 + 1 1 + 2 + 2 1 + 3 + 34
2 4 2 4 2
3
up to n terms is equal to 1
2 2 (c) [n (2 n - 1)(4n + 1) - 3]
n + n -1 n +n 3
(a) 2 (b)
2( n + n + 1) 2( n 2 + n + 1) (d) None of these
11. If a1, a2, a3,...., an, .... are in A.P. such that a4 – a7 + a10 = m,
n2 - n + 1 n2 - n then the sum of first 13 terms of this A.P., is :
(c) (d)
n2 + n + 1 2( n 2 + n + 1) (a) 10 m (b) 12 m
7. The sum to n terms of the series (c) 13 m (d) 15 m
2 + 5 +14 + 41 + ........ is 12. If a, b, c are in G.P. and x, y are the arithmetic means between
(a) 3n -1 + 8n - 3 (b) 8.3n + 4n - 8 a c
a, b, and b, c respectively, then + is equal to
x y
8
(c) 3n +1 + n + 1 (d) None of these (a) 0 (b) 1
3
8. If x = 1 + a + a2 + ....................to infinity and 1
y = 1 + b + b2 + ...................to infinity, where a, b are proper (c) 2 (d)
2
fractions, then 1 + ab + a2b2 + .....to infinity is equal : 13. Given a sequence of 4 numbers, first three of which are in
xy xy G.P. and the last three are in A.P. with common difference six.
(a) (b)
x + y -1 x - y -1 If first and last terms of this sequence are equal, then the
xy xy last term is :
(c) (d) (a) 16 (b) 8
x - y +1 x + y +1
(c) 4 (d) 2

RESPONSE 4. 5. 6. 7. 8.
GRID 9. 10. 11. 12. 13.

Space for Rough Work


Mathematics M-35

14. If the sum to infinity of the series, 1 + 4x + 7x2 + 10x3 +........, 1 3


is 35/16, where | x | < 1, then x equals to (a) log 2 (b) log
2 5
(a) 19/7 (b) 1/5
(c) 1/4 (d) None of these 5 1 5
(c) log (d) log
3 5 7 3 2 3
15. The sum + + + .... upto 11 terms is:
12 2
1 +2 2
1 + 22 + 32
2 21. The sum of n terms of two arithmetic series are in the ratio
2n + 3 : 6n + 5, then the ratio of their 13th terms is
7 11 (a) 53 : 155 (b) 27 : 87
(a) (b) (c) 29 : 83 (d) 31 : 89
2 4
22. Let a, b, c, be in A.P. with a common difference d. Then
11 60
(c)
2
(d)
11 e1 / c , e b / ac , e1 / a are in :
16. If x > 1, y > 1, z > 1 are in G.P. then (a) G.P. with common ratio ed
(b) G.P with common ratio e1/d
1 1 1
, , are in : (c) G.P. with common ratio e d /( b -d )
2 2
1 + log x 1 + log y 1 + log z
(d) A.P.
(a) A.P. (b) H.P.
23. If the ratio of H.M. and G.M. of two quantities is 12 : 13 then
(c) G..P. (d) None of these the ratio of the numbers is
17. If p, q, r are in A.P., a is G.M. between p and q and b is G.M. (a) 1 : 2 (b) 2 : 3
between q and r, then a2, q2, b2 are in (c) 3 : 4 (d) None of these
(a) G.P. (b) A.P.
24. If pth, qth and rth terms of an A.P. are equal to corresponding
(c) H.P (d) None of these
terms of a G.P. and these terms are respectively x, y, z, then
18. If S, P and R are the sum, product and sum of the reciprocals
of n terms of an increasing G.P respectively and S n = Rn.Pk, xy–z. yz–x. zx–y equals
then k is equal to (a) 0 (b) 1
(a) 1 (b) 2 (c) 2 (d) None of these
(c) 3 (d) None of these
19. If a, b, c, d are in G.P. then
25. For the series å t n , S n = 3t n - 2,
(a) a + b, b + c, c + d are in G.P. n n
(b) (b – c)2 + (c – a)2 + (d – b)2 = (a – d)2
(c) (a2 + b2 + c2) (b2 + c2 + d2) = (ab + bc + cd)2
where Sn = å t n . Then å Sn
n =1 n =1
(d) All are correct
20. The sum of the infinite series æ 3n ö
(a) 6ç - 1÷ - 2n (b) 6 n - 1 - 2n
ç 2n ÷
è ø
1 æ 1 1ö 1 æ 1 1 ö 1æ 1 1ö
ç + ÷- + + + - ... is equal to
2 è 3 4 ø 4 çè 32 4 2 ÷ø 6 çè 33 43 ÷ø æ 2n ö
(c) 2n - 6 ç + 1÷ (d) None of these
ç 3n ÷
è ø

14. 15. 16. 17. 18.


RESPONSE
19. 20. 21. 22. 23.
GRID
24. 25.
Space for Rough Work
EBD_7504
M-36 NTA JEE Main

26. If a1, a2, ........an+1 are in A.P. then


4
28. After striking the floor a certain ball rebounds th of its
1 1 1 5
+ + .......... + is
a1a2 a2 a3 an an +1 height from which it has fallen. The total distance that the
ball travels before coming to rest if it is gently released from
n -1 1 a height of 120m is
(a) (b)
a1an +1 a1an +1 (a) 960 m (b) 1000 m
(c) 1080 m (d) infinite
n +1 n 29. A G.P. consists of an even number of terms. If the sum of all
(c) a1an +1
(d) a1an +1 the terms is 5 times the sum of terms occupying odd places,
27. The value of x + y + z is 15 if a, x, y, z, b are in A.P. while the then the common ratio is
(a) 5 (b) 1
1 1 1 5 (c) 4 (d) 3
value of x + y + z is 3 if a, x, y, z, b are in H.P. Then the
If loge 5, loge (5x – 1) and loge æç 5 x - ö÷ are in A.P then the
11
value of a and b are 30.
è 5ø
(a) 2 and 8 (b) 1 and 9
(c) 3 and 7 (d) None of these values of x are
(a) log5 4 and log5 3 (b) log3 4 and log4 3
(c) log3 4 and log3 5 (d) log5 6 and log5 7

RESPONSE 26. 27. 28. 29. 30.


GRID

MATHEMATICS CHAPTERWISE SPEED TEST-68


Total Questions 30 Total Marks 120
Attempted Correct
Incorrect Net Score
Cut-off Score 38 Qualifying Score 55
Success Gap = Net Score – Qualifying Score
Net Score = (Correct × 4) – (Incorrect × 1)

Space for Rough Work


MATHEMATICS Speed
Straight Lines and Pair of Straight Lines TEST
No. of Questions
30
Maximum Marks
120
Time
1 Hour
69
Chapter-wise

GENERAL INSTRUCTIONS
• This test contains 30 MCQ's. For each question only one option is correct. Darken the correct circle/ bubble in the
Response Grid provided on each page.
• You have to evaluate your Response Grids yourself with the help of solutions provided at the end of this book.
• Each correct answer will get you 4 marks and 1 mark shall be deduced for each incorrect answer. No mark will be given/
deducted if no bubble is filled. Keep a timer in front of you and stop immediately at the end of 60 min.
• The sheet follows a particular syllabus. Do not attempt the sheet before you have completed your preparation for that
syllabus.
• After completing the sheet check your answers with the solution booklet and complete the Result Grid. Finally spend time
to analyse your performance and revise the areas which emerge out as weak in your evaluation.

1. If (– 4, 5) is one vertex and 7x – y + 8 = 0 is one diagonal of a 2 2 2


square, then the equation of second diagonal is æ 15 ö æ 10 ö æ 6 ö
ç ÷ +ç ÷ =ç ÷ , then the equation of the line
(a) x + 3y = 21 (b) 2x – 3y = 7 è AB ø è AC ø è AD ø
(c) x + 7y = 31 (d) 2x + 3y = 21 is
2. Two lines are given by (x – 2y)2 + k (x – 2y) = 0. The value of (a) 2x + 3y + 22 = 0 (b) 5x – 4y + 7 = 0
k, so that the distance between them is 3, is : (c) 3x – 2y + 3 = 0 (d) None of these
(a) k = 0 (b) k = ±3 5 4. The number of lines that are parallel to 2x + 6y + 7 =0 and
have an intercept of length 10 between the coordinate axes
(c) k = ± 5 (d) k = 3 is
3. A line through A (– 5, – 4) meets the line x + 3y + 2 = 0,
(a) 1 (b) 2
2x + y + 4 = 0 and x – y – 5 = 0 at B, C and D respectively. If
(c) 4 (d) Infinitely many

RESPONSE GRID 1. 2. 3. 4.

Space for Rough Work


EBD_7504
M-38 NTA JEE Main

5. The distance of the point (1, 2) from the line x + y + 5 = 0 10. The straight line y = x – 2 rotates about a point where it cuts
measured along the line parallel to 3x – y = 7 is equal to the x-axis and becomes perpendicular to the straight line
(a) (b) 40 ax + by + c = 0. Then its equation is
4 10
(a) ax + by + 2a = 0 (b) ax – by – 2a = 0
(c) 40 (d) 10 2 (c) bx + ay – 2b = 0 (d) ay – bx + 2b = 0
6. If p1, p2 are the lengths of the normals drawn from the origin 11. The number of possible straight lines, passing through
on the lines x cos q + y sin q = 2a cos 4q and (2, 3) and forming a triangle with coordinate axes, whose
x sec q + y cosec q = 4a cos 2q area is 12 sq. units, is
(a) 1 (b) 2
respectively, and mp12 + np22 = 4a 2 . Then
(c) 3 (d) 4
(a) m = 1, n = 1 (b) m = 1, n = 4 12. The slopes of the lines represented by
(c) m = 4, n = 1 (d) m = 1, n = – 1 x2 + 2hxy + 2y2 = 0 are in the ratio 1 : 2, then h equals
7. For what value of ‘p’ , y2 + xy + px2 – x – 2y + p = 0 represent
2 straight lines : 1 3
(a) ± (b) ±
2 2
2
(a) 2 (b)
3 (c) ±1 (d) ±3
13. The distance of the line 2x + y = 3 from the point (–1, 3) in the
1 1 direction whose slope is 1 is
(c) (d)
4 2
2 2
8. One vertex of an equilateral triangle is (2,3) and the equation (a) (b)
of line opposite to the vertex is x + y = 2, then the equation 3 3
of remaining two sides are
2 2 2 5
(a) y – 3 = (2 ± 3 ) (x – 2) (b) y + 3 = (2 ± 3 ) (x + 2) (c) (d)
3 3
(c) y + 3 = ((3 ± 2 ) (x +2) (d) y – 3 = (3 ± 2 ) (x – 2) 14. The equation of the straight line, the portion of which
9. The point on the line x + y = 4 which lie at a unit distance intercepted between the coordinate axes being divided by
from the line 4x + 3y = 10, are the point (–5, 4) in the ratio 1 : 2, is
(a) (3, 1), (– 7, 11) (b) (3, 1), (7, 11) (a) 8x + 5y = 60 (b) 8x – 5y = 60
(c) (–3, 1), (–7, 11) (d) (1, 3), (–7, 11) (c) –8x + 5y = 60 (d) None of these

RESPONSE 5. 6. 7. 8. 9.
GRID 10. 11. 12. 13. 14.

Space for Rough Work


Mathematics M-39

15. The reflection of the point (4, –13) in the line 5x + y + 6 = 0 , is (a) x - 3y = 0 (b) 3x- y =0
(a) (–1, –14) (b) (3, 4)
(c) (1, 2) (d) (–4, 13) (c) x + 3 y = 0 (d) 3x+ y =0
16. The combined equation of the pair of lines through the point
(1, 0) an d parallel to th e lines represented by 20. If the image of point P(2, 3) in a line L is Q(4, 5), then the
image of point R(0, 0) in the same line is:
2 x 2 - xy - y 2 = 0 is (a) (2,2) (b) (4, 5)
(c) (3, 4) (d) (7, 7)
(a) 2 x 2 - xy - y 2 - 4 x - y = 0 21. The coordinates of a point which is at +3 distance from
2 2 points (1, –3) of line 2x + 3y + 7 = 0 is
(b) 2 x - xy - y - 4 x + y + 2 = 0
æ 9 6 ö æ 9 9 ö
(c) 2 x 2 + xy + y 2 - 2 x + y = 0 (a) çè1 - ,- 3+ ÷ (b) çè 1 + , 1- ÷
13 13 ø 13 13 ø
(d) None of these
17. P is a point on either of the two lines y - 3 | x |= 2 at a æ 6 6 ö æ 9 6 ö
(c) çè 3 - , 3+ ÷ (d) çè1 + , -3- ÷
distance of 5 units from their point of intersection. The 13 13 ø 13 13 ø
coordinates of the foot of the perpendicular from P on the 22. If one of the diagonals of a square is along the line x = 2y
bisector of the angle between them are and one of its vertices is (3, 0), then its sides through this
æ 4+5 3 ö æ 4-5 3 ö vertex are given by the equations
ç 0, ÷ ç 0, ÷
(a) ç 2 ÷ø or ç 2 ÷ø depending on which the (a) y - 3x + 9 = 0, 3y + x - 3 = 0
è è
point P is taken (b) y + 3x + 9 = 0, 3y + x - 3 = 0
æ 4+5 3 ö (c) y - 3x + 9 = 0, 3y - x + 3 = 0
ç 0, ÷
(b) ç 2 ÷ø (d) y - 3x + 3 = 0, 3y + x + 9 = 0
è
23. Given a family of lines a(2x + y + 4) + b(x – 2y – 3) = 0, the
æ 4-5 3 ö
ç 0, ÷ number of lines belonging to the family at a distance 10
(c) ç 2 ÷ø
è from P(2, –3) is
æ5 5 3ö (a) 0 (b) 1
ç , ÷ (c) 2 (d) 4
(d) ç2 2 ÷
è ø 24. The line parallel to the x- axis and passing through the
18. The distance between the parallel lines intersection of the lines ax + 2by + 3b = 0 and
bx – 2ay – 3a = 0, where (a, b) ¹ (0, 0) is
9 x 2 – 6 xy + y 2 + 18 x - 6 y + 8 = 0 is
3
2 1 (a) below the x - axis at a distance of from it
2
(a) (b)
10 10 2
(b) below the x - axis at a distance of from it
3
4
(c) (d) None of these 3
10 (c) above the x - axis at a distance of from it
2
19. Equation of the hour hand at 4 O' clock is
2
(d) above the x - axis at a distance of from it
3

RESPONSE 15. 16. 17. 18. 19.


GRID 20. 21. 22. 23. 24.

Space for Rough Work


EBD_7504
M-40 NTA JEE Main

25. The equation 28. The points (1,3) and (5,1) are two opposite vertices of a
rectangle. The other two vertices lie on the line y = 2x + c,
8 x 2 + 8 xy + 2 y 2 + 26 x + 13 y + 15 = 0 represents a pair of
then one of the remaining vertices is
straight lines. The distance between them is (a) (4,4) (b) (2,2)
(a) 7/ 5 (b) 7/2 5 (c) (0,2) (d) (4,2)
29. (sin q, cos q) and (3, 2) lies on the same side of the line
(c) 7 /5 (d) None of these x + y = 1, then q lies between
26. A straight line L through the point (3, –2) is inclined at an (a) (0, p/2) (b) (0, p)
(c) (p/4, p/2) (d) (0, p/4)
angle 60° to the line 3x + y = 1. If L also intersects the
30. The perpendicular distance between the straight lines
x-axis, then the equation of L is 6x + 8y + 15 = 0 and 3x + 4y + 9 = 0 is
(a) y + 3x + 2 - 3 3 = 0 (b) y - 3x + 2 + 3 3 = 0 3 3
(a) units (b) unit
3 y - x + 3 + 2 3 = 0 (d) 2 10
(c) 3y + x - 3 + 2 3 = 0
27. The equation of a straight line, which passes through the 3 2
point (a, 0) and whose perpendicular distance from the point (c) unit (d) unit
4 7
(2a, 2a) is a, is
(a) 3x – 4y – 3a = 0 (b) x – a = 0
(c) both (a) and (b) (d) Neither of (a) and (b)

RESPONSE 25. 26. 27. 28. 29.


GRID 30.

MATHEMATICS CHAPTERWISE SPEED TEST-69


Total Questions 30 Total Marks 120
Attempted Correct
Incorrect Net Score
Cut-off Score 38 Qualifying Score 55
Success Gap = Net Score – Qualifying Score
Net Score = (Correct × 4) – (Incorrect × 1)

Space for Rough Work


MATHEMATICS Speed
Conic Sections TEST
No. of Questions
30
Maximum Marks
120
Time
1 Hour
70
Chapter-wise

GENERAL INSTRUCTIONS
• This test contains 30 MCQ's. For each question only one option is correct. Darken the correct circle/ bubble in the
Response Grid provided on each page.
• You have to evaluate your Response Grids yourself with the help of solutions provided at the end of this book.
• Each correct answer will get you 4 marks and 1 mark shall be deduced for each incorrect answer. No mark will be given/
deducted if no bubble is filled. Keep a timer in front of you and stop immediately at the end of 60 min.
• The sheet follows a particular syllabus. Do not attempt the sheet before you have completed your preparation for that
syllabus.
• After completing the sheet check your answers with the solution booklet and complete the Result Grid. Finally spend time
to analyse your performance and revise the areas which emerge out as weak in your evaluation.

1. If from any point P, tangents PT, PT¢ are drawn to two given 3. A line meets the co-ordinate axes in A and B. A circle is,
circles with centres A and B respectively; and if PN is the circumscribed about the triangle OAB. If the distances from
perpendicular from P on th eir radical axis, then A and B of the tangent to the circle at the origin be m and n,
then the diameter of the circle is
PT 2 - PT ' 2 =
(a) PN. AB (b) 2PN. AB (a) m(m + n) (b) m + n
(c) 4PN. AB (d) None of these
2. The parametric form of equation of the circle x2 + y2 – 6x (c) mn (d) m2 + n 2
+ 2y – 28 = 0 is
4. The angle between the tangents drawn from the origin to
(a) x = -3 + 38 cos q, y = -1 + 38 sin q the parabola y2 = 4a (x – a) is
(b) x = 28 cos q, y = 28 sin q (a) 90° (b) 30°
(c) x = -3 - 38 cos q, y = 1 + 38 sin q (c) tan –1
1
(d) 45°
2
(d) x = 3 + 38 cos q, y = -1 + 38 sin q

RESPONSE GRID 1. 2. 3. 4.

Space for Rough Work


EBD_7504
M-42 NTA JEE Main

5. The distance between the foci of an ellipse is 10 and its


latus rectum is 15. Its equation referred to its axes as axes of 10. AB is a focal chord of x 2 - 2 x + y - 2 = 0 whose focus is S.
coordinates is
If AS = l1 . then BS is equal to
(a) 3x2 + 4y2 = 300 (b) 2x2 + y2 = 50
2 2
(c) 10x + 15y = 300 (d) None of these
6. Two points P and Q are taken on the line joining the points 4l1
(a) l 1 (b)
A(0, 0) and B(3a, 0) such that AP = PQ = QB. Circles are 4l1 - 1
drawn on AP, PQ and QB as diameters. The locus of the
point S, the sum of the squares of the tangents from which l1 2l1
to the three circles is equal to b2, is (c) 4l1 - 1 (d)
4l1 - 1
(a) x 2 + y 2 - 3ax + 2a 2 - b 2 = 0
11. The point ([P + 1], [P]) (where [x] is the greatest integer less
2 2 2 2 than or equal to x), lying inside the region bounded by the
(b) 3( x + y ) - 9ax + 8a - b = 0
circle x2 + y2 – 2x – 15 = 0 and x2 + y2 – 2x – 7 = 0, then
(c) x 2 + y 2 - 5ax + 6a 2 - b 2 = 0 (a) P Î [–1, 0) È [0, 1) È [1, 2)
2 2 2 (b) P Î [–1, 2) – {0, 1}
(d) x + y - ax - b = 0
7. The coordinates of the middle point of the chord which the (c) P Î (–1, 2)
circle x2 + y2 + 4x – 2y – 3 = 0 cuts off on the line y = x + 2, are (d) None of these
æ -3 1 ö æ3 1ö 12. A line is drawn through the point P(3, 11) to cut the circle
(a) ç , ÷ (b) ç , ÷
è 2 2ø è2 2ø x2 + y2 = 9 at A and B. Then PA × PB is equal to
(a) 9 (b) 121
æ - 3 -1 ö æ 3 -1 ö
(c) ç , ÷ (d) ç , ÷ (c) 205 (d) 139
è 2 2 ø è2 2 ø
8. A hyperbola having the transverse axis of length 2 sin q, is 13. Let z = 1 – t + i t 2 + t + 2 , where t is a real parameter. The
confocal with the ellipse 3x2 + 4y2 = 12. Then its equation is
(a) x2 cosec2 q – y2 sec2 q = 1 locus of z in the argand plane is
(b) x2 sec2 q – y2 cosec2 q = 1 (a) an ellipse (b) hyperbola
(c) x2 sin2 q – y2 cos2 q = 1 (c) a straight line (d) None of these
(d) x2 cos2 q – y2 sin2 q = 1
9. If three points E, F, G are taken on the parabola y2 = 4ax so 14. The conic represented by the equation ax + by = 1 is
that their ordinates are in G.P., then the tangents at E and G
intersect on the (a) ellipse (b) hyperbola
(a) directrix (b) axis (c) parabola (d) None of these
(c) ordinate of F (d) tangent at F

RESPONSE 5. 6. 7. 8. 9.
GRID 10. 11. 12. 13. 14.

Space for Rough Work


Mathematics M-43

15. What is the area of the greatest rectangle that can be 20. If a circle passes through the point (a, b) and cuts the circle
2 2
x y x 2 + y 2 = 4 orthogonally, then the locus of its centre is
inscribed in the ellipse 2 + 2 = 1 ?
a b
(a) ab (b) 2 ab (a) 2ax - 2by - (a 2 + b2 + 4) = 0
(c) ab/2 (d) ab
(b) 2ax + 2by - (a 2 + b2 + 4) = 0
2 2 2
16. The line 3x + 2y + 1 = 0 meets the hyperbola 4x - y = 4a
in the points P and Q. The coordinates of the point of (c) 2ax - 2by + (a 2 + b2 + 4) = 0
intersection of the tangents at P and Q are
(d) 2ax + 2by + (a 2 + b2 + 4) = 0
(a) (-3a 2 , 8a 2 ) (b) (3a 2 , 8a 2 )
21. If a variable point P on an ellipse of eccentricity e is joined
(c) (3a 2 , - 8a 2 ) (d) None of these to the foci S1 and S2 then the incentre of the triangle PS1S2
17. The lengths of the tangent drawn from any point on the lies on
circle 15x 2 + 15y 2 - 48x + 64 y = 0 to the two circles (a) The major axis of the ellipse
(b) The circle with radius e
5x2 + 5y2 – 24x + 32y + 75 = 0 and 5x2 + 5y2 – 48x + 64y + 300
= 0 are in the ratio of
(a) 1 : 2 (b) 2 : 3 3 + e2
(c) Another ellipse of eccentricity
(c) 3 : 4 (d) None of these 4
18. The equation of the parabola whose focus is (0, 0) and the (d) None of these
tangent at the vertex is x – y + 1 = 0 is
22. If the coordinates of four concyclic points on the
(a) x 2 + y 2 + 2xy - 4x + 4y - 4 = 0
rectangular hyperbola xy = c2 are (ct i , c / t i ), i = 1,2,3,4 then
2
(b) x - 4x + 4y - 4 = 0
(a) t 1t 2 t 3 t 4 = -1 (b) t 1t 2 t 3 t 4 = 1
(c) y 2 - 4x + 4y - 4 = 0
(c) t 1t 3 = t 2 t 4 (d) t1 + t 2 + t 3 + t 4 = c 2
2 2
(d) 2x + 2 y - 4xy - x + y - 4 = 0
23. From the origin, chords are drawn to the circle
19. The curve described parametrically by x = 2 – 3 sec t,
y = 1 + 4 tan t represents : (x – 1)2 + y2 = 1, then equation of locus of middle points of
these chords, is -
3
(a) An ellipse centred at (2, 1) and of eccentricity (a) x2 + y2 = 1 (b) x2 + y2 = x
5 2 2
(c) x + y = y (d) None of these
(b) A circle centred at (2, 1) and of radius 5 units
24. If P º (x, y), F1 º (3, 0), F2 º (–3, 0) and
8 16x2 + 25y2 = 400, then PF1 +PF2 equals
(c) A hyperbola centred at (2, 1) & of eccentricity
5 (a) 8 (b) 6
5 (c) 10 (d) 12
(d) A hyperbola centred at (2, 1) & of eccentricity
3
RESPONSE 15. 16. 17. 18. 19.
GRID 20. 21. 22. 23. 24.

Space for Rough Work


EBD_7504
M-44 NTA JEE Main

25. The combined equation of the asymptotes of the hyperbola


x 2 y2 x 2 y2
2x2 + 5xy + 2y2 + 4x + 5y = 0 is – (a) + =1 (b) + =1
(a) 2x2 + 5xy + 2y2 + 4x + 5y + 2 = 0 80 5 / 4 20 5
(b) 2x2 + 5xy + 2y2 + 4x + 5y – 2 = 0 x 2 y2
(c) 2x2 + 5xy + 2y2 = 0 (c) + =1 (d) Both (a) and (b)
100 5
(d) None of these 29. A double ordinate of the parabola y2 = 4ax is of length 8a. It
subtends an angle at the vertex equal to
26. The common chord of x 2 +y 2 -4 x -4 y = 0 and
p p
x 2 +y 2 = 16 subtends at the origin an angle equal to (a) (b)
2 4
p p
(a) (b) p 2p
6 4 (c) (d)
p p 6 3
(c) (d) 30. The equation of the image of circle x2 + y2 + 16x – 24y +
3 2
27. The equation of one of the common tangents to the parabola 183 = 0 by the line mirror 4x + 7y + 13 = 0 is

y2 = 8x and x 2 + y 2 - 12x + 4 = 0 is (a) x 2 + y 2 + 32x - 4 y + 235 = 0


(a) y = –x + 2 (b) y = x – 2 (b) x 2 + y 2 + 32x + 4 y - 235 = 0
(c) y = x + 2 (d) None of these
28. If the axes of an ellipse coincides with the co-ordiante axes (c) x 2 + y 2 + 32x - 4 y - 235 = 0
and it passes through the point (4, –1) and touches the line
x + 4y – 10 = 0 then the eq. is (d) x 2 + y 2 + 32x + 4 y + 235 = 0

RESPONSE 25. 26. 27. 28. 29.


GRID 30.

MATHEMATICS CHAPTERWISE SPEED TEST-70


Total Questions 30 Total Marks 120
Attempted Correct
Incorrect Net Score
Cut-off Score 37 Qualifying Score 50
Success Gap = Net Score – Qualifying Score
Net Score = (Correct × 4) – (Incorrect × 1)
Space for Rough Work
MATHEMATICS Speed
Limits and Derivatives TEST
No. of Questions
30
Maximum Marks
120
Time
1 Hour
71
Chapter-wise

GENERAL INSTRUCTIONS
• This test contains 30 MCQ's. For each question only one option is correct. Darken the correct circle/ bubble in the
Response Grid provided on each page.
• You have to evaluate your Response Grids yourself with the help of solutions provided at the end of this book.
• Each correct answer will get you 4 marks and 1 mark shall be deduced for each incorrect answer. No mark will be given/
deducted if no bubble is filled. Keep a timer in front of you and stop immediately at the end of 60 min.
• The sheet follows a particular syllabus. Do not attempt the sheet before you have completed your preparation for that
syllabus.
• After completing the sheet check your answers with the solution booklet and complete the Result Grid. Finally spend time
to analyse your performance and revise the areas which emerge out as weak in your evaluation.

3. Let a and b be the roots of ax2 + bx + c = 0,


æ 100 x2 ö
ç x æ 2ö ÷ 1 - cos(ax 2 + bx + c )
1. lim + ç cos ÷ = Then lim is equal to :
x ®¥ ç e x è xø ÷
è ø x®a ( x - a )2

(a) e –1 (b) e –4 1
(a) 0 (b) (a - b) 2
(c) (1 + e–2) (d) e –2 2
a2
1 1 (c) (a - b ) 2 (d) None of these
2. If y = b-a g -a
+ a -b g -b
2
1+ x +x 1+ x +x nx
æ a1/ x + a1/ x + .......... + a1/ x ö
The value of lim ç 1 2 n
1 dy 4. ÷
+ then is equal to x ®¥ è n ø
a-g b-g
1+ x +x dx
ai > 0, i = 1, 2, ...... n, is
(a) 0 (b) 1
(a) a1 + a2 + ........... + an (b) ea1 +a2 +¼an
(c) ( a + b + g ) x a+b+ g-1 (d) None of these
a1 + a2 + ..... + a n
(c) (d) a1a2 a3 .......an
n

RESPONSE GRID 1. 2. 3. 4.

Space for Rough Work


EBD_7504
M-46 NTA JEE Main

5. If {x} denotes the fractional part of x, then


é sin [ x - 3] ù
{x}
10. lim ê ú , where [ . ] denotes greatest integer
e - {x} - 1 x® 0 ë [ x - 3] û
lim , where [a] denotes the integral part of
x ®[a ] {x}2 function is
a, is equal to (a) 0 (b) 1
(c) does not exist (d) sin 1
1
(a) 0 (b)
2 4 + 3an an = n , then
11. If a1 = 1 and an +1 = , n ³ 1 and if nlim
®¥
(c) e – 2 (d) None of these 3 + 2an
l/x the value of a is
æ a x + bx + c x ö
6. lim ç ÷ ; ( a , b, c, l > 0) is equal to – (a) (b)
x® 0 è 3 ø 2 – 2
(c) 2 (d) None of these
(a) 1; if l = 1 (b) abc; if l = 1
12. Let f ( x ) = a ( x ) b ( x ) g ( x ) for all real x, where
(c) abc ; if l = 1/3 (d) (abc)2/3 ; if l = 2
a ( x ) , b ( x) and g ( x ) are differentiable functions of x.
7. A triangle has two of its vertices at P ( a, 0 ) , Q ( 0, b ) and
If f ¢ ( 2) = 18 f ( 2) , a ¢ ( 2) = 3a ( 2) , b¢ ( 2) = -4b( 2) and
the third vertex R ( x, y ) is moving along the straight line
g ¢ ( 2 ) = kg ( 2 ) , then the value of k is
dA
y = x. If A be the area of the triangle, then is equal to (a) 14
dx
(b) 16
a -b a -b (c) 19
(a) (b)
2 4 (d) None of these
æ a + bö a+b sin x 4 - x 4 cos x 4 + x 20

(c) è 2 ÷ø (d)
4
13. lim
x® 0 4
is equal to
x 4 (e2 x 1 - 2 x 4 )
1
(a) 0 (b) – 1/6
8. lim (4 n + 5n ) n is equal to
n®¥ (c) 1/6 (d) does not exist
(a) 4 (b) 5 log( 2 + x ) - x 2 n sin x
14. Let f ( x ) = lim . Then
4 n®¥ 1 + x 2n
(c) 5e (d) e
5
(a) lim f ( x ) ¹ lim f (x )
cosec x x ®1+ x ®1-
ì1 + tan x ü
9. lim í ý is equal to
x ®0 î 1 + sin x þ (b) lim f ( x ) = sin 1
x ®1+
1 lim f (x ) = does’t exist
(a) (b) 1 (c)
e x ®1-

(c) e (d) e2 (d) None of these

RESPONSE 5. 6. 7. 8. 9.
GRID 10. 11. 12. 13. 14.

Space for Rough Work


Mathematics M-47

(c) Statement -1 is false, Statement -2 is true


n r3 - 8
15. The limit lim P is equal to (d) Statement - 1 is true, Statement- 2 is false
n ® ¥ r =3 r3 + 8 19. If Sn denotes the sum of n terms of a GP whose common
2 1 dS n
(a) (b) ratio is r , then ( r - 1) is equal to
7 12 dr
19 (a) ( n - 1) Sn + n Sn-1 (b) ( n - 1) Sn - n Sn -1
(c) (d) None of these
52 (c) ( n - 1) Sn (d) None of these
2x
æ l m ö x +1
16. If lim ç 1 + + = e 2 then æ 3x - 4 ö 3
x® ¥ è x x 2 ÷ø 20. The value of lim ç ÷ is equal to :
x ®¥ è 3 x + 2 ø
(a) l = -1, m = 2 (a) e –1/3 (b) e –2/3
(c) e –1 (d) e –2
(b) l = 2, m = 1
x tan 2 x - 2 x tan x
(c) l = 1, m = any real number 21. Value of lim is
x®0 (1 - cos 2 x ) 2
(d) l = m = any real number (a) 2 (b) –2
(c) 1/2 (d) –1/2
1 - cos x æ pö
17. If f ( x ) = , then f ¢ ç ÷ is equal to
è 2ø
1 - sin x 1+ 2 + x - 3
(a) 1 (b) 0 22. The value of lim is
x®2 x-2
(c) ¥ (d) does not exist
1 1
sin( f ( x )) (a) (b)
18. Statement-1: lim , 8 3 4 3
x ®a x-a

where f ( x ) = ax 2 + bx + c, is finite and non-zero, then (c) 0 (d) None of these


1 (4 x – 1)3
f (x) 23. The value of lim , is
e -1 x® 0 x2
lim 1
does not exist. sin log(1 + 3x )
x ®a 4
f (x)
e +1
4 4
(a) (ln 4)2 (b) (ln 4)3
sin( f ( x)) 3 3
Statement-2 : lim can take finite value only
x ®a x - a
3 3
(c) (ln 4) 2 (d) (ln 4) 3
0 2 2
when it takes form.
0
1, cos3 x
(a) Statement-1 is true, Statement-2 is true, Statement-2 is 24. The value of lim is
x ®0 x sin x cos x
a correct explanation for Statement -1
(b) Statement -1 is true, Statement -2 is true ; Statement-2 2 3 3 3
(a) (b) (c) (d)
is NOT a correct explanation for Statement - 1 5 5 2 4

RESPONSE 15. 16. 17. 18. 19.


GRID 20. 21. 22. 23. 24.

Space for Rough Work


EBD_7504
M-48 NTA JEE Main

28. The value of


1 - cos x 2
25. The value of lim is 8 ìï x2 x2 x2 x 2 üï
x ®0 1 - cos x lim 1 - cos - cos + cos cos ý is
í
x ®0 x8 ï 2 4 2 4 ïþ
î
1
(a) (b) 2
2 1 1
(a) (b)
8 16
(c) 2 (d) None of these 1 1
(c) (d)
32 64
26. lim (cosec x )1/ log x is equal to :
x® 0 ra ra
29. If z r = cos 2 + i sin 2 , where r = 1, 2, 3, ...., n, then
(a) 0 (b) 1 n n
1 lim z1 z2 z3 ...zn is equal to
(c) (d) None of these n®¥
e
(a) cos a + i sin a (b) cos(a / 2) - i sin(a / 2)
(sin nx ) [(a - n )nx - tan x]
27. If lim = 0 , then the value
eia
3
x® 0 x2 (c) eia / 2 (d)
of a 30. The values of constants a and b so that
1 1 æ x 2 +1 ö
(a) (b) n- Lt ç - ax - b ÷ = 0 is
n n n ®¥ç x + 1 ÷
è ø
1 (a) a = 0, b = 0 (b) a = 1, b = –1
(c) n+ (d) None of these
n (c) a = – 1, b = 1 (d) a = 2, b = –1.

RESPONSE 25. 26. 27. 28. 29.


GRID 30.

MATHEMATICS CHAPTERWISE SPEED TEST-71


Total Questions 30 Total Marks 120
Attempted Correct
Incorrect Net Score
Cut-off Score 38 Qualifying Score 50
Success Gap = Net Score – Qualifying Score
Net Score = (Correct × 4) – (Incorrect × 1)
Space for Rough Work
MATHEMATICS Speed
Mathematical Reasoning TEST
No. of Questions
30
Maximum Marks
120
Time
1 Hour
72
Chapter-wise

GENERAL INSTRUCTIONS
• This test contains 30 MCQ's. For each question only one option is correct. Darken the correct circle/ bubble in the
Response Grid provided on each page.
• You have to evaluate your Response Grids yourself with the help of solutions provided at the end of this book.
• Each correct answer will get you 4 marks and 1 mark shall be deduced for each incorrect answer. No mark will be given/
deducted if no bubble is filled. Keep a timer in front of you and stop immediately at the end of 60 min.
• The sheet follows a particular syllabus. Do not attempt the sheet before you have completed your preparation for that
syllabus.
• After completing the sheet check your answers with the solution booklet and complete the Result Grid. Finally spend time
to analyse your performance and revise the areas which emerge out as weak in your evaluation.

1. Let p, q and r be any three logical statements. Which one of 3. For integers m and n, both greater than 1, consider the
the following is true? following three statements :
P : m divides n
(a) ~ [ p Ù (~ q)] º (~ p) Ù q Q : m divides n2
(b) ~ [( p Ú q ) Ù (~ r ) º (~ p) Ú (~ q) Ú (~ r ) R : m is prime,
then
(c) ~ [ p Ú (~ q)] º (~ p) Ù q (a) QÙR® P (b) PÙQ ® R
(d) ~ [ p Ú (~ q)] º (~ p) Ù ~ q (c) Q®R (d) Q®P
2. ~(p ® q)® [(~p) Ú (~ q)] is 4. If ( p Ù ~ r ) Þ (q Ú r) is false and q and r are both false,
(a) a tautology
then p is
(b) a contradiction
(c) neither a tautology nor contradicion (a) True (b) False
(d) cannot come any conclusion. (c) May be true or false (d) Data sufficient

RESPONSE GRID 1. 2. 3. 4.

Space for Rough Work


EBD_7504
M-50 NTA JEE Main

5. Consider the following statements (c) [p Ù q) Ù (~ p) is a contradiction


p : x, y Î Z such that x and y are odd. (d) ~ [p Ú q] º (~ p) Ú (~ q)
q : xy is odd. Then, 11. The contrapositive of p ® (~q ® ~r) is –
(a) p Þ q is true (b) : q Þ p is true (a) (~ q Ù r) ® ~ p (b) (q ® r) ® ~p
(c) Both (a) and (b) (d) None of these (c) (q Ú ~r) ® ~ p (d) None of these
6. If S*(p, q, r) is the dual of the compound statement S(p,q,r)
12. ~ ( p Þ q) Û ~ pÚ ~ q is
and S (p,q,r) = ~ p Ù [~ (q Ú r)] then S*(~p, ~q, ~r) is
equivalent to – (a) A tautology
(a) S (p, q, r) (b) ~ S (~p, ~q, ~r) (b) A contradiction
(c) ~ S (p, q, r) (d) S*(p, q, r) (c) Neither a tautology nor a contradiction
7. The dual of statement (p Ù q) Ú ~ q º pÚ ~ q is (d) Cannot come to any conclusion
(a) (p Ù q) Ù ~ q º p Ù ~ q 13. Which of the following is wrong ?
(b) (p Ú q) Ù ~ q º p Ù ~ q (a) p ® q is logically equivalent to ~ p Ú q
(c) (p Ù q) Ú ~ q º pÚ ~ q (b) If the truth values of p, q, r are T, F, T respectively, then
(d) (p Ù q) Ú ~ q º pÚ~ q. the truth value of (p Ú q) Ù (q Ú r) is T
8. The converse of the statement if x< y then x 2 < y2 is (c) ~ (p Ú q Ú r) @ ~ p Ù ~ q Ù ~ r
(a) If x is not less then y then x2 is not less than y2 (d) The truth value of p Ù ~ (p Ú q) is always T.
(b) If x2 < y2 then x < y 14. The false statement of the following is
(c) If x2 ³ y2 then x ³ y (a) p Ù (~ p) is a contradiction
(d) None of these (b) ( p Þ q) Û (~ q Þ~ p) is a contradiction
9. If p and q are true statement and r, s are false statements, (c) ~ (~ p) Û p is a tautology
then the truth value of ~ [(pÙ~r) Ú (~q Ú s)] is (d) p Ú (~ p) Û p is a tautology
(a) true (b) false 15. In the truth table for the statement ( ~ p ® ~ q) Ù ( ~ q ® ~ p),
(c) false if p is true (d) None of these the last column has the truth value in the following order is
10. Identify the false statement (a) TTTF (b) FTTF
(a) ~ [p Ú (~ q)] º (~ p) Ú q (c) TFFT (d) TTTT
(b) [p Ú q] Ú (~ p) is a tautology

RESPONSE 5. 6. 7. 8. 9.
GRID 10. 11. 12. 13. 14.
15.

Space for Rough Work


Mathematics M-51

16. If p is any statement, t is tautology and c is a contradiction, 22. The negation of (p Ú q)Ù (q Ú ~ r) is
then which of the following is not correct? (a) (~ p Ù ~ q) Ú (q Ù ~ r)
(a) pÚ (~ p) = c (b) pÚ t = t (b) (~ p Ù ~ q) Ú (~ q Ù r)
(c) p Ù t = p (d) p Ù c = c. (c) (~ p Ù ~ q) Ú (~ q Ù r)
17. The logically equivalent proposition of p Û q is (d) (p Ù q) Ú (~ q Ù ~ r).
23. Let p: Kiran passed the examination,
(a) (p Ù q) Ú (p Ù q) (b) (p Þ q) Ù (q Þ p)
q: Kiran is sad
(c) (p Ù q) Ú (q Þ p) (d) (p Ù q) Þ (q Ú p) The symbolic form of a statement “It is not true that Kiran
18. The inverse of the statement (p Ù ~ q) ® r is passed therefore she is sad” is
(a) ~ (p Ú ~q) ® ~ r (b) (~p Ù q) ® ~ r (a) (~ p® q) (b) (p ® ~q)
(c) ~ (p® ~ q) (d) ~ ( p«q)
(c) (~p Ú q) ® ~ r (d) None of these
19. If x = 5 and y = – 2, then x – 2y = 9. Then contrapositive of 24. The conditional ( p Ù q) Þ p is
this proposition is (a) A tautology
(a) If x – 2y ¹ 9, then x ¹ 5 or y ¹ –2. (b) A fallacy i.e., contradiction
(b) If x – 2y = 9 then x ¹ 5 and y ¹ –2 (c) Neither tautology nor fallacy
(c) x – 2y = 9 if and only if x = 5 and y = – 2 (d) None of these
(d) None of these 25. If p, q are true and r is false statement, then which of the
20. The contrapositive of the statement, ' If I do not secure following is true statement?
(a) (p Ù q) Ú r is F
good marks then I cannot go for engineering', is (b) (p Ù q) ® r is T
(a) if I secure good marks, then I go for engineering (c) (p Ú q) Ù (p Ú r) is T
(b) if I go for engineering then I secure good marks (d) (p ® q) « (p ® r) is T
(c) if I cannot go for engineering then I donot secure 26. Dual of following statement are given which one is not
correct?
good marks
(a) (p Ú q) Ù (r Ú s), (pÙq) Ú (rÙ s)
(d) None of these (b) [p Ú (~q)] Ù (~ p),[ p Ù (~ q)] Ú (~p)
21. The statement p ® (q®p) is equivalent to (c) (p Ù q) Ú r, (p Ú q) Ù r
(a) p ® (p® q) (b) p ® (p Ú q) (d) (p Ú q) Ú s, Ù (p Ù q) Ú s.
(c) p ® (p Ù q) (d) p ® (p «q)

RESPONSE 16. 17. 18. 19. 20.


GRID 21. 22. 23. 24. 25.
26.

Space for Rough Work


EBD_7504
M-52 NTA JEE Main

27. Let p, q, r be three statements. Then ~ [ p Ú (q Ù r )] is equal Statement-1 : The given argument is a valid argument.
to Statement-2 : For three statements p, q and r
(a) (~ p Ù ~ q) Ù (~ p Ù ~ r ) p Þ q and q Þ r then p Þ r
(b) (~ pÚ ~ q) Ù (~ p Ú ~ r ) (a) Statement -1 is false, Statement-2 is true
(c) (~ pÙ ~ q) Ú (~ pÙ ~ r ) (b) Statement -1 is true, Statement-2 is true; Statement -2 is
(d) (~ pÚ ~ q) Ú (~ p Ù ~ r ) a correct explanation for Statement-1
(c) Statement -1 is true, Statement-2 is true; Statement -2
28. Consider the following statements
is not a correct explanation for Statement-1
p : A tumbler is half empty.
q : A tumbler is half full. (d) Statement -1 is true, Statement-2 is false
Then, the combination form of “p if and only if q” is 30. Consider the following statements
(a) a tumbler is half empty and half full P : Suman is brilliant
(b) a tumbler is half empty if and only if it is half full Q : Suman is rich
(c) Both (a) and (b) R : Suman is honest
(d) None of these The negation of the statement “Suman is brilliant and
29. Consider the following argument : dishonest if and only if Suman is rich” can be expressed as
“If it is cloudy tonight then it will rain tomorrow and if it (a) ~ (Q « ( P Ù ~ R)) (b) ~ Q «~ P Ù R
rains tomorrow, I shall be on leave tomorrow, and the (c) ~ ( P Ù ~ R) « Q (d) ~ P Ù (Q «~ R )
conclusion is if it is cloudy tonight then I shall be on leave
tomorrow.”

RESPONSE
27. 28. 29. 30.
GRID

MATHEMATICS CHAPTERWISE SPEED TEST-72


Total Questions 30 Total Marks 120
Attempted Correct
Incorrect Net Score
Cut-off Score 40 Qualifying Score 58
Success Gap = Net Score – Qualifying Score
Net Score = (Correct × 4) – (Incorrect × 1)
Space for Rough Work
MATHEMATICS Speed
Statistics TEST
No. of Questions
30
Maximum Marks
120
Time
1 Hour
73
Chapter-wise

GENERAL INSTRUCTIONS
• This test contains 30 MCQ's. For each question only one option is correct. Darken the correct circle/ bubble in the
Response Grid provided on each page.
• You have to evaluate your Response Grids yourself with the help of solutions provided at the end of this book.
• Each correct answer will get you 4 marks and 1 mark shall be deduced for each incorrect answer. No mark will be given/
deducted if no bubble is filled. Keep a timer in front of you and stop immediately at the end of 60 min.
• The sheet follows a particular syllabus. Do not attempt the sheet before you have completed your preparation for that
syllabus.
• After completing the sheet check your answers with the solution booklet and complete the Result Grid. Finally spend time
to analyse your performance and revise the areas which emerge out as weak in your evaluation.

1. Consider any set of observations x 1, x2, x3, ...., x101; it being


given that x1 < x2 < x3 < ... < x100 < x101 ; then the mean n
(c) S³ r (d) None of these
deviation of this set of observations about a point k is n -1
minimum when k equals 3. For (2n + 1) observations x1, –x1, x2, –x2, .........xn, –xn and
0 where x’s are all distinct. Let S.D. and M.D. denote the
(a) x1 (b) x51 standard deviation and median respectively.Then which of
the following is always true?
x1 + x 2 + ... + x101 (a) S.D < M.D.
(c) (d) x50 (b) S.D.> M.D.
101
(c) S.D. = M.D.
1 n 2 (d) Nothing can be said in general about the relationship
2. Let r be the range and S 2 = å ( xi - x ) be the S.D. of
n - 1 i =1 of S.D. and M.D.
a set of observations x1,x2, ....xn, then 4. If the mean deviation of the numbers 1, 1 + d,
1 + 2d, .... 1 + 100d from their mean is 255, then d is equal to :
n n (a) 20.0 (b) 10.1
(a) S£ r (b) S=r
n -1 n -1 (c) 20.2 (d) 10.0

RESPONSE GRID 1. 2. 3. 4.

Space for Rough Work


EBD_7504
M-54 NTA JEE Main

5. In an experiment with 15 observations on X, the following


results were available Sx2 = 2830, Sx = 170. On observation æaö a
(a) ç ÷s (b) s
that was 20 was found to be wrong and was replaced by the ècø c
correct value 30. Then the corrected variance is
(a) 78.00 (b) 188.66
(c) 177.33 (d) 8.33 æ a2 ö
ç ÷s
6. The quartile deviation of the following items : (c) ç c2 ÷ (d) None
è ø
12, 7, 15, 10, 16, 17, 25 is
(a) 4.5 (b) 13.5 12. The variance of 20 observations is 5. If each observation is
(c) 9 (d) 3.5 multiplied by 2, then the new variance of the resulting
7. Consider the first 10 positive integers. If we multiply each observation is
number by (– 1) and then add 1 to each number, the variance (a) 23 × 5
of the numbers so obtained is (b) 22 × 5
(a) 8.25 (b) 6.5 (c) 2 × 5
(c) 3.87 (d) 2.87 (d) 24 × 5
8. What is the standard deviation of the following series 13. The upper quartile for the following distribution

0-10 10-20 20-30 30-40 Size of items 1 2 3 4 5 6 7


Measurements
Frequency 1 3 4 2 Frequency 2 4 5 8 7 3 2

(a) 81 (b) 7.6 is given by the size of


(c) 9 (d) 2.26
9. Coefficient of variation of two distributions are 50 and 60 æ 31 + 1 ö é æ 31 + 1 ö ù
(a) ç 4 ÷ th item (b) ê 2 ç 4 ÷ ú th item
and their arithmetic means are 30 and 25, respectively. Then, è ø ë è øû
difference of their standard deviations is
(a) 0 (b) 1 é æ 31 + 1 ö ù é æ 31 + 1 ö ù
(c) 1.5 (d) 2.5 (c) ê3 ç 4 ÷ ú th them (d) ê 4 ç 4 ÷ ú th item
ë è øû ë è øû
10. The mean and S.D. of the marks of 200 candidates were
found to be 40 and 15 respectively. Later, it was discovered 14. The mean deviation from the mean of the A.P.
that a score of 40 was wrongly read as 50. The correct mean a, a + d, a + 2d, ........ a, a + 2nd is
and S.D. respectively are n (n + 1) d
(a) 14.98, 39.95 (b) 39.95, 14.98 (a) n (n + 1) d (b)
2n + 1
(c) 39.95, 224.5 (d) None of these
11. The standard deviation of a variate x is s. The standard n (n + 1) d n (n - 1) d
(c) (d)
ax + b
2n 2n + 1
deviation of the variable ; a, b, c are constants, is
c
RESPONSE 5. 6. 7. 8. 9.
GRID 10. 11. 12. 13. 14.

Space for Rough Work


Mathematics M-55

15. The first and the third quartiles of the data given below :
n n
(a) (b)
Marks No. of Students 4 3
0-10 4 2n
10-20 8 (c) (d) None of these
20-30 11 5
30-40 15 20. The standard deviation of n observations x1 , x2 ,..., xn is
40-50 12
50-60 6 n n
60-70 3 2. If å xi = 20 and å xi2 = 100, then n is
are respectively i =1 i =1
(a) 21.5, 43.8 (b) 26.26, 49.69 (a) 10 or 20 (b) 5 or 10
(c) 22.5, 45.2 (d) 23, 45 (c) 5 or 20 (d) 5 or 15
16. The mean and SD of 63 children on an arithmetic test are (e) 25
respectively 27.6 and 7.1. To them are added a new group of 21. The coefficient of variation from the given data
26 who had less training and whose mean is 19.2 and SD 6.2.
The values of the combined group differ from the original as Class Interval 0-10 10-20 20-30 30-40 40-50
to (i) the mean and (ii) the SD is Frequency 2 10 8 4 6
(a) 25.1, 7.8 (b) 2.3, 0.8
(c) 1.5, 0.9 (d) None of these is :
17. The standard deviations of two sets containing 10 and 20 (a) 50 (b) 51.9 (c) 48 (d) 51.8
members are 2 and 3 respectively measured from their 22. Coefficient of variation of two distribution are 60 and 70,
common mean 5. The SD for the whole set of 30 members is and their standard deviations are 21 and 16, respectively.
What are their arithmetic means?
2 (a) 35, 22.85 (b) 22.85, 35.28
(a) (b) 6
3 (c) 36, 22.85 (d) 35.28, 23.85
23. All the students of a class performed poorly in Mathematics.
æ 22 ö
(c) çè ÷ø (d) 3 The teacher decided to give grace marks of 10 to each of the
3 students. Which of the following statistical measures will
18. The marks of some students were listed out of 75. The SD of not change even after the grace marks were given ?
marks was found to be 9. Subsequently the marks were raised (a) mean (b) median
to a maximum of 100 and variance of new marks was (c) mode (d) variance
calculated. The new variance is
24. Let x1 , x 2 , .......... xn be n observations such that
(a) 144 (b) 122
(c) 81 (d) None of these å xi2 = 400 and å xi = 80. Then the possible value of n
19. If the variable takes values 0,1, 2, 3, ..., n with frequencies among the following is
n (a) 15 (b) 18
proportional to C0 , n C1 , n C2 ,..., n Cn respectively, the
(c) 9 (d) 12
variance is

RESPONSE 15. 16. 17. 18. 19.


GRID 20. 21. 22. 23. 24.

Space for Rough Work


EBD_7504
M-56 NTA JEE Main

å 9i =1 ( xi - 5) = 9 å 9i=1 ( xi - 5)
2 99
25. If and = 45, then the 29. If the variance of 1, 2, 3, 4, 5, ..., 10 is , then the standard
12
standard deviation of the 9 items x1, x2 , ..., x9 is
deviation of 3, 6, 9, 12, ..., 30 is
(a) 9 (b) 4
297 3
(c) 3 (d) 2 (a) (b) 33
26. The S.D. of the following data is nearly 4 2

x i 140 145 150 155 160 165 170 175 3 99


(c) 99 (d)
fi 4 6 15 30 36 24 8 2 2 12

(a) 8.64 (b) 7.26 30. Let x , M and s2 be respectively the mean, mode and
(c) 7.05 (d) None of these variance of n observations
27. The first of two samples has 100 items with mean 15 and SD x1, x2, ...., xn and di = – xi – a, i = 1, 2, ...., n,
3. If the whole group has 250 items with mean 15.6 and where a is any number.
Statement I: Variance of d1, d2,... dn is s2.
SD = 13.44 the SD of the second group is
Statement II: Mean and mode of d1, d2, .... dn are - x - a
(a) 5 (b) 4
and – M – a, respectively.
(c) 6 (d) 3.52
(a) Statement I and Statement II are both false
28. The sum of squares of deviations for 10 observations taken (b) Statement I and Statement II are both true
from mean 50 is 250. Then co-efficient of variation is
(c) Statement I is true and Statement II is false
(a) 50% (b) 10%
(d) Statement I is false and Statement II is true
(c) 40% (d) None of these

RESPONSE 25. 26. 27. 28. 29.


GRID 30.

MATHEMATICS CHAPTERWISE SPEED TEST-73


Total Questions 30 Total Marks 120
Attempted Correct
Incorrect Net Score
Cut-off Score 38 Qualifying Score 50
Success Gap = Net Score – Qualifying Score
Net Score = (Correct × 4) – (Incorrect × 1)

Space for Rough Work


MATHEMATICS Speed
Probability TEST
No. of Questions
30
Maximum Marks
120
Time
1 Hour
74
Chapter-wise

GENERAL INSTRUCTIONS
• This test contains 30 MCQ's. For each question only one option is correct. Darken the correct circle/ bubble in the
Response Grid provided on each page.
• You have to evaluate your Response Grids yourself with the help of solutions provided at the end of this book.
• Each correct answer will get you 4 marks and 1 mark shall be deduced for each incorrect answer. No mark will be given/
deducted if no bubble is filled. Keep a timer in front of you and stop immediately at the end of 60 min.
• The sheet follows a particular syllabus. Do not attempt the sheet before you have completed your preparation for that
syllabus.
• After completing the sheet check your answers with the solution booklet and complete the Result Grid. Finally spend time
to analyse your performance and revise the areas which emerge out as weak in your evaluation.

1. In four schools B1, B2, B3, B4 the percentage of girls students (a) P (only one of them occurs)
is 12, 20, 13, 17 respectively. From a school selected at random, = P (E1E 2 E 3 + E1E 2 E 3 + E1E 2 E 3 )
one student is picked up at random and it is found that the
(b) P (none of them occurs)
student is a girl. The probability that the school selected is
B2, is = P (E1 + E2 + E3 )
10 13 17 (c) P (atleast one of them occurs)
6
(a) (b) (c) (d) = P (E1 + E 2 + E3 )
31 31 62 62
2. The chance of one event happeing is the square of the (d) P (all the three occur) = P (E1 E 2 E3 ) where P (E1)
chance of a second event, but the odds against the first are
the cube of the odds against the second. The chance of the denotes the probability of E 1 an d E1 denotes
first event is COMPLEMENT of E1
4. The probability that in the random arrangement of the letters
1 1 2 4 of the word ‘UNIVERSITY’, the two I’s does not come
(a) (b) (c) (d) together is
3 9 3 9
3. Let E1, E2, E3 be three arbitrary events of a sample space S. 4
(a) (b) 1/ 5
Consider the following statements. Which of the following 5
statements is correct ? (c) 1/10 (d) 9/10

RESPONSE GRID 1. 2. 3. 4.
Space for Rough Work
EBD_7504
M-58 NTA JEE Main

5. Two numbers x and y are chosen at random (without


replacement) from amongst the numbers 1, 2, 3, .....2004. The æ1ö æ 7ö
(a) ç ÷ (b) ç ÷
è5ø è 10 ø
probability that x 3 + y3 is divisible by 3 is
æ 1ö æ 3ö
1 2 (c) ç ÷ (d) ç ÷
(a) (b) 10
è ø è 10 ø
3 3
11. The probability that a leap year will have 53 Friday or
1 1 53 Saturday, is
(c) (d)
6 4 2 3
(a) (b)
7 7
6. A card is drawn from a pack of 52 cards. A gambler bets that
it is a spade or an ace. What are the odds against his win- 4 1
ning this bet? (c) (d)
7 7
(a) 17 : 52 (b) 52 : 17 12. The probability that the two digit number formed by digits
(c) 9 : 4 (d) 4 : 9 1, 2, 3, 4, 5 is divisible by 4 is
7. If n objects are distributed at random among n persons, the
probability that at least one of them will not get anything is 1 1
(a) (b)
30 20
(n - 1)! (n - 1)!
(a) 1- n -1 (b) 1
n nn (c) (d) None of these
5
(n - 1)! 13. If 12 identical balls are to be placed in 3 identical boxes, then
(c) 1- (d) None of these
nn the probability that one of the boxes contains exactly 3 balls
8. If M and N are any two events. The probability, that exactly is :
one of them occurs, is 12 11
æ 1ö æ 1ö
(a) P(M) + P(N) – P(M Ç N) (a) 220 ç ÷ (b) 22 ç ÷
è 3ø è 3ø
(b) P(M) + P(N) + P(M Ç N)
11 10
(c) P(M) + P(N) 55 æ 2 ö æ 2ö
(c) çè ÷ø (d) 55 ç ÷
(d) P(M) + P(N) – 2 P(M Ç N) 3 3 è 3ø
9. A four digit number is formed by the digits 1, 2, 3, 4 with no 14. If two numbers p and q are choosen randomly from the set
repetition. The probability that the number is odd is : {1, 2, 3, 4} with replacement, then the probability that
1 p 2 ³ 4q is equal to
(a) zero (b)
3 1 3
(a) (b)
1 4 16
(c) (d) None of these
4 1 7
10. Of a total of 600 bolts, 20% are too large and 10% are too (c) (d)
2 16
small. The remainder are considered to be suitable. If a bolt
is selected at random, the probability that it will be
suitable is:

RESPONSE 5. 6. 7. 8. 9.
GRID 10. 11. 12. 13. 14.

Space for Rough Work


Mathematics M-59

15. If the integers m and n are chosen at random between 1 and 20. A bag has 13 red, 14 green and 15 white balls, p1 is the
100, then the probability that a number of the form 7m + 7n is probability of drawing exactly 2 white balls when four balls
divisible by 5 equals are drawn. Then the number of balls of each colour are
1 doubled. Let p2 be the probability of drawing 4 white balls
1
(a) (b) when 8 ball are drawn, then
4 7 (a) p1 = p2 (b) p1 > p2
1 1 (c) p1 < p2 (d) None of these
(c) (d)
8 49 3 1
21. If, P( B) = , P(A Ç B Ç C ) =
1 2 4 3
16. If A and B are two events such that P (A) = and P( B) = ,
2 3 1
then which of the following is not correct? and P( A Ç B Ç C ) = , then P (B Ç C) is
3
2 1
(a) P ( A È B) ³ (b) P(A Ç B' ) ³ 1 1
3 3 (a) (b)
12 6
1 1 1 1
(c) £ P( A Ç B) £ (d) £ P( A 'Ç B) £ 1 1
6 2 6 2 (c) (d)
17. In a class of 125 students 70 passed in Mathematics, 55 in 15 9
Statistics and 30 in both. The probability that a student 22. A and B are events such that P(A È B)=3/4, P(A Ç B)=1/4,
selected at random from the class has passed in only one P( A ) =2/3 then P ( A Ç B) is
subject is (a) 5/12 (b) 3/8
13 3 (c) 5/8 (d) 1/4
(a) (b)
25 25 1+ 4 p 1 - p 1- 2p
17 8 23. If , and are the probabilities of three
(c) (d) 4 2 2
25 25 mutually exclusive events, then value of p is
18. Let A, B, C be three events. If the probability of occurring
1 1
exactly one event out of A and B is 1– a, out of B and C is (a) (b)
1– 2a and out of C and A is 1 – a, and that of occurring three 2 3
events simultaneously is a2, then the probability that at 1 2
least one out of A, B, C will occur is (c) (d)
4 3
1 1 24. Two numbers x and y are chosen at random (without
(a) (b) <
2 2 replacement) from amongst the numbers 1, 2, 3, .....2004. The
1
> probability that x 3 + y3 is divisible by 3 is
(c) (d) None of these
2
1 2
19. If P (A) = P (B) = x and P(A Ç B) = P(A'ÇB' ) = 1/3 , then (a) (b)
3 3
x=?
(a) 1/2 (b) 1/3 1 1
(c) (d)
(c) 1/4 (d) 1/6 6 4

RESPONSE 15. 16. 17. 18. 19.


GRID 20. 21. 22. 23. 24.

Space for Rough Work


EBD_7504
M-60 NTA JEE Main

25. Let A, B, C be three events such that P (A) = 0.3, P(B) = 0.4, 28. Given that n is odd, the number of ways in which three
P(C) = 0.8, P(A Ç B) = 0.08, P(A Ç C) = 0.28, P(A Ç B Ç C) = numbers in A.P. can be selected from 1, 2, 3, ..., n is
0.09. If P(A È B È C) ³ 0.75, then
3(n - 1) 3(n + 1) 2
(a) 0.23 £ P( B Ç C) £ 0.48 (b) 0.45 £ P(B Ç C) £ 0.75 (a)
n ( n - 2)
(b)
2n (n - 1) (n - 2)
(c) 0.48 £ P(B Ç C) £ 0.75 (d) None of these.
26. Two integers x and y are chosen with replacement out of the n-2 3(n - 1)
(c) n (n - 1)
(d)
2n ( n - 2)
set {0, 1, 2, 3, ...........10}. Then the probability that | x – y | >5
is 29. A coin is tossed m + n times, with m > n. The probability of
81 30 getting m consecutive heads is
(a) (b)
121 121 n+2 n +1
(a) m +1 (b)
2 2 m +1
25 20 n
(c) (d) (c) (d) None of these
121 121 2 m +1
27. A natural number x is chosen at random from the first 100 30. Let A and B be two event such that P(A ÈB) ³ 3/4 and
natural numbers. Then the probability, for the equation 1/8 £ P (AÇB) £ 3/8.
100 Statement-1 : P(A) + P(B) ³ 7/8
x+ > 50 to be true is Statement-2 : P(A) + P(B) £ 11/8.
x
(a) Both the statements are True and Statement-2 is the
1 11 correct explanation of Statement-1.
(a) (b)
20 20 (b) Both the statements are True but Statement-2 is NOT
the correct explantion of Statement-1.
1 3
(c) (d) (c) Statement-1 is true and Statement-2 is false.
3 20 (d) Statement-1 is false and Statement-2 is true.

RESPONSE 25. 26. 27. 28. 29.


GRID 30.

MATHEMATICS CHAPTERWISE SPEED TEST-74


Total Questions 30 Total Marks 120
Attempted Correct
Incorrect Net Score
Cut-off Score 40 Qualifying Score 55
Success Gap = Net Score – Qualifying Score
Net Score = (Correct × 4) – (Incorrect × 1)

Space for Rough Work


MATHEMATICS Speed
Relations and Functions TEST
No. of Questions
30
Maximum Marks
120
Time
1 Hour
75
Chapter-wise

GENERAL INSTRUCTIONS
• This test contains 30 MCQ's. For each question only one option is correct. Darken the correct circle/ bubble in the
Response Grid provided on each page.
• You have to evaluate your Response Grids yourself with the help of solutions provided at the end of this book.
• Each correct answer will get you 4 marks and 1 mark shall be deduced for each incorrect answer. No mark will be given/
deducted if no bubble is filled. Keep a timer in front of you and stop immediately at the end of 60 min.
• The sheet follows a particular syllabus. Do not attempt the sheet before you have completed your preparation for that
syllabus.
• After completing the sheet check your answers with the solution booklet and complete the Result Grid. Finally spend time
to analyse your performance and revise the areas which emerge out as weak in your evaluation.

1. Let R = {(x, y) : x, y Î N and x2 – 4xy + 3y2 = 0}, where N is (a) f (x) > f (y) > f (z) (b) f (x) < f (y) < f (z)
the set of all natural numbers. Then the relation R is : (c) f (y) < f (x) < f (z) (d) f (y) < f (z) < f (x)
(a) reflexive but neither symmetric nor transitive.
p
(b) symmetric and transitive. 4. Domain of definition of the function f ( x ) = sin -1 (2 x ) +
(c) reflexive and symmetric. 6
(d) reflexive and transitive. for real valued x, is
2 2
2. Let P = {( x, y ) :| x + y | = 1, x, y Î R} . Then P is é 1 1ù é 1 1ù
(a) Reflexive (b) Symmetric
(a) ê- 4 , 2 ú (b) ê- 2 , 2 ú
ë û ë û
(c) Transitive (d) Anti-symmetric
3. Let f : {x, y, z} ® {1, 2, 3} be a one-one mapping such that
only one of the following three statements is true and æ 1 1ö é 1 1ù
(c) ç- , ÷ (d) ê- 4 , 4 ú
remaining two are false : f (x) ¹ 2, f (y) = 2, f (z) ¹ 1, then è 2 9ø ë û

RESPONSE GRID 1. 2. 3. 4.

Space for Rough Work


EBD_7504
M-62 NTA JEE Main

5. If R be a relation < from A = {1, 2, 3, 4} to B = {1, 3, 5} i.e.,


ìï x | x | -4, x ÎQ
(a, b) Î R Û a < b, then RoR -1 is 10. If f : R ® R, f (x) = í , then f (x) is
ïî x | x | - 3 x ÏQ
(a) {(1, 3), (1, 5), (2, 3), (2, 5), (3, 5), (4, 5)}
(b) {(3, 1), (5, 1), (3, 2), (5, 2), (5, 3), (5, 4)} (a) one to one and onto (b) many to one and onto
(c) {(3, 3), (3, 5), (5, 3), (5, 5)} (c) one to one and into (d) many to one and into
(d) {(3, 3), (3, 4), (4, 5)} 3x + 4
11. If f : B ® A is defined by f ( x ) = and g : A ® B is
ìï2x + a ; x ³ -1 5x - 7
6. If f (x) = í 2 and 7x + 4 ì3ü
ïîbx + 3 ; x < -1 defined by g ( x ) = , where A = R – í ý and
5x - 3 î5þ
ìx + 4 ; 0 £ x £ 4
g (x) = í ì7 ü
î-3x - 2 ; - 2 < x < 0 B = R – í ý and IA is an identity function on A and IB is
î5þ
If domain of g (f (x)) is [–1, 4], then – identity function on B, then
(a) a = 0, b > 5 (b) a = 2, b > 7 (a) fog = IA and gof = IA (b) fog = IA and gof = IB
(c) a = 2, b > 10 (d) a = 0, b Î R (c) fog = IB and gof = IB (d) fog = IB and gof = IA
7. Let S be the set of all straight lines in a plane. A relation R is 12. Let f be a real valued function with domain R satisfying
defined on S by aRb Û a ^ b then R is : 1
0 £ f ( x ) £ and for some fixed a > 0,
(a) reflexive but neither symmetric nor transitive 2
(b) symmetric but neither reflexive nor transitive 1
f (x + a ) = - f ( x) - (f ( x)) 2 " x Î R ,
(c) transitive but neither reflexive nor symmetric 2
(d) an equivalence relation then the period of the function f(x) is
8. A function whose graph is symmetrical about the y-axis is (a) a (b) 2a
given by (c) non-periodic (d) None of these
13. Let f (x) = [x]2 + [x + 1] – 3 where [x] = the greatest integer
(a) f ( x ) = sin[log( x + x 2 + 1 )] function. Then
sec 4 x + cos ec 4 x (a) f (x) is a many-one and into function
(b) f (x) = (b) f (x) = 0 for infinite number of values of x
x 3 + x 4 cot x
(c) f (x) = 0 for only two real values
(c) f ( x + y) = f ( x) + f ( y) " x , yÎ R (d) Both (a) and (b)
(d) None of these 14. f (x) = | x – 1 |, f : R+ ® R and g (x) = ex, g : [–1, ¥) ® R. If the
9. Let R be a reflexive relation on a finite set A having function fog (x) is defined, then its domain and range
n-elements, and let there be m ordered pairs in R. respectively are
Then (a) (0, ¥) and [0, ¥) (b) [–1, ¥) and [0, ¥)
(a) m ³ n (b) m £ n é 1 ö é1 ö
(c) m = n (d) None of these (c) [–1, ¥) and ê1 - , ¥÷ (d) [–1, ¥) and ê - 1, ¥÷
ë e ø ëe ø

RESPONSE 5. 6. 7. 8. 9.
GRID 10. 11. 12. 13. 14.

Space for Rough Work


Mathematics M-63

15. If X = {x1, x2, x3} and Y = {x1, x2, x3,x4,x5} then find which is x
a reflexive relation of the following ? 21. If f (x) = , then (fofo........of )(x) is equal to :
x -1 19 times
(a) R1 : {(x1, x1), (x2, x2)} 19
x æ x ö
(b) R1 : {(x1, x1), (x2, x2), (x3, x3)} (a) (b) ç
x -1 è x - 1÷ø
(c) R3 : {(x1, x1), (x2, x2), (x1, x3),(x2, x4)}
19x
(d) R3 : {(x1, x1), (x2, x2),(x3, x3),(x4, x4)} (c) (d) x
x -1
16. A binary operation * on the set {0, 1, 2, 3, 4, 5} is defined as 22. Which of the function defined below is one-one?
ì a+b , if a + b < 6 (a) f : (0, ¥) ® R , f (x) = x2 – 4x + 3
a*b = í
îa + b - 6 , if a + b ³ 6 (b) f : [0, ¥) ® R , f (x) = x2 + 4x – 5
the identity element is x 1
(a) 0 (b) 1 (c) 2 (d) 3 (c) f : R ® R , f (x) = e + x
e
17. Let R = {(1, 3), (2, 2), (3, 2)} and S = {(2, 1), (3, 2), (2, 3)} be (d) f : R ® R, f (x) = ln(x 2 + x + 1)
two relations on set A = {1, 2, 3}. Then RoS =
(a) {(1, 3), (2, 2), (3, 2), (2, 1), (2, 3)} 2 10 x - 10 - x
23. The inverse of f (x) = is
(b) {(3, 2), (1, 3)} 3 10 x + 10- x
(c) {(2, 3), (3, 2), (2, 2)}
(d) {(2, 3), (3, 2)} 1 1+ x 1 2 + 3x
(a) log10 (b) log10
2 3 1- x 2 2 - 3x
18. If g(f (x)) = | sin x| and f(g(x)) = (sin x ) , then
1 2 + 3x 1 2 - 3x
(a) f(x) = sin2 x, g(x) = x (c) log10 (d) log10
3 2 - 3x 6 2 + 3x
(b) f(x) = sin x, g(x) = | x | 24. A function f from the set of natural numbers to integers
(c) f(x) = x2, g(x) = sin x ì n -1
(d) f and g cannot be determined. ï , when n is odd
defined by f (n) = í 2 is
x-m n
19. Let f : R ® R be a function defined by f (x) = , ï - , when n is even
x-n î 2
where m ¹ n , then (a) neither one-one nor onto (b) one-one but not onto
(c) onto but not one-one (d) one-one and onto both
(a) f is one-one onto (b) f is one-one into
(c) f is many-one onto (d) f is many-one into æ pö æ pö
25. If f(x) = sin2x + sin2 ç x + ÷ + cos x cos ç x + ÷ and
20. If f(x) is defined on (0, 1), then the domain of definition of è 3 ø è 3ø
æ5ö
f (e x ) + f (ln | x |) is g ç ÷ = 1, then go f(x) =
è 4ø
(a) (–e, –1) (b) (-e, - 1) È (1, e) (a) 1 (b) 0
(c) sin x (d) None of these
(c) (-¥, - 1) È (1, ¥) (d) (–e, e)

15. 16. 17. 18. 19.


RESPONSE 20. 21. 22. 23. 24.
GRID 25.
Space for Rough Work
EBD_7504
M-64 NTA JEE Main

29. Statement-1 : If f : R ® R and g : R ® R be two mappings


ì ü
The domain of the function f ( x) = sin -1 ílog 2 æç x 2 ö÷ ý is
1
26. such that f (x) = sin x and g (x) = x2, then fog ¹ gof.
î è2 øþ
Statement-2 : (fog)x = f (x)g(x) = (gof)x
(a) [-2, - 1) È (1, 2] (b) (-2, - 1] È [1, 2] (a) Statement -1 is true, Statement-2 is true; Statement -2 is
(c) [-2, - 1] È [1, 2] (d) (-2, - 1) È (1, 2) a correct explanation for Statement-1.
(b) Statement -1 is true, Statement-2 is true; Statement -2
ax + b
27. Let f (x) = , then fof (x) = x, provided that: is not a correct explanation for Statement-1.
cx + d
(c) Statement -1 is false, Statement-2 is true.
(a) d = a (b) a = b = c = d = 1 (d) Statement -1 is true, Statement-2 is false.
(c) a = b = 1 (d) d = –a
30. Statement-1 : If f(x) = | x – 1| + | x – 2 | + | x – 3| where 2 < x < 3
e | x| - e - x is an identity function.
28. Let f : R ® R be a function defined by f ( x ) = x .
e + e -x Statement-2 : f : A ® A defined by f (x) = x is an identity
Then
function.
(a) f is both one-one and onto
(b) f is one-one but not onto (a) Statement -1 is false, Statement-2 is true
(b) Statement -1 is true, Statement-2 is true; Statement -2 is
(c) f is onto but not one-one
a correct explanation for Statement-1
(d) f is neither one-one nor onto.
(c) Statement -1 is true, Statement-2 is true; Statement -2
is not a correct explanation for Statement-1
(d) Statement -1 is true, Statement-2 is false

RESPONSE 26. 27. 28. 29. 30.


GRID

MATHEMATICS CHAPTERWISE SPEED TEST-75


Total Questions 30 Total Marks 120
Attempted Correct
Incorrect Net Score
Cut-off Score 35 Qualifying Score 50
Success Gap = Net Score – Qualifying Score
Net Score = (Correct × 4) – (Incorrect × 1)
Space for Rough Work
MATHEMATICS Speed
Inverse Trigonometric Functions TEST
No. of Questions
30
Maximum Marks
120
Time
1 Hour
76
Chapter-wise

GENERAL INSTRUCTIONS
• This test contains 30 MCQ's. For each question only one option is correct. Darken the correct circle/ bubble in the
Response Grid provided on each page.
• You have to evaluate your Response Grids yourself with the help of solutions provided at the end of this book.
• Each correct answer will get you 4 marks and 1 mark shall be deduced for each incorrect answer. No mark will be given/
deducted if no bubble is filled. Keep a timer in front of you and stop immediately at the end of 60 min.
• The sheet follows a particular syllabus. Do not attempt the sheet before you have completed your preparation for that
syllabus.
• After completing the sheet check your answers with the solution booklet and complete the Result Grid. Finally spend time
to analyse your performance and revise the areas which emerge out as weak in your evaluation.

1. sin–1(sin 5) > x2 – 4x holds if


æ x 3 ö æ 2x – K ö
(a) x = 2 – 9 – 2p 3. If A = tan –1 çè ÷ø and B = tan –1 çè K 3 ÷ø , then the
2K – x
(b) x = 2 + 9 – 2p value of A – B is
(c) x > 2 + 9 – 2p (a) 0º (b) 45º
(d) x Î (2 – 9 – 2 p ,2 + 9 – 2 p )
(c) 60º (d) 30º
4. The value of
-1 -1 -1
2. Range of f(x) = sin x + tan x + sec x is æ1 ö
tan,1 ççç (tan 2A) ∗ tan,1 (cot A) ∗ tan,1 (cot3 A)÷÷÷ is
æ p 3p ö é p 3p ù è2 ø
(a) çè 4 , 4 ÷ø (b) ê4, 4 ú p p p
ë û (a) 0 if <A< (b) p, if 0 < A <
4 2 4
ì p 3p ü (c) both (a) and (b) (d) None of these
(c) í , ý (d) None of these
î4 4 þ

RESPONSE GRID 1. 2. 3. 4.

Space for Rough Work


EBD_7504
M-66 NTA JEE Main

æ
-1 1 - x

p (n - 1)d (n - 1)d
5. If cos ç 2÷
< , then (a) (b)
è1+ x ø 3 a1 + a n 1 + a 1a n

æ 1 nd a n - a1
é 1 1 ù 1 ö (c) (d) a + a
(a) x Îê- , ú (b) x Î çç - , ÷÷
1 + a 1a n n 1
ë 3 3û è 3 3ø
11. The sum of the infinite series
æ 1 ö cot–1 2 + cot–1 8 + cot–1 18 + cot–1 32 + .... is
(c) x Î çç 0, ÷÷ (d) None of these
è 3ø p
(a) p (b)
n p 2
6. If cot – 1 > , n Î N, then the maximum value of n is :
p 6 p
(a) 1 (b) 5 (c) (d) None of these
4
(c) 9 (d) None of these
2n 2p
2n 12. If cos –1 x + cos –1 y = , th en the value
7. If åcos-1 xi = 0 then å
i =1
xi is
sin–1x + sin–1 y is equal to
7
i =1
(a) n (b) 2n 4p 3p
n(n + 1) (a) (b)
(c) (d) None of these 7 7
2
2p 5p
æ 1 - x2 ö æ 2ö (c) (d)
-1 1 - y p 7 7
8. If cos- 1 ç 2 ÷ + cos ç 2÷
= , where xy < 1, then
è 1+ x ø è 1+ y ø 2 13. The value of
(a) x – y – xy = 1 (b) x – y + xy = 1 p
ìï 1 - sin x + 1 + sin x üï
(c) x + y – xy = 1 (d) x + y + xy = 1 cot -1 í ý (0 < x < 2 ) is
9. The range of the function îï 1 - sin x - 1 + sin x þï
f (x) = sin -1 (log [ x ]) + log(sin - 1[ x ]); (where [.] denotes
x
the greatest integer function) is (a) p- (b) 2p - x
(a) R (b) [1, 2) 2

(c) ì pü (d) {– sin 1}


ílog ý x x
î 2þ
(c) (d) –p
2 2
10. If a1 , a 2 , a 3 ,........a n is an A.P. with common difference d;
( d > 0) then 14. If sin–1 a + sin–1b + sin–1c = p, then find the value of

é æ d ö æ ö a 1 - a2 + b 1 - b2 + c 1 - c 2 .
d
tan ê tan -1 çç ÷÷ + tan -1 çç ÷ + ...
÷ (a) abc (b) a + b + c
ëê è 1 + a 1a 2 ø è 1 + a 2a 3 ø
1 1 1
(c) ´ ´ (d) 2abc
-1 æç d öù a b c
...... + tan ç 1 + a a ÷÷ú is equal to
è n -1 n øúû

RESPONSE 5. 6. 7. 8. 9.
GRID 10. 11. 12. 13. 14.

Space for Rough Work


Mathematics M-67

3 1 20. In a DABC, if A = tan -1 2 and B = tan -1 3 , then C =


15. If a = sin -1 + sin -1
2 3 p p
(a) (b)
3 4
3 1
and b = cos -1 + cos -1 , then : p 3p
2 3 (c) (d)
6 4
(a) a<b (b) a =b
(c) a >b (d) a + b = 2p 21. If sin -1 x + sin -1 y + sin -1 z = p , then

æ x2 x3 ö x 4 + y 4 + z 4 + 4 x 2 y 2 z 2 = k ( x 2 y 2 + y 2 z 2 + z 2 x 2 ).
16. If sin-1 çx - + - ... ÷÷ where k =
ç 2 4
è ø (a) 1 (b) 2
æ 2 x4 ö p (c) 4 (d) None of these
x6
+ cos-1 çç x - + - ... ÷÷ = for 0 < |x| < 2 , then x 22. The sum of the infinite series
è 2 4 ø 2
equals æ 1 ö æ 2 – 1ö æ 3– 2ö
sin –1 ç ÷ + sin –1 ç ÷ + sin –1 ç ÷ + ...
(a) 1/2 (b) 1 è 2ø è 6 ø è 12 ø
(c) -1/2 (d) -1
æ n – ( n – 1) ö
-1 æ 1 ö -1 -1 æ1 ö -1 +... + sin –1 ç ÷ + ... is
17. If sin çè ÷ø + sec (2) + 2tan çè ÷ø + sec (5) è {n (n + 1)} ø
5 3
p p
-1 æ 1 ö -1 (a) (b)
+ sin çè ÷ø + 2 tan ( 3) = kp, then k = 8 4
2
p
(a) 1 (b) 2 (c) (d) p
(c) 4 (d) 5 2
¥ æ 1 ö æ x ö
If sin–1 (x – 1) + cos–1 (x – 3) + tan–1 çè ÷
18. å tan–1 çè 1+ r + r2 ÷ø = ............ 23.
2 - x2 ø
r =1
= cos–1 k + p, then the value of k =
p p (a) 1 (b) -1 / 2
(a) (b)
2 4 (c) 1 / 2 (d) None of these
2p é 1 æ 5 öù
(c)
3
(d) None of these 24. tan êcos -1 - sin -1 ç ÷ ú is equal to
ë 82 è 26 ø û
19. If 0 < a < b < c, then
2 4
ab + 1ö
-1 æ æ bc + 1ö æ ca + 1ö (a) (b)
cot ç + cot -1 ç + cot -1 ç =
è a - b ÷ø è b - c ÷ø è c - a ÷ø 23 31

(a) 0 (b) p 29 6
(c) (d)
(c) 2p (d) None of these 3 13

RESPONSE 15. 16. 17. 18. 19.


GRID 20. 21. 22. 23. 24.

Space for Rough Work


EBD_7504
M-68 NTA JEE Main

25. If q and f are the roots of the equation 8 x 2 + 22 x + 5 = 0 , 28. What is the value of
then tan (tan–1x + tan–1y + tan–1z) – cot (cot–1x + cot–1y + cot–1z) ?
(a) 0 (b) 2 (x + y + z)
(a) both sin -1 q and sin -1 f are real
3p 3p
(b) both sec -1 q and sec -1 f are real (c) (d) +x+y+z
2 2
(c) both tan -1 q and tan -1 f are real 1 2 4 p
(d) None of these 29. Statement-1 : tan -1 + tan -1 + tan -1 + ....¥ =
3 3 33 4
æ a cos x - b sin x ö
26. tan -1 ç ÷ is equal to Statement-2 : If xy < 1 then tan - 1 x + tan -1 y = tan -1
x+ y
è b cos x + a sin x ø 1 - xy
æ a p pö (a) Statement -1 is true, Statement-2 is true; Statement -2 is
ç where tan x > -1, < x < ÷ a correct explanation for Statement-1.
è b 2 2ø
(b) Statement -1 is true, Statement-2 is true; Statement -2
-1 a -1 a 1
(a) tan - tan -1 x (b) tan - is not a correct explanation for Statement-1.
b b x (c) Statement -1 is false, Statement-2 is true.
-1 a -1 b (d) Statement -1 is true, Statement-2 is false.
(c) tan -x (d) tan - tan -1 x
b a
-1 æ 1 ö -1 æ 1 ö
æ ö 30. Statement 1 : sin çè ÷ø > tan çè ÷
a e pø
27. If cot(cos -1 x) = sec ç tan -1 ÷ , then x is equal to
è b2 - a2 ø Statement 2 : sin–1x > tan–1 y for x > y, x, y Î (0, 1)
(a) Statement -1 is true, Statement-2 is true; Statement -2 is
b a a correct explanation for Statement-1.
(a) 2 2 (b) (b) Statement -1 is true, Statement-2 is true; Statement -2
2b - a 2b - a 2
2
is not a correct explanation for Statement-1.
2b2 - a 2 2b2 - a 2 (c) Statement -1 is false, Statement-2 is true.
(c) (d)
a b (d) Statement -1 is true, Statement-2 is false.
RESPONSE 25. 26. 27. 28. 29.
GRID 30.

MATHEMATICS CHAPTERWISE SPEED TEST-76


Total Questions 30 Total Marks 120
Attempted Correct
Incorrect Net Score
Cut-off Score 38 Qualifying Score 55
Success Gap = Net Score – Qualifying Score
Net Score = (Correct × 4) – (Incorrect × 1)
Space for Rough Work
MATHEMATICS Speed
Matrices TEST
No. of Questions
30
Maximum Marks
120
Time
1 Hour
77
Chapter-wise

GENERAL INSTRUCTIONS
• This test contains 30 MCQ's. For each question only one option is correct. Darken the correct circle/ bubble in the
Response Grid provided on each page.
• You have to evaluate your Response Grids yourself with the help of solutions provided at the end of this book.
• Each correct answer will get you 4 marks and 1 mark shall be deduced for each incorrect answer. No mark will be given/
deducted if no bubble is filled. Keep a timer in front of you and stop immediately at the end of 60 min.
• The sheet follows a particular syllabus. Do not attempt the sheet before you have completed your preparation for that
syllabus.
• After completing the sheet check your answers with the solution booklet and complete the Result Grid. Finally spend time
to analyse your performance and revise the areas which emerge out as weak in your evaluation.

1. If Bn – A = I
écos q - sin q ù
3. If A = ê T
ú , then A + A = I2, if
é26 26 18 ù é1 4 2 ù ë sin q cos q û
and A = ê 25 37 17 ú , B = êê 3 5 1 úú ,
ê ú
p
êë52 39 50úû êë7 1 6úû (a) q = np , n Î Z (b) q = ( 2n + 1) , n Î Z
2
then n = p
(a) 2 (b) 3 (c) q = 2np + ,nÎ Z (d) None of these
3
(c) 4 (d) 5
é1 1ù é0 1ù
2. If A = ê 100 4. If A = ê ú , I is the unit matrix of order 2 and a, b are
ú then A : ë0 0û
ë1 1û
arbitrary constants, then (aI + bA)2 is equal to
(a) 2100A (b) 299A
(a) a2I + abA (b) a2I + 2abA
(c) 2101A (d) None of these 2 2
(c) a I + b A (d) None of these

RESPONSE GRID 1. 2. 3. 4.

Space for Rough Work


EBD_7504
M-70 NTA JEE Main

5. If A is a square matrix, then AAT is a


é 3 1 ù
(a) skew-symmetric matrix (b) symmetric matrix ê ú
2 2 ú é1 1ù
(c) diagonal matrix (d) None of these 11. If P = ê , A= ê T
ú and Q = PAP , then P
ê 1 3ú ë 0 1û
é cos a sin a ù ê– ú
ë 2 2 û
6. If f (a) = ê - sin a cos a ú and if a, b, g, are angle of a
ë û (Q2005)PT equal to
triangle, then f (a). f (b). f(g) equals é 3 / 2 2005ù
é1 2005ù
ê0 ê ú
1 úû
(a) I2 (b) –I2 (a) (b)
ë ë 1 0 û
(c) 0 (d) None of these
7. Let A, B and C be n × n matrices. Which one of the following é 1 2005ù é1 3 / 2ù
is a correct statement? (c) ê ú (d) ê ú
ë 3/2 1 û ë0 2005 û
(a) If AB = AC, then B = C
é1 0 0 ù
(b) If A 3 + 2 A 2 + 3A + 5I = 0 ; then A is invertible.
12. If A = êê0 1 0 úú and I is the unit matrix of order 3, then
(c) If A 2 = 0 , then A = 0 êëa b -1úû
(d) None of these
A2 + 2A4 + 4A6 is equal to
écos a - sin a ù (a) 7A8 (b) 7A7
8. If Aa = ê ú , then
ë sin a cos a û (c) 8I (d) 6I
(a) Aa . A(–a) = I (b) Aa . A(–a) = O æ 1 2ö æ a 0ö
13. Let A = ç ÷ and B = ç , a, b Î N . Then
(c) Aa . Ab = Aab (d) Aa . Ab = Aa – b è 3 4ø è 0 b÷ø
(a) there cannot exist any B such that AB = BA
9. If A is a square matrix such that (A – 2I) (A + I) = O, then A–1 = (b) there exist more than one but finite number of B¢s such
A–I A+I that AB = BA
(a) (b) (c) there exists exactly one B such that AB = BA
2 2 (d) there exist infinitely many B¢s such that AB = BA
14. Given that
(c) 2 (A – I) (d) 2A + I é1 w w2 ù é k 1 1 ù é0 0 0ù
ê ú
10. A square matrix P satisfies P2 = I – P, where I is the identity ê w w2 1 ú êê 1 1 1 úú = êê0 0 0úú
matrix. If Pn = 5I – 8P, then n is equal to êë w2 1 w úû êë 1 1 1 úû êë0 0 0úû
(a) 4 (b) 5 then k =
(a) 6 (b) 1 (c) 8 (d) 9
(c) 6 (d) 7

RESPONSE 5. 6. 7. 8. 9.
GRID 10. 11. 12. 13. 14.
Space for Rough Work
Mathematics M-71


20. For each real number x such that – 1 < x < 1, let A (x) be the
é
ê 0 - tan
If A = ê 2 ú and I is the identity matrix of order é1 – xù x+ y
15. ú matrix (1 – x ) –1 ê and z = Then
ê tan a ú
0 ú ë– x 1û 1+ xy
êë 2 úû
(a) A(z) = A(x) + A(y) (b) A(z) = A(x)[A(y)]–1
é cos a - sin a ù
2, then (I – A) ê ú is equal to
ë sin a cos a û (c) A(z) = A(x) A(y) (d) A(z) = A(x) – A(y)
(a) I + A (b) I – A (c) A – I (d) A é 0 -1ù
21. If A = ê 16
ú , then A is equal to :
é 6 8 5ù ë1 0 û
ê ú é0 -1ù é0 1ù
16. If A = ê 4 2 3ú is the sum of a symmetric matrix B and
(a) ê1 0 ú (b) ê1 0 ú
ëê9 7 1úû ë û ë û
skew-symmetric matrix C, then B is é -1 0 ù é1 0 ù
(c) ê 0 1ú (d) ê0 1ú
é 6 6 7ù é 0 2 –2ù ë û ë û
ê 6 2 5ú ê –2 5 –2ú
(a) ê ú (b) ê ú éa b ù
êë7 5 1úû êë 2 2 0 úû 22. If ê g -a ú is square root of identity matrix of order 2 then –
ë û
é6 6 7ù é 0 6 –2ù (a) 1 + a2 + bg = 0 (b) 1 + a2 – bg = 0
ê –6 2 –5ú ê 2 0 –2ú
(c) ê ú (d) ê ú (c) 1 – a2 + bg = 0 (d) a2 + bg = 1
êë –7 5 1 úû êë –2 -2 0 úû
23. If A and B are matrices of same order, then ( AB¢ - BA ¢ ) is a
17. If A is a square matrix such that A2 = I, then (a) skew symmetric matrix (b) null matrix
(A – I)3 + (A + I)3 – 7A is equal to (c) symmetric matrix (d) unit matrix
(a) A (b) I – A (c) I + A (d) 3A
18. If B is an idempotent matrix, and A = I – B, then é2 1ù é- 3 2 ù é1 0 ù
(a) A2 = A (b) A2 = I 24. If ê úAê ú= ê0 1 ú , then the matrix A equals
(c) AB = I (d) BA = I ë3 2û ë 5 - 3û ë û
écos a - sin a 0 ù
ê ú é1 1 ù é1 1ù
19. Let F (a ) = ê sin a cos a 0 ú then F(a) . F(b) is equal to (a) ê1 0ú (b) ê ú
êë 0 0 1 úû
ë û ë0 1û
é1 0ù é0 1ù
æaö (c) ê1 1 ú (d) ê1 1ú
(a) F(ab) (b) Fçç ÷÷ ë û ë û
èbø
(c) F(a + b) (d) F(a - b)

RESPONSE 15. 16. 17. 18. 19.


GRID 20. 21. 22. 23. 24.
Space for Rough Work
EBD_7504
M-72 NTA JEE Main

25. If A is symmetric as well as skew-symmetric matrix, then A is


(a) Diagonal (b) Null éa n 0 0ù éna 0 0 ù
(c) Triangular (d) None of these ê n ú ê ú
(c) ê0 a 0ú (d)
26. If AB = A and BA = B, then B2 is equal to ê 0 na 0 ú
ê0 0 a n úú êë 0 0 na úû
(a) B (b) A êë û
(c) 1 (d) O
27. If A and B are two square matrices such that
B = – A–1 BA, then (A + B)2 = æ 2 -1ö æ 4 1ö
29. If A = çè -7 4 ÷ø and B = çè 7 2÷ø then which statement is
(a) O (b) A 2 + B2
2 2
(c) A + 2 AB + B (d) A + B true ?
é a 0 0ù (a) AAT = I (b) BBT = I
ê ú
28. Let A = ê0 a 0ú , then An is equal to (c) AB ¹ BA (d) (AB)T = I
êë0 0 a úû 30. If A is any square matrix, then which of the following is
éa n skew-symmetric?
0 0ù éa n 0 0ù
ê n ú ê ú (a) A + AT (b) A – AT (c) AAT (d) ATA – A
(a) ê0 a 0ú (b) ê0 a 0ú
ê0 0 a úú ê0 0 a úú
êë û êë û

RESPONSE 25. 26. 27. 28. 29.


GRID 30.

MATHEMATICS CHAPTERWISE SPEED TEST-77


Total Questions 30 Total Marks 120
Attempted Correct
Incorrect Net Score
Cut-off Score 40 Qualifying Score 55
Success Gap = Net Score – Qualifying Score
Net Score = (Correct × 4) – (Incorrect × 1)
Space for Rough Work
MATHEMATICS Speed
Determinants TEST
No. of Questions
30
Maximum Marks
120
Time
1 Hour
78
Chapter-wise

GENERAL INSTRUCTIONS
• This test contains 30 MCQ's. For each question only one option is correct. Darken the correct circle/ bubble in the
Response Grid provided on each page.
• You have to evaluate your Response Grids yourself with the help of solutions provided at the end of this book.
• Each correct answer will get you 4 marks and 1 mark shall be deduced for each incorrect answer. No mark will be given/
deducted if no bubble is filled. Keep a timer in front of you and stop immediately at the end of 60 min.
• The sheet follows a particular syllabus. Do not attempt the sheet before you have completed your preparation for that
syllabus.
• After completing the sheet check your answers with the solution booklet and complete the Result Grid. Finally spend time
to analyse your performance and revise the areas which emerge out as weak in your evaluation.

1. The value of the determinant (c) is equal to 0


(d) None of these
cos 2 54º cos2 36° cot135° 3. Consider the system of linear equations
sin 2 53° cot135° sin 2 37° is equal to a1 x + b1 y + c1 z + d1 = 0,
a2 x + b2 y + c2 z + d 2 = 0,
cot135° cos 2 25° cos 2 65°
a3 x + b3 y + c3 z + d3 = 0,
(a) –2 (b) –1 Let us denote by D (a, b, c) the determinant
(c) 0 (d) 1
a1 b1 c1
2. If a be a repeated root of the quadratic equation f(x) = 0 and
A(x), B(x), C(x) be polynomials of degrees 3, 4 and 5 a2 b2 c 2 , if D (a, b, c) # 0, then the value of x in the
a3 b3 c3
A( x) B( x ) C ( x) unique solution of the above equations is
respectively, then A(a) B(a ) C (a) D (b, c, d ) -D (b, c, d )
A '(a) B '(a) C '(a ) (a) (b)
D ( a , b, c ) D ( a, b, c )
(a) is divisible by f(x) for all x D (a, c, d ) D ( a , b, d )
(c) (d) –
(b) is not divisible by f(x) for all x D ( a, b, c ) D ( a, b, c )

RESPONSE GRID 1. 2. 3.

Space for Rough Work


EBD_7504
M-74 NTA JEE Main

1 sin A sin 2 A 2r - 1 m
Cr 1
4. If in a triangle ABC, 1 sin B sin 2 B = 0 then the triangle is 2 m
9. If D r = m - 1 2 m +1 then value of
1 sin C sin 2 C
sin 2 ( m 2 ) sin 2 ( m ) sin 2 ( m + 1)
(a) equilateral or isosceles m

(b) equilateral or right-angled å Dr is


r=0
(c) right angled or isosceles
(a) 0 (b) 4
(d) None of these
(c) 3 (d) 1
5. If [ ] denotes the greatest integer less than or equal to the 10. If b2 – ac < 0, a < 0 then the value of
real number under consideration and –1 < x < 0; 0 < y < 1;
1 < z < 2 , then the value of the determinant a b ax + by
b c bx + cy is
[x] + 1 [ y] [z] ax + by bx + cy 0
[ x ] [ y ] + 1 [ z ] is
[x] [ y ] [ z] + 1
(a) Zero (b) Positive
(c) Negative (d) b2 + ac
(a) [z] (b) [y]
é3 7 ù
(c) [x] (d) None of these 11. If A = ê ú , then the value of the determinant
6. If x, y, z are complex numbers, and ëê1 2 úû
|A2013 – 3A2012| is equal to
0 -y -z
(a) 8 (b) – 8
D= y 0 - x then D is (c) 9 (d) –7
z x 0 12. Suppose a, b, gÎR are such that sin a, sin b, sin g ¹ 0 and
(a) purely real (b) purely imaginary
sin 2 a sin a cos a cos 2 a
(c) complex (d) 0
D = sin 2 b sin b cos b cos 2 b then D cannot exceed
7. If the system of linear equations :
x1 + 2x2 + 3x3 = 6 sin 2 g sin g cos g cos 2 g
x1 + 3x2 + 5x3 = 9
1
2x1 + 5x2 + ax3 = b (a) 1 (b) 0 (c) - (d) None of these
2
is consistent and has infinite number of solutions, then : 13. The number of distinct real roots of
(a) a = 8, b can be any real number
sin x cos x cos x
(b) b = 15, a can be any real number p p
cos x sin x cos x = 0 in the interval - £ x £ is
(c) a Î R - {8} and b Î R - {15} 4 4
cos x cos x sin x
(d) a = 8, b = l5
(a) 0 (b) 2
1 ( x – 3) ( x – 3)2 (c) 1 (d) 3
14. For what value of p, is the system of equations :
8. The determinant 1 ( x – 4) ( x – 4 ) 2 vanishes for p3x + (p + 1)3 y = (p + 2)3
px + (p + 1) y = p + 2
1 ( x – 5) ( x – 5) 2 x+y= 1
(a) 3 values of x (b) 2 values of x consistent ?
(c) 1 values of x (d) no value of x (a) p = 0 (b) p = 1
(c) p = – 1 (d) For all p > 1

4. 5. 6. 7. 8.
RESPONSE
9. 10. 11. 12. 13.
GRID
14.
Space for Rough Work
Mathematics M-75

15. The value of


a2 bc
x -x 2 x -x 2 (c) = (d) None of these
(a + a ) (a - a ) 1 p 2 qr
(b x + b- x )2 (b x - b - x )2 1
is 20. If f ( x ) = ax 6 + bx 5 + cx 4 + dx 3 + ex 2 + fx + g
x -x 2 x -x 2
( c +c ) (c - c ) 1
x 2 - 2x + 3 7x + 2 x+4
(a) 0 (b) 2abc = 2x + 7 x2 - x + 2 3x then g =
(c) a 2 b 2 c 2 (d) None of these 3 2x - 1 x 2 - 4x + 7
16. The value of the determinant
(a) –200 (b) 100
1 a a2 (c) 112 (d) –108
cos(n –1) x cosnx cos (n + 1) x 21. Let A, B and C be n × n matrices. Which one of the following
is zero, if is a correct statement?
sin (n –1) x sinnx sin (n + 1) x
(a) If AB = AC, then B = C
(a) sin x = 0 (b) cos x = 0
(b) If A 3 + 2 A 2 + 3A + 5I = 0 ; then A is invertible.
1 + a2
(c) a = 0 (d) cos x = (c) If A 2 = 0 , then A = 0
2a
17. Value of the determinant (when n Î N) (d) None of these
n! (n + 1) ! (n + 2) ! a 5x p 3a 3b c
(n + 1) ! (n + 2) ! (n + 3) ! b 10 y 5 x 2y z
D= is 22. If = 125 , then find the value of
(n + 2) ! (n + 3) ! (n + 4) ! c 15z 15 p 5 5

( n!)2 ( 2n3 - 8n 2 )
(a) 25 (b) 125
(a)
(c) 5 (d) 10
(b) ( 2n!)3 ( 3n 2 + 4n - 5) 23. If a system of equation – ax + y + z = 0
x – by + z = 0
(c) ( n!)3 ( 2n 3 + 8n 2 + 10n + 4 ) x + y – cz = 0 (a, b, c ¹ –1)
(d) None of these 1 1 1
has a non-zero solution then + + =
18. Let Sk = a k + b k + g k , then 1+ a 1+ b 1+ c
S0 S1 S2 (a) 0 (b) 1
D = S1 S2 S3 is equal to (c) 2 (d) 3
S2 S3 S4 24. If a, b, c are sides of a triangle and
(a) S6 (b) S5 – S3
a2 b2 c2
(c) S6 – S4 (d) None of these
(a + 1) 2 (b + 1) 2 (c + 1) 2 = 0 , then
19. Let a1, a2 and b1, b2 be the roots of ax2 + bx + c = 0 and
px2 + qx + r = 0 respectively. If the system of equations (a - 1) 2 (b - 1) 2 (c - 1) 2
a1y + a2z = 0 and b1y + b2z = 0 has a non-trivial solution,
then (a) DABC cannot be equilateral triangle
(b) DABC is a right angled isosceles triangle
b2 ac c2 ab (c) DABC is an isosceles triangle
(a) = (b) =
2 pr 2 pq
q r (d) None of these

RESPONSE 15. 16. 17. 18. 19.


GRID 20. 21. 22. 23. 24.

Space for Rough Work


EBD_7504
M-76 NTA JEE Main

é1 ù
écos a - sin a 0ù ê 0 0ú
é a 0 0ù a
25. Let F(a ) = êê sin a cos a 0úú where a Î R . Then ê ú
ê
ê 1
ú
29. -1
Statement 1 : If A = ê 0 b 0ú then A = ê 0 0ú
êë 0 0 1úû b ú
êë 0 0 c úû ê 1 úú
ê 0 0
[F(a)]-1 is equal to ëê c úû

(a) F(-a) (b) F(a -1 ) Statement 2 : The inverse of a diagonal matrix is a diagonal
matrix.
(c) F(2a) (d) None of these (a) Statement -1 is true, Statement-2 is true; Statement -2 is
26. Consider the system of equations in x, y, z as a correct explanation for Statement-1.
x sin 3q – y + z = 0, x cos 2q + 4y + 3z = 0, (b) Statement -1 is true, Statement-2 is true; Statement -2
2x + 7y + 7z = 0. If this system has a non-trivial solution, is not a correct explanation for Statement-1.
then for integer n, values of q are given by (c) Statement -1 is false, Statement-2 is true.
æ (d) Statement -1 is true, Statement-2 is false.
(–1)n ö æ (–1)n ö 30. Statement-1 : If three lines L1 : a1x + b1 y + c 1 = 0,
(a) pçn + ÷ (b) pçn + ÷
è 3 ø è 4 ø L2 : a2x + b2y + c2 = 0 and L3 : a3x + b3y + c3 = 0 are concurrent
æ (–1)n ö np a1 b1 c1
(c) pçn + ÷ (d)
è 6 ø 2 lines, then a 2 b2 c2 = 0 .
27. If adj B = A, P = Q =1 , then adj (Q-1B P -1 ) is a3 b3 c3
(a) PQ (b) QAP a1 b1 c1
(c) PAQ (d) PA–1Q Statement-2 : If a 2 b2 c2 = 0 then the lines L1, L2,
28. Let l and a be real. The set of all values of x for which the
system of linear equations a 3 b3 c3
l x + (sin a) y + (cos a) z = 0 L3 must be concurrent.
x + (cos a) y + (sin a) z = 0 (a) Statement -1 is true, Statement-2 is true; Statement -2
– x + (sin a) – (cos a) z = 0 has a non-trivial solution, is is a correct explanation for Statement-1.
(a) [0, 2 ] (b) [- 2, 0 ] (b) Statement -1 is true, Statement-2 is true; Statement -2
is not a correct explanation for Statement-1.
(c) [- 2 , 2 ] (d) None of these (c) Statement -1 is false, Statement-2 is true.
(d) Statement -1 is true, Statement-2 is false.

RESPONSE 25. 26. 27. 28. 29.


GRID 30.

MATHEMATICS CHAPTERWISE SPEED TEST-78


Total Questions 30 Total Marks 120
Attempted Correct
Incorrect Net Score
Cut-off Score 36 Qualifying Score 53
Success Gap = Net Score – Qualifying Score
Net Score = (Correct × 4) – (Incorrect × 1)
Space for Rough Work
MATHEMATICS Speed
Continuity and Differentiability TEST
No. of Questions
30
Maximum Marks
120
Time
1 Hour
79
Chapter-wise

GENERAL INSTRUCTIONS
• This test contains 30 MCQ's. For each question only one option is correct. Darken the correct circle/ bubble in the
Response Grid provided on each page.
• You have to evaluate your Response Grids yourself with the help of solutions provided at the end of this book.
• Each correct answer will get you 4 marks and 1 mark shall be deduced for each incorrect answer. No mark will be given/
deducted if no bubble is filled. Keep a timer in front of you and stop immediately at the end of 60 min.
• The sheet follows a particular syllabus. Do not attempt the sheet before you have completed your preparation for that
syllabus.
• After completing the sheet check your answers with the solution booklet and complete the Result Grid. Finally spend time
to analyse your performance and revise the areas which emerge out as weak in your evaluation.

1. Let f : R ® R be a function such that | y '' | | y '' |


(a) (b)
æ x + y ö f ( x) + f ( y)

è 3 ø
÷=
3
, f (0) = 0 and f ' (0) = 3 . Then
1+ y '
(1 + y ' ) 2 3

(a) f(x) is a quadratic function 2 y '' y ''


(b) f(x) is continuous but not differentiable (c) (d)
( )
2 3
(c) f(x) is differentiable in R 1+ y ' 2 1 + y '2
(d) f(x) is bounded in R
4. Let a function f : R ® R satisfy the equation
ì( cos x )1/ x , x ¹ 0 f (x + y) = f(x) + f(y) for all x, y, If the function f(x) is
2. If the function f (x) = í is continuous at x = 0,
î k, x = 0 continuous at x = 0, then
then the value of k is (a) f(x) = 0 for all x
(a) 1 (b) –1 (b) f(x) is continuous for all positive real x
(c) 0 (d) e (c) f(x) is continuous for all x
(d) None of these
1
3. If x2 + y2 = a2 and k = , then k is equal to
a

RESPONSE GRID 1. 2. 3. 4.

Space for Rough Work


EBD_7504
M-78 NTA JEE Main

2x 10. The value of p for which the function


5. Differential coefficient of tan -1 with respect to
1 - x2 ì (4 x - 1)3
2x
-1 ï ,x¹0
sin will be ï x é x2 ù
1 + x2 f ( x) = í sin log ê1 + ú
ï p ëê 3 úû
(a) 1 (b) – 1 ï
î 12(log 4)3 , x = 0
(c) – 1/2 (d) x may be continuous at x = 0, is
(a) 1 (b) 2
6. The values of a, b and c which make the function
(c) 3 (d) None of these
ì f (b) - f (a)
ï sin(a + 1)x + sin x , x < 0 11. In the mean value theorem = f ¢ (c) , if a = 0,
ï x b-a
ï
f (x) = í c ,x = 0 b = 1/2 and f (x) = x (x – 1) (x – 2), the value of c is –
ï 2
ï x + bx - x 15
ï ,x > 0 (a) 1 - (b) 1 + 15
î bx 3 / 2 6
continuous at x = 0 are 21
(c) 1 - (d) 1 + 21
-3 1 3 1 6
(a) a= , c = , b= 0 (b) a= , c= , b¹0
2 2 2 2 2 y 2 (2) + 1/ 8
x
12. If y = (1 + 1/ x ) , then is equal to –
-3 1 (log 3/ 2 - 1/ 3)
(c) a= , c = , b ¹ 0 (d) None of these
2 2 (a) 3 (b) 4
7. Let f(x), g(x) be two continuously differentiable functions (c) 1 (d) 2
satisfying therelationshipsf ¢(x) = g(x) and f ²(x) = – f(x). ¥
xn
Let h(x) = [f(x)]2 + [g(x)]2. If h(0) = 5, then h (10)=
(a) 10 (b) 5
13. If f ( x ) = å n!
(log a ) n , then at x = 0, f(x)
n =0
(c) 15 (d) None of these
8. The function f(x) = [x]2 – [x2] (where [y] is the greatest integer (a) has no limit
function less than or equal to y), is discontinuous at : (b) is discontinuous
(a) all integers (c) is continuous but not differentiable
(d) is differentiable
(b) all integers except 0 and 1
(c) all integers except 0 e1 / x - e -1 / x
14. Let f ( x ) = g ( x ) . , where g is a continuous
(d) all integers except 1 e1 / x + e -1 / x
function then lim f ( x) does not exist if
tan px 2 + ( x + 1) n sin x x®0
9. If f ( x ) = lim 2 n
, then
n ®¥ x + (x + 1) (a) g(x) is any constant function
(a) f is continuous at x = 0 (b) g(x) = x
(b) f is differentiable at x = 0 (c) g(x) = x2
(c) f is continuous but not differentiable at x = 0 (d) g(x) = x h (x), where h(x) is a polynomial
(d) None of these

RESPONSE 5. 6. 7. 8. 9.
GRID 10. 11. 12. 13. 14.

Space for Rough Work


Mathematics M-79

20. Which of the following functions is differentiable at x = 0?


x x - x-x ö
-1 æ
15. If f (x) = cot ç
è 2 ÷ , then f '(1) is equal to
ø
(a) ( )
cos x + x (b) ( )
cos x - x

(a) –1 (b) 1 (c) sin ( x ) + x (d) sin ( x ) - x


(c) log 2 (d) –log 2
16. Let f : R ® R be a function defined by f (x) = max {x, x3}. The (e x - 1)2
set of all points where f (x) is NOT differentiable is 21. Let f (x) = for x ¹ 0, and f(0) = 12. If f is
æ xö æ xö
(a) {-1, 1} (b) {-1, 0} sin ç ÷ log ç1 + ÷
è aø è 4ø
(c) {0, 1} (d) {-1, 0, 1}
2
continuous at x = 0, then the value of a is equal to
17. The function f ( x ) = (sin 2x ) tan 2x
is not defined at (a) 1 (b) –1
p p (c) 2 (d) 3
æpö
x= . The value of f ç ÷ so that f is continuous at x =
4 è4ø 4 22. If the equation a n x n + a n -1x n -1 + ............. + a1 x = 0
is a1 ¹ 0, n ³ 2, has a positive root x = a , then the equation
1 n -2
(a) e (b) na n x n -1 + (n – 1) a n -1x + ......... + a1 = 0 has a positive
e root, which is
(c) 2 (d) None of these (a) greater than a
18. If g is the inverse function of f and f '( x) = sin x, then (b) smaller than a
g '( x ) is (c) greater than or equal to a
(d) equal to a
(a) cosec { g ( x)} (b) sin { g ( x)}
ì sin [ x ]
1 ï , for x > 0
(c) -
sin { g ( x)} (d) cos { g ( x)} ï [ x] + 1
ï p
23. If f (x) = í cos [ x ]
ì1 - sin 3 x p 2 for x < 0 ; where [x] denotes the
ï ,x < ï ,
ï 3 cos x
2 2 ï [ x]
ïï ï k, at x = 0
p î
19. Let f ( x ) = íp, x =
ï 2 greatest integer less than or equal to x, then in order that f
ï q (1 - sin x ) p be continuous at x =0, the value of k is
ï ,x >
ïî (p - 2 x )
2 2 (a) equal to 0 (b) equal to 1
(c) equal to –1 (d) indeterminate
p tan[ x ]p
If f(x) is continuous at x = , (p, q) = 24. If f (x) = , where [.] denotes the greatest
2 [1+ | log(sin 2 x + 1) |]
æ1 ö integer function and |.| stands for the modulus of the function,
(a) (1, 4) (b) ç , 2 ÷ then f(x) is
è2 ø
(a) discontinuous "x Î I
æ1 ö (b) continuous " x
(c) ç , 4÷ (d) None of these
è2 ø (c) non differentiable " x Î I
(d) a periodic function with fundamental period 1.

RESPONSE 15. 16. 17. 18. 19.


GRID 20. 21. 22. 23. 24.

Space for Rough Work


EBD_7504
M-80 NTA JEE Main

(c) The existence of lim (f (x) + g(x)) does not imply of


1 - x 2 n dy x ®c
If 1 - x 2n 2n n n
25. + 1- y = a( x - y ) , then 1 - y 2n dx
is existence of lim f (x) and lim g(x) .
x ®c x ®c
equal to (d) All of these
(a) 1 (b) x/y 29. Statement-1 : If g (x) is a differentiable function g (1) ¹ 0,
g (–1) ¹ 0 and Rolles theorem is not applicable to
x n -1 x2 - 1
f (x) = in [–1,1], then g (x) has atleast one root in
(c) (d) None of these g(x)
y n -1 (–1, 1).
26. The equation ex–8 + 2x – 17 = 0 has Statement-2 : If f (a) = f (b), then Rolles theorem is applicable
(a) two real roots (b) one real root for x Î (a, b).
(c) eight real roots (d) four real roots (a) Statement- 1 is True, Statement-2 is True, Statement-2
is a correct explanation for Statement -1
27. If f '' (x) = – f (x) and g (x) = f ' (x) and (b) Statement -1 is True, Statement -2 is True ; Statement-
2 2 2 is NOT a correct explanation for Statement - 1
æ æ xö ö æ æ xö ö
F( x ) = ç f ç ÷ ÷ + ç g ç ÷ ÷ and given that (c) Statement -1 is False, Statement -2 is True
è è 2ø ø è è 2 ø ø (d) Statement - 1 is True, Statement- 2 is False
F (5) =5, then F (10) is equal to – 30. Statement-1 : f (x) = | x | sin x, is differentiable at x = 0.
(a) 5 (b) 10 Statement-2 : If f (x) is not differentiable and g (x) is
(c) 0 (d) 15 differentiable at x = a, then f (x) . g (x) can still be differentiable
at x = a.
28. Choose the correct statements – (a) Statement-1 is true, Statement-2 is true, Statement-2 is
(a) If f ' (a+) and f ' (a–) exist finitely at a point, then f is a correct explanation for Statement -1
continuous at x = a. (b) Statement -1 is True, Statement -2 is True ; Statement-
(b) The function f (x) = 3 tan 5x – 7 is differentiable at all 2 is NOT a correct explanation for Statement - 1
points in its domain. (c) Statement -1 is False, Statement -2 is True
(d) Statement - 1 is True, Statement- 2 is False

RESPONSE 25. 26. 27. 28. 29.


GRID 30.

MATHEMATICS CHAPTERWISE SPEED TEST-79


Total Questions 30 Total Marks 120
Attempted Correct
Incorrect Net Score
Cut-off Score 41 Qualifying Score 60
Success Gap = Net Score – Qualifying Score
Net Score = (Correct × 4) – (Incorrect × 1)
Space for Rough Work
MATHEMATICS Speed
Applications of Derivatives TEST
No. of Questions
30
Maximum Marks
120
Time
1 Hour
80
Chapter-wise

GENERAL INSTRUCTIONS
• This test contains 30 MCQ's. For each question only one option is correct. Darken the correct circle/ bubble in the
Response Grid provided on each page.
• You have to evaluate your Response Grids yourself with the help of solutions provided at the end of this book.
• Each correct answer will get you 4 marks and 1 mark shall be deduced for each incorrect answer. No mark will be given/
deducted if no bubble is filled. Keep a timer in front of you and stop immediately at the end of 60 min.
• The sheet follows a particular syllabus. Do not attempt the sheet before you have completed your preparation for that
syllabus.
• After completing the sheet check your answers with the solution booklet and complete the Result Grid. Finally spend time
to analyse your performance and revise the areas which emerge out as weak in your evaluation.

1. The point P of curve y2 = 2x3 such that the tangent at P is 3. Let AP and BQ be two vertical poles at points A and B
perpendicular to the line 4x – 3y + 2 = 0 is given by : respectively. If AP = 16 m, BQ = 22 m and AB = 20 m, then
(a) (2, 4) (b) (1, the distance of a point R on AB from the point A such that
2)
RP2 + RQ2 is minimum, is
æ1 1ö æ1 1 ö (a) 5 m (b) 6 m
(c) ç , ÷ (d) çè , - ÷ø (c) 10 m (d) 14 m
è 2 2ø 8 16
4. If the tangent at any point on the curve x4 + y4 = a4 cuts off
2. f (x) = 2x2 – log | x | (x ¹ 0) is monotonic increasing in the
intercepts p and q on the co-ordinate axes then the value of
interval
p–4/3+q–4/3 is
(a) (1/2, ¥) (b) (– ¥, –1/2) È (1/2, ¥)
(c) (– ¥, –1/2) È (0, 1/2) (d) (–1/2, 0) È (1/2, ¥) (a) a –4/3 (b) a –1/2
(c) a 1/2 (d) None

RESPONSE GRID 1. 2. 3. 4.

Space for Rough Work


EBD_7504
M-82 NTA JEE Main

5. The minimum value of the function (a) [1, ¥) (b) (0, 1) È (1, 4)
(c) (–2, 4) (d) (1, 3) È (3, 5)
3/2 -3/2 æ 1ö
f (x) = x + x - 4 ç x + ÷ for all permissible real x, is 10. Let F(x) = x3 + ax2 + bx + 5 sin2 x be an increasing function in
è xø the set of real number R. Then a and b satisfy the
(a) – 10 (b) –6 condition.
(c) –7 (d) – 8 (a) a2 – 3b – 15 > 0 (b) a2 – 3b + 15 > 0
2
6. A particle moves along the curve 6y = x3 + 2. The point ‘P’ (c) a + 3b – 15 < 0 (d) a > 0 and b > 0
on the curve at which the y-coordinate is changing 8 times - | x|
11. The equation of the tangent to the curve y = e at the
æ 31 ö point where the curve cuts the line x = 1 is
as fast as the x-coordinate, are (4, 11) and ç -4, - ÷ .
è 3ø (a) e(x + y) = 1 (b) y + ex = 1
(a) x-coordinates at the point P are ± 4 (c) y + x = e (d) None of these
-31 12. A stone is dropped into a quiet lake and waves move in a
(b) y-coordinates at the point P are 11 and circle at a speed of 3.5 cm/sec. At the instant when the
3
radius of the circular wave is 7.5 cm. How fast is the en-
(c) Both (a) and (b)
closed area increasing?
(d) None of the above
(a) 32.5 p cm2/sec (b) 31.5 p cm2/sec
7. Which of the following statements is false?
(c) 52.5 p cm2/sec (d) None of these
(a) The length of sub-tangent to the curve x2y2 = 16a4 at
the point (–2a, 2a) is 2a. x y
13. The straight line + = 2 touches the curve
(b) x + y = 3 is a normal to the curve x2 = 4y a b
n n
(c) Curves y = –4x2 and y = e–x/2 are orthogonal. æxö æyö
(d) If a Î (–1, 0), then tangent at each point of the curve ç ÷ + ç ÷ = 2 at the point (a, b) for
èaø èbø
2 3 (a) n = 1, 2 (b) n = 3, 4, –5
y= x - 2ax 2 + 2x + 5 makes an acute angle with (c) n = 1, 2, 3 (d) Any value of n
3 14. The function f (x) = 3 cos4x + 10 cos3x + 6 cos2x – 3,
the positive direction of x-axis. (0 £ x £ p) is –
8. Find the coordinates of a point of the parabola
y = x2 + 7x + 2 æ p 2p ö
which is closest to the straight line y = 3x – 3. (a) Increasing in çè , ÷ø
2 3
(a) (–2, –8) (b) (2, 8)
(c) (–2, 8) (d) (2, –8) æ p ö æ 2p ö
(b) Increasing in çè 0, ÷ø È çè , p÷ø
9. The set of all values of a for which the function 2 3
æ cos 2 x sin 2 x ö æ p 2p ö
f (x) = (a 2 - 3a + 2) ç - ÷ + (a - 1)x + sin1 (c) Decreasing in çè , ÷ø
è 4 4 ø 2 3
does not possess critical points is (d) All of above

RESPONSE 5. 6. 7. 8. 9.
GRID 10. 11. 12. 13. 14.

Space for Rough Work


Mathematics M-83

15. The diagonal of a square is changing at the rate of 0.5 cm/


20. LL' is the latus rectum of the parabola y 2 = 36x and PP' is
sec. Then the rate of change of area, when the area is
400 cm2, is equal to double ordinate drawn between the vertex and the latus
rectum. The area of the trapezium PP'L'L is maximum when
(a) 20 2 cm 2 / sec (b) 10 2 cm 2 / sec the distnace of PP' from the vertex is
(a) 1 (b) 4
1 10 (c) 9 (d) 36
(c) cm2 / sec (d) cm 2 / sec 21. The value(s) of ‘a’ for which the area of the triangle included
10 2 2
between the axes and any tangent to the curve xa y = la is a
x2 y 2 constant is/are :
16. If the curves + = 1 and y3 = 16x intersect at right
a 4 1
angles, then a value of a is : (a) (b) 1
2
4
(a) 2 (b) 3
3 (c) (d) 2
1 3 2
(c) (d) 22. The minimum value of the function
2 4
17. The difference between the greatest and least values of the a2 b2
y= + , a > 0, b > 0 in (0, a) is :
é -p p ù x a-x
function f(x) = sin 2x – x, on ê , ú is
ë 2 2û
1
p (a) a + b (b)
(a) (b) p a+b
2
3p p 1 1
(c) (d) (c) (a + b ) 2 (d) (a + b)
2 4 a a2
18. If the normal at the point " t1" on the curve xy = c2 meets the
curve again at " t2", then 23. If y = m log x + nx2 + x has its extreme values at x = 2
and x = 1, then 2m + 10n =
(a) t13 t2 = 1 (b) t13 t2 = –1 (a) –1 (b) –4
(c) t1 t23 = – 1 (d) t1 t23 =1 (c) –2 (d) –3
19. The largest distance of the point (a, 0) from the curve 24. Find the greatest value of the function
2x2 + y2 – 2x = 0, is given by sin 2 x é pù
f ( x) = on the interval ê0, 2 ú
(a) (1 - 2a + a 2 ) (b) (1 + 2a + 2a 2 ) æ pö ë û
sin ç x + ÷
è 4ø
(c) (1 + 2a - a 2 ) (d) (1 - 2a + 2a 2 ) (a) 1 (b) 2
(c) 3 (d) None of these

RESPONSE 15. 16. 17. 18. 19.


GRID 20. 21. 22. 23. 24.

Space for Rough Work


EBD_7504
M-84 NTA JEE Main

a 2 -1 (a) f {g( x )} ³ f {g(0)} (b) g{f ( x )} £ g{f (0)}


25. If f ( x ) = x 3 - 3x + 5 is a decreasing function of x in R
2
a +1 (c) f {g(2)} = 7 (d) None of these
then the set of possible values of a (independent of x) is 29. Statement-1 : The minimum distance of the fixed point
(a) (1, ¥) (b) (-¥, - 1) 1
(0, y0) where 0 £ y 0 < , from the curve y = x2 is y0.
(c) [-1, 1] (d) None of these 2
x2 Statement-2 : Maxima and minima of a function is always a
26. Tangent is drawn to ellipse
27
(
+ y 2 = 1 at 3 3 cos q, sin q ) root of the equation f ' (x) = 0.
[where q Î (0, p/2)]. Then the value of q such that sum of (a) Statement- 1 is True, Statement-2 is True, Statement-2
intercepts on axes made by this tangent is minimum, is is a correct explanation for Statement -1
(b) Statement -1 is True, Statement -2 is True ; Statement-
(a) p 3 (b) p 6 (c) p 8 (d) p 4 2 is NOT a correct explanation for Statement - 1
2 2
27. The condition that the curves ax + by = 1 (c) Statement -1 is False, Statement -2 is True
and a1x2 + b1y2 = 1 may cut each other orthogonally is (d) Statement - 1 is True, Statement- 2 is False
a - a 1 b - b1 a + a1 b + b1 30. Statement-1 : If f(0) = 0, f ¢(x) = ln (x + 1 + x 2 ), then f(x) is
(a) = (b) =
aa1 bb1 aa1 bb1 positive for all x Î R0
a - a1 b - b1 Statement-2: f(x) is increasing for x > 0 and decreasing for x < 0.
(c) = (d) none of these (a) Statement- 1 is True, Statement-2 is True, Statement-2
a + a 1 b + b1
is a correct explanation for Statement -1
28. Let f and g be functions from the interval [0, ¥ ) to the (b) Statement -1 is True, Statement -2 is True ; Statement-
2 is NOT a correct explanation for Statement - 1
interval [0, ¥), f being an increasing and g being a
(c) Statement -1 is False, Statement -2 is True
decreasing function. If f {g(0)} = 0 then (d) Statement - 1 is True, Statement- 2 is False

RESPONSE 25. 26. 27. 28. 29.


GRID 30.

MATHEMATICS CHAPTERWISE SPEED TEST-80


Total Questions 30 Total Marks 120
Attempted Correct
Incorrect Net Score
Cut-off Score 38 Qualifying Score 53
Success Gap = Net Score – Qualifying Score
Net Score = (Correct × 4) – (Incorrect × 1)
Space for Rough Work
MATHEMATICS Speed
Integrals TEST
No. of Questions
30
Maximum Marks
120
Time
1 Hour
81
Chapter-wise

GENERAL INSTRUCTIONS
• This test contains 30 MCQ's. For each question only one option is correct. Darken the correct circle/ bubble in the
Response Grid provided on each page.
• You have to evaluate your Response Grids yourself with the help of solutions provided at the end of this book.
• Each correct answer will get you 4 marks and 1 mark shall be deduced for each incorrect answer. No mark will be given/
deducted if no bubble is filled. Keep a timer in front of you and stop immediately at the end of 60 min.
• The sheet follows a particular syllabus. Do not attempt the sheet before you have completed your preparation for that
syllabus.
• After completing the sheet check your answers with the solution booklet and complete the Result Grid. Finally spend time
to analyse your performance and revise the areas which emerge out as weak in your evaluation.

1.
sin 8 x - cos 8 x
ò 1 - 2 sin 2 x cos 2 x dx is equal to (
x + cos -1 3x )
2
3. If
ò 2
dx = A 1 - 9 x 2 + B (cos–13x)3 + c,
1 - 9x
where c is integration constant, then the values of A and B
1 1
(a) sin 2 x + C (b) - sin 2 x + C are :
2 2
1 1 1 1
(a) A = - , B = - (b) A = - , B =
1 2 9 9 9 9
(c) - sin x + C (d) - sin x + C
2 1 1
(c) A= , B=- (d) None of these
p 9 9
2 æ e sin x ö
d
2. Let In = ò sin
n
x dx , n is a positive integer. Then 4. Let F ( x) = ç ÷, x > 0.
dx ç x ÷
0 è ø
43 3
(a) In : In – 2 = n : (n – 1) (b) In > In – 2 If ò e sin x dx = F ( k ) - F (1) then one of the possible
x 1
(c) n (In – 2 – In) = In – 2 (d) None of these
values of k, is
(a) 64 (b) 15 (c) 16 (d) 63

RESPONSE GRID 1. 2. 3. 4.

Space for Rough Work


EBD_7504
M-86 NTA JEE Main

sin x 3 3
-1 æ
ò 5 ö2 1æ 1 ö2
3
5. If f ( x ) = cos (t ) dt , then f ¢(x) is equal to (c) ç1 + 2 ÷ + C (d) ç1 + ÷ + C
2x 15 è x ø 15 è x 2 ø
(a) cos (sin3 x) cos x – 2 cos (8x3)
ln 3
(b) sin (sin3 x) sin x – 2 sin (8x3) x sin x 2
(c) cos (cos3 x) cos x – 2 cos (x3) 10. The value of ò sin x 2 + sin(ln6 - x 2 )
dx is
(d) cos (sin 3 x) – cos (8x3) ln 2
a a 2a 1 3 1 3
(a) ln (b) ln
6. If ò f (2a - x)dx = m and ò f (x)dx = n, then
ò f (x)dx is 4 2 2 2
0 0 0 3 1 3
equal to (c) ln (d) ln
2 6 2
(a) 2m + n (b) m + 2n p
(c) m – n (d) m + n 2
sin 2 n x
é 10 -2n ù é 10 2n +1 ù 11. If a n = ò dx then
7. ê
êå ò sin 27 x dx ú + ê
ú ê åò
sin 27 x dx ú =
ú
0
sin x

ë n =1 - 2n -1 û ë n =1 2n û a 2 - a 1 , a 3 - a 2 , a 4 - a 3 ,............ are in
(a) 272 (b) –54 (c) 54 (d) 0 (a) A.P. (b) G.P.
(c) H.P. (d) None of these
æ xö
tan (ln x) tan ç ln ÷ tan (ln 2)
è 2ø 1 + nx n -1 - x 2 n
8. ò dx = 12. The value of ò e x dx is
x (1 - x n ) 1 - x 2n
æ ö
ç sec (ln x) ÷ 1- xn 1 + x 2n
(a) ex +C (b) ex +C
ln ç ÷ +C 1- xn 1 - x 2n
(a) ç sec æ ln x ö ÷
çè çè ÷
2 ø ÷ø 1 - x 2n 1 - x 2n
(b) ln (sec ln x) + C (c) ex +C (d) ex +C
1 - x 2n 1- x n
æ æ xö ö ¥ ¥
(c) ln ç sec ln ç ÷ x tan (ln x) ÷ + C 1
è è 2 ø ø 13. If
ò e -ax dx =
a ò
, then x n e -ax dx is
0 0
æ ö
ç sec (ln x) ÷ (-1) n n! ( -1) n (n - 1)!
(d) ln ç ÷ +C (a) (b)
ç sec æ ln x ö x tan (ln 2) ÷ a n +1 an
çè çè ÷ ÷ø
2ø n!
(c) n +1 (d) None of these
a
5 + x2
9. ò x4
dx is equal to 1
cos x
1
sin x
14. Let J = ò dx and K = ò dx . Then which one of
3 3
0
x 0
x
1æ 5 ö2 -1 æ 1 ö2
(a) ç1 + 2 ÷ + C (b) ç1 + 2 ÷ + C the following is true ?
15 è x ø 15 è x ø (a) K < 2/3 and J < 2 (b) K > 2/3 and J > 2
(c) K < 2/3 and J > 2 (d) K > 2/3 and J < 2
RESPONSE 5. 6. 7. 8. 9.
GRID 10. 11. 12. 13. 14.
Space for Rough Work
Mathematics M-87

¥
é 2 ù -1 x (1 + x + x 2 )
The value of ò e tan
15. ò êë e x úû dx is equal to ([x] = greatest integer £ x) 20.
1+ x2
dx is
0
-1
(a) x etan x+C (b) tan -1 x + C
2
(a) log e 2 (b) e2 (c) 0 (d) -1
e (c) e tan x + 2 x + C (d) None of these
16. The line y = a intersects the curve y = g (x), atleast at two 21. Given that f and g are continuous functions on [0 a]
x 2 satisfying f(a–x) = f(x) and g(x)+g(a–x) = 2.
x2
points. If ò g (t) dt = 2 ò
+ t 2g(t) dt then possible value a
2 x Then ò0 f (x) g (x) dx is equal to :
of a is/are –
a a

(a)
æ 1 1ö
çè - , ÷ø (b)
é 1 1ù
êë - 2 , 2 úû
(a) ò0 f (x) dx (b) ò0 g(x) dx
2 2
(c) 0 (d) None of these
æ 1 1ö ì 1 1ü
(c) çè - , ÷ø - {0} í- , 0, ý (d) 6x + 7
2 2 î 2 2þ 22. If ò dx
(x - 5)(x - 4)
17. Let the equation of a curve passing through the point (0,1)
3
be given by y= ò x 2 .e x dx. If the equation of the curve is 9
= A x 2 - 9x + 20 + B log x + x 2 - 9x + 20 - +C
2
written in the form x = f(y) then f(y) is
log e (3y - 2) log e (3y - 2)
Then the values of A and B are
(a) (b) 3
(a) 6, 34 (b) 3, 9
(c) 3 log e (2 - 3y ) (d) None of these (c) 12, 17 (d) None of these

p 2 b |x|
18. The integral ò0 | sin x - cos x | dx is equal to : 23. The value of the integral òa x
dx , where a < b is

(a) b – a (b) |b| – |a|


(a) 2 2 (b) 2( 2 - 1)
(c) a + b (d) |a| + |b|
(c) 2 +1 (d) None of these
19. Let f be a positive function if 24. lim é 1 1 1 ù
n ®µ ê + +.....+ =
ë 2n+1 2n+2 2n+n úû
k k
I1 = ò xf {x (1 - x )} dx and I 2 = ò f {x (1 - x)} dx 1 2
1-k 1-k (a) log e (b) log e
3 3
where 2k – 1 >0, then I1 : I2 is equal to
1 3 4
(a) 2 (b) k (c) (d) 1 (c) log e (d) log e
2 2 3

RESPONSE 15. 16. 17. 18. 19.


GRID 20. 21. 22. 23. 24.
Space for Rough Work
EBD_7504
M-88 NTA JEE Main

25. Let f : R ® R be a differentiable function and f (1) = 4. Then


29. Let g(x) be on integrable function and ò g(x) dx = g(x) .
f (x)

ò 2t dt Statement-1 : ò g(x) (f (x) - f ''(x)) dx


4
the value of lim , if f ' (1) = 2 is – = g(x)(f (x) - f '(x)) + C
x ®1 x -1
(a) 16 (b) 8 (c) 4 (d) 2 Statement-2 : ò g(x) (f (x) + f '(x)) dx = g(x)f (x) + C
1 (a) Statement-1 is true, Statement-2 is true, Statement-2 is
26. ò0[f ( x)g"( x ) - f " (x ) g (x )] dx is equal to : a correct explanation for Statement -1
(b) Statement -1 is true, Statement -2 is true ; Statement-2
[Given f(0) = g(0) = 0] is NOT a correct explanation for Statement - 1
(a) f(1) g(1) – f(1)g'(1) (b) f(1) g'(1) + f '(1)g(1)
(c) Statement -1 is false, Statement -2 is true
(c) f(1) g'(1) – f '(1)g(1) (d) None of these
(d) Statement - 1 is true, Statement- 2 is false
x

ò
10
27. Let f : (0, ¥) ® R and F( x ) = f ( t ) dt . If F(x2) = x2(1 + x),
0
30. Statement-1 : ò {x - [x]}dx = 5
then f(4) equals 0

5 na a

ò f (x)dx = n ò f (x)dx if f(x + a) = f(x)


(a) (b) 7 (c) 4 (d) 2 Statement-2 :
4
1000 x -[x] a 0
28. ò0 e dx is (a) Statement -1 is false, Statement-2 is true
(b) Statement -1 is true, Statement-2 is true; Statement -2 is
e1000 - 1 a correct explanation for Statement-1
(a) e1000 - 1 (b)
(c) Statement -1 is true, Statement-2 is true; Statement -2
e -1
is not a correct explanation for Statement-1
e -1
(c) 1000(e – 1) (d) (d) Statement -1 is true, Statement-2 is false
1000
RESPONSE 25. 26. 27. 28. 29.
GRID 30.

MATHEMATICS CHAPTERWISE SPEED TEST-81


Total Questions 30 Total Marks 120
Attempted Correct
Incorrect Net Score
Cut-off Score 36 Qualifying Score 53
Success Gap = Net Score – Qualifying Score
Net Score = (Correct × 4) – (Incorrect × 1)
Space for Rough Work
MATHEMATICS Speed
Applications of Integrals TEST
No. of Questions
30
Maximum Marks
120
Time
1 Hour
82
Chapter-wise

GENERAL INSTRUCTIONS
• This test contains 30 MCQ's. For each question only one option is correct. Darken the correct circle/ bubble in the
Response Grid provided on each page.
• You have to evaluate your Response Grids yourself with the help of solutions provided at the end of this book.
• Each correct answer will get you 4 marks and 1 mark shall be deduced for each incorrect answer. No mark will be given/
deducted if no bubble is filled. Keep a timer in front of you and stop immediately at the end of 60 min.
• The sheet follows a particular syllabus. Do not attempt the sheet before you have completed your preparation for that
syllabus.
• After completing the sheet check your answers with the solution booklet and complete the Result Grid. Finally spend time
to analyse your performance and revise the areas which emerge out as weak in your evaluation.

1. Let f(x) be a continuous function such that the area bounded 3. The area of the region bounded by the curve y = x | x |,
by the curve y = f(x), x-axis and the lines x-axis and the ordinates x = 1, x = -1 is given by :
a2 a p æ pö (a) zero (b)
1
x = 0 and x = a is + sin a + cos a, then f ç ÷ = 3
2 2 2 è 2ø
1 2
(a) 1 (b) (c) (d) 1
2 3
1 4. The area of the region
(c) (d) None of these
3 {( x, y) : 0 £ y £ x 2 + 1, 0 £ y £ x + 1, 0 £ x £ 2} is
2. The area above the x–axis enclosed by the curves
x2–y2 = 0 and x2 + y – 2 = 0 is 23
(a) (b) 2 2+5
5 7 6
(a) (b) 9
3 3 (c) (d) none of these
2
8 10
(c) (d)
3 3
RESPONSE GRID 1. 2. 3. 4.

Space for Rough Work


EBD_7504
M-90 NTA JEE Main

5. The area of the region (in sq. units), in the first quadrant 3 3
bounded by the parabola y = 9x2 and the lines x = 0, y = l and (a) (b)
5 4
y = 4, is :
(a) 7/9 (b) 14/3 7 5
(c) (d)
(c) 7/3 (d) 14/9 8 6
6. The area bounded by the x-axis, the curve y = f(x) and the 10. The triangle formed by the tangent to the curve f(x) = x2 + bx
- b at the point (1, 1) and the coordinate axes, lies in the first
lines x =1, x =b, is equal to b 2 + 1 - 2 for all b > 1, then quadrant. If its area is 2, then the value of b is
f(x) is (a) -1 (b) 3
(c) -3 (d) 1
(a) x -1 (b) x +1 11. The area included between the parabolas y2 = 4a (x + a) and
x y2 = 4b(x – a), b > a > 0, is
(c) x2 +1 (d) 4 2 2 a
1+ x 2 (a) b sq. units
3 b-a
7. If the area enclosed by y 2 = 4ax and line y = ax is 1/3 sq.
8 8 2 b
(b) a sq. units
units , then the area enclosed by y = 4x with same parabola 3 b -a
is
4 2 2 b
(a) 8 sq. units (b) 4 sq. units (c) a sq. units
(c) 4/3 sq. units (d) 8/3 sq. units 3 b -a
8. The sine and cosine curves intersects infinitely many times 8 8 2 a
giving bounded regions of equal areas. The area of one of (d) b sq. units
3 b-a
such region is
(a) 2 (b) 2 2 12. The area of the figure bounded by the curves y = | x – 1| and
(c) 3 2 (d) 4 2 y = 3 – | x |.
(a) 3 sq. units (b) 2 sq. units
9. The figure shows as triangle AOB and the parabola y = x 2. (c) 4 sq. units (d) 5 sq. units
The ratio of the area of the triangle AOB to the area of the 13. Area bounded by the curve y = logex, x = 0, y £ 0 and x-axis
region AOB of the parabola y = x 2 is equal to is :
Y 1
(a) 1 sq. unit (b) sq. unit
2
A(– a, a2) B(a, a2) (c) 2 sq. unit (d) None of these
14. The area bounded by the curves y = xe x , y = xe - x and
the line x = 1 is
X 2 2
O(0, 0) (a) (b) 1 -
e e
1 1
(c) (d) 1 -
e e

RESPONSE 5. 6. 7. 8. 9.
GRID 10. 11. 12. 13. 14.

Space for Rough Work


Mathematics M-91

x2 y2 20. The area enclosed by the curve x = a cos 3 t , y = b sin 3 t


15. The area common to the ellipse + = 1 and
a2 b2 and the positive directions of x-axis and y-axis is
pab pab
x2 y2 (a) (b)
+ = 1 , 0 < b < a is 4 32
b2 a2
3pab 5pab
b a (c) (d)
(a) ( a + b )2 tan -1 (b) ( a + b )2 tan -1 32 32
a b 21. Area bounded by the curves y = ex, y = e–x and the straight
b a line x = 1 is (in sq. units)
(c) 4ab tan -1 (d) 4ab tan -1 1 1
a b (a) e + (b) e + + 2
e e
16. Area between the curve y = 2x 4 - x 2 , x axis and the
1 1
ordinates of two minima of curve is (c) e+ -2 (d) e - + 2
e e
7 9
(a) (b) 22. Find the area bounded by the curve x = 2 – y – y2 and y-axis.
120 120 9 9
11 13 (a) – (b)
(c) (d) 2 2
120 120 (c) 9 (d) – 9
1 23. The line y = mx bisects the area enclosed by lines x =0,
17. The area bounded by y –1 = |x|, y = 0 and |x| = will be :
2
y = 0 and x = 3/2 and the curve y = 1 + 4x – x2. Then the
3 3
(a) (b) value of m is
4 2
13 13
5 (a) (b)
(c) (d) None of these 6 2
4
18. What is the area of the parabola y2 = 4bx bounded by its 13 13
latus rectum ? (c) (d)
5 7
(a) 2b2/3 square unit (b) 4b2/3 square unit 24. Area between the curve y = cos2 x, x-axis and ordinates
(c) b2 square unit (d) 8b2/3 square unit x = 0 and x = p in the interval (0, p) is
19. The area bounded by the curve y = ln (x) and the lines
y = 0, y = ln (3) and x = 0 is equal to : 2p
(a) (b) 2 p
(a) 3 (b) 3 ln (3) – 2 3
(c) 3 ln (3) + 2 (d) 2 p
(c) p (d)
2

RESPONSE 15. 16. 17. 18. 19.


GRID 20. 21. 22. 23. 24.

Space for Rough Work


EBD_7504
M-92 NTA JEE Main

29. Statement-1:
25. The area bounded by the curve y 2 = 4a 2 ( x - 1) and the
p/2
lines x = 1 and y = 4a is equal to : (sin x )5 / 2 p
(a)
16 a
sq unit (b) 5a sq unit
ò (sin x) 5/ 2
+ (cos x ) 5/ 2
dx =
4
0
3
Statement-2: Area bounded by y = 3x and y = x2 is
17 a
(c) sq unit (d) None of these 9/2 sq. units
4 (a) Statement-1 is true, Statement-2 is true, Statement-2 is
26. The coordinates of the point P(x, y) of y = e -|x| so that a correct explanation for Statement -1
the area formed by the coordinate axes and the tangent at P (b) Statement -1 is true, Statement -2 is true ; Statement-2
is greatest, are is NOT a correct explanation for Statement - 1
(a) (1, 1/e) (b) (1, –1/e) (c) Statement -1 is false, Statement -2 is true
(c) (1, e) (d) (–1, e)
(d) Statement - 1 is true, Statement- 2 is false
p 30. Statement-1 : The area bounded by the curves
27. The area under the curve y = | cos x – sin x |, 0 £ x £ , and
2 y = x2 – 3 and y = kx + 2 is least, if k = 0.
above x-axis is : Statement-2 : The area bounded by the curves
(a) 2 2 (b) 2 2 -2
y = x2 – 3 and y = kx + 2 is k 2 + 20 .
(c) 2 2 + 2 (d) 0 (a) Statement-1 is true, Statement-2 is true, Statement-2 is
28. The area of the triangle formed by the tangent and normal at a correct explanation for Statement -1
( )
thepoint 1, 3 on the circle x2 + y2 = 4 and the x-axis is (b) Statement -1 is true, Statement -2 is true ; Statement-2
is NOT a correct explanation for Statement - 1
(a) 3 sq. units (b) 2 3 sq. units (c) Statement -1 is false, Statement -2 is true
(c) 3 2 sq. units (d) 4 sq. units (d) Statement - 1 is true, Statement- 2 is false

RESPONSE 25. 26. 27. 28. 29.


GRID 30.

MATHEMATICS CHAPTERWISE SPEED TEST-82


Total Questions 30 Total Marks 120
Attempted Correct
Incorrect Net Score
Cut-off Score 35 Qualifying Score 50
Success Gap = Net Score – Qualifying Score
Net Score = (Correct × 4) – (Incorrect × 1)
Space for Rough Work
MATHEMATICS Speed
Differential Equations TEST
No. of Questions
30
Maximum Marks
120
Time
1 Hour
83
Chapter-wise

GENERAL INSTRUCTIONS
• This test contains 30 MCQ's. For each question only one option is correct. Darken the correct circle/ bubble in the
Response Grid provided on each page.
• You have to evaluate your Response Grids yourself with the help of solutions provided at the end of this book.
• Each correct answer will get you 4 marks and 1 mark shall be deduced for each incorrect answer. No mark will be given/
deducted if no bubble is filled. Keep a timer in front of you and stop immediately at the end of 60 min.
• The sheet follows a particular syllabus. Do not attempt the sheet before you have completed your preparation for that
syllabus.
• After completing the sheet check your answers with the solution booklet and complete the Result Grid. Finally spend time
to analyse your performance and revise the areas which emerge out as weak in your evaluation.

d2y dy 1 é e8 x 7ù 1 é e8 x 7ù
1. If y = (x + 1 + x 2 )n, then (1 + x2) +x is (a) y= ê - x+ ú (b) y= ê + x+ ú
dx 2 dx 8 ë 8 8û 8 ë 8 8û
(a) n 2y (b) –n2y
1 é e8 x 7ù 1 é e8 x 7ù
(c) –y (d) 2x2y (c) y= ê + x- ú (d) y= ê - x- ú
8 ë 8 8û 8 ë 8 8û
2. The solution of the differential equation
dy y y æ a2 – x2 – y 2 ö
+ log y = 2 (log y )2 is æ x dx + y dy ö
=
dx x x 4. The solution of ç
è x dy – y dx ÷ø ç ÷ is
è x2 + y 2 ø
æ 1ö
(a) y = log ( x 2 + cx ) (b) log y = x ç cx 2 + ÷ (a) ( x 2 + y 2 ) = a sin {(tan–1 y/x) + constant}
è 2ø
(b) ( x 2 + y 2 ) = a cos {(tan–1 y/x) + constant}
æ 1ö
(c) x = log yç cx 2 + ÷ (d) None of these
è 2ø (c) ( x 2 + y 2 ) = a {tan (sin–1y/x) + constant}
d3y d2y (d) ( x 2 + y 2 ) = a {tan(cos–1 y/x) + constant}
3. The solution of the differential equation - 8 =0
dx 3 dx 2
1
safisfying y (0) = , y '(0) = 0 and y ''(0) = 1 is
8

RESPONSE GRID 1. 2. 3. 4.

Space for Rough Work


EBD_7504
M-94 NTA JEE Main

5. The gradient of the curve passing through (4, 0) is given by (a) f ( x ) = y( x + c) (b) f ( x) = cxy
dy y 5x (c) f ( x ) = c(x + y) (d) yf (x ) = cx
- + = 0 ifthe point (5, a) lies on the curve,
dx x ( x + 2)(x - 3) 10. The solution of the differential equation sec2 x tan y dx +
then the value of a is sec2 y tan x dy = 0 is:
67 7 tan y
(a) (b) 5 sin (a) tan y tan x = c (b) =c
12 12 tan x
7 tan 2 x
(c) 5 log (d) None of these (c) =c (d) None of these
12 tan y
6. The differential equations of all conics whose axes coincide 11. The solution of the differential equation
with the co-ordinate axis
dy y
2 log x + = sin 2x is
d2y æ dy ö dy dx x
(a) xy + xç ÷ + y =0
dx 2 è dx ø dx 1 1
(a) y log | x |= C - cos x (b) y log | x |= C + cos 2x
2
d y æ dy ö dy
2 2 2
(b) xy + xç ÷ + x =0
dx 2 è dx ø dx 1 1
(c) y log | x |= C - cos 2x (d) xylog | x |= C - cos 2x
2 2 2
d2y æ dy ö dy
(c) xy + xç ÷ - y =0 2 + sin x æ dy ö
dx 2 è dx ø dx 12. If y = y(x) and ç ÷ = - cos x, y (0) = 1,
1 + y è dx ø
2
d2y æ dy ö dy æ pö
(d) xy - xç ÷ + y =0 then y ç ÷ equals
dx 2 è dx ø dx è 2ø
7. The equation of the curve satisfying the equation (a) 1/3 (b) 2/3
dy (c) –1/3 (d) 1
( xy - x 2 ) = y 2 and passing through the point (–1, 1) is 13. The solution of the differential equation
dx
(a) y = (log y - 1) x (b) y = (log y + 1) x æ e -2 x y ö dx
ç - ÷ = 1 is given by
(c) x = (log x - 1) y (d) x = (log x + 1) y çè x x ÷ø dy
y æ xö
8. If for the differential equation y ' = + f ç ÷ , the general (a) ye 2 x
= 2 x +c (b) ye -2 x
= x +c
x è yø
(c) y = x (d) y = 3 x
x
solution is y = , then f (x/y) is given by 14. The line normal to a given curve at each point (x, y) on the
log | Cx |
curve passes through the point (3, 0). If the curve contains
(a) – x2 / y2 (b) y 2/x 2 the point (3, 4) then its equation is
(c) x2 / y2 (d) – y2/x2 (a) x 2 + y 2 + 6x - 7 = 0 (b) 2(x 2 + y2 ) - 12x - 7 = 0
dy yf ' (x ) - y 2
9. The solution to the differential equation = (c) x 2 + y 2 - 6x - 7 = 0 (d) None of these
dx f (x )
where f (x) is a given function is

RESPONSE 5. 6. 7. 8. 9.
GRID 10. 11. 12. 13. 14.

Space for Rough Work


Mathematics M-95

15. Differential equation of all conics of the form ax 2 + by = 1, d


20. If y + (xy) = x (sin x + log x), then
a and b being parameters is : dx
2 2 2 x x c
d2y æ dy ö dy (a) y = cos x + sin x + cos x + log x – + 2
(a) xy + xç ÷ - y =0 x x 2 3 9 x
2
dx è dx ø dx
2 2 x x c
2 (b) y = – cos x – sin x + 2 cos x + log x – +
d2y æ dy ö dy x x 3 9 x2
(b) xy - xç ÷ + y =0
2
dx è dx ø dx 2 2 x x c
(c) y = – cosx + sin x + 2 cos x – log x – + 2
2 x x 3 9 x
d2y æ dy ö dy
(c) xy - xç ÷ - y =0 (d) None of these
2 dx dx
dx è ø 21. The equation of the curve passing through the point
(d) None of these æ 1ö
ç a , - ÷ and satisfying the differential equation
dy y f( y / x) è aø
16. The solution of the differential equation = +
dx x f '( y / x)
dy æ dy ö
is y-x = aç y 2 + ÷ is
(a) x f (y/x) = k (b) f (y/x) = kx dx è dx ø
(c) y f (y/x) = k (d) f (y/x) = ky (a) ( x + a )(1 + ay) = -4a 2 y (b) ( x + a )(1 - ay) = 4a 2 y
(k is arbitrary constant)
(c) ( x + a )(1 - ay) = -4a 2 y (d) None of these
dy
17. The solution of cos y = e x + sin y + x 2 esin y is 22. The real value of n for which the substitution y = un will
dx
4 dy 4 6
x3 x3 transform the differential equation 2 x y + y = 4 x into
(a) e x - e - sin y + =C (b) e - x - e - sin y + =C dx
3 3 a homogeneous equation is
(a) 1/2 (b) 1
x3 x3 (c) 3/2 (d) 2
(c) e x + e - sin y +
= C (d) e x - esin y - =C
3 3 23. The solution of the differential equation
18. Which of the following equations represents the curve for dy y y
+ log y = 2 ( log y ) is
2
which the intercept cut-off by any tangent on y-axis is
proportional to the square of the ordinate of the point of dx x x
tangency? æ 1ö
(a) y = log ( x 2 + cx ) (b) log y = x ç cx 2 + ÷
1 1 è 2ø
(a) x + y = cxy (b) + =c
x y æ 1ö
(c)x = log y ç cx 2 + ÷ (d) None of these
A B A B è 2ø
(c) x + y = xy (d)
x y
+ =1
24. The family of curves satisfying the differential equation
19. The equation of the curve which is such that the portion of dy
- 2 y tan x + y 2 tan 4 x = 0 is
the axis of x-cut off between the origin and tangent at any dx
point is proportional to the ordinate of that point is
(a) x = y (a – b log y) (b) log x = by2 + a (a) y sec 2 x = 5 tan 2 x + c (b) y sin 5 x + 5 cos 2 x
(c) x2 = y (a – b log y) (d) None of these (c) 5 sec 2 x = y(tan5 x + c) (d) None of these

RESPONSE 15. 16. 17. 18. 19.


GRID 20. 21. 22. 23. 24.

Space for Rough Work


EBD_7504
M-96 NTA JEE Main

28. An integrating factor of the differential equation


If a curve passes through the point æç 2, ö÷ and has slope
7
25.
è 2ø dy
= y tan x – y2 sec x is equal to :
æ 1 ö dx
ç 1 - ÷ at any point (x, y) on it, then the ordinate of the
è x2 ø (a) tan x (b) sec x
(c) cosec x (d) cot x
point on the curve whose abscissa is – 2 is : 29. Statement-1 : The differential equation of the form
3 3 yf (xy) dx + xf (xy) dy = 0 can be converted to homogeneous
(a) – (b) forms by substitution xy = v.
2 2
Statement-2 : All differential equation of first order and first
5 5 degree become homogeneous, if we put y = vx.
(c) (d) –
2 2 (a) Statement-1 is true, Statement-2 is true, Statement-2 is
a correct explanation for Statement -1
x 2 + y2
26. The curve that satisfies the differential equation y' = (b) Statement -1 is True, Statement -2 is True ; Statement-
2 xy 2 is NOT a correct explanation for Statement - 1
and passes through (2, 1) is a hyperbola with eccentricity : (c) Statement -1 is False, Statement -2 is True
(a) 2 (b) 3 (d) Statement - 1 is True, Statement- 2 is False
dy
(c) 2 (d) 5 30. Statement-1 : The general solution of + y = 1 is yex = ex + c
dx
27. The family of curves for which the area of the triangle formed Statement-2 : The number of arbitrary constants in the
by the x-axis, the tangent drawn at any point on the curve general solution of the differential equation is equal to the
and radius vector of the point of tangency is constant equal order of differential equation.
to 2a2, is given by (a) Statement -1 is false, Statement-2 is true
a2 (b) Statement -1 is true, Statement-2 is true; Statement -2 is
x = cy ± a2
(a) (b) y = cx ± a correct explanation for Statement-1
y x (c) Statement -1 is true, Statement-2 is true; Statement -2 is
not a correct explanation for Statement-1
(c) x 2 ± ay 2 = cy (d) a 2 x 2 ± y 2 = cy (d) Statement -1 is true, Statement-2 is false

RESPONSE 25. 26. 27. 28. 29.


GRID 30.

MATHEMATICS CHAPTERWISE SPEED TEST-83


Total Questions 30 Total Marks 120
Attempted Correct
Incorrect Net Score
Cut-off Score 38 Qualifying Score 55
Success Gap = Net Score – Qualifying Score
Net Score = (Correct × 4) – (Incorrect × 1)
Space for Rough Work
MATHEMATICS Speed
Vector Algebra TEST
No. of Questions
30
Maximum Marks
120
Time
1 Hour
84
Chapter-wise

GENERAL INSTRUCTIONS
• This test contains 30 MCQ's. For each question only one option is correct. Darken the correct circle/ bubble in the
Response Grid provided on each page.
• You have to evaluate your Response Grids yourself with the help of solutions provided at the end of this book.
• Each correct answer will get you 4 marks and 1 mark shall be deduced for each incorrect answer. No mark will be given/
deducted if no bubble is filled. Keep a timer in front of you and stop immediately at the end of 60 min.
• The sheet follows a particular syllabus. Do not attempt the sheet before you have completed your preparation for that
syllabus.
• After completing the sheet check your answers with the solution booklet and complete the Result Grid. Finally spend time
to analyse your performance and revise the areas which emerge out as weak in your evaluation.

r r r r r r
1. If | a | = 7, | b | = 11 and | a + b |= 10 3, then | a - b | is equal 4. If b and c are any two non-collinear mutually perpendicular
to
(a) 40 (b) 10 unit vectors and a is any vector, then
(c) 4 10 (d) 2 10
r r r a . ( b ´ c)
2. If a, b & c are three non-coplanar non-zero vectors, then ( a × b) b + ( a . c) c + ( b ´ c) is equal to :
| b ´ c |2
rr r r rr r r rr r r
(a.a ) (b ´ c ) + (a.b ) (c ´ a ) + (a.c ) (a ´ b ) is equal to
rrr r rrr r (a) a (b) 2a
(a) [b c a] a (b) [c a b ] b
rrr r (c) 3a (d) None
(c) [a b c ] c (d) None of these
r r r 5. Let a, b, c are three non-coplanar vectors such that
3. If a = iˆ + ˆj + kˆ , b = 4iˆ + 3 ˆj + 4kˆ , and c = iˆ + aˆj +b kˆ are r1 = a – b + c, r2 = b + c – a, r3 = c + a + b,
r
linearly dependent vectors and | c | = 3 , then r = 2a – 3b + 4c. If r = l1r1 + l2r2 + l3r3, then
(a) l1 = 7 (b) l1 + l3 = 6
(a) a = 1, b = –1 (b) a = 1, b = ±1
(c) l1 + l2 + l3 = 4 (d) l3 + l2 = 2
(c) a = –1, b = ±1 (d) a = ±1, b = 1

RESPONSE GRID 1. 2. 3. 4. 5.

Space for Rough Work


EBD_7504
M-98 NTA JEE Main

r
6. The values of a for which the vector a = î + 3ˆj + (sin 2a)k̂ (a)
1 ˆ
(6i + 13 ˆj + 18kˆ) (b)
2 ˆ
(6i + 12 ˆj - 8kˆ)
makes an obtuse angle with the z-axis and the vectors 3 3
r a (c) 1 (- 6iˆ - 8 ˆj - 9kˆ)
2
b = (tan a)î - ˆj + 2 sin k̂ (d) (- 6iˆ - 12 ˆj + 8kˆ)
3 3
2 r r r r r r r r
11. Let, a = a1iˆ + a2 ˆj + a3kˆ , b = b1iˆ + b2 ˆj + b3kˆ &
r a r r r r
and c = (tan a)î + (tan a )ˆj - 3 cos ec k̂ r
2 c = c1iˆ + c2 ˆj + c3kˆ be three non-zero vectors such that c
are mutually orthogonal are r r
is a unit vector perpendicular to both a & b . If the angle
(a) np - tan -1 2, n Î I 2
a1 a2 a3
-1
(b) (4n + 2)p - tan 2, n Î I r r p
-1 between a & b is , then b1 b2 b3 is equal to
(c) 2np - tan 2, n Î I 6 c1 c2 c3
(d) (2n + 1)p - tan -1 2, n Î I (a) 0 (b) 1
r r r r r r 1 r 2 r 2 3 r 2 r 2
7. If ((a ´ b ) ´ (c ´ d )).( a ´ d ) = 0 , then which of the following (c) |a | |b | (d) |a | |b |
is always true ? 4 4
r r
(a) ar, b , cr, d are necessarily coplanar 12. Let a , b and c be three non-coplanar vectors, and let
r r r r
(b) either a or d must lie in the plane of b and c p , q and r be the vectors defined by the relations
r r r r
(c) either b or c must lie in the plane of a and d b´c c´a a´b
r r r r p = , q = and r = .
(d) either a or b must lie in the plane of c and d [a b c] [a b c] [a b c]
8. If A, B, C, D are four points in space satisfying Then the value of the expression
AB . CD = k[| AD |2 + | BC |2 - | AC |2 - | BD |2 ] , ( a + b ) . p + ( b + c ). q + ( c + a ) . r is equal to
then the value of k is (a) 0 (b) 1
1 (c) 2 (d) 3
(a) 2 (b) r
3 13. The vector a = a iˆ + 2 ˆj + bkˆ lies in the plane of the vectors
r r r
1 b = iˆ + ˆj and c = ˆj + kˆ and bisects the angle between b
and cr . Then which one of the following gives possible
(c) (d) 1
2
uur uur uur uur values of a and b?
9. a , b and c are three vectors with magnitude | a | = 4, (a) a = 2, b = 2 (b) a = 1, b = 2
uur uur uur (c) a = 2, b = 1 (d) a = 1, b = 1
| b | = 4, | c | = 2 and such that a is perpendicular to
uur uur uur uur uur uur 14. Let p, q, r be three mutually perpendicular vectors of the
(b + c ), b is perpendicular to (c + a ) an d c is
uur uur uur uur uur same magnitude. If a vector x satisfies the equation
perpendicular to (a + b ) . It follows that | a + b + c | is
equal to : p ´ {( x - q ) ´ p} + q ´ {( x - r )) ´ q}
(a) 9 (b) 6
+ r ´ {( x - p) ´ r} = 0 then x is given by
(c) 5 (d) 4
1 1
10. If 4iˆ + 7 ˆj + 8kˆ , 2iˆ + 3 ˆj + 4kˆ and 2iˆ + 5 ˆj + 7kˆ are the (a) ( p + q - 2 r) (b) ( p + q + r)
position vectors of the vertices A, B and C respectively of 2 2
triangle ABC. The position vector of the point where the 1 1
(c) ( p + q + r) (d) (2 p + q - r )
bisector of angle A meets BC is : 3 3

RESPONSE 6. 7. 8. 9. 10.
GRID 11. 12. 13. 14.
Space for Rough Work
Mathematics M-99

r r r 20. The points D, E, F divide BC, CA and AB of the triangle ABC


15. Let a, b & c be non-coplanar unit vectors equally inclined
in the ratio 1 : 4, 3 : 2 and 3 : 7 respectively and the point K
rr r uuur uuur uur uuur
to one another at an acute angle q. Then | [a b c] | in terms divides AB in the ratio 1 : 3, then (AD + BE + CF) : CK is
of q is equal to equal to
(a) (1 + cos q) cos 2q (b) (1 + cos q) 1 - 2cos 2q (a) 1 : 1 (b) 2 : 5
(c) 5 : 2 (d) None of these
(c) (1 - cos q) 1 + 2cos q (d) None of these
21. Let A = 2 î + k̂ , B = î + ĵ + k̂ and C = 4 î - 3 ĵ + 7 k̂ . The
16. If the pth, qth and rth terms of a G.P. are positive numbers a,
b and c respectively, then find the angle between the vector R which satisfies the equations
vectors log a i + log b j + log c 2 kˆ and
2ˆ 2ˆ
R ´ B = C ´ B and R . A = 0 is given by
( q - r ) ˆi + ( r - p ) ˆj + ( p - q ) kˆ - î - 8ˆj + 2k̂
(a) - 2î + k̂ (b)
p p
(a) (b) 1
6 4 (c) (î - ĵ + 2k̂ ) (d) None of these
6
p p
(c) (d) 22. What is the interior acute angle of the parallelogram whose
3 2
1 ˆ 1 ˆ ˆ
17. Force i + 2 j – 3k , 2i + 3 j + 4k and –iˆ – ˆj + kˆ are acting at
ˆ ˆ ˆ ˆ ˆ ˆ sides are represented by the vectors i+ j + k and
the point P (0, 1, 2). The moment of these forces about the 2 2
point A (1, – 2, 0) is 1 ˆ 1 ˆ ˆ
(a) 2iˆ – 6 ˆj + 10kˆ (b) –2iˆ + 6 ˆj –10kˆ i- j+ k ?
2 2
(c) 2iˆ + 6 ˆj –10kˆ (d) None of these (a) 60° (b) 45°
r r
18. If a and b are non-collinear vectors, then the value of a for (c) 30° (d) 15°
r r r r r r
which the vectors u = (a - 2)a + b and v = (2 + 3a)a - 3b 23. If three vectors a , b , c are such that a ¹ 0 and
are collinear is :
a ´ b = 2 a ´ c , | a |=| c |= 1, | b |= 4 and the angle
3 2
(a) (b) -1 1
2 3 between b and c is cos then b - 2 c = l a where
4
3 2
(c) – (d) – l is equal to
2 3 (a) ± 2 (b) ± 4
r r r r r
19. If a = iˆ + ˆj, bˆ = 2jˆ – kˆ and r ´ a = b ´ a , 1 1
(c) (d)
r 2 4
r r r r r r
r ´ b = a ´ b , then what is the value of r ?
|r| 24. If ar and b are two non-zero non-collinear vectors then
r r r r r r r r r
2 [a , b , iˆ] iˆ + 2 [a , b , ˆj ] ˆj - 2 [a , b , kˆ] kˆ +[a, b , a ] is equal
(iˆ + 3jˆ – k)
ˆ (iˆ – 3jˆ + k)
ˆ
(a) (b) to
11 11 r r r
(a) 2(a ´ b ) (b) ar ´ b
(iˆ + 3jˆ + k)
ˆ (iˆ – 3jˆ – k)
ˆ r
(c) (d) (c) ar + b (d) None of these
11 11

RESPONSE 15. 16. 17. 18. 19.


GRID 20. 21. 22. 23. 24.
Space for Rough Work
EBD_7504
M-100 NTA JEE Main
uuur uuur r r r
25. In a triangle ABC, three forces of magnitudes 3AB , 2AC 29. Statement 1 : If a = 2i + k , b = 3 j + 4 k and c = 8i - 3 j
uuur r
and 6CB are acting along the sides AB, AC and CB are coplanar then cr = 4ar - b
respectively. If the resultant meets AC at D, then the ratio r r r r
Statement 2 : A set of vectors a1 , a2 , a3 ... an is said to be
DC : AD will be equal to :
linearly independent if every relation of the form
(a) 1 : 1 (b) 1 : 2 r r r
l1a1 + l 2 a2 + ..... + l n an = 0 implies that
(c) 1 : 3 (d) 1 : 4
l 1 = l 2 = ....l n = 0 (scalars).
26. For any vector p , the value of
(a) Statement-1 is true, Statement-2 is true, Statement-2 is
3
2
{ }
| p ´ î | 2 + | p ´ ˆj |2 + | p ´ k̂ | 2 is
a correct explanation for Statement -1
(b) Statement -1 is True, Statement -2 is True ; Statement-
2 is NOT a correct explanation for Statement - 1
2 2
(a) p (b) 2 p (c) Statement -1 is False, Statement -2 is True
(d) Statement - 1 is True, Statement- 2 is False
2 2 r r r
(c) 3 p (d) 4 p 30. Statement 1 : Let A(a), B(b) and C(c) be three points
r r
such that a = 2iˆ + ˆj + kˆ, b = 3iˆ - ˆj + 3kˆ and
27. If | a + b |=| a - b | then the vectors a and b are
r
adjacent sides of c = -iˆ + 7 ˆj - 5kˆ then OABC is a tetrahedron.
r r r
(a) a rectangle (b) a square Statement 2 : Let A(a), B(b) and C(c) be three points
(c) a rhombus (d) None of these r r
such that a , b and cr are non-coplanar then OABC is a
28 . If vectors ai + j + k , i + bj + k and iˆ + ˆj + ckˆ (a ¹ b ¹ c ¹ 1)
ˆ ˆ ˆ ˆ ˆ ˆ
tetrahedron, where O is the origin.
1 1 1 (a) Statement-1 is true, Statement-2 is true, Statement-2 is
are coplanar, then find + + .
1- a 1 - b 1- c a correct explanation for Statement -1
(b) Statement -1 is True, Statement -2 is True ; Statement-
(a) 0 (b) 1 2 is NOT a correct explanation for Statement - 1
(c) Statement -1 is False, Statement -2 is True
(c) –1 (d) 2 (d) Statement - 1 is True, Statement- 2 is False

RESPONSE 25. 26. 27. 28. 29.


GRID 30.

MATHEMATICS CHAPTERWISE SPEED TEST-84


Total Questions 30 Total Marks 120
Attempted Correct
Incorrect Net Score
Cut-off Score 35 Qualifying Score 48
Success Gap = Net Score – Qualifying Score
Net Score = (Correct × 4) – (Incorrect × 1)
Space for Rough Work
MATHEMATICS Speed
Three Dimensional Geometry TEST
No. of Questions
30
Maximum Marks
120
Time
1 Hour
85
Chapter-wise

GENERAL INSTRUCTIONS
• This test contains 30 MCQ's. For each question only one option is correct. Darken the correct circle/ bubble in the
Response Grid provided on each page.
• You have to evaluate your Response Grids yourself with the help of solutions provided at the end of this book.
• Each correct answer will get you 4 marks and 1 mark shall be deduced for each incorrect answer. No mark will be given/
deducted if no bubble is filled. Keep a timer in front of you and stop immediately at the end of 60 min.
• The sheet follows a particular syllabus. Do not attempt the sheet before you have completed your preparation for that
syllabus.
• After completing the sheet check your answers with the solution booklet and complete the Result Grid. Finally spend time
to analyse your performance and revise the areas which emerge out as weak in your evaluation.

x - 2 y + 1 z -1 3. The length intercepted by a line with direction ratios


1. The line, = = intersects the curve xy = c2, 2, 7, –5 between the lines
3 2 -1
z = 0 if c is equal to
1 x -5 y -7 z + 2 x +3 y-3 z -6
(a) ± 1 (b) ± = = and = = is
3 3 -1 1 -3 2 4
(c) ± 5 (d) None of these (a) 75 (b) 78
2. Two systems of rectangular axes have the same origin If a (c) (d) None of these
plane cuts them at the distance a, b, c and a¢, b¢, c¢ 83
respectively from the origin, then 4. From the point (1, –2, 3) lines are drawn to meet the sphere
x2 + y2 + z2 = 4 and they are divided internally in the ratio
1 1 1 æ 1 1 1 ö 2 : 3. The locus of the point of division is
+ + = k çç + + ÷÷ , where k =
2 2 2 2 2
a b c è a¢ b¢ c¢ 2 ø (a) 5x 2 + 5y 2 + 5z 2 - 6x + 12 y + 22 = 0
(a) 1 (b) 2
(b) 5(x 2 + y 2 + z 2 ) = 22
1
(c) 4 (d) (c) 5x 2 + 5y 2 + 5z 2 - 2xy - 3yz - zx - 6x + 12y + 5z + 22 = 0
2
(d) 5x 2 + 5y 2 + 5z 2 - 6x + 12y - 18z + 22 = 0

RESPONSE GRID 1. 2. 3. 4.
Space for Rough Work
EBD_7504
M-102 NTA JEE Main

5. If two lines L1 and L2 in space, are defined by


l - 5m + 3n = 0, 7l 2 + 5m2 - 3n2 = 0 are
{
L1 = x = l y + ( )
l -1 , z = ( )
l - 1 y + l and} 1 2 3 -1 2 3
(a) , , (b) , ,
L2 = { x = ( ) ( ) m}
14 14 14 14 14 14
m y + 1- m , z = 1 - m y +
1 -2 3 1 2 -3
then L1 is perpendicular to L2, for all non-negative reals l (c) , , (d) , ,
and m, such that : 14 14 14 14 14 14
l¹m 10. The equation of a sphere is x2 + y2 + z2 –10z = 0. If one end
(a) l + m =1 (b)
point of a diameter of the sphere is (– 3, – 4, 5), what is the
(c) l + m = 0 (d) l = m other end point ?
6. The locus of a point, such that the sum of the squares of its (a) (– 3, – 4, – 5) (b) ( 3, 4, 5)
distances from the planes x + y + z = 0, x – z =0 and x – 2y + (c) (3, 4, – 5) (d) (– 3, 4, – 5)
z = 0 is 9, is 11. A line makes the same angle a with each of the x and y axes.
If the angle q, which it makes with the z-axis, is such that
(a) x 2 + y2 + z2 = 3 (b) x 2 + y 2 + z 2 = 6
sin2q = 2 sin2a , then what is the value of a ?
(c) x 2 + y 2 + z 2 = 9 (d) x 2 + y 2 + z 2 = 12 (a) p/4 (b) p/6
(c) p/3 (d) p/2
7. If l1, m1, n1 and l2, m2, n2 be the direction cosines of two
12. If Q is the image of the point P(2, 3, 4) under the reflection in the
mutually perpendicular lines, Then the direction cosines of
plane x – 2y + 5z = 6, then the equation of the line PQ is
the line perpendicular to both of them are
(a) (m1n2 – m2n1), (n1l2 – n2l1), (l1m2 – l2m1) x-2 y-3 z-4 x - 2 y -3 z - 4
(a) = = (b) = =
(b) l1 + l2, m1 + m2, n1 + n2, -1 2 5 1 -2 5
(c) l1 l2, m1 m2, n1 n2
x-2 y-3 z-4 x - 2 y -3 z - 4
l1 m1 n1 (c) = = (d) = =
, , -1 -2 5 1 2 5
(d) l2 m 2 n 2 13. The foot of the perpendicular from (2, 4, –1) to the line
8. A variable plane passes through a fixed point (1, 2, 3). The 1 1
locus of the foot of the perpendicular from the origin to this x +5 = ( y + 3) = – ( z – 6)
4 9
plane is given by
(a) x2 + y2 + z2 – 14 = 0 (a) (– 4, 1, – 3) (b) (4, –1, –3)
(b) x2 + y2 + z2 + x + 2y + 3z = 0 (c) (–4, –1, 3) (d) (– 4, – 1, –3)
(c) x2 + y2 + z2 – x – 2y – 3z = 0 14. The equation of the plane which makes with co-ordinate
(d) None of these axes, a triangle with its centroid (a, b, g) is
9. The direction cosines l, m, n, of one of the two lines (a) ax + b y + gz = 3 (b) ax + b y + gz = 1
connected by the relations x y z x y z
(c) + + =3 (d) + + =1
a b g a b g

RESPONSE 5. 6. 7. 8. 9.
GRID 10. 11. 12. 13. 14.

Space for Rough Work


Mathematics M-103

15. The equation of two lines through the origin, which intersect 21. The distance of the point (1, –2, 3) from the plane
x -3 y-3 z p x y z -1
the line = = at angles of each, are x - y + z = 5 measured parallel to the line = = is
2 1 1 3 2 3 -6
(a) 1 (b) 2
x y z x y z
(a) = = ; = = (c) 4 (d) 2 3
1 2 1 1 1 2
22. A variable plane which remains at a constant distance 3p
x y z x y z from the origin cut the coordinate axes at A, B and C. The
(b) = = ; = =
1 2 -1 -1 1 -2 locus of the centroid of triangle ABC is
(a) x–1 + y–1 + z–1 = p–1 (b) x–2 + y–2 + z–2 = p–2
x y z x y z
(c) = = ; = = (c) x + y + z = p (d) x2 + y2 + z2 = p2
1 2 -1 1 -1 -2 23. The radius of the sphere
(d) None of the above
16. A rectangular parallelopiped is formed by drawing planes x 2 + y 2 + z 2 = 49 , 2x + 3y - z - 5 14 = 0 is
through the points (–1, 2, 5) and (1, –1, –1) and parallel to the 2 6
(a) 6 (b)
coordinate planes. The length of the diagonal of the
parallelopiped is (c) 4 6 (d) 6 6
(a) 2 (b) 3 24. Two spheres of radii 3 and 4 cut orthogonally The radius
(c) 6 (d) 7 of common circle is
17. The planes 3x – y + z + 1= 0, 5x + y + 3z = 0 intersect in 12
the line PQ. The equation of the plane through the point (a) 12 (b)
5
(2, 1, 4) and the perpendicular to PQ is
(a) x + y - 2z = 5 (b) x + y + 2z = –5 12
(c) (d) 12
5
(c) x + y + 2z = 5 (d) x + y – 2z = –5
x – 2 y –1 z + 2
x -1 y - 2 z -1 25. Let the line = = lie in the plane
18. The line = = and the plane x + 2 y + z = 6 3 –5 2
1 -2 3 x + 3y – az + b = 0. Then (a, b) equals
meet in (a) (–6, 7) (b) (5, –15)
(a) no point (b) only one point (c) (–5, 5) (d) (6, –17)
(c) infinitely many points (d) None of these 26. Equation of line in the plane p º 2x – y + z – 4 = 0 which is
19. If from a point P (a, b, c) perpendiculars PA and PB are perpendicular to the line l whose equation is
drawn to yz and zx planes, then the equation of the plane x-2 y-2 z-3
OAB is = = and which passes through the point
1 -1 -2
(a) bcx + cay +abz = 0 (b) bcx + cay – abz = 0 of intersection of l and p is –
(c) bcx – cay + abz = 0 (d) –bcx + cay + abz = 0
20. Under what condition do the planes x - 2 y -1 z - 1 x -1 y - 3 z - 5
(a) = = (b) = =
bx – ay = n, cy – bz = l , az – cx = m intersect in a line? 3 5 -1 3 5 -1
(a) a + b + c = 0 (b) a = b = c x + 2 y +1 z +1 x - 2 y -1 z - 1
(c) al + bm + cn = 0 (d) l + m + n = 0 (c) = = (d) = =
2 -1 1 2 -1 1

RESPONSE 15. 16. 17. 18. 19.


GRID 20. 21. 22. 23. 24.
25. 26.

Space for Rough Work


EBD_7504
M-104 NTA JEE Main

27. If the plane 2ax – 3ay + 4az + 6 = 0 passes through the 1 2 2 1 2 2


midpoint of the line joining the centres of the spheres (a) , , (b) - , ,
3 3 3 3 3 3
x 2 + y 2 + z 2 + 6 x - 8 y - 2 z = 13 and
1 2 2 1 2 2
(c) - , ,- (d) - , – ,
x 2 + y 2 + z 2 - 10 x + 4 y - 2 z = 8 then a equals 3 3 3 3 3 3
(a) – 1 (b) 1 30. Statement 1 : Let q be the angle between the line
(c) – 2 (d) 2
28. The equation of a plane passing through the line of x - 2 y -1 z + 2
= = and the plane x + y – z = 5.
intersection of the planes x + 2y + 3z = 2 and x – y + z = 3 and 2 -3 -2
2 -1 1
at a distance from the point (3, 1 ,–1) is Then q = sin
3 51
(a) 5x – 11y + z = 17 (b) 2x + y = 3 2 - 1 Statement 2 : Angle between a straight line and a plane is
the complement of angle between the line and normal to the
(c) x + y + z = 3 (d) x - 2 y = 1 - 2 plane.
29. A mirror and a source of light are situated at the origin O and (a) Statement-1 is true, Statement-2 is true, Statement-2 is
at a point on OX respectively. A ray of light from the source a correct explanation for Statement -1
strikes the mirror and is reflected. If the direction ratios of (b) Statement -1 is True, Statement -2 is True ; Statement-
the normal to the plane are 1, –1, 1, then direction cosines of 2 is NOT a correct explanation for Statement - 1
the reflected rays are (c) Statement -1 is False, Statement -2 is True
(d) Statement - 1 is True, Statement- 2 is False

RESPONSE 27. 28. 29. 30.


GRID

MATHEMATICS CHAPTERWISE SPEED TEST-85


Total Questions 30 Total Marks 120
Attempted Correct
Incorrect Net Score
Cut-off Score 35 Qualifying Score 52
Success Gap = Net Score – Qualifying Score
Net Score = (Correct × 4) – (Incorrect × 1)

Space for Rough Work


MATHEMATICS Speed
Probability TEST
No. of Questions
30
Maximum Marks
120
Time
1 Hour
86
Chapter-wise

GENERAL INSTRUCTIONS
• This test contains 30 MCQ's. For each question only one option is correct. Darken the correct circle/ bubble in the
Response Grid provided on each page.
• You have to evaluate your Response Grids yourself with the help of solutions provided at the end of this book.
• Each correct answer will get you 4 marks and 1 mark shall be deduced for each incorrect answer. No mark will be given/
deducted if no bubble is filled. Keep a timer in front of you and stop immediately at the end of 60 min.
• The sheet follows a particular syllabus. Do not attempt the sheet before you have completed your preparation for that
syllabus.
• After completing the sheet check your answers with the solution booklet and complete the Result Grid. Finally spend time
to analyse your performance and revise the areas which emerge out as weak in your evaluation.

1. If two events A and B are such that P(A) = 0.3, P(B) = 0.4 drawn, then the probability that it has come from box B2, is
35 14
and P(A Ç B) = 0.5 then P æç
B ö (a) (b)
= 78 39
è A È B ÷ø
10 12
(a) 0.9 (b) 0.5 (c) (d)
(c) 0.6 (d) 0.25 13 13
2. A bag contains three white, two black and four red balls. If 4. 6 coins are tossed together 64 times. If throwing a head is
four balls are drawn at random with replacement, the considered as a success then the expected frequency of at
probability that the sample contains just one white ball is least 3 successes is
16 8 (a) 64 (b) 21
(a) (b) (c) 32 (d) 42
81 81
2 3 1
(c)
32
(d)
4 5. If P ( A ) = , P ( B) = and P ( A Ç B ) = , then
81 81 5 10 5
3. For k = 1, 2, 3 the box Bk contains k red balls and (k + 1) P ( A¢ | B¢ ) . P ( B¢ | A¢ ) is equal to
1 1 1 5 5
white balls. Let P(B1 ) = , P(B2 ) = and P(B3 ) = . A box
2 3 6 (a) (b)
6 7
is selected at random and a ball is drawn from it. If a red ball is 25
(c) (d) 1
42

RESPONSE GRID 1. 2. 3. 4. 5.
Space for Rough Work
EBD_7504
M-106 NTA JEE Main

6. A binomial variate X has mean = 6 and variance = 2 the (a) P(X < 1) = P(X > 1)
probability that 5 £ X £ 7 is
1
4 1622 (b) P( < X < 1) < P(X > 1)
(a) (b) 2
2 6661
3
4672 (c) P(X > ) < P(X < 1)
(c) (d) none 2
6561 (d) All above are correct
7. c
Let E denote the complement of an event E. Let E, F, G be 12. In a test, an examinee either guesses or copies or knows the
pairwise independent events with P(G) > 0 and answer to a multiple choice question with four choices. The
P(EÇFÇG) = 0. Then P(Ec Ç Fc | G) equals 1
(a) P(Ec) + P(Fc) (b) P(Ec) – P(Fc) probability that he makes a guess is . The probability that
c
3
(c) P(E ) – P(F) (d) P(E) – P(Fc) 1
8. Suppose X is a random variable which takes values he copies is and the probability that his answer is correct
6
0, 1, 2, 3, ... and P(X = r) = pqr, where 0 < p < 1, q = 1 – p and 1
r = 0, 1, 2, ... then : given that he copied it is . The probability that he knew
8
(a) P( X ³ a ) = q a the answer to the question given that he correctly answered
(b) P( X ³ a + b | X ³ a ) = P( X ³ b ) it, is
24 1
(c) P(X = a + b | X ³ a ) = P(X = b) (a) (b)
(d) All of the above 29 4
9. n letters to each of which corresponds on addressed 3 1
envelope are placed in the envelope at random. Then the (c) (d)
4 2
probability that n letter is placed in the right envelope, will 13. Two events E and F are independent. If P(E) = 0.3,
be :
P ( E È F ) = 0.5, then P(E | F) – P(F | E) equals
1 1 1 1 1
(a) – + – + ....(–1) n 2 3
1! 2! 3! 4! n! (a) (b)
7 35
1 1 1 1 1
(b) + + – + .... 1 1
2! 3! 4! 5! n! (c) (d)
70 7
1 1 1 1 n 1
(c) – + – + ....( -1) 2
2! 3! 4! 5! n! 14. The probability of a man hitting a target is . He fires at the
(d) None of these 5
10. A bag contains n balls. It is given that the probability that target k times (k, a given number). Then the minimum k, so
among these n balls exactly r balls are white is propor- that the probability of hitting the target at least once is more
tional to r2 (0 £ r £ n). A ball is drawn at random and is 7
found to be white. Then the probability that all the balls in than , is :
10
the bag are white, will be: (a) 3 (b) 5
2n 4n (c) 2 (d) 4
(a) (n + 1) 2 (b) (n + 1)2 15. If A and B are two events such that P(A) ¹ 0 and P(B) ¹ 1 ,
2n 4n æ Aö
(c) (n + 3)2
(d) (n + 3)2 then P ç ÷ =
è Bø
11. A random variable X assumes values which are rational
n n +1 æ Aö æ Aö
numbers of the form and , where n = 1, 2, 3, ..... (a) 1– P çè ÷ø (b) 1– P ç B ÷
n +1 n B è ø
n +1
æ n ö æ n +1 ö æ 1 ö 1 – P(A È B) P(A)
If Pç X = ÷ = Pç X = ÷=ç ÷ , then : (c) (d)
è n + 1 ø è n ø è2ø P(B) P(B)

RESPONSE 6. 7. 8. 9. 10.
GRID 11. 12. 13. 14. 15.
Space for Rough Work
Mathematics M-107

16. Suppose X follows a binomial distribution with parameters


n and p, where 0 < p < 1, if P(X = r)/P(X = n – r) is independent 21. If X follows a binomial distribution with parameters n = 8
of n and r, then
1
1 1 and p = , then P(| X - 4 |£ 2) is
(a) p = (b) p = 2
2 3
1 119 119
(c) p = (d) none of these (a) (b)
128 228
4
17. A student appears for tests I, II and III. The student is 19 18
successful if he passes either in tests I and II or tests I and (c) (d)
IV. The probabilities of the student passing in tests I, II, III 128 128
22. A bag contains 4 red and 4 blue balls. Four balls are drawn
1
are p, q and respectively. The probability that the student one by one from the bag, then find the probability that the
2 drawn balls are in alternate colour.
1 35 2
is successful is then the relation between p and q is
2 (a) (b)
6 35
given by
(a) pq + p = 1 (b) p2 + q = 1 3 6
(c) pq – 1 = p (d) none of these. (c) (d)
35 35
18. A man takes a step forward with probability 0.4 and backward
with probability 0.6. The probability that at the end of eleven 23. If X is a Poisson variate such that P(X = 1) = P (X = 2), then
steps he is one step away from the starting point is P (X = 4) is equal to
5 1 1
2 5.35 æ 6 ö (a) 2 (b)
(a) (b) 462 ´ ç ÷ 2e 3e 2
510 è 25 ø
2 1
35 (c) 2 (d)
(c) 231´ (d) none of these 3e e2
510 24. 3% of the electric bulbs manufactured by a company are
19. If E 1 and E 2 are two events such that P(E1 ) = 1/4, defective. Using Poisson distribution (approximation), the
P(E2/E1) = 1/2 and P(E1/ E2) = 1/4, then choose the incorrect probability that a sample of 100 bulbs will contain exactly
statement. one defective, is
(a) E1 and E2 are independent (a) .05 (b) .15
(b) E1 and E2 are exhaustive (c) e-1 (d) e–2
(c) E2 is twice as likely to occur as E 1 (Given antilog (.1742) = 1.5)
(d) Probabilities of the events E 1 Ç E2 , E1 and E2 are in
G.P. 25. A random variable X has the probability distribution
æ 1ö X 1 2 3 4 5 6 7 8
20. If X and Y are independent binomial variates B ç 5, ÷ and
è 2ø
p(X) 0.15 0.23 0.12 0.10 0.20 0.08 0.07 0.05
æ 1ö
B ç 7, ÷ , then P (X + Y = 3) is
è 2ø For the events E = {X is a prime number} and F = {X < 4},
35 55 then P(E È F) is
(a) (b)
47 1024
(a) 0.50 (b) 0.77
220 11
(c) (d) (c) 0.35 (d) 0.87
512 204

RESPONSE 16. 17. 18. 19. 20.


GRID 21. 22. 23. 24. 25.
Space for Rough Work
EBD_7504
M-108 NTA JEE Main

26. The probability of India winning a test match against


1 2
1 (a) 2 (b)
Westindies is assuming independence from match to e e2
2
match the probability that in a 5 match series India's second 3
(c) (d) none of these
win occurs at the third test, is e2
(a) 2/3 (b) 1/2 29. A and B are two independent witnesses (i.e. there is no
(c) 1/4 (d) 1/8 collision between them) in a case. The probability that A will
27. The probability that an item produced by a factory is speak the truth is x and the probability that B will speak the
defective is p. From a certain lot, a sample of n items is truth is y. A and B agree in a certain statement. The probability
drawn with replacement. If it contains no defective items, that the statement is true is
the lot is accepted, while if it has more than two defective
items, the lot is rejected. If the sample has one or two x–y xy
(a) (b)
defective items, an independent sample of m items is drawn x+y 1 + x + y + xy
with replacement from the lot and combined with the
previous sample. If the combined sample does not contain x–y xy
(c) (d)
more than two defective items, the batch is accepted. The 1 – x – y + 2 xy 1 – x – y + 2 xy
probability that the batch is accepted is (q = 1 – p) 30. Statement-1 : A random variable X follows binomial
n (n - 1) 2 m+ n - 2
(a) qn + npqn–1 (qm + mpqm–1) + p q a2
2 distribution with mean and variance then is a positive
a -b
(b) npqn–1 (qm + mpqm–1)
integer
n n ( n - 1) 2 m + n - 2 Statement-2 : In a binomial distribution Mean > Variance.
(c) q + p q
2 (a) Statement -1 is false, Statement-2 is true
(d) None of these (b) Statement -1 is true, Statement-2 is true; Statement -2 is
28. Suppose that the probability that an item produced by a a correct explanation for Statement-1
particular machine is defective equals 0.2. If 10 items (c) Statement -1 is true, Statement-2 is true; Statement -2
produced from this machine are selected at random, the is not a correct explanation for Statement-1
probability that not more than one defective is found is (d) Statement -1 is true, Statement-2 is false

RESPONSE GRID 26. 27. 28. 29. 30.

MATHEMATICS CHAPTERWISE SPEED TEST-86


Total Questions 30 Total Marks 120
Attempted Correct
Incorrect Net Score
Cut-off Score 37 Qualifying Score 54
Success Gap = Net Score – Qualifying Score
Net Score = (Correct × 4) – (Incorrect × 1)
Space for Rough Work
MATHEMATICS Speed
Properties of Triangles TEST
No. of Questions
30
Maximum Marks
120
Time
1 Hour
87
Chapter-wise

GENERAL INSTRUCTIONS
• This test contains 30 MCQ's. For each question only one option is correct. Darken the correct circle/ bubble in the
Response Grid provided on each page.
• You have to evaluate your Response Grids yourself with the help of solutions provided at the end of this book.
• Each correct answer will get you 4 marks and 1 mark shall be deduced for each incorrect answer. No mark will be given/
deducted if no bubble is filled. Keep a timer in front of you and stop immediately at the end of 60 min.
• The sheet follows a particular syllabus. Do not attempt the sheet before you have completed your preparation for that
syllabus.
• After completing the sheet check your answers with the solution booklet and complete the Result Grid. Finally spend time
to analyse your performance and revise the areas which emerge out as weak in your evaluation.

1. In a triangle ABC, 2a2 + 4b2 + c2 = 4ab + 2ac, then cos B is


D D D
equal to 3. If s = a + b + c, r= , r1 = , r2 = and
1 s s-a s-b
(a) 0 (b)
8 D
3 7 r3 = , then the value of 1 + 1 + 1 + 1 is
(c) (d) s -c r12 r22 r32 r 2
8 8
2. The angles of elevation of the top of a tower (A) from the a 2 + b2 + c 2
(a) 0 (b)
top (B) and bottom (D) at a building of height a are 30° and D2
45° respectively. If the tower and the building stand at the
D2 a 2 + b 2 + c2
same level, then the height of the tower is (c) (d)
a 2 + b2 + c 2 D
a 3 4. In triangle ABC given 9a2 + 9b2 – 17c2 = 0.If
(a) a 3 (b)
3 -1
cot A + cot B m
= , then the value of (m + n) equals
a(3 + 3) cot C n
(c) (d) a( 3 –1) (a) 13 (b) 5
2
(c) 7 (d) 9

RESPONSE GRID 1. 2. 3. 4.

Space for Rough Work


EBD_7504
M-110 NTA JEE Main

5. ABCD is a rectangular field. A vertical lamp post of height


sin 3B
12 m stands at the corner A. If the angle of elevation of its top 10. If in a DABC, 2b2 = a2 + c2, then is equal to
sin B
from B is 60° and from C is 45°, then the area of the field is

(a) 48 2 sq.m (b) 48 3 sq.m c2 - a 2 c2 - a 2


(a) (b)
2ca ca
(c) 48 sq.m (d) 12 2 sq.m 2 2
æ c2 - a 2 ö æ c2 - a 2 ö
6. If the angle A of a triangle ABC is given by the equation (c) ç ca ÷ (d) ç 2ca ÷
è ø è ø
5 cos A + 3 = 0 , then sin A and tan A are the roots of the
equation 11. Two men are on the opposite side of a tower. They measure
the angle of elevation of the top of the tower 45° and 30°
(a) 15x 2 - 8x - 16 = 0 (b) 15x 2 - 8 2 x + 16 = 0
respectively. If the height of the tower is 40 m, find the
(c) 15x 2 - 8x + 16 = 0 (d) 15x 2 + 8x - 16 = 0 distance between the men.
7. A balloon is observed simultaneously from three points A, (a) 40 m (b) 40 3 m
B and C on a straight road directly under it. The angular (c) 68.280 m (d) 109.28 m
elevation at B is twice and at C is thrice that of A. If the 12. A vertical pole consists of two parts, the lower part being
distance between A and B is 200 metres and the distance one third of the whole. At a point in the horizontal plane
between B and C is 100 metres, then the height of balloon is through the base of the pole and distance 20 meters from it,
given by the upper part of the pole subtends an angle whose tangent
(a) 50 metres (b) 50 3 metres 1
is . The possible heights of the pole are
2
(c) 50 2 metres (d) None of these
(a) 20 m and 20 3 m (b) 20 m and 60 m
8. A, B, C are the angles of a triangle, then
(c) 16 m and 48 m (d) None of these
sin 2 A + sin 2 B + sin 2 C - 2 cos A cos Bcos C = 13. In an equilateral triangle, the inradius, circumradius and one
(a) 1 (b) 2 of the ex-radii are in the ratio
(c) 3 (d) 4 (a) 2 : 3 : 5 (b) 1 : 2 : 3
9. The base of a cliff is circular. From the extremities of a (c) 3 : 7 : 9 (d) 3 : 7 : 9
diameter of the base the angles of elevation of the top of the 14. An observer on the top of a tree, finds the angle of
cliff are 30° and 60°. If the height of the cliff be 500 metres, depression of a car moving towards the tree to be 30°. After
then the diameter of the base of the cliff is 3 minutes this angle becomes 60°. After how much more
2000 / 3m time, the car will reach the tree?
(a) 1000 3m (b)
(a) 4 min. (b) 4.5 min.
(c) 1000 / 3m (d) 2000 2m (c) 1.5 min (d) 2 min.

RESPONSE 5. 6. 7. 8. 9.
GRID 10. 11. 12. 13. 14.
Space for Rough Work
Mathematics M-111

15. The shadow of a tower is found to be 60 metre shorter when 20. In a DABC , ÐB = p / 3 and ÐC = p / 4 . If D divides BC
the sun’s altitude changes from 30° to 60°. The height of the
tower from the ground is approximately equal to sin ÐBAD
internally in ratio 1 : 3, then =?
(a) 62 m (b) 301 m sin ÐCAD
(c) 101 m (d) 52 m (a) 1 / 6 (b) 1/3
sin B (c) 1 / 3 (d) 2/3
16. In any triangle ABC, if cos A = , then
2sin C 21. Each side of an equilateral triangle subtends an angle of 60°
(a) a = b = c (b) c = a at the top of a tower h m high located at the centre of the
(c) a = b (d) b = c triangle. If a is the length of each of side of the triangle, then
17. Let in a triangle ABC, the line joining orthocentre and
circumcentre be parallel to the side BC. (a) 3a 2 = 2h 2 (b) 2a 2 = 3h 2
Statement-1 : tan A, tan B, tan C are in A.P. (c) a 2 = 3h 2 (d) 3a 2 = h 2
Statement-2 : If tan A, tan B, tan C are in A.P. then
22. An aeroplane flying horizontally 1 km above the ground is
tan A tan C = 3.
observed at an elevation of 60° and after 10 s the elevation
(a) Statement -1 is true, Statement-2 is true; Statement -2 is
is observed to be 30°. The uniform speed of the aeroplane in
a correct explanation for Statement-1
kilometre per hour is
(b) Statement -1 is true, Statement-2 is true; Statement -2
is not a correct explanation for Statement-1 (a) 60 3 (b) 240
(c) Statement -1 is false, Statement-2 is true (c) 240 3 (d) 480
(d) Statement -1 is true, Statement-2 is false a 2 - b2
23. In a triangle ABC, ÐC = 90°, then is equal to :
18. In a DABC , if angle C is obtuse, then a 2 + b2
(a) tan A tan B < 1 (b) tan A tan B £ 1 (a) sin (A + B) (b) sin (A – B)
(c) tan A tan B > 1 (d) None of these
æA-Bö
19. A tower of height b subtends an angle at a point O on the (c) cos (A + B) (d) sin ç ÷
è 2 ø
level of the foot of the tower and at a distance a from the
foot of the tower. If a pole mounted on the tower also 24. The length of the shadow of a pole inclined at 10° to the
subtends an equal angle at O, the height of the pole is vertical towards the sun is 2.05 metres, when the elevation
of the sun is 38°. The length of the pole is
æ a 2 - b2 ö æ a2 + b2 ö
(a) bç ÷ (b) bç ÷ 2.05sin 38° 2.05sin 42°
ç a 2 + b2 ÷ ç a 2 - b2 ÷ (a) (b)
è ø è ø sin 42° sin 38°
2.05cos 38°
æ a 2 - b2 ö æ a2 + b2 ö (c) (d) None of these
(c) aç ÷ (d) aç ÷ cos 42°
ç a2 + b2 ÷ ç a 2 - b2 ÷
è ø è ø

RESPONSE 15. 16. 17. 18. 19.


GRID 20. 21. 22. 23. 24.
Space for Rough Work
EBD_7504
M-112 NTA JEE Main

25. A circular ring of radius 3 cm hangs horizontally from a


point 4 cm vertically above the centre by 4 strings attached a 2 + b2 + c 2
28. Statement-1 : In any right angled triangle is
at equal intervals to its circumference. If the angle between R2
two consecutive strings be q then cos q equal to always equal to 8.
Statement-2 : a2 = b2 + c2
4 4 (a) Statement -1 is true, Statement-2 is true; Statement -2 is
(a) (b)
5 25 a correct explanation for Statement-1
(b) Statement -1 is true, Statement-2 is true; Statement -2
16
(c) (d) None is not a correct explanation for Statement-1
25 (c) Statement -1 is false, Statement-2 is true
26. If A, B, C are acute positive angles such that A + B+ C = p (d) Statement -1 is true, Statement-2 is false
and cot A cot B cot C = K , then 29. From a point a metre above a lake the angle of elevation of a
1 1 cloud is a and the angle of depression of its reflection is b.
(a) K£ (b) K³ The height of the cloud is
3 3 3 3
a sin(a + b) a sin(a + b)
1 1 (a) metre (b) metre
(c) K < (d) K > sin(a - b) sin(b - a )
9 3
(c) a sin(a – b) (d) None of these
27. A balloon is coming down at the rate of 4 m/min and its metre
angle of elevation is 45° from a point on the ground which sin(a + b)
has been reduced to 30° after 10 minutes. Balloon will be on 30. Angles of a triangle are in the ratio 4 : 1 : 1. The ratio between
the ground at a distance of how many meters from the its greatest side and perimeter is
observer 3 1
(a) (b)
(a) 20 3 m (b) 20(3 + 3) m 2+ 3 2+ 3
3 2
(c) 10(3 + 3) m (d) None of these (c) (d)
3+2 2+ 3
RESPONSE 25. 26. 27. 28. 29.
GRID 30.

MATHEMATICS CHAPTERWISE SPEED TEST-87


Total Questions 30 Total Marks 120
Attempted Correct
Incorrect Net Score
Cut-off Score 40 Qualifying Score 58
Success Gap = Net Score – Qualifying Score
Net Score = (Correct × 4) – (Incorrect × 1)
Space for Rough Work
PHYSICS Speed
Full Syllabus Test-1 TEST
No. of Questions
30
Maximum Marks
120
Time
1 Hour
88
Subject-wise

GENERAL INSTRUCTIONS
• This test contains 30 MCQ's. For each question only one option is correct. Darken the correct circle/ bubble in the
Response Grid provided on each page.
• You have to evaluate your Response Grids yourself with the help of solutions provided at the end of this book.
• Each correct answer will get you 4 marks and 1 mark shall be deduced for each incorrect answer. No mark will be given/
deducted if no bubble is filled. Keep a timer in front of you and stop immediately at the end of 60 min.
• The sheet follows a particular syllabus. Do not attempt the sheet before you have completed your preparation for that
syllabus.
• After completing the sheet check your answers with the solution booklet and complete the Result Grid. Finally spend time
to analyse your performance and revise the areas which emerge out as weak in your evaluation.

1. A block A of mass m1 rests on a horizontal table. A light D


string connected to it passes over a frictionless pulley at
the edge of table and from its other end another block B of
mass m2 is suspended. The coefficient of kinetic friction Signal C R
between the block and the table is µk. When the block A is
sliding on the table, the tension in the string is (a) 10.62 MHz (b) 10.61 kHz
(m 2 – m k m1 ) g m1m 2 (1 + m k )g (c) 5.31 MHz (d) 5.31 kHz
(a) (m1 + m 2 ) (b) (m1 + m 2 ) 4. In a double slit experiment, the two slits are 1 mm apart and
the screen is placed 1 m away. A monochromatic light
m1m 2 (1 – m k )g (m 2 + m k m1 )g wavelength 500 nm is used. What will be the width of each
(c) (m1 + m 2 ) (d) (m1 + m 2 ) slit for obtaining ten maxima of double slit within the central
maxima of single slit pattern ?
2. The relation between energy U, pressure P and volume V (a) 0.1 mm (b) 0.5 mm (c) 0.02 mm (d) 0.2 mm
for an ideal gas in an adiabatic process is given by, relation 5. A circuit is connected as shown in the figure with the switch
U = a + bP V. Find the value of adiabatic exponent (g) of this S open. When the switch is closed, the total charge that
gas. flows from y to x is
b +1 b +1 a +1 a
(a) (b) (c) (d)
b a b a+b
3. A diode detector is used to detect an amplitude modulated
wave of 60% modulation by using a condenser of capacity
250 picofarad in parallel with a load resistance 100 kilo ohm.
Find the maximum modulated frequency which could be
detected by it.
(a) zero (b) 54 m C (c) 27 m C (d) 81 m C

RESPONSE GRID 1. 2. 3. 4. 5.
Space for Rough Work
EBD_7504
PT-2 NTA JEE Main

6. From the top of a tower, a particle is projected with a velocity 12. A point P lies on the axis of a ring of mass M and radius R at
of 19.6 m/s at an angle of 30° with the horizontal. If g = 9.8 a distance R from its centre C. A small particle starts from P
m/s2, the particle will move at right angles to its initial and reaches at C under gravitational attraction only. Its
direction of motion after a time of speed at centre C will be :
(a) 1 s (b) 4 s (c) 6 s (d) 8 s
2Gm
7. Two cars moving in opposite directions approach each other (a)
R R
with speed of 22 m/s and 16.5 m/s respectively. The driver of
the first car blows a horn having a frequency of 400 Hz. The R
C P
frequency heard by the driver of the second car is [velocity
2Gm æ 1 ö
of sound = 340 m/s] : 1-
R çè ÷
(b)
(a) 361 Hz (b) 411 Hz (c) 448 Hz (d) 350 Hz 2ø
8. A moving body with a mass m1 and velocity u strikes a
stationary body of mass m2. The masses m1 and m2 should 2Gm
(c) ( 2 - 1)
be in the ratio m1/m2 so as to decrease the velocity of the R
first body to 2u/3 and giving a velocity of v to m 2 assuming
a perfectly elastic impact. The ratio m1/m2 is (d) zero
13. The figure shows a thin metalic rod whose one end is
(a) 5 (b) 1 / 5 (c) 1 / 25 (d) 25
pivoted at point O. The rod rotates about the end O in a
9. If the susceptibility of dia, para and ferro magnetic materials plane perpendicular to the uniform magnetic field with
are cd, cp, cf respectively, then angular frequency w in clockwise direction. Which of the
(a) cd < cp < cf (b) cd < cf < cp following is correct ?
× × × × × ×
(c) cf < cd < cp (d) cf < cp < cd
× × × × × ×
10. A hollow cylinder has a charge q coulomb within it. If f is
× × × × × ×
the electric flux in units of volt meter associated with the
curved surface B, the flux linked with the plane surface A in × × × O× × ×
units of volt meter will be × × × × × ×
B
× × × × × ×

C A (a) The free electrons of the rod move towards the outer
end
q q 1æ q ö (b) The free electrons of the rod move towards the pivoted
f - f (d) ç - f÷
(a) 2e 0 (b) (c) e0 2 è e0 end.
3 ø
11. A mass M, attached to a horizontal spring, executes S.H.M. (c) The free electrons of the rod move towards the mid-
with amplitude A1. When the mass M passes through its point of the rod.
mean position then a smaller mass m is placed over it and (d) The free electrons of the rod do not move towards any
both of them move together with amplitude A2. The ratio of end of the rod as rotation of rod has no effect on motion
æ A1 ö of free electrons.
çè A ÷ø is: 14. For a prism kept in air, it is found that for an angle of incidence
2
1 60°, the angle of Prism A, angle of deviation d and angle of
M +m æ M ö2 emergence ‘e’ become equal. Then the refractive index of
(a) (b) çè ÷
M M + mø the material of prism is
1
M (a) 1.73 (b) 1.15 (c) 1.5 (d) 1.33
æ M + mö 2
(c) çè ÷ (d)
M ø M +m

RESPONSE 6. 7. 8. 9. 10.
GRID 11. 12. 13. 14.

Space for Rough Work


Physics PT-3

15. A circular hole of diameter R is cut from a disc of mass M 19. Two radioactive nuclei P and Q, in a given sample decay
and radius R. The circumference of the cut passes through into a stable nucleous R. At time t = 0, number of P species
the centre of the disc. The moment of inertia of the remaining are 4 N0 and that of Q are N0. Half-life of P (for conversion to
R) is 1 minute whereas that of Q is 2 minutes. Initially there
portion of the disc about an axis perpendicular to the disc are no nuclei of R present in the sample. When number of
and passing through its centre is nuclei of P and Q are equal, the number of nuclei of R present
in the sample would be
æ 15 ö 2 æ 1ö 2
(a) çè ÷ø MR (b) çè ÷ø MR 9N 0 5N0
32 8 (a) 3N0 (b) (c) (d) 2N0
2 2
20. In the circuit shown below, what will be the readings of the
æ 3ö 2 æ 13 ö 2 voltemeter and ammeter?
(c) çè ÷ø MR (d) çè ÷ø MR
8 32
100 W
16. A gaseous mixture consists of 16 g of helium and 16 g of
Cp L C R
oxygen. The ratio of the mixture is
Cv
V
(a) 1.62 (b) 1.59 (c) 1.54 (d) 1.4 A
300V 300V
17. Truth table for system of four NAND gates as shown in
figure is : 100V
A
220V, 50Hz

(a) 800 V, 2 A (b) 300 V, 2 A


Y (c) 220 V, 2.2 A (d) 100 V, 2 A
21. Energy of an electron in an excited hydrogen atom is –3.4
eV. Its angular momentum will be
B (a) 3.72 × 10–34Js (b) 2.10 × 10–34Js
(c) 1.51 × 10 Js–34 (d) 4.20 × 10–34Js
A B Y A B Y
0 0 0 0 0 0 22. Starting from rest, a body slides down a 45º inclined plane in
twice the time it takes to slide down the same distance in the
0 1 1 0 1 0 absence of friction. The coefficient of friction between the
(a) (b)
1 0 1 1 0 1 body and the inclined plane is:
1 1 0 1 1 1 (a) 0.33 (b) 0.25 (c) 0.75 (d) 0.80
A B Y A B Y 23. A uniform magnetic field of induction B is
confined to a cylindrical region of radius R. The magnetic
0 0 1 0 0 1
0 1 1 0 1 0 dB
field is increasing at a constant rate of (tesla/second).
(c) (d) dt
1 0 0 1 0 1
An electron of charge e, placed at the point P on the periphery
1 1 0 1 1 1 of the field experiences an acceleration
18. In an experiment, the angles are required to be measured
using an instrument. 29 divisions of the main scale exactly 1 eR dB
(a) towards left
coincide with the 30 divisions of the vernier scale. If the 2 m dt
smallest division of the main scale is half-a-degree (0.5°), B
1 eR dB R
then the least count of the instrument is : (b) towards right
2 m dt
(a) one degree (b) half degree eR dB
(c) one minute (d) half minute (c) towards left P
m dt
(d) zero

RESPONSE 15. 16. 17. 18. 19.


GRID 20. 21. 22. 23.

Space for Rough Work


EBD_7504
PT-4 NTA JEE Main

24. An iron tyre is to be fitted on to a wooden wheel 1m in 27. Two identical charged spheres suspended from a common
diameter. The diameter of tyre is 6 mm smaller than that of point by two massless strings of lengths l, are initially at a
wheel. The tyre should be heated so that its temperature distance d (d << l) apart because of their mutual repulsion.
increases by (the coefficient of cubical expansion of iron is The charges begin to leak from both the spheres at a constant
3.6 × 10–5/°C) rate. As a result, the spheres approach each other with a
velocity v. Then v varies as a function of the distance x
(a) 167°C (b) 334°C (c) 500°C (d) 1000°C
between the spheres, as :
25. The radiation corresponding to 3 ® 2 transition of hydrogen
1 1
atom falls on a metal surface to produce photoelectrons. - -1
These electrons are made to enter a magnetic field of 3 × 10– (a) v µ x 2 (b) v µ x (c) v µ x 2 (d) v µ x
4 T. If the radius of the largest circular path followed by 28. All batteries are having emf 10 volt and internal resistance
these electrons is 10.0 mm, the work function of the metal is negligible. All resistors are in ohms. Calculate the current in
close to: the right most 2W resistor of the circuit
(a) 1.8 eV (b) 1.1 eV (c) 0.8 eV (d) 1.6 eV 2
10
26. Figure shows a small mass connected to a string, which is
2 2 10 2 2
attached to a vertical post. If the ball is released when the
string is horizontal as shown, the magnitude of the total
acceleration of the mass as a function of the angle q is:
10
10

2 10
q
25 25 12 6
(a) A (b) A (c) A (d) A
12 6 25 25
29. A sphere rolls down an inclined plane without slipping. What
fraction of its total energy is rotational ?
2 3 4 5
(a) (b) (c) (d)
(a) g sin q (b) g cos q 7 7 7 7
30. Two identical piano wires kept under the same tension T
have a fundamental frequency of 600 Hz. The fractional
(c) g 3cos 2 q + 1 (d) g 3sin 2 q + 1 increase in the tension of one of the wires which will lead to
occurrence of 6 beats/s when both the wires oscillate
together would be
(a) 0.02 (b) 0.03 (c) 0.04 (d) 0.01

RESPONSE 24. 25. 26. 27. 28.


GRID 29. 30.

SUBJECTWISE SPEED TEST-88 - PHYSICS


Total Questions 30 Total Marks 120
Attempted Correct
Incorrect Net Score
Cut-off Score 45 Qualifying Score 60
Success Gap = Net Score – Qualifying Score
Net Score = (Correct × 4) – (Incorrect × 1)
Space for Rough Work
PHYSICS Speed
Full Syllabus Test-2 TEST
No. of Questions
30
Maximum Marks
120
Time
1 Hour
89
Subject-wise

GENERAL INSTRUCTIONS
• This test contains 30 MCQ's. For each question only one option is correct. Darken the correct circle/ bubble in the
Response Grid provided on each page.
• You have to evaluate your Response Grids yourself with the help of solutions provided at the end of this book.
• Each correct answer will get you 4 marks and 1 mark shall be deduced for each incorrect answer. No mark will be given/
deducted if no bubble is filled. Keep a timer in front of you and stop immediately at the end of 60 min.
• The sheet follows a particular syllabus. Do not attempt the sheet before you have completed your preparation for that
syllabus.
• After completing the sheet check your answers with the solution booklet and complete the Result Grid. Finally spend time
to analyse your performance and revise the areas which emerge out as weak in your evaluation.

1. Two long parallel wires carry currents i1 and i2 such that 4. A coil of resistance 50 W is connected across a 5.0 V
i1 > i2. When the currents are in the same direction, the battery. 0.1 s after the battery is connected, the current in
magnetic field at a point midway between the wires is the coil is 60 mA. The inductance of the coil is
6 × 10–6 T. If the direction of i2 is reversed, the field becomes (a) 5.5 H (b) 1.5 H
i (c) 2.5 H (d) 9.5 H
3 × 10–5 T. The ratio of 1 is
i2 5. In the given circuit with steady current, the potential drop
1 2 3 across the capacitor must be
(a) (b) 2 (c) (d)
2 3 2
2. Two blocks are connected over a massless pulley as shown A V R B
in fig. The mass of block A is 10 kg and the coefficient of
kinetic friction is 0.2. Block A slides down the incline at
constant speed. The mass of block B in kg is C
V

2V 2R
A

30º B

(a) 3.5 (b) 3.3 (c) 3.0 (d) 2.5 2V V V


(a) (b) (c) (d) V
3. E, M, J and G denote energy, mass, angular momentum and 3 3 2
gravitational constant respectively. The dimensions of
EJ 2
are that of :
M 5G 2
(a) angle (b) length (c) mass (d) time

RESPONSE GRID 1. 2. 3. 4. 5.
Space for Rough Work
EBD_7504
PT-6 NTA JEE Main

6. A glass marble dropped from a certain height above the 10. An asteroid of mass m is approaching earth initially at a
horizontal surface reaches the surface in time t and then distance of 10Re with speed vi. It hits the earth with a speed
continues to bounce up and down. The time in which the
vf then (Re and Me are radius and mass of earth)
marble finally comes to rest is
é1+ e ù é1 - e ù 2Gm æ 1ö
tê (d) t ê (a) v 2f = vi2 + ç1 - ÷
(a) ent (b) e2t (c)
ú
ë1 - e û
ú
ë1 + e û M e R è 10 ø
7. A thin wire of length L and uniform linear mass density r is
bent into a circular loop with centre at O as shown. The 2GM e æ 1ö
(b) v 2f = vi2 + çè1 + ÷ø
moment of inertia of the loop about the axis xx’ is : Re 10
x x’
2GM e æ 1ö
90° (c) v 2f = vi2 + çè1 - ÷ø
Re 10

O
2Gm æ 1ö
(d) v 2f = vi2 + ç1 - ÷
R e è 10 ø
11. Let there be a spherically symmetric charge distribution with
rL3 rL3 5rL3 3rL3
(a) (b) (c) (d) æ5 rö
8p 2 16 p 2 16 p 2 8p 2 charge density varying as r(r ) = r0 çè - ÷ø upto r = R ,
8. A stone projected with a velocity u at an angle q with the 4 R
horizontal reaches maximum height H1. When it is projected
and r(r ) = 0 for r > R , where r is the distance from the
æp ö
with velocity u at an angle çè - q÷ø with the horizontal, it origin. The electric field at a distance r(r < R) from the origin
2
is given by
reaches maximum height H2. The relation between the
horizontal range R of the projectile, heights H1 and H2 is r0 r æ 5 r ö 4pr0 r æ 5 r ö
(a) ç - ÷ (b) ç - ÷
(a) R = 4 H1H 2 (b) R = 4(H1 – H2) 4e 0 è 3 R ø 3e 0 è 3 R ø

H12 r0 r æ 5 r ö r0 r æ 5 r ö
R= (c) ç - ÷ (d) ç - ÷
3ε 0 è 4 R ø
(c) R = 4 (H1 + H2) (d)
H 22 4ε0 è 4 R ø
9. Carbon monoxide is carried around a closed cycle abc in 12. The Young's modulus of a wire of length L and radius r is
which bc is an isothermal process as shown in the figure . Y N/m2. If the length and radius are reduced to L/ 2 and r/
The gas absorbs 7000 J of heat as its temperture increases 2, then its young's modulus will be
from 300 K to 1000 K in going from a to b. The quantity of (a) Y/2 (b) Y (c) 2 Y (d) 4 Y
heat rejected by the gas during the process ca is
13. In a CE transistor amplifier, the audio signal voltage across
P the collector, resistance 2kW is 2V. If the base resistance is
P2 b 1kW and the current amplification of the transistor is 100,
the input signal voltage is :
(a) 0.1 V (b) 1.0 V (c) 1 mV (d) 10 mV
P1 a c 14. Hydrogen atom in ground state is excited by a
V monochromatic radiation of l = 975 Å. Number of spectral
O V1 V2 lines in the resulting spectrum emitted will be
(a) 4200 J (b) 5000 J (c) 9000 J (d) 9800 J (a) 3 (b) 2 (c) 6 (d) 10

RESPONSE 6. 7. 8. 9. 10.
GRID 11. 12. 13. 14.

Space for Rough Work


Physics PT-7

15. Two communicating vessels contain mercury. The diameter 20. Which one of the following curves represents the variation
of one vessel is n times larger than the diameter of the other. of impedance (Z) with frequency f in series LCR circuit?
A column of water of height h is poured into the left vessel.
The mercury level will rise in the right-hand vessel by (s =
relative density of mercury and r = density of water) Z Z

(a) (b)

f f

n2 h h
(a)
( n + 1) 2
s
(b)
( n + 1) s
2
Z Z
(c) (d)
h h
(c) (d)
( n + 1)2 s n2 s
16. When a metallic surface is illuminated by a light of
f f
wavelength l, the stopping potential for the photoelectric
current is 3 V. When the same surface is illuminated by light
of wavelength 2l, the stopping potential is 1V. The threshold 21. A mixture consists of two radioactive materials A1 and A2
wavelength for this surface is with half lives of 20 s and 10 s respectively. Initially the
(a) 4l (b) 3.5 l (c) 3l (d) 2.75l mixture has 40 g of A1 and 160 g of A2. The amount of the
17. The molar heat capacity C for an ideal gas going through a
two in the mixture will become equal after :
given process is given by C = a/T , where a is a constant.
If g = CP/CV , the work done by one mole of gas during (a) 60 s (b) 80 s (c) 20 s (d) 40 s
heating from T0 to h T0 through the given process will be 22. The earth’s magnetic field lines resemble
that of a dipole at the centre of the earth. If the magnetic
1 æ h - 1ö
(a) ln h (b) a ln h – çè g - 1÷ø RT0 moment of this dipole is close to 8 × 1022 Am2, the value of
a
earth’s magnetic field near the equator is close to (radius of
æ h - 1ö the earth = 6.4 × 106 m)
(c) a ln h – (g – 1) RT0 (d) a ln h + çè g - 1÷ø RT0
(a) 0.6 Gauss (b) 1.2 Gauss
18. The electric field of an electromagnetic wave travelling through (c) 1.8 Gauss (d) 0.32 Gauss
vaccum is given by the equation E = E0 sin (kx – wt). The
quantity that is independent of wavelength is 23. The total length of a sonometer wire between fixed ends is
k 110 cm. Two bridges are placed to divide the length of wire
(a) kw (b) (c) k2 w (d) w in ratio 6 : 3 : 2. The tension in the wire is 400 N and the mass
w
19. A point mass oscillates along the x-axis according to the law per unit length is 0.01 kg/m. What is the minimum common
x = x0 cos(wt - p / 4) . If the acceleration of the particle is frequency with which three parts can vibrate?
(a) 1100 Hz (b) 1000 Hz
written as a = A cos(wt + d ) ,then
(c) 166 Hz (d) 100 Hz
A = x0w , d = 3p / 4 (b) A = x0, d = -p / 4
2
(a)

(c) 2
A = x0w , d = p / 4 (d) A = x0w 2 , d = -p / 4

RESPONSE 15. 16. 17. 18. 19.


GRID 20. 21. 22. 23.

Space for Rough Work


EBD_7504
PT-8 NTA JEE Main

24. Determine the modulation index and carrier swing of an FM -mp


wave having a frequency deviation of 25 kHz and a (a) v = v0 e -mp (b) v = v0 e r
modulating signal of 5 kHz.
(a) 7, 70 kHz (b) 8, 80 kHz
(c) 5, 50 kHz (d) 10, 100 kHz (c) v = v0 emp (d) v = 0
25. For a gas, difference between two specific heats is 5000 J/ 28. An object at 2.4 m in front of a lens forms a sharp image on
mole°C. If the ratio of specific heat is 1.6, the two specific a film 12 cm behind the lens. A glass plate 1 cm thick, of
heats in J/mole-°C are refractive index 1.50 is interposed between lens and film
with its plane faces parallel to film. At what distance
(a) C P = 1.33´ 10 4 , C V = 2.66 ´ 10 4
(from lens) should object shifted to be in sharp focus of
(b) C P = 13.3´ 10 4 , C V = 8.33 ´103 film?
(a) 7.2 m (b) 2.4 m (c) 3.2 m (d) 5.6 m
(c) C P = 1.33´ 10 4 , C V = 8.33 ´103 29. In circuit shown below, the resistances are given in ohms
and the battery is assumed ideal with emf equal to 3 volt.
(d) C P = 2.6´ 10 4 , C V = 8.33 ´10 4
The voltage across the resistance R 4 is
26. A single slit Fraunhoffer diffraction pattern is formed with
white light. For what wavelength of light the third secondary 50W
maximum in the diffraction pattern coincides with the second
secondary maximum in the pattern for red light of R1 R3 60W R 4 30W
wavelength 6500 Å?
(a) 4400 Å (b) 4100 Å (c) 4642.8 Å (d) 9100 Å 3V 50W R2
27. In the figure shown PRQ is a curved vertical wall with rough R5 30W
inner surface with the floor is smooth. The radius of the
curved wall is r and its length is quarter of a circle. A particle
of mass m is projected on the floor at P, grazing the rough (a) 0.4 V (b) 0.6 V (c) 1.2 V (d) 1.5 V
wall as shown , with an initial velocity v0. The velocity of the 30. A copper wire of length 40cm, diameter 2mm and resistivity
particle at Q will be 1.7 × 10–8 Wm forms a square frame. If a uniform magnetic
field B exists in a direction perpendicular to the plane of
dB
square frame and it changes at a steady rate = 0.02 T/s,
dt
then the current induced in the frame is
(a) 9.3 × 10–2 A (b) 9.3 × 10–1 A
–2
(c) 3.3 × 10 A (d) 19.3 × 10–2 A

RESPONSE 24. 25. 26. 27. 28.


GRID 29. 30.

SUBJECTWISE SPEED TEST-89 - PHYSICS


Total Questions 30 Total Marks 120
Attempted Correct
Incorrect Net Score
Cut-off Score 45 Qualifying Score 60
Success Gap = Net Score – Qualifying Score
Net Score = (Correct × 4) – (Incorrect × 1)
Space for Rough Work
PHYSICS Speed
Full Syllabus Test-3 TEST
No. of Questions
30
Maximum Marks
120
Time
1 Hour
90
Subject-wise

GENERAL INSTRUCTIONS
• This test contains 30 MCQ's. For each question only one option is correct. Darken the correct circle/ bubble in the
Response Grid provided on each page.
• You have to evaluate your Response Grids yourself with the help of solutions provided at the end of this book.
• Each correct answer will get you 4 marks and 1 mark shall be deduced for each incorrect answer. No mark will be given/
deducted if no bubble is filled. Keep a timer in front of you and stop immediately at the end of 60 min.
• The sheet follows a particular syllabus. Do not attempt the sheet before you have completed your preparation for that
syllabus.
• After completing the sheet check your answers with the solution booklet and complete the Result Grid. Finally spend time
to analyse your performance and revise the areas which emerge out as weak in your evaluation.

1. A force is given by F = at + bt2 , where t is time, the 3. A particle undergoes simple harmonic motion having time
dimensions of a and b are respectively period T. The time taken in 3/8th oscillation is
(a) [M L T–4] and [M L T–1] 3 5
(b) [M L T–1] and [M L T0] (a) T (b) T
(c) [M L T–3] and [M L T–4] 8 8
(d) [M L T–3] and [M L T0] 5 7
2. Which of the following time-displacement graph is not (c) T (d) T
12 12
possible in nature? 4. The angular velocity of a body changes from w1 to w2
t t without applying torque but by changing moment of inertia.
The ratio of initial radius of gyration to the final radius of
(a) (b) gyration is
s s (a) w2 : w1 (b) w2 2 : w12
t t
(c) w2 : w1 (d) 1/ w2 :1/ w1
5. The work function of aluminium is 4.2 eV. If two photons
(c) (d) each of energy 3.5 eV strike an electron of aluminium, then
s s emission of electron will
(a) depend upon the density of the surface
(b) possible
(c) not possible
(d) None of these

RESPONSE GRID 1. 2. 3. 4. 5.
Space for Rough Work
EBD_7504
PT-10 NTA JEE Main

6. If a capacitor of capacitance ‘C’ is connected in series with a g


an inductor of inductance L, then the angular frequency will (a) > (b) <
g a
be
a g
1 L (c) = (d) >
(a) (b) g a
LC C
11. What should be the minimum value of refractive index of
(c) LC (d) LC the material of the prism for the reflections to take place as
7. Two wires A and B of the same material, having radii in the shown in the figure?
ratio 1 : 2 and carry currents in the ratio 4 : 1. The ratio of drift
speed of electrons in A and B is
45º
(a) 16 : 1 (b) 1 : 16
(c) 1 : 4 (d) 4 : 1
R
8. If for a gas, = 0.67 , this gas is made up of molecules
CV
which are
(a) diatomic (a) 1.7 (b) 1.4
(b) mixture of diatomic and polyatomic molecules (c) 1.2 (d) 2.7
(c) monoatomic 12. Electric lines of force about a negative point charge are
(d) polyatomic (a) circular, anti-clockwise
9. A metallic bar is heated from 0ºC to 100ºC. The coeficient of (b) circular, clockwise
linear expansion is 10–5 K–1. What will be the percentage (c) radial, inwards
increase in length? (d) radial, outwards
(a) 0.01% (b) 0.1% 13. A gun fires two bullets at 60º and 30º with horizontal. The
(c) 1% (d) 10% bullets strike at some horizontal distance. The ratio of
10. A rough vertical board has an acceleration a along the maximum height for the two bullets is in the ratio of
horizontal so that a block of mass M pressing against it (a) 2 : 1 (b) 3 : 1
does not fall. The coefficient of friction between block and (c) 4 : 1 (d) 1 : 1
the board is 14. A generator has an e.m.f. of 440 Volt and internal resistance
a of 400 Ohm. Its terminals are connected to a load of 4000
Ohm the voltage across the load is
M (a) 220 volt (b) 440 volt (c) 200 volt (d) 400 volt
15. The radius vector, drawn from the sun to a planet, sweeps
out equal areas in equal intervals of time. This is the
statement of
(a) Kepler’s first law (b) Kepler’s second aw
(c) Newton’s first law (d) Kepler’s third law

RESPONSE 6. 7. 8. 9. 10.
GRID 11. 12. 13. 14. 15.

Space for Rough Work


Physics PT-11

16. The path difference between the two waves : 20. Two particles of mass m1 and m2 (m1 > m2) attract each
æ 2 px ö other with a force inversely proportional to the square of
y1 = a1 sin ç wt - ÷ the distance between them. If the particles are initially held
è l ø
at rest and then released, the centre of mass will
æ 2px ö (a) move towards m1 (b) move towards m2
and y 2 = a 2 sin ç wt - + f ÷ will be
è l ø (c) remain at rest (d) Nothing can be said
2p 2 p æ pö 21. The r.m.s. velocity of oxygen molecule at 16ºC is 474 m/sec.
(a) f (b) çf - ÷ The r.m.s. velocity in m/s of hydrogen molecule at 127ºC is
l l è 2ø
(a) 1603 (b) 1896 (c) 2230.59 (d) 2730
l 2p æ pö
(c) f (d) çf + ÷ 22. To demonstrate the phenomenon of interference, we require
2p l è 2ø two sources which emit radiation
17. The diagram shows the energy levels for an electron in a (a) of the same frequency
certain atom. Which transition shown represents the (b) of different wavelengths
emission of a photon with the most energy? (c) of the same frequency and having a definite phase
n=4 relationship
n=3 (d) of nearly the same frequency
23. A galvanometer coil has a resistance of 15W and gives full
n=2 scale deflection for a current of 4 mA. To convert it to an
ammeter of range 0 to 6 A
(a) 10 mW resistance is to be conncected in parallel to the
n =1
I II III IV galvanometer
(a) IV (b) III (c) II (d) I (b) 10 mW esistance is to be connected in series with the
galvanometer
18. The direction of propagation of electromagnetic waves is
(c) 0.1 W resistance is to be connected in parallel to the
given by the direction of
ur ur galvanometer
(a) Vector E (b) Vector B (d) 0.1 W resistance is to be connected in series with the
ur ur galvanometer
(c) Vector ( E ´ B ) (d) None of these
19. A body of mass 10 kg and velocity 10 m/s collides with a 24. A uniform rod of mass m, length l , area of cross-section A
stationary body of mass 5 kg. After collision both bodies stick has Young’s modulus Y. If it is hanged vertically, elongation
to each other, velocity of the bodies after collision will be under its own weight will be
mgl 2mgl mgl mgY
3 18 9 20 (a) (b) (c) (d)
(a) m/s (b) m/s (b) m/s (d) m/s 2AY AY AY Al
10 3 20 3

RESPONSE 16. 17. 18. 19. 20.


GRID 21. 22. 23. 24.

Space for Rough Work


EBD_7504
PT-12 NTA JEE Main

25. The magnifying power of a telescope is 9. When it is 28. A plane wave of wavelength 6250 Å is incident normally on
adjusted for parallel rays, the distance between the objective a slit of width 2 × 10–2 cm. The width of the principal maximum
and the eye piece is found to be 20 cm. The focal length of on a screen distant 50 cm will be
lenses are (a) 312.5 × 10–3 cm (b) 312.5 × 10–3 m
(a) 18 cm, 2 cm (b) 11 cm, 9 cm (c) 312.5 × 10–3 m (d) 312 m
(c) 10 cm, 10 cm (d) 15 cm, 5 cm 29. In a p-type semi-conductor germanium is doped with
26. If two soap bubbles of different radii are connected by a (a) aluminium (b) boron
tube. Then (c) gallium (d) all of these
(a) air flows from the smaller bubble to the bigger 30. The table given below represents the truth table for which
(b) air flows from bigger bubble to the smaller bubble till of the following combinations of logic gates?
the sizes are interchanged
A B Y
(c) air flows from the bigger bubble to the smaller bubble
0 0 1
till the sizes become equal
0 1 0
(d) there is no flow of air.
1 0 1
27. An object undergoing SHM takes 0.5 s to travel from one
1 1 0
point of zero velocity to the next such point. The distance
between those points is 50 cm. The period, frequency and (a) A
B Y
amplitude of the motion is
(a) 1s, 1Hz, 25 cm (b) 2s, 1Hz, 50 cm A
(b) Y
(c) 1s, 2Hz, 25 cm (d) 2s, 2Hz, 50 cm B

A
(c) B Y

(d) None of these

RESPONSE 25. 26. 27. 28. 29.


GRID 30.

SUBJECTWISE SPEED TEST-90 - PHYSICS


Total Questions 30 Total Marks 120
Attempted Correct
Incorrect Net Score
Cut-off Score 45 Qualifying Score 60
Success Gap = Net Score – Qualifying Score
Net Score = (Correct × 4) – (Incorrect × 1)
Space for Rough Work
PHYSICS Speed
Full Syllabus Test-4 TEST
No. of Questions
30
Maximum Marks
120
Time
1 Hour
91
Subject-wise

GENERAL INSTRUCTIONS
• This test contains 30 MCQ's. For each question only one option is correct. Darken the correct circle/ bubble in the
Response Grid provided on each page.
• You have to evaluate your Response Grids yourself with the help of solutions provided at the end of this book.
• Each correct answer will get you 4 marks and 1 mark shall be deduced for each incorrect answer. No mark will be given/
deducted if no bubble is filled. Keep a timer in front of you and stop immediately at the end of 60 min.
• The sheet follows a particular syllabus. Do not attempt the sheet before you have completed your preparation for that
syllabus.
• After completing the sheet check your answers with the solution booklet and complete the Result Grid. Finally spend time
to analyse your performance and revise the areas which emerge out as weak in your evaluation.

1. The respective number of significant figures for the numbers (a) 0° (b) 30°
23.023, 0.0003 and 2.1 × 10–3 are (c) 45° (d) 60°
(a) 5, 1, 2 (b) 5, 1, 5 4. A particle starting from rest falls from a certain height.
(c) 5, 5, 2 (d) 4, 4, 2 Assuming that the acceleration due to gravity remain the
2. If the kinetic energy of a particle is increased by 300%, the same throughout the motion, its displacements in three
momentum of the particle will increase by successive half second intervals are S1, S2 and S3 then
(a) 20% (b) 200% (a) S1 : S2 : S3 = 1 : 5 : 9
(c) 100% (d) 50% (b) S1 : S2 : S3 = 1 : 3 : 5
3. The pulleys and strings shown in the figure are smooth and (c) S1 : S2 : S3 = 9 : 2 : 3
of negligible mass. For the system to remain in equilibrium. (d) S1 : S2 : S3 = 1 : 1 : 1
The angle q should be
5. The Bulk modulus for an incompressible liquid is
(a) zero (b) unity
(c) infinity (d) between 0 and 1
q

2m
m m

RESPONSE GRID 1. 2. 3. 4. 5.
Space for Rough Work
EBD_7504
PT-14 NTA JEE Main

6. A transformer is employed to D
(a) convert A.C. into D.C. m1 m2 m3 m4
(b) convert D.C. into A.C. C
B
(c) obtain a suitable A.C. voltage
(d) obtain a suitable D.C. voltage
7. As a result of change in the magnetic flux linked to the A
closed loop shown in the fig, an e.m.f. V volt is induced in (a) m1 = m2 (b) m2 = m3
the loop. (c) m3 = m4 (d) m4 = m1
11. The meniscus of a liquid contained in one of the limbs of a
narrow U-tube is held in an electromagnet with the meniscus
in line with the field. The liquid is seen to rise. This indicates
that the liquid is
(a) ferromagnetic (b) paramagnetic
(c) diamagnetic (d) non-magnetic
The work done (in joule) in taking a charge Q coulomb 12. Under a constant torque the angular momentum of a body
once along the loop is changes from A to 4A in 4 second. The torque on the body
(a) QV (b) 2QV will be
(c) QV/2 (d) zero
1 4 3
8. What is the acceleration of a projectile at its heighest point (a) 1A (b) A (c) A (d) A
4 3 4
(a) maximum (b) minimum
13. The kinetic theory of gases
(c) zero (d) g
(a) explains the behaviour of an ideal gas
9. A sphere of radius R has uniform volume charge density.
(b) describes the motion of a single atom or molecule
The electric potential at a point (r < R) is
(c) relates the temperature of the gas with K.E. of atoms of
(a) due to the charge inside a sphere of radius r only the gas
(b) due to the entire charge of the sphere (d) all of the above
(c) due to the charge in the spherical shell of inner and 14. An electric bulb marked 40 W and 200V, is used in a circuit of
outer radii r and R, only supply voltage 100V. Now its power is
(d) independent of r (a) 10 W (b) 20 W (c) 40 W (d) 100 W
10. A ray of light passes through four transparent media with 15. The height at which the acceleration due to gravity becomes
refractive indices m1, m2, m3 and m4 as shown in the figure. g
The surfaces of all media are parallel. If the emergent ray CD (where g = the acceleration due to gravity on the surface
9
is parallel to the incident ray AB, we must have of the earth) in terms of R, the radius of the earth, is
R
(a) (b) R / 2 (c) 2 R (d) 2 R
2

RESPONSE 6. 7. 8. 9. 10.
GRID 11. 12. 13. 14. 15.

Space for Rough Work


Physics PT-15

16. A thermodynamic system is taken from state A to B along 20. The oscillating electric and magnetic field vectors of
ACB and is brought back to A along BDA as shown in the PV electromagnetic wave are oriented along
diagram. The net work done during the complete cycle is (a) the same direction and in phase
given by the area (b) the same direction but have a phase difference of 90º
(c) mutually perpendicular directions and are in phase
P (d) mutually perpendicular directions but has a phase
B difference of 90º
P2
C 21. If the cold junction of a thermo-couple is kept at 0°C and the
D hot junction is kept at T°C then the relation between neutral
P1
A temperature (Tn) and temperature of inversion (Ti) is
A¢ B¢
(a) Tn = 2Ti (b) Tn = Ti – T
V
(c) Tn = Ti + T (d) Tn = Ti/2
(a) P1ACBP2P1 (b) ACBB'A'A 22. Let E a be the electric field due to a dipole in its axial plane
(c) ACBDA (d) ADBB'A'A distant l and Eq be the field in the equatorial plane distant l',
17. A man is watching two trains, one leaving and the other then the relation between Ea and Eq will be
coming with equal speed of 4 m/s. If they sound their (a) E a = 4E q (b) E q = 2E a
whistles each of frequency 240 Hz, the number of beats heard
by man (velocity of sound in air = 320 m/s) will be equal to (c) E a = 2E q (d) E q = 3E a
(a) 12 (b) 0 (c) 3 (d) 6 23. Two coherent monochromatic light beams of intensities I
18. When a wave travels in a medium the particles displacement and 4 I are superimposed. The maximum and minimum
is given by the equation y = 0.03 sin p(2t – 0.01x), where x possible intensities in the resulting beam are
and y are in seconds. The wavelength of the wave is (a) 5I and I (b) 5I and 3I
(a) 200 m (b) 100 m (c) 20 m (d) 10 m (c) 9I and I (d) 9I and 3I
19. The value of current I in the circuit shown in figure is 24. When an electron jumps from the fourth orbit to the second
orbit, one gets the
R2= 5W (a) second line of Lyman series
(b) second line of Paschen series
(c) second line of Balmer series
3V R1= 5W R3= 5W
(d) first line of Pfund series
25. A uniform magnetic field acts at right angles to the direction
R4= 5W of motion of electron. As a result, the electron moves in a
circular path of radius 2 cm. If the speed of electron is
(a) 1.8 A (b) 0.8 A doubled, then the radius of the circular path will be
(c) 0.2 A (d) 1.6 A (a) 2.0 cm (b) 0.5 cm (c) 4.0 cm (d) 1.0 cm

RESPONSE 16. 17. 18. 19. 20.


GRID 21. 22. 23. 24. 25.

Space for Rough Work


EBD_7504
PT-16 NTA JEE Main

26. A photoelectric cell is illuminated by a point source of light 28. Two flat circular coils have a common center, but their planes
1 m away. When the source is shifted to 2 m, then are at right angles to each other. The inner coil has 150 turns
(a) number of electrons emitted is a quarter of the initial and radius of p cm. The outer coil has 400 turns and a radius
number of 2p cm. The magnitude of the resultant magnetic induction
(b) each emitted electron carries one quarter of the initial at the common centers of the coils when a current of 200 mA
energy is sent through each of them is
(c) number of electrons emitted is half the initial number (a) 10–3 Wb/m2 (b) 2 × 10–3 Wb/m2
(d) each emitted electron carries half the initial energy
(c) 5 × 10–3 Wb/m2 (d) 7 × 10–3 Wb/m2
27. A hollow cylinder has a charge q coulomb within i. If f is 29. Optical fibres transmit light along its axis, by the process of
the electric flux in units of voltmeter associated with the (a) total internal reflection
curved surface B, the flux linked with the plane surface A in (b) refraction
units of voltmeter will be
B (c) interference
(d) diffraction
C A
30. When a p-n junction diode is reverse biased the flow of
current across the junction is mainly due to
(b) f
q
(a)
2e0 3 (a) diffusion of charges
q 1æ q ö (b) drift of charges
(c) -f (d) ç - f÷ (c) depends on the nature of material
e0 2 è e0 ø
(d) both drift and diffusion of charges

RESPONSE
26. 27. 28. 29. 30.
GRID

SUBJECTWISE SPEED TEST-91 - PHYSICS


Total Questions 30 Total Marks 120
Attempted Correct
Incorrect Net Score
Cut-off Score 45 Qualifying Score 60
Success Gap = Net Score – Qualifying Score
Net Score = (Correct × 4) – (Incorrect × 1)
Space for Rough Work
CHEMISTRY Speed
Full Syllabus Test-1 TEST
No. of Questions
30
Maximum Marks
120
Time
1 Hour
92
Subject-wise

GENERAL INSTRUCTIONS
• This test contains 30 MCQ's. For each question only one option is correct. Darken the correct circle/ bubble in the
Response Grid provided on each page.
• You have to evaluate your Response Grids yourself with the help of solutions provided at the end of this book.
• Each correct answer will get you 4 marks and 1 mark shall be deduced for each incorrect answer. No mark will be given/
deducted if no bubble is filled. Keep a timer in front of you and stop immediately at the end of 60 min.
• The sheet follows a particular syllabus. Do not attempt the sheet before you have completed your preparation for that
syllabus.
• After completing the sheet check your answers with the solution booklet and complete the Result Grid. Finally spend time
to analyse your performance and revise the areas which emerge out as weak in your evaluation.

1. The structure : H 2 C = C - CH = CH 2
(b)
CH3 H |
CH3
C C H
CH3 C (c) CH 2 = CH - CH = CH 2
COOH
CH3 (d) HC º C - CH = CH 2
shows 4. The compound A on heating gives a colourless gas and a
(a) tautomerism residue that is dissolved in water to obtain B. Excess of CO 2
(b) geometrical isomerism is bubbled through aqueous solution of B, C is formed which
is recovered in the solid form. Solid C on gentle heating
(c) optical isomerism
gives back A. The compound is
(d) geometrical and optical isomerism
(a) CaSO4.2H2O (b) CaCO3
2. Ionisation of weak acid can be calculated by the formula
(c) Na2CO3 (d) K2CO3
Ka 100 5. The fraction of total volume occupied by the atoms present
(a) 100 (b)
C 1 + 10(pKa - pH) in a simple cube is
(c) Both (a) and (b) (d) None of these p p
3. Which of the following will yield a mixture of (a) (b)
3 2 4 2
2-chlorobutene and 3-chlorobutene on treatment with HCl ?
(a) CH 2 = C = CH – CH3 p p
(c) (d)
4 6

RESPONSE GRID 1. 2. 3. 4. 5.
Space for Rough Work
EBD_7504
CT-2 NTA JEE Main

6. The straight chain polymer (silicones) is formed by 9. Beryllium shows diagonal relationship with aluminium.
(a) hydrolysis of CH3 SiCl 3 followed by condensation Which of the following similarity is incorrect ?
polymerisation (a) Be forms beryllates and Al forms aluminates
(b) Be(OH)2 like Al(OH)3 is basic.
(b) hydrolysis of (CH3)4Si by addition polymerisation
(c) Be like Al is rendered passive by HNO 3 .
(c) hydrolysis of (CH3)2SiCl2 followed by condensation
polymerisation (d) Be2C like Al4C3 yields methane on hydrolysis.
10. Carbon cannot be used in the reduction of Al2O3 because
(d) hydrolysis of (CH3)3SiCl followed by condensation
(a) the enthalpy of formation of CO2 is more than that of
polymerisation
Al2O3
7. The time taken for 90% of a first order reaction to complete (b) pure carbon is not easily available
is approximately (c) the enthalpy of formation of Al 2O3 is very high
(a) 1.1 times that of half-life (d) it is an expensive proposition
(b) 2.2 times that of half-life 11. In the synthesis of glycerol from propene, compounds
formed are
(c) 3.3 times that of half-life (a) Glycerol b-chlorohydrin and allyl chloride
(d) 4.4 times that of half-life (b) Glyceryl trichloride and glycerol a-chlorohydrin
8. Which of the following volume (V) - temperature (T) plots (c) Allyl alcohol and a-chlorohydrin
represents the behaviour of one mole of an ideal gas at one (d) Allyl alcohol and monosodium glycerolate
12. The missing reagents R1 and R2 in the following series of
atmospheric pressure ? reactions are
V(L) -
(38.8 L, 373K) R
CH 3 CH 2 Br ¾¾®
R
1 [ ] ¾¾® 2 CH 3 CHP + Ph 3
(22.4L, (a) PhLi and Ph3P respectively
(a) (b) Ph3P and PhLi respectively
273K)
(c) Ph3P and C2H5ONa respectively
T(K) (d) Either (b) or (c)
13. At 25°C molar conductance of 0.1 molar aqueous
V(L)
(28.6 L, 373K) solution of ammonium hydroxide is 9.54 ohm–1 cm2mol–1
and at infinite dilution its molar conductance is
(b) (22.4L, 238 ohm–1 cm2 mol–1. The degree of ionisation of ammonium
273K) hydroxide at the same concentration and temperature is :
(a) 20.800% (b) 4.008%
T(K) (c) 40.800% (d) 2.080%
V(L) 14. Red precipitate is obtained when ethanol solution of
(30.6 L, 373K)
dimethylglyoxime is added to ammoniacal Ni(II). Which of
(c) (22.4L, the following statements is not true ?
273K) (a) Red complex has a square planar geometry.
(b) Complex has symmetrical H-bonding
T(K) (c) Red complex has a tetrahedral geometry.
V(L) (d) Dimethylglyoxime functions as bidentate ligand.
15. The ratio of number of oxygen atoms (O) in 16.0 g ozone
(22.4L, (O3), 28.0 g carbonmonoxide (CO) and 16.0 oxygen (O2) is
(d) (Atomic mass : C = 12, O = 16 and Avogadro’s constant
273K)
NA = 6.0 × 1023 mol–1)
(14.2 L, 373K) (a) 3 : 1 : 2 (b) 1 : 1 : 2
T(K) (c) 3 : 1 : 1 (d) 1 : 1 : 1

RESPONSE 6. 7. 8. 9. 10.
GRID 11. 12. 13. 14. 15.
Space for Rough Work
Chemistry CT-3

16. The secondary structure of a protein refers to 20. Which one of the following sets forms the biodegradable
(a) fixed configuration of the polypeptide backbone polymer?
(b) a– helical backbone (a) CH2 = CH – CN and CH2 = CH – CH = CH2
(c) hydrophobic interactions (b) H2N – CH2 – COOH and H2N–(CH2)5 – COOH
(d) sequence of a– amino acids. (c) HO – CH2 – CH2 – OH and
17. Aniline when treated with conc. HNO3 gives
NH2 NH2 HOOC COOH
NO2 NH2
NO2
(a) (b) (d) CH = CH2 and CH2 = CH – CH = CH2

NH2 21. When a sample of gas is compressed at constant temperature


O from 15 atm to 60 atm, its volume changes from 76 cm3 to
NH2 20.5 cm3. Which of the following statements are possible
explanations of this behaviour?
(a) The gas behaves non-ideally
(c) (d) (b) The gas dimerises
(c) The gas is adsorbed into the vessel walls
O (a) 1, 2 and 3 (b) 1 and 2 only
18. Which one of the following statements is FALSE?
(c) 2 and 3 only (d) 1 only
(a) The correct order of osmotic pressure for 0.01 M
22. An unknown compound A has a molecular formula C4H6,
aqueous solution of each compound is
when A is treated with an excess of Br2, a new substance B
BaCl 2 > KCl > CH3COOH > Sucrose with formula C4H6Br2 is formed. A forms a white precipitate
(b) The osmotic pressure (p) of a solution is given by the with ammonical silver nitrate solution. A may be
equation p = MRT, where M is the molarity of the (a) Butyne-1 (b) Butyne-2
solution (c) Butene-1 (d) Butene-2
(c) Raoult’s law states that the vapour pressure of a 23. Which of the following is not the correct order for the
component over a solution is proportional to its mole stated property ?
fraction (a) Ba > Sr > Mg; atomic radius
(d) Two sucrose solutions of same molality prepared in (b) F > O > N : first ionization enthalpy
different solvents will have the same freezing point (c) Cl > F > I; electron affinity
depression (d) O > Se > Te; electronegativity
19. In the solid state, SO3 may have structure 24. DG in Ag2O ® 2 Ag + 1/2O2 at a certain temperature is
O O –10 kJ mol–1. Pick the correct statement
S (a) Ag2O decomposes to Ag and O2
(b) Ag and O2 combines to form Ag2O
O O
(a) O O (c) Reaction is in equilibrium
S S (d) Reaction does not take place
O O O 25. If electron, hydrogen, helium and neon nuclei are all moving
with the velocity of light, then the wavelength associated
O O O
with these particles are in the order
(b) S S S (a) Electron > hydrogen > helium > neon
O O O O O O (b) Electron > helium > hydrogen > neon
(c) a & b both (c) Electron < hydrogen < helium < neon
(d) None of these (d) Neon < hydrogen < helium < electron

RESPONSE 16. 17. 18. 19. 20.


GRID 21. 22. 23. 24. 25.

Space for Rough Work


EBD_7504
CT-4 NTA JEE Main

26. Methyl cyanide is less basic than methylamine because: H


(a) there is a triple bond between carbon and nitrogen H3C C
atoms 29. Consider the two molecules : (I) C C CH3
(b) its molecular weight is higher than methylamine
(c) the lone pair of electrons in nitriles belongs to sp-orbital O O
and lone pair of electrons in amines belongs to H
sp3 -orbital H H
(d) None of these H3C C CH3
27. The relative rate of catalytic hydrogenation of the following (II) C C .
alkenes is H
O O
H
Which of the following statements about them is true ?
(a) II is capable of forming intramolecular hydrogen
I II III IV bonding whereas I is not
(a) I > II > III > IV (b) III > II > I > IV (b) Both are capable of forming equally stable
intramolecular hydrogen bonding
(c) IV > I > II > III (d) IV > I > II = III (c) I is likely to form more stable intramolecular hydrogen
28. When chlorine reacts with cold and dilute solution of sodium bonding than II
hydroxide, the products obtained are (d) Both of them cannot form intramolecular hydrogen
bonding
(a) Cl– + ClO– (b) Cl– + ClO -2 30. Which one of the following statement is not true?
(a) In vulcanization the formation of sulphur bridges between
(c) Cl– + ClO3- (d) Cl– + ClO -4 different chains make rubber harder and stronger.
(b) Natural rubber has the trans -configuration at every
double bond
(c) Buna-S is a copolymer of butadiene and styrene
(d) Natural rubber is a 1, 4 - polymer of isoprene

RESPONSE
26. 27. 28. 29. 30.
GRID

SUBJECTWISE SPEED TEST-92 - CHEMISTRY


Total Questions 30 Total Marks 120
Attempted Correct
Incorrect Net Score
Cut-off Score 45 Qualifying Score 60
Success Gap = Net Score – Qualifying Score
Net Score = (Correct × 4) – (Incorrect × 1)
Space for Rough Work
CHEMISTRY Speed
Full Syllabus Test-2 TEST
No. of Questions
30
Maximum Marks
120 1 Hour
Time 93
Subject-wise

GENERAL INSTRUCTIONS
• This test contains 30 MCQ's. For each question only one option is correct. Darken the correct circle/ bubble in the
Response Grid provided on each page.
• You have to evaluate your Response Grids yourself with the help of solutions provided at the end of this book.
• Each correct answer will get you 4 marks and 1 mark shall be deduced for each incorrect answer. No mark will be given/
deducted if no bubble is filled. Keep a timer in front of you and stop immediately at the end of 60 min.
• The sheet follows a particular syllabus. Do not attempt the sheet before you have completed your preparation for that
syllabus.
• After completing the sheet check your answers with the solution booklet and complete the Result Grid. Finally spend time
to analyse your performance and revise the areas which emerge out as weak in your evaluation.

1. The standard EMF for the cell reaction, 4. The electronegativity difference between N and F is greater
than that between N and H yet the dipole moment of
Zn + Cu 2+ ¾
¾® Cu + Zn 2+ is 1.1 volt at 25°C. NH3 (1.5 D) is larger than that of NF3 (0.2D). This is because
The EMF for the cell reaction, when 0.1 M Cu2+ and (a) in NH3 the atomic dipole and bond dipole are in the
0.1 M Zn2+ solutions are used, at 25°C is same direction whereas in NF3 these are in opposite
(a) 1.10 V (b) 0.10 V directions
(c) –1.10 V (d) –0.110 V (b) in NH3 as well as NF3 the atomic dipole and bond dipole
are in opposite directions
2. H3C CH3 ¾¾¾¾¾¾
(i) Alc. KOH
® P is (c) in NH3 the atomic dipole and bond dipole are in the
(ii) SO2Cl2 , D
opposite directions whereas in NF3 these are in the
I same direction
(a) 3-Chloropropene (b) 2-Chloropropene (d) in NH3 as well as in NF3 the atomic dipole and bond
(c) 1-Chloropropene (d) 1, 2-Dichloropropane dipole are in the same direction
3. The electrons, identified by quantum numbers n and l, 5. End product (a) of the following sequence of reaction is :
(I) n = 3, l = 2 (II) n = 5, l = 0 Fe Excess CH 3Br
(III) n = 4, l = 1 (IV) n = 4, l = 2 NO2 ¾¾¾¾ ® ¾¾¾¾¾¾ ®A
30%HCl
(V) n = 4, l = 0
can be placed in order of increasing energy, as NH2 N(CH3)2
(a) (b)
(a) I < V < III < IV < II (b) I < V < III < II < IV
(c) V < I < III < II < IV (d) V < I < II < III < IV NO2
+ –
(c) NH3Br (d) OCH3

RESPONSE GRID 1. 2. 3. 4. 5.
Space for Rough Work
EBD_7504
CT-6 NTA JEE Main

6. The second order Bragg diffraction of X-rays with l = 1.0Å White precipitate of AgCl appears on adding which of the
following?
from a set of parallel planes in a metal occurs at an angle of
60°. The distance between the scattering planes in the crystal is (a) NH3 (b) aqueous NaCl
(a) 0.575Å (b) 1.00Å (c) 2.00Å (d) 1.15Å (c) aqueous HNO3 (d) aqueous NH4Cl
7. 14 g of N2 and 36 g of ozone are at the same pressure and 13. Which among the following is substitution reaction ?
temperature . Their volumes will be related as OMgX
(a) C = O + RMg X ¾ ¾® C
(a) 2VN = 3VO3 (b) 3VN = 2VO3 R
2 2 O
O
(c) 3VN = 4VO3 (d) 4VN = 3VO3
2 2
8. Which one of the following reactions of xenon compounds (b) + | O ¾
¾® O
is not feasible?
3 O O
(a) 3XeF4 + 6H 2 O ¾¾ ® 2Xe + XeO3 +12HF + O 2
2 Anhy . ZnCl 2
(b) 2XeF2 + 2H 2 O ¾¾ ® 2Xe + 4HF + O 2 (c) R 3COH ¾¾ ¾ ¾ ¾
¾® Turbidity
Conc . HCl
(c) XeF6 + RbF ¾¾ ® Rb[XeF7 ]
(d) XeO3 + 6HF ¾¾ (d) All of the above
® XeF6 + 3H 2O
14. Regarding F– and Cl– which of the following statements is/
9. Identify the correct statement
(a) Elemental sodium can be prepared and isolated by are correct?
electrolysing an aqueous solution of sodium chloride (i) Cl– can give up an electron more easily than F –
(b) Elemental sodium is a strong oxidising agent (ii) Cl– is a better reducing agent than F–
(c) Elemental sodium is insoluble in ammonia (iii) Cl– is smaller in size than F–
(d) Elemental sodium is easily oxidised (iv) F– can be oxidized more readily than Cl –
10. Arrange the following alcohols in order of increasing ease (a) (i) and (ii) (b) (i), (ii) and (iv)
of dehydration (c) (iii) and (iv) (d) Only (i)
CH 3CH 2 OH C 6 H 5 CH 2 OH 15. Benzaldehyde reacts with ethanoic KCN to give
I II
(a) C6H5CHOHCN (b) C6H5CH(OH)COC6H5
Cl 3CCH 2 OH F3CCH 2 OH (c) C6H5CHOHCOOH (d) C6H5CHOHCHOHC6H5
III IV
16. In a closed system : A(s) 2B(g) + 3C(g) , if the partial
(a) II < I < IV < III (b) IV < III < II < I pressure of C is doubled, then partial pressure of B will be:
(c) IV < III < I < II (d) II < I < III < IV
(a) two times the original value
11. Which of the following species is basic & reducing?
(b) one-half of its original value
(a) SO 32 - (b) SO 24 -
(c) S2 O 24 - (d) HSO -4 1
(c) times the original value
12. Consider the following equilibrium 2 2
+ (d)
AgCl + 2NH 3 ƒ éë Ag ( NH 3 )2 ùû + Cl - 2 2 times its original value

6. 7. 8. 9. 10.
RESPONSE
11. 12. 13. 14. 15.
GRID
16.

Space for Rough Work


Chemistry CT-7

17. Under the influence of an electric field, the particles in a sol 21. In order to refine “blister copper” it is melted in a furnace
migrate towards cathode. The coagulation of the same sol is and is stirred with green logs of wood. The purpose is
studied using NaCl, Na2SO4 and Na3PO4 solutions. Their (a) to expel the dissolved gases in blister copper.
coagulating values will be in the order (b) to bring the impurities to surface and oxidize them.
(a) NaCl > Na2SO4 > Na3PO4 (c) to increase the carbon content of copper.
(b) Na2SO4 > Na3PO4 > NaCl (d) to reduce the metallic oxide impurities with hydrocarbon
(c) Na3PO4 > Na2SO4 > NaCl gases liberated from the wood.
(d) Na2SO4 > NaCl > Na3PO4 22. One of the equivalent energy resonance structure for formate
18. A perfect gas is found to obey the relation PV3/2 = constant. If anion is shown below
the gas is compressed to half of its volume at temperature T

: : : :
A O
adiabatically, the final temperature of the gas will be
H –C
(a) 2T 2 (b) 4T (c) T 2 (d) 2T O:
B
19. Mn 2 + forms a complex with Br - ion. The magnetic moment
of the complex is 5.92 B.M. What would be the probable Which C – O bond is longer ?
formula and geometry of the complex? (a) A
(a) [MnBr6 ]4 - , octahedral (b) B
(b) [MnBr4 ]2 - , square planar (c) Both are equal
(d) Structure shown is not correct
(c) [MnBr4 ]2 - , tetrahedral
23. The colour of CuCr2O7 solution in water is green because?
(d) [MnBr5 ]3- , trigonal bipyramidal
(a) Cr2 O72 - ions are green
20. Which of the following resonance structure is lowest
in energy? (b) Cu 2+ ions are green
(c) Both Cr 2O2– 2+
7 and Cu ions are green
H O
:

H O : : : :
| || | (d) Cu2+ ions are blue and Cr2 O72 - ions are yellow
(a) H–C –C (b) H – C – C
| | + 24. CH 3 Br + Nu - ¾
¾® CH 3 - Nu + Br -
: :

O– H O– H
B
H HB
:

The decreasing order of the rate of the above reaction with


nucleophiles (Nu–) A to D is
[Nu– = (a) PhO–, (b) AcO–, (c) OH–, (d) CH3O–]
:

H O : :
| | (a) A > B > C > D (b) B > D > C > A
(c) H–C –C
| + (c) D > C > A > B (d) D > C > B > A
: :

O– H
B
H 25. P4O10 is not used to dry NH3 gas because
(a) P4O10 reacts with moisture in NH3
(a) A (b) B (b) P4O10 is not a drying agent
(c) C (d) All have same energy (c) P4O10 is acidic and NH3 is basic
(d) P4O10 is basic and NH3 is acidic

RESPONSE 17. 18. 19. 20. 21.


GRID 22. 23. 24. 25.

Space for Rough Work


EBD_7504
CT-8 NTA JEE Main

26. Among the following, the most acidic is :


(b) O = = =O
(a) CH3COOH (b) ClCH2COOH
(c) Cl2CHCOOH (d) Cl2CHCH2COOH O
27. The Ce (Z = 58) belongs to IIIrd group of periodic table. If it
furnish one a particle to form an element 'X', then X belongs to (c)
(a) III rd group (b) IInd group
st
(c) I group (d) zero group
O
28. The heat of sublimation of iodine is 24 cal g -1 at 50°C. If
O
specific heat of solid iodine and its vapour are 0.055 and
0.031cal g -1 respectively, the heat of sublimation of iodine (d)
at 100°C is
(a) 22.8 cal g–1 (b) 25.2 cal g–1 O
(c) – 22.8 cal g–1 (d) – 25.2 cal g–1 30. Aluminium chloride exists as dimer, Al2Cl6 in solid state as
29. Which of the following will show tautomerism ? well as in solution of non-polar solvents such as benzene.
O When dissolved in water, it gives
(a) [Al(OH) 6 ]3- + 3HCl (b) [Al(H 2 O) 6 ]3+ + 3Cl -
(a) Al 2 O3 + 6HCl
(c) Al 3+ + 3Cl - (d)

RESPONSE
26. 27. 28. 29. 30.
GRID

SUBJECTWISE SPEED TEST-93 - CHEMISTRY


Total Questions 30 Total Marks 120
Attempted Correct
Incorrect Net Score
Cut-off Score 45 Qualifying Score 60
Success Gap = Net Score – Qualifying Score
Net Score = (Correct × 4) – (Incorrect × 1)
Space for Rough Work
CHEMISTRY Speed
Full Syllabus Test-3 TEST
No. of Questions
30
Maximum Marks
120
Time
1 Hour
94
Subject-wise

GENERAL INSTRUCTIONS
• This test contains 30 MCQ's. For each question only one option is correct. Darken the correct circle/ bubble in the
Response Grid provided on each page.
• You have to evaluate your Response Grids yourself with the help of solutions provided at the end of this book.
• Each correct answer will get you 4 marks and 1 mark shall be deduced for each incorrect answer. No mark will be given/
deducted if no bubble is filled. Keep a timer in front of you and stop immediately at the end of 60 min.
• The sheet follows a particular syllabus. Do not attempt the sheet before you have completed your preparation for that
syllabus.
• After completing the sheet check your answers with the solution booklet and complete the Result Grid. Finally spend time
to analyse your performance and revise the areas which emerge out as weak in your evaluation.

4. In a compound AOH, electronegativity of ‘A’ is 2.1, the


CH3 compound would be
(a) Acidic
1. IUPAC name of is
(b) Neutral towards acid & base
CH2CH3
(c) Basic
(a) 1-methyl-3 ethyl cyclohexane (d) Amphoteric
(b) 1-ethyl-3 methyl benzene 5. Which of the following orders is wrong?
(c) 1-ethyl-3 methyl cyclo hexane (a) Electron affinity– N < O < F < Cl
(d) Cyclo hexane-1-ethyl-3-methyl (b) Ist ionisation potential – Be < B < N < O
2. Which of the following structures does not contain any (c) Basic property– MgO < CaO < FeO < Fe 2O3
chiral C atom but represent the chirality in the structure. (d) Reactivity–Be < Li < K < Cs Cl
(a) 2 – Ethyl – 3 – hexene (b) 2, 3-Pentadiene
(c) 1,3 – Butadiene (d) Pent – 3 – en – 1 – yne
6. The dipole moment of chlorobenzene is 1.5 D. The
3. N2 and O2 are converted to mono cations N2+ and O2+
respectively, which of the following is wrong?
(a) In N2+, the N – N bond weakens Cl
(b) In O2+, the O – O bond order increases Cl
(c) In O2+, paramagnetism decreases dipole moment of is
(d) N2+ becomes diamagnetic Cl Cl
(a) 2.86 D (b) 2.25 D
(c) 1.5 D (d) 0 D

1. 2. 3. 4. 5.
RESPONSE GRID
6.

Space for Rough Work


EBD_7504
CT-10 NTA JEE Main

7. Following substances are in solid state : 12. AB, A2 and B2 are diatomic molecules. If the bond enthalpies
(A) Methane (B) Cesium chloride of A2, AB and B2 are in the ratio 1:1 :0.5 and enthalpy of
(C) Ice (D) Lithium formation of AB from A2 and B2 is –100 kJ mol–1 . What is
Which non-conductive solid when melts converts into the bond energy of A2 :
conductive liquid? (a) 200 kJ mol–1 (b) 100 kJ mol–1
(a) C, D (b) Only C (c) 300 kJ mol–1 (d) 400 kJ mol–1
(c) Only B (d) A, B and C 13. Equal volume of 0.1 M urea and 0.1 M glucose are mixed.
8. On applying pressure to the equilibrium The mixture will have
ice water, which phenomenon will happen (a) Lower osmotic pressure
(a) More ice will be formed (b) Same osmotic pressure
(b) More water will be formed (c) Higher osmotic pressure
(c) Equilibrium will not be disturbed (d) None of these
(d) Water will evaporate 14. If the following half cells have the E° values as
9. Let n1 be the frequency of the series limit of the Lyman series,
n2 be the frequency of the first line of the Lyman series, and n3 Fe +3 + e – ¾
¾® Fe +2 ; E° = + 0.77V and Fe +2 + 2e - ¾
¾® Fe ;
be the frequency of the series limit of the Balmer series, then – E° = – 0.44V. The E° of the half cell Fe +3 + 3e – ¾
¾® Fe will
1 be
(a) n3 = (n – n ) (b) n2 – n1 = n3
2 1 3 (a) 0.33 V (b) 1.21 V
(c) n1 – n2 = n3 (d) n1 + n2 = n3 (c) 0.04 V (d) 0.605 V
15. The oxidation states of sulphur in the anions
Given, HF + H 2 O ¾¾® Ka + —;
10. ¾ H 3O + F SO 32 - , S2 O 24- and S2 O 62 - follow the order
K
F — + H 2 O ¾¾
¾b ® HF + OH — .
(a) SO 32 - < S 2 O 24 - < S2 O 62 -
Which relation is correct
(b) S 2O 24- < S2 O62- < SO 32-
1
(a) Kb = Kw (b) Kb =
Kw (c) S 2O 62- < S2 O 24- < SO 32-
Ka
(c) Ka × Kb = Kw (d) = Kw (d) S 2O 24- < SO 32- < S2 O 62-
Kb
16. Which of the following statements is not correct?
11. In an amino acid, the carboxyl group ionises at pK a1 = 2.34 (a) C– Cl bond in vinyl chloride is less polar than in CH3Cl
and ammonium ion at pK a 2 = 9.60. The isoelectric point of (b) C – Cl bond in vinyl chloride is stronger than in CH3Cl
the amino acid is at pH (c) C – Cl bond in vinyl chloride is shorter than in CH3Cl
(a) 5.97 (b) 2.34 (d) Vinyle chloride undergo nucleophilic substitution more
(c) 9.60 (d) 6.97 readily than CH3Cl.

RESPONSE 7. 8. 9. 10. 11.


GRID 12. 13. 14. 15. 16.

Space for Rough Work


Chemistry CT-11

21. In Lassaigne’s test, the organic compound is fused with a


NO2
piece of sodium metal in order to
17. (a) increase the ionisation of the compound
¾¾¾
Br
2 ® X ¾¾¾¾
3 ®Y
CH ONa (b) decrease the melting point of the compound
Fe,D CH3OH, D (c) increase the reactivity of the compound
(d) convert the covalent compound into a mixture of ionic
compounds
Cl
22. An aqueous solution of colourless metal sulphate M gives
Product (Y) of this reaction is – a white precipitate with NH4OH. This was soluble in excess
of NH4OH. On passing H2S through this solution a white
NO2 NO2
ppt. is formed. The metal M in the salt is
(a) Ca (b) Ba
(a) (b) (c) Al (d) Zn
23. Choose the correct order of T (True) and F (False) –
OCH3 Br
(1) When the pH of rain water is below 3.6, it is called acid
Cl OCH3
rain.
NO2 NO2 (2) Ozone hole occurs over Antarctica mainly during
September–October and it gets replenished in
November–December.
(c) (d) (3) Methylcyclohexane is an ozone-depleting molecule.
(4) COD is always larger than BOD.
OCH3 (a) TTTF (b) FTFT
OCH3 (c) FFFF (d) FTTT
18. Although Al has a high oxidation potential it resists 24. Which of the following compound can not used in
corrosion because of the formation of a tough, protective preparation of iodoform?
coat of (a) CH3CHO (b) CH3COCH3
(a) Al(NO3)2 (b) AlN (c) HCHO (d) 2-propanol
(c) Al2O3 (d) Al2(CO3)2 25. The correct priorities for the substituents shown below,
19. A metal which is not affected by conc. H2SO4, HNO3 or according to the E-Z sequence rule is
alkalis forms a compound X. This compound X can be used
to give a complex which finds its application for toning in I. – CN II. – CBr (CH 3 ) 2
photography? The metal is
(a) Au (b) Ag O
||
(c) Hg (d) Cu III. – COOH IV. – CH 2 - C - OCH 3
20. If Cl2 gas is passed into aqueous solution of KI containing
some CCl4 and the mixture is shaken, then”. O
(a) Upper layer becomes violet ||
(b) Lower layer becomes violet V. – C-H
(c) Homogenous violet layer is formed
(d) None (a) II, III, V, I, IV (b) V, II, I, IV, III
(c) III, IV, I, II, V (d) II, V, I, IV, III

RESPONSE 17. 18. 19. 20. 21.


GRID 22. 23. 24. 25.

Space for Rough Work


EBD_7504
CT-12 NTA JEE Main

26. Identify X in the sequence given : 28. An alkene upon ozonolysis yield
CHO – CH2– CH2– CH2 – CHO only. The alkene is
NH2 (a) CH2= CH – CH2 –– CH2 –– CH2 –– CH2 –– CH3
CHCl3 HCl
(Y) X + methanoic acid (b)
KOH (300 K)

Cl
(c)

(a) NH2 Cl (b) C N Cl


(d)

29. 1 mol of N 2 and 3 mol of H 2 are placed in a closed container


(c) N C Cl (d) CH3–NH Cl
at a pressure of 4 atm. The pressure falls to 3 atm at the same
temperature when the following equilibrium is attained
27. Select the rate law that corresponds to the data shown for N 2 (g ) + 3H 2 (g) 2 NH 3 (g ) .
the following reaction A + B ¾¾® C
The K p for the dissociation of NH 3 is
Expt. No. (1) (2) Initial Rate 3´ 3
(a)
3
atm - 2 (b) 0.5 ´ (1.5) 3 atm 2
1 0.012 0.035 0.10 0.5 ´ (1.5)

2 0.024 0.070 0.80 0.5 ´ (1.5)3 (1.5) 3


(c) atm 2 (d) atm - 2
3´ 3 0.5
3 0.024 0.035 0.10 30. 0.5 g mixture of K2Cr2O7 and KMnO4 was treated with excess
4 0.012 0.070 0.80 of KI in acidic medium. I2 liberated required 100 cm3 of 0.15N.
Na2S2O3 solution for titration. The percentage amount of
(a) Rate = K[B]3 (b) Rate = K [B]4 K2Cr2O7 in the mixture is
(c) Rate = K [A] [B]3 (d) Rate = K [A]2 [B]2 (a) 85.36 % (b) 14.64 %
(c) 58.63 % (d) 26.14 %

RESPONSE
26. 27. 28. 29. 30.
GRID

SUBJECTWISE SPEED TEST-94 - CHEMISTRY


Total Questions 30 Total Marks 120
Attempted Correct
Incorrect Net Score
Cut-off Score 45 Qualifying Score 60
Success Gap = Net Score – Qualifying Score
Net Score = (Correct × 4) – (Incorrect × 1)
Space for Rough Work
CHEMISTRY Speed
Full Syllabus Test-4 TEST
No. of Questions
30
Maximum Marks
120
Time
1 Hour
95
Subject-wise

GENERAL INSTRUCTIONS
• This test contains 30 MCQ's. For each question only one option is correct. Darken the correct circle/ bubble in the
Response Grid provided on each page.
• You have to evaluate your Response Grids yourself with the help of solutions provided at the end of this book.
• Each correct answer will get you 4 marks and 1 mark shall be deduced for each incorrect answer. No mark will be given/
deducted if no bubble is filled. Keep a timer in front of you and stop immediately at the end of 60 min.
• The sheet follows a particular syllabus. Do not attempt the sheet before you have completed your preparation for that
syllabus.
• After completing the sheet check your answers with the solution booklet and complete the Result Grid. Finally spend time
to analyse your performance and revise the areas which emerge out as weak in your evaluation.

1. An atom X belongs to 4th period of the periodic table and 4. Which of the following shows the tendency to form
has highest number of unpaired electrons in comparison to peroxide?
the other elements of the period. The atomic number of X is (a) Lithium (b) Magnesium
(a) 23 (b) 25 (c) Beryllium (d) Radium
(c) 24 (d) 33 5. A 1.0 M solution with respect to each of the metal halides
2. In O2- , O 02 and O 22 - molecular species, the total number of AX 3 , BX 2 , CX 3 and DX 2 is electrolysed using platinum
antibonding electrons respectively are : electrodes. If E° = 1.50 V, E° = 0.3 V,, E°C3+ /C =
A3+ / A B2+ /B
(a) 7, 6, 8 (b) 1, 0, 2
– 0. 74 V, E° = – 2.37 V..
(c) 6, 6, 6 (d) 8, 6, 8 D2+ /D
3. Aluminothermy used for on the spot welding of large iron The correct sequence in which the various metals are
structure is based on the fact that- deposited at the cathode is
(a) As compared to iron, aluminium has greater affinity for (a) A, B, C, D (b) A, B, C
oxygen. (c) D, C, B, A (d) C, B, A
(b) As compared to aluminium, iron has greater affinity for 6. Arrange hypophosphorous acid (H3 PO2), phosphorous
oxygen. acid (H3PO3) and Phosphoric acid (H3PO4) in the decreasing
(c) Reaction between aluminium and oxygen is endothermic. order of acidic strength
(d) Reaction between iron and oxygen is endothermic. (a) H3PO3 > H3PO4 > H3PO2
(b) H3PO4 > H3PO3 > H3PO2
(c) H3PO4 > H3PO2 > H3PO3
(d) H3PO4 » H3PO3 » H3PO2

1. 2. 3. 4. 5.
RESPONSE GRID
6.

Space for Rough Work


EBD_7504
CT-14 NTA JEE Main

7. Which of the following reactions corresponds to the (a) p1 < p2 (b) p1 > p2
definition of enthalpy of formation ?
(a) C(diamond) + O2(g) ® CO2(g) p1 p2
(c) p1 = p2 (d) =
(b) C(graphite) + O2(l) ® CO2(g) p1 + p 2 p1 + p 2
(c) C(graphite) + O2(g) ® CO2(g)
13. In the reaction of KMnO4 with an oxalate in acidic medium,
(d) C(g) + O2(g) ® CO2(g)
8. Among the reactions given below for B2H6, the one which MnO-4 is reduced to Mn 2 + and C 2 O 24 - is oxidised to
does not take place is
(a) B2H6 + HCl ¾¾ ® B2H5Cl + H2 CO2. Hence, 50 ml of 0.02 M KMnO4 is equivalent to
D (a) 100 ml of 0.05 M H 2C 2O 4
(b) 2B2H6 + 6NH3 ¾¾® B3N3H6 (borazine)
(c) B2H6 + 2N(CH3)3 ¾¾
® 2(CH3)3 NBH3 (b) 50 ml of 0.05 M H 2C 2O 4
H O+ (c) 25 ml of 0.2 M H 2C 2O 4
(d) B2H6 + 6C2H4 ¾¾¾® 3
3C2H5OH + 2B(OH)3
9. The pure crystalline substance on being heated gradually (d) 50 ml of 0.10 M H 2C 2O 4
first forms a turbid liquid at constant temperature and still at
14. In which reaction, there is change in oxidation number of N
higher temperature turbidity completely disappears. The
behaviour is a characteristic of substance forming. (a) 2NO2 ® N2O4
(a) Allotropic crystals (b) NH4OH ® NH4+ + OH–
(b) Liquid crystals (c) N2O5 + H2O ® 2HNO3
(c) Isomeric crystals
(d) 2NO2 + H2O ® HNO3 + HNO2
(d) Isomorphous crystals.
10. Silver bromide when dissolve in hypo solution gives complex 15. Potassium permanganate acts as an oxidant in neutral,
..... in which oxidation state of silver is .... alkaline as well as acidic media. The final products obtained
from it in the three conditions are, respectively
(a) Na3[Ag(S2O3)2], (I) (b) Na3[Ag(S2O3)3], (III)
(a) MnO42–, Mn3+ and Mn2+
(c) Na3[Ag(S2O3)2], (II) (d) Na3[Ag(S2O3)4], (I)
11. Which of the following can be termed as a mixed complex? (b) MnO2, MnO2 and Mn2+
(a) K4 [Fe(CN)6] (b) [Cu(NH3)4] SO4 (c) MnO2, MnO2+ and Mn3+
(c) [Co(NH3)4NO2Cl] Cl (d) K2FeO4 (d) MnO, MnO2 and Mn2+
12. The relationship between the values of osmotic pressures 16. The number of p electrons present in 6.4 g of calcium carbide
of solutions obtained by dissolving 6.00 g L–1 of CH3COOH is – (NA = Avagadro’s number)
(p1) and 7.45 g L–1 of KCl (p2) is (a) 4 NA (b) 0.4 NA
(c) 0.1 NA (d) 0.2 NA

RESPONSE 7. 8. 9. 10. 11.


GRID 12. 13. 14. 15. 16.

Space for Rough Work


Chemistry CT-15

21. An element (atomic mass =100g/mol) having bcc structure


C–CºH has unit cell edge 400pm. The density (in g/cm3) of the
17. +CH 3 - CH 2 MgBr gives
element is
(a) 10.376 (b) 5.19
C º CMgBr + CH3 - CH2
(a) (c) 7.289 (d) 2.144
22. An organic compound A (C4H10O) has two enantiomeric
forms and on dehydration it gives B(major product) and C
C º CMgBr + C2H6
(b) (minor product). B and C are treated with HBr/ Peroxide and
the compounds so produced were subjected to alkaline
hydrolysis then-
MgBr + CH3 - CH2 - C º CH
(c) (a) B will give an isomer of A
(b) C will give an isomer of A
C2H5 + HC º CMgBr (c) Neither of them will give isomer of A
(d)
(d) Both B and C will give isomer of A
18. The rate of SN1 reaction is fastest in the hydrolysis of 23. A reaction is found to be second order w.r.t. one of the
which of the following halides ? reactants & has rate constant of 0.5 mol–1 dm3 min–1. If
(a) C6H5CH2Br (b) CH3Br initial concentration is 0.2 mol dm–3 then t1/2 of reaction is
(c) (CH3)2CHBr (d) (CH3)3CBr (a) 5 min (b) 10 min
19. Two elements A & B form compounds having molecular (c) 15 min (d) 20 min
formulae AB2 and AB4. When dissolved in 20.0 g of benzene
1.00g of AB2 lowers f.p. by 2.3°C whereas 1.00g of AB4 CH 2 O
lowers f.p. by 1.3°C. The molal depression constant for 24. O CH 2
benzene in 1000g is 5.1. The atomic masses of A and B are CH 2 O
(a) 52, 48 (b) 42, 25
(c) 25, 42 (d) None The above shown polymer is obtained when a carbonyl
compound is allowed to stand. It is a white solid. The polymer
20. To detect iodine in presence of bromine, the sodium extract
is
is treated with NaNO 2 + glacial acetic acid + CCl 4 . Iodine
(a) trioxane (b) formose
is detected by the appearance of
(c) paraformaldehyde (d) metaldehyde.
(a) yellow colour of CCl 4 layer
25. Concentration of NH4Cl and NH4OH in a buffer solution is
(b) purple colour of CCl4
in the ratio of 1 : 1, Kb for NH4OH is 10–10. The pH of the
(c) brown colour in the organic layer of CCl 4 buffer is
(d) deep blue colour in CCl4 (a) 4 (b) 5
(c) 9 (d) 11

RESPONSE 17. 18. 19. 20. 21.


GRID 22. 23. 24. 25.

Space for Rough Work


EBD_7504
CT-16 NTA JEE Main

26. Aniline when diazotized in cold and when treated with 29. Phospholipids are esters of glycerol with
dimethyl aniline gives a coloured product. Its structure
(a) two carboxylic acid residues and one phosphate group
would be
(b) one carboxylic acid residue and two phosphate groups
(a) CH3NH N=N NHCH3
(c) three phosphate groups
(b) CH3 N=N NH2 (d) three carboxylic acid residues

(c) (CH3)2N N=N Cº N


Ether
+ C6H5MgBr ¾¾¾ H3O+
30. ® A ¾¾¾ ¾® Ba
(d) (CH3)2N NH

27. An organic amino compound reacts with aqueous nitrous


O N–MgBr
acid at low temperature to produce an oily nitrosoamine.
C C
The compound is:
(a) (b)
(a) CH3 NH2 (b) CH3CH2NH2
(c) CH3CH2NH.CH2CH3 (d) (CH3 CH2)3N
28. The standard reduction potential of N–H N–OH
C C
Li+/Li, Ba2+/Ba, Na+/Na and Mg2+/Mg are –3.05, –2.73, –
(c) (d)
2.71 and –2.37 volts respectively. Which one of the following
is strongest oxidising agent?
(a) Na+ (b) Li+
(c) Ba2+ (d) Mg2+

RESPONSE
26. 27. 28. 29. 30.
GRID

SUBJECTWISE SPEED TEST-95 - CHEMISTRY


Total Questions 30 Total Marks 120
Attempted Correct
Incorrect Net Score
Cut-off Score 45 Qualifying Score 60
Success Gap = Net Score – Qualifying Score
Net Score = (Correct × 4) – (Incorrect × 1)
Space for Rough Work
MATHEMATICS Speed
Full Syllabus Test-1 TEST
No. of Questions
30
Maximum Marks
120
Time
1 Hour
96
Subject-wise

GENERAL INSTRUCTIONS
• This test contains 30 MCQ's. For each question only one option is correct. Darken the correct circle/ bubble in the
Response Grid provided on each page.
• You have to evaluate your Response Grids yourself with the help of solutions provided at the end of this book.
• Each correct answer will get you 4 marks and 1 mark shall be deduced for each incorrect answer. No mark will be given/
deducted if no bubble is filled. Keep a timer in front of you and stop immediately at the end of 60 min.
• The sheet follows a particular syllabus. Do not attempt the sheet before you have completed your preparation for that
syllabus.
• After completing the sheet check your answers with the solution booklet and complete the Result Grid. Finally spend time
to analyse your performance and revise the areas which emerge out as weak in your evaluation.

1. If f : R ® R, g : R ® R be two given functions, then 4. If |ak| < 3, 1 £ k £ n, then all the complex numbers z satisfying
f (x) = 2 min (f(x) – g(x), 0) equals the equation 1 + a1 z + a2z2 +....+an zn = 0
(a) f(x) + g(x) – |g(x) – f(x)| 1
(b) f(x) + g(x) + |g(x) – f(x)| (a) lie outside the circle z =
4
(c) f(x) – g(x) + |g(x) – f(x)| 1
(d) f(x) – g(x) – |g(x) – f(x)| (b) lie inside the circle z =
4
2. The number of solutions of the equation 1
(c) lie on the circle z =
1 1 4
sin 5 x - cos5 x = - ( sin x ¹ cos x ) is 1 1
cos x sin x (d) lie in < z <
3 2
(a) 0 (b) 1
5. If a1, a2, a3,...., an is an AP with common difference d, then
(c) infinite (d) none of these
3. If A, B, C are acute positive angles such that A + B+ C = p é -1 æ d ö -1 æ d ö
and cot A cot B cot C = K , then tan ê tan ç 1 + a a ÷ + tan ç 1 + a a ÷ + ....
ë è 1 2ø è 2 3ø
1 1
(a) K £ (b) K ³
3 3 3 3 æ d öù
+ tan -1 ç ÷ ú is equal to
1 1 è 1 + a n -1 a n ø úû
(c) K< (d) K>
9 3 ( n - 1) d ( n - 1) d
(a) a1 + a n
(b) 1 + a1a n
nd a n - a1
(c) (d)
1 + a1 a n a n + a1

RESPONSE GRID 1. 2. 3. 4. 5.

Space for Rough Work


EBD_7504
MT-2 NTA JEE Main

6. One of the diameters of the circle circumscribing the 11. The coefficient of xn in the polynomial
rectangle ABCD is 4y = x + 7. If A and B are the points (–3, 4)
( x + n C 0 )( x + 3 . n C1 )(x + 5 . n C 2 ) .... (x+(2n + 1) nCn) is
and (5, 4) respectively, then the area of the rectangle is
(a) 16 sq. units (b) 24 sq. units (a) n . 2n (b) n . 2n + 1
(c) (n + 1) . 2n (d) n . 2n + 1
(c) 32 sq. units (d) none of these
7. The angle between tangents to the parabola y2 = 4 ax at the 12. Which of the following is NOT CORRECT?
points where it intersects with the line x – y – a = 0 is (a) The greatest integer less than or equal to ( 2 + 1) 6 is 197.
p p
(a) (b) (b) The integer next above ( 3 + 1) 2 m contains 2 m +1 as
3 4
p p factor
(c) (d) (c) The greatest integer less than or equal to the number
6 2
8. The probability that a certain beginner at golf gets a good (7 + 4 3 ) m is a multiple of 2.
1
shot if he uses the correct club is , and the probability of (d) If R = (6 6 + 14)2n+1 and f = R – [R] where [R] is
3
integer and 0 £ f < 1 then Rf = 202n + 1.
1 13. If the quadratic equation z2 + (a + ib) z + c + id = 0 where a,
a good shot with an incorrect club is . In his bag there are
4 b, c, d are non-zero real number, has a real root then
5 different clubs, only one of which is correct for the shot. If (a) abd = b2c + d2 (b) abc = bc2 + d2
2 2
he chooses a club at random and takes a stroke, the (c) abd = bc + ad (d) none of these
probability that he gets a good shot is 14. If a line segment AM = a moves in the plane XOY remaining
parallel to OX so that the left end point A slides along the
1 1 4 7
(a) (b) (c) (d) circle x2+ y2 = a2, the locus of M is
3 12 15 12 (a) x 2 + y 2 = 4a 2 (b) x 2 + y 2 = 2ax
9. In a certain town 25% families own a phone and 15% own a car
whereas 65% own neither a phone nor a car. 2000 families own (c) x 2 + y 2 = 2ay (d) x 2 + y 2 - 2ax - 2ay = 0
both a car and a phone. Consider the following statements in
this regard
15. For the series å t n , S n = 3t n - 2,
(a) 10% families own both a car and a phone n n
(b) 35% families own either a car or a phone. where Sn = å t n . Then å Sn
(c) 40,000 families live in the town. n =1 n =1
Which one of the statements are correct?
æ 3n ö
(a) a and b (b) a and c (c) b and c (d) a, b & c (a) 6ç - 1÷ - 2n (b) 6 n - 1 - 2n
10. The chord of contact of tangents from a point P to a circle ç 2n ÷
è ø
passes through Q. If l1 and l2 are the length of the tangents æ 2n ö
from P and Q to the circle, then PQ is equal to (c) 2n - 6 ç + 1÷ (d) none of these
ç 3n ÷
l +l l -l è ø
(a) 1 2 (b) 1 2 (c) l12 + l 22 (d) l1l2
2 2

RESPONSE 6. 7. 8. 9. 10.
GRID 11. 12. 13. 14. 15.

Space for Rough Work


Mathematics MT-3

16. Which of the following is a contradiction?


ì (1 + ax)1/x , x < 0
(a) (p Ù q)Ù ~ (p Ú q) (b) p Ú (- p Ù q) ï
(c) (p Þ q) Þ p (d) None of these ïï b , x=0
17. The mean and S.D. of the marks of 200 candidates were 22. Let f ( x) = í 1/3
found to be 40 and 15 respectively. Later, it was discovered ï ( x + c) - 1
ï ,x>0
1/2
that a score of 40 was wrongly read as 50. The correct mean ïî ( x + 1) - 1
and S.D. respectively are
(a) 14.98, 39.95 (b) 39.95, 14.98 If f (x) is continuous at x = 0, then find the values of a, b and c.
2 2
(c) 39.95, 224.5 (d) None of these (a) a = loge , b = , c = 1
18. At a certain conference of 100 people, there are 29 Indian 3 3
women and 23 Indian men. Of these Indian people 4 are 3 2
(b) a = loge , b = , c = 1
doctors and 24 are either men or doctors. There are no foreign 2 3
doctors. How many foreigners and women doctors are 2 2
attending the conference? (c) a = loge , b = ,c= 2
(a) 48, 1 (b) 34, 3 (c) 46, 4 (d) 42, 2 3 3
2 3
(d) a = loge , b = ,c= 1
é cos q sin q ù 1 3 2
19. If A = ê - sin q cos q ú then lim An is
ë û n ®¥ n æ bö
23. The cost of running a bus from A to B, is ` ç av + ÷ , where
(a) a null matrix (b) an identity matrix è vø
é 0 1ù v km/h is the average speed of the bus. When the bus travels
(c) ê -1 0 ú (d) None of these at 30 km/h, the cost comes out to be ` 75 while at 40 km/h, it
ë û is ` 65. Then the most economical speed (in km/ h) of the
bus is :
20. If exp{(sin 2 x + sin 4 x + sin 6 x + .....¥) log e 2} satisfies
(a) 45 (b) 50 (c) 60 (d) 40
the equation x 2 -9x +8 = 0, then the value of ln 3
x sin x 2
cos x p 24. The value of ò sin x 2 + sin(ln6 - x 2 )
dx is
, 0 < x < is ln 2
cos x + sin x 2
1 3 1 3 3 1 3
3 -1 3 +1 (a) ln (b) ln (c) ln (d) ln
(a) 3 +1 (b) 3 - 1 (c) (d) 4 2 2 2 2 6 2
2 2
x2 y2
n r 25. The area common to the ellipse + = 1 and
21. The limit of the series å as n approaches a2 b2
2 4
r =1 1 + r + r
infinity, is x2 y2
1 1 1 + = 1 , 0 < b < a is
(a) (b) (c) (d) 1 b2 a2
2 4 3
b a
(a) ( a + b )2 tan -1 (b) ( a + b )2 tan -1
a b
b a
(c) 4ab tan -1 (d) 4ab tan -1
a b

RESPONSE 16. 17. 18. 19. 20.


GRID 21. 22. 23. 24. 25.

Space for Rough Work


EBD_7504
MT-4 NTA JEE Main

26. The solution of the differential equation r r r


28. If a, b & c are three non-coplanar non-zero vectors, then
dy rr r r rr r r rr r r
( x 2 - yx 2 ) + y 2 + xy 2 = 0 is : (a.a ) (b ´ c ) + (a.b ) (c ´ a ) + (a.c ) (a ´ b ) is equal to
dx rrr r rrr r
(a) [b c a] a (b) [c a b ] b
æ xö 1 1 æ yö 1 1
(a) log ç ÷ = + + c (b) log ç ÷ = + + c rrr r
è yø x y è xø x y (c) [a b c ] c (d) None of these
29. The direction cosines l, m, n, of one of the two lines
1 1 1 1 connected by the relations
(c) log( xy ) = + + c (d) log( xy ) + + = c
x y x y l - 5m + 3n = 0, 7l 2 + 5m2 - 3n2 = 0 are
27. Let p, q, r be three mutually perpendicular vectors of the 1 2 3 -1 2 3
(a) , , (b) , ,
14 14 14 14 14 14
same magnitude. If a vector x satisfies the equation
1 -2 3 1 2 -3
(c) , , (d) , ,
p ´ {( x - q ) ´ p} + q ´ {( x - r )) ´ q} 14 14 14 14 14 14
+ r ´ {( x - p) ´ r} = 0 then x is given by 30. 6 coins are tossed together 64 times. If throwing a head is
considered as a success then the expected frequency of at
1 1 least 3 successes is
(a) ( p + q - 2 r) (b) ( p + q + r)
2 2 (a) 64 (b) 21 (c) 32 (d) 42
1 1
(c) ( p + q + r) (d) (2 p + q - r )
3 3

RESPONSE
26. 27. 28. 29. 30.
GRID

SUBJECTWISE SPEED TEST-96 - MATHEMATICS


Total Questions 30 Total Marks 120
Attempted Correct
Incorrect Net Score
Cut-off Score 45 Qualifying Score 60
Success Gap = Net Score – Qualifying Score
Net Score = (Correct × 4) – (Incorrect × 1)
Space for Rough Work
MATHEMATICS Speed
Full Syllabus Test-2 TEST
No. of Questions
30
Maximum Marks
120
Time
1 Hour
97
Subject-wise

GENERAL INSTRUCTIONS
• This test contains 30 MCQ's. For each question only one option is correct. Darken the correct circle/ bubble in the
Response Grid provided on each page.
• You have to evaluate your Response Grids yourself with the help of solutions provided at the end of this book.
• Each correct answer will get you 4 marks and 1 mark shall be deduced for each incorrect answer. No mark will be given/
deducted if no bubble is filled. Keep a timer in front of you and stop immediately at the end of 60 min.
• The sheet follows a particular syllabus. Do not attempt the sheet before you have completed your preparation for that
syllabus.
• After completing the sheet check your answers with the solution booklet and complete the Result Grid. Finally spend time
to analyse your performance and revise the areas which emerge out as weak in your evaluation.

x - ai p
1. If A i = , i = 1, 2, ...., n and if a 1 < a2 < a3 < .... < an. (a) b (b) -b
x - ai 2
lim (c) p–b (d) – b
Then x ®a m (A1 A2 .... An) 1 £ m £ n
x
(a) is equal to (–1)m 4. If Im = ò
1
( log x )m dx satisfies the relation Im = k – lIm–1,
(b) is equal to (–1)m + 1
then
(c) is equal to (–1)m –1
(a) k = e (b) l = m
(d) does not exist
2. If a variable takes values 0, 1, 2,..., n with frequencies 1
(c) k= (d) none of these
e
n n ( n - 1) n - 2 2
q n , q n -1p, q p ,..., p n , where p + q = 1, then 1/ n
1 1.2 éæ
the mean is
1 ö æ 22 ö æ n 2 ö ù
5. The value of lim êç 1 + 2 ÷ ç 1 + 2 ÷ ... ç 1 + 2 ÷ ú is
(a) np (b) nq n ®¥ ëêè n ø è n ø è n ø ûú
(c) n(p + q) (d) none of these e p /2
3. If x1, x2, x3, x4 are roots of the equation (a) (b) 2 e2 ep/2
2 e2
x4 – x3 sin 2 b + x2 cos 2 b – x cos b – sin b = 0.
then tan–1 x1 + tan–1 x2 + tan–1 x3 + tan–1 x4 = 2
(c) 2
ep /2 (d) none of these
e

RESPONSE GRID 1. 2. 3. 4. 5.

Space for Rough Work


EBD_7504
MT-6 NTA JEE Main

11. If z1, z2 and z3 are complex numbers such that


dy y ( x - y In y )
6. Solution of the differential equation = is
dx x ( x In x - y ) 1 1 1
z1 = z2 = z3 = + + = 1, then z1 + z2 + z3 is
z1 z2 z3
x In x + y In y x In x - y In y
(a) = c (b) =c
xy xy (a) equal to 1 (b) less than 1
(c) greater than 3 (d) equal to 3
In x In y In x In y
(c) + =c (d) - =c 12. Assume that A i (i = 1, 2, ........, n) are the vertices of a
x y x y
regular polygon inscribed in a circle of radius unity then
7. If the line y = 3x cuts the curve n –1 2
x3 + y3 + 3xy + 5x2 + 3y2 + 4x + 5y – 1 = 0 at the points A, B, the value of å A1A i + 1 , is
C then, OA. OB. OC is i =1

4 (a) 2n (b) n
(a) ( 3 3 - 1) (b) 3 3 + 1 (c) – n (d) – 2n
13
13. If n C r -1 + n +1C r -1 + n + 2 C r -1 + .... + 2n C r -1
2
(c) +7 (d) none of these
3 2 n +1
A die is thrown 2n + 1 times, n Î N. The probability that = C - n C r , then the value of r and the minimum
8. r 2 -132
faces with even numbers show odd number of times is value of n are
2n + 1 1 (a) 10 (b) 11
(a) (b) less than
2n + 3 2
(c) 12 (d) 13
1
(c) greater than (d) none of these n -1
Let S = å
2 k +2
14. P2 , then
x 3 x k =0
9. Period of the function sin + cos5 is :
2 5 (a) n divides 3S (b) n + 1 divides 3S
(a) 2 p (b) 10 p (c) n + 2 divides 3S (d) All are correct
(c) 8 p (d) 5 p 15. The sum of each of two sets of three terms in A.P. is 15. The
common difference of the first set is greater than that of the
10. If the point (a, b) on the curve y = x is closest to the
second by 1 and the ratio of the products of the terms in the
point (1, 0), then the value of ab is first set and that of the second set is 7 : 8. The ratio of the
1 2 smallest terms in two sets of terms is
(a) (b)
2 2 3 11 2 1
(a) or (b) or
1 2 4 12 3 2
(c) (d)
4 4 2 3
(c) or (d) none
3 4

RESPONSE 6. 7. 8. 9. 10.
GRID 11. 12. 13. 14. 15.

Space for Rough Work


Mathematics MT-7

16. It two arithmetic means A1, A2, two geometric means G1,G2, 19. Which of the following is wrong ?
and two harmonic means H1,H2, be inserted between two (a) p ® q is logically equivalent to ~ p Ú q
G1G 2 (b) If the truth values of p, q, r are T, F, T respectively, then
given numbers then = the truth value of (p Ú q) Ù (q Ú r) is T
H1H 2
(c) ~ (p Ú q Ú r) @ ~ p Ù ~ q Ù ~ r
A1 + A 2 H1 + H 2
(a) (b) (d) The truth value of p Ù ~ (p Ú q) is always T.
H1 + H 2 G1 + G 2
20. n letters to each of which corresponds on addressed
G1 + G 2 envelope are placed in the envelope at random. Then the
(c) (d) None of these
A1 + A 2 probability that n letter is placed in the right envelope, will
17. P is a point on either of the two lines y - 3 | x |= 2 at a be :
distance of 5 units from their point of intersection. The 1 1 1 1 1
(a) – + – + ....(–1) n
coordinates of the foot of the perpendicular from P on the 1! 2! 3! 4! n!
bisector of the angle between them are 1 1 1 1 1
(b) + + – + ....
æ 4+5 3 ö æ 4-5 3 ö 2! 3! 4! 5! n!
ç 0, ÷ ç 0, ÷
(a) ç 2 ÷ or ç 2 ÷ø depending on which the
è ø è 1 1 1 1 n 1
(c) – + – + ....( -1)
point P is taken 2! 3! 4! 5! n!
æ 4+5 3 ö (d) None of these
ç 0, ÷
(b) ç 2 ÷ø
è 21. Let A = {x : x Î R, x < 1}; B = {x : x Î R, x - 1 ³ 1} and
æ 4-5 3 ö A È B = R - D, then the set D is
ç 0, ÷
(c) ç 2 ÷ø
è (a) {x :1 < x £ 2} (b) {x :1 £ x < 2}
æ5 5 3ö (c) {x :1 £ x £ 2} (d) None of these
ç , ÷
(d) ç2 2 ÷
è ø 22. The domain of ; f(x) = cos(sin x ) + log x {x} ; {.} denote
18. Let P (a sec q , b tanq) and Q (a sec f, b tan f), where the fractional part, is
x2 y2 (a) [1, p) (b) ( 0, 2p ) - [1, p)
q + f = p / 2, be two points on the hyperbola 2
- 2
= 1. If
a b
(h, k) is the point of intersection of the normals at P and Q, æ pö
(c) çè 0, ÷ø - {1} (d) (0, 1)
then k is equal to 2

a 2 + b2 æ a 2 + b2 ö é1 - 1ù éa 1 ù
(a) (b) –ç ÷ 23. If A = ê ú ,B= ê ú and
a è a ø ë2 1 û ëb -1û
(A + B)2 = A2 + B2 + 2AB, then values of a and b are
æ a 2 + b2 ö (a) a = 1, b = –2 (b) a = 1, b = 2
a 2 + b2 –ç (c) a = –1, b = 2 (d) a = –1, b = –2
(c) (d) ÷
b è b ø

RESPONSE 16. 17. 18. 19. 20.


GRID 21. 22. 23.

Space for Rough Work


EBD_7504
MT-8 NTA JEE Main

24. The integral value(s) of a for which the equation


27. LL' is the latus rectum of the parabola y 2 = 36x and PP' is
2 2 2 2 double ordinate drawn between the vertex and the latus
( x + x + 2) - (a - 3)( x + x + 2)(x + x + 1)
rectum. The area of the trapezium PP'L'L is maximum when
+ (a - 4)(x 2 + x + 1) 2 = 0 the distnace of PP' from the vertex is
has at least one real root is /are (a) 1 (b) 4
(c) 9 (d) 36
(a) 5 (b) 6
(c) 4 (d) none of these 28. The area bounded by the curve y 2 = 4a 2 ( x - 1) and the
lines x = 1 and y = 4a is equal to :
ìï51/ x , x < 0 16 a
25. Let f (x) = í and l Î R, then at x = 0 (a) sq unit (b) 5a sq unit
ïîl[x], x ³ 0 3
17 a
(a) f is discontinuous (c) sq unit (d) none of these
4
(b) f is continuous only, if l = 0
(c) f is continuous only, whatever l may be 29. The dot product of a vector with the vectors ˆi + ˆj - 3kˆ ,
(d) None of these ˆi + 3jˆ - 2kˆ and 2iˆ + ˆj + 4kˆ are 0, 5 and 8 respectively. The
26. If s = 2t3 – 6t2 + at + 5 is the distance travelled by a vector is
particle at time t and if the velocity is – 3 when its (a) ˆi + 2ˆj + kˆ (b) - ˆi + 3jˆ - 2kˆ
acceleration is zero, then the value of a is
(a) –3 (b) 3 (c) ˆi + 2ˆj + 3kˆ (d) ˆi - 3jˆ - 3kˆ
(c) 4 (d) – 4 30. A set contains (2n + 1) elements. The number of subsets of
the set which contain at most n element is
(a) 2n (b) 2n + 1
(c) 2 n–1 (d) 22n

RESPONSE 24. 25. 26. 27. 28.


GRID 29. 30.

SUBJECTWISE SPEED TEST-97 - MATHEMATICS


Total Questions 30 Total Marks 120
Attempted Correct
Incorrect Net Score
Cut-off Score 45 Qualifying Score 60
Success Gap = Net Score – Qualifying Score
Net Score = (Correct × 4) – (Incorrect × 1)
Space for Rough Work
MATHEMATICS Speed
Full Syllabus Test-3 TEST
No. of Questions
30
Maximum Marks
120
Time
1 Hour
98
Subject-wise

GENERAL INSTRUCTIONS
• This test contains 30 MCQ's. For each question only one option is correct. Darken the correct circle/ bubble in the
Response Grid provided on each page.
• You have to evaluate your Response Grids yourself with the help of solutions provided at the end of this book.
• Each correct answer will get you 4 marks and 1 mark shall be deduced for each incorrect answer. No mark will be given/
deducted if no bubble is filled. Keep a timer in front of you and stop immediately at the end of 60 min.
• The sheet follows a particular syllabus. Do not attempt the sheet before you have completed your preparation for that
syllabus.
• After completing the sheet check your answers with the solution booklet and complete the Result Grid. Finally spend time
to analyse your performance and revise the areas which emerge out as weak in your evaluation.

1. Equation of straight line ax + by + c = 0 where 3a+4b+ c = 0,


which is at maximum distance from (1, –2), is 2 y z
(a) 3x + y – 17 = 0 (b) 4x + 3y – 24 = 0 4. If x ¹ 2, y ¹ 2, z ¹ 2 and x 2 z = 0 , then the value of
(c) 3x + 4y – 25 = 0 (d) x + 3y – 15 = 0 x y 2
2. The function 2 y z
log(1 + ax) - log(1 - bx) + + =
f (x) = is not defined at x = 0, the 2- x 2- y 2-z
x
value of which should be assigned to f at x = 0, so that it is (a) 1 (b) 0 (c) 3 (d) 4
continuous at x = 0, is : 5. Box contains 2 one rupee, 2 five rupee, 2 ten rupee and 2 twenty
(a) a – b (b) a + b
rupee coin. Two coins are drawn at random simultaneously. The
(c) log a + log b (d) log a – log b
probability that their sum is ` 20 or more, is
3. The solution of the equation 2z = | z | + 2i, where z is a
complex number, is (a) 1/4 (b) 1/2 (c) 3/4 (d) 1/8

3 3 æ 1ö
(a) z= -i (b) z= +i 6. If ç x i , x ÷ , i =1, 2, 3, 4 are four distinct points on a circle,
3 3 è i ø
then the value of x1 . x2 . x3 . x4 is :
3 (a) – 1 (b) 4 (c) 0 (d) 1
(c) z= ±i (d) None of these
3

1. 2. 3. 4. 5.
RESPONSE GRID
6.
Space for Rough Work
EBD_7504
MT-10 NTA JEE Main

q2
dq 501p æ x cos3 x - sin x ö
7. The value of the definite integral, ò = 12. Value of ò esin x ç ÷ dx is
q1
1 + tan q K è cos 2 x ø
1003p p (a) x esin x – esin x sec x + C
where q2 = and q1 = . The value of K equals
2008 2008 (b) x ecos x – esin x sec x + C
(a) 2007 (b) 2006 (c) 2009 (d) 2008 (c) x2 esin x + esin x sec x + C
(d) 2x esin x – esin x tan x + C
x2 y2 æ 1 ö
8. The line lx + my + n = 0 cut the ellipse + = 1 in -1
a 2
b2 13. sin–1 ç ÷ + cot (3) =
è 5 ø
points whose eccentric angle differ by p/2. Then the value p p p p
of a2l2 + b2m2 is- (a)
6
(b) (c)
3
(d)
4 2
(a) 2n2 (b) 2n (c) 2m2 (d) 2m
9. The expansion of (1 + x)n has 3 consecutive terms with 14. If a and b are the real roots of the equation
coefficients in the ratio 1 : 2 : 3 and can be written in the form
nC : nC n
x2 – (k – 2) x + (k2 + 3k + 5) = 0 (k Î R) .
k k+1 : Ck+2. The sum of all possible values of (n + k)
is
Find the maximum and minimum values of (a 2 + b2 ) .
(a) 18 (b) 21 (c) 28 (d) 32
10. The mean and standard deviation of 6 observations are 8 and (a) 18, 50/9 (b) 18, 25/9
4 respectively. If each observation is multiplied by 3, find the (c) 27, 50/9 (d) None of these
new standard deviation of the resulting observations. 15. The sum of the coefficient of all the terms in the expansion
of (2x – y + z)20 in which y do not appear at all while x
(a) 12 (b) 18 (c) 24 (d) 144
appears in even powers and z appears in odd powers is
11. Negation of the statement p ® (q Ù r) is
(a) ~ p ® ~ (q Ú r) (b) ~ p ® ~ (q Ù r) 2 20 - 1 320 - 1
(a) 0 (b) (c) 219 (d)
(c) (q Ù r) ® p (d) p Ù (~ q Ú ~r) 2 2
16. All the five digit numbers in which each successive digit
exceeds is predecessor are arranged in the increasing order.
The (105)th number does not contain the digit
(a) 1 (b) 2
(c) 6 (d) All of these

RESPONSE 7. 8. 9. 10. 11.


GRID 12. 13. 14. 15. 16.

Space for Rough Work


Mathematics MT-11

tan x tan x - sin x sin x 22. 1+ 2.2 + 3.22 + 4.23 + ....+ 100.299 equals-
17. lim equals (a) 99.2100 (b) 100.2100
x ®0 x3. x
(c) 1 + 99.2100 (d) None of these
(a) 1/4 (b) 3/4 (c) 1/2 (d) 1
n é -1 2 2 ù
18. cos3x. sin2x = å a m sin mx is an identity in x. Then - ê 2 -1 2 ú
m =1 23. If k ê ú is an orthogonal matrix then the value of
3 1 êë 2 2 -1úû
(a) a3 = , a2 = 0 (b) n = 5, a1 =
8 4 k is
n 3
(c) å a m = (d) All the above 1 1
4 (a) ± (b) ±
m =1 2 3
r r r
19. If a, b, c are non-coplanar unit vector such that (c) ±2 (d) ±3
r r r 1 r r
a ´ (b ´ c) = (b + c) then the angle between the vectors
2 -1 é æ 1+ x ö ù
r r 24. The first derivative of the function êcos ç sin ÷ + xx ú
a, b is ëê è 2 ø ûú
(a) 3p/4 (b) p/4 (c) p/8 (d) p/2 with respect to x at x = 1 is :
(a) 3/4 (b) 0
(c) 1/2 (d) – 1/2
20. Solve : tanq + tan(q + (p/3)) + tan(q + (2p/3)) = 3
(a) (3n + 1) (p/12), where n ÎI æ οö æ οö
25. The maximum value of sin çç x ∗ ÷÷ ∗ cos çç x ∗ ÷÷ in the
(b) (4n + 1) (p/6), where n ÎI èç 6 ÷
ø èç 6 ÷ø
(c) (4n + 1) (p/12), where n ÎI
æ οö
interval ççç0, ÷÷÷ is attained at:
(d) None of these
21. Let function f : R ® R be defined by f (x) = 2x + sin x for è 2ø
x Î R , then f is ο ο
(a) (b)
(a) one-one and onto 12 6
(b) one-one but not onto ο ο
(c) onto but not one-one (c) (d)
3 2
(d) neither one-one nor onto

RESPONSE 17. 18. 19. 20. 21.


GRID 22. 23. 24. 25.

Space for Rough Work


EBD_7504
MT-12 NTA JEE Main

26. If the substitution x = tan–1 (t) transforms the differential


28. The function f ( x ) = x ( x + 3)e -(1/ 2) x satisfies all the
2
d y dy
equation 2
+ xy + sec 2 x = 0 into a differential conditions of Rolle’s theorem in [-3,0] . The value of c is
dx dx
(a) 0 (b) –1
d2 y dy (c) –2 (d) –3
equation (1 + t 2 ) 2
+ (2t + y tan -1 (t)) = k then k is
dt dt
sin 2 x cos 2 x
equal to 29. The value of ò0 sin -1 t dt + ò0 cos -1 t dt is
(a) –2 (b) 2 (c) –1 (d) 0
27. Let f : R ® R and fn (x) = f (fn–1 (x)) n ³ 2, n Î N, p p
the roots of equation f3(x) f2(x) f (x) – 25f2(x) f (x) + 175 f (x) (a) p (b) (c) (d) 1
2 4
= 375 which also satisfy equation f (x) = x will be
30. The co-ordinates of the point, where the line
(a) 5 (b) 15
(c) 10 (d) Both (a) and (b) x - 6 y +1 z + 3
= = meets the plane x + y –z = 3 is :
-1 0 4
(a) (2, 1, 0 ) (b) (7, –1, 7)
(c) (1, 2 , –6) (d) (5, –1, 1)

RESPONSE 26. 27. 28. 29. 30.


GRID

SUBJECTWISE SPEED TEST-98 - MATHEMATICS


Total Questions 30 Total Marks 120
Attempted Correct
Incorrect Net Score
Cut-off Score 45 Qualifying Score 60
Success Gap = Net Score – Qualifying Score
Net Score = (Correct × 4) – (Incorrect × 1)

Space for Rough Work


MATHEMATICS Speed
Full Syllabus Test-4 TEST
No. of Questions
30
Maximum Marks
120
Time
1 Hour
99
Subject-wise

GENERAL INSTRUCTIONS
• This test contains 30 MCQ's. For each question only one option is correct. Darken the correct circle/ bubble in the
Response Grid provided on each page.
• You have to evaluate your Response Grids yourself with the help of solutions provided at the end of this book.
• Each correct answer will get you 4 marks and 1 mark shall be deduced for each incorrect answer. No mark will be given/
deducted if no bubble is filled. Keep a timer in front of you and stop immediately at the end of 60 min.
• The sheet follows a particular syllabus. Do not attempt the sheet before you have completed your preparation for that
syllabus.
• After completing the sheet check your answers with the solution booklet and complete the Result Grid. Finally spend time
to analyse your performance and revise the areas which emerge out as weak in your evaluation.

1. Let L1 be a straight line passing through the origin and L2 4. The area bounded by the curves y = log x, y= log | x |,
be the straight line x + y = 1. If the intercepts made by the y = | log x | is :
circle x2 + y2 – x + 3y = 0 on L1 and L2 are equal, then which (a) 4 sq. unit (b) 6 sq. unit
of the following equation can represent L1 ? (c) 10 sq. unit (d) None of these
(a) x + 7y = 0 (b) x – y = 0 5. Number of permutations 1, 2, 3, 4, 5, 6, 7, 8 and 9 taken all at
(c) x – 7y = 0 (d) Both (a) and (b) a time are such that the digit. 1 appearing somewhere to the
2. In a group of 1000 people, each people speak either Hindi, left of 2, 3 appearing to the left of 4 and 5 somewhere to the
Bengali or both Hindi and Bengali. There are 750 who can left of 6, is
speak Hindi and 400 who can speak Bengali. Number of (e.g., 815723946 would be one such permutation)
people, who can speak Bengali only ? (a) 9 . 7! (b) 8! (c) 5! . 4! (d) 8! . 4!
6. If the function f : [0, 16] ® R is differentiable. If 0 < a < 1 and
(a) 600 (b) 250 (c) 150 (d) 400
16
3. The equation sin x + x cos x = 0 has at least one root in 1 < b < 2, then ò f (t) dt is equal to
0
æ p ö
(a) çè - , 0÷ø (b) (0, p)
2 (a) 4 [a3f (a4) – b3 f (b4)] (b) 4 [a3f (a4) + b3 f (b4)]
(c) 4 [a4f (a3) + b4 f (b3)] (d) 4 [a2f (a2) + b2 f (b2)]
æ 3p ö æ pö
(c) çè p , ÷ø (d) çè 0, ÷ø
2 2

1. 2. 3. 4. 5.
RESPONSE GRID
6.
Space for Rough Work
EBD_7504
MT-14 NTA JEE Main

7. If a, b, c are in A. P., then the value of 1 x


x +1 x + 2 x + a (b) log | x + 2 | – log (x2 + 4) + sin–1 2 + C
2
x + 2 x +3 x + b 1 x
is:
x +3 x +4 x +c (c) log | x + 2 | – log (x2 + 4) + cos–1 2 + C
2
(a) 3 (b) – 3 x
1
(c) 0 (d) None of these (d) log | x + 2 | – log (x3 + 4) + tan –1 2 + C
2
8. Let ‘a’ denote the root of equation
12. Vertices of a parallelogram taken in order are A (2, –1, 4),
æ 1+ x 2ö B (1, 0, –1), C (1, 2, 3) and D. Distance of the point
cos (cos–1 x) + sin –1 sin ç ÷ = 2 sec–1 (sec x) P (8, 2, –12) from the plane of the parallelogram is
è 2 ø
4 6 32 6
then possible values of [ | 10a | ] where [ . ] denotes the (a) (b)
9 9
greatest integer function will be
(a) 1 (b) 5
16 6
(c) 10 (d) Both (a) and (c) (c) (d) None of these
9
9. The value of the expression r r r
13. Given A = 2iˆ + 3jˆ + 6k,
ˆ B = iˆ + ˆj - 2kˆ and C = iˆ + 2jˆ + kˆ .
sin 2 y 1 + cos y sin y r r r r r
1- + - is equal to - Compute the value of | A ´ [A ´ (A ´ B)].C | .
1 + cos y sin y 1 - cos y
(a) 343 (b) 512 (c) 221 (d) 243
(a) 0 (b) 1 dy
14. Solution of the differential equation x2y–x3 = y 4cos x,
(c) sin y (d) cos y dx
when y(0) = 1 is :
10. If x2 – 2x cos q + 1 = 0, then the value of x2n – 2xn cos nq + 1,
(a) y3 = 3x3 sin x (b) x3 = 3 y3 sin x
n Î N is equal to
(a) cos 2nq (c) x 3 ¹ y3 sin x (d) None of these
(b) sin 2nq 15. Which of the following functions is inverse of itself ?
(c) 0
1- x
(d) some real number greater than 0 (a) f (x) = (b) f (x) = 3log x
1+ x
8
11. Evaluate ò (x + 2)(x 2 + 4) dx (c) f (x) = 3x(x+1) (d) None of these
16. The locus of the centres of the circles which cut the circles
1 x x2 + y2 + 4x – 6y + 9 = 0 and x2 + y2 – 5x + 4y – 2 = 0
(a) log | x + 2 | – log (x2 + 4) + tan –1 2 +C orthogonally is
2
(a) 9x + 10y – 7 = 0 (b) x – y + 2 = 0
(c) 9x – 10y + 11 = 0 (d) 9x + 10y + 7 = 0

RESPONSE 7. 8. 9. 10. 11.


GRID 12. 13. 14. 15. 16.

Space for Rough Work


Mathematics MT-15

17. The sum to infinity of the series 23. Which of the following is always true ?
1 1 1 1 (a) (~p Ú ~ q) º (p Ù q)
+ + + + ....., is equal to
1 1 + 2 1 + 2 + 3 1+ 2 + 3 + 4
(b) (p ® q) º (~q ® ~p)
(a) 3 (b) 1 (c) 2 (d) 3/2 (c) ~ (p ® ~q) º (p Ù ~q)
sin x + log(1 - x) (d) ~ (p « q) º (p ® q) ® (q ® p)
18. lim is equal to -
x®0 x2
(a) 0 (b) 1/2 24. The domain of function f(x) = 2 x - 3x is -
(c) –1/2 (d) Does not exist
(a) (–¥, 0] (b) R
19. A box contains 6 red, 5 blue and 4 white marbles. Four
(c) [0,¥) (d) No value of x
marbles are chosen at random without replacement. The
probability that there is atleast one marble of each colour 25. The value of the expression
among the four chosen, is
æ 1ö æ 1 ö æ 1ö æ 1 ö
çè 1 + ÷ø çè1 + 2 ÷ø + çè 2 + w ÷ø çè 2 + 2 ÷ø
48 44 88 24 w w w
(a) (b) (c) (d)
91 91 91 91
æ 1ö æ 1 ö æ 1ö æ 1 ö
15 + ç3 + ÷ çè 3 + 2 ÷ø + .... + çè n + w ÷ø çè n + 2 ÷ø
æ 3 3ö è wø w w
20. If in the expansion of çè x - 2 ÷ø , the r th term is
x where w is an imaginary cube root of unity, is
independent of x, then r equals- n (n 2 - 2) n (n 2 + 2)
(a) 8 (b) 9 (a) (b)
3 3
(c) 10 (d) None of these
21. From Mean value theorem n (n 2 - 1)
(c) (d) None of these
f(b) – f(a) = (b – a) f ' (x1) where a < x1 < b and 3
26. If s, s' are the length of the perpendicular on a tangent from
1
f(x) = then x equal to: the foci, a, a' are those from the vertices is that from the
x
x2 y2
2ab b-a a+b centre and e is the eccentricity of the ellipse, + =1,
(a) (b) (c) ab (d) a 2
b2
a+b b+a 2
22. The function f(x) = x4 – 62x2 + ax + 9 attains its maximum ss ¢ - c 2
value on the interval [0, 2] at x = 1. Then the value of a is: then =
aa ¢ - c 2
(a) 120 (b) –120
(a) e (b) 1/e (c) 1/e2 (d) e2
(c) 52 (d) None of these.

RESPONSE 17. 18. 19. 20. 21.


GRID 22. 23. 24. 25. 26.

Space for Rough Work


EBD_7504
MT-16 NTA JEE Main

27. One percent of the population suffers from a certain disease.


æ 5x + 1 ö
There is blood test for this disease, and it is 99% accurate, (c) cos ç
è 10x 2 – 3 ÷ø
in other words, the probability that it gives the correct answer
is 0.99, regardless of whether the person is sick or healthy. A (d) None of these
person takes the blood test, and the result says that he has 29. The trace Tr(A) of a 3 × 3 matrix A = (a ij ) is defined by the
the disease. The probability that he actually has the disease, relation Tr(A) = a11 + a22 + a33 (i.e., Tr(A) is sum of the main
is diagonal elements). Which of the following statements
(a) 0.99% (b) 25% (c) 50% (d) 75% cannot hold ?
(a) Tr(kA) = kTr(A) (k is a scalar)
æ 5x + 1 ö dy (b) Tr(A + B) = Tr(A) + Tr(B)
28. If y = f ç ÷ and f¢(x) = cosx, then =
è 10x – 3 ø
2 dx (c) Tr(I3) = 3
(d) Tr(A2) = Tr(A)2
æ 5x + 1 ö d æ 5x + 1 ö
(a) cos ç p /2 n
an
è 10x 2 – 3 ÷ø dx çè 10x 2 – 3 ÷ø å
30. Let a n = ò (1 - sin t)n sin 2t dt then nlim
®¥ 1 n
is equal to
0
5x + 1 æ 5x + 1 ö
(b) cos ç (a) 1/2 (b) 1 (c) 4/3 (d) 3/2
10x 2 – 3 è 10x 2 – 3 ÷ø

RESPONSE
27. 28. 29. 30.
GRID

SUBJECTWISE SPEED TEST-99 - MATHEMATICS


Total Questions 30 Total Marks 120
Attempted Correct
Incorrect Net Score
Cut-off Score 45 Qualifying Score 60
Success Gap = Net Score – Qualifying Score
Net Score = (Correct × 4) – (Incorrect × 1)

Space for Rough Work


Speed
Full Test-1 TEST
No. of Questions
90
Maximum Marks
360
Time
3 Hours
100
Full Syllabus

GENERAL INSTRUCTIONS
• This test contents 120 MCQ’s :
– Physics (120 marks) : Question No. 1 to 30 are of 4 marks each.
– Chemistry (120 marks) : Question No. 31 to 60 are of 4 marks each.
– Mathematics (120 marks) : Question No. 61 to 90 are of 4 marks each.
• Darken the correct circle/ bubble in the Response Grid provided at the end of this test.
• You have to evaluate your Response Grids yourself with the help of solutions provided at
the end of this book.
• Each correct answer will get you 4 marks and 1 mark shall be deduced for each incorrect answer. No mark will be given/
deducted if no bubble is filled. Keep a timer in front of you and stop immediately at the end of 3 hours.
• After completing the sheet check your answers with the solution booklet and complete the Result Grid. Finally spend time
to analyse your performance and revise the areas which emerge out as weak in your evaluation.

PHYSICS 5GmM 2GmM


(a) (b)
1. A glass prism of refractive index 1.5 is immersed in water 6R 3R
4
(refractive index ) as shown in figure. A light beam incident GmM 3GmM
3
normally on the face AB is totally reflected to reach the face (c) (d)
2R 2R
BC, if
3. A uniform wire of length l and radius r has a resistance of
5 B A r
(a) sin q ³ q 100 W. It is recast into a wire of radius . The resistance of
9 2
2 new wire will be :
(b) sin q ³ (a) 1600 W (b) 400 W
3 (c) 200 W (d) 100 W
8
(c) sin q ³ C 4. A piece of wood from a recently cut tree shows 20 decays
9
1 per minute. A wooden piece of same size placed in a museum
(d) sin q ³ (obtained from a tree cut many years back) shows 2 decays
3
2. What is the minimum energy required to launch a satellite of per minute. If half life of C14 is 5730 years, then age of the
mass m from the surface of a planet of mass M and radius R wooden piece placed in the museum is approximately:
in a circular orbit at an altitude of 2R? (a) 10439 years (b) 13094 years
(c) 19039 years (d) 39049 years

Space for Rough Work


EBD_7504
FT-2 NTA JEE Main

5. What is the maximum value of the force F such that the 8. Consider a spherical shell of radius R at temperature T. The
block shown in the arrangement, does not move? black body radiation inside it can be considered as an ideal
U
gas of photons with internal energy per unit volume u =
F V
4 1æUö
1 µ T and pressure p = ç ÷ . If the shell now undergoes
m= 3è V ø
60º 2 3 an adiabatic expansion the relation between T and R is :
m = 3kg
1 1
(a) T µ (b) T µ 3
R R
(c) T µ e–R (d) T µ e–3R
(a) 20 N (b) 10 N (c) 12 N (d) 15 N
6. A particle falls freely near the surface of the earth. Consider 9. If the ratio of the concentration of electrons to that of holes
a fixed point O (not vertically below the particle) on the 7 7
in a semiconductor is and the ratio of currents is ,
ground. Then pickup the incorrect alternative 5 4
(a) The magnitude of angular momentum of the particle then what is the ratio of their drift velocities?
about O is increasing 5 4 5 4
(a) (b) (c) (d)
(b) The magnitude of torque of the gravitational force on 8 5 4 7
the particle about O is decreasing 10. Two particles A and B of equal mass M are moving with the
same speed v as shown in the figure. They collide completely
(c) The moment of inertia of the particle about O is inelastically and move as a single particle C. The angle q
decreasing that the path of C makes with the X-axis is given by:
(d) The magnitude of angular velocity of the particle about Y
O is increasing
7. A thin liquid film formed between a U-shaped wire and a C
light slider supports a weight of 1.5 × 10–2 N (see figure).
The length of the slider is 30 cm and its weight negligible. q
X
The surface tension of the liquid film is
A 45°
30°
(a) 0.0125 Nm–1 B

(b) 0.1 Nm–1 Film

(c) 0.05 Nm–1 3+ 2 3- 2


(a) tanq = (b) tanq =
1- 2 1- 2
(d) 0.025 Nm–1
W 1- 2 1- 3
(c) tanq = (d) tanq =
2(1 + 3) 1+ 2

Space for Rough Work


FULL TEST-1 FT-3

11. The temperature of the two outer surfaces of a composite (a) 400 nm (b) 700 nm
slab, consisting of two materials having coefficients of (c) 500 nm (d) 667 nm
thermal conductivity K and 2K and thickness x and 4x, 14. In the LC circuit, the current in the direction shown and the
respectively, are T2 and T1 (T2 > T1 ) . The rate of heat transfer charges on the capacitor plates have the signs shown. At
this time
C
æ A(T2 - T1 ) K ö I
through the slab, in a steady state is ç
è ÷ø f ,
x
+Q –Q
with f equal to
L
2 x 4x
(a)
3
(a) I is increasing and Q is increasing
1
(b) 2K T1 (b) I is increasing and Q is decreasing
2 K
(c) I is decreasing and Q is increasing
(c) 1 (d) I is decreasing and Q is decreasing
15. A car, starting from rest, accelerates at the rate f through a
1 distance S, then continues at constant speed for time t and
(d)
3 f
then decelerates at the rate to come to rest. If the total
12. The fundamental frequency of a sonometer wire of length l 2
is n0. A bridge is now introduced at a distance of Dl (<< l) distance traversed is 15 S , then
from the centre of the wire. The lengths of wire on the two 1
(a) S = ft 2 (b) S = f t
sides of the bridge are now vibrated in their fundamental 6
modes. Then, the beat frequency nearly is – 1 1 2
(c) S = ft 2 (d) S = ft
(a) n0Dl/l (b) 8n0Dl/l 4 72
16. Figure shows a network of capacitors where the numbers
(c) 2n0Dl/l (d) n0Dl/2l
indicates capacitances in micro Farad. The value of capacitance
13. In Young's double slit experiment shown in figure S1 and S2 C if the equivalent capacitance between point A and B is to be
are coherent sources and S is the screen having a hole at a 1 mF is :
point 1.0mm away from the central line. White light (400 to C 1
700nm) is sent through the slits. Which wavelength passing A
through the hole has strong intensity?
8 6 4

S1 Centre of
screen 2 2 12
0.5mm
S2 Screen
1.0mm B
hole 32 31
(a) mF (b) mF
50cm S 23 23
33 34
(c) mF (d) mF
23 23
Space for Rough Work
EBD_7504
FT-4 NTA JEE Main

17. The position of a projectile launched from the origin at t = 0 (a) 6.25 A, 3.75 V (b) 3.00 A, 5 V
r
( )
is given by r = 40iˆ + 50 ˆj m at t = 2s. If the projectile was (c) 3.75 A, 3.75 V (d) 6.25 A, 6.25 V
launched at an angle q from the horizontal, then q is 21. Consider an optical communication system operating at a
(take g = 10 ms–2) wavelength of 800 nm. Suppose, only 1% of the optical
source frequency is the available channel bandwidth for
2 3
(a) tan -1 (b) tan
-1 optical communication. How many channels can be
3 2 accommodated for transmitting audio signals requiring a
-1 7 -1 4 bandwidth of 8 kHz ?
(c) tan (d) tan
4 5 (a) 4.8 × 108 (b) 48
18. The current voltage relation of a diode is given by
(c) 6.2 × 108 (d) 4.8 × 105
I = (e1000V T
-1) mA, where the applied voltage V is in volts
22. A monoatomic ideal gas is filled in a nonconducting
and the temperature T is in degree kelvin. If a student makes container. The gas can be compressed by a movable
an error measuring ±0.01 V while measuring the current of 5 nonconducting piston. The gas is compressed slowly to
mA at 300 K, what will be the error in the value of current in 12.5% of its initial volume.
mA? Find final temperature of the gas if it is T0 initially –
(a) 0.2 mA (b) 0.02 mA
(a) 4T0 (b) 3T0
(c) 0.5 mA (d) 0.05 mA
(c) 2/3 T0 (d) T0
19. A thin horizontal circular disc is rotating about a vertical
axis passing through its centre. An insect is at rest at a point 23. A point charge +Q is positioned at the center of the base of
near the rim of the disc. The insect now moves along a a square pyramid as shown. The flux through one of the
diameter of the disc to reach its other end. During the journey four identical upper faces of the pyramid is –
of the insect, the angular speed of the disc
(a) continuously decreases
(b) continuously increases
(c) first increases and then decreases
(d) remains unchanged
20. An ideal ammeter (zero resistance) and an ideal voltmeter
+Q
(infinite resistance) are connected as shown. The ammeter
and voltmeter reading for R1 = 5 W, R2 = 15 W, R3 = 1.25 W
and E = 20 V are given as
Q Q
(a) 16e 0 (b) 4e
R1=5W 0

R2=15W
3 1.25 W
R= Q
(c) 8e 0 (d) None of these

E = 20 V

Space for Rough Work


FULL TEST-1 FT-5

24. A jar is filled with two non-mixing liquids 1 and 2 having 27. Currents of a 10 ampere and 2 ampere are passed through
densities r1 and, r2 respectively. A solid ball, made of a two parallel thin wires A and B respectively in opposite
material of density r3 , is dropped in the jar. It comes to directions. Wire A is infinitely long and the length of the
equilibrium in the position shown in the figure.Which of wire B is 2 m. The force acting on the conductor B, which is
the following is true for r1, r1and r3? situated at 10 cm distance from A will be
(a) r3 < r1 < r2 (a) 8 × 10–5 N (b) 5 × 10–5 N
r1

(c) 8p × 10–7 N (d) 4p × 10–7 N


(b) r1 > r3 > r2 r3
28. An insect crawls up a hemispherical surface very slowly. The
coefficient of friction between the insect and the surface is 1/
(c) r1 < r2 < r3
3. If the line joining the centre of the hemispherical surface to
(d) r1 < r3 < r2 the insect makes an angle a with the vertical, the maximum
possible value of a so that the insect does not slip is given
25. A point particle of mass 0.1 kg is executing SHM of amplitude by
of 0.1m. When the particle passes through the mean position,
its kinetic energy is 18 × 10–3 J. The equation of motion of
this particle when the initial phase of oscillation is 45° can a
be given by –

æ pö
(a) y = 0.1 cos ç 6t + ÷ (a) cot a = 3 (b) sec a = 3
è 4ø
(c) cosec a = 3 (d) cos a = 3
æ pö
(b) y = 0.1 sin ç 6t + ÷
è 4ø 29. The length of a magnet is large compared to its width and
æ pö breadth. The time period of its oscillation in a vibration
(c) y = 0.4 sin ç t + ÷ magnetometer is 2s. The magnet is cut along its length into
è 4ø
three equal parts and these parts are then placed on each
æp ö
(d) y = 0.2 sin ç + 2t ÷ other with their like poles together. The time period of this
è2 ø
26. What happens when the applied load increases and upto combination will be
breaking stress in the experiment to determine the Young's 2
modulus of elasticity ? (a) 2 3s (b) s
3
(a) The area of wire goes on decreasing and wire extends 2
and breaks. (c) 2 s (d) s
3
(b) The area of wire goes on increasing and wire breaks.
30. Hot water cools from 60°C to 50°C in the first 10 minutes and
(c) The wire extends and area remains constant. to 42°C in the next 10 minutes. The temperature of the
(d) The area remains same and wire length is also same. surroundings is:
(a) 25°C (b) 10°C
(c) 15°C (d) 20°C

Space for Rough Work


EBD_7504
FT-6 NTA JEE Main

CHEMISTRY 35. At 675K, H2(g) and CO2(g) react to form CO(g) and H2O (g),
Kp for the reaction is 0.16. If a mixture of 0.25 mole of H2(g)
31. Let n1 be the frequency of the series limit of the Lyman series,
and 0.25 mol of CO2 is heated at 675K, mole % of CO(g) in
n2 be the frequency of the first line of the Lyman series, and n3
be the frequency of the series limit of the Balmer series, then – equilibrium mixture is –
(a) 7.14 (b) 14.28
1
(a) n3 = (n – n ) (b) n2 – n1 = n3 (c) 28.57 (d) 33.33
2 1 3
36. Choose the correct option for
(c) n1 – n2 = n3 (d) n1 + n2 = n3
Anhydrous
¾¾¾¾® I
HI
O | Conc.
OH ¾¾¾¾® II
32. ¾¾® HI
N D
(a) I and II are identical
Number of a-H in alkene which is major product in this (b) I and II are different
reaction is – (c) Mechanism of formation of I and II are not known
(a) 2 (b) 4 (d) None of these
(c) 5 (d) 7 37. Reducing the pressure from 1.0 atm to 0.5 atm would change
33. Match List I with List II and select the correct answer : the number of molecules in one mole of ammonia to
List I (Ions) List II (Shapes)
(a) 25% of its initial value
A. ICl-2 1. Linear (b) 50% of its initial value
B. BrF2+ 2. Pyramidal (c) 75% of its initial value
(d) None of the above
C. ClF4- 3. Tetrahedral 38. Structure of some important polymers are given. Which one
represents Buna-S?
D. AlCl-
4 4. Square planar
CH 3
5. Angular |
A B C D (a) (–C H2 – C = CH – C H2 –)n
(a) 1 2 4 5 (b) (–CH2 – CH = CH – CH2 – CH – CH2 –)n
(b) 4 5 2 3 |
(c) 1 2 4 3 C6H5
(d) 5 1 3 4 (c) (–CH2 – CH = CH – CH2 – CH – CH2 –)n
34. Which one of the following is the correct statement? |
CN
(a) Boric acid is a protonic acid.
Cl
(b) Beryllium exhibits coordination number of six. |
(c) Chlorides of both beryllium and aluminium have (d) (–CH2 – C = CH – CH2 –)n
bridged chloride structures in solid phase.
(d) B2H6.2NH3 is known as ‘inorganic benzene’.

Space for Rough Work


FULL TEST-1 FT-7

39. In electrolysis of NaCl when Pt electrode is taken, then H2 is 46. Aqueous solution of (M) + (NH4)2S ® yellow ppt (B)
liberated at cathode while with Hg cathode it forms sodium (NH ) S
¾¾¾¾¾4 2 2 ® insoluble.
amalgam. This is because
(a) Hg is more inert than Pt The cation present in (M) is –
(b) more voltage is required to reduce H+ at Hg than at Pt (a) CdS (b) SnS2
(c) Na is dissolved in Hg while it does not dissolve in Pt 2+
(c) Cd (d) Sn2+
(d) conc. of H+ ions is larger when Pt electrode is taken
40. In the spinel structure, oxides ions are cubical-closest 47. Both geometrical and optical isomerisms are shown by
packed whereas 1/8th of tetrahedral voids are occupied by (a) [Co(en)2Cl2]+ (b) [Co(NH3)5Cl]2+
(c) [Co(NH3)4Cl2] + (d) [Cr(ox)3]3–
A2+ cation and 1/2 of octahedral voids are occupied by B3+
cations. The general formula of the compound having spinel 48. The instantaneous rate of disappearance of MnO4– ion in
structure is:
the following reaction is
(a) A2B2O4 (b) AB2O4
(c) A2B4O2 (d) A4B2O2 4.56 × 10–3 Ms–1
41. Which of the following option is having maximum number 2MnO4– + 10I– + 16H+ ® 2Mn2+ + 5I2 + 8H2O
of unpaired electrons –
(a) A tetrahedral d6 ion The rate of appearance I2 is :
(b) [Co(NH3)6]3+ (a) 4.56 × 10–4 Ms–1 (b) 1.14 × 10–2 Ms–1
(c) A square planar d7 ion
(d) A co-ordination compound with magnetic moment of (c) 1.14 × 10–3 Ms–1 (d) 5.7 × 10–3 Ms–1
5.92 B.M. 49. Which of the following xenon-oxo compounds may not be
42. The value of P° for benzene is 640 mm of Hg. The vapour
obtained by hydrolysis of xenon fluorides?
pressure of solution containing 2.5gm substance in 39gm
benzene is 600mm of Hg the molecular mass of X is – (a) XeO2F2 (b) XeOF4
(a) 65.25 (b) 130 (c) XeO3 (d) XeO4
(c) 40 (d) 80
aq. KOH
43. Which of the following carbide does not release any 50. Me – CH = CH2 + CHCl3 ¾¾¾¾ ® A (Major products) is –
excess, D
hydrocarbon on reaction with water.
(a) SiC (b) Be2C O
(c) CaC2 (d) Mg2C3
44. An ether (A), C5H12O, when heated with excess of hot (a) OK
concentrated HI produced two alkyl halides which when
treated with NaOH yielded compounds (B) and (C). Oxidation OK
(b)
of (B) and (C) gave a propanone and an ethanoic acid
respectively. The IUPAC name of the ether (A) is: O
(a) 2-ethoxypropane
(b) ethoxypropane
(c) methoxybutane (c)
(d) 2-methoxybutane
O
45. For the reaction taking place at certain temperature NH4HS
ƒ NH3 (g) + H2S (g), if equilibrium pressure is X bar, then
DG° would be –
(a) – 2 RT ln X (b) – RT ln (X – ln 2)
(d) OH
(c) – 2RT (ln X – ln 2) (d) None of these
OH

Space for Rough Work


EBD_7504
FT-8 NTA JEE Main

51. The decomposition of N2O5 in carbon tetrachloride was End product (B) of above reaction is –
followed by measuring the volume of O2 gas evolved :
2N 2 O5 ( CCl4 ) ® 2N2 O4 ( CCl 4 ) + O 2 ( g ) . CHO
CO2 K
The maximum volume of O2 gas obtained was 100 cm3. In
500 minutes, 90 cm3 of O2 were evolved. The first order rate (a) CH2 OH (b)
constant (in min–1) for the disappearance of N2O5 is : CH2OH

2.303 2.303 100


(a) (b) log
500 500 90
CH2OH CO2 K
2.303 90 100
(c) log (d)
500 100 10 ´ 500 (c) (d)
52. The basic character of the transition metal monoxides CH2 OH CO2 K
follows the order
(Atomic Nos.,Ti = 22, V = 23, Cr = 24, Fe = 26)
56. Precautions to be taken in the study of reaction rate for the
(a) TiO > VO > CrO > FeO reaction between potassium iodate (KIO3) and sodium
(b) VO > CrO > TiO > FeO sulphite (Na2SO3) using starch solution as indicator at
(c) CrO > VO > FeO > TiO different concentrations and temperature –
(d) TiO > FeO > VO > CrO (a) The concentration of sodium thiosulphate solution
should always be less than the concentration of the
53. NH4ClO4 + HNO3 (dilute) ¾® X + HClO4
potassium iodide solution.
X ¾¾¾
heat
® Y (gas) Gas (Y) is – (b) Freshly prepared starch solution should be used.
(c) Experiments should be performed with the fresh
(a) O2 (b) N2 solutions of H2O2 and KI.
(c) NO2 (d) N2O (d) All of these
54. Which of the following is/are formed when ozone reacts 57. Predict the product (A) of the following reaction
with the unburnt hydrocarbons in polluted air ?
(i) Formaldehyde (ii) Acrolein
(iii) Peroxyacetyl nitrate (iv) Formic acid H+
(a) (i) and (iv) (b) (ii) only A
D
(c) (iii) only (d) (i), (ii) and (iii)

(a) (b)

O3 Conc. KOH
55. ¾¾® (A) ¾¾¾¾¾ ® (B) (c) (d)
Zn (1 mole)

Space for Rough Work


FULL TEST-1 FT-9

58. A gas undergoes change from state A to state B. In this 64. The area above the x–axis enclosed by the curves x2–y2 = 0
process, the heat absorbed and work done by the gas is 5 J and x2 + y – 2 = 0 is
and 8 J, respectively. Now gas is brought back to A by
another process during which 3 J of heat is evolved. In this 5 7
(a) (b)
reverse process of B to A : 3 3
(a) 10 J of the work will be done by the gas.
(b) 6 J of the work will be done by the gas. 8 10
(c) (d)
(c) 10 J of the work will be done by the surrounding on 3 3
gas. 65. Number of permutations 1, 2, 3, 4, 5, 6, 7, 8 and 9 taken all at
(d) 6 J of the work will be done by the surrounding on gas. a time are such that the digit. 1 appearing somewhere to the
59. In the given molecule choose the correct order of their angle. left of 2, 3 appearing to the left of 4 and 5 somewhere to the
O left of 6, is
(e.g., 815723946 would be one such permutation)
y C x
(a) 9 . 7! (b) 8!
H z O–H (c) 5! . 4! (d) 8! . 4!
(a) x > y > z (b) x < y < z 66. If the function f : [0, 16] ® R is differentiable. If
(c) x = y > z (d) Can’t be predicted 16
60. A current of 2.0 A passed for 5 hours through a molten metal
salt deposits 22.2 g of metal (At wt. = 177). The oxidation
0 < a < 1 and 1 < b < 2, then ò f (t) dt is equal to–
0
state of the metal in the metal salt is
(a) 4 [a3f (a4) – b3 f (b4)]
(a) +1 (b) +2
(b) 4 [a3f (a4) + b3 f (b4)]
(c) +3 (d) +4
(c) 4 [a4f (a3) + b4 f (b3)]
MATHEMATICS (d) 4 [a2f (a2) + b2 f (b2)]
61. Let L1 be a straight line passing through the origin and L2 67. Three distinct points P (3u2, 2u3), Q (3v2, 2v3) and R (3w2,
be the straight line x + y = 1. If the intercepts made by the 2w3) are collinear then –
circle x2 + y2 – x + 3y = 0 on L1 and L2 are equal, then which (a) uv + vw + wu = 0 (b) uv + vw + wu = 3
of the following equation can represent L1 ? (c) uv + vw + wu = 2 (d) uv + ww + wu = 1
(a) x + 7y = 0 (b) x – y = 0
68. Let ‘a’ denote the roots of equation
(c) x – 7y = 0 (d) Both (a) and (b)
62. Let a1, a2 and b1, b2 be the roots of ax2 + bx + c = 0 and px2 + æ 1+ x2 ö
qx + r = 0 respectively. If the system of equations a1y + a2z cos (cos–1 x) + sin–1 sin ç –1
÷ = 2 sec (sec x)
= 0 and b1y + b2z = 0 has a non-trivial solution, then è 2 ø
then possible values of [ | 10a | ] where [ . ] denotes the
b2 ac c2 ab
(a) = (b) = greatest integer function will be
q2 pr r2 pq (a) 1 (b) 5
(c) 10 (d) Both (a) and (c)
a2 bc 69. The image of the pair of lines represented by:
(c) = (d) None of these
2 qr
p ax2 + 2hxy + by2 = 0 by the line mirror y = 0 is
63. If f ''( x ) < 0, "x Î (a, b), then f '( x) = 0 occurs (a) ax2 – 2hxy – by2 = 0
(a) exactly once in (a, b) (b) bx2 – 2hxy + ay2 = 0
(b) atmost once in (a, b) (c) bx2 + 2hxy + ay2 = 0
(c) atleast once in (a, b) (d) ax2 – 2hxy + by2 = 0
(d) None of these
Space for Rough Work
EBD_7504
FT-10 NTA JEE Main

70. If x2 – 2x cos q + 1 = 0, then the value of 77. The sum of the first n terms of the series
x2n – 2xn cos nq + 1, n Î N is equal to – 12 + 2.2 2 + 3 2 + 2.4 2 + 5 2 + 2.6 2 + ...
(a) cos 2nq
(b) sin 2nq n(n + 1)2
(c) 0 is when n is even. When n is odd the sum is
(d) some real number greater than 0 2
2
1
x2
1
x3
2
x2
é n(n + 1) ù n2 (n + 1)
71. If I1 = ò 2 dx , I 2 = ò 2 dx , I 3 = ò 2 dx and I 4 = (a) êë 2 úû (b)
0 0
2
1
2
x 3 n(n + 1)2 3n( n + 1)
ò 2 dx then (c) (d)
4 2
1
78. The straight line joining any point P on the parabola y2 =
(a) I 2 > I1 (b) I1 > I 2 4ax to the vertex and perpendicular from the focus to the
(c) I 3 = I 4 (d) I 3 > I 4 tangent at P, intersect at R, then the equation of the locus of
72. Vertices of a parallelogram taken in order are R is –
A (2, –1, 4), B (1, 0, –1), C (1, 2, 3) and D. Distance of the (a) x2 + 2y2 – ax = 0 (b) 2x2 + y2 – 2ax = 0
point P (8, 2, –12) from the plane of the parallelogram is – (c) 2x2 + 2y2 – ay = 0 (d) 2x2 + y2 – 2ay = 0
79. A box contains 6 red, 5 blue and 4 white marbles. Four
4 6 32 6 marbles are chosen at random without replacement. The
(a) (b) probability that there is atleast one marble of each colour
9 9
among the four chosen, is –
16 6
(c) (d) None of these 48 44
9 (a) (b)
r r r 91 91
73. Given A = 2iˆ + 3jˆ + 6k,
ˆ B = iˆ + ˆj - 2kˆ and C = iˆ + 2jˆ + kˆ .
88 24
r r r r r (c) (d)
Compute the value of | A ´ [A ´ (A ´ B)].C | . 91 91
(a) 343 (b) 512 80. If z + 1/z = 2 cos q, then the value of
|(z2n – 1)/(z2n + 1)|
(c) 221 (d) 243 (a) | tan n q | (b) tan n q
74. The value of the definite integral (c) | cot n q | (d) cot n q
3p / 4 81. The two curves x3 – 3xy2 + 2 = 0 and 3x2y – y3 = 2
ò [(1 + x)sin x + (1 - x) cos x ] dx is – (a) cuts at right angle
(b) touch each other
0
p
(a) 2( 2 + 1) (b) 2 +1 (c) cut at an angle
3
(c) 2 2 (d) 0 p
(d) cut at an angle
75. Area of triangle formed by common tangents to the circles 4
x2 + y2 – 6x = 0 and x2 + y2 + 2x = 0 is– 82. If the pair of lines ax 2 + 2 (a + b)xy + by 2 = 0 lie along
(a) 3 3 (b) 2 3 diameters of a circle and divide the circle into four sectors
such that the area of one of the sectors is thrice the area of
(c) 9 3 (d) 6 3 another sector then
76. The locus of the centres of the circles which cut the circles (a) 3a 2 - 10 ab + 3b 2 = 0
x2 + y2 + 4x – 6y + 9 = 0 and 2 2
x2 + y2 – 5x + 4y – 2 = 0 orthogonally is – (b) 3a - 2ab + 3b = 0
(a) 9x + 10y – 7 = 0 (b) x – y + 2 = 0 (c) 3a + 10 ab + 3b 2 = 0
2

(c) 9x – 10y + 11 = 0 (d) 9x + 10y + 7 = 0 (d) 3a 2 + 2ab + 3b 2 = 0


Space for Rough Work
FULL TEST-1 FT-11

83. Which of the following is a contradiction? (a) 0.99% (b) 25%


(a) (p Ù q)Ù ~ (p Ú q) (b) p Ú ( - p Ù q) (c) 50% (d) 75%
(c) (p Þ q) Þ p (d) None of these 88. Set of values of m for which two points P and Q lie on the line
p
sin q + sin 2q y = mx + 8 so that Ð APB = Ð AQB = where A º (– 4, 0),
84. Period of is 2
cos q + cos 2q B º (4, 0) is –
(a) 2p (b) p
2p p (a) ( -¥, - 3) È ( 3, ¥ ) - {-2, 2}
(c) (d)
3 3 (b) [ - 3, - 3] - {-2, 2}
2
85. If 1, w, w ,........ w n-1 are the n roots of unity, then
(c) (-¥, -1) È (1, ¥) - {-2, 2}
(1 - w) (1 - w2 )........(1 - wn-1 ) equals
(a) 0 (b) 2 (d) {- 3, 3}
(c) n (d) n2 89. The trace Tr(A) of a 3 × 3 matrix A = (aij ) is defined by the
æ 16 ö relation Tr(A) = a11 + a22 + a33 (i.e., Tr(A) is sum of the main
86. If the tangent at the point çç 4 cos q, sin q ÷÷ to the ellipse diagonal elements). Which of the following statements cannot
è 11 ø hold ?
16x + 11y = 256 is also a tangent to the circle x2 + y2 – 2x = 15,
2 2
(a) Tr(kA) = kTr(A) (k is a scalar)
then the value of q is
(b) Tr(A + B) = Tr(A) + Tr(B)
p p
(a) ± (b) ± (c) Tr(I3) = 3
4 3
(d) Tr(A2) = Tr(A)2
p p
(c) ± (d) ± p /2 n
an
6 2 90. Let a n = ò (1 - sin t) sin 2t dt then n ®¥ å n is equal
n lim
87. One percent of the population suffers from a certain disease. 1
0
There is blood test for this disease, and it is 99% accurate, to
in other words, the probability that it gives the correct answer
is 0.99, regardless of whether the person is sick or healthy. A (a) 1/2 (b) 1
person takes the blood test, and the result says that he has (c) 4/3 (d) 3/2
the disease. The probability that he actually has the disease,
is –

FULL SYLLABUS SPEED TEST-100


Total Questions 90 Total Marks 360
Attempted Correct
Incorrect Net Score
Cut-off Score 125 Qualifying Score 165
Success Gap = Net Score – Qualifying Score
Net Score = (Correct × 4) – (Incorrect × 1)
Space for Rough Work
EBD_7504
Response Grid
PHYSICS CHEMISTRY MATHEMATICS

1. 31. 61.
2. 32. 62.
3. 33. 63.
4. 34. 64.
5. 35. 65.
6. 36. 66.
7. 37. 67.
8. 38. 68.
9. 39. 69.
10. 40. 70.
11. 41. 71.
12. 42. 72.
13. 43. 73.
14. 44. 74.
15. 45. 75.
16. 46. 76.
17. 47. 77.
18. 48. 78.
19. 49. 79.
20. 50. 80.
21. 51. 81.
22. 52. 82.
23. 53. 83.
24. 54. 84.
25. 55. 85.
26. 56. 86.
27. 57. 87.
28. 58. 88.
29. 59. 89.
30.
Speed
Full Test-2 TEST
No. of Questions
90
Maximum Marks
360
Time
3 Hours
101
Full Syllabus

GENERAL INSTRUCTIONS
• This test contents 120 MCQ’s :
– Physics (120 marks) : Question No. 1 to 30 are of 4 marks each.
– Chemistry (120 marks) : Question No. 31 to 60 are of 4 marks each.
– Mathematics (120 marks) : Question No. 61 to 90 are of 4 marks each.
• Darken the correct circle/ bubble in the Response Grid provided at the end of this test.
• You have to evaluate your Response Grids yourself with the help of solutions provided at
the end of this book.
• Each correct answer will get you 4 marks and 1 mark shall be deduced for each incorrect answer. No mark will be given/
deducted if no bubble is filled. Keep a timer in front of you and stop immediately at the end of 3 hours.
• After completing the sheet check your answers with the solution booklet and complete the Result Grid. Finally spend time
to analyse your performance and revise the areas which emerge out as weak in your evaluation.

q q
PHYSICS (a) ˆj (b) - ˆj
4p 2 e 0 r 2 4p e 0 r 2
2
1. A 1 kg block attached to a spring vibrates with a frequency q ˆj q
of 1 Hz on a frictionless horizontal table. Two springs (c) - (d) ˆj
identical to the original spring are attached in parallel to an 2p e 0 r 2
2
2p e 0 r 2
2

8 kg block placed on the same table. The frequency of 3. The magnetic field due to a current carrying circular loop of
vibration of the 8 kg block is : radius 3 cm at a point on the axis at a distance of 4 cm from
1 1 the centre is 54 mT. What will be its value at the centre of
(a) Hz (b) Hz
4 2 2 loop?
(a) 125 mT (b) 150 mT
1
(c) Hz (d) 2Hz (c) 250 mT (d) 75 mT
2
2. A thin semi-circular ring of radius r has a positive charge q 4. The velocity of projection of a body is increased by 2%,
ur other factors remaining unchanged, what will be the
distributed uniformly over it. The net field E at the centre O percentage change in the maximum height attained?
is
(a) 1 % (b) 2 %
j
(c) 4 % (d) 8 %

i
O Space for Rough Work
EBD_7504
FT-14 NTA JEE Main

Hydrogen ( 1 H ) , Deuterium ( 1 H ) , singly ionised Helium 8. The amplitude of velocity of a particle acted on by a force F
1 2
5.
( 2 He4 ) and doubly ionised lithium ( 3 Li6 ) all have one
+ ++
1
cos w t along the x direction, is given by x =
electron around the nucleus. Consider an electron transition [aw 2 - bw + c]
from n = 2 to n = 1. If the wavelengths of emitted radiation
where a, b, c, are constants and b2 < 4 ac. For what value of
are l1, l 2 , l3 and l 4 respectively then approximately w does the resonance occur ?
which one of the following is correct?
b b
(a) w= (b) w =
(a) 4l1 = 2l 2 = 2l 3 = l 4 2a a
(c) w=c (d) w = 0
(b) l1 = 2l 2 = 2l 3 = l 4
9. Three bars each of area of cross - section A and length L are
(c) l1 = l 2 = 4l 3 = 9l 4 connected in series. The thermal conductivities of their
materials are K, 2K, 1.5K. If the temperatures of the external
(d) l1 = 2l 2 = 3l 3 = 4l 4 ends of the first and last bar are 200 & 18ºC, then the
6. A small object of uniform density rolls up a curved surface temperatures of both the junctions are
with an initial velocity v. It reaches up to a maximum height (a) T1 = 116ºC, T2 = 74ºC
3v2 (b) T1 = 120ºC, T2 = 84ºC
of with respect to the initial position. The object is (c) T1 = 132ºC, T2 = 98ºC
4g
(a) Ring (d) T1 = 164ºC, T2 = 62ºC
10. In the following diagram the reading of the ammeter is (when
(b) Solid sphere
the internal resistance of the battery is zero)
(c) Hollow sphere v
(d) Disc A 10V
7. One end of a massless rope, which passes over a massless
and frictionless pulley P is tied to a hook C while the other 4W
end is free. Maximum tension that the rope can bear is 360 N. 5W
With what value of maximum safe acceleration (in ms-2) can a
man of 60 kg climb on the rope?
40 10
P (a) A (b) A
29 9
C (c) 5 (d) 2 A
A
3

(a) 16 (b) 6 (c) 4 (d) 8

Space for Rough Work


FULL TEST-2 FT-15

11. A 2V battery is connected across AB as shown in the 14. In a uniform magnetic field of induction B a wire in the form
figure. The value of the current supplied by the battery of a semicircle of radius r rotates about the diameter of the
when in one case battery’s positive terminal is connected circle with an angular frequency w. The axis of rotation is
to A and in other case when positive terminal of battery perpendicular to the field. If the total resistance of the circuit
is connected to B will respectively be: is R, the mean power generated per period of rotation is

D1 5W ( B pr w )2 ( B pr 2 w ) 2
(a) (b)
2R 8R

D2 10 W B pr 2 w ( B pr w 2 ) 2
(c) (d)
2R 8R
15. The insulation property of air breaks down at
E = 3 × 106 volts/ metre. The maximum charge that can be
given to a sphere of diameter 5 m is approximately
A B
(a) 2 × 10–2 coulomb (b) 2 × 10–3 coulomb
(a) 0.4 A and 0.2 A (b) 0.2 A and 0.4 A
(c) 2 × 10–4 coulomb (d) 2 × 10–5 coulomb
(c) 0.1 A and 0.2 A (d) 0.2 A and 0.1 A
16. A compound microscope has an objective and eye-piece as
12. If a bar magnet of magnetic moment 80 units be cut into two thin lenses of focal lengths 1 cm and 5 cm respectively. The
halves of equal lengths, the magnetic moment of each half distance between the objective and the eye-piece is 20 cm.
will be The distance at which the object must be placed in front of
(a) 80 units (b) 40 units the objective if the final image is located at 25 cm from the
(c) 160 units (d) 20 units eye - piece, is numerically
13. A thin wire of length L and uniform linear mass density r is 95 95
bent into a circular loop with centre at O as shown. The (a) cm (b) 89 cm
6
moment of inertia of the loop about the axis XX' is
5 25
X X¢ (c) cm (d) cm
rL3 6 6
(a) 90° 17. When light of 3000 Å is incident on sodium chloride, the
8p 2
O stopping potential is 1.85 volt and when light of 4000 Å is
3 incident, then the stopping potential becomes 0.82 volt. The
rL
(b) threshold wavelength for sodium is: (h = 6.6 × 10–34 J-sec.)
16p 2 (a) 5451 Å (b) 5154 Å
(c) 5320 Å (d) 5211 Å
5rL3 18. A steel wire is suspended vertically from a rigid support.
(c) When loaded with a weight in air, it extends by la and when
16p 2
the weight is immersed completely in water, the extension is
reduced to lw. Then the relatively density of material of the
3rL3 weight is
(d)
8p 2 la la
(a) (b)
lw la - l w
lw lw
(c) (d)
la - l w la

Space for Rough Work


EBD_7504
FT-16 NTA JEE Main

19. In four complete revolution of the cap, the distance travelled 23. In diagrams (1 to 4), variation of volume with changing
on the pitch scale is 2 mm. If there are 50 divisions on the pressure is shown. A gas is taken along the path ABCD.
circular scale, then the least count of the screw gauge is The change in internal energy of the gas will be
(a) 0.001 mm (b) 0.01 mm
(c) 0.10 mm (d) 1.0 mm V D V D
C C
20. The x-t graph of a particle undergoing simple harmonic
motion is shown below. The acceleration of the particle at t
= 4 / 3 s is A A
B B

(1) P (2) P
1
x (cm)

0 8 12 t(s) D C
D
4 C

–1
V V
A B A
B
2
3 2 –p
(a) p cm/s 2 (b) cm/s 2 (3) P (4) P
32 32
(a) Positive in all the cases (1) to (4)
p2 3 2 (b) Positive in cases (1), (2), (3) but zero in case(4)
(c) cm/s 2 (d) – p cm/s2
32 32 (c) Negative in cases (1), (2), (3) but zero in case (4)
(d) Zero in all the cases
21. An electromagnetic wave of frequency 1 × 1014 hertz is
24. Three masses m, 2m and 3m are moving in x-y plane with
propagating along z-axis. The amplitude of electric field is 4
speed 3u, 2u and u respectively as shown in figure. The
V/m. If e0 = 8.8 × 10–12 C2/N-m2, then average energy density
three masses collide at the same point at P and stick together.
of electric field will be: The velocity of resulting mass will be:
(a) 35.2 × 10–10 J/m3 (b) 35.2 × 10–11 J/m3
(c) 35.2 × 10 J/m–12 3 (d) 35.2 × 10–13 J/m3 y
22. In a Young’s double slit experiment with light of wavelength 2m, 2u
l the separation of slits is d and distance of screen is D
such that D >> d >> l. If the fringe width is b, the distance 60°
from point of maximum intensity to the point where x
m, 3u P 60°
intensity falls to half of maximum intensity on either side
is: 3m, u

b b
(a)
6
(b)
3 (a)
u ˆ
12
(
i + 3jˆ ) (b)
12
(
u ˆ
i - 3jˆ )
b b
(c) (d)
4 2 (c)
u ˆ
12
(
-i + 3jˆ ) (d)
12
(
u ˆ
-i - 3jˆ )

Space for Rough Work


FULL TEST-2 FT-17

25. A satellite of mass m revolves around the earth of radius R (c) 12.5 mA, 5 mA, 7.5 mA
at a height x from its surface. If g is the acceleration due to (d) 12.5 mA, 7.5 mA, 5 mA
gravity on the surface of the earth, the orbital speed of the 29. The Thallium-201 half-life is 74 hours. If the sample has an
satellite is activity of 80 millicuries initially, what will be the activity
gR 2 gR after 9.25 days ?
(a) (b)
R+ x R-x (a) 2.5 mCi (b) 5 mCi
(c) 10 mCi (d) 20 mCi
æ gR 2 ö 1/ 2 30. An engine approaches a hill with a constant speed. When it
(c) gx (d) ç ÷ is at a distance of 0.9 km, it blows a whistle whose echo is
è R + xø
heard by the driver after 5 seconds. If the speed of sound in
26. The plots of intensity versus wavelength for three black air is 330 m/s, then the speed of the engine is :
bodies at temperatures T1, T2 and T3 respectively are as (a) 32 m/s (b) 27.5 m/s
shown. Their temperature are such that (c) 60 m/s (d) 30 m/s

(a) T1 > T2 > T3 T3 CHEMISTRY


I T1 T2 31. An azeotropic mixture of two liquids boil at a lower
(b) T1 > T3 > T2
temperature than either of them when
(c) T2 > T3 > T1 (a) It is saturated.
l
(b) It does not deviate from Raoult’s law.
(d) T3 > T2 > T1
(c) It shows negative deviation from Raoult’s law.
27. An electric charge 10–3 λ C is placed at the origin (0, 0) of X (d) It show positive deviation from Raoult’s law.
– Y co-ordinate system. Two points A and B are situated at 32. Which of the following statement is wrong ?
(a) Geometrical isomers are diastereomers.
( 2, 2) and (2, 0) respectively. The potential difference
between the points A and B will be (b) Enantiomers have same chemical and
physical properties.
(a) 4.5 volts (b) 9 volts
(c) Conformers are non interconvertible to
(c) Zero (d) 2 volt each other.
28. A Zener diode is connected to a battery and a load as shown
(d) 3-methyl hexane shows optical isomerism.
below:
33. Four species are listed below:
4 kW A IL
i. HCO3– ii. H3 O+
IZ iii. HSO4– iv. HSO3F
I
10 V = VZ RL = 2kW Which one of the following is the correct sequence of their
60 V
acid strength?
(a) iv < ii < iii < i (b) ii < iii < i < iv
B
(c) i < iii < ii < iv (d) iii < i < iv < ii
The currents, I, IZ and IL are respectively.
(a) 15 mA, 5 mA, 10 mA
(b) 15 mA, 7.5 mA, 7.5 mA

Space for Rough Work


EBD_7504
FT-18 NTA JEE Main

34. The enthalpy of combustion of H2(g), to give H2O(g) is –


249 kJ mol–1 and bond enthalpies of H – H and O = O are 433
kJ mol–1 and 492 kJ mol–1 respectively. The bond enthalpy of
O – H is
(a) 464 kJ mol-1 (b) –464 kJ mol-1 (iii)

(c) 232 kJ mol-1 (d) –232 kJ mol-1


35. A certain metal when irradiated to light
(v = 3.2× 1016 Hz) emits photoelectrons with twice kinetic
energy as did photoelectrons when the same metal is
(iv)
irradiated by light (v = 2.0 × 1016 Hz) The v0 (threshold
frequency) of metal is (a) only (i) (b) (i) and (iv)
(a) 1.2 × 1014 Hz (b) 8 × 1015 Hz (c) (i), (iii) and (iv) (d) all the four
(c) 1.2 × 1016 Hz (d) 4 × 1012 Hz 40. The process of the extraction of Au and Ag is based on their
solubility in
36. Malachite has the formula Cu2CO3(OH)2. What percentage
(a) NH3 or NH4OH (b) KCN or NaCN
by mass of malachite is copper?
(c) HCl or HNO3 (d) H2SO4
(a) 25% (b) 50.9% 41. Which of the following compounds having highest and
(c) 57.5% (d) 63.5% lowest melting point respectively ?
37. Which one of the following pairs is mismatched (1) CsF (2) LiF
(a) Fossil fuel burning - release of CO2 (3) HCl (4) HF
(b) Nuclear power - radioactive wastes Correct answer is
(c) Solar energy - Greenhouse effect (a) 2, 3 (b) 1, 4
(d) Biomass burning - release of CO2 (c) 4, 3 (d) 2, 1
38. Which of the following undergoes hydrolysis by SN1 42. Silver bromide when dissolve in hypo solution gives complex
mechanism : ..... in which oxidation state of silver is ....
(a) CH3CH2CH2CH2Cl (b) CH3CH2CH2Cl (a) Na3[Ag(S2O3)2], (I)
(c) CH3 – O – CH2Cl (d) CH3Cl (b) Na3[Ag(S2O3)3], (III)
39. Which of the following hydrocarbon can react with maleic (c) Na3[Ag(S2O3)2], (II)
anhydride ? (d) Na3[Ag(S2O3)4], (I)
43. The freezing point of a solution, prepared from 1.25 g of a
non-electrolyte and 20 g of water, is 271.9 K. If molar
depression constant is 1.86 K mole–1, then molar mass of
the solute will be
(i)
(a) 105.7 (b) 106.7
(c) 115.3 (d) 93.9

(ii)

Space for Rough Work


FULL TEST-2 FT-19

44. Consider the reaction: 49. In the reaction,


H 2SO3 (aq) + Sn 4 + (aq) + H 2O (l )
HOCl B
¾¾ ® Sn 2+ (aq) + HSO-4 (aq) + 3H + (aq) CH 2 < CH 2 ¾¾ ¾↑ A ¾¾
↑ CH 2 OH
|
Which of the following statements is correct? CH2OH
(a) Sn 4+ is the oxidizing agent because it undergoes
oxidation. The molecule ‘A’ and the reagent ‘B’ are
(b) Sn 4+ is the reducing agent because it undergoes (a) H C – CH and hot water
2 2
oxidation. O
(c) H2SO3 is the reducing agent because it undergoes (b) CH3CH2Cl and NaOH
oxidation. (c) CH3CH2OH and H2SO4
(d) H2SO3 is the reducing agent because it undergoes (d) CH 2 – CH2 and NaHCO3
reduction. | |
45. An organic compound (A) reacts with sodium metal and Cl OH
forms (B). On heating with conc. H2SO4, (A) gives diethyl 50. Alkali metals dissolve in liquid NH3 then which of the
following observations is not true?
ether, (A) and (B) are (a) H2 gas is liberated.
(a) C2H5OH and C2H5ONa (b) Solution turns into blue due to solvated
(b) C3H7OH and CH3ONa electrons.
(c) It becomes diamagnetic.
(c) CH3OH and CH3ONa (d) Solution becomes conducting.
(d) C4H9OH and C4H9ONa 51. Which of the following species involves sp3 hybridisation?
(a) SbF6– (b) PCl5
46. Which one of the following esters cannot undergo Claisen
(c) N2Cl4 (d) SiF62–
self-condensation?
52. At very high pressures, the compressibility factor of one
(a) CH 3 - CH 2 - CH 2 - CH 2 - COOC 2 H 5 mole of a gas is given by :
(b) C6H5COOC2H5 Pb Pb
(a) 1 + (b)
(c) C6H5CH2COOC2H5 RT RT
Pb b
(d) C6H11CH2COOC2H5 (c) 1 - (d) 1 -
RT (VRT)
47. In recovery of silver from photographic film, you have 53. How many unit cells are present in a cube-shaped ideal
decided to dissolve the silver ion with dilute nitric acid. crystal of NaCl of mass 1.00 g ?
Addition of dilute HCl to precipitate AgCl seems to result in [Atomic masses : Na = 23, Cl = 35.5]
unacceptable losses. You might improve recovery by addition (a) 5.14 × 1021 unit cells
of_______ in the latter step. (b) 1.28 × 1021 unit cells
(a) NaNO3 (b) NaCl (c) 1.71 × 1021 unit cells
(d) 2.57 × 1021 unit cells
(c) Ag2SO4 (d) sodium acetate 54. The electrode potential of Mg2+/Mg electrode, in which
48. One mole of fluorine is reacted with two moles of hot and conc. of Mg2+ is 0.01M, is
concentrated KOH. The products formed are KF, H2O and ( E o Mg 2 + / Mg = -2.36 V)
O2. The molar ratio of KF, H2O and O2 respectively is
(a) 2.36 V (b) –2.36 V
(a) 1 : 1 : 2 (b) 2 : 1 : 0.5 (c) 2.42 V (d) –2.42 V
(c) 1 : 2 : 1 (d) 2 : 1 : 2

Space for Rough Work


EBD_7504
FT-20 NTA JEE Main

55. Among the following compounds (I-III) the correct order of


reactivity with electrophile is MATHEMATICS
61. It has been found that A and B play a game 12 times, A wins
OCH3 NO2
6 times, B wins 4 times and they draw twice. A and B take
part in a series of 3 games. Find the probability that they will
win alternately.
5 6
(a) (b)
I II III 36 35
(a) II > III > I (b) III < I < II 4 7
(c) (d)
(c) I > II > III (d) I = II > III 35 36
56. One desires to prepare a positively charged sol of silver 3 x x
62. Period of the function sin + cos5 is :
iodide. This can be achieved by 2 5
(a) adding a little AgNO3 solution to KI (a) 2 p (b) 10 p
solution in slight excess. (c) 8 p (d) 5 p
(b) adding a little KI solution to AgNO3 63. The values of the parameter a such that the roots a, b of the
solution in slight excess. a b
(c) mixing equal volumes of equimolar equation 2x2 + 6x + a = 0 satisfy the inequality + < 2 are
b a
solutions of AgNO3 and KI.
(d) none of these. (a) a > 0 (b) a < 9/2
57. Which of the following is an artificial edible colour? (c) a < 0 or a > 9/2 (d) None of these
(a) Melamine (b) Saffron 64. The least difference between the roots, in the first quadrant
(c) Carotene (d) Tetrazine æ pö
ç 0 £ x £ ÷ , of the equation
58. Lassaigne’s test for nitrogen is positive for which è 2ø
compound?
4 cos x (2 - 3 sin 2 x ) + (cos 2x + 1) = 0 is
(a) NH2OH (b) NH2NH2
(c) H2NCONH2 (d) All the three p p
(a) (b)
59. It is always advisable not to keep egg yolk or mustard in 6 4
silver cutlery because p p
(a) silver reacts with water of egg yolk to form AgOH. (c) (d)
3 2
(b) silver reacts with sulphur of egg yolk forming black
Ag2S. y
dt d2y
(c) silver reacts with egg yolk forming Ag2SO4 which is a 65. If x = ò , then is equal to :
0 1+ t2 dx 2
poisonous substance.
(d) silver attracts UV light of the atmosphere, thereby
spoiling the food. (a) y (b) 1+ y 2
60. Which of the following complexes will give white precipitate
with BaCl2 (aq)? x
(c) (d) y 2
(a) [Co(NH3)4SO4]NO2 (b) [Cr(NH3)4SO4]Cl 1+ y 2
(c) [Cr(NH3)5Cl]SO4 (d) Both (b) & (c)

Space for Rough Work


FULL TEST-2 FT-21

66. The number of real roots of the equation 71. Number of ways to distribute 10 distinct balls to 3 persons.
1 + a1 x + a2 x2 + …. + an xn = 0 One getting 2, 2nd getting 3 and 3rd getting 5 balls is
10! 10!
1 (a) (b)
where |x| < and |an| < 2, is 2!.3!.5! 2!.(3!)2 .5!
3
(a) n if n is even 10!
(c) (d) 10!
(b) 0 for any natural number n 2!.5!
(c) 1 if n is odd 72. The general solution of the differential equation,
(d) None of these. æ dy ö
sin 2x ç - tan x ÷ - y = 0 , is :
p è dx ø
- p
67. If e 2 <q< , then (a) y tan x = x + c
2
(a) cos log q > log cos q (b) y cot x = tan x + c
(b) cos log q < log cos q (c) y tan x = cot x + c
(c) cos log q = log cos q
(d) y cot x = x + c
2
(d) cos log q = log cos q sin B
3 73. In any triangle ABC, if cos A = , then
2 sin C
68. If [x] denotes the greatest integer £ x, then (a) a = b = c (b) c = a
(c) a = b (d) b = c
1 74. If w is a cube root of unity, then a root of the following
lim ([12 x] + [ 22 x] + [32 x] + ........+ [ n 2 x]) =
x ®3 n 3
x +1 w w2
x x equation w x + w2 1 =0 is
(a) (b)
2 3 w2 1 x+w
x (a) x = 1 (b) x = w
(c) (d) 0
6 (c) x = w 2 (d) x = 0 .
69. For natural numbers m, n if 75. The lines lx + my + n = 0, mx + ny + l = 0 and
nx + ly + m = 0 are concurrent if
(1 - y )m (1 + y ) n = 1 + a1 y + a2 y 2 + ¼ and (a) l – m – n = 0
a1 = a2 = 10, then (m, n) is (b) l + m – n = 0
(c) l – m + n = 0
(a) (20, 45) (b) (35, 20) (d) l2 + m2 + n2 = lm + mn + nl
(c) (45, 35) (d) (35, 45)
76. If (x, y) are the co-ordinates of a point in the plane , then
s n
3 4 2
70. The value of å å n Cs s Cr is
r=0 s =1 5 8 2 = 0 represent
r £s
x y 2
(a) 3n – 1 (b) 3n +1 (a) a. st. line || to y-axis (b) a st. line || to x-axis
(c) 3n (d) 3(3n – 1) (c) a st. line (d) a circle

Space for Rough Work


EBD_7504
FT-22 NTA JEE Main

r r r r r r r r
77. The equation of the circle whose radius is 5 and which touches 82. The value of [A - B, B - C, C - A] where | A |= 1, | B |= 2
the circle x2 + y2– 2x – 4y–20 = 0 at the point ( 5,5) is r
(a) x2 + y2 + 18 x + 16 y + 120 = 0 and | C |= 3 is
(b) x2 + y2 – 18 x – 16 y + 120 = 0 (a) 1 (b) 6
(c) x2 + y2 – 18 x + 16 y + 120 = 0 (c) 0 (d) 3
(d) x2 + y2 + 18 x – 16 y + 120 = 0 r r
83. If A = ˆi + ˆj + k,
ˆ B = 4iˆ + 3jˆ + 4kˆ and
æ 3ö x2 y 2
78. The normal at ç 2, ÷ to the ellipse, + = 1 touches a r
è 2ø 16 3 C = ˆi + aˆj + b kˆ are linearly independent vectors and
parabola, whose equation is
r
(a) y2 = – 104 x (b) y2 = 14 x C = 3 , then
2
(c) y = 26x (d) y2 = – 14x
79. If p : I study and q : I fail (a) a = 1, b = –1 (b) a = 1, b = ± 1
(c) a = –1, b = ± 1 (d) a = ± 1, b = 1
Then negation of 'I study or I fail' is
(a) I do not study and I do not fail 2 x -3
84. If f(x) = , g (x) = and
(b) I do not study or I do not fail x -3 x+4
(c) Either I study and I do not rail or I study and I do not 2(2 x + 1)
fail h(x) = – , then
x 2 + x - 12
(d) I study and I do not fail
lim [f(x) +g(x) +h(x)] is equal to
80. The expression x ®3
(a) –2 (b) –1
é æ 3p ö ù
3 êsin 4 ç - a ÷ + sin 4 (3p + a )ú 2
ë è 2 ø û (c) - (d) 0
7
85. p : Every quadratic equation has one real root and q : Every
é æp ö ù
-2 êsin 6 ç + a ÷ + sin 6 (5p - a ) ú is equal to quadratic equation has two real roots, then truth value of p
ë è 2 ø û and q are
(a) 0 (b) 1 (a) p is true and q is false
(c) 3 (d) sin 4a + cos6a (b) p is false and q is true
81. The general solution of the trigonometric equation sin x – (c) p and q both true
cos x = 1 is given by (d) p and q both false
86. Let A, B and C be n × n matrices. Which one of the following
(a) x = 2n p, n Î I is a correct statement?
p p (a) If AB = AC, then B = C
(b) x = n p + (–1)n + , n Î I
4 4
(b) If A 3 + 2 A 2 + 3A + 5I = 0 ; then A is invertible.
p (c) If A 2 = 0 , then A = 0
(c) x =2n p + ,n ÎI
2 (d) None of these
p
(d) x = n ,n ÎI
2

Space for Rough Work


FULL TEST-2 FT-23

87. If P an d Q be two given poin ts on th e cur ve 89. If for a real number y, [y] is the greatest integer less than or
1 uuur uuur 3p / 2
y= x + such that OP.iˆ = 1 and OQ.iˆ = -1 where î is a equal to y, then the value of the integral òp / 2 [2 sin x ]dx is
x
unit vector along the x–axis , then the length of vector (a) –p (b) p
2OP + 3OQ is (c) p /2 (d) -p/2
(a) 5 5 (b) 3 5 90. If f(x) = ax2 + b, b ¹ 0, x £ 1;
= bx2 + ax + c, x > 1,
(c) 2 5 (d) 5
then f(x) is continuous and differentiable at x = 1 if
88. The area bounded by parabola y2 = x, st. line
y = 4 and y-axis is (a) c = 0, a = 2b (b) a = b, c arbitrary (c)
16 64 a = b, c = 0 (d) a = b, c ¹ 0
(a) (b)
3 3
(c) 7 2 (d) None

FULL SYLLABUS SPEED TEST-101


Total Questions 90 Total Marks 360
Attempted Correct
Incorrect Net Score
Cut-off Score 125 Qualifying Score 165
Success Gap = Net Score – Qualifying Score
Net Score = (Correct × 4) – (Incorrect × 1)
Space for Rough Work
EBD_7504
Response Grid
PHYSICS CHEMISTRY MATHEMATICS

1. 31. 61.
2. 32. 62.
3. 33. 63.
4. 34. 64.
5. 35. 65.
6. 36. 66.
7. 37. 67.
8. 38. 68.
9. 39. 69.
10. 40. 70.
11. 41. 71.
12. 42. 72.
13. 43. 73.
14. 44. 74.
15. 45. 75.
16. 46. 76.
17. 47. 77.
18. 48. 78.
19. 49. 79.
20. 50. 80.
21. 51. 81.
22. 52. 82.
23. 53. 83.
24. 54. 84.
25. 55. 85.
26. 56. 86.
27. 57. 87.
28. 58. 88.
29. 59. 89.
30.
HINTS & SOLUTIONS (PHYSICS – Chapter-wise Tests)
Speed Test-1
1. (a) A and B have different dimensions. Hence, only A and æ 4 ö
B in a ratio form and is meaningful. çè ÷
100 ø (100g)
2. (d) No of divisions on main scale = N = 3 3
No of divisions on vernier scale = N + 1 æ 1 ö (10cm)
çè ÷ø
size of main scale division = a 10
Let size of vernier scale division be b 4 100g
= ´ (10)3 ·
then we have 100 (10cm) 3
aN = 40 unit
aN = b (N + 1) Þ b =
N +1 8. (d) For angular momentum, the dimensional formula is
aN [ML2T–1]. For other three, it is [ML2T–2].
Least count is a – b = a –
N +1 9. (d) Let X = ML–1 T–1
é N +1 - N ù a
= aê ú = N +1 DX æ DM DL DT ö
ë N +1 û Then, ´ 100 = ç + + ÷ ´ 100
X è M L T ø
Stress Force / Area
3. (a) Y = = Þ Y = Pressure. DM DL DT
Strain Dimensionless As we know, = 1%, = 1.5% and = 3%
4. (a) T = Pa Db Sc M L T
= (1 + 1.5 + 3) % = 5.5 %.
M0L0T1 = (ML–1 T–2)a (ML–3)b (MT–2)c
= Ma+b+c L–a–3b T–2a–2c q2
Applying principle of homogeneity 10. (d) eo =
a + b + c = 0; – a – 3b = 0; – 2a – 2c = 1 (r 2 )4 pF
on solving, we get a = – 3/2, b = 1/2, c = 1 Þ unit of eo is (coulomb)2/ newton-metre2
a
5. (b) As =P [ML2 T -2 ][ML2 T -1 ]2
V2 11. (a) = [M 0 L0 T 0 ] = angle.
5 -1 3 - 2 2
[M ][M L T ]
dyne
\ a = PV = 2 (cm3 ) 2 = dyne cm 4 12. (a) The current voltage relation of diode is
cm 2
6. (a) Here, Mass of a body, M = 5.00 ± 0.05 kg I = (e1000 V/T – 1) mA (given)
When, I = 5 mA, e1000 V/T = 6mA
Volume of a body, V = 1.00 ± 0.05 m3
1000
M Also, dI = (e1000 V /T

Density, r = T
V
(By exponential function)
Dr DM DV
Relative error in density is, = + 1000
r M V = (6 mA) ´ ´ (0.01) = 0.2 mA
300
Percentage error in density is 13. (c) Impulse = change in momentum
Dr DM DV 14. (b) Measured length of rod = 3.50 cm
´ 100 = ´ 100 + ´ 100
r M V For vernier scale with 1 Main Scale Division = 1 mm
9 Main Scale Division = 10 Vernier Scale Division,
æ 0.05 ö æ 0.05 ö
=ç ´ 100 ÷ + ç ´ 100 ÷ = 1% + 5% = 6% Least count = 1 MSD –1 VSD = 0.1 mm
è 5 ø è 1 ø
15. (c) Weber is the unit of magnetic flux in S.I. system.
7. (b) In CGS system, 1 Wb(S.I unit) = 108 maxwell
g
d=4 3 é MLT -2 ù
(d) F µ v Þ F = kv Þ [k ] = éê ùú =
F
cm 16. ê -1 ú
= [ML0T -1 ]
ëvû êë LT úû
The unit of mass is 100g and unit of length is 10 cm, so
0.5
æ 100g ö 17. (d) Least count of screw gauge = mm = 0.01mm
4ç 50
è 100 ÷ø
density = 3
\ Reading = [Main scale reading + circular scale
æ 10 ö reading × L.C] – (zero error)
çè cm÷ø = [3 + 35 × 0.01] – (–0.03) = 3.38 mm
10
EBD_7504
S-P-2 SOLUTIONS

Q2 1 e2
19. (c) We know that is energy of capacitor so it represent 27. (c) From F =
2C 4 p eo r2
the dimension of energy = [ML2T–2]. e2
20. (a) Number of significant figures in 23.023= 5 \ = 4 p F r 2 (dimensionally)
Number of significant figures in 0.0003 = 1 eo
Number of significant figures in 2.1 × 10 –3 = 2 e2 4pFr 2 (MLT-2 )L2
= = = [M 0 L0 T 0 A 0 ] ,
drift velocity Vd (ms -1 ) m 2 s -3 eo hc hc ML2T-1[LT-1]
21. (b) Mobility m = = =
electric field E (Vm -1 ) V
e2
æ joule(J) ö is called fine structure constant & has value
e o hc
çèQ Volt = V = coulomb(C) ÷ø
1
2 -1 2 -1
.
m s C m s As 137
= = [Coulomb,c = As] 28. (d) Let unit ‘u’ related with e, a0, h and c as follows.
J kg m 2 s -2
[ u] = [e]a [a0]b [h]c [C]d
= kg–1 s2 A = M–1 T2 A
22. (b) [momentum] = [M][L][T–1] = [MLT–1] Using dimensional method,
[M–1L–2T+4A+2] = [A1T1]a[L]b[ML2T–1]c[LT–1]d
E [M][LT-1]2
Planck’s constant = = = ML2T -1 [M–1L–2T+4A+2] = [Mc Lb+2c+d Ta–c–d Aa]
n T -1
a = 2, b = 1, c = – 1, d = – 1
a 3b2 DP Da Db
23. (d) P = , × 100% = 3 × 100% + 2 ×
cd P a b e 2 a0
\ u=
Dc Dd hc
100% + × 100% + × 100%.
c d 29. (d) 30 Divisions of vernier scale coincide with 29 divisions
= 3 × 1% + 2 × 2% + 3% + 4% = 14% of main scales
24. (c) Diameter D = M.S.R. + (C.S.R) × L.C. 29
0.5 Therefore 1 V.S.D = MSD
D = 2.5 + 20 × 30
50 29
D = 2.70 mm Least count = 1 MSD – 1VSD = 1 MSD - MSD
30
The uncertainty in the measurement of diameter 1 1
DD = 0.01 mm. = MSD = ´ 0.5° = 1 minute.
30 30
We know that
Mass M M 30. (d) Let dimensions of length is related as,
r= = = 3 z
Volume V 4 æ Dö é e2 ù
πç ÷ x
L = [c] [G] ê y
ú
3 è 2ø
ëê 4pe 0 ûú
Dr DM DD
\ ´ 100 = ´ 100 + 3 ´ 100 e2
r M D = ML3T–2
4pe0
0.01
=2+3× ´ 100 = 3.1% L = [LT–1]x [M–1L3T–2]y[ML3T–2]z
2.70
25. (d) Dimensionally e0L = Capacitance (c) [L] = [Lx + 3y + 3z M –y + z T–x – 2y – 2z]
DV C DV q Comparing both sides
\ e0 L = = =I
–y + z = 0 Þ y = z ...(i)
Dt Dt Dt
26. (d) As we know, time period of a simple pendulum x + 3y + 3z = 1 ...(ii)

L 4p2 L –x – 4z = 0 (Q y = z) ...(iii)
T = 2p Þg= 2 From (i), (ii) & (iii)
g T
1
The maximum percentage error in g z = y = , x = –2
2
Dg DL æ DT ö 1/2
´ 100 = ´ 100 + 2 ç ´ 100 ÷ é e2 ù
g L è T ø Hence, L = c -2
êG × ú
êë 4pe 0 úû
Speed Test-2

1. (b) x = at3 and y = bt3 1


a - b´
6. (d) x= (1 - e b ) = a (1 - e -1 ) = a (1 - 1 )
dx dy
vx = = 3a t 2 and v y = = 3b t 2 b b b e
dt dt
a (e - 1) a ( 2 .718 - 1) a (1 .718 ) a 2
= = = = 0 .637 ~- a/b
\ v= v 2x + v2y 2 4 2 4
= 9a t + 9b t b e b 2 .718 b 2 .718 b 3
dx
velocity v = = ae - bt , v0 = a
= 3t 2 a 2 + b 2 dt
dv
2. (d) Let 'S ' be the distance between two ends and 'a' be accleration a = = - abe - bt & a 0 = - ab
dt
the constant acceleration a
As we know v2 – u2 = 2aS At t = 0, x = (1 - 1) = 0 and
b
v2 - u2 1 a a 1 2
or, aS = At t = , x = (1 - e -1 ) = (1 - ) = a / b
2 b b b e 3
Let vc be velocity at mid point. a
At t = ¥, x =
2 2 S b
Therefore, vc - u = 2a a
2 It cannot go beyond this, so point x > is not reached
vc2 = u 2 + aS b
by the particle.
a
v2 - u 2 At t = 0, x = 0, at t = ¥ , x = , therefore the particle
vc2 = u 2 + b
2 does not come back to its starting point at t = ¥.
7. (d) Ist part: u = 0, t = 5s, v = 108 km/hr = 30 m/s
u 2 + v2
vc = v = u + at Þ 30 = 0 + a × 5 Þ a = 6 m/s2
2
3. (a) Acceleration of the particle a = 2t – 1 1 2 1
s = ut + at = 0 ´ 5 + ´ 6 ´ 52 = 75 m
The particle retards when acceleration is opposite to 2 2
velocity. IIIrd part: s = 45m, u = 30m/s, v = 0
Þ a . v < 0 Þ (2t – 1) (t2 – t) < 0 Þ t (2t – 1) (t – 1) < 0 v2 - u 2 -30 ´ 30
Now t is always positive a= = = -10m / s 2
2s 2 ´ 45
\ (2t – 1) (t – 1) < 0
v = u + at Þ 0 = 30 – 10 × t Þ t = 3s
1 IInd part :
or 2t – 1 < 0 and t – 1 > 0 Þ t < and t > 1.
2 s = s1 + s2 + s3
This is not possible 395 = 75 + s2 + 45 Þ s2 = 275 m
or 2t – 1 > 0 & t – 1 < 0 Þ 1/2 < t < 1 275
4. (b) When a ball is dropped on a floor, t= = 9.16 = 9.2s.
30
1 Total time taken = (5 + 9.2 + 3) sec = 17.2 sec
y = gt 2 ..... (1)
2 dv dv
So the graph between y and t is a parabola. Here as 8. (a) = - kv 3 or 3 = - k dt
dt v
time increases, y decreases.
When the ball bounces back, then 1
Integrating we get, - = - kt + c ...(1)
1 2v 2
y = ut + gt 2 ..... (2)
2 1
The graph between y and t will be a parabola. But here At t = 0, v = v0 \ - =c
2v2o
as time increases, y also increases. So (b) represents Putting in (1)
the graph.
1 1 1 1
5. (d) Relative speed of police with respect to thief - 2 = - kt - 2
or 2 - 2 = - kt
= 10 – 9 = 1 m/s 2v 2v0 2v0 2v
Instantaneous separation = 100 m é 1 ù
[ ]
1 2
D istance or or 1 + 2v 02 kt = v 0
Time = v2
Velocity
EBD_7504
S-P-4 SOLUTIONS

t
v 20 v0 é gt 2 ft 3 ù
or v 2 = or v = = êv0 t + + ú
1 + 2 v 20 kt 1 + 2v 02 kt 2 3 úû
ëê 0
9. (d) Let the two balls Pand Q meet at height x m from the gt 2 ft 3
ground after time t s from the start. or, x = v0 t + +
2 3
We have to find distance, BC = (100 – x)
g f
Q C At t = 1, x = v0 + + .
2 3
(100 – x)m 5
100 m 12. (c) Case-1 : u = 50 ´ m / s,
–1 B 18
25 m s
xm v = 0,s = 6m, a = a
P
A v 2 - u 2 = 2as
2
For ball P æ 5ö
Þ 02 - ç 50 ´ ÷ = 2 ´ a ´ 6
S = x m, u = 25 m s–1, a = –g è 18 ø
1 2 2
From S = ut + at æ 5ö
2 Þ - ç 50 ´ ÷ = 2 ´ a ´ 6 ....(i)
è 18 ø
1 2 5
x = 25t – gt .......... (i) Case-2 : u = 100 km/hr = 100 ´ m/sec
2 18
For ball Q v = 0, s = s, a = a \ v 2 - u 2 = 2as
S = (100 – x) m, u = 0, a = g
2
1 2 2 æ 5ö
\ 100 – x = 0 + gt .......... (ii) Þ 0 - ç100 ´ ÷ = 2as
2 è 18 ø
Adding eqns. (i) and (ii), we get 2
100 = 25t or t = 4 s æ 5ö
Þ - ç100 ´ ÷ = 2as … (ii)
From eqn. (i),
è 18 ø
1 Dividing (i) and (ii) we get
x = 25 × 4 – ´ 9.8 ´ (4)2 = 21.6 m
2 100 ´ 100 2 ´ a ´ s
= Þ s = 24m
Hence distance from the top of the tower 50 ´ 50 2´a´6
= (100 – x) m = (100 – 21.6 m) = 78.4 m 13. (c) Height of tap = 5m and (g) = 10 m/sec2.
uuur uuur v For the first drop,
10. (c) From figure, OA = 0iˆ + 30 ˆj, AB = 20iˆ + 0 j
1 1
20 m 5 = ut + gt 2 = (0 ´ t ) + ´10t 2 = 5t2 or t2 = 1 or t = 1.
B 2 2
A It means that the third drop leaves after one second of
45°
30 m the first drop. Or, each drop leaves after every 0.5 sec.
Distance covered by the second drop in 0.5 sec
30 2 m
1 1
C = ut + gt 2 = (0 ´ 0.5) + ´ 10 = (0.5) 2 = 1.25m .
O 2 2
uuur v Therefore, distance of the second drop above the
BC = -30 2 cos 45°iˆ - 30 2 sin 45° ˆj = -30iˆ - 30 j ground = 5 – 1.25 = 3.75 m.
uuur uuur uuur uuur
\ Net displacement, OC = OA + AB + BC = -10iˆ + 0 ˆj 14. (b) Relative speed of each train with respect to each other
uuur be, n = 10 + 15 = 25 m/s
| OC |= 10 m . Here distance covered by each train = sum of their
dx lengths = 50 + 50 = 100 m
11. (c) We know that, v = Þ dx = v dt
dt 100
x t \ Required time = = 4 sec .
Integrating, ò dx = ò v dt 25
0 0 15. (c) Q t = x +3
t
or x = ò (v0 + gt + ft 2 ) dt Þ x = t – 3 Þ x = (t – 3)2
0
Physics S-P-5

Þt=3 20. (b) x = 40 + 12 t – t3


\ x = (3 – 3)2 dx
Þ x = 0. v= = 12 - 3t 2
dt
16. (d) Initial velocity, u = 10 ms–1
Final velocity, v = 20 ms–1 12
For v = 0; t = = 2 sec
Distance, s = 135 m 3
Let, acceleration = a So, after 2 seconds velocity becomes zero.
Using the formula, v2 = u2 + 2as Value of x in 2 secs = 40 + 12 × 2 – 23
v2 - u 2 = 40 + 24 – 8 = 56 m
a= a
2s 21. (c) We have, Sn = u + (2n - 1)
2
(20)2 - (10)2 400 - 100
= = a
2 ´ 135 2 ´ 135 or 65 = u + (2 ´ 5 - 1)
300 150 10 -2 2
a= = = ms 9
2 ´ 135 135 9 or 65 = u + a ..... (1)
Now, using the relation, v = u + at 2
v - u 20 - 10 10 a
t= = = ´ 9sec = 9s. Also, 105 = u + (2 ´ 9 - 1)
a 10 / 9 10 2
5 50 17
17. (d) Speed, u = 60 ´ m/s = m/s or 105 = u + a ..... (2)
18 3 2
5 100 Equation (2) – (1) gives,
d = 20m, u' = 120 ´ = m/s
18 3 17 9
Let declaration be a then (0) 2 – u2 = –2ad 40 = a - a = 4a or a = 10 m/s2.
2 2
or u2 = 2ad … (1) Substitute this value in (1) we get,
2 2
and (0) – u' = –2ad' 9
or u¢2 = 2ad¢ …(2) u = 65 - ´ 10 = 65 - 45 = 20 m / s
2
(2) divided by (1) gives, \ The distance travelled by the body in 20 s is,
d'
4= Þ d ' = 4 ´ 20 = 80m 1 1
d s = ut + at 2 = 20 ´ 20 + ´ 10 ´ (20) 2
2 2
18. (b) Time taken by the stone to reach the water level
= 400 + 2000 = 2400 m.
2h
t1 = æ tö dv æ tö
g 22. (c) Here, f = f0 çè1 - ÷ø or, = f 0 ç1 - ÷
Time taken by sound to come to the mouth of the well, T dt è Tø

or, dv = f 0 æç1 - ö÷ dt
h t
t2 =
v è Tø
2h h é æ t öù
\ Total time t1 + t 2 = + \ v = ò dv = ò ê f0 ç1 - ÷ údt
g v
ë è T øû
19. (b) The two car (say A and B) are moving with same
velocity, the relative velocity of one (say B) with respect æ t2 ö
or, v = f 0 ç t - ÷ + C
to the other A, ® è 2T ø
® ®
v BA = v B - v A = v – v = 0
So the relative separation between them (= 5 km) always where C is the constant of integration.
remains the same. At t = 0, v = 0.
Now if the velocity of car (say C) moving in opposite
æ 0ö
®
direction to A and B, is v C relative to ground then the \ 0 = f0 ç 0 - ÷ + C Þ C = 0
è 2T ø
velocity of car C relative to A and B will
® ® ® æ t2 ö
be v rel. = v C - v \ v = f0 ç t - ÷
® è 2T ø
But as v is opposite to vc
If f = 0, then
So, the time taken by it to cross the cars A and B
æ tö
d 4 T
t = Þ
v rel. 60
EBD_7504
S-P-6 SOLUTIONS

Hence, particle's velocity in the time interval t = 0 and t 6x v max


= T is given by v av =
10x
t =T T
é æ t öù vav 3
vx = ò dv = ò ê f 0 çè1 - T ÷ø údt =
vmax 5
t =0 t =0 ë û
T 27. (a) xA = xB
éæ t2 ö ù
= f 0 êç t - ÷ ú 1 2
2T ø ú at Þ a = tan 45° = 1 m/s2
ëêè û 0
10.5 + 10t =
2
æ T2 ö æ Tö
= f 0 ç T - ÷ = f0 ç T - ÷ 20 ± 400 + 84
è 2T ø è 2ø t2 – 20t – 21 = 0 Þ t = Þ t = 21 sec.
2
1
= f 0T . 1 2
2 28. (a) Q h = gt
2
u 2 - v2
23. (a) Using v2 = u2 – 2gh i.e., h = , 1
2g \ h1 = g(5)2 = 125
2
2 2
æ uö æ uö
çè ÷ø - çè ÷ø 1
AB =
2 3
u/4 h1 + h2 = g(10)2 = 500
C ® 2
2g
u/3 Þ h2 = 375
B
®

2 2
æ uö æ uö u/2 1
çè ÷ø - çè ÷ø A
®

h1 + h2 + h3 = g(15)2 = 1125
3 4 2
and BC = O u
®

2g Þ h3 = 625
h2 = 3h1 , h3 = 5h 1
2 2 2 2
æ uö æ uö æ 1 ö æ 1ö h2 h3
AB çè ÷ø - çè ÷ø çè ÷ø - çè ÷ø or h1 = =
2 3 20
2 3 3 5
\ = = =
BC æ u ö 2 æ u ö 2 2
æ 1ö æ 1 ö
2 7 29. (d) Relative velocity of parrot w.r.t the train
çè ÷ø - çè ÷ø çè ÷ø - çè ÷ø = 10 – (–5) = 15 ms–1.
3 4 3 4
8+8 Time taken by parrot to cross the train = 150 = 10 s
24. (a) Velocity of boat = = 8 km h -1 15
2
1 2
Velocity of water = 4 km h -1 30. (d) Distance from A to B = S = ft1
2
8 8 8 Distance from B to C = ( ft1 ) t
t= + = h = 160 minutes
8-4 8+ 4 3
u2 ( ft1 )2
25. (b) u = 0, t1=10s, t2 = 20s Distance from C to D = = = ft12 = 2 S
2a 2( f / 2)
1 2 A f B C f /2 D
Using the relation, S = ut + at
2
t1 t 2t 1
Acceleration being the same in two cases,
1 1 15 S
S1 = a × t12 , S2 = a × t 22
2 2 Þ S + f t1t + 2 S = 15 S
2
S1 æ t1 ö æ 10 ö
2
1 Þ f t1t = 12 S ............. (i)
\ = ç ÷ = çè ÷ø =
S2 è t 2 ø 20 4 1 2
f t1 = S ............ (ii)
S2 = 4S1 2
x + 2x + 3x t
26. (b) v av = t + t + t Dividing (i) by (ii), we get t1 =
1 2 3 6
2
2x 2x 6x 1 ætö f t2
t1 = , t2 = , t3 = Þ S= fç ÷ =
vmax vmax vmax 2 è 6ø 72
Speed Test-3

r r r r
1. (b) | A ´ B |= A B sin q , A.B = A B cos q 6. (b)
r r r r
| A ´ B |= 3 A.B Þ AB sin q = Ö3 AB cos q
or, tan q = Ö3, \ q = 60º
r r
\| A ´ B |= A 2 + B 2 + 2 AB cos 60º

= A 2 + B 2 + AB
r ˆ The velocity of first particle, v1 = v
2. (b) u = i + 2 ˆj = u x iˆ + u y ˆj Þ u cos q = 1, u sin q = 2
The velocity of second particle, v2 = at
1 gx 2 r r r
y = x tan q - Relative velocity, v12 = v1 - v2
2 u x2
= v 2 + ( at ) - 2v ( at cos a )
2 2
1 2 or v12
2
\ y = 2 x - gx = 2 x - 5x
2 dv12
3. (c) At the two points of the trajectory during projection, For least value of relative velocity, =0
the horizontal component of the velocity is the same. dt
Þ ucos 60° = vcos 45° d é 2
or v + a 2t 2 - 2vat cos a ù = 0
1 1 147 dt ë û
Þ 147 ´ = v ´ Þ m /s
2 2 2 or 0 + a × 2t – 2vacos a = 0
2

Vertical component of u = u sin 60° v cos a


or t =
147 3 a
m
=
2 2u sin 30° 2(10) (1 / 2) 2
Vertical component of v = v sin 45° 7. (d) t= = = sec
g cos 30° 10 ( 3 / 2) 3
147 1 147
= ´ = m
2 2 2 1
R = 10 cos 30° t – g sin 30° t2
147 147 3 2
but vy = uy + ayt Þ = - 9.8t
2 2 10 3 æ 2 ö 1 æ 1ö 4 10 20
147 = ç ÷ - (10) ç ÷ = 10 - = m
Þ 9.8t = ( 3 - 1) Þ t = 5.49s. 2 è 3ø 2 è 2ø 3 3 3
2
r v2
4. (c) Clearly a = ac cos q(-iˆ) + ac sin q(- ˆj ) 8. (c) a= = 1 cm/s. Centripetal acceleration is directed
r
-v 2 v2
= cos q iˆ - sin q ˆj towards the centre. Its magnitude = 1. Unit vector at
R R the mid point on the path between P and Q is
Y
-(xˆ + y)
ˆ / 2.
P( R, q)
9. (b) Two vectors are
R
q r ˆ ˆ
O
X A = cos wti + sin wtj
r wt wt
B = cos ˆi + sin ˆj
r r 2 2
r
5. (c) Using the law of vector addition, (d + e ) is as shown r
For two vectors A and B to be orthogonal A.B = 0
in the fig.
r r wt wt
d A.B = 0 = cos wt.cos 2 + sin wt.sin 2

æ wt ö æ wt ö
= cos ç wt - ÷ = cos ç ÷
è 2 ø è 2 ø
wt p p
f So, = \ t=
w
r r r
\ d +e = f
EBD_7504
S-P-8 SOLUTIONS

ur A
10. (a) v1 = 50 km h –1 due North; \ cos q = –
ur B
uur
v2 = 50 km h –1 due West. Angle between v1 and B2 B
Hence, from (i) = A2 + B2 – 2A2 Þ A= 3
uur A 2
v2 = 90º
A 3
ur Þ cos q = – = – \ q = 150°
- v1 = 50 km h -1 due South B 2
14. (b) Suppose velocity of rain
\ Change in velocity r
uur ur uur ur v R = v x ˆi - v y ˆj
= | v2 - v1 | = | v2 + (- v1 ) |
and the velocity of the man
= v22 + v12 = 502 + 502 = 70.7 km/h r
v m = u ˆi
The direction of this change in velocity is in South-West. \ Velocity of rain relative to man
11. (b) v = 6 î + 8ˆj r r r
v Rm = v R - v m = ( v x - u ) ˆi - v y ˆj
According to given condition that rain appears to fall
vertically, so (vx – u) must be zero.
10 8 \ vx – u = 0 or vx = u
When he doubles his speed,
uur
q v'm = 2u ˆi
r r uur
uur 6 Now v Rm = v R - v' m
Comparing with v = vx ˆi + v yˆj , we get
vx = 6ms–1 and vy = 8 ms–1
( )
= v x ˆi - v y ˆj - (2uˆi )
= ( v x - 2u ) ˆi – v y ˆj
Also, v 2 = v x 2 + v y 2 = 36 + 64 = 100
r
or v = 10 ms–1 The v Rm makes an angle q with the vertical
8 6 r
sin q = and cos q = x - componend of v Rm
10 10 tan q = r
y - componend of v Rm
v 2 sin 2q 2v 2 sin q cos q
R= = ( v x - 2u )
g g
= -v y
8 6 1
R = 2 ´10 ´ 10 ´ ´ ´ = 9.6 m
10 10 10 u - 2u
12. (d) s = t3 + 5 = -v
y
ds which gives
Þ velocity, v = = 3t 2
dt
u
dv vy =
Tangential acceleration at = = 6t tan q
dt
v 2 9t 4
Radial acceleration ac = =
R R
At t = 2s, at = 6 ´ 2 = 12 m/s2
9 ´ 16
ac = = 7.2 m/s2
20
\ Resultant acceleration Thus the velocity of rain
r
= at2 + ac2 = (12) 2 + (7.2) 2 = 144 + 51.84 v = v ˆi - v iˆ
R x y
= 195.84 = 14 m/s2 u ˆ
= u ˆi - j.
B tanq
13. (b) = A 2 + B2 + 2ABcos q ...... (i)
2 15. (c) For projectile A
B sin q
\ tan 90° = Þ A + B cos q = 0 u 2A sin 2 45°
A+
2g
Physics S-P-9

For projectile B Time taken to each the shortest distance between


u 2B sin 2 q OB 50 2
Maximum height, HB = A and B = uuuur = = 5h
2g VBA 10 2
As we know, HA = HB
u 2 sin 2 45° u 2
u 2A sin 2 45° u 2B sin 2 q 19. (b) H = = ...(1)
= 2g 4g
2g 2g
u 2 sin 90° u 2
sin 2 q u 2A R= =
= g g
sin 2 45° u B2
2 R u2
æu ö \ = ...(2)
sin 2 q = ç A ÷ sin 2 45° 2 2g
è uB ø H
2 2 \ tan a =
æ 1 ö æ 1 ö 1 R/2
sin 2 q = ç ÷ ç ÷ =
è 2 ø è 2 ø 4 u2
1 -1 æ 1 ö 4g 1 æ 1ö
sin q = Þ q = sin ç ÷ = 30° = = \ a = tan -1 ç ÷
2 è2ø u 2 2 è 2ø
dv 2g
16. (a) ar = w2 R & at = =0
dt
2 u
2 æ 22 ö H
or, ar= (2pn)2R = 4p2n2R = 4p ç ÷ (1) a
è 44 ø 45°
R/2 R/2
20. (b) Here, x = 4sin(2pt) ...(i)
y = 4cos(2pt) ...(ii)
Squaring and adding equation (i) and (ii)
x2 + y2 = 42 Þ R = 4
Motion of the particle is circular motion, acceleration

anet = ar = p2ms–2 and direction along the radius towards ur v2


vector is along – R and its magnitude =
the centre. R
17. (c) Position vector Velocity of particle, v = wR = (2p) (4) = 8p
r = cos wt + sin wt ŷ ur uur ur uur
r x̂ 21. (d) | A + B |2 = | A - B |2
\ Velocity, vr = –wsin wt x̂ + wcos wt ŷ ur ur r r ur uur
and acceleration, | A + B |2 = | A |2 + | B |2 + 2 A . B = A2 + B 2 + 2 AB cosq
r r r r r uur ur uur
a = –w2 cos wt x̂ + w sin wt ŷ = –w2 r
r r
r × r = 0 hence r ^ v and | A - B |2 = | A |2 + | B |2 - 2 A . B
r v
r is directed towards the origin. = A2 + B 2 - 2 AB cos q
a
ur
18. (a) V A = 10 –i$ ( ) So, A2 + B2 + 2AB cos q
= A2 + B2 – 2AB cos q
ur
V B = 10 $j () 4 AB cos q = 0 Þ cos q = 0
ur
V BA = 10 $j + 10 ˆi = 10 2 km / h \ q = 90º
So, angle between A & B is 90º.
Distance OB = 100 cos 45° = 50 2 km
22. (c) According to law of sines or Lami’s Theorem
N($j)

w 10 km/h A 100 km
45°

100 km O
V = 10 2 km / h vA vB vB / A
=
sin 45°
S
EBD_7504
S-P-10 SOLUTIONS

23. (a) Range of a projectile is maximum when it is projected at 1


an angle of 45° and is given by = = 4 km/hr
15 / 60
u2
Rmax = , where u is the velocity of projection Speed of water v = 5 2 - 4 2 = 3 km/hr
g
2 u 2 sin 2 q
ÞR= u \ u 2 = Rg … (i) 27. (a) H1 =
g 2g
Now, to hit a target at a distance (R/2) from the gun, we u 2 sin 2 (90° - q) u 2 cos 2 q
and H 2 = =
must have 2g 2g
R u 2 sin 2q u 2 sin 2 q u 2 cos2 q (u 2 sin 2q ) 2 R2
= , where q is the angle of projection. H1H 2 = ´ = =
2 g 2g 2g 16 g 2 16
R Rg sin 2q \ R = 4 H1 H 2
Þ = ; from (i)
2 g 28. (a) Total area around fountain
1 2
A = p Rmax
Þ sin 2q = Þ sin 2q = sin 30°
2 v 2 sin 2q v 2 sin 90° v 2
Þ 2q = 30° \ q = 15° Where Rmax = = =
g g g
24. (a) Distance covered in one circular loop = 2pr
v4
= 2 × 3.14 × 100 = 628 m \ A= p 2
g
628 r r r
Speed = = 10 m / sec 29. (c) P = vector sum = A + B
62.8
Displacement in one circular loop = 0 r r r
Q = vector differences = A - B
0 r r
Velocity = =0 Since P and Q are perpendicular
time R r r
uuur uuuur uuur \ P.Q=0
25. (a) PQ + QR = PR
b' r r r r
uuur uur r Þ (A + B).(A - B) = 0 Þ A2 = B2 Þ A = B
\ QR = b ' - b q
P Q 30. (b) y = bx2
uur ur uur ur uur ur b Differentiating w.r.t to t an both sides, we get
Now | b ' - b |2 = (b ' - b ).(b ' - b )
dy dx
= b2x
= b '2 - 2bb ' cos q + b 2 dx dt
vy = 2bxvx
= 2b2 (1 - cos q) [Q b ' = b] Again differentiating w.r.t to t on both sides we get
uur r dv y dx dv
b ' - b = 2b 1 - cos q = 2bv x + 2bx x = 2bv 2x + 0
dt dt dt
æ qö q dv x
= 2b ç 2 sin ÷ = 2bsin
è 2 ø 2 [ = 0, because the particle has constant
dt
26. (a) v acceleration along y-direction]
dv y
Now, = a = 2bv 2x ;
5 km/hr 4 km/hr dt
2 a
vx =
2b
Speed along the shortest path a
vx =
2b
Speed Test-4

1. (d) Here m = 0.5 kg ; u = – 10 m/s;


F -av2
t = 1/50 s ; v = + 15 ms–1 Acceleration = =
Force = m (v– u)/t = 0.5 (10 + 15) × 50 = 625 N M M
2. (d) Mass of rocket (m) = 5000 Kg 7. (c) Let T be the tension in the branch of a tree when
Exhaust speed (v) = 800 m/s monkey is descending with acceleration a. Then mg –
Acceleration of rocket (a) = 20 m/s2 T = ma; and T = 75% of weight of monkey
Gravitational acceleration (g) = 10 m/s2 æ 75 ö æ1ö g
We know that upward force =ç ÷mg = ç ÷mg or a = .
è 100 ø è4ø 4
F = m (g + a) = 5000 (10 +20)
8. (a) Coefficient of static friction,
= 5000 × 30 = 150000 N.
We also know that amount of gas ejected 1
ms = tan 30° = = 0.577 @ 0.6
3
æ dm ö F 150000
ç ÷= = = 187.5 kg / s 1 2
è dt ø v 800 S = ut + at
2
3. (b) From figure,
1 1
Acceleration a = Ra …(i) 4= a(4)2 Þ a = = 0.5
2 2
and mg – T = ma …(ii) a [Q s = 4m and t = 4s given]
From equation (i) and (ii) R a = gsinq – mk(g) cosq
æ aö 0.9
T × R = mR2a = mR2 çè ÷ø Þ mk = = 0.5
R T 3
or T = ma u
T 9. (d) v = u - at Þ t = [ As v = 0]
Þ mg – ma = ma a
g m a u ´ m 30 ´ 1000
Þ a= t= = = 6sec
2 mg F 5000
4. (a) Limiting friction between block and slab 2 1
= µsmAg = 0.6 ×10 × 9.8 = 58.8 N 10. (d) m AW = ; m BW = .
3 3
But applied force on block A is 100 N. So the block will
slip over a slab.
Now kinetic friction works between block and slab T T
Fk = µkmAg = 0.4 × 10× 9.8 = 39.2 N B NB
m 2mg
This kinetic friction helps to move the slab 2m
A 2
39.2 39.2 mg
\ Accleration of slab = = = 0.98 m / s2
mB 40 mg mg 2 2mg 2mg
5. (c) The tension in both strings will be same due to symmetry. 2 45° 45° 2

Diagram shows the various forces acting on the masses


Tcosq Tcosq and their resolution in the direction of motion. Let us
q q consider the two masses to be a system. The forces
T T
trying to move the system such that A moves upwards
T Tsinq B Tsinq T and B moves downwards
A C
Ö2 mg 2mg mg mg
mg mg = - =
For equilibrium in vertical direction for body B we have 2 2 2
The forces trying to stop this motion (i.e., maximum
2 mg = 2T cos q frictional force)
\ 2 mg = 2(mg ) cos q [Q T = mg, (at equilibrium] = fA + fB = m A N A + m B N B
1 2 mg 1 2mg 4 mg
\ cos q = Þ q = 45° = ´ + ´ =
2 3 2 3 2 3 2
6. (b) Thrust on the satellite, Since the stopping force is more therefore the mass
- vdM
F= = - v(a
dt
EBD_7504
S-P-12 SOLUTIONS

dM 17. (d) The particle is moving in circular path


11. (c) = 0.1 kg/s, vgas = 50 m/s , R Rsinq
dt
q
Mass of the rocket = 2 kg. Mv = constant Rcosq

dM dv dv 1 dM mg
–v +M =0. \ = v q
h
dt dt dt M dt
1 From the figure, mg = R sin q … (i)
Þ Acceleration = ´ 50 ´ 0.1 = 2.5 m/s 2
2
mv 2
m/3 = R cos q … (ii)
12. (d) m F r
The acceleration of the system is From equations (i) and (ii) we get
rg r
F 3F tan q = 2 but tan q =
a= = m/6 m/6 v h
m 4m m F
m+ A v 2 (0.5) 2
3 \h = = = 0.025m = 2.5cm
The tension in the middle of the rope (i.e., at point A) is g 10
18. (b) The acceleration of mass m is due to the force T cos q
æ mö 7m 3F
ÞT=
7F
T = çm+ ÷a = ´ T cos q
è 6ø 6 4m 8 \ T cos q = ma Þ a= ... (i)
m
13. (c) Maximum force by surface when friction works
F
also, F = 2T sin q Þ T= ... (ii)
F = f 2 + R 2 = (µR) 2 + R 2 = R µ 2 + 1 2 sin q
Minimum force = R when there is no friction From (i) and (ii)
æ F ö cos q F
Hence ranging from R to R µ2 + 1 [where, R = mg] a= ç ÷
è 2sin q ø m
14. (c) Motion with constant momentum along a straight line. T T
F F x
According to Newton's second law rate of change of = = a
2m tan q 2m a 2 - x 2 Tsinq Tsinq a
momentum is directly proportional to force applied.
T
15. (b) For the motion of both the blocks q Tcosq
é a2 - x2 ù
m1a = T – mkm1g êQtan q = ú
m2g – T = m2a ê x ú x x
ë û
a 19. (b) Considering the equilibrium of B
T
mk m1g m1 –mBg + T = mBa
mk Since the block A slides down with constant speed.
a = 0.
m2 Therefore T = mBg
a
Considering the equilibrium of A, we get
m 2g – m k m1g 10a = 10g sin 30º – T – mN
a= m2g where N = 10g cos 30°
m1 + m 2
N a
æ m 2 g – m k m1g ö T
m2g – T = (m2) ç m + m ÷
è 1 2 ø A mN T a

solving we get tension in the string si n3
10g B
m m (1 + m k ) g 10g cos30º
T= 1 2
m1 + m 2 mBg
10g
16. (d) Acceleration of block while sliding down upper half =
10
g sin f; \ 10 a = g - T - m ´ 10 g cos 30 º
retardation of block while sliding down lower half = – 2
(g sin f - mg cos f) but a = 0, T = mBg
For the block to come to rest at the bottom, acceleration 0 .2 3
in I half = retardation in II half. 0 = 5g - m B g - × 10 ×g
2
g sin f = -(g sin f
Þ m = 2 tan f
Physics S-P-13

20. (a) The Earth pulls the block by a force 25. (a) As the ball, m = 10 g = 0.01 kg rebounds after striking
Mg. The block in turn exerts a force the wall
Mg on the spring of spring balance s2 Mkgf \ Change in momentum = mv – (–mv) = 2 mv
S1 which therefore shows a reading Inpulse = Change in momentum = 2mv
of M kgf. Impulse 0.54 N s
s1 Mkgf \n= = = 27 m s-1
The spring S1 is massless. Therefore 2m 2 × 0.01 kg
it exerts a force of Mg on the spring 26. (b) From the F.B.D.
M
of spring balance S2 which shows N = mg cos q
the reading of M kgf. Mg
21. (a) When tension in the cable is equal to the weight of F = ma = mg sin q – mN
cable, the system is in equilibrium. It means the system Þ a = g(sin q – m cos q)
is at rest or moving with uniform velocity.
N mN
22. (c) Tension at the highest point
mv2
Ttop = – mg = 2mg (\ vtop = 3gr ) mg sin q m g cos q
r xmg
Tension at the lowest point q
Tbottom = 2mg + 6mg = 8mg
Now using, v2 – u2 = 2as
Ttop 2mg 1
\ = = . or, v 2 = 2 ´ g (sin q - m cos q)l
Tbottom 8mg 4
23. (d) Writing free body-diagrams for m & M, (l = length of incline)
M or, v = 2gl (sin q - m cos q)
m
K
F 27. (d) As shown in the figure, the three forces are represented
N
by the sides of a triangle taken in the r samer order.
N
a Therefore the resultant force is zero. Fnet = ma.
T T M Therefore acceleration is also zero i.e., velocity remains
m F unchanged.
28. (d) Given F = 600 – (2 × 105 t)
mg Mg The force is zero at time t, given by
we get T = ma and F – T = Ma
0 = 600 – 2 × 105 t
where T is force due to spring
600
Þ F – ma = Ma or,, F = Ma + ma Þ t= 5
= 3 ´ 10 –3 seconds
F 2 ´ 10
\ a= . t 3´10 –3
M +m
Now, force acting on the block of mass m is ò
\ Impulse = Fdt = ò (600 – 2 ´ 105 t ) dt
æ F ö = mF 0 0
ma = m ç .
è M + m ÷ø m + M
3´10 –3
é 2 ´ 10 t ù 5 2
= ê600t – ú
24. (a) At limiting equilibrium, m = tanq 2
ëê úû 0
dy x 2
tanq = m = = (from question) = 600 ´ 3 ´ 10 –3 – 105 (3 ´10 –3 ) 2
dx 2
= 1.8 – 0.9 = 0.9Ns
29. (c) The block begins to slide if
F cos 37° = µ (mg – F sin 37°)
m
5t [cos 37° + µ sin 37°] = µ mg
q y
é 4 3ù
5t ê + ú = 70 or t0 = 10 sec. or t0/2 = 5 sec.
ë5 5û
30. (d) According to law of conservation of momentum the
third piece has momentum
Q Coefficient of friction m = 0.5 = 1´ –(3iˆ + 4j)
ˆ kg ms–1
Impulse = Average force × time
x2
\ 0.5 = Impulse
2 Þ Average force =
Þ x=+1 time
–(3iˆ + 4ˆj)kg ms –1
x3 1 =
Now, y = = 10 –4 s
6 6
EBD_7504
S-P-14 SOLUTIONS

Speed Test-5

1. (b) k = 5 × 103 N/m Vf 1


=
1
2
( )
1
W = k x22 - x12 = ´ 5 ´ 103 é(0.1) 2 - (0.05) 2 ù
2 ë û
or
Vi 2

5000 2gh 1
= ´ 0.15 ´ 0.05 = 18.75 Nm =
or, 2
2 v02 + 2gh
10
2. (a) Given: Mass of particle, M = 10g = kg or, 4gh = v02 + 2gh
1000
radius of circle R = 6.4 cm \ v0 = 20ms–1
Kinetic energy E of particle = 8 × 10–4J
acceleration at = ? M

1
mv 2 = E
2
1 æ 10 ö 2
Þ ç ÷ v = 8 × 10–4
2 è 1000 ø M
Þ v2 = 16 × 10–2
Þ v = 4 × 10–1 = 0.4 m/s
Now, using
v2 = u2 + 2ats (s = 4pR) 5. (c) Work done in stretching the rubber-band by a distance
æ 22 6.4 ö dx is
(0.4)2 = 02 + 2at ç 4 ´ ´ ÷ dW = F dx = (ax + bx2)dx
è 7 100 ø
7 ´100 Integrating both sides,
Þ at = (0.4)2 × = 0.1 m/s2 L L
8 ´ 22 ´ 6.4 aL2 bL3
3. (b) We know that F × v = Power W = ò axdx + ò bx 2 dx = +
2 3
\ F ´ v = c where c = constant 0 0
6. (b) We know that DU = – W for conservative forces
dv æ mdv ö
m ´v = c çQ F = ma = ÷ x
DU = - ò Fdx or DU = - ò k xdx
x
dt è dt ø
0 0
v t
1 kx 2
m ò vdv = c ò dt Þ mv 2 = ct Þ U(x) – U(0) = -
2 2
0 0
Given U(0) = 0
2c 1 2 kx 2
v= ´t
m U(x) = -
2
dx 2c 1 2 dx This is the equation of a parabola, which is symmetric
= ´t where v = to U-axis in negative direction.
dt m dt
dw
x t 7. (d) As we know power P =
2c 1 dt
ò dx = m ´ ò t 2 dt Þ w = Pt = mv
1 2
0 0 2
3
2c 2t 2 3 2Pt
x= ´ Þ xµt 2 So, v =
m 3 m
4. (a) When ball collides with the ground it loses its 50% of dv
2P 1
energy Hence, acceleration a = . =
dt
m 2 t
1 Therefore, force on the particle at time ‘t’
KEf 1 mVf2
= 2 1
\ Þ = 2Km 2 1 Km mK –1/2
KEi 2 1 2 2 = ma = . = = t
2 2 t 2t 2
Physics S-P-15

2u A
m1u = m1 + m2 v 2h1
8. (a) (By condition of linear t BC + t CB = 2
3 g C
h
momentum)
1 2e 2 h 2h D
Þ m1u = m 2 v ...... (i) =2 = 2e
3 g g h1
h2
| v - v2 | 2h
Also e = 1 t BD + t DB = 2e 2
| u 2 - u1 | g
2u 5 B
Þ v- =u Þ v= u ...... (ii) \ Total time taken by the body in coming to rest
3 3
1 5 m 2h 2h 2h
From (i) and (ii), m1u = m 2 u Þ 1 = 5 = + 2e + 2e 2 + .........
3 3 m2 g g g
9. (b) As we know work done in stretching spring
2h 2h
1 = + 2e [1 + e + e 2 + .........]
w = kx 2 g g
2
where k = spring constant 2h 2h 1 = 2h é1 + e ù æ 1 + e ö
+ 2e ´ =t
g êë1 - e ûú èç 1 - e ø÷
=
x = extension g g 1- e
Case (a) If extension (x) is same, 13. (d) When C strikes A
1 1 1 1
mv 0 2 = mv' 2 + kx 0 2 ( v ' = velocity of A)
W = K x2
2 2 2 2
So, WP > WQ (Q KP > KQ) 2m
m v0 m
C A B
F2
Case (b) If spring force (F) is same W =
2K
So, WQ > WP kx 0 2 = m( v 0 2 - v' 2 ) ............. (i)
1 1
10. (c) 2mv '2 = kx 0 2
2 2
(When A and B Block attains K.E.)
10m 1
h1 \ kx 0 2 = mv'2 ......... (ii)
2
From (i) and (ii),
2 k 2
Just before impact, energy kx 0 2 = mv0 2 - mv' 2 = mv 0 - x 0
2
E = mgh = 10mg ............. (1) k
Just after impact Þ kx 0 2 + x 0 2 = mv0 2
2
25
E1 = mgh - mgh = 0.75 mgh 3 2 v02
100 kx 0 2 = mv 0 2 \ k = m 2
Hence, mgh 1 = E1 (from given figure) 2 3 x0
mgh1 = 0.75 mg (10) 14. (a) By conservation of energy
h1 = 7.5m 1 2
mg (3h) = mg (2h) + mv (v = velocity at B)
11. (a) Given, h = 60m, g = 10 ms–2, 2
1
Rate of flow of water = 15 kg/s mgh = mv2 ; v = 2gh
\ Power of the falling water 2
= 15 kgs–1 × 10 ms–2 × 60 m = 900 watt. From free body diagram of block at B
Loss in energy due to friction mv2
10 B h
= 9000 ´ = 900 watt.
100
\ Power generated by the turbine
= ( 9000 – 900) watt = 8100 watt = 8.1 kW N
mg
2h
12. (c) t AB =
g ; N = mg
EBD_7504
S-P-16 SOLUTIONS
r K.E.(max.) + P.E.(min.) = 2 (Given)
15. (d) Given force F = 2tiˆ + 3t 2 ˆj
According to Newton's second law of motion, 1 9
r \ K.E.(max.) = 2 + =
dv 4 4
m = 2tiˆ + 3t 2 ˆj (m = 1 kg)
dt 1 2
r K.E.max . = mv max .
v t 2
r
ò dv = ò ( 2tiˆ + 3t ˆj) dt
2
Þ
1 9 3
0 0 Þ ´1´ v 2max. = Þ vmax . =
r 2 4 2
Þ v = t 2 ˆi + t 3 ˆj 19. (b) Constant power of car P0 = F.v = ma.v
r r
Power P = F·v = (2t iˆ + 3t 2 ˆj) · (t 2 ˆi + t 3 ˆj) dv
P0 = m .v
= (2t3 + 3t5)W dt
16. (a) According to conservation of linear momentum, P0 dt = mvdv Integrating
MbVb = MblVbl + MbVb1 ....(i)
mv 2
where vb is velocity of bullet before collision P0 .t =
2
v1b velocity of bullet after collision and vbl is the 2P0t
velocity of block. v=
m
K.E. of block = P.E. of block
Q P0 , m and 2 are constant
1 2 \
M bl Vbl = Mbl gh (h = 0.2m) vµ t
2
20. (b) Let initial velocity of the bullet be v.
Solving we get Vbl = 2ms–1
Now from eq (i) By linear momentum conservation
20 × 10–3 × 600 = 4 × 2 + 20 × 10–3 Vb1 m æm ö
v = ç + m÷ v1
2 è2 ø
Solving we get Vb1 = 200 m/s
(v1 = combined velocity)
1 v
17. (d) sin q = v1 = .......... (1)
x 3
From free body diagram of the body retardation = µg
v 2
F æ vö
a 0 = ç ÷ - 2m gd Þ v = 3 2mgd
è 3ø
21. (a) x = 3t –4t2 + t3
x 1 dx
q = 3 - 8t + 3t 2
sin dt
mg
q d2x
Acceleration = = -8 + 6t
F – mg sin q = ma dt 2
Acceleration after 4 sec
æg ö = –8 + 6 × 4 = 16 ms–2
F = m (g sin q + a) = m çè + a ÷ø ....... (1)
x Displacement in 4 sec
Displacement of the body till its velocity reaches v = 3 ×4 – 4 × 42 + 43 = 12 m
\ Work = Force × displacement
v2
v2 = 0 + 2as Þ s = = Mass × acc. × disp.
2a = 3 × 10–3 × 16 × 12 = 576 mJ
m v2
Now, work done = F s cos 0° = (g + ax) ´ 2
x 2a 22. (b) v2 = u2 + 2gh = (10) + 2 ´10 ´19.5 = 490
2
mv K.E. at the ground
= (g + ax)
2ax 1 1 5 49
18. (a) Velocity is maximum when K.E. is maximum = mv 2 = ´ ´ 490 = J
2 2 1000 40
For minimum. P.E.,
5 æ - 50 ö 1
dV P.E. = mgh = ´ 10 ´ ç ÷=- J
= 0 Þ x3 - x = 0 Þ x = ± 1 1000 è 100 ø 40
dx
49 æ 1 ö 50
1 -ç- ÷ = = 1.25J
Þ Min. P.E. = - 40 è 40 ø 40
4
Physics S-P-17

23. (c) Volume of water to raise = 22380 l = 22380×10–3m3 27. (a) Amount of water flowing per second from the pipe
mgh V rgh V rgh m m l æmö
P= = Þ t= = = . = ç ÷v
t t P time l t èlø

22380 ´10-3 ´ 103 ´ 10 ´10


Power = K.E. of water flowing per second
t= = 15 min 1æmö
10 ´ 746 = ç ÷v ×v
2
24. (c) Change in momentum along the wall 2è l ø
= mv cos60º – mv cos 60º = 0 1æmö 3
= ç ÷v
Change in momentum perpendicular to the wall 2è l ø
= mv sin60º – (– mv sin60º) = 2mv sin60º 1
= ´100 ´ 8 = 400 W
2
Change in momentum 28. (a) Velocity of 50 kg. mass after 5 sec of projection
\ Applied force =
Time v = u - gt =100 -9.8 ´5 =51m / s
2 mv sin 60º At this instant momentum of body is in upward
= direction
0.20
pinitial = 50 ´ 51 = 2550 kg m / s
2 ´ 3 ´ 10 ´ 3
= = 50 ´ 3 3 After breaking 20 kg piece travels upwards with
2 ´ 0.2
150 m/s let the speed of 30 kg mass is v
= 150 3 newton p final = 20 ´150 + 30 ´ v
25. (b) Let the block compress the spring by x before stopping. By the law of conservation of momentum
Kinetic energy of the block = (P.E of compressed spring) pinitial = pfinal
+ work done against friction. Þ 2550 = 20 ´150 + 30 ´ v Þ v = -15m / s
1 1 i.e. it moves in downward direction.
´ 2 ´ (4)2 = ´ 10,000 ´ x 2 + 15 ´ x
2 2 1
10,000 x2 + 30x – 32 = 0 29. (d) K.E. = mv 2
2
Þ 5000 x 2 + 15 x - 16 = 0 Further, v 2 = u 2 + 2as = 0 + 2ad = 2ad
= 2(F / m) d
-15 ± (15)2 - 4 ´ (5000)(-16)
\ x= 1
2 ´ 5000 Hence, K.E. = m ´ 2(F / m) d = Fd
= 0.055m = 5.5cm. 2
or, K.E. acquired = Work done
26. (b) Wmg = DK Þ – mg l = ½ mv2 – ½ mu2 = F × d = constant.
i.e., it is independent of mass m.
or mv2 = m(u2 – 2 g l) or v = u 2 - 2gl ˆj 30. (a) Gravitational potential energy of ball gets converted
r into elastic potential energy of the spring.
u = u î
1 2
mg(h + d) = kd
2
1 2
v Net work done = mg(h + d) - kd = 0
2

u h
r r
\ v - u = u 2 - 2gl ˆj - u î d
r r
\| v - u |= [(u 2 - 2gl) + u 2 ]½ = 2(u 2 - gl)
EBD_7504
S-P-18 SOLUTIONS

Speed Test-6

1. (d) Initially centre of mass is at the centre. When sand is With increase in the value of n, the centre of mass shift
poured it will fall and again after a limit, centre of mass towards the end x = L such that first the shifting is at a
will rise. higher rate with increase in the value of n and then the
2 rate decreases with the value of n.
2. (a) Here a = R These characteristics are represented by graph (a).
3
L L L n
4 a æ xö
Now, M 3
pR 3 ò xdm ò x (l dx) ò k çè L÷ø .xdx
= xCM = 0 = 0 =0
M¢ a3 L L L n
æxö
4 3
pR
3 2M
ò dm ò l dx ò k çè L ÷ø dx
= 3 3
= p. M¢ = 0 0 0
æ 2 ö 2 3p é x n+ 2 ù
L
çè R ÷ø kê
3 nú
êë (n + 2) L úû 0 L (n + 1)
Moment of inertia of the cube about the given axis, = =
2
é k x n +1 ù
L n+2
2M æ 2 ö
´ç R÷ 2 ê nú
M¢ a 3p è 3 ø = 4MR
2
ëê (n + 1) L ûú 0
I= =
6 6 9 3p
3. (d) Minimum velocity for a body rolling without slipping L
For n = 0 , xCM = ; n = 1,
2
2 gh
v= 2L 3L
K2 xCM = ; n = 2, xCM = ;....
1+ 3 4
R2 1 2
6. (c) Kinetic energy (rotational) KR = Iw
2 K2 2
For solid sphere, = 1
2 5 2
R Kinetic energy (translational) KT = Mv
2
2 gh 10 (v = Rw)
\ v= = gh
K 2 7 M.I.(initial) Iring = MR2; winitial = w
1+
R2 M.I.(new) I¢(system) = MR 2 + 2mR 2
r Mw
w¢(system) =
4. (c) o o M + 2m
Solving we get loss in K.E.
From conservation of angular momentum about any Mm
= w2 R 2
fix point on the surface, (M + 2m)
mr2w0 = 2mr2w L
w0 r 7. (c) For toppling Mg = F1 × h
Þ w = w0 / 2 Þ v =
2
[Q v = r w] 2
For sliding
5. (a) · When n = 0, x = k where k is a constant. This means
µMg = F2
that the linear mass density is constant. In this case
the centre of mass will be at the middle of the rod i.e., at For sliding to occur first
L/2. Therefore (c) is ruled out F1 > F2
· n is positive and as its value increases, the rate of mgL
increase of linear mass density with increase in x or > mMg or L > 2mh
2
increases. This shows that the centre of mass will shift 8. (b) For no angular acceleration tnet = 0
towards that end of the rod where n = L as the value of Þ F1 × 5 = F2 × 30 (given F2 = 4N) Þ F1 = 24 N
n increases. Therefore graph (b) is ruled out.
n 1 æ K2 ö
æ xö mv 2 ç 2 ÷
· The linear mass density l = k çè ÷ø Rotational KE 2 èR ø K2
L 9. (b) = = .
2
æ K2 ö K2 + R2
ç1 + 2 ÷
è R ø
Physics S-P-19

m2l
10. (d) Initial position of cm = 1 2 L3 Arw 2
m1 + m2 so rotational K.E = Iw =
2 24
m1Dx1 + m2 Dx2 m1v0t + 0 15. (c) If a body rolls on a horizontal surface, it possesses
Also xcm = = both translational and rotational kinetic energies. The
m1 + m2 m1 + m2
net kinetic energy is given by B
m2l mv t
\ final position = + 10 1 æ K2 ö
m1 + m2 m1 + m2 K net = mv2 ç1 + 2 ÷ , v
2 h
è R ø
11. (a) R
m q = 30°
R where K is the radius of gyration. A C
So from law of conservation of energy,
(i) (ii) 1 æ K2 ö
Moment of inertia of a ring about a given axis is mv2 ç1 + 2 ÷ = mgh ,
2 è R ø
I = MR2 where h is the height attained by the sphere.
3M 1 æ 2ö
Mass of the remaining portion of the ring = i.e., ´ 2 ´ (10) 2 ç1 + ÷ = 2 ´ 9.8 ´ h.
4 2 è 5ø
Moment of inertia of the remaining portion of the ring
about a given axis is 1 æ7ö
i.e., ´ 100 ´ ç ÷ = 9.8h
3 2 è5ø
I¢ = MR 2
4 700
or h= = 7.1m
Given I¢ = kMR2 98
\ k = 3/4. 16. (b) Angular momentum of M2 about O is
12. (b) Applying angular momentum conservation L2 = M2vR clockwise
Angular momentum of M1 about O is
V0 L1 = M1vR clockwise O
m Ldisc = Iw clockwise
\ Ltotal = L1 + L2 + Ldisc
1 M2vR
= M2vR + M1vR + MR2w
2
æ R0 ö Now, v = Rw as rope does not slip
mV0R0 = (m) (V1) ç ÷
è 2 ø é 1 ù
\ L = ê M 2 + M1 + M ú vR
\ v1 = 2V0 ë 2 û
1 Therefore k = 2
2
Therefore, new KE = m (2V0)2 = 2mv0 17. (b) Couple produces purely rotational motion.
2
1 2 18. (c) Velocity of P = (NP)w = (NM + MP)w
13. (c) Iw = Loss of potential energy
2 = r(r + sin q)w = v(1 + sin q)
2
1 ml mgl 19. (b)
´ w2 = (1 + cos q)
2 3 2 y
3g æ qö 6g q
Þ w2 = ç 2 cos 2 ÷ Þ w = cos
l è 2ø l 2 3 (0,3,0)
14. (c) If rotation axis is passing through its middle point & is
^ to its plane, then moment of inertia about YY' is (0,0,0)
Y 20. (a) x¢ 1 2 x
L
(2,0,0)
P Q
Y'
2 (–2,–2,0)
ML
I= where M = volume × density = (L×A)×r
12
L3 Ar y¢
so I =
12
EBD_7504
S-P-20 SOLUTIONS

I1 = I2 = 0, because these particles are placed on x-axis v2 sin 2q


The M.I. of system about x-axis, = I 1 + I2 + I3 + I4 Horizontal range AB =
g
= 0 + 0 + 3 × (3)2 + 4 × (–2)2 = 27 + 16 = 43 kg – m2
2
21. (a) æ 2gh ö
22. (c) After collision velocity of COM of A becomes zero and çç 2 ÷÷ sin(2 ´ 30°)
1 + k / R2
that of B becomes equal to initial velocity of COM of A. =è ø
But angular velocity of A remains unchanged as the g
two spheres are smooth. Solving we get AB = 2.08 m
23. (b) I = 1.2 kg m2, Er = 1500 J,
a = 25 rad/sec2, w1 = 0, t = ? mr 2
28. (b) Tr = a1 ....... (1)
As Er 2
1 2 2E r 2 ´ 1500 mr 2
= Iw , w = = = 50 rad / sec Tr = a ....... (2)
2 I 1.2 2
a1 = a ....... (3)
From w2 = w1 + at
50 = 0 + 25 t, \ t = 2 seconds T
24. (a) F = 20t – 5t2
r a1 a a
b
FR r
\a = = 4t - t 2
I
dw T acm
Þ = 4t - t 2
dt
Acceleration of point b = acceleration of point a
w t
(
Þ ò d w = ò 4t - t dt
2
) ra1 = acm – ra
Hence, 2ra = acm
....... (4)
0 0
3
t 29. (b) Since no external torque act on gymnast, so angular
Þ w = 2t 2 - (as w = 0 at t = 0, 6s) momentum (L=I ω ) is conserved. After pulling her arms
3
q 6 æ & legs, the angular velocity increases but moment of
t3 ö
ò d q = ò ç 2t 2 - ÷ dt

inertia of gymnast, decreases in, such a way that angular
0 0 è momentum remains constant.
Þ q = 36 rad
36
Þ n= <6
2p
25. (b) m
26. (b) The M.I. about the axis of rotation is not constant as 30. (c)
the perpendicular distance of the bead with the axis of (2/p) r
rotation increases. z
Also since no external torque is acting. r
dL
\ text = Þ L = constant Þ Iw = constant
dt
Since, I increases, w decreases.
27. (b) Velocity of the tennis ball on the surface of the earth or
Moment of inertia about z-axis, Iz = mr2
ground
(about centre of mass)
2gh Applying parallel axes theorem,
v= ( where k = radius of gyration of spherical
k2 Iz = Icm + mk2
1+ 2
R 2
m4r 2 æ 4ö
æ2 ö 2
Icm = Iz – m ç r÷ = mr - = mr 2 ç1 - 2 ÷
2 èp ø p 2 è p ø
shell = R)
3 i.e., k = 4
Speed Test-7

1. (d) The gravitational force due to the whole sphere at A r +l


GMm GMm
r +l
1
point is F= òr
lx 2
dx =
l òr
x2
dx
GM e m o , where m is the assumed rest mass at r +l r +l
F1 = 0 GMm GMm é x -2+1 ù
ò
-2
(2R ) 2 = x dx =
point A. l r
l êë -2 + 1úû r
In the second case, when we made a cavity of radius r+l
GMm -1 r + l GMm é 1 ù GMm
(R/2), then gravitational force at point A is =- éë x ùû = - ê ú =
l r l ë x ûr r (r + l )
GM e m o
F2 = \ F2/F1= 1/9
( R + R / 2) 2 mv 2 GmM GM
4. (d) = g= 2
2. (b) We choose reference point, infinity, where total (R + x ) (R + x ) 2 also R
energy of the system is zero.
So, initial energy of the system = 0 mv 2 æ GM ö R 2
\ = mç
1 1 Gm 1m 2 (R + x) è R 2 ÷ø (R + x)2
Final energy = m 1 v12 + m 2 v 22 -
2 2 d mv 2 R2
From conservation of energy, \ = mg
(R + x ) (R + x ) 2
Initial energy = Final energy
1/ 2
1 1 Gm 1 m 2 gR 2 æ gR 2 ö
\ 0 = m 1 v12 + m 2 v 2 2 -
2 2 d \ v2 = Þ v = çç ÷
÷
R+x è R + x ø
1 1 Gm 1 m 2
or m 1 v12 + m 1 v 22 = ...(1) 3
2 2 d 5. (b) v= ve
By conservation of linear momentum 4
v m m 2
1 1 æ3 ö 9
m1v1 + m 2 v 2 = 0 or 1 = - 2 Þ v 2 = – 1 v1 K.E. = mv 2 = m ç v e ÷ = mve2
v2 m1 m2 2 2 è4 ø 32
Putting value of v2 in equation (1), we get 9 æ 2GM ö
= m
æ m v ö
2
2Gm1m 2 32 çè R ÷ø
m1v12 + m 2 çç - 1 1 ÷÷ = 9 GMm GMm
è m2 ø d K.E. = ; P.E. = -
16 R R
m1m 2 v12 + m12 v12 2Gm1m 2 7 GMm
= Total energy = K.E. + P.E. = -
m2 d 16 R
Let the height above the surface of earth be h, then
2Gm 22 2G
v1 = = m2 GMm
d ( m1 + m 2 ) d (m1 + m 2 ) P.E. = -
h
2G 7 GMm GMm 16R
Similarly v 2 = - m1 - =- \ h=
d ( m1 + m 2 ) 16 R h 7
6. (a) When closer to the sun, velocity of planet will be greater.
3. (a) The force of attraction between sphere and shaded
So time taken in covering a given area will be less.
æm ö
ç dx ÷ 7. (a)
l
position dF = GM è ø 8. (d) Acceleration due to gravity on earth's surface
2
x M
r l g=G
R2
This implies that as radius decreases, the acceleration
due to gravity increases.
M m Dg DR
= -2 DR
But = -1%
g R R
('–' sign is due to shrinking of earth)
x
= 2%
EBD_7504
S-P-22 SOLUTIONS

9. (c) The gravitational field due to the ring at a distance In case of satellites P.E = – 2 K.E
3r is given by and T.E = P. E + K. E
Gm( 3r ) 3Gm k k k
E= ÞE= Total energy = - =-
2 2 3/ 2 2 2R R 2R
[r + ( 3r) ] 8r
18. (d) Variation of g with altitude is,
3GmM
Attractive force = é 2h ù
8r 2 g h = g ê1 - ú ;
10. (d) Time period does not depend upon the mass of satellite ë Rû
variation of g with depth is,
2 pr 2pr 2 pr 3 / 2 2p
11. (a) T = = = = é dù
v0 2
( gR / r )1/ 2
gR 2 w g d = g ê1 - ú
ë Rû
gR2 gR 2 Equating gh and gd, we get d = 2h
Hence, r 3 / 2 = or r 3 = 2
w w 19. (a) The total momentum will be zero and hence velocity
or, r = (gR2 / w2)1/3 will be zero just after collisiion. The pull of earth will
12. (b) Acceleration due to gravity, make it fall down.
g µ r (if r < RE) and mgx
1 20. (c) Net force towards centre of earth = mg ¢ =
gµ R
(if r > RE)
r2 Normal force N = mg ¢ sin q
13. (d) Both decreases but variation are different. mgx R
14. (d) Thus pressing force N =
R 2x
15. (c) Let mass of smaller sphere (which has to be removed)
is m Mg
N= constant and independent of x.
2
R
Radius = (from figure) Tangential force, F = ma = mg¢ cos q
2
R2
M m - x2
= gx 4
4 3 4 æRö
3 Q = g ¢ cos q =
pR pç ÷ R x
3 3 è2ø
N
M
Þm=
8 mg
Mass of the left over part of the sphere R/2
x
M 7
M' = M- = M
8 8
Therefore gravitational field due to the left over part of
the sphere
gx
=
GM '
=
7 GM a= R 2 - 4x 2
R
x2 8 x2
R
R GMm é1 1 ù Curve is parabolic and at x = , a = 0 . Hence (c).
16. (b) P.E. = ò dr = -GMm ê - ú 2
R0 r2 ë R R0 û 21. (b) Loss in potential energy = Gain in kinetic energy
1 2 GMm æ 3 GMm ö 1
The K.E. acquired by the body at the surface = m v - -ç- ÷ = mv
2
2 R è 2 R ø 2
1 é1 1 ù
\ mv2 = - GMm ê - ú GMm 1 GM
2 Þ = mv 2 Þ v = = gR
ë R R0 û 2R 2 R
æ 1 1ö 22. (d)
v = 2G M ç - ÷ 23. (d) Angular momentum, L = Iw; moment of inertia of sphere
è R0 R ø along the axis passing through centre of mass,
mv 2 k 2 k 2 2p
17. (b) = or mv = I= MR 2 and w = .
R R2 R 5 T

Kinetic energy = 4p MR 2
L=
5T
Physics S-P-23

1
24. (b) gµ (R + h ) 3
R2 27. (c) T = 2p
R decreasing g increase hence, curve b represents GM
correct variation.
25. (b) The Gravitational field due to a thin spherical shell of R3 (1.01R )3
T1 = 2 p , T2 = 2p
radius R at distance r. GM GM
GM
F= (If r > R) T2 - T1
r2 ´100 = 1.5%
For r = R i.e. on the surface of the shell T1
GM 28. (b) Due to inertia of motion it will move tangentially to the
F= 2 original orbit with same velocity.
R
For r < R i.e., inside the shell 1
F=0 29. (c) gµ so we will not get a straight line.
R2
Also F = 0 at a point where Force due to Earth = Force
due to mars
30. (b) Potential energy of particle at the centre of square

r=R r æ ö
26. (a) The gravitational potential at the centre of uniform ç GMm ÷
= -4 ç
spherical shell is equal to the gravitational potential at a ÷
ç ÷
the surface of shell i.e., è 2 ø
-GM
V= , where a is radius of spherical shell
a æ ö
GMm ÷ 1 8 2 GM
Now, if the shell shrinks then its radius decrease then \ -4 ç + mv2 = 0 Þ v 2 =
density increases, but mass is constant. so from above ç a ÷ 2 a
è 2ø
expression if a decreases, then V increases.
EBD_7504
S-P-24 SOLUTIONS

Speed Test-8

1. (c) We know that Young's modulus -V 1


F L and DV = g V DT = 3 a.V.T or = ...(2)
Y= ´ DV 3a. T.
pr 2 l
Since Y, F are same for both the wires, we have, From eqs. (1) and (2), B = P /(3a.T) or T = P
1 L1 1 L 2 3aB
=
r12 l 1 r22 l 2 9. (a)
l1 r22 ´ L1
2
(D 2 / 2) ´ L1 lateral strain ( b )
or, = = 10. (c) Poisson’s ratio, s =
l2 r12 ´ L 2
2
(D1 / 2) ´ L 2 longitudinal strain ( a )
For material like copper, s = 0.33
l1 D 22 ´ L1D 22 L 1
or, = = ´ 2 = And, y = 3k (1 – 2 s)
l2 D12 ´ L 2
(2 D 2 ) 2 2 L2 8
9 1 3
So, l1 : l2 = 1 : 8 Also, = +
y k n
Dp 1 DV y = 2n (1+ s)
2. (b) B = Þ µ [Dp = constant]
DV / V B V Hence, n < y < k
3. (a) From the graph, it is clear that for the same value of 1
load, elongation is maximum for wire OA. Hence OA is 11. (d) W1 = kx 2
2
the thinnest wire among the four wires. 1
4. (c) Using the usual expression for the Young’s modulus, and W2 = k ( x + y )2
2
the force constant for the wire can be written as 1 1
F YA \ W= W2 - W1 = k ( x + y ) 2 - kx 2
= 2 2
k= where the symbols have their usual
Dl L 1
meanings. Now the two wires together will have an = ky (2 x + y )
2
é k1k2 ù D
effective force constant ê k + k ú . Substituting the
ë 1 2û 12. (c)
corresponding lengths and the Young’s moduli we get F
the answer.
5. (b) Solids are least compressible whereas gases are highly
compressible. h
1 2
6. (d) At extension l1, the stored energy = Kl1
2 Shearing strain is created along the side surface of the
1 2 punched disk. Note that the forces exerted on the disk are
At extension l2, the stored energy = Kl2 exerted along the circumference of the disk, and the total
2 force exerted on its center only.
Work done in increasing its extension from l1 to l2 Let us assume that the shearing stress along the side surface
1 of the disk is uniform, then
K (l22 – l12 )
=
2 F> ò dFmax = ò s max dA = s max ò dA
7. (c) Young’s modulus of elasticity is surface surface surface

F/A æDö
Y= = ò s max .A = smax .2p ç ÷ h
DL / L è2ø
FL 8 æ1 -2 ö -2
\ DL = = 3.5 ´10 ´ ç ´ 10 ÷ ´ 0.3 ´10 ´ 2p
AY è 2 ø
L = 3.297 ´ 104 ; 3.3 ´ 10 4 N
So, D L µ
A
D L 2 L 2 A1 2 2 F
\ = ´ = ´ =4 A Þ Dl = Fl
D L1 L1 A 2 1 1 13. (b) As Y = Dl
AY
DL2 = 4 × DL1 = 4 × 1 = 4 cm l
V
8. (b) Bulk modulus B =
Physics S-P-25

21. (a) Ratio of radii r1:r2 = 1:2


Fl 2
Therefore Dl = µ l2 Ratio of area, A1:A2 = pr12 : pr22
VY
A1 : A2 = 1 : 4
Hence graph of Dl versus l2 will give a straight line. Now, Stress1 : Stress2 = 4 : 1
14. (c) According to questions, So, Strain1 : Strain2 = 4 : 1
ls r y Dls l1 4
= a, s = b, s = c, =? Þ 4l2 = l1 = 8
lb rb yb Dl b \ l =
2 1
Fl Fl \ l2 = 2 mm
As, y = Þ Dl =
ADl Ay Increase in length of B is 2 mm.
3mgl s 22. (d)
Dl s = [Q Fs = (M + 2M)g] rq 6mm ´ 30°
prs2 .ys 23. (b) rq = l f Þ f = = = 0.18 °
l 1m
2Mgl b
Dl b = [Q Fb = 2Mg] 24. (a) Given: F = 100 kN = 105 N
prb2 .y b Y = 2 × 1011 Nm–2
3Mgl s l0 = 1.0 m
Dl s pr 2 .y 3a radius r = 10 mm = 10– 2 m
= s s = 2
\ Dl b 2Mg.l b 2b C Stress
From formula, Y =
prb2 .yb Strain
Stress F
Dr / r 1 Dr 1 Dl Þ Strain = =
15. (d) = 0.5 = , = Y AY
Dl / l 2 r 2 l
16. (c) 105 105
= =
17. (c) x pr 2Y 3.14 ´ 10-4 ´ 2 ´ 1011
1
=
628
1
Therefore % strain = ´ 100 = 0.16%
628
25. (c) The given graph does not obey Hooke's law. and there
is no well defined plastic region. So the graph represents
F elastomers.
Fcom Fext
From the figure, it is clear that d
26. (c)
Fcom < Fext.
F 4 ´ 9.8 For a beam, the depression at the centre is given by,
18. (b) K= = = 19.6 ´ 102
x 2 ´ 10 -2 æ fL ö
d=ç ÷
1 2 2
Work done = 19.6 ´10 ´ (0.05) = 2.45 J è 4Ybd3 ø
2 [f, L, b, d are constants for a particular beam]
ph( r24 - r14 ) ph r 4
19. (a) C1 = , C2 = 1
2l 2l i.e. d µ
Y
Initial volume = Final volume
27. (b) Using Hooke’s law, F = kx we can write
\ p[r22 - r12 ]lr = pr 2 lr 4 = k(a – l0) … (i)
and 5 = k(b – l0) … (ii)
Þ r 2 = r22 - r12 Þ r 2 = (r2 + r1 )(r2 - r1 ) If l be the length under tension 9N, then
9 = k(l – l0) … (iii)
Þ r 2 = (8.02 + 7.98)(8.02 - 7.98) After solving above equations, we get
l = (5b – 4a).
Þ r 2 = 16 ´ 0.04 = 0.64 cm Þ r = 0.8 cm
28. (b)
C1 r24 - r14 4
[8.02] - [7.98] 4
1
\ =
4
=
4 29. (a) U / volume = Y ´ strain 2 = 3600 J m–3
C2 r [0.8] 2
20. (a)
EBD_7504
S-P-26 SOLUTIONS

Wl 3 W l 3 ´12 4 W l3
30. (a) d= , where W = load, l = length of beam and I is \ d1 = = (for sq. cross section)
3 YI 3 Y b4 Y b4
geometrical moment of inertia for rectangular beam,
W l3 4 W l3
b d3 and d 2 = =
Ι= where b = breadth and d = depth 3 Y(p r 4 / 4) 3Y (p r 4 )
12
For square beam b = d (for circular cross-section)
b4 d1 3 p r 4 3pr4 3
\ Ι1 = Now = = =
12 d2 b 4 2 2
(p r ) p
æ πr4 ö
For a beam of circular cross-section, Ι2 = ç ÷ (Q b 2 = π r 2 i.e., they have same cross-sectional area)
è 4 ø
Speed Test-9

1. (d) The excess pressure inside a bubble of soap is given by 4T


7. (a) Inside pressure must be greater than outside pres-
4T . It means excess pressure inside the bubbles A r
p= sure in bubble.
r
and C is more than inside B. So, air will go towards B
pa
from A and C. So, A and C will start collapsing with the
increasing volume of B. pa
2T cos q 1 h 2 r1 2
2. (c) h = rrg Þ h µ r Þ h = r = 3
1 2 This excess pressure is provided by charge on bubble.
æ 3 ö
çQ r1 = r, r2 = r + 50% of r = r ÷ 4T s 2 4T Q2 é Q ù
è 2 ø = ; r = 2 4 ê s= ú
2 r 2e 0 16p r ´ 2e 0 ë 4 pr 2 û
2 æ3 ö æ2 ö
New mass m2 = pr2 h 2r = p ç r1÷ ç h1 ÷ r
è2 ø è3 ø Q = 8pr 2rTe 0

=
2
(
3 2
)
pr1 h1 r = m
3
2
8.
9.
(a) Because film tries to cover minimum surface area.
(c) Vertical distance covered by water before striking
3. (a) When a body falls through a viscous liquid, its velocity
increases due to gravity but after some time its velocity ground = (H – h). Time taken is, t = 2 ( H - h ) / g :
becomes uniform because of viscous force becoming Horizontal velocity of water coming out of hole at P,
equal to the gravitational force. Viscous force itself is a
variable force which increases as velocity increases, so u = 2 gh
curve (a) represents the correct alternative.
4. (c) Sum of volumes of 2 smaller drops \ Horizontal range = ut = 2gh ´ 2(H - h) / g
= Volume of the bigger drop
= 2 h (H - h )
4 4
2. pr 3 = p R 3 Þ R = 21/ 3 r
3 3 8hL
Surface energy = T.4pR2 10. (a) Fluid resistance is given by R =
pr4
= T4p22/3r2 = T.28/3 pr2.
5. (c) Angle of contact q When two capillary tubes of same size are joined in
parallel, then equivalent fluid resistance is
T - TSL
cos q = SA æ 8hL ö 9
TLA 8h L 8h ´ 2L
R S = R1 + R 2 = 4
+ 4 = çç ÷÷ ´
when water is on a waxy or oily surface pR p (2R) è pR 4 ø 8
TSA < TSL cos q is negative i.e., 90° < q < 180°
P pPR 4 8 8 é pPR 4 ù
i.e., angle of contact q increases Rate of flow = = ´ = X êas X = ú
RS 8hL 9 9 êë 8 hL ûú
And for q > 90º liquid level in capillary tube fall. i.e., h
decreases 11. (d) At equilibrium, weight of the given block is balanced
6. (c) Pressure at interface A must be same from both the by force due to surface tension, i.e.,
sides to be in equilibrium. 2L. S = W
W 1.5 ´ 10 -2 N
or S = = = 0.025 Nm -1
2L 2 ´ 0.3 m
R d2 Rsina 12. (b)
a
2
R 2 r ( d1 - d 2 ) g
a Rcosa 13. (c) Terminal velocity, vT =
Rsin a – Rsin a 9h
A
d1 vT (10.5 - 1.5) 9
2
= Þ vT = 0.2 ´
\ (R cos a + R sin a)d2g = (R cos a – R sin a)d1g 0.2 (19.5 - 1.5) 2 18
d1 cos a + sin a 1 + tan a \ vT = 0.1 m / s
Þ d = cos a - sin a = 1 - tan a 2
2
the pressure decreases.
EBD_7504
S-P-28 SOLUTIONS

15. (c) Velocity of ball when it strikes the water surface T2 – T1


\ cos (p – q) =
n = 2gh ...(i) T
Terminal velocity of ball inside the water T2 – T1
\ – cos q =
2 (r - 1) T
v = r2g ...(ii) T1 – T2
9 h \ cos q =
T
2 r2g 24. (c) Upthrust = weight of 40 cm3 of water
Equation (i) and (ii) we get 2 gh = (r - 1)
9 h = 40 g = down thrust on water
2 25. (b)
2 4 æ r -1 ö
Þh= r ç ÷ g 26. (d) From the figure it is clear that liquid 1 floats on liquid 2.
81 è h ø
The lighter liquid floats over heavier liquid. Therefore
16. (a) dv = 8 cm/s and dx = 0.1 cm
we can conclude that r1 < r2
dv 8
Velocity gradient = = = 80 / s. Also r3 < r2 otherwise the ball would have sink to the
dx 0.1 bottom of the jar.
40 Also r3 > r1 otherwise the ball would have floated in
17. (c) Volume of ball = = 50 cm3
0.8 liquid 1. From the above discussion we conclude that
Downthrust on water = 50 g.
r1 < r3 < r2.
Therefore reading is 650 g.
27. (a) When the bubble gets detached,
4T
18. (b) Pressure inside tube = P = P0 + Buoyant force = force due to surface tension
r
\ P2 < P1 (since r 2 > r1)
Hence pressure on side 1 will be greater than side 2. So
air from end 1 flows towards end 2
R
q

rq
2 1 T×dl
Subhemispherical Hemispherical Force due to excess pressure = upthrust
radius = r2 radius = r1 2T
19. (c) Access pressure in air bubble =
R
20. (a) As A1v1 = A2v2 (Principle of continuity) 2T 4p R 3
2
(p r ) = rw g
or, l 2 2 gh = pr 2 2 g ´ 4h (Efflux velocity = 2gh ) R 3
2 2 R 4 rw g 2 2rw g
l2 l2 l Þ r = Þ r=R
\ r2 = or r= = 3T 3T
2p 2p 2p 28. (d) Let Vi be the volume of the iceberg inside sea water
21. (a) The condition for terminal speed (vt) is and V is the total volume of iceberg.
Weight = Buoyant force + Viscous force Total weight of iceberg
Fv B=Vr2 g = weight of water displaced by iceberg.
V.r ice g = Vi .r water .g Þ Vi = rice = 0.92
V rwater 1.03
Thus the fraction of total volume of iceberg above the
sea level
é æ 0.92 ö ù
ê V – çè 1.03 ÷ø V ú
W=V r 1g æ V - Vi ö
=ç ´ 100% = ë û
´ 100%
Vg ( r1 - r 2 ) è V ÷ø V
\ V r1 g = V r2 g + kvt2 \ vt = æ 0.92 ö 0.11
k = ç1 - ÷ ´ 100% = ´ 100% » 11%
22. (a) è 1.03 ø 1.03
23. (d) T1 + T cos (p – q) = T2 29. (b) Bernoulli’s theorem.
T 30. (a) Inflow rate of volume of the liquid = Outflow rate of
volume of the liquid
p–q pR 2 V VR 2
2
=
T1 npr nr 2
Speed Test-10

1. (b) Material expands outward and so x, r increases. Due to F / A stress


linear expansion diameter of rod will increase. 7. (a) E= = where Dl=(l'–l) = lat so F = EAat
Dl / l strain
S 4p r 2 8. (b)
2. (a) E = sT4 = sT 4 9. (b) The wall of two layers A and B are connected in series.
S0 4p R 2 Then, heat flowing per second across
r2 both wall layers are same i.e
=s T4 A B
R2 qA L L qB
3. (c) Q = mcDT
Q = mc (T – T0) ......(i)
Q = Kt whereas K is heating rate
\ from 50 to boiling temperature, T increases Dq A Dq B
linearly.
H1 = H 2
At vaporization, equation is Q = mL
K A ADqA K B ADqB
so, temperature remains constant till vaporisation is = Þ K ADq A = K BDq B
L L
complete
Þ 3K B DqA = K B DqB (as KA = 3KB)
After that, again Eqn (i) is followed and temperature
increases linearly Þ 3DqA = DqB ...................(i)
4. (b) When hot water temperature (T) and surrounding Also DqA + DqB = 20° C ...................(ii)
temparature (T0) readings are noted, and log(T – T0) From (i) & (ii)
is plotted versus time, we get a straight line having \ Dq A +3 Dq A = 20°C Þ 4 DqA = 20°C
a negative slope; as a proof of Newton’s law of Þ DqA = 5°C
cooling.
10. (d) l = 10 cm = 0.1 m, d = 0.2 mm,
5. (a) From Wein’s displacement law r = 0.1 mm = 1 × 10–4 m,
lmT = constant e = 0.2, T = 2000 K, s = 5.67 × 10–8 W/m2 K4
Þ lm µ T–1 According to stefan's law of radiation, rate of emission
6. (c) of heat for an ordinary body,
E = sAeT4 = s(2 p r l) eT4
= 5.67 × 10–8 × 2 × 3.14 × 1 × 10–4 × 0.1 × 0.2 × (2000)4
H1 H1
= 11.4 W
\ Power radiated by the filament = 11.4 W [A = 2prl]
11. (c) Given Δl / l = 0.10% = 0.001 and ΔT =100º C
H H2 H Δl
The given arrangement of rods can be redrawn as Now = αΔT
l
follows or 0.001 = a × 100
2K1K2 or a = 10–5 /°C
K= Further g = 3a = 3 × 10–5/°C
K1+K2
ΔV
\ ´ 100 = (3 ´ 10 - 3 ) (100) = 0.30%
K1 K2 V
l l 12. (d) T - dT

dr
·
T1 r
K3 r1
It is given that H1 = H2
KA(q1 - q2 ) K3 A(q - q ) K KK T
Þ =
2l
EBD_7504
S-P-30 SOLUTIONS

Consider a shell of thickness (dr) and of radiius (r) and 18. (a) Suppose, height of liquid in each arm before rising the
let the temperature of inner and outer surfaces of this temperature is l.
shell be T and (T – dT) respectively.
dQ t1
= rate of flow of heat through it t1
dt t2 t2
l1
KA[(T - dT ) - T ] - KAdT l l
= = l2
dr dr
2 dT With temperature rise height of liquid in each arm
= -4pKr (Q A = 4 pr 2 )
dr increases i.e. l1 > l and l2 > l
To measure the radial rate of heat flow, integration
l l2
technique is used, since the area of the surface through Also l = 1 =
which heat will flow is not constant. 1 + g t1 1 + g t2
l –l
r2 T2 Þ l1 + g l1t2 = l2 + g l2 t1 Þ g = 1 2 .
æ dQ ö 1 l2 t1 – l1t2
Then, ç
è dt ÷ø ò r2 dr = -4p K ò dT
19. (a) The rate of heat loss per unit area due to radiation
r1 T1 = Îs (T4–T04)
dQ é 1 1 ù = 0.6 × 5.67 × 10–8 [(400)4–(300)4] = 595 Jm–2s–1.
ê - ú = -4pK [T2 - T 1 ] l 2l
dt ë r1 r2 û t µ t'µ
20. (d)
dQ -4pKr1r2 (T2 - T1 ) A A/2
or = t¢ l/A
dt (r2 - r1 ) =4
t l/A
dQ r r t¢ = 4 × t
\ µ 1 2
dt (r2 - r1 ) 3/t¢ = 48s
13. (a) According to Newton’s law of cooling if temperature 21. (a) The heat flow rate is given by
difference between body & surrounding is large, then
dQ kA(q1 - q)
rate of cooling is also fast hence curve A shows correct =
behaviour. dt x
14. (a) Among glass, wood and metals, metals expand more x dQ x dQ
Þ q -q = Þ q = q1 -
for same rise in temperature. 1 kA dt
kA dt
where q1 is the temperature of hot end and q is
1 temperature at a distance x from hot end.
15. (b) According to Wien’s law l m µ and from the figure
T The above equation can be graphically represented by
(lm)1 < (lm)3 < (lm)2 therefore T1 > T3 > T2. option (a).
22. (d) Let T be the temperature of the interface. As the two
16. (a) Moment of inertia of a rod,
1 sections are in series, the rate of flow of heat in them
I= ML2
12 will be equal.
Differentiating w.r.t. to DL, we get T1 1 2 T2
DI 1
= ´ 2ML
DL 12
1 DI DL K1 K2
DI = 2MLD L \ =2
12 I L K A(T - T ) K 2 A(T - T2 )
DL \ 1 1 = ,
As we know, DL = LaDt or = aDt l1 l2
L where A is the area of cross-section.
DL
Substituting the value , we get or, K1 A(T1 - T )l 2 = K 2 A(T - T2 )l1
L
DI or, K1T1l 2 - K1T l 2 = K 2T l1 - K 2T2 l1
= 2aD t
I or, ( K 2 l1 + K1l 2 )T = K1T1l 2 + K2T2 l1
17. (c) The lengths of each rod increases by the same amount
K1T1l 2 + K 2T2 l1 K l T + K 2 l1T2
\ Dl a = Dl s Þ l1a a t = l 2 a s t \ T = = 1 2 1 .
K 2 l1 + K1l 2 K1l 2 + K 2 l1
l 2 aa l a
Þ = Þ 2 +1 = a +1 23. (d) Radius of small sphere = r
l1 a s l1 as Thickness of small sphere = t
l 2 + l1 a a + a s l1 as Radius of bigger sphere = 2r
Þ = Þ = Thickness of bigger sphere = t/4
l1
Physics S-P-31

Let K1 and K2 be the thermal conductivities of larger Thus in first case


and smaller sphere.
m ´ C ´ (61° – 59°) -KA éæ 61° + 51° ö ù
For bigger sphere, = ê çè ÷ø - 30°ú (i)
4 3
4 dx ë 2 û
K1 4p (2r)2 ´ 100 3 p(2r) rL In second case,
=
t/4 25 ´ 60 m ´ C ´ (51° - 49°) - KA éæ 51° + 49° ö ù
For smaller sphere, = ê çè ÷ø - 30° ú (ii)
t dx ë 2 û
4 3 Dividing equation (i) by equation (ii)
pr rL
K 2 ´ 4pr 2 ´ 100 3
= t 30
t 16 ´ 60 =
4 20
K1 8 or t = 6 minutes.
\ K = 25
2 Q dθ dθ
24. (a) V + DV = (L + DL)3 = (L + aLDT)3 27. (d) = K1 A1 = K 2A2
t dx dx
= L3 + (1 + 3aDT + 3a2DT2 +a3DT3)
2
Þ a and a3 terms are neglected. 28. (c) Since specific heat = 0.6 kcal/g × °C = 0.6 cal/g × °C
\ V (1 + gDT) = V (1 + 3aDT) From graph it is clear that in a minute, the temperature is
1 + gDT = 1 + 3aDT raised from 0°C to 50°C.
\ g = 3a. Þ Heat required for a minute = 50 × 0.6 × 50 = 1500 cal.
25. (d) According to the principle of calorimetry. Also from graph, Boiling point of wax is 200°C.
Heat lost = Heat gained 29. (b) Temperature of B will be higher because, due to expan-
mLv + mswDq = mwswDq sion centre of mass B will come down same heat is sup-
plied but in B, Potential energy is decreased therefore
Þ m × 540 + m × 1 × (100 – 80) internal energy gain will be more.
= 20 × 1 × (80 – 10) 30. (c) In parallel combination, the equivalent thermal
Þ m = 2.5 g conductivity is given by
Therefore total mass of water at 80°C K1 A1 + K 2 A2 + K3 A3 + ....... + K n An
K=
= (20 + 2.5) g = 22.5 g A1 + A2 + A3 + ....... + An
26. (c) From Newtwon's law of cooling For two rods of equal area,
dQ dT (K + K2 ) A
= - KA K= 1 (if A1 = A2 = A)
dt dx 2A
Area of cross-section A and thickness dx is the same. K + K2
Also dQ = mCdq Þ K= 1
2
EBD_7504
S-P-32 SOLUTIONS

Speed Test-11

1. (a) U = a + bPV ......(1) æ nRg ö


QBC = nC p DT = ç T0
In adiabatic change, è g – 1÷ø
nR nR W é 2ln 2 – 1 ù
dU = – dW = (T2 - T1 ) = (d T ) Efficiency, h= =ê ú
g -1 g -1 Q ë g / ( g – 1) û
nR 5. (c) Coefficient of performance of a refrigerator,
ÞU =
ò dU = g - 1 ò dT Q2 T2
b= = (Where Q2 is heat removed)
æ nR ö PV W T1 - T2
or U =ç ÷ T +a = + a ......(2)
è g - 1ø g -1 Given: T2 = 4°C = 4 + 273 = 277 k
where a is the constant of integration. T1 = 30°C = 30 + 273 = 303 k
Comparing (1) and (2), we get
600 ´ 4.2 277
1 b +1 \ b= =
b= Þg = . W 303 - 277
g -1 b
Þ W = 236.5 joule
2. (d) For path ab : ( DU )ab = 7000 J
W 236.5 joule
By using DU = mCV DT Power P = = = 236.5 watt.
t 1sec
5 6. (a) Change in internal energy do not depend upon the
7000 = m ´ R ´ 700 Þ m = 0.48
2 path followed by the process. It only depends on initial
For path ca : and final states i.e., DU1 = DU2
( DQ)ca = ( DU )ca + ( DW )ca ...(i) 7. (c) T1 = 273 + 27 = 300K
Q ( DU )ab + ( DU )bc + ( DU )ca = 0 T2 = 273 + 927 = 1200K
For adiabatic process,
Q 7000 + 0 + ( DU )ca = 0 Þ ( DU )ca = -7000 J ...(ii)
P1–g Tg = constant
Also ( DW )ca = P1 (V1 - V2 ) = mR (T1 - T2 ) Þ P11–g T1g = P21–g T2g
= 0.48 × 8.31 × (300 – 1000) = –2792.16J ...(iii) 1-g g 1-g g
On solving equations (i), (ii) and (iii) æP ö æ T1 ö æP ö æ T2 ö
Þç 2÷ =ç ÷ Þç 1÷ =ç ÷
( DQ )ca = - 7000 - 2792.16 = - 9792.16 J » - 9800 J èP ø 1 èT ø 2 èT ø 2 èT ø 1
1-1.4 1.4 -0.4
3. (a) Here Q = 0 and W = 0. Therefore from first law of æ P1 ö æ 1200 ö æP ö
=ç Þç 1 ÷ = (4)1.4
thermodynamics DU = Q + W = 0 çè P ÷ø
2 è 300 ÷ø è P2 ø
\ Internal energy of the system with partition = 0.4
Internal energy of the system without partition. æ P2 ö
çè P ÷ø = 41.4
n1Cv T1 + n2 Cv T2 = (n1 + n2 )Cv T 1
æ 1.4 ö æ7ö
n1T1 + n2 T2 ç ÷ ç ÷
\T = P2 = P1 4è 0.4 ø = P1 4è 2 ø
n1 + n2
PV PV = P1 (27) = 2 × 128 = 256 atm
1 1
But n1 = and n2 = 2 2
RT1 RT2
8. (b) P
PV T1
1 1 ´ T + P2V2 ´ T (V, T1)
1 2
RT1 RT2
\T=
PV
1 1 + P2V2 (32 V, T2)
RT1 RT2
1 1 + P2V2 )
T1T2 ( PV T2
=
1 1 2 + P2V2T1
PV T V
4. (a) WAB = 0, WBC = PDV = nRDT = –nRT0 We have, TV g -1 = constant
V Þ T V g -1 = T (32V )g -1
WCA
Physics S-P-33

7 1
For diatomic gas, g = 4
5 So, P = KT
3
2
\ g -1 = uRT 1
5 or = KT 4 [As PV = u RT]
2 V 3
\T = (32) 5 .T2 Þ T1 = 4T2
1 4 3 3
T2 pR T = constant
Now, efficiency = 1 - T 3
1 1
T2 1 3 Therefore, Tµ
= 1- = 1 - = = 0.75. R
4T2 4 4 11. (c) Heat required to change the temperature of vessel by a
9. (c) The equation for the line is small amount dT
P – dQ = mCdT
Total heat required
3Po 4
æ T ö
4
3
100 ´10 –3 ´ 32 é T 4 ù
c – Q = m ò 32 ç ÷ dT = ê ú
q 20 è 400 ø (400)3 ë 4 û 20
2Po
Po Þ Q = 0.001996 kJ
q
Po Work done required to maintain the temperature of sink
Vo to T2
V
Vo 2Vo
Q1 – Q2 æT ö
- P0 W = Q1 – Q2 = Q2 = ç 1 –1÷ Q2
P = V V + 3P Q2 T
è 2 ø
0

Þ W = æç 1 2 ö
-P0 T –T
[slope = , c = 3P0] ÷ Q2
V0 è T2 ø
PV0 + P0V = 3P0V0 ...(i) For T2 = 20 K
But PV = nRT 300 – 20
W1 = ´ 0.001996 = 0.028 kJ
nRT 20
\P= ...(ii) For T2 = 4 K
V

nRT W2 = 300 – 4 ´ 0.001996 = 0.148 kJ


From (i) & (ii) V0 + P0V = 3P0V0 4
V
As temperature is changing from 20k to 4 k, work done
\ nRT V0 + P0V2 = 3P0V0V ...(iii) required will be more than W1 but less than W2.
dT 12. (b) For an adiabatic process, the temperature-volume
For temperature to be maximum =0 relationship is
dV
g -1
Differentiating e.q. (iii) by 'V' we get æV ö
T1V1g -1 = T2V2 g -1 Þ T1 = T2 ç 2 ÷
dT è V1 ø
nRV0 + P0(2V) = 3P0V0
dV V1
Here g = 1.4 (for diatomic gas). V2 = , T1 = Ti , T2 = aTi
dT 32
\ nRV0 = 3P0V0 – 2 P0V 1.4 -1
dV é1ù é1ù
0.4
aTi
\ Ti = aTi ê ú \ Ti = aTi ê 5 ú =
dT 3P0 V0 - 2P0 V ë 32 û ë2 û 4
= nRV0
=0 \ a=4
dV
3V0 3P0 W T
V= \ P= [From (i)] 13. (a) As we know h = =1– 2
2 2 Q1 T1
9Po Vo 300 K 3
\ Tmax = [From (iii)] Þ h =1– =
4nR 1200K 4
1æUö 3 W 4 4
10. (a) As, P = ç ÷ = Þ Q1 = W ´ Þ Q1 =12.6 ´106 ´
3èVø 4 Q1 3 3
U
But = KT 4
V
EBD_7504
S-P-34 SOLUTIONS

14. (a) Initially In the second case


V1 = 5.6l, T1 = 273K, P1 = 1 atm, DQ = 12 cal = 12 × 4.2 J = 50.4 J
5 DW = DQ – DU = 50.4 – 35 = 15.4 J
g= (For monatomic gas)
3 18. (d) T
The number of moles of gas is
5.6l 1 2T0
Q1
n= =
22.4l 4 Q3
Finally (after adiabatic compression) T0
Q2
V2 = 0.7l S
For adiabatic compression S0 2S 0

T1V1g-1 = T2V2 g-1 1 3


Q1 = T0 S0 + T0 S0 = T0 S0
g-1 5 2 2
-1
æV ö æ 5.6 ö 3 Q2 = T0(2S0 – S0) = T0S0 and Q3 = 0
\ T2 = T1 ç 1 ÷ = T1 ç ÷
è V2 ø è 0.7 ø W Q1 - Q2 Q2 TS 1
2/ 3 h= = = 1- = 1- 0 0 =
= T1 (8) = 4T1 Q1 Q1 Q1 3 3
T S
We know that work done in adiabatic process is 2 0 0
19. (c) Change in internal energy is equal to work done in
nRDT 9
W= = RT1 adiabatic system
g -1 8 DW = –DU (Expansion in the system)
15. (c) Curve A, B shows expansion. For expansion of a gas,
1
Wisothermal > Wadiabatic = - ( PV - P V )
g -1 1 1 2 2
Pisothermal > Padiabatic
Tisothermal > Tadiabatic 1
DU = ( P2V2 – PV
1 1)
1– g
Þ Slope of curve for isothermal change < slope of
curve for adiabatic change. Here, V1 = V ,V2 = 2V
So, curve B shows isothermal change and curve A 1 PV
shows adiabatic change. \ DU = [ P ´ 2V – PV ] =
1– g 1– g
1 PV
16. (a) Initially the efficiency of the engine was which Þ DU = -
6 g -1
1 T2
increases to when the sink temperature reduces 20. (b) Efficiency of Carnot engine, h = 1-
3 T1
by 62º C. where T1 and T2 be the temperature of source and sink
1 T respectively.
h = = 1- 2 , when T2 = sink temperature
6 T1 T2 40 60 3
\ = 1 - h = 1- = = (Q h = 40%)
T1 = source temperature T1 100 100 5
5 3 3
Þ T2 = T1 T2 = T1 = ´ 500 K = 300 K ...(i)
6 5 5
Secondly, (Q T1 = 500 K)
Let T 1¢ be the temperature of the source for the same
1 T - 62 T 62 5 62
= 1- 2 = 1- 2 + = 1- + sink temperature when efficiency h¢ = 50%
3 T1 T1 T1 6 T1
T 50 1
or, T1 = 62 × 6 = 372K =372– 273 = 99ºC \ 2 = 1- h ¢ = 1- =
T ¢
1
100 2
5
& T2 = ´ 372= 310 K= 310 - 273 = 37º C
6 T1¢ = 2T2 = 2 × 300 K = 600 K (Using eq. (i))
17. (b) In the first-case adiabatic change, 21. g
(a) For an adiabatic change PV = constant
DQ = 0, DW = –35 J P1V1g = P2V2g
From 1st law of thermodynamics, As molar specific heat of gas at constant volume
DQ = DU + DW, 3
Cv = R
or 0 = DU – 35 2
\ DU = 35 J 5
= R;
2
Physics S-P-35

CP (5 / 2) R 5 DQ = heat absorbed by gas


g= = = DW = work done by gas.
CV (3 / 2) R 3
–20J = DU – 8J
\ From eqn. (1)
DU = –12J = UFinal – Uinitial
g 5/3 Uinitial = 30J.
æV ö æ 6ö
P2 = ç 1 ÷ P1 = ç ÷ ´ 105 N / m 2 UFinal = 30 – 12 = 18J.
è V2 ø è 2ø
T1 - T2
= (3)5/3 × 105 = 6.19 × 105 N/m2 27. (b) is maximum in case (b).
T1
Work done
28. (a) Efficiency of a process is defined as the ratio of work
1 done to energy supplied. Here,
= [6.19 ´ 105 ´ 2 ´ 10-3 - 10 -5 ´ 6 ´ 10-3 ]
1 - (5 / 3)
h = DW = Area under p - V diagram
é 2 ´ 102 ´ 3 ù DQ DQ AB + DQBC
= -ê (6.19 - 3)ú
ëê 2 ûú p0V0
\ h =
= –3 × 102 × 3.19 = –957 joules nCv DT1 + nC p DT2
[–ve sign shows external work done on the gas] p0V0
=
22. (b) dW = P D V = 1 .01 ´ 10 5 [1671 - 1] ´ 10 - 6 Joule 3 5
nR(TB – TA ) + nR (TC – TD )
1.01 ´ 167 2 2
= cal. p0V0
4.2 =
3 5
= 40cal. nearly (2 p0V0 – p0V0 ) + (4 p0V0 – 2 p0V0 )
2 2
Δ Q = mL = 1 ´ 540, p0V0 1
= = = 15.4%
3 5 6.5
ΔQ = ΔW + ΔU p0V0 + × 2 p0V0
2 2
or D U = 540 - 40 = 500 cal.
29. (a) V1 = 1l = 1000 cm3, P1 = 72 cm of Hg.
23. (d) T1 = 18º C = (273 + 18) = 291 K V2 = 900 cm3, P2 = ?
and V2 = V1 / 8 Q The process is isothermal
We know that T V g -1 = constant \ P1V1 = P2V2
72 × 1000 = P2 × 900
or , T2 V2 g -1 = T1 V1g -1 P2 = 80 cm of Hg
g -1 \ Stress = P – P = 80 – 72 = 8 cm of Hg.
æV ö 2 1
\ T2 = T1 çç 1 ÷÷ = 291 ´ (8)1 .4 -1 5
è V2 ø 30. (c) P V = constant Þ PV5/3 = constant Þ g =
3 5
3
= 668.5 K = 395.5º C Þ monoatomic gas
24. (a) Initial and final condition is same for all process For adiabatic process
DU1 = DU2 = DU3 P PiVi 1
from first law of thermodynamics
DQ = DU + DW
Work done 2
DW1 > DW2 > DW3 (Area of P.V. graph) adiabatic
So DQ1 > DQ2 > DQ3
25. (b) Efficiency of carnot engine PfVf
T2 1 T2
n = 1 – T i.e., 10 = 1 – T V
1 1
1
Þ
T2 1 9
= Þ 1=
T 10 ´105 ´ 8 ´10 -3 - 105 ´ 10-3
=1– Pf Vf - Pi Vi 32
T1 10 10 T2 9 W= =
æ T1 ö 1- g 5
1-
\ w = Q2 ç T – 1÷ 3
è 2 ø
25 - 100 75 ´ 3
æ 10 ö æ1ö
i.e., 10 = Q2 ç – 1÷ 10 = Q2 ç ÷ \ W = ( 3 - 5 ) / 3 = 2 = 112.5J
è 9 ø è9ø
Þ Q2 = 90J From first law of thermodynamics q = DU + w \ DU = – w
\ DU = – 112.5 J
So, 90 J heat is absorbed at lower temperature.
Now applying first law of thermodynamics for process
26. (b) q2 = DU + Pi(Vf – Vi)
DQ = D U + D W – 3 = 587.55 J
EBD_7504
S-P-36 SOLUTIONS

Speed Test-12

P1 V1 P2 V2 V
1. (b) = Here, P1 = 200kPa tµ
T1 T2 T
T1 = 22°C = 295 K T2 = 42°C = 315K As, TVg–1 = K
So, t µ Vg + 1/2
2 g +1
V2 = V1 + V1 = 1.02V1 Therefore, q =
100 2
7. (a) According to given Vander Waal's equation
200 ´ 315V1
\ P2 = = 209.37kPa nRT an 2
295 ´1.02V1 P= - 2
V - nb V
2. (d) Molar mass of the gas = 4g/mol
Speed of any quantity x Work done,
V2 V2 2 V
gRT g ´ 3.3 ´ 273 dV dV
V= Þ 952 =
4 ´ 10-3
W= ò PdV = nRT ò
V - nb
- an2 ò 2
m V1 V 1 V V 1
16 8
Þ g = 1.6 = = V
é1ù 2
= nRT [ loge (V - nb)]V2 + an2 ê ú
10 5 V
CP 8 1 ëV ûV1
Also, g = C = 5
V
æ V - nb ö 2 éV1 - V2 ù
8´ 5 = nRT loge ç 2 ÷ + an ê ú
So, CP = = 8JK–1mol–1 V
è 1 - nb ø ë V1V2 û
5
n1Cv1 + n2Cv2 8. (b) From graph, T2V = const. .....(1)
3. (a) For mixture of gas, Cv = As we know that TVg–1 = const
n1 + n2
1
3 1 5 5 g-1
4 ´ R + ´ R 6R + R Þ VT = const. ....(2)
= 2 2 2 = 4 = 29 R ´ 2 = 29 R
On comparing (1) and (2), we get
æ 1ö 9 9´4 18
çè 4 + ÷ø 2
Þ g = 3/2
2
5R 1 7 R 3P Pg
4´ + ´ Also v rms = and vsound =
n1C p1 + n2 C p2 2 2 2 r r
and C p = =
(n1 + n2 ) æ 1ö
çè 4 + ÷ø Þ
v rms
=
3
= 2
2
vsound g
7
10 R + R R
= 4 = 47 R 9. (a) C = C v mix + .....(1)
9 18 1- n
2 n1C v1 + n 2 C v2
Cp Now, Cvmix =
47 R 18 n1 + n 2
Þ
\ = ´ = 1.62
Cv 18 29 R
3R 5R
4. (b) Vapour pressure does not depend on the amount of 1´ +2´
= 2 2 = 13R
substance. It depends on the temperature alone.
1+ 2 6
æV ö æV ö
5. (a) Q q1 < q2 Þ tan q1 < tan q2 Þ çè T ÷ø < çè T ÷ø 13R R 1
1 2 From (1), 3R = + Þn=-
6 1- n 5
V 1 10. (c) As no heat is lost,
From PV = mRT ; µ
T P Loss of kinetic energy = gain of internal energy of gas
æ 1ö æ 1ö 1 2 1 2 m R
Hence çè P ÷ø < çè P ÷ø Þ P1 > P2 . mv = nCV D T Þ mv = × DT
1 2 2 2 M g –1
1
6. (a) t= Mv 2 ( g –1)
æNö K
2pd2 ç ÷
èVø
Physics S-P-37

3 16. (d) Internal energy of 2 moles of oxygen


11. (a) k BT = K av ...(i)
2 æ5 ö 5
Uo 2 = µ ç RT÷ = 2. RT = 5RT
where Kav is the average kinetic energy of the proton. è2 ø 2
2K av Internal energy of 4 moles of Argon.
\ T = 3k æ3 ö 3
B U Ar = µ ç RT÷ = 4. RT = 6RT
è2 ø 2
2 ´ 4.14 ´ 10 –14 J \ Total internal energy
T= = 2 ´ 109 K.
3 ´ 1.38 ´ 10–23 JK –1 U = U O2 + U Ar = 11RT
12. (a) Let T be the temperature of the mixture, then 17. (a) V = 22.4 litre = 22.4 × 10–3 m3, J = 4200 J/kcal
U = U1 + U2 by ideal gas equation for one mole of a gas,
f
Þ (n1 + n 2 ) RT PV 1.013 ´ 105 ´ 22.4 ´ 10 -3
2 R= =
T 273
f f
= (n1 ) (R) (T0 ) + (n 2 ) (R) (2T0 ) R 1.013 ´ 105 ´ 22.4
2 2 Cp - C v = = = 1.979 kcal/kmol K
Þ (2 + 4)T = 2T0 + 8T0 (Q n1 = 2, n2 = 4) J 273 ´ 4200
18. (a) 1 mole = 22.4 L at S.T.P.
5
\ T = T0 4.5g 4.5
3 = 22.4 ´ = 5.6 L
13. (b) Root-mean square-velocity is given by 18g 18
3RT æTö m1C p1 + m 2C p2
v rms = i.e., v rms µ ç ÷ 19. (b) (C p )mix =
M èMø m1 + m 2
5 7
( v rms )O 2 é TO M H2 ù (C p1 ( He) = R and C p2 ( H 2 ) = R)
\ = ê 2 ´ ú 2 2
( v rms )H 2 êë TH 2 M O2 úû 5 7
1´ R + 1´ R
(Cp)mix = =
2 2 3R = 3 × 2 = 6 cal/mol.°C
éæ 1200 ö æ 2 öù 1 1 +1
= êç ÷ ´ ç ÷ú =
ëè 300 ø è 32 øû 2 \ Amount of heat needed to raise the temperature from
1 1930 0°C to 100°C
\ ( v rms )O 2 = ( v rms )H 2 ´ =
2 2 (DQ ) p = mC pD T = 2´ 6´ 100 = 1200 cal
= 965 m/s
20. (c) P-V diagram of the gas is a straight line passing
14. (c) Given
through origin. Hence P µ V or PV–1 = constant
CP - CV = 5000 J / mole °C .......(i) Molar heat capacity in the process PVx = constant is
CP R R
= 1.6 .......(ii) C= + ; Here g = 1.4 (For diatomic gas)
CV g -1 1 - x
From Equation (i) & (ii), R R
ÞC= + Þ C = 3R
CP CV 5000 1.4 - 1 1 + 1
Þ - =
CV CV CV n1
21. (a) Number of moles of first gas = N
A
5000
Þ 1.6 - 1 = n2
CV Number of moles of second gas = N
A
5000 n3
Þ CV = = 8.33 ´ 103 Number of moles of third gas = N
0.6 A
Hence CP = 1.6 CV = 1.6 × 8.33 × 103 If there is no loss of energy then
CP = 1.33 × 104 P1V1 + P2V2 + P3V3 = PV
PV æ mö PV æ R ö n1 n n n + n + n3
15. (c) = nR = ç ÷ R or =ç ÷ m RT1 + 2 RT2 + 3 RT3 = 1 2 RTmix
T è Mø T è Mø NA NA NA NA

PV n1T1 + n2T2 + n3T3


i.e., versus m graph is straight line passing Þ Tmix = n1 + n2 + n3
T
through origin with slope R/M, i.e. the slope depends 22. (d) We know that P V = n R T = (m/M) R T
on molecular
different gases.
EBD_7504
S-P-38 SOLUTIONS

æmö æ m ö 13R
Now P ´ ç ÷ = ç ÷ R T ...(1) Þ = C v1 + 2C v2
è d ø èMø 2
where d = density of the gas 3R 5R
Possible values are, Cv1 = , C v2 =
2 2
P R T k NAT \ Gases are monatomic (like He) and diatomic (like
= = ... (2)
d M M N2)
whereR = k NA, k is Boltzmann constant. 27. (c) According to Dalton’s Law
M P = P1 + P2.....................
But = m = mass of each molecule so here 1 litre of orygen at a pressure of 1atm, & 2 litre of
NA
nitrogen at a pressure of 5 atm are introduced in a vessel
P´m of 1 litre capacity.
d=
kT
V1P1 + P2 V2 1 ´ 1 + 2 ´ .5
3KT so P = = = 2atm
23. (b) For 1 molecule of a gas, vrms = volume of vessel 1
m
where m is the mass of one molecule dU d éM N ù é -6M 12N ù
28. (b) F= =- ê - ú = -ê + ú
3 12
dr dr ë r R û ë r2 r13 û
For N molecule of a gas, v1= 3KT ´ N In equilibrium position, F = 0
m
6M 12N 2N
3KI ´ N \ - 13 = 0 or, r 6 =
For 2N molecule of a gas v2 = ´N r2 r M
(2m)
\ Potential energy at equilibrium position
v1
\ =1 M N M2 M2 M2
v2 U= = = - =
(2N / M) (2N / M) 2 2N 4N 4N
m
24. (d) PV = RT 29. (a) Pressure of the gas will not be affected by motion of
M
the system, hence by
6
Initially, PV = R ´ 500
M 3P 3P 1
vrms = Þc2 = Þ P = pc 2
P (6 - x) r r 3
Finally, V = R ´ 300 (if x g gas leaks out)
2 M 30. (d) Let the mass of the gas be m.
6 5 At a fixed temperature and pressure, volume is fixed.
Hence, 2 = ´ \ x = 1 gram
6-x 3 m
Density of the gas, r =
nRDT 1000 ´ 8.3 ´ 7 V
25. (a) W = Þ - 146000 =
1- g 1- g r m m
Now = =
58.1 58.1 P PV nRT
or 1 - g = - Þ g = 1+ = 1.4
m
146 146 Þ = x (By question)
Hence the gas is diatomic. nRT
n1Cv1 + n 2 Cv2 Þ xT = constant Þ x1T1 = x2T2
26. (c) Cv mix =
n1 + n 2 é\ ù
x1T1 283 ê
13R n1Cv1 + 2n1Cv2 é n1 1 ù Þ x2 Þ = x T1 = 283K úú
Þ = êQ n = 2 ú T2 383 ê
êëT2
6 n1 + 2n1 ë 2 û = 383K úû
Speed Test-13

1. (a) For an SHM, the acceleration a = -w 2 x where w 2 is 1 æ 1ö


3
a or, =ç ÷ (Using eq. (i))
a constant. Therefore, is a constant. The time period n è 3ø
x
aT \ n = 33
T is also constant. Therefore, is a constant. 5. (c) The net force becomes zero atthe mean point.
x
2. (b) The kinetic energy of a particle executing S.H.M. is Therefore, linear momentum must be conserved.
given by \ Mv1 = (M + m)v2
k k
1 2 2 2 MA1 = ( M + m) A2
K= ma w sin wt M m+M
2
æ k ö
where, m = mass of particle \ çç v = A M ÷÷
a = amplitude è ø
w = angular frequency Q A1 M = A2 M + m
t = time 1 A m+M
1 \ A = M
Now, average K.E. = < K > = < mw2 a2 sin2 wt > 2
2 6. (c)
1 Figure shows the rod at an angle lq
= mw2a2 <sin2 wt >
2 q with respect to its equilibrium 2
1 æ 1ö æ 2 1ö position. Both the springs are
= mw2a2 çè ÷ø çè Q < sin q > = ÷ø q
2 2 2 lq l/2
stretched by length . M
1 1 2
= mw 2 a 2 = ma2 (2pn) 2 (Q w = 2 pn)
4 4 The restoring torque due to the
2 2 2 springs
or, < K > = p ma n lq
t = –2 (Restoring force) ×
m 4p m 2 4 p2 m perpendicular distance 2
3. (b) t1 = 2p or t12 = or k1 =
k1 k1 t12 æ lq ö l l2
t = -2k ç ÷ ´ = - k q ... (i)
4 p2m 4p 2 m è 2 ø 2 2
Similarly, k 2 = and (k1 + k 2 ) =
t 02 If I is the moment of inertia of the rod about M then
t 22
d 2q
4 p2m 4 p 2m 4 p2m 1 1 1 t = Ia = I … (ii)
\ = + or = + dt 2
t 02 t12 t 22 t 02 t12 t 22
From (i) & (ii) we get
4. (b) Amplitude of a damped oscillator at any instant t is given
by d 2q l 2q d 2q k l2 -k l2
I 2
= -k Þ =- q= q
A = A0e–bt/2m dt 2 dt 2 I 2 2
M l /12 2
where A0 is the original amplitude d 2q 6k
From question, Þ 2
=- q
dt M
A Comparing it with the standard equation of rotational
When t = 2 s, A = 0
3 SHM we get
A0 d 2q
\ = A0e–2b/2m = -w 2q Þ w 2 = 6k Þ w = 6k
3
dt 2 M M
1
or, = e–b/m … (i) 6k 1 6k
3 Þ 2 pv = Þ v=
A M 2p M
When t = 6 s, A = 0
n Mg
7. (c) = P0
A0 A
\ = A0e–6b/2m
n g
PV = PV g
1
or, = e–3b/m
n
EBD_7504
S-P-40 SOLUTIONS

Piston \U max + 0 = E Þ U max = E


Mg = P0A … (1)
Further,
P0 Ax0 g = PA( x0 - x ) g
x 1
K .E. = mw 2 a 2 cos 2 wt
g 2
P0 x0 x0 Cylinder
P=
( x0 - x ) g containing But by question, K .E. = K0 cos 2 wt
ideal gas
Let piston is displaced by distance x 1
\ K 0 = mw 2 a 2
2
æ P xg ö
Mg - ç 0 0
÷ A = Frestoring 1 2 2
çè ( x - x) g ÷ø Hence, total energy, E = mw a = K 0
0 2
æ x0g ö \U max = K 0 & E = K0 .
P0 A ç 1 - ÷ = Frestoring [ x ? x] 11. (b) Distance covered by lift is given by
çè ( x - x ) ÷øg 0
0 y = t2
gP Ax \ Acceleration of lift upwards
F=- 0
x0 d2 y d g
\ Frequency with which piston executes SHM. = 2
= (2t) = 2 m / s 2 =
dt dt 5
1 gP0 A 1 gP0 A 2 l
f = = Now, T = 2p
2p x0 M 2p MV0 g
8. (b) Equation of displacement is given by l l 5
T ' = 2p = 2p = T.
x = A sin(w t + f ) g 6 6
g+ g
5 5
F0 F0 bt
where A = = 2 2
-
2 2 2 m (w 0 - w ) 2m
m (w 0 - w ) 12. (c) Q A = A 0e
Here damping effect is considered to be zero (where, A0 = maximum amplitude)
According to the questions, after 5 second,
1
\x µ b(5)
2 2 -
m (w 0 - w ) … (i)
0.9A 0 = A 0e 2m
9. (a) Here, After 10 more second,
x = x0 cos (wt – p / 4 ) b(15)
-
dx æ pö A = A0 e 2m …(ii)
\ Velocity, v = = - x0w sin çw t - ÷
dt è 4ø From eqns (i) and (ii)
Acceleration, A = 0.729 A0
\ a = 0.729
dv 2 æ pö
a= = - x0w cos çw t - ÷ 13. (d) In simple harmonic motion, starting from rest,
dt è 4ø At t = 0 , x = A
x = Acoswt ..... (i)
é æ pö ù
= x0 w 2 cos ê p + ç wt - ÷ ú When t = t , x = A – a
ë è 4øû When t = 2 t , x = A –3a
3p ö From equation (i)
æ
= x0 w 2 cos ç wt + ÷ ...(1) A – a = Acosw t ......(ii)
è 4ø A – 3a = A cos2w t ....(iii)
Acceleration, a = A cos (wt + d) ...(2) As cos2w t = 2 cos2 w t – 1...(iv)
Comparing the two equations, we get From equation (ii), (iii) and (iv)
3p A - 3a æ A-a ö
2
A = x0w2 and d = . = 2ç
4 ÷ -1
A è A ø
10. (c) We have, U + K = E
where, U = potential energy, K = Kinetic energy, E = A - 3a 2 A2 + 2a 2 - 4 Aa - A2
Þ =
Total energy. A A2
2a2 – 4Aa
Physics S-P-41

Þ A = 2a Time period of oscillation,


a 1 m1 + m 2 1.5 2p p
Þ = T = 2p = 2p = = s
A 2 k 600 20 10
Now, A – a = A coswt
bt
A- a –
Þ cos wt = 16. (d) As we know, E = E0 e m
A b15
2p p –
1 15 = 45e m
Þ cos wt = or t=
2 T 3 [As no. of oscillations = 15 so t = 15sec]
Þ T= 6t
b15
14. (b) Loss in PE of mass = gain in PE of the spring 1 –
=e m
1 2 3
\ mg (h + x) = kx
2 Taking log on both sides
kx 2 – 2mgx – 2mgh = 0 b 1
= ln3
2 2 m 15
2mg ± 4m g - 4k (-2mgh)
Þ x= 17. (b) y
2k

mg m 2g 2 2mgh B a
or x= ± + P
k k2 k q
mg m 2g 2 2 mgh mg l
x = A– = + -
k 2 k k
k

é 2kh ù
mg A x
ê 1+
= - 1ú O x M
êë mg
k úû
dq
15. (a) Applying linear momentum conservation, = 2 \ q = 2t
dt
0.5 × 3 = (1 + 0.5) v or v = 1 m/s
Let BP = a, \ x = OM = a sin q = a sin (2t)
By conservation of energy, Hence M executes SHM within the given time period
mean position and its acceleration is opposite to x that means towards
left.
18. (a) The displacement of a particle in S.H.M. is given by
///////////////

3m/s y = a sin (wt + f)


0.5kg 1 kg
dy
//////////////////////////////////////////////////// velocity = = wa cos (wt + f)
dt
The velocity is maximum when the particle passes
After collision through the mean position i.e.,

1 1 1.5 æ dy ö
çè ÷ø
(1 + 0.5) v2 = kA 2 Þ A = ´v dt max = w a
2 2 k
mean position
The kinetic energy at this instant is given by
2
1 æ dy ö 1
m çè ÷ø = mw2 a2 = 8 × 10–3 joule
2 dt max 2
///////////////

1
or × (0.1) w2 × (0.1)2 = 8 × 10–3
0.5kg 1 kg 2
////////////////////////////////////////////////////
Solving we get w = ± 4
Substituting the values of a, w and f in the equation of
S.H.M., we get
y = 0.1 sin (± 4t + p/4) metre.
1.5 1 19. (d) Let the extension in the spring be x0 at equilibrium. If
Þ A= ´1 = m = 0.05m F0 be the tension in the string then F0 = kx0. Further if
600 20
T0 = mg and
A = 5 cm.
EBD_7504
S-P-42 SOLUTIONS

Let the mass m be displaced through a slight æ4 ö 1000


displacement x downwards. Let the the new tension in Net force = çè - 1÷ø ´ 1000 Vg = Vg
3 3
the string and spring be T and F respectively.
1000 Vg g
æ xö g eff = =
Þ F = ç x 0 ÷ and F = 2T 4 4
è 2ø 3 ´ ´ 1000 V
3
æ xö
Þ 2T = k ç x 0 ÷ l
è 2ø \ t = 2p
g/4
kx t = 2t0
Þ (T - T0 ) =
4 24. (a) The two springs are in parallel.
m m \ Effective spring constant, K = K1 + K2
Þ Time period = 2p = 4p
k k Now, frequency of oscillation is given by
4
1 K 1 K1 + K 2
f = or,, f = ....(i)
l 2p m 2p m
20. (c) As we know, time period, T = 2p
g When both k1 and k2 are made four times their original
When additional mass M is added then values, the new frequency is given by
l + Dl 4 K1 + 4 K 2
TM = 2p 1
g f '=
2p m
l + Dl æ TM ö
2
l + Dl
TM = or ç = 1 4( K1 + 4K 2 ) æ 1 K1 + K 2 ö
l è T ÷ø l = = 2ç
T 2p m è 2p m ÷ø
æ TM ö
2
Mg é Mgl ù
êëQ Dl = AY úû
or, ç = 1+ =2f; from eqn. (i)
è T ÷ø AY 25. (a) K.E. of a body undergoing SHM is given by,
1 é æ TM ö ù A
2
1 2 2 2
\ = êç ÷ - 1ú K .E. = ma w cos wt
Y êë è T ø úû Mg 2
21. (b) For block A to move in S.H.M. 1 2 2
T .E. = ma w
N 2
Given K.E. = 0.75 T.E.
A
2 p
Þ 0.75 = cos wt Þ w t =
6
mg x p p´2 1
mean Þt= Þt= Þ t= s
position 6´w 6 ´ 2p 6
mg – N = mw2x 1 2 2 2
where x is the distance from mean position 26. (a) K. E. = mw (a – y )
For block to leave contact N = 0 2
At mean position, y= 0
g
Þ mg = mw2 x Þ x = 2 1 2 2
w \ K. E. = mw a
2
1 2 2 1
22. (d) K.E = k ( A - d ) P. E. = mw y
2 2
2 2
1 2
and P.E. = kd a 1 a2 1 2 2
2 At , y = ; P. E. = mw2 = mw a
At mean position d = 0. At extrement positions d = A 2 2 4 8

l (1 / 2)mw 2 a 2
l \ Ratio =
23. (a) t = 2p ; t 0 = 2p (1 / 8)mw2 a 2
g eff g
K.E. at mean position 4
Buoyant = =
force 1000 Vg PE at a / 2 1
27. (a) Under the action of first force, F1 = mw12 y
4 Under the action of second force,

Weight
Physics S-P-43

Under the action of resultant force, Force applied by the spring is F = – kx


F1 + F2 = mw y2 Þ F = – k (2Lq)
(q is the angular displacement from the equilibrium
Þ mw 2 y = mw12 y + mw 22 y position). Further
Þ w 2 = w12 + w 22 r r
t =| l ´ F |= 4L2 k sin q = -4L2 k q
2 2 2
æ 2p ö æ 2p ö æ 2p ö
Þç ÷ = ç ÷ +ç ÷ ..
è Tø è T1 ø è T2 ø Also, t = Ia = I q = - 4L2 kq
2 2 .. 3k
æ 4 ö æ 3ö Þ q+ q=0
T12T22 çè ÷ø × çè ÷ø M
5 5 12
ÞT= = = .
T12 + T22 æ 4ö
2
æ 3ö
2
25 3k
Þ w0 =
çè ÷ø + çè ÷ø M
5 5
28. (d) F = – bV, b depends on all the three i.e, shape and size 30. (c) When the bob moves from maximum angular
of he block and viscosity of the medium. displacement q to mean position, then the loss of
gravitational potential energy is mgh
1 4 where h = l(1 – cos q)
29. (c ) I = M(2L) 2 = ML2
3 3
EBD_7504
S-P-44 SOLUTIONS

Speed Test-14

1. (a) Equation of the harmonic progressive wave given by : Dl


y = a sin 2p (bt – cx). l = 1.5 m, = 0.01,
l
Here u = b
r = 7.7 × 103 kg/m3 (given)
2p 1
k= = 2p c take, = c g = 2.2 × 1011 N/m2 (given)
l l
Dl
1 b Putting the value of l, , r and g in eqn. (i) we get,
\ Velocity of the wave = ul = b = l
c c
dy 2 103
f = ´
dt
= a 2pb cos 2p (bt – cx) = aw cos (wt – kx) 7 3 or f » 178.2 Hz
Maximum particle velocity = aw= a2pb = 2p ab
v
b 2 1 5. (a) f0 =
given this is 2 ´ i.e. 2pa = or c = 2l
c c pa
Now beat frequency = f1 – f2
2. (b) Speed of pulse at a distance x /////////////////
v é ù
from bottom, v = gx . v v vê 1 1 ú
= - = ê - ú
While traveling from mid point x æl ö æl ö 2 ê l - Dl l + Dl ú
2 ç - Dl÷ 2 ç + Dl÷
to the top, frequency remains è2 ø è2 ø ë2 2 û
unchanged. =
é 2 2 ù
v1 v2 g (L / 2) gL (f 0 l) ê -
= = Þ l 2 = 2l 0 ë l - 2 Dl l + 2Dl úû
l1 l 2 Þ l0 l2
é l + 2Dl - l + 2 Dl ù æ 4 Dl ö 8f 0 Dl
H = 2 f0l ê ú » 2f 0 l ç »
2 2 è l 2 ÷ø l
A 0.9 km B I ëê l - 4 (Dl) ûú
3. (d)
ENGINE L 6. (a) Here, original frequency of sound, f0 = 100 Hz
C L Speed of source Vs = 19.4 cos 60° = 9.7
Let after 5 sec engine at point C
19.4
AB BC
t= +
330 330
0.9 ´ 1000 BC
5= +
330 330
\ BC = 750 m
60°
Distance travelled by engine in 5 sec
S O
= 900 m – 750 m = 150 m 19.4 cos 60° = 9.7
Therefore velocity of engine
From Doppler's formula
150 m
= = 30 m/s æ V - V0 ö
5sec f = f0 ç ÷
4. (b) Fundamental frequency, è V - Vs ø
v 1 T 1 T æ V-0 ö
f = = = f = 100 ç ÷
2l 2l m 2l Ar è V - ( +9.7) ø
V
é T mù f = 100
êQ v = and m = ú æ 9.7 ö
ë m lû V ç1 - ÷
è V ø
Tl T Y Dl æ 9.7 ö
Also, Y = Þ =
ADl A l f = 100 ç 1 + ÷ = 103Hz
è 330 ø
1 gDl
Þ f =
2l lr
Physics S-P-45

7. (a) For fundamental mode, 1/ 2


f' 2r - rw æ 2r - rw ö
= f '= f ç
1 T f 2r è 2r ø÷
f=
2l m 1/ 2
Taking logarithm on both sides, we get é 2r - 1 ù
= 300 ê ú Hz
æ 1ö æ Tö ë 2r û
log f = log ç ÷ + log ç ÷ 11. (a) Given wave equation is y(x,t)
è 2l ø è mø
= e(- ax +bt 2 + 2 ab xt )
2

æ1 ö 1 æTö
= log çè ÷ø + log ç ÷
2l 2 èmø =e
-[( ax )2 + ( b t ) 2 + 2 a x . b t ]

æ1ö 1 -( ax + bt ) 2
or log f = log ç ÷ + [log T - log m] =e
è 2l ø 2
2
Differentiating both sides, we get æ
b ÷
ö
- çè x + tø
df 1 dT = e a
= (as l and m are constants)
f 2 T It is a function of type y = f (x + vt)
dT df
Þ = 2´ b
T f Þ Speed of wave =
a
Here df = 6
f = 600 Hz f0v
12. (c) f2 =
dT 2 ´ 6 v + v0
\ = = 0.02 The wave which reaches wall f1 is reflected.
T 600
8. (b) Frequency received by listener from the rear source, f0 v
f1 =
v -u v -u v v-u v - v0
n¢ = ´n = ´ =
v v l l The reflected frequency is f1 as the wall is at rest.
Frequency received by listener from the front source, f0 v fv 2 f 0 vv0
Beats = f1 - f 2 = - 0 =
v+u v v+u v - v0 v + v0 v 2 - v0 2
n¢¢ = ´ =
v l l 13. (b) Given f A = 1800Hz
No. of beats = n'' – n'
vt = v
v+u v -u v + u - v + u 2u fB = 2150 Hz
= - = =
l l l l
Reflected wave frequency received by A, f A¢ = ?
9. (c) Given : Wavelength (l) = 5000 Å
velocity of star (v) = 1.5 × 106 m/s. Applying doppler’s effect of sound,
We know that wavelength of the approaching star (l') = vs f
f¢ =
c -v vs - v t
l
c
æ f ö
l' c -v v here, v t = vs ç1 - A ÷
or, = = 1- è fB ø
l c c
v l ' l - l ' Dl æ 1800 ö
or, = 1 - = = . Therefore, = 343 ç1 - ÷
c l l l è 2150 ø
v 1.5 ´ 106 vt = 55.8372 m/s
Dl = l ´ = 5000 ´ = 25 Å. Now, for the reflected wave,
c 3 ´ 108
[where Dl = Change in the wavelength] æ v + vt ö
\ f A¢ = ç s ÷ fA
10. (a) In air : T = mg = rVg è vs - v t ø
1 rVg æ 343 + 55.83 ö
\ f= ... (i)
2l m =ç ÷ ´1800
è 343 - 55.83 ø
In water : T = mg – upthrust
= 2499.44 » 2500Hz
V Vg
= Vrg – rw g = (2r - rw ) 14. (c) Length of pipe = 85 cm = 0.85m
2 2 Frequency of oscillations of air column in closed organ
Vg pipe is given by,
(2
1 2
\ f '=
2l m
EBD_7504
S-P-46 SOLUTIONS

(2 n - 1)u For rotational equilibrium of massless rod, taking torque


f = £ 1250 about point O.
4L
TAB × x = TCD (L – x) ... (ii)
(2n - 1) ´ 340
Þ £ 1250 For translational equilibrium,
0.85 ´ 4 TAB + TCD = mg ... (iii)
Þ 2n – 1 < 12.5 » 6 On solving, (i) and (iii), we get
é æ t ö x ù mg 4mg
15. (d) y = 0.02(m)sin ê 2 p ç ÷ - 0.50(m) ú TCD = \ TAB =
ë è 0.04( s ) ø û 5 5
Substituting these values in (ii), we get
But y = a sin(wt - kx ) L
4mg mg
´x= ( L - x) Þ x =
2p 1 5 5 5
\ w= Þ n= = 25 Hz
0.04 0.04
v 3v l1 1
2p 18. (b) = , \ =
k= Þ l = 0.5 m 4 l1 2 l 2 l2 6
0.50 19. (b) Here, T = 0.05 sec, v = 300 ms–1.
\ velocity, v = nl = 25 × 0.5 m/s = 12.5 m/s
v
Velocity on a string is given by Now l = = vT = (300 ´ 0.05)m
v
T or, l = 15 m
v= 2 = (12.5)2 × 0.04 = 6.25 N
m \ T = v ´m Phase of the point at 10 m from the source
æI ö æ I2 ö 2p 2p 4p
16. (a) We have, L1 = 10log ç 1 ÷ ; L2 = 10 log ç ÷ = ´x= ´ 10 = rad
è I0 ø è I0 ø l 15 3
Phase of the point at 15 m from the source
æI ö æI ö
\ L1 – L2 = 10 log ç 1 ÷ - 10log ç 2 ÷ 2p 2p
è I0 ø è I0 ø ´x = ´ 15 = 2p rad
l 15
æI I ö æI ö \ The phase difference between the points
or, DL = 10 log ç 1 ´ 0 ÷ or, DL = 10log ç 1 ÷
è I0 I 2 ø è I2 ø 4 p 2p
= 2p - = rad
3 3
æI ö æI ö
or, 20 = 10log ç 1 ÷ or, 2 = log ç 1 ÷ 20. (b) From figure, tension T1 = m2g
è I2 ø è I2 ø T2 = (m1 + m2)g
Rigid support
I1 I As we know
or, = 102 or, I2 = 1 . Velocity µ T2
I2 100 T So,
Þ Intensity decreases by a factor 100. lµ T
L m1
1 TAB l1 T
17. (a) Frequency of Ist harmonic of AB = = 1
2l m Þ l2 T1
T2
1 TCD m2
Frequency of 2nd harmonic of CD =
l m l2 m1 + m2
Þ =
Given that the two frequencies are equal. l1 m2
1 TAB 1 TCD TAB
\ = Þ = TCD
2l m l m 4
l1 l2 l3
Þ TAB = 4TCD ... (i) 21. (b)
A B
1 T
n=
l AB 2l m
=l l CD = l
2 1
TAB TCD or, n µ or nl = constant, K
l
O \ n1l1 = K,
B x L–x D

m
Physics S-P-47

K K K K 25. (b) l1 = 50 cm. l2 = 51 cm.


or, n = n + n + n
1 2 3 v1 T2 273 + 20
vµ T Þ = = Þ v2 = 319.23.
1 1 1 1 v2 T1 273
or, n = n + n + n
1 2 3 v 2 319.23
22. (b) Let the string vibrates in p loops, wavelength of the n1 = = = 640 Hz.
l1 0.50
pth mode of vibration is given by
v2 319.23
2l n2 = = = 625.94 = 626 Hz.
lp = l 2 51 ´ 10-2
p
No. of beats = n2 – n1 = 14 Hz
æ 4px ö
Given, y = 2 sin ç ÷ cos(96pt) 26. (c)
è 15 ø
l1 +e
é æ 4px ö æ 4px ö l2 +e
or y = 2 êsin ç + 96pt ÷ + sin ç – 96 pt ÷
ë è 15 ø è 15 ø
Comparing it with standard equation, we get
96 p 4p
u= = 48 Hz and k =
2p 15
1 2 ´ 60 4p
= ´
48 p 15 ´ 96 p For first resonance For second resonance
Þ p = 16. l 3l
l1 + e = l2 + e =
23. (b) n¢ = nv .......(1) 4 4
v - vs But v = nl
nv 4 3v
n¢¢ = .......(2) \ v = n (l 2 + e ) Þ l2 + e = ...(i)
v + vs 3 4v
n ¢ v + vs v
From (1) and (2), n ¢¢ = v - v .......(3) \ v = n 4( l1 + e) Þ l1 + e = ... (ii)
s 4n
n¢ 5 Subtracting (i) and (ii),
According to question, =
n ¢¢ 4 v = 2n (l 2 - l1 ) \ Dv = 2n ( Dl 2 + Dl1 )
vs = ? v = 330 m/s .........(4) = 2 × 512 × (0.1 + 0.1) cm/s = 204.8 cm/s
From eq. (3) and (4) 27. (a) The frequency of vibration of a string is given by,
5 é 330 + vs ù 1 T
=ê ú f= where m is mass per unit length.
4 ë 330 - vs û 2l m
9vs = 330 1 T 1 T
\ vs = 36.6 m/s f1 = , f2 = ,
2l1 m 2l2 m
24. (d) Load supported by sonometer wire = 4 kg
Tension in sonometer wire = 4 g
1 T (l1 - l2 )
If m = mass per unit length f2 – f1 =
2 m l1 l2
1 T
then frequency u =
2l m T 20
= –3
= 2 ´ 102 = 1.414 × 100
1 4g m 10
Þ 416 =
2l m = 141.4
When length is doubled, i.e., l¢ = 2l l1 - l2 (51.6 - 49.1) ´ 102
Let new load = L l1 l2 = 51.6 ´ 49.1
As, u¢ = u
2.5 ´ 102 1
= =
1 Lg 1 4g 50 ´ 50 10
\ =
2l ¢ m 2l m 1 1
\ f2 – f1= ´ 141.4 ´ = 7 beats
2 10
1 Lg 1 4g 28. (b) Fundamental frequency of closed organ pipe
Þ =
4l m 2l m

Þ L = 2´ 2
EBD_7504
S-P-48 SOLUTIONS

Fundamental frequency of open organ pipe (2A)w = 3.14


V 3.14
V0 = Amplitude 2A = = 0.1m
2l0 2 ´ (3.14) ´ 5
3V Equation of standing wave is
Second overtone frequency of open organ pipe = y = (0.1) sin (p/2) x sin (10p) t
2l0
From question, 30. (b) y = 60 cos (180t – 6x) ....(1)

V 3V 2p
= w = 180, k = 6 Þ =6
4lc 2l0 l

Þ l0 = 6lc = 6 × 20 = 120 cm w 2 p l 180


v= = ´ = = 30 m / s
k T 2p 6
1.2
29. (d) m= = 0.6 kg / m Differentiating (1) w.r.t. t,
2
dy
f = 5 Hz, l = 2l = 4m v= = -60 ´ 180 sin(180 t - 6 x )
dt
v = nl = 5 × 4 = 20 m/s
v max = 60 ´180 mm / s
T
Using v = Þ T = 202 ´ 0.6 = 240 N = 10800 mm / s = 0.0108 m / s
m
æ ¶y ö v max 0.0108
çè ÷ø = 3.14 m / s = = 3.6 ´10 - 4
¶t max v 30
Speed Test-15

Charge work done in moving through distance S,


1. (a) Surface charge density (s) = ® ®
Surface area W = qE . S = (qE) ´ S´ cosq
– Q + 2Q = Q \ 10 J = (0.5 C) × E × 2 cos 60°
-2Q E = 10 × 2 = 20 NC–1 = 20 Vm–1
So sinner = –2Q
4pb2
(b) From figure, q = p + a , where
a b 7.
Q c 3
and sOuter = 2
+2Q
4pc E 2 æ p sin q ö æ 4p Î0 r 3 ö 1
2. (a) Electric field intensity at the centre of the disc. tan a = =ç ÷ç ÷ = tanq
E1 è 4 p Î0 r 3 ø è 2p cos q ø 2
s
E= (given)
2 Î0
Electric field along the axis at any distance x from the æ pö 1 p 3
tan ç q - ÷ = tan =
centre of the disc è 3ø 2 3 2

æ
s x ö p -1 æ 3 ö
ç1 - ÷ E E
E' =
ç
2 Î0 ÷ or q - 3 = tan ç 2 ÷ a 1
è x2 + R 2 ø è ø E2 p/3
From question, x = R (radius of disc) P
æ 3ö
æ ö or a = tan -1 ç ÷
\E' = s ç 1-
R
÷ ç 2 ÷
è ø p/3
2 Î0 ç ÷ –q +q
è R2 + R2 ø O
p æ 3ö
s æ 2R - R ö or q = + tan -1ç ÷
= ç ÷ 3 ç 2 ÷
2 Î0 çè è ø
2R ÷ø
8. (b) Let us consider a spherical shell of radius x and
4 thickness dx.
= E
14
\ % reduction in the value of electric field
æ 4 ö dx
çè E - E÷ø ´ 100 x
14 1000
= = % ; 70.7% O
E 14
r Shell
3. (d) Since electric field E decreases inside water, therefore
r r
flux f = Ε.A also decreases.
Charge on this shell
4. (c) When a dipole is placed in a uniform electric field, two
equal and opposite forces act on it. Therefore, a torque æ xö
dq = r.4px2dx = r0 ç1 - ÷ .4px 2dx
acts which rotates the dipole. è Rø
5. (a) For the distances close to the charge at x = 0 the field is \ Total charge in the spherical region from centre
very high and is in positive direction of x-axis. As we to r (r < R) is
move towards the other charge the net electric field
r
becomes zero at x = a thereafter the influence of charge æ xö
q = ò dq = 4pr 0 ò ç1- ÷ x 2dx
at x = 2a dominates and net field increases in negative è Rø
0
direction of x-axis and grows unboundedly as we come
r
closer and closer to the charge at x = 2a. é x3 x 4 ù
6. (d) Force acting on the charged particle due to electric = 4pr0 ê - ú
® êë 3 4R úû 0
field = qE
S é r3 r 4 ù
= 4pr0 ê - ú
ëê 3 4R úû

60°
EBD_7504
S-P-50 SOLUTIONS

Let us assume flux linked with the plane surfaces A and


1 .q
\ Electric field at r, E = C be
4pe 0 r 2 fA = fC = f'
Therefore,
1 4pr0 r 3 é 1 r ù
= . ê - q
4pe 0 r 2
ë 3 4R úû = 2f '+ f B = 2f '+ f
e0
r0 é r r 2 ù 1æ q ö
= ê - ú Þ f' = ç - f÷
e0 êë 3 4R úû 2 è e0 ø
9. (a) The flux is zero according to Gauss’ Law because it is a p
13. (d) E =
open surface which enclosed a charge q. 4pe 0 .r 3
10. (b) O a 1
–q q C Apparently, E µ p and E µ 3 µ r -3 .
r
14. (a) Let us consider a spherical shell of radius x and thickness
r dx. The volume of this shell is 4px2(dx). The charge
Electric field at C due to electric dipole enclosed in this spherical shell is
1 2p
= along OC
4pe 0 r3 dx
Electric field at C due to induced charge must be equal x

and opposite to electric field due to dipole as net field


at C is zero.
11. (a) l = linear charge density;
Charge on elementary portion dx = l dx.
dE dq = (4p x 2 )dx ´ kx a

q \ dq = 4p kx 2 + a dx .
O For r = R :
The total charge enclosed in the sphere of radius R is
q
a
dq

R
R 3+ a
dx
x A ò0
Q = 4p k x 2 + a dx = 4p k
3+ a .
ldx
Electric field at O, dE = \ The electric field at r = R is
4pe 0 a 2
1 4pkR3+ a 1 4pk 1+ a
Horizontal electric field, i.e., perpendicular to AO, will be E1 = = R
4pe 0 (3 + a ) R 2 4 pe0 3 + a
cancelled.
Hence, net electric field = addition of all electrical fields For r = R/2 :
in direction of AO The total charge enclosed in the sphere of radius R/2 is
= SdE cos q R/2
4pk ( R / 2)3+ a
ÞE=ò
ldx
cos q
Q' = ò0 4pk x 2 + a dx =
3+ a
4pe 0 a 2
\ The electric field at r = R/2 is
dx 1+ a
Also, dq = or dx = adq 1 4pk ( R / 2)3+ a 1 4 pk æ R ö
a E2 = = ç ÷
p/2 4 pe 0 3 + a ( R / 2) 2 4pe 0 3 + a è 2 ø
l cos qd q l
E= ò 4 pe a
=
4 pe a
[ sin q]p– p/ 2/ 2 1
-p / 2 0 0 Given, E2 = E1
8
l l
= 4pe a [1– (–1)] = 2pe a 1 4 pk æ R ö 1
1+ a
1 4 pk 1+ a
0 0 \ çè ÷ø = ´ R
q 4pe0 (3 + a ) 2 2 3 4pe 0 3 + a
12. (d) Since f total = f A + fB + fC = ,
e0 Þ 1+a=3 Þ a=2
where q is the total charge. 15. (a) Since lines of force starts from A and ends at B, so A is
+ve and B is –ve. Lines of forces are more crowded near
surface B is f = fB
Physics S-P-51

L 1 (2q) (- q) F
16. (c) T0 = 2p Now, force F¢ = Þ F¢ =
4pe0 2 8
g r
When the plates are charged, the net acceleration is, 22. (d) Electric flux, f = EA cos q ,
g' = g +a where q = angle between E and normal to the surface.
qE æ qE ö p
g' = g + çè a = ÷ Here q =
m mø 2
L Þ f=0
\ T = 2p
qE 23. (c) Force of interaction
g+
m +q +q
1/ 2
T æ g ö 1 6p1p 2
\ = F= .
T0 çç g + qE ÷
÷
4p Î0 (x)4 p1 p2
è m ø
–q –q
17. (d) Electric displacement vector, D = eE x
As, e = e 0 K \ D = e0K E r
24. (c) E inside the charged sphere
18. (b) Force on charge q1 due to q2 is
q1q2 P
F12 = k 2 R
b
O P Q
Force on charge q1 due to q3 is
qq
F13 = k 1 3 r
a2 Ein = 0 …(i)
The X - component of the force (Fx) on r
q1 is F12 + F13 sin q E on the surface of the charged
sphere
q1q2 q1q2 r 1 q r 1
\ Fx = k 2
+k sin q Es = 2
i.e., Es µ nˆ …(ii)
b a2 4p Î0 R R2
q2 q3 r
\ Fx µ + sin q E on any point away from the uniformly c h a r g e d
2
b a2 sphere is given
r 1 q
E= nˆ
F12 4p Î0 r 2
q
F13 sin q r 1
E µ 2 nˆ … (iii)
r
Q R is the radius of the sphere, which is constant,
F13 r
F13cos q thus E is maximum and constant at the surface of the
4 3 sphere. But decreases on moving away from the
19. (c) Net downward force on the drop = pr (r - r0 ) g surface of the uniformally charged sphere.
3
For equilibrium, electric force must be upwards i.e. 25. (c) K.E. = Force × distance = qE.y
charge on the drop is positive. 26. (b) The path of the particle will be as shown in figure. At the
point of minimum distance (D) the velocity of the particle
4 3 4pr 3 (r - r0 ) g will be ^ to its position vector w.r.t. +Q.
neE = pr (r - r0 ) g i.e. n =
3 3eE 1 1 kQq
r r mu 2 + 0 = mv 2 + ............. (1)
20. (a) f = E. A = 4iˆ.(2iˆ + 3 ˆj ) = 8 V-m 2 2 rmin

1 (4q) (-4q)
21. (b) F = 4pe v D
0 r2
when C is touched with A, then charge on A & C each = B
u
2q after that C is touched with B, charge on
r min
2q + (-4q)
B= = -q d
2
A
+Q C
EBD_7504
S-P-52 SOLUTIONS

Q Torque on q about Q is zero hence angular momentum æ q ö


about Q will be conserved. dq = ç ÷ dl
Þ mvrmin = mud ............. (2) è pr ø
q
By eq. (2) in eq. (1) = (rd q) (Qdl = rd q)
2
pr
1 1 æ ud ö kQq æqö
mu 2 = m ç +
2 ÷
2 è rmin ø rmin = ç ÷ dq
èpø
æ Electric field at O due to dq is
1 2 d 2 ö mu 2 d
mu ç 1 - =
Þ 2 2 ÷ rmin {Q kQq = mu2d } 1 dq 1 q
è rmin ø dE = . = . dq
4p Î0 r 2 4 p Î0 pr 2
2
Þ rmin - 2rmin d - d 2 = 0
The component dE cos q will be counter balanced by
2d ± 4d 2 + 4d 2 another element on left portion. Hence resultant field at O is
Þ rmin = = d (1 ± 2)
2 the resultant of the component dE sinq only.
Q distance cannot be negative p
q
\ rmin = d (1 + 2) \ E = ò dE sin q = ò 2 2
sin qd q
0 4 p r Î0
27. (b) It is possible to create or destroy charged particles but
q
it is not possible to create or destroy net charge. The
charge of an isolated system is conserved.
=
2 2
[ -cos q]p0
4p r Î0
uur ur
28. (b) Flux = E . A. q
q
uur ur = (+1 + 1) = 2 2
E is electric field vector & A is area vector.. 2 2
4p r Î0 2p r Î0
uur ur The direction of E is towards negative y-axis.
Here, angle between E & A is 90º.
uur ur r q
So, E . A = 0 ; Flux = 0 \ E=- 2 2 ˆj
2p r Î0
29. (c) Let us consider a differential element dl. charge on this
element. 30. (c) Let n be the number of electrons missing.
j 1 q2
F= ×
4pe 0 d 2
+
+

+ dl
+

+ dq
+
Þ q = 4pe 0 d 2 F = ne
+ dE q
+
cos q
i
O 4pe 0 Fd 2
dE \ n=
e2
dE sin q
Speed Test-16

1. (d) As volume remains constant, therefore,


C1 C2
4 4 Here, C = C
p R 3 = n ´ p r3 \ R = n 1 / 3 r. 3 4
3 3
Hence, no charge will flow through 20mF
New potential = V´= nq nq
= C1 C2 C'
4 p e o R 4 p e o ( n1 / 3 r )
q
= n2/3
= n 2 / 3 V. X Y ÞX Y
4 p eor
2. (c) Charges reside only on the outer surface of a conductor C3 C4 C''
with cavity.
C1 and C2 are in series, also C3 and C4 are in series.
Î0 A Î A 2 Î0 A (1 + K )
3. (a) C a = and Cb = 0 = Hence, C' = 3 mF, C'' = 3 mF
d d d d
+ C' and C'' are in parallel.
2 2K Hence net capacitance = C' + C'' = 3 + 3 = 6 mF
A A
Î0 Î0 K Î A 8. (c)
2 + 2 R
and C c = = 0 (1 + K)
d d 2d
Î0 A +q –q
or C b = 2(1 + K ) > C a
d A C B D
Î0 A 1 + K 2L L
or C c = > C a \ C b and C c > C a .
Potential at C = VC = 0
d 2
4. (d) Potential at D = VD
5. (c) Since, potential difference across C2 is greater than C1.
æ -q ö kq 2 kq
é q ù = kç ÷+ =-
Þ C1 > C2 êQ V = and q is same in series ú è L ø 3L 3 L
ë C û Potential difference
6. (b) In shell, q charge is uniformly distributed over its
surface, it behaves as a conductor. -2 kq 1 æ 2 qö
+
VD – VC = 3 L = 4p Î çè - . ÷ø
3 L
0
+
+ q Þ Work done = Q (VD – VC)
R + 2 1 qQ - qQ
+
+ =- ´ =
+ 3 4pe0 L 6pe 0 L
+
9. (a) Potential at B, VB is maximum
q
V= potential at surface = and inside VB > VC > VA
4pe 0 R
As in the direction of electric field potential de-
q creases.
V=
4pe 0 R 10. (b) At. equipotential surface, the potential is same at any
Because of this it behaves as an equipotential surface. point i.e., VA = VB as shown in figure. Hence no work is
C1 6mF required to move unit change from one point to another
i.e.,
C3 C5 6mF
7. (d) X Y W
A 6m F 20m F C2 VA - VB = =0Þ W =0
C4 unit ch arg e
6mF
Equivalent circuit equipotential
surface
VA VB
6m F C2
C1 6m F
6m F

X 20mF
6mF Y
C3
VAVB
6mF
EBD_7504
S-P-54 SOLUTIONS

11. (b) Now it is given that capacitance of vacuum = C0.


(i) Electrostatic field is zero inside a charged conductor or Q
neutral conductor. Thus, C =
V
(ii) Electrostatic field at the surface of a charged conductor
s.s
must be normal to the surface at every point. = (Let surface area of plates = s)
(iii) There is no net charge at any point inside the conductor v
and any excess charge must reside at the surface. s.s
=
(iv) Electrostatic potential is constant throughout the s æ ld ö
volume of the conductor and has the same value (as ln ç1 + ÷
l è K0 ø
insde) on its surface. d 1
(v) Electric field at the surface of a charged conductor is = sl. (Q in vacuum e0 =1)
d æ ld ö
r s ln ç1 + ÷
E= nˆ è K0 ø
e0 ld æ sö
C= . C0 çè here, C0 = ÷ø
e0 A æ ld ö d
12. (c) Ceff = since effective capacitance between ln ç1 + ÷
d A B è K 0ø
plates A and E is zero. 17. (b) Potential difference across the branch de is 6 V. Net
C D
1 2 e0 A 2 capacitance of de branch is 2.1 µF
\ U = CV = V E F So, q = CV
2 2d
Þ q = 2.1 × 6 µC
Þ q = 12.6 µ C
13. (c) 180V S Potential across 3 µF capacitance is
12.6
V= = 4.2 volt
3
Q2 Q1 Potential across 2 and 5 combination in parallel is 6 –
14. (c) R1 R2 4.2 = 1.8 V
r So, q' = (1.8) (5) = 9 µC
18. (d) Initial charge on capacitors C1 and C2 is given by,
q1 = C1V1 = 60 pC
Q2 Q1 q2 = C2V2 = 60pC
Vr = +
4 pe 0 r 4pe 0 R1 When S1 and S3 are closed, capacitors C1 and C2 get
connected in series. As a result charge on them should
1 æ Q 2 Q1 ö be same and so the charge do not redistribute on them.
Vr = ç + ÷
4 pe0 è r R1 ø So potential on them remains same.
15. (b) Charge on a particle, q = 2 e. q q
K.E. = work done = q × V = 2e × 106 V = 2 MeV. 19. (a)
16. (c) The value of dielectric constant is given as,
K = K 0 + lx
R R
d
And, V = ò Edr 1 2
0
d

d
s At (1) using, potential (V1 ) = Vself + Vdue to (2)
V=ò dx
K
0 1 éq q ù
Þ V1 = ê - ú
d
1 4pe 0 êë R R 2 + d 2 úû
= s ò ( K + lx ) dx
0
0 At (2) using potential (V2 ) = Vself + Vdue to (1)
s
= éln ( K 0 + ld ) - ln K 0 ùû é- q ù
lë Þ V2 =
1
ê +
q
ú
4pe 0 êë R R2 + d 2 úû
s æ ld ö
= l ln ç1 + K ÷
è 0ø
Physics S-P-55

éq q ù 25. (c) Let plate A plate B be carrying charges Q1 and Q2 re-


1 q q
= ê + - - ú spectively. When they are brought closer, they induce
4 pe 0 êë R R R2 + d 2 R 2 + d 2 úû equal and opposite charges on each other i.e. – Q2 on
1éq q ù plate A and – Q1 on plate B. Therefore, net charge on
= ê - ú plate A = Q1 – Q2 and net charge on plate B = – (Q1 –
2pe 0
êë R R 2 + d 2 úû Q2), so the charge on the capacitor = Q1 – Q2.
20. (c) All the charge given to inner sphere will pass on to the \ Potneital different between the plates
outer one. So capacitance that of outer one is 4p Î0 b .
21. (d) When S and 1 are connected Q1 – Q 2
V=
The 2mF capacitor gets charged. The potential C
difference across its plates will be V. 26. (c) The direction of electric field is always perpendicular
The potential energy stored in 2 mF capacitor to the direction of electric field and equipotential
surface maintained at high electrostatic potential to
1 1
Ui = CV 2 = ´ 2´ V 2 = V 2 other equipotential surface maintained at low
2 2 electrostatic potential.
When S and 2 are connected The positively charged particle experiences the
The 8mF capacitor also gets charged. During this electrostatic force in the direction of electric field i.e.,
charging process current flows in the wire and some from high electrostatic potential to low electrostatic
amount of energy is dissipated as heat. The energy potential. Thus, the work done by the electric field on
loss is the positive charge, so electrostatic potential energy
of the positive charge decreases because speed of
1 C1 C2 2
DU = 2 C + C (V1 - V2 ) charged particle moves in the direction of field due to
r
1 2 force qE.
Here, C1 = 2mF, C2 8 mF, V1= V, V2 = 0 27. (a) The equivalent circuit is shown in figure.
1 2´8 4 CAB = 3mF.
\ DU = ´ (V - 0) 2 = V 2
2 2+8 5 B
The percentage of the energy dissipated
4 2 2µF
DU V
= U ´ 100 = 5 1µF 1µF
´ 100 = 80%
i
V2 2µF
22. (c)
23. (c) Equipotential surfaces are normal to the electric field A
lines. The following figure shows the equipotential 28. (c) If we increase the distance between the plates its
surfaces along with electric field lines for a system of capacity decreases resulting in higher potential as we
two positive charges. know Q = CV. Since Q is constant (battery has been
disconnected), on decreasing C, V will increase.
1 Q 1 æ -2Q ö
29. (a) V = V1 + V2 + V3 = . + ç ÷
4p Î0 R 4p Î0 è R ø
1 æ 3Q ö 1 æ 2Q ö
+ ç ÷= ç ÷
4p Î0 è R ø 4p Î0 è R ø
30. (c) Applying conservation of energy,
Î0 A
24. (d) C = 1 2m. s. Dt
d CV 2 = m. s Dt ; V =
A ® common area, Here A = A1 2 C
EBD_7504
S-P-56 SOLUTIONS

Speed Test-17

1. (d) From the curve it is clear that slopes at points A, B, C, x


dx
D have following order A > B > C > D.
And also resistance at any point equals to slope of the
V-i curve.
So order of resistance at three points will be
RA > RB > RC > RD
V
Resistance of this elemental part is,
2. (d) R
rdx r0 xdx
dR = 2
=
R pa pa 2
L
r0 L2
r0 xdx
R R = ò dR = ò pa 2 2pa 2
=
0
R R
R R R R
R V V ´ 2pa 2
Current through cylinder is, I = =
R r0 L2
R
B
2V
A Potential drop across element is, dV = I dR = x dx
L2

dV 2V
R/3 E(x) = = x
dx L2
5. (a) The simplified circuit is
R/3 R/3 10V 4 B x(assume)
10 x–10
2 10 2 4
A R/3 B
10V 20V
3R R 10
´ 20
R2 P
Rnet between AB = 3 3 = = 4W O(assume)
3R R 4R
+ We have to find I.
3 3
Let potential of point P be 0. Potential at other points
rl 2 m are shown in the figure apply Kirchoff’s current law at
3. (d) R = 2
. But m = pr2 ld \ pr =
pr ld B where potential is assume to be x volt.
2 2 x - 10 x - 10 x - 20 (x - 10) - 0
rl 2 d R = rl1 d R = rl2 d + + + =0
\R = , 1 , 2 4 2 4 2
m m1 m2 Þ x – 10 + 2x – 20 + x – 20 + 2x – 20 = 0
35
rl32 d Þ 6x = 70 Þ x = volt
R3 = 3
m3
35
20 -
l12 2
l2 l3 2 \ I= 3 = 25 A
R1 : R2 : R3 = : : 4 12
m1 m2 m3
6. (c) In series, Rs = nR
25 9 1 1 1 1
R1 : R2 : R3 = : : = 125 :15 :1 In parallel, = + + ...n terms
1 3 5 R R R
4. (a)
Physics S-P-57

15. (d) In steady state capacitor is fully charged and no current


output
7. (a) Efficiency is given by h = flows through it.
input
1 1 1 1 6 + 3 + 2 11 6
5 ´ 15 ´ 14 = + + = = Þ Req = W
= = 0.875 or 87.5 % R eq 1 2 3 6 6 11
10 ´ 8 ´ 15
rl 6 ´ 11
8. (c) R= Current through battery = = 11A
A 6
When wire is cut into 4 pieces and connected in parallel. Q = CV = 0.5 × 10–6 × 6 = 3.0 × 10–6 C = 3mC
R 16. (d) The total volume remains the same before and after
R eff. = Þ PC = 16P stretching.
16
V 2 V2 V2 V2 Therefore A ´ l = A ' ´ l '
PA : PB : PC : PD ::
: : :
R R / 4 R / 16 R / 2 Here l ' = 2l
9. (c) By principle of symmetry and superposition,
A´l A´l A
I R \ A' = = =
2 ´ ´ R 0 = I R eq. Þ R eq. = 0 l' 2l 2
6 3 Percentage change in resistance
I
(Current in AB is due to division in current entering æ l' l ö
6 rç - ÷
R f - Ri A' A ø
= ´100 = è ´100
I Ri l
at A and current is due to current returning from r
6 A
infinity of grid).
10. (b) (i) When key between the terminals 1 and 2 is éæ l ' A ö ù é æ 2l A ö ù
= êç ´ ÷ - 1ú ´ 100 = êç A ´ ÷ - 1ú ´ 100
plugged in, ëè A ' l ø û êëè 2 l ø úû
P.D. across R = IR = k l1
= 300%
Þ R = k l1 as I = 1A
e v e El
(ii) When key between terminals 1 and 3 is plugged in, 17. (a) Vd =´ t or, Vd = . t [Q V = E l]
P.D. across (X + R) = I(X + R) = k l2 m l m l
Þ X + R = k l2 \ Vd µ E
\ X = k (l2 – l1) 18. (d) r ´ s = 1 = constant
\ R = kl1 and X = k (l2 – l1) r l1
11. (c) Resistance of bulb is constant 19. (c) R = , now l2 = 2l1
A1
V2 Dp 2DV DR
P= Þ = + A2 = p(r2)2 = p (2r1)2 = 4p r12 = 4A1
R p V R
r(2 l1 ) r l1 R
Dp \ R2 = = =
= 2 × 2.5 + 0 = 5% 4 A1 2 A1 2
p
\ Resistance is halved, but specific resistance remains
Ig R Ig the same.
12. (b) S = ÞS= R
nIg - I g (n - 1)I g 20. (d) From the principle of potentiometer, V µ l
13. (c) I = neAVd V l
Þ = ; where
I E L
Vd = = 5 × 10–3 m/sec V = emf of battery, E = emf of standard cell.
neA
l1 X 12 L = length of potentiometer wire
14. (b) If l1 = length from one end then 1 - l = R = 18
1 El 30E
12 V= =
l1 = m = 40cm. L 100
30
E
and l'1 = length from one end in second case
l1¢ X 12 i
= =
1 - l1¢ R ¢ 8 i
12 r
l1¢ = m=
20
EBD_7504
S-P-58 SOLUTIONS

Applying Kirchhoff ’s rule in loop abcfa


21. (b) As P = Ι 2 R, so P1 = (1.01 Ι)2 R = 1.02 I 2 R = 1.02 P.
e1 – (i1 + i2) R – i1 r1 = 0.
It means % increase in power
26. (b) Current, I = (2.9 ´ 1018 + 1.2 ´1018 ) × 1.6 × 10–19
æP ö
= ç 1 - 1÷ ´ 100 = 2%. = 0.66A towards right.
èP ø
6´3
6V 27. (b) Total resistance = = 2W
6+3
2W
1.5W 6
22. (a) Current in circuit = = 3A
6W
2
3W
Therefore current through bulb 1 is 2A and bulb 2 is
6V 6V 1A. So bulb 1 will glow more
28. (d) E = V + Ir
3/2W 3/2W 3W V = 12 – 3 = 9 volt
29. (a) Copper rod and iron rod are joined in series.
3W 3W
l
\ R = RCu + RFe = (r1 + r2)
6 A
hence Req = 3/2; \I = = 4A æ lö
3/ 2 çèQ R = r ÷ø
A
dE
23. (b) Thermo-electric power S = = a + bT From ohm’s law V = RI
dT
= (1.7 × 10–6 × 10–2 + 10–5 × 10–2) ¸
24. (a) Let, we connect 24 cells in n rows of m cells, then if I is
0.01 × 10–4 volt
the current in external circuit then
= 0.117 volt (Q I = 1A)
mE 30. (a)
I= ...(1) R2
mr / n + R R1
I
For I to be maximum, (mr + nR) should be minimum.
mr
It is minimum for R = ...(2)
n
So maximum current in external circuit is
mE ...(3) R
I=
2R
2e
m I=
=6 R + R1 + R 2
here R =3, r = 0.5 so equation (2) become
n Potential difference across second cell
so n = 2, m = 12
= V = e - iR 2 = 0
R
25. (d) a b
2e
i1+ i2 e – .R2 = 0
R + R1 + R 2
e1 r1
f
i1 i1
c R + R1 + R 2 - 2R 2 = 0
i2
R + R1 - R 2 = 0
e d
i2 r2 e2 \ R = R 2 - R1
Speed Test-18

1. (c) K.E. of electron = 10 eV


150
= = 15 mA
1 2 10
Þ mv = 10 eV
2 V = voltage to be measured = 150 volts
1 (such that each division reads 1 volt)
Þ (9.1 ´ 10 -31 )v 2 = 10 ´ 1.6 ´10 -19
2 150
Þ R= - 5 = 9995W
-19 15 ´10 -3
2 2 ´ 10 ´1.6 ´ 10
Þ v = 5. (b) Magnetic field due to segment ‘1’
9.1´ 10-31 uur m I
B1 = 0 [sin 90° + sin 0°] (–k)$
Þ v2 = 3.52 × 1012 Þ v = 1.88 × 106 m
4pR
Also we know that for circular motion
– m0 I ur
mv2
=Bev Þ r =
mv
= 11 cm
=
4pR
()
k$ = B3
r Be Magnetic field due to segment 2
2. (a) At a distance x consider small element of width dx.
Magnetic moment of the small element is B2 =
4R
–i =( )
m0 I $ – m0 I $
4pR
pi ( )
æq ö
çè dx÷ø w Z
l
dm = .p x 2
2p
dx
2

O Y
1
x I I
3
l/2
qw 2
x dx ; M = qwl = qpfl
2 2
M= ò 2l 24 12 X
-l / 2
ur
m0i1 m i m \ B at centre
3. (d) B1 = – + 0 2 = 0 (i2 – i1)
2 pr / 2 2 pr / 2 pr ur ur ur ur
B = B1 + B2 + B3 =
– m0I $
4pR
pi + 2k$ ( )
= 6 ´ 10 –6 T ...(i)
6. (d) Torque on the solenoid is given by
When the current is reversed in I2,
t = MB sin q
m0 (i1 + i2 ) m (i + i ) where q is the angle between the magnetic field and the
B2 = – = – 0 1 2 = 3´ 10–5 T ...(ii)
2pr / 2 pr axis of solenoid.
Dividing (ii) by (i) we get M = niA
\ t = niA B sin 30°
–(i1 + i2 ) 30 1
= =5 = 2000 ´ 2 ´ 1.5 ´ 10 -4 ´ 5 ´ 10 -2 ´
i2 – i1 6 2
– (i1 + i2) = 5i2 – 5i1 Þ 6i2 = 4i1 = 1.5 ´ 10 -2 N - m
7. (c) Time period of cyclotron is
i1 3
= 1 2pm 2pm mu p
i2 2 T= = ; B= u; R = =
u eB e eB eB
4. (c) Resistance of Galvanometer,
2pmu
Current sensitivity 10 Þ p = eBR = e ´ R = 2pmuR
G= Þ G= = 5W e
Voltage sensitivity 2 2
)2
= 2p2mu2R2
Here i g
EBD_7504
S-P-60 SOLUTIONS

8. (a) Rg = 50W, Ig = 25 × 4 × 10–AW = 10–2 A or FCA = 3 x 10-5 N (towards left).


Range of V = 25 volts Similarly, the wires B and C attract each other as they
V = Ig(Re + Rg) also carry the currents in same direction. the force on C
V
due to current in B is towards right hand side. Therefore,
\ Re = - Rg = 2450W the force on C due to B is given by
Ig
m 0 2i B i C
R FBC = . l
4p rBC
A B

10-7 ´ 2 ´ 20 ´ 10 ´ 0.15
Ig Re Rg =
9. (d) Here, current is uniformly distributed across the cross- 0.10
section of the wire, therefore, current enclosed in the or FBC = 6 × 10–5 N (towards right)
Therefore, the net force on C is
æ aö
amperean path formed at a distance r1 ç = ÷ F = (6 × 10–5 – 3 × 10–5)
è 2ø
= 3 × 10–5 N (towards right).
11. (b) \ Both galvanometers are in parallel
8´8
\ R eq = = 4W
8+8
a/2
P1 P2 e 4
\ Net current i = = = 1A
R eq 4
The current in both coils will be divided equally be-
cause their resistances are equal.
Now from tangent law,
æ p r2 ö B = H tan q
= ç 12 ÷ ´ I , where I is total current
èpa ø m 0 Ni
= H tan q
2R
\ Magnetic field at P1 is
N1 R 2 tan q1
m0 ´ current enclosed ´ =
B1 = N 2 R1 tan q 2
Path

æ p r2 ö 2 16 tan 30°
´ =
m 0 ´ ç 12 ÷ ´ I N 2 8 tan 60° Þ N2 = 12
è pa ø m ´ I r1
Þ B1 = = 0 12. (d) Magnetic field at the centre of the current loop is
2p r1 2p a 2
µ 0 2 pI
Now, magnetic field at point P2, B=
4 pR
m0 I m I µ0 2 p q u µ 2 pqu
B2 = . = 0 . or, B = , R= 0
2p (2a) 4pa 4 pR 4 pB
B1 m 0 Ir1 4pa Substituting the given values, we get
\ Required ratio = = ´
B2 2pa 2 m 0 I 4p ´ 10-7 ´ 2p ´ 2 ´ 10-6 ´ 6.25 ´ 1012
R= = 1.25 m
4p ´ 6.28
a
2 r1 2 ´ 2
13. (a) The magnetic field at O due to current in DA is
= = = 1.
a a mo I p
B1 = ´ (directed vertically upwards)
10. (b) The wires A and C carry current in same direction, 4p a 6
therefore they attract each other. The force on C due to
The magnetic field at O due to current in BC is
is A towards the wire A and is given by.
mo I p
m 0 2i A iC B2 = ´ (directed vertically downwards)
FCA = . 4p b 6
4p rAC
The magnetic field due to current AB and CD at O is
-7
10 ´
l=
Physics S-P-61

Therefore the net magnetic field is Therefore,


B = B1 - B2 (directed vertically upwards)

mo I p m o I p
= - ´
4 p a 6 4p b 6
dB
mo I æ 1 1 ö mo I
= ç - ÷= (b - a) p
24 è a b ø 24ab m0 I m0I
Bnet = ò dB sin q = ò sin qd q =
14. (c) To keep the main current in the circuit unchanged, the 2p 2 R 0 p2 R
resistance of the galvanometer should be equal to the r
18. (a) The direction of B is along ( - kˆ )
net resistance.
\ The magnetic force
æ GS ö ur r ur
\G = ç + S¢ F = Q (v ´ B) = Q (viˆ) ´ B( - kˆ) = QvBjˆ
è G + S ÷ø r
GS Þ F is along OY.
ÞG- = S¢
G+S 1 2qV
2 19. (a) mv 2 = qV ; v =
G 2 m
\ S¢ = .
G +S
I I C × × × × ×
G G
×q × r× × ×

× × × × ×
D
S × × × × × B
15. (b) According to the figure the magnitude of force on the × × × × ×
segment QM is F3 –F1 and PM is F2. v q
q A d

mv m 2qV
r= =
qB qB m

1 2mV
r=
B q

d q
In D CBD, sin q = = Bd
r 2mV
Therefore, the magnitude of the force on
ìï q üï
q = sin -1 í Bd ý
segment PQ is (F3 – F1 ) 2
+ F22 îï 2mV ïþ
16. (c) As electron move with constant velocity without 20. (a) Let us consider a thickness dx of wire. Let it be at a
deflection. Hence, force due to magnetic field is equal distance x from the centre O.
and opposite to force due to electric field.
E 20
qvB = qE Þ v = = = 40 m / s dx
B 0.5
x
dq a
17. (d) Current in a small element, dI = I O
p
b
Magnetic field due to the element
m 0 2dI
dB =
4p R
The component dB cos q, of the field is cancelled by N
Number of turns per unit length =
b-a
EBD_7504
S-P-62 SOLUTIONS

N dq L é dq ù
\ Number of turns in thickness dx =
b-a
dx Þ 2T
2
= BI
2p
dq êëQ 2 = small úû
Small amount of magnetic field is produced at O due to BIL
thickness dx of the wire. ÞT =
2p
m 0 NI dx
\ dB = 23. (d) D
2 (b - a) x
On integrating, we get, I2
2 p-q
b m0 NI dx m0 NI
B = òa =
2 b-a x 2 (b - a) A q B

b dx m0 NI C
òa x
=
2 (b - a )
[loge x] ba I1
I
m NI b E
B= 0 loge
2 (b - a) a Magnetic field at the centre due to current in arc
21. (c) The angular momentum L of the particle is given by ABC is
L = mr2w where w = 2pn. m 0 I1
w wq B1 = q (Directed upwards)
\ Frequency n = ; Further i = q × n = 4p r
2p 2p Magnetic field at the centre due to current in arc
wq 2 ADB is
Magnetic moment, M = iA = ´ pr ;
2p µ0 I 2
wqr 2 M wqr 2 q B2 = ( 2p - q) (Directed downwards)
\M= So, = = 4p r
2 L 2 mr 2 w 2 m Therefore net magnetic field at the centre
22. (c) Let us consider an elemental length dl subtending an m I q m I
angle dq at the centre of the circle. Let FB be the B = 0 1 - 0 2 (2p - q)
4 p r p 4p r
magnetic force acting on this length. Then E E EA
Also, I1 = = =
R1 rl1 / A rr q
T cos dq T cos dq
2 2 E E EA
dq and I2 = = =
2
dq
2 R2 rl 2 / A rr (2p - q)
T Tsin dq T
2 Tsin dq
m0 é EA q EA (2p - q) ù
2 \ B= ê ´ - ´ ú=0
I R I 4p ë r r q r rr (2p - q) r û
dq dq
2 2 24. (a) In mass spectrometer, when ions are accelerated through
potential V
1 2
mv = qV ..........(i)
2
FB = BI (dl) directed upwards as shown As the magnetic field curves the path of the ions in a
semicircular orbit
é arc(dl ) ù
= BI (Rdq) êQ angle (dq) = ú mv 2 BqR
ë radius Rû Bqv = Þ v= .......... (ii)
R m
æ Lö é Lù Substituting (ii) in (i)
= BI ç ÷ d q êQ 2pR = L Þ R = 2p ú
è 2p ø ë û 1 é BqR ù
2
Let T be the tension in the wire acting along both ends m = qV
of the elemental length as shown. On resolving T, we 2 êë m úû
æ dq ö q 2V
find that the components. T cos ç ÷ cancel out and or =
è 2 ø m B 2 R2
Since V and B are constants,
æ dq ö
the components. T sin ç ÷ add up to balance FB. q 1
è 2 ø \ µ

At equilibrium 2T
Physics S-P-63

Qx 27. (a) I = 50 k; Ig = 20k, where k is the figure of merit of


25. (b) Electric field at P is E =
4 p Î0 (x 2 + r 2 )3 / 2 20 k . R g
galvanometer; S = Ig Rg(I – Ig); so 12 =
Magnetic field P is (50 k - 20 k )
m0 2pir 2 m 0 2pQfr 2 On solving we get Rg = 18 ohms.
B= [Q i = Qf ] 28. (d) Let I be current and l be the length of the wire.
4p (x 2 + r 2 )3 / 2 = 4p (x 2 + r 2 )3 / 2
m 0 In m 0 I ´ p
Q For Ist case : B = = where 2pr = l
2r l
and n = 1
r l
P For IInd case : l = n(2pr ') Þ r ' =
x 2np

m0 nI m0 nI n 2 m 0 pI
B' = = = = n 2B
f = frequency of revolution 2r ' l l
2
2np
1
Electric energy density = e0 E2 29. (b) When a charged particle enters a magnetic field at a
2
direction perpendicular to the direction of motion, the
B2 path of the motion is circular. In circular motion the
Magnetic energy density = direction of velocity changes at every point (the
2m0
magnitude remains constant).
1 Therefore, the tangential momentum will change at
e0 E2
Electric field density 2 every point. But kinetic energy will remain constant as
=
Magnetic energy density B2
1 2
2m 0 it is given by mv and v2 is the square of the
2
E2 x 2c2 9 10
magnitude of velocity which does not change.
= 2 2 = 2 2 4 = 2 ´ 10 = 9.1 × 109. 30. (a) FE = qE (Force due to electric field)
c B 4p f r p
FB = evB sin q = qvB sin 0 = 0
26. (d) m = 1.67 × 10–27 kg; e = 1.60 × 10–19 C;
(Force due to magnetic field)
n = 10 MHz = 107 Hz; R = 60 cm = 0.6 m
-27 E
2 p mn 2 p ´ 1.67 ´ 10 ´ 107
B= = = 0.656 Tesla
e 1.60 ´ 10-19 +q
2 2 2 2 -19 2 2 B
B e R (0.656) ´ (1.60 ´ 10 ) (0.6)
Emax = = Force due to electric field will make the charged particle
2m 2 ´ 1.67 ´ 10 -27 released from rest to move in the straight line (that of
11.874 ´ 10-13 electric field). Since the force due to magnetic field is
= 11.874 × 10–13 J = = 7.421 MeV zero, therefore, the charged particle will move in a
1.6 ´ 10-13 straight line.
EBD_7504
S-P-64 SOLUTIONS

Speed Test-19

1. (b) As the axes are perpendicular, mid point lies on axial 1


line of one magnet and on equatorial line of other æ t ö2
magnet.
\ l = 2ç ÷
ç 3 B.i ÷
è ø
m 0 2 M 10-7 ´ 2 ´ 1 6. (a) M = 60 Am2
\ B1 = = =2 ´10-7 r
4 p d3 13 t = 1.2 × 10–3 Nm, BH = 40 × 10–6 Wb/m2
r r r
m M t = M ´ BH Þ t = MBH sin q
and B2 = 0 3 = 10-7
4p d Þ 1.2 × 10–3 = 60 × 40 × 10–6 sin q

\ Resultant field = B12 + B 22 = 5 ´ 10 -7 T 1.2 ´ 10-3 1


Þ sin q = = = sin 30°
-6 2
2. (c) Initial magnetic moment of each magnet = m × l. 60 ´ 40 ´ 10
Þ q = 30°
As is clear from Fig., S1 and N2 neutralize each other.
7. (b) Electro magnet should be amenable to magnetisation
Effective distance between and demagnetization
\ retentivity and coercivity should be low.
N1 and S2 = l 2 + l2 = l 2
\ M¢ = ml 2 . I I 1
8. (b) T = 2p = 2p where I = ml 2
3. (b) For a diamagnetic material, the value of µr is less than M ´B MB 12
one. For any material, the value of Îr is always greater When the magnet is cut into three pieces the pole
than 1. strength will remain the same and
4. (a) The time period of oscillation of a freely suspended 1 æ mö æ l ö I
2
magnet is given by M.I. (I¢) = ç ÷ ç ÷ ´ 3 =
12 è 3 ø è 3 ø 9
I We have, Magnetic moment (M)
T = 2p
MH = Pole strength (m) × l
I \ New magnetic moment,
2p
T MH æ lö
Thus, = M ' = m ´ ç ÷ ´ 3 = ml = M
T' I è 3ø
2p
MH ' T 2
Given, T = 4 sec, T ' = 2sec , \T'= = s.
9 3
4 H' 9. (a) Given that : B1 = 1.2 × 10–2 T, orientation of dipole with
So, = the field B1, q1 = 15°
2 H
Hence, orientation of dipole with B2,
H'
or =2 q2 = 60° – 15° = 45° (figure)
H
or H' = 4H B2
5. (b) t = MB sin q 60°
45°
t = iAB sin 90º A

t N 15°
B1
\ A= l l S
iB
Also, A = 1/2 (BC) (AD) As the dipole is in equilibrium, therefore, the torque on
l/2 l/2 the dipole due to the two fields must be equal and
B C
D
l opposite. If M be the magnetic dipole moment of the
dipole, then
2
1 1 ælö 3 2 t1 = t2 or MB1 sin q1 = MB2 sin q2
But (BC)(AD) = (l ) l 2 - ç ÷ = l
2 2 è2ø 4 B1 sin q1 1.2 ´ 10-2 sin15°
or, B2 = =
3 2 t sin q2 sin 45°
Þ (l ) =
4
4.4 × 10–3 Tesla
Physics S-P-65

6 T1 I1 M 2 2I M
10. (a) Here, 2 l = 8 cm , l = 4 cm , d = = 3 cm . = ´ = ´
2
At neutral point,
T2 I 2 M1 I 2M
m M 25 / 4
H=B= 0 Þ T2 = = 2sec
4 p (d + l 2 )3 / 2
2
21/ 4
M M 17. (b) Force on a charged particle is given by F = qvB. Here
= 10 - 7 = v = 0 and also resultant B is zero.
-2 3 1250
(5 ´ 10 )
\ Force = 0
\ M = 1250 H = 1250 ´ 3.2 ´ 10 -5 Am 2 18. (a) Given, B = 4 × 10–5 T
RE = 6.4 × 106 m
M 1250 ´ 3.2 ´ 10 -5 Dipole moment of the earth M = ?
m= = A m.
2l 8 ´ 10 -2
m M
B= 0 3
1 4p d
= 0.5 Am = 0.5 ´ ab amp ´100 cm
10
4p´10-7 ´ M
= 5 ab-amp cm. 4 ´10-5 =
( )
3
11. (a) cd < cp < cf 4p´ 6.4 ´ 106
For diamagnetic substance cd is small and negative
\ M @ 1023 Am2
(10–5)
For paramagnetic substances cp is small and positive 19. (d) Relative magnetic permeability
(10–3 to 10–5) magnetic permeability of material (μ)
For ferromagnetic substanes c f is very large mr =
permeability of free space (μ 0 )
(103 to 105)
12. (d) PQ6 corresponds to the lowest potential energy among For paramagnetic materials mr > 1.
all the configurations shown. 20. (d) Given, I = 9 × 10–5 kg m2, B = 16p2 × 10–5 T
13. (d) A magnetic needle kept in non uniform magnetic field 15 3
T= = s
experience a force and torque due to unequal forces 20 4
acting on poles. In a vibration magnetometer
V 3 é 3ù I 4p 2 I
14. (d) tan d = = êëQ tan 37º = 4 úû Time period, T = 2p or M =
H 4 MB BT 2
3 2 -5
\ V= H 4 p ´ 9 ´ 10
4
M = 2
= 4 A m2
æ ö
3
V = 6 × 10–5 T 16 p 2 ´ 10 -5 ´ ç ÷
è4ø
4 –5 –5 21. (b)
H = ´ 6 ´ 10 T = 8 ´ 10 T N
3
\ Btotal = V2 + H2 = (36 + 64) ´ 10–5
= 10 × 10–5 = 10–4T. S P N R
15. (b) Ferromagnetic substance has magnetic domains Q
whereas paramagnetic substances have magnetic d d
R BP
dipoles which get attracted to a magnetic field.
Diamagnetic substances do not have magnetic dipole BR
but in the presence of external magnetic field due to BQ S
their orbital motion of electrons these substances are
repelled. 2 2
æ m 0 2M ö æ m0 M ö
BP2 çè 4 p 3 ÷ø + çè 4 p 3 ÷ø
2
16. (c) Initially magnetic moment of system BR = + BQ =
d d
M 1 = M 2 + M 2 = 2M and moment of inertia m0 M ì m M ü
= 5 = 5 B , í BQ = 0 = Bý
I1 = I + I = 2I. 4p d3 î 4p d 3
þ
Finally when one of the magnet is removed then 22. (d) d1 = 40°, d2 = 30°, d = ?
M2 = M and I2 = I cot d = cot 2 d1 + cot 2 d2 = cot 2 40º + cot 2 30º
I cot d = 1.192 + 3 = 2.1
So, T = 2p
M B .
EBD_7504
S-P-66 SOLUTIONS

23. (a) Iron is ferromagnetic. 28. (c) Magnetic field in solenoid B = m0n i
24. (b) The field is entering into the surface so flux is negative. B
25. (d) Work done, W = MB (cosq1 – cosq2) Þ = ni
m0
Here q1 = 0°, q2 = 60° (Where n = number of turns per unit length)
\ é 1 ù MB ...(i) B Ni
W = MB ê1 - ú = =
ë 2û 2 Þ
m0 L
Torque, t = MBsinq = MBsin60° 100i
Þ 3 ´ 103 =
MB 3 10 ´ 10-2
= = 3W (Using eq. (i)) Þ i = 3A
2
29. (b) The time period of a rectangular magnet oscillating in
26. (a) Given M = 8 × 1022 Am2
d = Re = 6.4 × 106m I
earth’s magnetic field is given by T = 2p
m0 2M mBH
Earth’s magnetic field, B = .
4p d3
where I = Moment of inertia of the rectangular
4p´10-7 2 ´ 8 ´ 1022 magnet
= ´
4p (6.4 ´ 106 )3 m = Magnetic moment
@ 0.6 Gauss BH = Horizontal component of the earth’s magnetic
27. (b) Given : M1 = 1.20 Am2 field
Case 1
N
BH I 1
T = 2p I = M l2
mBH where 12
B1
S B2 S Case 2
O Magnet is cut into two identical pieces such that each
N N piece has half the original length. Then
r r
I'
T ' = 2p
S m ' BH
2
M 2 = 1.00 Am 2 1 æ M ö æ lö I m
where I ' = çè ÷ø çè ÷ø = and m ' =
12 2 2 8 2
20
r= cm = 0.1m
2 T' I' m
Bnet = B1 + B2 + BH \ = ´
T m' I
m 0 ( M1 + M 2 )
Bnet = + BH
4p r3 I /8 m 1 1
= ´ = =
m/2 I 4 2
10 -7 (1.2 + 1)
= 3
+ 3.6 ´ 10 -5 30. (c) The net energy dissipated per unit volume of the material
(0.1) during a complete cycle of magnetisation is equal to
= 2.56 ´ 10 -4 Wb/m
2
area of B.H curve.
Speed Test-20

1. (b) 8. (b) Mutual inductance depends on the relative position


2. (d) The self inductance of a long solenoid is given by and orientation of the two coils.
9. (d) The back e.m.f. in a motor is induced e.m.f., which is
L = mr m0 n 2 Al
maximum, when speed of rotation of the coil is maximum
Self inductance of a long solenoid is independent of
10. (d) Here, induced e.m.f. w l
the current flowing through it. 2l
3. (b) The self-inductance of a coil L = m0n2Al where 3l dx
x
m0 = permeability of air, e= ò (wx) Bdx
n = number of turns per unit length, 2l
A = Area of cross-section and [(3l) 2 – (2l)2 ] 5Bl 2 w
l = length of the solenoid = Bw =
2 2
This depends on the geometry of the inductor such as
cross-sectional area, length and number of turns and dB 2T
not on the material, even if it is made of a super 11. (b) At t = 2s, B = 4T, =
conducting material. If the superconductor is below dt s
the critical temperature, the current will continuously A = 20 × 30cm2
flow and the inductance may not have the property of dA
inductance any more. \ L1 = L2; L3= 0. = 600 × 10–4 m2, = -(5 ´ 20) cm 2 / s
dt
4. (a) The induced emf will remain constant only in the case
of rectangular and square loops. In case of the circular = – 100 × 10–4 m2/s
and the elliptical loops, the rate of change of area of df é d(BA) ù é BdA AdB ù
the loops during their passage out of the field is not E=- = -ê ú = -ê +
dt ë dt û ë dt dt úû
constant, hence induced emf will not remain constant
for them. = – [4 × (– 100 × 10–4) + 600 × 10–4 × 2]
5. (b) Induced e.m.f. in the ring opposes the motion of the = – [– 0.04 + 0.120] = – 0.08 V
magnet. 12. (d) f = n BA cos q = 10 B a2 cos wt
6. (a) When a north pole of a bar magnet moves towards the
coil, the induced current in the coil flows in a direction
such that the coil presents its north pole to the bar
e=-
df
dt
=-
d
dt
( )
10 B a 2 cos wt = 10 B a 2 sin w t (w ) .

magnet as shown in figure (a). Therefore, the induced 13. (a) As the magnetic field increases, its flux also increases
current flows in the coil in the anticlockwise direction. into the page and so induced current in bigger loop
When a north pole of a bar magnet moves away from will be anticlockwise. i.e., from D to C in bigger loop
the coil, the induced current in the coil flows in a and then from B to A in smaller loop.
direction such that the coil presents its south pole to 14. (a)
the bar magnet as shown in figure (b).
15. (c) e = Blv = 2´ 10-1 ´ v = 0.2 v

e 0.2v
I= = 10 -3 Þ = 10 -3
R 4
N N [Since effective resistance R of bridge is
(a) (b)
6´6
Therefore induced current flows in the coil in the R= =3 W
6+6
clockwise direction. so total resistance = 1 + 3 = 4W]
7. (d) Given : f = 4t 2 + 2t + 1 wb Þ v = 2 cm s–1
df d 16. (d) The magnetic field is increasing in the downward
\ = (4t 2 + 2t + 1) = 8t + 2 = | e | direction. Therefore, according to Lenz’s law the current
dt dt
I1 will flow in the direction ab and I2 in the direction dc.
| e | 8t + 2 8t + 2
Induced current, I = = = A d dI NM I
R 10W 10 17. (a) E= ( NMI ) Þ E = NM Þ E=
At t = 1 s, dt dt t

8 ´1+ 2 E MI
I= A= =
10 N t
EBD_7504
S-P-68 SOLUTIONS

18. (d) Rate of decrease of area of the semicircular ring order to oppose the cause) in the ring (assumed). Hence
there will be a force towards left on the electrons.
dA
- = (2R)V W W
dt 23. (a) x= Þ V= Þ W = QV
Acoording to Faraday's law of induction induced emf Q Q
df dA 24. (c) L = 2mH, i = t2e–t
e=- = -B = - B(2RV)
dt dt di
-L = - L[ - t 2 e -t + 2te - t ]
E= dt
× × × × ×
when E = 0,
× × × × ×
–e–t t2 + 2te–t = 0
× × × × × or, 2t e–t = e–t t2
× × × × × Þ t = 2 sec.
M Q vt 25 (c) If the current increases with time in loop A, then
2R magnetic flux in B will increase. According to Lenz's
The induced current in the ring must generate law, loop -B is repelled by loop -A because current in
magneticfield in the upward direction. Thus Q is at loop B will be antiparallel to that in A.
higher potential. - df
26. (b) dq = [Q ff = -f i ]
19. (c) Induced emf = vBH l = 1.5 × 5 × 10–5 × 2 R
= 15 × 10–5 2f 2m Ia æ 2a ö m Ia.ln 2
= 0.15 mV \ dq = i = 0 ln ç ÷ = 0
R R.2p è a ø Rp
20. (c) The area swept by radius OC in one half circle is pr2/2.
The flux change in time T/2 is thus (pr2 B/2). The é 2a
m I m Ia 2a ù
é 2p ù ê f = ò 0 a dx = 0 ln ú
induced emf is then e = pr2B/T = Bwr2/2 êQ T = êë 2px 2p a ú
ë w úû a û
2
The induced current is then I = e/R = Bwr /2R
- df
27. (b) The individual emf produced in the coil e =
21. (c) Df = e = iR Þ Df = (iDt )R = QR dt
Dt |e| 1 df
Df \ The current induced will be i = Þi=
ÞQ= R R dt
R
dq dq 1 df 1 BA
22. (a) If we consider the cylindrical surface to be a ring of But i = Þ = Þ ò dq = ò d f Þ q =
radius R, there will be an induced emf due to changing dt dt R dt R R
field. Ndf df
r 28. (a) Since e = – if is fast, so e is large.
df dB dt dt
ò E.d l = - dt = - A dt
df (W2 - W1 )
dB dB R dB 29. (b) = R tot = (R + 4R)W = 5R W
Þ E (2pR) = - A = -p R 2 Þ E= - dt t
dt dt 2 dt
\ Force on the electron ndj - n(W2 - W1 )
i= = .
eR dB R tot dt 5Rt
F = - Ee =
2 dt (Q W2 & W1 are magnetic flux)
1 eR dB m N N A 4p´ 10-7 ´ 300 ´ 400 ´ 100 ´10-4
Þ acceleration = 30. (d) M= 0 1 2 =
2 m dt l 0.2
As the field is increasing being directed inside the paper,
hence there will be anticlockwise induced current (in = 2.4p ´ 10 -4 H
Speed Test-21

1. (a) The charging of inductance given by, 100 5


C= =
æ - ö
Rt w12 L ´ 60 3w12 L
i = i0 ç1 - e L ÷ 1 3
çè ÷ø w2 = = w1
5 5

-
Rt
-
Rt 3w12 L
i0 1
= i0 (1 - e L ) Þ e L = 4. (d)
2 2
200V,50Hz
Taking log on both the sides, 1A
~
Rt
- = log1 - log 2
L

L 300 ´ 10 -3
Þ t= log 2 = ´ 0.69 L
R 2
From the rating of the bulb, the resistance of the bulb
Þ t = 0.1 sec. can be calculated.
2. (a) For series R – C circuit, capacitive reactance, 2
Vrms
R= = 100W
2 P
æ 1 ö
Zc = R2 + ç ÷ For the bulb to be operated at its rated value the rms
è Cw ø current through it should be 1A
Xc = 1/Cw Vrms
Also, Irms =
Z
200
R \ 1=
C 100 2 + (2 p50.L) 2

~ 3
HL=
p
AC Source 5. (c) Growth in current in LR2 branch when switch is closed
V V is given by
Current i = =
Zc 2
æ 1 ö E
R2 + ç ÷ i= [1 - e - R2t / L ]
è Cw ø R2
V 1 R2t
Vc = iXc = ´ di E R2 - R2t / L E -
2 Cw Þ = . .e = e L
æ 1 ö dt R2 L L
R2 + ç ÷
è Cw ø Hence, potential drop across L
V æ E -R t / L ö -R t / L
Vc = = ç e 2 ÷ L = Ee 2
(RCw)2 + 1 èL ø
2t
If we fill a di-electric material like mica instead of air - -3
then capacitance C­ Þ Vc¯ = 12e 400´10 = 12e–5tV
So, Va > Vb 6. (a) At angular frequency w, the current in RC circuit is given
3. (a) If voltage across resistor is 50V then this should be the by
resonance condition. Vmax
At resonance, XL = XC imax = .......(i)
2
2 æ 1 ö
1 1 R +
è w Cø
w2L = ; w2 =
w2 C LC i rms Vrms Vmax
Also, At w = w1 Also = = .....(ii)
2 2 2 9
100 60 100 ö R +
I= = ÷ 2 2
w C
XL XC ; w ÷ø
EBD_7504
S-P-70 SOLUTIONS

From equation (i) and (ii) we get Time constant for series combination
1
RC
2 5 wC 3 XC 3 =
3R = Þ = Þ = 2
w 2C2 R 5 R 5
In first case :
æ - tö
R
- 1
t
V0
7. (a) I = I o ç1 - e L ÷ V = V0 e 2 RC = ...(1)
çè ÷ø 2
In second case :
(When current is in growth in LR circuit)
t2
- V0
æ ( RC / 2)
- ´2 ö
æ 5 ....(2)
- tö V = V0 e =
R
E 5 2
= ç1 - e L ÷ = ç1 - e 10 ÷
R çè ÷ø 5 çè ÷ø From (1) and (2)
t1 t2
= (1 – e–1) =
2 RC ( RC / 2)
8. (d) When capacitance is taken out, the circuit is LR.
t 10
wL Þ t2 = 1 = = 2.5 sec.
\ tan f = 4 4
R
11. (c) When the capacitor is completely charged, the total
1 200 energy in the LC circuit is with the capacitor and that
Þ wL = R tan f = 200 ´ =
3 3
1 Q2
Again , when inductor is taken out, the circuit is CR. energy is E =
2 C
1 When half energy is with the capacitor in the form of
\ tan f =
w CR electric field between the plates of the capacitor we get
1 1 200
Þ = R tan f = 200 ´ = E 1 Q '2
wc 3 3 = where Q ' is the charge on one plate of the
2 2 C
2 capacitor
2 æ 1 ö
Now, Z = R + ç - wL ÷ 1 1 Q 2 1 Q '2 Q
è wC ø \ ´ = Þ Q' =
2 2 C 2 C 2
2
æ 200 200 ö V0 4V0
= (200) 2 + ç - ÷ = 200 W 12. (d) V = t Þ V= t
è 3 3ø T/4 T
1/ 2
ìT/4 2 ü
Power dissipated = Vrms I rms cos f ï ò t dt ï
4V0 4V0 ï 0 ï V0
Vrms R æ Rö Þ Vrms = < V2 > = 2
<t > = í ý =
= Vrms . . çQ cos f = ÷ T T ï T /4 ï 3
ï ò
Z Z è Zø dt ï
î 0 þ
V 2rms R (220)2 ´ 200
= = 13. (b) At t = 0 , no current will flow through L and R1
Z2 (200)2
220 ´ 220 V
= = 242 W \ Current through battery = R
200 2
1 2 At t = ¥ ,
9. (a) Energy stored in magnetic field = Li
2 R1R2
1 q2 effective resistance, Reff =
Energy stored in electric field = R1 + R2
2 C
V
1 1 q2 \ Current through battery = R
\ Li 2 = eff
2 2 C
V ( R1 + R2 )
1 =
Also q = q0 cos wt and w = R1R2
LC
p 14. (a) L = 10 mHz = 10–2 Hz
On solving t = LC f = 1MHz = 106 Hz
4
10. (c)
= 2RC
Physics S-P-71

1 19. (c) Impedance at resonant frequency is minimum in series


f2 = LCR circuit.
4p 2 LC
2
1 1 10 -12 æ 1 ö
ÞC= = = = 2.5 pF So, Z = R 2 + ç 2pfL - ÷
4p 2 f 2 L 4 ´ 10 ´ 10 -2 ´ 1012 0.4 è 2pfC ø
15. (d) The average power in the circuit where cos f = power
factory 20. (a) Since Vs = N s
< P > = Vrms × Irms cos f Vp N p
180 Where
f = p/3 = phase difference = = 60
3 Ns = No. of turns across primary coil = 50
1 Np = No. of turns across secondary coil
2 = 1 volt = 1500
Vrms =
2 2 df d
and V p = = (f 0 + 4t ) = 4
1 dt dt
2 = æ 1ö A 1500
Irms = ç ÷ Þ Vs = ´ 4 = 120 V
2 è 2ø 50
Es n æn ö
p 1 21. (a) = s or E s = E p ´ ç s ÷
cos f = cos = Ep np ç np ÷
3 2 è ø
1 1 1 1 æ 200 ö
<P>= ´ ´ = W \ E s = 120 ´ ç ÷ = 240 V
2 2 2 8 è 100 ø
16. (a) Initially, when steady state is achieved,
Ι p ns æ np ö æ 100 ö
= or Ι s = Ι p ç ÷ \ Is = 10 ç = 5 amp
i=
E
Ι s n p èn ø è 200 ÷ø
s
R
Let E is short circuited at t = 0. Then 22. (b) We know that, i = i0 (1 - e-t / t )
E 3
or i0 = i0 (1 - e-4 / t )
At t = 0, i0 = 4
R
1
Let during decay of current at any time the current or e -4 / t =
4
flowing is - L di - iR = 0 or e4 / t = 4
dt
4
i t \ = ln 4
di R di R t
Þ
i
= - dt Þ
L ò i ò L
= - dt
2
i0 0 or t= s
ln 2
R
i R - t E 5
Þ log e = - t Þ i = i0 e L 23. (a) I0 = = = 0.1 A
i0 L R 50
I = 60 mA = 60 × 10–3 A, t = 0.1
R -100 ´10-3
E - t 100 100 ´10 -3 1 æ - tö
R
Þi= e L = e =
R 100 e Now, I = I0 ç1 - e L ÷
17. (d) Current is maximum when XL = XC çè ÷ø
1 1 1
Þ wL = Þ w= = æ æ - ö
5
- ´ 0.1 ö
50
wC LC -6
0.5 ´ 8 ´ 10 \ 60 × 10–3 = 0.1 ç1 - e L ÷ = 0.1 1 - e ÷
ç L
çè ÷ø çè ÷ø
1 or 1 – e–5/L = 0.6
= -3 = 500 rad/s.
2 ´ 10 4
\ e–5/L = 1 – 0.6 = 0.4 = or e5/L = 10/4
18. (a) If w = 50 × 2p then wL = 20W 10
If w¢ = 100 × 2p then w¢L = 40W Taking log of both sides
Current flowing in the coil is 5
= 2.303 [log10 10 – log10 4]
L
200 200 200 = 2.303 [1.0000 – 0.6021] = 2.303 × 0.3979 = 0.9164
I= = =
Z R 2 + (w¢L)2 (30)2 + (40) 2 5
I = 4A.
EBD_7504
S-P-72 SOLUTIONS

24. (d) These three inductors are connected in parallel. The


net impedence Z = R 2 + ( X L - X C )2 = R
equivalent inductance Lp is given by
1 1 1 1 1 1 1 3 R
= + + = + + = =1 power factor cos f = =1
L p L1 L2 L3 3 3 3 3 Z
27. (a) Here, C = 100 mF = 100 × 10–6 F, R = 40 W,
\ Lp = 1 Vrms = 110 V, f = 60 Hz
Peak voltage,
1
25. (d) At resonance, wL = . The circuit behaves as if it V0 = 2 . Vrms = 100 2 = 155.54 V
wC
contains R only. So, phase difference = 0 Circuit impedance,
At resonance, impedance is minimum Zmin = R and 1
current is maximum, given by Z= R2 + 2 2
w C
E E
I max = =
Z min R 1
= 40 2 +
It is interesting to note that before resonance the current (2 ´ p ´ 60 ´ 100 ´ 10 -6 ) 2
leads the applied emf, at resonance it is in phase, and
after resonance it lags behind the emf. LCR series circuit = 1600 + 703.60 = 2303.60 = 48 W
is also called as acceptor circuit and parallel LCR circuit hence, maximum current in coil,
is called rejector circuit. V0 155.54
I0 = = = 3.24 A
Z 48
26. (c)
28. (a) Laminated core provide less area of cross-section for
when L is removed from the circuit the current to flow. Because of this, resistance of the
core increases and current decreases thereby
R
decreasing the eddy current losses.
p/3
XC p 29. (c) Charge on he capacitor at any time t is given by q = CV
= tan
R 3 Z1 (1– et/t)
XC
at t = 2t
p
X C = R tan ...(1) q = CV (1 – e–2)
3
when C is remove from the circuit 30. (b) V = 200V; r = 10W
R¢ = 10 + 100W = 110W
XL p Z2
= tan
R 3 XL V 220
I= = = 2A
p/3 R¢ 100
p
X C = R tan R ...(2)
3 P = I2R = 4 × 100 = 400 W
Speed Test-22

1. (b) Q The E.M. wave are transverse in nature i.e.,


r r = 8.85 ´10-12 ´ (720)2
k ´E r
= =H …(i) = 4.58 ´10-6 J / m3
mw
r r r df
r B
where H = 9. (c) Ñò B.d l =b m 0 Î4ac
0
dt
m
r r æ ö dE æ dE ö
k ´H r or B ´ 2pr = m 0 Î0 A ç ÷ \ B µ çè ÷ø
and = -E … (ii) è dt ø dt
we
r r r r 10. (b) Comparing with the equation of wave.
k is ^ H and k is also ^ to E Ey = E0 cos (wt – kx)
r r r r r
or In other words X || E and k || E ´ B w = 2 pf = 2p × 106 \ f = 106 Hz
2. (d) The direction of propagation of electromagnetic wave 2p
is perpendicular to both electric field E and magnetic = k = p × 10–2 m–1, l = 200 m
l
field B, i.e., in the direction of E × B by right thumb rule. 11. (b) Wavelength of monochromatic green light
The diagram given below
= 5.5 × 10–5 cm
x
Power
E(x) B(–y) E(x) B(–y) Intensity I =
Area
z 100 ´ ( 3 /100 ) 3
= = Wm -2
4p ( 5 )
E(–x) 2 100 p
y B (y) B(y)
E(–x)

So, electromagnetic wave is along the z-direction which Now, half of this intensity (I) belongs to electric field
is give the cross product of E and B direction is and half of that to magnetic field, therefore,
r r
perpendicular to E and B from E to B . i.e., (E × B) in z- I 1
= e 0 E 02 C
direction. 2 4
3. (a) Direction of energy progration of EM-waves is given
by or 2I
E0 =
r r r r e0 C
D = K ( E ´ B) or –kˆ = K ( E ˆj ´ B )
Clearly direction of magnetic field is along positive x-
axis. æ 3 ö
2´ç p÷
4. (b) EM waves carry momentum and hence can exert è 100 ø 6
= = ´ 30 = 7.2
( )
pressure on surfaces. They also transfer energy to the æ 1 ö 8 25
surface so p ¹ 0 and E ¹ 0. ç ÷ ´ 3 ´10
è 4p ´ 9 ´109 ø
5. (b)
2p \ E 0 = 2.68 V / m
6. (b) Here, k = , w = 2pu
l
1
k 2p / l 1 1 12. (c) E0 = CB0 and C =
\ = = = (Q c = u l) m0 e 0
w 2pu pu c
where c is the speed of electromagnetic wave in 1
vacuum. It is a constant whose value is 3 × 106 m s–1 Electric energy density = e 0 E02 = m E
2
7. (b)
8. (a) Erms = 720 1 Bo 2
Magnetic energy density = = mB
The average total energy density 2 m0
Thus, mE = mB
1 1
= Î0 E 20 = Î0
2 2
EBD_7504
S-P-74 SOLUTIONS

13. (a) Velocity of light 21. (d) For an E.M. wave power is transmitted in a direction
perpendicular to both the fields.
E E 9.3
C= Þ B= = = 3.1´ 10-8 T 22. (b)
B C 3 ´ 108 23. (b) All components of electromagnetic spectrum travel in
vacuum with velocity 3 × 108 m/s.
1
14. (c) Speed of EM waves in vacuum = = constant 24. (c) Electromagnetic waves are the combination of mutually
m 0 Î0 perpendicular electric and magnetic fields.

1 E
15. (d) e 0 E 02 is electric energy density.. 25. (a) Momentum of light falling on reflecting surface p =
C
2
As surface is perfectly reflecting so momentum reflect
B2
is magnetic energy density.. E
2m 0 p1 = –
C

1 B2
So, total energy = e 0 E 02 + 0 E E
2 2m 0 =P
C C
16. (c) Gamma rays < X-rays < Ultra violet < Visible rays
< Infrared rays < Microwaves < Radio waves. So, momentum transferred
E æ E ö 2E
17. (b) The average energy stored in the electric field – –
C çè C ÷ø
= P – P1 = =
C
1
UE = e0 E 2 26. (d)
2
The average energy stored in the magnetic field = UB = E0 2p
27. (a) = c . also k = and w = 2pv
B0 l
1 B2
, These relation gives E0k = B0w
2 m0
According to conservation of energy UE = UB 1
28. (c) Speed of light of vacuum c = and in another
m 0 e0
B2
e 0m 0 =
E2 1
medium v =
me
B 1
= e 0 m0 =
E C c me c
\ = = mr K Þ v = .
18. (d) To generate electromagnetic waves we need v m 0 e0 mr K
accelerating charge particle.
19. (c) Pressure = I/C 29. (b)
20. (b) Audible waves are not electromagnetic wave. 30. (d) Wave is uv rays.
Speed Test-23

1. (a) From mirror formula A A æ A + dm ö


or sin × cot = sin ç
1 1 1 dv v 2 æ du ö 2 2 è 2 ÷ø
+ = so, =- 2ç ÷
v u f dt u è dt ø A
dv æ f ö du
2 A cos 2 æ A + dm ö
or sin × = sin ç
Þ = -ç
dt è u - f ÷ø dt 2 A è 2 ÷ø
sin
dv 1 2
Þ = m/s
dt 15 A é p æ A + dm ö ù
2. (d) r or cos = cos ê - ç ÷ú
2 ë2 è 2 øû
h = 12 cm A p æ A + dm ö
Þ = -ç ÷
2 2 è 2 ø
C or A = p - A - d m Þ d m = p - 2A .
1 1 3 5. (b) f0 = 100 cm, fe = 5 cm
sin C = = = .
m 4/3 4 When final image is formed at least distance of distinct
Now r = h tan C vision (d), then
f æ f ö 100 æ 5ö
3 36 M = 0 ç1 + e ÷ = ç 1 + ÷ [Q D = 25 cm]
= 12 ´ = cm fe è dø 5 è 25 ø
7 7
6
3. (d) For 1st lens, u1 = – 30, f1 = + 10cm, M = 20 ´ = 24
5
1 1 1 6. (c) I1 is the image of object O formed by the lens.
Formula of lens, + =
v1 30 10 1 1 1
- = ; u1 = – 15, f1 = 10
or, v1 = 15 cm at I1 behind the lens. v1 u1 f
The images I1 serves as virtual object for concave lens. Solving we get, v1 = 30 cm.
For second lens, which is concave, u2 = (15 – 5) = 10 I1 acts as source for mirror
cm. I1 acts as object. f2 = – 10 cm. \ u2 = – (45 – v1) = – 15 cm.
The rays will emerge parallel to axis as the virtual object I2 is the image formed by the mirror
is at focus of concave lens, as shown in the figure. Image 1 1 1 1 1
of I1 will be at infinity. These parallel rays are incident on \ v = f - u = - 10 + 15
2 m 2
the third lens viz the convex lens, f3 = + 30 cm. These
\ v2 = –30cm.
parallel rays will be brought to convergence at the focus
of the third lens. L I2
\ Image distance from third lens = f3 = 30 cm u3=15cm v2 =30cm

u2 =15cm
O
A
I1
u1 =15cm v1 =30cm
I3
v3 45cm
The height of I2 above principal axis of lens is
4. (a) We have, v
= 2 ´ 1 + 1 = 3cm.
u2
æ A + dm ö
sin ç
è 2 ÷ø I2 acts as a source for lens, u3 = –(45 – v2) = –15 cm.
m= Hence, the lens forms an image I 3 at a distance
æ Aö v3 = 30 cm to the left of lens.
sin ç ÷
è 2ø
The height of I3 below the principal axis of lens
æ A + dm ö v
= 3 ´ 3 = 6 cm.
sin ç ÷
A è
Þ cot =
2
sin 302 + 62 = 6 26 cm.
EBD_7504
S-P-76 SOLUTIONS

7. (d) The focal length of the lens 10. (b) If the angular limit of resolution of human eye is R then
1 1 1
= - = +
1 1 20 + 1 21 1.22l 1.22 ´ 5 ´ 10 -7
= = R= = rad
f v u 12 240 240 240 a 2 ´ 10 -3
240
f = cm 1.22 ´ 5 ´ 10 -7 180
21 = ´ ´ 60 mi nute = 1 minute
-3 p
2 ´ 10
æ 1ö
Shift = t ç 1 - ÷ h
è m ø 11. (b) tan 45° = Þ h = 60 m
60
æ 1 ö 1
= 1ç 1 - ÷ = 1´ Tower
è 3/2 ø 3
1 35 h
Now v' = 12 - = cm
3 3 45°
Now the object be distance u¢. 45° 60 m
1 3 21 1 é 3 21 ù
= - = - Image
u ¢ 35 240 5 êë 7 48 úû
1 1 é 48 - 49 ù
=
u¢ 5 êë 7 ´ 16 úû sin 45°
12. (a) For point A, a m g =
u¢ = –7 ×16 × 5 = – 560 cm = – 5.6 m sin r
Q 1
n Þ sin r =
90 – a 2 am g
a For point B, sin (90° – r) = gma where,
8. (c)
q P (90° – r) is critical angle.
45º Air
Applying Snell’s law at Q
90° –r r A
sin 90° 1
n= =
sin(90º -a ) cos a 90° – r
1 B
\ cos a =
n
Glass
2 n2 - 1
1
\ sin a = 1 - cos a = 1 - 2 = ...(1)
n n
Applying Snell’s Law at P 1
sin q 2 \ cos r = g m a =
n= Þ sin q = n ´ sin a = n - 1 ; from (1) a mg
sin a
2 1
æ 2 ö 4 1 Þ a mg =
\ sin q = ç ÷ - 1 = - 1= cos r
è 3ø 3 3
1 1
æ 1 ö =
or q = sin -1 ç ÷ 1 - sin r 2 1
è 3ø = 1-
9. (b) From the fig. 2 am g 2
Angle of deviation, 2 a m 2g
2 1
d = i+e- A Þ a mg = 1
=
A 1- 2 a m 2g - 1
Here, e = i d
i e 2 a m 2g
3
and e = A 3
4 Þ 2 a m 2g - 1 = 2 Þ a mg =
2
3 3 A 13. (a) anl = 1.6, anw = 1.33
\d= A+ A– A =
4 4 2 f = 20 cm
For equilateral prism, A = 60° We have,
60° 1 æ 1 1 ö
\ d= = 30° ( ) - ÷
R2 ø
2 1
Physics S-P-77

1 æ 1 1 ö The image is of the same size and inverted.


= (1.6 - 1) ç - ..... (1) 17. (b) For the image of point
20 è R1 R 2 ÷ø
P to be seen by the N
1 æ 1 1 ö observer, it should be
Also, = ( w n l - 1) ç - ÷ formed at point Q. M r 2h
f' è R1 R 2 ø
In DQNS,
i
æ n öæ 1 1 ö NS = QS = 2h
= ç a l - 1÷ç - ÷ Q 45° S 2h
\ ÐNQS = 45° A
è a nw ø è R1 R 2 ø h
\ r = 45°
1 æ 1.6 öæ 1 1 ö
=ç - 1÷ - At point M P B
f ' è 1.33 ø çè R1 R 2 ÷ø ..... (2)
2h
1 sin r sin 45° 1
Dividing equation (1) by (2) m= = = ...(i)
2 sin i sin i 2 × sin i
f' 0.6
Þ = In D PMB,
20 (1.2 - 1)
PM2 = 4h2 + h2 = 5h2
0.6 ´ 20 h
f'= = 60 cm. \ sin i = ...(ii)
0.2 5h
Hence it's focal length is three times longer than in air. From (i) and (ii)
14. (a) Given i = 60°
1 5
A= d=e µ =
2 2
d = i + e – A Þ d = i ( Q e = A) 18. (c) As shown in the figure, the system is equivalent to
combination of three thin lens in contact
æ A + dm ö
sin ç 1 1 1 1
è 2 ÷ø \ = + +
m= f f1 f2 f3
A
sin By lens maker's formula
2
1 æ 3 öæ 1 1 ö 1
Here angle of deviation is min. (Q i = e) = ç – 1÷ç – ÷ = –
f1 è 2 ø è ¥ 25 ø 50
æ 60° + 60° ö 1 æ 4 öæ 1 1 ö 3
sin ç ÷ø = ç – 1÷ ç + ÷=
è 2 f 2 è 3 ø è 25 20 ø 100
m= = 1.73 1 2
60° 1 æ 3 öæ 1 1ö 1
sin = ç – 1÷ ç – ÷=–
2 f 3 è 2 ø è –20 ¥ ø 40
15. (b) u = –50 cm = –0.5 m
1 1é 1 3 1 ù 1 é –8 + 12 – 10 ù 1 é- 6ù
v = –30 cm = –0.3 m = ê– + – ú= ê =
f 5 ë 10 20 8 û 5 ë 80 ú
û 5 êë 80 úû
1 1 1 -1 1 -0.2 400
P= = - = + = = -1.33 D . or f =– cm = –66.6cm
f v u 0.3 0.5 0.15 6
16. (b) Object distance u = – 40 cm Hence the system behaves as a concave lens of focal
length 66.6 cm.
Focal length f = – 20 cm 19. (a) A
According to mirror formula
60°
1 1 1 1 1 1 Q d
+ = or = - R
i1 i2
u v f v f u r1 r2
1 1 1 1 1
or + - = +
v -20 ( -40 ) -20 40
1 -2 + 1 1
= =- or v = - 40 cm. B C
v 40 40 Given AQ = AR and ÐA = 60°
Negative sign shows that image is infront of concave
\ ÐAQR = ÐARQ = 60°
mirror. The image is real.
\ r = r = 30°
Magnification, m face AB.
EBD_7504
S-P-78 SOLUTIONS

sin i1 = µ sin r 1 Using mirror formula we get


1 3 1 1 1
Þ sin i1 = Ö 3 sin 30° = 3 ´ = = + , f = – 18.31 cm
2 2 f v u
\ i1 = 60° 23. (a) Here fo = 2 cm and fe = 3 cm.
Similarly, i2 = 60° Using lens formula for eye piece
In a prism, deviation
-1 1 1
d = i1 + i2 – A = 60° + 60° – 60° = 60° Þ + = Þ ue = – 3 cm
ue µ 3
20. (a) The ray will come out from CD if it suffers total internal
reflection at surface AD, i.e., it strikes the surface AD at But the distance between objective and eye piece is
15 cm (given)
critical angle C ( the limiting case).
\ Distance of image formed by the objective
P D = v = 15 – 3 = 12 cm.
A Let u be the object distance from objective, then for
C n n2 objective lens
1
Q 90°–C 1 1 1 -1 1 1
a - + = or + =
u v f u 12 2

B C -1 1 1 5 12
Þ = - = , u = - = -2.4 cm
Applying Snell's law at P u 2 12 12 5
n2 Apparent depth 1
n1 sin C = n2 or sin C = 24. (b) Since =
n1 Realdepth m
Applying Snell's law at Q Þ Apparent depth = d/m
n2 sin a = n1 cos C So mark raised up = Real depth – Apparent depth
d æ 1 ö æ m -1 ö
n1 ìï æ n ö ïü = d- = d çç1 - ÷÷ = çç ÷÷d
Þ sin a = cos ísin -1 ç 2 ÷ ý m è m ø è m ø
n2 ïî è n1 ø ïþ
25. (a) Considering refraction at the curved surface,
én ìï æ n ö üïù u = – 20, µ2 = 1
or a = sin -1 ê 1 cos ísin -1 ç 2 ÷ ýú µ1 = 3/2 , R = + 20
ëê n2 îï è n1 ø þïûú
m 2 m1 m 2 - m1
21. (d) In the later case microscope will be focussed for O'. So, it is Applying - =
required to be lifted by distance OO'. v u R
OO' = real depth of O – apparent depth of O. 1 3 / 2 1 - 3/ 2
Þ - = Þ v = -10
v -20 20
O' i.e., 10 cm below the curved surface or 10 cm above the
Image
} actual position of flower.
O 26. (b) Deviation = zero
3 é real depth ù So, d = d1 + d2 = 0
= 3- êµ = ú
1.5 apparent depth û Þ (m1 – 1)A1 + (m2 – 1) A2 = 0
ë
Þ A2 (1.75 – 1) = – (1.5 – 1) 15°
é1 . 5 - 1 ù 3 ´ . 5
= 3ê ú= = 1 cm 0.5
ë 1. 5 û 1. 5 Þ A2 = - ´ 15°
0.75
22. (c) The image I ' for first refraction (i.e., when the ray comes
or A2 = – 10°.
out of liquid) is at a depth of
Negative sign shows that the second prism is inverted
33.25 é Re al depth ù
= = 25cm êQ Apparent depth = ú
with respect to the first.
1.33 ë m û 27. (a) Magnification
Now, reflection will occur at concave mirror. For this I' Angle subtended by
behaves as an object. f0 final image on the eye
\ u = – (15 + 25) = – 40 cm = =
f e Angle subtended by
é 25 ù the object on eye (or objective)
and v = - ê15 +
ë 1.33 úû 0.3m b 30 cm b
Þ = Þ =
25 3cm 0.5° 3cm 0.5 °
Where
1.33
Physics S-P-79

28. (b) From lens formula


360 360
1 æ 1 1 ö 29. (b) When q = 90° then = =4
= (a µg - 1) ç - q 90
fa è R1 R 2 ÷ø
is an even number. The number of images formed is
1 æ 1 1 ö given by
= (1.5 - 1) çç - ÷
÷ ...(i)
fa R
è 1 R 2ø
360 360
When lens is immersed in liquid, then n= -1 = -1 = 4 -1 =3
q 90
1 æ 1 1 ö
= (l µ g - 1) çç - ÷
÷
fl è R1 R 2 ø æ mg ö
ç - 1÷÷
ç
a mg 1.5 1 æ 1.5 ö æ 1 1 ö Pa è m a ø= +5
l mg = = 30. (b) = = -5
m 1 .7 Þ f = ç 1.7 - 1÷ çç - ÷÷
P1 æ m g ö - 100 / 100
a l è øè 1R R 2ø
l ç ÷
ç m - 1÷
1 - 0.2 æ 1 1 ö è 1 ø
= ç - ÷ ...(ii)
fl ç
1.7 è R1 R 2 ÷ø æ mg ö mg
- 5 çç - 1÷÷ = -1
Dividing eq. (i) by (ii), we get è m1 ø ma
fl 0.5
= = -4.25 or fl = – 4.25fa 1 .5 -1 1 .5 5
fa 0.2 -1 = (1 .5 - 1) = -0 .1 ; m1 = =
- m1 5 0.9 3
1. 7
Hence, focal length will increase and will change in
sign.
EBD_7504
S-P-80 SOLUTIONS

Speed Test-24

1. (d) Optical path difference 7. (a)


Dx = (m 2 – m1)t . 8. (b) Dx1 = (m1 - 1)t = (1.5 - 1)t = 0.5t
2. (b) æ4 ö 2
3. (b) Separation between slits are (r1=) 16 cm and (r2=) 9 cm. and Dx2 = (m2 - 1) ´ 2t = ç - 1÷ ´ 2t = t .
è3 ø 3
Actual distance of separation
As Dx2 > Dx1 , so shift will be along –ve y-axis.
= r1r2 = 16 ´ 9 = 12cm 9. (b) The width of the central maximum is given by
4. (a) For a circularly polarised light electric field remains 2lD
constant with time. b=
d
Þ If d ® 2d, then b decreases.
2
|E| é sin a ù
Also, intensity I = I0 ê
ë a úû

t pd sin q
where a=
l
5. (b) Let nth fringe of 2500 Å coincide with (n – 2)th fringe
\ I increases as d increases
of 3500Å.
\ 3500 (n – 2) = 2500 × n \ The central maximum will become narrower and brighter.
10. (c) Here Angle of incidence, i = 57
1000 n = 7000, n = 7
tan 57° = 1.54
\ 7th order fringe of 1st source will coincide with 5th
uglass = tan i
order fringe of 2nd source.
It means, Here Brewster’s law is followed and the
reflected ray is completely polarised.
6. (d) Let P is a point infront of one slit at which intensity is
Now, when reflected ray is analysed through a polaroid
to be calculated. From figure,
then intensity of light is given by malus law.
i.e. I = I0 cos2q
S1 P on rotating polaroid ‘q’ changes. Due to which intensity
first decreases and then increases.
d O B 11. (c) Here A 2 = a12 + a 22 + 2a1a 2 cos d
Q a1 = a2 = a
S2
2 2 2æ 2d ö
D \ A = 2a (1 + cos d) = 2a çè 1 + 2cos - 1÷
2 ø
Path difference = S2P – S1P d
Þ A 2 µ cos 2
æ 1d ö 2 2
= D2 + d 2 - D = D ç1 + ÷-D
d
è 2 D2 ø Now, I µ A
2
\ I µ A 2 µ cos 2
2
é d2 ù d2 d
= D ê1 + - 1ú = \ I µ cos 2 .
ë 2D 2 û 2D 2
Dl
d2 d 5l l 12. (c) b= , where D is the distance between the slits &
Dx = = = = d
2 ´10d 20 20 4 screen and d is the separation between the slits.
Place difference,
2Dl 4 Dl
b' = = = 4b
2p l p d/2 d
Df = , =
l 4 2 13. (c) Wavelength for which maximum obtained at the hole
So, resultant intensity at the desired point 'p' is has the maximum intensity on passing. So,
f
I = I0cos2 = I0
2
Physics S-P-81

20. (c)
xd 1 ´ 10-3 ´ 0.5 ´ 10 -3
l= = 21. (b) For first minima at P
nD n ´ 50 ´ 10-2 AP – BP = l
1 ´ 10-6 1000nm l
= =
n n AP – MP =
2
n = 1, l = 1000 nm ® Not in the given range
n = 2, l = 500nm
14. (b)
15. (c) For path difference l, phase difference= 2p rad. P

l p
For path difference , phase difference = rad.
4 2
As K = 4I0 so intensity at given point where path A f
M f O
l
difference is
4 B

2 æ pö æ p ö
K¢ = 4I0 cos ç ÷ ç cos = cos 45º ÷ 2p l
è 4ø è 4 ø So phase difference, f = ´ = p radian
l 2
K 22. (c) In Fraunhoffer diffraction, for minimum intensity,
= 2I0 =
2 l
Dx = m
l 2
16. (c) Angular width = = 10 -3 (given)
d For first minimum, m = 1
\ No. of fringes within 0.12° will be l
\ Dx =
0.12 ´ 2p 2
n= @ [2.09]
360 ´ 10-3 lD
23. (a) As b = and l b < l y ,
\ The number of bright spots will be three. d
17. (b) For reflection at the air-soap solution interface, the phase \ fringe width b will decrease
difference is p 24. (c) The nearest white spot will be at P, the central maxima.
2d d d
\ y= - =
3 2 6

Air
Soap solution d/2
µ = 1.4 2d/3
Glass P
y O
µ = 1.5

For reflection at the interface of soap solution to glass


also there will be a phase difference of p
\ The condition for max. intensity = 2µt = nl 25. (d) Order of the fringe can be counted on either side of the
for n, nl1 = (n – 1)l2 central maximum. For example, no. 3 is first order bright
n 420 nm = (n – 1) 630 nm fringe.
n(630 - 420) = 630, \ n(210) = 630 nm . l
DX C = l , D X A =
630 2
\ n= =3
210 l
D X C - DX A = = 300nm
This is the maximum order where they coincide 2
3 ´ 420 D
2 ´ 1.4 ´ t = 3 ´ 420 Þ t = = 450 nm. 26. (c) Distance of nth maxima, x = nl µl
2 ´ 1.40 d
18. (c) As l < l \ x blue < x green
19. (b)
EBD_7504
S-P-82 SOLUTIONS

db d æ lL ö
I0 I0/2 For b to be minimum =0 ça + ÷= 0
da da è a ø
27. (a)
a = lL
b min = lL + lL = 2 lL = 4lL
1 3 2
The intensity of light transmitted through third polaroid, r
29. (d) Let any R its components are
I r r r r
I = 0 cos 2 q R = Rx + Ry + Rz
2
\ intensity of light transmitted through the last polaroid r r r r
with | R |= R = | R |= R = Rx 2 + Ry 2 + Rz

æI ö
I' = ç 0 cos 2 q ÷. cos 2 (90 - q) = I 0 cos 2 q. sin 2 q
è 2 ø 2 Rx Ry Rz
& cosq x = , cosq y , cosqz =
R R R
I 0 ´ 4 sin 2 q. cos 2 q I 2
= = 0 ´ ( 2 sin q × cos q ) there cosqx, cosqy and cosqz one called direction
2´ 4 8 cosines.
r
æI ö Hence x + y + z = c (= R )
= ç 0 ÷ sin 2 2q
è8 ø
2 2 2
28. (a) Given geometrical spread = a So, magnitude of c = I + I + I = 3
l lL 1
Diffraction spread = ´ L = and cos q x =
a a 3
lL
The sum b = a + 30. (a)
a
Speed Test-25

1. (a) The energy possessed by photons of wavelength


2 ´ 1.6 ´ 10-19 ´ 15000
1240 v=
550 nm is = 2.25 eV 9.1 ´ 10-31
550
The energy possessed by photons of wavelength v = 7.26 × 107 m/s
1240 4. (b) As l decreases, y increases and hence the speed of
450 nm is = 2.76 eV
450 photoelectron increases. The chances of photo electron
The energy possessed by photons of wavelength to meet the anode increases and hence photo electric
1240 current increases.
350 nm is = 3.54 eV 5. (c) Since, stopping potential is independent of distance
350
For metal plate p : hence new stopping potential will remain unchanged
fp = 2 eV. i.e., new stopping potential = V0.
All the wavelengths are capable of ejecting electrons. 6. (a) Give that, only 25% of 200W converter electrical energy
Therefore, the current is maximum. Also as the work into light of yellow colour
function is lowest in p, the kinetic energy of ejected
electron will be highest and therefore, the stopping æ hc ö 25
çè ÷ø ´ N = 200 ´
potential is highest. l 100
For metal plate q :
Where N is the No. of photons emitted per second, h
fq = 2.5 eV.
Photons of wavelength 550 nm will not be able to eject = plank’s constant, c, speed of light.
electrons and therefore, the current is smaller than p. 200 ´ 25 l
The work function is greater than q therefore the N= ´
stopping potential is lower in comparison to p. 100 hc
For metal plate r :
fr = 3 eV 200 ´ 25 ´ 0.6 ´ 10 -6
= = 1.5 × 1020
Only wavelength of 350 nm will be able to eject electrons 100 ´ 6.2 ´ 10-34 ´ 3 ´ 108
and therefore, current is minimum. Also the stopping
potential is least. E
2. (a) For one photocathode 7. (d) As P =
C
1 2 hc
hf1 - W = mv ....(i) lp = ...(i)
2 1 E
For another photo cathode
h2
1 2 le2 = ...(ii)
hf 2 - W = mv2 ....(ii) mE
2
From equations (i) and (ii)
Subtracting (ii) from (i) we get
lp µ le2
1 2 1 2
( hf1 - W ) – ( hf 2 - W ) = mv - mv
2 1 2 2
8. (a) The work function has no effect on photoelectric current
so long as hn > W0. The photoelectric current is
m 2 2 proportional to the intensity of incident light. Since there
\ h( f1 - f 2 ) = (v 1 - v 2 )
2 is no change in the intensity of light, hence I1 = I2.
2 2 2h 9. (a) hn = W0 + Ek = 3.5 + 1.2 = 4.7 eV
\ v1 - v 2 = (f - f )
m 1 2 1
10. (a) eV = mv2
3. (a) The maximum kinetic energy of an electron accelerated 2
1
through a potential difference of V volt is
2
mv2 =eV 2eV 2 ´ 1.6 ´10 -19 ´ 20
Þv= =
m 9.1´ 10-31
2eV
\ maximum velocity v = = 2.65 ´10 6 m / s
EBD_7504
S-P-84 SOLUTIONS

11. (c) n ® 2 – 1 18. (c) As we know


E = 10.2 eV 1
kE = E – f lµ
V
Q = 10.20 – 3.57
h u0 = 6.63 eV 1 1 1 1 1
\ : : = 1: :
100 200 300 2 3
6.63 ´ 1.6 ´ 10-19 19. (b) According to Einsten’s photoelectric effect, the K.E.
u0 = = 1.6 × 1015 Hz of the radiated electrons
6.67 ´ 10-34
K.Emax = E – W
12. (b) According to Einstein’s photoelectric equation, hv =
1
f0 + Kmax mv 2 = (1 – 0.5) eV = 0.5 eV
2 1
We have
1
hv = f0 + 0.5 ...(i) mv 2 = (2.5 – 0.5) eV = 2 eV
2 2
and 1.2hv = f0 + 0.8 ...(ii)
v1 0.5 1
Therefore, from above two equations f0 = 1.0 eV. = = = 1/ 2
v2 2 4
1 hc 2( hc - lf )
13. (c) mv 2 = -f Þ v = 20. (b) By using hv – hv0 = Kmax
2 l lm
14. (d) hu – hu0 = EK, according to photoelectric equation, Þ h (v1 – v0) = K1 ..... (i)
when u = u0, EK = 0. And h(v2 – v0) = K2 ..... (ii)
Graph (d) represents EK – u relationship.
v1 - v0 K1 1 kv - v 2
1 Þ = = , Hence v 0 = 1 .
15. (a) According to relation, E = mv
2 v 2 - v0 K 2 K K -1
2
21. (d) V = 3000 volt.
2E 2eV
=v 1
m mv2 = eV Þ v =
2 m
h
l= 2 ´1.6 ´ 10-19 ´ 3000
2mE \ v=
9.1 ´ 10 -31
Because m1 < m3 < m2
= 32.6 × 106 = 3.26 × 107 m/s.
So for same l, E1 > E 3 > E 2 . 22. (d) Number of emitted electrons NE
16. (d) W0 = hn1 – eV1 µ Intensity
= h n2 – e V2 1
eV2 = h(n2 – n1) + eV1 µ
(Distance) 2
h ( n 2 - n1 ) Therefore, as distance is doubled, NE decreases by
V2 = + V1
e (1/4) times.
17. (a) As we know, 23. (b)
hc 24. (d) Photoelectrons are emitted if the frequency of incident
eVs = –Y light is greater than the threshold frequency.
l
hc hc
3eVo = –Y ...(1) 25. (d) - f = eV0
l l
hc
eVo = –Y ...(2)
2l hc f
v0 = -
3hc el e
3eVo = – 3Y ...(3)
2l For metal A For metal B
Multiplying eqn. (2) by (3) and subtracting it from
fA 1 fB 1
eqn.(1) = =
hc l hc l
hc
Y=
4l 1
As the value of (increasing and decreasing) is not
So, threshold wavelength, l
hc
l th = =
Y or lesser work function (f).
Physics S-P-85

26. (b) Work function f of metal = 2.28 eV h h h


Wavelength of light l = 500 nm = 500 × 10–9m de-Broglie wavelength, l = = =
mv m(eEt / m ) eEt
hc
KEmax = –f Rate of change of de-Broglie wavelength
l
6.6 ´ 10-34 ´ 3 ´ 108 dl h æ 1 ö -h
KEmax = -7 – 2.82 = ç - ÷÷ =
ç
5 ´ 10 dt eE è t ø e E t 2
2
KEmax = 2.48 – 2.28 = 0.2 ev 28. (a) Retarding potential depends on the frequency of
h h incident radiation but is independent of intensity.
lmin = =
p 2m ( KE )max 29. (a) From formula
20 h
´ 10-34 l=
3 2 mKT
=
2 ´ 9 ´10 -31 ´ 0.2 ´1.6 ´10 -19 6.63 ´10 -34
= m
25 2 ´ 1.67 ´10 -27 ´1.38 ´10 -23 T
lmin = × 10–9
9 [By placing value of h, m and k)
= 2.80 × 10–9 nm \ l ³ 2.8 × 10–9 m 30.8
= Å
eE T
27. (a) Here, u = 0 ; a = ;v=?;t=t
m
c 3 ´ 108
\ v = u + at = 0 +
eE
t 30. (b) l0 = = = 6 ´ 10-7 m = 6000Å
m v0 5 ´ 1014
EBD_7504
S-P-86 SOLUTIONS

Speed Test-26

mv2 e2 æ 1 b ö n 2 (n 2 - 1)
(c) As F = = Again, 6 = , in second case.
1. ç + ÷ 2
r 4p Î0 è r 2 r 3 ø
nh nh Or n 22 - n 2 - 12 = 0 or (n 2 - 4)(n 2 + 3) = 0.
and mvr = Þv=
2p 2 pmr Take positive root, or n 2 = 4

æ nh ö 1 e2 æ 1 b ö
2 2pKZe 2
\ mç ´ = Now velocity of electron u =
÷ ç + ÷ nh
è 2pmr ø r 4p Î0 è r 2 r3 ø
u1 n 2 4
\ = = .
1 b mn 2 h 2 4p Î0 u 2 n1 3
or, 2 + 3 =
r r 4p2 m2 e2 r 3 kze 2
2 5. (b) E = – 3.4 eV and r =
a0 n 1 b 2E
or, = + angular momentum = mvr
3 2
r r r3
1
æ Î h2 ö Þ mv2 = E = 3.4´ (10-19 ´ 1.6)
çQa 0 = 0 Given ÷ 2
ç mpe 2 ÷
è ø Þ m2v2 = (9.1´ 10-31 )2 ´ 3.4´ 1.6 ´ 10 -19
For nth orbit = 99.008 × 10–50
\ rn = a0n2 – b
mv = 9.95028 × 10–25
2. (c) Magnetic moment of the hydrogen atom, when the
æ 9 ´ 109 ´ 1´ (1.6 ´ 10-19 ) 2 ö
electron is in nth excited state, i.e., n¢ = (n + 1)
\ L = (9.95028 × 10–25) çç 2 ´ (3.4)
÷
÷
As magnetic moment Mn = InA = in(prn2) è ø
= 2.10 × 10–34 Js.
mz 2 e5
i n = eVn = 6. (b) Radius of circular path followed by electron is given
4e02 n 3 h 3
by,
n 2h2
æ 1 ö mu 2meV 1 2m
rn = çk = ÷ r= = = V
4p2 kzme 2 è 4p Î0 ø qB eB B e
Solving we get magnetic moment of the hydrogen atom B2 r 2 e
for nth excited state Þ V= = 0.8V
2m
æ e ö nh For transition between 3 to 2.
Mn ' = ç ÷
è 2m ø 2p æ 1 1 ö 13.6 ´ 5
E = 13.6 ç - ÷ = = 1.88eV
1 N sin 4 (q1 / 2) è 4 9ø 36
3. (c) Nµ 4 ; 2 =
sin q / 2 N1 sin 4 (q2 / 2) Work function = 1.88 eV – 0.8 eV = 1.08 eV » 1.1eV
7. (d) Radius of n th orbit r n µ n2, graph between rn and n is a
N2 sin 4 (60° / 2)
or = rn æ n ö
2
5 ´ 10 6
sin 4 (120° / 2) ær ö
parabola. Also, = ç ÷ Þ log e ç n ÷ = 2log e (n)
r1 è 1 ø è r1 ø
N2 sin 4 30°
or =
5 ´ 106 sin 4 60° Comparing this equation with y = mx + c,
4 4
æ 1ö æ 2 ö 5
Graph between log e æç n ö÷ and loge (n) will be a straight
r
orN 2 = 5 ´ 106 ´ ç ÷ ç ÷ = ´ 10 6
è 2ø è 3 ø 9 è r1 ø
4. (a) Number of emission spectral lines line, passing from origin.
n(n - 1) Similarly it can be proved that graph between
N=
2 æf ö
log e ç n ÷ and log (n) is a straight line. But with
n1 (n1 - 1) è f1 ø e
\3 = , in first case.
2 negative slops.
Or n12 - n1 - 6 = 0 or (n1 - 3)(n1 + 2) = 0 hc hc 6.62 ´ 10-34 ´ 3 ´ 108
E= Þl= =
Take positive root. 12.5 ´ 1.6 ´ 10 -19
\ n1 = 3
Physics S-P-87

1 æ 1 1ö 1 4 æ 1 1ö
1 æ 1 1 ö \ = Rç 2 – 2 ÷ Þ =
= Rç - ÷ ç – ÷
çn 2 n 2 ÷ l è n1 n2 ø l 3 ´ 122 ´ 10 è 32 ¥ ø
–9
l è 1 2 ø
3 ´ 122 ´ 9 ´ 10 –9
(where Rydberg constant , R = 1.097 × 107) \ l= = 823.5nm
4
æ1 ö 12. (b) KEmax = 10eV
1 1
or, = 1.097 ´ 107 ç - ÷ f = 2.75 eV
993 ´ 10 -10 ç 2
n 22 ÷
è1 ø Total incident energy
Solving we get n 2 = 3 E = f + KEmax = 12.75 eV
Spectral lines \ Energy is released when electron jumps from the
Total number of spectral lines = 3 excited state n to the ground state.
Two lines in Lyman series for n1 = 1, n2 = 2 and n 1 = 1, Q E4 – E1 = {– 0.85 – (–13.6) ev}
n2 = 3 and one in Balmer series for n 1 = 2 , n 2 = 3 = 12.75eV
n=3
\ value of n = 4
Balmer 13. (a) Energy of ground state 13.6 eV
n=2
Energy of first excited state
Lyman Lyman
n=1 13.6
=- = -3.4 eV
4
nh Energy of second excited state
9. (b) l= ,| E |µ Z 2 / n2 ; n = 3
2p 13.6
=- = -1.5 eV
Þ lH = lLi and |EH| < |EL i| 9
10. (a) Given potential energy between electron and proton Difference between ground state and 2nd excited state
r = 13.6 – 1.5 = 12.1 eV
= eV0 log [Q | U |= eV ] So, electron can be excited upto 3rd orbit
r0
No. of possible transition
d é r ù eV0 1 1 ® 2, 1 ® 3, 2 ® 3
\ |F| = êeV0 loge ú = ´
dr ë r0 û r0 r So, three lines are possible.
But this force acts as centripetal force 14. (c)
n=3
mv 2 eV0 eV Case (I)
\ = Þ mv 2 = 0 ...(i)
r rr0 r0
Energy states

nh
By Bohr’s postulate, mvr = ....(ii) n=2
2p Case (II)
From (i) and (ii),
II
m2 v 2 r 2 n2 h2 r0
= n=1
mv 2 4p 2 ´ V0 e 1
2 2 The wave number ( n) of the radiation =
n h r0 l
Þ r2 = Þrµn
4pV0 me é 1 1ù
For n th orbit rn µ n = R¥ ê 2 - 2 ú
êë n1 n2 úû
11. (b) The smallest frequency and largest wavelength in Now for case (I) n1 = 3, n2 = 2
ultraviolet region will be for transition of electron from
orbit 2 to orbit 1. 1 é 1 -1 ù
= R¥ ê , R = Rydberg constant
æ 1
l1 ë 9 4 úû ¥
1 1ö
\ = Rç 2 – 2 ÷ 1 é 4 - 9 ù -5R¥
l è n1 n2 ø = R¥ ê =
l1 ë 36 úû 36
1 é1 1ù é 1 ù 3R
Þ = R ê 2 – 2 ú = R ê1 – ú = - 36
122 ´ 10 m –9
ë1 2 û ë 4û 4 Þ l1 =
5 R¥
Þ R= 4
m –1 1 é 1 1 ù -3R¥
3 ´ 122 ´ 10–9 = R¥ ê - ú =
l2 ë 4 1û 4
The highest frequency and smallest wavelength for
infrared region will be for transition of electron from ¥ -4
to 3rd orbit.
EBD_7504
S-P-88 SOLUTIONS

l1 -36 3R¥ 19. (a) For Lyman series


Þ = ´
l 2 5 R¥ -4 é1 1 ù
u = RC ê 2 - 2 ú
l1 27 ë1 n û
= where n = 2, 3, 4, .......
l2 5 For the series limit of Lyman series, n = ¥
é 1 1 ù é1 1 ù
15. (b) E = Rhc ê 2 - 2 ú \ u1 = RC ê - ú = RC ...(i)
êë n1 n2 úû ë1 ¥ 2 û
2
For the first line of Lyman series, n = 2
E will be maximum for the transition for which
é1 1 ù 3
é 1 \ u2 = RC ê - ú = RC ...(ii)
1 ù
ê - ú is maximum. Here n2 is the higher energy ë12 22 û 4
êë n12 n2 2 úû
For Balmer series
level. é1 1 ù
u = RC ê 2 - 2 ú
é ù ë2 n û
Clearly, ê 1 - 1 ú is maximum for the third transition, where n = 3, 4, 5 .....
2
êë n12
n2 úû For the series limit of Balmer series, n = ¥
i.e. 2 ® 1. I transition represents the absorption of
é1 1 ù RC
energy. \ u3 = RC ê - ú= 4 ...(iii)
2
16. (a) Speed of electron in nth orbit ë2 ¥2 û
From equation (i), (ii) and (iii), we get
2p KZe 2 u1 = u2 + u3 or u1 - u2 = u3
Vn =
nh é 1
1 1 ù 1
Z 20. (c) Wave number = RZ 2 ê - ú Þ l µ
V = (2.19 × 106 m/s) l 2 2
êë n1 n úû Z2
n
By question n = 1 and n 1 = 2
2
V = (2.19 × 106) (Z = 2 & n = 3) Then, l1 = l2 = 4l3 = 9l4
3
V = 1.46 × 106 m/s 21. (a) At closest distance of approach, the kinetic energy of
the particle will convert completely into electrostatic
1 1 c–v potential energy.
17. (b) =
l' l c+v 1
Kinetic energy K.E. = mv2
2
Here, l ' = 706 nm, l = 656 nm KQq
Potential energy P.E. =
2 2 r
c–v æ l ö æ 656 ö
\ =ç ÷ =ç ÷ = 0.86 1 KQq 1
c+ v è l'ø è 706 ø mv 2 = Þ rµ
2 r m
v 0.14 22. (d) For an atom following Bohr’s model, the radius is given
Þ =
c 1.86 by
Þ v = 0.075 × 3 × 108 = 2.25 × 107m/s r m2
rm = 0 where r0 = Bohr’s radius and m = orbit
18. (a) For emission, the wave number of the radiation is given Z
number.
as For Fm, m = 5 (Fifth orbit in which the outermost electron
1 æ 1 1ö is present)
= Rz 2 ç 2 - 2 ÷
l è n1 n2 ø r0 52 1
\ rm = = nr0 (given) Þ n = 4
R = Rydberg constant, Z = atomic number 100
æ1 1ö æ 1ö 1 24 1 é 1 1 ù
23. (c) = Rê 2 - 2ú
= R çè 2 - 2 ÷ø = R çè 1 - ÷ø Þ = R l
1 5 25 l 25 ë n1 n 2 û
linear momentum 1 é1 1 ù
Þ = 1.097 ´ 107 ê 2 - 2 ú Þ n 2 = 4
h 24 970.6 ´ 10 -10
P = = h × R× (de-Broglie hypothesis) ë1 n2 û
l 25
24hR n(n - 1) 4 ´ 3
Þ mv = N= = =6
25 2 2
Physics S-P-89

24. (a) Energy of electron in nth orbit is 1 L2


Z2 E= (m1r12 + m2 r2 2 ) 2
En = - (Rch) 2 = - 54.4 eV 2 I
n nh
For He+ is ground state L=n (According Bohr's Hypothesis)
2n
(2) 2
E1 = - (Rch) = -54.4 Þ Rch = 13.6
(1)2 1 L2
E= (m1r12 + m2 r22 )
\ For Li++ in first excited state (n = 2) 2 (m1r12 + m2 r22 )2
(3)2
E' = – 13.6 × = - 30.6 eV 1 L2 n 2 h2
(2) 2 E= =
2 (m1r12 + m2 r22 ) 8p2 (m1r12 + m2 r22 )
25. (b) For 2nd line of Balmer series in hydrogen spectrum
1 æ 1 1ö 3 (m1 + m2 )n 2 h 2 é m2 r m2 r ù
= R (1) ç 2 - 2 ÷ = R E= êQ r1 = ; r2 = ú
l è2 4 ø 16 8 p2 r 2 m1m2 ë m1 + m 2 m1 + m2 û
2
é1 æ 1 1 ö 3R ù k k mv
For Li2+ê = R ´9 ç 2 - 2 ÷ = ú 28. (b) When F = = centripetal force, then =
ël èx 12 ø 16 û r r r
which is satisfied by n = 12 ® n = 6. Þ mv2 = constat Þ kinetic energy is constant
26. (a) Velocity of electron in nth orbit of hydrogen atom is Þ T is independent of n.
29. –
(d) When one e is removed from neutral helium atom, it
given by :
becomes a one e– species.
2pKZe2 For one e– species we know
Vn =
nh
–13.6Z 2
Substituting the values we get, En = eV/atom
n2
2.2 ´ 106 1
Vn = m/s or Vn µ For helium ion, Z = 2 and for first orbit n = 1.
n n
–13.6
As principal quantum number increases, velocity \ E1 = ´ 22 = – 54.4 eV
decreases. (1) 2
27. (d) The energy of the system of two atoms of diatomic \ Energy required to remove this e– = + 54.4 eV
\ Total energy required = 54.4 + 24.6 = 79 eV
1 2 30. (a) As the electron comes nearer to the nucleus the
molecule E = Iw
2 potential energy decreases
where I = moment of inertia æ – k .Ze 2 ö
L çQ = P.E. and r decreases÷
è r ø
w = Angular velocity = ,
I
L = Angular momentum é 1 1 kZe2 ù
The K.E. will increase êQ K.E. = | P.E. | = ú
1 ë 2 2 r úû
I = (m1r12 + m2 r22 )
2
é 1 kZe2 ù
1 2 2 2 The total energy decreases êT.E. = – ú
Thus, E = (m1r1 + m2 r2 )w … (i) ëê 2 r ûú
2
EBD_7504
S-P-90 SOLUTIONS

Speed Test-27

1. (c) Energy is released in a process when total binding energy 9. (c) The range of energy of b-particles is from zero to some
(BE) of products is more than the reactants. By calculations maximum value.
we can see that this happens in option (c). 10. (a) Mass defect = ZMp + (A –Z)Mn–M(A,Z)
Given W = 2Y B.E.
BE of reactants = 120 × 7.5 = 900 MeV
or, = ZMp + (A–Z) Mn–M(A,Z)
c2
BE of products = 2 × (60 × 8.5) = 1020 MeV.
B.E.
\ M (A, Z) = ZMp + (A–Z)Mn–
TaTb c2
2. (c) Tav = 11. (d)
Ta + Tb BE/A

If a and B are emitted simultaneously.


3. (c) No. of nuclide at time t is given by N = Noe–lt
Where No = initial nuclide
This equation is equivalent to y = ae–kx
Thus correct graph is
A
N
From the graph of BE/A versus mass number A it is
clear that, BE/A first increases and then decreases with
increase in mass number.
180 a b
t 12. (a) 72 A ® 70 A1176 ¾¾
¾¾ ® 71 A 2176
4. (b) By conservation of energy, a g
®69 A3172 ¾¾®69 A 4172
¾¾
2.M 2 1 2M 2
( M + Dm ) c2 = c + . v ,
2 2 2 13. (a) 1
® 11H + -1e 0 + n + Q
¾¾
0n
where v is the speed of the daughter nuclei
The mass defect during the process
2 M 2 2 Dm Dm = mn - mH - me = 1.6725 × 10–27
Þ Dmc = v \v=c
2 M
– (1.6725 × 10–27+ 9 × 10–31kg)
5. (d) Radioactivity at T1 , R1 = l N1 = – 9 × 10–31 kg
Radioactivity at T2, R2 = l N2 The energy released during the process
\ Number of atoms decayed in time E = Dmc2
(T1 – T2) = (N1 –N2) E = 9 × 10–31× 9 × 1016 = 81 × 10–15 Joules

(R 1 - R 2 ) (R1 - R 2 ) T 81 ´ 10 -15
= = µ (R1 - R 2 ) T E= = 0.511MeV
l 0.693 1.6 ´ 10 –19
6. (a) Radioactive decay is a continuous process. Rate of 14. (a) As we know, R = R0 (A)1/3
radioactive decay cannot be controlled. Nuclear fission where A = mass number
can be controlled but not of nuclear fusion. RAI = R0 (27)1/3 = 3R0
7. (d) p1 = p 2 5
RTe = R0 (125)1/3 = 5R0 = R
m1v1 = m 2 v2 3 AI
2m1 = m2 15. (c)
16. (a) From eqn. (ii), B has 2 units of charge more than C.
3
4 3 4 3 R1 = 1: 2 From eqn. (i), A loses 2 units of charge by emission of
2r. pR1 = r. pR 2 ; 3 ; R1 : R2 = 1 : 21/3 alpha particle. Hence,
3 3 R2
A and C are isotopes as their charge numbers are same.
8. (b)
Physics S-P-91

17. (d)
R1 = R2 e l (t2 -t1 ) = R2 e -l (t1 -t2 )
18. (c) We assume that mass number of nucleus when it was
at rest = A 24. (a) Iodine and Yttrium are medium sized nuclei and
therefore, have more binding energy per nucleon as
Q Mass number of a -particle = 4
compared to Uranium which has a big nuclei and less
\ Mass number of remaining nucleus = A – 4
B.E./nucleon.
As there is no external force, so momentum of the
25. (d) Nuclear force is not the same between any two nucleons.
system will remain conserved.
26. (c) Binding energy per nucleon for fission products is higher
4v relative to Binding energy per nucleon for parent nucleus,
Þ 0 = (A - 4)v¢ + 4v Þ v' = -
(A - 4) i.e., more masses are lost and are obtained as kinetic
negative sign represents that direction is opposite to energy of fission products. So, the given ratio < 1.
the direction of motion of a -particle. 27. (d) For A + B ® C + e, e is positive. This is because Eb
19. (d) Because radioactivity is a spontaneous phenomenon. for C is greater than the Eb for A and B.
20. (b) The number of a- particles released =8 Again for F ® D + E + e, e is positive. This is
Therefore the atomic number should decrease by 16
because Eb for D and E is greater than Eb for F .
The number of b–-particles released = 4
28. (a)
Therefore the atomic number should increase by 4.
Also the number of b+ particles released is 2, which ( m1 - m2 )v1 + 2 m2 v2
29. (a) v¢1 =
should decrease the atomic number by 2. m1 + m2
Therefore the final atomic number As v2 is zero, m2 > m1, v '1 is in the opposite direction.
= Z –16 + 4 – 2 = Z –14 m1 = 1, m2 = A.
= 92 – 14 = 78 ( A - 1)
\ | v ¢1 |= v
ln 2 ln 2 ( A + 1) 1
21. (a) T1/ 2 = \l =
l T1/ 2 The fraction of total energy retained is
In2 ln 2 l T 1/ 2mv ¢12 ( A - 1)2
Þ lA = ,l B = Þ A= B. =
TA TB l B TA 1/ 2mv12 ( A + 1)2

æ Eö n ´ 200 ´ 106 ´ 1.6 ´ 10 -19 30. (c) Average life of the nuclei is
22. (a) P = n ç ÷ Þ 1000 =
è tø t 1
n tav = ....(i)
Þ = 3.125 ´ 1013. l
t Half life of the nuclei
23. (d) Let at time t1 & t2, number of particles be N1 & N2. So,
0.693
dN1 dN 2 t1/2 = ....(ii)
R1 = = -l N1 ; R2 = = -l N 2 l
dt dt from (i) and (ii)
R1 lN1 N1
= = = e l ( t2 -t1 ) t1/2
R2 lN 2 N1e -l (t2 -t1 ) tav =
0.693
EBD_7504
S-P-92 SOLUTIONS

Speed Test-28

1. (d) Here, ni = 1016 m–3, nh = 5 × 1022 m–3 60 - 10 50


As nenh = n2i = 3
= = 12.5 mA
4 ´ 10 4 ´ 103
ni2 (1016 m -3 )2 From circuit diagram,
\ ne = = = 2 ´ 109 m -3 I = IZ + IL
n h 5 ´ 1022 m -3
2. (d) DIE = 8.0 mA Þ 12.5 = IZ + 5
DIC = 7.9 mA Þ IZ = 12.5 – 5 = 7.5 mA.
DI 7.9 Vo R 5 ´103 ´ 62
a= C = = 0.9875 ; 0.99 8. (b) = o ´b = = 10 ´ 62 = 620
DIE 8.0 Vin R in 500
a 0.9875 Vo = 620 × Vin= 620 × 0.01 = 6.2 V
Also, b = = = 79
1 - a (1 - 0.9875) \ Vo = 6.2 volt.
3. (b) Output of upper AND gate = AB 9. (c) P(0) 0 X(0)
Q(1)
Output of lower AND gate = AB Z(0)
\ Output of OR gate, Y = A B + BA R(0) 0 1 Y(1)
This is boolean expression for XOR gate. S(1)
4. (c) In n-type semiconductors, electrons are the majority Vm 25
charge carriers. 10. (a) Im = =
R f + R L (10 + 1000)
= 24.75 mA
5. (c) No. of electrons reaching the collector,
Im 24.75
96 Idc = = = 7.87 mA
nC = ´ 1010 = 0.96 ´ 1010 p 3.14
100
n ´e Im 24.75
Emitter current, IE = E Irms = = = 12.37 mA
2 2
t
Pdc = Idc2 × RL = (7.87 × 10–3)2 × 103 = 61.9 mW
n ´e
Collector current, IC = C Pac = Irms 2(Rf + RL) = (12.37 × 10–3)2 × (10 + 1000)
t = 154.54 mW
\ Current transfer ratio,
Rectifier efficiency
IC n C 0.96 ´ 1010
a= = = = 0.96 Pdc 61.9
IE n E 1010 h=
Pac
× 100 = × 100 = 40.05%
154.54
6. (a) A 1
Y = A.AB B.AB

11. (a) Conductivity, σ = = e(n eμ e + n h μ h )


Y2 = A.AB ρ
2.13 = 1.6 × 10–19(0.38 + 0.18) ni
Y1 = AB (Since in intrinsic semi-conductor, ne = nh= ni)
\ density of charge carriers, n i
B 2.13
Y3 = B.AB = = 2.37 ´ 1019 m -3
1.6 ´ 10 -19 ´ 0.56
By expanding this Boolen expression
12. (b) D2 is forward biased whereas D1 is reversed biased.
Y = A.B + B.A So effective resistance of the circuit
Thus the truth table for this expression should be (a). R = 4 + 2 = 6W
7. (d) Here, R = 4 kW = 4 × 103 W 12
Vi = 60 V \i = = 2 A.
6
Zener voltage Vz = 10 V 13. (b) It is a p-n-p transistor with R as base.
RL = 2 kW = 2 × 103 W 14. (c) (W + X) × (W + Y) = W + (X × Y)
V 10 15. (d) In pure semiconductor electron-hole pair = 7 × 1015/m3
Load current, IL = Z = = 5 mA
RL 2 ´ 103 ninitial = nh + ne = 14 × 1015 after doping donor Impurity

Current through R 21 and


ND
ne = = 2.5 × 1021
2
Physics S-P-93

So, nfinal = nh + ne The output voltage, across the load RC


V0 = IC RC = 2
Þ nfinal » ne » 2.5 × 1021 (Q ne >> nh)
The collector current
n final - n initial
Factor = 2
n initial IC = 3
= 10-3 Amp
2 ´ 10
2.5 ´ 1021 - 14 ´1015 2.5 ´1021 Current gain (b)
= » = 1.8 × 105
14 ´1015 14 ´1015 IC
16. (d) In common emitter configuration current gain = = 100
IB
-hf e -50
Ai = = IC 10-3
1 + hoe RL 1 + 25 ´ 10-6 ´ 1 ´ 103 = –48.78 IB = = = 10 -5 Amp
100 100
17. (c)
Input voltage (Vi)
R out
18. (d) Voltage gain G = b Vi = RB IB = 1 × 103 × 10–5 = 10–2 Volt
R in
R out Vi = 10 mV
Þ G = 25 R ...(i)
in 22. (b) The power gain in case of CE amplifier,
b Power gain = b2 × Resistance gain
Transconductance gm =
R in R
= b2 ´ o
b 25 Ri
Þ Rin = =
gm 0.03 = (10)2 × 5 = 500.
Putting this value of Rin in eqn. (i)
23. (d) A
R
G = 25 out × 0.03 ...(ii)
25
R out
\ G' = 20 × 0.02 ...(iii) C
20
From eqs. (ii) and (iii) B
2
Voltage gain of new transistor G' = G The truth table for the above logic gate is :
3
19. (a) Positive terminal is at higher potential (–5V) and A B C
negative terminal is at lower potential –10 V. 1 1 1
20. (d) Energy band gap range is given by,
1 0 1
hc
Eg = 0 1 1
l
For visible region l = (4 × 10–7 ~ 7 × 10–7)m 0 0 0
6.6 ´ 10 -34 ´ 3 ´ 108 This truth table follows the boolean algebra C = A + B
Eg = which is for OR gate
-7
7 ´ 10
24. (c) A crystal structure is composed of a unit cell, a set of
19.8 ´ 10-26 atoms arranged in a particular way; which is periodically
=
7 ´ 10 -7 repeated in three dimensions on a lattice. The spacing
2.8 ´ 10-19 between unit cells in various directions is called its
= lattice parameters or constants. Increasing these lattice
1.6 ´ 10-19 constants will increase or widen the band-gap (Eg),
Eg = 1.75 eV which means more energy would be required by
21. (d) electrons to reach the conduction band from the
IC valence band. Automatically Ec and Ev decreases.
25. (b) I = nA evd or I µ nvd
RC O/P = 2 Volt Ie n v n I v 7 4 7
\ = e e or e = e ´ h = ´ =
Ih n h vh n h Ih ve 4 5 5
Input RB
DV 2.1 - 2 1
26. (b) R= = = = 0.25W
-3 4
´ 10
EBD_7504
S-P-94 SOLUTIONS

27. (b) A Y1 29. (a) Here, diodes D1 and D2 are forward biased and D3 is
B reverse biased.
Therefore current through R3
Y
V 6 1
i= = = A = 50 mA .
R ' 120 20
Y2
30. (a) Current gain (a) = 0.96
Y1 = A + B, Y2 = A . B Ie = 7.2 mA
Y = (A + B)gAB = A g A + A g B + Bg A + Bg B Ic
= a = 0.96
= 0 + AgB + BgA + 0 = AgB + BgA Ie
28. (d) Copper is a conductor, so its resistance decreases on
I c = 0.96 ´ 7.2 mA = 6.91 mA
decreasing temperature as thermal agitation decreases;
whereas germanium is semiconductor therefore on Ie = Ic + Ib
decreasing temperature resistance increases. Þ Ib = Ie – Ic = 7.2 – 6.91 = 0.29 mA
Speed Test-29

1. (d) The maximum line-of-sight distance between the 16. (d) For good demodulation,
transmitting and receiving antennas is 1 1
<< RC or, RC >>
dM = 2 RhT + 2 RhR f f
where hT and hR are the heights of transmitting and æP ö
17. (a) G = 10 log10 ç 0 ÷ dbs
receiving antennas respectively.
è Pn ø
\ dM = 2 Rh + 2 Rh = 2 2 Rh = 8Rh G µ P0 (® power radiated by isotropic antenna)
(Q hT = hR = h) 1
Gµ (® power radiated by practical antenna)
2. (b) The frequencies present in amplitude modulated Pn
wave are : B
Carrier frequency = wc 18. (b) Modulation index =
A
Upper side band frequency = wc + wm B = 25, A = 60
Lower side band frequency = wc – wm.
25
3. (b) Band width Þ M.I. = = 0.416 Þ m% = 41.6%
= 2 × frequency 60
= 2 × 5000 19. (d) Frequency of EM wave u = 830 kHz
= 10 kHz = 830 × 103 Hz.
4. (c) For x(t), BW = 2(Dw + w) Magnetic field, B = 4.82 × 10–11 T
c
Dw is deviation and w is the band width of modulating As we know, frequency, u =
signal. l
c 3 ´ 108
\ BW = 2(90 + 5) = 190 or l = =
v 830 ´ 103
For x2 (t), BW = 2 × 190 = 380 l ; 360 m
5. (c) Comparing given expression with And, E = BC = 4.82 × 10–11 × 3 × 108
(e)AM = Ec(1 + ma cos wmt) cos wct = 0.014 N/C
peak value of carrier wave, Ec= 10V. 20. (a) Here, fc = 1.5 MHz = 1500 kHz, fm = 10 kHz
6. (b) p µ (l/l)2
\ Low side band frequency
7. (d)
= fc – fm = 1500 kHz – 10 kHz = 1490 kHz
8. (c)
Upper side band frequency
9. (d)
10. (b) For given transmission band 88-108 MHz (Df)max = 75 = fc + fm = 1500 kHz + 10 kHz = 1510 kHz
kHz 21. (b) The frequency of AM channel is 1020 kHz whereas for
the FM it is 89.5 MHz (given). For higher frequencies
given ( Df )actual = 18.75 kHz
(MHz), space wave communication is needed. Very tall
towers are used as antennas.
( Df )actual 18.75
\ % modulation m = ´ 100 = = 25% 22. (a) The critical frequency of a sky wave for reflection from
( )max
D f 75 a layer of atmosphere is given by fc = 9(Nmax)1/2
11. (a) Comparing (x AM)t = 100 [1 + 0.5 t] coswct for 0<t<1 Þ 10 × 106 = 9(Nmax)1/2
2
with standard AM signal x AM = Ec [1+ma cos wmt] æ 10 ´ 106 ö
÷ ; 1.2 ´10 m
12 –3
Þ N max =ç
cos wct è 9 ø
We have modulating signal t and ma = 0.5. Pt 12 12
23. (c) Pc = = = = 9.6 kW
12. (b)
1+
m a2
1+
( 0.5) 2 1.25
13. (d) When ma > 1 then carrier is said to be over modulated.
2 2
14. (d)
24. (c) Above critical frequency (fc), an electromagnetic wave
24 8 penetrates the ionosphere and is not reflected by it.
15. (d) Here Vmax = = 12 mV and Vmin = = 4 mV
2 2 25. (c) Modulation index
Vmax – Vmin 12 – 4 8 1 E A
Now, m =
Vmax + V
EBD_7504
S-P-96 SOLUTIONS

Equation of modulated signal [Cm(t)] ma 2


= E(C) + maE(C) sin wmt 28. (c) Average side-band power Pav = Pc
4
= A (1+ sin wmt) sin wCt Here ma = 0.5
(As E(C) = A sin wCt) Pc = 10
26. (a) Modulation is a process of superposing a low 0.5 ´ 10 ´ 10
frequency audio signals (called modulating signal) on \ Pav = = 6.25
4
a high frequency radio wave called carrier wave. 29. (a)
27. (b) Ec = 100 V, ma = 0.4, R = 100 W,
R 68
E 2 (100) 2 30. (d) h= = = 0.872
Pc = c = = 50 watt R + R L 68 + 10
2R 2 ´ 100
Þ Power gain = h G
æ m 2 ö é (0.4) 2 ù
P = ç1 + a ÷ Pc = ê1+ ú ´ 50 = 54 watt Power gain 16
ç 2 ÷ 2 ûú Þ directive gain G = = = 18.35
è ø ëê h 0.872
HINTS & SOLUTIONS (CHEMISTRY – Chapter-wise Tests)
Speed Test-30
1. (b) (b) 6.023 × 1023 = 1 mole
Wt. in g Thus 6.023 × 1022 = 0.1 mole
2. (a) No. of moles = Weight of N2 = 0.1 × 28 = 2.8 g
Mol. wt
200 (c) Weight of silver = 0.1 g
No. of moles in 200 mg = (d) Weight of oxygen = 32 × 0.1 = 3.2 g
1000 ´ 44
= 4.5 × 10–3 moles 9. (d) 1 Mole of Mg3(PO4)2 contains 8 mole of oxygen atoms
No. of moles in 1021 molecules \ 8 mole of oxygen atoms º 1 mole of Mg3(PO4)2 mole
of Mg3(PO4)2
10 21
= = 1.67 × 10–3 moles 1
6.02 ´ 1023 0.25 mole of oxygen atom º ´ 0.25 mole of Mg3(PO4)2
No. of moles left = (4.5 – 1.67) × 10–3 = 2.88 × 10–3 8
-2 mole of Mg (PO )
3. (a) 2BCl3 + 3H 2 ® 2B + 6HCl = 3.125 ´ 10 3 4 2
10. (d) Since molarity of solution is 3.60 M. It means 3.6 moles
3
or BCl3 + H 2 ® B + 3HCl of H2SO4 is present in its 1 litre solution.
2 Mass of 3.6 moles of H2SO4
Now, since 10.8 gm boron requires hydrogen
= Moles × Molecular mass
3
= ´ 22.4L at N.T.P = 3.6 × 98 g = 352.8 g
2 \ 1000 ml solution has 352.8 g of H2SO4
hence 21.6 gm boron requires hydrogen Given that 29 g of H2SO4 is present in = 100 g of solution
3 22.4 \ 352.8 g of H2SO4 is present in
´ ´ 21.6 < 67.2L at N.T. P..
2 10.8 100
4. (c) M. Wt of Na2SO4.10 H2O is 322 g which contains 224 g = ´ 352.8 g of solution
29
oxygen. \ 32.2 g will contain 22.4 g oxygen. = 1216 g of solution
6.02 ´ 1020 Mass 1216
5. (b) Moles of urea present in 100 ml of sol.= Density = = = 1.216 g/ml = 1.22 g/ml
6.02 ´ 10 23 Volume 1000
6.02 ´ 10 20 ´ 1000 PV PV
\M = = 0.01M 11. (b) From 1 1 = 2 2
T1 T2
6.02 ´ 10 23 ´ 100
[ Q M = Moles of solute present in 1L of solution] V1 ´ 640 620 ´ 300
6. (d) Relative atomic mass = =
(273 + 47) (273 + 27)
Mass of one atom of the element 620 ´ 300 ´ 320
th V1 = = 310 cc
1/12 part of the mass of one atom of Carbon - 12 640 ´ 300
Mass of one atom of the element 0.33
12. (d) Weight of Iron in 67200 = ´ 67200 = 221.76
or ´ 12 100
mass of one atom of the C - 12
Now if we use 1/6 in place of 1/12 the formula becomes 221.76
Number of atoms of Iron = = 3.96 º 4
56
Relative atomic mass =
13. (a) 20 volume H2O2 means that 1mL of this H2O2 solutions
Mass of one atom of element produces 20 mL of O2 at N.T.P. on decomposition by
´6
Mass of one atom of carbon heat.
\ Re lative atomic mass decrease twice \ For 20 mL of O2, the volume of 20 volume H2O2
7. (c) NaCl + H 2SO 4 ¾¾ ® NaHSO4 + HCl required = 1mL
xg 4.9g 6g 1.825g For 1 mL of O2, the volume of 20 volume
According to law of conservation of mass "mass is 1
H2O2 required = mL
neither created nor destroyed during a chemical 20
change" For 5000 mL or 5L of O2, the volume of 20
\ Mass of the reactants = Mass of products 1
volume H2O2 required = ´ 5000 mL = 250 mL
x + 4.9 = 6 + 1.825 20
or x = 2.925 g 14. (a) 2Al(s) + 6HCl(aq) ® 2Al3+(aq) + 6Cl– (aq) + 3H2(g)
8. (c) (a) Weight of H2
2
= 0.2 × 2 = L of H2 at S.T.P
EBD_7504
S-C-2 SOLUTIONS

3 ´ 22.4 Equivalent weight


\ 1 mole of HCl produces = L weight of metal
6 = ´ 35.5
of H2 at S.T.P weight of chlorine
= 11.2 L of H2 at STP 25.25
= ´ 35.5 = 12
15. (a) 95% H2SO4 by weight means 100g H2SO4 solution 74.75
contains 95g H2SO4 by mass. 2 ´ V.D.
Molar mass of H2SO4 = 98g mol–1 Valency of metal =
Equivalent wt. of metal + 35.5
95 2 ´ 94.8
Moles in 95g = = 0.969 mole = =4
98 12 + 35.5
Volume of 100g H 2SO 4 \ Formula of compound = MCl4
19. (d) Q 18 gm, H2O contains = 2 gm H
mass 100g
= = \ 0.72 gm H2O contains
density 1.834g cm –3
2
= 54.52 cm3 = 54.52 × 10–3 L = ´ 0.72 gm = 0.08 gm H
18
Moles of solute Q 44 gm CO2 contains = 12 gm C
Molarity =
Volme of solute in L \ 3.08 gm CO2 contains
0.969 12
= = 17.8 M = ´ 3.08 = 0.84 gm C
54.52 ´ 10 –3 44
16. (c) 50 ml of 16.9% solution of AgNO3 0.84 0.08
\C: H= :
12 1
æ 16.9 ö
ç 100 ´ 50 ÷ = 8.45 g of Ag NO3 = 0.07 : 0.08 = 7 : 8
è ø \ Empirical formula = C7H8
8.45g æ 8.45 g ö 20. (a) Na 2CO3 + 2HCl ¾¾ ® 2NaCl + H 2 O + CO 2
nmole = =ç ÷ 1mol 1mol
(107.8 + 14 + 16 ´ 3) g / mol è 169.8g / mol ø
= 0.0497 moles NaHCO3 + HCl ¾¾
® NaCl + H 2O + CO 2
1mol 1mol
50 ml of 5.8% solution of NaCl contain
Na2CO3 + NaHCO3 + NaCl + HCl ¾¾ ®
æ 5.8 ö
NaCl = ç ´ 50 ÷ = 2.9 g 848g 84g 68g (excess)
è 100 ø 8mol 1mol
E55555555555555555F
2.9g 1kg washingsoda
nNaCl = = 0.0495 moles
(23 + 35.5) g/ mol ¾¾
® 8CO2 + CO 2
AgNO3 + NaCl ® AgCl¯ + Na + Cl – fromNa 2CO3 from NaHCO 3
1 mole 1 mole 1 mole Thus, on complete reaction with HCl, 1kg of washing
\ 0.049 mole 0.049 mole 0.049 mole of AgCl soda will evolve 9 mol of CO2.
21. (a) 2.6 has two significant figures.
w
n= ®w = (n AgCl) × Molecular Mass 0.260 has three significant figures.
M
0.002600 has four significant figures.
= (0.049) × (107.8 + 35.5)
2.6000 has five significant figures.
= 7.02 g
22. (b) Given
- mass of solute (w) = 120 g
17. (b) Number of valence electrons in a N 3 ion = 1
-
mass of solvent (w) = 1000 g
Now, 1 mol or 42 g of N 3 has = 6.023 × 1023 ions Mol. mass of solute = 60 g
- density of solution = 1.12 g/ ml
So, 42 g of N 3 has 6.023 × 4 × 1023 valence e–
From the given data,
23 Mass of solution = 1000 + 120 = 1120 g
1 g of N 3 has 6.023 ´ 1 ´10 valence e–
-
Mol. mass Mol. mass
42 Q d= or V =
23 V d
4.2 g of N 3 has 4.2 ´ 6.023 ´ 1´ 10 valence e– i.e.,
-
1120
42 Volume of solution V = = 1000 ml or = 1 litre
1.12
0.1 NA valence e–.
W 120
18. (c) 74.75% of chlorine means 74.75g chlorine is present Now molarity (M) = = = 2M
in 100g of metal chloride. Mol. mass ´ V ( lit ) 60 ´1
Weight of metal = 100g – 74.75g 23. (d) In an unknown compounds containing N and H
= 25.25g
Chemistry S-C-3

Element Percentage Atomic ratio Simple ratio


[O] required for 1 mol. of Fe(C2O4) is 1.5, 5 [O] are
obtained from 2 moles of KMnO4
12.5 12.5 \ 1.5 [O] will be obtained from
H 12.5% = 12.5 =2
1 6.25 2
87.5 6.25 = ´ 1.5 = 0.6 moles of KMnO4.
N 87.5 = 6.25 =1 5
14 6.25 28. (c) C2H4 + 3 O2 ¾ ¾® 2CO2 + 2H2O
2 × vapour density = Mol. wt = mol wt. = 16 × 2 = 32. 28 g 96 g
Molecular formula = n × empirical formula mass Q 28 g of C2H4 undergo complete combustion by
32
= 96 g of O2
n= =2 \ 2.8 kg of C2H4 undergo complete combustion by
16 = 9.6 kg of O2.
\ Molecular formula of the compound will be = (NH2)2 29. (a) C3H 8 + 5O 2 ® 3CO 2 + 4H 2O
= N2H4
a 3a
24. (a) BaCl 2 + H 2SO 4 ¾¾ ® BaSO 4 + 2HCl
13
208 g 98 g 233 g 73 g C 4 H10 + O 2 ® 4CO 2 + 5H 2 O
2
20.8 g 9.8 g 23.3 g 7.3 g (3 – a) 4 (3 – a)
25. (c) 5CO + I 2 O 5 ® 5CO 2 + I 2 But, 3a + 4 (3 – a) = 10
25.4 \ a = 2 (Propane ) and 3 – 2 = 1 (Butane )
Moles of I 2 O 5 = 30. (c) Element % Relative no. Simplest ratio of
254
= 0.01 º 0.05 moles of CO of atoms atoms
Weight of CO = 0.05 × 28 = 1.4 g; C 49.3 49.3/12 = 4.1 4.1/2.74 = 1.5
Weight of CO 2 = 2 – 1.4 = 0.6 g 1.5 × 2 = 3
0.6 H 6.84 6.84/1 = 6.84 6.84/2.74 = 2.5
Hence % of CO 2 = × 100 = 30% = 2.5 × 2
2 =5
7.5
26. (b) PV = nRT \ 5.6 × 1 = ´ 0.0821´ 273 O 43.86 43.86/16 = 2.74 2.74/2.74 = 1
M. Wt. 1×2=2
M. Wt = 30.12 Hence gas is NO.
27. (b) The required equation is \ Empirical formula = C3H5O2
Empirical formula mass
2KMnO 4 + 3H 2SO 4 ¾
¾® = (3 × 12) + (5 × 1) + (2 × 16) = 36 + 5 + 32 = 73
K 2 SO 4 + 2MnSO 4 + 3H 2 O + 5 [O ] Molecular mass = 2 × Vapour density
nascent oxygen = 2 × 73 = 146
molecular mass
2Fe(C2 O 4 ) + 3H 2SO 4 + 3 [ O ] ¾¾
® n= = 146/73 = 2
empirical formula mass
Fe 2 (SO 4 ) 3 + 2CO 2 + 3H 2 O Molecular formula = Empirical formula × 2
= (C3H5O2) × 2 = C6H10O4
EBD_7504
S-C-4 SOLUTIONS

Speed Test-31

1. (c) The species CO, NO+, CN– and C22– contain 14 electrons 109677.76
Þ n12 = = 9 Þ n1 = 3
each. 12186.3
2. (d) NaCl : No. of e– in Na+ = At. No. of Na–1 \ The line belongs to Paschen series.
= 11 – 1 = 10 h
No. e– in Cl– = At. No. of Cl + 1 7. (d) de Broglie wavelength, l =
mv
= 17 + 1 = 18
CsF : No. of e– in Cs+ = 55 – 1 = 54 l1 m 2 v2 1 1 v2
= ; = ´
No. of e– in F– = 9 + 1 = 10 l 2 m1v1 4 9 v1
NaI : No. of e– in Na+ = 11 – 1 = 10 v2 9
No. of e– in I– = 53 + 1 = 54 =
v1 4
K2S : No. of e– in K+ = 19 – 1 = 18
No. of e– in S2– = 16 + 2 = 18 v1 4
=
3. (c) For electron in the ground state, v2 9
h h 1
mvr = Þ mv = KE = mv 2
2p 2pr 2
h KE1 m1 v12 9
2
Now, mv = æ 4ö 16
l = ´ = ´ç ÷ =
KE 2 m 2 v22 1 è 9ø 9
h h
So, = Þ l = 2pr
l 2pr 8. (c) Fe (III) = [Ar] 3d 5 unpaired electrons = 5;
l = 2 ´ 3.14 ´ 0.53Å = 3.328Å Magnetic moment = 5(5 + 2) ;
= 3.328 ´ 10 -10 m
Ratio = 7: 3
= 0.3328 ´10 -9 m = 0.3328 nm
4. (a) For He+, Co(II) = [Ar] 3d 7 unpaired electrons = 3;
1 æ 1 1 ö Magnetic moment = 3(3 + 2)
v= = RH Z 2 ç 2 - 2 ÷
l è2 4 ø Ratio = 7: 3
æ 1 1ö æ 1 1 ö
= RH (2)2 ç 2 - 2 ÷ = RH ç 2 - 2 ÷ E = hu =
ch
;and u =
c
è2 4 ø è (1) (2) ø 9. (b)
l l
For H,
3.0 ´108
æ ö 8 ´ 1015 =
1 1 1 l
v= = RH ç - ÷
l ç n12 n2 ÷ 3.0 ´ 108
è 2ø \ l= = 0.37 ´ 10 -7 = 37.5 ´ 10 -9 m = 4 ´ 101
For same frequency, 8 ´ 1015
h h
æ 1 æ ö 10. (d) lp = ; l Li =
1 ö 1 1 2eVm p 2 ´ 3eVmLi
RH ç - ÷ = R ç - ÷
è (1) 2 (2 )2 ø H
çè n12 n22 ÷ø
h
=
\
1 1 1 1 2 ´ 3eV ´ 9m p
- = -
n12 n22 12 2 2
l Li3+ 2eVm p 1
\ n1 = 1 & n2 = 2 Hence, = =
5. (d) For Balmer n 1 = 2 and n 2 = 3; lp 2 ´ 3eV ´ 9m p 3 3
æ 1 1 ö 5R 11. (b) From the given data, we have
n = R çç - ÷÷ = cm -1 (EC – EB) + (EB – EA) = (EC – EA)
2
è2 32 ø 36
6. (c) Series limit is the last line of the series, i.e. n2 = ¥. æ hc hc ö hc é 1 1 1 ù
or ç + ÷= ê or + = ú
1 é 1 1 ù é 1 1 ù R è l1 l 2 ø l3 ë l1 l 2 l3 û
\u = = R ê 2 - 2ú= Rê 2 - 2ú= 2
l êë n1 n 2 úû êë n1 ¥ ûú n1 l1l 2 é l1 + l 2 1 ù
or l3 = êQ = ú
1 ë l .l l 3û
Q u = 12186.3 = 1 2
Chemistry S-C-5

h æ 1 1 ö hc
12. (a) We know Dp.Dx ³ 16. (a) DE = 2.178 ´ 10-18 ç 2 - 2 ÷ =
4p è1 2 ø l
since Dp = Dx (given)
3 hc
\ Dp.Dp =
h Þ 2.178 ´ 10-18 ´ =
4p 4 l

or mDv mDv. =
h
[\ Dp= mDv] 6.62 ´ 10-34 ´ 3 ´ 108
=
4p l
2 h
or ( Dv ) = 6.62 ´ 10-34 3 ´ 108 ´ 4
4 pm 2 l=
h 1 h 2.178 ´ 10-18 ´3
or Dv = = = 1.214 × 10–7m
4pm 2 2 m p
13. (a) (n + l) rule the higher the value of (n + l), the higher is 17. (c) Not more than two electrons can be present in same
the energy. When (n + l) value is the same see atomic orbital. This is Pauli's exclusion principle.
value of n. 18. (a) 2nd excited state will be the 3rd energy level.
I II III IV 13.6 13.6
(n + l) (4 + 1) (4 + 0) (3 + 2) (3 + 1) En = eV or E = eV = 1.51 eV.
2 9
n
5 4 5 4
19. (b) Calculating number of electrons
\ IV < II < III < I 3-
® 5 + 8 ´ 3 + 3 = 32 ü
BO 3 ¾¾
Z2 ï
14. (b) I. E = ´ 13.6eV ...(i) 1. 2-
CO3 ¾¾® 6 + 8 ´ 3 + 2 = 32ý iso-electronic species
n2
I1 Z12 n22
-
NO3 ¾¾® 7 + 8 ´ 3 + 1 = 32 ï
or I = 2 ´ 2 ...(ii)
þ
2 n1 Z 2
Given I1 = – 19.6 × 10–18 , Z1 = 2,
2-
® 16 + 8 ´ 3 + 2 = 42ü
SO3 ¾¾
ï
n1 = 1 , Z2 = 3 and n2 = 1 2- ï
2. CO3 ¾¾
® 32 ý not iso-electronic species
Substituting these values in equation (ii). ï
-
NO3 ¾¾
® 32 ïþ
19.6 ´10 -18 4 1
– = ´
I2 1 9
9 CN
-
® 6 + 7 + 1 = 14ü
¾¾
or I 2 = -19.6 ´ 10 -18 ´ ï
4 ï
3. N2 ¾¾
® 7 ´ 2 = 14 ý iso-electronic species
= – 4.41 × 10–17 J/atom ï
-
15. (c) As per Bohr’s postulate, C2 ¾¾
® 6 ´ 2 + 2 = 14 ï
þ
nh
mvr =
2p ® 15 + 8 ´ 4 + 3 = 50 ü
PO34- ¾¾
ï
nh 2- ï
So, v = 4. SO 4 ¾¾
® 16 + 8 + 2 = 50 ý iso-electronic species
2pmr
- ï
1 2 ClO4 ¾¾
® 17 + 8 ´ 4 + 1 = 50ï
KE = mv þ
2
Hence the species in option (b) are not
1 æ nh ö2 isoelectronic.
So, KE = m ç ÷
2 è 2pmr ø 20. (d) (DE), The energy required to excite an electron in an
atom of hydrogen from n = 1 to n = 2 is DE (difference in
ao ´ n2
Since, r = energy E2 and E1)
z
Values of E2 and E1 are,
So, for 2nd Bohr orbit
-1.312 ´106 ´ (1) 2
a ´ 22 E2 = = –3.28 × 105 J mol–1
r= o = 4ao (2)2
1
DE is given by the relation,
1 æ 22 h 2 ö
KE = m çç 2 2 ÷ E1 = – 1.312 × 106 J mol–1
2 è 4p m ´ (4ao ) ÷ø
2
\ DE = E2 – E1 = [–3.28 × 105]– [–1.312 × 106 ] J mol–1
h2 = (–3.28 × 105 + 1.312 × 106) J mol–1
KE =
32p 2 mao2 –1
EBD_7504
S-C-6 SOLUTIONS

æ 1 Substituting this in equation (i)


1 1ö
21. (a) n= = RH Z = ç 2 – 2 ÷ h m
l è n1 n2 ø l=
In Balmer series n1 = 2 & n2 = 3, 4, 5.... Last line of m 2 KE
the spectrum is called series limit. 1
Limiting line is the line of shortest wavelength and l=h ...(i)
2m( K .E.)
high energy when n2 = ¥
1 R 3.29 ´ 1015 3.29 ´ 1015 1
\ n = = H2 = = i.e. l µ
l n1 22 4 KE
\ when KE become 4 times wavelength become 1/2.
= 8.22 ´ 1014 s –1 27. (a) The electronic configuration of Rubidium (Rb = 37) is
22. (d) Energy required to break one mole of Cl – Cl bonds in
Cl2 1s 2 2s 2 2 p 6 3s 2 3 p 6 3d 10 4s 2 4 p 6 5s1
Since last electron enters in 5s orbital
242 ´ 103 hc
= = 1
6.023 ´ 10 23 l Hence n = 5, l = 0, m = 0, s = ±
2
6.626 ´ 10-34 ´ 3 ´ 108 28. (c) The kinetic energy of the ejected electron is given by
=
l the equation
-34
6.626 ´ 10 ´ 3 ´ 108 ´ 6.023 ´ 10 23 1
hn = h no + mv 2 Q n =
c
\l= 2 l
242 ´ 108
= 0.4947 × 10–6 m = 494.7 nm hc hc 1 2
or = + mv
h l lo 2
23. (b) l= 1 2 hc hc
mv mv = -
h = 6.6 × 10–34 J/s 2 l lo
m = 1000 kg æl -lö
= hc ç o ÷
36 ´ 103 è llo ø
v = 36 km/hr = m/sec = 10 m/sec
60 ´ 60 2hc æ lo - l ö
6.6 ´ 10-34 \ v2 = ç ÷
\ l= = 6.6 ´ 10-38 m m è llo ø
3
10 ´ 10
24. (c) rn = a0n2 2hc æ l o - l ö
or v= ç ÷
r = a0 × (3)2 = 9a0 m è ll o ø
nh nh = 3h h 29. (d) Total energy of a revolving electron is the sum of its
mvr = ; mv = = kinetic and potential energy.
2p 2 pr 2 p ´ 9 a0 6 p a0
Total energy = K.E. + P. E.
h n
l= = ´ 6p a0 = 6pa0 e 2 æ e2 ö
mv h = + -
2r çè r ÷ø
r 2y 2
25. (c) l = 2 represent d orbital for which e2
=-
a0 2r
30. (b) Energy of 1 mole of photons,
26. (b) de – Broglie wavelength is given by :
h E = N0 × h u
l= ... (i) N0 ´ h ´ c
mv
=
1 2
l
K.E. = mv
2 6.023 ´ 10 23 ´ 6.63 ´ 10 -34 ´ 3 ´ 10 8
=
v2 =
2 KE 253.7 ´ 10 -9
m = 472.2 kJ
2 KE Energy converted into KE = (472.2 – 430.53) kJ
v= (472.2 - 430.53)
m % of energy converted into KE =
472.2
= 8.76 %
Speed Test-32

1. (d) All the given species contains 10 e – each i.e. 9. (b) Along the period, I.P. generally increases but not regularly.
isoelectronic. Be and B are exceptions. First I.P. increases in moving
For isoelectronic species anion having high negative from left to right in a period, but I.P. of B is lower than Be.
charge is largest in size and the cation having high 10. (d)
positive charge is smallest. 11. (d) The electronic configuration of the given ions are as
2. (c) Covalent radius is radius of an atom in its bound state follows.
i.e., in fluorine it is half of distance between two 2+ 2 2 6
12Mg = 1s , 2s 2p (No unpaired electron)
covalently bonded fluorine atoms; van der Waal radii 3+ 2 2 6 2 6 1
is one-half of the distance between the nuclei of two 22Ti = 1s , 2s 2p , 3s 3p 3d (One unpaired electron)
3+ 2 2 6, 2 6 2
identical non-bonded isolated atoms. These atoms are 23V = 1s , 2s 2p 3s 3p 3d
attracted toward each other through weak van der (Two unpaired electrons)
Waal’s force hence van der Waal radii are very large. 2+ 2 2 6 2 6 6
26Fe = 1s , 2s 2p ,3s 3p 3d
3. (c) All the given species are isoelectronic. In case of
(Four unpaired electrons)
isoelectronic species ionic radii increases with increase
in negative charge on anions. 12. (b) IE2 of Mg is lower than that of Na because in case of
4. (b) On moving along the period, ionization enthalpy Mg+, 3s-electron has to be removed whereas in case
increases. of Na+, an electron is removed from the stable inert
In second period, the order of ionization enthalpy should gas configuration which is difficult.
be as follows : 13. (b) According to modern periodic law, the properties of the
F > O > N. elements are repeated after certain regular intervals when
But N has half-filled structure, therefore, it is more stable these elements are arranged in order of their increasing
than O. That is why its ionization enthalpy is higher atomic numbers.
than O. Thus, the correct order of IE is 14. (a) For isoelectronic species, size of anion increases as
F > N > O. negative charge increases whereas size of cation
5. (c) In a period the value of ionisation potential increases decreases with increase in positive charge. Further ionic
from left to right with breaks where the atoms have radii of anions is more than that of cations. Thus the
some what stable configuration. In this case N has half correct order is Ca++ < K+ < Ar < Cl– < S– –
filled stable orbitals. Hence has highest ionisation 15. (b) The alkali metals are highly reactive because their first
energy. Thus the correct order is ionisation potential is very low and hence they have great
B< C< O < N tendency to loses electron to form unipositive ion.
and not as given in option (c) On moving down group- I from Li to Cs ionisation
6. (a) Atomic number (Z) = 120 enthalpy decreases hence the reactivity increases. The
IUPAC name = Unbinilium halogens are most reactive elements due to their low
Symbol = Ubn bond dissociation energy, high electron affinity and high
7. (b) The acidic character of non metal oxides increases across enthalpy of hydration of halide ion. However their
a period from left to right and decreases down a group. reactivity decreases with increase in atomic number
So, acidic character will follow the order: 16. (b) In hydrides of 15th group elements, basic character
oxide of nitrogen > oxides of sulfur > oxides of carbon. decreases on descending the group i.e.
Among oxides of carbon acidic character increases with NH3 > PH3 > AsH3 > SbH3.
17. (b) The right sequence of I.E1 of Li < B < Be < C.
the oxidation number of carbon. So, +4CO 2 is more acidic 18. (b) As we move down in a group electron gain enthalpy
than CO. Hence the sequence of acidic character is N2O5 becomes less negative because the size of the atom
> SO2 > CO2 > CO increases and the distance of added electron from the
8. (c) The ionization energy increases with decrease in size. nucleus increases. Negative electron gain enthalpy of
Further the element having stable configuration has F is less than Cl. This is due to the fact that when an
higher ionisation energy than expected. Hence the electron is added to F, the added electron goes to the
ionization energy of nitrogen (Z = 7) is more than oxygen smaller n = 2 energy level and experiences significant
(Z = 8) and carbon (Z = 6) because it has half-filled repulsion from the other electrons present in this level.
p-orbitals. In Cl, the electron goes to the larger n = 3 energy level
6 and consequently occupies a larger region of space
C = 1s 2 2s 2 2 p 2 ; 7
Hence the correct Cl > F > Br > I.
EBD_7504
S-C-8 SOLUTIONS

19. (a) On moving along a period ionic radii decreases due to H2O + CO2 –––––® H2CO3
increase in effective nuclear charge. Silica (SiO2) is insoluble in water and acts as a very
20. (a) As the nuclear charge increases, the force of attraction weak acid.
between the nucleus and the incoming electron SnO2 is amphoteric as it reacts with both acid and base.
increases and hence the elecron gain enthalpy becomes SnO2 + 2H2SO4–––––® Sn(SO4)2 + 2H2O
more negative, hence the correct order is SnO2 + 2KOH–––––® K2SnO3 + H2O
Ca < Al < C < O < F 26. (a) Incoming electrons occupies the smaller n = 2 shell,
21. (a) First period has H and He only out of which He is inert, also negative charge on oxygen (O–) is another factor
hence, H behaves as a highly electropositive as well as due to which incoming electron feel repulsion.
electronegative. Hence electron repulsion outweigh the stability gained
22. (a) O– – and F– are isoelectronic. Hence have same number by achieving noble gas configuration.
of shells, therefore greater the nuclear charge smaller 27. (b) In the isoelectronic species, all isoelectronic anions
will be the size i.e. belong to the same period and cations to the next
O– – > F– period.
+ 3+
further Li and B are isoelectronic. therefore 28. (c) I represents Li, II represents K
Li+ > B3+ III represents Br, IV represents I
Hence the correct order of atomic size is. V represents He
O-- > F– > Li+ > B3+ So, amongst these, II represents most reactive metal
23. (d) As the size increases the basic nature of oxides changes and V represents least reactive non-metal.
to acidic nature i.e., acidic nature increases. 29. (d)
SO 2 > P 2 O 3 > SiO 2 > Al 2 O 3 30. (d) Helium (He) 1s2 ® Highest ionisation
Acidic Weak Amphoteric energy due to noble gas
acidic
in nature.
SO2 and P2O3 are acidic as their corresponding acids
Fluorine (F) 1s2, 2s22p3 ® High electronegativity in
H2SO3 and H3PO3 are strong acids.
nature due to small size
24. (a) For isoelectronic species ionic radii decreases as the
and –1 oxidation state.
charge on ion decreases. Further on moving down in a
Rubidium (Rb) ® Most electronegative
group ionic radii increases. Hence the correct order is
element due to large
O2– < N3– < S2– < P3– atomic size.
25. (a) CaO is basic as it form strong base Ca(OH)2 on reaction Lithium (Li) ® Strongest reducing
with water. agent due to small size
CaO + H2O –––––® Ca(OH)2 and positive oxidation
CO2 is acidic as it dissolve in water forming unstable state (+1)
carbonic acid.
Speed Test-33

2– result is reduction of dipole moment. It has been shown


O
in the following figure :
1. (c) CO 23 – Its structure is C–O
O .. ..
N N
2. (a) Greater the difference in electronegativity between the
two atoms, larger will be polarity and hence dipole F
moment. Thus (a) has maximum dipole moment. H H F
H F
H 6. (c) Since F form H-bond [HF2]– exists. Therefore KHF2
H3C H
H–C=O C=C gives K+ + HF2–
H3C H
(a) (b) 7. (b) On changing N2 to N+2, B.O. decreases from 3 to 2.5
(very less polar) whereas on changing O2 to O+2, B.O. increases from 2
(C–O bond is
to 2.5. In former case, the bond dissociation energy
more polar)
decreases and in the latter case, it increases.
H CH3 Cl CH3
8. (a) Molecular orbital configuration of
C C
C C N 2– 2 2 2 2
2 = s1s s *1s s 2 s s * 2 s –
H3C H H3C Cl
(c) (d) ìï p 2 px2 ìï p * 2 p1x
í s2 p2z í
2 1
Symmetrical molecules (µ = 0) ïîp 2 p y ïîp * 2 p y

3. (c) NH3 undergoes H-bonding and hence has the highest 10 – 6


Bond order = =2
b.p. Among the remaining hydrides i.e. PH3, AsH3 and 2
SbH3 as we move from PH3 to BiH3, the molecular ì p2 px2 ìï p * 2 p1x
mass increases. As a result the van der waal’s forces of N 2– = s1s 2 s *1s 2 s 2s 2s * 2s 2 ïí s2 pz2 í
2 0
attraction increases and the boiling point increases îï p 2 p y ïî p *2 p y
regularly from PH3 to BiH3.
10 - 5
Bond order = = 2.5
Cl 2
4. (b) BiCl3: Cl Bi ; sp2 - Hybridisation
Cl ìï p2 px2
(Trigonal planar geometry); Bond angle = 120º N 2 = s1s 2 s *1s 2 s 2 s 2s * 2 s 2 í , s 2 pz2
2
ïîp 2 p y
sp3 10 – 4
P Bond order = =3
In PCl3 (Pyramidal 2
Cl Cl Cl geometry) Bond angle = \ The correct order is = N 2– –
o
below 109 28’ 2 < N2 < N2
sp3
As and decreases from
In AsCl3 (Pyramidal 1
PCl3 to BiCl3 9. (a) Hybridisation = [No. of valence electrons of central
Cl Cl Cl geometry) 2
sp3 atom + No. of monovalent atoms attached to it +
In BiCl3 Bi (Pyramidal
Cl Cl Cl geometry) Negative charge if any – Positive charge if any]
1
In these, order of bond angle : NO2– H = [5 + 0 + 1 - 0] = 3 = sp 2
2
BCl3 > PCl3 > AsCl3 > BiCl3
1
5. (a) In NH3 the atomic dipole (orbital dipole due to lone NO3– H = [5 + 0 + 1 - 0] = 3 = sp 2
2
pair) and bond dipole are in the same direction whereas
1
in NF3 + 0] = 4 = sp3
EBD_7504
S-C-10 SOLUTIONS

1 3 16. (a) According to VSEPR theory order of repulsion in


NH4+, H = [5 + 4 + 0 - 1] = 4 = sp between lp – lp, lp – bp and bp – bp is as under
2
SCN– = sp lp – lp > lp – bp > bp – bp
i.e., NO2– and NO3– have same hybridisation.
17. (a) In NH3 and BF4- the hybridisation is sp3 and the bond
10. (b) Electronegativity difference is 4.0 - 1.20 = 2.8 percentage
angle is almost 109º 28'.
ionic character is 72.24% when the electronegativity
difference is 1.7, the % ionic character is approx 51%. 18. (d) In XeF2 Total number of valence electrons of Xe = 8,
two electrons shared with 2F atoms, 6 electrons left
11. (c) In PO34- ion, formal charge on each O-atom of P – O hence 3 lone pairs, in XeF4 4 shared with 4 F atoms 4
Total charge 3 left hence 2 lone pairs; in XeF6 6 shared with 6 F atoms
bond = = - = - 0.75 2 left hence 1 lone pair.
Number of O atom 4
12. (c) The electronic configuration of the given molecules éæ No. of electrons ö
are : 1 êç ÷
19. (a) Hybridisation = êç in valence
2 ç ÷+
N +2 = s1s 2 , s *1s 2 , s 2 s 2 , s * 2s 2 , p 2 px2 = p p 2y ,s 2 p1z ; êè shell of atom ÷ø
ë
1 unpaired e–
æ No.of monovalent ö æ charge on ö
O2 = s1s 2 , s *1s 2 , s2s2 , s * 2s 2 , s 2 p2z , p 2 p2x » p 2 p2y , ç ÷ – ç ÷+
è atoms around it ø è cation ø
p * 2 p1x » p * 2 p1y ; 2 unpaired e–s
æ charge on ö
ç ÷
O22- 2 2 2
= s1s ,s *1s ,s 2s ,s * 2s 2
, s 2 pz2 , p 2 p2x » p 2 p2y , è anion ø
(a) For AlH3,
p*2 px2 » p*2 p 2y ; no unpaired e–s
Hybridisation of Al atom = 1 [ 3 + 3 - 0 + 0]
B2 = s1s 2 , s *1s 2 , s2s 2 , s * 2 s 2 , p 2 p1x » p 2 p1y ; 2
= 3 = sp 2
2 unpaired e–s
13. (c) ClO3– and SO3–2 both have same number of electrons For AlH4–,
(42) and central atom in each being sp3 hybridised. Both
are having one lone pair on central atom hence they are Hybridisation of Al atom = 1 [ 3 + 4 - 0 + 1]
pyramidal. 2
14. (a) The number of lone pairs of electrons on central atom = 4 = sp3
in various given species are (b) For H2O,
Species Number of lone pairs on Hybridisation of O atom
central atom 1
IF7 0
=
2
[6 + 2 - 0 + 0] = 4 = sp3
IF5 1
For H3O+, Hybridisation of O atom
ClF3 2
1
XeF2 3 =
2
[ 6 + 3 - 1 + 0] = 4 = sp3
Thus the correct increasing order is
IF7 < IF5 < ClF3 < XeF2 (c) For NH3
0 1 2 3 Hybridisation of N atom
15. (b) CH4 NH3 H2O
1
H
=
2
[ 5 + 3 - 0 + 0] = 4 = sp3
C 109°28' N O For NH +4 , Hybridisation of N atom
H H H 107°H H
H 104.5° H
H
1
Tetrahedral; Trigonal Bent =
2
[ 5 + 4 - 1 + 0] = 4 = sp3
pyramidal
Note: The geometry of H2 O should have been Thus hybridisation changes only in option (a).
tetrahedral if there are all bond pairs. But due to presence

Hence bond angle


Chemistry S-C-11

20. (b) H 3 Consequently such compounds are soluble in non-


O sp polar solvents like ether, benzene etc. and are only
sparingly soluble in water whereas meta and para
isomers are more soluble in water & less soluble in
non-polar solvents.
2
B sp
H
O O
Å
N
O
H
sp3 O sp3 O intra-molecular H-bonding
H
27. (b) According to Fajan’s rules smaller, highly charged
21. (b) Hybridisation in PO 34– = ½ [5 + 0 + 3 –0] = 4 sp3. In cation has greatest covalent character while large cation
π bonding only d orbital of P, p orbital of O can be with smaller charge has greatest ionic character.
involved. Since hybrid atomic orbitals do not 28. None of the given option is correct.
form π bond. The molecular orbital configuration of the given
22. (b) The bond angle decreases on moving down the group molecules is
due to decrease in bond pair-bond pair repulsion. H2 = s1s2 (no electron anti-bonding)
NH3 PH3 ASH3 SbH3 BiH3 Li2 = s1s2 s* 1s2 s2s2(two anti-bonding electrons)
107º 94º 92º 91º 90º
This can also be explained by the fact that as the size
1
{ 1
B2 = s1s2 s* 1s2 s2s2 s* 2s2 p2p y = p2pz }
of central atom increases sp3 hybrid orbital becomes (4 anti-bonding electrons)
more distinct with increasing size of central atom i.e. Though the bond order of all the species are same (B.O
pure p- orbitals are utilized in M–H bonding = 1) but stability is different. This is due to difference
23. (a) For any species to have same bond order we can expect in the presence of no. of anti-bonding electron.
them to have same number of electrons. Calculating Higher the no. of anti-bonding electron lower is the
the number of electrons in various species. stability hence the correct order is H2 > Li2 > B2
29. (a) The structure of IF6– is distorted octahedral
O2– (8 + 8 + 1 = 17) ; CN - (6 + 7 + 1 = 14)
This is due to presence of a “weak” lone pair.
NO+(7 + 8 – 1= 14); CN+ (6 + 7 –1 = 12)
We find CN– and NO+ both have 14 electrons so they F
have same bond order. Correct answer is (a). F F
+
24. (d) s 2b s *a2 s 2b s*2 2 2 1
a ( p b = p b ) s b (N 2 = 13electrons)
I
it contains one unpaired electron hence paramagnetic 164°
25. (b) The delocalised pp - pp bonding between filled p- F F
orbital of F and vacant p-orbital of B leads to shortening
of B–F bond length which results in higher bond F
dissociation energy of the B–F bond. 30. (b) The molecular orbital configuration of the molecules
given is
F Total no. of electrons in NO = 7(N) + 8(O) = 15
B F Hence E.C. of NO = KK[s(2s)]2 [s *(2s)]2[s2 pz ]2
F [p (2 p x )]2 [p(2 p y )]2[ p *(2 p x )]1
Due to presence of one unpaired electron NO is
Vacant Filled paramagnetic.
2p-orbital 2p -orbital Except NO all are diamagnetic due to absence of
F F unpaired electrons.
+
B=F B–F
+
F F
+ +1/3
F F
+1/3
B–F B F
+1/3
F
26. (b)
EBD_7504
S-C-12 SOLUTIONS

Speed Test-34

1. (c) V1 = V2 at const. pressure 1


T1 T2 8. (b) Rate of diffusion µ
Molecular mass
22.4 V2
Þ = , V2 = 30.6 litre Q Molecular mass of HCl > Molecular mass of NH3
273 373
\ HCl diffuses at slower rate and white ammonium
2. (a) From the graph we can see the correct order of pressures chloride is first formed near HCl bottle.
p1 > p3 > p2
3. (b) Gases become cooler during Joule Thomson’s expansion only U CO 2 TCO 2 ´ M N 2 O
if they are below a certain temperature known as inversion 9. (a) =
U N 2O M CO 2 ´ TN 2 O
temperature (Ti). The inversion temperature is characteristic of
each gas and is given by x TCO2 44
Þ = ´ Þ TN 2O = 16TCO 2
2a 4x 44 TN 2O
Ti = , where R is gas constant
bR 10. (b) 2L 3L
Given a = 0.244 atm L2 mol–2
b = 0.027 L mol–1
R = 0.0821 L atm deg–1 mol–1
2 ´ 0.244
\ Ti = = 220 K nA n'A
0.027 ´ 0.0821 =
2 3
nB =
3 n'B 5
4. (d) u a T or u1 / u2 = T1 / T2
(n A + n¢A ) M A + (nB + nB¢ ) M B
27 + 273 300 1 Mean molar mass =
= = = nA + n¢A + nB + n¢B
927 + 273 1200 2
77 M A + 123M B
u2 = 2u1 =
200
5. (d) Let the mass of methane and oxygen = m gm. 12 12
DA rB é r B ù ær ö
Mole fraction of O2 11. (d) = =ê ú ; \ DA = DB ç B ÷
DB rA ër A û è rA ø
Moles of O2
= 12. (c)
Moles of O 2 + Moles of CH 4
m / 32 m / 32 1 24.6
= = =
m / 32 + m /16 3m / 32 3
Partial pressure of O2 = Total pressure × mole fraction of
1 1 PV 21.6
O2, PO2 = P × = P
3 3 20.1
6. (c) The expression of root mean square speed is
3RT
urms = 3.0
M 2.0 1/V
Hence, 0
1/ 2
u rms (H 2 ) é 3R(50K) /(2g mol -1) ù æ a ö
=ê ú =1 ç P + 2 ÷ (V ) = RT
u rms (O2 ) êë 3R(800K) /(32g mol -1) úû è V ø
PV + a/V = RT
7. (a) Given T = 27°C = 27 + 273 = 300 K a
V = 10.0 L PV = RT -
Mass of He = 0.4 g V
Mass of oxygen = 1.6 g y = RT – a(x)
Mass of nitrogen = 1.4 g 21.6 - 20.1
So, slope = – a = = – a = – 1.5
n He = 0.4/4 = 0.1 2-3
n O2 = 1.6/32 = 0.05 a = 1.5
n N2 = 1.4/28 = 0.05 3
K ´ 313
n total = n He + n O2 + n N2 = 0.1 + 0.05 + 0.05 = 0.2 K .E of neon at 40 °C 313
13. (a) = 2 =
n RT 0.2 ´ 3 293
P= = K ´ 293
V 2
Chemistry S-C-13

14. (c) Higher the critical temperature more easily will be the 1 ´ 0.7
gas liquify. Now since most easily liquifiable gas show = = 0.031
0.0821 ´ 273
larger deviation, NH3 will show maximum deviation from
ideal behaviour. Increase in volume is by = 0.031 – 0.022
15. (c) According to Avogadro’s law "At same temperature = 0.009 mole of gas
and pressure Final no. of moles of CO i.e. n(CO final)
Volume µ no. of moles" n(CO = 2n(CO – n(CO
w w w final) 2 initial) 2 final)
n H2 = ; n O2 = ; n CH4 = = 2(0.022 – n(CO ...(i)
2 32 16 2 final)
Q VH : VO : VCH = n H 2 : n O 2 : n CH 4 n(CO final) = 0.044 – 2n(CO ...(ii)
2 2 4 2 final)
w w w \ Now, n(CO final) + n(CO final) = 0.031
=: : = 16 : 1 : 2
2 32 16 n(CO final) = 0.031 – n(CO final)...(ii)
16. (a) Given, n H 2 = n O 2 and t H 2 = t O2 2
Substituting (ii) in eq. (i)
According to Graham's law of diffusion for two different
n(CO final) = 0.044 – 2[0.031 – n(CO final)]
gases.
n(CO final) = 0.044 – 0.062 + 2n(CO final)
rH 2 v /t MO 2 32
= 1 1 Þ = n(CO final) = 0.018 mol.
rO2 v 2 / t 2 MH 2 2
nRT 0.018 ´ 0.0821 ´ 273
Volume of CO = V = =
1/ 2 P 1
= 16 = 4 = 0.40 Litre
1/ x
and volume of CO2 = 0.7 litre – 0.4 litre
x
=4 = 0.3 litre
2 \ CO2 = 300 mL, CO = 400 mL
\x=8
\ Fraction of O2 = 1/8 2RT
21. (c) Most probable speed (C*) =
17. (c) As temperature rises the most probable speed increases M
and the fraction of molecules possessing most probable 8RT
Average Speed (C) =
speed decreases. pM
18. (b) According to kinetic theory the gas molecules are in a 3RT
state of constant rapid motion in all possible directions Root mean square velocity (C) =
M
colloiding in a random manner with one another and
with the walls of the container and between two 2RT 8RT 3RT
C*: C : C = : :
successive collisions molecules travel in a straight line M pM M
path but show haphazard motion due to collisions. 4 3
= 1: : = 1:1.128 :1.225
19. (b) According to Boyle's law, PV = constant p 2
\ log P + log V = constant 22. (b) According to Graham’s Law Diffusion:
log P = – log V + constant r1 d2
Hence, the plot of log P vs log V is straight line with =
r2 d1
negative slope.
Vol. of gas diffused (V )
Since rate of diffusion =
Time taken for diffusion(t )
r1 V1 / t1
log P \ =
r2 V2 / t2
log V r1 V1 / t1 d2
20. (a) CO 2 + C ¾¾
® 2CO or = =
r2 V2 / t2 d1
Stoichoimetry ratio is 1 : 2
20 / 60 16 / 2 1
AT STP, P = 1 atm, T = 273 K, R = 0.0821 = = =
V2 / 30 32 / 2 2
PV
Initial moles of CO2; n(CO2initial) = Q Mol. wt = 2 × V.D
RT
Mol.wt
1 ´ 0.5 \ V.D =
= = 0.022 mole 2
0.0821 ´
EBD_7504
S-C-14 SOLUTIONS

3RT 27. (d) On applying Dalton's law,


23. (d) r.m.s. velocity Vr m s = Partial pressure of a component
M
= Mole fraction × Total pressure
V1 T1 Given, mass of N2 = 56 g, mass of O2 = 96 g
i.e., V = T2 Total pressure = 10 atm
2
56 96
5 ´ 104 1 T1 n
N2 = = 2, n
O2 = =3
= = 28 32
10 ´ 104 2 T2 n
N2 2
x
\ T2 = 4T1 N2 = = = 0.4,
n n
N 2 + O2 2+3
3RT n
O2 3
24. (d) Vrms = x
O2 = = = 0.6
M n
N 2 + n O2 2+3
Vrms(O ) = Vrms(He) \ PN2 = 0.4 × 10 = 4 atm, PO2 = 0.6 × 10 = 6 atm
2
3RTO2 3RTHe p A vA 8 ´ 12 96
= 28. (c) Moles of A, (n A) = = =
M O2 M He RT RT RT
TO 2 THe pB v B 8 ´ 5 40
or = Moles of B, (n B) = = =
M O2 M He RT RT RT
300 ´ 32 Total pressure × total volume = (nA + nB) × RT
\ TO 2 = = 2400K
4 1
PV p ´ (12 + 8) = (96 + 40)RT
25. (b) Compressibility factor ( Z ) = RT
RT p = 6.8
(For one mole of real gas)
Partial pressure of A = p × mole fraction of A
van der Waals equation
a æ 96 96 + 40 ö
(P + )(V - b ) = RT = 6.8 ç ÷
V2 è RT RT ø
At low pressure, volume is very large and hence = 4.8 atm
correction term b can be neglected in comparison to Partial pressure of B = 6.8 – 4.8 = 2 atm.
very large volume of V. 70.6g
i.e. V - b » V 29. (d) Number of moles of O2 = = 2.21 mol
32g mol -1
æ a ö
çè P + 2 ÷ø V = RT Number of moles of Ne =
167.5g
= 8.375 mol
V
20g mol-1
a
PV + = RT
V 2.21
Mole fraction of O2 = = 0.21
a 2.21 + 8.375
PV = RT -
V Mole fraction of Ne = 1 – 0.21 = 0.79
PV a Partial pressure of a gas = Mole fraction × total pressure
=1- Partial pressure of O2 = 0.21 × 25 = 5.25 bar
RT VRT
Partial pressure of Ne = 0.79 × 25 = 19.75 bar
a
Hence, Z = 1 - 2RT
VRT 30. (b) V0 = = R ( 273 + t )
26. (a) According to Boyle’s law M
3(2t + 273 + 273)R
V1 P2 750 360 U rms =
= ; = 1
V2 P1 V2 840
= 6( t + 273) R = 6 V0
V2 = 1750 ml = 1.750 L
Speed Test-35

1. (d) Given DH = 41 kJ mol–1 = 41000 J mol–1 2 × – 46 = 712 + 3 × (436) – 6x


T = 100°C = 273 + 100 = 373 K – 92 = 2020 – 6x
n=1 6x = 2020 + 92
DU = DH – DnRT = 41000 – (1 × 8.314 × 373) 6x = 2112
= 37898.88 J mol–1 ; 37.9 kJmol–1 x = + 352 kJ/mol
2. (a) Internal energy is dependent upon temperature and 7. (b) DHS-S + 2DH H -S = 239 2DH H -S = 175
according to first law of thermodynamics total energy of
Hence, DHS-S = 239 - 175 = 64 kcal mol -1
an isolated system remains same, i.e., in a system of
constant mass, energy can neither be created nor Then, DH for 8S(g ) ® S8(g ) is 8 × (–64) = –512k cal
destroyed by any physical or chemical change but can 8. (b) Enthalpy of reaction
be transformed from one form to another = B.E(Reactant)– B.E(Product)
DE = q + W
= éë B.E (C= C) + 4 B.E.(C–H) + B.E.(H -H) ùû
For closed insulated container, q = 0, so, DE = + W, as
work is done by the system - éë B.E.(C - C) + 6 B.E.(C - H) ùû
3. (b) Maximum work done by a system at constant pressure
and constant temperature is –DG. It can be derived as = [606.1 + (4 × 410.5) + 431.37)] – [336.49 + (6 × 410.5)]
follows : = –120.0 kJ mol–1
From first law of thermodynamics, 9. (a) Given, for reaction
q = DE + wexp. + wnon exp. ® H + (aq.) + OH – ( aq.);
(i)H2O (l) ¾¾
At constant pressure, wexp = PDV
DH r = 57.32 kJ
q = DE + PDV + wnon-exp.
q = DH + wnon exp. ....... (1) 1
(ii) H 2 ( g ) + O 2 ( g ) ¾¾ ® H 2 O(l );
For the reversible change at const temp., 2
q rev DH r = –286.20 kJ
DS = or qrev = TDS
T For reaction (i)
so from equ. (1), TDS = DH + wnon exp. DH r = DH°f (H + .aq)+DH°f (OH – .aq)–
Þ DH – TDS = –wnon exp. ...... (2)
For a change taking place under constant pressure and DH°f (H2 O, l)
constant temperature, 57.32 = 0 + DH°f (OH – , aq) – DH°f (H 2 O, l) …(iii)
DH – TDS = DG
For reaction (ii)
so from equ (2)
DG = –wnon exp. DH r = DH°f (H 2 O, l) –
(it means wnon exp. or wuseful) 1
or wmax = –DG DH°f (H 2 , g) – DH°f (O2 , g )
2
Latent heat of fusion DH –286.20 = DH°f (H 2 O, l)
4. (a) DS = =
Melting point T On replacing this value in eq. (iii) we have
2930 57.32 = D H°f (OH – , aq) – (–286.20)
= J K–1 mol–1 = 9.77 J K–1 mol–1
300
DH°f (OH - , aq) = –286.20 + 57.32 = –228.88 kJ
5. (b) Desired equation is H 2 O (l) ® H 2 O(g), DH = ? 10. (b) At equilibrium DG = 0
(Equation II - Equation I) Hence, DG = DH – TeDS = 0
DH = –245.5 kJ – (–286 kJ) = 40.5 kJ ΧH
® 2NH 3 DH = 2 ´ -46.0 kJ mol–1
\ DH = TeDS or Te <
6. (b) N 2 + 3H 2 ¾¾ ΧS
Let x be the bond enthalpy of N – H bond then For a spontaneous reaction
[Note : Enthalpy of formation or bond formation enthalpy DG must be negative which is possible only if DH – TDS < 0
is given which is negative but the given reaction involves \ DH < TDS
bond breaking hence values should be taken as positive.]
ΧH
DH = S Bond energies of reactants – S Bond energies
of products
EBD_7504
S-C-16 SOLUTIONS

11. (b) 2C8 H18(g) + 25O 2(g) ¾¾® 16CO 2(g) + 18H 2 O (g) 225 = [1410 + 330] – [700 + BEC º C]
The given reaction is a combustion reaction. Hence 225 = 1740 – 700 – BEC º C
this will be an exothermic reaction. BEC º C = 815 kJ mol–1
i.e. DH = –ve 17. (a) Given DH 35.5 kJ mol–1
Further Dn = + ve i.e. DS = +ve
DS = 83.6 JK–1 mol–1
DG = DH – TDS = –DH – TDS = –ve
Q DG = DH – TDS
12. (a) DG = DH - T DS, DH + ve, DS is + ve; TDS > DH for For a reaction to be spontaneous, DG = –ve
spontaneous process. It will make DG, –ve i.e., DH < TDS
13. (b) We have to calculate the enthalpy of the reaction DH 35.5 ´ 103 Jmol -1
\ T> =
OH (g) ® O(g) + H(g) DS 83.6 JK -1
From the given reactions, this can be obtained as follows. So, the given r eaction will be spontaneous at
é 1 1 ù T > 425 K
– ê H 2 (g) + O2 (g) ® OH(g) ú ; DH = – 42.09 kJ mol–1 18. (b) DG = DH – TDS
ë2 2 û
Since DG = DH – TDS for an endothermic reaction,
1 1 DH = +ve and at low temperature D S = + ve
+ [H (g) ® 2H(g)] ; DH = × 435.89 kJ mol–1
2 2 2 Hence DG = (+) DH – T ( + )DS
1 1 and if T D S < DH (at low temp)
+ [O2(g) ® 2O(g)]; DH = × 495.05 kJ mol–1 DG = +ve (non spontaneous)
2 2
Add But at high temperature, reaction becomes
spontaneous i.e. DG = –ve.
OH(g) ® H(g) + O(g) ; DH = 423.38 kJ mol-1 because at higher temperature TDS > DH.
19. (c) The standard enthalpy of the combustion of glucose
14. (c) CH 4 + 2O 2 ¾® CO2 + 2H2O can be calculated by the eqn.
x 2x
C6H12O6(s) + 6O2(g) ® 6CO2(g) + 6H2O(l)
C3 H8 + 5O2 ¾® 3CO2 + 4H2O DHC = 6 × DHf(CO2) + 6 × DHf (H2O) – DHf [C6H12O6]
(5- x ) 5(5- x )
DH° = 6 (–400) + 6(–300) – (–1300)
2x + 5(5– x) = 16
Þ x = 3L DH° = –2900 kJ/mol
\ Heat released For one gram of glucose, enthalpy of combustion

3 2 2900
= ´ 890 + ´ 2220 = 317 DH° = – =- 16.11kJ / gm
22.4 22.4 180
15. (a) C + O2 ® CO2 + 393.5 kJ/mol 20. (c) CH2 = CH2 (g) + H2 (g) ® CH3 - CH3
12g 44g Enthalpy change = Bond energy of reactants – Bond
44g is formed from 12g of carbon energy of products.
D H = 1(C = C) + 4 (C – H) + 1 (H - H) - 1 (C - C) - 6 (C - H)
12 ´ 35.2 = 1 (C = C) + 1 ( H – H) – 1 (C – C) – 2 (C– H)
35.2g is formed from g of C
44 = 615 + 435 – 347 – 2 × 414 = 1050 – 1175 = –125 kJ.
= 9.6 g of C = 9.6/12 = 0.8 mole 21. (c) For a reaction to be at equilibrium DG = 0. Since
1 mole release heat 393.5 kJ DG = DH – T DS so at equilibrium DH – TDS = 0
0.8 mole release heat = 393.5 × 0.8 or DH = T DS
= 314.8 kJ » 315 kJ
For the reaction
16. (d) (i) 2C(s) + H2(g) ¾¾
® H – C º C – H(g) 1 3
X 2 + Y2 ¾¾ ® XY3 ; DH = –30kJ (given)
DH = 225 kJ mol–1 2 2
Calculating DS for the above reaction, we get
(ii) 2C(s) ¾¾
® 2C(g) DH = 1410 kJ mol–1
é1 3 ù
1410 DS = 50 - ê ´ 60 + ´ 40ú JK -1
C( s) ¾¾
® C( g )DH = = 705kJmol -1 ë2 2 û
2
=50 – (30 + 60) JK–1 = – 40 JK–1
(iii) H2(g) ¾¾® 2H(g) DH = 330 kJ mol–1 At equilibrium, TDS = D H [Q DG = 0]
From equation (i) : \ T ´ (–40) = –30 ´ 1000 [Q 1kJ = 1000J]
225 = éë 2 ´ DHC(s) ¾¾
® C( g )
+ 1 ´ BE H – H ù
û T=
–30 ´1000
or or 750 K
–40
2ICl( g )
Chemistry S-C-17

DrH = [DH(I2(s) ®I2(g)) + DHI–I + DHCl–Cl] – [DHI – Cl] 26. (a) –Wirreversible = Pext (V2 – V1)
= 151.0 + 242.3 + 62.76 –2 × 211.3 = 33.46 = 10 atm (2L – 1L)
= 10 atm – L
33.46
Χf H°(ICl) < < 16.73 kJ / mol V2
2
1 1
–Wreversible = ò Pex dv
H2 + X2 ¾ V1
23. (a) ¾® HX
2 2
Let the bond enthalpy of X – X bond be x. V2
= 2.303 nRT log V
1
1 1
DH f (HX ) = – 50 = DH H– H + DH X - X – DH H - X 2
2 2 = 1 ´ 2.303 ´ 0.0821 atm–L /K /mol × log
1
1 1 -x = 16.96 atm–L
= 2x + x – 2x =
2 2 2 Wreversible 16.96
-1
= = 1.69 » 1.7
\ x = 50 × 2 = 100 kJ mol Wirreversible 10.00
1 27. (b) For 5 moles of gas at temperature T,
24. (a) N 2 (g) + O 2 ® N 2 O (g )
2 PV1 = 5RTT
1 .-. + ..
N º N(g) + ( O = O ) ® N = N = O (g) For 5 moles of gas at temperature T – 2,
2 .. ..
PV2 = 5R(T – 2)
DH f o = [Energy required for breaking of bonds ] \ P (V2 - V1 ) = 5R(T – 2 – T);
- [Energy released for forming of bonds]
PDV = – 10R,
1 – PDV = 10R
= (D H N º N + D HO = O - (D H N= N + DH N= O )
2 When DV is negative, W is + ve.
1 28. (a) C 2 H 5 OH(l ) + 3O 2 ( g ) ¾¾
® 2CO2 ( g ) + 3H2 O(l)
= (946 + ´ 498) - (418 + 607) = 170 kJ mol -1
2 Bomb calorimeter gives DU of the reaction
Resonance energy = 170 - 82 = 88 kJ mol-1 Given, DU = –1364.47 kJ mol–1
Dng = – 1
1 1
25. (b) H 2 + Cl 2 ¾¾
® HCl 1 ´ 8.314 ´ 298
2 2 DH = DU + DngRT = -1364.47 -
1000
DH HCl = å B.E. of reactant = – 1366.93 kJ mol–1
- å B.E. of products
29. (b)

1 1 1
30. (b) (
D S° = S° CO 2 + 2 ´ S° H 2O - S° CH 4 + 2 ´ S°O 2 )
-90 = ´ 430 + + ´ 240 - B.E. of HCl = (213.6 + 2 × 69.9) – (186.2 + 2 × 205.2)
2 2 2
= – 242. 8 J K–1 mol–1.
\ B.E. of HCl = 215 + 120 + 90
= 425 kJ mol –1
EBD_7504
S-C-18 SOLUTIONS

Speed Test-36

1. (b) Kp = Kc (RT)Dn since Dn = 0, Kp = Kc. AgBr


2. (a) Kw at 25°C = 1 × 10–14 Ksp = [Ag+] [Br–] = 5.0 × 10–13
At 25ºC 5.3 ´ 10 –13
[Ag + ] =
Kw = [H+] [OH–] = 10–14 [Br – ]
At 100°C (given) AgI
Kw = [H+] [OH–] = 55 × 10–14 Ksp = [Ag+] [I–] = 8.3 × 10–17
Q for a neutral solution
[H+] = [OH–] 8.3 ´ 10–17
[Ag + ] =
\ [H+]2 = 55 × 10–14 [I – ]
or [H+] = (55 × 10–14)1/2
If we take éë Cr2 O 4 ùû = [Cl ] = [Br ] = [I ] = 1
–2 – – –
Q pH = – log [H+]
On taking log on both side than maximum [Ag+] will be required in case of Ag2CrO4.
– log [H+] = –log (55 × 10–14)1/2 6. (b) MY M+ + Y–
1 2
KSP = s = 6.2 × 10–13
pH = - log 55 + 14 log10
2
pH = 6.13 s = 6.2´10-13
[Salt] s = 7.87 × 10–7 mol L–1
3. (c) pH = pKa + log
[Acid] NY 3 N3+ + 3Y–
[Salt] KSP = s × (3s)3 = 27s4 = 6.2 × 10–13
5 = 4 + log [Q pKa = – log Ka]
[Acid] 1/4
æ 6.2 ´ 10-13 ö
Given, Ka = 1 × 10– 4 s= ç ÷
\ pKa = – log (1× 10– 4) = 4 è 27 ø
Now from Handerson equation s = 3.89 × 10–4 mol L–1
\ molar solubility of NY3 is more than MY in water.
[Salt]
pH = pKa + log 7. (b) DG°NO(g) = 86.6k J/mol = 86600 J/mol
[Acid]
Putting the values G° NO2 (g) = x J/mol
[Salt] T = 298, KP = 1.6 × 1012
5= 4 + log
[Acid] DG° = – RT ln KP
Given equation,
[Salt]
log = 5–4=1 ˆˆ† 2NO (g)
2NO(g) + O2 (g) ‡ˆˆ
[Acid] 2
\ 2DG°NO – 2DG°NO = – R (298) ln (1.6 × 1012)
Taking antilog 2
2DG°NO – 2 × 86600 = – R (298) ln (1.6 × 1012)
[Salt]/[Acid] = 10 = 10 : 1 2
2DG°NO = 2 × 86600 – R (298) ln (1.6 × 1012)
4. (a) The reaction given is an exothermic reaction thus 2

according to Le chatalier ’s principle lowering of 1


temperature, addition of F2 and Cl2 favour the forward DG°NO = [2 × 86600 – R(298) ln (1.6 × 1012]
2 2
direction and hence the production of ClF3. = 0.5 [2 × 86600 – R (298) ln (1.6 × 1012)]
5. (c) Ag2CrO4 8. (d) Clausius – Clapeyron's equation
Ksp = [Ag+]2 é Cr2 O4–2 ù = 1.1 × 10–12 d ln P -DH v
ë û =
dT RT 2
1.1 ´ 10 –12 9. (a)
[Ag + ] =
éCr2 O4–2 ù
ë û
ˆˆ† 2NH3 ; K1 =
(i) N 2 + 3H 2 ‡ˆˆ
[ NH3 ]2
AgCl
Ksp = [Ag+] [Cl–] = 1.8 × 10–10
[ N2 ][ H2 ]3
1.8 ´ 10 –10 [ NO]2
2 =
+
[Ag ] =
[Cl – ] [ N 2 ][ O 2 ]
Chemistry S-C-19

1
(iii) H 2 + O 2 ¾¾ ® H 2 O; K 3 =
[ H 2 O] Kp
or, K p = a 2 P \a = when 1 - a 2 = 1
2 [ H 2 ][ O 2 ]1/2 P
Applying (II + 3 × III – I) we will get
14. (d) Max. pressure of CO2 = Pressure of CO2 at equilibrium
5 K
2NH3 + O2 ‡ˆˆ ˆˆˆ †ˆ 2NO + 3H 2O;
For reaction,
2 SrCO3(s) ‡ˆˆˆˆ† SrO(s) + CO2
[ NO]2
´
[ H 2O ]3 [ NH3 ]2 Kp = PCO2 = 1.6 atm = maximum pressure of CO2
K=
[ N2 ][ O 2 ] [ H 2 ]3 ´ [ O2 ]3 / 2 [ N 2 ][ H 2 ]3 volume of container at this stage.
nRT
\ K = K2 × K33 / K1 V= …(i)
P
10. (b) Initially on increasing temperature rate of reaction will Since container is sealed and reaction was not earlier at
increase, so % yield will also increase with time. But at equilibrium.
equilibrium % yield at high temperature (T2) would be \ n = constant.
less than at T1 as reaction is exothermic so the graph is PV 0.4 ´ 20
n= = …(ii)
T1 RT RT
Put equation (ii) in equation (i)
T2 é 0.4 ´ 20 ù RT
%yield

V= ê = 5L
ë RT úû 1.6
15. (b) C(s) + CO2(g) ¾® 2CO(g)
Apply law of mass action,
time
ˆˆ† 2Ag + ( aq ) + C 2O 42 - ( aq ) (PCO ) 2 (10PCO2 )2
11. (c) Ag 2C 2 O 4 (s) ‡ˆˆ KP = or 63 =
2s s PCO2 PCO2
KSP = [Ag+]2 [C2O42–] (Given KP = 63) and PCO = 10PCO2
[Ag+] = 2.2 × 10–4 M 100(PCO2 ) 2
Given that: or 63 = or 63 = 100 PCO2
PCO2
\ Concentration of C2O42– ions,
63
-4 PCO2 = = 0.63 atm
éë C2 O24 - ùû = 2.2 ´ 10 M
100
2 PCO = 10PCO2 = 10 × 0.63 = 6.3 atm
-4
= 1.1 ´10 M Ptotal = PCO2 + PCO = 0.63 + 6.3 = 6.93 atm.
\ KSP = (2.2 × 10–4)2 (1.1 × 10–4) ˆˆ† H∗ ∗ HA,
16. (d) H 2A ‡ˆˆ
= 5.324 × 10–12 [H + ][HA - ]
\ K = 1.0 × 10 –5 = (Given)
12. (b) For a gaseous phase reaction Kp and Kc are related as 1
[H 2 A]
Dn g
K p = K c ( RT )
HA- ¾¾
® H+ + A--
For the given reaction,
[H + ][A -- ] (Given)
1 \ K 2 = 5.0 ´ 10 -10 =
CO(g) + O2 (g) ® CO2(g) [HA - ]
2
Dng = 1– (1 + 0.5) = – 0.5 or -
1 [H + ]2 [A 2 - ]
2 K= = K1 ´ K2
1
[H 2 A]
- = (1.0 × 10–5) × (5 × 10–10) = 5 × 10–15
\ Kp = Kc ( RT ) 2
1
17. (d) Let the weak monoacidic base be BOH, then the reaction
Kp - that occurs during titration is
or = ( RT ) 2
Kc BOH + HCl ® BCl + H2O
ˆˆ† PCl 3 + Cl 2
13. (b) PCl 5 ‡ˆˆ Equilibrium : B+ + H 2 O ‡ˆˆ ˆˆ† BOH + H +
c (1- h) c.h c.h
1– a a a Using the normality equation, N1V1 = N 2 V2
a a (acid) (base)
P´ P a 2P
\ Kp = 1 + a 1 + a Substituting various given values, we get
=
1- a
1+ a
EBD_7504
S-C-20 SOLUTIONS

2 15
or V1 = 2.5´ ´ = 2.5 × 3 = 7.5 ml = 10- 5 ´ 10 - 3 = 10- 4
5 2 \ pH + pOH = 14
Then the concentration of BCl in resulting solution is \ pH = 14 – 4 = 10
given by 19. (c) Let s = solubility
2 ˆˆ† Ag ∗ ∗ IO3,
AgIO3 ‡ˆˆ
´ 2.5 1 s s
[BCl] = 5 = or 0.1 M
10 10 Ksp = [Ag+] [IO3–] = s × s = s2
[Total volume = 2.5 + 7.5 = 10 ml] Given Ksp = 1 × 10–8
Kw
Since K h < \ s = K sp = 1´10-8
Kb
1´10,14 = 1.0 × 10–4 mol/lit = 1.0 × 10–4 × 283 g/lit
[ Kh < = 10–2
1´10,12 (Q Molecular mass of Ag IO3 = 283)
0.1h 2 0.1h 2
Thus K h < or 10,2 < 1.0 ´10-4 ´ 283 ´100
(1, h) (1, h) = gm /100ml
1000
or 10–2 – 10–2 h = 0.1 h 2 = 2.83 × 10–3 gm/ 100 ml
or 0.1 h2 + 10–2 h – 10–2 = 0 20. (a) (HSO4)– can accept and donate a proton
(Solving this quadratic equation for h, we get) (HSO4)– + H+ ® H2SO4 (acting as base)
é (HSO4)– – H+ ® SO42–. (acting as acid)
-b ± b 2 - 4ac ù
êx = ú 21. (a) Mg(OH)2 ® [Mg2+] + 2[OH–]
Using ê 2a ú
ë û x 2x
Ksp = [Mg] [OH]2 = [x][2x]2 = x.4x2 = 4x3.
-10-2 ± (10-2 )2 + 4 ´10 -1 ´ 10 -2
h= ˆˆ† Cr 3+ (aq.) + 3OH - (aq.)
22. (b) Cr(OH)3 (s) ‡ˆˆ
2 ´ 0.1 s 3s
,2 ,4 ,3 (s) (3s)3 = Ksp
,10 ± 10 ∗ 4´10
=
2´ 0.1 27 S 4 = K sp
,0.01 ± .0001 ∗ 0.004 1/4
æ K sp ö æ 1.6 ´10-30 ö
1/4
= s = çç ÷÷ = çç ÷
0.2 ÷
è 27 ø è 27 ø
-0.01 ± 0.0041 23. (d) [Cu(H2O)4]2+ + 4NH3 ‡ˆˆ ˆˆ† [Cu(NH3)4]2+ + 4H2O
=
0.2 involves lose and gain of electrons. H2O is coordinated
,0.01 ± 0.064 to Cu by donating electrons (LHS). It is then removed
= by withdrawing electrons.
0.2
0.54 24. (b) pH of an acidic solution should be less than 7. The reason
= [Neglecting the negative term] is that from H2O. [H+] = 10–7M which cannot be neglected
0.2
= 0.27 in comparison to 10–8M. The pH can be calculated as.
from acid, [H+] = 10–8M.
\ [H+] = c.h = 0.1 × 0.27 = 2.7 × 10–2 M
from H2O, [H+] = 10–7M
Thus the correct answer is [d].
\ Total [H+] = 10–8 + 10–7
18. (c) M1V1 = M2V2
= 10–8 (1 + 10) = 11× 10–8
1 × 0.10 = M2 × 100
\ pH = – log [H ] = –log 11×10–8
+
M2 = 0.001 = 10–3
= –[log11 + 8 log 10]
BOH ‡ˆˆ ˆˆˆ† +
ˆ B + OH
- = –[1.0414 – 8] = 6.9586
C 0 0 25. (d) Q pH = 1 ; H+ = 10–1 = 0.1 M
C(1- a ) C a Ca
pH = 2 ; H+ = 10–2 = 0.01 M
Ca ´ Ca \ M1 = 0.1 V1 = 1
Kb = M2 = 0.01 V2 = ?
C(1 - a)
From
Kb = Ca2 (Q1 - a » 1) M1V1 = M2V2
0.1 × 1 = 0.01 × V2
a = Kb / C
V2 = 10 litres
Kb \ Volume of water added = 10 – 1 = 9 litres
[OH- ] = C a = ´
C
2 (g)
Chemistry S-C-21

Given Kc = 1.8 × 10–6 at 184 ºC


Hg2Cl2 2Hg 2+ + 2Cl-
R = 0.0831 kj/mol. k 2s 2s
Kp= 1.8 × 10–6 × 0.0831 × 457 = 6.836 × 10–6 Ksp = (2s)2 × (2s)2 = 16s4
[ Q 184°C = (273 + 184) = 457 k, 1/ 4
Dn = (2 + 1, –1) = 1] æK ö
s = ç sp ÷
Hence it is clear that Kp > Kc è 16 ø

27. (c) éë Zr3 ( PO 4 ) 4 ùû ‡ˆˆ


ˆˆ† 3Zr 4+ + 4PO 43- BaSO4 Ba ++ + SO 4--
3S 4S s s
Ksp = (3S)3 (4S)4 Ksp = s2
= 27S 3 × 256 S 4
= 6912 S 7. s= K sp
1/7
æ K sp ö CrCl3 Cr3+ + 3Cl -
\ S = çç ÷÷ s 3s
è 6912 ø
28. (c) In aqueous solution BA(salt) hydrolyses to give Ksp = s × (3s)3 = 27 s4
1/ 4
s = æç sp ö÷
ˆˆ† BOH + HA
BA + H2O ‡ˆˆ K
Base acid è 27 ø
Now pH is given by Hence the correct order of solubilities of salts is
1 1 1 1/ 4 1/ 4 1/ 5
pH = pKw + pKa - pKb æ K sp ö æ Ksp ö æ K sp ö
2 2 2 K sp > ç ÷ >ç ÷ >ç ÷
è 16 ø è 27 ø è 108 ø
substituting given values, we get
30. (b) NaCN is a salt of strong base and weak acid ; pH
1
pH = (14 + 4.80 - 4.78) = 7.01 1 1
2 = 7+ pK a + log C
2 2
29. (b) Cr2(SO4)3 2Cr +++ + 3SO4-- pKa for HCN = 14–4.70 = 9.30
2s 3s
1 1
Ksp = (2s)2 (3s)3 = 4s2 × 27s3 = 108s5 \ pH = 7 + ´ 9.30 + log 0.5; pH = 11.5
2 2
1/ 5
æ K sp ö
s= ç ÷
è 108 ø
EBD_7504
S-C-22 SOLUTIONS

Speed Test-37

1. (d) O.N. of N in NO2 and N2O4 is +4 C 2 O 4– – ¾¾


® 2CO 2 .... (iii)
\ difference is zero. On balancing
O.N. of P in P2O5 and P4O10 is +5
C 2 O 4– – ¾¾
® 2CO 2 + 2e – .... (iv)
\ difference is zero
On multiplying eqn. (ii) by 5 and (iv) by 2 and then
O.N. of N in N2O is +1 and in NO is +2. The difference is 1
adding we get
O.N. of S in SO2 is +4 and in SO3 is +6. The difference is +2.
2MnO 4– + 5C2 O4– – + 16H + ¾¾
®
-
0 +1
2. (d) Zn + 2Ag CN ¾ –2e
¾ ¾® Ag + Zn (CN)2
0 +2 2Mn ++ + 10CO2 + 8H 2 O
+ 2e - 8. (d) Order of decreasing electrode potentials of Mg, K, Ba
The oxidation state shows a change only in (d) and Ca is
3. (b) Magnesium provides cathodic protection and prevent Mg > Ca > Ba > K
rusting or corrosion. It can be explained by their standard reduction potentials.
+7 +2 E 0 + = - 2.925
4. (a) 2K Mn O 4 + 3H 2SO 4 ® K 2SO 4 + 2 Mn SO 4 + 3H 2 O + 5O K |K
+2 +3 E = - 2.90
Ba 2+ |Ba
2 Fe SO 4 + H 2SO 4 + O ® Fe 2 (SO 4 ) 3 + H 2O
E 0 2+ = - 2.87
O.N. of Mn changes from +7 to +2 (Reduction) Ca |Ca
O.N. of Fe changes from +2 to +3 (Oxidation) E0 2+ = - 2.37
Mg |Mg
+6 +4
5. (c) 2HI -1 + H 2SO 4 ¾ ¾® I 02 + SO 2 + 2H 2 O . In this Highly negative value of E 0red shows the least value of
reaction oxidation number of S is decreasing from + 6 to electrode potential.
9. (c) On balancing the given reaction, we find
+4 hence undergoing reduction and for HI oxidation
3Na2HAsO3 + NaBrO3 + 6HCl
Number of I is increasing from –1 to 0 hence underegoing
¾¾® 6NaCl + 3H3AsO4 + NaBr
oxidation therefore H2SO4 is acting as oxidising agent. 10. (c) In SO3– –
x + 3(– 2) = – 2; x = + 4
ˆˆ† M, for this reaction, high negative
6. (a) (i) Mnn+ + ne– ‡ˆˆ
In S2O4– –
value of E° indicates lower reduction potential, that means 2x + 4(– 2) = – 2
M will be a good reducing agent. 2x – 8 = – 2
2x = 6; x = + 3
Stronger reducing agent Þ Easy to oxidise
In S2 O62–
ß 2x + 6(– 2) = – 2
Lower reduction potential Ü higher oxidation potential 2x = 10; x = + 5
hence the correct order is
(ii) Element F Cl Br I
S2O4– – < SO3– – < S2O6– –
Reduction potential +2.87 +1.36 +1.06 +0.54 11. (a) CrO2 Cl2
(E° volt) Let O. No. of Cr = x
As reduction potential decreases from fluorine to iodine, \ x + 2 (–2) + 2 (–1) = 0
oxidising nature also decreases from fluorine to iodine. x–4–2=0
(iii) The size of halide ions increases from F – to I –. The \ x= +6
bigger ion can loose electron easily. Hence the reducing 12. (a) If an electronegative element is in its lowest possible
nature increases from HF to HI. oxidation state in a compound or in free state. It can
7. (a) X MnO 4– + Y C2 O4– – + Z H + function as a powerful reducing agent.
e.g. I –
Z
X Mn ++ + 2Y CO 2 + H 2O 13. (d) Xe = 53.5 % \ F = 46.5%
2 Relative number of atoms Xe
First half reaction
53.5 46.5
MnO 4– ¾¾ ® Mn + + .... (i) = = 0.4 and F = = 2.4
131.2 19
On balancing
Simple ratio Xe = 1 and F = 6 ; Molecular formula is XeF6
MnO 4– + 8H + + 5e – ++ .... (ii)
Second h
2. It gets
Chemistry S-C-23

superficially coated with a green layer of basic carbonate -1 +6 0 +4


CuCO3. Cu (OH)2. 23. (d) 2HI + H 2SO 4 ® I2 + SO2 + 2H 2 O
Oxidised Re duced
15. (b) In iodometry, K2Cr2O7 liberates I2 from iodides (NaI or RA OA
KI). Which is titrated with Na2S2O3 solution. 24. (c) A reaction, in which a substance undergoes simultaneous
K 2 Cr2O7 + I – + H + ¾¾
® Cr 3+ + I 2 oxidation and reduction, is called disproportionation
Here, one mole of K2Cr2O7 accepts 6 mole of electrons. reaction. In these reactions, the same substance
molecular weight simultaneously acts as an oxidising agent and as a
\ Equivalent weight = reducing agent. Here Cl undergoes simultaneous
6
+4 oxidation and reduction.
16. (c) H 2SO3 ( aq ) + Sn 4+ ( aq ) + H 2 O ( l ) ¾¾
® 2KOH + Cl2 ® KCl+ KOCl + H 2O.
+6 0 -1 +1
Sn 2+ ( aq ) + HSO4- ( aq ) + 3H + 25. (b) Na2S4O6 has the structure :
Hence H2 SO 3 is the reducing agent because it O O
undergoes oxidation. + –
Na O – S*– S – S – S*– O Na
– +

-1 +1
17. (d) Cl0 + 2NaOH ® NaCl+ NaClO+ H O O O
2 2
+6 +7 +4
O.N. of two S* atoms are +5 each and that of other two S
3M nO4-- + 2H 2O ® 2 Mn O 4- + M nO 2 + 4OH – atoms is zero each.
+4 +5 +3 26. (b) CrO2Cl2, MnO4–; O.N. of Cr and Mn are +6 and +7
2 N O 2 + H 2 O ® HN O 3 + H NO 2 respectively.
All undergo disproportionation +2 0 2.5
18. (a) The reaction given is 27. (a) 2 S 2O 2- ( aq ) + I ( s ) ® S O2- ( aq ) + 2I - ( aq )
3 2 4 6
Cr2O72– + Fe2+ + C2O42– ¾¾ ® Cr3+ + Fe3+ + CO2 +2 0
Cr2O72– ¾¾ ® 2Cr3+ S2O32- ( aq ) + 2Br2 ( l ) + 5H 2O ( l ) ®
On balancing +6
14H+ + Cr 2O72– + 6e– ¾¾ ® 2Cr3+ + 7H2O ......(i) 2SO 24- ( aq ) + 4Br - ( aq ) + 10H + ( aq )
Fe2+ ¾¾ ® Fe3+ + e– ......(ii) Hence, bromine is a stronger oxidising agent than I2, as
C2O4 ¾¾2– ® 2CO2 + 2e– .......(iii) it oxidises S of S2 O32 - to SO 24 - whereas I2 oxidises it
On adding all the three equations.
Cr2O72– + Fe2+ + C2O42– + 14H+ + 3e– only into S4 O62 - ion.
¾¾ ® 2Cr3+ + Fe3+ + 2CO2 + 7H2O 28. (a) Higher the value of reduction potential higher will be the
Hence the total no. of electrons involved in the reaction oxidising power whereas the lower the value of reduction
=3 potential higher will be the reducing power.
19. (d) H2S, the oxidation state of S is – 2. So it cannot accept +5 +6 +6
more electrons because on accepting 2 electrons S 29. (c) KClO 3 + H 2C 2 O 4 + H 2SO 4 ® K 2SO 4
accquires a noble gas configuration. So, it can acts only –1
as a reducing agent by loosing electron. On the other + KCl + CO 2 + H 2 O
hand, the oxidation state of S in SO2 is + 4 which is an i.e. maximum change in oxidation number is observed in
intermediate oxidation state of sulphur so it can reduce Cl (+5 to –1).
as well oxidise. 30. (d) Zinc gives H2 gas with dil H2SO4/HCl but not with HNO3
20. (c) In all the given compounds oxidation number of non metal because in HNO3, NO3– ion is reduced and give NH4NO3,
is + 4. As C belongs to group IV and it is in its maximum N2O, NO and NO2 (based upon the concentration of
oxidation state. So, reduction in oxidation number of HNO3)
nonmetal is not possible only in CO2. As we know that [ Zn + 2HNO 3 ¾
¾® Zn ( NO 3 ) 2 + 2H] ´ 4
reduction is always accompained by an increase in (nearly 6%)
oxidation number of reducing agent. So, CO2 cannot acts
as reducing agent among the given choices. HNO 3 + 8H ¾
¾® NH 3 + 3H 2 O
21. (b) The balanced equation is
2C2 H6 + 7O2 ® 4CO 2 + 6H 2O. NH 3 + HNO 3 ¾
¾® NH 4 NO 3
Ratio of the coefficients of CO2 and H2O is 4 : 6 or 2 : 3.
4Zn+10HNO3 ¾¾ ® 4Zn(NO3 ) 2 +NH 4 NO3 +3H 2O
22. (b) The compound undergo oxidation itself and reduces
others is known as reducing agent. In this reaction O. N. Zn is on the top position of hydrogen in electrochemical
of Ni changes from 0 to + 2 and hence Ni acts as a reducing series. So Zn displaces H2 from dilute H2SO4 and HCl
agent.
+ H2
EBD_7504
S-C-24 SOLUTIONS

Speed Test-38

1. (c) Hg will not displace hydrogen since it is present below 13. (a) Normality of 10V of H2O2
hydrogen in ECS.
68 ´10
2. (c) The correct option is (c). In H2O, hydrogen has + 1 = 17 ´ N \ N= 1.78
22.4
oxidation no. In CaH2, hydrogen has – 1 oxidation no.
3. (c) Occlusion is the phenomenon of adsorption of 14. (b) In the structure of ice each molecule of H 2 O is
hydrogen by metal. surrounded by three H2O molecules in hexagonal honey
4. (a) It is due to hydrogen bonding when H2O forms a cage comb manner which results an open cage like structure.
like structure in solid ice and density is reduced. As a result there are a number of 'hole' or open spaces.
In such a structure lesser number of molecules are
5. (d) Heavy water is stable.
packed per ml. When ice melts a large no. of hydrogen
6. (a) A fusion reaction is difficult to occur because positively
bonds are broken. The molecules therefore move into
charged nuclei repel each other. At very high
the holes or open spaces and come closer to each other
temperature of the order of 106 to 107 K, the nuclei may
than they were in solid state. This result sharp increase
have sufficient energy to overcome the repulsive forces
in the density. Therefore ice has lower density than
and fuse. It is for this reason, fusion reactions are also
water.
called thermonuclear reactions. Hence, hydrogen can
electrolysis
be fused to form helium at high temperature and high 15. (b) 2HSO4– (aq) ¾¾¾¾¾ ® HO3SOOSO3H(aq)
pressure.
® 2HSO -4 (aq) + 2H+(aq) + H2O2(aq)
Hydrolysis
7. (b) H - (aq)+ H 2O(l) ¾¾ ® OH - (aq)+ H2 ( g ) ¾¾¾¾¾
base 1 acid 1 base 2 acid2 16. (c) A diamagnetic substance contains no unpaired
In this reaction H– acts as bronsted base as it accepts electron.
one proton (H+) from H2O and form H2. H2 is diamagnetic as it contains all paired electrons
8. (c) H2O2. The key reactions are
PbO + H 2S ® PbS + H 2 O H 2 = s b2 , H +2 = s 1b , H 2- = s b2 , s *1 +
a ; He 2 = s b2 , s *1
a
(black) (diamagnetic) (paramagnetic) (paramagnetic) (paramagnetic) (paramagnetic)

PbS + 4H 2 O 2 ® PbSO 4 + 4H 2 O
(white) H 2 = s b2 , H +2 = s 1b , H 2- = s b2 , s *1 + 2
a ; He2 = s b , s a
*1
(diamagnetic) (paramagnetic) (paramagnetic) (paramagnetic) (paramagnetic)
When blackened statues are treated with H2O2, the
PbS is oxidised to PbSO4, which is colourless (White). 17. (b) Q 22.4 L of H2 at STP = 1 mole of H2
9. (d) The water gas is a combination of carbon and
1
hydrogen. It is used in manufacturing of methanol. \ 0.224 L of H2 at STP = ´ 0.224 = 0.01 moles.
22.4
CO ( g ) + 2H 2 ( g ) ¾¾¾¾
Cobalt
® CH3OH ( l) 18. (a) Only elements having reduction potential less than -
Catalyst
0.41V liberate hydrogen with cold water.
10. (a) Elements of group 7, 8 and 9 of d-block do not form
hydrides at all. The inability of metals of group 7, 8 and 19. (a) In one electron species, such as H-atom, the energy of
9 of periodic table to form hydrides is referred to as orbital depends only on the principal quantum number,
hydrides gap of d-block. In these compounds H atoms n.
are supposed to occupy interstitial position in the metal Hence answer (d)
lattices. They are also called non-stoichiometric i.e. is < 2s = 2p < 3s = 3p = 3d < 4s
hydrides. = 4p Þ 4d = 4g
11. (a) Cu + 4HNO3(conc.) ¾® Cu(NO3)2 + 2NO2 + 2H2O 20. (d) Chlorine has lone pair which it can donate to form
coordinate bond while hydrogen cannot.
® C2 H 5O - Na + + 1 / 2H 2 ­
C2 H 5OH + Na ¾¾
21. (a) MgCO3 is insoluble in water.
Mg + 2H 2 O(steam) ¾¾
® Mg(OH) 2 + H 2 ­ 22. (a) True peroxide contains O-O linkage and O 22 - ion. They

® C6 H 5O - Na + + 1 / 2H 2 ­
C6 H 5OH + Na ¾¾ give hydrogen peroxide with dil H 2SO 4 .
BaO 2 + H 2SO4 ( dil.) ® BaSO4 + H 2 O2
NaH + H 2 O ¾¾
® NaOH + H 2 ­
23. (d) Very pure hydrogen can be prepared by the action of
12. (c) Hydrogen is not used in the synthesis of
Chemistry S-C-25

NaH + H2O ¾¾® NaOH + H2 26. (d) Polyphosphates (e.g., sodium hexa metaphosphate)
(very pure Hydrogen) form soluble complexes with cations such as Ca2+,
D Mg2+ and so they are used as water softening agents.
24. (c) 2H 2 O 2 ¾¾® 2 H 2 O + O 2
2KI + H 2O2 ® 2KOH + I 2 Na 2 [Na 4 (PO3 )6 ] + 2Ca 2+ ¾¾
® Na 2 [Ca 2 (PO3 ) 6 ]+ 4Na +
"Calgon " Hardness "Soluble"
2KMnO4 + 3H 2SO4 + 5H 2O2 ® Sod. hexa metaphosphate

27. (c) Ortho and para forms of hydrogen resemble in their


K 2SO 4 + 2MnSO 4 + 8H 2 O + 5O 2
chemical properties.
25. (a) H2O2 acts as a reducing agent only in presence of
strong oxidising agents (i.e., MnO4–) in acidic as well 28. (c) H 2O2 has oxidizing and reducing properties both.
as alkaline medium. 29. (b) Na2 zeolite + CaCl 2 ® Ca zeolite + 2 NaCl
2KMnO4 + 3H 2SO 4 + 5H 2O 2 ¾¾
® 30. (c) CH4(g) + H2O(g) ¾¾¾
1270 K
® CO + 3H2
Ni
K 2SO 4 + 2MnSO 4 + 8H 2 O + 5O 2
Mixture of CO and H2 is called water gas.
EBD_7504
S-C-26 SOLUTIONS

Speed Test-39

1. (d) Alkali metals are highly electropositive and thus highly 10. (c) According to Fajan's rule,
reducing .Therefore reduction , double decomposition Size of cation µ Ionic character..
and displacement methods for their extraction are not Recall that size of metal (ion) increases while going
down the group, and decreases on crossing the periods
suitable .Only electrolytic methods are useful for their
from left to right. Thus Rb+ (group I) is largest and
extraction. Be2+ (group II) is smallest in size. Hence RbCl has
2. (a) A reducing agent is a substance which can loose greatest, and BeCl2 has lowest ionic character.
electron and hence a reducing agent should have low 11. (d) Na2O2 is peroxide of sodium not super oxide. The
ionisation energy. Now since ionisation energy
formula of sodium superoxide is NaO2.
decreases from Li to Cs, the reducing property should
12. (c) Ca and CaH2 both react with H2O to form H2 gas,
increase from Li to Cs. The only exception to this is
lithium. This is because the net process of converting Ca + 2H 2 O ¾
¾® Ca (OH) 2 + H 2
an atom to an ion takes place in 3 steps. CaH 2 + 2H 2 O ¾
¾® Ca ( OH ) 2 + 2 H 2
(i) M(s) ® M(g) DH = Sublimation energy
(ii) M(g) ® M+(g) + e– DH = Ionisation energy whereas K gives H2 while KO2 gives O2 and H2O2
(iii) M+(g)+H2O ® M+ (aq) DH = Hydration energy 2K + 2 H2O ¾ ¾® 2KOH + H2
The large amount of energy liberated in hydration of Li 2KO2 + 2H 2O ¾¾
® 2KOH + O2 + H 2O2
(because of its small size) makes the overall DH negative. Similarly, Na gives H2 while Na2 O2 gives H2O2
This accounts for the higher oxidation potential of 2 Na + 2 H 2 O ¾
¾® 2 NaOH + H 2
lithium i.e., its high reducing power.
3. (a) According to Fajan rules, ionic character increases with Na 2 O 2 + 2H 2 O ¾¾® 2 NaOH + H 2 O 2
increase in size of the cation and decrease in size of the Likewise Ba gives H2 while BaO2 gives H2O2
anion. Thus, CsF has higher ionic character than NaCl Ba + 2 H2O ¾ ¾® Ba (OH)2 + H2
and hence bond in CsF is stronger than in NaCl. BaO2 + 2H2O ¾ ¾® Ba (OH)2 + H2 O2
4. (b) BaCO3 forms a yellow ppt of barium chromate. BaCO3 13. (a) Basic strength of the oxides increase in the order
forms a white precipitate of BaSO4. BaCl2 is soluble in Li2O < Na2O < K2O < Rb2O < Cs2O. The increase in
water. basic strength is due to the decrease in I.E. and increase
5. (a) The weaker the base, the less stable is its carbonate. in electropositive character.
The melting points of the halides decrease in the order
Since LiOH is the weakest base, hence Li2CO3 has the
NaF > NaCl > NaBr > NaI, as the size of the halide ion
lowest thermal stability. increases. The decrease in melting point is due to
6. (b) As outermost electronic configuration of alkali metals is increase in the covalent character with increase in the
ns1 and also their size are largest in their respective size of anion according to Fajan's rule.
periods so their 1st I.E will be lowest among the given
options. As second electron is to be removed from 14. (d) Zn + + + 2NaOH ¾¾® Zn (OH) 2 + 2 Na +
complete shell or noble gas core, so the IInd I.E. must be
highest among the given options. So, option (b) is correct Zn (OH ) 2 + 2 NaOH ¾
¾® Na 2 ZnO 2 + 2H 2 O
choice.
7. (a) As we move down the group, the lattice energies of ¾® 2 Na + + ( ZnO 2 ) 2 -
Na 2 ZnO 2 ¾
carbonates remain approximately the same. However
the hydration energies of the metal cation decreases D
from Be++ to Ba++, hence the solubilities of carbonates 15. (b) CaCO3( s ) ¾¾® CO 2( g ) ­ + CaO ( s )
of the alkaline earth metal decrease down the group A colourless B
mainly due to decreasing hydration energies of the CaO(s) + H2 O ¾¾ ® Ca (OH )2(aq)
cations from Be++ to Ba++. B
8. (a) 4KO2 + 2 CO2 ® 2 K2CO3 + 3O2 . Ca (OH )2 + 2CO2 ¾¾ ® Ca (HCO3 )2(aq)
KO2 is used as an oxidising agent. It is used as air purifier C
in space capsules. Submarines and breathing masks as Ca (HCO3 )2(s) ¾¾ ® CaCO3(s) + CO2(g) + H 2 O(g)
it produces oxygen and remove carbon dioxide. A
\ Correct choice : (b)
9. (d) Halides of group I and II impart characteristic colour to
the flame due to low IE of the central atom. However, 16. (d) In Castner Kellner cell, sodium amalgam is formed at
ionization energy of Be and Mg atoms is high due to mercury cathode.
their small size, hence they can't be excited to higher
levels by Bunsen 2;
impart colour
Chemistry S-C-27

25. (d) In alkaline earth metals, ionic size increases down the
Mg 3 N 2 + 6H 2 O ¾
¾® 3Mg (OH) 2 + 2 NH 3 ­ group. The lattice energy remains constant because
(colourless)
sulphate ion is so large, so that small change in cationic
CuSO 4 + 4 NH 3 ¾
¾® [Cu ( NH 3 ) 4 ]SO 4 size does not make any difference. On moving down
Blue complex the group the degree of hydration of metal ions
decreases very much leading to decrease in solubility.
18. (d) K and Mg, both form oxides
\ BeSO 4 > MgSO 4 > CaSO 4 > SrSO 4 > BaSO 4
K + O 2 ® KO 2 ;2Mg + O 2 ® 2MgO
26. (a) The alkali metals dissolve in liquid ammonia without
Mg form nitride also 3Mg + N 2 ® Mg 3 N 2 evolution of hydrogen. The metal loses electrons and
K does not form nitride. combine with ammonia molecule.
M––® M+ (in liquid ammonia)
19. (c) The atom becomes larger on descending the group, so
+ e– (ammoniated)
the bonds becomes weaker (metallic bond), the
M + (x + y) NH3 ®[M(NH3)x]+ + e–(NH3)y
cohesive force/energy decreases and accordingly Solvated electron
melting point also decreases. It is ammoniated electron which is responsible for colour.
20. (d) Lesser the lattice energy, more will be the solubility in 27. (c)
H2O. 28. (c) Mg is more reducing in nature than carbon
21. (d) For a good quality cement, the ratio of silica (SiO2)
to alumina (Al2O3) should be between 2.5 and 4 and CO 2 + 2Mg ® 2MgO + C
the ratio of lime (CaO) to the total of the oxides of 29. ® Na + + e -
(b) Q For Na ¾¾ IE1 = 5.1 eV
silicon (SiO2), aluminium (Al2O3) and iron (Fe2O3)
+ -
should be as close as possible to 2. \ For Na + e ¾¾ ® Na EF = -5.1 eV
22. (b) 2Na + 2H 2 O ¾¾® 2NaOH + H 2 ­ (because the reaction is reverse)
30. (b) Li2 = s1s2 s*1s2 s2s2
2K + 2H 2O ¾¾® 2KOH + H 2 ­
1
All alkali metals decompose water with the evolution \ Bond order = (4 - 2) = 1
2
of hydrogen.
Li2+ = s1s2 s*1s2 s2s1
Ca + 2H 2 O ¾¾® C a(OH) 2 + H 2 ­
1
Sr + 2H 2O ¾¾® Sr(OH) 2 + H 2 ­ B.O. = (3 - 2) = 0.5
2
Be + 2H2O ¾¾® No reaction Li2– = s1s2 s*1s2s2s2s*2s1
Ca, Sr, Ba and Ra decompose cold water readily with 1
evolution of hydrogen. Mg decomposes boiling water B.O. = (4 - 3) = 0.5
2
but Be is not attacked by water even at high The bond order of Li2+ and Li2– is same but Li2+ is more
temperatures as its oxidation potential is lower than stable than Li2– because Li2+ is smaller in size and has 2
the other members. electrons in antibonding orbitals whereas Li2– has 3
23. (c) Monovalent sodium and potassium ions and divalent electrons in antibonding orbitals. Hence Li2+ is more
magnesium and calcium ions are found in large stable than Li2–.
proportions in biological fluids.
24. (b) Alkali metals readily lose electron to give monovalent
M+ ion. Hence they are never found in free state in nature.
EBD_7504
S-C-28 SOLUTIONS

Speed Test-40

1. (b) In Boric acid each B atom is sp 2 hybridized and contains good conductor of electricity but also reduce the melting
BO 33- units which are held together by hydrogen bonds. point of the mixture to around 1140 K.
10. (c) Alum furnishes Al3+ ions which bring about coagulation
2. (c) AlCl3 exists as a dimer (Al2Cl6). It is a strong Lewis acid of negatively charged clay particles, bacteria etc.
as it has an incomplete octet and has a tendency to gain 11. (c) Due to non-availability of vacant d-orbitals, it cannot
electrons. AlCl3 undergoes hydrolysis easily and forms exceed its coordination number more than four. Thus
an acidic solution. carbon never forms complexes e.g., [CCl6]2– deos not
AlCl3 + 3H2O ® Al(OH)3 + 3HCl exist but [SiCl6]2– exists.
Option (c) is true that AlCl3 sublimes at 180ºC. 15.2°C
ˆˆˆˆˆ†
12. (b) a-tin ‡ˆˆˆˆ
164°C
ˆˆˆˆˆ
ˆ b-Sn ‡ˆˆˆˆ †
232°C
ˆˆˆˆˆ
ˆ g -Sn ‡ˆˆˆˆ †
ˆ Liquid tin
3. (c) The approximate composition of cement is : (Grey) (White) (Britle)

Lime (CaO) = 50 – 60% (most common and stable) (Rhombic)


Silica (SiO2) = 20 – 25 % Although transitions of white to grey tin occurs at any
temperature below 15.2°C, it becomes rapid only at
Alumina (Al2O3) = 5 – 10 %
–50°C, unless a catalyst is present. During the conversion
Magnesia (MgO) = 2 – 3% of white tin to grey tin (in cold countries) volume
Ferric oxide (Fe2O3) = 2 – 3% increases. Grey Sn is very brittle easily crumbles down to
4. (d) K 2SO 4 .Al 2 (SO4 )3 .24H 2 O ¾¾¾®
water powder. This phenomenon is called tin diseases, tin past
or tin plague.
2K + + 4SO24 - + 2Al3+ + 24H 2O 13. (a) Al 2 (SO 4 )3 + 6NH 4 OH ® 2Al(OH)3 + 3(NH 4 ) 2SO 4
5. (c) The IE1 of Ga is more than that of Al because of the small Al(OH) 3 + NaOH ® Na + [Al(OH) 4 ]-Soluble complex it
atomic size and greater effective nuclear charge of Ga.
6. (d) Ceramics are inorganic , non-metallic, solid minerals. They is insoluble in NH 4 OH
come in a variety of forms, including silicates (silica, SiO2 14. (d) CO 2 + Ca(OH) 2 ¾¾
® CaCO3 ¯+ H 2O
with metal oxides), oxides (oxygen and metals), carbides (A) white ppt
(carbon and metals), aluminates (alumina, Al2O3 with metal
oxides) and nitrides. CaCO 3 + CO 2 + H 2 O ¾
¾® Ca ( HCO 3 ) 2
The given ceramics are B4C (carbides), SiC (carbides), Calcium bicarbonat e
(Souble in water )
Si3N4 (nitrides) and thus, none of these is an oxide.
Heat
All of these are non-oxide ceramics. Ca ( HCO 3 ) 2 ¾¾¾® CaCO 3 + CO 2 + H 2 O
7. (b) Because they react with water to form methane gas (B)
Al4C3 + 12H2O –––® 4 Al(OH)3 + 3CH4 A and B are CO 2 and CO 2 .
Aluminium carbide Methane 15. (c) Hydrolysis of substituted chlorosilanes yield
corresponding silanols which undergo polymerisation.
Be 2C + 4H 2 O ¾¾
® 2Be(OH)2 + CH 4
Beryllium carbide Methane CH3 Cl H OH – 2HCl CH3 OH
Si + Si
D , air
8. (a) 2Al + 6HCl ¾¾¾ CH3 Cl H OH CH3 OH
® 2AlCl3 + 3H 2
Dialkyl silandiol
2Al + 3Cl 2 ¾¾
® 2AlCl3 Polymerisation of dialkyl silandiol yields linear
1000°C thermoplastic polymer.
Al 2O3 + 3C + 3Cl 2 ¾¾¾¾
® 2AlCl3 + 3CO
vapours CH3 CH3

Cooling
HO — Si — OH + H O — Si — OH
Anhydrous AlCl3
CH3 CH3
D
AlCl3 .6H 2O ¾¾ ® Al(OH)3 + 3HCl + 3H 2O
CH3 CH3
Thus AlCl3 cannot be obtained by this method
9. (c) Fused alumina (Al2O3) is a bad conductor of electricity. HO — Si — O — Si — OH
Therefore, cryolite (Na AlF ) and fluorspar (CaF ) are
CH3 CH3
Chemistry S-C-29

16. (d) Inertness of ns 2 electrons of the valence shell to Cl m Cl Cl Cl Cl


participate in bonding on moving down the group in 2p
20
heavier p-block elements is called inert pair effect. Be 98° Be 263 pm Be Be
As a result, Pb(II) is more stable than Pb(IV) 82°
Cl Cl Cl Cl Cl
Sn(IV) is more stable than Sn(II)
\ Pb(IV) is easily reduced to Pb(II) and can acts as an Cl Cl Cl
oxidising agent whereas Sn(II) is easily oxidised to Sn(IV)
and can acts as a reducing agent. Al Al
17. (c) CO2 forms carbonic acid H2CO3, when dissolved in Cl Cl Cl
water, CO is neutral, whereas other two GeO2 and SnO2 24. (c) In graphite, carbon is sp2 hybridized. Each carbon is thus
are solids. linked to three other carbon atoms forming hexagonal
D rings. Since only three electrons of each carbon are used
18. (b) Na2B4O7. 10 H2O ¾¾¾¾® Na2B4O7
-10H 2O in making hexagonal ring, fourth electron of each carbon
D
is free to move. This makes graphite a good conductors
Na 2 B4O7 ¾¾
® 2 NaBO2 + B2 O3 of heat and electricity.
anhydrous sod.metaborate Boric anhydride Further graphite has a two dimensional sheet like structure.
These various sheets are held together by weak van der
CuO + B2 O3 ¾¾
® Cu(BO 2 )2 Waal’s force of attraction. Due to these weak forces of
cupric meta borate(Blue beed)
attraction, one layer can slip over the other. This makes
19. (b) H 3 BO 3 ¾100°C 160°C graphite soft and a good lubricating agent.
¾ ¾® HBO 2 ¾¾ ¾® H
25. (d) 2CaF2 + SiO 2 + H 2 SO 4 ¾¾
®
H 2 B4 O 7 + H 2O ¾
¾® 2 B 2 O 3 + H 2 O
hydrolysis
SiF4 + H2 O + CaSO 4 ¾¾¾¾¾ ® CaSiF6
Cl Cl Cl
20. (c) Al Cl AlCl3 (dimer) 26. (a) Glass is a translucent or transparent amorphous
Cl Al Al
supercooled solid solution or we can say super cooled
21. (b) F and Cl are more oxidising in nature and can achieve Pb liquid of silicates and borats having a general formula
in (IV) O.S. but Br2 and I 2 can not achieve Pb in (IV) R2O. MO . 6 SiO2. where R = Na or K and M = Ca, Ba, Zn
or Pb.
O.S. secondly Pb 4 + is strong in oxidising nature and in 27. (b) The hydrolysis of Trialkylchlorosilane
R3 SiCl yields dimer :
its presence, Br - and I - can not exist. R R
22. (d) In SiO2 (quartz), each of O-atom is shared between two
SiO44– tetrahedra. R Si O Si R
R R
28. (a) The linking of identical atoms with each other to form
long chains is called catenation. However, this property
decreases from carbon to lead. Decrease of this property
is associated with M-M bond energy which decreases
from carbon to lead.
23. (c) The correct formula of inorganic benzene is B3N3H6 so 29. (c) The feldspars are most abundant aluminosilicate minerals
(d) is incorrect statement in the Earth surface. The silicon atoms and aluminium atoms
OH occupy the centres of interlinked tetrahedra of SiO44 - and
|
Boric acid (H3BO3 or B - OH ) is a lewis acid so (a) is AlO54- . These tetrahedra connect at each corner to other
|
OH tetrahedra forming an intricate, three dimesional, negatively
charged framework. The sodium cations sit within the voids
incorrect statement.
in this structure.
The coordination number exhibited by beryllium is 4 and
not 6 so statement (b) is incorrect. 30. (c) CO2 being more dense covers the igniting material more
Both BeCl2 and AlCl3 exhibit bridged structures in solid effectively than N2.
state so (c) is correct statement.
EBD_7504
S-C-30 SOLUTIONS

Speed Test-41

1. (b) Coloured impurities are removed by adsorbing over 9. (b) If there is a small difference (10 or less) in the boiling
activated charcoal. points of liquids fractional distillation is used e.g. ac-
CH3 O etone b.p. 333 K and methanol b.p. 338 K.
5 4 3 2 1 10. (a) Cyclohexane (iv) is non-planar and has chair
2. (a) CH3 – CH – C – CH2 – CH2OH
conformation. In this conformation, the bond angle is
3. (b) The chemistry of the Lassaigne’s test of nitrogen is the normal tetrahedral angle (109°, 28’) and thus has
D no angle strain and hence is most stable. The rest of
Na + C + N ¾¾® NaCN
From organic the molecules are nearly planar and hence their stability
compound
depends upon the angle strain in accordance with
2NaCN + FeSO 4 ¾¾
® Fe(CN) 2 + Na 2SO 4 Baeyer’s strain theory. Since cyclopropane has higher
Fe(CN)2 + 4NaCN ¾¾
® Na 4[Fe(CN)6 ]
Sod. ferrocyanide æ 109° 28'– 60° ö
angle strain ç = 24 ,°44' ÷ th an
3Na 4 [Fe(CN)6 ] + 4Fe3+ ¾¾
® Fe 4 [Fe(CN) 6 ]3 + 12Na + è 2 ø
(Ferric ferrocyanide)
(Prussian blue) æ 109° 28'–108° ö
cyclopentane ç = 0°, 44' ÷ . Therefore
4. (c) The atom or group which has more power to attract è 2 ø
electrons in comparision to hydrogen is said to have
-I effect. Thus higher the electronegativity of atom cyclopentane (iii) is more stable than cyclopropane
stronger will be the -I effect. As electronegativity of N, (i). Further, because of the presence of a double bond
O and F follow the order N < O < F hence based upon in a three membered ring, cyclopropene (ii) is the least
electronegative character order of-I effect is stable. Thus the order of stability is (iv) > (iii) > (i) >(ii).
– NR2 < – OR < – F. - OH
11. (a) CH3* CH (OH)COOH ¾¾¾® CH 3CH(H )COOH
5. (a) This method is applied for the purification of substances Lactic acid +H
No chiral carbon
which (i) are insoluble in water, (ii) are volatile in steam, Chiral carbon is present
(optically inactive )
(optically active)
(iii) are associated with non steam volatile impurities,
(iv) have high molecular weights and (v) possess a 12. (b) Wt. of organic substance = 0.25 g
fairly high vapour pressure at about the boiling point V1 = 40 mL, T1 = 300 K
of water e.g. Aniline. P1 = 725 – 25 = 700 mm of Hg
6. (a) Carbinol is methyl alcohol (CH 3 OH), hence P2 = 760 mm of Hg (at STP)
3 2 1
vinylcarbinol should be CH = CHCH OH (prop-2- T2 = 273 K
2 2
enol)
P1V1 P2 V2
.. H H =
H +
T1 T2
C C H2C – CH = CH 2 V2 (Volume of nitrogen at STP)
..

H H
..

7. (a)
H Hyperconjugation in propene
273 ´ 700 ´ 40
Carbocation Alkyl group
= = 33.52 mL
Hyperconjugation in CH2CH3
+ 300 ´ 760
H Percentage of nitrogen

H2C – CH2 28× volume of N 2 at STP ×100


= 22400 × wt. of organic substance
Hyperconjugation
in free radical
8. (a) It is a bridge compound 28 ´ 33.52 ´ 100
= = 16.76%
1 22400 ´ 0.25
6 O
7 2
13. (d) Na 2 [Fe(CN)5 NO] + S2 - ® Na 4 [Fe(CN)5 NOS]
5 3 Sodium thio-nitroprusside
4 (violet)
Chemistry S-C-31

14. (c) Th e mixture of sugars is a h omogenous one. 18. (b) Hydrazine (NH2NH2) does not contain carbon and
Homogeneous mixtures of a solvent and one or more hence on fusion with Na metal, it cannot form NaCN;
solutes (dissolved substances) are often separated by consequently hydrazine does not show Lassaigne’s
chromatography. Chromatography works to separate test for nitrogen.
a mixture because the components of a mixture 19. (d) In staggered conformation any two hydrogen atoms
distribute themselves differently when they are in on adjacent carbon atoms are as far apart as possible
contact with a “two phase system”. One phase is there by minimising repulsion between the electron
stationary and the other is moving or mobile. The clouds of s-bonds of two non-bonded H-atomic
stationary phase may be a solid packed in a tube or a (torsional strain)
piece of paper. The mobile phase may be liquid of H
gaseous. HH
H H
Me
Me H H H
H2 H H H H
Pd-BaSO4
H
15. (b) H
Staggered form Eclipsed form
Me H
No torsional strain
OH OH
Me H Me | |
Me
H 20. (a) PhCOCOPh ¾LiAlH
¾ ¾¾ 4®
Ph - C x - C x - Ph The
H2
| |
Pd-BaSO H H H
H H product contains two similar asymmetric carbon atoms
Due to cis-addition of H2 to the triple bond, the reduced and two optically active and one optically inactive meso
product has a plane of symmetry and hence is optically form.
inactive. 21. (c) Steam distillation is the most suitable method of
16. (d) The amount of s-character in various hybrid orbitals is separation of 1 : 1 mixture of ortho and para nitrophenols
as follows. as there is intramolecular hydrogen bonding in
sp = 50%, sp2 = 33% and sp3 = 25% o-nitrophenol.
Therefore s character of the C – H bond in acetylene 22. (b) In Kjeldahl’s method nitrogen is converted into
(sp) is greater than that of the C – H bond in alkene (sp2 (NH4)2 SO4, then to NH3
hybridized) which in turn has greater s character of the 23. (b) The resonance structures have same positions of nuclei
C – H bond than in alkanes. Thus owing to a high s and same number of unpaired electrons.
character of the C – H bond in alkynes, the electrons 24. (a) In SN Ar reactions, a carbanion is formed as an intermediate,
constituting this bond are more strongly held by the so any substituent that increases the stability of carbanion
carbon nucleus with the result the hydrogen present and hence the transition state leading to its formation will
on such a carbon atom can be easily removed as enhance the SNAr reactions. To compare the rates of
proton. The acidic nature of three types of C – H bonds substitution in chlorobenzene, chlorobenzene having
follows the following order electron-withdrawing group, and chlorobenzene having
-C º C - H > - C = C - H > - C - C - H electron-releasing group, we compare the structures
Further, as we know that conjugate base of a strong carbanion I (from chlorobenzene), II (from chlorobenzene
acid is a weak base, hence the correct order of basicity containing electron-withdrawing group) and III (from
is chlorobenzene containing electron-releasing group).
- - -
H - C º C < CH2 = CH < CH 2 - CH3 Z Cl Z Cl
Br Br
– –
17. (b) is optically active
G
I I I II
due to absence of plane of symmetry and center of symmetry.
EBD_7504
S-C-32 SOLUTIONS

of the ring, stabilises carbanion, facilitates SN reaction 32 0.233


(activation effect) 26. (b) % of S = ´ ´ 100 = 10 %
233 0.32
Z Cl 27. (c) Sterioisomerism involve those isomers which contain
same ligands in their co-ordination spheres but differ
– in the arrangement of these ligands in space. Stereo-
isomerism is of two type geomerical isomerism and
optical isomerism. In geomerical isomerism ligands
G occupy different positions around the central metal
atom or ion.
III
In optical isomerism isomers have same formula but
G releases electrons, intensifies –ve charge, differ in their ability to rotate directions of the plane of
destabilizes carbanion, retards S N reaction polarised light.
(deactivation)
NO2 is activating group and CH3 and OCH3 are Cl
28. (b) – – – –
deactiving group. C Cl > C6H5CH2 > (CH3)2 CH > (CH3)3C
Hence, the correct order of nucleophilic substitution Cl
reactions –ve charge –M effect +I effect of CH 3 group
highly dispersed delocalises intensifies the –ve charge
Cl due to – I effect –ve charge
Cl Cl
Cl -
.. .. +
29. (c) CH2 = CH - Cl : a double bond is
.. « CH 2 - CH = Cl
> > > : ..

formed between C and Cl. Hence it is less reactive due to


NO2 CH3 OCH3 resonance
30. (a) % of element Relative Simple ratio
25. (d) The stability of carbanions is affected due to resonance, no. of atoms
inductive effect and s - character of orbitals. Greater the
number of groups having + I group (alkyl group) lesser 38.8
C 38.8 = 3.2 1
stable would be the carbanion. 12
Further stability of carbanion decreases with decrease 16
H 16.0 = 16.0 5
in s-character. Benzene carbanions are stablized due to 1
resonance, hence the correct order is
45.28
N 45.28 = 3.2 1
14
R - C = C > R 2C = CH> > R C – CH
3 2

The correct order of stability of given carbanion is in the


order I > III > II > IV.
Speed Test-42

1. (d) A doubly bonded carbon atom having an alkyl group 6. (a) Electrolysis of a concentrated aqueous solution of either
is oxidised to aldehyde which is further oxidised to sodium or potassium salts of saturated mono-
carboxylic acid. carboxylic acids yields higher alkane at anode.
2RCOOK ¾¾ ¾ ¾¾® 2 RCOO - + 2 K +
Electrolyt ic
4 (i) KMnO ,OH -
CH3CH 2CH = CH CH3 ¾¾¾¾¾¾¾
+
® Oxidation Anode Cathode
(ii) H - –
At anode 2RCOO ® 2RCOO + 2e ¾
¾®
CH3CHO + CH3CH 2CHO R — R + 2CO 2
¯ ¯ At Cathode 2K + + 2e – ® 2 K
CH3COOH CH3CH 2COOH 2K + H2O®2KOH + H2 ­

CH 2 Br
CH 2 NBS
2. (a) CH 2 = CH 2 + H - O - SO2 OH ¾
¾® 7. (c) ¾¾¾ ®
H 2SO 4
(Kolbe's Method)
Å 8. (b) Sum of masses of C and H = 10.5 + 1.0 = 11.5
CH 3 CH 2 + OSO 2 OH ¾¾
® CH3CH 2OSO2OH
Ethyl hydrogen sulphate 10.5
% of C = ´ 100 = 91.3%
Addition of sulphuric acid takes place according to 11.5
Markownikoff’s rule. Alkanes do not absorb cold conc. 1.0
% of H = ´ 100 = 8.7%
H 2SO4 . 11.5
3. (c) The given molecular formula suggests that the aldehyde Element % Ratio of atoms Simplest ratio
formed will be acetaldehyde hence the alkene will be C 91.3 91.3/12 = 7.61 7.61/7.61 = 1 × 7 = 7
CH3CH = CHCH3 H 8.7 8.7/1= 8.7 8.7/7.61 = 1.14 × 7= 8
2- butene \ Empirical formula = C7H8

O PV
O3 H H We know that n =
¾¾® C C RT

O O mass PV
=
molar mass RT

2.81 1 ´1
= ; M = 92.168 » 92
Zn / H 2O
¾¾¾¾® 2CH3CHO M 0.082 ´ 400

O O \ n=
84 + 8
=1
O 92
O
4. (c) + O3 O
O \ Molecular formula = ( C7 H8 ) ´ 1 = C7 H8
O
O O 9. (c) Zn / alcohol
CH 3CHBrCHBrCH 3 ¾¾ ¾ ¾¾® CH 3CH = CHCH 3
Benzene triozonide

CH3 CH3 Since two bromine atoms are lost from different carbon
| | ether atom, the reaction is known as b-elimination.
5. (c) H3C – C – CH2Br + 2Na + Br – CH2– C – CH3 –2NaBr Br
| | - HBr
CH3 CH3 10. (b) ¾¾¾¾¾ ®
(alc)KOH
neo-pentyl bromide Br
1, 2-dibromo-
CH3 CH3 cyclohexane
| |
H3C – C – CH2 – CH2 – C – CH3 - HBr
| | ¾¾¾¾¾
®
(alc) KOH
CH CH Br
1, 3-cyclo-
hexadiene
EBD_7504
S-C-34 SOLUTIONS

11. (a) Toluene on oxidation with mild oxidising agent like 16. (d) Writing the reaction we get
chromyl chloride (CrO2Cl2) gives benzaldehyde.
CH3 CHO CH 3 MgX + CH 3 – C º C – H ¾¾
®

CrO Cl CH3 – C º CMgX + CH4 ( g )


¾¾¾¾®
2 2
So we find that CH4 is produced in this reaction.
toluene benzaldehyde 17. (d)

H SO H3C
12. (c) CH 3 - CH = CH 2 + H 2O ¾¾¾¾
2 4 ® CH - CH - CH
3 3
| H3C
O3 O=C
OH CH3 CH3
2° alcohol Zn + H2O
OHC
CH3 CH3
| | B H
H SO 18. (d) 6 CH3 - CH = CH 2 ¾¾¾®
2 6
CH 3 - C = CH 2 + H 2O ¾¾¾¾
2 4 ® CH - C - CH
3 3 ether, 0°C
| 1, Pr opene
OH
3 ° alcohol H O
Addition follows Markownikoff’s rule. 2(CH3CH 2 CH 2 )3 B ¾¾¾®
2 2
OH -
13. (b) The combustion reaction of ethylene is
6CH 3CH 2 CH 2 OH + 2H 3BO 3
5
C 2 H 2 + O2 ® 2CO2 + H 2 O Pr opanol
2
Both HC 19. (a) Huckel rule is not obeyed. It has only four electrons.
CH and CO2 have sp hybridization.
14. (c) We know that Further it does not have continuous conjugation.
NaHCO 3 20. (b) –Cl group is o-, p-directing due to +R effect ; however it
CH2 CH2 + HOCl CH2OH CH2OH
is deactivating due to strong –I effect of Cl (difference
CH2Cl CH2OH from other o-, p-directing groups which are activating).
\ M = CH 2 OH - CH 2 Cl and R = NaHCO3 The net result is that chlorobenzene undergoes o-, p-
15. (c) Hydration of alkynes give ketones. substitution, but with difficulty
OH
H2O, H 2SO2
H3 C - C º CH H3 C - C = CH 2
HgSO 4 (A)
O
Tautomerism
H3 C - C - CH3
(B)

Å
NaNH2 ® HCº CNa ¾¾¾¾¾¾¾¾¾¾¾¾¾¾¾¾
3 2 H C –CH · Br
HCº CH ¾¾¾¾¾¾¾¾¾¾¾ ® HCºC – CH 2 – CH3
liq. NH 3
21. (a) (X)
1-Butyne NaNH2 liq. NH3
Å
3 2 H C - CH – Br
H3 C – CH 2 – Cº C – CH 2 – CH3 H3 C – CH 2 – CºC N a
3-Hexyne (Y)
22. (d) It is a streoselective reaction and in it a trans product is formed.
Na, NH ( l )
C–R ¾¾¾¾¾
3 ®
C R 'OH

H R
C C
H
[It is a reduction. Metal dissolved reduction
Na/C2H5OH reaction is stereo selective].
CH3
CH2 1 equiv. HBr
Å Br – Br
23. (d) H2C HC Å
(Major)
Chemistry S-C-35

24. (a) Acetylene reacts with the other three as: 28. (a) The intermediate 2º carbocation shows resonance
CH2 +
CH º CNa Na
CH º CH +HCl Br + é ù
liq. NH3 CH 2 = CH - CH = CH 2 ¾¾¾ ® êCH 2 Br - CH - CH = CH 2 ú
CHCl ë û
CH3
+HCl + ù
CHCl CHCl2 « CH 2 Br - CH = CH - CH 2 úû
[AgNO +NH OH]
[AgNO3+NH4OH] AgCº CAg + NH4NO3
CH º CH white ppt.
Br -
¾¾
¾® CH 2 Br - CH = CH - CH 2 Br
25. (c) H 1,4 - Dibromo - 2- butene

CH CH CH2
+
CH CH CH2 CH2-CH=CH2
(i) Hg(OAC)2
29. (a)
(ii) NaBH 4

HCl CH2–CH–CH3
3° allylic carbocation

Cl OH
CH CH2 CH3
CH3 CH3

30. (b) Br / FeBr Mg / ether


¾¾2¾¾¾

¾¾ ¾ ¾
¾®
Br
(Since tertiary carbocation is more stable)
CH3 CH 3
26. (d) H 3C - C º C - H + CH 3CH 2 MgCl ¾
¾®

H 3C - C º C -MgCl + CH 3CH 3 CH3 CH3

H C = O
¾¾2 ¾¾
¾®
MgBr H 3O +
H CH 2OH
CH 3 CH3
C º C - CH 3
CH2Br
27. (c) Ethene is obtained by electrolysis of dipotassium
succinate as follows
– Method (1) will give
CH2COOK CH2COO +
ionization + 2K CH2Br
– CH2Br
CH2COOK CH2COO
Pot. Succinate which is allylic bromination.
ionization
ˆˆˆˆˆˆ† 2OH - + 2H +
2H 2O ‡ˆˆˆˆˆˆ
At anode :

CH2COO CH2COO CH2

– 2e + 2CO2
CH2COO

CH2COO CH2
Unstable
At cathode :

® [ 2H ]
2H + + 2e - ¾¾
EBD_7504
Speed Test-43

1. (b) High concentration of SO2 leads to stiffness of O3 + h u ¾


¾® O 2 + O
flower buds.
2. (a) Radiation coming from sun or outerspace have high NO 2 + O ¾
¾® NO + O 2
energy or short wavelength, which are allowed to enter 2 O3 + h u ¾
¾® 3 O 2 (Net reaction)
by green house gases. However, radiation emitted by
The presence of oxides of nitrogen increase the
earth is in infrared region, having long wavelength, are
reflected back by the envelope of green house gases. decomposition of O 3 .
3. (b) In cold water, dissolved oxygen can reach a 17. (a) PSCs react with chlorine nitrate and HCl to give HOCl
concentration upto 10 ppm, whereas oxygen in air is and Cl2.
about 200, 000 ppm. 18. (d) Using plastic cans for neatly storing substances will
4. (b) Pesticides and synthetic fertilizers pollute the soil. not come under green chemistry. The plastic materials
5. (c) Water is often treated with Cl2 to kill germs. are non-biodegradable.
6. (b) CO is a primary pollutant. 19. (d) Addition of phosphate fertilizers to water leads to
7. (c) CO and oxides of Nitrogen are poisonous gases nutrient enrichment (eutrophication)
present in automobile exhaust gases. 20. (b) Thermal pollution is caused by power plants. Power
8. (b) This represents a great step forward for green plant requires a larger quantity of water for cooling.
chemistry. The water after cooling is left in the water body. The
9. (a) Normal rain water has pH 5.6. Thunderstorm results in temperature of left water is generally very high and
the formation of NO and HNO3 which lowers the pH. affects aquatic life.
10. (c) The irritant red haze in the traffic and congested 21. (a) Based on the features given gas must be SO2 .
places is due to presence of oxides of nitrogen. 22. (b) Sewage water is purified by micro-organisms.
11. (c) Photochemical smog is formed in noon in summer. 23. (d) CO is highly toxic and impairs respiration. CO combine
12. (b) Strength of sewage or degree of water pollution is with haemoglobin of blood and reduces its O2 carry
measured in terms of BOD (Biochemical oxygen capacity.
demand) value. 24. (d) Acid rain is the rain water containing sulphuric acid
13. (a) Photochemical smog is caused by oxides of sulphur and nitric acid which are formed from the oxides of
and nitrogen. sulphur and nitrogen present in the air as pollutants
14. (c) Freons and chlorofluorocarbon find their way to and rain water has a pH range of 4-5.
stratosphere through without being destroyed in the 25. (b)
troposphere due to their low reactivity. In the 26. (d) The oxidised hydrocarbons and ozone in presence of
stratosphere the Cl and Br atoms are liberated from the humidity cause photochemical smog.
parent compounds by the action of ultraviolet light Hydrocarbons + O2, NO2, NO, O, O3 ® Peroxides,
e.g. formaldehyde, peroxyacetyl-nitrate (PAN), acrodein etc.
CF2 Cl 2 + h n ¾¾ ® CF2 Cl + Cl Hence chlorofluorocarbons are not common component
of photochemical smog.
CFCl3 + h n ¾¾
® CFCl 2 + Cl
27. (a) Microorganisms present in the soil is a sink for CO.
The Cl atoms can destroy ozone molecules through a 28. (b) The ideal value of D.O for growth of fishes is 8 mg/ l .
variety of catalytic cycles.
7mg /l is desirable range, below this value fishes get
15. (d) Aerosols use CFC and flying jets release NO. Both
susceptible to disease. A value of 2 mg/ l or below is
causes depletion of ozone layer. lethal for fishes.
16. (c) The ozone layer, existing between 20 to 29. (c) The excess of fluorine in water causes fluorosis. The
35 km above the earth’s surface, shield the earth from symptoms of fluorosis are mottling of teeth (yellowish
the harmful U. V. radiations from the sun. streaks) and abnormal bones liable to fracture etc. It is
Depletion of ozone is caused by oxides of nitrogen an example of endemic disease.
N2O + h u ¾
¾® NO + N 30. (d) Minamata is caused by Hg poisoning.
reactive nitric oxide
NO + O 3 ¾
¾® NO 2 + O 2
Speed Test-44

1. (c) 17. (d)


2. (b) ZM Z´ M
r=
NA V 18. (d) r = ,
No ´ a3
rN A V 8.92 ´ 6.02 ´ 1023 ´ (362)3 ´10 -30
Z= = Z ´ 27
M 63.55 2.7 = \ Z=4
=4 6.02 ´ 10 ´ (4.05) 3 ´ 10 -24
23
\ It has fcc unit cell
Hence it is face centred cubic unit lattice.
3. (c)
d1 (a 2 )3 z1 æ 3 ö 4
3 Again 4r = a 2 = 5.727 Å
4. (c) d = ´ =ç ÷ ´ = 1.46 \ r = 1.432 Å
2 (a1 )3 z 2 è 3.5 ø 2
a
5. (a) Wurtzite has face centred cubic structure in which each 19. (a) d hkl = 2
Zn2+ ion is attached to four S2– ions and each S2– ion ( h + k + l 2 )1 / 2
2

remains in contact with four Zn 2+ ions. Hence


1 1
coordination number of each ion is 4. Hence d100 = 1, d110 = , d111 =
6. (c) The increase in pressure results in decrease in size of 2 3
+ - 20. (c) For cubic geometry the limiting ratio is
ions (more in case of anion than cation), the r / r
increases and the coordination number also increase. r+
> 0.731 i.e., (c)
r+ 60 r-
7. (a) ratio is = 0.441, Hence LiF has NaCl structure 21. (a) Statement (a) is correct.
r- 136
with CN = 6. 22. (a) For bcc lattice body diagonal = a 3 .
8. (a) Diamond is like ZnS. In diamond cubic unit cell, there
The distance between the two oppositely charged ions
are eight corner atoms, six face centered atoms and
a
four more atoms inside the structure. = 3
Number of atoms present in a diamond cubic cell 2
1 1 387 ´ 1.732
= 8´ ∗ 6 ´ ∗ 4 < 8 = = 335pm
8 2 2
(corners) (face (inside 23. (d) Number of formulas in cube shaped crystals
centered) body) 1. 0
= ´ 6.02 ´ 10 23 since in NaCl type of structure 4
9. (a) Fraction of unoccupied sites in NaCl crystal 58.5
2.165 ´ 103 formula units form 'a' cell
= 1-
2.178 ´ 103 23
2.178´103 , 2.165´103 0.13´103 \ units cells = 1.0 ´ 6.02 ´10 = 2.57 × 1021 unit cells.
< < 58.5 ´ 4
2.178´103 2.178
130 24. (c) As CsCl is body-centred, d = 3a / 2.
< = 5.96 × 10–3 25. (c) AB is just like NaCl. Thus twelve A+ are at edges and 1
2178
10. (a) MnO2 within body of fcc i.e. in octahedral voids and six B– at
11. (c) I2 (s) faces and 8 at corner.
12. (d) 26. (b) Among the given crystals, only silicon exists as a
13. (a) covalent solid. It has diamond like structure.
14. (b) It is the property of liquid crystal. 27. (c) The p.f. for body centred cube = 0.68 (see text).
15. (d) To get a n – type semiconductor from silicon, it should r+
28. (d) Calculate the ratio to get the limiting ratio value and
be doped with a substance with valency 5. r-
e.g., silicon is doped with phosphorus to form a consult the table. All are correct.
n – type semiconductor. 29. (c)
16. (d) nl = 2d sin q ; 30. (b) In Na2O there is antifluorite structure. Here negative
ions form the ccp arrangement so that each positive
æ 3ö ion is surrounded by 4 negative ions and each negative
2 ´ 1Å = 2 ´ dsin 60
EBD_7504
S-C-38 SOLUTIONS

Speed Test-45

1. (b) According to Le-chateliers principle, for an exothermic 6. (d) According to Henry’s law,
reaction (DH< 0) increase in temperature decreases the m=k×p
solubility. given KH = 1.4 × 10–3
o o
2. (b) Ptotal = p A ´ x A + p B ´ x B pO2 = 0.5 or
= 80.0 × 0.4 + 120.0 × 0.6 = 104 mm Hg pO2 = K H ´ x O2
The observed Ptotal is 100 mm Hg which is less than 0.5
\ x O2 =
104 mm Hg. Hence the solution shows negative 1.4 ´ 10 - 3
deviation. m
No. of moles; n =
3. (c) p A = poA ´ x A = Total pressure × y M
A
m
0.7 ´ 10 -4 =
p B = poB ´ x B = Total pressure × yB 32
Where x and y represent mole fraction in liquid and m = 22.4 ´ 10 -4 g = 2.24 mg
vapour phase respectively. 7. (c) Let a be the give of dissociation then
poB x B yB NaCl Na + Cl -
\ = , 1- a a +a
poA x A yA
1- a + a + a
1 - yA
Van’t Hoff's factor i = = 1+ a
poB (1 - x A ) 1
=
poA x A yA Again Van’t Hoff’s factor
Normal mol . wt 58.5
1 po æ 1 ö = = = 1.83
– 1 = Ao ç - 1÷ Observed mol.wt 31.8
xA p B è yA ø
Equating to both values of i, \1 + a = 1.83
p oA 1 po \ a = 0.83
= – Ao
p oB yA pB 8. (c) These two components A and B follows the condition
of Raoult’s law if the force of attraction between A and
1 po 1 æ po ö B is equal to the force of attraction between A and A or
or, = Ao . + çç 1 - Ao ÷÷ B and B.
xA p B yA è pB ø
9. (c) There is no change in vapour pressure.
p oA 1 p oB - p oA 10. (b) ps = x A (poA – poB ) + poB ;
= . +
p oB yAp oB o
ps = pA ´ x A - poB ´ x A + poB ;
This is equation of straight line.
o
ps = pA ´ x A - pBo (1 - x B ) + pBo
p oA p oB - p oA
Slope = , Intercept =
p oB p oB \ ps = poA ´ xA + pBo ´ xB.
4. (b) DTf = 0.3° C This is condition for ideal solution.
11. (d) DTf = 0 – (– 0.00732°) = 0.00732
K ´ WB ´ 1000 DTf = i × Kf × m
DTf = 0.3° C = f
M B ´ WA
ΔTf 0.00732
i= = =2
1.86 ´ WB ´ 1000 K f × m 1.86× 0.002
0.3 =
62 ´ 5000 12. (b) As DTf = Kf. m
\ WB = 50 g DTb = Kb. m
The amount used should be more than 50 g. DT f DTb
Hence, we have m = =
m æ m ö RT Kf Kb
5. (b) p V = n RT = RT or M 2 = ç 2 ÷
M è V ø p Kf
Where M2
and m2
Chemistry S-C-39

Þ [DTb = 100.18 - 100 = 0.18°C] C6H12O6 and C12H22O11 are not ionised so they have
single particle.
1.86 Hence, Al2(SO4)3 have maximum value of depression
= 0.18 × = 0.654°C
0.512 in F.P or lowest F.P
As the Freezing Point of pure water is 0°C, 24. (b) Sodium sulphate dissociates as
DTf = 0 –Tf Na 2SO 4 (s) ¾¾ ® 2Na + + SO -4 -
0.654 = 0 – Tf hence van’t hoff factor i = 3
\ Tf = – 0.654
Thus the freezing point of solution will be – 0.654°C. Now D T f = i k f .m
= 3 × 1.86 × 0.01 = 0.0558 K
13. (b) [DH mix < 0]
14. (d) C2H5I and C2H5OH form non-ideal solution. 25. (a) K 4 [Fe(CN) 6 ] and Al 2(SO 4 )3 both dissociates to
15. (b) For this solution intermolecular interactions between give 5 ions or i = 5
n-heptane and ethanol are weaker than n-heptane-n- ˆˆ† 4K+ + [Fe(CN)6]4–
K4 [Fe(CN)6] ‡ˆˆ
heptane & ethanol-ethanol interactions hence the ˆˆ† 2Al3+ + 3SO 4--
and Al2 (SO4 )3 ‡ˆˆ
solution of n-heptane and ethanol is non-ideal and
shows positive deviation from Raoult’s law. 26. (b) 1 molal solution means 1 mole of solute dissolved in
16. (c) 1000 gm solvent.
17. (a) Liquid solvent and solid solvent are in equilibrium. \ nsolute = 1 wsolvent = 1000g
18. (d) 1000
\ nsolvent = = 55.56
19. (b) The molality involves weights of the solute and the 18
solvent. Since weight does not change with the
1
temperature, therefore molality does not depend upon Xsolute = = 0.0177
the temperature. 1 + 55.56
27. (b) Concentration of
wt ´1000
20. (d) Molarity (M) = 25.3 1000
mol. wt. ´ vol (ml) Na 2CO3 = ´ = 0.955 M
106 250
wt. 1000
2= ´ [Na + ] = 2 × 0.955 = 1.91 M
63 250
63 é CO32– ù = 0.955 M
wt. = gm ë û
2 28. (d) Azeotrope of HCl + H2O contains 20.2% HCl.
100 29. (d) DTf = i.k f .m ; DTb = i.k b .m
wt. of 70% acid = ´ 31.5 = 45 gm
70
DTf k f
21. (c) Vant Hoff factor i = 4 in case of (NH4)3PO4, =
DTb k b
(NH4)3 PO4 3NH +4 + PO34-
DTf = 0 - ( -0.186°C) = 0.186°C
22. (d) DTb = iKb m
Given, (DTb)X > (DTb)Y 0.186 1.86 0.52 ´ 0.186
= Þ DTb = = 0.052
\ iX Kb m > iYKb m DTb 0.52 1.86
(Kb is same for same solvent) 30. (a) Mole fraction of any component A
iX > iY
So, x is undergoing dissociation in water. 1= u/c No. of moles of A
x=
23. (a) Depression in F.P. µ No. of particles. Total No. of moles
Al 2 (SO 4 )3 provides five ions on ionisation As total no. of moles > no. of moles of A
thus x can never be equal to one or zero.
¾® 2 Al 3+ + 3SO 42 –
Al 2 (SO 4 ) 3 ¾
while KCl provides two ions
ˆˆ† K + + Cl-
KCl ‡ˆˆ
EBD_7504
S-C-40 SOLUTIONS

Speed Test-46

1. (c) Conductivity of an electrolyte depends on the mobility


° = 3387000
of ions and concentration of ions. The motion of an or Ecell
ionic species in an electric field is retarded by the 32 ´ 96500
oppositely charged ions due to their interionic 3387000 3387
attraction. On dilution, concentration of electrolyte = or V = 1.0968 V
3088000 3088
decreases and the retarding influence of oppositely Thus option (c) is correct answer.
charged ions decreases. Therefore mobility of ions 9. (d) Standard Gibbs free energy is given as DG° = – nE°F
increases.
If E°cell < 0 i.e., – ve
2. (d) Degree of dissociation, DG° > 0
L 8.0 Further DG° = – RT ln Keq
a= = = 2 ´ 10 -2
L ¥ 400 \ DG° > 0 and Keq < 1
10. (b) Fe +2 / Fe ; Eº = –0.44
Ca 2
( )
1 2
Ka = » Ca 2 = ´ 2 ´ 10-2 Fe+3 / Fe+2 ; Eº = 0.77
(1 - a ) 32
The metals having higher negative electrode potential values
= 1.25 ´ 10 -5 can displace metals having lower values of negative electrode
potential from their salt solutions.
3. (b) HCl completely dissociates to give Hr and Cl - ions,
hence act as very good electrolyte. While others are non- 11. (d) WA WB 1.6 Wt. of Ag
= ; =
electrolytes. EA EB 8 108
4. (a) DG = –nF E°cell = –2 × 96500 × 1.1 J = 212.3 kJ.
5. (d) \ Wt . of Ag = 21.6 g
¥ 2 -1 12. (a) Ecell = Eright (cathode) - Eleft (anode).
6. (a) L m = 57 + 73 = 130 Scm mol
7. (d) Kohlrausch’s Law states that at infinite dilution, each 3
13. (c) CH3OH (l) + O (g) ® CO2 (g) + 2H2O (l)
ion migrates independently of its co-ion and 2 2
contributes to the total equivalent conductance of an DGr = [DGf (CO2 , g ) + 2DGf (H 2O, l)] -
electrolyte a definite share which depends only on its
own nature. é 3 ù
From this definition we can see that option (d) is the êë DGf (CH3OH, l )+ 2 DGf (O 2 , g ) úû
correct answer.
8. (c) Writing the equation for pentane-oxygen fuel cell at = – 394.4 + 2 (–237.2) – (–166.2) – 0
respective electrodes and overall reaction, we get = – 394.4 – 474.4 + 166.2 = – 702.6 k J
At Anode: 702.6
+ - % efficiency = ´ 100 = 97%
C5 H12 + 10H 2O ® 5CO 2 + 32H + 32e 726
(pentane)
14. (b) DG = - nE° F
At Cathode:
RT C 2
8O 2 + 32H + + 32e - ® 16H 2O For concentration cell, E = ln
nF C1
Overall :C5H12 + 8O2 ® 5CO2 + 6H 2O
In it R, T, n and F are constant
Calculation of DG° for the above reaction
So E is based upon ln C2 / C1
DG° = [5×(–394.4) + 6× (–237.2)]
– [–8.2] RT
= – 1972.0 – 1423.2 + 8.2 = – 3387.0 kJ Now DG = -nEF = - nF ´ ln C2 / C1
nF
= – 3387000 Joules. = –RTlnC2/C1
From the equation we find n = 32 At constant temperature DG is based upon ln (C2/C1).
° and substituting
Using the relation, DG° = – nFEcell 2 ´ 112g
15. (a) 112 ml of H2 at STP = (Since 22400 ml at STP
various values, we get 22400

– 3387000 =
Chemistry S-C-41

Eq.wt ´ i ´ t X
Amount deposited = Conductivity of NaOH = .Z
96500 Y

2 ´112 1 ´ 965 ´ i X 1000 XZ 4


D m (NaOH) = .Z ´ = 10
\ 22400 = 96500 ; Y 0.1 Y
22. (c) From the given data we find Fe3+ is strongest oxidising
i = 1 amp agent. More the positive value of E°, more is the
16. (b) For Zn2+ ® Zn tendency to get oxidized. Thus correct option is (c).
23. (c) At infinite dilution each ion makes a definite contribution
E = E° -
2.303RT
log
[ Zn ] towards molar conductance which is given by
Zn 2 + /Zn Zn 2+ /Zn nF é Zn 2+ ù
ë û L om = v + l + + v - l -
24. (a) In case of equivalent conductance of strong electrolyte
0.06 1 there is little increase with dilution.
= -0.76 - log = -0.76 - 0.03
2 [ 0.1] 25. (d) A device that converts energy of combustion of fuels,
directly into electrical energy is known as fuel cell.
E = -0.79V
Zn 2 + /Zn
0.0592 [Mn 2 + ]
17. (a) The given order of reduction potentials (or tendencies) 26. (d) E = E° - log
is Z > Y > X. A spontaneous reaction will have the 2 [ H + ]4
following characteristics
0.0592 ´ 4 [Mn 2 + ] 1 / 4
Z reduced and Y oxidised = E° - log
Z reduced and X oxidised 2 [H + ]
Y reduced and X oxidised
= E° – 0.0592 ´ 2 (log[Mn 2 + ]1 / 4 + pH)
Hence, Y will oxidise X and not Z.
D E = E 2 - E1 = 0.0592 × 2 (pH1 - pH2 )
18. (b) For, M + + X - ¾ o
¾® M + X , E cell = 0.44 – 0.33 = 0.11V is
= 0. 118 × 1
positive, hence reaction is spontaneous.
= 0.118V
19. (a) MnO-4 will oxidise Cl - ion according to the equation 27. (d) Here n = 4, and [H+] = 10– 3 (as pH = 3)
Applying Nernst equation
2 MnO -4 + 16H + + 10Cl - ¾
¾® 2 Mn 2 + + 8H 2O + 5Cl 2 ­
The cell corresponding to this reaction is as follows: 0.059 [Fe2+ ]2
E = Eº – log + 4
n [H ] ( pO2 )
Pt , Cl 2 (1 atm) |Cl- || MnO4- , Mn 2+ , H + | Pt
0.059 (10-3 )2
Eocell = 1.51 -1.40 = 0.11 V = 1.67 - log
4 (10-3 ) 4 ´ 0.1
Eocell being +ve, DG ° will be -ve and hence the above
0.059
= 1.67 - log107 = 1.67 – 0.103 = 1.567 V
reaction is feasible. MnO-4 will not only oxidise Fe 2 + 4
ion but also Cl - ion simultaneously.. 28. (b) In H 2 - O 2 fuel cell, the combustion of H2 occurs to
create potential difference between the two electrodes
20. (c) The cell reaction is as follows : 29. (a)
+
30. (a) H2 ¾¾ ® 2H+ + 2e–
A® A + e – 1 atm 10-10
+
B + e® B
( )
2
+ +
E 0.059 10-10
Adding A + B ® A + B H 2 /H + =0– log
2 1
21. (b) Conductivity (X) = conductance (c) × cell constant
E = +0.59 V
X H 2 /H +
\ Cell constant =
Y
EBD_7504
S-C-42 SOLUTIONS

Speed Test-47

1. (b) k = Ae -Ea / RT 7. (c) For a first order reaction


Ea 1 2.303 a
log k = log A - . k= log10
2.303R T t a-x
1 when t = t½
Plot of log k Vs.
T
2.303 a
-E a k= log10
Straight line Slope = t½ a-a /2
2.303R
2. (d) Rate constant k = 0.6 × 10–3 mole per second. (unit mol 2.303 ln 2
L–1S–1 shows zero order reaction) or t ½ = log10 2 =
k k
For a zero order reaction
[A] = [A]0 – kt 8. (b) Rate = kC 2A C½
B
and [A0] – [A] = [B] = kt
= 0.6 × 10–3 × 20 × 60 = 0.72 M Rate = k[2C A ]2 [4C B ]½ = 4 ´ 2[C A ]2 [C B ]½

1
3. (c) t1 / 2 µ , where n is the order of reaction 9. (a) A (g ) + 2 B(g ) ¾
¾® C(g ) + D (g )
( p ) n -1
Rate = k[A][B]2 = k(0.60)(0.80)2 .......................(i)
n -1 n -1
2 æ 50 ö 1 æ1ö
=ç ÷ or =ç ÷ when p c = 0.20 atm pA is reduced to 0.40 and
4 è 100 ø 2 è 2ø
pB = 0.40 (See stoichiometric representation)
\ n=2 Rate = k[0.40] [0.40]2..............................(ii)
4. (b) Let x torr of C2H4O decompose after 20 min. Then,
80 – x + 2x =120 ; x = 40 torr = 50% of initial pressure. 0.40 ´ 0.40 ´ 0.40 1
Hence t1/2 = 20 min. For 75% reaction, fraction left \ (ii) divide by (i) = =
0.60 ´ 0.80 ´ 0.80 6
2
25 1 æ 1 ö 10. (a) From data 1 and 3, it is clear that keeping (B) const,
= = =ç ÷
100 4 è 2 ø When [A] is doubled, rate remains unaffected. Hence
No. of half lives = 2. Time needed for 75% reaction., rate is independent of [A]. from 1 and 4, keeping [A]
2 × 20 = 40 min constant, when [B] is doubled, rate become 8 times.
5. (b) The presence of enzyme (catalyst) increases the speed Hence
of reaction by lowering the energy barrier, i.e. a new rate µ [ B]3 .
path is followed with lower activation energy. 11. (a) Since initial velocity is ten times the permissible value
\ A0 = 10A
ET 0.693 0.693
E'T l= =
t1/ 2 30
Ea
Products 2.303 A 2.303 10A
log 0 =
Energy

Ea t1/2 = log
1
l A 0.693 / 30 A
2.303 ´ 30
Reactants + catalyst = ´ log10 = 100 days.
0.693
Progress of reaction
12. (a) Presence of catalyst does not affect enthalpy change
Here ET is the threshold energy. of reaction DH R = E f - E b = 180 – 200 = – 20 kJ/mol
Ea and Ea is energy of activation of reaction in absence 13. (d) Since the slow step is the rate determining step hence
1
and presence of catalyst respectively. if we consider option (A) we find
n -1
æ -2 ö n -1
(t1/ 2 )1 æ a2 ö
6. (c) =
(t1/ 2 )2 çè a1 ÷ø
Chemistry S-C-43

Now if we consider option (B) we find 16. (b) For a first order reaction
0.693 0.693
Rate = k [ Cl2 ] éë HS ùû
-
...(i) t1/2 = ; k= = 0.5 × 10–3s–1
k 1386
For equation, 2.303 1 2.303 4
ˆˆ† H+ + HS– 17. (c) t1/ 4 = log = log
H S ‡ˆˆ
2 k 3/ 4 k 3

é H + ù é HS- ù 2.303 2.303


= (log 4 - log 3) = (2log 2 - log 3)
K = ë ûë û k k
H 2S
2.303 0.29
= (2 ´ 0.301 - 0.4771) =
K [ H 2 S] k k
-
or éë HS ùû = 18. (b, d)
H+
According to Arrhenius equation
Substituting this value in equation (i) we find
k 2 Ea æ 1 1 ö
Rate = k [ Cl2 ]
[ H S] [ Cl ][ H 2S]
K 2 = k' 2
ln = -
k1 R çè T1 T2 ÷ø
H+ éH+ ù
ë û
Ea æ 1 1 ö
Thus slow step should involve 1 molecule of Cl 2 and = – ç - ÷
R è T2 T1 ø
1 molecule of H2S.
hence only , mechanism (A) is consistent with the given
k1 E æ1 1 ö
rate equation. ln =- a ç - ÷
k2 R è T1 T2 ø
14. (d) From 1st and 2nd sets of data - no change in rate is
observed with the change in concentration of ‘C’. So 19. (d) Overall order = sum of orders w.r.t each reactant.
the order with respect to ‘C’ is zero. Let the order be x and y for G and H respectively
From 1st and 4th sets of data
Dividing eq. (4) by eq. (1) [G]mole [H]mole rate(mole
Exp.No.
litre-1 litre -1 litre- time -1 )
-3 x
1.25 ´ 10 é 0.005 ù 1 a b r

5.0 ´ 10 -3 ë 0.010 úû 2 2a 2b 8r
or 0.25 = (0.5)x or (0.5)2 = (0.5)x 3 2a b 2r
\ x=2
The order with respect to ‘A’ is 2 from the 1st and 3rd Q For (1) and (3), the rate is doubled when conc. of
sets of data dividing eq. (1) by eq. (3) G is doubled keeping that of H constant i.e.,
rate µ [G] \ x = 1
5.0 ´ 10-3
y
é 0.005 ù From (2) and (3), y = 2

1.0 ´ 10- 2 ë 0.010 úû \ Overall order is 3.
or (0.5)1 = (0.5)y Þ y = 1 20. (c) Rate1 = k [A]n [B]m; Rate2 = k [2A]n [½B]m
The order with respect to ‘B’ is 1 Rate 2 k[ 2A] n [½ B] m
\ =
So the order with respective the reactants A, B and C is Rate1 k[ A]n [ B] m
2, 1 and 0.
= [2]n [½]m = 2n.2–m = 2n–m
15. (b) DH = Ea(f ) - Ea(b)
2000
-
Thus energy of activation for reverse reaction depend 21. (d) Given, k1 = 1016.e T
upon whether reaction is exothermic or endothermic
If reaction is exothermic, 1000
-
DH = - ve Ea(b) > Ea(f ) and k2 = 1015.e T

If reaction is endothermic when k and k are equal at any temperature T, we


DH = + ve E a(b) < E
EBD_7504
S-C-44 SOLUTIONS

2000 1000 When concentrations of both A and B are doubled,


- -
10 .e16 T 15
= 10 .e T rate increases by 8 times hence order with respect to B
is 2.
-
2000
-
1000 \ rate = k [A]1 [B]2
15 15
or 10 ´ 10 .e T
= 10 .e T
25. (b) 2 N 2O 5 ® 4 NO 2 + O 2
-
2000
-
1000 From the unit of rate constant it is clear that the reaction
or 10.e T = e T follow first order kinetics. Hence
by rate law equation, r = k [N2O5]
2000 1000 wherer = 1.02 × 10–4, k = 3.4 × 10–5
or ln 10 - =-
T T 1.02 × 10–4 = 3.4 × 10–5 [N2O5]
[N2O5] = 3M
2000 1000 26. (c) Third order
or ln 10 = -
T T 27. (a) If we write rate of reaction in terms of concentration of
NH3 and H2,then
1000
or 2.303 log 10 = 1 d[ NH 3 ] 1 d[H 2 ]
T Rate of reaction = =-
2 dt 3 dt
or 2.303 ×1×T=1000 [\ log 10= 1]
d[ NH 3 ] 2 d[H 2 ]
So, =-
1000 dt 3 dt
or T = K
2.303
28. (a) E a ( F.R .) ¹ E a (B.R .) Ea can be calculated.
22. (b)
23. (a) Arrhenius equation is given by 29. (b) For reaction 3A ¾ ¾® B + C
k = Ae - Ea /(2.303 RT ) If it is zero order reaction r = k [A]0, i.e the rate remains
same at any concentration of 'A'. i.e independent upon
Taking log on both sides, we get
concentration of A.
Ea
log k = log A –
2.303 RT 2.303 a 2.303 1 2.303
30. (d) k= log = log = log 8
1 t a-x 24 1 24
Arrhenius plot a graph between log k and whose 8
T
- Ea
slope is .
2.303 R
24. (a) When concentration of A is doubled, rate is doubled.
Hence order with respect to A is one.
Speed Test-48

1. (a) Smaller the charge on anion, lesser will be its


x x 1
coagulating power. = kP1/ n or log = log k + log P;
\ KBr have Br– with least charge of – 1 on Br thus m m n
KBr is least effective in coagulating Fe(OH)3.
2. (c)
3. (b) x/m
Ps
4. (d)
5. (b) According to Freundlich equation. P
x x x 1
µ p1/ n or = kp1/ n plot of log vs log P is linear with slope =
m m m n
x x 1 19. (d) Tannin used in leather industry contains negatively
or log = log kp1/ n or log = log k + log p charged colloidal particles.
m m n
20. (b) When oppositely charged sols are mixed their charges
6. (b) 1 micron m = 10–6 m and 1 millimicron = 10–7 m.
are neutralised. Both sols may be partially or completely
7. (d) All are irreversible colloidal systems. precipitated.
8. (d) Cetyl trimethyl ammonium bromide, 21. (b)
+ - 22. (d)
[C16 H 33 (CH 3 )3 N Br ] is a cationic micelle
23. (a) According to Langmuir's model of adsorption of a gas
on a soild surface the mass of gas adsorbed(x)per gram
9. (a) o- Polarhead of the adsorbent (m) is directly proportional to the
n- Non-polar tail pressure of the gas (p) at constant temperature i.e.
(micelle) x
µp
m
10. (c) It is due to impact of molecules of dispersion medium on 24. (c) The more the liquefiable nature of a gas, the more is the
the colloidal particles. enthalpy of adsorption. Water is more liquefiable.
11. (c) 25. (a) A catalyst speeds up both forward and backward
12. (c) The movement towards anode shows that sol is negative. reaction with the same rate.
For coagulation of negative sol. Cation with higher So, equilibrium constant is not affected by the presence
charge is more effective. of a catalyst at any given temperature.
13. (d) Fe(OH)3 is positive sol. K3[Fe(CN)6] will provide 26. (c) As Sb2S3 is a negative sol, so Al2(SO4)3 will be the
[Fe(CN)6]3– for coagulation. most effective coagulant due to higher positive charge
14. (c) Alum. coagulates mud particles and helps in purifying on Al (Al3+) – Hardy-Schulze rule.
water. 27. (d)
15. (a) On increasing temperature adsorption of a gas on 28. (d) At high pressure the extent of adsorption follows zero
surface of solid decreases. Solid adsorb greater amount order kinetics.
of substances at lower temperature. 29. (c) The critical micelle concentration is the lowest
16. (c) For physiosorption the D H value is 10 - 40 kJ/mol. concentration at which micelle formation appears.
When surfactants are present above that CMC, they
17. (a) On shaking with the dispersion medium, colloids can act as emulsifiers that will solubilise a compound
directly form the colloidal sol. Hence they are called which is normally insoluble in the solvent being used.
intrinsic colloids. i.e., glue.
30. (b) Peptisation is disintegration of colloidal aggregate.
18. (d) According to Freundlich adsorption isotherm.
At intermediate pressure, extent of adsorption
EBD_7504
Speed Test-49

1. (a) The conversion of metal sulphide to metal oxide involves 18. (a) Cuprous oxide formed during roasting of cuprous
the process of roasting (i.e., x is roasting). sulphide is mixed with few amount of cuprous sulphide
The metal oxides can then be converted to impure metal and heated in a reverberatory furnace to get metallic
by reduction. Of the given choices in (a) and (b) the copper.
reduction process is that of smelting. (i.e., ‘y’ is smelting)
The conversion of impure metal to pure metal involves 2Cu 2O + Cu 2S ® 6Cu + SO2
a process of purification. Thus it is electrolysis. 19. (a) Zone refining is based on the difference in solubility of
2. (b) impurities in molten and solid state of the metal. This
3. (c) method is used for obtaining metals of very high purity.
4. (c) 20. (b) The reactions involved in cyanide extraction process
5. (d) are :
6. (b) Ag 2 S + 4NaCN ® 2Na [Ag(CN)2] + Na2S
7. (a) Dalomite on calcination gives CaO, MgO which (argentite)
provides basic lining in furnace. 4Na2S + 5O 2 + 2H2O ® 2Na2SO4 + 4NaOH + 2S
8. (c) Oxiding
agent
9. (c) Zn being more reactive than Ag and Au, displaces them.
2Na[Ag(CN)2] + Zn ® Na2 [Zn(CN)4] + 2 Ag ¯
Leaching
4Ag + 8NaCN + 2H2O + O2 ¾¾¾¾® (reducing
agent)
4Na[Ag(CN)2] + 4NaOH
21. (b) Al2O3 cannot be reduced by carbon.
Soluble Sodium dicyanoargentate (I)
22. (d)
Soluble cyanide compound can be treated with Zn to 23. (b) The given reaction is the method named as Van Arkel
give metal by displacement. for the purification of titanium
2Na[Ag(CN)2] + Zn ¾¾¾¾¾¾ Displacement 24. (c) Ca and K are strong reducing agents, hence their oxides
®
Na2[Zn(CN)4] + 2Ag¯ cannot be reduced with carbon
10. (d) Decomposition of carbonates and hydrated oxides. 25 (b) Zr and Ti are purified by van Arkel method.
11. (c)
Zr(s) + 2I 2 (g) ¾¾
® ZrI 4 (g)
12. (b) Pig iron or cast iron contains 3 – 5% carbon and varying
amounts of Mn, Si, P and S which makes the iron hard On the hot
ZrI4 (g) ¾¾¾ ¾¾
® Zr(s) + 2I 2 (g)
and brittle. filament

13. (b) Cuprite is Cu2O and Argentite is Ag 2 S . 26. (c) (iii) and (iv) reactions occur in the temperature range of
900 – 1500K in blast furnace.
14. (b) Cyanide process is for gold (I-D); floatation process -
27. (d) Zincite is ZnO.
pine oil (II-B); Electrolytic reduction - Al (III-C); Zone
28. (a) FeO is capable forming slag with SiO2
refining -Ge (IV-A).
15. (b) For a spontaneous reaction , D Gº must be negative SiO 2 + FeO ® FeSiO 3
and it can be possible only in this case when x < y 29. (d)
16. (c) Fused alumina (Al2O3) is a bad conductor of electricity. 30. (b) The two equation are:
Therefore, cryolite (Na3AlF6) and fluorspar (CaF2) are 4 2
Al(s) + O 2 (g) ® Al2 O3 (s), D f Gº = -827kJ mol -1
added to purified alumina which not only make alumina 3 3
a good conductor of electricity but also reduce the … (1)
melting point of the mixture to around 1140 K. 4 2
Cr(s) + O 2 (g) ® Cr2 O3 (s), D f Gº = - 540kJ mol -1
17. (d) The sulphide ore is roasted to oxide before reduction 3 3
because the DGof of most of the sulphides are greater … (2)
Subtracting equation (ii) from equation (i) we have,
than those of CS2 and H2S, therefore neither C nor H
4 2
can reduce metal sulphide to metal. Further, the standard Al(s) + Cr2 O 3 (s),
free energies of formation of oxide are much less than 3 3
those of SO2 D r G° = - 287kJ mol -1
Speed Test-50

1. (a) Hypophosphorous acid is H3PO2 in which O.S. of P is Oxidation


+1
2. (b) The bond angle decreases on moving down the group - + -1 +5 redution
3HO Cl ¾¾® 2HCl+ HClO 3
due to decrease in bond pair-bond pair repulsion.
20. (b) XeOF4 is square pyramidal.
NH3 PH3 AsH3 SbH3 BiH3
107º 94º 92º 91º 90º 21. (c) Yellow ammonium sulphide is ( NH 4 ) 2 S x
This can also be explained by the fact that as the size 22. (c) The correct order of increasing bond angle is
of central atom increases sp3 hybrid orbital becomes Cl2 O < ClO 2- < ClO 2
more distinct with increasing size of central atom i.e. Cl2O 2 ClO2 ClO2
2
2

pure p-orbitals are utilized in M–H bonding. .


O . . Cl . .

..
..
3. (b) CuSO 4 + 4NH 3 ® [Cu( NH 3 ) 4 ] SO 4 Cl

Blue complex due to Cu(NH3 )2 + . . 110° . .


Cl
. . 118°
O
4 Cl .O. O O

..

..
..

..
..
..

..
.. .. ..

..
..

In ClO2 there are 2 lone pairs of electrons present on
4. (a)
5. (c) The nitroprusside ion is [Fe(CN)6NO+]2–. The magnetic the central chlorine atom. Therefore the bond angle in
moment measurements reveal the presence of 4 ClO2– is less than 118° which is the bond angle in ClO2
unpaired electrons in Fe which must be then in Fe++ which has less number of electrons on central chlorine
(3d6) and not Fe+++(3d5) atom.
6. (c) [Fe(H2O)5NO]2+ ion is formed 23. (d) The products of the concerned reaction react each
7. (d) other forming back the reactants.
8. (a) 4HCl + O2 ® 2Cl2 + 2H2O XeF6 + 3H 2 O ¾¾ ® XeO3 + 6HF .
air cloud of white fumes 24. (b) The hybridization of XeO3F2 is sp3d and its structure
1 is trigonal bipyramidal in which oxygen atoms are
9. (a) ICl7. The hybridisation is (7 + 7 + 0 - 0) = 7 (sp3d3 ) situated on the plane and the fluoride atoms are on the
2 top and bottom.
10. (a) Due to large enthalpy of vaporisation SO2 can be used
25. (d)
as refrigerant
11. (c) Oxides which are more ionic in nature (salt - like) are 26. (a) 2 S2 O 3-2 + I 2 ® S 4O 6-2 + 2I -
known as saline oxides e.g. oxides of alkali metals Tetrathion ate
..
12. (b) Among the given compounds, the NH 3 is most ba- 27. (c) FeSO 4 . 7H 2 O ® FeSO 4 + 7H 2 O
D
sic. Hence has highest proton affinity 2FeSO 4 ¾¾® Fe 2 O 3 + SO 2 + SO 3
13. (c) Helium is heavier than hydrogen although it is non- 28. (d) HO Cl < HO Cl O < HO Cl O 2 < HO Cl O3
inflammable +1 +3 +5 +7
14. (c) It is H2SO5. In case of oxyacids of similar element as the oxidation
number of the central atom increases, strength of acid
OH
| also increases.
15. (b) HO - P - H it can form two series of salts by 29. (b) The molecular geometry of BrF5 is square pyramidal
||
O with asymmetric charge distribution on the central atom.
replacement of H attached to oxygen F F
16. (b) RCOO– is not pseudo halide.
Br
17. (d) In gaseous state the HCl is covalent in nature while in
aqueous solution it ionises to give H+ and C l ions F F
F
H
| O
O
=

18. (c) Hypophosphorous acid H - O - P ® O 30. (d) NºN ® O N–N =


|
=

H (i) (ii) O
Two H-atoms are attached to P atom. O O
O O
®
=

19. (b) During disproportionation same compound undergo ®


=

O
EBD_7504
S-C-48 SOLUTIONS

Speed Test-51

1. (d) (n – 1)d 5ns2 attains the maximum O.S. of + 7. 1


2. (b) Mn2+ (3d5) is more stable them Mn3+ (3d4). 15. (c) AgNO 3 ® Ag + NO 2 + O2
2
3. (c)
4. (d) Super conductors are derived from compounds of 16. (a) 2AgNO3 + Na 2S2O3 ® Ag 2S2O3 + 2NaNO3
transition metals 17. (b) Oxidation of water takes place in presence of Mn in
biological process
5. (b) Mn - 3d 5 4s2
18. (a)
The no. of various oxidation states possible are + 2, (a) V = 3d 3 4s 2 ; V2+ = 3d 3 = 3 unpaired electrons
+ 3, + 4, + 5, + 6 and + 7.
6. (c) The transition metals and their compounds are used as Cr = 3d 5 4s 1 ; Cr 2+ = 3d 4 = 4 unpaired electrons
catalysts. Because of the variable oxidation states they Mn = 3d 5 4s2 ; Mn2+ = 3d 5 = 5 unpaired electrons
may form intermediate compound with one of the Fe = 3d 6 4s 2 ; Fe2+ = 3d 6 = 4 unpaired electrons
reactants. These intermediate provides a new path with
Hence the correct order of paramagnetic behaviour
lowe activation energy. V2O5 + SO2 ® V2O4 + SO3
2V2O4+ O2 ® 2V2O5 V2+ < Cr 2+ = Fe2+ < Mn2+
7. (a) According to their positions in the periods, these values (b) For the same oxidation state, the ionic radii generally
are in the order: decreases as the atomic number increases in a
Yb+3 < Pm+3 < Ce+3 < La+3 particular transition series. hence the order is
At. Nos. 70 61 58 57 Mn++ > Fe++ > Co++ > Ni++
This is due to lanthanide contraction. (c) In solution, the stability of the compound depends upon
8. (a) When a solution of potassium chromate is treated with electrode potentials, SEP of the transitions metal ions
an excess of dilute nitric acid. Potassium dichromate are given as
and H2O are formed.
Co3+ / Co = + 1.97, Fe3+ / Fe = + 0.77 ;
2K2CrO4 + 2HNO3 –® K2Cr2O7 + 2KNO3 + H2O
Hence Cr2O7– and H2O are formed. Cr3+ / Cr2+ = – 0.41, Sc 3+ is highly stable as it does not
9. (b) Mn2+ (d5)is more stable than Mn3+ (d4), thus show + 2 O. S.
(d) Sc – (+ 2), (+ 3)
E° 3+ 2+ = + ve
Mn /Mn Ti – (+ 2), (+ 3), (+ 4)
+3 – +7 –
Cr – (+ 1), (+ 2), (+ 3), (+ 4), (+ 5), (+ 6)
– 4e
10. (c) Mn2O3 ¬¾ ¾¾ [KMnO 4 ] ¾–¾
e
¾® [MnO4 ]–1 Mn – (+ 2), (+ 3), (+ 4), (+ 5), (+ 6), (+ 7)
– –
– 3e
i.e. Sc < Ti < Cr = Mn
– 5e
+4 19. (b) In lanthanides, there is poorer shielding of 5d electrons
2+
Mn MnO2 by 4 f electrons resulting in greater attraction of the
nucleus over 5d electrons and contraction of the atomic
11. (d) O.N. of Cr in all the species is + 6. Configuration of Cr radii.
(VI) is (Ar) 3d 0 . Hence no d-d electron transition. 20. (d)
12. (c) The number of unpaired electrons in Ni2+(aq) = 2 Eu La Gd Am
Water is weak ligand hence no pairing will take place O.S +2 +3 +3 +3,+4,+5,+6
spin magnetic moment 21. (a)

= n(n + 2) = 2(2+ 2) 2KMnO 4 + H 2SO 4 (Conc ) ® K 2SO 4 + Mn 2 O 7 + H 2O


Explosive
= 8 = 2.82 22. (c)
13. (d) 2Cu 2 O + Cu 2S ¾
¾® 6Cu + SO 2
AgCl ( s ) + 2NH 4 OH ( aq ) ® éë Ag ( NH 3 )2 ùû Cl ( aq ) + 2H 2O ( l )
self reduction.
23. (a) Paramagnetic moment is directly proportional to number
14. (b) Eu 2 + has electronic configuration [ Xe] 4f 7 hence
of unpaired electrons present in the complex.
Chemistry S-C-49

24. (d) d 5 –––– strong ligand field 26. (c) Parke’s process It is for extraction of silver
27. (a) Cu, Hg and Ag are attacked by conc acids but gold is
not attacked. AuCl3 forms H [AuCl 4 ] a complex with
t2g eg
HCl which is used for tonning in photography.
.. +
m = n n + 2 = 3 = 1.73BM 28. (a) N H3 + H + (acid medium) N H4
d 3–– in weak as well as in strong field
29. (a) When KI is added to mercuric iodide it disssolve in it
and form complex.
t2g eg HgI 2 ∗ KI ® K 2 [HgI 4 ]
red, solid (so lub le)
m = 3(5) = 15 = 3.87 B.M. (inso lub le)

d 4– in weak ligand field On heating HgI2 decomposes as


HgI 2 ƒ Hg + I 2
(violet vapours)
t2g eg
30. (c) Eo = 0.34 V
Cu +2 / Cu
m = 4(8) = 24 = 4.89
d 4– in strong ligand field other has – ve E oR.P.

Eo
Co + + / Co = – 0.28 V
t2g eg
Eo = – 0.25V
Ni + + / Ni
m = 2(4) = 8 = 2.82.
Eo = – 0.44V
25. (b) In neutral or faintly alkaline medium thiosulphate is Fe ++ / Fe
quantitatively oxidized by KMnO4 to SO42–
8KMnO4 + 3Na2S2O3 + H2O ¾®
3K2SO4 + 8MnO2 + 3Na2SO4 + 2KOH
EBD_7504
S-C-50 SOLUTIONS

Speed Test-52

1. (b) According to spectrochemical series C2 O24 - > F- . \ No. of moles of AgCl obtained
= 3 × No. of moles of complex
2. (b) CN– is coordinated to cobalt as the ligand and
coordinated compounds have different properties 0.03
\n= =3
than the individual species. 0.01
3. (d) 13. (b) The chemical formula of Pentaminenitrochromium (III)
4. (d) [Ni X4]2–, the electronic configuration of Ni 2+ is chloride is
éCr ( NH3 ) NO2 ù Cl2
ë 5 û
It can exist in following two structures
é Cr ( NH 3 ) NO2 ù Cl2 and
ë 5 û
It contains two unpaired electrons and the hybridisation
is sp3 (tetrahedral). é Cr ( NH3 ) ONO ù Cl 2
ë 5 û
5. (b) The crystal field splitting in tetrahedral complexes is Therefore the type of isomerism found in this
lower than that in octahedral complexes, and compound is linkage isomerism as nitro group is linked
4
D t = - D0 . through N as –NO2 or through O as – ONO.
9
14. (c) Nickel ions are frequently detected by the formation of
6. (b)
red precipitate of the complex of nickel
7. (a) [Co(NH3)5CO3]ClO4. Six monodentate ligands are dimethylglyoxime, when heated with dimethylglyoxime.
attached to Co hence C. N. of Co = 6;
CH3 C NOH
O. N. = x + 5 × (0) + 1 × (–2) + 1× (–1) = 0 \ x = + 3 ; + Ni++
electronic configuration of Co3+[Ar] 3d64s0 hence CH3 C NOH
number of d electrons is 6. All d electrons are paired Dimethylglyoxime
due to strong ligand hence unpaired electron is zero.
8. (a) EDTA has hexadentate four donor O atoms and 2 donor OH O
N atoms and for the formation of octahedral complex CH3 C N N C CH3
one molecule is required Ni
9. (c) [Mn(CN) 6 ]3– and [Fe(CN) 6 ] 3– are inner orbital CH3 C N N C CH3
complexes and paramagnetic while [Co(C2O4)3]3– is
O OH
diamagnetic in nature.
10. (a) Ti(C2H4)4 is an organometallic compound due to Ti Nickel dimethylglyoxime
directly attached to C- atom
15. (d) [Cr(H2O)4Cl2] Cl + AgNO3 ¾¾ ® AgCl +
11. (d) Tetraethyl lead Pb(C2H5)4 is not p bonded complex. It
is s bonded organometallic compound [Cr(H2O)4Cl2]NO3
12. (a) ® xCl-
CoCl3.6NH3 ¾¾ Molarity =
wt
´
1000
2.675g mol. mass vol.
wt vol. 0.01 ´100
xCl - + AgNO3 ¾¾
® x AgCl ¯ = molarity ´ =
4.78g mol. mass 1000 1000
Number of moles of the complex = 0.001
16. (c) Gd (64) = [Xe] 4f 7 5d1 6s2
2.675
= = 0.01 moles Gd3+ = [Xe] 4f 7 5d0 6s0
267.5
i.e. no. of unpaired electrons = 7
Number of moles of AgCl obtained
m = n(n + 2) = 7(7 + 2)
4.78
= = 0.03 moles
143.5
Chemistry S-C-51

17. (b) In the above paramagnetic state the geometry of the


18. (d) isomers complex is sp3 giving tetrahedral geometry.
The diamagnetic state is achieved by pairing of
[Ru(NH3 ) 4 Cl2 ]+ ,[Co(NH3 )5 Cl]2 + , electrons in 3d orbital.
cis and trans none
3d 4s 4p
2+ +
[Ir(PR 3 ) 2 H(CO)] , [Co(en ) 2 Cl 2 ]
cis and trans cis and trans and
optical isomers
dsp2
19. (d) Due to some backbonding by sidewise overlapping
between d-orbitals of metal and p-orbital of carbon, Thus the geometry of the complex will be dsp2 giving
the F–C bond has s and p character. square planar geometry.
20. (c) The correct structure of EDTA is
4+
HOOC–H2C
N – CH2 – CH2 – N
CH2–COOH 25. (b) é Pt ( NH3 ) ù Cl 4 ƒ é Pt ( NH3 ) ù + 4Cl- ;
ë 6û ë 6û
HOOC–H2C CH2–COOH
21. (a) [PdCl4]2– is dsp2 hybridized and square planar in shape. -
Ag + + C l ® AgCl (ppt)
22. (b) B 26. (b) The correct IUPAC Name of [Mn(CN)5]2–.
V G Pentacyanomangnate (III).
27. (a) [NiL4]2–

3d 4s 4p
R Y Ni 2+ ( d 8 )
O
For a given metal ion, weak field ligands create a complex
[NiL4 ]2 –
with smaller D, which will absorbs light of longer l and
thus lower frequency. Conservely, stronger field ligands sp3
create a larger D, absorb light of shorter l and thus
higher v i.e. higher energy. i.e, no. of unpaired electron = 2
Red < Yellow < Green < Blue hybridization – sp3.
l = 650 nm 570 nm 490 nm 450 nm 28. (b)
So order of ligand strength is 29. (c) In [Co(CO)5NH3]2+ CO is bonded to cobalt through
L1 < L3 < L2 < L4 s and p bonding.
23. (d) 30. (d) K2[Ni(EDTA)]. Since EDTA is hexadentate and
24. (c) In both states (paramagnetic and diamagnetic) of the chelating and coordinates from six sides forming more
given complex, Ni exists as Ni2+ whose electronic stable complex.
configuration is [Ar] 3d 84s0.
3d 4s 4p
Ni2+ :

sp3
EBD_7504
S-C-52 SOLUTIONS

Speed Test-53

1. (c) 12. (c) CH3OH does not undergo iodoform reaction, all others
+
do so.

NH2 N º NCl Cl CH3
13. (b) CH C = O + CH3CH2MgBr
3
HNO2 Cu2Cl2
CH3 OMgBr
HCl Sandmeyer’s reaction C
(Diazotization) CH3 CH2CH3
H 2O
2. (b)
3. (a) CH3 OH
4. (a) The reaction is dehydrohalogenation CH3 C
CH2CH3
Br 2- methyl butan-2ol
(3° alcohol)
C6 H 5
alc. KOH
C6 H 5
CH3 CH3
1-phenyl cyclopentene
14. (a) CH3 – C – Cl + 2Na + Cl – C – CH3
5. (a) Only 1° alkyl halides (i.e. CH3Br) undergo SN2 reaction. CH3
CH3
6. (a) t-Butyl chloride
7. (d) –Cl is o, p–directing.

Wurtz
Rxn
8. (b)
CH2Cl CH2CN CH3 CH3 CH3 CH3
NaCN
¾¾ ¾
¾® CH3 – C – C –CH2Cl CH – C – C – CH3
DM F Mono 3
CH3CH3 Chlorination CH3 CH3
I I
Nuclear substitution will not take place. 15. (b)
9. (b) The compound has two similar assymmetric CH3 CH3 CH3
C-atoms. It has plane of symmetry and exist in meso
form. Cl2 Cl
16. (c) ¾¾¾¾ ® +
FeCl3

Plane of symmetry
Cl
CH3 CH2Cl CHCl2
Cl2 ,hv Cl2 ,hv
¾¾¾¾ ® ¾¾¾¾ ®
Meso - 2, 3 dichlorobutane
10. (d)
CCl3
OCH3 OCH3
Cl2 , hv
Br – ¾¾¾¾ ¾®
NaNH2 NH 2
11. (d)
(–HBr)
17. (c) The acid character follows the order :
Benzyne intermediate CH3COOH > C6H5OH > H2O > CH3OH
The basic character will follow the order
OCH OCH3 OCH3 OCH3 CH3COO– < C6H5O– < O–H < CH3O–
The stronger the acid, the weaker the conjugate base
NH2
NH3 formed.
+ – 18. (b) Reaction between alkyl halide, aryl halide and sodium
known as Wurtz fitting reaction
Chemistry S-C-53

C6H5Cl + 2Na + ClCH3 ¾® C6 H5CH3 + 2NaCl CH 3


Toluene
Anhyd.
|
25. (d) C6 H 6 + CH3CH 2 CH 2Cl ¾¾¾¾ ® C6 H 5 - CH - CH 3
Cl AlCl3 Isopropyl benzene
|
19. (c) 2Cl - C - CH3 + 6Ag ¾¾
® CH3C º CCH3 + 6AgCl 26. (d) –NO2 group withdraws electrons from o- and p-
| 2 - butyne
positions and hence activates the –Cl present on that
Cl
position towards nucleophilic substitution.
1, 1, 1-trichloroethane
Hunsdiecke r reaction
20. (b) Compound which are mirror image of each other and 27. (d) C 6 H 5 COOAg + Br2 ¾¾ ¾¾ ¾¾ ¾¾®
are non superimposable are termed as enantiomers.
CH3 CH3 C 6 H 5 Br + CO 2 + AgBr
H OH HO H
and Cl
HO H H OH N(CH 3 ) 2
CH3 ( CH ) NH
CH3 28. (a) + (CH 3 ) 2 NLi ¾¾¾
3 ¾
¾® + LiCl
These are enantiomers
21. (d) Iodoform test is given by methyl ketones, acetaldehyde
and methyl secondary alcohols. heat
29. (d) Cl3C - CH 2 CH 3 + KOH ¾¾¾
®
CH3 CH CH2 OH
(OH)3 C - CH 2CH3 + 3KCl
CH3
Isobutyl alcohol is a primary alcohol hence does'nt
give positive iodoform test.

Alc.KOH
22. (b) BrCH2 – CH2Br ¾¾¾¾
- HBr
¾
® CH2 = CHBr O
||
NaNH
¾¾¾¾ 2®
CH º CH CH3CH 2 C - OH
- HBr
Elimination of HBr from CH2 = CHBr requires a stronger 30. (a) A strong nucleophile favours the SN2 reaction and a
base because here, Br acquires partial double bond weak nucleophile favours the SN1 reaction.
character due to resonance. First reaction is SN1 reaction because C2H5OH is used
23. (c) C6H5CHCH3Br being an optically active secondary as solvent which is a weak nucleophile.
alkyl bromide undergoes SN2 nucleophilic substitution Second reaction is SN2 reaction because C2H5O– is
reaction. Hence it undergoes complete inversion of strong nucleophile.
configuration.
H H

C6H5 – DMF
¾¾¾® Cl — C C6H5
C — Br + Cl

H3C CH3
24. (c) Chloral on reaction with chlorobenzene in the presence
of a catalytic amount of sulphuric acid forms DDT
(dichlorodiphenyl Trichloro ethane).

H H Cl
Cl3C – C = O +
Trichloro H Cl
acetaldehyde

Cl
Cl3C – CH
EBD_7504
S-C-54 SOLUTIONS

Speed Test-54

1. (a) Lucas reagent is conc. HCl + anhyd. ZnCl2. +


2. (c) ˆˆ† (CH3 )3 C+ + H 2 O
Step 2 : (CH3 )3 C - OH 2 ‡ˆˆ
3. (a) Methyl alcohol (CH3OH) is also known as carbinol. Hence 3° Carbocation
vinyl carbinol is CH2 = CH – CH2OH
4. (a) 3-methyl pentanol-3 will be dehydrated most readily ˆˆ† (CH 3 )3 C - Cl
Step 3 : (CH3 )3 C+ + Cl- ‡ˆˆ
since it produces tertiary carbonium ion as intermediate. t -Butyl chloride
10. (d) The two components should be (CH3 )3 CONa +
CH3 (CH3)3CBr. However, tert-alkyl halides tend to undergo
| elimination reaction rather than substitution leading to
CH3 - CH 2 - C - CH 2 - CH3 the formation of an alkene, Me2C = CH2
|
OH 11. (d)
12. (d)
CH3 distill
(C 2 H 5O) 2 Mg + H 2 O ¾¾¾® 2C 2 H 5 OH + Mg (OH ) 2
+ |
H
¾¾¾
® CH3 - CH 2 - C - CH 2 - CH 3 (distillate ) (residue )
Å
13. (a) MnO2 being a mild oxidising agent stops the
5. (c)
oxidation of – CH2OH group at aldehyde stage.
O
14. (b) The tertiary alkyl halide undergo elimination reaction
6. (c) CH CHCH2Cl to give alkenes
2
1 2 3

7. (d) 2-Phenylethanol, C6 H5CH 2CH 2 OH , is a 1º alcohol CH3


which can be prepared from C6H5MgBr by treating with ½
ethylene oxide (note that HCHO will introduce only one CH3 —C—X + NaOC2H5 ¾®
carbon atom, i.e. it will give C6H5 CH2OH and not ½
CH3
C6H5CH2CH2OH).
O
C 6 H 5MgBr + ¾ ¾® C 6 H 5 CH 2 CH 2 OH ;
CH3
C 6 H 5 MgBr + HCHO ¾
¾® C 6 H 5 CH 2 OH ½
CH3 —C== CH 2
8. (b) Methyl vinyl ether under anhydrous condition at room 2-methyl propene
temperature undergoes addition reaction.
HBr
OH OH OH
CH 2= CH - OCH3 ¾¾¾
® CH3 - CH - O - CH3
| CH3 COOH
Br 15. (d)
9. (b) Tertiary alcohols react fastest with conc. HCl and o-cresol Salicylic acid Phenol
anhydrous ZnCl2 (lucas reagent) as its mechanism
proceeds through the formation of stable tertiary Electron releasing groups (–CH3, –OCH3, –NCH3 etc)
carbocation. intensify the negative charge of phenoxide ion, i.e.,
destablises it hence decrease ionization of parent
Mechanism
phenol. Therefore decreases acidity while electron
donating groups (–NO2, –COOH, –CHO etc.) increases
CH3 acidity.
|
Step 1 : CH3 — C — OH + H - Cl 16. (b) Number of active hydrogen in a compound corresponds
| to the number of moles of CH4 evolved per mole of the
CH3 compound.
2 Methyl Propan-2-ol CH MgI
- NH 2 , - SH, - OH or - C º CH ¾¾¾¾®
3
CH 4 (2CH 4 from
+
ˆˆ† (CH ) C — OH + Cl- (2CH 4 from - NH 2 )
Chemistry S-C-55

17. (b) 23. (b)


18. (b) 24. (b) The commercial alcohol is made unfit for drinking by
mixing in it some copper sulphate (to give it colour)
ONa and pyridine (a foul smelling liquid). It is known as
19. (a) + CO2 ¾¾
® denaturation of alcohol.
Oxidation
Sodium 25. (b) CH3CH 2 CH 2CH 2OH¾¾¾¾¾ ® CH3CH 2 .CH 2CHO
Phenoxide

OH OH O OH OCH 3
H SO
¾¾¾¾
2 4® CH3OH | |
26. (a) CH 2 C - CH 3 ¾¾¾¾ ® H 2 C - C - CH 3
COONa H 2SO4
COOH | |
Salicylic acid CH 3 CH 3
27. (c) Electron withdrawing group stabilises the benzene ring
(CH 3 CO) 2 O due to delocalisation of charge.
– CH3 and – CH2OH are electron donating group and
hence decrease the stability of benzene ring – OCH3
O is weaker electron withdrawing group than – COCH3.
||
O - C - CH3 Hence – COCH3 group more stabilize the phenoxide
ion at p-position.
+ CH3COOH 28. (d) This method is suitable for the preparation of a wide
COOH variety of unsymmetrical ethers. The nucleophilic
Aspirin substitution of halides with alkoxide leads to desired
(Acetyl Salicylate) product.
29. (c) Due to presence of methyl alcohol in liquor.
20. (b) C2 H 5 Br + C 2 H 5ONa ¾¾¾¾
® C2 H5 - O - C2H 5
- NaBr
Sod. ethoxide diethyl ether 30. (d) CH 3 - CH 2 - CH - CH - CH 2 - CH 3 ¾HIO
¾¾ 4®
21. (b) | |
OH OH
CH3 COOH
HNO3
22. (c) 2 CH 3 - CH 2 - CHO
(Oxidation)
CH3 COOH
o-Xylene

CO
Phenol
O Phenolpthalein
H2SO4
CO (Y)
Phthalic anhydride, X
EBD_7504
S-C-56 SOLUTIONS

Speed Test-55

1. (c)
2. (a) Ketones on oxidation give carboxylic acids with lesser O O O
number of carbon atoms, i.e., C C C
O O O
[O]
CH 3 COCH 3 ¾¾
¾® CH 3 COOH
O
+CO 2 + H 2O C–CH3 CH2–CH3
3. (b)
9. (b) Zn-Hg/HCl
H CH3 H O ¾¾ ¾ ¾¾®
CH 3
O3
CH3– CH 2– C C CH 3– CH 2– C– C
CH3 Phenyl methyl Ethyl
CH3 O O ketone benzene
4. (a) O O
CH3– C – CH 3+ CH3 – CH 2 – CHO (–H 2O) This reaction is known as Clemmensen's reduction.
5. (c) Cannizzaro reaction - when an aldehyde containing no 10. (a)
a – H undergo reaction in presence of 50% KOH. It
disproportionates to form a molecule of carboxylic acid COCH3
and a molecule of alcohol. C6H6
O CH3COOH +PCl5 ® CH3COCl AnhydAlCl
3
[A]
CH=O CH2–OH Friedel Craft
C–O
reaction
2 50% KOH + OH OMgBr
H+ MgBrC2H 5
Cl C2H5 – C – CH3 C2H5 – C – CH3
Cl Cl ether
hydrolysis

OH O
6. (d) (C)
O2N NO 2 HN NH
11. (d) Since, C when heated with Br2 in presence of KOH
O O produces ethylamine, hence it must be propanamide
NO2 and hence the organic compound (A) will be propanoic
(Picric acid) (Barbituric acid) acid. The reactions follows.
NH
CH3 - CH 2 - COOH ¾¾¾
3
®
(A)
HO
- + D
O O COOH O CH3 - CH 2 - CO O N H 4 ¾¾
®
HO (B)
O C CH3
OH KOH + Br
HO CH 3 - CH 2 - CONH 2 ¾¾¾¾¾
2
®
Hoffmann
(Aspirin) (C) bromamide
(Ascorbic acid) reaction

7. (d) It is Clemmensen’s reduction CH 3 - CH 2 - NH 2


O (Ethylamine )
|| Zn - Hg
CH3 - C - CH 2 - CH3 Conc. HCl CH 3 CH 2 - CH 2 CH 3
OH
8. (c) Due to resonance in carboxylate ion, the double bond 12. (d) (a) 2C6H5CHO KCN
KCN
C6H5–CH–C–C6H5
character of C = O bond in carboxylic acids is greatly , CC2HH5OH
H OO OH
O
(Benzoin)
Chemistry S-C-57

O O PhCOPh < Ph.COCH3 < CH3COCH3 < HCHO


(b)
H
+ The lower reactivity of ketones is due to presence of
O O two alkyl group which shows +I effect. The reactivity
of ketones decreases as the size of alkyl group
H O H increases.

D COONa
20. (b) 2HCOONa ¾¾® | + H2
360°
COONa
OH OH OH OH
sod.oxalate
Phenolphthalein
(c)
21. (b) EtONa
¾¾ ¾
¾®

O O
|| - ||
( CH3 )2 CH CH2 Br
CH3 - C - CH - C - OEt ¾¾¾ ¾¾¾¾ ¾®
Na+
K RT
70
>3 H O H
Fries rearrangement
O O
|| || Ketonic
CH 3 - C - CH C OEt ¾¾¾¾®
hydrolysis
|
CH 2 - CH - CH 3
|
COOCH3 CH 3
OH O CH3
(d) (Oil of wintergreen) || |
C H3 - C - CH 2-C H 2- C H -C H 3 + CO 2 + EtOH
1 2 3 4 5 6
OH
13. (b) NaBH4 selectively reduces the aldehyde group to |
alcohol without affecting double bond in a organic 22. (b) CH 3 - CH 2 - C - Cl ¾
¾®
compound. So, X is NaBH4. |
OCH 3
NaBH
C6 H5 CH = CHCHO ¾¾¾¾
4 ® C H CH = CHCH OH
6 5 2 O
||
O CH 3 - CH 2 - C - OCH 3 + HCl
( -H O )
14. (b) CH3–C=O + H2N–NH–C–NH2 ¾¾ ¾
2 ¾®
23. (a) The correct order of increasing acid strength
O
H (Me)2CH.COOH < CH3COOH < MeOCH2COOH
CH3–CH=N–NH–C–NH2
acetaldehyde semicarbazone < CF3COOH
Electron withdrawing groups increase the acid strength
15. (a)
and electron donating groups decrease the acid
COOH COOC2H5 strength.
24. (d)
HCl
16. (c) + C2H5OH + H2O 25. (b) Acetic acid does not have –CHO grouping, while all
Ethanol
dry
others (HCOOH, HCHO and CH3CHO) have –CHO
This process is known as esterification. grouping
17. (a) 26. (b) Pinacolone is oxidation product of pinacol.
OH OH CH3 O
18. (a) | | | ||
oxidation with
19. (a) Addition of HCN to carbonyl compounds is CH 3 - C - C H 3 - C H 3 ¾¾ ¾¾ ¾¾® CH 3 - C - C - CH 3
| | rearrangem ent |
nucleophilic addition reaction. The order of reactivity CH3 CH3 CH3
of carbonyl compounds is Pinacol Pinacolone
3-dimethyl-2-butanone
higher), Then
EBD_7504
S-C-58 SOLUTIONS

27. (c) Aldehydes having a- H atom, when treated with 29. (d) I2 and Na2CO3 react with acetophenone (C6H5COCH3)
aluminium ethoxide ( in place of NaOH or KOH), to give yellow ppt. of CHI 3 but benzophenone
undergo Cannizzaro type of reaction with a difference (C6H5COC6H5) does not and hence can be used to
that the product isolated is an ester rather than salt of distinguish between them.
acid or alcohol. Such reaction in called Tischenko
reaction. COOC2H5 H2N
30. (c) CH 2 + CO
2CH3CHO ¾¾¾¾¾¾
®
Al(OC2 H5 )3 COOC2H5 H2 N
Acetaldehyde
Malonic ester
[CH3COOH + CH3CH2OH] ® CH 3COOC 2 H 5
Ethylaceta te D
CO - NH
¾¾
® CH 2 CO + C2H5OH
28. (a) It is an example of Claisen condensation. The product CO - NH
is acetoacetic ester. Barbituric acid
O O
|| ||
CH 3 C - OC 2 H 5 + H - CH 2 - C - OC 2 H 5 ¾
¾®

O O
|| ||
CH 3 - C - CH 2 - C - OC 2 H 5
(ethyl acetoaceta te )
Speed Test-56

1. (c) Since the organic amino compound on reaction with 15. (a) Reduction of alkyl isocyanides in presence of LiAlH4
nitrous acid at low temperature produces an oily yields secondary amines containing methyl as one of the
nitrosoamine so the organic amino compound is a alkyl group.
secondary aliphatic amines.
r LiAlH 4
2. (b) R - N = C + 4[H] ¾¾¾¾ ® R - NH - CH 3
2°amine
3. (a) The yield in Sandmayer reaction is found to be better
than Gattermann reaction. r LiAlH4
e.g., CH3 - N = C + 4[H] ¾¾¾¾ ® CH3 - NH - CH3
Reduction dimethyl amine
4. (d) CH3CN CH3 CH2 NH2
HONO whereas, alkyl cyanides give 1° amine on reduction.
CH3 CH2 OH + N2+ H2O
16. (d) C6 H 5 NH 2 + COCl2 + KOH ® C 6 H 5 NH.COCl + HCl
5. (c) Aliphatic amines are more basic than aromatic amines.
D
Resonance decreases the basic character due to ¾¾® C6 H 5 NCO + HCl
delocalisation of shared pair of electrons on nitrogen
17. (c) Benzaldehyde reacts with primary aromatic amine to
within benzene nucleus.
form schiff's base
6. (d)
7. (b) C6 H 5CH = O + C 6 H 5 NH 2 ¾
¾®
Benzaldehyde Aniline
+ –
N2 Cl OH
C 6 H 5 C H = NC 6 H 5 + H 2 O
¾¾¾®
H 2O Benzyliden e aniline
+ N2+HCl
8. (b)
CH3
NO2
CH3
p- cresol
HNO 3, H2SO 4 Cl 2/FeCl3
9. (d) 18. (b)
333 K

10. (b) CH2–NH2 compound is most basic due to


NO2 NH2
localized lone pair of electron on nitrogen atom while other
compounds have delocalized lone pair of electron. Cl /Fe reduction

NaCN Ni / H
11. (a) CH 3 CH 2 Cl ¾¾ ¾¾ ® CH 3 CH 2 CN ¾¾ ¾2¾
® Cl Cl
X
(a) When chlorination is done earlier than nitration,
(CH 3CO) 2 O chlorobenzene formed at first step would introduce –
CH 3CH 2CH 2 NH 2 ¾¾¾¾¾¾
® CH 3CH 2 CH 2 NHCOCH 3
Y Z
NO2 group in ortho-position, not in m-position
(b) Again if –NO2 group is reduced earlier than the
12. (d) (1) CH3 CH2 – CH2 – CH2 – NH2
chlorination step, –NH2 group formed on reduction will
(ii) with KOH (alcohol) and CHCl3 produces bad smell again introduce –Cl in o-position
(2) CH3C º CH 19. (c)
(iii) gives white ppt with ammonical AgNO3 20. (d) Reduction with iron scrap and hydrochloric acid is
(3) CH3 CH2 COOCH3 preferred because FeCl2 formed gets hydrolysed to
(i) alkaline hydrolysis release hydrochloric acid during the reaction. Thus,
(4) CH3 CHOH – CH3 only a small amount of hydrochloric acid is required
to initiate the reaction.
(iv) with Lucas reagent cloudiness appears after 5 minutes
= C + H 2 O ¾¾ H+
13. (a) R 2 NNO + H 2O ® R 2 NH + HNO 2 21. (d) CH 3CH 2 N ® ¾® CH 3CH 2 NH 2

14. (d) + HCOOH


EBD_7504
S-C-60 SOLUTIONS

NaOH Thus the correct order of basicities is


22. (a) CH3CONH2 ¾¾¾¾
Br
® CH3 NH 2
2 CH3
NH O
(Hofmann bromamide reaction) || | ||
CH3 - C - NH2 > CH3-NH > CH3CH2 NH 2 > CH3- C - NH2
23. (a) Acetamide is basic due to the presence of lone pair of
electrons of N it is also acidic because its conjugate 25. (c) CH3NC (methyl isocyanide) on reduction with LiAlH4
base shows resonance. gives secondary amine.
26. (c) Only 1° amides (i.e. RCONH2) in the present case
O O O-
|| || | undergo Hofmann bromamide reaction.
® CH 3- C - NH - ¬¾® CH 3- C = NH
base
CH3- C - NH 2 ¾¾¾

(more stable due to –ve charge on O) RCONH2 + Br2 + 4KOH ¾¾


®
Benzonitrile (C6H5CºN) acts as an electrophile (Lewis R - NH 2 + 2KBr + K 2CO3 + 2H 2O
acid) due to electron deficiency of C of CN as well as (Hofmann's bromamide reaction)
nucleophile (Lewis base) in nature due to presence of
lone pair electrons on N hence it is neutral Triethylamine 27. (d) C 6 H 5 NH 2 + CS 2 ® C 6 H 5 NH .CS.SH ¾HgCl
¾¾ ¾2®
and phenol are basic and acidic in nature respectively.
24. (b) Guanidine is most basic because its conjugate acid is C 6 H 5 N = C = S + HgS + 2HCl
stabilized by two equivalent resonance structures,. The reaction is called mustard oil reaction.

NH NH2 28. (b) O+ H 2 N.CH 2CH 3 ® N.CH 2 CH 3


|| | +
H+
CH 3- C - NH 2 ¾¾¾
® CH3- C - NH 2 ¬¾® CH 3- C - NH 2 ¬¾® CH 3- C = N H 2H 2 / Pt
¾¾ ¾ ¾® NH.CH 2CH 3
reduction

+ NH NH 2
2 O O
|| | +
|| || - Å
CH 3- C - NH 2 ¬¾® CH 3- C = N H 2
R - C - N- N º N:
29. (c) R - C - Cl + NaN3 ® ®

:
Further 2º amines are more basic than 1º, while amides
are least basic due to delocalisation of the lone pair of
electrons on N R – N = C = O + N2
. .- 30. (c) Amides are hydrolysed to give acids and NH 3 or
:O : : O:
|| . . | + amines.
CH 3- C- N H 2 ¬¾® CH 3- C = N H 2
Speed Test-57

1. (a) 20. (d) 2, 4- Dinitroflurobenzene, also known as Sanger's


2. (c) reagent, reacts with the H2N - group of the peptide to
3. (b) from 2, 4- dinitrophenyl (DNP) derivative of the peptide.
4. (d) The DNP derivative of the peptide is hydrolysed to
5. (a) give DNP derivative of the single amino acid
6. (d) Glucose being an aldose responds to Tollen’s test while
H 2 NCHCONH CHCONH CHCOOH +
fructose, although a ketose, undergoes rearrangement | | |
in presence of basic medium (provided by Tollen’s R1 R2 R3
reagent) to form glucose, which then responds to
Tollen’s test.
O2N F ¾
¾®
7. (d) Since sucrose is dextrorotatory while hydrolysis
product of sucrose, having equimolar mixture of glucose
and fructose, is laevorotatory. Hence the hydrolysed NO2
product of sucrose is known as invert sugar and the 2, 4, Dinitrofluoro benzene
hydrolysis of sucrose is known as inversion.
8. (b) All proteins are not found in L-form but they may be
O2N NHCHCONHC HCONH C HCOOH
present in form of D or L. | | |
9. (a) Butter, being a lipid, dissolves in chloroform. R1 R2 R3
NO2
10. (b) Human body can synthesize 10 of the 20 amino acids
obtained by hydrolysis of various proteins (DNP derivative of polypeptide)
11. (c) Chemically amylose is a long unbranched chain with
+
200-1000 a-D-(+ -glucose units held by C1-C4 H
glycosidic linkage.
12. (a)
13. (a) On the basis of structure of guanine and
O2N NH CHCOOH
complementary bases present in them, we can say that |
if the sequence of bases in one strand of DNA is I, then R1 +
NO2 (DNP derivative)
the sequence in the second strand should be II
A:T : G : C :T :T : G :A I H 2 NCHCOOH + H 2 NCHCOOH
T : A : C : G : A : A : C : T II | |
R2 R3
14. (c) –
O O (Amino acid mixture)
+
15. (a) Due to resonance, – C – NH – – C = NH – , 21. (d) Zwitter ion contains both +ve and –ve charge. Proton
of –COOH group is transferred to the –NH2 group.—
C – N bond acquires some double bond character, hence
NH3+ group is acidic since it can donate a proton and
shorter in length
—COO– group is basic since it can accept a proton.
16. (c) Proline is an acidic amino acid.
22. (c) Glucose is considered as a typical carbohydrate which
17. (a) Isoelectric point (pH) contains –CHO and –OH group.
pKa1 + pK a 2 2.34 + 9.60 23. (a)
= = = 5.97
2 2 24. (a) DNA has double stranded a-helical structure
18. (b) 25. (c)
OH 26. (d) The correct structure of thymine is
H H
O OH OH

19. (b) HO
N CH3
HO OH
H H
H
It is a b
EBD_7504
S-C-62 SOLUTIONS

27. (b) The DNA sequence that codes for a specific protein is
called a Gene and thus every protein in a cell has a 30. (a) CHO CH=N.NHPh
corresponding gene. 3PhNHNH2
CHOH CH=N.NHPh + PhNH2+ NH 3
28. (d) Enzymes are very specific biological catalysts
possessing well - defined active sites (CHOH)3 (CHOH)3
29. (b) In DNA and RNA heterocyclic base and phosphate
ester are at C1' and C5' respectively of the sugar CH2OH CH2OH
molecule.
Speed Test-58

1. (c) 2. (d) 3. (d) 13. (b) High density polythene is formed when addition
4. (a) Molecules of a polymer, being large in size, scatter light polymerisation of ethene takes place in a hydrocarbon
5. (c) Polythene is a linear polymer solvent in presence of catalyst such as ziegler-Natta
6. (d) Nylon–6 can be manufactured from catalyst.
O Cl
|
NH2OH 14. (b) CH 2 = C - CH = CH 2 (chloroprene) is the monomer
of neoprene.
O Cl Cl
| |
NOH C Polymerisation
conc H2SO4 CH 2 = C - CH = CH 2 ¾¾¾¾¾® [ CH 2 - C = CH - CH 2 ] n
NH K 2S 2 O 8
(Beckmann
rearrangement) C Chloroprene Neoprene
H 15. (b) Buna – N is a copolymer of butadiene
H
caprolactum (CH2=CH–CH=CH2) and acrylonitrile (CH2 = CHCN).
H2O, D 16. (b) Nylon 6, 6 has amide linkage capable of forming
hydrogen bonding.
é Où 17. (d) The concerned chemical reactions are
ê || úú ,(n,1)H 2O
ê polymerization
–ëê NH , (CH 2 )5 , C –ûú ↔¾¾¾ ¾¾¾ (i) CaC 2 + 2H 2 O ® Ca (OH) 2 + C 2H2
Nylon,6 64 kg Ethyne , 26kg
é ∗ ù (ii) C 2 H 2 + H 2 ® C2H 4
ê H 3 N, (CH 2 )5 , COO, ú
ë û Ethylene , 28 kg
7. (c) Ebonite is a hard highly vulcanized rubber, containing
20-30%, rubber (iii) nC2 H 2 ® [-CH 2 - CH 2 -]n
8. (a) Formula of the monomer indicated in bracket, n´28 kg n´28 kg polythene
or 28 kg or 28 kg
(CH3)2C=CH2, corresponds to 2-methylpropene
Thus 64 kg of CaC2 gives 26 kg of acetylene which in turn
9. (d) 10. (b)
gives 28 kg of ethylene whose 28 kg gives 28 kg of the
11. (a) Neoprene is a polymer of chloroprene. Hence, correct polymer, polythene.
representation is 18. (c)
Cl 19. (b) nHOOC(CH 2 ) 4 COOH + nH 2 N(CH 2 )6 NH 2
| adipic acid Hexamethylene
O2 or peroxides
nCH 2 = CH — C = CH 2 ¾¾¾¾¾¾® diamine
Chloroprene O O
525K
¾¾¾® [– C – (CH 2)4 – C –NH–(CH2) 6 – NH –]n
æ -CH 2 - C = CH - CH 2 - ö Polymerisation Nylon 6, 6
ç | ÷
ç ÷ 20. (b) 21. (a)
è Cl øn
Neoprene 22. (b) Nylon-66 is a polyamide, hence it has only –CONH–
linkage and no –COO– linkage
12. (b) Biodegradable polymer is Nylon-2- Nylon-6 which is 23. (c)
copolymer of glycine (H2N – CH2– COOH) and amino 24. (d) Orlon is a trade name of polyacrylonitrile
caproic acid (H2N–(CH2)5 – COOH). 25. (d)
nH2N – CH2 – COOH + nH2N – (CH2)5– COOH 26. (a) Melamine plastic crockery is a copolymer of HCHO
and Melamine.
Glycine Amino caproic acid 27. (b)
28. (d) Nylon and cellulose, both have intermolecular
hydrogen bonding, polyvinyl chloride has dipole-
O O
dipole interactions, while natural rubber has van der
–( HN – CH2 – C – HN – (CH2)5 – C )–n Waal forces which are weakest.
29. (a) Neoprene is a polymer of chloroprene
EBD_7504
Speed Test-59

1. (a) Given drug is used as antacid. 16. (a)


2. (c) 17. (d)
3. (d) 18. (d) Bacteriostatic drugs inhibit the growth of organism
while bactericidal drugs kill the microorganisms.
4. (c)
19. (c) Narrow spectrum antibiotics are effective against
5. (a) Gram–positive or Gram-negative bacteria. Limited
6. (b) It is acetyl salicylic acid i.e., aspirin, analgesic and spectrum antibiotics are effective against a single
antipyretic. organism or disease.
7. (a) Whiteware possesses good strength, translucency, 20. (c) Linear alkylbenzenesulphonates (LAB) carrying phenyl
and low porosity. chains at secondary positions are biodegradable
8. (b) 21. (b) The term “antihistamine” refers only to H1 antagonists,
9. (c) Glyceryl dinitrate is not used as fixative in perfumes. which is also known as H1-receptor antagonists and
10. (b) H1-antihistamine.
11. (d) Drugs that mimic the natural messenger by switching 22. (d)
on the receptor are called agonists. While drugs that 23. (c) Magnesium hydrosilicate forms base of Talcum
binds to the receptor site and inhibit its natural function powder.
are called antagonists. 24. (b) It is the known structure of Penicillin G
12. (d) The mixture of chloroxylenol and terpenol is dettol 25. (b) Aluminium salts present in deodorants act as
which is used as antiseptic. antibacterial agents.
13. (c) We know th at N-acetyl-para-aminophenol (or 26. (a) Pheromones are the chemical substances used by
paracetamol) is an antipyretic which can also be used insects for communication.
as an analgesic to relieve pain. 27. (b) Dilute solutions of boric acid and hydrogen peroxide
are weak antiseptics.
14. (a)
28. (d) Sucrose is natural sweetner.
15. (a) Benzalkonium chloride, also known as 29. (c) Broad spectrum antibiotics act on different antigens.
alkyldimethylbenzylammonium chloride is nitrogenous
cationic surface active agent belonging to the 30. (a)
quaternary ammonium group. It is used as antiseptic.

Cl
+
N
CnH2n+1
n = 8, 10, 12, 14, 16, 18
HINTS & SOLUTIONS (MATHEMATICS – Chapter-wise Tests)
Speed Test-60
1. (c) {5} is a subset of A as 5 Î A Now n( M Ç P 'Ç C ') = n[ M Ç ( P È C ) ']
But, {1, 2} is not a subset of A as elements 1, 2 Ï A.
2. (a) Let U be the set of consumers questioned X, the set of = n( M ) - n[( M Ç ( P È C )]
consumers who liked the product A and Y, the set of = n( M ) - n[( M Ç P) È ( M Ç C )]
consumers who liked the product B. Then n (U) = 1000,
n (X) = 720, n (Y) = 450 = n( M ) - n ( M Ç P ) - n ( M Ç C ) + n ( M Ç P Ç C )
n (XÈY) = n (X) + n (Y) – n (XÇY) = 1170 – n (XÇY) = 23 – 12 – 9 + 4 = 27 – 21 = 6
\ n (X Ç Y) = 1170 – n (X È Y) n( P Ç M 'Ç C ') = n[ P Ç (M È C ) ']
Clearly n (X Ç Y) is least
When n (X È Y) is maximum. = n( P) - n[ P Ç ( M È C )]
Now, XÈY Ì U
= n( P) - n[( P Ç M ) È ( P Ç C )]
\ n(XÈY) £ n (U) = 1000
\ the maximum value of n (XÈY) is 1000. = n ( P ) - n( P Ç M ) - n( P Ç C ) + n ( P Ç M Ç C )
Thus the least value of n (XÇY) is 170 = 24 – 12 – 7 + 4 = 9
3. (d) For a set S, the partition of S is a set of subsets of S, such
that they are pair-wise disjoint and their union is S. n(C Ç M 'Ç P ')
In the options (a) & (b), the subsets are not disjoint. = n(C ) - n(C Ç P ) - n(C Ç M ) + n(C Ç P Ç M )
In the option (c), the subsets are disjoints but their = 19 – 7 – 9 + 4 = 23 – 16 = 7
union is not equal to the given set.
8. (c) We have
Only the option (d) meets with both the requirements.
min n (A È B) = max {n(A), n (B)} = max {3, 6} = 6
4. (a) From Venn-Euler’s Diagram.
max n (A È B) = n (A) + n (B) = 9
\ 6 £ n (A È B) £ 9
(A È B)'
9. (b) A D B = (A – B) È (B – A)
U
= {1, 2} È {3, 4, 9}
(A 'Ç B) = {1, 2, 3, 4, 9}
10. (a) See the following Venn diagram
A B I M F
23
4–x

\ (A È B) ' È (A 'Ç B) = A ' x


29
5. (d) A = {(n, 2n) : n Î N} and B = {(2n, 3n)}: n Î N W
Listing few members of each set
n (I) = 29 + 23 = 52
A = {(1, 2), (2, 4), (3, 6),....} n (F) = 100 – 52 = 48
B = {(2, 3), (4, 6), (6, 9)......}
n(M È D) = n(M) + n(D) - n(M Ç D)
There is no member common to both these sets, hence.
A ÇB= f 24 = 23 + 4 - n(M Ç D)
6. (a) bN = {bx : x ÎN} \ n(M Ç D) = 3
cN = {cx : x ÎN}
\ bN Ç cN = {x : x is multiple of b and c both} \ n ( W Ç D) = 4 - 3 = 1
= { x: x is multiple of l.c.m. of b and c } 11. (c) Suppose a Î X and aÎA
= { x : x is multiple of b c} Þ aÎ X È A Þ aÎ Y È A
[given b and c are relatively prime \ l.c.m. of b and Þ aÎ Y and aÎA (Q XÈA = YÈA)
c = bc] Þ aÎYÇA Þ YÇA is non-empty
\ bN Ç cN = {bc x : x ÎN} = dN (Given)
This contradicts that YÇA = f
\ d = bc.
So, X = Y
7. (d) n( M ) = 23, n( P) = 24, n(C ) = 19
12. (d) (a) x Î A - B Û x Î A and x Ï B
n( M Ç P) = 12, n( M Ç C ) = 9, n( P Ç C ) = 7
Û x Î A and x Î B' Û x Î A Ç B'
n( M Ç P Ç C ) = 4 \ A – B = A Ç B' ...(i)
We have to find n( M Ç P 'Ç C '), n( P Ç M 'Ç C '), x ÎA and x ÎB'
Û x ÏA' and xÎB' Û xÎB' and xÏA'
n(C Ç M 'Ç P ')
EBD_7504
S-M-2 SOLUTIONS

\ A – B = B' – A' ...(ii) = n(A) - n(A Ç B)


Clearly (a) is not correct. Also from (i) (c) is not correct.
Replacing A by P and B by Q È R, we have
Next let xÎA – (A – B)
Û xÎA and xÏA – B n (P Ç (Q È R) ') = n(P) - n(P Ç (Q È R)) etc.
Û xÎA and [x ÏA or x ÎB] Hence all options are correct.
A – (A – B) = A Ç B 15. (a) Minimum value of n = 100 – (30 + 20 + 25 +15)
Û [xÎA and x ÏA] or [ x ÎA and xÎB] = 100 – 90 = 10
\ A – (A – B) = f È (A Ç B) = A Ç B 16. (c) n (A' Ç B') = n (A È B)' = n (U) – n (A È B)
\ (b) is also incorrect = n (U) – [n (A) + n (B) – n (A Ç B)]
The result (d) is correct as can be seen in the following = 700 – [200 + 300 – 100] = 300
Venn diagram 17. (d) Here first statement is true wheareas the second
statement is false.
U 18. (b) C stands for set of students taking economics
A B
E C
a b c
A–B BÇA B–A
g
d e
f
A È B = (A – B) È (A Ç B) È (B – A) M
a + b + c + d + e + f + g = 40; a + b + d + g = 16
13. (a) B A b + c + e + g = 22; d + e + f + g = 26
b + g = 5; e + g = 14; g = 2
B¢ – A ¢ and A – B Go by backward substitution
e = 12, b = 3, d + f = 12, c + e = 17 Þ c = 5;
a + d = 11
a +d + f = 18 Þ f = 7 \ d = 12 – 7 = 5
14. (d) Let P = set of families buying A,
Q = set of families buying B 19. (c) Numbers which are divisible by 5 are 5, 10, 15, 20, 25,
and R = set of families buying C. 30, 35, 40, 45, 50, 55, 60, 65, 70, 75, 80 they are 16 in
numbers. Now, Numbers which are divisible by 7 are
\ n (P) = 40% of 10,000 = 4,000, similarly 7, 14, 21, 28, 35, 42, 49, 56, 63, 70, 77 they are 11 in
n(Q) = 2, 000 , n(R) = 1,000 numbers.
n(P Ç Q) = 500, n (Q Ç R) = 300 Also, total odd numbers = 40
n (P Ç R) = 400 and n (P Ç Q Ç R) = 200 Let C represents the students who opt. for cricket, F
(i) Number of families buying only A = n(P Ç Q¢ Ç R¢) for football and H for hockey.
= n (P Ç (Q È R)') = n(P) –n(P Ç (Q È R)) \ we have n(C) = 40, n(F) = 16, n(H) = 11
Now, C Ç F = Odd numbers which are divisible by 5.
= n(P) - [n(P Ç Q) + n(P Ç R) - n((P Ç Q) I (P Ç R))] CÇ H = Odd numbers which are divisible by 7.
= n(P) - n(P Ç Q) - n(P Ç R) + n(P Ç Q Ç R). F Ç H = Numbers which are divisible by both 5 and 7.
= 4,000 – 500 – 400 + 200 = 3,300. n(C Ç F), 8, n(C Ç H) = 6,
(ii) Number of families buying only B n(FÇ H) = 2, n (C Ç F Ç H) = 1
We Know
= n(Q) - n(P Ç Q) - n(Q Ç R) + n(P Ç Q Ç R)
n(CÈFÈH) = n(C) + n(F) + n(H)
[see (i)] – n(C Ç F) – n(C Ç H)
= 2,000 – 500 – 300 + 200 = 1,400. – n(F Ç H) + n(C Ç H Ç F)
(iii) Number of families buying none of A, B and n(CÈFÈH) = 67 – 16 + 1 = 52
C = n(P ' ÇQ ' ÇR ') = n(P ' Ç(Q ÈR) ') \ n(C¢ Ç F¢ Ç H¢)
= n{(P È (Q È R)) '} = 10000 - n(P È Q È R) = Total students – n(C È F È H)
n(C¢ Ç F¢ Ç H¢)= 80 – 52 = 28
= 10, 000 - [n(P) + n(Q) + n(R) - n(P Ç Q) 20. (c) a + e + f + g = 23 H
b + d + f + g = 15 f b B
-n(Q Ç R) - n(P Ç R) + n(P Ç Q Ç R)] a g
c + d + e + g = 20 e d
= 10,000 – [4,000 + 2,000 + 1,000 – 500 – 300 – 400 + 200] c
f + g = 7; d + g = 5
= 10, 000 – 6,000 = 4,000. C
e+ g= 4
Note : For sets A, B, we have
a + b + c + d + e + f + g = 60 – 15 = 45
(A Ç B) È (A Ç B') = A Ç (B È B') = A Ç U = A By substitutions,
and (A Ç B) Ç (A Ç B') = A Ç (B Ç B') = A Ç f = f a + e = 16, b + d = 8, b + f = 10, c + e = 15, c + d = 16
Also, b + c + d = 22
\ n(A) = n(A Ç B) f = 25
Mathematics S-M-3

From these, we get 27. (d) We have


b = 6, a = 15, c = 14 e = 1, d = 2, f = 4 and g = 3 n (A È B È C) = n (A) + n (B) + n (C) –
Clearly(a) is not correct n (A Ç B) – n(BÇC) – n (C Ç A) + n (AÇB Ç C)
for (b) a + f = 19 Þ (b)is incorrect = 10 +15 + 20 – 8 – 9 – n (C Ç A) + n (A Ç B ÇC)
for (c) e = 1 Þ (c) is correct = 28 – {n(C Ç A) – n (A Ç B Ç C)} ...(i)
21. (b) Given set can be written as Since n (C Ç A) ³ n (A Ç B Ç C)
(A – B) È (B – A) = (A È B) – (A Ç B) We have n (C Ç A) – n (A Ç B Ç C) ³ 0...(ii)
(By definition of symmetric difference) From (i) and (ii)
Hence, (A \ B) È (B \ A) = (A È B) \ (A Ç B) n (A È B È C) £ 28 ...(iii)
22. (c) A = {1, 3, 5, 15}, B = {2, 3, 5, 7} C = {2, 4, 6, 8 } Now, n(A È B) = n (A) +n (B) – n (A Ç B)
= 10 + 15 – 8 = 17
\ A È C = {1, 2, 3, 4, 5, 6, 7, 8,15} and n (B È C) = n (B) + n (C) – n (B Ç C)
(A È C) Ç B = {2, 3, 5} = 15 + 20 – 9 = 26
Since, n (A È B È C) ³ n (AÈC) and
23. (b) 2m - 2n = 112 Þ 2n (2 m- n - 1) = 16.7 n (AÈBÈC) ³ n (BÈC), we have
\ 2n (2m -n - 1) = 24 (23 - 1) n (AÈBÈC) ³ 17 and n (AÈBÈC) ³ 26
Hence n (AÈBÈC) ³ 26 ...(iv)
Comparing we get n = 4 and m – n = 3 From (iii) and (iv) we obtain
Þ n = 4 and m = 7 26 £ n (AÈBÈC) £ 28
24. (a) Let the number of students who take only Math be x Also n (AÈBÈC) is a positive integer
and only Chemistry be y. \ n(AÈBÈC) = 26 or 27 or 28
M C 28. (a) U U
x 30
y A B A B

So, from the Venn diagram, we have total number of C C


students who take Math = x + 30
and take Chemistry = y + 30.
(i) A È B È C (ii) (A Ç Bc Ç Cc)
According to question, we have
10 U
30 = ( x + 30)
100 A B
Þ x = 270 and
C
12
30 = ( 30 + y )
100
Þ y = 220 (iii) Cc
x + y + 30 = 270 + 220 + 30 = 520. From Fig. (i), (ii) and (iii), we get
(A È B È C) Ç (A Ç Bc Ç Cc)c Ç Cc = (B Ç Cc)
25. (d) n(A) = 1000, n(B) = 500, n(A Ç B) ³ 1, 29. (b) n(A) = 40% of 10,000 = 4,000
n(A È B) = p n(B) = 20% of 10,000 = 2,000
n(A È B) = n(A) + n(B) – n(A Ç B) n(C) = 10% of 10,000 = 1,000
p = 1000 + 500 – n (A Ç B) n( A Ç B) = 5% of 10,000 = 500
1 £ n(A Ç B) £ 500 n( B Ç C ) = 3% of 10,000 = 300
Hence p £ 1499 and p ³ 1000
n(C Ç A) = 4% of 10,000 = 400
1000 £ p £ 1499
n( A Ç B Ç C ) = 2% of 10, 000 = 200
26. (c) Given set is {(a, b) : 2a 2 + 3b2 = 35, a, b Î Z } c c c
We want to find n( A Ç B Ç C ) = n[ A Ç ( B È C ) ]
2
We can see that, 2(±2) + 3(±3) = 35 2
= n( A) - n[ A Ç ( B È C )]
and 2(±4)2 + 3(±1)2 = 35 = n( A) - n[( A Ç B ) È ( A Ç C )]
\ (2, 3), (2, – 3), (–2, – 3), (–2, 3), (4, 1), (4, – 1), = n( A) - [n( A Ç B) + n( A Ç C ) - n( A Ç B Ç C )]
(– 4, –1), (–4, 1) are 8 elements of the set . \ n = 8.
= 4000 – [500 +400 – 200] = 4000 – 700 = 3300.
30. (b) Both statements are correct but second statement is
not proper explaination of statement-1.
EBD_7504
S-M-4 SOLUTIONS

Speed Test-61
1. (b) We have domain of R = the set of first components of
the ordered pairs in R = {0, 0, 1, 2, 2, 2, 1, 0, 0} = {0, 1, 2} 3
8. (a) f ( x) = 2
+ log10 ( x 3 - x)
range of R = the set of second components of the 4- x
ordered pairs in R
= {0, 1, 1, 1, 2, 0, 0, 2, 1} = {0, 1, 2} 4 - x ¹ 0; x 3 - x > 0;
2

2. (c) f (x) = ( x - 2) ( x - 3) + -( x - 4) ( x + 2) x ¹ ± 4 and - 1 < x < 0 or 1 < x < ¥


The first part is real outside (2, 3) and the second is real
in [–2, 4] so that thedomain is [–2, 2] È [3, 4]. \ D = (- 1, 0) È (1, ¥ ) - { 4} –
+ –
+

1 –1 0 1
æ 1ö D = (- 1, 0) È (1, 2) È (2, ¥ ).
3. (d) 3 f ( x) - f ç ÷ = log x 4 ; x º
è xø x 9. (c) Here A and B sets having 2 elements in common, so
4
æ1ö æ1ö A × B and B × A have 22 i.e., 4 elements in common.
3 f ç ÷ - f ( x) = log ç ÷
è øx è xø Hence, n[(A × B) Ç (B × A)] = 4
After solving we get f(x) = log x
10. (d) x2 + y2 = 9 Þ y2 = 9 – x2 Þ y = ± 9 – x 2
f (e - x ) = log e e - x = - x
1 x=0Þy= ± 9–0 = ±3ÎZ
4. (b) f ( x) = , define if | x | – x > 0
x -x x = ± 1 Þ y = ± 9 –1 = ± 8 Ï Z
Þ | x | > x, Þ x < 0 x= ±2Þy= ± 9–4 = ± 5 ÏZ
Hence domain of f(x) is (– ¥ , 0)
x=±3Þy= ± 9–9 = 0ÎZ
5. (d) | x 2 | -5 | x | +6 = (| x | -2) (| x | -3)
is real for 0 £ |x| £ 4 x = ± 4 Þ y = ± 9 – 16 = ± –7 Ï Z and so on.
\ f(x) is real for all 0 £ |x| £ 2 or 3 £ |x| £ 4.
6. (b) In the definition of function \ R = {(0, 3), (0, –3), (3, 0), (–3, 0)}
Domain of R = {x : (x, y) Î R} = {0, 3, –3}
x(x - p) x(p - q) Range of R = {y : (x, y) Î R} = {3, –3, 0}.
f (x) = + =p
q-p (p - q)
Putting p and q in place of x, we get 11. (c) f ( xy ) = 1 + x 2 y 2
p(p - p) p(p - q) f (x) f (y) = 1 + x 2 1 + y 2 = 1+ x 2 y 2 + x 2 + y 2
f (p) = + =p
q-p (p - q)
Þ f (p) = p ³ 1+ x 2 y 2 = f (xy)
q(q - p) q(p - q) \ f (xy) £ f (x) f ( y)
and f (q) = + =q 12. (d) For f (x) to be defined, we must have
q-p (p - q)
Þ f (q) = q x - 1 - x 2 ³ 0 or x ³ 1 - x2 > 0
Putting x = (p + q) 1
\ x2 ³ 1 – x2 or x ³
2
.
(p + q)(p + q - p) (p + q)(p + q - q) 2
f (p + q) = + Also, 1 – x2 ³ 0 or x2 £ 1.
(q - p) (p - q)
(p + q)q (p + q)(p) pq + q 2 - p 2 - pq 1 æ 1 ö æ 1 ö
Now, x ³ Þ çè x - ÷ø çè x +
2
= + = ÷ ³0
(q - p) (p - q) (q - p) 2 2 2ø
1 1
Þ x£- or x ³
q 2 - p2 (q - p)(q + p) 2 2
= = = p + q = f (q) + f (p)
q-p (q - p) Also, x2 £ 1 Þ (x – 1) ( x + 1) £ 0
So, f (p) + f (q) = f (p + q) Þ –1 £ x £ 1
7. (a) f (2a – x) = f (a – (x – a)) = f (a) f (x – a) – f (0) f (x) 1
Thus, x > 0, x ³ and x2 £ 1
2
= f (a) f (x –a) – f (x) = – f (x) 2
[Q x = 0, y = 0, f (0) = f 2 (0) - f 2 (a) é 1 ù
ÞxÎê ,1ú
Þ f 2 (a) = 0 Þ f (a) = 0] ë 2 û
Þ f (2a - x) = - f ( x )
Mathematics S-M-5

22. (b) Let f (x) = log (g(x))


13. (b) Period of sinx = 2π Þ period of sin 3 x = 2π \ f (x1) + f (x2) = log(g(x1)) + log(g(x2))
x = log(g(x1) · g(x2))
period of sin 3 x = π Þ period of sin3 = 2π \ Option (b) is correct
2
23. (a) f ( x + y ) = f (x ) + f ( y ) .
period of cos5 x = 2π Þ period of cos5 x = π
Function should be f (x) = mx
5 x f (1) = 7; \ m = 7, f ( x ) = 7 x
Þ period of cos = 5π
5 n n 7 n ( n + 1)
Thus required period = LCM of 2π & 5π = 10π S f (r ) = 7 S r =
r =1 1 2
14. (d) n(A) = 4, n(B) = 3 24. (d) {x2} – 2 {x} ³ 0
n(A) × n(B) × n(C) = n(A × B × C) Þ {x} ({x} – 2} ³ 0
4 × 3 × n(C) = 24 Þ n(C) = 24/12 = 2 Þ {x}£ 0 or {x} ³ 2
15. (b) We have (x, y) Î R iff x + y < 6 Second case is not possible.
Given the value x = 1, we get possible values of y = 1, 2, 3, 4. Hence {x} = 0, as {x}£ [0, 1). Hence range of f (x)
Thus 1R1, 1R2, 1R3, 1R4. Similarly we may find other contains only one element 0.
values. The set of such ordered pairs is
x -1
R = {(1, 1), (1, 2), (1, 3), (1, 4), (2, 1), (2, 2), 25. (b) Given f ( x ) =
(2, 3), (3, 1), (3, 2), (4, 1)} x +1
\ n (R ) = 10 2x -1
\ f (2 x) =
2x + 1
16. (c) f ( x ) = log( x + x 2 + 1)
2(2 x - 1)
= (multiply and divide by 2)
{ }
ìï - x 2 + x 2 + 1üï 2(2 x + 1)
f ( - x ) = log - x + x 2 + 1 = log í ý
ïî x + x 2 + 1 ïþ 4 x - 2 3x + x - 3 + 1 3( x - 1) + x + 1
= = =
4x + 2 3x + x + 3 - 1 3( x + 1) + x - 1
= - log( x + x 2 + 1) = - f ( x )
Þ f (x) is an odd function. é x -1ù
3ê ú + 1 3 f ( x) + 1
x0 a ë x + 1û
17. (d) x0 = a, x1 = f (x) = = ; = x -1 =
1 - x0 1 - a +3 f ( x) + 3
a x +1
x1 1 - a = a
x2 = f ( x1 ) = = x2 - x + 1
1 - x1 1 - a 1 - 2a 26. (d) Let y =
1- a x2 + x + 1
a Þ x2(y – 1) + x(y + 1) + (y – 1) = 0
\ x2009 = = 1 Þ 1 – 2009 a = a
1 - 2009 a - (y + 1) ± (y + 1) 2 - 4(y - 1) 2
Þ x=
1 2(y - 1)
Þ a=
2010
æ 1 ö -(y + 1) ± -3y 2 + 10y - 3
18. (a) f (x) is defined if – log1/2 çè 1 + 1/ 4 ÷ø –1 > 0 =
2(y - 1)
is real iff
x
æ 1 ö y–1¹ 0Þy¹1
Þ log1/2 çè 1 + 1/ 4 ÷ø < –1 If y = 1 then original equation gives x = 0, so taking
x
–1 y= 1
1 æ 1ö Also 3y2 – 10y + 3 £ 0
Þ 1 + 1/ 4 > ç ÷
x è 2ø Þ (3y – 1) (y – 3) £ 0
1 é1 ù é1 ù
Þ 1/ 4 > 1 Þ y Î ê , 3ú \ Range is ê , 3ú
x ë3 û ë3 û

( )
Þ 0< x<1
19. (b) For f (x) to be defined, we must have 27. (b) We have, f (x) = exp 5x - 3 - 2 x2
x2 – 3x + 2 = (x – 1) (x – 2) > 0 Þ x < 1 or > 2
Domain of f = (– ¥,1) È ( 2, ¥). 5 x - 3- 2 x 2
i.e. , f (x) = e
20. (a) It is obvious.
For Domain of f (x), 5 x - 3 - 2 x 2 should be +ve.
æ x2 + e ö æ x2 + 1 - 1 + e ö æ e -1 ö
21. (b) f (x) = ln ç ÷ = ln ç ÷ = ln ç 1+ 2 ÷ i.e., 5x - 3 - 2 x2 ³ 0
2 2 è ø
è x +1ø
taking –ve sign common)
EBD_7504
S-M-6 SOLUTIONS

Þ 2 x( x - 1) - 3( x - 1) £ 0 putting y = 1,
Þ (2 x - 3)( x - 1) £ 0 f(2) = 8 = 2 + 2 + k Þ k = 4
Þ 2x - 3 £ 0 or x -1³ 0 \ f (1 + y) = 2 + 2y2 + 4y = 2(y + 1)2
3 \ f (x) = 2x2
Þ x£ or x ³1 29. (d) Here R = {(x, y) : | x2 – y2 | < 16 }
2 and given A = {1, 2, 3, 4, 5}
3 é 3ù \ R = {(1, 2) (1, 3) (1, 4); (2, 1) (2, 2) (2,3) (2, 4);
\ 1£ x £ i.e., x Î ê1, ú
2 ë 2û (3, 1); (3, 2) (3,3) (3, 4); (4, 1) (4,2) (4, 3); (4, 4), (4, 5), (5, 4)
3 (5, 5)}
Hence, domain of the given function is [1, ].
2 30. (c) If n = 0, then h (n) is not defined, so, ‘h’ is not a function.
28. (a) f (x + y) = f(x) + 2y2 + kxy All other are functions.
f(1 + y) = 2 + 2y2 + ky, putting x = 1

Speed Test-62

1. (c) Given y = cos2 x + sec2 x


1 æ 1 ö Now, x 2 + 4 y 2 = 4cos2 q + 4sin2 q
çèQ cos x =
2
Þ y = cos x + ÷ 2 2
= 4(cos q + sin q) = 4
2
cos x sec x ø
1 5. (d) The given equation can be written as
2
Þ y = cos x + + 2- 2
cos2 x 1 - 2 sin 2 x cos 2 x = a
2
æ 1 ö Þ sin 2 2x = 2(1 - a ) Þ 2(1 - a ) £ 1
Þ y = çè cos x - ÷ +2
cos x ø and 2(1 - a ) ³ 0 Þ 1 / 2 £ a £ 1
Þ y = (cos x - sec x)2 + 2
q q æ 1 + cos q ö
As (cos x – sec x)2 = 0 or positive 6. (d) Consider tan (1 + secq) = tan ç ÷
\ y = 2 or y ³ 2 2 2 è cos q ø

æ 3q ö æqö sin q / 2 2cos2 q / 2 sin q


2 sin ç ÷ cos ç ÷ = . …= = tan q …(1)
sin q + sin 2q è 2 ø è 2 ø = tan æ 3q ö cos q / 2 cos q cos q
2. (c) = ç ÷
cos q + cos 2q æ 3q ö æ qö è 2 ø \ f1(q) = tanq/2 (1 + secq) (1 + sec 2q)
2 cos ç ÷ cos ç ÷ = [tan q/2 (1 + sec q)] (1 + sec 2q)
è ø2 è ø
2
2p = (tan q) (1 + sec 2q) [from(1)]
Hence period = = tan 2q [replacing q by 2q as above]
3
3. (c) Given an angle q which is divided into two parts A and Þ f1(q) = tan 21 q …(2)
Similarly, f2(q) = tan 22 q, f3(q)
B such that A – B = k and A + B = q ,
= tan 23q, f4(q) = tan24q etc.
tan A k
and tan A : tan B = k : 1, i.e. = æ pö æ pö p
tan B 1 Þ f 2 ç ÷ = tan ç 22 ÷ = tan = 1
tan A + tan B k + 1 è 16 ø è 16 ø 4
Þ =
tan A - tan B k - 1 æ pö æ pö p
(by componendo and dividendo) f 3 ç ÷ = tan ç 23 ÷ = tan = 1
è 32 ø è 32 ø 4
sin (A + B) k + 1 sin q k + 1
Þ = Þ = æ pö æ pö p
sin (A - B) k - 1 sin k k - 1 f 4 ç ÷ = tan ç 24 ÷ = tan = 1
è 64 ø è 64 ø 4
k -1 7. (b) The given expression can be written as
Þ sin k = sin q
k +1 (cos6x + cos4x) + 5(cos4x + cos2 x) +10(cos2 x +1)
4. (a) Given that 2y cos q = x sin q ...(i)
cos5x + 5cos3x +10cos x
and 2 x sec q - y cosec q = 3 ...(ii)
2x y 2 cos 5 x cos x + 5.2 cos 3 x cos x + 10.2 cos 2 x
- =3 =
Þ cos5 x + 5cos 3x + 10 cos x
cos q sin q
Þ 2 x sin q - y cos q - 3sin q cos q = 0 ...(iii) 2 cos x(cos5 x + 5cos 3 x + 10 cos x )
= = 2 cos x
Solving (i) and (iii), we get y = sin q and x = 2 cos q cos5 x + 5cos 3 x + 10 cos x
Mathematics S-M-7

8. (a) We have sin a + sin b + sin g – sin ( a + b + g) sin 2B = 3 sin A cos A ...(2)
= sin a + sin b + sin g – sin a cos b cos g Now, cos (A + 2B) = cos A cos 2B – sin A sin 2B
– cos a sin b cos g – cos a cos b sin g = cos A (3 sin 2 A) – sin A (3 sin A cos A) = 0
+ sin a sin b sin g [using eqs. (1) and (2)]
= sin a (1 – cos b cos g) + sin b (1 – cos a cos g) p
+ sin g(1–cos a cosb) + sin a sin b sin g > 0 Þ A + 2B =
\ sin a + sin b + sin g > sin (a + b + g) 2
1
Trick :Put a = 30, b = 30, g = 60 and check... 14. (b) Let f (x) = sin x cos x = sin 2 x
2
sin(a + b + g ) 1 1 1
Þ <1 We know -1 £ sin 2 x £ 1 Þ – £ sin 2 x £
sin a + sin b + sin g 2 2 2
9. (b) The expression Thus the greatest and least value of of f(x) are
æ p öæ 3p öæ 3p öæ pö 1 1
= ç1 + cos ÷ç1 + cos ÷ç1 - cos ÷ç1 - cos ÷ and - respectively
è 10 øè 10 øè 10 øè 10 ø 2 2
é 7p æ 3p ö 3p æp ö
êQ cos = cosç p - ÷ = - cos 15. (c) tan (cot x) = cot (tan x) = tan çè - tan x ÷ø
10 è 10 ø 10 2
ë
p
9p æ pö pù Þ cot x = n p + – tan x
and cos = cosç p - ÷ = - cos ú 2
10 è 10 ø 10 û [Q tan q = tan a Þ q = np + a]
p
æ p öæ 3p ö 2 p 3p Þ cot x + tan x = n p +
= ç1 - cos 2 ÷ç1 - cos 2 ÷ = sin . sin 2 2
è 10 øè 10 ø 10 10
cos x sin x p
2 Þ + = (2n + 1)
æ 5 -1 5 + 1ö sin x cos x 2
= sin 2 18°. sin 2 54° = ç . ÷ = 1
ç 4 4 ÷ 16 1 p
è ø Þ = (2n + 1)
sin x cos x 2
10. (b) Consider sin A - 6 cos A = 7 cos A 1 (2n + 1) p
Þ =
Þ sin A = ( 7 + 6) cos A sin 2 x 4
4
( 7 + 6)( 7 - 6) Þ sin 2x =
Þ sin A = cos A (2n + 1)p
( 7 - 6)
x
Þ 7 sin A = cos A + 6 sin A 16. (c) Put t = >0,
y 2
11. (a) Let 3 + 1 = r cos a, and 3 – 1 = r sin a 1 æ 1ö
Consider t + = ç t - ÷ + 2 ³ 2 equality holding
( ) ( )
2 2
\ r 2 = 3 + 1 + 3 – 1 = 8 i.e. a = p/12 è t tø
From the equation, r cos (q – a) = 2 iff t = 1
Þ cos (q – p /12) = 1/ 2 = cos (p/4) 1
\ q = 2np ± p/4 + p/12 Also, t + = 2sin q £ 2, so that t should necessarily
t
12. (a) We have, sin p( x 2 + x) = sin px 2 be 1, i.e., x = y.
Þ p(x 2 + x ) = np + (-1) n px 2 17. (b) p n - p n -2 = (cos n q + sin n q) - (cos n - 2 q + sin n - 2 q)
2 2
\ Either x + x = 2m + x Þ x = 2m Î I = cos n - 2 q(cos 2 q - 1) + sin n - 2 q(sin 2 q - 1)

or x 2 + x = k - x 2 , where k is an odd integer = - sin 2 q cos n - 2 q - cos 2 q sin n - 2 q

- 1 ± 1 + 8k = - sin 2 q cos 2 q(cos n -4 q + sin n -4 q)


Þ 2x 2 + x - k = 0 Þ x =
4 = - sin 2 q cos 2 qp n - 4 = kp n -4
For least positive non-integral solution
1 Þ k = - sin 2 q cos 2 q
is x =
, when k = 1 18. (a) Given f (x) = cos (log x)
2
13. (b) Given, 3 cos2A +2 cos2 B = 4 \ f (xy) = cos (log xy)
f (xy) = cos [log x + log y] ....(i)
Þ 2 cos 2 B - 1 = 4 - 3 cos 2 A - 1
æ xö æ ö
And f ç ÷ = cos log x
Þ cos 2B = 3 (1 - y ÷ø
and 2 cos B sin B
EBD_7504
S-M-8 SOLUTIONS

æ xö \ 3S 4 – 2 S 6
f ç ÷ = cos (log x – log y) ....(ii)
è yø = 3(cos 4 q + sin 4 q) - 2(cos4 q + sin 4 q – cos 2 q sin 2 q )
Adding (i) and (ii), we get
æ xö = cos 4 q + sin 4 q + 2 cos2 q sin 2 q
f (xy) + f ç ÷ = cos (log x + log y) + cos (log x – logy)
è yø = (cos2 q + sin 2 q)2 = (1)2 = 1
= 2 cos (log x). cos (log y)
23. (a) We have | 4 sin x - 1 |< 5
æ xö
Þ f (xy) + f ç ÷ = 2 f (x). f (y) Þ - 5 < 4 sin x - 1 < 5
è yø
1 ì æ xö ü æ 5 -1 ö
Then the value of f (x)f (y) – í f ç ÷ + f ( xy)ý Þ -ç ÷ < sin x < 5 + 1
2 î è yø ç 4 ÷ 4
þ è ø
1
= f ( x ) f ( y ) - .2 { f ( x). f ( y )} = 0 æ-pö æ 3p ö
2 Þ sin ç ÷ < sin x < sinç ÷
19. (b) Let g(x) = 6 sin x – 8 cos x + 5 è 10 ø è 10 ø
Max. value of g(x) = 6 2 + 8 2 + 5 = 5 + 10 = 15 æ p 3p ö
Þ x Îç- , ÷ [Q x Î (-p, p)]
2 2
Min. value of g(x) = - 6 + 8 + 5 = 5 - 10 = -5 è 10 10 ø
1 æ 1 1ö sin x cos x
\ The range of f ( x ) = is R - ç - , ÷ 24. (c) f (x) = | sec x | - | cos ec x |
g (x ) è 5 15 ø
Þ it is an unbounded function. Range f(x) = sin x . | cos x | – cos x | sin x | = ?
Þ f(x) has no maximum and no minimum values.
ì é πù
20. (a) Since a = b = g Þ cos 2 a = cos2 b = cos 2g ï 0 x Î ê0, ú
ï ë 2û
Q cos 2 a + cos2 b + cos 2 g = 1
ï æπ ö
Þ 3cos 2 a = 3cos 2 b = 3cos2 g = 1 ï- sin 2 x x Îç , π÷
è2 ø
ï
f ( x) = í
2 2 2 1 æ 3π ö
Þ cos a = cos b = cos g = ï 0 x Î ç π, ÷
3 ï è 2ø
2 ï
2 2 2
\ sin a = sin b = sin g = ï sin 2 x æ 3π ö
x Î ç , 2π÷
3 ïî è 2 ø
3 (1/ 3) 1 p
\ cos q = = Þ q=
3 (2 / 3) 2 3
– sin 2x 0
21. (a) sin 12° sin 24° sin 48° sin 84°
1
= (2 sin 12° sin 48°) (2 sin 24° sin 84°) 0 sin 2x
4
1
= (cos 36° – cos 60°) (cos 60° – cos 108°)
2 So range is [–1, 1]
1æ 1ö æ1 ö sin( x + y ) a + b
= çè cos 36° – ÷ø çè + sin 18° ÷ø 25. (b) Let =
4 2 2 sin( x - y ) a - b

=
1 ì1
í
4 î4
( )
1ü ì1 1
5 +1 – ýí +
2þ î 2 4
ü 1
5 –1 ý =(
þ 16
) If
a c
= , then by componendo and dividendo, we
b d
and cos 20° cos 40° cos 60° cos 80°
a +b c +d
1 have = .
= [cos(60° – 20°) cos 20° cos(60° + 20°)] a -b c -d
2 Applying componendo and dividendo, we get
1 é1 ù 1 1 1 1
= ê cos 3 ( 20° )ú = cos 60° = ´ = . sin ( x + y) + sin ( x - y ) (a + b) + (a - b)
=
2 ë4 û 8 2 8 16
sin ( x + y) - sin ( x - y ) (a + b) - (a - b)
22. (c) Let Sn = cos n q + sin n q ; S4 = cos 4 q + sin 4 q
2sin x cos y 2a
Þ =
S6 = cos6 q + sin 6 q = (cos2 q)3 + (sin 2 q)3 2cos x sin y 2b
= (cos2 q + sin 2 q)(cos 4 q + sin 4 q – cos 2 q ´ sin 2 q) [using sin (A + B) and sin (A – B)]

[
Mathematics S-M-9

26. (b) Q a cos x + b sin x = c we have, cos q = cos a Þ q = 2np ± a


æ 1 – tan 2 ( x / 2) ö \ For general value of q , cos 2q = 0
æ 2 tan ( x / 2) ö p
Þ aç ÷ + bç =c Þ cos 2q = cos p
è 1 + tan 2 ( x / 2) ø è 1 + tan 2 ( x / 2) ÷ø 2
Þ 2q = 2 m p ±
p 2
Þ q = mp ± or 2 cos q + 1 = 0 ;
2 æ xö æ xö 4
Þ (a + c ) tan çè ÷ø – 2b tan çè ÷ø + (c – a ) = 0
2 2 -1 2p
Þ cos q = Þ cos q = cos
æ aö æ bö 2b 2 3
\ tan ç ÷ + tan ç ÷ = 2p
è 2ø è 2 ø (a + c) So, q = 2mp ±
3
æ aö æ bö c - a
and tan ç ÷ tan ç ÷ = æ pö æ pö
è 2ø è 2ø a + c 29. (b) Let, y = sin ç x + ÷ + cos ç x + ÷
è ø6 è ø 6
æ aö æ bö
tan ç ÷ + tan ç ÷ é 1 æ pö 1 æ pö ù
æ a + bö è 2ø è 2ø = 2 ê sin ç x + ÷ + cos ç x + ÷ ú
Now, tan ç =
è 2 ÷ø æ aö æ bö ë 2 è 6ø 2 è 6øû
1 – tan ç ÷ tan ç ÷
è 2ø è 2ø
é p æ pö p æ pö ù
2b = 2 êsin sin ç x + ÷ + cos cos ç x + ÷ ú
a +c = b=
ë 4 è 6ø 4 è 6øû
= Independent of c
æ c – aö a
1– ç
è a + c ø÷ é æ p pö ù é æ p öù
= 2 êcos ç x + – ÷ ú = 2 êcos ç x – ÷ ú
Also, ë è 6 4 ø û ë è 12 øû

– (a 2 – b 2 ) £ a cos x + b sin x £ (a 2 + b2 ) p
Þ x– =0 [Q y to be max.]
12
\ – (a + b ) £ c £ (a + b )
2 2 2 2
p
27. (b) tan2 q sec2q ( cot2q – cos2q) Þ x=
12
= sec2 q (tan2q cot2q – tan2q cos2q) 30. (b) If a is the smallest positive angle for which sin a = x,
æ sin q 2 ö then b = p – a, g = 2p + a and d = 3p – a
2
= sec q çç 1 - cos 2 q ÷ = sec 2 q (1 - sin 2 q) a b g d
2 ÷ So, 4 sin + 3 sin + 2 sin + sin
è cos q ø 2 2 2 2
= sec2 q. cos2q =1
28. (a) Given cos q + cos 2q + cos 3q = 0 a a a a
= 4sin + 3cos - 2sin - cos
Þ (cos 3q + cos q) + cos 2q = 0 2 2 2 2
Þ 2 cos 2q. cos q + cos 2q = 0 a a
= 2 sin + 2 cos = 2 1 + sin a = 2 1 + x
Þ cos 2q.(2 cos q + 1) = 0 2 2

Speed Test-63

1. (d) Since P(1) : 2 < 1 is false Now, 102(k + 1) – 1 + 1 = 102k – 1 102 + 1


P(2) : 22 < 1× 2 is false = (11l – 1)100 + 1 [Using (i)]
P(3) : 23 < 1× 2 × 3 is false = 1100l – 99 = 11(100l – 9) = 11m,
P(4) : 24 < 1× 2 × 3 × 4 is true where m = 100l – 9 Î N
2. (a) For n = 1, P(1) : 65 + k is divisible by 64. Þ 102(k + 1) – 1 + 1 is divisible by 11
Thus k, should be –1 Þ P(k + 1) is true.
Since 65 – 1 = 64 is divisible by 64. Thus, P(k + 1) is true, whenever P(k) is true.
3. (a) Let P(n) be the statement given by Hence, by the principle of mathematical induction,
P(n) : 102n – 1 + 1 is divisible by 11 P(k) is true for all n Î N, i.e. 102n – 1 + 1 is divisible
For n = 1, P(1) : 10(2 × 1) – 1 + 1 = 11, by 11 for all n Î N.
which is divisible by 11. 4. (c) Since P(5) is true and P(k + 1) is true, whenever P (k)
So, P(1) is true. is true.
Let P(k) be true, i.e. 102k – 1 + 1 is divisible by 11 5. (b) When k = 1, LHS = 1 but RHS = 1 + 10 = 11
Þ 102k – 1 + 1 = 11l, for some l Î N … (i) \ T(1) is not true
We shall now show that P(k + 1) is true. For this, we Let T(k) is true.
have to show that ..... + (2 k - 1) = k 2 + 10
EBD_7504
S-M-10 SOLUTIONS

Now, 1 + 3 + 5 + ..... + (2k - 1) + (2k + 1) [2(m + 1)]!


<
2 2
= k + 10 + 2k + 1 = ( k + 1) + 10 [(m + 1)!]2
Hence, for n ³ 2, P(n) is true.
\T(k +1) is true. 13. (a) x2n–1 + y2n–1 is always contain equal odd power. So it
That is T(k) is true Þ T(k + 1) is true. is always divisible by x + y .
But T(n) is not true for all n Î N , as T(1) is not true. 14. (a) It can be proved with the help of mathematical induction
6. (b) S(k) = 1+3+5+...+(2k – 1) = 3 + k2 n
that < a(n) £ n.
S (1) :1 = 3 + 1, which is not true 2
Q S (1) is not true. 200
\ P.M.I cannot be applied \ < a(200) Þ a(200) > 100 and
Let S(k) is true, i.e. 2
a(100) £ 100.
1 + 3 + 5.... + (2 k - 1) = 3 + k 2 15. (d) Let the given statement be P (n), then
Þ 1 + 3 + 5.... + (2k - 1) + 2k + 1 P (1) Þ 21 > 12 which is true
P (2) Þ 22 > 22 which is false
= 3 + k 2 + 2k + 1 = 3+ ( k + 1) 2 P (3) Þ 23 > 32 which is false
P (4) Þ 24 > 42 which is false
\ S (k ) Þ S (k + 1) P (5) Þ 25 > 52 which is true
7. (b) Check through option, the condition 2n (n - 1)! < nn P (6) Þ 26 > 62 which is true
\ P (n) is true when n ³ 5
is satisfied for n > 2
8. (a) Putting n = 2 in 32n –2n + 1 then, 16. (b) n(n 2 - 1) = (n - 1)(n)( n + 1)
32´ 2 - 2 ´ 2 + 1 = 81- 4 + 1 = 78, which is divisible It is product of three consecutive natural numbers, so
by 2. according to Langrange’s theorem it is divisible by 3!
9. (a) The product of two consecutive numbers is always i.e., 6
even. 17. (b) For n = 1, we have
49n + 16n + l = 49 + 16 + l = 65 + l
10. (b) a1 = 7 < 7. Let am < 7 = 64 + ( l + 1), which is divisible by 64 if l = – 1
For n = 2, we have
Then am + 1 = 7 + am Þ a2m + 1 49n + 16n + l = 492 + 16 × 2 + l = 2433 + l
= 7 + am < 7 + 7 < 14. = 64 × 38 + ( l + 1), which is divisible by 64 if l = – 1
Þ a m + 1 < 14 < 7; So by the principle of Hence, l = – 1
mathematical induction an < 7 " n. 18. (a) Let P (n) = n (n2 – 1) then
P(1) = 1 (0) = 0 which is divisible by every n Î N
11. (c) Check through option, the condition 3n > n3 is true P (3) = 3 (8) = 24 which is divisible by 24 and 8
when n ³ 4 . P (5) = 5 (24) = 120 which is divisible by 24 and 8
Hence P (n) is divisible by 24.
4n (2n)! 19. (c) Let n = 1, then option (a), (b) and (d) eliminated. Only
12. (d) Let P(n) : <
n + 1 (n !)2 option (c) satisfied.
For n = 2, 20. (a) P(n) : 2.7n + 3.5n – 5 is divisible by 24.
For n = 1,
4 2
4! 16 24 P(1) : 2.7 + 3.5 – 5 = 24, which is divisible by 24.
P(2) : < Þ <
2 + 1 (2) 2
3 4 Assume that P(k) is true,
which is true. i.e. 2.7k + 3.5k – 5 = 24q, where q Î N ... (i)
Let for n = m ³ 2, P(m) is true. Now, we wish to prove that P(k + 1) is true whenever
P(k) is true, i.e. 2.7k + 1 + 3.5k + 1 – 5 is divisible by 24.
4m (2m)! We have,
i.e. <
m + 1 ( m !) 2 2.7k + 1 + 3.5k + 1 – 5 = 2.7k . 71 + 3.5k . 51 – 5
= 7[2.7k + 3.5k – 5 – 3.5k + 5] + 3.5k . 5 – 5
4m+1 4m 4(m + 1) = 7[24q – 3.5k + 5] + 15.5k – 5
Now, = · = (7 × 24q) – 21.5k + 35 + 15.5k – 5
m+2 m +1 m + 2
= (7 × 24q) – 6.5k + 30 = (7× 24q) – 6(5k – 5)
(2m)! 4(m + 1) = (7 × 24q) – 6(4p) [ Q (5k – 5) is a multiple of 4]
< ·
(m !) 2 (m + 2) = (7 × 24q) – 24p = 24(7q – p)
= 24 × r; r = 7q – p, is some natural number ... (ii)
(2m)!(2m + 1)(2m + 2)4(m + 1)(m + 1) 2 Thus, P(k + 1) is true whenever P(k) is true.
=
(2m + 1)(2m + 2)(m !)2 (m + 1) 2 ( m + 2) Hence, by the principle of mathematical induction,
P(n) is true for all n Î N.
[2(m + 1)]! 2(m + 1)2
=
[(m + 1)!]2
Mathematics S-M-11

1 1 1 23. (d) Let the statement P(n) be defined as


21. (b) Let P(n) : + + ..... +
1× 2 × 3 2 × 3 × 4 n ( n + 1) (n + 2) n5 n3 7n
P(n) : + + is a natural number for all n Î N.
n (n + 3) 5 3 15
= Step I : For n = 1,
4 (n + 1) (n + 2)
1 1 7
For n = 1, P(1) : + + =1ÎN
5 3 15
1 1 Hence, it is true for n = 1.
L.H.S. = =
1× 2 ×3 6 Step II : Let it is true for n = k,
1(1 + 3) 1 k5 k3 7k
and R.H.S. = = i.e. + + =lÎN ... (i)
4 (1 + 1) (1 + 2 ) 6 5 3 15
\ P(1) is true. Step III : For n = k + 1,
Let P(k) is true, then
(k + 1)5 (k + 1)3 7 ( k + 1)
1 1 1 + +
P(k) : + + ..... + 5 3 15
1× 2 × 3 2 × 3 × 4 k ( k + 1) (k + 2 )
1 5
k (k + 3) = (k + 5k4 + 10k3 + 10k2 + 5k + 1)
= ... (i) 5
4 (k + 1) (k + 2)
1 3 7 7
For n = k + 1, + (k + 3k2 + 3k + 1) + k+
3 15 15
1 1
P(k + 1) : + + ..... æ k5 k3 7 ö
1× 2 × 3 2 × 3 × 4 = ç + + k ÷ + (k4 + 2k3 + 3k2 + 2k)
è 5 3 15 ø
1 1
+ +
k (k + 1) (k + 2) (k + 1) (k + 2) (k + 3) 1 1 7
+ + +
5 3 15
( k + 1) (k + 4) = l + k4 + 2k3 + 3k2 + 2k + 1
=
4 (k + 2 ) (k + 3) [using equation (i)]
1 1 which is a natural number, since l k Î N.
L.H.S. = + + ..... Therefore, P(k + 1) is true, when P(k) is true.
1× 2 × 3 2 × 3 × 4 Hence, from the principle of mathematical induction,
1 1 the statement is true for all natural numbers n.
+ + 24. (c) For n = 1, we have
k (k + 1) (k + 2) (k + 1) (k + 2) (k + 3) xn+1 + (x + 1)2n–1 = x2 + (x + 1) = x2 + x + 1,
k (k + 3) 1 which is divisible by x2 + x + 1
= + For n = 2, we have
4 (k + 1) ( k + 2 ) (k + 1) (k + 2) (k + 3) xn+1 + (x + 1)2n–1 = x3 + (x + 1)3 = (2x + 1) (x2 + x + 1),
[from (i)] which is divisible by x2 + x + 1.
Hence, option (c) is true.
(k + 1)2 (k + 4) (k +1)(k + 4) 25. (d) Let P(n) be the statement given by
= = = R.H.S.
4 (k + 1)(k + 2)(k + 3) 4 (k + 2)(k + 3) P(n) : 52n + 2 – 24n – 25 is divisible by 576.
Hence, P(k + 1) is true. For n = 1,
Hence, by principle of mathematical induction for all P(1) : 52 + 2 – 24 – 25 = 625 – 49 = 576,
n Î N, P(n) is true. which is divisible by 576.
22. (c) Let P(n) : 7n – 3n is divisible by 4. \ P(1) is true.
For n = 1, Let P(k) be true,
P(1) : 71 – 31 = 4, which is divisible by 4. Thus, P(n) i.e. P(k) : 52k + 2 – 24k – 25 is divisible by 576.
is true for n = 1. Þ 52k + 2 – 24k – 25 = 576l ... (i)
Let P(k) be true for some natural number k, We have to show that P(k + 1) is true,
i.e. P(k) : 7k – 3k is divisible by 4. i.e. 52k + 4 – 24k – 49 is divisible by 576
2k + 4
We can write 7k – 3k = 4d, where d Î N ... (i) Now, 5 – 24k – 49
Now, we wish to prove that P(k + 1) is true whenever = 52k + 2 + 2 – 24k – 49 = 52k + 2 . 52 – 24k – 49
P(k) is true, i.e. 7k + 1 – 3k + 1 is divisible by 4. = (576l + 24k + 25) . 25 – 24k – 49 [from (i)]
Now, 7(k + 1) – 3(k + 1) = 7(k + 1) – 7.3k + 7.3k – 3(k + 1) = 576.25l + 600k + 625 – 24k – 49
= 7(7k – 3k) + (7 – 3)3k = 7(4d) + 4.3k [using (i)] = 576.25l + 576k + 576
= 4(7d + 3k), which is divisible by 4. = 576{25l + k + 1}, which is divisible by 576.
Thus, P(k + 1) is true whenever P(k) is true. \ P(k + 1) is true whenever P(k) is true.
Î N.
EBD_7504
S-M-12 SOLUTIONS

26. (a) Let us write the statement 1


1 1 1 1 n (1 + 2 + 3 + ..... + k) + (k + 1) < (2k + 1)2 + (k + 1)
+ + + ..... + = 8
P(n) :
1.2 2.3 3.4 n(n + 1) n + 1 [using equation (i)]
1
=
1 1 1
Þ = is true thus P(n) is
(2k + 1)2 k + 1 (2k + 1) + 8k + 8
2
we note that P(1) : = + =
1.2 1 + 1 2 2 8 1 8
true for n = 1 2
Suppose that P(k) is true for some natural number ‘k’ 4k + 1 + 4k + 8k + 8
=
1 1 1 1 k 8
+ + + ..... + = ......(1)
4k 2 + 12k + 9 ( 2k + 3)
2
1.2 2.3 3.4 k ( k + 1) k + 1
= =
1 1 1 1 1 8 8
Now, + + + ..... + +
k (k + 1) ( k + 1)( k + 2) 2
ëé 2 (k + 1) + 1ùû
1.2 2.3 3.4 (2k + 2 + 1)2
= =
k 1 8 8
= + [From (1)]
k + 1 (k + 1)(k + 2) 2
ëé2 ( k + 1) + 1ùû
Þ 1 + 2 + 3 + ..... + k + (k + 1) <
k (k + 2) + 1 k 2 + 2k + 1 8
= =
(k + 1)(k + 2) (k + 1)(k + 2) Therefore, P(k + 1) is true when P(k) is true.
Hence, from the principle of mathematical
(k + 1)2 k +1 k +1 induction, the statement is true for all natural
= = = numbers n.
(k + 1)(k + 2) k + 2 (k + 1) + 1
II. Let the statement P(n) be defined as
Thus P(k + 1) is true whenever P(k) is true. Hence. by the P(n) : n(n + 1) (n + 5) is a multiple of 3.
principle of mathematical induction P(n) is true for all natural Step I : For n = 1,
numbers. P(1) : 1(1 + 1) (1 + 5) = 1 × 2 × 6 = 12 = 3 × 4,
27. (d) Let P(n) be the statement given by which is a multiple of 3, that is true.
P(n) : 32n when divided by 8, the remainder is 1. Step II : Let it is true for n = k,
or P(n) : 32n = 8l + 1 for some l Î N i.e. k(k + 1) (k + 5) = 3l
For n = 1, Þ k(k2 + 5k + k + 5) = 3l
P(1) : 32 = (8 × 1) + 1 = 8l + 1, where l = 1 Þ k3 + 6k2 + 5k = 3l ... (i)
\ P(1) is true. Step III : For n = k + 1, (k + 1) (k + 1 + 1) (k + 1 + 5)
Let P(k) be true. = (k + 1) (k + 2) (k + 6) = (k2 + 2k + k + 2) (k + 6)
Then, 32k = 8l + 1 for some l Î N ... (i) = (k2 + 3k + 2) (k + 6)
We shall now show that P(k + 1) is true, for which = k3 + 6k2 + 3k2 + 18k + 2k + 12
we have to show that 32(k + 1) when divided by 8, the = k3 + 9k2 + 20k + 12
remainder is 1. = (3l – 6k2 – 5k) + 9k2 + 20k + 12
Now, 32(k + 1) = 32k . 32 = (8l + 1) × 9 [Using (i)] [using equation (i)]
= 72l + 9 = 72l + 8 + 1 = 8(9l + 1) + 1 = 3l + 3k2 + 15k + 12
= 8m + 1, where m = 9l + 1 Î N = 3(l + k2 + 5k + 4), which is a multiple of 3.
Þ P(k + 1) is true. Therefore, P(k + 1) is true when P(k) is true.
Thus, P(k + 1) is true, whenever P(k) is true. Hence, from the principle of mathematical
Hence, by the principle of mathematical induction induction, the statement is true for all natural
P(n) is true for all n Î N. numbers n.
28. (c) I. Let the statement P(n) be defined as Hence, both the statements are true.
1 1 1 1
P(n) : 1 + 2 + 3 + ..... + n < (2n + 1)2 29. (a) Statement-1 : Let P(n) : + + ..... + > n
8
1 2 n
Step I : For n = 1,
For n = 2,
1 1
P(1) : 1 < (2.1 + 1)2 Þ 1 < × 32 1 1
8 8 P(2) : +
> 2, which is true.
1 2
9 Assume P(k) is true,
Þ 1< , which is true.
8
Step II : Let it is true for n = k. 1
1 1
i.e. + + ..... + > k ... (i)
1
1 2 k
1 + 2 + 3 + ..... + k < (2k + 1)2 ... (i) For n = k + 1, we have to show that
8
Step III : For n = k + 1, 1 1 1 1 1
+ + > k + 1 ... (ii)
k k +1
Mathematics S-M-13

æ 1 1 1 1 ö 1
From (iii) and (iv),
L.H.S. = ç + + + ..... + ÷ + ... (iii) 1 1 1 1 1
è 1 2 3 kø k +1 + + + ..... + + > k
Statement-2 : For n = k, 1 2 3 k k +1

k (k + 1) < k + 1 > k +1
1
+ [Using (i)]
k +1
Þ k k +1 < k +1 k +1
Hence, (ii) is true for n = k + 1
Þ k < k +1 Hence, P(n) is true for n ³ 2
So, Statement-1 and Statement-2 are correct and
Q k + 1 > k for k ³ 2 Statement-2 is the correct explanation of Statement-1.
30. (b) Let P(n) be the statement given by
k P(n) : 41n – 14n is a multiple of 27
Þ 1>
k +1 For n = 1,
i.e. P(1) = 411 – 141 = 27 = 1 × 27,
Þ k>
k
k +1
, (Multiplying by k ) which is a multiple of 27.
\ P(1) is true.
Let P(k) be true, i.e. 41k – 14k = 27l ... (i)
Þ
(k + 1) – 1 Þ k > k +1 –
1 For n = k + 1,
k> 41k + 1 – 14k + 1 = 41k 41 – 14k 14
k +1 k +1
= (27l + 14k) 41 – 14k 14 [using (i)]
1 = (27l × 41) + (14k × 41) – (14k × 14)
Þ k+ > k +1 ... (iv) = (27l × 41) + 14k (41 – 14)
k +1 = (27l × 41) + (14k × 27)
= 27(41l + 14k),
which is a multiple of 27.
Therefore, P(k + 1) is true when P(k) is true. Hence,
from the principle of mathematical induction, the
statement is true for all natural numbers n.

Speed Test-64

2n (a + b)3 - 3ab(a + b )
é (1 + i ) ù =
1. (b) Given: ê ú =1 ...(i)
ab
ë (1 - i ) û
p(3q - p 2 )
1+ i =
Let A = ...(ii) q
1- i
( p 2 - 3q) q
1+ i 1+ i 2i \ The given expression = =-
So A = ´ = p(3q - p 2 ) p
1 - i 1 + i 1 - i2
q
Now i = – 1 and i4 = 1
2
2i 3. (a) Here, a + b = p and ab = q
\ A= =i
2 æ 2
2 a
ö æ b2 ö
So, from equations (i) and (ii), Let a1 = a çç - b ÷ and b1 = b 2 ç
÷ ç
- a ÷ then
÷
(i)2n = 1 = i4 è b ø è a ø
Þ 2n = 4 \ n = 2. Sum of the roots
2. (a) Since, a and b are the roots of the equation
x2 + px + q = 0, therefore æ a2 ö æ b2 ö
a1 + b1 = a 2 ç - b÷ + b2 ç - a÷
a + b = – p and ab = q ç b ÷ ç a ÷
è ø è ø
Now, (wa + w2b)(w2a + wb)
= a2 + b2 + (w4 + w2)ab (Q w3 = 1)
2 2
= a + b – ab.
2
(Q w + w2 = –1)
æ a2 b2 ö
= (a 2 - b 2 ) ç
ç b
- ÷ = 1 (a - b)(a + b) a 3 - b3
a ÷ø ab
( )
= (a + b) – 3ab è
= p2 – 3q
a 2 b2 a 3 + b3
=
1
ab
(
(a + b )(a - b )2 a 2 + b 2 + ab )
Also, + =
b a
}{(a + b) 2
- ab}
EBD_7504
S-M-14 SOLUTIONS

1 2 2 = cos 4q cos 3q – sin 4q sin 3q


= q ( p ) {p - 4q}{p - q} + i (sin 4q cos 3q + sin3q cos 4q)
Product of the roots = cos (4q + 3q) + i sin (4q + 3q) = cos 7q + i sin 7q
æ a2 ö æ b2 ö 3 | z |2 3 3z . z
a1b1 = a 2 ç - b ÷ ×b2 ç - a÷ 8. (d) z 3 + =0 Þz + = 0 Þ z 3 + 3z = 0
ç b ÷ ç a ÷ z z
è ø è ø
Let z = reiq
(
= - ab a 2 - b 2 ) = -ab (a + b) (a - b)
2 2 2
Þ r3ei3q + 3re–iq = 0
= - ab ( a + b ) 2 [(a + b) - 4ab ] = -p 2q (p 2 - 4q )
2 Þ ei4q =-1 [Q r = 3]
\ The required equation is Þ cos 4q + i sin 4q = -1
p
( )( )
x 2 - p 2 - 4q p 2 - q x - p 2 q p 2 - 4q = 0
q
( ) Þ cos 4q = -1 ...(i)
Now 0 £ q < 2p
or qx 2 - p(p 2 - q) (p 2 - 4q) x - p 2 q 2 (p 2 - 4q) = 0 Þ 0 £ 4q < 8p
\ q = p, 3p, 5p, 7p
4. (d) The given equation is m 2 x 2 + (2 m - m 2 ) x + 3 = 0 9. (c) Given z = x + iy is a variable complex number
2m - m 2
m-2 3 z -1 p
\a + b = - 2
= and ab = 2 where arg
z +1 4
=
m m m
Now, consider
a b 4 a 2 + b 2 4 (a + b) 2 - 2ab 4
Now + = Þ = Þ = z - 1 x + iy - 1 x - iy + 1
b a 3 ab 3 ab 3 = ´ (By rationalizing)
Substituting the values, we get z + 1 x + iy + 1 x - iy + 1
2 [ x + ( iy - 1)][ x - (iy - 1)] x 2 - (iy - 1)2
æ m - 2ö 3 = =
çè ÷ - 2. 2 4 ( x + 1 + iy )( x + 1 - iy ) ( x + 1) 2 - i 2 y 2
m ø m =
3 3 z - 1 x 2 + y 2 - 1 + 2iy
m2 =
z + 1 x2 + y2 + 1 + 2x
m 2 - 4m + 4 - 6 4
Þ = Þ m 2 - 4m - 6 = 0
3 3 x2 + y 2 - 1 2 yi
= +
m1 and m2 are roots of this equation, therefore 2 2
x + y + 2x +1 x + y2 + 2x + 1
2
m1+ m2 = 4 and m1m2 = –6
m12 m 22 m13 + m 23 é 2y ù
The given expression is, + = ê 2 2 ú
æ z - 1ö x + y + 2x +1 ú
m 2 m1 m1m 2 \ arg ç ÷ = tan -1 ê
è z + 1ø ê x2 + y2 - 1 ú
(m + m2 )3 - 3m1m2 (m1 + m2 ) ê 2 ú
= 1 2
ë x + y + 2x +1 û
m1m2
(4)3 - 3.( -6).(4) 68 æ 2y ö
= =- = tan -1 ç 2 ÷
-6 3 è x + y 2 - 1ø
z1 3 +i 3 æ3+ 3 ö æ3- 3 ö
5. (a) = = çç ÷÷ + çç ÷÷ i æ z - 1ö p
But given arg ç =
z2 3 +i è 4 ø è 4 ø è z + 1÷ø 4
which is represented by a point in first quadrant.
2y p 2y p
2 \ tan -1 Þ 2= = tan
4 ( 2 - i ) ± 16 ( 2 - i ) + 8 (1 + i )( 5 + 3i) 2 2
x + y -1 4 2
x + y -1 4
6. (a) Roots =
4 (1 + i ) 2y é p ù
4-i -i 3 - 5i -1 - i Þ 2 2
=1 êQ tan 4 = 1ú
= or = or x + y -1 ë û
1+ i 1+ i 2 2
Þ x2 + y2 - 1 = 2 y Þ x2 + y2 - 2 y = 1
3 - 5i 9 + 25 17 -1 - i 1 +1 1
= = = and = = 10. (b) Let a > 0, b > 0, c > 0
2 4 2 2 4 2 Given equation ax2 + bx + c = 0
(cos q + i sin q )4 -b ± b2 - 4ac
7. (b) 3 = (cosq + i sinq)4 (cosq – i sinq)–3 we know that D = b2 – 4ac and x =
(cos q - i sin q) 2a
= (cos 4 q + i sin 4q) {cos (–q) + i sin (– q)}–3 Let b2 - 4ac > 0, b > 0
= (cos 4 q + i sin 4q
= (cos 4q + i sin 4q - 4ac < b 2
Mathematics S-M-15

Þ Roots are negative a b g


2 14. (b) + + = 1+ i
- b ± i 4ac - b a b c
Let b 2 - 4 ac < 0 , then x =
2a Squaring both side,
Þ roots are imaginary and have negative real part.
a2 b2 g2 æ ab bg ga ö
(Qb > 0) . + + +2ç + + = 2i
a 2
b 2
c è ab bc ca ÷ø
2
11. (c)
y 2
C (z+wz) a b2 g2 æ cab + abg + bga ö
i.e. + + +2ç ÷ø = 2i
a 2
b 2
c è
2 abc
z)
(w

A (z)
2
a b2 g2 æ a b c ö
B

x \ 2
+ 2
+ 2
= 2i çè\ a + b + g = 0÷ø
O a b c
15. (
(a) Since 7 + 4 3 7 - 4 3 = 1, )( )
\ The given equation becomes
1 x2 -4 x + 3 x2 -4 x + 3
Let the point A represents z and B represents wz, then y+ = 14 where y = ( 7 - 4 3 ) + 7+4 3
2p y
ÐAOB = . If C represents z + wz then OACB is a
3 Þ y2 – 14y + 1 = 0 Þ y = 7 ± 4 3
parallelogram.
\ Area of D ABC = Area of D OAB Now y = 7 + 4 3 Þ x2 – 4x + 3 = –1 Þ x = 2, 2

1 2p 1 3 3 Also y = 7 – 4 3 Þ x2 – 4x + 3 = 1 Þ x = 2 ± 2
= |z| |wz| sin = |z| |z| [|wz| = |z|] = | z |2 16. (a) From the given equations
2 3 2 2 4
Now given that area of D ABC = 4 3 . b c m n
a + b = - , ab = , g + d = - , gd =
a a l l
3
\ | z |2 = 4 3 bm
4 Now, (ag + bd) + (ad + bg ) = (a + b)( g + d) =
Þ | z | = 4 . That is z lies on a circle or radius 4 al
2 2 2 2
12. (b) Z and (ag + bd)(ad + bg ) = a gd + abg + abd + b gd
= (a 2 + b 2 ) gd + ( g 2 + d 2 ) ab
|z| | z –2 |
{ } {
= (a + b) 2 - 2ab gd + (g + d)2 - 2gd ab }
æ b 2 2c ö n æ m 2 2n ö c n (b 2 - 2ac) c(m 2 - 2nl)
O
2
2 =ç - ÷ +ç - ÷ = +
ç a 2 a ÷ l ç l2 l ÷a a 2l al2
| z | + |z – 2| ³ 2 è ø è ø
If origin, z and 2 represent the vertices of a triangle then its \ The required equation is,
side lenghts are given by |z|, |z–2| and 2. x 2 - {(ag + bd) + (ad + bg )} x + (ag + bd)(ad + bg ) = 0
Now in a triangle sum of two sides ³ third side
\| z | + | z - 2 | ³ 2 bm ln( b 2 - 2 ac ) + ac (m 2 - 2 nl )
Þ x2 - x+ =0
\ Minimum value of |z|+|z–2| is 2 al a 2 l2
Also |z| + |z – 2| = |z| + |2 – z| ³ |z + 2 – z| = 2
Þ a 2 l 2 x 2 - ablmx + b 2 ln + acm 2 - 4ac ln = 0
[|z1| + |z2| ³ |z1 + z2|]
13. (b) We have two equations -1 + i 3 -1 - 3 i
17. (c) Let = w then = w2
| z | -4 = 0 and | z - i | - | z + 5i |= 0 2 2
Putting z = x + iy, these equations become where w = cube root of unity
100 100
| x + iy |= 4 i.e. x 2 + y 2 = 16 ...(1) æ -1 + - 3 ö æ -1- - 3 ö
Consider ç ÷ +ç ÷
ç 2 ÷ ç 2 ÷
and | x + iy - i |=| x + iy + 5i | è ø è ø
100 100
or x 2 + ( y - 1) 2 = x 2 + ( y + 5) 2 i.e y = – 2 ...(2) æ -1 + 3 i ö æ -1 - 3 i ö
=ç ÷ +ç ÷
Hence the complex numbers z satisfying the given equations è 2 ø è 2 ø
are z1 = (2 3 , - 2) and z = (-2 3 , - 2) = w100 + w 200 = w + w 2 = -1
2
that is, z1 = 2 3 - 2i, z =0)
EBD_7504
S-M-16 SOLUTIONS

= a2b2 + 2(bb + c)
3 x 2 + 9 x + 17 = a2b2 + 2a2b2 = 3a2b2 > 0 [Using (ii)]
18. (b) y=
3x 2 + 9 x + 7 Thus, f (x) is a polynomial such that
3x 2 ( y - 1) + 9 x( y - 1) + 7 y - 17 = 0 f (a) < 0 and f (b) > 0. Therefore, there exists g satisfying
D ³ 0 Q x is real a < g < b such that f ( g ) = 0
23. (d) Given equation is
81( y - 1) 2 - 4 ´ 3( y - 1)(7 y - 17) ³ 0 (x – a) (x – b) + (x – b) (x – c) + (x – c) (x – a) = 0
Þ ( y - 1)( y - 41) £ 0 Þ 1 £ y £ 41 Þ (x2 – bx – ax + ab) + (x2 – cx – bx + bc)
+ (x2 – ax – cx + ac) = 0
\ Max value of y is 41 Þ [x – (a + b)x + ab] + [x2 – (b + c)x + bc]
2

æ 1 2 ö æ 3 + 4i ö + [x2 – (a + c)x + ac] = 0


19. (d) Let z = ç +
è 1 - 2i 1 + i ÷ø çè 2 - 4i ÷ø
2
Þ 8x – 2(a + b + c)x + ab + bc + ca = 0
Since roots are equal
é 1 + i + 3 - 6i ù é 3 + 4i ù \ We have
=ê úê ú b2 – 4ac = 0 Þ b2 = 4ac
ë (1 - 2i )(1 + i ) û ë 2 - 4i û
4(a + b + c)2 = 12(ab + bc + ca)
é 4 - 5i ùé 3 + 4i ù é 32 + i ù (a + b + c)2 = 3(ab + bc + ca)
= ê =ê ú a2 + b2 + c2 = ab + bc + ca.
ë 3 - i úû
ëê 2 - 4i ûú ë 2 - 14i û
24. (a) As a + ib = 1 Þ a 2 + b 2 = 1
32 + i 2 + 14i 64 + 448i + 2i - 14
= ´ =
2 - 14i 2 + 14i 4 + 196 1 + b + ai (1 + b + ai)2 (1 + b) 2 – a 2 + 2(1 + b)ai
= = =
50 + 450i 1 9 1 + b – ai (1 + b – ai)(1 + b + ai) (1 + b) 2 + a 2
= = + i
200 4 4 (1 – a 2 ) + b 2 + 2b + 2ai + 2abi b 2 + b 2 + 2b + 2ai + 2abi
20. (a) Discriminant of the equation = 2 2
=
1 + (a + b ) + 2b 1 + 1 + 2b
2 a 2 x 2 - 2 abx + b 2 = 0 is - 4 a 2 b 2 < 0 and that of the 2
equation b + b + ai + abi b(1 + b ) + a (1 + b)i
= = = b + ai
1+ b 1+ b
p 2 x 2 +3pqx +q 2 = 0 is 5p 2 q 2 > 0 25. (b) Given, x2 + 2px + q = 0
There cannot be any common root. \ a + b = – 2p …(i)
21. (d) By Coni’s formula B and D is obtained by ab = q …(ii)
y And ax2 + 2bx + c = 0
B 1 2b
\ a+ =– …(iii)
b a
60° a c
C A(2, 1)
and = …(iv)
(0, 1) 60° b a
x
Now, (p2 – q)(b2 – ac)
O
D éæ 1ö
2
ù
ê a+ ú
éæ a + b ö 2
ù ç b ÷ aú 2
æ pö = êç ÷ - ab ê
ú çè ÷ - a
z -i BC çè ± 3 ÷ø i ë è -2 ø û êë 2 ø b úû
= e
z + i - i CD
2
(a - b)2 æ 1ö
æ p pö = a - ÷ · a2 ³ 0
Þ z - i = 2 ç cos ± i sin ÷ (Q BC = CD)
è 16 çè bø
3 3ø
\ Statement 1 is true.
Þ z = i + 1 ± 3i = 1 + i(1 ± 3)
æ a + bö a
22. (d) a is a root of a2x2 + bx + c = 0 Again, now pa = – çè ÷ a = - (a + b)
Þ a2a2 + ba + c = 0 2 ø 2
Þ ba + c = – a2 a2 ...(i) aæ 1ö
b is a roots of a2x2 – bx – c = 0 and b = - ç a + ÷
2è bø
Þ bb + c = a2b2 ...(ii)
Let f (x) = a2x2 + 2bx + 2c. Then, 1
f (a) = a2a2 + 2ba + 2c = a2a2 + 2(ba + c) Since, pa ¹ b Þ a + ¹a+b
b
Þ f (a) = a2a2 – 2a2a2 [Using (i)]
= – a2a2 < 0 Þ b2 ¹ 1, b ¹ {–1, 0, 1}, which is correct,
and f (b) = a2b2 + 2bb + 2c a
a ¹ aab
b
Mathematics S-M-17

Þ
æ 1ö
a çb - ÷ ¹ 0
28. (d) ( z – 1) ( z – 5) + ( z – 1) (z – 5)
è bø
= 2 Re éë( z – 1) ( z – 5 ) ùû
1
Þ a ¹ 0 and b – ¹0 éëQ z1 z2 + z 2 z1 = 2 Re ( z l z 2 ) ùû
b
= 2 Re[(1 + i) (–3 – i)] = 2(–2) = –4
Þ b ¹ {–1, 0, 1}
[Given z = 2 + i]
Statement 2 is true.
29. (d) Given that a, b are the roots of the given equation
Both Statement 1 and Statement 2 are true. But
2x2 + 6x + b = 0, then
Statement 2 does not explain statement 1.
1 + 2w + 3w2 2 + 3w + 3w2 6
+ Sum of the roots = a + b = – =–3
26. (b) We have, 2
2 + 3w + w2 3 + 3w + 2w2
b
We know that w3 = 1 And product of the roots = ab =
2
w3 + 2w + 3w2 2w3 + 3w + 3w2 D = B2 – 4AC
+ D = 62 – 4 × 2 × 6
2
2 + 3w + w 3 + 3w + 2w2 D = 36 – 8b
w(w2 + 2 + 3w) w(2w2 + 3 + 3w) D>0 (Q b < 0)
= + = w + w =2w
2 + 3w + w2 (3 + 3w + 2w2 ) a b a 2 + b2 (a + b) 2 – 2ab
Then, + = =
27. (c) Since, a and b be the roots of the equation b a ab ab
ax2 + bx + c = 0, then
b
b c (–3) 2 – 2 ´
a + b = - and ab = = 2 = 9 – b = 18 – 2 b = 18 – 2b
a a b b b b b
Now, sum of roots = |a| + |b| 2 2
=–a–b (Q b < a < 0) 18
æ bö b
= – 2 < -2 (Q b < 0)
= –ç- ÷ = (Q |a| + |b| > 0) b
è aø a
a b
\ + < –2
c b a
and product of roots = |a| |b| =
a
(cos x + i sin x ) (cos y + i sin y )
Hence, required equation is 30. (a) Given
(cot u + i ) (1 + i tan v )
b c
x2 – x+ =0 (cos x + i sin x ) (cos y + i sin y )
a a =
Þ |a| x2 – |b| x + |c| = 0 (cos u + i sin u)(cos v + i sin v )
= sin u cos v [cos (x + y – u – v)
+ i sin (x + y – u – v)]

Speed Test-65

3
1. (a) The inequality is | x + 2 | - | x - 2 |< x - . Dividing the 3
Þx>
9
2 (iii) If x ³ 1 , then ( x + 2) - ( x - 1) < x -
2 2
problem into three intervals :
3 3 9 9 æ9 ö
(i) If x < –2, then - ( x + 2) + ( x - 1) < x - Þx > - Q > 1. Þ common solution is x > Þ x Î ç , ¥ ÷
2 2 è2 ø
2 2
æ9 ö
\ Solution set is x Î ç , ¥ ÷
3 è 2 ø
But - > -2 , hence no common values Þ x Î f
2
2. (b) Given (y 2 + 5y + 3)(x 2 + x + 1) < 2x
3 5
(ii) If -2 £ x < 1, then ( x + 2) + ( x - 1) < x - Þ x < - 2x
2 2 Þ y 2 - 5y + 3 < 2
...(1)
5 x + x +1
But - < -2
2
EBD_7504
S-M-18 SOLUTIONS

2x x 2 + 6x + 9 x 2 + 6x + 9
Let 2
= z Þ zx 2 + (z - 2)x + z = 0 Þ log 2 > log 2 ( x + 1) Þ > ( x + 1)
x + x +1 2( x + 1) 2( x + 1)
2
Q x Î R Þ (z - 2) 2 - 4z.z ³ 0 Þ 3z 2 + 4z - 4 £ 0 Þ -2 £ z £ - x 2 + 2x + 7 2
3 Þ > 0 Þ ( x + 1)(x - 2x - 7) < 0
2( x + 1)
2x 2
Þ -2 £ £ clearly the inequality (1) holds
2
x + x +1 3 Þ x 2 - 2x - 7 < 0 [Q x + 1 > 0]
5- 5 5+ 5 Þ -1 - 2 2 < x < -1 + 2 2 ,
if y 2 - 5y + 3 < -2 Þ y 2 - 5y + 5 < 0 Þ <y<
2 2
3. (c) We have, |2x – 3| < |x + 5| but x > -1 Þ -1 < x < -1 + 2 2
Þ |2x – 3| – |x + 5| < 0
7. (b) Let 3 x = y , then the inequality is
ì3 - 2 x + x + 5 < 0, x £ -5
ï 3 | y 2 - 3y - 15 |< 2 y 2 - y …(1)
ï3 - 2 x - x - 5 < 0, - 5 < x £
Þ í 2 1
ï The inequality holds if 2y 2 - y > 0 Þ y < 0 or y >
3 2
ï2 x - 3 - x - 5 < 0, x >
î 2 1
Q y = 3x £/ 0 Þ y >
2
ì x > 8, x £ - 5
ï Now the inequality on solving,
2 3
ïx > - , - 5 < x £
Þ í 3 2 - (2 y 2 - y) < y 2 - 3y - 15 < 2 y 2 - y
ï 3
ï x < 8, x > Þ 3y 2 - 4y - 15 > 0 and y 2 + 2 y + 15 > 0
î 2
æ 2 3ö æ 3 ö 5
Þ x Î çè - , ÷ø È çè , 8÷ø Solution of first inequality 3y 2 - 4 y - 15 > 0 is y < -
3 2 2 3
æ 2 ö or y > 3
Þ x Î ç - , 8÷
è 3 ø Solution of second inequality y 2 + 2 y + 15 > 0 is y Î R
4. (c) x + 7 < 2x + 3 2x + 4 < 5x + 3
The common solution is

1 y > 3 Þ 3 x > x Þ x > 1 Þ x Î (1, ¥)


Þ 4< x 1 < 3x Þ x >
3 8. (c) Y
1
so, x > and x > 4 \ x lies in (4, ¥). y=6
3
5. (d) From the figure it is clear that there are 3 lines. (0, 3)
Line which passes from (0, 14) and (19, 14) is y = 14 x+y=3
\ In the shaded region 0 £ y £ 14
Line which passes from (5, 0) and (0, 14) is X
O (3, 0)
14 x + 5y = 70
\ In the shaded region 14x + 5y ³ 70 9. (b) Given equations are
Line which passes from (5, 0) and (19, 14) is
x – y– 5 = 0
3x + 2y – z = 4, …(i)
\ In the shaded region x – y £ 5 2x – y + 2z = 6 …(ii)
Thus, inequations are 14 x + 5y ³ 70, x – y £ 5, y £ 14. x + y + z < 7. …(iii)
2 By (i) + 2 × (ii),
6. (a) The log functions are defined if x + 6x + 9 > 0 and
2(x + 1) 7x + 3z = 16,
\ x = 1, z = 3.
(x + 3) 2 From (i), y = 2x + 2z – 6 = 2. Since x = 1,
x +1 > 0 Þ > 0 and x + 1 > 0 Þ x > –1
2( x + 1) y = 2, z = 3 satisfy the inequality (iii), the solution is
x 2 + 6x + 9 x = 1, y = 2, z = 3.
Now the inequality is log < - log 2 ( x + 1)
2 -1 2(x + 1) 10. (d) We can add two inequalities of the same type. So by
adding above two inequalities we get
x 2 + 6x + 9
Þ - log 2 < x + y < 38
2( x + 1)
Mathematics S-M-19

11. (b) Y 350 £ 300 + x £ 375


50 £ x £ 75
4x–3y + 2 = 0
17. (a) The solution set of the system in the set of the
2x + 3y – 5 = 0 integers x satisfying
O X
a b
£ x < . Since each of 1, 2, 3 satisfies the two
9 8
12. (c) | x - 1 |£ 3 Þ -3 £ x - 1 £ 3 Þ -2 £ x £ 4
inequalities,
and | x - 1 | ³ 1Þ x - 1 £ -1 or x - 1 ³ 1 Þ x £ 0 or x ³ 2
a
Taking the common values of x, we get x Î [-2, 0] È [2, 4] 0< £ 1 Þ 0 £ a 9,
9
13. (c) We have
b
MA 3< £ 4 Þ 24 < b £ 32.
IQ = × 100 8
CA So a has 9 choices and b has 8 choices, i.e. there are
MA 9 × 8 = 72 required ordered pairs (a, b).
Þ IQ = × 100 [Q CA = 12 years]
12 18. (a) Let x and x + 2 be two odd natural numbers.
25 we have, x > 10 …(i)
= MA and x + (x + 2) < 40 …(ii)
3 On solving (i) and (ii), we get
Given, 80 £ IQ £ 140 10 < x < 19
25 So, required pairs are (11, 13), (13, 15), (15, 17) and
Þ 80 £ MA £ 140 (17, 19)
3
Þ 240 £ 25MA £ 420 19. (a) | x - 1 | -1 £ 1 Þ -1 £ | x - 1 | -1 £ 1
240 420 Þ 0 £| x - 1 |£ 2 Þ -2 £ x - 1 £ 2
Þ £ MA £
25 25 Þ -1 £ x £ 3 Þ x Î [-1, 3]
Þ 9.6 £ MA £ 16.8
20. Profit = Revenue – Cost
14. (b) Given in-equations are
Revenue – cost > 0 for some profit
–17 £ 3x + 10 £ –2
(5x + 20) – (2x + 80) > 0
\ –27 £ 3 x £–12
3x – 60 > 0
\ –9 £ x £ –4 …(i)
x > 20
–22 £ 5x + 13 £ 3 Thus, the manufacturer have to sale more than 20 items to
\ –35 £ 5x £ –10 earn some profit.
\ –7 £ x £ –2 …(ii) 21. (b) Let the shortest side be x cm.
–19 £ 2x – 9 £ –3 Then, by given condition, second length = x + 3 cm
Third length = 2x cm
\ –10 £ 2x £ 6
Also given, total length = 91
\ –5 £ x £ 3 …(iii) Hence, sum of all the three lengths should be less
The common range of values that satisfies all the than or equal to 91
x + x + 3 + 2x £ 91
equations is [– 5, – 4] Þ 4x + 3 £ 91
Y Subtracting (–3) to each term,
15. (b) –3 + 4x + 3 £ 91 – 3
2x + y = 2 Þ 4x £ 88
(0, 2) 4x 88 88
x–y=3 Þ £ Þ x£
(1, 0) 4 4 4
X Þ x £ 22 cm ... (i)
O (3, 0)
(5/3, – 4/3) Again, given that
(0, –3) Third length ³ second length + 5
Þ 2x ³ (x + 3) + 5
16. (c) Let Ankur got ‘x’ marks in fifth subject. Þ 2x ³ x + (3 + 5)
Transferring the term x to L.H.S.,
90 + 70 + 75 + 65 + x 2x – x ³ 8
So, average marks =
5 Þ x³8 ... (ii)
90 + 70 + 75 + 65 + x From equations (i) and (ii), length of shortest board
Given 70 £
£ 22.
EBD_7504
S-M-20 SOLUTIONS

22. (d) Y 25. (d) Y


(0, 4.5) (0, 4)
3x + 4y = 18 2x + y = 4
2x + 3y = 3 (2, 0)
(0, 1) X
O
O (3/2, 0) (6, 0) X
x + 2y = 0
23. (a) The equations, corresponding to inequalities
3x + 2y £ 6 and 6x + 4y ³ 20, are 3x + 2y = 6 and x+2
26. (a) The inequality is log 0.2 £ 1 . The L.H.S is valid if
6x + 4y = 20. So the lines represented by these x
equations are parallel. Hence the graphs are disjoint.
x+2
Y > 0 Þ x( x + 2) > 0 Þ x < -2 or x > 0 .
x
(0,5) Solving the inequality, we get (note that base < 1)
x+2 1 x+2 1 4 x + 10
³ 0. 2 = Þ - ³0Þ ³0
x 5 x 5 5x
(3,0)
5
(10/3,0) x (2 x + 5) ³ 0 Þ x £ - or x ³ 0 .
X 2
O (2,0) 6x + 4y = 20
3x + 2y = 6 5
Taking the intersection, we get x £ - or x > 0
2
x+3 + x
24. (a) We have >1 æ 5ù
x+2 Þ x Î çç - ¥, - ú È (0, ¥)
x + 3+ x è 2û
Þ -1 > 0 27. (b) Let numbers are 2x and 2x + 2
x+2
Then, according to the question,
x+3 -2
Þ >0 5
x+2 2x > 5 Þ x >
Now two cases arise : 2
Case I : When x + 3 ³ 0, i.e., x ³ – 3. Then and 2x + 2 > 5 Þ 2x > 5 – 2
3
x+3 -2 x+3- 2 Þ 2x > 3 Þ x >
>0 Þ >0 2
x+2 x+2
and 2x + 2x + 2 < 23 Þ 4x < 23 – 2
x +1
Þ >0 21
x+2 Þ 4x < 21 Þ x <
Þ {(x + 1) > 0 and x + 2 > 0} 4
or {x + 1 < 0 and x + 2 < 0} Now, plotting all these values on number line
Þ {x > – 1 and x > – 2}
or {x < – 1 and x < –2}
Þ x > – 1 or x < – 2
–¥ 3 5 21 +¥
Þ x Î (-1, ¥) or x Î ( - ¥, - 2)
2 2 4
Þ x Î ( -3, -2) È ( -1, ¥) [Since x ³ -3 ] …(i)
Case II : When x + 3 < 0, i.e., x < – 3 æ 5 21 ö
x+3 -2 -x - 3 - 2 From above graph, it is clear that x Î ç , ÷ in
>0 Þ >0 è2 4 ø
x+2 x+2 which integer values are x = 3, 4, 5.
-( x + 5) x+5 When x = 3, pair is (2 × 3, 2 × 3 + 2) = (6, 8)
Þ >0 Þ <0 When x = 4, pair is (2 × 4, 2 × 4 + 2) = (8, 10)
x+2 x+2
Þ (x + 5 < 0 and x + 2 > 0 ) When x = 5, pair is (2 × 5, 2 × 5 + 2) = (10, 12)
or (x + 5 > 0 and x + 2 < 0) \ Required pairs are (6, 8), (8, 10), (10, 12).
Þ (x < – 5 and x > – 2) 28. (c) In given all equations, the origin is present in shaded
or (x > – 5 and x < – 2). It is not possible. area, answer (c) satisfy this condition.
Þ x Î (– 5, – 2) ...(ii) 29. (d) As shown in graph drawn for x + y = 1 and x – y = 1 the
Combining (i) and (ii), the required solution is origin included in the area. Hence the bounded region
x Î (– 5, – 2) È (– 1, ¥ ) situated in all four quadrant.
Mathematics S-M-21

Y 3
The solution set is x ³ implies that
x + y = 1 (0, 1) 4
x–y=1 4m + 3 3
6m – 1 > 0 and 2 ( 6 m - 1 ) = 4 ,
X
O
(1, 0) \ 2(4m + 3) = 3(6m – 1)
i.e. (18 – 8)m = 6 + 3,
(0,–1)
9
2 m + x 4 mx - 1 \ m = .
£ 10
30. (b) From it follows that
3 2
4m + 2x £ 12mx –3,
2(6m – 1) x ³ 4m + 3.

Speed Test-66

1. (c) Starting with the letter A, and arranging the other four 6. (d) Tn denotes the number of triangles which can be formed
letters, there are 4! = 24 words. These are the first 24 by using the vertices of a regular polygon of n sides.
words. Then starting with G, and arranging A, A, I, and \ Tn = nC3 Þ Tn+1 = n+1C3
4! 24
N in different ways, there are = = 12 words. Þ Tn+1 – Tn = n+1C3 – nC3 = 28 (Given)
2! 1! 1! 2
Hence, total 36 words. (n + 1)! n!
- = 28
Next, the 37th word starts with I. There are 12 words 3!(n + 1 - 3)! 3!(n - 3)!
starting with I. This accounts up to the 48th word. The
49th word is NAAGI. The 50th word is NAAIG. (n + 1)n(n - 1) n(n - 1)(n - 2)
Þ - = 28
2. (b) First we have to select 2 men for bow side and 3 for 6 6
stroke side. The number of selections of the crew for n(n - 1)
two sides = 5C2 × 3C3 Þ ´ 3 = 28 Þ n(n – 1) = 28 × 2
For each selection there are 4 persons on both sides, 6
who can be arranged in 4! × 4!ways. Þ (n – 8) (n + 7) = 0 Þ n = 8, – 7
Required number of arrangement n can never be less than zero
= 5C2 × 3C3 × 4! × 4! = 5760 Þn=8
3. (c) Words start with D are 6! = 720, start with E are 720. 7. (a) x+2Px+2 = a Þ a = ( x +2 )!
start with MD are 5! = 120 and start with ME are 120. xP
x! x–11P
Now the first word starts with MO is nothing but 11 =bÞb= s and x–11 =c
(x -11)!
MODESTY. Hence rank of MODESTY is 1681.
4. (c) Let the sides of the game be A and B. Given 5 married Þ c = (x – 11)!
couples, i.e., 5 husban ds an d 5 wives. Now, a = 182 bc
2 husbands for two sides A and B can be selected out x!
of 5 = 5C2 = 10 ways. \ (x + 2)! = 182 (x– 11)!
( x - 11)!
After choosing the two husbands their wives are to
Þ (x + 2) (x + 1) = 182 = 14 × 13
be excluded (since no husband and wife play in the
\ x + 1 = 13 \ x = 12
same game). So we have to choose 2 wives out of
8. (a) We know that in any triangle the sum of two sides is
remaining 5 – 2 = 3 wives i.e., 3C2 = 3 ways.
always greater than the third side.
Again two wives can interchange their sides A and B
\ The triangle will not be formed if we select segments
in 2! = 2 ways.
of length (2, 3, 5), (2, 3, 6) or (2, 4, 6).
By the principle of multiplication, the required number
Hence no. of triangles formed = 5C3 – 3.
of ways = 10 × 3 × 2 = 60
9. (a) [1.3.5...... (2n – 1)]2n
5. (d) Triangles with vertices on AB, BC and CD are
n
3×4×5 = 60 1.2.3.4.5.6....(2n – 1)(2 n)2n = (2n)!2 =
(2n)!
Triangles with vertices on AB, BC and DA are = n n!
2.4.6......2n 2 (1.2.3....n)
3×4×6 = 72 10. (c) There are 3p points, so possible no. of triangles
Triangles with vertices on AB, CD and DA are = 3pC3
3×5×6 = 90 But the points on he same line do not form a triangle,
Triangles with vertices on BC, CDand DA are such selections are = 3 . pC3
4×5×6 = 120 3pC – 3 . pC = p2 (4p – 3)
3 3
\
EBD_7504
S-M-22 SOLUTIONS

11. (c) Leaving the ground floor and second floor, their are Number of triangle with one side common with octagon
10 floors in which three groups of people can left the = 8 × 4 = 32
lift cabin in 10P3 ways, i.e. 720 ways. (Consider side A1A2. Since two points A3, A8 are
n -1 adjacent, 3rd point should be chosen from remaining 4
12. (d) There are in the set (1, 2, 3, ..... n) (n being odd),
2 points.)
n +1 Number of triangles having two sides common with
even numbers, odd numbers and for an A.P., the octagon : All such triangles have three consecutive
2
sum of the extremes is always even and hence the vertices, viz., A1A2A3, A2A3A4, ..... A8A1A2.
choice is either both even or both odd and this may be Number of such triangles = 8
done in \ Number of triangles with no side common
n -1 n +1 = 56 – 32 – 8 = 16.
( n - 1) 2
2 C
2 + 2 C
2 = ways 18. (d) When A has B or C to his right we have the order :
4 AB or AC ...(1)
Note that, if a, b, c are in A.P. a + c = 2b. Hence, if a, b, When B has C or D to his right, we have the order :
c are integer the sum of extreme digits (a and c) is even.
13. (b) According to given restriction: BC or BD ...(2)
B must not precede A (immediately or otherwise), Taking these two possibilities together, we must have
Þ A must follow B, i.e., B should addressed the meeting ABC or ABD or AC and BD.
at first place For ABC, D, E, F to arrange along a circle, number of
So, rest of the four speakers can address in 4! ways. way = 3 ! = 6, where three persons A,B,C together are
\ Required number of ways = 4! = 24 treated as single.
14. (b) Total number of possible predictions = 3 10 For ABD, C, E, F, the number of ways = 6. For AC, BD,
No. of predictions which have r wrong and 10 – r correct E,F the number of ways = 6.
entries = 10Cr 210 – r Hence, total number of ways = 18.
4
\ Desired no. of ways = 310 – å 10 C r 210 - r 19. (c) Let m + 5 Pm + 1 =
11
2
(
(m - 1) m + 3Pm )
r =0
[Note that above no. of ways is also equal to ( m + 5)! 11 é (m + 3)!ù
5 Þ = (m - 1) ê ú
å 10C r 2 r
4! 2 ë 3! û
r =0 (m + 5) (m + 4) 11
15. (b) Two circles intersect in 2 points. Þ = (m - 1)
4 2
\ Maximum number of points of intersection
= 2 × number of selections of two circles from 8 circles (m + 4) (m + 5) = 22(m – 1)
= 2 × 8C2 = 2 × 28 = 56 Þ m2 – 13m + 42 = 0
Statement 2 : 4 lines intersect each other in 4C2 = 6 Þ m2 – 7m – 6m + 42 = 0
points Þ m(m – 7) – 6 (m – 7) = 0
4 circles intersect each other in 2 × 4C2 = 12 points. Þ m = 6, 7
Further, one lines and one circle intersect in two points. Hence m1 + m2 = 13
So 4 lines will intersect four circles in 32 points. 20. (b) There are 8 chairs on each side of the table. Let the
Maximum number of points = 6 + 12 + 32 = 50 sides be represented by A and B. Let four persons sit
16. (d) A chessboard is made up of 9 equispaced horizontal on side A, then number of ways of arranging 4 persons
and vertical line. To make a 1 × 1 square, we must choose on 8 chairs on side A = 8P4 and then two persons sit
two consecutive horizontal and vertical lines from
among these. This can be done in 8 × 8 = 82 ways. on side B. The number of ways of arranging 2 persons
A 2 × 2 square needs three consecutive horizontal and on 8 chairs on side B = 8P2 and the remaining 10
vertical lines, and we can do this in 7 × 7 = 72 ways. persons can be arranged in remaining 10 chairs in 10!
Continuing in this manner, the total number of square is ways.
Hence the total number of ways in which the persons
8(8 + 1)[( 2 ´ 8) + 1]
8 2 + 7 2 + 6 2 + ..... + 2 2 + 12 = can be arranged
6
= 204. 8! 8! 10!
17. (d) Number of all possible triangles = 8P4 × 8P2 × 10! =
4! 6!
= Number of selections of 3 points from 8 vertices
= 8C3 = 56 21. (c) One possible arrangement is 2 2 1
A6 A5 Three such arrangements are possible. Therefore, the
number of ways is (5C2)(3C2)(1C1)(3) = 90
A4
A7 The other possible arrangements 1 1 3
Three such arrangements are possible. In this case,
A8 A the number of ways is (5C1)(4C1)(3C3)(3) = 60

A
Mathematics S-M-23

22. (d) Number of white balls = 10 NOTE : Such problems are called problems of
Number of green balls = 9 deragement. Hence, using the formula of deragement.
and number of black balls = 7 The required number of ways of placing all letters in
\ Required probability = (10 + 1) (9 + 1) (7 + 1) – 1 wrong envelope
= 11.10.8 –1 = 879
é 1 1 1 1 ù 4 ! 4! 4 !
= 4! ê1- + - + ú = - + = 12 - 4 + 1 = 9
ïì n! ïü ë 1! 2! 2! 4!û 2! 3! 4!
í ý
îï r!( n - r ) !þï = 56
n
23. (c) Now,
Cr 26. (a) Given: 12Pr = 11P6 + 6.11P5
n
C r-1 n! 28 we know that
Q n–1P + r . n–1P = r! nC
( )(
r - 1 ! n - r + 1) ! r r–1 r
\ 11P6 + 6. 11P5 = 6! 12C6 [r = 6, n = 12]
n - r +1 Þ 12P6 = 6! 12C6
Þ =2 …(i)
r 12! 12!
Þ n = 3r – 1 …(ii) \ = 6! which are equal
n
6! 6!6!
C r +1
70 n-r 5 \ r=6
Also, =
Þ =
Cr n
56 r +1 4 39
27. (b) C 3r -1 - 39 C = 39 C - 39 C 3r
[Putting (r + 1) in place of r in (i) on the LHS] r2 r 2 -1
Þ 4n = 9r + 5 …(iii) Þ 39 C 3r -1 + 39 C3r = 39 C + 39 C
Eliminating n from (ii) and (iii), we get r 2 -1 r2
r = 3. 40 40
Þ C3r = C
r2
24. (a) Any number greater than a million must be of 7 or
more than 7 digits. Here number of given digits is seven, Þ r 2 = 3r or r 2 = 40 - 3r Þ r = 0, 3 or – 8, 5
therefoer we have to form numbers of seven digits 3 and 5 are the values as the given equation is not
only. defined by r = 0 and r = –8. Hence, the number of values
Now there are seven digits of which 3 occurs thrice of r is 2.
and two occurs twice. 28. (a) There are six letters in MOTHER, all different,
7! i.e. arrangement can be made out of the letters of the
\ number of numbers formed = = 420 word MOTHER taken four at a time with M present in
2 ! 3!
every arrangement.
But this also includes those numbers of seven digits So, rest 3 letters can be arrangement from 5 letters
whose first digit is zero and so in fact they are only six So, total number of ways = 4 × 5P3
digit numbers.
Number of numbers of seven digits having zero in the 5! 4 ´ 5 ´ 4 ´ 3 ´ 2 = 240
= 4´ =
6! (5 – 3)! 2
first place = 1 × = 60. 29. (a) The number of words starting from A are 5! = 120
2! 3! The number of words starting from I are 5! = 120
Hence required number = 420 – 60 = 360 The number of words starting from KA are 4! = 24
25. (a) Let us first consider 2 letters and 2 envelopes, then The number of words starting from KI are 4! = 24
there is only one way to place both the letters in wrong The number of words starting from KN are 4! = 24
envelope. The number of words starting from KRA are 3! = 6
Next, we consider 3 letters and 3 directed envelopes. The number of words starting from KRIA are 2! = 2
The number of ways of putting all letters in wrong The number of words starting from KRIN are 2! = 2
envelopes The number of words starting from KRISA are 1! = 1
= Total number of possible arrangements – Number of The number of words starting from KRISNA are 1! = 1
ways in which all letters are in correct envelopes Hence, rank of word ‘KRISNA
– Number of ways in which 1 letter in correct envelope = 2(120) + 3(24) + 6 + 2(2) + 2(1) = 324
= 3 ! – 1 – 3C1 × 1 = 2
[ Q The case of two letters in correct envelope and L O Y
30. (b) L O Y 1 1 2
one in wrong envelope is not possible]
Further, we consider 4 letters and 4 directed envelopes. 2 2 Þ 1 2 1
4
The number of ways of putting all letters in wrong ³1 ³1 2£ 2 1 1
envelopes 2 2 0
= Total number of possible arrangements – number of Required number of ways
ways in which all letters are in correct envelope – = 2C1 × 2C1 × 2C2 + 2C1 × 2C2 × 4C1
Number of ways in which 1 letter is in correct + 2C2 × 2C1 × 4C1 + 2C2 × 2C2 × 4C0
envelopes (3 in wrong envelope)
– Number of ways in which 2 letters are in correct 4´3
=2×2× +2 ×1×4 +1×2× 4+1× 1×1
envelope (2 in wrong envelope) = 4 ! – 1 – 4C × 1 = 9. 2
EBD_7504
S-M-24 SOLUTIONS

Speed Test-67

1. (b) (101)50 - (99) 50 = (100 + 1)50 - (100 - 1)50 15 n


Cr 15 15
år n
= å (n + 1 - r ) = å (16 - r )
= 2[ 50 C1 (100) 49 + 50 C3 (100) 47 + ...... + 50 C 49 (100)] 1 Cr -1 n =1 1
50 49 49 50 1
>2. C1 . (100) = 2 ´ 50(100) = (100) = 16 × 15 – × 15 ´ 16
2
Þ (101) > (99) + (100) Þ y > x Þ x < y .
50 50 50
n (n + 1)
2. (c) Putting the value of C0, C2, C4....., we get By using sum of n natural numbers =
2
n(n - 1) n (n - 1)( n - 2)( n - 3) 1 = 240 – 120 = 120
= 1+ + + ..... =
3.2! 5.4! n +1 4
æ x 2ö
é (n + 1)n (n - 1) (n + 1) n(n - 1)( n - 2)( n - 3) ù 6. (c) ç1 + - ÷
êë(n + 1) + + + .....ú è 2 xø
3! 5! û
2
Put n + 1 = N æ x 2ö æ x 2ö
= 4C0 + 4C1 ç - ÷ + 4C2 ç - ÷
1 é N ( N - 1)( N - 2) N ( N - 1)( N - 2)( N - 3)( N - 4) ù è 2 xø è 2 xø
= êë N + + + ...ú
N 3! 5! û 3 4
æ x 2ö æ x 2ö
+ 4C3 ç - ÷ + 4C4 ç - ÷
=
1 N
N
{
C1 + N C3 + N C5 + .... } è 2 xø è 2 xø
æ x 2ö é x2 4 ù
=
1 N -1
N
2 {=
2n
n +1
}{Q N = n + 1}
= 4C0 + 4C1 ç - ÷ + 4C2 ê
è 2 xø êë 4
-2+ 2
x
ú
úû
3. (d) Here, Pn = n C 0 . n C1 . n C 2 .... n C n é æ xö
3 2
æ x ö æ 2ö æ xö æ 2ö
2
æ 2ö ù
3
+ 4 C3 ê 3C0 ç ÷ - 3C1 ç ÷ ç ÷ + 3C2 ç ÷ ç ÷ - 3C3 ç ÷ ú
êë è 2ø è 2ø è xø è 2ø è xø è xø ú
and Pn +1 = n +1C 0 . n +1
C1 . n +1
C 2 .... n +1C n +1 û
é æ xö
4 3
æ x ö æ 2ö
2
æ xö æ 2ö
2
æ xö æ 2ö
3
æ 2ö ù
4
n +1
P C 0 . n +1C1 . n + 2C 2 .... n +1C n +1 + 4C4 ê 4 C0 ç ÷ - 4C1 ç ÷ ç ÷ + 4 C2 ç ÷ ç ÷ - 4C3 ç ÷ ç ÷ + 4C4 ç ÷ ú
\ n +1 = ëê
è 2 ø è 2 ø è x ø è 2 ø è x ø è ø
2 xè ø è ø
x ú
û
n
Pn C0 . n C1 . n C 2 .... n C n The term independent of x in above
æ n +1 C1 öæ n +1 C 2 öæ n +1 C 3 ö æ n +1 Cn +1 ö = 4C0 + 4C2 (-2) + 4C4 . 4C2 = 1 - 12 + 6 = -5
ç ÷ç ÷ç ÷..... ç ÷
=ç n ÷ç n ÷ç n ÷ ç nC ÷ 7. (b) 1 + 2x + 3x2 + .... = (1 + x) – 2
è C 0 øè C1 øè C 0 ø è n ø
(1 + 2 x + 3x2 + ....) { }
-2 -7 / 2
-3 / 2 7
n +1 Þ = (1 + x ) = (1 + x )
æ n + 1 öæ n + 1 öæ n + 1 ö æ n + 1 ö (n + 1)
=ç ÷ç ÷ç ÷.....ç ÷ =
\ The coefficient of x5 in (1 + 2x + 3x2 + ........)– 7/2
è 1 øè 2 øè 3 ø è n + 1 ø (n + 1) !
= Coefficient of x5 in (1 + x)7
6 = 7C5 = 21
4. (d)
50
C4 + å 56- r C3 8. (a) The number of subsets of the set which contain at
r =1 most n elements is
2n + 1C + 2n + 1C + 2n + 1C + .... + 2n + 1C = K (say)
é 55 C3 + 54 C3 + 53C3 + 52 C3 ù 0 1 2 n
= 50
C4 + ê We have
ú
2K = 2 (2n + 1C0 + 2n + 1C1 + 2n + 1C2 + .... + 2n + 1Cn)
êë + 51C3 + 50 C3 úû
= (2n + 1C0 + 2n + 1C2n + 1) + (2n + 1C1 + 2n + 1C2n)
n +1 + ... + (2n + 1Cn + 2n + 1Cn + 1) (Q nCr = nCn – r)
We know éë Cr + Cr -1 = Cr ù
n n
û = 2n + 1 C0 + 2n + 1 C1 + 2n + 1 C2 + .... + 2n + 1C2n + 1
=2 2n + 1 ÞK=2 2n
= (50 C4 +50 C3 ) +51C3 +52 C3 +53 C3 + 54 C3 + 55 C3 9. (c) Q x3 and higher powers of x may be neglected
= (51 C4 + 51C3 ) +52 C3 + 53 C3 +54 C3 + 55 C3 3 3
(1 + x) 2 - æçè1 +

Proceeding in the same way, we get ÷

55 \
C4 + 55 C3 = 56 C4 . 1

5. (b) General term of the given series is (1 - x ) 2


n
Cr éæ 3 1 ö ù
r =n+1–r .
2 x 2 ÷ - æ 1 + 3 x + 3.2 x ö ú
n -1 ê 2
Cr -1 ÷ø ç ÷ú
By taking 2! è 2 2! 4 ø úû
Mathematics S-M-25

15. (a) To find


é 1 3 ù 30C 30C – 30C 30 C + 30 C 30C – ....
ê x 2 . 2 2 ú é -3 2 ù -3 2 0 10 1 11 2 12
= ê1 + + x úê x ú= x + 30 C2030C30
ë 2 2! ûë8 û 8 We know that
(as x3 and higher powers of x can be neglected) (1 + x)30 = 30C0 + 30C1x + 30C2x2
50 + .... + 30C20x20 + ....30C30x30 ....(1)
10. (d) å 50 Cr (2 x - 3)r (2 - x )50 - r (x – 1)30 = 30C0x30 – 30C1x29 +....+ 30C10x20
r =0
– 30C11x19 + 30C12x18 +.... 30C30x0 ....(2)
= [(2 – x) + (2x – 3)]50
= (x – 1)50 Multiplying eqn (1) and (2) and equating the coefficients
= (1 – x)50 of x20 on both sides, we get
30C = 30C 30 C – 30 C 30C 30 30
= 50C0 – 50C1 x ....... – 50C25 x25 + ........ 10 0 10 1 11 + C2 C12– ....
Coefficient of x25 is – 50C25 + 30C20 30C30
11. (d) a0 + a1 + a2 + ..... = 22n and a0 + a2 + a4 + .... = 22n –1 \ Req. value is 30C10
an = 2nCn = the greatest coefficient, being the middle 16. (c) Let the consecutive coefficient of
coefficient (1 + x)n are nCr–1, nCr, nCr + 1
an– 3 = 2nCn– 3 = 2nC2n – (n– 3) = 2nCn+3 = an+3 From the given condition,
12. (a) In the expansion of (1 + ax)4 nC : nC : nC
r–1 r r + 1 = 6 : 33 : 110
Middle term = 4C2(ax)2 = 6a2x2 Now nCr – 1: nCr = 6 : 33
In the expansion of (1 - ax)6,
Middle term = 6C3( - ax)3 = – 20a3x3 n! r!(n - r)! 6
Þ ´ =
It is given that (r -1)! (n - r +1)! n! 33
Coefficient of the middle term in (1 + ax)4 = Coefficient
of the middle term in (1 – ax)6 r 2
Þ = Þ 11r = 2n – 2r + 2
Þ 6a2 = – 20a3 n - r + 1 11
3 Þ 2n – 13r + 2 = 0 ....(i)
Þ a = 0, a = - and nCr : nCr + 1= 33 : 110
10
13. (c) The number of selection = coefficient of x8 in n! (r + 1)! (n - r - 1)! 33 3
(1 + x + x2 + .... + x8) (1 + x + x2 + ...... + x8). (1 + x)8 Þ ´ = =
r!(n - r)! n! 110 10
(1– x 9 )2
= coefficient of x8 in (1 + x )8
(1 – x )2 (r + 1) 3
Þ = Þ 3n – 13r – 10 = 0 ...(ii)
= coefficient of x8 in (1 + x)8 in (1 + x8) (1 – x)–2 n - r 10
= coefficient of x8 in Solving (i) & (ii), we get n = 12
(8C0 + 8C1x + 8C2x2 + ..... + 8C8x8)
× (1 + 2x + 3x2 + 4x3 + ..... + 9x8 +....) 17. (d) Tr +1 in the expansion
= 9. C0 + 8 · 8C1 + 7. 8C2 + .... + 1. 8C8
8
11 r
= C0 + 2C1 + 3C2 + .... + 9C8 [Cr = 8Cr] é 2 1 ù æ 1ö
ê ax + bx ú = 11Cr (ax 2 )11 - r ç ÷
Now C0x + C1x2 + .... + C8x9 = x (1 + x)8 ë û è bx ø
Differentiating with respect to x, we get
C0 + 2C1x + 3C2x2 + .... 9C8x8 = (1 + x)8 + 8x (1 + x)7 = 11 Cr (a)11 - r (b) - r ( x)22 - 2r - r
Putting x = 1, we get C0 + 2C1 + 3C2 + ..... + 9C8
For the coefficient of x7, we have
= 28 + 8.27. = 27 (2 + 8) = 10.27.
22 – 3r = 7 Þ r = 5
2
14. (b) = 2 x2 + 1 - 2 x2 - 1 \ Coefficient of x7 = 11 C5 (a)6 (b)- 5 ...(i)
2 2
2x + 1 + 2x -1
Again Tr+1 in the expansion
\ given expression
11 r
= ( 2 x 2 + 1 + 2 x 2 - 1)6 + ( 2 x 2 + 1 - 2 x 2 - 1)6 é 1 ù æ 1 ö
ê ax - 2 ú = 11Cr (ax 2 )11 - r ç -
we know that, ë bx û è bx 2 ÷ø

(a + b)6 + (a - b)6 = 2[a 6 + 6 C 2 a 4b 2 + 6 C4 a2 b 4 + 6 C6 b6 ] 11


= Cr (a )
11 - r r
( -1) ´ (b ) ( x ) -r
( x) - 2r 11 - r

For the coefficient of x–7, we have


\( 2 x 2 + 1 + 2 x 2 - 1)6 + ( 2 x 2 + 1 - 2 x 2 - 1)6
11 – 3r = – 7 Þ 3r = 18 Þ r = 6
= 2[(2 x 2 + 1)3 + 15(2 x 2 + 1) 2 (2 x 2 - 1) \ Coefficient of x - 7 = 11C6 a5 ´ 1 ´ (b) - 6
+15(2 x 2 + 1)(2 x 2 - 1) + (2 x 2 - 1)3 ] \ Coefficient of x7 = Coefficient of x–7
Which is a polynomial of degree 6. Þ 11
C5 (a)6 (b) - 5 = 11C6 a5 ´ (b) - 6
EBD_7504
S-M-26 SOLUTIONS

18. (a) We have 2007


79 + 97 = (8 – 1)9 + (8 + 1)7 = (1 + 8)7 – (1 – 8)9 æ 1 ö
ç 223 ÷ = 39 = 32 ×37 = 9 (2187)
= ç3 ÷
= [1+ 7C1 8+ 7C2 82 + .....+ 7 C7 87 ]
è ø
- [1 - 9C18 + 9C2 82 - ..... - 9C9 89 ] \ k = 2187
22. (a) (1 + x)4n = 4nC0 + 4nC1 x + 4nC2 x2 +4nC3 x3
= 7C18 + 9C18 + [ 7C2 + 7C3.8 + .... - 9C2 + 9C3.8 - .....]82
+ 4nC4 x4 + ..... + 4nCun x4n
= 8 (7 + 9) + 64 k = 8..16 + 64 k = 64 q, where q = k + 2
Thus, 79 + 97 is divisible by 64. Put x = 1 and x = – 1, then adding.
19. (d) (1 - y )m (1 + y )n 24n–1 = 4nC0 + 4nC2 + 4nC4 + ..... + 4nC4n ..... (i)
Now put, x = i
= [1- m C1 y + m C2 y 2 - ......] [1 + n C1 y + n C2 y 2 + .....]
(1 + i)4n = 4nC0 + 4nC1i – 4nC2 – 4nC3i + 4nC4 +
= 1 + (n - m) y + ìí m(m - 1) + n(n - 1) - mn üý y 2 + ..... ........ + 4nC4n
î 2 2 þ
By comparing coefficients with the given expression, Compare real and imaginary part, we get
we get (–1)n (2)2n = 4nC0 – 4nC2 + 4nC4 – 4nC6 + .... + 4nC4n ... (ii)
\ a1 = n - m = 10 and Adding (i) and (ii), we get
m 2 + n2 - m - n - 2mn Þ 4nC0 + 4nC4 + .... + 4nC4n = (–1)n (2)2n–1 + 24n–2
a2 = = 10
2 23. (d) We know that, (1 + x)20 = 20C0 + 20C1x + 20C2 x2
So, n – m = 10 and (m - n) - (m + n) = 20 2 + ...... 20C10 x10 + ..... 20C20 x20
Put x = –1, (0) = C0 – 20C1 + 20C2 – 20C3 + ......
20
Þ m + n = 80 \ m = 35, n = 45 + 20C10 – 20C11 .... + 20C20
20. (d) We have Þ 0 = 2[20C0 – 20C1 + 20C2 – 20C3 + ..... – 20C9] + 20C10
S = C 0 + (C 0 + C1 ) + (C 0 + C1 + C 2 ) + Þ 20C10 = 2[20C0 – 20C1 + 20C2 – 20C3
+ ...... – 20C9 + 20C10]
.... + (C0 + C1 + ... + Cn ) 1
Þ 20C0 – 20C1 + 20C2 – 20C3 + .... + 20C10 = 20C
= (C0 + C0 +....n +1 times) + (C1 + C1 + .... n times) 2 10
24. (c) We have, 7103 = 7 (49)51 = 7 (50 – 1)51
(C2 + C2 + .... n - 1times) + .....+ (Cn-1 + Cn-1 ) + Cn = 7 (5051 – 51C1 5050 + 51C2 5049 – ... – 1)
= (n + 1)C0 + nC1 + ( n - 1)C 2 + .... + 2C n -1 + Cn = 7 (5051 – 51C1 5050 + 51C2 5049 – ...) – 7 + 18 – 18
= 7 (5051 – 51C1 5050 + 51C2 5049 – ...) – 25 + 18
= C0 + 2C1 + 3C2 + ..... + (n + 1)C n [Q Cr = Cn -r ] = k + 18 (say) where k is divisible by 25,
General Term Tr +1 = (r + 1) Cr \ remainder is 18.
1 1 1 1
- -
Tr +1 = r n Cr + n Cr = n . n -1Cr -1 + n Cr æ a ö2 æ a ö2 æ a + x ö 2 æa-xö 2
25. (d) ç ÷ +ç ÷ =ç ÷ +ç ÷
n èa+xø èa -x ø è a ø è a ø
\ S = å Tr +1 = n [ n -1 C0 + n -1C1 + ..... + n -1Cn -1 ]
r =0 1 1
- -
æ xö 2 æ xö 2
+[ n C0 + n C1 + ..... + n C n ] = ç1 + ÷ + ç1 - ÷
è aø è aø
= n .2n -1 + 2n = (n + 2) 2n -1
é 1 x 3 x2 ù é 1 x 3 x2 ù
21. (a) We know that, = ê1 - + ú + ê1 + + ú
(a – 1)n = nC0.an – nC1.an–1 + nC2an–2 – êë 2 a 8 a 2 úû êë 2 a 8 a 2 úû
........ + (–1)n–1.nCn–1a + (–1)n nCn
n é x ù 3 x2
(a - 1)
êQ x << a , \ << 1ú = 2 + .
\ = nC0an–1 – nC1an–2 + nC2an–3 – ë a û 4 a2
a
(-1)n n 26. (b) We have,
........ + (–1)n–1.nCn–1 + Cn
a 2r + 2 n Cr 2r + 2 1 n
(a - 1) n - (-1) n tr + 1 = = . Cr
\ f (n) = (r + 1) (r + 2) r + 2 r + 1
a
(a - 1)2007 + 1 (a - 1)2008 - 1 2r + 2 1 n +1
Now, f (2007) + f (2008) = + = . Cr + 1
a a r + 2 n +1

(a - 1)2007 (1 + a - 1) 2r + 2 æ 1 n +1 ö
=
2007 = .ç C r + 1 ÷÷
a ø
Mathematics S-M-27

2r + 2 1 n+2 ( x - ai )n = (T0 - T2 + T4 - ......) - i(T1 - T3 + T5 - ....)


= . Cr + 2
n +1 n + 2 .........(ii)
Multiplying (ii) and (i) we get required result
é 1 n 1 n +1 ù i.e., ( x 2 + a 2 ) n = (T0 - T2 + T4 - ....) 2 + (T1 - T3 + T5 - ....) 2
êQ Cr = Cr + 1 ú
ë r + 1 n +1 û 29. (c) ( x + n C 0 )(x + 3. n C1 )(x + 5 . n C 2 ).....(x + (2n + 1) . nCn)
Putting r = 0, 1, 2, ......, n and adding we get,
n
The given expression = x n +1 + x n {n C 0 + 3 . n C1 + 5 . n C 2 + ..... + ( 2n + 1) . C n } + ....
1 Coeff. of xn = n C 0 + 3 . n C1 + 5. n C 2 + ..... + (2 n + 1) . n C n
= {22 . n + 2C2 + 23 . n + 2C3 +....
(n + 1) (n + 2)
= 1 + ( n C1 + 2 . n C1 ) + ( n C 2 + 4 . n C 2 ) + ....
+ 2n + 2 . n + 2Cn + 2}
1 + ( n C n + 2n . n C n )
= {(1 + 2)n + 2 – n + 2C0 – 2. n + 2C1}
(n + 1) (n + 2) n n n n n
= (1 + C1 + .... + C n ) + 2( C1 + 2 C 2 + .... + n . C n )
3n + 2 - 2(n + 2) - 1 3n + 2 - 2n - 5 é n(n - 1) n (n - 1)(n - 2) ù
= = = 2 n + 2ên + 2 . + 3. + ... + n .1ú
(n + 1) (n + 2) (n + 1) (n + 2) 2! 3!
ë û
27. (b) (1 – 9x + 20x ) = [(1 – 4x) (1 – 5x)]–1
2 –1
n n -1 n -1 n -1
1 é (1 - 4x) - (1 - 5x) ù 1 = 2 + 2n[1 + C1 + C 2 + .... + C n -1 ]
= ê ú = [(1 – 5x)–1 – (1 – 4x)–1] n n -1
x ë (1 - 4x).(1 - 5x) û x = 2 + 2n . 2 = 2 n (1 + n ) = (n + 1) . 2 n
30. (d) Since the coefficient of (r +1)th term in the expansion of
1
= [(5 – 4)x + (52 – 42) x2 + (53 – 43) x3 (1 + x)n = nCr
5 \ In the expansion of (1+ x)18
+......+ (5n – 4n) xn +......] coefficient of (2r + 4)th term = 18C2r + 3,
\ coeff. of xn = 5 n + 1 – 4n + 1 Similarly, coefficient of (r–2)th term in the expansion of
28. (b) From the given condition, replacing a by ai and – ai (1+ x)18 = 18Cr–3
respectively, we get If nCr = nCs then r + s = n
( x + ai )n = (T0 - T2 + T4 - ......) + i (T1 - T3 + T5 - ....) So, 18
C2 r +3 = 18 Cr -3 gives
.........(i) 2r + 3 + r – 3 = 18
and Þ 3r = 18 Þ r = 6.

Speed Test-68

1. (a) Given a 2 , b 2 . c 2 are in A.P.. a b c


Þ , , are in A.P..
b+c c+a a +b
Þ a 2 + (ab + bc + ca ), b 2 + (ab + bc + ca )
2. (b) The (n + 2)th term of A.P.
c 2 + (ab + bc + ca ) are in A.P.. x1 = a + (n + 1) (b – a) ...(i)
The (n + 2)th term of G.P.
Þ (a + b) (a + c), (b + c) (b + a ), (c + a )(c + b) are in A.P..
n +1
æbö
1 1 1 x2 = a ç ÷ ...(ii)
Þ , , are in A.P.. èaø
b+c c+a a +b
[Divide by (a + b) (b + c) ( c + a)] The (n + 2)th term of H.P.
Again, a 2 , b 2 . c 2 are in A.P.. 1
x3 =
1 æ1 1ö
1 1 1 + (n + 1) ç - ÷
Þ , , are in A.P.. a èb aø
b+c c+a a +b
Now, x1, x2, x3 are in G.P.
a +b+c a +b+c a +b+c
Þ , , are in A.P.. if x 22 = x1x 3 That is
b+c c+a a+b
2n + 2
a b c æbö a + (n + 1)(b - a )
Þ + 1, + 1, + 1 are in A.P.. if a 2 ç ÷ =
b+c c+a a a 1 æ1 1ö
ç - ÷
èb aø
EBD_7504
S-M-28 SOLUTIONS

b 2n + 2 (n + 1) b - na n -1
1 3
or
a 2n
=
b + (n + 1) (a - b)
ab \ S2 = å t n -1 = 2 Sn 2 - 2 Sn + 2S1
n =1
2n + 2 2n + 2
b -a n +1 1 ( n - 1) n ( 2n - 1) 3 n ( n - 1)
\ = = - + 2( n - 1)
2n 2n n
ab(b -a ) 2 6 2 2
é 2n 2 - n 3n ù
[ ]
a +b 2ab
3. (a) We have =m = (n - 1) ê - + 2ú = n - 1 2n 2 - n - 9n + 24
2 a+b
ëê 12 4 úû 12
(a + b) 2
or
4ab
= m ...........(i)
=
n -1 2
6
[
n - 5n + 12 =
6
]
n 3 - 6n 2 + 17n - 12

(a + b) 2
or -1 = m -1 n 3 - 6n 2 + 17n - 12
4ab \ x n = 2 + S2 = 2 +
6
(a - b) 2
or = m - 1 ..............(ii) n 3 - 6n 2 + 17n
4ab =
6
Dividing (i) by (ii), we get
So, x20 = 990
( a + b) 2 m a+b m 6. (b) The general term is
= or =
( a - b) 2 m -1m -1a -b k k
xk = 2 4
=
Applying componendo and dividendo, 1+ k + k 1 + 2k + k 4 - k 2
2

a m + m -1 k 1 2k
We get = = =
b 2 2 2 2 (k 2 + 1 + k )( k 2 + 1 - k )
m - m -1 ( k + 1) - k
4. (a) Let the first set of number be a – d, a, a + d.
1é 1 1 ù 1
Then a – d + a + a + d = 15 Þ a = 5 = - 2 = [ f ( k ) - f ( k + 1)]
The second set of numbers will be 2 ë k - k + 1 k + k + 1 ûú 2
ê 2

b – (d – 1), b, b + (d – 1). Again,


1
b - (d - 1) + b + b + (d - 1) = 15 Þ b = 5 where f (k ) =
2
k - k +1
Hence, the sets of numbers are
5 – d, 5, 5 + d and 6 – d, 5, 4 + d. n 1 n
Further, from the given condition \ Sn = å xk = å [ f (k ) - f (k + 1)]
k =1 2 k=1
(5 - d)5(5 + d) 7 25 - d 2 1 1 1é 1 ù
= Þ = = [ f (1) - f ( n + 1)] = ê1 - 2
(6 - d) 5(4 + d) 8 2 ë n + n + 1 úû
2 8
24 + 2d - d 2
Þ d 2 + 14d - 32 = 0 Þ d = 2,-16 7. (d) Let, Sn = 2 + 5 +14 + 41 + .......+ xn
\ The two sets are 3, 5, 7 and 4, 5, 6 or Sn = 2 + 5 + 14 + ....... + xn -1 + xn
21, 5, –11 and 22, 5, –12.
0 = 2 + [ 3 + 9 + 27 + ..........to ( n - 1) terms ] - xn
3 -11 3 11
\ Ratio of their smallest term is or i.e. or 3(3n -1 - 1) 1 1 n
4 - 12 4 12 \ xn = 2 + = + .3
5. (c) Let S1= 2 + 3 + 5 +9 + 16 +............+ xn 3 -1 2 2
S1 = 2 + 3 + 5 + 9 + ..........x n -1 + x n 1 1 n
\Sn = å xn = å1+ å 3
2 2
O = 2 + [1 + 2 + 4 + 7 + ......
+ to (n - 1) term] - x n =
1
2
1
(
n + 3 + 32 + 33 + ........ + 3n
2
)
\ x n = 2 + [1 + 2 + 4 + 7 + .....to (n - 1) terms]
Again let
S2 = 1 + 2 + 4 + 7 + ........+ t n-1
n 1 3(3n - 1) n 3 n
= + .
2 2 3 -1
= + 3 -1
2 4
( )
8. 2 3
(a) If a, ar, ar , ar ......... are in G.P., then
S2 = 1 + 2 + 4 + 7..... + t n - 2 + t n -1
a
O = 1 + [1 + 2 + 3 + ....... + (n - 2) term] - t n-1 sum of infinite G.P.= a + ar + ..... + ¥ =
1- r
(n - 2)(n - 1) n 2 - 3n + 4 where 'a' is the first term and 'r' is the common ratio of
t n -1 = 1 + =
2
Mathematics S-M-29

12. (c) Given that a, b, c are in G.P.


Given x = 1 + a + a 2 + .....¥
This is a GP, with common ratio 'a'. So, b 2 = ac … (i)
1 x -1 a+b
Þx= Þ x - ax = 1 Þ a = x= … (ii)
1- a x 2
Again, y = 1 + b + b2 + ......¥ This is also a G.P., with b+c
y= … (iii)
common ratio 'b'. 2
1 y -1 x c 2a 2c 2(ab + bc + 2ca )
Þ y= Þb= Now + = + =
1- b y a y a + b b + c ab + ac + b2 + bc

Now, consider 1 + ab + a 2b 2 + .....¥


which is again a GP with common ratio 'ab'.
=
2(ab + bc + 2ca)
(ab + ac + ac + bc )
= 2 Q b2 = ac { }
1 1 13. (b) Let a, b, c, d be four numbers of the sequence.
\ Sum = = Now, according to the question b2 = ac and c – b = 6
1 - ab x -1 y -1
1- . and a – c = 6
x y
Also, given a = d
xy xy
= = éa + bù
xy - xy + x + y - 1 x + y - 1 \ b2 = ac Þ b 2 = a ê (Q 2c = a + b)
ë 2 úû
1.3 1.3.5 Þ a2 – 2b2 + ab = 0
9. (d) Let , S = 1 + + + ...¥ Now, c – b = 6 and a – c = 6,
6 6.8
gives a – b = 12 Þ b = a – 12
S 1 1.3 1.3.5 \ a2 – 2b2 + ab = 0 Þ a2 – 2(a – 12)2 + a(a – 12) = 0
Þ = + + + ...¥
4 4 4.6 4.6.8 Þ a2 – 2a2 – 288 + 48a + a2 – 12a = 0
Þ 36a = 288 Þ a = 8
1 S 1 1 1 1 1.3 1 1.3.5 Hence, last term is d = a = 8.
Þ - = - . - . - ...¥
2 8 2 2 4 2 4.6 2 4.6.8 14. (b) S = 1 + 4x + 7x2 + 10x3 +........
x.S = x + 4x2 + 7x3 +........
1 æ 1 ö 1 æ 1 öæ 1 ö
-1 -1 - 2÷ Subtract
1 S 1 2 çè 2 ÷ø 2 çè 2 ÷çøè 2 ø S (1 – x) = 1 + 3x + 3x2 + 3x3 +.......
Þ - = 1- + -
2 8 2 1.2 1.2.3
æ 1 ö
S (1 – x) = 1 + 3x ç , Q |x|<1
1 æ 1 öæ 1 öæ 1 ö è 1 - x ÷ø
ç - 1÷ç - 2 ÷ ç - 3 ÷
2 è 2 øè 2 øè 2 ø ......¥ 1 + 2x
+ S=
1.2.3.4
(1 - x) 2
1
- S / 8 = (1 - 1)
1/ 2
Þ =0 1+ 2x 35
2 Given : =
2 16
(1 - x)
S / 8 = 1/ 2 Þ S = 4
Þ 16 + 32x = 35 + 35x2 – 70x
n n (n + 1) Þ 35x2 – 102x + 19 = 0
10. (a) We have Sn = = = n +1
1 n + 1 -1 Þ 35x2 – 7x – 95x + 19 = 0
1- Þ 7x (5x – 1) – 19 (5x – 1) = 0
n +1
Þ (5x – 1) (7x – 19) = 0
Þ S2n = (n + 1) 2 1 19
Þ x= ,
So, S12 + S22 + ... + S22 n -1 2 2
= 2 + 3 + ... + (2n ) 2 5 7
But | x | < 1, \ x = 1/5
1 15. (c) Given sum is
= (2n )(2n + 1)(4n + 1) - 1
6 3 5 7
1 2
+ 2 2
+ + .....
= [ n ( 2n + 1)( 4 n + 1) - 3] 1 1 +2 1 + 22 + 32
2
3
11. (c) If d be the common difference, then 2n + 1 6
nth term = Tn = =
m = a4 – a7 + a10 = a4 – a7 + a7 + 3d = a7 n ( n + 1) (2n + 1) n( n + 1)
13 13 6
S13 = [a1 + a13 ] = [ a1 + a7 + 6d ] é1 1 ù
2 2 or Tn = 6 ê - ú
13
= [2a7 ] =
2
EBD_7504
S-M-30 SOLUTIONS

1 1 6n 6
\ Sn = å Tn = 6 å -6å = - = (ar - ar 2 ) 2 + (ar 2 - a ) 2 + (ar 3 - ar) 2
n n +1 n n +1
2 2
6 6n = a 2 r 2 (1 - r ) 2 + a 2 (1 - r 2 ) + a 2 r 2 (1 - r 2 )
=6- =
n +1 n +1
= a 2 (1 - r) 2 [r 2 + (1 + r ) 2 + r 2 (1 + r ) 2 ]
So, sum upto 11 terms means
6 ´ 11 66 33 11 = a 2 (1 - r) 2 [r 2 + (1 + r) 2 (1 + r ) 2 ]
S11 = = = =
11 + 1 12 6 2 é 2 ù
16. (b) If x > 1, y > 1, z > 1 are in G.P. = a 2 (1 - r) 2 êr 2 + (1 + r 2 ) + 2r (1 + r 2 )ú
ë û
1 1 1
Then , , 2
1 + log x 1 + log y 1 + log z = a 2 (1 - r ) 2 (r + 1 + r 2 )
x > 1, y > 1, z > 1, 2 2
\ y2 = xz ...(i) = a 2 (1 - r 3 ) = (a - ar 3 ) = (a - d ) 2
Taking log on both sides of equ. (i), we get Further, (a 2 + b 2 + c 2 ) (b 2 + c 2 + d 2 )
2 log y = log x + log z
Þ 2 + 2 log y = 2 + log x + log z = ( a 2 + a 2 r 2 + a 2 r 4 ) (a 2 r 2 + a 2 r 4 + a 2 r 6 )
Þ 2 (1 + log y) = (1 + log x) + (1 + log z)
Clearly, (1 + log x), (1 + log y) (1 + log z) are in A.P. [
= a 4 r 2 (1 + r 2 + r 4 ) (1 + r 2 + r 4 ) = a 2 r(1 + r 2 + r 4 ] 2

1 1 1 2
Then , , are in H.P.. = ( a.ar + ar.ar 2 + ar 2 .ar 3 ) = (ab + bc + cd ) 2
1 + log x 1 + log y 1 + log z
17. (b) Since p, q, r are in A.P. 20. (d) Consider
p+r 1 æ 1 1ö 1 æ 1 1 ö 1 æ 1 1 ö
\ q= ç + ÷- + + + - ...
2 è 3 4 ø 4 çè 32 42 ÷ø 6 çè 33 43 ÷ø
...(i)
2
Since a is the G.M. between p, q
\ a2 = pq ...(ii) æ1 1 1 1 1 1 ö æ1 1 1 1 1 1 ö
= çè . - . 2 + . 3 ...÷ø + ç . - . 2 + . 3 -÷ ...
Since b is the G.M. between q, r 2 3 4 3 6 3 è2 4 4 4 6 4 ø
\ b2 = qr ...(iii)
From (ii) and (iii) 1æ1 1æ 1 ö 1æ 1 ö ö 1 æ 1 1 1 1 1 ö
- + ... + - . + . ..
2 çè 3 2 çè 32 ÷ø 3 çè 33 ÷ø ÷ø 2 çè 4 2 42 3 43 ÷ø
=
a2 b2
p= , r=
q q 1æ x 2 x3 ö 1 æ y 2 y3 ö
= ç x - + ...÷ + ç y - + ...÷
a 2 b2 2è 2 3 ø 2è 2 3 ø
\ (i) gives 2q = +
q q 1 1
where x = ,y=
2 2 2 2 2
Þ 2q = a + b Þ a , q , b are in A.P.. 2 3 4
n (n -1)
1 1
= log(1 + x) + log(1 + y)
a (1 - r n ) 2 2
18. (b) S= , P = a n .r 2
1- r 1 æ 1ö 1 æ 1ö 1 5
=log ç1 + ÷ + log ç1 + ÷ = log
1 1 1 1- r n 2 è 3 ø 2 è 4 ø 2 3
R= + + + ......n terms =
a ar ar 2 a (1 - r ) r n -1 21. (a) Sum of an A.P. is given by
n
æ Sö
n Sn = [2a + ( n - 1) d ]
Sn = R n P k Þ ç ÷ = P k 2
è Rø where 'a' is the first term and 'd' is the common difference
Þ (a r ) = Pk
2 n–1 n of A.P.
Þ P2 = Pk Þ k = 2 Let Sn1 be the sum of n terms of Ist A.P..
b c d
19. (d) We have = = =r and Sn2 be the sum of n terms of IInd A.P..
a b c
b = ar, c = br = ar 2, d = cr = br 2 = ar3 Given that the sum of n terms of two arithmetic series is
in the ratio 2n + 3 : 6n + 5
b + c ar + ar 2 c + d br + cr 2 Sn1
Now, = = r and = =r 2n + 3
a+b a + ar b+c b+c Þ = ..... (i)
Sn2 6n + 5
\a + b, b + c, c + d are in G.P.
Again (b - c) 2 + (c - a ) ) d1 ] = 2 n + 3 and
Mathematics S-M-31

n tn 3
2
[ 2a2 + (n - 1)d2 ] = 6n + 5
Sn2 = 25. (a) t n = Sn - Sn -1 = 3t n - 3t n -1 Þ
t n -1
=
2
From Eq. (i) , we get
3
n Hence the series is geometric with common ratio .
Sn1 [2a1 + (n - 1)d1 ] 2
2n + 3
= 2 = Also, S1 = 3t 1 - 2 Þ t 1 = 3t1 - 2 Þ t1 = 1
Sn2 n 6n + 5
[2a2 + (n - 1)d2 ]
2 n
æ3ö
n ç ÷ -1 ìïæ 3 ö n üï
2a1 + (n - 1)d1 2n + 3 a (r - 1) è 2 ø
Þ = Now, Sn = = = 2 íç ÷ - 1ý
2a2 + (n - 1)d2 6n + 5 r -1 3 ïîè 2 ø ïþ
-1
For a = 13, n = 2a – 1 = 2 × 13 – 1 = 25 2

2a1 + (25 - 1) d1 53 a1 + 12d1 53 \S1 + S2 + S3 + ...... + Sn


\ = Þ a + 12d = 155
2a2 + (25 - 1)d2 155 2 2
é 3 ì 3 2 ü ù
22. (c) a, b, c are in A.P. Þ 2b = a + c êìí - 1üý + ïíæç ö÷ - 1ïý + ú
êî 2 þ ïè 2 ø ï ú
î þ
Now, e1/ c ´ e1 / a = e(a + c) / ac = e 2b / ac = (e b / ac )
2 = 2ê ú
ê ìïæ 3 ö3 üï ìïæ 3 ö n üïú
ê íç ÷ - 1ý + ..... + íç ÷ - 1ýú
\ e1 / c , e b / ac , e1 / a in G.P. with common ratio ê ïîè 2 ø ïþ ïîè 2 ø ú
ë þïû
e b / ac 2
-d 2 )
= = e (b -a ) / ac = e d /( b -d )( b + d ) = e d /( b éìï 3 æ 3 ö 2 æ 3 ö 3 æ 3 ö üï ù
1/ c n
e = 2 êí + ç ÷ + ç ÷ + ........... + ç ÷ ý - n ú
[Q a, b, c are in A.P. with common êï 2 è 2 ø è 2 ø è 2 ø ïþ úû
ëî
difference d \ b – a = c – b = d]
2ab é3ì 3 n ü ù
H .M 12 ê ïíæç ö÷ - 1ïý ú
23. (d) Given that = a + b 12 ê 2 ïîè 2 ø ú
G.M 13 Þ ab = 13
ïþ é ìïæ 3 ö n üï ù
= 2ê - nú ê3 íç ÷ - 1ý - n ú
ê 3 ú =2
ê
-1
ú êë ïîè 2 ø ïþ úû
a+b 13 2
or = ëê ûú
2 ab 12
(a + b) + 2 ab 13 + 12 25 æ 3n ö
Þ = =
(a + b) - 2 ab 13 - 12 1 = 6 ç n - 1÷ - 2n
è2 ø
( a + b )2 52 a+ b 5 26. (d) a1, a2 , a3 ,........., an+1 are in A.P..
Þ = Þ =
-2 1 a- b 1
( a - b) and common difference = d
( a + b) + ( a - b) 5 +1 1 1 1
Þ = Let S = + + ....... +
( a + b) - ( a - b ) 5 -1 a1a2 a2 a3 an an +1

1/ 2 1ì d d d ü
6 æaö
2 a 6 ÞS= í + + .......... + ý
Þ =
Þ ç ÷ = Þ a :b = 9: 4 d î a1a2 a2 a3 an an +1 þ
2 b 4 èbø 4
24. (b) Let first term of an A.P. be a and c.d. be d and first term 1 ì a2 - a1 a3 - a2 a -a ü
of a G.P. be A and c.r. be R,then ÞS= í + + .......... + n +1 n ý
a + (p–1) d = ARp–1 = x d î a1a2 a2 a3 an an +1 þ
Þ p – 1 = (x– a) /d ...(1)
a + (q – 1) d = ARq–1 = y 1ì1 1 1 1 1 1 ü
ÞS= í - + - + .......... + - ý
Þ q – 1 = (y–a)/d ...(2) d î a1 a2 a2 a3 an an +1 þ
a + (r– 1) d= ARr–1 = z
Þ r – 1 = (z – a) / d ...(3) 1ì1 1 ü 1 ì an +1 - a1 ü
ÞS= í - ý= í ý
\ Given expression d î an an+1 þ d î a1an+1 þ
= (ARp–1)y–z, (ARq–1) z–x, (ARr–1)x–y
= A0 R(p–1)(y–z)+(q–1)(z–x)+(r–1)+(x–y) 1 ì nd ü n
= A0R[(x–a)(y–z)+(y–a)(z–x)+(z–a)(x–y)]/d ÞS= í ý=
d î a1an +1 þ a1an +1
[By (1), (2) and (3)]
= A0R0 = 1
EBD_7504
S-M-32 SOLUTIONS

27. (b) As x, y, z, are A.M. of a and b


é 4 æ 4 ö2 ù
æa+bö = 120 + 2 ´120 ê + ç ÷ + .......to ¥ú
\x + y + z = 3ç ÷ êë 5 è 5 ø úû
è 2 ø
3 é 4 ù
\15 = (a + b )Þ a + b = 10 ...(1) ê ú
2 = 120 + 240 ê 5 ú = 120 + 240 ´ 4 = 1080 m
4
ê1 - ú
1 1 1 1 1
Again , , are A.M. of and ëê 5 ûú
x y z a b
29. (c) Let the G.P. be a, ar, ar2, ..........
1 1 1 3æ1 1ö S = a + ar + ar2 + ..........+ to 2n term
\ + + = ç + ÷
x y z 2èa bø a(r 2 n - 1)
=
5 3 a+b r -1
\ = . The series formed by taking term occupying odd places
3 2 ab
is S1 = a + ar2 + ar4 + .......... to n terms
10 10
Þ = Þ ab = 9 ...(2) a é(r 2 )n - 1ù
9 ab ë û Þ a(r 2n - 1)
S1 = S1 =
Solving (1) and (2), we get 2
r -1 r2 -1
a = 9, 1, b = 1, 9 Now, S = 5S1
28. (c) Clearly, the total distance described
a ( r 2 n - 1) a ( r 2 n - 1)
é 4 4 4 4 4 4 ù
= 120 + 2 ê120 ´ + 120 ´ ´ + 120 ´ ´ ´ + ......to ¥ ú
or =5
5 5 5 5 5 5 r -1 r2 -1
ë û
Except in the first fall the same ball will travel twice in each 5
Þ 1=
step the same distance one upward and second downward r +1
travel. Þ r+1= 5\r=4
\ Distance travelled æ 11ö
30. (a) loge 5 + log e ç 5x - ÷ = 2 loge (5x - 1)
è 5ø
First fall Þ 5x+1 – 11 = 52x + 1 – 2 × 5x Þ 52x – 7.5x + 12 = 0
Let 5x = t, t2 – 7t + 12 = 0
Þ t = 4, 3
Rebound and 5x = 4, 5x = 3
second fall x
log55 = log54 log55x = log53
x = log54 x = log53
120m 4
120 ´ m
5

Speed Test-69

1. (c) One vertex of square is (– 4, 5) and equation of one


diagonal is 7x – y + 8 = 0 4 31
c = 5- =
Diagonal of a square are perpendicular and bisect each 7 7
other Hence equation of the other diagonal is
Let the equation of the other diagonal be y = mx + c 1 31
where m is the slope of the line and c is the y-intercept. y = - x+
7 7
Since this line passes through (– 4, 5) or x + 7y = 31.
\ 5 = –4m + c… (i) 2. (b) The lines are given by
Since this line is at right angle to the line (x – 2y)2 + k (x – 2y) = 0 Þ ( x - 2 y)( x - 2 y + k ) = 0
7x – y + 8 = 0 or y = 7x + 8, having slope = 7,
That is x – 2y = 0 and x – 2y + k = 0
-1 These are parallel. The distance between the two lines
\ 7 × m = – 1 or m=
7
Putting this value of m in equation (i) we get k
) \| k |= 3 5 Þ k = ±3 5
Mathematics S-M-33

3. (a) The parametric equation of a line through A is Þ 1 + r cos q + 2 + r sin q + 5 = 0


x+5 y+4 1 3
= =r Þ 1+ r +2+r + 5 = 0 Þ r = 2 10
cos q sin q 10 10
Let AB = r 1, AC = r 2 and AD = r3 6. (b) p12 = 4a 2 cos 2 4q
Then the coordinates of B, C, D are
(–5, + ricos q, – 4 + ri sin q), i = 1, 2, 3 16a 2 cos 2 2q 2 2 2 2
Now B lies on the line x + 3y + 2 = 0 p 22 = = 16a cos 2q cos q sin q = a2 sin2 4q
sec 2 q + cos ec 2 q
\ –5 + r 1cos q + 3 (–4 + r 1 sin q) + 2 = 0
15 \ p12 + 4p 22 = 4a 2
= cos q + 3 sin q 7. (c) We have the equation
r1
y2 + xy + px2 – x – 2y + p = 0
C lies on 2x + y + 4 = 0 We know any general equation
\ 2(-5 + r2 cos q) + (-4 + r2 sin q) + 4 = 0 ax2 + by2 + 2hxy + 2gx + 2fy + c = 0 ... (1)
represents two straight lines if
10 abc + 2fgh – af 2 – bg2 – ch2 = 0 ... (2)
Þ = 2 cos q + sin q
r2 On comparing given equation with (1), we get
D lies on x – y – 5 = 0 1 1
a = p, b = 1, h = , g = - , f = – 1, c = p
6 2 2
\ -5 + r3 cos q + 4 - r3 sin q - 5 = 0 Þ = cos q - sin q . Put these value in equation (2)
r3
1 1
2 2 2
p × 1 × p + 2 × – 1 × – × – p × (–1)2 – 1
æ 15 ö æ 10 ö æ6ö 2 2
From the given condition çç ÷÷ + çç ÷÷ = ç ÷ 2 2
çr ÷ æ 1ö æ1ö
è r1 ø è r2 ø è 3ø × ç - ÷ - p´ç ÷ = 0
è 2ø è 2ø
we get, (cos q + 3 sin q) 2 + ( 2 cos q + sin q) 2 = (cos q - sin q) 2
1 1 p 5p 1
Þ p2 +
- p - - = 0 Þ p2 - + = 0
2 2 4 4 4 4
Þ (2 cos q + 3 sin q) 2 = 0 Þ tan q = -
3 Þ 4p2 – 5p + 1 = 0 Þ (4p – 1) (p – 1) = 0
\ Equation of the line is 1
Þ p = 1,
2 4
y + 4 = - (x + 5) Þ 2 x + 3y + 22 = 0 8. (a) Since the two sides make an angle of 60° each with side
3 x + y = 2. Therefore equations of these sides will be
4. (b) The equation of any line parallel to 2x + 6y + 7 = 0 is
2x + 6y + k = 0 -1 ± tan 60° -1 ± 3
y–3= (x – 2) = (x – 2)
æ k ö æ kö 1 m (-1) tan 60° 1± 3
This meets the axes at A ç - , 0÷ and B ç 0, - ÷
è 2 ø è 6ø Þ y – 3 = (2 ± 3 ) (x – 2)
By hypothesis, AB = 10 9. (a) Let the point (h, k) lie on a line x + y = 4
then h + k = 4 ...(i)
k2 k2 10k 2
Þ + = 10 Þ = 10 4h + 3k - 10
4 36 36 and 1 = ± Þ 4h + 3k = 15 ...(ii)
4 2 + 32
Þ 10k2 = 3600Þ k = ±6 10
and 4h + 3k = 5 ...(iii)
Hence there are two lines given by On solving (i) and (ii); and (i) and (iii), we get the
2 x + 6 y ± 6 10 = 0 required points (3, 1) and (–7, 11).
5. (c) The slope of the line 3x – y = 7 is tan q = 3. Trick : Check with options. Obviously, points (3, 1)
and (–7, 11) lie on x + y = 4 and perpendicular
P 3 distance of these points from 4x + 3y = 10 is 1.
or = Þ H = 9 + 1 = 10
B 1
b
3 1 10. (d) Slope of the line in the new position is , since it is ^
\ sin q = ,cos q = a
10 10 to the line ax + by + c = 0 and it cuts the x-axis at (2,0). Hence,
The eqn of line passing through (1, 2) and parallel to b
y = 3x – 7 is the required line passes through (2, 0) and its slope is .
a
x -1 y - 2
= ...(i) Required eq. is
cos q sin q
Let r be the required distance. b
y-0 = (x - 2) Þ ay = bx - 2b Þ ay - bx + 2b = 0
\ (1 + r cos q, 2 + r sin q) lies on x + y + 5 = 0 a
EBD_7504
S-M-34 SOLUTIONS

11. (c) Let line be y – 3 = m (x – 2) x y


14. (c) Let the equation of the line be + =1.
3 a b
y intercept is (3 – 2m), x intercept is (2 – )
m Y
Area = 12
B (0, b)
1 3 2
\ 12 = 2 - | 3 - 2m |
2 m b
P
1 (–5, 4)
9
Þ 12 - - 4m = + 24 A a O
X
m
(a, 0)
\ 4m2 + 12m + 9 = 0 Þ m = – 3/2
So, the coordinates of A and B are (a, 0) and (0, b)
9 respectively. Since the point (–5, 4) divides AB in the
or 12 - - 4m = -24 Þ 4m2 – 36m + 9 = 0; D > 0 ratio 1 : 2
m
Þ There are two values of m. Hence total 3 values of m. 1.0 + 2.a
\ –5=
12. (b) We know that if m1 and m2 are the slopes of the lines 1+ 2
represented by ax2 + 2hxy + by2 = 0, 1.b + 2.0 15
and 4 = Þa=– and b = 12
2h 2 +1 2
then sum of slopes = m1 + m2 = - and
b 2 y
So the line is – x+ = 1, i.e. – 8x + 5y = 60
a 15 12
product of slopes = m1 m2 = . 15. (a) Let Q(a, b) be the reflection of P(4, – 13) in the line
b 5x + y + 6 = 0
Consider the given equation which is
æ a + 4 b - 13 ö
x2 + 2hxy + 2y2 = 0 Then the mid-point R ç , ÷ lies on
è 2 2 ø
On comparing this equation with ax2 + 2hxy + by2 = 0,
we have a = 1, 2h = 2h and b = 2 5x + y + 6 = 0
Let the slopes be m1 and m2. æ a + 4 ö b - 13
\ 5ç + + 6 = 0 Þ 5a + b + 19 = 0...(i)
Given : m1: m2 = 1: 2 è 2 ÷ø 2
Let m1 = x and m2 = 2x Also PQ is perpendicular to 5x + y + 6 = 0
2h b + 13 æ 5 ö
\ m1 + m2 = - Þ x + 2 x = - h Þ h = – 3x ...(i) Therefore ´ ç - ÷ = – 1 Þ a – 5b – 69 = 0..(ii)
2 a - 4 è 1ø
1 1 Solving (i) and (ii), we get a = – 1, b = – 14.
a
and m1m2 = Þ x . 2x = Þ x = ± ...(ii) 16. (b) We have the equation 2 x 2 - xy - y 2 = 0
b 2 2
3 Þ (2x + y) (x – y) = 0
\ From eqs. (i) and (ii), we have h = ± . If (h, k) be the point then remaining pair is
2 (2x + y + h) (x – y + k) = 0
13. (c) The equation of the line through (–1, 3) and having the
Where, 2x + y + h = 0 and x – y + k = 0
x +1 y - 3 It passes through the point (1, 0)
slope 1 is = =r. \ 2× 1+ 0 + h =0Þ 2+ h= 0 Þ h=–2
cos q sin q
Any point on this line at a =1 and 1 – 0 + k = 0 Þ 1+ k = 0 Þ k = – 1
m \ Required pair is (2x + y – 2) (x – y – 1) = 0
distance r from P (–1, 3) is P (–1, 3) Þ 2 x 2 - 2 xy - 2 x + xy - y 2 - y - 2 x + 2 y + 2 = 0

(-1 + r cos q, 3 + r sin q) \ 2 x 2 - xy - y 2 - 4 x + y + 2 = 0


2x + y = 3
This point is on the line 2x + y = 3 if ì x, x ³ 0
17. (b) Since | x | = í - x , x < 0 ,
2(- 1 + r cos q ) + 3 + r sin q = 3 ...(i) î
But tan q = 1; Þ q = 45° therefore the equations of two lines are
(i) becomes, y = 3 x + 2, x ³ 0 and y = - 3 x + 2, x < 0
Clearly y-axis the only bisector of the angle between these
1 1 3r 2 2 two lines. There are two points P and Q on these lines at a
- 2 + 2 r. + 3 + r. =3 Þ = 2; r =
2 2 2 3 distance of 5 units from A. Clearly M is the foot of the
perpendicular from P and Q on y-axis (bisector). AM = AP
Mathematics S-M-35

æ æx y ö
5 3 ö÷ Mid-point ç 1 , 1 ÷ lies on the line (i)
Hence, coordinates of M are çç 0, 2 + è 2 2ø
è 2 ÷ø
\ x1 + y1 = 14 ...(ii)
y1
Y Slope of RS = ; Since RS ^ line L
x1
Q
M
P y
\ 1 ´ (-1) = -1 \ x1 = y1 ...(iii)
x1
From (ii) and (iii), x1 = y1 = 7
y = 3x + 2 Hence the image of R = (7, 7)
y = - 3x + 2
A (0,2)
2 2
21. (a) Slope of given line is = – , \ tanq = –
3 3
O X
Hence 90° < q < 180°
2 3
\ sin q = , cosq = –
13 13
Equation of the line in parametric form,
18. (a) 9 x 2 – 6 xy + y 2 + 18 x - 6 y + 8 = 0
Þ ((3x)2 - 2 ´ (3x ) ´ y + y 2 ) + 6(3 x - y ) + 8 = 0 x -1 y +3
= =r
Þ (3x - y )2 + 6(3 x - y ) + 8 = 0 æ 3 ö æ 2 ö
ç- ÷ ç ÷
Let 3x - y = z è 13 ø è 13 ø
\ z2 + 6z + 8 = 0 Putting r = 3, we get the co-ordinate of desired point as
Þ z2 + 4z + 2 z + 8 = 0 9 6
Þ z ( z + 4) + 2( z + 4) = 0 x–1=– ,y+3=
13 13
Þ ( z + 2)( z + 4) = 0 9 6
Þ z = -2, z = -4 or x = 1 – , y = –3 +
13 13
3x - y + 2 = 0 ... (i) or 3x - y + 4 = 0 22. (a) Clearly the point (3, 0) does not lie on the diagonal
If P1 be the distance of line (i) from the origin, then x = 2y. Let m be the slope of a side passing through (3, 0).
2 2 Then its equation is
P1 = =
9 +1 10 y – 0 = m (x – 3) ....(i)
Also, if P2 be the distance of line (ii) from the origin, Since the angle between a diagonal and a side of a square
then
p
4 is . Therefore angle between
P2 = 4
10
So, distance between lines p
x = 2y & y – 0 = m( x - 3) is also
4 2 2 4
P = P2 - P1 = - =
10 10 10 1
19. (c) Since the hour, minute and second hand always pass m-
p 2 Þ m = 3, - 1
through origin because one end of these hands is Consequently, tan = ±
always at origin. Now at 4 O' clock, the hour hand 4 m 3
1+
makes 30° angle in fourth quadrant. So the equation 2
of hour hand is Y \ from (i), we get the required equations
1
y = 3(x - 3) Þ y - 3x + 9 = 0 or
1 2
y = mx Þ y = - x 1
3 y = - (x - 3) Þ 3y + x - 3 = 0
X 3
O 30° 3
Þ x + 3y = 0
4 Y

C
20. (d) Mid-point of P(2, 3) and Q(4, 5) = (3, 4) y
x=2
Slope of PQ = 1, Slope of the line L = – 1 B
Mid-point (3, 4) lies on the line L. D
Equation of line L,
y – 4 = – 1(x – 3) Þ x + y – 7 = 0 ...(i) X
Let image of point R(0, 0) be S(x1, y1) O A (3, 0)
æx y ö
Mid-point of RS =
EBD_7504
S-M-36 SOLUTIONS

23. (b) The length of perpendicular from P (2, –3) on the given
family of lines 2am - 2a - ma
=a (given)
a (4 - 3 + 4) + b(2 + 6 - 3) m2 + 1
= = ± 10 (given)
(2a + b) 2 + (a - 2b) 2 Þ (m - 2) 2 = (m 2 + 1) Þ m 2 - 4 m + 4 = m 2 + 1

Þ 5a + 5b = ± 10(5a 2 + 5b 2 ) 3
Þ 0 m 2 - 4m + 3 = 0 Þ m = ,¥
4
Þ 25(a + b) 2 = 50(a 2 + b 2 ) Þ 25(a - b) 2 = 0 Þ a = b The required equation of lines are, from (i)
For which we get only line 3x – y + 1 = 0 3x – 4y – 3a = 0 and x – a = 0.
24. (a) The line passing through the intersection of lines 28. (a) Let (1,3) and (5,1) represent vertices A and C. The middle
ax + 2by = 3b = 0 and bx - 2ay - 3a = 0 is point G(3,2) must lie on the diagonal BD, whose equation
is y = 2x + c
ax + 2by + 3b + l (bx – 2ay – 3a) = 0
\ 2 = 2.3 + c Þ c = –4
Þ (a + b l ) x + (2b – 2a l )y + 3b – 3 l a = 0 \ equation of BD is y = 2x – 4
As this line is parallel to x-axis.
1
\ a + b l = 0 Þ l = – a/b Also GA = GB = GC = GD = AC = 5
a 2
Þ ax + 2by + 3b – (bx – 2ay – 3a) = 0 We have to find two points along BD at distances
b
2a 2 3a 2 ± 5 from G. For this we convert equation of BD into
Þ ax + 2by + 3b – ax + y+ =0 distance form.
b b
Slope of line BD = tan q = 2
æ 2a 2 ö 3a 2
y ç 2b + ÷ + 3 b + =0 2 1
è b ø b \ sin q = and cos q =
5 5
æ 2b 2 + 2a 2 ö æ 3b2 + 3a 2 ö \ Distance form of the line BD through G is
yç ÷ = - ç ÷
è b ø è b ø x -3 y- 2
= =r
-3(a 2 + b 2 ) -3 1 2
y= =
2(b 2 + a 2 ) 2 5 5
So it is 3/2 units below x-axis.
25. (b) The distance between the parallel straight lines given Put r = ± 5 to get the vertices of B and D as (4,4) and
by (2,0)
29. (d) As (sinq, cosq) and (3, 2) lie on the same side of
g 2 - ac x + y – 1 = 0, they should be of same sign.
ax 2 + 2hxy + by 2 + 2 gx + 2 fy + c = 0 is 2
a(a + b) \ sinq + cosq – 1 > 0 as 3 + 2 – 1 > 0
Here, a = 8, b = 2, c = 15, g = 13. æ pö
So, required distance Þ 2 sin çè q + ÷ø > 1
4
169 - 120 7 7 æ pö 1 p
= 2 = 2´ =
80 4 5 2 5 Þ sin çè q + ÷ø > Þ0<q<
4 2 4
26. (b) Let the slope of line L be m. 30. (b) 6x + 8y + 15 = 0 ... (i)
m+ 3 and 3x + 4y + 9 = 0 ... (ii)
Y
Then = 3 6x + 8y + 15 = 0
1 - 3m 3x + y = 1
L
(0, 1)
Þ m + 3 = ± ( 3 – 3m) X¢ O X d
60°
Þ 4m = 0 or 2m = 2 3 (3, –2)

Þ m = 0 or m = 3 6x + 8y + 18 = 0
Q L intersects x-axis, \m= 3 Multiply equation (ii) by 2, we get
6x + 8y + 18 = 0
\ Equation of L is y + 2 = 3 (x – 3) Distance between the straight lines
or 3 x – y – (2 + 3 3 ) = 0 c 2 – c1 18 – 15 3
27. (c) Equation of line passing through (a, 0) is y = m(x – a) = = unit
2 2 10
Þ mx - y - ma = 0 ....(i)
a +b (6 ) 2
+ (8)
2
Speed Test-70

1. (b) Let the two given circles be


\ Equation of the circle is x 2 + y 2 - ax - by = 0 .
x2 + y2 + 2g1x + c = 0 ...(1)
and x2 + y2 + 2g2x + c = 0 ...(2) Equation of the tangent at the origin is ax + by = 0 ...(i)
Their centres are A (–g1, 0) and B (– g2, 0) Let AL and BM be the perpendicular from A and B on (i),
\ AB = g1 – g2
Let P be the point (x1, y1). Then, a2 b2
then AL = = m and BM = =n
PT = x12 + y12 + 2g1 x1 + c ; a 2 + b2 a 2 + b2

Þ m + n = a 2 + b 2 = diameter of the circle.


PT = x 12 + y12 + 2g 2 x1 + c
4. (a) Given parabola is y2 =4a (x – a)
Radical axis of (1) and (2) is 2 (g1 – g2) x = 0 or x = 0, Equation of tangent : SS1 = T2
PN = length of ^ from P on radical axis = x1. [y2 – 4a (x – a)] [y12 – 4a (x1 – a)]
\ PT 2 - PT ' 2 é
2
æ x + x1 öù
= ( x12 + y12 + 2g1x1 + c) - (x12 + y12 + 2g 2 x1 + c) = ê yy1 - 4a ç - a÷ ú
ë è 2 øû
= 2x1 (g1 – g2) = 2 PN. AB At x1 = 0, y1 = 0, we have
2. n
(d) Eq of circle is (y2 – 4ax + 4a2) 4a2 = (–2ax + 4a2)2
Þ y2 – 4ax + 4a2 = x2 + 4a2 – 4ax Þ x2 – y2 = 0
x2 + y2 – 6x + 2y – 28 = 0
2 h 2 - ab = 2 0 - 1.(-1) = ¥
2g = – 6 Þ g = – 3 and 2f = 2 Þ f = 1 Þ tan q = 1 + (-1)
a+b
c = – 28 Þ q = 90°

x2 y2
\r= g2 + f 2 - c = 9 + 1 + 28 5. (a) Let the equation of the ellipse be + =1.
a2 b2
If S and S' be the foci, then SS' = 10.
= 38
But SS' = CS + CS' = 2ae, C being the centre
centre : = (–g, –f) = (3, – 1) = (h, k) \ 2ae = 10, or ae = 5 ...(1)
\ x = h + r.cos q, and y = k + r sin q
b2
Also 2 = 15 ...(2)
Þ x = 3 + 38 cos q, y = –1 + 38 sin q a

Also b 2 = a 2 (1 - e 2 ) = a 2 - a 2 e 2 = a 2 - 25 [using (1)]


3. (b) Y
B(0, b) By (2), 2b 2 = 15a; or 2(a 2 - 25) = 15a
B
n 5
C(a/2,b/2) \a = - or a = 10.
M 2
But a cannot be negative,
O X
A (a, 0) 15 ´10
\ a = 10; \ b 2 = = 75 .
m 2
L
The equation to the ellipse is therefore

Let the coordinates of A be (a, 0) and of B be (0, b), then AOB x 2 y2


being a right angled triangle the centre of the circumscribed + = 1; or 3x 2 + 4 y 2 = 300 .
100 75
æa bö 6. (b) Points P and Q are (a, 0) and (2a, 0)
circle is mid-point ç , ÷ of AB and radius is
è2 2ø So, the circles are
( x - 0)( x - a ) + ( y - 0)( y - 0) = 0 Þ x2 + y2 – ax = 0
a 2 b2
OC = +
4 4 0) = 0 Þ x2 + y2 – 3ax + 2a2 = 0
EBD_7504
S-M-38 SOLUTIONS

( x - 2a )(x - 3a ) + ( y - 0)( y - 0) = 0 \ (2at2)2 = (2at1) (2at3) or t 22 = t 1t 3 ...(2)


2 2 2
Þ x + y - 5ax + 6a = 0 The tangents at E and G are
Now, S be (h, k) then
t1y = x + at12 ...(3)
2 2 2 2 2 2 2 2 2
h + k - ah + h + k - 3ah + 2a + h + k - 5ah + 6a = b
Þ 3(h 2 + k 2 ) - 9ah + 8a 2 = b 2 and t3y = x + at 32
7. (a) Equation of chord PQ is y = x + 2 2
Solving (3) and (4), we get x = at1t3 = at 2 [from (2)]
or x – y + 2 = 0 ...(1)
2
Since the x-coordinate of the point of intersection is at 2 , the
2
point lies on the line x = at 2 i.e. on the ordinate of
C (–2, 1)
2
F ( at 2 , 2at2).
y=x+2
P M Q 10. (c) Given curve is x 2 - 2 x + y - 2 = 0
Centre of circle is C (–2, 1).
Þ x2 - 2 x + y - 2 + 1 = 1
Draw CM ^ PQ, then M is the mid point of PQ.
Equation of any line ^ to PQ is x + y + k = 0
Þ ( x - 1)2 = - y + 3 = -1( y - 3)
If it passes through C(–2, 1) then –2 + 1 + k = 0 or k = 1
Equation of CM is x + y + 1 = 0 ...(2) 1
which is downward parabola with a =
3 1 4
Solving (1) and (2), we obtain x = - and y = .
2 2 We know, if l1 and l2 are the length of the segment of
æ -3 1 ö any focal chord then length of semi-latus rectum is
\ Coordinates of M are ç , ÷
è 2 2ø 2l1l2
8. (a) Equation of the ellipse is 3x2 + 4y2 = 12 l1 + l2

x2 y2 Here AS =l1 and BS = l2 (say) are the segments.


Þ + =1 .... (1)
4 3 2l1 ( BS ) l1
\ we have = 2a Þ BS =
l1 + BS 4l1 - 1
3 1
Eccentricity e1 = 1 - = 11. (d) Since the ([P + 1], [P]) lies inside the circle
4 2
x2 + y2 – 2x – 15 = 0 [But [x + n] = [x] + n, n Î N]
So, the foci of ellipse are (1, 0) and (– 1, 0) \ [P + 1]2 + [P]2 – 2[P + 1] – 15 < 0
Let the equation of the required hyperbola be ([P] + 1)2 + [P]2 – 2([P] + 1) – 15 < 0
2[P]2 – 16 < 0, [P]2 < 8 ...(1)
x2 y2
- =1 .... (2) From the second circle
a2 b2 ([P] + 1)2 + [P]2 – 2([P] + 1) – 7 > 0
Given 2a = 2 sin q Þ a = sin q Þ 2[P]2 – 8 > 0, [P]2 > 4 ...(2)
Since the ellipse (1) and the hyperbola (2) are confocal, so From (1) & (2), 4 < [P]2 < 8, which is not possible
\ for no values of ‘P’ the point will be within the region.
the foci of hyperbola are (1, 0) and (– 1, 0) too. If the
eccentricity, of hyperbola be e2 then 12. (b) Given circle, x 2 + y 2 = 9

ae2 = 1 Þ sin q e2 = 1 Þ e2 = cosec q B


A
\ b = a (e – 1) = sin q (cosec q – 1) = cos q
2 2 2 2 2 2
2
P O x2 + y 2 = 9
\ Required equation of the hyperbola is (0, 0)
(3,11) 3
T
x2 y2
– = 1 Þ x cosec q – y sec q = 1
2 2 2 2
PA ´ PB = PT 2 [ By Geometry]
sin 2 θ cos 2 θ
9. (c) Given parabola is y2 = 4ax ...(1) PA ´ PB = (3)2 + (11)2 - 9 = 121
Let the coordinates of E, F and G be respectively 13. (b) Let z = 1 - t + i t 2 + t + 2
( at 1
2
, 2at1 ) , ( at2 , 2 at
2
) and ( at 2
, 2at ) We know that z = x + iy
Since ordinates of E, F
+2
Mathematics S-M-39

compare real and imaginary part, we get Area of the rectangle ABCD
x=1–t Þ t=1–x = (a cos q + a cos q) (b sin q + b sin q)
2 = 2a cos q × 2b sin q = 2ab sin 2q
and y = t + t + 2
= 2ab × 1 = 2ab
2 2 16. (a) Let the required point be (x1, y1). The given line
Þ y =t +t+2
3x + 2y + 1 = 0 ...(1)
Þ y 2 = (1 – x )2 + (1 - x ) + 2 is chord of contact of the point so it must be same as
Þ y2 = 1 + x 2 - 2x + 1 - x + 2 the line
T = 0, i.e. 4xx1 – yy1 = 4a2 ...(2)
Þ y 2 = x2 - 3x + 4 Comparing the coefficients of (1) and (2), we get
2
2 æ 3ö 7 4x1 - y1 4a 2
Þ y =çx- ÷ + = = Þ x1 = - 3a 2 , y1 = 8a 2
è 2 ø 4 3 2 -1
Which is a hyperbola.
17. (a) Let P(h, k) be a point on the circle
14. (c) The given conic is ax + by = 1 15x 2 + 15y 2 - 48x + 64 y = 0
Squaring both sides, ax + by + 2 abxy = 1 Then the lengths of the tangents from P(h, k) to
5x 2 + 5y 2 - 24x + 32 y + 75 = 0 and
or ax + by - 1 = -2 abxy
5x 2 + 5y 2 - 48x + 64y + 300 = 0 are
Squaring again, (ax + by - 1) 2 = 4abxy
24 32
or a 2 x 2 - 2abxy + b 2 y 2 - 2ax - 2by + 1 = 0 … (1) PT1 = h 2 + k 2 - h+ k + 15
5 5
Comparing the equation (1) with the equation
48 64
Ax 2 + 2Hxy + By 2 + 2Gx + 2Fy + C = 0 and PT2 = h 2 + k 2 - h+ k + 60
5 5
\ A = a2 , H = –ab, B = b2 , G = – a, F = –b, C = 1
Then, D = ABC + 2 FGH - AF 2 - BG 2 - CH 2 48 64 24 32 32 24
or PT1 = h- k- h + k + 15 = k- h + 15
15 15 5 5 15 15
= a 2 b 2 - 2a 2 b 2 - a 2 b 2 - a 2 b 2 - a 2 b 2 = -4a 2 b 2 ¹ 0
(Since (h, k) lies on 15x 2 - 15y 2 - 48x + 64y = 0
and H 2 = a 2 b 2 = AB
48 64
So we have D ¹ 0 and H2 – AB = 0. Hence the given equation \h2 + k2 - h+ k = 0)
represent a parabola. 15 15

x2 y2 and PT2 =
48
h-
64
k-
48
h+
64
k + 60
15. (b) Given equation of ellipse, +
=1
a 2 b2 15 15 5 5
Let A (a cos q, b sin q) be any point on ellipse 96 128 24 32
(1st quadrant) = - h+ k + 60 = 2 - h+ k + 15 = 2PT1
15 15 15 15
Coordinate of B = [a cos (p – q), b sin (p – q)]
Þ PT1 : PT2 = 1 : 2
= (– a cos q, b sin q) (2nd quadrant)
18. (a) 0
Coordinate of C = [a cos (p + q), b sin (p + q)] Y 1=
K y+
(3rd quadrant) x–
Coordinate of D = [a cos (2p – q), b sin (2p – q)] ix
ctr A
re
= (a cos q, – b sin q) (4th quadrant) di
S X
Y
Axis
The length of the perpendicular drawn from the given focus
(–a cos q, b sin q) (a cos q, b sin q) upon the given line
B A
0 - 0 +1 1
X¢ X x - y + 1 = 0 is =
O (1) 2 + (-1) 2 2.
C D The directrix is parallel to the tangent at the vertex.
(–a cos q, –b sin q) x - y + l = 0 , where l is
EBD_7504
S-M-40 SOLUTIONS

But the distance between the focus and the directrix = 2 ×


x2 y2
(the distance between the focus and the tangent at the vertex) Let the ellipse be + =1 ....(1)
1 a2 b2
= 2´ = 2.
2 b2
Then e 2 = 1 - ....(2)
0-0+l a2
Hence = 2.
(1) 2 + (-1) 2 Let a point P on (1) be (a cosq, b sinq).
The coordinates of foci are S1 (ae, 0) and S2 (–ae, 0).
\ l = 2. [l must be positive see figure]
\ The directrix is the line x – y + 2 =0. Hence, S1P = a (1 - e cos q)
Let (x, y) be a moving point on the parabola. By the focus- S2 P = a (1 + e cos q) and S1S2 = 2ae
directrix property of the parabola, its equation is If (h, k) be the coordinates of in centre then
2
æ x-y+2ö 2ae ´ a cos q + a (1 - e cos q) ´ -ae + a (1 + e cos q) ´ ae
( x - 0) 2 + ( y - 0) 2 = çç ± ÷÷ h=
è 2 ø 2ae + a (1 - e cos q) + a (1 + e cos q)
or x 2 + y 2 + 2 xy - 4 x + 4 y - 4 = 0 .
2ae cos q
19. (d) Given, x = 2 – 3 sec t, y = 1 + 4 tan t = ....(3)
1+ e
x-2 y -1 be sin q
Þ sec t = , tan t = k=
-3 4 ....(4)
1+ e
Since, sec2t – tan2 t = 1 2
h2 k2 æ e ö
2 2 Squaring and adding (3) & (4) we have, + =ç ÷
( x - 2) ( y - 1) 4a 2 b 2
è1+ e ø
\ - =1,
9 16 \ The locus of the point (h, k) is
which is a hyperbola with centre at (2,1) and eccentricity e, x2 y2 e
+ = 1 , where l =
.
2 2 2 2 2 1+ e
given by 16 = 9(e - 1) 4a l b l
Which is another ellipse with eccentricity
25 5
Þ 9e 2 = 25 Þ e 2 = Þe=
9 3 b2 3 + e2
= 1- =
20. (b) Let the variable circle is 4a 2 4

x 2 + y 2 + 2 gx + 2 fy + c = 0 ......(1) 22. (b) Let the points lie on the circle

It passes through (a, b) x 2 + y 2 + 2gx + 2fy + 2fy + k = 0, then

\ a 2 + b2 + 2 ga + 2 fb + c = 0 ......(2) c2 c
c 2 t i2 + + 2gct i + 2f +k =0
t i2 ti
(1) cuts x 2 + y 2 = 4 orthogonally
\ 2( g ´ 0 + f ´ 0) = c - 4 Þ c = 4 Þ c 2 t i4 + 2gct 3i + kt 3i + 2fct i + c 2 = 0

\ from (2) a 2 + b2 + 2 ga + 2 fb + 4 = 0 Its roots are t 1 , t 2 , t 3 , t 4 so

\ Locus of centre (–g,–f) is c2


t1t 2 t 3 t 4 = =1
a 2 + b2 - 2ax - 2by + 4 = 0 c2

or 2ax + 2by = a 2 + b2 + 4 2g 2gc


Also, t 1 + t 2 + t 3 + t 4 = - 2
=-
c c
21. (c)
23 (b) Here equation of the given circle is x2 + y2 – 2x = 0
This clearly passes through origin
P(a cos q, b sin q) Hence if (x1, y1) be midpoint of the chord then its
equation is given by T = S1
Þ xx1 + yy1 – (x + x1) = x12 + y12 – 2x1
S2 S1 or xx1 + yy1 – x = x12 + y12 – x1
This passes through the origin (0, 0)

x2 + y2 = x
Mathematics S-M-41

x2 y 2 Þ (3m 2 + 1) 2 = 8(m 4 + m 2 ) Þ m 4 - 2m 2 + 1 = 0 Þ m = ±1
24. (c) The ellipse can be written as, + =1
25 16 Hence, the required tangents are y = x + 2 and y = –x – 2.
Here a2 = 25, b2 = 16, but b2 = a2 (1 – e2)
Þ 16/25 = 1– e2 x2 y2
28. (d) Let the equation of ellipse be + =1
Þ e2 =1 – 16/25 = 9/25 Þ e = 3/5 a2 b2 Y
Foci of the ellipse are (± ae, 0) = (± 3, 0), The point P (4, –1) lies on it P(4,–1)
i.e., F1 and F2
\ We have PF1 + PF2 = 2a = 10 for every point P on the 16 1 C X
\ + =1 ...(1)
ellipse. 2 2
a b
25. (a) Let the equation of asymptotes be Equation of a tangent to the ellipse is
2x2 + 5xy + 2y2 + 4x + 5y + l = 0 ......... (1)
This equation represents a pair of straight lines, therefore 1 5
abc + 2fgh – af2 – bg2 – ch2 = 0 x + 4y – 10 = 0 Þ y = - x +
4 2
25 25
\ 4l + 25 - -8-l ´ =0 2 2
2 4 æ5ö æ 1ö
\ç ÷ = a 2 ç - ÷ + b2 [using c2 =a2m2 +b2]
9l 9 è2ø è 4ø
Þ - + = 0Þl = 2
4 2
Putting the value of l in eq. (1), we get 25 a 2 a2
2x2 + 5xy + 2y2 + 4x + 5y + 2 = 0 Þ = + [from (1)]
4 16 a 2 - 16
this is the equation of the asymptotes.
26. (d) The centre of two circles are C1 (2, 2) and C2 (0, 0). The Þ a 4 - 100a 2 + 1600 = 0 Þ a 2 = 80 or a 2 = 20
radii of two circles are r1 = 2 2 and r 2 = 4
Þ a = 4 5 or a = 2 5
Y
From equation (1)
B
5
C1 If a2 = 80, b2 = and if a2 = 20, b2 = 5
4
\ Equation of the ellipse
X
C2 A
x 2 y2 x 2 y2
+ = 1 or + =1
80 5 / 4 20 5
29. (a)
The eq. of the common chord of the circles x2 + y2 – 4x Y
– 4y = 0 and x2 + y2 = 16 is x + y = 4 which meets the P (4a,4a)
circle x2 + y2 = 16 at points A(4, 0) and B(0, 4). Obviously
OA ^ OB. Hence, the common chord AB makes a right 45°
K A 45° N X
angle at the centre of the circle x 2 + y 2 = 16 . Where,
O is the origin and the centre C 2 of the second circle. Q (4a,–4a)

2
27. (c) Any tangent to parabola y2 = 8x is y = mx + ...(i)
m Let A be the vertex of the given parabola y2 = 4ax.
PNQ is the double ordinate of length 8a.
It touches the circle x 2 + y 2 - 12x + 4 = 0 , if the length \ PN = NQ = 4a
of perpendicular from the centre (6, 0) is equal to radius Now the y co-ordinates of P and Q are 4a and –4a.
Therefore from the equation of the parabola y2 = 4ax, we get
32 . (4a)2 = 4ax, or x = 4a.
\ the co-ordinates of P and Q are (4a, 4a) and (4a, –4a).
2 i.e. AN = PN = NQ
6m + 2
m æ 1ö Thus, ÐNAP = 45°, ÐNAQ = 45°
\ = ± 32 Þ ç 3m + ÷ = 8(m 2 + 1)
m2 +1 è mø
EBD_7504
S-M-42 SOLUTIONS

30. (d) The centre D of the required circle is the image of the centre

C(–8,12)
C of the given circle in the line mirror. If D be (a, b) then

a + 8 b - 12 é 4 ´ -8 + 7 ´ 12 + 13 ù
= = -2 ê ú [See straight line]
L
4 7 ë 42 + 72 û
a + 8 b - 12 -2 ´ 65
Or, = = = -2
4 7 65
\a = -16, b = -2
D
\ Required circle is ( x + 16) 2 + ( y + 2) 2 = 52
The centre of the given circle is (–8, 12) and radius is 5.
The image of the circle will have the same radius, i.e. the
radius of the required circle is 5.

Speed Test-71

é 100 x2 ù æ ax2 + bx + c ö
ê x æ 2ö ú 2sin2 ç ÷
+ ç cos ÷
1. (d) Consider xlim
®¥ ê e x è xø ú 1 - cos(ax 2 + bx + c) è 2 ø
ëê ûú 3. (c) lim = lim
x®a ( x - a )2 x®a ( x - a )2
x2
x100 é æ 2öù é a( x - a )( x - b) ù
2
= lim + lim êcos ç ÷ ú
x ®¥ ex x ®¥ ë è xøû ê sin 2 ú a 2 ( x - b) 2
= lim 2 ê ú ´
x ®a ê a ( x - a )( x - b) ú 4
x100
= lim = 0 (Using L¢ Hopital’s rule) ë 2 û
x®¥ ex [using ax2 + bx + c = a (x – a) (x – b)]
x2
æ 2ö a2
and lim ç cos ÷ is of (1¥) form = (α - β) 2
x ®¥ è xø 2
æ 2 ö 1
lim x 2 ç cos - 1÷ 4. (d) Putting x = , we get
x ®¥ è x ø y
=e
n/ y
4
(cos t –1) æ a y + a y + ..... + a y ö
lim
2 æ 2 2ö L = limit = lim ç 1 2 n
÷
=et ® 0 t çè Put = t Þ x = ÷ø y®0 è n ø
(Q x®¥ y®0)
x t

=e
- lim ç
æ 1-cos t ö
t ®0 çè t 2 ÷ø
÷.4 æ sin t ö
– lim ç
® è 2t ÷ø
4
= e -2
\ log e L = lim
y ®0
n
y
1 y
.log e a1
n
( y
a+2 )
æ 0ö
+ a+ny ç ÷
.....
è 0ø
= e t 0
2. (a) We have, æ a y log a + a y log a + ..... + a y log a ö
1 1 2 2 n n
1 1 1 ç y y y ÷
y= + + è a1 + a 2 + ...... + a n ø
x x b g
x x a g
x a
xb = n lim
1+ a + a 1+ b + b 1+ g + g y ®0 1
x x x x x x [using L¢Hopital rule]
xa xb xg = n.
log( a1a2 ....an )
= + +
a
x +x +x b g a
x +x +x b g a
x +x +x b g n
\ log L = log( a1. a2 .....an ) Þ L = a1. a 2 .a3 ......a n
x a + xb + x g
= =1
x a + xb + x g
dy
\ =0
dx
Mathematics S-M-43

e{x} - {x} - 1 = y lim (1 + 0)1 n = y ´ 10 = y ´ 1 = y


5. (d) Let [a] = n, then xlim n®¥
®n - {x}2 In given question, y = 4, x = 5
e{n - h} - {n - h} - 1 e1- h - (1 - h ) - 1 1
= lim 2
= lim 2 = e– 2 \ lim (4 n + 5n ) n = 4
h ®0 {n - h} h ®0 (1 - h ) n®¥
cosec x
e{x} - {x} - 1 e{n + h} - {n + h} - 1 ì1 + tan x ü
and xlim = lim 9. (b) Consider xlim í ý
® 0 î 1 + sin x þ
®n + {x}2 h ®0 {n + h}2
1/ cos x
h 2 h3 é cos x ù
h 1+ h + + + .... - h - 1 æ
ê 1+ sin x ö sin x ú
e - h -1 2! 3! 1
= lim = lim = ê çè cos x ÷ø ú
h ®0h2 h ®0 h 2
2 ê
= lim ë ûú
\ Limit does not exist. 1/ sin x
x®0 (1 + sin x )
l/x
æ a x + bx + cx ö
6. (d) Consider lim ç ÷ sin x 1
x ®0 è 3 ø [Q tan x = and cosec x = ]
cos x sin x
æ a x +b x + c x ö l æ a x +b x + c x -3 ö n
-1÷ ´ l lim ç ÷ æ 1ö
lim ç We know, lim ç 1 + ÷ = e
x ®0 çè 3 ÷ø x x ®0 èç 3x ø÷ n® 0 è nø
=e =e
1/ cos x
l æ a x -1 b x -1 c x -1 ö é cos x ù
+ +
lim ç
3 x®0 çè x x x ÷ø
÷
ê æ 1 + sin x ö sin x ú
=e ê çè cos x ÷ø ú
= el\3 (log a + log b + log c) êë úû
\ lim 1/ sin x
=e
λ / 3(log abc )
= (abc )λ / 3
x®0 (1 + sin x )
7. (c) Area of D PQR = A 1
é cos x ù cos x
-1 êæ ö sin x ú
= é x ( b - 0 ) + 0 ( 0 - y ) + a ( y - b ) ùû
2 ë êç 1 ÷ ú
ê ç1 + cos x ÷ ú
-1 êç ÷ ú
= [ bx + ax - ab] ( as y = x ) êè sin x ø ú
2
= lim ë û
dA -1 x®0 é
æ 1 ö
cosec xù
\ = ( a + b) ê ç1 + ú
dx 2 ÷
êë è cosec x ø úû

1
lim
e
cos
x®0 x e
= = = 1.
Q (0, b) e e
R (x, y) é sin [ x - 3] ù
x 10. (c) lim ê ú
y= x ® 0 ë [ x - 3] û

For x ® 0+ , [ x - 3] = -3
O sin [ x - 3] sin ( -3) sin 3
\ = = Î (0,1)
P (a, 0) [ x - 3] -3 3
sin [ x - 3]
n n 1n \ lim+ =0
8. (a) We have lim ( y + x ) , 0 < x < y x®0 [ x - 3]
n® ¥
For x ® 0- , [ x - 3] = -4
n ù1 n é æ n ù1/ n
é x xö
= lim y ê1 + n ú = y lim ê1 + ç ú sin [ x - 3] sin 4
n®¥ ê y ûú n®¥ ê è y ÷ø ú \ = lies in (–1, 0)
ë ë û [ x - 3] 4
n
æ xö - 1 \ Limit does not exist.
As n ® ¥, ç ÷
è yø
EBD_7504
S-M-44 SOLUTIONS

4 + 3an 4 + 3an ì
11. (a) We have an +1 = 3 + 2a Þ lim an +1 = lim 3 + 2a ïlog (2 + x ) if | x |< 1
n n®¥ n®¥ n ï -2 n
ï x log( 2 + x ) - sin x
f (x ) = í lim = - sin x if | x |> 1
4 + 3a ïn ®¥ x -2 n + 1
Þ a= Þ2a 2 = 4 Þa = 2 ï1
3 + 2a ï [log( 2 + x ) - sin x ] if | x |= 1
î2
a ¹ – 2 because each an > 0, therefore lim an = a > 0.
Thus lim f ( x ) = lim (- sin x ) = - sin 1
12. (c) We have, f ( x ) = a ( x ) b ( x ) g ( x ) , for all real x x ®1+ x ®1

Þ f ¢ ( x ) = a¢ ( x ) b ( x ) g ( x ) + a ( x) b¢ ( x ) g ( x ) and lim f ( x ) = lim log( 2 + x ) = log 3.


x ®1- x ®1
+a ( x ) b ( x ) g ¢ ( x ) n
r3 - 8 æ 33 - 8 ö æ 43 - 8 ö æ 3 ö
÷........ç n - 8 ÷
Þ f ¢ ( 2) = a ¢ ( 2) b ( 2) g ( 2) + a ( 2) b ¢ ( 2) g ( 2)
15. (a) lim
n®¥
Õ r3 + 8 = Lim ç ÷ç
n ®¥ç 33 + 8 ÷ ç 43 + 8 ÷ ç n3 + 8 ÷
r =3 è øè ø è ø
+a ( 2 ) b ( 2 ) g ¢ ( 2 ) æ 3 - 2 32 + 4 + 2(3) ö
= lim ç . ÷
18 f ( 2 ) = 3a ( 2 ) b ( 2 ) g ( 2 ) - 4a ( 2 ) b ( 2 ) g ( 2 ) n®¥ è 3 + 2 32 + 4 - 2(3) ø

+ k a ( 2) b( 2) g ( 2) æ 4 - 2 4 2 + 4 + 2(4) ö æ n - 2 n 2 + 4 + 2n ö
ç . ÷ .....ç . ÷
[Q f ¢ ( 2 ) = 18 f ( 2 ) , a ¢ ( 2 ) = 3a ( 2 ) ,b ¢ ( 2 ) ç 4 + 2 4 2 + 4 - 2(4) ÷ ç n + 2 n 2 + 4 - 2n ÷
è ø è ø
= - 4b ( 2) and g ¢ ( 2) = kg ( 2)]
æ3-2 4-2 5-2 n - 2 ö æ 32 + 4 + 2(3)
= lim ç . . ........ ÷ç .
Þ 18 f ( 2 ) = ( - 1+ k ) a ( 2 ) b ( 2 ) g ( 2 ) n®¥ è 3 + 2 4 + 2 5 + 2 n + 2 ø è 32 + 4 - 2(3)

= ( k - 1) f ( 2 ) 42 + 4 + 2(4) n 2 + 4 + 2n ö
2
...... 2 ÷
4 + 4 - 2(4) n + 4 - 2n ø
[Q f ( 2 ) = a ( 2 ) b ( 2 ) g ( 2 ) ]
Þ k - 1 = 18 æ 1.2.3.4.5.6.7....... ö æ 19.28.39.52.63..... ö 1.2.3.4 2
=ç ÷ç ÷= =
\ k = 19 è 5.6.7.8......... ø è 7.12.19.28.39.52...... ø 7.12 7
2x
sin x 4 - x 4 cos x 4 + x 20 æ l mö
13. (c) lim 16. (c) lim ç1 + + ÷ [1¥ form]
x® 0 2 x4 x ®¥ è x x2 ø
x 4 (e 1 - 2 x4 )
æ l m ö æ 2m ö
Lim ç 2 l + ÷
sin t - t cos t + t 5 lim ç 1+ + -1÷ 2 x
x ®¥ è x x 2 ø = e x ®¥ è x ø
= e 2l for any value
= lim =e
2t
t ®0 t (e - 1 - 2t ) of m

t3 t5 æ t2 t4 ö Given e 2l = e 2 Þ l = 1 \ l = 1 and m is any real


t- + ..... - t ç 1 - + ...... ÷ + t 5 number.
3! 5! ç 2! 4! ÷
è ø
lim
æ ö 17. (d) f ( x ) =
(1 - sin x )( sin x) - (1 - cos x )( - cos x)
= t ®0 4t 2 8t 3 16t 4
t ç 1 + 2t +
ç 2!
+
3!
+
4!
+ .... - 1 - 2t ÷
÷ (1 - sin x ) 2
è ø
sin x - sin 2 x + cos x - cos 2 x sin x + cos x - 1
t3 t3 t5 t5 = =
- + + - + ..... + t 5 (1 - sin x ) 2
(1 - sin x ) 2
= lim 6 2 5! 4!
t ®0 8t 4 æ pö 1+ 0 -1 0
2t 3 + + ...... f ¢ç ÷ = =
3! \ è 2 ø (1 - 1) 2 0
1 1
- + æ pö
6 2 = - -1 + 3 = 1 Therefore, f ¢ çè ÷ø does not exist.
= 2
2 12 6
14. (a) For | x | < 1, x 2n ® 0as n ® ¥ and for | x | > 1, sin( f ( x))
18. (a) For lim , denominator tends to 0, hence
x-a
x ®a
1 the numerator must also tend to 0 for limit to be finite.
® 0 as n ® ¥. So
x 2n Then, a is a root of the equation ax 2 + bx + c = 0 or
f (a ) = 0 .
Mathematics S-M-45

Also, consider f (a + ) ® 0+ and f (a - ) ® 0- 2 x tan 3 x


= lim
x® 0 4 sin 4 x (1 - tan 2 x )
e1/ f ( x ) - 1 1 - e -1/ f ( x)
Þ lim = lim -1/ f ( x )
=1
x ®a + e1/ f ( x ) + 1 x ®a + 1 + e 1 x 1 1 1 1 1 1
= lim . 3 . = .1. 3 . =
e1/ f ( x) - 1 2 x® 0 sin x cos x 1 - tan x 2 1 1 - 0 2
2
and lim- = -1
x ®a e1/ f ( x) + 1 [1 + 2 + x - 3]
Thus, both the statements are true and statement 2 is 22. (a) The required limit = lim
x® 2 ( x - 2) é 1 + 2 + x + 3 ù
the correct explanation of statement 1. ëê úû

19. (b) We have, S n =


(
a r n -1 ) ( x+2 -2 )( x+2 +2 )
= lim

Þ ( r - 1) S n = a r n - a
r -1 x ®2
( x - 2) ( 1+ 2+ x + 3 )( x+2 +2 )
On differentiating both sides w.r.t r, we get ( x + 2) - 4
( 1+ )(
= lim
dS
( r - 1) n + Sn = nar n-1 - 0
dr
x®2
( x - 2) 2+ x + 3 x+2 +2 )
dS 1
Þ ( r - 1) n = nar n -1 - S n
dr
= n é nth term of GP ù - Sn
= xlim
®2
( 1+ 2+ x + 3 )( x+2 +2 )
ë û
1 1 1
= =
.
= n éëSn - Sn-1 ùû - Sn 2 3 4 8 3
= ( n - 1) Sn - n Sn-1
(4 x – 1)3
x +1 x +1 23. (b) lim
x ®0 x2
æ 3x - 4 ö 3 æ 3x + 2 - 6 ö 3 sin log(1 + 3 x )
20. (b) Consider lim çè ÷ = lim çè ÷ 4
x ®¥ 3 x + 2 ø x®¥ 3x + 2 ø
x +1 x +1 (4 x – 1) 3 ( x / 2) 2 3x 4
æ 3x + 2 6 ö 3 æ -6 ö 3 = lim . . .
= lim çè - ÷ = lim çè 1 + ÷ x®0 x 3 2
sin x / 4 log(1 + 3 x ) 3
x®¥ 3 x + 2 3 x + 2 ø x®¥ 3x + 2 ø
4 4
-6 x +1 = (loge 4)3.1.log e (e ) = (log e 4)3.
3 x +2 ö 3 x +2 ´ 3 3 3

{ }
æ
= lim ç 1 + -6 -6
÷ 1 - cos3 x (1 - cos x) (1 + cos x + cos2 x )
x ®¥ è 3x + 2 ø 24. (c) lim = lim
x ® 0 x sin x cos x x ®0 x sin x cos x
- 6 x +1
´ æ x ö æ xö
3 x +2 3 æ 1ö 2 sin 2 ç ÷
= lim [ e] Q lim
çè x ®¥ çè 1 + ÷ø = e÷ è 2ø (1 + cos x + cos 2 x )
x® ¥ x ø = lim ´
x®0 æ xö æ xö cos x
-2 x - 2 x ( -2 -2 / x ) - 2 -2 / x x.2 sin ç ÷ cos ç ÷
3 x +2 x (3+ 2 / x ) è 2ø è 2ø
= lim e = lim e = lim e 3+2/ x = e– 2/3
x ®¥ x ®¥ x®¥
æ xö
sin ç ÷
x tan 2 x – 2 x tan x è 2 ø 1 + cos x + cos 2 x 1 3
21. (c) Consider lim = lim ´ = ´3 = .
x®0 (1 – cos 2 x ) 2 x®0 æ x ö æ xö 2 2
2ç ÷ cos ç ÷ cos x
è 2ø è 2ø
x tan 2 x - 2 x tan x
= lim (Q cos 2x = 1 – 2 sin 2x)
x ®0 4sin 4 x x2
2 sin 2
x é 2 tan x ù 1 - cos x 2 2
= lim ê - 2 tan x ú 25. (c) xlim = lim
x® 0 4 sin 4 x ë 1 - tan 2 x û ® 0 1 - cos x x®0 2 x
2 sin
2
2 x tan x é1 - 1 + tan 2 x ù
= lim ê ú x2
x®0 4sin 4 x ëê 1 - tan 2 x ûú 2 | sin |
= lim 2
EBD_7504
S-M-46 SOLUTIONS

x2 ( - h) 2 8 ìï æ x2 ö æ x 2 ö üï
2 | sin | 2 | sin | = xlim í 2sin 2 ç ÷ 2sin 2 ç ÷ ý
LHL = lim 2 = lim 2 ®0 x8 ïî è 4ø è 8 ø ïþ
x® 0 -
x h® 0 ( -h)
2sin 2 2sin 2
2 2 (using cos 2q = 1 - 2 sin 2 q )

h2 h2 æ hö
2 é æ x2 ö æ x2 ö ù
2 sin sin ç ÷ ê sin 2 ç ÷ sin 2 ç ÷ ú
lim 2 = 2
lim 2 ´ è 2ø ´2 ê è 4ø è 8 øú
h® 0 h 2 h® 0 h 2 2 = lim ê32 ´ ú
2 sin 2 æ hö x®0 ë x 4
x4 û
2 2 çè sin ÷ø
2
é x4 x4 ù
=
2
´ 1 ´ 1´ 2 = 2 ê sin sin ú
2 32. lim ê 16 ´ 64 ú
=
h2
x ®0 ê x4 x4 ú
2 | sin | ê 16. 64. ú
RHL = lim 2 = 2 \ LHL = RHL = 2 ë 16 64 û
h ®0 h
2sin 2 1 1 1 sin q
2 = 32 ´ ´ = (Using lim = 1)
[Note that the question contains mod sign, hence we 16 64 32 q® 0 q
checked for LHL and RHL] rα rα
1/ log x 29. (c) z r = cos + i sin 2
26. (c) Let y = lim(cosec x) n 2
n
x ®0
Taking log on both sides, we get α α
z1 = cos 2
+ i sin 2 ;
log cosec x é ¥ ù n n
log y = lim êë ¥ form úû
x®0 log x 2α 2α
- cot x z2 = cos 2
+ i sin 2 ;...
= lim (By L' Hopital rule) n n
x ® 0 1/ x
na na
Þ zn = cos + i sin 2
x æ 1 ö n 2
n
= - lim çèQ cot x = ÷
x ® 0 tan x tan x ø
Þ log y = –1 consider nlim
®¥
( z1 z 2 z 3 .... z n )

1 é ìa ü
Þ y = e -1 = = lim êcos í 2 (1 + 2 + 3 + .... + n )ý
e n®¥ ë în þ
1
Hence, required limit = ìa üù
e + i sin í (1 + 2 + 3 + ... + n)ýú
2
(sin nx ) [(a - n)nx - tan x ] în þû
27. (c) Let lim =0
x®0 x2 é an(n + 1) an( n + 1) ù
= lim ê cos + i sin
æ n x é 3 3ö ìï x üï ù 3 n ®¥ ë 2n 2
2n 2 úû
ç nx - 3! ÷ ên(a - n) x - í x + 3 + ......ý ú
Þ lim0
è ø êë
2
îï þï ûú
=0 = lim
(
cosα 1 + 1 )
n + (
i sinα 1 + 1
n )
x® x x ®¥ 2 2
(By using expansion of sin x and tan x) ia
Þ n2 (a – n) – n = 0 Þ an – n2 – 1 = 0 a a
= cos + i sin = e 2
2 2
n2 + 1 1
Þ a= =n+ æ x2 + 1 ö
n n 30. (b) lim ç - ax - b÷ = 0
28. (c) Given expression is x ®¥ è x + 1 ø
8 ïì x2 x2 x2 x 2 ïü
lim í1 - cos - cos + cos cos ý ( x 2 + 1) - ( ax + b )( x + 1)
x ®0 x8 ï
î 2 4 2 4 ïþ Þ lim =0
x®¥ x +1
8 ìï x2 x2 æ x 2 ö üï x 2 (1 - a ) - (a + b) x - b + 1
= xlim í1 - cos - cos ç1 - cos ÷ ý
®0 x8 ïî 2 4 è 2 øï Þ lim =0
þ x®¥ x +1

8 ìïæ x2 ö æ x 2 ö üï Þ 1 - a = 0, a + b = 0 Þ a = 1, b = -1.
= xlim 1 - cos
8 íç
®0 x ïîè
Mathematics S-M-47

Speed Test-72

1. (c) Statement given in option (c) is correct. 12. (c)


~ [p Ú (~ q) ] = (~ p) Ù ~ (~ q)
= (~ p) Ù q ~ ( p Þ q) Û
p q p Þ q ~ ( p Þ q) ~ p ~ q ~ pÚ ~ q
2. (a) ~ pÚ ~ q
8 64 T T T F F F F T
3. (a) (b) = 2, = 16 ; but 4 is not prime.
4 4 T F F T F T T T
F T T F T F T F
Hence P Ù Q ® R, false F F T F T T T F
(6)2 36
(c) = = 3 ; but 12 is not prime Last column shows that result is neither a tautology
12 12 nor a contradiction.
Hence Q ® R, false 13. (d) The truth tables of p ® q and ~ p Ú q are given below:
2 4
(4) 16 p q ~p p ® q ~ (p Ú q )
(d) = = 2 ; is not an integer
8 8 8
T T F T T
Hence Q ® P, false
4. (a) Given result means p Ù ~ r is true, q Ú r is false. T F F F F
5. (a) Let p : x, y Î Z such that x and y are odd. F T T T T
q : xy is odd. F F T T T
To check the validity of the given statement, assume
that if p is true, then q is true. Clearly, truth tables of p ® q and ~ p Ú q are same.
p is true means that x and y are odd integers. Then, So, p ® q is logically equivalent to ~ p Ú q.
x = 2m + 1, for some integer m. Hence, option (a) is correct.
y = 2n + 1, for some integer n. If the truth value of p, q, r are T, F, T respectively, then
Thus, xy = (2m + 1) (2n + 1) = 2(2mn + m + n) + 1 the truth values of p Ú q and q Ú r are each equal to T.
This shows that xy is odd. Therefore, the given Therefore, the truth value of (p Ú q) Ù (q Ú r) is T.
statement is true. Hence, option (b) is correct.
Also, if we assume that q is not true. This implies that We have, ~ (p Ú q Ú r) @ ( ~ p Ù ~ q Ù ~ r)
we need to consider the negation of the statement q. So, option, (c) is correct.
This gives the statement. If p is true and q is false, then p Ú q is true.
: q : product xy is even. Consequently,
This is possible only, if either x or y is even. This ~ (p Ú q) is false and hence p Ù ~ (p Ú q) is false.
shows that p is not true. Thus, we have shown that Hence, option (d) is wrong.
: q Þ: p 14. (b) p Þ q is logically equivalent to ~ p Þ ~ q
6. (c) S (p, q, r) = ~ p Ù [~ (q Ú r)] \ ( p Þ q) Û (~ q Þ ~ p) is a tautology but not a
So, S (~p, ~q, ~r) º ~ (~p) Ù [~ (~q v ~r)] º p Ù (q Ú r) contradiction.
S*(p, q, r) º ~ p Ú [~ (q Ù r)] 15. (c)
S* (~p, ~q, ~r) º p Ú (q Ú r) 16. (a)
Clearly, S* (~p, ~q, ~r) º ~ S (p, q, r) 17. (b) (p Þ q) Ù (q Þ p) means p Û q .
7. (b) 18. (c) The inverse of the proposition (p Ù ~ q) ® r is
8. (b) ~ (p Ù ~ q) ® ~ r
9. (b) º ~ p Ú ~ (~q) ® ~ r
10. (d) Since ~ (p Ú q) º ~ pÙ ~ q (By De-Morgans’ law) º~pÚq®~r
19. (a) Let p, q and r be three propositions given by
\ ~ (p Ú q) ¹ ~ p Ú ~ q p : x = 5, q : y = –2 and r : x – 2y = 9
\ (d) is the false statement Then, the given statement is (p Ù q) ® r
11. (a) We know that the contropositive of p ® q is Its contrapositive is
~ q ® ~ p. So contrapositive of p ® (~q ® ~r) is ~ r ® ~ (p Ù q)
~ (~q ® ~r) ® ~p i.e., ~ r ® ~ p Ú ~ q
i.e., If x – 2y ¹ 9, then x ¹ 5 or y ¹ –2
º ~ q Ù [~ (~r)] ® ~p 20. (b)
[Q ~ (p ® q) º p Ù ~q]
º ~ q Ù r ® ~p
EBD_7504
S-M-48 SOLUTIONS

21. (b) Let us make the truth table for the given statements, as 27. (c) Consider ~ [p Ú (q Ù r)] = ~p Ù ~ (q Ù r)
follows : = ~p Ù (~q Ú ~ r)
p q p Ú q q ®p p ® (q ® p ) p ® (p Ú q ) = (~p Ù ~q) Ú (~p Ù ~ r)
T T T T T T 28. (b) The given statements are
T F T T T T p : A tumbler is half empty.
F T T F T T q : A tumbler is half full.
F F F T T T
We know that, if the first statement happens, then the
From table we observe second happens and also if the second happens, then
p ® (q®p) is equivalent to p®(pÚq) the first happens. We can express this fact as
22. (c) ~ [ ( p Ú q) Ù (q Ú ~ r)] º ~ ( p Ú q) Ú ~ (q Ú ~ r) If a tumbler is half empty, then it is half full.
º (~ p Ù ~ q) Ú (~ q Ù r) If a tumbler is half full, then it is half empty.
23. (b)
We combine these two statements and get the
24. (a) following. A tumbler is half empty, if and only if it is
p q p Ù q ( p Ù q) Þ p
half full.
T T T T
29. (b) Define the statements
T F F T p = It is cloudy tonight
F T F T q = it will rain tomorrow
r = I shall be on leave tomorrow
F F F T
The assumptions are p Þ q, q Þ r and the conclusion
\ ( p Ù q) Þ p is a tautology.. is p Þ r validity can be checked using truth table.
25. (c) (p Ú q) Ù (p Ú r) 30. (a) Suman is brilliant and dishonest if and only if Suman is
º (T Ú T) Ù (T Ú F) rich is expressed as
º T Ù T Q « ( P Ù ~ R)
º T Negation of it will be ~ (Q « ( PÙ ~ R))
26. (d)

Speed Test-73

1. (b) Mean deviation is minimum when it is considered about


the item, equidistant from the beginning and the end i.e. n
ÞS£ r .
n –1
101+ 1
the median. In this case median is th i.e. 51st 3. (b) On arranging the given observations in ascending
2 order, we get
item i.e., x51.
0
{
1 n All negative terms All positive terms
2. (a) We have r = max | xi – x j | and S =
n –1
å ( xi – x )2
2 ( n+1) th term
i¹j i=1 The median of given observations = (n+1)thterm = 0
2
\ S.D.>M.D.
2 æ x + x + .... + xn ö
Now, consider ( xi – x ) = ç xi – 1 2 ÷ Mean =
101 + d(1 + 2 + 3 + ......+100)
è n ø 4. (b)
101
1 d × 100 × 101
= 2
[( xi – x1 ) + ( xi – x2 ) + .... + ( xi – xi –1)] =1+ =1 + 50 d
n 101 × 2
1 Q Mean deviation from the mean = 255
+ ( xi – xi + 1) + ... + ( xi – xn )] £ [(n – 1)r ]2
n 2 1
Þ [| 1 - (1 + 50d ) | + | (1 + d ) - (1 + 50 d ) |
[Q| xi – x j |£ r ] 101
+ | (1 + 2d ) -(1 + 50d ) | +....+ | (1 + 100 d ) - (1 + 50 d ) |]
n
Þ ( xi – x ) £ r Þå ( xi – x ) £ nr
2 2 2 2 = 255
i =1 Þ 2d [1 + 2 + 3 + ... + 50] = 101´ 255

1 n
nr 2 nr 2 50 ´ 51 101´ 255
Þ å ( xi – x )2 £ Þ S2 £ Þ 2d ´ = 101´ 255 Þ d = = 10.1
n -1 50 ´ 51
i =1
Mathematics S-M-49

5. (a) Given Sx = 170, Sx2 = 2830 s2 60 ´ 25


Increase in Sx = 10, then S x ¢ = 170 + 10 = 180 Þ 60 = ´ 100 Þ s 2 = Þ s2 = 15
25 100
Increase in Sx2 = 900 – 400 = 500, then Thus, s1 – s2 = 15 – 15 = 0
Sx¢2 = 2830 + 500 = 3330 10. (b) Corrected Sx = 40 × 200 – 50 + 40 = 7990
2 \ Corrected x = 7990 / 200 = 39.95
1 2 æ Sx ¢ ö
\ Variance = Sx ¢ –ç ÷
n è n ø Incorrect Sx 2 = n[s 2 + x 2 ] = 200[15 2 + 40 2 ]
2 =365000
3330 æ 180 ö Corrected Sx2 = 365000 – 2500 + 1600 = 364100
= –ç ÷ = 222 –144 = 78.
15 è 15 ø
364100
6. (d) The given data in ascending order of magnitude is \ Corrected s = – (39.95) 2
7, 10, 12, 15, 16, 17, 25 200
Here, lower quartile Q1 = 10, = (1820.5 –1596) = 224.5 = 14.98.
median = 15 and upper quartile Q3 = 17
ax + b a b
Q 3 - Q1 17 - 10 11. (b) Let y = Þ y= x+
\ Q.D. = = = 3. 5 c c c
2 2
7. (a) First ten positive integers are 1, 2, 3, 4, 5, 6, 7, 8, 9 and 10. a b
Þ y = Ax + B where A = and B =
Sum of these numbers ( å xi ) = 1 + 2 + ... + 10 = 55 So, y = Ax + B and hence
c c

Sum of squares of these numbers ( å xi )


2
y - y = Ax + B - (Ax + B) = A(x - x)
= 12 + 22 + … + 102 = 385 Þ ( y - y) 2 = A 2 ( x - x ) 2

Standard deviation ( s ) =
å x i2 - æ 1 ö
ç å xi ÷
2
Þ å ( y - y) 2 = A 2 å ( x - x ) 2
n èn ø 2 2 2
Þ ns y = A (ns x ) Þ s y = A s x
385
- ( 5.5 ) = 38.5 - 30.25 = 8.25
2
= a
10 Hence standard deviation is multiplied by A =
c
\ Variance (s2) = 8.25
12. (b) Let the observations be x1, x2, ...., x20 and x be their
8. (c)
Class fi yi
d i = y i – A,
fid i 2 mean. Given that, variance = 5 and n = 20. We know
fid i
A = 25 that,
1 20
0-10
10-20
1
3
5
15
–20
–10
–20
–30
400
300
( )
Variance s 2 = å ( xi - x )
n i=1
2

20–30 4 25 0 0 0 1 20 20
å ( x i - x ) or
2
å ( xi - x )
2
i.e. 5 = = 100 ...(i)
30-40 2 35 10 20 200 20 i =1 i =1
Total 10 –30 900 If each observation is multiplied by 2 and the new
resulting observations are yi, then
2 2 1
S fi di2 æ S fi di ö 900 æ –30 ö yi = 2xi i.e., xi = y
s2 = –ç ÷ = –ç ÷ 2 i
S fi è S fi ø 10 è 10 ø
s2 = 90 – 9 = 81 Þ s = 9. 1 20 1 20 1 20
9. (a) We know that,
Therefore, y = å i 20 å i 20 å xi
n i =1
y = 2x = 2.
i =1 i =1
s
Coefficient of variation = ´ 100 1
x i.e., y = 2x or x = y
s1 2
\ CV of 1st distribution = ´ 100
30 On substituting the values of xi and x in eq. (i), we get
s1 20 2 20
Þ 50 = ´ 100 [CV of 1st distribution = 50 (given)] æ1 1 ö
å çè 2 yi - 2 y ÷ø = 100 i.e., å ( yi - y ) = 400
2
30
i =1 i =1
Þ s1 = 15
Thus, the variance of new observations
s2
Also, CV of 2nd distribution = ´ 100 1
25 = ´ 400 = 20 = 2 2 ´ 5
20
EBD_7504
S-M-50 SOLUTIONS

( ) ( )=
th
é æ n + 1ö ù n1 s12 + d12 n2 s 22 + d 22
13. (c) Upper Quartile = Size of ê3 çè ÷ item 22
ë 4 ø úû 17. (c) Using, s =
n1 + n2 3
th
é æ 31+1ö ù 18. (a) Given, s = 9
= Size of ê3çè ÷ item, [Qn = å f = 31].
ë 4 ø úû Let a student obtains x out of 75. Then his marks out
14. (b) The mean of the series 4x 4
of 100 are . Each observation is multiplied by
1 3 3
X= {a + ( a + d ) + ( a + 2d ) + ..... + ( a + 2nd )}
2n + 1 4
New s = ´ 9 = 12, Variance = s 2 = 144
1 ì 2n + 1 ü 3
= í (2a + 2nd )ý = a + nd n n
2n + 1 î 2 þ n
Therefore, mean deviation from mean å r . n Cr å r. r n -1
Cr -1
r =0 r= 0
19. (a) m1¢ = =
1 2n 1 2n n n
å
2n + 1 r = 0
( a + rd ) - ( a + nd ) = å | r- n | d
2n + 1 r= 0
å n
Cr å n
Cr
r=0 r= 0
[2 (1 + 2 + ... + n) + 0] d n (n + 1)d n
= =
2n + 1 2n + 1 n å n -1
Cr -1
15. (c) Here, we construct the cumulative frequency table r= 1 n . 2n -1 n
= n
= n
=
Class Frequency Cumulative 2 2
frequency å n
Cr
r =0
0–10 4 4
10–20 8 12 n
20–30
30–40
11
15
23
38
å r 2 . n Cr 1 n
å { r ( r - 1) + r }
r =0 n
40–50 12 50 m ¢2 = = Cr
n n
2
50–60 6 56 å n
Cr r= 0
60–70 3 59 r =0
Total 59 1 n
n
n 59
=
2 n å r ( r - 1) n Cr + 2
For Q1. Here n = 59 Þ = = 14.75 r =0
4 4
1 n n ( n - 1) n - 2 n
\ Class of first quartile is 20 – 30 =
n
2 r =0
å r ( r - 1) r ( r - 1) Cr - 2 +
2
14.75 – 12 27.5
Þ Q1 = 20 + ´10 = 20 + = 22.5
11 11 n ( n - 1) n
= . 2n - 2 +
n 2
3n 3 ´ 59 2
For Q3. Here = = 44.25
4 4 n ( n - 1) n
\ Class of third quartile is 40–50 = +
4 2
44.25 - 38 62.5
Þ Q 3 = 40 + ´10 = 40 + = 45.2 Variance m 2 = m ¢2 - ( m1¢ ) 2
12 12
16. (a) Mean and SD s of the combined group are
n ( n - 1) n æ n ö 2 n
63 ´ 27.6 + 26 ´ 19.2 = + -ç ÷ =
m= = 25.1 4 2 è 2ø 4
63 + 26 Hence (a) is the correct answer
Thus, AM is decreased by 27.6 - 25.1 = 2.5
63 ´ ( 7.1) + 26 ´ ( 6.2 )
2 2
2
s2 = S xi2 æ S xi ö
20. (c) We know SD = -ç
89 n è n ÷ø
63 ( 25.1 - 27.6) + 26 ( 25.1 - 19.2)
2 2
+ \ 100 æ 20 ö 100 400
2
89 2= -ç ÷ = -
Þ s = 7.8 (approx) n è nø n n2
Hence, (a) is the correct answer. 100 400
Þ 4= - 2
n
Þ n = 20, 5
Mathematics S-M-51

21. (c)
Classmid-value (x) f fx d = x–M d2 fd2
å xi - 45 = Þ å xi = 54
Similarly,
0 – 10 5 2 10 –20.7 428.49 856.98
10 – 20 15 10 150 – 10.7 114.49 1144.9 å xi2 - 10 ´ 54 + 25 ´ 9 = 45
20 – 30 25 8 200 – 0.7 0.49 3.92
30 – 40 35 4 140 9.3 86.49 345.96 Þ å xi2 = 360
40 – 50 45 6 270 19.3 372.49 2234.94
2
Sf = 30 Sfx = 770 Sfd2 = 4586.7 360 æ 54 ö 324
Þ s= -ç ÷ = =2
9 è 9 ø 81
Sfx 770
Now, M (A.M) = = = 25.7 26. (b) Let us assume an arbitary mean a = 155. Following table
Sf 30
Now, standard deviation (S.D) is constructed :
X i - 155
Sfd 2 4586.70 Xi fi ui =
5
u i2 fiu i f i u i2
= = = 12.36 140 4 –3 9 –12 36
Sf 30
145 6 –2 4 –12 24
SD 12.36 150 15 –1 1 –15 15
\ Coeff of SD = = = 0.480
M 25.7 155 30 0 0 0 0
\ Coeff of variation = Coeff of S.D × 100 160 36 1 1 36 36
= 0.480 × 100 = 48. 165 24 2 4 48 96
22. (a) C.V. (1st distribution) = 60, s1 = 21 170 8 3 9 24 72
C.V. (2nd distribution) = 70, s2 = 16 175 2 4 16 8 32
Let x1 and x2 be the means of 1st and 2nd distribution, Total 125 77 311
respectively, Then æ 2ö
ρ
C.V. (1st distribution) = 1 ≥100 \ Variance = s = c
2 2ç
ç
å fi u i2 - æç å fi u i ö÷ ÷
x1 n ç n ÷ ÷
ç è ø ÷ø
21 21 è
\ 60 = ≥100 or x1 = ≥100 = 35
x1 60 æ 311 æ 77 ö 2 ö
ç 311 25 ´ 77 ´ 77
ρ2 = 25 ´ ç -ç ÷ ÷ = 25 ´ -
and C.V. (2nd distribution) = ≥100 125 è 125 ø ÷ 125 125 ´ 125
x2 è ø
16 16 = 62.2 – 9.4864 = 52.7136 Þ S.D.
i.e., 70 = x ≥100 or, x2 = ≥100 = 22.85
2 70 = 52.7136 = 7.26 nearly

23. (d) If initially all marks were xi then si2 =


å ( xi - x )2
i
27. (b) Use s = 2 ( ) (
n1 s12 + d12 + n2 s 22 + d 22 )
N n1 + n2
Now each is increased by 10
where d1 = m1 - a, d2 = m2 - a, a being the mean of
å ( xi - x )
2
å [ ( xi +10) -( x +10)]2
i i
the whole group. Let m2 = mean of the second group
si2 = = = si2
N N 100 ´ 15 + 150 ´ m2
Hence, variance will not change even after the grace \ 15.6 = Þ m2 = 16
250
marks were given.
24. (b) We know that for positive real numbers x1, x2, ...., xn,
we have
(
Thus, 13.44 = 100 ´ 9 + 150 ´ s
2
)
+100 ´ ( 0.6) + 150 ´ ( 0.4)
2 2
2 2
å xi2 æ å xi
³ç
ö
÷ Þ
400 æ 80 ö
³ çè ÷ø 250
Þ s=4
n çè n ÷ø n n Hence, (b) is the correct answer,
Þ n ³ 16 . So only possible value for n = 18 28. (b) 250
9
S.D. (s) = = 25 = 5
10
25. (d) Let å ( xi - 5) = 9 s
i =1
Hence, coefficient of variation = ´ 100
9 9 mean
Þ å xi - å 5 = 9 5
i =1 i =1 = ´ 100 = 10%.
50
9

Þ å xi - (9 ´
i =1
EBD_7504
S-M-52 SOLUTIONS

99 = -x - a
29. (b) Variance of 1, 2, 3, 4, 5, ... 10 is Since, di = – xi – a and we multiply or subtract each
12
observation by any number the mode remains the
99 same. Hence mode of –xi – a i.e. di and xi are same.
\ variance of 3, 6, 9, 12, ... 30 is 9 ´
12 Now variance of d1, d2,...., dn
99 3 1 n
\ S.D. of 3, 6, 9, 12, ... 30 = 9´ =
12 2
33 = å [di - ( - x - a)]2
n i =1
x1 + x2 + x3 + ... + xn
30. (b) x = 1 n
n = å [ - xi - a + x + a]2
n i =1
1 n
s2 = å ( xi - x )2 1 n
n i =1 = å ( - xi + x ) 2
n i =1
Mean of d1, d2, d3, ...., dn
d1 + d 2 + d3 + .... + d n 1 n
=
n
= å ( x - xi )2 = s2
n i =1
( - x1 - a ) + ( - x2 - a ) + ( - x3 - a ) + .... + ( - xn - a )
=
n
é x1 + x2 + x3 + .... + xn ù na
= -ê ú- n
ë n û

Speed Test-74

1. (b) Total no. of students in four schools 1


= 12 + 20 + 13 + 17 = 62. \ from (i) p = q 2 =
Now, one student is selected at random. 9
\ Total outcomes = 62C1 1
\ p( E1 ) = p =
Now, no. of students in school B2 = 20. 9
No. of ways to select a student from B2 = 20C1 . 3. (d) The three events are E1, E2, E3 of sample space S.
(a) P (only one of them occurs)
20C1 20 10
\ Required probability = = = = P ( E1E 2 E3 + E1E 2 E3 + E1E 2 E 3 )
62C 1 62 31
2. (b) Let the two events be E1 and E2. Let their chances be (b) P (none of them occurs) = P ( E1E 2 E3 )
p and q respectively. Then p = q2 .....(i) (c) P (atleast one of them occurs)
The chances of not happening of the events are 1 – p and P (only one occurs + any two occurs + all the three
1 – q respectively. occur)

Odds against the first event =


1- p (d) P (all the three occur) = P ( E1E 2E3 )
p 4. (a) Total no. of arrangements of the letters of the word
1- q 10!
UNIVERSITY is .
Odds against the second event = 2!
q
No. of arrangements when both I's are together = 9!
3 So. the no. of ways in which 2 I’s do not together
1 - p æ 1- q ö 1 - q 2 (1 - q)3
Given = ç ÷ Þ = [From (i)] 10!
p ç q ÷ q2 q3 = - 9!
è ø 2!
\ Required probability
æ1- q ö é (1 - q )2 ù = 0
Þç ÷ ê(1 + q ) - ú 10!
- 9!
ç q2 ÷ êë q úû 10!- 9! 2!
è ø
= 2! =
10! 10!
Q q ¹ 1 and q ¹ 0
2!
1
\ q (1+ q ) = 1 - 2q + q 2 - 2] 8 4
= =
´ 9! 10 5
Mathematics S-M-53

5. (a) We divide the number in three groups 420 7


3k + 1 type {1, 4, 7, .................., 2005} Thus required probability = =
3k + 2 type {2, 5, 8, .................., 2006} 600 10
3k + 3 type {3, 6, 9, .................., 2007} 11. (b) In a leap year there are 366 days in which 52 weeks and
x3 + y3 is divisible by 3 if x and y both belong to 3rd group or two days. The combination of 2 days may be : Sun-
one of them belongs to the first group and the other to the Mon, Mon-Tue, Tue-Wed, Wed-Thu, Thu-Fri, Fri-Sat,
second group. So favourable number of cases Sat-Sun.
= 669 C2 +669 ´669 2 2
P (53 Fri) = ; P (53Sat) =
7 7
Total number of cases = 2007 C2
1
669 ´ 668 and P (53 Fri and 53 Sat) =
+ 669 ´ 669 7
\ Desired probability = 2 \ P (53 Fri or Sat) = P (53 Fri) + P (53 Sat)
2007 ´ 2006 – P (53 Fri and Sat)
2 2 2 1 3
669 ´ 2006 1 =+ – =
= = 7 7 7 7
2007 ´ 2006 3 12. (c) Given digits are 1, 2, 3, 4, 5
6. (c) Probability of the card being a spade or an ace Total no. of 2 digits numbers formed = (5)2 = 25
16 4 Favourable cases are 12, 24, 32, 44, 52
= = . Hence odds in favour is 4 : 9. No. of favourable cases = 5
52 13
So the odds against his winning is 9 : 4. 5 1
\ Required Probability = =
7. (a) The first object can be given to any of the n persons. 25 5
But the second, third and other objects, too, can go to any 13. (c) Note:- The question should state ‘3 different’ boxes
of the n persons. Therefore the total number of ways of instead of ‘3 identical boxes’ and one particular box has 3
distributing the n objects randomly among n persons is nn. balls. Then the solution would be:
There are nPn = n! ways in which each person gets exactly 12
one object, so the probability of this happening is C3 ´ 29
Required probability =
n! (n - 1) ! 312
= .
nn n n -1 55 æ 2 ö
11
Hence the probability that at least one person does not get = ç ÷
3 è 3ø
(n - 1) ! 14. (d) Total number of outcomes
any object is 1 - .
n -1 S = {(1, 1), (1, 2), (1, 3), (1, 4), (2, 1), (2, 2), (2, 3),
n
(2, 4), (3, 1), (3, 2), (3, 3), (3, 4), (4, 1), (4, 2), (4, 3), (4, 4)}
8. (d) Probability of exactly M occurs = P(M Ç N) n(S) = 16
and probability of exactly N occurs = P(M Ç N) Number of favourable outcomes
E = {(2, 1), (3, 1), (3, 2), (4, 1), (4, 2), (4, 3), (4, 4)}
\The probability that exactly one of them occurs is
n(E) = 7
P(M Ç N)+P(M Ç N)
n(E) 7
= P(M) – P(M Ç N)+P(N) – P(M Ç N) Required Probability = =
n(S) 16
= P(M)+P(N) – 2P(M Ç N) 15. (c) 7m + 7n = [(5 + 2)m + (5 + 2)n] º 5 ´ integer + 2m + 2n
m n m n
9. (d) Total number of numbers = 4! = 24 \ 7 + 7 is divisible by 5 iff 2 + 2 is divisible by 5 and so
m n
For odd nos. 1 or 3 has to be at unit's place unit place of 2 + 2 must be 0 since it cannot be 5.
If 1 is at unit place, then total number of numbers m possible n
= 3! = 6 1 3,7, 11,15, .... = 25
And if 3 is at units place, then total number of numbers 2 4, 8,12,..... = 25
= 3! = 6 3 1, 5, 9, ..... = 25
\ Total number of odd number = 6 + 6 = 12 4 2, 6,10, .... = 25
Since 21 + 23 º 23 + 21 so (1, 3) and (3, 1) are same as
12 1 favourable cases.
\ Required probability = =
24 2
10. (b) Total number of bolts = 600 25 ´ 50 1
\p= =
Number of too large bolts = 20% of 600 ´
100 100 8
20 ´ 600 16. (b) Obviously P (A È B) ³ max {P( A), P ( B)} = 2 ....(1)
= = 120 3
100
Number of too small bolts = 10% of 600 = 60 1
Number of suitable A), P(B)} =
2
EBD_7504
S-M-54 SOLUTIONS

1 2
But , P (A Ç B) = P( A ) + P( B) - P (A È B) ³ P( A ) + P (B) - 1 = Þ P ( A ) + P ( B) - P ( A Ç B) =
6 3
[Q maximum value of P(A È B) = 1]
(Q P(A È B) = P(A) + P(B) - P(A Ç B) )
1 1
\ We get £ P( A Ç B) £ ....(2) 1 2 1
6 2 Þ x+ x- = Þ 2x = 1 Þ x =
1 1 1 3 3 2
Now, P(A Ç B' ) = P(A) - P(A Ç B) £ - = ....(3) 15
2 6 3 C2 15 ´ 14 ´ 4! 1
20. (b) p1 = = =
Also P(A'ÇB) = P(B) - P(A Ç B) 42
C4 2!´ 42 ´ 41´ 40 ´ 39 41´ 26 and
1 1 2 1 2 1
Since , £ P( A Ç B) £ Þ - ³ P( B) - P( A Ç B) ³ - 30
C4 15 ´ 14 ´13 ´ 12 ´ 8!
6 2 3 6 3 2 p2 = =
1 1
84
C8 4!´ 84 ´ 83 ´ 82 ´ .....´ 77
Þ £ P(A 'ÇB) £ ....(4)
6 2 15 ´14 ´13 ´12 ´ 8 ´ 7 ´ 6 ´ 5
Clearly (b) does not hold correct. = < p1 Þ p > p .
84 ´ 83 ´ 82 ´ 81´ 79 ´ 78 ´ 77 1 2
17. (a) M Stat = 30 21. (a) From venn diagram, we can see that

S = 125 A C

AÇBÇC
B
A Ç BÇC
M = 70 Stat = 55
P (M) + P (Stat) – 2P (M Ç Stat) P(B Ç C) = P(B) - P(A Ç B Ç C ) - P(A Ç B Ç C )
70 55 30 65 13 3 1 1 1
= + -2 = = = - - = .
125 125 125 125 25 4 3 3 12
18. (c) Probability that exactly one event out of A and B occur 22. (a) P (A È B) = P (A) + P (B) – P (A Ç B);
is P(A) + P(B) - 2P(A Ç B) 3 1
\ P(A) + P(B) - 2P(A Ç B) = 1 - a ......(1) Þ =1 – P( A ) + P(B) –
4 4
Similarly, P(B) + P(C) - 2P(B Ç C) = 1 - 2a ......(2) 2 2
Þ 1=1– + P(B) Þ P(B) = ;
and P(C) + P(A) - 2P(C Ç A ) = 1 - a ......(3) 3 3
Now, Probability that at least one out of A, B, C will occur is 2 1 5
Now, P( A Ç B ) = P(B) – P ( A Ç B ) = – = .
P(A È B È C) = P (A) + P(B) + P (C) - P(A Ç B) 3 4 12
- P(B Ç C) - P(C Ç A) + P(A Ç B Ç C) 1 + 4p 1 - p 1 - 2p
23. (a) , , are probabilities of the three
1 4 2 2
= [{P(A ) + P(B) - 2P( A Ç B)} + {P( B) + P(C) - 2P( B Ç C)} mutually exclusive events, then
2
+ {P(C) + P(A ) - 2P(C Ç A)}] + P(A Ç B Ç C) 1 + 4p 1- p 1 - 2p
0£ £ 1, 0 £ £ 1, 0 £ £1
1 4 2 2
= [(1 - a ) + (1 - 2a ) + (1 - a )] + a 2 1 + 4 p 1 - p 1 - 2p
2
and 0 £ + + £1
3 1 1 1 4 2 2
= a 2 - 2a + = a 2 - 2a + 1 + = (a - 1) 2 + >
2 2 2 2 1 3 1 1 1 5 1 1
\- £ p £ , - 1 £ p £ 1, - £ p , £ p £ \ £p£
[Q a ¹ 1] 4 4 2 2 2 2 2 2
[The intersection of above four intervals]
19. (a) Since, P(A 'Ç B ') = 1 1
3 \p =
2
1 1 24. (a) We divide the number in three groups
So, P( A È B) = Þ 1 - P ( A È B) =
3 3 3k + 1 type {1, 4, 7, .................., 2005}
3k + 2 type {2, 5, 8, .................., 2006}
2
Þ P ( A È B) =
3
Mathematics S-M-55

x3 + y3 is divisible by 3 if x and y both belong to 3rd group or (n –2, n – 1, n), hence, (n – 2) in number
one of them belongs to the first group and the other to the Numbers with common difference = 2, (1,3,5); (2,4,6); ..........
second group. So favourable number of cases ( n – 4, n – 2, n) hence, (n – 4) in number.
Number with common difference = 3, (1,4,7); (2,5,8)........
= 669 C2 +669 ´669 (n – 6, n – 3, n), hence, (n – 6) in number
Total number of cases = 2007 C2 ..................................................................................
..................................................................................
669 ´ 668
+ 669 ´ 669 n - 1 æ1, n + 1 , n ö
\ Desired probability = 2 Numbers with common difference = ç ÷,
2 è 2 ø
2007 ´ 2006
2 n -1
hence 1 in number [Note that n is odd, so is integer]
669 ´ 2006 1 2
= = \ Total number of favourable case
2007 ´ 2006 3
= (n - 2) + (n - 4) + (n - 6) + .... + 3 + 1
25. (a) Since P(A È B È C) ³ 0.75 , therefore
0.75 £ P(A È B È C) £ 1 1 æ n -1 ö (n - 1) 2
= ç ÷ [1 + (n - 2)] =
2è 2 ø 4
Þ 0.75 £ P( A) + P( B) + P (C) - P( A Ç B) - P( B Ç C) -
n -1
[Q total number of terms in above A.P. is ]
P(A Ç C) + P(A Ç B Ç C) £ 1 2
Þ 0.75 £ 0.3 + 0.4 + 0.8 - 0.08 - P(B Ç C) - 0.28 + 0.09 £ 1 ( n - 1) 2
Þ 0.75 £ 1.23 - P (B Ç C) £ 1 Þ - 0.48 £ -P( B Ç C) £ -0.23 Required probability = 4
n
Þ 0.23 £ P(B Ç C) £ 0.48. C3
26. (b) Total number of selections = 11 × 11
Now |x – y| > 5 clearly x ¹ 5 (n - 1) 2
If x = 0, then y > 5 Þ 5 favourable cases 4 3(n - 1)
= =
If x = 1, then y > 6 Þ 4 favourable cases n (n - 1) (n - 2) 2n (n - 2)
If x = 2, then y > 7 Þ 3 favourable cases 6
If x = 3, then y > 8 Þ 2 favourable cases ALTERNATE : There are in the set {1, 2, 3, ....., n} (n being
If x = 2, then y > 9 Þ 1 favourable cases
Symmetical cases will be obtained for x = 6, 7, 8, 9,10 n -1 n +1
odd), even number, odd numbers; and for an
\ Favourable cases = 30 2 2
A.P., the sum of the extremes is always even and hence the
30
\ Desired probability = choice is either (both) 2 even or (both) 2 odd and this may
121 be done in
27. (b) Given equation n -1 n +1
100 (n - 1) 2
2+ C2 =
2 C 2 ways.
x+ > 50 Þ x 2 - 50x + 100 > 0 Þ ( x - 25) 2 > 525 4
x
Þ x - 25 < - (525) or x - 25 > (525) (n - 1) 2
4 3(n - 1)
Þ x < 25 - (525) or x > 25 + (525) Hence, the required probability = =
n
C3 2 n (n - 2)
As x is positive integer and (525) = 22.91 , we must have 29. (a) The sequence of m consecutive heads can begin with
the ith toss, provided 1 £ i £ n + 1. If it begins at the first
x £ 2 or x ³ 48
Let E be the event for favourable cases and S be the sample m
æ1ö
space. toss, the probability of m consecutive head is ç ÷ . If the
è2ø
\ E = {1, 2, 48, 49, ......100}
sequence begins with the (i + 1) th toss for 1 £ i £ n, the ith
\ n(E ) = 55 and n(S) = 100 toss must be tails, with heads on the next m tosses. The
n (E ) 55 11 m m +1
= = 1æ1ö æ1ö
Hence the required probability P(E) = . probability of this event is ç ÷ =ç ÷
n (S) 100 20 2è2ø è2ø
28. (d) Exhaustive number of cases = n C3 Hence the required probability is
Three numbers in A.P. can be selected in the following m m +1
æ1ö æ1ö 2+n
ç ÷ + nç ÷ = .
EBD_7504
S-M-56 SOLUTIONS

30. (d) P(AÈB) = P(A) + P(B) – P(AÇB) Þ P(A) + P(B) £ 1/8 + 3/4 = 7/8
\ 1 ³ P(A) + P(B) – P(AÇB) ³ 3/4 As the maximum value of P(AÇB) is 3/8, we get
Þ P(A) + P(B) – 1/8 ³ 3/4 1 ³ P(A) + P(B) – 3/8
[since minimum value of P(AÇB) is 1/8] Þ P(A) + P(B) £ 1 + 3/8 = 11/8

Speed Test-75

1. (d) R = {(x, y) : x, y Î N and x2 – 4xy + 3y2 = 0} A


Now, x2 – 4xy + 3y2 = 0 Þ (x – y) (x – 3y) = 0
\ x = y or x = 3y
\ R = {(1, 1), (3, 1), (2, 2), (6, 2), (3, 3), (9, 3),......}
C
Since (1, 1), (2, 2), (3, 3),...... are present in the relation, B
therefore R is reflexive. (3) Transitivity :
Since (3, 1) is an element of R but (1, 3) is not the In a plane, let AB, BC and CA be three lines, such
element of R, therefore R is not symmetric that
Here (3, 1) Î R and (1, 1) Î R AB ^ BC and BC ^ CD
Þ (3, 1) Î R
Þ AB P CD Þ a R b, R is not transitive.
(6, 2) Î R and (2, 2) Î R Þ (6, 2) Î R
Hence, R is symmetric but neither reflexive nor
For all such (a, b) Î R and (b, c) Î R transitive.
Þ (a, c) Î R 8. (d) A function whose graph is symmetrical about the
Hence R is transitive. y-axis must be even
2. (b) Obviously, the relation is not reflexive and transitive Since sin x and log( x + x 2 + 1 ) are odd function
but it is symmetric, because x2 + y2 = 1 Þ y2 + x2 = 1
3. (c) Let f (x) ¹ 2 be true and f (y) = 2, f (z) ¹ 1 are false therefore sin(log( x + x 2 + 1)) must be odd.
Þ f (x) ¹ 2, f (y) ¹ 2, f (z) = 1
Þ f (x) = 3, f (y) = 3, f (z) = 1 but then function is many
sec 4 x + cos ec 4 x
one, similarly two other cases. Also, is an odd function.
x 3 + x 4 cot x
p p 1 é 1 1ù
4. (a) - £ sin -1 ( 2 x ) £ Þ - £ 2 x £ 1 Þ x Î ê- , ú Now, let f ( x + y) = f ( x) + f ( y) " x, y Î R
6 2 2 ë 4 2û
5. (c) We have, R = {(1, 3); (1, 5); (2, 3); (2, 5); (3, 5); (4,5)} \ f (0 + 0) = f (0) + f (0) \ f(0) = 0
R–1 = {(3, 1); (5, 1); (3, 2); (5, 2); (5, 3); (5,4)} f ( x - x ) = f ( x ) + f (- x ) or 0 = f(x) + f(– x)
Hence RoR–1 = {(3, 3); (3, 5); (5, 3); (5, 5)}
i.e f ( -x ) = -f ( x) \ f(x) is odd
6. (a) f (4) = g (4) Þ 8 + a = 8 Þ a = 0
f (–1) = – 2 for a = 0 9. (a) Since R is reflexive relation on A, therefore (a,a) Î R
f (–1) > f (4) for all a ÎA.
b+ 3 > 8 Þb> 5 The minimum number of ordered pairs in R is n.
7. (b) We have to test the equivalencity of relation R on S. Hence , m ³ n .
(1) Reflexivity :
In a plane any line be parallel to itself not perpen-
10. ( )
(d) f (2) = f 31/ 4 Þ many to one function

dicular. Hence aRb , R is not reflexive. and f (x) ¹ - 3 " x Î R Þ into function
(2) Symmetry :
æ ( 3x + 4 ) ö
In a plane if a line AB is perpendicular to the other 7 çç ÷+4
line BC, then BC is also perpendicular to AB, i.e., æ 3x + 4 ö è ( 5x - 7 ) ø÷
11. (b) We have, gof (x) = g ç ÷=
aRb Þ AB ^ BC è 5x - 7 ø æ ( 3x + 4 ) ö
5 çç ÷÷ - 3
And bRa Þ BC ^ AB è ( 5x - 7 ) ø
Hence R is symmetric.
41x
= =x
41
Mathematics S-M-57

16. (a) The operation table for * is given as


æ 7x + 4 ö
Similarly, fog (x) = f ç ÷
è 5x - 3 ø * 0 1 2 3 4 5
0 0 1 2 3 4 5
æ ( 7x + 4 ) ö 1 1 2 3 4 5 0
3 çç ÷÷ + 4
è ( 5x - 3) ø 2 2 3 4 5 0 1
= æ 3 3 4 5 0 1 2
5 çç
( 7x + 4 ) ö - 7 4 4 5 0 1 2 3
÷÷
è ( 5x - 3) ø 5 5 0 1 2 3 4

21x + 12 + 20x - 12 41x From the table, we note that


= = =x
35x + 20 - 35x + 21 41 a * 0 = 0 * a = a, " a Î { 0,1, 2, 3, 4, 5}
Thus, gof (x) = x, " x Î B and fog (x) = x, " x Î A, Hence, 0 is the identity for operation.
which implies that gof = IB and fog = IA. 17. (c) Here R = {(1, 3), (2, 2); (3, 2)}, S = {(2, 1); (3, 2); (2, 3)}
2
1 1 1 ì1 ü Then RoS = {(2, 3), (3, 2); (2, 2)}
12. (b) f (x + a) = - f (x) - f (x)2 = - í - f (x) ý
2 2 4 î2 þ 18. (a) g(f(x)) = |sin x| indicates that possibly f(x) = sinx, g(x) = |x|
Assuming it correct, f(g(x)) = f(|x|) = sin |x|, which is not
2
1 1 ì1 ü correct.
Þ f (x + 2a) = - - í - f (x + a) ý
( )2 indicates that possibly
2 4 î2 þ
f (g ( x )) = sin x
2
1 1 ì1 1 ü
= - - í - + f (x) - (f (x)) 2 ý g( x ) = x f ( x ) = sin 2 x or
2 4 î2 2 þ

1 1 1 1
g( x ) = sin x , f ( x ) = x 2
= - - f (x) + (f (x))2 = - - f (x) = f (x)
2 4 2 2 2
Then g(f (x )) = g(sin x) = sin x =| sin x |
13. (d) f (x) = [x]2 + [x + 1] – 3 = {[x] + 2} {[x] – 1} (for the first combination), which is given.
So, x = 1, 1.1, 1.2, .......... Þ f (x) = 0
\ f (x) is many one. Hence f(x) = sin2x, g (x) = x
only integral values will be attained. [Students may try by checking the options one by one]
\ f (x) is into. 19. (b) Let f : R ® R be a function defined by

ì1 - x, 0 < x < 1 x-m


14. (b) f (x) = | x - 1| = í f (x) =
x-n
î x - 1, x ³ 1
g (x) = ex, x ³ –1 For any (x, y) Î R
Let f (x) = f (y)
ì1 - g(x), 0 < g(x) < 1 i.e. - 1 £ x < 0
(fog) (x) = í x-m y-m
îg(x) - 1, g(x) ³ 1 i.e. 0 £ x Þ = Þ x=y
x-n y-n
ìï1 - e x , - 1 £ x < 0
= í x \ f is one – one
ïîe - 1, x³0 Let a Î R such that f (x) = a
\ domain = [–1, ¥) x-m
fog is decreasing in [–1, 0) and increasing in [0, ¥) Þ a= Þ (x – n) a = x – m
x-n
1
fog(-1) = 1- and fog (0) = 0 Þ xa –na =x–m Þ xa - x = n a - m
e
As x ® ¥, fog (x) ® ¥, Þ x(a - 1) = na - m
\ range = [0, ¥) na - m
Þ x= . for a = 1, x Ï R
1 æ y ö a -1
\ x= loge ç ÷
2 è 2–yø So, f is not onto.
20. (a) Since the domain of f is (0, 1),
15. (b) (a) Non-reflexive because (x3, x3) Ï R1
(b) Reflexive \ 0 < e x < 1 and 0 < ln | x | < 1
(c) Non-Reflexive e 0 < | x | < e1
EBD_7504
S-M-58 SOLUTIONS

Þ -¥ < x < 0 and 1 < | x | < e 1 é5 æ pö p æ p öù


= ê - 2 cos ç 2 x + ÷ cos + cos ç 2 x + ÷ ú
Þ x Î (-¥, 0) and x Î (-e, - 1) È (1, e) 2 ë2 è 3ø 3 è 3 øû

Þ x Î (-e, - 1) 5
= for all x.
x 4
21. (a) Given f (x) =
x -1 æ5ö æ5ö
gof (x) = g (f(x)) = g ç ÷ = 1 [Q g ç ÷ = 1
æ x ö è 4ø è 4ø
\ (fo f) (x) = f {f (x)} = f ç ÷
è x -1 ø (given)] Hence, gof(x) = 1, for all x.
26. (c) For f(x) to be defined, we must have
x x x
x -1 x -1 æ1 ö 1
= x - 1 = = = x. - 1 £ log 2 ç x 2 ÷ £ 1 Þ 2 -1 £ x 2 £ 21 [Q the base = 2 > 1]
x x - x +1 1 è 2 ø 2
-1
x -1 x -1 x -1
Þ 1£ x2 £ 4 …(1)
x
Þ (f o f o f )(x) = f (f o f )(x) = f (x) = Now, 1 £ x 2 Þ x 2 - 1 ³ 0 i.e ( x - 1)(x + 1) ³ 0
x -1
Þ x £ -1or x ³ 1 …(2)
x
Þ (f o f o f .....o f ) (x) = f (f o f)(x) = f (x) = Also, x 2 £ 4 Þ x 2 - 4 £ 0 i.e ( x - 2)( x + 2) £ 0
144244 3 x -1
19 times Þ -2 £ x £ 2 …(3)
22. (b) By definition only f (x) = x2 + 4x – 5 with domain From (2) and (3), we get the domain of f
[0, ¥) is one to one. = ((-¥, - 1] È [1, ¥)) Ç [-2, 2] = [-2, - 1] È [1, 2]

2 10 x - 10 - x 3y + 2 ax + b
27. (d) f ( x ) =
23. (b) If y = x - x , 102x = 2 - 3y cx + d
3 10 + 10
ì ax + b ü
aí ý+b
2 + 3y 2 + 3x cx + d þ
fof (x ) = î
1 1
or x = log10 \ f –1 (x) = log10 .
2 2 - 3y 2 2 - 3x ì ax + b ü
cí ý+d
î cx + d þ
24. (d) f :N®I
a 2 x + ab + bcx + bd
f (1) = 0, f (2) = -1, f (3) = 1, f (4) = -2, Þ =x
acx + bc + cdx + d 2
f (5) = 2, and f (6) = -3 so on.
Þ (ac + dc) x 2 + (bc + d 2 - bc - a 2 ) x
A B – ab – bd = 0, " x Î R
Þ (a + d)c = 0, d 2 - a 2 = 0
1 0
2 –1
and (a + d)b=0
3 1 Þ a+ d = 0 Þd= –a
4 –2 28. (d) We have, If x < 0 |x| = –x
5 2
–3
e-x - e-x
6
\ f ( x) = =0 \f ( x ) = 0 " x < 0
e x + e -x
In this type of function every element of set A has unique
\ f (x) is not one-one
image in set B and there is no element left in set B.
Hence f is one-one and onto function. e x - e -x
Next if x ³ 0, |x| = x \f ( x ) =
25. (a) We have ex + e-x
f ( x ) = sin 2 x + sin 2 (x + p / 3) + cos x cos( x + p / 3)
e x - e -x e 2x - 1 1+ y
Let y = Þ y= \ e 2x =
1 - cos 2 x 1 - cos(2 x + 2p / 3) e x + e -x e2x + 1 1- y
= +
2 2
1+ y 2y
1 For x ³ 0, e 2 x ³ 1 \ ³1Þ ³0
+ {2cos x cos( x + p / 3)} 1- y 1- y
2
Þ y( y - 1) £ 0, y ¹ 1 Þ 0 £ y < 1
1 é5 ì æ 2p ö ü æ p öù
= ê - ícos 2 x + cos \ f(x) is not onto
2 ëê 2 î
Mathematics S-M-59

29. (d) Since, (fog) x = f {g(x)} = f (x2) = sin x2 30. (b) 2 < x < 3 Þ x - 1 > 0
and (gof) x = g {f(x)} = g(sin x) = sin x 2
x-2 >0
Þ fog ¹ gof x -3 < 0
Þ f (x) = x - 1 + x - 2 + 3 - x = x
Þ f is an identity function

Speed Test-76

3p 5p p p
1. (d) <5< and cot A < 1 if <A<
2 2 4 2
Þ sin–1 (sin 5) = 5 – 2p
cot A + cot 3 A
Given sin–1 (sin 5) > x2 – 4x tan -1 (cot A) + tan -1 (cot 3 A) = p + tan -1 ,
Þ x2 – 4x + 4 < 9 – 2p 1 - cot 4 A
Þ (x – 2)2 < 9 – 2p
p
Þ – 9 – 2p < x – 2 < 9 – 2 p If 0 < A <
4
Þ 2 – 9 – 2p < x < 2 + 9 – 2p and tan–1(cotA) + tan –1 (cot3A)
2. (d) f ( x) = sin -1 x + tan -1 x + sec -1 x cot A + cot 3 A p p
= tan -1 if <A<
4
1 - cot A 4 2
Range of f ( x) = (Range of sin -1 x ) Ç
cot A + cot 3 A cot A cos ec 2 A. sin 4 A
(Range of tan–1x) Ç (Range of sec -1 x ) Also, =
1 - cot 4 A sin 4 A - cos 4 A
é p p ù æ p pö
ë 2 2û 2 2
p
2 { p
= ê - , ú Ç çè - , ÷ø Ç [0, ) È ( , p]
2 } =
sin A cos A
(sin A + cos 2 A )(sin 2 A - cos 2 A)
2
æ p pö
2 2 {
p
2
p
= çè - , ÷ø Ç [0, ) È ( , p]
2 } =-
sin 2A
2 cos 2 A
1
= - tan 2A
2
ìæ p p ö p ü ìæ p p ö p ü Hence,
= íçè - , ÷ø Ç [0, ) ý È íçè - , ÷ø Ç ( , p]ý
î 2 2 2 þ î 2 2 2 þ
æ1 ö
p p tan -1 ç tan 2 A÷ + tan -1 (cot A) + tan -1 (cot 3 A) ,
= [0, ) È f = [0, ) è2 ø
2 2
ì p
tan A – tan B ïp if 0 < A <
4
3. (d) tan (A – B) = =í
1 + tan A tan B p p
ï0 if < A <
3x 2x – K î 4 2

=
2 K– x 3K -1 -1
[Since, tan ( - x ) = - tan x ]
3x 2 x – K
1+ æ 2ö
2K – x 3K -1 1 - x ÷ p
5. (b) We have, cos ç <
ç1+ x2 ÷ 3
3Kx – (2 x – K)(2K – x ) è ø
=
(2K – x ) 3K + 3 x (2 x – K) æ 1 - x2 ö p
p
2 2 Þ- < cos -1 ç ÷<
3Kx – (4Kx – 2 x – 2K + Kx) 3 è 1 + x2 ø 3
=
2 3K 2 – 3Kx + 2 3x 2 – 3Kx
1 - x2 p 1 1 - x2
2 2 Þ 0 £ cos -1 < Þ < £1
2 x – 2Kx + 2 K 1
1 + x2 3 2 1 + x2
= = = tan 30º
2 2
2 3 x – 2 3Kx + 2 3k 3
Þ 1 + x 2 < 2(1 - x 2 ) £ 2(1 + x 2 )
\ A – B = 30º
p 1 1 1
4. (c) We know that cot A > 1 if 0 < A < Þ 0 £ x2 < Þ- <x<
3
EBD_7504
S-M-60 SOLUTIONS

-1 n p p n n æ a -a ö
6. (b) cot > Þ < cot -1 < p Þ -¥ < < 3 + tan -1 ç n n-1 ÷
p 6 6 p p è 1+ a n-1a n ø
Þ n < p 3 (Q n > 0) Þ n £ 5 (Q 5 < p 3 < 6)
( )(
= tan -1 a 2 - tan -1 a 1 + tan -1 a 3 - tan -1 a 2 + .... )
( )
-1 -1
7. (b) Since 0 £ cos xi £ p, \ cos xi = 0 for all i. -1
+ tan a n - tan -1 a n -1
2n
\ xi = 1 for all i \ å xi = 2n æ a -a ö
= tan -1 a n - tan -1 a1 = tan -1 çç n 1 ÷÷
i=1 è 1 + a n a1 ø
8. (d) We have
æ (n - 1) d ö
æ 1- x2 ö -1 æ 1 - y ö
2
π = tan -1 çç ÷
÷
cos -1 ç 2 ÷ + cos ç
ç 2 ÷
÷ = è 1 + a 1a n ø
è1+ x ø è 1+ y ø 2
Put x = tan q and y = tan f, we get é æ d ö æ d ö
\ tan ê tan -1çç ÷÷ + tan -1 ç ÷
ç 1 + a a ÷ + .....
æ 1- tan 2 q ö êë è 1 + a1a 2 ø è 2 3ø
-1 æ 1 - tan f ö
2
p
cos -1 çç 2 ÷÷ + cos çç 2 ÷ ÷= .
è 1 + tan q ø è 1 + tan f ø 2 æ d öù (n - 1)d
... + tan -1 çç ÷÷ú =
π è 1 + a n -1a n øûú 1 + a 1a n
Þ cos–1 (cos 2q) + cos–1 (cos 2f) = 11. (c) Let S¥ = cot–12 + cot–1 8 + cot–1 18 + cot–1 32 + ....
2
\ Tn = cot–1 2n2
p p
Þ 2(q + f) = Þ q+f = 1
2 4 = tan –1
2n 2
p
So, tan–1 x + tan–1 y = æ 2 ö æ (2n + 1) – (2 n –1) ö
4 = tan –1 ç 2 ÷ = tan –1 ç
è 4n ø è 1 + (2n + 1)(2n – 1) ÷ø
æ x+y ö
Þ tan -1 ç -1
÷ = tan 1 = tan–1 (2n + 1) – tan–1 (2n – 1)
è 1 - xy ø ¥
Þ x + y = 1 – xy Þ x + y + xy = 1 \ Sn = å {tan (2 n + 1) – tan (2n – 1)}
–1 –1

9. (c) sin–1 (log[x]) is defined if -1 £ log[ x ] £ 1 and [ x] > 0 n =1


= tan–1 ¥ – tan–1 1
1
Þ £ [ x ] £ e Þ [x] = 1, 2 Þ x Î [1, 3) p p p
e = – =
2 4 4
Again, log(sin -1[ x]) is defined if 2p
-1
12. (d) Let cos–1 x + cos–1 y =
sin [ x ] > 0 and -1 £[ x ] £ 1 7
Þ [ x ] > 0 and - 1 £ [ x] £ 1 Þ 0 < [ x] £ 1 æp -1 ö æ p -1 ö 2p
Þ ç - sin x ÷ + ç - sin y ÷ =
Þ x Î[1, 2) è2 ø è2 ø 7
\ Domain of f ( x) = [1, 2) 2p 5p
Þ sin–1 x + sin–1 y = p - = .
For 1 £ x < 2, [x] = 1 7 7
2
\ f ( x) = sin -1 0 + log p = log p , "x Î[1, 2) 13.
æ x xö
(a) Since, 1 ± sin x = ç cos ± sin ÷
2 2 è 2 2ø

\ Range of f ( x) = ì
ílog ý ìï 1 - sin x + 1 + sin x üï
î 2þ \ cot -1 í ý
îï 1 - sin x - 1 + sin x þï
-1 æ d ö æ
÷÷ + tan -1 ç
d ö
÷ + ...
10. (b) We have, tan çç ç ÷ éæ x xö æ x xö ù
è 1 + a1a 2 ø è 1 + a 2a 3 ø
ê çè cos 2 - sin 2 ÷ø + çè cos 2 + sin 2 ÷ø ú
= cot -1 ê ú
-1 æç d ö
÷ ê æ cos x - sin x ö - æ cos x + sin x ö ú
+ tan ç ÷
è 1 + a n -1a n ø êë çè 2
÷ ç
2ø è 2
÷
2 ø úû
æ a -a ö æ a -a ö ì æ xöü
= tan -1 ç 2 1 ÷ + tan -1 ç 3 2 ÷ +¼ ì xü x
è 1+ a1a 2 ø è 1+ a 2a 3 ø = cot -1 í - cot ý = cot-1 ícot ç p - ÷ ý = p -
î 2þ î è 2øþ 2
Mathematics S-M-61

14. (d) Let sin–1 a = x \ a = sin x


p p ì 1 1 ü
sin–1 b = y \ b = sin y or + + 2 í tan -1 + cot -1 ý = kp
sin–1 c = z \ c = sin z 2 2 î 3 3þ
\ a 1 - a2 + b 1 - b2 + c 1 - c 2 æ pö
or p + 2ç ÷ = k p
= sin x cos x + sin y cosy + sinz cosz è 2ø
= (1/2) (sin2x + sin2y + sin 2z) =(1/2) (4sin x sin y sin z) or p + p = kp
= 2 sinx siny sinz = = 2abc or 2p = kp
15. (a) We have or k=2
æ 3 3 ö÷ æ -1 1 1ö 1
a + b = ç sin -1 + cos -1 + sin + cos -1 ÷ 1 1
ç 2 2 ÷ çè 3 3ø 1 1 r( r + 1) -
è ø = = r r +1
18. (b) =
p p 1 + r + r 2 1 + r (r + 1) 1 + 1 1 æ 1 ö
= + =p r (r + 1)
1+ . ç ÷
2 2 r è r + 1ø
p -1 æ 1 ö 1 1
-1 -1
Since sin x + cos x = for all x \ tan çè 2÷
= tan-1 - tan-1
2 1+ r + r ø r r +1
p 1 p 1 ¥
Also, a = + sin -1 < + sin -1 æ 1 ö p
3 3 3 2 \ åtan-1 çè 1+ r + r2 ÷ø = tan-11 = 4
r=1
é pù
as sin q is increasing in ê0, ú ab + 1
ë 2û 19. (c) Qa - b < 0 , so cot -1 = cot -1 b - cot -1 a + p
a-b
p p p p
\ a< + = Þ b> >aÞ a <b bc + 1
3 6 2 2 b – c < 0, so cot -1 = cot -1 c - cot -1 b + p
b-c
æ x2 x3 ö
16. (b) sin–1 çç x - + - ... ÷÷ + -1 ca + 1
è 2 4 ø c – a > 0, so cot = cot -1 a - cot -1 c
c-a
æ x4 x6 ö p Adding we get
cos–1 çç x - - ... ÷÷ =
2
+
è 2 4 ø 2 ab + 1 bc + 1 ca + 1
cot -1 + cot -1 + cot -1 = 2p
x 2 x3 x 4 x6 a-b b-c c-a
This is true only when x - + - … = x2 - + ......
2 4 2 4 20. (b) We have, A = tan -1 2 Þ tan A = 2
and B = tan -1 3 Þ tan B = 3
x x2
Þ = since A, B, C are angles of a triangle, A + B + C = p
x x2
1+ 1+ Þ C = p - (A + B)
2 2
-1 é 2 + 3 ù
x x2 Now, A + B = tan -1 2 + tan -1 3 = p + tan ê ú
(Common ratios are - & - & |common ratios | < 1, in ë1 - 2.3 û
2 2
the given interval) é -1 -1 -1 é x + y ù ù
ê\ tan x + tan y = p + tan ê úú
2x 2x 2 ê ë1 - xy û ú
= Þ x = 0 or x = 1 Þ x = 1,
2 + x 2 + x2 êfor x > 0, y > 0 and xy > 1 ú
ë û
{x cannot be zero as 0 < |x| < 2 }.
p 3p
17. (b) The given question can be written as = p + tan -1 (-1) = p - tan -1 1 = p - =
4 4
æ 1ö æ 1ö
sin -1 ç ÷ + sec-1(5) + sec-1(2) + sin -1 ç ÷ 3p p
è 5ø è 2ø \ From (1), C = p - =
4 4
æ 1 ö
+2 tan -1 ç ÷ + 2 tan -1 ( 3) = k p 21. (b) We have, sin -1 x + sin -1 y = p - sin -1 z
è 3ø
or x (1- y2 ) + y (1- x2 ) = z
or
ì -1 æ 1 ö
î è 5ø
-1 æ 1 ö ü
è 5ø þ
-1
{
ísin ç ÷ + cos ç ÷ ý + sec (2) + cosec ( 2 )
-1
}
or x2 (1 - y 2 ) = z2 + y2 (1 - x2 ) - 2 yz (1 - x2 )
ì -1 æ 1 ö -1 æ 1 öü
+ í2 or ( x 2 - z 2 - y 2 )2 = 4 y 2 z 2 (1 - x 2 )
î
EBD_7504
S-M-62 SOLUTIONS

or x4 + y4 + z4 - 2x2 z2 + 2 y2 z2 - 2x2 y2 é a ù
- tan x ú
-1 æ a cos x - b sin x ö -1 ê b
÷ = tan ê a
2 2 2 2 2
+4x y z -4 y z = 0 26. (c) tan ç ú
è b cos x + a sin x ø ê1 + tan x ú
or x 4 + y 4 + z 4 + 4 x 2 y 2 z 2 = 2( x 2 y 2 + y 2 z 2 + z 2 x 2 ) ë b û
\ k=2 a a
= tan -1 - tan -1 (tan x) = tan -1 - x
–1
æ r – ( r – 1) ö b b
22. (c) Q Tr = sin ç ÷
è r ( r + 1) ø æ ö
27. (a) cot(cos -1 x) = sec ç tan -1 a
÷ …(i)
æ r – ( r – 1) ö è b2 - a2 ø
= tan –1 ç ÷
è 1 + r ( r – 1) ø a
-1
Let q = tan
n æ r – ( r – 1) ö b - a2
2
S n = å tan –1 ç ÷
r =1 è 1 + r ( r – 1) ø A
n
= å {tan –1 r – tan –1 ( r – 1)} a b
r =1 \ tan q = a
b2 - a 2
= tan –1 n – tan –1 0 q
= tan –1 n – 0 B 2 2 C
b –a
–1 p
\ S¥ = tan ¥ = 2 2 2 2 2
2 In DABC, AC = a + ( b - a )
23. (c) sin–1 (x – 1) Þ –1 £ x – 1 £ 1 Þ 0 £ x £ 2
cos–1 (x – 3) Þ –1 £ x – 3 £ 1 Þ 2 £ x £ 4 AC = b

æ x ö
b
tan–1 çè ÷ Þ x Î R, x ¹ 2, - 2 \ sec q = … (ii)
2 - x2 ø b2 - a 2

-12 From (i), cot (cos -1 x ) = sec q


sin–1 (2 – 1) + cos–1 (2 – 3) + tan = cos-1 k + p
2- 4 b
Þ sin–1 1 + cos–1 (– 1) + tan–1 (–1) = cos–1 k + p cot (cos -1 x) = [ using (ii)]
p p b2 - a 2
+ p - = cos -1 k + p
2 4 æ b ö
cos -1 x = cot -1 ç ÷ … (iii)
p 1
Þ cos–1 k = Þ k = è b2 - a 2 ø
4 2
æ b ö
é æ 1 ö -1 æ 5 ö ù -1
24. (a) tan êcos -1 ç ÷ø - sin çè ÷ú
Again let a = cot ç ÷
ë è 82 26 ø û è b 2 - a2 ø
= tan (tan–19 – tan–15) P
ì -1 æ 9 - 5 ö ü 2 2
2b –a
2
= tan í tan çè ÷ý = b
1 + 9 ´ 5 ø þ 23 cot a = 2 2
î b –a
b2 - a 2 a
2 1 5 Q b R
25. (c) 8 x + 22 x + 5 = 0 Þ x = - , -
4 2
\ PR 2 = ( b2 - a 2 )2 + b2
1 5
Q - 1 < - < 1 and - < -1
4 2 PR = 2b 2 - a 2
-1 æ 1 ö -1 æ 5 ö b
\ sin ç - ÷ exists but sin ç - ÷ does not exist \ cos a = … (iv)
è 4 ø è 2ø 2b - a 2
2

æ 5ö æ 1ö From (iii),
sec -1 ç - ÷ exists but sec -1 ç - ÷ does not exist,
è 2ø è 4ø cos -1 x = a or x = cos a
æ 1ö æ
tan -1 ç - ÷ and tan -1 ç [using (iv)]
è 4 ø è
Mathematics S-M-63

28. (a) tan (tan–1 x + tan –1 y + tan–1 z) \ Sum to n terms = Sn = tan -1 2 n - tan -1 1
– cot (cot–1 x + cot–1 y + cot–1 z)
Sum to infinite terms
= tan (tan–1 x + tan –1 y + tan–1 z)
p p p
æp p p ö = lim S n = lim tan -1 2 n - tan -1 1 = - =
– cot ç - tan -1 x + - tan -1 y + - tan -1 z ÷ n ®¥ n ®¥ 2 4 4
è2 2 2 ø
x
æ -1 -1 pö 30. (a) sin -1 x = tan -1 > tan -1 x > tan -1 y
çQ tan x + cot x = ÷ 1- x 2
è 2ø
= tan (tan–1 x + tan –1 y + tan–1 z) \ statement-2 is true
– cot{3p/2 – (tan –1 x + tan –1 y + tan–1 z) e<p
= tan (tan x + tan –1 y + tan–1 z)
–1
1 1
– tan (tan–1 x + tan –1 y + tan–1 z) = 0 >
e p
29. (b) Statement-2 is clearly true.
Now any general term of the series by statement-2.

2 k -1 2 k - 2 k -1 æ 1 ö æ 1 ö æ 1 ö
sin -1 ç ÷ > tan -1 ç ÷ > tan -1 ç
t k = tan - 1 = tan - 1 è eø è eø è p ÷ø
1 + 2 2 k -1 1 + 2 k × 2 k -1
statement-1 is true
= tan -1 2 k - tan -1 2 k -1

Speed Test-77

1. (a) Q Bn – A = I
(b) Let A = éê1 1ùú
1 1
\ Bn = I + A 2.
ë û
é1 0 0 ù é 26 26 18 ù
ê ú ê ú A2 = é1 1ù é1 1ù = 2 é1 1ù = 2A
B = ê 0 1 0 ú + ê 25 37 17 ú
n
ëê1 1ûú êë1 1ûú ëê1 1ûú
êë 0 0 1 úû êë52 39 50 úû
A3 = 22 éê11 11ùú , A 4 = 23 é1 1ù
êë1 1úû
é 27 26 18 ù ë û
ê 25 38 17 ú A3 = 22 A, A4 = 23A
Bn = ê ú
\ A n = 2n -1 é1 1ù Þ A = 2
100 100 -1
ëê 52 39 51úû êë1 1úû
A

é 1 4 2ù
n
é 27 26 18 ù \ A 100 = 299A
ê 3 5 1 ú = ê 25 38 17 ú écos q - sin q ù
or ê ú ê ú ....... (i) 3. (c) We have, A = ê ú
ë sin q cos q û
ëê 7 1 6úû ëê 52 39 51ûú
\ n ¹1 T é cos q sin q ù
Now put n = 2, then \ A = ê - sin q cos q ú
ë û
2 Now, AT + A = I2 (given)
é 1 4 2ù é 1 4 2 ù é 1 4 2ù
2 ê3 5 1 ú = ê3 5 1 ú ê3 5 1ú é cos q sin q ù é cos q - sin q ù é1 0 ù
B = ê ú ê úê ú Þê ú+ê ú=ê ú
êë 7 1 6úû êë 7 1 6 úû êë7 1 6 úû ë - sin q cos q û ë sin q cos q û ë 0 1 û
é 2 cos q 0 ù é1 0 ù
é1 + 12 + 14 4 + 20 + 2 2 + 4 + 12 ù Þê 0 =
ë 2 cos q úû êë0 1 úû
= êê 3 + 15 + 7 12 + 25 + 1 6 + 5 + 6 úú
1
êë 7 + 3 + 42 28 + 5 + 6 14 + 1 + 36 úû Þ 2 cosq = 1 Þ cos q =
2
é 27 26 18 ù p
Þ q = 2np + , n Î Z
= ê 25 38 17 ú 3
ê ú
êë 52 39 51úû 4. (b) (aI + bA)2 = a2I2 + b2A2 + 2ab AI
= a2I2 + b2 A2 + 2abA
Which is equal to R.H.S. of eq. (i).
\ n=2 aI + bA)2 = a2I + 2abA.
EBD_7504
S-M-64 SOLUTIONS

Also, P6 = P (5P – 3I)


é1 - 1 1ù é1 2 1ù Þ P6 = 5P2 – 3P
ê ú ê ú
5. (b) Let A = 2 1 0 , then A' = ê- 1 1 - 1ú Þ P6 = 5 (I – P) – 3P
ê ú
êë1 - 1 2úû êë 1 0 2 úû Þ P6 = 5I – 8P
So, n = 6
é1 - 1 1ù é 1 2 1 ù é3 1 4ù 11. (a) Given Q = PAPT
ê úê ú ê ú Þ PTQ = APT, (Q PPT = I)
\ AA' = ê2 1 0ú ê- 1 1 - 1ú = ê1 5 1 ú Þ PTQ2005P = APTQ2004 P = APTQ2003PA
êë1 - 1 2úû êë 1 0 2 úû êë4 1 4úû (Q Q = PAPT Þ QP = PA)
= APT Q2002PA2 = APTPA2004
é cos a sin a ù é cos b sin b ù 2005
6. (b) Hence f(a) f(b) = ê - sin a cos a ú ê - sin b cos b ú é1 1ù é1 2005ù
ë û ë û = AIA2004 =A2005 =ê ú =ê
ë0 1û ë0 1 úû
é cos a cos b - sin a sin b cos a sin b + sin a cos b ù
= ê - sin a cos b - cos a sin b - sin a sin b + cos a cos b ú é1 0 0 ù é1 0 0 ù é1 0 0 ù
ë û
12. (a) A = êê0 1 0 úú êê0 1 0 úú = êê0 1 0 úú
2
é cos(a + b) sin(a + b) ù
=ê ú êëa b -1úû êëa b -1úû êë0 0 1 úû
ë - sin(a + b) cos(a + b) û
A2 = A4 = A6 = I3 Þ A2 + 2A4 + 4A6
é cos(a + b + g ) sin(a + b + g ) ù
similarly f(a) f(b) f(g) = ê - sin(a + b + g ) cos(a + b + g ) ú é1 0 0ù é 2 0 0ù é 4 0 0 ù
ë û ê 0 1 0ú + ê 0 2 0ú + ê 0 4 0ú
é cos p sin p ù = ê ú ê ú ê ú
= ê ú as a + b + g = p êë 0 0 1úû êë 0 0 2úû êë 0 0 4úû
ë - sin p cos p û
é -1 0 ù é1 0 ù é7 0 0 ù
= ê 0 -1ú = - ê0 1 ú = – I2 ê 0 7 0 ú = 7I = 7A8
ë û ë û = ê ú 3
7. (b) We have a theorem that if a square matrix A satisfies êë 0 0 7 úû
the equation
é1 2 ù é a 0ù
a0 + a1 x + a 2 x 2 + ....... + a n x n = 0, 13. (d) A=ê ú B=ê ú
ë3 4û ë0 bû
where a0 ¹ 0 then A is invertible.
Since A, B an d C are n × n matrices and é a 2b ù
AB = ê ú
A satisfies the equation x3 + 2x 2 + 3x + 5 = 0 as ë3a 4b û
A 3 + 2 A 2 + 3A + 5I = 0 , therefore, A is invertible. é a 0 ù é1 2 ù é a 2 a ù
BA = ê úê ú=ê ú
écos a - sin a ù é cos a sin a ù ë 0 b û ë3 4û ë3b 4b û
8. (a) A a × A ( -a ) = ê úê ú
ë sin a cos a û ë- sin a cos a û Hence, AB = BA only when a = b
\ There can be infinitely many B¢s for which AB = BA
é cos 2 a + sin 2 a sin a cos a - sin a cos a ù
=ê ú é 1 w w 2 ù é k 1 1 ù é0 0 0 ù
êësin a cos a - sin a cos a cos 2 a + sin 2 a úû ê ú
14. (b) êw w 2 1 ú êê 1 1 1 úú = êê0 0 0úú
é1 0 ù êë w 2 1 w úû êë 1 1 1 úû êë0 0 0úû
=ê ú=I
ë0 1 û
9. (a) (A – 2I) (A + I) = 0 é k + w + w 2 1 + w + w2 1 + w + w 2 ù é0 0 0ù
ê ú
Þ AA – A – 2I = 0 (Q AI = A) kw + w 2 + 1 w + w 2 + 1 w + w 2 + 1 ú = ê0 0 0ú
Þ ê 2 ê ú
æA-Iö A-I ê kw + 1 + w w 2 + 1 + w w2 + 1 + w ú êë0 0 0úû
Þ Aç ÷=I \ = A -1 ë û
è 2 ø 2
10. (c) Q P3 = P(I – P) é 1 + w + w2 + k - 1 0 0ù é 0 0 0 ù
= PI – P2 = PI – (I – P) ê 0 0ú = ê 0 0 0 ú
1 + w + w 2 + kw - w
= P– I + P = 2P – I Þ ê ú ê ú
ê1 + w + w 2 + kw 2 - w 2 0 0ú ê 0 0 0 ú
Now, P4 = P.P3 ë û ë û
Þ P4 = P(2P – I)
Þ P4 = 2P2 – P é k -1 0 0ù é 0 0 0 ù
Þ P4 = 2I – 2P – P ê ú
(k - 1)w 0 0ú = ê 0 0 0 ú
Þ P4 = 2I –3P Þ ê ê ú
and P5 = P (2I – 3P) ê 0 0ú êë 0 0 0 úû
(k - 1)w 2
Þ P5
Þ P5 = 5P – 3I 0 or k = 1
Mathematics S-M-65

é 6 8 5ù
é0 - t ù æ aö 16. (a) If A = ê 4 2 3 ú is the sum of a symmetric matrix B
15. (a) Here, A = ê t 0 ú , where t = tan çè ÷ø êë 9 7 1 úû
ë û 2
and skew symmetric matrix C,
æ aö
1 - tan 2 ç ÷ é6 4 9 ù
è 2 ø 1- t2 Transpose of A = ê8 2 7 ú
Now, cos a = =
2 æ a ö 1+ t2 êë5 3 1 úû
1 + tan ç ÷
è 2ø
1 é é 6 8 5ù é6 4 9 ù ù
So that B = ê ê 4 2 3ú + ê 8 2 7 ú ú
æaö 2 ê ê 9 7 1ú ê5 3 1 ú ú
2 tan ç ÷ ëë û ë ûû
and sin a = è 2 ø = 2t
æaö 2
1 + tan 2 ç ÷ 1 + t 1 é 12 12 14 ù
è2ø B= ê 12 4 10 ú
2 êë 14 10 2 ûú
é cos a - sin a ù
= (I - A) ê ú é6 6 7ù
ë sin a cos a û \ B= ê 6 2 5 ú
êë 7 5 1 úû
é1 - t 2 -2t ù
ê ú 17. (a) A2 = I
2
æ é 1 0ù é 0 - t ù ö ê 1 + t 1+ t2 ú Now, (A – I)3 + (A + I)3 – 7A
ê ú
= ç ê 0 1ú - ê + t 0 ú÷ ê 2t 1 - t2 ú
= A3 – I3 – 3A2I + 3AI2 + A3 + I3 + 3A2I + 3AI2 – 7A
èë û ë ûø = 2A3 + 6AI2 – 7A = 2A2A + 6AI – 7A
ê ú
ë1 + t 2 1+ t2 û éQ A 2 = I ù
= 2IA +6A – 7A = 2A + 6A – 7A = A ë û
é1 - t 2 -2t ù 18. (a) Since B is an idempotent matrix, \ B2 = B.
ê ú Now, A2 = (I – B)2 = (I – B)(I – B)
ê1 + t 2 1+ t2 ú
é 1 tù ê ú = I - IB - BI + B2 = I - B - B + B2 = I - 2B + B2
= ê - t 1ú
ë û ê 2t 1 - t2 ú
ê ú = I - 2B + B = I - B = A
ë1 + t 2 1+ t2 û \ A is idempotent.
écos a - sin a 0ù écos b - sin b 0ù
é 1 - t 2 + 2t 2 -2t + t (1 - t 2 ) ù ê úê ú
ê ú 19. (c) F(a) . F(b) = ê sin a cos a 0ú ê sin b cos b 0ú
ê 1+ t2 1+ t2 ú êë 0 0 1úû êë 0 0 1úû
ê ú
= ê - t (1 - t 2 ) + 2t 2t 2 + 1 - t 2 ú
ê ú é cos(a + b) - sin(a + b) 0ù
ë 1+ t2 1+ t2 û F (a ) . F(b) = ê sin(a + b ) cos(a + b ) 0ú = F (a + b )
ê ú
é 1 + t2 -2t + t - t 3 ù ëê 0 0 1úû
ê ú
ê 1 + t2 1 + t2 ú æ x + y ö é 1 + xy ù
ê ú 20. (c) A( z ) = A çè 1 + xy ÷ø = ê (1– x )(1 – y ) ú
= ê - t + t 3 + 2t 2t 2 + 1 - t 2 ú ë û
ê ú
ë 1 + t2 1 + t2 û é æ x + y öù
ê 1 –ç
è 1 + xy ÷ø úú
- t (1 + t 2 ) ù
é 1 + t2 ê
ê ú ê æ x+ yö ú
ê 1 + t2 1 + t2 ú ê– ç ÷ 1 ú
ê ú é1 - t ù ë è 1 + xy ø û
= ê t (1 + t 2 ) 1 + t2 ú = êt 1 ú
ë û \ A(x). A(y) = A(z).
ê ú
ë 1 + t2 1 + t2 û é0 -1ù
21. (d) We have A = ê ú
é 1 0ù é 0 - t ù ë1 0 û
Also, I + A = ê ú+ê ú
ë0 1û ë t 0 û æ 0 -1ö æ 0 -1ö
Now, A2 = A . A = ç
é 0 + 1 - t + 0ù è 1 0÷ø çè 1 0÷ø
= êt + 0 0 + 1 ú
ë û æ -1 0ö
=ç =–I
é1 - t ù è 0 -1÷ø
= êt 1 ú
ë û
EBD_7504
S-M-66 SOLUTIONS

25. (b) Let A = [aij]n´ m. Since A is skew–symmetric aii = 0


æ 1 0ö
where I = ç is identity matrix (i = 1, 2, ......, n ) and aji = – aji (i ¹ j)
è 0 1÷ø Also, A is symmetric so aji = aji " i and j
(A2)8 = (– I)8 = I \ aji = 0 " i ¹ j
Hence, A16= I Hence aij = 0 " i and j Þ A is a null zero matrix
éa b ù é a b ù éa b ù é1 0ù 26. (a) Since BA = B, \ (BA)B = BB = B 2
ê g -a ú = I2 ; ê g -a ú ê g =
22. (d)
ë û ë ûë -a úû êë 0 1 úû Þ B(AB) = B2 Þ BA = B2 (Q AB = A)
Þ a2 + bg = 1 Þ B = B2 (Q BA = B)
27. (b) B = – A–1 BA Þ AB = – BA Þ AB + BA = 0
23. (a) ( AB¢ - BA¢ )¢ = ( AB¢ )¢ - ( BA ¢ )¢ \ (A + B)2 = A2 + AB + BA + B2 = A2 + B2

= ( B¢ )¢ A¢ – ( A ¢ )¢ B¢ = BA ¢ - AB¢ = - ( AB¢ - BA ¢ ) éa 0 0ù éa 0 0ù éa 2 0 0ù
ê2 ú ê ú ê ú
Hence, ( AB¢ - BA ¢ ) is a skew-symmetric matrix.
28. (c) A = 0 a 0 0 a 0 = ê 0
ê úê ú a2 0ú
êë0 0 a úû êë0 0 a úû ê0 0 a 2 úú
é2 1ù é- 3 2 ù ëê û
24. (a) Let B = ê ú and C = ê 5 - 3ú
ë 3 2 û ë û éa 2 0 0ù éa 0 0ù éa
3
0 0ù
-1 -1 3ê 2 ú ê ú ê 3 ú
Given BAC = I Þ B (BAC) = B I A =ê0 a 0ú ê0 a 0ú = ê 0 a 0ú
-1 -1
ê0 0 a 2 úú êë0 0 a úû ê 0 0 a 3 úú
Þ I(AC) = B Þ AC = B êë û êë û
-1 -1
Þ ACC -1 = B -1C -1 Þ AI = B -1C -1 \ A = (B )(C ) æ 2 -1ö æ 2 -7 ö æ 1 0ö
29. (d) Here A AT = çè -7 4 ÷ø çè -1 4 ÷ø ¹ çè 0 1÷ø
-1 1 é2 - 1ù é 2 - 1ù
Now B = 4 - 3 ê- 3 =
2 úû êë- 3 2 úû (BBT)11 = (d)2 + (a)2 ¹ 1
ë
(AB)11 = 8 – 7 = 1, (BA)11 = 8 – 7 = 1
1 é - 3 - 2 ù é3 2 ù \ AB ¹ BA may be not true
C -1 = ê ú=ê ú
9 - 10 ë- 5 - 3û ë5 3 û æ 2 -1ö æ 4 1ö
Now AB = ç -7 4 ÷ ç 7 2÷
é 2 - 1ù é3 2ù é1 1ù è ø è ø
\ (B -1 )(C -1 ) = ê úê ú=ê ú
ë- 3 2 û ë5 3û ë1 0û æ 8-7 2 - 2 ö æ 1 0ö æ 1 0ö
é1 1 ù = ç -28 + 28 -7 + 8÷ = ç 0 1÷ ; (AB)T = ç 0 1÷ = I
è ø è ø è ø
\A = ê ú
ë1 0û 30. T T T T T T T
(b) (A – A ) = A –(A ) = A – A = – (A – A )
Hence, (A – AT) is skew-symmetric.

Speed Test-78

C1 ® C1 + C2 + C3
cos 2 54° cos 2 36° cot135°
2 cos2 54° + sin 2 54° - 1 sin2 54° -1
1. (c) Let D = sin 53° cot135° sin 2 37°
2 2
= cos 37° - 1 + sin 37° -1 sin 2 37°
cot135° cos2 25° cos 2 65°
-1 + cos2 25° + sin 2 25° cos2 25° sin 2 25°
cos2 54° cos2 (90° - 54°) -1
0 sin 2 54° – 1
2 2
= sin (90° - 37°) -1 sin 37°
0 –1 sin 2 37° = 0
2 =
-1 cos 25° cos2 (90 - 25°) 0 cos 2 25° sin 2 25°
2. (a) We must have f (x) = k (x – a)2 where k is a constant.
cos 2 54° sin 2 54° -1 Now in order that the given determinant [Say, D(x)] be
divisible by f(x) we must show that both D(x) and D'(x)
= cos 2 37° -1 sin 2 37°
vanish at x = a. Now
-1 cos 2 25° sin 2 25°
Mathematics S-M-67

A(a ) B( a) C( a ) 6. (b) We have


D(a ) = A(a ) B( a ) C( a ) = 0 0 - y -z 0 y z
A ¢ ( a ) B¢ ( a ) C ¢ ( a ) D = y 0 -x = - y 0 x
z x 0 -z -x 0
Also
A '( x ) B'( x) C'( x) A( x ) B( x) C( x) [Interchanging rows and columns]
D ¢( x ) = A(a ) B(a ) C( a ) + 0 0 0 0 - y -z
3
A ¢ ( a ) B¢ ( a ) C ¢ ( a ) A ¢ ( a ) B¢ ( a ) C ¢( a ) = ( -1) y 0 - x [Taking –1 common from each row]
z x 0
A( x ) B( x ) C( x ) A '( x ) B'( x ) C '( x ) = –D
+ A(a ) B( a ) C(a ) = A(a ) B(a ) C( a )
\ D + D = 0 Þ 2 Re( D ) = 0
0 0 0 A ¢ ( a ) B¢ ( a ) C ¢ ( a )
which clearly gives D' (a) = 0, since first and third row \ D is purely imaginary.
become identical. 7. (d) Given system of equations can be written in matrix
3. (a) From the given system of equations, form as AX = B where
D1 D D
x= , y= 2, z= 3 æ1 2 3ö æ6ö
D D D A = ç 1 3 5 ÷ and B = ç 9 ÷
where, D = D (a, b, c) ç2 5 a÷ çb÷
è ø è ø
D1 = D (d, b, c)
D2 = D (a, d, c) Since, system is consistent and has infinitely many
D1 = D (a, b, d) solutions
D(d , b, c ) \ (adj. A) B = 0
Now, x =
D(a, b, c)
æ 3a - 25 15 - 2a 1 ö æ 6 ö æ 0 ö
- d1 b1 c1 ç a - 6 -2 ÷ ç 9 ÷ = ç 0 ÷
Þ ç 10 - a
where, D (d, b, c) = -d 2 b2 c2 è -1 -1 1 ÷ø çè b ÷ø çè 0 ÷ø
-d3 b3 c3 Þ – 6 – 9 + b= 0 Þ b = 15
b1 - d1 c1 b1 c1 -d1 and 6(10 – a) + 9(a – 6) – 2(b) = 0
Þ 60 – 6a + 9a – 54 – 30 = 0
= - b2 -d 2 c2 = + b2 c2 -d 2
Þ 3a = 24 Þ a = 8
b3 -d3 c3 b3 c3 -d 3 Hence, a = 8, b = 15.
= D(b, c, d ) 8. (d) The given determinant vanishes, i.e.,
D(b, c, d )
Hence, x = 1 x - 3 ( x - 3)2
D (a, b, c)
1 x - 4 ( x - 4)2 = 0
2
1 sin A sin A
1 x - 5 ( x - 5)2
4. (a) 1 sin B sin 2 B = 0
1 sin C sin 2 C
Expanding along C1, we get
Þ (sin A – sin B)(sin B – sin C)(sin C – sin A) = 0 (x – 4)(x – 5)2 – (x – 5)(x – 4)2 – {(x – 3)(x – 5)2
– (x – 5)(x – 3)2} + (x – 3)(x – 4)2
Þ sin A = sin B or sin B = sin C or sin C = sin A
– (x – 4)(x – 3)2 = 0
\ atleast two of A, B, C are equal.
Hence the triangle is isosceles or equilateral. Þ (x – 4)(x – 5)(x – 5 – x + 4)
5. (a) Since, –1 < x < 0 – (x – 3)(x – 5)(x – 5 – x + 3)
\ [x] = –1 +(x – 3)(x – 4) (x – 4 –x + 3) = 0
0 < y < 1 \ [y] = 0 Þ – (x – 4)(x – 5) + 2(x – 3)(x – 5) – (x – 3)(x – 4) = 0
1 < z < 2 \ [z] = 1 Þ – x2 + 9x – 20 + 2x2 – 16x + 30 – x2 + 7x – 12 = 0
0 0 1 Þ – 32 + 30 = 0 Þ –2 = 0
\ Given determinant = -1 1 1 = 1 = [z] Which is not possible, hence no value of x satisfies the
-1 0 2 given condition.
EBD_7504
S-M-68 SOLUTIONS

m 1 cos x cos x
2r - 1 Cr 1
9. (a) D r = m2 - 1 2m m +1 Þ (sin x + 2cos x) 1 sin x cos x = 0
1 cos x sin x
sin 2 ( m2 ) sin 2 ( m) sin 2 (m + 1)
1 cos x cos x
m m m Þ (sin x + 2cos x) 0 sin x - cos x 0 =0
å (2r - 1) å m
Cr å1 0 0 sin x - cos x
r =0 r =0 r =0
m
Þ (sin x + 2 cos x )(sin x - cos x ) 2 = 0
Þ å Dr = m2 - 1 2m m +1
r =0
sin2 ( m2 ) sin 2 (m) sin2 (m + 1) Þ sin x + 2cos x = 0, sin x - cos x = 0
Þ tan x = -2, tan x = 1 .
é p pù
m2 - 1 2m m +1 For tan x = -2 , x does not lie on ê- , ú
ë 4 4û
= m2 - 1 2m m +1 =0 p
For tan x = 1, x = , hence only one root.
4
sin 2 ( m2 ) sin 2 (m ) sin 2 ( m + 1) 14. (c) The given system of equations are :
p3x + (p +1)3 y = (p +2)3 ...(1)
a b ax + by px + (p +1)y = (p +2) ....(2)
10. (b) Let D = b c bx + cy x +y = 1 ....(3)
ax + by bx + cy 0 This system is consistent, if values of x and y from first
two equation satisfy the third equation.
a b ax + by
= b c bx + cy p3 (p + 1)3 (p + 2)3
0 0 - ( ax 2 + 2bxy + cy 2 ) which Þ p (p + 1) (p + 2) = 0
[Applying R 3 ® R 3 - xR1 - yR 2 ] 1 1 1
2 2 2
= ( b - ac )( ax + 2bxy + cy )
Now, b 2 - ac < 0 and a < 0 p3 (p + 1)3 - p3 (p + 2)3 - p3
Þ Discriminant of ax2 + 2bxy + cy2 is negative and a < 0. Þ p 1 2 =0
Þ ax 2 + 2bxy + cy 2 < 0 for all, x, y Î R [See Quadratics] 1 0 0
2 2 2 Þ 2 (p + 1)3 – 2p3 – (p + 2)3 + p3 = 0
Þ D = (b - ac)(ax + 2bxy + cy ) > 0 .
11. (d) | A2013 – 3A2012 | = | A2012 | | A – 3I | = | A |2012 | A – 3I Þ 2 (p3 +1+3p2 +3p) –2p3 – (p3 + 8 + 12p + 6p2)+ p3 =0
| Þ 2p3 + 2 + 6p2 + 6p – 2p3– p3 – 8 – 12p – 6p2 + p3 = 0
Þ – 6 – 6p = 0
0 7 Þ p=–1
= (1) 1 -1 = (1) (– 7) = – 7. 15. (a) Given determinant is
12. (a) We can write D as, (a x + a - x )2 (a x - a - x )2 1
1 cot a cot 2 a ( b x + b- x ) 2 (b x - b - x ) 2 1
D = sin a sin b sin g 1 cot b cot 2 b
2 2 2
(c x + c - x )2 (c x - c - x )2 1
1 cot g cot 2 g

2 2 2
= sin a sin b sin g (cot b - cot a) a 2 x + a -2 x + 2 a 2 x + a -2 x - 2 1
-2 x
(cot g - cot a)(cot g - cot b)
2x
= b +b +2 b2 x + b-2 x - 2 1
= sin(a - b) sin(a - g ) sin(b - g ) c2 x + c-2 x + 2 c2 x + c-2 x + 2 1
It is clear from here that D cannot exceed 1.
Applying C1 ® C1 – C2
[Qsin q >/ 1, for any q Î R ]
13. (c) Applying C1 ® C1 + C 2 + C 3 , we get 4 a 2 x + a -2 x - 2 1 1 a 2 x + a -2 x - 2 1
= 4 b2 x + b-2 x - 2 1 = 4 1 b2 x + b-2 x - 2 1
sin x + 2 cos x cos x cos x
sin x + 2 cos x sin x cos x = 0 4 c2 x + c-2 x - 2 1 1 c2 x + c-2 x - 2 1
sin x + 2 cos x
Mathematics S-M-69

16. (a) Given determinant


-q r
1 a a2 and b1 + b2 = , b1b2 = ...(2)
p p
co s ( n - 1) x cos nx c os ( n + 1) x
=0 Since the given system of equation has a non-trivial solution
sin ( n - 1) x sin n x sin ( n + 1) x
a1 a 2
1 + a 2 - 2a cos x a a2 \ = 0 i.e. a1b2 – a2b1 = 0
b1 b 2
Þ 0 cos nx cos (n + 1) x = 0
0 sin nx sin (n + 1) x a1 a 2 a1 + a 2 a1a 2
or = = =
b1 b 2 b1 + b 2 b1b 2
By applying C1 ® C1 + C3 – 2 cos x C2
By expanding
(1 + a2 – 2a cos x) [cos nx sin (n + 1) x pb pc b 2 ac
Þ = Þ =
– sin nx cos (n + 1) x]= 0 qa ra q 2 pr
Now, (1 + a2 – 2a cos x) sin (n + 1 – n) x = 0
3 2 4
Þ (1 + a 2 - 2a cos x)sin x = 0
7 2 0
20. (d) Putting x = 0 in both sides g = .
1 + a2 3 -1 7
sin x = 0 or cos x =
2a
1 2 4
æ 1 + a2 ö
As a ¹ 1 \ç ÷ >1 91 2 0
è 2a ø Apply C1 ® C1 + C2 + C3 =
1 -1 7
Þ cos x > 1 It is not possible.
\ sin x = 0 Apply R1 ® R1 – R2
17. (c) Here
0 0 4
1 (n + 1)
(n + 2) (n + 1)
(n + 3) (n + 2) = 9 1 2 0 = 9 ´ 4(-3) = -108
(n + 1) (n + 2) (n + 1)
D = (n!)3 (n + 1) 1 -1 7
(n + 2) (n + 1) (n + 3) (n + 2) (n + 4) (n + 3) 21. (b) We have a theorem that if a square matrix A satisfies
(n + 1) (n + 2)(n + 1)
the equation
= (n!)3 (n +1)2 (n + 2)
a0 + a1 x + a2 x 2 + ....... + a n x n = 0,
1 1 1
n +1 n+2 n +3 where a0 ¹ 0 then A is invertible.
×
(n + 2) (n + 1) (n + 3) (n + 2) (n + 4) (n + 3) Since A, B an d C are n × n matrices and
A satisfies the equation x3 + 2x 2 + 3x + 5 = 0 as
Operating C2 – C1, C3 – C2 and expanding
= (n!)3 (n + 1)2 (n + 2). 2 A 3 + 2 A 2 + 3A + 5I = 0 , therefore, A is invertible.
= (n!)3 (2n3 + 8n 2 + 10n +4) as on simplification.
3a 3b c 3a x p
S0 S1 S2 x 2y z 3b 2 y 5
18. (d) D = S1 S2 S3 22. (a) = [changing rows into
S2 S 3 S 4 p 5 5 c z 5
columns]
1+1+1 a + b + g a 2 + b2 + g 2
3a x p a 5x p
= a + b + g a + b 2 + g 2 a 3 + b3 + g 3
2
1 3 1
3b 2 y 5 ´ b 10 y 5 1
a 2 + b2 + g 2 a3 + b3 + g 3 a 4 + b4 + g 4 =3 =3 5 = (125) = 25
3c 3z 15 c 15z 15 5
The above determinant can be expressed as product of
two determinants. Thus,
-a 1 1
1 1 1 1 1 1 23. (b) D= 1 -b 1 = 0 for non-zero solution
D = a b g a b g = [(b - a )(g - a )(g - b)]2
1 1 -c
a 2 b2 g 2 a 2 b2 g 2
Þ abc – a – b – c – 2 = 0
19. (a) Since a1, a2 and b1, b2 are the roots of ax2 + bx + c = 0 Þ abc = a + b + c + 2
and px2 + qx + r = 0 respectively, therefore
1 1 1
-b c
a1 + a2 = , a1 +c
a
EBD_7504
S-M-70 SOLUTIONS

3 + 2 (a + b + c) + (ab + bc + ac) Þ sin 3q [28 – 21] + 1 [7 cos 2q – 6] + [7 cos 2q – 8] = 0


= Þ 3 sin q – 4 sin 3 q + 2(1 – 2 sin2 q) – 2 = 0
1 + (a + b + c) + (ab + bc + ac) + abc
Þ sin q (4 sin2 q + 4 sin q – 3) = 0
3 + 2 ( a + b + c) + (ab + bc + ac ) Either sin q = 0 or 4 sin 2 q + 6 sin q – 2 sin q – 3 = 0
= =1 Þ (2 sin q – 1) (2 sin q + 3) = 0
1 + 2 ( a + b + c ) + 2 + ab + bc + ac
1 3
a2 b2 c2 \ sin q = ,sin q ¹ – [Q sin q > – 1]
2 2
24. (c) Let D = (a + 1) 2 (b + 1) 2 (c + 1) 2 ; p
(a - 1) 2
(b - 1) 2
(c - 1) 2 \ q = np or q = np + (–1)n 6

Apply R 2 ® R 2 - R 3 and take 4 common é (–1)n ù


Þ q = p ên + ú.
ë 6 û
a2 b2 c2
D=4 a b c 27. (c) Given adj B= A, P = Q = 1
;
(a - 1) 2 (b - 1) 2 (c - 1) 2 Consider , adj (Q-1B P -1 )
= (adj P–1) (adj B) (adj Q–1)
Apply R 3 ® R 3 + 2R 2 - R 1
= (adjP) -1 A.(adjQ)-1
2 2 2
a b c
æ 1 ö
D=4 a b c -1
= 4 (a – b) (b – c) (c – a). = (P–1)–1 A(Q–1)–1 çQ P = P .adjP÷
1 1 1 è ø
= PAQ .
Now given D = 0 Þ 4(a - b)( b - c)(c - a ) = 0 28. (c) Since the system has a non-trivial solution,
Clearly, atleast one of the factors must be zero. Hence the
l sin a cos a
triangle must be isosceles.
ALTERNATE : therefore 1 cos a sin a = 0
-1 sin a - cos a
If a = b then D = 0 as C1 and C2 become identical. Similarly
if b = c or c = a, D = 0 . Þ l (– cos2a – sin2a) – (–sina cosa – sina cosa)
– (sin2a – cos2a) = 0
écos a - sin a 0ù écos(-a) - sin(-a) 0ù Þ –l + sin 2a + cos 2a = 0 Þ l = sin 2a + cos 2a
25. (a) F(a) . F(-a) = êê sin a úê
cos a 0ú ê sin(-a) cos(-a) 0ú
ú

æ
êë 0 0 1úû êë 0 0 1úû Þ l= 2 cos ç 2a - ÷
è 4ø
écos a - sin a 0ù é cos a sin a 0ù æ pö
ê úê ú Since –1 < cos ç 2a - ÷ < 1 " ÎR
F(a) . F(-a) = ê sin a cos a 0ú ê- sin a cos a 0ú è 4ø
êë 0 0 1úû êë 0 0 1úû
\ – 2 < l< [
2 i.e. l Î - 2 , 2 ]
é cos 2 a + sin 2 a + 0 cos a sin a - cos a sin a + 0 0 + 0 + 0ù
ê ú é bc 0 0 ù
= êsin a cos a - sin a cos a + 0 sin 2 a + cos 2 a + 0 0 + 0 + 0ú -1 1 1 ê ú
ê 29. (b) A = adjA = 0 ca 0
0+ 0+0 0+0+ 0 0 + 0 + 1úú det A abc ê ú
ëê û êë 0 0 ab úû
é1 0 0 ù
ê ú é 1 ù
= ê0 1 0 ú = I [Q cos 2 a + sin 2 a = 1] 0 0 ú
ê a
êë0 0 1 úû ê ú
1
-1
= êê 0 0 ú
ú
F( a) . F( -a) = 1 \[F( a)] = F(-a ) b
ê ú
ê 1 ú
26. (c) Given system of equations 0 0
ëê c ûú
x sin 3q – y + z = 0 ù
are homogeneous system of The inverse of a diagonal matrix is a diagonal matrix.
x cos 2q + 4 y + 3z = 0 úú Both true but Statement 2 is not correct reason of
linear equation
2 x + 7 y + 7z = 0 úû Statement 1.
30. (d) If D = 0 then two of rows or column are proportional
Since system has non-trivial solution
which is possible even if three lines are parallel or two
sin 3q –1 1 of them are coincident.
cos 2q 4 3 = 0
\
2 7
Mathematics S-M-71

Speed Test-79

1. (c) We have
du
æ x + y ö f ( x ) + f ( y) Þ u=2q Þ = 2 ...... (a)
fç ÷= , f(0) =0 and f ¢ (0) = 3 dq
è 3 ø 3
In equation (ii), put x = tan q
æ 3x + 3h ö
fç ÷ - f (x) é 2 tan q ù
f (x + h ) - f ( x) è 3 ø \ v = sin -1 ê = sin -1 (sin 2q )
f ¢(x ) = lim = lim 2 ú
h®0 h h ®0 h ë1 + tan q û

f (3x) + f (3h ) f (3x) + f (0) dv


- Þ v = 2q Þ = 2 ........ (b)
3 3 f (3h ) - f (0)
= lim = lim =3 dq
h ®0 h h ®0 3h
\ f(x) = 3x + c, Q f(0) = 0 Þ c = 0 From equations (a) and (b),
\ f(x) = 3x du du d q 1
1 = ´ = 2´ = 1
1/ x dv d q dv
2. (a) lim ( cos x ) = k Þ lim log ( cos x ) = log k 2
x ®0 x ®0 x \ Required differential coefficient will be 1.
1
Þ lim lim log cos x = log k 6. (c) In the definition of the function, b ¹ 0, then f(x) will be
x ®0 x x ®0
undefined in x > 0.
1
Þ lim ´ 0 = log e k Þ k = 1. Q f(x) is continuous at x = 0, \ LHL = RHL = f(0)
x® 0 x

3. (b) x 2 + y 2 = a 2 Þ 2 x + 2 yy¢ = 0 Þ y¢ = - x / y sin( a + 1) x + sin x x + bx 2 - x


lim = lim =c
Þ yy¢ + x = 0 Þ x ®0 x x ®0 bx 3 / 2
x<0 x <0
æ 1 + y¢ ö 2
Þ yy ¢¢ + y ¢ 2 + 1 = 0 Þ y = - ç ....(i)
è y ¢¢ ÷ø
æ sin( a + 1) x sin x ö 1 + bx - 1
Þ lim ç + ÷ = lim =c
1 1 1 1 é xù x ® 0è x x ø x ®0 bx
\k = =
a
= =
êQ y ¢ = - y ú
x2 + y 2 x2 y 1 + y12 ë û
y 1+ 2
y (1 + bx ) - 1
Þ (a + 1) + 1 = lim =c
x ®0 bx ( 1 + bx + 1)
- y ¢¢ | y '' |
= = 1
2
(1 + y ¢ ) 1 + y ¢ 2 (1 + y ¢ 2 ) 3/ 2 Þ a + 2 = lim =c
x ®0 1 + bx + 1
4. (c) Since f(x) is continuous at x = 0 \ lim f ( x ) = f (0)
x ®0 1 3 1
Take any point x = a, then at x = a Þ a+2 = =c \ a =- ,c= , b¹0
2 2 2
lim f ( x ) = lim f (a + h )
x ®a h®0 7. (b) Since f’(x) = g(x), f' (x) = g' (x)
Put f' (x) = – f(x). Hence g' (x) = –f(x)
= lim [f (a ) + f (h )] [Q f (x + y ) = f ( x ) + f ( y )]
h ®0 we have h' (x) = 2f(x) f’(x) + 2g(x) g' (x)
= f (a ) + lim f (h ) = f (a ) + f (0) = f (a + 0) = f (a ) = 2[f(x) g(x) + g(x) [–f(x)]] = 2 [f(x) g(x) – f(x) g(x)] = 0
h ®0 \ h(x) = C, a constant
\ f(x) is continuous at x = a. Since x = a is any arbitrary \ h(0) = C i.e. C = 5
point, therefore f(x) is continuous for all x.
h(x) = 5 for all x. Hence h (10) = 5.
2x
5. (a) Let u = tan -1 ...... (i)
1- x2 8. (d) f(x) = [x]2 – [x2] = (–1)2 – (0)2 = 0, – 1< x < 0 Þ 0 < x 2 < 1

2x = 0 – 0 = 0, 0 £ x < 1 and = 1 – 1 = 0, 1 £ x < 3


and v = sin -1 ...... (ii)
1 + x2
and = 1 – 3 = –2, 3 £ x < 4
In equation (i) put, x = tan q
\ From above it is clear that the function is discontinuous at
é 2 tan q ù
\ u = tan -1 ê –1
ë1 -
EBD_7504
S-M-72 SOLUTIONS

0 + 1´ 0 Now, put value of y (2) and y1(2)


9. (d) f (0) = =0 2
0 +1 æ 9ö æ 1 3ö 1
Þ y2 (2) = ç ÷ ç - + log ÷ -
tan px 2 + ( x + 1) n sin x è 4ø è 3 2ø 8
lim f ( x) = lim lim
x ®0 - x ® 0- n ® ¥ x 2 + (x + 1) n 2
æ 1ö æ 3 1ö
4 ç y2 (2) + ÷ = 9 ç log – ÷
tan px 2 è 8ø è 2 3ø
= lim ( If x ® 0 - , x + 1 < 1)
x ®0 - x2 Þ Required expression = 3
=p \ LHL ¹ f (0) ¥ ¥
xn ( x log a ) n
\ f(x) is not continuous at x = 0 hence not differentiable also. 13. (d) We have, f ( x ) = å n!
(log a ) n = å n!
n =0 n =0
10. (d) For f (x) to be continuous at x = 0, we should have
x
lim f (x) = f (0) = 12(log 4)3 = e x log a = e log a = a x
¥®0

æ xö f (0 - h ) - f (0) a -h - 1
3 Lf ¢(0) = lim = lim = loge a
æ x ö çè p ÷ø
px 2 h ®0 -h h ®0 - h
lim f (x) = lim ç 4 - 1÷ ´ ·
¥®0 x®0 è x ø æ xö æ 1 2ö f (0 + h ) - f (0) a h -1
çè sin p ÷ø log çè1 + 3 x ÷ø Rf ¢(0) = lim = lim = log e a
h ®0 h h ®0 h
æ ö Since Lf ¢(0) = Rf ¢(0), \ f(x) is differentiable at x = 0
x2
= (log 4)3 · 1 · p ·
lim ç ÷ Since every differentiable function is continuous, therefore,
x®0 1 2 1
ç x - x 4 + ¼÷ f(x) is continuous at x = 0.
è3 18 ø
3
= 3p (log 4) · Hence p = 4. e1 / x - e -1 / x 1 - e -2 / x
14. (a) lim = lim =1
f (b) - f (a) x ® 0+ e1 / x + e -1 / x x ® 0+ 1 + e -2 / x
11. (c) f ¢(c) =
b-a
e1 / x - e -1 / x e2 / x - 1
3/8-0 3 and lim = lim = -1.
1/ x -1 / x
Þ 3c2 - 6c + 2 = = x ®0- e +e x ®0- e2 / x + 1
1/ 2 - 0 4
Hence lim f (x ) exists if lim g(x) = 0 .
21 21 æ 1 ö 21 x®0 x ®0
Þ c = 1± Þ c = 1+ Ï ç 0, ÷ Þ c = 1 -
è ø If g(x) = a ¹ 0 (constant) then
6 6 2 6
x lim f (x) = a and lim f (x) = - a.
æ 1ö x ®0 + x ®0 -
12. (a) Let y = ç 1 + ÷
è xø Thus lim f ( x) doesn’t exist in this case.
Taking logarithm of both sides, we get x®0

é æ 1ö ù \ lim f (x) exists in case of (b), (c) and (d) each.


log y = x êlog ç1 + ÷ ú x ®0
ë è xø û
æ x -x ö
1 x2 æ 1 ö æ 1ö 15. (a) Let f (x) = cot -1 x - x
Þ y1 ( x ) = ç - ÷ + log çè1 + ÷ø ç 2 ÷
y x + 1 è x2 ø x è ø
Take out x–x common
1 æ 1ö
= - + log ç1 + ÷
è xø ......... (1) æ x 2 x - 1ö
x +1
f (x) = cot -1 ç ÷
Since, y (2) = (1 + 1/2)2 = 9/4 è 2xx ø
æ 1 3ö Put xx = tan q
so, y1 (2) = (9/4) çè - + log ÷ø
3 2 ì tan 2 q - 1üï
Again differentiate eq (1) w.r.t (x), we get \ f (x) = cot–1 ïí –1
ý = cot (– cot 2q)
îï 2 tan q þï
y ( x ) y2 ( x ) – [ y1 ( x )]2 1 1
= –
( y ( x )) 2
(1 + x ) 2
x ( x + 1) 2 tan q
= p – cot–1 (cot 2q) [Q tan 2q = ]
By putting x = 2, we get 1 - tan 2 q
y (2) y2 (2) – ( y1 Þ f (x) = p – 2q = p – 2 tan –1 (xx)
( y (2)) 2
Mathematics S-M-73

Differentiate w.r.t. x, we get q[1 - sin{( p / 2) + h}] q (1 - cosh) q


f [( p / 2) + ] = lim = lim =
2 2 2
h ®0 [ p - 2{(p / 2) + h}] h ®0 4h 8
f '(x) = - .x x (1 + log x)
1 + x2 x 1 q 1
\ At x = 1 \p = = Þ p = , q = 4.
2 8 2
-2
f ' (1) =
(1 + 0) = -1 . 20. (d) x is non-differentiable function at
1+ 1
16. (d) f (x) = max. {x, x3} x = 0 as L.H.D = –1 and R.H. D = 1
ì x, x ³ 0
x < -1
ìx ;
ï 3
Q x = í- x, x < 0
î
ïx ; -1 £ x £ 0
=í But cos h is differentiable
ïx ; 0 £ x £1
ï x3 ; \ Any combination of two such functions will be non-
î x ³1 differentiable . Hence option (a) and (b) are ruled out.
Now, consider sin x + x
ì1 ; x < -1
ï 2 sin -h + - h
ï3 x ; -1 £ x £ 0 L ' = hlim
\ f ' (x) = í ®0 -h
ï1 ; 0 £ x £1
ï3 x 2 ; x ³1
sin h
î = lim - 1 = -1 - 1 = –2
h ®0 - h
Clearly f is not differentiable at – 1, 0 and 1.
17. (b) f is continuous at x = p / 4 , if lim f ( x ) = f (p / 4) . sin h + h
R ' = lim
x ®p / 4 h® 0 h
2
Now, L = lim (sin 2 x ) tan 2x sin h
= lim +1 = 1+1 = 2
x ®p / 4 h® 0 h
Þ log L = lim tan 2 2x log sin 2x Consider sin x - x
x ®p / 4
sin - h - - h
log sin 2 x æ ¥ ö L ' = lim
= lim ç ÷ h® 0 -h
x ®p / 4 cot 2 2 x è ¥ ø
sin h
2 cot 2x 1 = lim +1 = 0
= lim =- h® 0 –h
x ®p / 4 - 2 cot 2x cos ec 2 2x.2 2
sin h - h
\ f (p / 4) = e -1 / 2 = 1 / e R ' = lim
or L = e -1 / 2 h® 0 h
18. (a) Given f -1 ( x) = g ( x) sin h
= lim -1 = 0
h
Þ x = f [ g ( x) ]
h® 0

Diff. both side w.r.t (x) Hence, sin x - x is differentiable at x = 0.


1 (e x - 1)2
Þ 1 = f ' [ g ( x)] .g '( x) Þ g '( x ) = 21. (d) Lt
f '( g ( x)) x ®0 æxö æ xö
sin ç ÷ log ç1 + ÷
Given, f '( x ) = sin x èaø è 4ø
\ f '( g ( x)) = sin [ g ( x)]
(e x - 1)2 2
1 .x
Þ = co sec [ g ( x) ] = Lt x
f '( g ( x )) x ®0 æ xö æ xö
sin ç ÷ log ç1 + ÷
x è aø è 4ø x
Hence, g '( x ) = cosec [ g ( x )] . . .
a æ xö x 4
çè ÷ø
- 1 - sin 3 [(p / 2) - h ] a 4
19. (c) f [( p / 2) ] = lim h ®0 3 cos 2 [(p / 2) - h ] Þ 4a = 12 Þ a = 3
3
1 - cos h 1
= lim =
h ®0 2 2
3 sin h
EBD_7504
S-M-74 SOLUTIONS

But f ' (b) = eb – 8 + 2, so that e b–8 = – 2 which is not


22. (b) Let f (x) = a n x n + a n -1x n -1 + ........... + a1x = 0 possible, Hence there is no real root other than 8.
The other given equation,
é æ xö æ xö æ xö æ xö ù
na n x n -1 + (n – 1) a n -1x
n -2
+ ....+ a1 = 0 = f ¢(x) 27. (a) F¢( x) = ê f ç ÷ . f ¢ ç ÷ + g ç ÷ g ¢ ç ÷ ú
ë è 2ø è 2ø è 2ø è 2ø û
Given a1 ¹ 0 Þ f(0) = 0 Here, g (x) = f ' (x)
Again f (x) has root a, Þ f ( a) = 0 and g' (x) = f '' (x) = – f (x)

so F¢( x ) = f çæ ÷ö g çæ ÷ö - f çæ ÷ö æ xö
\ f (0) = f(a) x x x
gç ÷ =0
\ By Rolle’s theorem, è 2ø è 2 ø è 2ø è 2ø
f ¢(x) = 0 has root between ( 0, a ) Þ F (x) is constant function
so F (10) = 5
Hence f ¢( x ) has a positive root smaller than a.
f (a + h) - f (a)
23. (a) If f is continuous at x = 0, then 28. (d) (a) lim exist finitely
h ® 0+ h
lim f ( x) = lim f ( x ) = f ( 0 )
x ®0- x ®0 + \ lim f (a + h) - f (a)
h ® 0+
Þ f ( 0 ) = lim f ( x )
x ®0 - æ f (a + h) - f (a) ö
= lim ç ÷ø h = 0
p h ® 0+ è h
cos
2
[ 0 - h]
k = lim f (0 - h) = lim Þ lim f (a + h) = f (a)
h® 0 h® 0 [0 - h] h ® 0+

p p Similarly, lim f (a + h) = f (a)


cos
2
[ -h ] cos [ -h - 1]
2
h ® 0-
k = lim = lim \ f is continuous at x = a
h® 0 [ ]
- h h ® 0 [ h - 1]
-
p
(b) Function is not differentiable at 5x = (2n + 1)
æ pö 2
cos ç - ÷
è 2 ø ;k = 0 only, which are not in domain
k = lim
h ®0 -1 1 1
24. (b) The denominator of the given function is always defined (c) Let f (x) = 2
and g (x) = –
,
Also, tan [x]p = tan n p = 0 [[x] = integer, say n]
x x2
\ f(x) = 0 " x lim f (x) + g(x) exists whatever lim f (x) and
x®0 x ®0
\ f(x) is continuous and differentiable for all x.
lim g(x) does not exist.
25. (c) Put x n = cos a , y n = cos b x ®0
29. (d) Statement 1 : As f (–1) = f (1) and Rolles theorem is not
æ a + bö æ a – bö applicable, then it implies f (x) is either discontinuous
2sin ç cos ç
sin a + sin b è 2 ÷ø è 2 ÷ø or f ' (x) does not exist at atleast one point in (–1, 1)
Þ a= =
cos a – cos b æ a + bö æ a – bö Þ g (x) = 0 at atleast one value of x in (–1, 1).
–2sin ç
è 2 ÷ø
sin ç
è 2 ÷ø Statement 2 is false. Consider the example in
statement-1.
æ a – bö 30. (a) f (x) = | x | sin x
= – cot ç
è 2 ÷ø
| 0 - h | sin (0 - h) - 0
Þ 2 cot–1 (– a) = a – b L.H.D. = lim
Þ cos–1(x n) – cos–1(y n) = 2 cot–1(– a) h ®0 h
- h sin h
y n-1 dy x n -1 1– x 2 n dy x n -1 = lim =0
Þ = Þ = n -1
h®0 h
1 - y2n dx 1 - x 2n 1 – y 2 n dx y
| 0 + h | sin (0 + h) - 0
26. (b) Clearly x = 8 satisfies the given equation. Assume that R.H.D. = lim
h®0 h
f (x) = ex – 8 + 2x – 17 = 0 has a real root a other than x = 8. We
f (x) is differentiable at x = 0
may suppose that a > 8 (the case for a < 8 is exactly similar).
Applying Rolle's theorem on [8, a],
we get b Î (8, a), such that f ' (b) = 0.
Speed Test-80

1. (d) Given equation of curve : y2 = 2x3 ...(i) Z = 2x2 – 40x + 1140.


and line : 4x – 3y + 2 = 0 dZ d2 Z
dy = 4x - 40 and =4
y2 = 2x3 Þ 2y = 6x 2 dx dx 2
dx For maximum or minimum, we must have
Slope of tangent dZ
= 0 Þ 4x - 40 = 0 Þ x = 10
dy 6x 2 3x 2 dx
= = = = m1 (say)
dx 2y y Q
and 4x – 3y + 2 = 0 Þ – 3y = – 4x – 2
4 2
P
3y = 4x + 2 Þ y = x +
3 3
4 22 m
Slope of line = = m 2 (say) 16 m
3
Now, m1 × m2 = – 1
3x 2 4 A R (20 – x) m B
\ ´ = –1 x
y 3
y = – 4x2 d2 Z
Squaring on both sides, we get Clearly, = 4 > 0 for all x
dx 2
y2 = 16x4 ...(ii)
\ Z is minimum when x = 10 m
equ. (i) – equ (ii) :
4. (a) 4x3 + 4y3 (dy/dx) = 0 Þ dy/dx = –x3/y3
2x3 = 16x4 Þ x=
1
or 0 Equation of tangent, Y – y = – x 3/y3 (X–x)
8 X Y
Þ y3Y + x3X = x4 + y4 = a4 Þ 4 3 + 4 3 = 1
If x = 0 then y = 0 a /x a /y
1 1 1 1 Here, p = a4/x3, q = a4/y3
If x = then y = –4 ´ ´ = –
8 8 8 16
æ1 1ö
Þ p -4 / 3 + q - 4 / 3 =
a -16 / 3
x -4
+
a -16 / 3
y -4
(
= a -16 / 3 x 4 + y 4 )
Therefore, the points are (0, 0) and ç , – ÷ .
è 8 16 ø
= a -16 / 3 (a 4 ) = a -4 / 3
2. (d) f '(x) = 4x – 1/x
f '(x) is monotonic increasing when f '(x) > 0 3/2 -3/2 æ 1ö
Þ 4x – 1/x > 0 5. (a) f (x) = x + x -4çx+ ÷
è xø
4x 2 - 1 3
Þ >0 æ 1 ö æ 1 ö
x f (x) = ç x + ÷ - 3 çè x + ÷
è xø xø
ìï4x 2 - 1 > 0 when x > 0
Þí 2 éæ 1 ö
2 ù
ïî4x - 1 < 0 when x < 0 -4 ê ç x + - 2 ú
÷
But x > 0, 4x2 – 1> 0 Þ x2 > 1/4 Þ | x | > 1/2 êè xø ú
ë û
Þ x Î (1/2, ¥ )
and x < 0, 4x2 – 1< 0 Þ x2 < 1/4 Þ | x | < 1/2 1
Let x+ = t (x > 0)
Þ x Î (–1/2, 0 ) x
\ x Î (–1/2, 0) È (1/2, ¥) Let g (t) = t3 – 3t – 4t2 + 8
3. (c) Let R be a point on AB such that AR = x m. Then, g (t) = t3 – 4t2 – 3t + 8
RB = (20 – x)m g' (t) = 3t2 – 8t – 3 = (t – 3) (3t + 1)
In D’s RAP and RBQ, we have g' (t) = 0 Þ t = 3 (t ¹ –1/3)
PR2 = x2 + 162 ... (i) g'' (t) = 6t – 8
RQ2 = 222 + (20 – x)2 g'' (3) = 10 > 0 Þ g (3) is minimum
\ PR2 + RQ2 = x2 + 162 + 222 + (20 – x)2 g (3) = 27 – 9 – 36 + 8 = – 10
= 2x2 – 40x + 1140
Let Z = PR2 + RQ2
EBD_7504
S-M-76 SOLUTIONS

6. (c) Given, 6y = x3 + 2
On differentiating w.r.t. t, we get (x + 2)2 + 1 Nr
= as is + ive
dy dx dx dx (10) Dr
6 = 3x 2 Þ 6´8 = 3x 2
dt dt dt dt dD 2(x + 2)
Þ 3x2 = 48 Þ x2 = 16 = = 0 \x = – 2
dx (10)
Þ x = ±4
When x = 4, then 6y = (4)3 + 2 and hence y is – 8 i.e. point is (–2, – 8)

66 d2D 2
Þ 6y = 64 + 2 Þ y = = 11 = =+ ive and hence min. at (–2, –8)
2 (10)
6 dx
When x = – 4, then 6y = (– 4)3 + 2 9. (b) f(x) = (a2 – 3a + 2) (cos2x/4 – sin2x/4)
Þ 6y = – 64 + 2 + (a – 1) x + sin 1
-62 -31 Þ f(x) = (a –1) (a –2) cos x/2 + (a –1) x +sin1
Þ y= =
6 3 1 x
Þ f ¢ (x) = – (a –1) (a – 2) sin + (a –1)
Hence, the required points on the curve are (4, 11) and 2 2
æ -31 ö é ( a - 2) xù
ç -4, ÷ Þ f ' ( x ) = (a - 1) ê1 - sin ú
è 3 ø ë 2 2û
7. (c) (a) x y2 = 16a4
2
If f(x) does not possess critical points, then
y f '(x) ¹ 0 for any x Î R
LST = 2
x T Þ xy = 4a é ( a - 2) xù
Þ (a - 1) ê1 - sin ú ¹ 0 for any x Î R
y + xy' = 0 ë 2 2û

-y y æa -2ö x
y¢ = ; LST = y / x = x Þ LST = 2a Þa ¹ 1 and 1 - ç ÷ sin = 0
x è 2 ø 2
\ (a) is true. must not have any solution in R.
dy x x 2
(b) = = 1 i.e. x = 2 Þ a ¹1 and sin = is not solvable in R
dx 2 2 a -2
\ (2, 1) is the point on the curve x2 = 4y at
which the normal is Þ a ¹ 1 and
2
>1 é For a < 2, f(x) < x ∗ sin1ù
ê ú
y – 1 = – 1 (x – 2) i.e. x + y = 3 \ (b) is true a -2 ê [ f '(x) < 1 ¹ 0 ú
ë û
(c) y = – 4x2, y = e–x/2 Þ a ¹1 and | a – 2| < 2
The curves are non-intersecting Þ a ¹ 1 and –2 < a –2 < 2
\ curves are not orthogonal i.e. (c) is false. Þ a ¹1 and 0 < a < 4 Þ a Î (0, 1) È (1, 4).
2 3 10. (c) We have f(x) = x3 + ax2 + bx + 5 sin 2 x
(d) y= x – 2ax2 + 2x + 5
3 Þ f ' ( x) = 3x 2 + 2ax + b + 5 sin 2x
dy Q f(x) is an increasing function
= 2x2 – 4ax + 2 = 2 (x2 – 2ax + 1)
dx \ f ' ( x ) > 0 Þ 3x 2 + 2ax + b + 5 sin 2x > 0,
= 2 (x – a)2 + 2 – 2a2 > 0 (Q sin 2x < 1)
[Q a Î (–1, 0) Þ 0 < a2 < 1]
\ (d) is true \ 0 < 3x 2 + 2ax + b + 5 sin 2x < 3x 2 + 2ax + b + 5
8. (a) Let (x, y) be the one point of parabola, y = x 2 + 7x + 2
its distance from the line Þ 3x 2 + 2ax + b + 5 > 0
y = 3x – 3 or 3x – y – 3 = 0 is
Þ 4a 2 + 4.3(b + 5) < 0 Þ a 2 + 3b - 15 < 0
2
3x - y - 3 3x - (x + 7x + 2) - 3
D= = [ Q ax 2 + bx + c > 0 for all real x if . a > 0 and
(10) (10) discriminant < 0].
11. (d) We have equation of tangent to any curve f(x) at
- x 2 - 4x - 5 (x1, y1) is
=
(10) dy
(y - y1 ) = (x - x1 )
2 2 dx (x1 , y1 )
x + 4x + 5 (x + 2) + 1
D= = -|x |
(10) (10) Given curve is y = e
Mathematics S-M-77

æ 1ö dA da
ç 1, e ÷ at x = 1, | x | = x = 2a ...(i)
è ø dt dt
when area A is 400 cm2 then a = 20
dy
So, y = e- x Þ = -e - x dA 0.5
dx \ = 2 ´ 20 ´ = 10 2 cm 2 / sec
dt 2
æ dy ö
\ ç ÷ = -e -1 x2 y 2 2 x 2 y dy
è dx øx =1 16. (b) + =1 Þ + . =0
a 4 a 4 dx
Therefore, equation of tangent is
dy - 4 x
1 -1 Þ = ...(i)
y- = (x - 1) Þ x + ey = 2 dx ay
e e
12. (c) The waves move in a circle at a speed of dy dy 16
y3 = 16x Þ 3 y 2 . = 16 Þ = ...(ii)
dr dx dx 3 y 2
v = 3.5 cm/sec =
dt Since curves intersects at right angles
where r is the instantaneous radius of circle.
Let A be the area of circular wave at time t - 4 x 16
\ ´ = -1
Then A = pr2 ay 3 y 2
On differentiating both sides w.r.t. 't', we get Þ 3ay3 = 64x
é dA ù dr 64 x 4
ê dt ú = 2p r dt Þ a= =
ë ûr 3 ´ 16 x 3
17. (b) f(x) = sin 2x – x Þ f ¢(x) = 2 cos 2x – 1
é dA ù Therefore, f ¢(x) = 0
ê dt ú = 2p´ 7.5´ 3.5 = 52.5 p
ë û 7.5 1
Þ cos 2x =
\ Rate of increasing area = 52.5p cm2/sec. 2
13. (d) The point (a, b) lies on both the straight line and the p p p p
Þ 2x = or - Þ x = - or
n n 3 3 6 6
æxö æyö
given curve ç ÷ + ç ÷ = 2. æ pö p p
èaø èbø Þ f ç - ÷ = sin ( -p) + =
Differentiating the equation, we get è 2ø 2 2

dy x n -1 bn æ pö æ 2p ö p 3 p
Þ f ç - ÷ = sin ç - ÷+ = - +
= – n . n -1 è 6ø è 6 ø 6 2 6
dx a y
æ dy ö b x y æpö æ 2p ö p 3 p
Þ f ç ÷ = sin ç ÷ - = -
\ç ÷ = – = the slope of + = 2 è6ø è 6 ø 6 2 6
è dx øat (a, b) a a b
æpö p p
Hence, it touches the curve at (a, b) whatever may be Þ f ç ÷ = sin ( p ) - = -
the value of n. è2ø 2 2
14. (a) f ¢ (x) = – 12 cos3 x sin x – 30 cos2x sin x – 12 cos x sin p p
x = – 6 sin x cos x (cos x + 2) (2 cos x + 1) Clearly, is the greatest value and - is the least.
2 2
p 2p p p
f ' (x) = 0, for x = 0, , ,p Therefore, difference = + = p
2 3 2 2
18. (b) We have, xy = c2
p 2p
Clearly, f ' (x) > 0 for <x< dy dy y
2 3 Þx + y. 1= 0 Þ =– ;
dx dx x
p 2p
And f ' (x) < 0 ; for 0 < x < or <x<p dy ù 1
2 3 =-
\ dx úû(ct t12
15. (b) Diagonal D = 2.a 1, c/t1 )
Differentiating w.r.t. t
æ cö
dD da The equation of the normal at ç ct1, t ÷ is
= 2 è 1ø
dt at
c
da 1 da 1 y– = t12 (x – ct1).
or = = ´ 0.5 cm / s t1
dt 2 dt 2
Let Area is denoted
EBD_7504
S-M-78 SOLUTIONS

Now, x = 0 Þ y= 1 + a
æ cö
since, this normal passes through ç ct 2 , t ÷ therefore. l l (1 + a )
è 2ø y= 0 Þ x = +l =
a a
c c
- = t12 (ct2 – ct1) 1 (1 + a )l l
t 2 t1 Area, A = ´ (1 + a ) = (1 + a ) 2
2 a 2a
Þ t13 t2 = – 1 (as t1 – t2 ¹ 0)
19. (d) Let (x, y) be the point on the curve dA l é a.2(1 + a ) - (1 + a ) 2 ù
2x2 + y2 –2x = 0. Then its distance from (a, 0) is given by For constant area, = ê ú=0
da 2 êë a2 úû
S = {( x - a ) 2 + y 2 } … (i) Þ (a – 1) (a + 1) = 0 Þ a = 1, a = –1
Þ S = x – 2ax + a + 2x – 2x [Using 2x + y – 2x = 0]
2 2 2 2 2 2
a2 b2
dS 22. (c) Given , y = +
Þ S2 = –x2 + 2x(1–a) + a2 Þ 2S = -2 x + 2(1 - a ) x a-x
dx
dy a2 b2 a2
For S to be maximum, Þ =- + =0 Þ x= ,
dx x 2 (a - x ) 2 a±b
dS
= 0 Þ –2x + 2 (1–a) = 0 Þ x = 1–a
dx a2
out of which only one x = is in (0, a).
d 2S a+b
It can easily checked that < 0 for x = 1 – a.
dx 2 d2 y 2a 2 2b 2
Also = +
> 0 in (0, a)
Hence, S is maximum for x = 1 – a. Putting x = 1– a in (i). dx 2 x 3 (a - x )3
We get, S = (1 - 2a + 2a 2 ) \ Minimum value attained is
20. (a) a+b 2 1
a +b+ b = ( a + b) 2
L ab a
P 23. (d) Let y = mlogx + nx2 + x
dy m
= + 2nx + 1
O N S dx x
dy
At x = 2, =0
P¢ dx

m
Let P be ( 9t2,
18t) then ON = 9t2 \ + 2n (2) + 1 = 0
2
Area of trapezium
dy
1 1 At x = 1, =0
A= (PP '+ LL ')NS = (36t + 36)(9 - 9t 2 ) dx
2 2
m + 2n + 1 = 0
= 162 (1 + t 2 )(1- t) Thus, we have
m + 8n + 2 = 0
dA \ 6n + 1 = 0
= 162[ 2(1 + t )(1 - t ) - (1 + t ) 2 ] = 162(1 + t )(1 - 3t )
dt
6n + 1 = 0 ü 1
1 \ ý Þn=-
A is maximum when t = 3m + 2 = 0 þ 6
3
2
2 m=-
æ1ö 3
\ For maximum area ON = 9ç ÷ = 1
è3ø -4 5
Hence, 2m + 10n = - =–3
21. (b) Given curve is xay = la ....(1) 3 3
(l, 1) is a point on the given curve.
Differentiating eq. (1) w.r.t. x, we get sin 2 x ìï (sin x + cos) 2 - 1 üï
24. (a) Let f ( x ) = = 2í ý
æ pö ïî sin x + cos x ïþ
dy sin ç x + ÷
ax a -1 y +x a =0 è 4ø
dx
æ y2 -1 ö
dy - ax a -1 y ay dy a = 2ç ÷, where y = sin x + cos x
Þ = =- at (l,1) =- ç y ÷
dx x a x dx l è ø

Eq. of tangent at (l
Mathematics S-M-79

Since the curves are orthogonal, m1m2 = –1


æ y2 -1 ö
Let f( y) = 2 ç ÷, and g(x) = sin x + cos x
æ - ax ö æ a 1x ö
ç y ÷ \ çç ÷÷ çç - ÷ = -1 Þ aa1x 2 = - bb1 y 2
è ø ÷ ....(3)
è by øè 1 ø b y
We have, g’(x) = cos x – sin x.
For max or min. g’(x) = 0 Þ tan x = 1 Solving (1) and (2) we get (a - a1 ) x 2 = (b1 - b) y 2 ....(4)
Þ x = p /4. For this value of x.
g’’(x) < 0. Thus, g(x) is max. at x = p/4 and hence the a - a 1 b - b1
Dividing by (3), =
domain of g(x) is [1, 2 ] i.e. y lies between 1 and 2 aa1 bb1
æ 1 ö 28. (b) f ' ( x ) > 0 if x ³ 0 and g' ( x) < 0 if x ³ 0
Now, f' ( y) = 2 çç1 + 2 ÷÷ > 0 for all yÎ[1, 2 ].
Let h (x ) = f (g( x))
è y ø
then h ' (x ) = f ' (g (x )).g' ( x) < 0 if x ³ 0
That is f (y) is increasing for all y Î [1, 2 ]
\ h(x) is decreasing function
Thus it attains the greatest value at 2 and is equal \ h (x ) £ h (0) if x ³ 0
æ ( 2 ) -1 ö2 \ f (g(x )) £ f (g(0)) = 0
to 2 ç ÷ =1
But codomain of each function is [0, ¥ )
ç 2 ÷
è ø
\ f (g(x )) = 0 for all x ³ 0
Hence, greatest value of f(x) on [0, p/2] = greatest value
\ f (g(x )) = 0
of f(y) on [1, 2 ] = 1.
Also g(f (x)) £ g(f (0)) [as above]
æ a 2 -1 ö 2
25. (c) f ' ( x ) = 3çç 2 ÷÷ x - 3 29. (d) Let the point on the parabola be (t, t2).
è a + 1ø Let d be the distance between (t, t2) and (0, y0), then
d2 = t2 + (t2 – y0)2 = t4 + (1 – 2y0) t2 + y02
f ' ( x ) < 0 for all x if a 2 - 1 £ 0 Þ -1 £ a £ 1
= z2 + (1 – 2y0) z + y02, z ³ 0
x cos q 1
26. (b) + y sin q = 1 . its vertex is at z = y0 –< 0.
3 3 2
the minimum value of d2 is at z = 0
Sum of intercepts 3 3 sec q + cos ecq = f (q) (say)
i.e. t2 = 0
\ d = y0
3 3 sin3 q - cos3 q
f ¢ (q) = \ Statement 1 is true. Statement 2 is false because
sin 2 q cos 2 q extremum can occur at a point where f ' (x) does not
Þ At q = p / 6, f (q ) is minimum. exist.
27. (a) The given curves are ax2 + by2 = 1 ....(1) 30. (b) f ¢(x) = ln (x + 1 + x 2 ) = – ln ( 1 + x 2 - x )
and a1x2 + b1y2 = 1 ....(2)
Þ f ¢ (x) > 0
dy dy ax Þ f(x) is increasing when x > 0.
from (1) 2ax + 2by =0Þ =- = m1 (say)
dx dx by Þ f(x) > f(0) Þ f(x) > 0.
Again f(x) is decreasing in (-¥, 0)
dy dy a x Þ f(x) > f(0) Þ f(x) > 0.
from (2) 2a1x + 2b1 y =0Þ = - 1 = m 2 (say)
dx dx b1 y

Speed Test-81

sin 8 x - cos 8 x 1.(sin 2 x - cos 2 x ) [(sin 2 x + cos 2 x ) 2


1. (b) I = ò 1 - 2 sin 2 x cos 2 x dx - 2 sin 2 x cos 2 x ]
=
ò 1 - 2 sin 2 x cos 2 x
(sin 4 x - cos 4 x )(sin 4 x + cos 4 x )
= ò 1 - 2 sin 2 x cos 2 x
dx
(sin 2 x - cos 2 x ) (1 - 2 sin 2 x cos 2 x )
2 2 2 2
= ò 1 - 2 sin 2 x cos 2 x
dx
(sin x - cos x )(sin x + cos x )
1
(sin 4 x + cos 4 x ) = - ò cos 2 x dx = - sin 2 x + C
= ò 1- 2
dx
EBD_7504
S-M-80 SOLUTIONS

p p sin x
d
2 2 f ¢( x) = ò cos(t 3 )dt
n -1 dx
ò sin ò sin
n
2. (c) In = x dx = x sin x dx 2x
0 0 = f (sinx) . cosx – f (2x) × 2
= cos(sin3x) . cosx – cos(2x)3 × 2
p
p = cos(sin3x) . cosx – 2cos(8x3)
2
6. (d) Put x = 2a – t
é - sin n -1 x cos x + (n - 1)sin n - 2 x cos 2 x dx
ù2
ë û0 ò so that dx = – dt
0 when x = a, t = a and when x = 2a, t = 0
2 a a
p
2
ò0 f (x) dx = ò f (x)dx + ò f (2a - t) dt = n + m
0 0
7. (d) General term of the series
= (n – 1) ò sin n - 2 x (1 - sin 2 x ) dx
10 -2 n
0

p p
å ò sin 27 xdx is
n =1 - 2 n -1
2 2
n -2
= (n – 1) ò sin ò
n -2 n 2 n +1
x dx - (n - 1) sin x dx 2n

ò sin 27 xdx = ò sin 27 ( - x )( -dx ) = - ò sin


27
0 0
I1 = x dx = – I
2
\ In = (n – 1) In – 2 – (n – 1) In - 2 n -1 2 n +1 2n

n -1 10 2 n +1
Þ n In = (n – 1) In – 2 Þ In =
n
In – 2
where I2 is general term of series å ò sin 27 xdx
Þ n (In – 2 – In) = In – 2 n =12n
Also, In : In – 2 = (n – 1) : n and In – 2 > In. So I1 + I2 = 0 for all n

(
x + cos -1 3x ) 2
æ xö
ln x = ln ç ÷ + ln 2
3. (a) I = ò 1 - 9x 2
dx 8. (d)
è 2ø
tan(ln x / 2) + tan (ln 2)
Put 3x = cos q Þ 3dx = - sin q dq Þ tan (ln x) = 1 - tan (ln x / 2) tan (ln 2)
cos q
+ q2
Þ tan (ln x) tan æç ln ö÷ tan (ln 2) = tan (ln x) – tan æ xö
1 3 x
I= -
3 ò sin q
sin q dq
è 2ø çè ln 2 ÷ø
– tan (ln 2)
1 é1 2ù 1 q3
=-
3 ò êë 3 cos q + q ú dq = - sin q -
û 9 9
+c
\ I=ò
tan (ln x)
dx - ò
tan (ln x / 2)
dx - ò
tan (ln 2)
dx
x x x
=-
1
9
1
1 - 9 x 2 - cos -1 3 x
9
( ) +c
3
æ xö
= ln sec (ln x) – ln sec ç ln ÷ – tan (ln 2) ln x
è 2ø
1
\ A = B= - ìï sec (ln x) üï
9 +C
= ln í tan ln 2 ý
4. (a) Let ïî sec (ln(x / 2)x ïþ
4 4
d esin x 3 sin x 3 3x 2 sin x 3
dx
F(x) =
x
Now ò x
e dx =
x3
eò dx 9. (c) Let I = ò
5 + x2
x4
dx
1 1
put x = 5 tan q
Let x 3 = t, 3x 2 dx = dt
when x = 1, t = 1& x = 4, t = 64 5 + x2
Þò dx
64 sin t x4
e
ò dt = [ f (t) ]1 = F(64) - F(1)
64
I=
t 5 + 5 tan 2 q
1 = ò( 5.sec 2 q d q
5 tan q )
4
K = 64.
sin x 5(1 + tan 2 q)
5. (a) f ( x) = ò cos(t )dt3 = ò (5) 2 tan 4 q
5.sec 2 q d q
2x
Mathematics S-M-81

p
5sec2 q
= ò 25tan 4 q
5 sec 2 q d q 2
sin {nx + (n - 1) x} sin {nx - (n - 1) x}
= ò sin x
dx
5 sec q 5 sec 2 q d q 0
= ò 25 tan 4 q
[Using identity sin2A – sin2B = sin (A+B) sin (A – B)]
p p
1 cos q
5 ò sin 4 q sin (2nx - x ) sin x
= dq 2 2

Put sin q = t Þ cos q dq = dt


= ò sin x
dx = ò sin(2n - 1) x dx
0 0
1 dt
= ò 4 p é p ù
5 t é cos(2n - 1)x ù 2 ê cos(2n - 1) 2 cos 0 ú
-1 1 -1é 1 ù = -ê ú = -ê - ú
+C = +C ë 2n - 1 û 0 ê 2n - 1 2n - 1ú
=
15 t 3 15 êë sin 3 q úû ë û
-1 1
= éë cosec2 q. cosec q ùû + C =
15 2n - 1
-1 é(
ë 1 + cot q ) . 1 + cot q ùû + C
2 2 1 1 1
= \ a 2 - a 1 = , a 3 - a 2 = , a 4 - a 3 = ..........etc.
15 3 5 7

-1 é1 + tan 2 q 1 + tan 2 q ù \ a 2 - a 1 , a 3 - a 2 , a 4 - a 3 ......... form an H.P..


= ê ´ ú+C
15 êë tan 2 q tan 2 q úû ì 1 + nx n -1 - x 2n ü
ï ï
3/ 2 é x2 ù
3/ 2
12. (d) ò ex í
ïî (1 - x n ) 1 - x 2n
ýdx
ïþ
1 é1 + tan q ù 2
1 ê1 + 5 ú
=- ê ú +C= - +C
15 ë tan 2 q û 15 ê x 2 ú é 1 - x 2n ù
ëê 5 ûú nx n -1 1 - x 2n
= ò ex ê n
+ ú = ex
n
+C
ê ú
3 ë 1- x (1 - x n ) 1 - x 2n û 1 - x
-1 é 5ù 2
= ê1+ 2 ú +C ¥ ¥ ¥
15 ë x û é e -ax ù n -1 e
-ax

1 l n3 2 x sin x 2
13. ò
(c) Let I n = x n e - ax = ê x n ×
êë
ú - nx
- a úû
×
-a
dx ò
10. (a) I =
2 ò ln 2
sin x 2 + sin(ln6 - x 2 )
dx 0 0 0

1 xn n
Let x2 = t Þ 2x dx = dt =- lim + I n -1
a x ®¥ e ax a
Also, when x = ln 2 , t = ln2
é xn ù
when x = ln3 , t = ln3 n
\ I n = I n -1 êQ lim ax = 0ú
1 l n3 sin t dt
a ëê x®¥ e úû
\I=
2 ò ln 2 sin t + sin(ln 6 - t ) ...(1)
=
n n -1
× I n -2 =
n (n - 1)(n - 2)
I n -3
b b a a a3
Using ò a f (x) dx = ò a f (a + b - x)dx ..............................................................
..............................................................
We get
¥
1 ln3 sin(ln6 - t ) n! n!
òe
- ax
= dx =
I= ò
2 ln 2 sin t + sin(ln6 - t )
dt ...(2)
a n
0
a n +1

Adding values of I in equations (1) and (2) 1 1 1


ln 3 We have, K = sin x dx < x dx =
2I=
1
òln2
1 1 3
1 dt = (ln3 - ln 2) = ln
14. (a)
ò x ò x ò x dx
2 2 2 2 0 0 0

1 3 2 3/ 2 1 2 2
ÞI= ln = [x ]0 = Þ K <
4 2 3 3 3
p 1 1
cos x 1
2 2
sin nx - sin (n - 1) x 2 Now, J = ò dx < ò dx = [2 x ]10 = 2
11. (c) a n - a n -1 = ò dx x 0
x
0
EBD_7504
S-M-82 SOLUTIONS

2 2 3 3
15. (a) We have, if e x > 2, < 1 . Also x > 0 Þ e x + 2 = 3y Þ e x = 3 y - 2
x
e e
Þ x 3 = log e (3 y - 2) Þ x = 3 log e (3y - 2)
2 é 2ù
Þ0< <1 \ If x > log e 2, ê ú = 0
ex ë ex û p
18. (b) We have cos x ³ sin x for 0 £ x £
4
Again if 0 < x < log e 2 then 1 < e x < 2
p p
1 1
2 2 and sin x ³ cos x for £x£
Þ 1 > x > Þ 2 > x > 1 or 1 < x < 2 4 2
e 2 e e p
é 2 ù \ ò 2 | sin x - cos x | dx
\ê ú =1 0
ëex û p p

é 2ù
¥ ¥ = ò 0
4
ò
(cos x - sin x ) dx + p2 (sin x - cos x ) dx
\ I = ò ê x ú dx = ò é2e - x ù dx
ë û 4
0 ëe û 0 p p
log 2 ¥ = [sin x + cos x ] 4
0 + [- cos x - sin x ]p2
é 2e - x ù dx + é2e - x ù dx
= ò ë û ò ë û 4
0 log 2
é 1 1 ù é 1 1 ù
log 2 ¥ =ê + - 0 - 1ú - ê0 + 1 - - ú
ë 2 2 û ë 2 2û
= ò (1) dx + ò (0) dx = log e 2
0 log 2 = 2 -1-1+ 2 = 2 2 - 2
x 2 k
x2
16. (c) ò g (t) dt =
2
2 ò
+ t 2 g(t) dt
x
19. (c) I1 = ò xf {x(1 - x)}dx
1-k
Differentiating w.r.t. x, we get
k
g (x) = x + (–x2 (g (x))
x
= ò (k + 1 - k - x)f [(k + 1 - k - x){1 - (k + 1 - k - x)}]dx
Þ g (x) = 1-k
1 + x2 [See Property No. 7]
Y k k k
= ò (1 - x ) f {(1 - x ) x} dx = ò f {x (1 - x)} dx - ò xf {x(1 - x )} dx
1- k 1- k 1- k
(1, 1/2)
I1 1
Þ I 1 = I 2 - I 1 Þ 2I 1 = I 2 Þ =
I2 2
X
20. (a) Put x = tan q Þ dx = sec 2 q d q

q 1 + tan q + tan 2 q
(–1, –1/2) I= òe
1 + tan q 2
. sec 2 q dq
q
æ 1 1ö
= e ò
(tan q + sec 2 q) dq
Clearly from graph, a Î ç - , ÷ - {0} -1
è 2 2ø = eq tan q + c = xe tan x
+C
3 a a
17. (b) y = ò x 2 . e x dx ò ò
21. (a) I = 0 f ( x) g(x) dx = 0 f (a - x ) g(a - x ) dx

Put x 3 = t Þ 3 x 2 dx = dt a

dt 1 t 1 x3
= ò0 f (x)[2 - g(x)] dx [From given conditions]

ò
t
y = e . = e +C= e +C
3 3 3 a
It passes through (0, 1) then
=2 ò0 f (x) dx - I
1 0 1 2 a a
e + CÞ C = 1 - =
1=
3 3 3
\2 I = 2 ò0 f ( x ) dx Þ I = ò0 f (x)dx
1 x3 2
\y = e +
3 3
Mathematics S-M-83

d n 1
22. (a) Let 6 x + 7 = l ( x - 5)( x - 4) + m 1 1 dx
dx = lim
n ®¥ n
å 2
= ò 2+x
i.e. 6 x + 7 = l ( 2 x - 9) + m which gives l = 3 and m = 34 n =1 2 + 0
n
6x + 7 3(2 x - 9) + 34

( x - 5)( x - 4)
dx = ò x 2 - 9 x + 20
dx
[ ]
= log( 2 + x )]10 = log 3 - log 2 = log
3
2
1
- dx f (x)
= 3 ò ( 2 x - 9 )( x 2 - 9 x + 20 ) 2 dx + 34 ò 2
x - 9 x + 20 ò 2t dt
(t 2 )f4(x) (f (x)) 2 - 42
lim 4 = lim = lim
1 25. (a)
- +1 x ®1 x -1 x ®1 x -1 x ®1 x -1
(x 2 - 9x + 20 ) 2 dx
= 3ò + 34 ò 2f (x).f ¢(x)
1 81 1
- +1 x 2 - 9x + - = lim (L-Hospital)
2 4 4 x ®1 1
= 2f (1). f ' (1) = 2 × 4 × 2 = 16
dx 26. (c) Integrating by parts.
= 6 x 2 - 9x + 20 + 34
ò
æ 9ö æ1ö
çx - ÷ -ç ÷
2 2
ò f (x )g"(x ) dx - ò f "(x ) g(x ) dx
è 2ø è2ø
= f ( x )g ' ( x ) - ò f ' ( x ) g ' ( x ) dx - f ' ( x ) g ( x ) + ò f ' ( x ) g ' ( x )dx

= 6 x 2 - 9x + 20 = f ( x ) g' ( x ) - f ' ( x ) g( x )
1 1
ì
ï 9 æ
2
9ö æ1ö
+34 log í x - + ç x - ÷ - ç ÷
2 ü
ï
ý+C
Hence, ò0 f (x ) g" ( x) dx - ò0 f " (x )g ( x ) dx
ïî 2 è 2ø è2ø ïþ = f (1)g ' (1) - f ' (1)g(1) - f (0)g' (0) + f ' (0)g(0)
9 = f (1)g' (1) - f ' (1)g(1)
2
=6 x 2 - 9 x + 20 + 34 log | x + x - 9 x + 20 - | +C 27. (c) F ¢(x) = f(x)
2
\ A = 6, B = 34.
23. (b) Three cases may arise
Also, F(t) = t 1 + t ( )
|x| x 3
Case (i) : let 0 £ a < b, then = =1 Þ F¢(t) = 1 + t1/2; F¢(4) = 1 + 3 = 4 Þ f(4) = 4
x x 2
28. (c) We know, ex – [x] is periodic function with period 1.
|x| b b
\ ò ò
dx = dx = b - a =| b | - | a | 1000
a x a
(Since |a| = a and |b| = b)
\ I= ò0 e x -[x]dx

| x | -x 1000 1000
Case (ii) : Let a < b £ 0, then = = -1
ò ò
x -[x]
x x e dx = e{x}dx
b |x| b 0 0
\ òa x
dx = ò
a
(-1)dx = a - b = - | a | + | b |
1 2 3 1000
(Since |a| = –a and |b| = –b)
Case (iii) : Let a< 0<b, then ò x
= e dx + e dx + ò x
ò e dx + ... + ò
x
e x dx
0 1 2 999
b |x| 0 |x| b |x|
òa dx = òa dx + ò0 dx 1 1 1

0
x
b
x x
ò ò
= e x dx + e x dx + e x dx + ...1000 terms ò
òa (-1) dx = ò0
0 0 0
= (1) dx = a + b = - | a | + | b |
1
(Since |a| = –a and |b| = b)
b |x|
ò
= 1000 e x dx = 1000 x 1
[e ]0 = 1000[e1 - e0 ]
Thus, in every case òa x
dx =| b | - | a |
= 1000 ( e – 1).
0

24. (c)
é 1
lim ê +
1
+¼+
1 ù 29. (b) ò g(x) (f (x) + f '(x)) dx
n ®¥ ë 2n + 1 2n + 2 2n + n úû
= ò g(x) f (x) dx + ò g(x) f '(x) dx
1é 1 1 1 ù
= lim + +¼+ = f (x) ò g(x) dx - ò (f '(x) ò g(x) dx) dx + ò g(x) f '(x)dx
n ®¥ n
ê 1 2 nú
ê2+ 2+
ë n n
EBD_7504
S-M-84 SOLUTIONS

= f (x)g(x) - ò f '(x)g(x) dx + ò g(x)f '(x) dx 10 1 1


æ x2 ö
1

(Q ò g(x) dx = g(x))
30. (b) ò ò
x - [ x ] dx = 10 x - [ x ] dx = 10 x dx = 10ç
ò
ç 2 ÷
è
÷ =5
øa
0 0 0
= f(x) g(x) + C na a
Now,
ò ò
f (x ) dx = n f (x )dx
ò g(x) (f (x) - f ''(x)) dx 0 0
If ‘a’ is period of f. Both Statement 1 and Statement 2
= ò g(x){(f (x) + f '(x)) - (f '(x) + f ''(x))}dx
are true and Statement 2 is correct reason of
= g(x) f(x) – g(x) f '(x) + C Statement 1.

Speed Test-82

a
a2 a p
1. (b) We have, ò f (x) dx = + sin a + cos a
2 2 2 Area of the bounded region OABCO
0 1
Differentiating w.r.t. a, we get = 2 × Area OBCO = 2x ò é(2 - x 2 ) - x ù dx
ë û
1 p 0
f(a) = a + (sin a + a cos a) – sin a
2 2 [Since y = 2 –x2 is the upper curve and y = x is the lower
curve]
p æç p ö÷ p 1 p 1
Put a = ;f = + – = 1
2 è ø2 2 2 2 2 é x3 x 2 ù é 1 1ù 7
2. (b) = 2ê2x - - ú = 2 ê2 - - ú =
êë 3 2 úû ë 3 2û 3
y 0
3. (c) The area of the region bounded by the curve y = f (x)
y = –x B (0, 2) y=x and the ordinates x = a, x = b is given by
b
(–1, 1) A C (1, 1) Area = òa y dx
x According to the question,
O
ïì x , x ³ 0
2
y=x x =í
We first draw the given curves 2
ïî - x , x < 0
The first curve x 2 - y 2 = 0 Þ y = ± x represents a pair of
Y
straight lines with slopes 1 and –1 passing through origin.
The second curve y = x2
Þ x 2 + y - 2 = 0 Þ x 2 = -y + 2 Þ x 2 = -( y - 2)
B
represents a parabola with vertex (0,2) axis as y–axis and x=1
C(1, 0)
concavity dawnwards (see the chapter of parabola in X
coordinates). Both the curves are plotted in the figure and x = –1 O A(1, 0)
the required area is shown by the shaded region. D
The points A and C are the points of intersection of y 2 = x 2
y = –x2
with x 2 + y - 2 = 0 .
Required area
Solving the two equations, we get y 2 + y - 2 = 0 [putting = area of region OAB + area of region OCD
value of x2 = y2] = 2 × Area of region OAB
Þ ( y + 2)( y - 1) = 0 1 2
giving y = –2 and 1, but y = –2 is discarded as the required = 2ò x 2dx = sq. units
0 3
area is above the x-axis. 4. (a) The required region is the intersection of the following
\y = 1 Þ x = ± 1 regions
The points A and C are respectively (–1, 1) and (1, 1) now } . It represents the region below
due to symmetry
Mathematics S-M-85

A 2 = {( x, y) : 0 £ y £ x + 1} . It represents the region below 4 ,4


the straight line y = x+1, and a
A 3 = {( x , y) : 0 £ x £ 2} . It represents the region lying
between the ordinates x = 0 and x = 2.
The required area is the region shown as the shaded region y = ax
in the following figure : 2
y = 4ax
Y
C(2, 3) 4
2 é y y2 ù 1
ò
y=x+1
Given ê - údy =
B(1, 2) ê a 4a úû

3

8 1 1 8 1
A (0, 1)
Þ - ´ 64 = Þ = Þa=8
a 12a 3 3a 3
y=x+1
So, the parabola is y 2 = 32x
X
O N
(2, 8)

By solving y =x +1 and y = x2+1, we get the points of


intersection A(0,1) and B (1,2).
Hence the required region is bounded by
y = f(x), y = 0 x = 0, x = 2, y = 4x
ìïx 2 + 1, 0 £ x £ 1 y2 = 32x
Where f(x) = í
ïî x + 1, 1 £ x £ 2 Area enclosed by y = 4x is
\ Required area 8
é y y2 ù é y 2 y3 ù
8

2 1 2 òê - ú dy = ê - ú = 8
ò0 f (x)dx = ò0 (x ò1 (x + 1) dx
2
A= + 1)dx + ê 4 32 úû
0ë êë 8 96 úû 0 3

1 2 p 5p
é x3 ù éx2 ù 23 8. (b) Point of intersection of y = sin x and y = cos x are , .
= ê 3 + xú + ê 2 + xú = 6 4 4
êë úû 0 êë úû1
X
4
y p/4 y = sin x
5. (d) Required area = ò 9
dy y = cos x
y =1
3p/2
4 Y
1
4
1 2 p/2
1/2
= ò y dy = ´ ( y 3/2 )
3 3 3
y =1 1
2 1/ 2 3 2 2 14
= [(4 ) - (11/ 2 )3 ] = [8 - 1] = ´ 7 = sq. units.
9 9 9 9 é p 5p ù
Since, sin x ³ cos x on the interval ê , ú
b ë4 4 û
6. (d) Given ò f (x)dx = b2 +1 - 2
\ Area of one such region = ò
5p / 4
(sin x – cos x ) dx
1 p/4
Differentiate with respect to b
= 2 2 sq. unit.
b x
f (b) = Þ f ( x) = 1
9. (b) Area of DAOB = ´ base ´ height
b2 + 1 x2 +1 2
7. (d) Point of intersection of y 2 = 4ax 1
= ´ 2a ´ a 2 = a 3 units
2
æ4 ö Area of region AOB
and y = ax are (0, 0) and ç , 4 ÷
èa ø
EBD_7504
S-M-86 SOLUTIONS

Y a 8b
é b - a 3 ù b-a
= 2 ê 2ay - y ú
ë 12ab û0
A(–a, a2) B(a, a )
2

é 3ù
a 8b b - a æ a 8b ö ú
= 2 ê 2a ´ - ç ÷
X ê b - a 12ab çè b - a ÷ø ú
O(0, 0) ë û
8b 2 4 8b 8a 2 8b
a2 a2 = 4a - a2 = sq. units
= 2 ò x dy = 2 ò y dy b-a 3 b-a 3 b -a
0 0
12. (c) The given curves are :
y = x - 1, x ³ 1 ü y = 3 - x, x > 0 ü
a2 ý and ý
é y3 / 2 ù 4 3 y = 1 - x, x £ 1 þ y = 3 + x, x < 0 þ
= 2ê ú = a units
ë 3 / 2 û0 3

a3 3 Y
\ ratio of areas = =
4 3 4
a C

y=
3

1–

y=
10. (c) f(x) = x2 + bx - b; f ¢(x) = 2x + b Þ f ¢(1) = b + 2

1
x

x–
Equation of tangent : y - 1 = (b + 2) (x - 1)


B

y=
x
x
Putting x = 0 Þ y = 1 - b - 2 = -b - 1 > 0 Þ b < -1

+
D

3
-1

y=
1 A
Putting y = 0 Þ x - 1 = - Þ x= +1
b+2 b+2 X' –4 –3 –2 –1 O 1 2 3 4 X
b +1
= > 0 Þ b < -2 or b > -1
b+2
Combining, the two conditions = b < -2 Y'
1 b +1
Now, |-b -1| = 2; (b + 1)2 = 4|b + 2| = -4b - 8
2 b +2 The required area = area of ABCD
Þ (b + 3)2 = 0 Þ b = -3 follows the condition b < -2
11. (b) We have y2 = 4a(x + a) ...(i), a parabola with vertex (– a, 0) 0 2
and y2 = 4b (x – a) ...(ii), a parabola with vertex (a, 0) = ò (3 + x ) dx + ò (3 - x) dx
-1 0
8b
Solving (i) and (ii), we get y = ± a 1 2
b-a
– ò (1 - x) dx – ò ( x - 1) dx
Y -1 1
5 1
= + 4 – 2 – = 4 sq. units.
2 2
13. (a) Given, curves are y = logex, x = 0, y £ 0 and x-axis.
\ The given shaded region shows the required area
X
(– 4, 0) (a, 0) Y
y = logex
x=0

a 8b
b -a O (1, 0) X
æ æ y2 ö æ y2 öö y=0
A=2 ò çç
ç ç 4b
+a÷ -ç
÷ ç 4b
- a ÷ ÷ dy
÷÷
0 èè ø è øø
a 8b
b-a æ ( b - a ) y2 ö
=2 ò ç 2a -
ç
0 è
Mathematics S-M-87

Now, Intersecting point of y = logex & x-axis is (1, 0)


\ Required area Y-axis
1 1
= ò (0 - log e x )dx = ò log e x dx
0 0
b
1
1
= x loge x ]0 - ò x dx
1
x a (a, 0)
0 X-axis
O a b
= x log e x ]0 - x ]0
1 1

= [ | x log e x - x |]0 = 1 sq. unit


1

14. (a) Y y = x e x y = xe–x

The area of shaded region


a
b a2
e) òa a 2 - x 2 dx -
1, 2
A( 1
0
B(1, e ) a
b é xù a2
X = ê x a 2 - x 2 + a 2 sin -1 ú -
O x=1 2a ë a û0 2
2
Given curves are y = xe x and b é 2 2 2 -1 a ù a
= a a - a + a sin -
2a êë a úû 2
y = xe - x
b ab a2
æ 1ö = × . +
Line x = 1 meets the curves at A(1, e) and B çè1, ÷ø . 2a a 2 + b 2 a 2 + b 2
e
Both the curves pass through origin. ab -1 b a 2 b2
sin -
1 2 2
a 2 + b2 2 a + b
2
( )
\ Required area = ò ( xe x - xe- x )dx
0 ab b
= tan -1
1 2 a
= ò x(e x - e- x )dx Required area = 8I = 4ab tan –1
b
sq. units
0 a
1
16. (a) The graph of y = 2x 4 - x 2 is shown below :
ò
x -x x -x 1
= x(e + e ) |0 - (e + e )dx
0

æ 1ö 1 1
= çe + ÷ - e
x
+ e- x –1/2 1/2
è eø 0 0 –1 1
2 2
1 1 2
= e+ - e + 1+ - 1 =
e e e
x2 y2 x2 y2 1
15. (c) We have, + = 1 and
= 1 both are + Minimum occurs at x = ±
a 2 b2 b2 a 2 2
ellipse with centre (0, 0), vertex (a, 0), (– a, 0) and 1/ 2 1/ 2
(0, b), (0, – b)
ò (– y)dx = 2 ò (x
2
\ Desired area = 2 - 2x 4 )dx
The curves intersect at ( ± a, ± a )
0 0
ab 1/ 2
where, a = é x3 2x 5 ù é1 1ù 7
2 - =
a + ú
80 û 120
EBD_7504
S-M-88 SOLUTIONS

17. (c) The given lines are, p/ 2 p/2


y – 1 = x, x ³ 0; y – 1 = – x, x < 0
1 1
A = 3ab ò ò
sin 4 t cos2 t dt = 3ab cos4 t sin2 tdt
y = 0; x = - , x < 0 ; x = , x ³ 0 0 0
2 2 p
so that the area bounded is as shown in the figure. 2
p 3pab
ò
\ 2A = 3ab cos2 t sin2 tdt = 3ab×
16
ÞA =
32
0
21. (c) Given curves are y = ex and y = e–x
Now, ex = e–x Þ x = 0
1

ò (e - e - x ) dx
x
\ Area = A =
0

= ( e x + e - x )0
1

Required area
= éë( e + e -1 ) - ( e0 + e -0 ) ùû = éêe + - (1 + 1) ùú
æ
12 1
12 x2 ö æ1 1ö 5
= 2 ò (1 + x ) dx = 2çç x + 2 ÷÷ = 2 ç 2 + 8 ÷ = 4 ë e û
0 è ø0 è ø
1
b =e+ - 2.
18. (d) Given equation y2 = 4bx = 2 ò0 4bx dx
22.
e
(b) Put 2 – y – y2 = 0
b Þ y = 1, – 2
2é ù 8 b éê 2 ù
3 3
= 4 b ´ êx 2 ú = b - 0ú This means, the curve intersects the y-axis at y = 1 and
3ê ú 3 ê ú y = – 2.
ë û0 ë û
1
\ area of parabola bounded by its latus rectum
Hence required area = ò xdy
8b 2 -2
= sq. units
3 1
19. (d) To find the point of intersection of curves y = ln (x) (2 - y - y 2 ) dy
and y = ln (3), put ln (x) = ln (3)
= ò
-2
Þ ln (x) – ln (3) = 0 Þ ln (x) – ln (3) = ln (1)
Y
x
Þ =1 Þ x = 3 y=1
3
Y O X
y = ln (x)
y = –2
y = ln (3)
X
O 1 3 1
é y 2 y3 ù 9
= ê2 y - - ú = sq. units
ë 2 3 û -2 2

23. (a) y = 1 + 4 x - x 2 = 5 - ( x - 2) 2
Y
3 3
y = mx
Required area = ò ln (3) dx - ò ln ( x) dx y = 1+ 4x – x2
0 1

= [ x ln (3) ]0 - [ x ln ( x ) - x ]1 = 2
3 3
X
20. (c) y = 0, when t = 0 and then x = a 3 2
So desire area 2
a 0
3/ 2 3/ 2
ò
A = ydx = ò b sin 3 t (-3a cos 2 t sin tdt ) 2
x )dx = 2 ò mx dx
0 p/ 2
0
Mathematics S-M-89

3 æ 9 ö 1 æ 27 ö 9 5
5é 2 ù
= + 2ç ÷ - ç ÷ = m × = ò ê 4ax - 2a ´ (x - 1)3 2 ú
2 è4ø 3è 8 ø 4 1ë 3 û1
13 æ 32a ö 16
On solving we get m = = ç 20a -
6 è ÷ - (4a - 0) = a sq.unit
3 ø 3
Y 26. (a) Graph of y = e -|x| is symmetrical about y-axis, so we

-x dy
consider x ³ 0 , then y = e Þ = -e x
(0,1) dx

P(x, y)
24. (d)
O
X 0
x= /2 x=
Tangent at P is Y - y = -e - x (X - x )
p
2
Required area = ò cos x dx Its x-intercept = x + ye x and y-intercept = y + xe - x
0 1
So area A = (x + ye x )( y + xe - x )
p /2 p 2
= ò cos 2 x dx + ò cos 2 x dx
1
0 p /2 = (x + 1)(1 + x )e - x [Q y = e–x]
2
p
1 p 1 1
= ´ +
2 2 2 ò (1 + cos 2x) dx =
2
( x + 1) 2 e - x
p /2
dA 1 1
p 1é sin 2x ù p 1 éæ pö ù
p = ( x + 1)e - x - ( x + 1) 2 e - x = ( x + 1)(1 - x ) e - x
= + êx + = + p - ÷ú dx 2 2
4 2ë 2 úû p /2 4 2 êëçè 2øû where A is maximum if x = 1
So P is (1, e–1) Due to symmetry, there is another point
p p p (–1, e–1)
= + =
4 4 2 27. (b) y = | cos x – sin x |
25. (a) Given curve : y2 = 4a2 (x – 1)
Lines : x = 1 and y = 4a Y
Now, (y – 0)2 = 4a2 (x – 1)
f(x) = cos x g(x) = sin x
This is a parabola with vertex A (1, 0).
x=1
4 a)
(1,
C B y = 4a X
(5, 4a) O p/4 p/2

O , if a > 0
(0, 0) A (1, 0)

p /4
Required area = 2 ò (cos x - sin x ) dx
0

p/4 é 2 ù
Required area = 2 [ sin x + cos x ]0 =2ê - 1ú = (2 2 - 2) sq. units
= area of shaded region ABC ë 2 û
5
= ò [ y(line) - y(parabola) ] dx 28. (b) The tangent on x 2 + y2 = 4 at (1, 3 ) is
1
5 éx +
ë 3y = 4 ùû and equation of normal at (1, 3 ) is
= ò éë 4a - 2a x - 1 ùû dx
1
y=x 3
EBD_7504
S-M-90 SOLUTIONS

Y 1 4
é x2 ù 1 é x2 ù
= 3ê ú + ê 4x - ú
ëê 2 ûú0 3 êë 2 ûú
1
(1, 3 )
1 1 é 1 ù
= 3´ + ê 4 ( 4 - 1) - (16 - 1) ú
2 3ë 2 û
X 3 1 é 15 ù 3 9
(0, 0) (4, 0) = + ê12 - ú = +
2 3ë 2û 2 2 3

3 3 3
= + = 2 3 sq. units
1 4
2 2
4- x 29. (a) Both the statements are true and statement 2 is correct
Required area = ò x 3 dx + ò 3
dx
explaination of statement-1.
0 1 30. (a) Here both the statements are correct and statement-2
is correct explaination of statements-1.

Speed Test-83

y = ( x + 1 + x 2 )n æ1ö -1 æ 1 ö
1. (a) \ The solution is, z ç ÷ =
x
è ø ò
ç ÷ dx + c
x2 è x ø
dy æ 1 ö
= n( x + 1 + x 2 )n -1 ç 1 + (1 + x 2 ) -1/ 2 . 2 x ÷ ; 1 æ 1 ö - x -2 æ 1ö
dx è 2 ø Þ ç ÷= + c Þ x = log y ç cx 2 + ÷
log y è x ø -2 è 2ø
dy ( 1 + x 2 + x) n( 1 + x2 + x) n
= n( x + 1 + x 2 )n -1 = d2y d 3 y dt
dx 1 + x2 1 + x2 3. (a) Let = t Þ = and the given equation
dx 2 dx 3 dx
dy dy dt
or 1 + x 2 = ny or 1 + x 2 y1 = ny (Q y 1= ) recuces to = 8t .
dx dx dx
Squaring, (1 + x 2 ) y12 = n 2 y 2 dt
Separating the variables, = 8dx . Integrating we get.
t
Differentiating, (1 + x 2 )2 y1 y2 + y12 .2 x = n2 .2 yy 1
ln t = 8 x + c1 Þ ln y¢¢ = 8 x + c1
d2 y Put x = 0, then y¢¢ = 1 Þ C1 = 0
(Here, y2 = ) or (1+x2)y2 + xy1 = n2y
dx2 \ln y ¢¢ = 8x Þ y¢¢ = e8x ,

2. (c) Divide the equation by y (log y) 2 e8 x


Again integrate, we get y ' = + c2
8
1 dy 1 1 1
+ × = 1
y (log y) dx log y x x 2
2
Again putting x = 0 and y¢ = 0 Þ c2 = -
8
1 -1 dy dz
Put log y = z Þ = e8 x 1 1 æ e8 x ö
y (log y ) 2 dx dx \y' = - Þ y= ç - x ÷ + c3
8 8 8è 8 ø
dz 1 1 [After integration]
Thus, we get, - + × z = 2 , linear in z
dx x x 1 7
By giving values x = 0, and y = , we get c3 =
8 64
dz æ 1 ö 1
Þ + ç- ÷z = - é e8 x
dx è x ø x2 1 7ù
Hence the final solution is y = ê - x+ ú
8 ë 8 8û
1
- ò dx 1
I.F. = e x = e -log x
Mathematics S-M-91

4. (a) Taking x = r cos q and y = r sin q, so that 2


y d2y æ dy ö dy
x2 + y2 = r2 and = tan q, Þ xy + xç ÷ - y =0
x dx 2 è dx ø dx
we have x dx + y dy = r dr
dy
and x dy – y dx = x2 sec2 qdq = r2 dq. 7. (a) We have, ( xy - x 2 ) = y2
The given equation can be transformed into dx
dx 1 dx 1 1 -1
r dr æ a2 – r2 ö Þ y2 = xy - x 2 Þ 2 - . =
= çè ÷ dy x dy x y y 2
r 2d q r2 ø
dr
1 - 1 dx du
= (a 2 – r 2 )
Putting = u so that 2 =
Þ x x dy dy
dq
dr du u 1
Þ = dq We obtain + = 2 . Which is linear..
(a – r 2 )
2 dy y y
Integrating both sides, then we get 1
ò y dy
æ rö I.F. = e = elog y = y
sin –1 ç ÷ = q + c
è aø
1
æ ( x2 + y 2 ) ö Hence the solution is uy = ò . ydy + C
–1 æ yö y2
Þ sin –1 ç ÷ = tan èç ÷ +c ...(i)
è a ø xø
y
or = log y + C or y = x(log y + C)
Þ ( x + y ) = a sin {tan–1(y/x) + c}
2 2
x
This passes through the point (–1, 1),
Þ ( x 2 + y 2 ) = a sin {tan–1 (y/x) + constant} \ 1 = –1 (log 1 + C) i.e C = –1
dy y 5x Thus, the equation of the curve is y = x(log y – 1).
5. (c) The differential equation is - =-
dx x ( x + 2)( x - 3) dv dy
8. (d) Putting v = y / x so that x +v=
æ 1ö
dx dx
ò ç - ÷dx
I. F = e è x ø = e -ln x =
1 dv
x We have x + v = v + f (1/ v)
dx
æ 1ö æ 1ö 5x æ x+2ö
Solution is y ç ÷ =
è xø ò çè x ÷ø ´ (x + 2)(x - 3) dx = ln çè x - 3 ÷ø + C Þ
dv
=
dx
f(1/ v) x
It passes through (4, 0), so C = – ln 6
dv
ì ( x + 2) ü
\ y = x ln í ý
Þ log | C x | = ò f(1/ v)
î 6( x - 3) þ (C being constant of integration)
æ 7ö x
Putting (5, a), we get a = 5 ln ç ÷ But y = is the general solution,
è 12 ø log | Cx |
6. (c) Any conic whose axes coincide with co-ordinate axis is x 1 dv
ax2 + by2 = 1 ..(i) So = = log | Cx | = ò
Diff. both sides w.r.t. 'x', we get y v f (1/ v )
Þ f (1/v) = – 1/v2
dy dy
2ax + 2by = 0 i.e. ax + by =0 ..(ii) (differentiating w.r.t. v both sides)
dx dx Þ f (x/y) = – y2 / x2
æ d 2 y æ dy ö 2 ö dy f ' ( x ) y2
Diff. again, a + b ç y 2 + çè dx ÷ø ÷ = 0 ..(iii) 9. (a) We have =
dx f ( x )
y-
f (x )
è dx ø
a ydy / dx dy f ' ( x) y2
From (ii), =- Þ - y=-
b x dx f (x ) f (x)
Divide by y2
a æ d 2 y æ dy ö 2 ö
From (iii), b = - ç y 2 + çè ÷ø ÷ dy f ' (x) 1
è dx dx ø y -2 - y -1 =-
dx f (x ) f ( x)
dy
y 2 2 dy dz
\ dx = y d y + æ dy =
ç dx dx
x dx 2 è dx
EBD_7504
S-M-92 SOLUTIONS

dz f ' ( x ) 1 dz f ' ( x ) 1 p
- - ( z) = - Þ + z= Put x =
dx f (x ) f (x) dx f (x ) f (x) 2
4 1
ò
f '( x )
dx ln(1 + y ) = - ln 3 + ln 4 = ln Þ y=
3 3
I.F. = e f (x)
= e log f ( x ) = f ( x)
æ e -2 x y ö dx
1 - =1
\ The solution is z (f ( x) ) = (f ( x) )dx + c ç ÷
ò f (x)
13. (a) çè x x ÷ø dy

Þ y -1 (f ( x ) ) = x + c Þ f (x ) = y(x + c) dy y e -2 x
Þ + = … (i)
10. (a) Given differential equation is dx x x
sec2 x tan y dx + sec2 y tan x dy = 0
On separating the variables dy
Þ + P( y) = Q
(dividing the equation by tan x tany) dx
This is linear differential equation.
sec2 x sec2 y
Þ dx = - dy e -2 x
tan x tan y 1
Here, P = and Q =
On integrating both sides, we get x x
1
sec2 x sec2 y I.F = eò Pdx = e
ò dx
ò tan x dx = - ò tan y dy \ x = e2 x

\ Solution is
Put tan x = u Þ sec2x .dx = du
and tan y = v Þ sec2y.dy = dv e -2 x
y.e 2 x
= ò e2 x
dx Þ y.e2 x = 2 x + c
.
du dv x
\ ò = -ò Þ log u = - log v + log c 14. (c) The equation of normal to a curve at a point (x, y) is
u v
c dy
Þ u = Þ u.v = c (Y - y) + (X - x ) = 0
v dx
\ Required solution is tan x. tan y = c Since it passes throgh the point (3, 0), we have
dy y sin 2x dy dy
11. (c) + = (0 - y ) + (3 - x ) = 0 Þ y = (3 - x ) Þ ydy = (3 - x )dx
dx x log x log x dx dx

dx y2 x2
ò x log x Integrating, we get = 3x - +C
I.F. = e 2 2
1 Þ x2 + y2 – 6x – 2c = 0
ò dt Since the curve passes through (3, 4), we have
\ I.F. = e t = elog t = t = log | x |
solution is given by 7
9 + 16 – 18 – 2c = 0 Þ c = .
y (I.F.) = ò Q.(I.F.) dx + C 2
\ x2 + y2 – 6x – 7 = 0 is the required equation of the curve.
sin 2x 15. (a) The given equation is ax2 + by2 = 1.
y log | x | = ò log | x |
(log | x |) dx + C
dy
Differentiating we get, 2ax + 2 by =0
dx
cos 2x
=- +C dy
2 Þ ax + by = 0, ....(1)
dx
12. (a) dy æ 2 + sin x ö
= - cos x, y (0) = 1 æ d 2 y æ dy ö 2 ö
dx çè 1 + y ø÷
Differentiating again, a + b ç y + ç ÷ ÷ = 0, ....(2)
ç dx 2 è dx ø ÷
dy - cos x è ø
Þ = dx From eqs. (1) and (2), we get
(1 + y) 2 + sin x
Integrating both sides by dy æ d 2 y æ dy ö 2 ö
a=- = -b ç y +ç ÷ ÷
Þ ln(1 + y ) = - ln(2 + sin x) + C x dx ç dx 2 è dx ø ÷
è ø
Put x = 0 and y = 1
2
d2y æ dy ö dy
Þ ln(2) = - ln 2 =0
Mathematics S-M-93

y 1
16. (b) Let =v ò dx
x I.F. = e x = e log x = x
dy dv k
\
dx
= v+ x
dx
The solution is z( x ) = ò x (x)dx + c
dy y f( y / x) x 1 c
Then, = + reduces to y -1x = kx + c Þ = kx + c Þ = 1+
dx x f '( y / x) y ky kx

dv f( v ) æ1ö æ cö
v+x = v+ ç ÷ ç- ÷ A B
f '(v) k kø
Þè ø+è
dx
=1 Þ + =1
y x x y
f '(v ) dx
Þ dv = 19. (a) Let the equation of the curve be y = f (x).
f(v) x
On integrating, we get y = f(x)
log f (v) = log x + log k Y
Þ f (v) = xk
P (x,y)
æ yö
Þ f ç ÷ = kx
è xø
dy O T M X
17. (c) cos y = ex .esin y + x2 esin y = esin y (x2 + ex)
dx It is given that OT µ y
cos y dy cos y Þ OT = by
ò esin y dy = ò (x
2
Þ = (x2 + ex) Þ + ex )dx Þ OM – TM = by
esin y dx
y
sin y = t Þ x- = by [Q TM = Length of the subtangent]
cosy dy = dt dy / dx
x3 dx dx x
Þ ò e - t dt = + e x + C¢ Þ x- y = b. y Þ - = -b
3 dy dy y
It is linear differential equation. Its solution is given by
e- t x 3 x3
Þ = + e x + C¢ Þ e x + e - sin y + =C x
-1 3 3 = – b log y + a Þ x = y (a – b log y)
18. (d) The equation of tangent at any point P(x,y) is y
dy 20. (d) The given differential equation can be written as
Y-y= (X - x) dy
dx x + 2y = x (sin x + log x)
dx
For Y intercept put X = 0 S dy 2
Þ + . y = sinx + logx
P (x, y) dx x
dy
which is linear in y i.e. of the type + Py = Q
dx
O T
2
dy Hence P =
Þ Y = y-x x
dx
ò Pdx = 2 log x = log x
2
\
dy
Given Y µ y 2 Þ y - x = ky 2 2
dx \ e ò Pdx = elog x = x 2
[k proportionality constant]
òx
2
\ Sol. is y. x2 = (sin x + log x ) dx + c
dy 1 ky 2 dy 1 -1 k
Þ - y=- Þ y -2 - y =-
dx x x dx x x x2 x2
= – x2 cos x + 2x sin x + 2 cos x + log x – +c
dy dz 3 9
Put y -1 = z Þ - y -2 = .
dx dx 2 2 x x c
i.e. y = – cos x + sinx + 2 cos x + log x – + 2
dz 1 k dz æ 1 ö k x x 3 9 x
Then - - z=- Þ +ç ÷ z =
dx x x dx è x ø x
EBD_7504
S-M-94 SOLUTIONS

dy æ dy ö 24. (c) Divide the equation by y2, we get


21. (c) We have y - x = aç y 2 + ÷ dy
dx è dx ø y -2 - (2 tan x )y -1 = - tan 4 x [see the Bernoulli’s equation]
2 dx
Þ ydx - xdy = ay dx + ady
dy dz
Þ y (1 - ay )dx = (x + a )dy Put y -1 = z Þ - y -2 =
dx dx
dx dy
Þ - =0 dz
x + a y (1 - ay) Hence, - - (2 tan x ). z = - tan 4 x
Integrating, we get dx
log( x + a ) - log y + log(1 - ay ) = log C dz
Þ + ( 2 tan x ) z = tan 4 x
(a + x )(1 - ay) dx
or log = log C i.e. ( x + a )(1- ay) = Cy
y Which is linear in z, Integrating factor,

æ 1ö I.F. = e ò 2 tan xdx = e 2 log|sec x | = sec 2 x


Since the curve passes through ç a , - ÷

C
è aø
ò
The solution is z(sec 2 x ) = (tan 4 x ) sec 2 xdx + a
\ 2a ´ (1 +1) = - i.e C = -4a 2 1
a y -1 (sec 2 x ) = tan 5 x + a Þ 5 sec 2 x = y(tan 5 x + c), C = 5a
5
So, ( x + a )(1 - ay ) = -4a 2 y
dy 1 æ 1 ö
22. (c) Q y = un 25. (a) Slope =
dx
= 1- 2 Þ
x
ò dy = ò ç1 -
è x2
÷ dx
ø
dy du
\ = nu n –1
dx dx 1
Þ y = x+ + C , which is the equation of the curve
dy x
On substituting the values of y and in the given
dx æ 7ö
equation, then since curve passes through the point ç 2, ÷
è 2ø
du
2 x 4 .u n .nu n –1 + u 4n = 4 x6 7 1
dx \ = 2 + +C Þ C=1
2 2
du 4 x 6 – u 4 n
Þ = 1
dx 2nx 4 u 2 n –1 \ y = x+ +1
x
Since, it is homogeneous. Then, the degree of 4x6 – u4n and
2nx4 u2n–1 must be same. 1 -3
\ 4n = 6 and 4 + 2n – 1= 6 when x = – 2, then y = - 2 + + 1=
-2 2
3 dy dv
Then, we get n = 26. (a) Putting y = vx and = v+x , the given equation
2 dx dx
23. (c) Divide the equation by y (log y )2 reduces to the form

1 dy 1 1 1 dv x 2 + v 2 x 2 1 + v 2 dv 1 + v 2 1 - v2
v+x = = Þx = -v =
+ × = dx 2x ( vx ) 2v dx 2v 2v
y (log y ) dx log y x x 2
2

2v dv dx
1 -1 dy dz Þ = 2
Put log y = z Þ 2
= 1- v 2 x Þ - log(1 - v ) = log x - a
y (log y ) dx dx
Þ log(1 - v 2 )x = a Þ x (1 - v 2 ) = c , where c = ea
dz 1 1
Thus, we get, - + × z = 2 , linear in z 1 1
dx x x When x = 2, v = , [when x = 2, y = 1, v = ]
2 2
dz æ 1 ö 1
Þ + ç- ÷ z = - 2 3
dx è x ø x so that c =
2
1
- ò dx 1 3x
I.F. = e x = e- log x = 2 2
So the equation of the curve is x - y =
x 2
2
æ 1ö -1 æ 1 ö æ 3ö 9
\ The solution is, z ç ÷ = ò 2 ç ÷ dx + c Þ ç x - ÷ - y2 =
è xø x è xø è 4ø 16

1 æ 1ö - which is a rectangular hyperbola with eccentricity 2 .


Þ ç ÷ =
log y è x ø 2]
Mathematics S-M-95

27. (a) Tangent drawn at any point P (x, y) is Diff both side, we get
dy -1 dy dz
Y-y= (X - x) ...(i) . =
dx y 2 dx dx
The triangle , whose area is given is D OPT (see the shaded
region in the adjacent figure) If coordinates of T are (X, 0) 1 dy
Put value of in the equation(1) , we get
then y 2 dx
y æ dz ö
- ç ÷ - (tan x ) z = - sec x
è dx ø
P(x, y)
æ dz ö
Þ çè ÷ø + (tan x ) z = sec x
dx
This is the linear diff equation in 'z' i.e.
x dz
O N T This is of the form + P. z = Q
dx

then integrating factor = e ò


Pdx
dx \ In the given question
X =x-y (Put Y = 0 in (i)) \ area of D OPT
dy
I.F. = e
ò tan x dx = elog(sec x ) = sec x
1
= | X × y | = 2a 2 (given) 29. (c) Q xy = v
2
dy dv
1 dx dx \ x +y=
\ | xy - y 2 | = 2a 2 Þ xy - y 2 = ±2a 2 dx dx
2 dy dy Then, the given equation reduces to
dx x 2a 2 v æ 1 æ dv öö
Þ - = ± 2 , which is linear in x f ( v ) + x f( v ) ç ç – y ÷ ÷ = 0
dy y x è
è x dx øø
y
1 v dv
ò - dy Þ f ( v ) + f ( v ) – y f( v ) = 0
1 x dx
The integrating factor is I.F. = e
y
= e -log y =

{ }
y v ( f (v ) – f(v )) dv
Þ + f( v ) = 0
æ1ö 2a 2 æ1ö x dx
The solution is x çç ÷÷ = ò ± 2 çç ÷÷ dy + c
èyø y èyø dx f(v )dv
Þ + =0
x vf ( v ) – f( v )
x 2a 2 y -2 a2 Which is variable seperable form.
Þ =± + c Þ x = cy ±
y -2 y
dy dy
28. (b) Consider the differential equation 30. (c) + y =1 Þ = dx
dx 1- y
dy
= y tan x - y 2 sec x dy
dx
Divide by y2 on both the sides, we get ò 1 - y = ò dx - log(1 - y) = x
1 æ dy ö tan x 1 - y = e - x , ye = e + c
x x
ç ÷= - sec x ...(1)
y 2 è dx ø y Order of differential equation is the number of orbitarary
constants.
1 Both are true but Statement 2 is not correct reason.
Let =z
y
EBD_7504
S-M-96 SOLUTIONS

Speed Test-84

r r r r
1. (d) a = 7, b = 11 and a + b = 10 3 1–b=0Þb=1
r r r r r r r
2 2 2 Thus | c |= 3 = 12 + a2 + b2 Þ a = ±1
Now a + b = a + b +2 a b cos q
4. (a) Since, b, c and b ´ c are mutually perpendicular
(10 3 )
2
\ = 49 + 121 + 2 × 7 × 7 cos q
vectors, therefore any vector a can be expressed in terms
\ 300 = 170 + 154 cos q
300 – 170 of b, c and b ´ c .
= cos q
154 Let a = x b + y c + z ( b ´ c) ....(1)
65
\ = cos q Taking dot product with b ´ c in eq. (1),
77
r r 2 r 2 r 2 r r
Now, a – b = a + b – 2 a b cos q we get, a. ( b ´ c) = 0 + 0 + z | b ´ c | 2 Þ z = a . ( b ´ c)
2
| b´ c |
2 2 65
= (7) + (11) – 2 × 7 × 11 × Taking dot product with b in eq. (1), we get
77
= 49 + 121 – 2 × 65 a . b = x b . b + y c . b + z .0 = x
= 170 – 130 = 40
r r Taking dot product with c in eq. (1), we get
\ a – b = 40 = 2 10
\ Option (d) is correct. a. c = 0 + y + 0 Þ y = a. c
r r r
2. (a) Since, a, b & c are non-coplanar a . ( b ´ c)
r r r r r r \ a = ( a . b) b + ( a . c ) c + ( b ´ c)
\ b ´ c , c ´ a, a ´ b are also non-coplanar.. | b ´ c |2
So, any vector can be expressed as a linear combination 5. (c) We have r = l1r1 + l2r2 + l3r3
of these vectors. Þ 2a – 3b + 4c = (l1 – l2 + l3)a
r r r r r r r + (– l1 + l2 + l3)b + (l1 + l2 + l3)c
Let, r = l (b ´ c ) + m (c ´ a ) + g (a ´ b )
Þ l1 – l2 + l3 = 2, –l1 + l2 + l3 = –3, l1 + l2 + l3 =
rr rrr 4
\ a.r = l [a b c ] + m (0) + g (0)1
rr (Q a, b, c are non-coplanar)
a. r
Þ l= r rr 7 1
[a b c ] Þ l1 = , l2 = 1, l3 = -
2 2
rr rr Therefore, l1 + l3 = 3 and l1 + l2 + l3 = 4.
b .r c .r
Similarly, m = r r r & g = r r r
[a b c ] [a b c ] 6. (d) a × k̂ < 0 Þ sin 2a < 0 …(1)
r r
If r = a b × c Þ tan 2 a - tan a - 6 = 0 Þ tan a = -2 or 3
r r r r r r r
Then l (b ´ c ) + m (c ´ a ) + g (a ´ b ) = a sin 2a < 0 if tan a = -2
rr r r rr r r rr r r r rrr
& (a.a ) (b ´ c ) + (b.a )(c ´ a ) + (c.a ) (a ´ b ) = a [a b c ] a
for which sin > 0 if a lies in the second quadrant
r r r 2
3. (d) If a = iˆ + ˆj + kˆ , b = 4iˆ + 3 ˆj + 4kˆ and c = iˆ + aˆj + bkˆ
r \ a = p + tan -1 (-2) = p - tan -1 2
are linearly dependent vectors (provided) | c |= 3 i.e.,
rrr General value is a = 2np + p - tan -1 2, n Î I
[a b c ] = 0 r r r r r r
7. (c) ((a ´ b ) ´ (c ´ d )).( a ´ d ) = 0
1 1 1 0 0 1 rrr r rrr r r r
([a c d ] b - [b c d ] a ).(a ´ d ) = 0
Þ 4 3 4 =0 Þ 1 –1 4 = 0
rr r rr r
1 a b 1– a a – b b [a c d ][b a d ] = 0
r r r r
By expanding Either c or b must lie in the plane of a and d .
Þ (a – b – a + 1)
Mathematics S-M-97

uuur
8. (c) Let b , c , d be the position vectors of B, C, D w.r.t. AB = –2i – 4ˆj – 4kˆ

A as origin. So, AB = b , CD = d - c , AD = d ,
= (–2)2 + ( –4 ) + (–4) 2
2

BC = c - b , AC = c and BD = d - b
= 4 + 16 + 16 = 36
uuur
Now, L.H.S. = b . ( d - c ) and | AB | = 6
uuur
2 2 2 2
RHS = k[| d | + | c - b | - | c | - | d - b | ] AC = 2iˆ + 5 ˆj + 7kˆ – 4iˆ – 7 ˆj – 8kˆ
= k[| d . d + c . c + b . b - 2 c . b AC = –2i – 2 ˆj – kˆ
- c . c - d . d - b .b +2d . b]
= (–2)2 + (–2) 2 + (–1) 2
1.
= 2 k[ b . ( d - c )] Þ k = = 4 + 4 +1 = 9
2 uuur
ur ur ur AC = 3
9. (b) Since, a , b and c are three vectors with magnitude ur
ur ur ur \ p.v. of D
| a |=| b |= 4 and | c |= 2,
ur ur ur 6(2iˆ + 5 ˆj + 7kˆ) + 3(2iˆ + 3 ˆj + 4kˆ)
As a is perpendicular to (b + c ) =
6+ 3
ur ur ur ur ur uurur
Þ a .(b + c ) = 0 or a .b + a .c = 0 ...(i) 12iˆ + 30 ˆj + 42kˆ + 6iˆ + 9 ˆj + 12kˆ
ur ur ur =
b is perpenducular to (c + a ) 9
ur ur ur urur urur 18iˆ + 39 ˆj + 54kˆ 3(6iˆ + 13 ˆj + 18kˆ)
Þ b .(c + a ) =Þ0 or b.c + b.a = 0 ...(ii) = =
ur ur ur 9 9
Þ c is perpendicular to (a + b )
6iˆ + 13 ˆj + 18kˆ)
ur ur ur ur ur ur ur =
Þ c .(a + b ) = 0 Þor c . a + c .b = 0 ...(iii) 3
From equations (i), (ii) and (iii), we get 2
uur ur ur ur ur ur a1 a2 a3
Þ 2(a . b + b .c + c .a ) = 0 rrr
11. (c) Q b1 b2 b3 = [a b c ]2
Further we know that c1 c2 c3
ur ur ur ur ur ur
ÞQ | a + b + c |2 =| a |2 + | b |2 + | c |2 r r r r r r
uuur ur uur ur uur ur = {(a ´ b ).c}2 = | a ´ b |2 | c |2 cos0°
+ 2a . b + 2b . c + 2c . a r r
ur ur ur [Q (a ´ b ) is parallel to cr ]
Þ| a + b + c |2 = 4 2 + 4 2 + 2 2 + 0 = 36 r2 r 2 r 1 r r
2 p
ur ur ur = | a | | b | sin (Q | c | = 1) = | a |2 | b |2
or | a + b + c | = 6 6 4
10. (a) Let O be the origin and the bisector of ÐA meets BC a .( b ´ c ) [a b c]
12. (d) a .p = = =1= b . q = c . r
BD AB [a b c] [a b c]
at D. Then = and position vector of D is given
DC AC
by b .( b ´ c ) 0
b.p = = =0= c.p = a .r
A [a b c] [a b c]
Therefore, the given expression is equal to
1 + 0 + 1 + 0 + 1 + 0 = 3.
[Also see the system of reciprocal vectors]
r r r
13. (d) Q a lies in the plane of b and c
C r
\ ar = b + lcr
B D
uuur uuur
AB OC + AC OB Þ aiˆ + 2 ˆj + bkˆ = iˆ + ˆj + l ( ˆj + kˆ)
OD = uuur
AB + AC Þ a = 1, 2 = 1+ l, b = l Þ a = 1, b = 1
uuur
AB = 2iˆ + 3jˆ + 4kˆ – 4iˆ – 7 ˆj – 8kˆ
D
B
E
S-M-98 SOLUTIONS

16. (d) Let A be the first term and x the common ratio of G.P.
14. (b) Let | p | = | q | = | r | = k
So, a = Ax p -1 Þ log a = log A + (p - 1) log x
Let p̂, q̂, r̂ be unit vectors along p, q, r respectively.
Similarly, log b = log A + (q - 1) log x
Clearly p̂, q̂, r̂ are mutually perpendicular vecotrs, so any and log c = log A + (r - 1) log x
vector x can be weitten as a1p̂ + a 2 q̂ + a 3r̂ .
If a = log a 2 î + log b 2 ˆj + log c 2 k̂
\ p ´ {( x - q ) ´ p} = ( p . p) ( x - q ) - { p.( x - q )} p
and b = (q - r ) î + ( r - p) ĵ + ( p - q ) k̂ then
= k 2 ( x - q ) - ( p. x ) p [Q p . q = 0]
a . b = 2[log a (q - r ) + log b ( r - p ) + log c( p - q )]
= k 2 ( x - q ) - kp̂.( a1p̂ + a 2 q̂ + a 3 r̂ ) kp̂ = k 2 ( x - q - a1p̂)
= 2[(q - r ){log A + (p - 1) log x} +(r - p){log A + (q - 1) log x}
Similarly, q ´ {( x - r ) ´ q} = k 2 ( x - r - a 2 q̂ ) +(p - q){log A + (r - 1) log x}]
2
and r ´ {( x - p) ´ r} = k ( x - p - a 3 r̂ ) = 2[(q - r + r - p + p - q) log A +(qp - pr - p + r + qr - pq
According to the given condition -r + p + pr - qr - p + q) log x ] = 0
2
k ( x - q - a1p̂ + x - r - a 2 q̂ + x - p - a 3 r̂ ) = 0 p
Hence, the angle between a and b is .
Þ k 2 {3 x - ( p + q + r ) - (a1p̂ + a 2 q̂ + a 3 r̂ )} = 0 2
r
17. (b) If F be the resultant force, then
Þ k 2 [ 2 x - ( p + q + r )] = 0 r
F = 2iˆ + 4 ˆj + 2kˆ
r uuur
1
Þ x= ( p + q + r ) [Q k ¹ 0] Also, r = AP = – iˆ + 3 ˆj + 2kˆ
2
uuur r uuur r uuur r
15. (c) OA = a, OB = b & OC = c are unit vectors and iˆ ˆj kˆ
equally inclined to each other at an acute angle q. r r
\ required moment = r ´ F = –1 3 2 = –2iˆ+ 6 ˆj –10kˆ
O 2 4 2

® ® ® ®
18. (b) Since, u and v are collinear, therefore k u + v = 0
B
® ®
C Þ[k(a – 2) + 2 + 3a] a + (k – 3) b = 0 ...(i)
G
® ®
A Since a and b are non-collinear, then for some
\ ABC is an equilateral triangle constant m and n,
® ®
2 2
and AB = OA + OB - 2OA.OB.cos q m a + n b = 0 Þ m = 0, n = 0
= 2 - 2 cos q = 2 1 - cos q Hence from equation (i)
k–3=0 Þ k=3
\ Area of D ABC
And k(a – 2) + 2 + 3a = 0
3 3 3 2
= AB2 = .2(1 - cos q) = (1 - cos q) Þ 3(a – 2) + 2 + 3a = 0 Þ a =
4 4 2 3
If G is the centroid of the D ABC, then r r r r r r
19. (a) As given, r ´ a = b ´ a Þ (r – b) ´ a = 0
1 r r r r r r r r r
OG = | a + b + c |
3 Þ r – b is parallel to a Þ r – b = n a, n Î R
r r r
1 2 2 2 rr rr rr Þ r = b+ na ...(i)
= a + b + c + 2a .b + 2b .c + 2c .a
3 r r r r r r r
Similarly, r ´ b = a ´ b can be written as r = a + mb
1 where m Î R ...(ii)
= 1 + 2 cos q
3 \ From equations (i) and (ii), we get
r r r r r r
\ [a b c ] = Volume of parallelopiped m = 1 =n and r a= b +
= OG × 2 ar (D ABC) r r r r r
Þ r = i + 3j – k and | r |= 9 + 1+ 1 = 11
1 3 r r r r
= 2. 1 + 2 cos q ´ (1 - cos q) r i + 3j – k
3 r
= (1 - cos q) 1 + 2
Mathematics S-M-99
uuur uuur
20. (b) Let AB = a, AC = b 1 é1 1 ù 1
= - + 1 = = cos 60°
uuur 4 a + b uuur 2b 2 êë 2 2 úû 2
So, AD = , AE = , \ q = 60°
5 5
uuur 3a uuur a 23. (b) Given, a ´ b = 2 a ´ c Þ a ´ ( b - 2 c ) = 0
AF = , and AK=
10 4 Since the vectors are non zero.

A \ b - 2 c is parallel to a . Let b - 2 c = l a

K Now | b - 2 c |2 = b 2 + 4c 2 - 4 b . c
3 1
F 2 = 16 + 4 - 4 ´ 4 ´ 1´ = 16
4
E
i.e., l2 a 2 = 16 Þ l2 = 16 Þ l = ±4
7 r r r r r r r r r
24. (a) 2[a , b , iˆ]iˆ + 2[a , b , ˆj ] ˆj - 2[a, b , kˆ] k̂ +[a, b , a ]
3
r r r r r r
= 2(a ´ b )iˆ.iˆ + 2(a ´ b ) ˆj. ˆj - 2(a ´ b )kˆ.kˆ + 0
r r r
(Q [a, a, b ] = 0)
C
B 1 D 4 r r r r r r
= 2(a ´ b ) + 2(a ´ b ) - 2(a ´ b ) [Q iˆ.iˆ = ˆj. ˆj = kˆ.kˆ = 1]
r r
uuur uuur uuur b + 4a + 2b – 5a + 3a –10b = 2(a ´ b )
AD +BE + CF
uuur = 5 5 10
A
CK a – 4 b 25. (b)
4
6b – 2a + 3a – 10b 2
= ´4 = . 3AB 2AC
10(a – 4b ) 5
21. (b) Let R = x î + yˆj + zk̂ . Then

R ´ B = C ´ B Þ ( R - C )´ B = 0 B C
6CB
î ĵ k̂
There forces 3AB , 2AC and 6CB acts along the
Þ x -4 y+3 z -7 = 0 sides of the triangle AB, AC and CB.
1 1 1 DC 1
Hence, =
AD 2
Þ ( y - z + 10) î + ( z - x - 3) ĵ + ( x - y - 7) k̂ = 0
26. (c) Suppose p = p1î + p 2 ĵ + p 3 k̂
Þ y - z = -10, z - x = 3, x - y = 7
Also R . A = 0 Þ 2 x + 0 . y + z = 0 Þ z = -2 x . Solving, p ´ î = p 2 ˆj ´ î + p 3 k̂ ´ î = - p 2 k̂ + p 3 ĵ
we obtain | p ´ î | 2 = p 22 + p 32
x = –1, y = –8, z = 2. Hence R = - î - 8 ĵ + 2 k̂ . Similarly, | p ´ ĵ | 2 = p 32 + p12 , | p ´ k̂ |2 = p12 + p 22

22. (a) Let a =


1 ˆ 1 ˆ ˆ
2
i+
2
j+k \
3
2
{
| p ´ î | 2 + | p ´ ĵ |2 + | p ´ k̂ | 2 }
2
1 ˆ 1 ˆ ˆ = 3( p12 + p 22 + p 32 ) = 3 p
and b = i- j+ k
2 2 27. (a) B C
a.b a +b
\ cos q = b
| a || b |
a –b
æ 1 ˆ 1 ˆ ˆö æ 1 ˆ 1 ˆ ˆö
ç i+ j + k ÷ .ç i- j+k÷ O a A
è 2 2 øè 2 2 ø
=
1 1 1 1 Let OA = a and OB = b .
+ +1 + +1
2 2
EBD_7504
S-M-100 SOLUTIONS

Þ a (b – 1)(c – 1) – (1 – a) {(c – 1) – (1 –b)} = 0


a + b = OA + OB = OC Þ| a + b |= OC Þ a (1 – b)(1 – c) + (1 – a) (1 – c) + (1 – a) (1 – b) = 0
Again a - b = OA - OB = BA Þ| a - b |= BA Þ (a – 1 + 1) (1 – b) (1 – c) + (1 – a) (1 – c)
+ (1 – a) (1 – b)= 0
Given | a + b |=| a - b |Þ OC = BA Þ (1 – b) (1 – c) + (1 – a) (1 – c) + (1 – a) (1 – b)
= (1 – a) (1 – b) (1 – c)
\ Diagonals of the parallelogram OACB are equal.
1 1 1
Þ + + =1
\ OACB is a rectangle. 1- a 1 - b 1- c
r r r
\ a and b are adjacent sides of a rectangle. 29. (b) Statement 1 : a , b , c are coplanar
r r r r
28. (b) Since vectors are coplanar Þ c = xa + yb solving we get Þ cr = 4ar - b
a 1 1 30. (c) Statement 1 is false and statement 2 is true.
r r r
\
1 b 1
=0 Since a.(b ´ c ) = 0
1 1 c r rr
\ a, b,c are coplanar..
a 1 1
1- a b -1 0
Þ =0 [Using R2 – R1, R3 – R2]
0 1 - b c -1

Speed Test-85

1. (c) We have, z = 0 for the point where the line intersects < -3 - 3s - 5 - 3t, 3 + 2s - 7 + t, 6 + 4s + 2 - t >
the curve.
i.e., < -8 - 3s - 3t, - 4 + 2s + t, 8 + 4s - t >
x - 2 y + 1 0 -1 If PQ is the desired line then direction ratios of PQ
Therefore, = =
3 2 -1 should be proportional to < 2, 7, –5>, therefore,
x-2 y +1 -8 - 3s - 3t -4 + 2s + t 8 + 4s - t
Þ = 1 and =1 = =
3 2 2 7 -5
Þ x = 5 and y = 1 Taking first and second numbers, we get
-56 - 21s - 21t = - 8 + 4s + 2t
Put these value in xy = c2, we get, 5 = c2 Þ c = ± 5 … (i)
Þ 25s + 23t = -48
2. (a) Let a, b, c be the intercepts when Ox, Oy, Oz are taken as Taking second and third members, we get
x y z 20 - 10s - 5t = 56 + 28s - 7 t
axes; then the equation of the plane is + + =1
a b c Þ 38s - 2t = -36 … (ii)
Also let a¢, b¢, c¢ be the intercepts when OX, OY, OZ are Solving (i) and (ii) for t and s, we get
taken as axes ; then in this case equation of the same plane s = –1 and t = –1.
The coordinates of P and Q are respectively
X Y Z
is + + =1 (5 + 3(-1), 7 - (-1), - 2 - 1) = (2, 8, - 3)
a¢ b¢ c¢
Now (1) and (2) are equations of the same plane and in both and (-3 - 3(-1), 3 + 2(-1), 6 + 4(-1)) = (0, 1, 2)
the cases the origin is same. Hence length of the \ The required line intersects the given lines in the
perpendicular drawn from the origin to the plane in both the points (2, 8, –3) and (0, 1, 2) respectively.
case must be the same. Length of the line intercepted between the given lines
1 1
i.e = = | PQ | = ( 0 - 2) 2 + (1 - 8) 2 + ( 2 + 3) 2 = 78 .
1 1 1 1 1 1
+ + + + 4. (d) Suppose any line through the given point (1, –2, 3) meets
a2 b2 c2 a' 2 b' 2 c' 2
the sphere x 2 + y 2 + z 2 = 4 in the point
1 1 1 1 1 1
or 2
+ 2
+ 2
= \k=1
2
+ 2
+ (x1, y1, z1). Then x12 + y12 + z12 = 4 ...(1)
a b c a¢ b¢ c¢2
3. (b) The general points on the given lines are respectively Now let the co-ordinates of the points which divides
the join of (1, –2, 3) and (x1, y1, z1) in the ratio 2 : 3 be
P(5 + 3t , 7 - t , - 2 + t ) and Q(-3 - 3s, 3 + 2s, 6 + 4s) . (x , y , z ). Then we have
Mathematics S-M-101

where q is the angle between the given lines.


2.x1 + 3.1 5x - 3 ù
x2 = or x1 = 2 ú p
2+3 2 But, q = and therefore, sin q = 1
ú 2
2.y1 + 3.(-2) 5y + 6 ú
y2 = or y1 = 2 \ l = (m1n2 – m2n1); m = (n1l2 – n2l1) and n = (l1m2 – l2m1)
2+3 2 ú ...(2) Hence, the direction cosines of the required line are
ú
2.z1 + 3.3 5z - 9 ú (m1n2 – m2n1) (n1l2 – n2l1), (l1m2 – l2m1)
z2 = or z1 = 2 8. (c) Let P(a, b, g) be the foot of the perpendicular from the
2+3 2 úû
origin O(0, 0, 0) to the plane So, the plane passes through
Putting the values of x1, y1, z1, from (2) in (1), we have P(a,b, g) and is perpendicular to OP. Clearly direction ratios
(5x 2 - 3) 2 + (5y 2 + 6) 2 + (5z 2 - 9) 2 = 4 ´ 4 of OP i.e., normal to the plane are a, b, g . Therefore, equation
of the plane is a (x – a) + b (y – b) + g (z – g) = 0
or 25(x 22 + y 22 + z 22 ) - 30x 2 + 60y 2 - 90z 2 + 110 = 0 This plane passes through the fixed point (1, 2, 3), so
a (1 – a) + b (2 – b) + g (3– g) = 0
or 5(x 22 + y 22 + z 22 ) - 6( x 2 - 2y 2 + 3z 2 ) + 22 = 0 or a2 + b2 +g2 – a – 2b – 3g = 0
\ The locus of (x2, y2, z2) is Generalizing a, b and g, locus of P (a, b, g) is
x2 + y2 + z2 – x – 2y – 3z = 0
5(x 2 + y 2 + z 2 ) - 6( x - 2y + 3z ) + 22 = 0 . 9. (a) From the first relation, l = 5m – 3n. Putting this value
5. (d) For L1, of l in second relation
x - ( l - 1) 7(5m - 3n ) 2 + 5m 2 - 3n 2 = 0
x = l y + ( l - 1) Þ y = ...(i)
l
Þ 180m 2 - 210mn + 60n 2 = 0
z- l
z = ( l - 1) y + l Þ y = ...(ii) or 6m 2 - 7mn + 2n 2 = 0
l -1
Note that it, being quadratic in m, n, gives two sets of
From (i) and (ii)
values of m, n, and hence gives the d.r.s. of two lines.
x - ( l - 1) y-0 z- l Now, factorising it, we get
= = ...(A)
l 1 l -1 6 m 2 - 3mn + 4 mn + 2 n 2 = 0
The equation (A) is the equation of line L1. or (2m - n ) (3m - 2n ) = 0
Similarly equation of line L2 is
Þ either 2m - n = 0, or 3m - 2n = 0
x - (1 - m ) y-0 z- m
= = ...(B) Taking 2m – n = 0 we get 2m = n.
m 1 1- m Also putting m = n/2 in l = 5m – 3n, we get
Since L1 ^ L2 , therefore l = (5n/2) – 3n Þ l = – n/2 Þ n = – 2l
l m + 1´1 + ( l - 1) (1 - µ) = 0 l m n
Thus, we get, –2l = 2m = n or = =
-1 1 2
Þ l + m =0 Þ l =- m Þ l=m
Þ d.r.s. of one line are –1, 1, 2.
6. (c) Let the variable point be (a, b, g ) then according to question Hence, the d,c,s. of one line are
2 2 2
æ | a + b + g | ö æ | a - g | ö æ | a - 2b + g | ö é -1 1 2 ù é 1 -1 - 2 ù
çç ÷÷ + çç ÷÷ + ç ÷÷ = 9 ê , , ú or ê , , ú
è 3 ø è 2 ø çè 6 ø ë 6 6 6û ë 6 6 6û
Taking 3m – 2n = 0, we get
Þ a 2 + b2 + g 2 = 9 .
2n
So, the locus of the point is x 2 + y 2 + z 2 = 9 3m = 2n or m = .
3
7. (a) Let l, m, n be the direction cosines of the line Putting this value in l = 5m – 3n, we obtain
perpendicular to each one of the given lines.
Then, ll1 + mm1 + nn1 = 0 ...(1) 2n n
l = 5´ - 3n = or n = 3l
ll2 + mm2 + nn2 = 0 ...(2) 3 3
Solving (1) and (2) by cross-multiplicatin, we get :
3m l m n
l m n Thus 3l = =nÞ = =
= = 2 1 2 3
(m1n 2 - m 2 n1 ) (n1l 2 - n 2 l1 ) (l1m 2 - l1m1 ) Þ the d.r’.ss of the second line are 1, 2, 3; and hence
é 1 2 3 ù
l 2 + m2 + n 2 l d.c.s. of second line are ê , , ú
= or
( m1n 2 - m 2 n 1 ) ë 14 14 14 û
å( m1n 2 - m 2 n1 ) 2
é -1 -2 -3 ù
m n 1 or ê , , ú
= =
(n1l 2 - n 2 l1 ) (l1m 2
EBD_7504
S-M-102 SOLUTIONS

10. (b) The equation of the given sphere is As we know, centroid of DABC with vertices
x2 + y2 + z2 – 10z = 0. (x1, y1, z1), (x2, y2, z2) and (x3, y3, z3) is given by
\ Its centre is (0, 0, 5).
Coordinates of one end point of a diameter of the æ x1 + x 2 + x 3 y1 + y 2 + y3 z1 + z 2 + z 3 ö
çè , , ,÷
sphere is given as ( – 3, – 4, 5). 3 3 3 ø
Let Coordinates of another end point of this diameter \ By using this formula, we have
(x1, y1, z1) a +0+0
= a Þ a = 3 a,
-3 + x1 3
\ =0 Þ x1 = 3
2 0+b+0
= b Þ b= 3b
-4 + y1 3
=0 Þ y1 = 4
2 0+0+c g
and = Þ c= 3g
5 + z1 3
and = 5 Þ z1 = 5 Now, put the values of a, b, c in equation (i), which
2
gives
\ Required coordinates are (3, 4, 5).
11. (a) Since l2 + m2 + n2 = 1 x y z
\ cos2 a + cos2 a + cos2 q = 1 .......... (i) + + =1
3a 3b 3g
(Q A line makes the same angle a with x and y-axes and
q with z-axis) \
x y z
+ + =3
Also, sin2 q = 2 sin2 a a b g
Þ 1 – cos2 q = 2(1 – cos2 a) (Q sin2 A + cos2 A = 1) 15. (b) Given equation of line is
Þ cos2 q = 2cos2 a –1 .......... (ii)
\ From Eq. (i) and (ii) x -3 y-3 z
= =
2 cos2 a + 2 cos2 a – 1 = 1 2 1 1
1 Þ DR’s of the given line are 2, 1, 1
Þ 4 cos2 a = 2 Þ cos2 a =
2 2 1 1
Þ DC’s of the given line are , ,
1 p 3p 6 6 6
Þ cos a = ± Þ a= ,
2 4 4 p
12. (b) Let Q be the image of the point P(2, 3, 4) in the plane x – 2y Since, required lines make an angle with the given
3
+ 5z = 6, then PQ is normal to the plane
\ direction ratios of PQ are <1, –2, 5 >
line
Since PQ passes through P(2, 3, 4) and has direction ratios 1, The DC’s of the required lines are
–2, 5 1 2 -1
-1 1 -2
, , , and, respectively..
x - 2 y-3 z -4 6 6 6 6 6 6
\ Equation of PQ is = =
1 -2 5 Also, both the required lines pass through the origin.
13. (a) Given equation of line is \ Equation of required lines are
1 1 x y z x y z
x +5 = ( y + 3) = – ( z – 6) = = and = =
4 9 1 2 -1 -1 1 -2
16. (d) The planes forming the parallelopiped are
x +5 y +3 z – 6
or = = = l ( say ) x = - 1, x = 1 ; y = 2, y = -1 and z = 5, z = –1
1 4 –9
x = l – 5, y = 4l – 3, z = – 9l + 6 Hence, the lengths of the edges of the parallelopiped
(x, y, z) º (l – 5, 4l – 3, –9l + 6) …(i) are 1– (–1) = 2, | – 1–2| = 3 and |–1–5| = 6
Let it is foot of perpendicualr (Length of an edge of a rectangular parallelopiped is the
So, d.r.’s of ^ line is distance between the parallel planes perpendicular to the
(l – 5 – 2, 4 l – 3 – 4, – 9l + 6 + 1) edge)
º (l – 7, 4l– 7, – 9l + 7) \ Length of diagonal of the parallelopiped
D.r.’s of given line is (1, 4, – 9) and both lines are ^
= 2 2 + 3 2 + 6 2 = 49 = 7.
\ (l–7). 1 + (4l – 7). 4 + (– 9 l+ 7) (–9) = 0
Þ 98l = 98 Þ l = 1 17. (d) Let {l, m, n} be the direction -cosines of PQ, then
\ Point is (– 4, 1, – 3). [Substituting l = 1 in (i)] 3l – m + n = 0 and 5l + m + 3n = 0
14. (c) Let us take a triangle ABC and their vertices A (a, 0, 0), l m n l m n
B (0, b, 0) and C (0, 0, c) \ = = i.e = =
- 3 -1 5 - 9 3 + 5 1 1 -2
Therefore the equation of plane is
Now a plane ^ to PQ will have l, m , n as the coefficients
x y z of x, y and z
+ + =1 .... (i)
a b c Hence the plane ^ to PQ is x + y – 2z = l
Now \ 2 + 1 –2.4 = l i.e l = – 5
is x + y – 2z = – 5
Mathematics S-M-103

18. (c) D.R. of given line are 1, –2, 3 and the d.r. of normal to the From (2) and (3), we get
given plane are 1, 2, 1. 1 1 1 1
Since 1 × 1 + (–2) × 2 + 3 × 1= 0, therefore, the line is + + =
Þ a -2 + b -2 + g -2 = p -2
2 2 2
parallel to the plane, Also, the base point of the line 9a 9b 9g 9p 2
(1, 2, 1) lies in the given plane. Generalizing a, b, g, locus of centroid P (a, b, g) is
(1 + 2 × 2 + 1 = 6 is true) x–2 + y–2 + z–2 = p–2
Hence, the given line lies in the given plane. 23. (b) The sphere x2 + y2 + z2 = 49
Alternatively, any point on the given line is has centre at the origin (0, 0, 0)
(t + 1, –2t + 2, 3t + 1). and radius 7.
It lies in the given plane Disance of the plane
x + 2y + z = 6 if t + 1 + 2 (–2t + 2) + 3t + 1 = 6 O
i.e. if 0t = 0, which is true for all real t. Hence every point 2x + 3y - z - 5 14 = 0
on the given line lies in the given plane i.e. the line lies in from the origin N
the plane. P
19. (b) A (0, b, c) in yz-plane and B(a, 0, c) in zx-plane. 2(0) + 3(0) - (0) - 5 14
Plane through O is px + qy + rz = 0. It passes through A =
and B. 2 2 + 32 + (-1) 2
\ 0p + qb + rc = 0 and pa + 0q + rc = 0
p q r - 5 14 5 14
Þ = = =k = =5=
bc ca - ab 14 14
Þ p = bck, q = cak and r = – abk. Thus in Figure
Hence required plane is bcx + cay – abz = 0. OP = 7, ON = 5
20. (c) The planes bx – ay = n, cy – bz = l and az – cx = m NP2 = OP2 – ON2 = (7)2 – (5)2 = 49 – 25 = 24
intersect in a line, if al + bm + cn = 0.
21. (a) Equation of the line through (1, –2, 3) parallel to the line \ NP = 2 6
x y z -1 Hence the radius of the circle = NP = 2 6
= = is
2 3 -6 C1
x -1 y + 2 z - 3 24. (b)
= = = r (say) ...(1)
2 3 -6 q 4
Then any point on (1) is (2r + 1, 3r – 2, –6r + 3)
If this point lies on the plane x – y + z = 5 then
O
1
(2r + 1) – (3r – 2) + (–6r + 3) = 5 Þ r = P
7
3
æ9 11 15 ö
Hence the point is ç , - , ÷ C2
è7 7 7ø
For the orthogonal section C1P and C2P are pendicular
æ9 11 15 ö where C1 and C2 are centres of sphere of radii 4 and 3
Distance between (1, –2, 3) and ç , - , ÷ respectively
è 7 7 7ø
3
æ 4 9 36 ö æ 49 ö Now C1P = 4 and C2P = 3, so tan q =
= ç + + ÷ = ç ÷ =1 4
è 49 49 49 ø è 49 ø \ Radius of circle of intersection
x y z 3 12
22. (b) Let equation of the variable plane be + + =1 OP = C1P sin q = 4 ´ =
a b c 5 5
This meets the coordinate axes at A(a, 0, 0), B (0, b, 0) and x - 2 y -1 z + 2
C(0, 0, c). 25. (a) Q The line = = lie in the plane
3 -5 2
Let P(a, b, g) be the centroid of the DABC .Then x + 3y – a z + b = 0
a +0+0 0+b+0 0+0+ c \ Point (2, 1, – 2) lies on the plane
a= ,b = ,g = i.e. 2 + 3 + 2a + b = 0
3 3 3
or 2a + b + 5 = 0
\ a = 3a, b = 3b , c = 3g ...(2) ....(i)
Plane (1) is at constant distance 3p from the origin, so Also normal to plane will be perpendicular to line,
0 0 0 \ 3 × 1 – 5 × 3 + 2 × (– a ) = 0
+ + –1 Þ a=–6
a b c 1 1 1 1
3p = Þ + + = ...(3) From equation (i) we have, b = 7
2 2
æ1ö æ1ö æ1
ç ÷ +ç ÷ +ç
èaø èbø èc
EBD_7504
S-M-104 SOLUTIONS

26. (b) Let direction ratios of the line be a, b, c , then -2 l 2


Þ =
2a – b + c = 0 2
3l + 4l + 14 3
a – b – 2c = 0
7
a b c Þ 3 l 2 + 4 l + 14 = 3l 2 Þ l = -
i.e., = = 2
3 5 -1
\ Required equation of plane is
\ direction ratios of the line are (3, 5, – 1)
7
Any point on the given line is (2 + l, 2 - l, 3 - 2l ) . It (x + 2y + 3z – 2) – (x – y + z – 3) = 0
lies on the plane p if 2
or 5x – 11y + z = 17
2 (2 + l ) - ( 2 - l) + ( 3 - 2l ) = 4
R
i.e., 4 + 2l - 2 + l + 3 - 2l = 4 29. (d)
i.e., l = -1
N
\ the point of intersection of the line and the plane is
(1, 3, 5)
q/2
x -1 y - 3 z - 5 O X
\ equation of the required line is = =
3 5 -1
27. (c) Plane 2ax - 3ay + 4az + 6 = 0 passes through the mid
point of the centre of spheres Let the ray of light comes along
x-axis and strikes the mirror at
x 2 + y 2 + z 2 + 6 x - 8 y - 2 z = 13 and the origin.
Direction cosines of normal are
x 2 + y 2 + z 2 - 10 x + 4 y - 2 z = 8 respectively
1 1 1 q 1
center of spheres are (– 3, 4, 1) and (5, – 2, 1). Mid point ,- , =
so. cos
of centres is (1, 1, 1). 3 3 3 2 3
Satisfying this in the equation of plane, we get Let the reflected ray has direction cosines l, m, n then
2a - 3a + 4a + 6 = 0 Þ a = -2. l +1 1 2 1
28. (a) The plane passing through the intersection line of = Þ l = -1 = -
q 3 3 3
given planes is 2 cos
2
(x + 2y + 3z – 2) + l( x - y + z - 3) = 0
m+0 1 2 n+0 1 2
or (1 + l )x + (2 - l)y + (3 + l )z + (-2 - 3l ) = 0 =- Þm=- = Þn=
q 3 3 2 cos q 3 3
2 2 cos
Its distance from the point (3, 1, –1) is 2 2
3
2- 3+ 2 1
3(1 + l) + 1(2 - l ) - 1(3 + l) + (-2 - 3l ) 30. (a) sin q = =
2 4+9+4 3 51
\ =
(1 + l )2 + (2 - l )2 + (3 + l )2 3 Statement 1 is true, statement 2 is true by definition.

Speed Test-86

1. (d) æ B ö P [B Ç (A È B)] æ B ö P (A Ç B)
Pç =
è A È B ÷ø P (A È B) Pç
è A È B÷ø
= P (A) + P(B) - P(A Ç B)

P [B Ç A) È (B Ç B)] 0.2 0.2 1


= = = = = 0.25
P (A) + P(B) - P(A Ç B) 0.7 + 0.6 - 0.5 0.8 4
Given 2. (c) The probability p of getting a white ball in a single draw
Þ P(A Ç B) = 0.5 3 1
is = . If X is the number of white balls drawn, then
Þ P(A) - P(A Ç B) = 0.5 9 3
Þ P(A Ç B) = P(A) - P(A Ç B) = 0.7 - 0.5 = 0.2 X ~ B(n, p), with n = 4. Therefore the probability of getting
exactly one white ball is
3
æ2ö 32
ç ÷ =
è3ø 81
Mathematics S-M-105

3. (b) In a box, B1 = 1R, 2W; B2 = 2R, 3W and B3 = 3R, 4W 7. (c) We have


1 1 1
Also, given that, P(B1 )= , P(B2 ) = and P(B3 )=
2 3 6
æ R ö G
P(B2 )P ç ÷
æ B2 ö è B2 ø
\ Pç ÷= F
è R ø æR ö æ R ö æ R ö E
P(B1 )P ç ÷ +P(B 2 )P ç ÷ +P(B3 )P ç ÷
è B1 ø è B2 ø è B3 ø
1 2 2 Q EÇ FÇ G= f
´
3 5 15 14
= = = .
1 1 1 2 1 3 1 2 1 39
´ + ´ + ´ + + P (Ec Ç Fc Ç G)
2 3 3 5 6 7 6 15 14 P (E c Ç Fc / G) =
P (G)
1 1 1
4. (d) Here p = . q = 1 – p = 1 - = P (G) - P (E Ç G) - P(G Ç F)
2 2 2 =
n = 6, N = 64. P(G)
6- r
r
æ1ö æ1ö æ1ö
6
[From venn diagram Ec Ç Fc Ç G = G – E Ç G – F Ç G]
Then p( r ) = n C r p r q n - r = 6 C r ç ÷ .ç ÷ = 6Cr ç ÷
è 2ø è 2ø è 2ø P (G) – P (E) P ( G) – P(G) P (F)
=
1 6 P (G)
\ f ( r ) = Np( r ) = 64. 6 C r . = Cr
64 [Q E, F, G are pairwise independent]
6 = 1– P (E) – P (F) = P (Ec) – P (F)
Now å p(r) = p(3) + p(4) + p(5) + p(6) ¥
pq a
3
1 1
8. (d) We have, P(X ³ a ) = å pq x =
1- q
= qa
= ( 6 C 3 + 6 C 4 + 6 C 5 + 6C 6 ) = (2 6 - 6 C 0 - 6 C1 - 6 C 2 ) x =a
26 26
P[(X ³ a + b) Ç (X ³ a )]
1 42 21 Next, P(X ³ a + b | X ³ a ) =
= ( 64 - 1 - 6 - 15) = = P( X ³ a )
26 64 32
6
21 P ( X ³ a + b) q a + b b
= P (X ³ a ) = a = q = P(X ³ b)
\ f (r ) r ³3 = N å
3
p(r) = 64. = 42
32 q
P[(X = a + b)P(X ³ a )]
2 3 1 Also, P(X = a + b)X ³ a ) =
5. (c) P( A) = , P ( B) = , P ( A Ç B) = P( X ³ a )
5 10 5
2 3 P(X = a + b) pq a + b
P ( A¢) = 1 - P ( A ) = 1 - = = = = pq b = P(X = b)
5 5 P( X ³ a ) q a
3 7
P ( B¢ ) = 1 - P ( B ) = 1 - = 9. (c) Probability of n balls = 1 – P (A1 È A2 È A3 È ..... È An)
10 10 Where A1..... An the event that the letter is placed at
2 3 1 1 right envelope.
P ( A È B) = P ( A ) + P ( B ) - P ( A Ç B ) = + - = = 1 – [S P (Ai) – S P (Ai Ç Ak) + S P (Ai Ç Aj
5 10 5 2
1 1 Ç Ak) ...... + (–1)n–1 P (Ai Ç Aj Ç An)]
P ( A¢ Ç B¢ ) = P ( A È B)¢ = 1 - P ( A È B ) = 1 - =
2 2 (n – 1)!
Here, P (Ai) =
P ( A¢ Ç B¢ ) P ( A¢ Ç B¢ ) n!
\ P ( A¢ | B¢ ) × P ( B¢ | A¢ ) = P B¢ .
( ) P ( A¢ ) P (A1 Ç A2 Ç A3 Ç ....... Ç An) =
(n – r)!
n!
1/ 2 1/ 2 1 50 25
= . = ´ = Þ SA1 Ç A 2 Ç A3 Ç ......... Ç A n
7 /10 3 / 5 4 21 42
6. (c) Here mean = np = 6 ; variance = npq =2 é1 1 1 n –1! ù
= 1 – ê – + ......(–1)
2 1 2 ë 1! 2! 3! n! úû
\ q= = ; \ p = 1 - q = ; \ n =9
9 3 3 1 1 1 1
= – + – .......... + (–1) n
Now, probability of 5 £ x £ 7 is equal to p(5) + p(6) + p(7) 2! 3! 4! n!

= 9 C5 p 5 q 4 + 9 C6 p 6 q 3 +
EBD_7504
S-M-106 SOLUTIONS

10. (b) Let Ei = 0, 1, 2, ... n be the event that the bag contains
æ E ö
exactly i white balls then p (Ei) k2i P( E 3 ).Pçç ÷
è E 3 ÷ø
n =
r
where å ki2 = 1 Þ k=
n(n + 1)(2n + 1)
æ E ö æ E ö
P( E1 ).Pçç ÷÷ + P( E 2 ).Pçç
æ E
÷÷ + P( E 3 ).Pçç
ö
÷÷
i= 0 è E1 ø è E2 ø è E3 ø
Let A be the event that a ball drawn is white 1
´1
2 3 2 24
æ E ö p(En ) p(A En ) K.n .1 n = =
pç n ÷ = = = 1 1 1 1 1 29
è Aø n
æiö K n
n(n + 1) ö
2 ´ + ´ + ´1
å ki2 çè n ÷ø å (i)3 æç ÷
3 4 6 8 2
i=0 n t =0 è 2 ø 13. (c) Since, E and F are independent events
\ P ( E Ç F) = P ( E ) P ( F)
æE ö 4n
pç n ÷ = Þ P(E|F) P(F) = P(E) P(F) and P(F|E)P(E) = P(E)P(F)
è A ø (n + 1)2 Þ P(E|F) = P(E) and P(F|E) = P(F)
¥ n +1 Now, P ( E È F ) = P ( E ) + P ( F ) - P ( E Ç F )
¥
æ n ö æ1ö
11. (d) P(X < 1) = å pç X =
n + 1
÷= å
ç ÷ =
1 = P(E) + P(F) – P(E)P(F)
n =1 è ø n =1è 2 ø Þ 0.5 = 0.3 + P(F) – 0.3P(F)
2
P(F) (1 – 0.3) = 0.5 – 0.3
1 0.2 2
Similarly, P(X > 1) = P ( F) = =
2 0.7 7
¥ n +1 2
æ1 ö æ1ö 1
Also, Pç < X < 1÷ =
è2
ç ÷
ø n =2 è 2 ø
å =
4
< P(X > 1) \ P(E|F) – P(F|E)= P(E) – P(F) = 0.3 -
7
3 2 1
Note that P(X > 3/2) = 0 < P (X < 1) = - =
12. (a) Let E1 be the event that the answer is guessed, E2 be the 10 7 70
event that the answer is copied, E 3 be the event that the 2 3 2 æ3ö 2
2 k
æ3ö 2 7
examinee knows the answer and E be the event that the 14. (a) + ´ + ç ÷ ´ + ...... + ç ÷ . >
examinee answers correctly. 5 5 5 è5ø 5 è 5 ø 5 10

2 é 3 æ3ö æ 3ö ù 7
2 k
1 1
Given P( E1 ) = , P( E 2 ) = , Þ ê1 + + ç ÷ + ...... + ç ÷ ú >
3 6 5 ë 5 è5ø è 5 ø û 10
Assume that events E1, E2 & E3 are exhaustive. k
æ3ö
\ P(E1 ) + P (E 2 ) + P (E 3 ) = 1 1- ç ÷ k
2 7 æ3ö 7
´ è ø >
Þ 5 Þ 1- ç ÷ >
1 1 1 5 3 10 è5ø 10
\ P(E 3 ) =1 -P(E1 ) -P(E 2 ) =1 - - = . 1-
3 6 2 5
k
æE ö æ3ö 3
Now, P çç ÷÷ º Probability of getting correct answer by Þ ç ÷ < Þ k³3
è5ø 10
è E1 ø Hence minimum value of k = 3
1 æ A ö P(A Ç B) P(A È B) 1 - P(A È B)
guessing = (Since 4 alternatives) 15. (c) Pç ÷ = = =
4 è Bø P(B) P(B) P(B)
æ E ö 1
P çç ÷÷ º Probability of answering correctly by copying = P( X = r ) n
C r p r (1 - p) n -r (1 - p) n - 2r
= =
è E2 ø 8 16. (a) P( X = n - r ) n
C n - r p n - r (1 - p) r p n -2 r
æ E ö n -2r n -2r
and Pçç ÷ º Probability of answering correctly by knowing = 1
÷ æ 1- p ö æ1 ö æ1ö
è E3 ø = çç ÷÷ = çç - 1÷÷ and çç ÷÷ – 1 > 0
è p ø èp ø è pø
æ E3 ö \ ratio will be independent of n and r if
Clearly, ç ÷ is the event he knew the answer to the
(1/p)–1 = 1
è E ø
question given that he correctly answered it. Using Baye’s Þ p = 1/2
17. (a) Let A, B and C be the events that the student is
æ E3 ö successful in tests I, II and III respectively.
theorem P ç ÷
è E ø
1 - P( B)}P(C) + P(A)P(B)P(C)
Mathematics S-M-107

Since X and Y are independent events


æ 1ö 1 1
= p . q ç 1 - ÷ + p(1– q) +p.q Þ X+Y=3
è 2ø 2 2 Þ X = 0, Y = 3; X = 1, Y = 2; X = 2, Y = 1; X = 0, Y = 3
1 1 1 \ P (X + Y = 3)
= pq + p (1 - q ) + pq = 1 ( pq + p - pq + pq) = 1 ( pq + p) 5 7 5 7
2 2 2 2 2 æ 1 ö 7 æ1ö 5 æ 1ö 7 æ1ö
1 1 = 5C0 ç ÷ . C3 ç ÷ + C1 ç ÷ C2 ç ÷
\ = ( pq + p) Þ 1 = pq + p è 2ø è 2ø è 2ø è 2ø
2 2
5 7 5 7
18. (b) As 0.4 + 0.6 = 1, the man either takes a step forward or a æ1ö æ1ö æ1ö æ1ö 55
step backward. Let a step forward be a success and a step +5 C2 ç ÷ 7C1 ç ÷ + 5C3 ç ÷ 7 C0 ç ÷ = .
è2ø è2ø è2ø è 2 ø 1024
backward be a failure. 21. (a) We have,
Then, the probability of success in one step
P(| X - 4 | £ 2) = P( -2 £ X - 4 £ 2) = P( 2 £ X £ 6)
2
= p = 0.4 = = 1 -[P(X = 0) + P(X = 1) + P(X = 7) + P(X = 8)]
5
3 é æ1ö ù
8 8 8 8
The probability of failure in one step = q = 0.6 = . æ1ö æ1ö æ1ö
5 = 1 - ê 8 C 0 ç ÷ + 8 C1 ç ÷ + 8 C 7 ç ÷ + 8 C 8 ç ÷ ú
êë è2ø è2ø è2ø è 2 ø úû
In 11 steps he will be one step away from the starting point if
the numbers of successes and failures differ by 1. 8
So, the number of successes = 6 æ1ö 18 119
The number of failures = 5 = 1 - ç ÷ (1 + 8 + 8 + 1) = 1 - 8 =
2
è ø 2 128
or the number of successes = 5,
22. (d) E1 : Event that first drawn ball is red, second is blue
The number of failures = 6
and so on.
\ the required probability = 11 C 6 p 6 q 5 + 11C5 p 5 q 6 E2 : Event that first drawn ball is blue, second is red and so
on.
6 5 5 6
11 æ2ö æ3ö æ2ö æ3ö 4 4 3 3 4 4 3 3
= C 6 ç ÷ .ç ÷ + 11C 5 ç ÷ .ç ÷ \ P(E1) = ´ ´ ´ and P(E2) = ´ ´ ´
è5ø è5ø è5ø è5ø 8 7 6 5 8 7 6 5
5 5 4 4 3 3 6
11! æ 2 ö æ 3 ö ì 2 3 ü P(E) = P(E1) + P(E2) = 2 × × × × =
= .ç ÷ .ç ÷ .í + ý 8 7 6 5 35
6 !5 ! è 5 ø è 5 ø î 5 5 þ 23. (c) Given that, P (X = 1) = P (X = 2)
5
11.10.9.8.7 2 5.35 æ 6 ö e –λ λ1 e – l l 2
= . 462 ´ ç ÷ \ = Þl=2
120 10 = è 25 ø 1! 2!
5
P ( E1 Ç E 2 ) e –2 (2)4 e –2 ´ 16 2
19. (b) P (E2/E1) = \ P ( X = 4) = = = 2
P (E1 ) 4! 24 3e

1 P ( E1 Ç E 2 ) 24. (b) [ p =
3
, n = 100 ; \ m = np = 3 ]
Þ = 100
2 14
1 e - m .m r e -e 31 3
Þ P (E1 Ç E2) = = P (E2) . P (E1/E2) \ from p(r) = we get P(1) = 1 = 3
8 r e
1
= P (E2) . = antilog[log3 - 3 log10 e] = anti log[0.4771 - 3 ´ 0.4343]
4
1 = anti log( 1 .1742) = 0.15 (nearly).
Þ P (E2) = 25. (b) P(E) = P ( 2 or 3 or 5 or 7)
2
1 = 0.23 + 0.12 + 0.20 + 0.07 = 0.62
Since P (E1 Ç E2) = = P (E1) . P (E2) P ( F ) = P(1 or 2 or 3) = 0.15 + 0.23 + 0.12 = 0.50
8
Þ events are independent P ( E Ç F ) = P(2 or 3) = 0.23 + 0.12 = 0.35
1 1 1 5 \ P( EUF ) = P( E) + P( F ) - P( E Ç F )
Also P (E1 È E2) = + - =
2 4 8 8 = 0.62 + 0.50 - 0.35 = 0.77
Þ E1 & E2 are non-exhaustive 26. (c) Since, India's second win occurs at the third test.
æ 1ö 1 1 Therefore, the sample space is
20. (b) B ç 5, ÷ Þ n = 5, p = , q = = [LWW, WLW]
è 2ø 2 2
where, L = losing the test
æ 1ö
B ç 7, ÷ Þ n = 7,
è 2ø at the 3rd test)
EBD_7504
S-M-108 SOLUTIONS

= P (LWW) + P (WLW) This value is very small so the Binomial probabilities are
= P (L) P (W) P (W) + P (W) P (L) P (W) approximated by Poisson probabilities then
(Q Prob. from match to match is independent). m = np = 10 ´ 0.2 = 2
æ 1 1 1ö æ 1 1 1 ö \ The probability that not more than one defective is found.
= ç ´ ´ ÷ + ç ´ ´ ÷ (given)
è 2 2 2ø è 2 2 2ø = P(k = 0) + P(k = 1) = e- m + me- m = e -2 + 2e -2 = 3e -2
1 1 2 1 29. (d) A and B will agree in a certain statement if both speak
= + = = truth or both tell a lie. We define following events
8 8 8 4
27. (a) Let X be the number of defective items in the first sample E1 = A and B both speak truth Þ P(E1) = xy
of size n, and Y the number of defective items in the sample E2 = A and B both tell a lie
of size m. Then X ~ B(n, p) and Y ~ B(m, p). The batch will be Þ P (E2) = (1 – x ) (1 – y)
accepted in the following cases : E = A and B agree in a certain statement
(X = 0); (X = 1, Y £ 1); (X = 2, Y = 0). Clearly, P(E / E1 ) = 1 and P(E / E 2 ) = 1
The probabilities of these events are as follows :
The required probability is P(E1 / E ) . Using Baye’s theorem
P(X = 0) = qn ; P(X = 1, Y £ 1) = P(X = 1) P (Y £ 1)
= P(X = 1)[P(Y = 0) + P(Y = 1)] P(E1 )P(E / E1 )
P(E1 / E ) =
= n C1pq n -1 (q m + m C1pq m -1 ) = npq n -1 (q m + mpq m -1 ) ; P(E1 )P(E / E1 ) + P(E 2 )P(E / E 2 )
and P(X = 2, Y = 0) = P(X = 2) P(Y = 0) xy.1 xy
= =
1 2 m+n -2 xy.1 + (1 – x )(1 – y ).1 1 – x – y + 2 xy
= ( n C 2 p 2 q n - 2 )( m C 0 q m ) = n (n - 1)p q
2 30. (c) If binomial distribution be B (n, p) then
Adding these three results, we see that the probability that Mean = np = a and Variance = npq = b
the batch is accepted is
b a-b
n n -1 m m -1 n (n - 1) 2 m+ n - 2 We get q = and p = 1 - q =
q + npq (q + mpq )+ p q a a
2
28. (c) We suppose the distribution to be Binomial with æa -bö a2
So np = a Þ n ç ÷=a Þn =
n = 10, p = 0.2, q = 1 – p =0.8 è a ø a -b
\ The probability that not more than one defective is found.
a2
ænö Thus is a positive integer.
= P( k = 0) + P( k = 1) = q n + çç ÷÷p q n -1 a -b
è1 ø
= (0.8)10 + 10(0.2)(0.8)9 = (0.8)9 [0.8 + 2] = 2.8(0.8) 9 .

Speed Test-87

1. (d) a2 + a2 + 4b2 – 4ab = 2ac – c2 a+ x


Þ (a – 2b)2 + (a – c)2 = 0 or 1 = or x 3 = a + x, Þ x( 3 - 1) = a
x 3 A
which is possible only when
a – 2b = 0 and a – c = 0 a
or x =
3 -1 x
a b c
or = = = l (say)
1 1/ 2 1 B 30° C
\ a = l , b = l/2, c = l
a a
a 2 + c2 - b2 45°
\ cos B = D
2ac E
l2 Therefore, height of the tower,
l2 + l 2 -
= 4 = 1- 1 = 7 a
2 a+ x = a+
2l 8 8 3 -1
AC 1 x é 3 - 1 + 1ù
2. (c) In DABC , tan 30° = or = a 3 3 + 1 a (3 + 3)
BC 3 BC = aê ú= ´ =
ë 3 -1 û 3 -1 3 +1 2
a+x
or BC = x 3 and in D ADE , tan 45° =
DE
Mathematics S-M-109

D 7. (d) x = h cot 3 a …(i)


3. (b) We know, r = where, r = In-radius and (x + 100) = h cot 2a …(ii)
s
D = area of triangle (x + 300) = h cot a …(iii)

= s(s - a )( s - b)(s - c ) , a, b, c are the sides of triangle


Therefore
2
1 ( s - a) 2 1 ( s - b)2 1 = ( s - c)
= , 2= , 2
r3 D2
r12 D2 r2 D2
1 s2
and =
r2 D2
From (i) and (ii),
1 1 1 1 – 100 = h (cot 3a – cot 2a),
Consider 2 + 2 + 2 + 2
r1 r2 r3 r From (ii) and (iii),
s 2 + ( s - a)2 + ( s - b) 2 + ( s - c ) 2 – 200 = h (cot 2a – cot a),
=
D2 æ sin a ö
100 = h ç
4 s + a + b + c 2 - 2 s(a + b + c )
2 2 2
a 2 + b2 + c 2 è sin 3a sin 2a ÷ø
= = .
D2 D2
æ sin a ö
cot A + cot B sin (A + B) sin C 200 = h ç
4. (a) = . è sin 2a sin a ÷ø
cot C sin A sin B cos C
sin 3a 200 sin 3a
sin 2 C c2 2ab or = Þ =2
= = . 2 sin a 100 sin a
sin A sin Bcos C ab a + b2 - c 2
Þ 3sin a - 4sin 3 a - 2sin a = 0
2 2
2c 2c 9 m Þ 4sin 3 a - sin a = 0 Þ sin a = 0
= = = =
a 2 + b2 - c 2 17c 2 4 n 1 æ pö p
- c2 or sin 2 a = = sin 2 ç ÷ Þ a =
9 4 è 6ø 6
Þ (m + n) = 9 + 4 = 13 p 3
5. (a) Let AE is a vertical lamp-post. Given AE = 12 m Hence, h = 200sin = 200 = 100 3 m
3 2
AE D
tan 45° = C 8. (b) sin 2 A + sin 2 B + sin 2 C
AC E 45°
Þ AC = AE = 12 m = 1 - cos 2 A + 1 - cos 2 B + sin 2 C
AE = 2 - cos 2 A - cos(B + C)cos(B - C)
Now, tan 60° =
AB = 2 - cos A[cos A - cos(B - C)]
AE 60°
Þ AB = = 4 3m
A B = 2 - cos A[ - cos(B + C) - cos(B - C)]
3
Consider, = 2 + cos A.2 cos Bcos C
BC = AC 2 - AB 2 = 144 - 48 = 96 = 4 6
\ sin2A + sin2B + sin2C – 2cos A cos B cos C = 2
500
Area = AB ´ BC = 4 3 ´ 4 6 = 48 2 sq.cm. 9. (b) d2 = h cot 30° = 500 3, and d1 =
3
3
6. (d) We are given 5 cos A + 3 = 0 Þ cos A = -
5
Now A being the angle of DABC , sin A cannot be negative. h
3 4 4
So, cos A = - Þ sin A = and then tan A = -
5 5 3 60°
30°
8 16
\ sin A + tan A = - and sin A tan A = - d1 d2
15 15
Hence, sinA and tan A are the roots of the equation 500 2000
Diameter d = 500 3 + 3= m
3 3
æ 8 ö æ 16 ö 10. (d) Consider a DABC with sides a, b, c then by cosine
x 2 + ç ÷x - ç ÷ = 0 or 15x 2 + 8x - 16 = 0
è 15 ø è 15 ø formula, we have
EBD_7504
S-M-110 SOLUTIONS
A
Þ H 2 - 4dH + 3d 2 = 0 Þ H 2 - 80 H + 3(400) = 0
Þ H = 20 or 60 m
c b 3 2 3a
13. (b) We have D = a ,s=
4 2
D a abc a3 a
B a C \ r= = , R= = =
s 2 3 4D 3a 2
3
c2 + a 2 - b2
cos B = D 3 / 4a 2 3
2ca and r1 = = = a
s-a a/2 2
a 2 + c2
But given 2b2 = a2 + c2 Þ b2 = aa 3
2 Hence, r : R : r1 = : : a =1: 2:3
2 3 3 2
æ a 2 + c2 ö 14. (c) d = h cot 30° – h cot 60° and time = 3 min.
c 2 + a2 - ç ÷
è 2 ø a 2 + c2 h(cot 30° - cot 60°)
\ cos B = = \ Speed = per minute
2ca 4ca 3
sin 3B 3sin B - 4sin3 B
Consider =
sin B sin B h
= 3 – 4 sin 2 B = 3 – 4 (1 – cos2 B) = – 1 + 4 cos2 B
60° 30°
2 2
é a 2 + c2 ù 1 æ a 2 + c2 ö d
Þ -1 + 4 ê ú = -1 + ç
ë 4ca û 4 è ca ÷ø It will travel distance h cot 60° in
h cot 60° ´ 3
-4(ac)2 + (a 2 + c 2 ) 2 = 1.5minute
= h(cot 30° - cot 60°)
4( ac) 2
h 1 h
15. (d) tan 30° = , =
2 ö2 x + 60 3 x + 60
(a 2 + c 2 ) 2 - 4a 2c 2 æ c2 - a
= =ç ÷ .
4(ac)2 è 2ac ø x + 60 = 3h, x = 3h - 60
A
40
11. (d) From D O1 AB, tan 45° = Þ x = 40m
x

60° 30°
B x C 60m D
h h
tan 60° = , x =
x 3
h
y Þ 3h - 60 = Þ h = 30 3 = 52 m (Approx.)
From D AO2 B, cot 30° = 3
40 16. (b) From cosine and sine formula, we have
Þ y = 40cot 30° = 40 3
b2 + c2 - a 2 sin A sin B sin C
cos A = and = = = k.
Distance between the men = 40 + 40 3 = 109.28 m 2bc a b c
12. (b) From the given figure we have Given, in any DABC,
H H sinB
= tan a and = tan b 2H
cos A =
3d d 2sin C
3 From the above given formula, we have
H H sin B = bk, sin C = ck
-
1 d 3d bk b
tan(β - α) = = H \ cos A = =
2 H2 2 ck 2 c
1+ 2 q b 3
Put the value of cos A in the formula, which gives
3d a
Mathematics S-M-111

Þ c2 = a 2 Þ c = a A
17. (b) If tan A, tan B, tan C are in A.P. then
2 tan A = tan A + tan C Þ 3 tan B = tan A tan B tan C
Q tan B ¹ 0 Þ tan A tan C = 3
A 60° 45°
B 1 D 3 C
From D ABD , (By sine Rule)
sin ÐBAD sin 60°
= …(i)
H O BD AD
Now, in D ADC ;
…(ii)
sin ÐCAD sin 45°
B C =
N D DC AD
Now in DABC, (i) divided by (ii)
OH is parallel to BC, so OD = HN
Þ R cos A = 2R cos B cos C sin ÐBAD 3 2 1 3 1
= ´ ´ = =
Þ - cos(B + C) = 2cos Bcos C sin ÐCAD 2 1 3 2 ´3 6
21. (b) Let QT be the tower of height (h) in D PRS. Now, each
Þ sin Bsin C = 3cos B cos C
triangle QPR, QRS, QSP are equilateral. Q
Þ tan B tan C = 3 Þ tan A + tan B + tan C = 3 tan A Thus QP = QS = QR = a.
Þ tan B + tan C = 2 tan A In D QTP,, R 60°
Þ tan B, tan A, tan C and in A.P.. QP2 = QT2 + PT2
2
18. (a) Since A + B + C = p æa ö
a 2 = h 2 + ç sec 30°÷ T
\ A + B = p - C Þ tan(A + B) = tan(p - C) è2 ø
S P
2 2
tan A + tan B a 4 a
Þ = - tan C a2 = h2 + . Þ a2 = h2 +
1 - tan A tan B 4 3 3

Þ
tan A + tan B a2 3a 2 - a 2
> 0 [Q angle C is obtuse \ tan C < 0] a2 -= h2 Þ = h2
1 - tan A tan B 3 3
p \ 2a2 = 3h2
But C is obtuse angle , so A and B will both be less than d
2
\ Both tan A and tan B are positive. 22. (c)
Hence from (1), 1 - tan A tan B > 0 Þ tan A tan B < 1
b 2(b / a) p+b
19. (b) tan a = , tan 2a = = H 1 km
a 1 - (b / a) 2
a P
2ba p+b p 60°
Þ 2 =
a - b2 a 30°

2ba 2 - a 2 b + b3
Þ = p b d = H cot 30° - H cot 60°
a2 - b2 a Time taken = 10 second
a
b(a 2 + b2 ) O a cot 30° - cot 60°
Þ p= \ speed = ´ 60 ´ 60 = 240 3
(a 2 - b 2 ) 10
20. (a) By sine rule, we have 23. (b) A + B = 180° – C = 90°
The sine of the angles are proportional to the lengths a = 2R sin A, b = 2R sin B, c = 2R sin C
of opposite sides.
a 2 - b 2 sin 2 A - sin 2 B
\ =
i.e., sin A = sin B = sin C a 2 + b 2 sin 2 A + sin 2 B
a b c sin( A + B) sin( A - B)
Given a D ABC in which ÐABD = 60° =
sin 2 A + sin 2 (90° - A )
BD 1
and ÐACD = 45° , also = [Q A + B = 90°]
DC 3
B
E
S-M-112 SOLUTIONS

sin 90° sin(A - B) æ tan A tan B tan C ö


= = sin(A - B) 1/ 3
sin 2 A + cos 2 A
Þ ç ÷ ³ (tan A tan B tan C)
è 3 ø
sin 38° sin( SPO)
24. (a) = Þ (tan A tan B tan C) 2 / 3 ³ 3
l 2.05
QP
2/3
æ1ö 1 1
Þ ç ÷ ³ 3 Þ ³ 33 / 2 Þ K £
èKø K 3 3
27. (b) B1B2 = h = (d tan 45° –d tan 30°) B1
Time taken = 10 min h
l B2
d æ 3 - 1ö
10° Rate = 4 =
10 çè 3 ÷ø 45°
30° B3
38° 40 3
Þd = = 20(3 + 3)m d
S 2.05 m O 3 -1
sin (180° - 38° - 90° - 10°) 2.05sin 38° p
= Þl = 28. (d) Q a2 = b2 + c2, then Ð A =
2.05 sin 42° 2
25. (c) OP = 4 \ a = 2R sin A = 2R
OA = OB = OC = OD = 3
a 2 + b2 + c 2 a 2 + a 2 2a 2 2(2R)2
Þ = = 2 = = 8
AP = BP = CP = DP = 32 + 4 2 = 5 R2 R2 R R2
ABCD is a square of side length 29. (b) (H + a) cot b = (H – a) cot a
6 a sin (a + b)
= =3 2 Þ H = sin (b - a )
2
C
P H-a
P a
a b
B O
Q
q P H
A D
C C
O [using componendo and dividendo]
B C 30. (c) Consider a triangle ABC.
Given, angles of a triangle are in the ratio 4 : 1 : 1.
1 3 Angles are 4x, x and x.
BC i.e., ÐA = 4x, ÐB = x, ÐC = x
q 2 3
sin = = 2 = Now, by angle sum property of D, we have
2 BP 5 5 2 ÐA + ÐB + ÐC = 180°
q 16 180°
2 Þ 4x + x + x = 180° Þ x = = 30°
\ cos q = 1 - 2 sin = 6
2 25 \ ÐA = 120°, ÐB = 30°, ÐC = 30°
We know, ratio of sides of DABC is given by
26. (a) Given A + B + C = p sin A : sin B : sin C = sin 120° : sin 30° : sin 30°
Þ tan A + tan B + tan C = tan A tan B tan C 3 1 1
= : : = 3 :1:1
Now, A.M ³ G.M. 2 2 2
3 3
=
Þ tan A + tan B + tan C ³ (tan A tan B tan C)1/ 3 Required ratio =
1 +1 + 3 2 + 3
.
3
HINTS & SOLUTIONS (PHYSICS – Subject-wise Tests)
Speed Test-88
1. (b) For the motion of both the blocks 4. (d) Here, distance between two slits,
m1a = T – mkm1g d = 1mm = 10–3m
m2g – T = m2a distance of screen from slits, D = 1 m
a wavelength of monochromatic light used,
T l = 500nm = 500 × 10–9m
mk m1g m 1 width of each slit a = ?
mk 2lD
Width of central maxima in single slit pattern =
a
m2 lD
a Fringe width in double slit experiment b =
d
10 l D 2 l D
So, required condition =
m2g d a
d 1
m 2g – m k m1g Þ a= = ´ 10–3 m = 0.2 mm
5D 5
a= m1 + m 2
5. (c) Before closing of the switch, i = 9 1A,
9
æ m 2 g – m k m1g ö
m2g – T = (m2) ç m + m ÷ V1 + V2 = 9 and 3V1 = 6V2,
è 1 2 ø \ V1 = 6V, V2= 3V.
solving we get tension in the string Charge q1 = 3 × 6 = 18 mC, and q2 = 6 × 3 = 18mC
The charge at the junction x, qi = 18 – 18 = 0.
m1 m 2 (1 + m k ) g
T= After closing the switch,
m1 + m 2 9
2. (a) U = a + bPV ......(a) i= = 1A, V = 3 × 1 = 3V and V = 6 × 1 = 6V,
9 1 2
In adiabatic change, q1 = 3 × 3 = 9 mC and q2 = 6 × 6 = 36 mC.
nR nR Now final charge at junction x,
dU = – dW = (T2 - T1 ) = (d T ) qf = – 9 + 36 = 27 mC.
g -1 g -1
The flow of charge from y to x,
nR Dq = qf – qi = 27 – 0 = 27mC.
Þ U = ò dU = g - 1 ò dT 6. (b) From the figure,
æ nR ö v = u x = 19.6 cos 30°
or U =ç T +a =
PV
+ a ......(b) u y = 19.6 cos 30° x
è g - 1ø÷
60°
g -1 /s
6m
where a is the constant of integration. 1 9. v
u=
Comparing (a) and (b), we get q=30° 60°
1 b +1 u x = 19.6 cos 30°
b= Þg = .
g -1 b v y = - u y + gt
3. (b) Given : Resistance R = 100 kilo ohm = - 19.6 sin 30° + gt
= 100 × 103 W
Capacitance C = 250 picofarad
= 250 × 10–12F
t = RC = 100 × 103 × 250 × 10–12 sec
= 2.5 × 107 × 10–12 sec vy –19.6 sin 30° + gt
= 2.5 × 10–5 sec tan60° = Þ Ö3 =
The higher frequency which can be detected with vx 19.6 cos 30°
tolerable distortion is 3(19.6 cos 30°) + 19.6 sin 30°
Þ t = = 4s
1 1 9.8
f = = Hz
2pma RC 2p ´ 0.6 ´ 2.5 ´ 10 -5 7. (c) As we known from Doppler's Effect

100 ´ 10 4 4 é v + v0 ù é 340 + 16.5 ù


= Hz = ´ 10 4 Hz = 10.61 KHz fapprent = f0 ê v - v ú = 400 ê ú
25 ´ 1.2p 1.2 p ë sû ë 340 - 22 û
This condition is obtained by applying the condition fapprent = 448 Hz
that rate of decay of capacitor voltage must be equal v = 22 m/s v0= 16.5 m/s
B
EBD_7504
S-PT-2 SOLUTIONS

2u 14. (a) Given i = 60°


8. m1u = m1
(a) + m2 v A= d=e
3 d = i + e – A Þ d = i ( Q e = A)
(By conservation of linear momentum)
æ A + dm ö
1 sin ç
Þ m1u = m 2 v ...... (i) è 2 ÷ø
3 m=
A
| v - v2 | sin
Also e = 1 2
| u 2 - u1 | Here angle of deviation is min.(Q i = e)
2u 5 æ 60° + 60° ö
Þ v- =u Þ v= u ...... (ii) sin ç
3 3 è ÷ø
2
m=
1 5 m 60° = 1.73
From (i) and (ii), m1u = m 2 u Þ 1 = 5 sin
3 3 m2 2
cd < cp < cf 15. (d) M.I. of complete disc about its centre O.
9. (a)
For diamagnetic substance cd is small and negative 1
ITotal = MR2 ...(i)
(10–5) 2
For paramagnetic substances cp is small and positive
(10–3 to 10–5)
For ferromagnetic substanes c f is very large R/2
O¢ Circular hole of
(103 to 105)
diameter R (radius = R/2)
q
10. (d) Since ftotal = f A + fB + fC = , R O
e0 Disc mass = M
where q is the total charge. radius = R
As shown in the figure, flux associated with the curved
surface B is f = fB
Let us assume flux linked with the plane surfaces A and
C be fA = fC = f' Mass of circular hole (removed)

( )
Therefore, M
= As M = pR 2t \ M µ R 2
q 1æ q ö 4
= 2f '+ f B = 2f '+ f Þ f ' = ç -f÷ M.I. of removed hole about its own axis
e0 2 è e0 ø
2
11. (c) The net force becomes zero atthe mean point. 1 æ M ö æ Rö 1
= çè ÷ø çè ÷ø = MR2
Therefore, linear momentum must be conserved. 2 4 2 32
\ Mv1 = (M + m)v2 M.I. of removed hole about O¢
k k æ k ö Iremoved hole = Icm + mx2 [about 0]
MA1 = ( M + m) A2 \ çv = A M ÷ 2
M m+M è ø MR 2 M æ Rö
= + çè ÷ø
A1 m+ M 32 4 2
Q A1 M = A2 M + m \ =
A2 M
MR 2 MR 2 3MR 2
= + =
-GMm GMm 1 2 2Gm æ 1 ö 32 16 32
12. (b) = + mv or ç 1- ÷ M.I. of complete disc can also be written as
2R R 2 R è 2ø
r r r ITotal = Iremoved hole + Iremaining disc
13. (b) The application of equation FB = q (V ´ B ) on the
3MR 2
element dl of the rod gives force on positive charge ITotal = + Iremaining disc ...(ii)
towards the outer end. Therefore electrons will move 32
towards pivoted end. From eq. (i) and (ii),
× × 1 3MR 2
MR 2 = + I remaining disc
2 32
× × ×
Þ Iremaining disc
dl

× × ®
V × MR 2 3MR 2 æ 13 ö
= - = ç ÷ MR2
× × × 2 32 è 32 ø
O
× × ×

× ×
Physics S-PT-3

n1Cv1 + n2Cv2 N0 N0
16. (a) For mixture of gas, Cv = NQ = =
4/2 4
n1 + n2 2
or population of R
3 1 5 5
4 ´ R + ´ R 6R + R æ N0 ö æ N 0 ö 9N 0
= 2 2 2 = 4 = 29 R ´ 2 = 29 R çè 4 N 0 - 4 ÷ø + çè N 0 - 4 ÷ø =
æ 1 ö 9 9´4 18 2
çè 4 + ÷ø
2 2 20. (c) V 2 = VR2 + (VL - VC )2 Þ VR=V=220 V
5R 1 7 R 220
n1C p1 + n2C p2 4´ + ´ Also i = = 2.2 A
and C p = 2 2 2 110
=
(n1 + n2 ) æ 1ö
çè 4 + ÷ø kze 2
2 21. (b) E = – 3.4 eV and r =
2E
7 angular momentum = mvr
10 R + R
= 4 = 47 R C p 47 R 18 1
mv2 = E = 3.4´ (10-19 ´ 1.6)
Þ
9 18 Þ\
Cv
= ´
18 29 R
= 1.62 2
2
Þ 2 v 2 = (9.1´ 10-31 )2 ´ 3.4´ 1.6 ´ 10-19
= 99.008 × 10–50
17. (a) A mv = 9.95028 × 10–25
Y = A.AB B.AB

Y2 = A.AB \ L = (9.95028 × 10–25)


æ 9 ´ 109 ´ 1´ (1.6 ´ 10-19 ) 2 ö
ç ÷
ç 2 ´ (3.4) ÷
Y1 = AB è ø
= 2.10 × 10–34 Js.
B 22. (c) In presence of friction a = (g sinq – mg cos q)
Y3 = B.AB \ Time taken to slide down the plane
By expanding this Boolen expression 2s 2s
t1 = =
Y = A.B + B.A a g (sin q - m cos q)
Thus the truth table for this expression should
be (a). 2s
18. (c) 30 Divisions of vernier scale coincide with 29 divi- In absence of friction t 2 =
g sin q
sions of main scales.
29 Given : t 1 = 2 t 2
Therefore, 1 V.S.D = M.S.D.
30 2s 2s´ 4
\ t12 = 4t 22 or =
29 g (sin q - m cos q) g sin q
Least count = 1 MSD – 1 VSD = 1 MSD – M.S.D sin q = 4 sinq – 4m cos q
30
1 1 3 3
´ 0.5° = 1 minute m= tan q = = 0.75 (since q = 45°)
= = 4 4
30 30
23. (a) If we consider the cylindrical surface to be a ring of
19. (b) Initially P ® 4N0
radius R, there will be an induced emf due to changing
Q ® N0
field.
Half life TP = 1 min.
TQ = 2 min. r df dB
Let after time t number of nuclei of P and Q are equal, ò E.dl = - dt = - A dt
that is
dB dB R dB
4N0 N0 4 1 Þ E (2pR) = - A = -p R 2 Þ E=-
= = dt dt 2 dt
; Þ 2t/1 = 4.2t/2
2t /1 2t / 2 2t /1 2t/2 \ Force on the electron
22.2t/2 = 2(2 + t/2) eR dB 1 eR dB
F = - Ee = Þ acceleration =
t t t 2 dt 2 m dt
Þ = 2 + Þ = 2 Þ t = 4 min
1 2 2 As the field is increasing being directed inside the paper,
hence there will be anticlockwise induced current (in
(4N 0 ) N0 order to oppose the cause) in the ring (assumed). Hence
NP = 4 /1
=
2 4 there will be a force towards left on the electrons.
at t = 4 min.
EBD_7504
S-PT-4 SOLUTIONS

24. (c) Initial diameter of tyre = (1000 – 6) mm = 994 mm, so


10V 4 B x(assume)
initial radius of tyre
10 x–10
994 2
R= = 497 mm 10 2 4
2
6 10V 20V
and change in diameter DD = 6 mm so DR = = 3 mm
2 10
20
After increasing temperature by Dq tyre will fit onto P
wheel O(assume)
Increment in the length (circumference) of the iron tyre
We have to find I.
g g Let potential of point P be 0. Potential at other points
DL = L ´ a ´ Dq = L ´ ´ Dq [As a = ]
3 3 are shown in the figure apply Kirchoff’s current law at
B where potential is assume to be x volt.
ægö x - 10 x - 10 x - 20 (x - 10) - 0
2pDR = 2pR ç ÷ Dq + + + =0
è 3ø
4 2 4 2
Þ x – 10 + 2x – 20 + x – 20 + 2x – 20 = 0
3 DR 3´ 3
ÞÞ Dq = = 35
g R 3.6 ´ 10 –5 ´ 497 35 20 -
Þ 6x = 70 Þ x = volt \ I = 3 = 25 A
Þ Dq ; 500°C 3 4 12
25. (b) Radius of circular path followed by electron is given
by, 1 1
29. (a) Rotational energy = I(w) 2 = (mK 2 )w 2
2 2
mu 2meV 1 2m
r= = = V 1
qB eB B e Linear kinetic energy = mw2 R 2
2
B2 r 2 e \ Required fraction
Þ V= = 0.8V
2m 1
(mK 2 )w2 K2
For transition between 3 to 2. = 2 = 2
1 1 K + R2
æ 1 1 ö 13.6 ´ 5 (mK 2 )w 2 + mw 2 R 2
E = 13.6 ç - ÷ = = 1.88eV 2 2
è 4 9ø 36
2 2
Work function = 1.88 eV – 0.8 eV = 1.08 eV » 1.1eV R
5 2
26. (d) at = g sin (90° – q) = cos q = = 2 2 2
2 2 7 (Q For a solid sphere, K = R )
v= 2gh = 2 gl sin q R + R2 5
5

v2 2 gl sin q 1 T
\ ac = = = 2g sin q 30. (a) For fundamental mode, f =
2l m
l l
Taking logarithm on both sides, we get
a= ac 2 + at 2 = g 3sin 2 q + 1 . æ Tö
æ 1ö
log f = log ç ÷ + log ç ÷
Fe O è 2l ø è mø
27. (c) From figure tan q = ;q
mg æ1 ö 1 æTö
l
q = log çè ÷ø + log ç ÷
kq 2
x 2l 2 èmø
=
2
x mg 2l æ1ö 1
or log f = log ç ÷ + [log T - log m]
or x3 µ q2 …(i) q q è 2l ø 2
or x3/2 µ q …(ii) x
Differentiating both sides, we get
Differentiating eq. (i) w.r.t. time df 1 dT
= (as l and m are constants)
dx dq dq f 2 T
3x2 µ 2q but is constant dT df
dt dt dt Þ = 2´
so x2(v) µ q Replace q from eq. (ii) T f
x2(v) µ x3/2 or v µ x–1/2 Here df = 6
f = 600 Hz
28. (a) The simplified circuit is
Physics S-PT-5

Speed Test-89
m0i1 m i m 5
1. (d) B1 = – + 0 2 = 0 (i2 – i1 ) = 2.303 [log10 10 – log10 4]
2 pr / 2 2 pr / 2 pr L
= 6 ´ 10 –6 T = 2.303 [1.0000 – 0.6021] = 2.303 × 0.3979 = 0.9164
...(i) 5
When the current is reversed in i2, \ L= = 5.5 H
0.9164
m0 (i1 + i2 ) m (i + i ) 5. (b) As the capacitor offers infinite resistance to steady
B2 = – = – 0 1 2 = 3´ 10–5 T ...(ii)
current so, the equivalent circuit is
2pr / 2 pr
Dividing (ii) by (i) we get V R
A B
–(i1 + i2 ) 30
= =5
i2 – i1 6
2R
i1 3
– (i1 + i2) = 5i2 – 5i1 Þ 6i2 = 4i1; =
i2 2 2V I
2. (b) Considering the equilibrium of B V VC
–mBg + T = mBa A B
Since the block A slides down with constant speed. Using ohm’s law, current in circuit is
a = 0.
V
Therefore T = mBg 2V – V = I (2R + R) Þ I =
Considering the equilibrium of A, we get 3R
10a = 10g sin 30º – T – mN The voltage drop across
where N = 10g cos 30° V 4 4
N a VAB = 2V - ´2R = V ; VAB = V = V + VC
3R 3 3
T
1
A mN T a Þ Voltage drop across C = V..
30º 3
sin
10g B
2h
10g cos30º t AB =
6. (c)
mBg
g
10g
10 2h1 2e 2 h 2h
\ 10 a = g - T - m ´ 10 g cos 30 º but a = 0, T = mBg t BC + t CB = 2 =2 = 2e
2 g g g
0 .2 3 2h
0 = 5g - m B g - × 10 ×g Þ mB = 3.268 » 3.3 kg t BD + t DB = 2e 2
2 g
3. (a) The dimensions of
\ Total time taken by the body in coming to rest
EJ 2 ( ML2T -2 ) (ML2T -1 )2
= =1 2h 2h 2h
5 2 ( M 5
) ( M -1 3 -2 2
L T ) = + 2e + 2e 2 + .........
M G g g g
So, it represents dimensions quantity like angle.
E 5 2h 2h
4. (a) I0 = = = 0.1 A = + 2e [1 + e + e 2 + .........]
R 50 g g
I = 60 mA = 60 × 10–3 A, t = 0.1
2h 2h 1 = 2h é1 + e ù æ 1 + e ö
æ - tö
R + 2e ´ =t
g êë1 - e ûú çè 1 - e ÷ø
=
Now, I = I0 ç1 - e L ÷ g g 1- e
çè ÷ø 7. (d) L = 2p R
x x¢
æ L
- ö
æ 5
- ´ 0.1 ö
50
ÞR=
\ 60 × 10 = 0.1 ç 1 - e
–3 L ÷ = 0.1 1 - e ÷
ç L 2p
çè ÷ø çè ÷ø
æ mR 2 ö O
or 1 – e–5/L = 0.6 I = çè + mR 2 ÷
2 ø
4
\ e–5/L = 1 – 0.6 = 0.4 = or e5/L = 10/4 3 3 Lö
2
3 rL3
÷ =
T pø 8 p2
EBD_7504
S-PT-6 SOLUTIONS

u 2 sin 2 q 1 q
H1 = E= .
8. (a) 4p Î0 r 2
2g

u 2 sin 2 (90° - q) u 2 cos 2 q 1 pr0 r 3 æ 5 r ö r r æ5 r ö


and H 2 = = = . 2 ç - ÷= 0 ç - ÷
2g 2g 4pÎ0 r è 3 R ø 4 Î0 è 3 R ø
12. (b) Young's modulus of wire does not vary with dimensions
u 2 sin 2 q u 2 cos2 q (u 2 sin 2q ) 2 R2 of wire.
H1H 2 = ´ = =
2g 2g 16 g 2 16 IC
13. (d)
\ R = 4 H1 H 2
RC O/P = 2 Volt
9. (d) For path ab : ( DU )ab = 7000 J
By using DU = mCV DT Input RB
5
7000 = m ´ R ´ 700 Þ m = 0.48
2
For path ca :
The output voltage, across the load RC
( DQ)ca = ( DU )ca + ( DW )ca ...(i) V0 = IC RC = 2
Q ( DU )ab + ( DU )bc + ( DU )ca = 0 The collector current (IC)
Q 7000 + 0 + ( DU )ca = 0 Þ ( DU )ca = -7000 J ..(ii) 2
IC = = 10-3 Amp
Also ( DW )ca = P1 (V1 - V2 ) = mR (T1 - T2 ) 2 ´ 103
= 0.48 × 8.31 × (300 – 1000) = – 2792.16 J ... (iii) IC
Current gain (b) = = 100
On solving equations (i), (ii) and (iii) IB
( DQ )ca = - 7000 - 2792.16 = - 9792.16 J » - 9800 J
IC 10-3
GM e m 1 2 IB = = = 10 -5 Amp
GM e m 1 2 100 100
10. (c) – + mvi = – + mv f
10 R e 2 Re 2 Input voltage (Vi)
Vi = RB IB = 1 × 103 × 10–5 = 10–2 Volt
2 2GM e æ 1ö Vi = 10 mV
\ v 2f = vi + ç1 - ÷
R e è 10 ø 14. (c) For the l = 975 Å
11. (a) Let us consider a spherical shell of radius x and 1 æ 1 1 ö
thickness dx. = Rç 2 - 2 ÷
l çn ÷
Charge on this shell è 1 n2 ø
æ5 xö 2
where R is the Rydberg constant
dq = r.4p x 2 dx = r0 ç 4 - R ÷ .4px dx Solving we get n 2 = n = 4
è ø (Q n1 = 1 ground state)
\ Total charge in the spherical region from centre to r (r Therefore number of spectral lines
< R ) is
n(n - 1) 4(4 - 1)
r = = =6
æ5 xö 2 2
q = ò dq = 4 pr0 ò ç - ÷ x 2 dx
0
è4 Rø 15. (b)
h y
x
A B
dx
x
pd 2 p(nd )2
x´ = y
4 4
2
\ x = n y
From Pascal’s law
rw gh = r Hg g ( x + y)
é 5 r3 1 r 4 ù
3æ5 r ö
= 4pr0 ê 4 . 3 - R . 4 ú = pr0r ç - ÷ or 1 × gh = sg ( n 2 y + y )
ëê ûú è3 Rø
\ Electric field at r,
Physics S-PT-7

16. (a) As we know, hu = hu0 + K.E.max N 01


hc hc N1 =
=+ eVs (2)t/20
or lincident l 0 The amount of A2, which remains, after t years
when lincident= l, Vs = 3V
N 02
and for lincident = 2l, Vs = 1V. N2 =
(2)t/10
hc hc
\ = + 3eV ...(i) According to the problem
l l0 N1 = N2
hc hc 40 160 æ t ö
and = + 1eV ...(ii) = ç - 2÷
2l l 0 ; t/20
= 2è 10 ø
t/20
(2) (2)t/10 2
On simplifying (i) and (ii)
t t t t
2hc 1 hc = -2; - =2
= Þ l 0 = 4l . 20 10 20 10
l0 2 l
t
hT0 =2
17. (b) hT0 20
Q= ò CdT = a ln
T0
= a lnh
t = 40 s
T0
22. (a) Given M = 8 × 1022 Am2
R d = Re = 6.4 × 106m
DU = CV DT = ( h - 1)T0
g -1 m0 2M
Earth’s magnetic field, B = .
é h - 1ù 4p d3
W = Q - DU = a ln h - ê ú RT0
ë g - 1û 4p´10-7 2 ´ 8 ´ 1022
= ´
2p 4p (6.4 ´ 106 )3
18. (b) Here, k = , w = 2pu
l
@ 0.6 Gauss
k 2p / l 1 1 23. (b) Total length of sonometer wire, l = 110 cm = 1.1 m
\ = = = (Q c = u l)
w 2pu pu c Length of wire is in ratio, 6 : 3 : 2 i.e. 60 cm, 30 cm,
where c is the speed of electromagnetic wave in vacuum. 20 cm.
It is a constant whose value is 3 × 106 m s–1 Tension in the wire, T = 400 N
19. (a) Here, Mass per unit length, m = 0.01 kg
Minimum common frequency = ?
x = x0 cos (wt – p / 4 )
As we know,
dx æ pö
\ Velocity, v = = - x0 w sin ç w t - ÷ 1 T 1000
dt è 4ø Frequency, n = = Hz
Acceleration, 2l m 11
1000
dv 2 æ pö Similarly, n1 = Hz
a= = - x0w cos ç wt - ÷ 6
dt è 4ø 1000
n2 = Hz
é æ pö ù 3
= x0 w 2 cos ê p + ç wt - ÷ ú 1000
ë è 4øû n3 = Hz
2
æ 3p ö Hence common frequency = 1000 Hz
= x0 w 2 cos ç wt + ÷ ...(a) 24. (c) Frequency deviation, Df = 25 kHz
è 4ø
Frequency of modulating signal, f m = 5 kHz
Acceleration, a = A cos (wt + d) ...(b)
Comparing the two equations, we get Df 25
Modulation index, m f = = =5
3p fm 5
A = x0w2 and d = . Carrier swing = 2 × Df = 2 × 25 kHz = 50 kHz
4
20. (c) Impedance at resonant frequency is minimum in series 25. (c) Given
LCR circuit. C P - CV = 5000 J / mole °C .......(i)
CP
2 = 1.6 .......(ii)
æ 1 ö CV
So, Z = R 2 + ç 2 pfL - ÷
è 2pfC ø From Equation (i) & (ii),
C P C V 5000 5000
21. (d) Let, the amount of the two in the mixture will become Þ - = Þ 1.6 - 1 =
equal after t years. CV CV CV CV
The amount of A
EBD_750
S-PT-8 SOLUTIONS

5000 Net resistance between Q & R


Þ CV = = 8.33 ´103 60 ´ 30
0.6 = R¢ = + 30 = 50 W
Hence C P = 1.6 C V = 1.6 ´ 8.33 ´ 103 60 + 30
So, equivalent resistance
CP = 1.33 ´ 104
( 2n + 1) l D 50 ´ 50
26. (c) x = R eq = + 50 = 75W
2a 50 + 50
( 4 + 1) D ´ 6500Å 3 1
For red light, x = Current in the circuit is I = = A
2a 75 25
( 6 + 1) D
1
For other light, x = ´ lÅ Potential drop across R1 = 50 ´ = 2V
2a 25
x is same for each.
5 \ Potential difference across PS or QR is same and given
\ 5 ´ 6500 = 7 ´ l Þ l = ´ 6500 = 4642.8 Å. by = 3 – 2 = 1 volt
7
-mmv 2 1
Current across QR = A
27. (b) ma = 50
r
p /2 Potential drop across R5 is given by
v
dv –mmv 2 dv –(rd q)
or mv
ds
=
r
or ò v
= ò r
1
´ 30 = 0.6 V
v0 0 50
-mp Hence, potential drop across R4 is
\ v = v0
e r = 1 – 0.6 = 0.4 V
28. (d) The focal length of the lens 30. (a) Area of the loop = 0.1 × 0.1 = 0.01 m 2
1 1 1 1 1 20 + 1 21 240 df - d
= - = + = = ; f = cm e=- = (BA)
f v u 12 240 240 240 21 dt dt
Magnitude of emf
æ 1ö æ 1 ö 1
Shift = t ç 1 - ÷ ; 1 ç 1 - ÷ = 1´ dB
è mø è 3/2 ø 3 e=A = (0.01m 2 ) (0.02 T / s) = 2 ´ 10-4 V
dt
Now v' = 12 - 1 = 35 cm Resistance of the loop is
3 3
Now the object distance be u'. l 1.7 ´ 10-8 ´ 40 ´ 10-2
R= r = = 2.16 × 10–3 W
1
=
3
-
21 1 é 3 21 ù 1 1 é 48 - 49 ù
= - =
A 3.14 ´ 10-6
;
u' 35 240 5 êë 7 48 úû u' 5 êë 7 ´ 16 úû Current induced in the loop
u' = –7 ×16 × 5 = – 560 cm = – 5.6 m
e 2 ´ 10 -4 V
50 W I= = = 9.3´ 10 -2 A
29. (a) P Q R 2.16 ´ 10 -3 W
R1 R3 60 W R 4 30 W
+
3V R2 50 W
– R5 30 W

S R

Speed Test-90
1. (c) [at] = [F] amd [bt2] = [F]
Þ [a] = [MLT–3] and [b] = [MLT–4] a 2p T
y= = a cos w t = a cos t or t = s
2. (b) At a particular time, two values of displacement are not 2 T 6
possible. So time to complete 3/8th oscillation
T T T 5T
= + =
3. (c) Time to complete 1/4th oscillation is s. Time to 4 6 12
4
1 4. (c) I1 w1 : I 2 w2 or M K12 w1 : M K 2 2 w2
complete th vibration from extreme position is
8
K æw ö
obtained from
Physics S-PT-9

5. (c) Work function of aluminium is 4.2 eV. The energy of 13. (b) The bullets are fired at the same initial speed
two photons can not be added at the moment photons
collide with electron all its energy will be dissipated H u 2 sin 2 60º 2g sin 2 60º ( 3 / 2)2 3
= ´ = = =
or wasted as this energy is not sufficient to knock it H¢ 2g u 2 sin 2 30º sin 2 30º (1/ 2)3 1
out. 14. (d) Total resistance of the circuit
Hence emmission of electron is not possible. = 4000 + 400 = 4400 W
6. (a) In case of L-C circuit, angular frequency of oscillation
V 440
1 Current flowing i = = = 0.1 amp.
w= R 4400
LC Voltage across load = R i
7. (a) Current flowing through the conductor, = 4000 × 0.1 = 400 volt.
I = n e v A. Hence 16. (c) Phase difference = f
2 l l
4 nevd1 p(1) vd 4 ´ 4 16 Path diff = ´ phase diff . = f
= or 1 = = . 2p 2p
1 nev d p(2)2 vd2 1 1
2
é 1 1 ù
R 3 17. (b) E = Rhc ê - ú
2
8. (c) Since = 6.7 Þ CV = R, hence gas is êë n1 n 2 2 úû
CV 2
E will be maximum for the transition for which
monoatomic.
é 1 1 ù
Dl ê 2 - 2 ú is maximum. Here n 2 is the higher
9. (b) = a D T = 10 -5 100
´ = 3
10
l êë n1 n 2 úû
Dl energy level.
´ 100% = 10 -3 100
´
l é 1 1 ù
-1
Clearly, ê - ú is maximum for the third
= 10 =0.1% êë n12 n 2 2 úû
10. (d) Force on M = Mg
Reaction force= Ma transition, i.e. 2 ® 1 . I transition represents the
absorption of energy.
force of friction = mR = m.Ma ur ur
18. (c) Vector ( E ´ B )
mMa 19. (d) Velocity after the collision
10 ´10 + 5 ´ 0 100 20
= = = m / sec .
15 15 3
20. (c) The centre of mass remains at rest because force of
attraction is mutual. No external force is acting.
3R ´ 289 æ 3RT ö
21. (c) v oxg . = çè vrms = M ÷ø
32
3R ´ 400
vH = so vH = 2230.59 m/sec
Mg 2
22. (c) For the observation of interference phenomenon, the
Force of friction will two source must be coherent & must have same
balance the weight. So frequency.
23. (a) G = 15W, ig = 4 mA, i = 6 A
g
mMa ³ Mg ; m ³ Required shunt,
a.
11. (b) From the figure, it is clear that angle of incidence on æ ig ö æ 4 ´ 10-3 ö
the reflecting surface is 45°. So, critical angle for S= ç ÷ G = èç 6 - 4 ´ 10-3 ø÷ ´ 15
è i - ig ø
glass air interface should be 45°.
Now, from the formula 4 ´ 10-3
= ´ 15 = 0.01W = 10mW (in parallel)
1 1 5.996
sin C = Þ sin45° =
m m Fl Fl mgl
24. (c) Y= Þ Dl = =
ADl YA YA
m = 2 = 1. 4
f0
12. (c) For a single negative point charge, electric lines of force 25. (a) =9, \ f0 = 9f e
fe
are radial and inwards.
\ f0 = 18 cm
EBD_7504
S-PT-10 SOLUTIONS

26. (a) Let pressure outside be P0


2T
\ P1 ( in smaller bubble ) = P0 +
r
sin q = 2la
2T
P2 ( in bigger bubble ) = P0 + ( R > r) l
R sin q = a
\ P1 > P2 a q o sin q = 0
hence air moves from smaller bubble to bigger bubble. –l
sin q = a
27. (a) Given : T/2 = 0.5 s
\ T = 1s Intensity
sin q = –2l
a
1 1
Frequency, f = = = 1Hz D
T 1
If A is the amplitude, then
2A = 50 cm Þ A = 25 cm. Screen position of various minima for Fraunhoffer
28. (a) Width of central maximum diffraction pattern of a single slit of width a.
2 l D 2 ´ 6250 ´ 10 -10 ´ 0.5 29. (d) In a p-type semic-onductor germanium is doped with
= = trivalent element. All the elements, like aluminium,
a 2 ´ 10 -4
boron & gallium are trivalent. So, the correct
= 3125´ 10 -6 m = 312.5 ´ 10 -3 cm. alternative is (d).

Speed Test-91
1. (a) Number of significant figures in 23.023= 5 Bulk Modulus = ¥
Number of significant figures in 0.0003 = 1 [As liquid is uncompressible, ΔV = 0 ]
Number of significant figures in 2.1 × 10–3
=2 W W
7. (a) x= ÞV = Þ W = QV
2. (c) New K.E., E' = 4E Q Q
p = 2 mE and p ¢ = 2 mE ¢ 8. (d) Acceleration is the same everywhere equal to 'g'.
9. (a) Due to the charge inside a sphere of radius r only.
p 2m ´ 4E 10. (d) Applying Snell's law at P,
= =2
p 2mE
1 sin i µ2
µ2 = = ...(1)
p' sin r1 µ1
-1 = 2 - 1 [on substrating 1 in both sides.]
p m1 m2 m3 m4
p- p r1 r2 i
´ 100 = (2 - 1) ´ 100 = 100% r2 R
p Q
i1 P r1
3. (c) If T is the tension in the string , Then
T = mg (For outer masses)
2T cos q = 2 mg (For inner mass)
Applying Snell's law at Q,
Eliminating T, we get
2 sin r1 m3
2(mg ) cos q = 2 mg m3 = = ...(2)
sin r2 m 2
or cos q = 1/ 2 Þ q = 45 ° Again applying Snell's law at R
1 æ 1 ö2 3 sin r2 m 4
4. (b) S1 = g ç ÷ , S1 + S2 = g(1)2 m4 = = ...(3)
2 è2ø sin i m3
Multiplying (i), (ii) and (iii), we get
2
1 æ3ö µ4 = µ1
g ç ÷ = S + S2 + S3 If the emergent ray is parallel to incident ray after
2 è2ø
travelling a number of parallel interfaces then the
refractive index of the first and the last medium is always
5. (c) Bulk Modulus =
Physics S-PT-11

11. (b) Paramagnetic liquid tends to flow from region of weaker The electric and magnetic fields oscillate in same phase.
magnetic fields to stronger magnetic fields. Ti + Tc
dL 4A - A 3 21. (d) Since Tn = =Neutral temperature
12. (d) Torque = = = A 2
dt 4 4
Ti + 0 Ti
[dL is change in angular momentum] Tn = =
13. (d) All the given phenomena are explained by kinetic 2 2
theory of gases. [Tc = 0°C = temperature of cold junction]
14. (a) Resistance of bulb 22. (c) We know that for short dipole,
2p
V 2 200 ´ 200 field at axial point, E a =
= = = 1000 W d3
P 40
p
V 2 100 ´ 100 and field at equatorial point, E q =
New power = = = 10W . d3
R 1000
So, E a = 2E q
g' R2
15. (d) We know that = 23. (c) Let I1 = I and I2 = 4I
g ( R + h) 2
( ) =( ) = (3 I )
2 2 2
I max = I1 + I 2 I + 4I = 9I
2
g /9 é R ù R 1
\ =ê \ =
ë R + h úû ( ) =( )
2 2
g R+h 3 I min = I1 - I 2 I - 4I =I
\ h = 2R
24. (c) When the electron jumps from any orbit to second
16. (c) Work done = Area under curve ACBDA
orbit, then wavelength of line obtained belongs to
17. (d) The apparent frquency of sound of both the trains
Balmer series.
will be changed.
The apparent frequency of sound of train coming in mv
25. (c) r= or r µ v
320 240 × 320 qB
= 240 × = = 243
320 - 4 316 As v is doubled, the radius also becomes double. Hence
radius = 2 × 2 = 4 cm
For apparent frequency of train going away
26. (a) Power µ no. of electrons emitted (N)
320
= 240 × = 237 1 1
320+4 Pµ 2
Þ Nµ
r r2
When these sounds are heard simultaneously beat is
listened. q
No. of beats per sec = 243 – 237 = 6. 27. (d) Since ftotal = f A + fB + fC = e
0
18. (a) y = 0.03 sin p(2t - 0.01x) is equation of a progressive where q is the total charge.
wave. As shown in the figure, flux associated with the curved
Comparing it with the standard form of equation surface B is f = fB
y = a sin ( w t – kx) Let us assume flux linked with the plane surfaces A and
w = 2 p ; k = 0.01 p C be
2p 2 fA = fC = f'
k= = 0.01p Þ l = = 200 m. Therefore,
l 0.01
19. (b) Three 5W resistors are in series. Their total resistance q 1æ q ö
= 15W. Now it is in parallel with 5W resistor, so total = 2f '+ f B = 2f '+ f Þ f ' = ç - f ÷
resistance, e0 2 è e0 ø
1 1 1 3+1 4 m0 In
= + = = 28. (a) B= Add vectorially..
R 5 15 15 15 2 pa
15 V 3 3´ 4 29. (a) The side-wall of optical fibre provides total internal
R= \ I= = = = 0.8A
4 R 15 / 4 15 reflection of beam of light incident on it.
20. (c) The direction of oscillations of E and B fields are 30. (b) When p-n junction is reverse biased, the flow of current
perpendicular to each other as well as to the direction is due to drifting of minority charge carriers across the
of propagation. So, electromagnetic waves are junction.
transverse in
EBD_7504
HINTS & SOLUTIONS (CHEMISTRY – Subject-wise Tests)
Speed Test-92

1. (c) CH3 H 6. (c) Hydrolysis of substituted chlorosilanes yield


corresponding silanols which undergo polymerisation.
C C H
* CH3 Cl H OH – 2HCl CH 3 OH
CH3 C
Si + Si
CH3 COOH CH3 Cl H OH CH 3 OH
It has asymmetric carbon atom (marked *) so it shows Dialkyl silandiol
optical isomerism. Two groups attached to C = C are Polymerisation of dialkyl silandiol yields linear
same (–CH3), so it does not show geometrical isomerism. thermoplastic polymer.
2. (c) For weak acid dissociation equilibria, degree of
dissociation (a) is given as : CH3 CH3

Ka Ka HO — Si — OH + H O — Si — OH
a= , \ % a = 100
C C
CH3 CH3
[H + ][A - ] [H + ]Ca [H + ]a
Also, K a = = =
[HA] C(1 - a) (1 - a ) CH3 CH3
a 1- a
log K a = log H + + log ; or pK a = pH + log HO — Si — O — Si — OH
1- a a
1- a 1- a
pK a - pH = log ; = 10pK a -pH CH3 CH3
a a
1 2.303 100
or, = 10pK a - pH + 1 7. (c) t 90% = log
a k 100 - 90
1 100 2.303 100
a= ,%a= t 50% = log
pK a -pH é1 + 10 pK a - pH ù k 100 - 50
[1 + 10 ]
ë û
H + t 90% log10
3. (a) CH 2 = C = CH - CH 3 ¾¾
¾® = \ t 90% = 3.3t 50%
t 50% log 2
é + + ù
= C = CH - CH3 ¾¾¾
® êCH 2 = C- CH 2CH3 + CH 2 = CH - CHCH3 ú 8. (c) Find the volume by either
2
ë û V = RT/P (PV = RT) or P1V1 = P2V2 and and match it
with the values given in graph to find correct answer.
Cl- Volume of 1 mole of an ideal gas at 273 K and 1 atm
¾¾¾
® CH 2 = C - CH 2CH3 + CH 2 = CH - CHCH3
| | pressure is 22.4 L and that at 373 K and 1 atm pressure
Cl Cl is calculated as ;
2 -Chlorobutene 3-Chlorobutene
RT 0.082 ´ 373
D V = = = 30.58 L ; 30.6 L
4. (b) CaCO3 (s) ¾¾® CO 2 (g) ­ + CaO(s) P 1
(A) colourless (B)
9. (b) Be(OH)2 and Al(OH)3 are amphoteric in nature.
® Ca ( OH ) 2 (aq)
CaO(s) + H 2O(g) ¾¾ 10. (c) The enthalpy of formation of Al 2O3 is very high and
(B) therefore, it cannot be reduced by carbon. It is reduced
® Ca ( HCO3 )2 (aq)
Ca ( OH )2 (aq) + 2CO2 (g) ¾¾ by electrolytic method.
(C) 11. (a)
Ca ( HCO3 )2 (s) ¾¾
D
® CaCO3 (s) + CO2 (s) + H 2 O(g) 12. (d) ¾® CH 3 CH 2 P + Ph 3Br -
CH 3CH 2 Br + Ph 3 P ¾
(C) (A) - +
C H Li or
5. (d) Number of atoms per unit cell = 1 ¾¾6 ¾
5¾¾® CH 3 CH = PPh 3 ¬¾® CH 3 C H P Ph 3
C 2 H 5ONa
a
Atoms touch each other along edges. Hence r = Lm 9.54
2 13. (b) a = = = 0.04008 = 4.008 %.
( r = radius of atom and a = edge length) L¥
m
238
4 3 14. (c) Nickel ions are frequently detected by the formation of
pr red precipitate of the complex of nickel dimethylglyoxime,
p
Therefore % fraction = 3
S-CT-2 SOLUTIONS

CH3 C NOH 2+ 18. (d) DTf = K f ´ m ´ i . Since Kf has different values for
+ Ni different solvents, hence even if the molality is same
CH3 C NOH
DTf will be different.
Dimethylglyoxime 19. (a)
OH O 20. (b) Biodegradable polymer is Nylon-2-Nylon-6 which is
copolymer of glycine (H2 N – CH2 – COOH) and
CH3 C N N C CH3 aminocaproic acid (H2N–(CH2)5 – COOH).
Ni nH2N – CH2 – COOH +
CH3 C N N C CH3 glycine nH2N – (CH2)5– COOH
aminocaproic acid
O OH
Nickel dimethylglyoxime

16 16 O O
15. (d) 16.0 g O3 = mole = ´ 6.023 ´ 1023 molecules
48 48
–( HN – CH2 – C – HN – (CH2)5 – C )–n
16
= 3´ ´ 6.023 ´ 1023 atoms = 6.023 × 1023 atoms nylon – 2 – nylon – 6
48
21. (d) Given, P1 = 15 atm, P2 = 60 atm
28
28.0 g CO = mole = 1 mole V1 = 76 cm3, V2 = 20.5 cm3.
28 If the gas is an ideal gas, then according to Boyle's law,
= 1 × 6.023 × 10 molecules = 1 × 6.023 × 1023 atoms
23
it must follow the equation,
= 6.023 × 1023 atoms P1V1 = P2V2
16 1 P1 × V1 = 15 × 76 = 1140
16.0 g O 2 = mole = mole P2 × V2 = 60 × 20.5 = 1230
32 2
\ P1V1 ¹ P2 V2
1
= × 6.023 × 1023 molecules \ The gas behaves non-ideally.
2 The given information is not sufficient to comment on
1 other statements.
= × 2 × 6.023 × 1023 atoms 22. (a) Conversion of C4H6 to C4H6Br 2 indicates that the
2
compound is either butyne-1 or butyne-2. However,
= 6.023 × 1023 atoms
white precipitate with ammonical silver nitrate solution
Therefore, the ratio is
indicates that it is a terminal alkyne, i.e. butyne-1 and
6.023 × 1023 : 6.023 × 1023 : 6.023 × 1023
not butyne-2.
i.e. 1 : 1 : 1
23. (b) On moving along the period, ionization enthalpy
16. (b) The secondary structure of a protein refers to the shape
increases.
in which a long peptide chain can exist. There are two
In second period, the order of ionization enthalpy
different conformations of the peptide linkage present
should be as follows : F > O > N
in protein, these are a-helix and b-conformation. The
But N has half-filled structure, therefore, it is more
a-helix always has a right handed arrangement. In b-
stable than O. That’s why its ionization enthalpy is
conformation all peptide chains are streched out to
higher than O. Thus, the correct order of IE is F > N > O.
nearly maximum extension and then laid side by side
24. (a) DG = –ve means the process is spontaneous.
and held together by intermolecular hydrogen bonds.
25. (a) l = h / mv ; for the same velocity, l varies inversely
The structure resembles the pleated folds of drapery
with the mass of the particle.
and therefore is known as b-pleated sheet.
26. (c) C H3 — C º N CH 3 — NH 2
NH2 O
sp sp3
27. (c)
17. (c) HNO3
28. (a) 2NaOH + Cl 2 ® NaCl + NaOCl + H 2O
Hence, the ions formed in the reaction are Cl– and OCl
O 29. (c) Structure I has two resonance forms, which make the
p-Benzoquinone hydrogen bonding more stable while II molecule has
no such resonance stabilization.

30. (b) CH3 H CH 2 CH2 CH3 H


C=C C=C C=C
CH2 CH 2 CH3 H CH2 CH2
Natural rubber
EBD_7504
Speed Test-93
1. (a) Since concentration of ions is the same hence 8. (d) The products of the concerned reaction react each
Ecell = E°cell. other forming back the reactants.
XeF6 + 3H 2 O ¾¾ ® XeO3 + 6HF .
2. (a) H3C CH3 CH2 CH3 9. (d) Elemental sodium is easily oxidised ( has low I.P.) and
alc. KOH
¾¾¾¾® CH acts as reductant.
I 10. (c) Higher the stability of the carbocation, more easily it is
formed and more will be the ease of its dehydration.
SO Cl , heat
¾¾¾¾¾¾¾
2 2
® Thus order of stability of the carbocations of the
allylic substitution
corresponding alcohols is
CH2 = CHCH2Cl
+ + + +
3. (c) Applying (n + l) rule F3C CH2 < Cl3C CH 2 < H3C CH2 < C6H5CH 2
(I) n+l=3+2=5 Least stable due Most stable due
(II) n+l=5+0=5 to intensification to resonance
of charge
(III) n + l = 4 + 1 = 5
(IV) n + l = 4 + 2 = 6 F3 CCH 2 OH < Cl3CCH 2 OH < H 3CCH 2 OH
(V) n+l=4+0=4 < C 6 H 5 CH 2 OH (Ease of dehydration)
Higher the value of (n + l), more is the energy associated 11. (a) A base can donate electrons hence all are bases but
with it. In case of same value of (n + l), the electron
with higher value of n has higher energy. SO 32 - can be oxidised to SO 24 - hence it is reducing also.
\ The order is : V < I < III < II < IV
12. (c) 2HNO3(aq) + [Ag(NH3)2]+ + Cl– ¾¾
®
4. (a) In NH3 the atomic dipole (orbital dipole due to lone
pair) and bond dipole are in the same direction whereas
AgCl ( s ) ¯ +2NH +4 + 2NO 3-
in NF3 these are in opposite direction so in the former
case they are added up whereas in the latter case net When nitric acid is added the solution becomes acidic
result is reduction of dipole moment. It has been shown and the complex ion dissociates and liberate silver ion
in the following figure : to recombine with chloride ion. This is the confirmatory
test for silver in group 1.
Anhy. ZnCl
N N 13. (c) R 3COH ¾¾¾¾¾¾
2
® R3CCl
Conc. HCl
H H H F F F It is substitution reaction. R3CCl being insoluble in
5. (b) Completing the given reaction : H 2 O give turbid solution.
Fe 14. (d)
NO2 ¾¾¾¾®
30%HCl 15. (b) When benzaldehyde is refluxed with aqueous alcoholic
potassium cyanide, two molecules of benzaldehyde
Excess CH 3Br
NH2 ¾¾¾ ¾ ¾¾ ® condense together to form benzoin.
H O
CH3 KCN (alc)
N —C +C—
CH3 D
O H
6. (d) nl = 2d sin q ;
æ 3ö H O
2 ´1Å = 2 ´ d sin 60° Þ d = 1.15Å çç sin 60° = ÷
è 2 ÷ø —C –C—
7. (b) At the same T and P, V µ n OH
Benzoin
14 1 1
nN 2 = = ; VN 2 µ 2 3 2
28 2 2 16. (c) K p = p B (1) pC (1) = p B (2) pC3 (2) = p B (2)
2
(2 pC (1) )3 ;
36 3 3
nO 3 = = ; VO 3 µ p2 1 p B (2) 1
48 4 4 B (2)
hence = , =
2 p2 8 pB (1) 2 2
B (1)
Hence , VN 2 / VO3 = , 3VN 2 = 2VO 3
3 17. (a) Since the sol particles migrate towards cathode, they
are positively charged. Hence, anions would be
Greater is the valence of
S-CT-4 SOLUTIONS

3 22. (c) Formate ion


18. (c) PV 2 = constant.
O O –
nRT H–C H–C
Again P =
V O – O
3
nRT Hence A, B bonds are of same length.
\ ´V2 = constant (K)
V 23. (d) Green colour of CuCr2O7 is due to blue colour of
1
K Cu 2+ ions and yellow colour of Cr2 O7 2- ions
or, TV 2 = = K' (constant)
nR 24. (c) The stronger the acid, the weaker the conjugate base
1 1
2 2 formed.
For two states, T1V1 = T2 V2 The acid character follows the order :
1 CH3COOH > C6H5OH > H2O > CH3OH
2 The basic character will follow the order
æV ö
T2 = T1çç 1 ÷÷ CH3COO– < C6H5O– < OH– < CH3O–
è V2 ø Hence, decreasing order of rate of given reactions with
V Nu– is D > C > A > B
But V2 = 1 (given)
2 NH
1 25. (c) P4 O10 + 2H 2 O ¾¾
® 4HPO ¾¾¾
3
3
® NH PO
4 3
2 basic
Acidic meta phosphoric meta ammonium
æ ö acid phosphate
çV ÷
\ T2 = T1 ç 1 ÷ =T 2
1
26. (c) Cl 2CHCOOH is most acidic because it has two
ç V1 ÷
ç ÷ chlorine at a-position.
è 2 ø
27. (b) 58Ce is lanthanoid. Lanthanoids are from 57 to 71 and
19. (c) Magnetic moment, n(n + 2) = 5.92 B.M. Þ n all are present in IIIrd group. Hence 58Ce forms 56X
(unpaired electrons) = 5 element on emission of one a particle which belongs
5
to IInd group.

Mn 2+
:
3d
.. .. 4p.. ..
4s
28. (a) DH 2 - DH1 = DCp (T2 - T1 )

DH 2 - 24 = (0.031 - 0.055)(100 - 50)


sp3 hybridization (tetrahedral)
20. (a) The order of stability of resonating structure carrying Þ DH 2 = 22.8 cal g -1
no charge is greater than that of structures carrying OH
O
minimum charge. Also, each atom having their octet
complete.
21. (d) 2Cu2S + 3O2 ® 2Cu2O + 2SO2 29. (a) are tautomers.
3Cu 2 O + CH 4 ® 6Cu + 2H 2 O + CO
(From green
O O
logs of wood)
30. (b) Al2Cl6 + 12H2O ƒ 2[Al(H2O)6]3+ + 6Cl–
EBD_7504
Speed Test-94

CH3
3
2 [H 3 O + ][F – ] [HF][OH – ]
1. (d) 1 CH CH Ethyl group comes first in alphab- 10. (c) Ka = and K b = – .
2 3 [HF][H 2 O] [F ][H 2 O]
C2H5
3 Therefore, Ka × Kb = [H3O+] [OH–] = Kw.
atical order but IUPAC name of 2 Br , 2-bromo- 11. (a) Isoelectric point (pH)
1
CH3 pKa1 + pK a 2
2.34 + 9.60
= = = 5.97
4-ethyl-1-methyl cyclohexane. It follows lowest sum 2 2
rule 12. (d) Let bond energy of A2 be x then bond energy of AB is
(i.e., lowest set of locant is preferred.) also x and bond energy of B2 is x/2.
Lowest sum of locant is = 1 + 2+ 3 = 6 Enthalpy of formation of AB is – 100 KJ/mole:
2. (b) The molecule 2,3 - pentadiene does not have any chiral 1 1
C but at the same time it does not have any mirror plane A 2 + B2 ® 2 AB; A 2 + B2 ® AB; D =- 100KJ
2 2
which makes the molecule chiral.
æ x xö 2x + x - 4x
3. (d) In N2+ , there is one unpaired electron hence it is or - 100 = ç + ÷ - x\- 100= \ x= 400 KJ
è 2 4ø 4
paramagnetic.
4. (b) In A — O — H, if EN of ‘A’ is 2.1 then it will be neutral, (n 1 + n 2 )
as XA – X0 = X0 – XH. (where X is EN) 13. (b) M total = st ,
v1 + v 2
5. (b) Correct order is B < Be < O < N.
n1 = n2 = 0.1, V1 = V2 = 1litre Þ M total = 0.1 M
Cl 120
1 0.77 + 2( -0.44) 0.77 - 0.88 0.11
6 2
Cl 14. (c) E0 = = =-
6. (c) Dipole moments of 2Cl and 5Cl 3 3 3
3
Cl 5 4 Cl » - 0.04
are vectorically cancelled. 15. (d) The chemical bond method gives the O.N.
– O O O O
It is due 1 Cl and 3 Cl m 2 O – – – –
– S = O: O – S – S – O : O–S–S–O
2 2 O +4 +3
= m1 + m 2 + 2m1 m 2 cos q +5 O O
d+ d– +
= (1.5) 2 + (1.5) 2 + 2 ´1.5 ´1.5 cos 120 \ m = 1.5 D

16. (d) CH2 = CH– Cl CH2 – CH = Cl
Vinyl chloride
7. (c) CsCl is ionic solid.
8. (b) Volume of ice > volume of water & thus increase in 17. (b) NO2 NO2
pressure favours forward reaction showing decrease
in volume. CH3ONa
X=
SNAr
æ 1 1 ö Br
9. (c) n = RcZ2 ç 2 - 2 ÷ Br
çn ÷ OCH3
è 1 n2 ø Cl
18. (c) Because the layer of Al2O3 (oxide) is inert, insoluble
æ1 1 ö
n1 = RcZ2 ç 2 - 2 ÷ = RcZ2 and impervious.
è1 ¥ ø 19. (a) Au, the gold is not attacked by acids and alkalis. It
2 forms AuCl3.AuCl3 further reacts with HCl to form
æ1 1 ö 3RcZ H[Au(Cl)4] which is used in photography.
n 2 = RcZ2 ç 2 - 2 ÷= 4
è1 2 ø
AuCl3 + HCl ¾¾
® H[Au(Cl)4 ]
2æ 1 ö RcZ2
1 Complex
n 3 = RcZ ç 2 - 2 ÷ =
è2 ¥ ø 4 20. (b) 2KI + Cl 2 ® 2KCl + I 2
\ n1 - n2 = n3 I 2 + CCl 4 ® Violet Colour
Note: The excess of Cl2 should be avoided. The layer
may become colorless due to conversion of
S-CT-6 SOLUTIONS

I 2 + 5Cl 2 + 6H 2 O ® 2HIO 3 + 10HCl


HCl
In case of Br 2 : Cl NC [Y]
300 K
Br2 + 2H 2 O + Cl 2 ® 2HBrO + 2HCl
21. (d) To convert covalent compounds into ionic compounds
such as NaCN, Na2S, NaX, etc. Cl + HCOOH

22. (d) Zn 2+ + 2NH 4 OH ¾¾


® Zn(OH) 2 + 2NH 4+ [X]
White ppt.
27. (a) Let the rate law be r = [A]x[B]y
Zn(OH)2 + 2NH 4 OH ¾¾
®(NH 4 ) 2 ZnO 2 + 2H 2 O 0.10 [0.024]x [0.035] y
Soluble Divide (3) by (1) =
0.10 [0.012]x [0.035] y
(NH 4 )2 ZnO2 + H 2 S ¾¾
® ZnS + 2NH 4 OH \ 1 = [2]x, x = 0
White ppt.
0.80 [0.024]x [0.070] y
23. (b) (1) When the pH of rain water is below 5.6, it is called Divide (2) by (3) =
acid rain. 0.10 [0.024]x [0.035]y
(2) Ozone hole occurs over Antarctica mainly during \ 8 = (2)y , y = 3
September–October and it gets replenished in Hence rate equation, R = K[A]0[B]3 =K[B]3
November–December. CH2– C – H
(3) Methylcyclohexane is not an ozone-depleting
molecule. O CH2– CH
Ozonolysis
(4) BOD (Biological oxygen demand) is a measure of 28. (b) CH2 CH2
organic pollutant present in the sample of water. O CH2– CH
Higher is the value of BOD, higher is the level of
CH2– C – H
organic pollution in water. The amount of oxygen
(in mg/L) consumed for oxidising all organic and 29. (b) N 2 (g) + 3H 2 (g) 2 NH 3(g)
oxidisable inorganic material in a sample of water 1–x 3 – 3x 2x at equilibrium
is called chemical oxygen demand (COD). Total moles,
COD is always larger than BOD.
1 – x + 3 – 3x + 2x = 4 – 2x = 3 (given)
24. (c) Formaldehyde can not produce iodoform, as only those
(Since, 4 moles = 4 atm given)
compound which contains either CH3 - CH - group
|
\ x = 0.5
OH p N2 ´ p3H2
or CH3 - CH - group on reaction with potassium K p for dissociation of NH 3 =
|| p 2 NH 3
O
3
iodide and sod. hypochlorite yield iodoform. æ 1 - 0 .5 ö é æ 3 - 3 ´ 0 .5 ö ù
ç ´ 3 ÷ ´ êç ÷ ´ 3ú
N C CH 3 è 3 ø ëè 3 ø û
=
| | | é 2 ´ 0 .5 ´ 3 ù
2
25. (a) (i) - C - N , (ii) - C - CH 3 , ê ú
| | | ë 3 û
N C Br
= 0.5 × (1.5) 3 atm 2
OH H O-C
| | | 30. (b) Let the amount of the K 2Cr2 O 7 in the mixture be x g,
(iii) - C - O , (iv) - C - C - OCH 3 ,
| | then amount of KMnO4 will be (0.5 – x) g
| |
O C H O
æ x 0.5 - x ö 100 ´ 0.15
\ ç + ÷ =
O-C è 49 31.6 ø 1000
| |
(v) - C - O where 49 is Eq. wt. of K 2Cr2 O 7 and 31.6 is Eq. wt. of
|
H KMnO4 .
Arrange (NNN), (BrCC), (OOO), (CHH), (OOH) in On solving, we get x = 0.073 g
increasing atomic number. The order is ii, iii, v, i, iv. 0.0732 ´ 100
% age of K 2Cr2 O 7 = = 14.64%
CHCl3 0.5
26. (a) Cl
EBD_7504
Speed Test-95

1. (c) The configuration of atom of 4th period with maximum 12. (a) Osmotic pressure, p = CRT ;
unpaired electrons is 1s2 2s2 2p6 3s2 3p6 3d5 4s1 . 6 7.45
n CH 3COOH = , n KCl =
Hence its atomic number is 24. 60 74.5
2. (a) The total numberof electrons in the molecular species Since KCl ionises. Therefore its effective conc., in
given, respectively are 17, 16 and 18. Write down the solution increases.
electronic configuration of the molecular species and 13. (b)
observe the number of electrons in antibonding orbitals 14. (d) In reaction (d) oxidation number changes from + 4 in
which are respectively are 7, 6 and 8. NO2 to + 3 in HNO2 and + sin HNO3
3. (a) Aluminium has greater affinity for oxygen and the 15. (b) In neutral and alkaline medium
reaction is highly exothermic.
4. (d) Among alkaline earth metals, barium and radium have MnO 4- + 2H 2 O + 3e - ¾¾
® MnO 2 + 4OH -
the tendency to form peroxides.
In acidic medium:
5. (b) The more the reduction potential, the more is the
deposition of metals at cathode. Cation having E° value MnO 4- + 8H + + 5e - ¾¾
® Mn 2 + + 4H 2 O
less than – 0.83 V (reduction potential of H2O) will not
deposit from aqueous solution. C
6. (d) There is very little difference in acid strength in the
series H3 PO 4 , H3 PO 3 and H3 PO 2 because the 16. (b) Ca , 6.4g of CaC2 contain p-electron = 4NA
hydrogen in these acids are not all bonded to C
phosphorus.
In the above three acids although the number of –OH C ºCH
groups (ionisable hydrogen increases, yet the acidity + CH 3– CH2MgBr
17. (b)
does not increase very much. This is due to the fact
that the number of unprotonated oxygen, responsible
for the enhancement of acidity due to inductive effect, C º CMg Br
remains the same with the result dissociation constant C2 H6 +
also remains nearly same.
7. (c) C(graphite) is the stable state of aggregation of carbon. 18. (a) Because of the formation of the most stable
8. (d) Reaction between diborane and alkene are carried out +
in dry ether under an atmosphere of N2 because B2H6 carbonium ion, C6 H5 - C H 2
and the products are very reactive. The products
19. (c) Let atomic masses of A and B be a and b amu
further treated with alkaline H2O2 to convert into respectively
alcohols.
\ Molar mass of AB2 = (a + 2b) g mol–1
alkaline
B2 H6 + 6C 2 H 4 ¾¾
® B(C 2 H 4 )3 ¾¾¾¾
®
H 2O 2
and Molar mass of AB4 = (a + 4b) g mol–1
reactive
For compound AB2
3CH3CH 2 OH + H3BO3 DTb = Kb´ WB ´ 1000/ WA ´ MB
9. (b) Liquid crystals on heating first become turbid and then 2.3 = 5.1 ´ 1 ´ 1000/ 20.0 ´ (a + 2b)....I
clear. For compound AB4
10. (a) AgX + 2 Na2 S2O3 ® Na3 [Ag(S2O3)2]+ NaX
1.3 = 5.1 ´ 1 ´ 1000/ 20.0 ´ (a + 4b)....II
Sodium argento thiosuphate
Solving (I) and (II), a = 25.49 b = 42.64
(soluble complex)
11. (c) By definition, a mixed complex contains more than one
type of ligands.
S-CT-8 SOLUTIONS

20. (b) 2NaI + 2 NaNO 2 + 4 CH 3COOH ¾


¾® I 2 + 2NO + NaNO ,HCl
26. (c) NH 2 ¾ ¾ ¾ ¾
2
¾®
4 CH 3COONa + 2 H 2 O 0- 5°C

The colour of CCl4 layer turns purple due to liberated


I2.
21. (b) For bcc lattice, number of atoms per unit cell = 2 NMe 2
N 2 Cl +
n´M 2 ´ 100 H
Now d = =
a 3 ´ No (4 ´ 10-8 cm)3 ´ 6.02 ´ 10 23

= 200/38.528 = 5.19 g/cc


N=N NMe 2
22. (b) Entiomers of C4H10O are

OH HO 27. (c) The secondary amines react with HNO2 to give the
| |
H3CH 2C - C - CH3 H 3C - C - CH 2 CH 3 oily nitroso derivative. Amongst the options, (c) is
| |
H the secondary amine.
H
28. (d) The strongest oxidising agent is one which has
maximum tendency to gain electrons, i.e. whose E°Red
OH is maximum
|
H SO
CH 3 CH 2- C H - CH 3 ¾¾2¾¾

29. (a) Phospholipids - Phosphate + glycerol + fatty acids + a
A
nitrogen containing base.
CH 3 CH = CH - CH 3 + CH 2 = CHCH 2 CH 3
General formula : CH 2 O . COR '
Major ( B ) Minor ( C )
|
CHO COR ' '
1 | OH
23. (b) In general t1/ 2 of reaction µ
(a 0 )n -1
|
CH 2 O - P - O - X
||
For a second order reaction, O

1 1 1
t1/ 2 = n -1
= = = 10 min X = OHCH 2CH 2 NH 2 ,
K(a 0 ) K(a 0 ) 0.5 ´ 0.2 Ethanolamine
30. (a) Alkyl or Aryl cyanide react with grignard reagent to
CH2 form ketones
On keeping O O
24. (a) 3 HCHO CH2 CH2 CºN
aq. solution O Ether
Trioxane +C6H5MgBr
(meta formaldehyde)

O
25. (a) pOH = pK b + log
[salt]
C
[base] H 3O+
C6 H5– C=NMgBr
= - log 10 -10 + log 1 = 10; pH = 14 - 10 = 4 C6H5

or C 6 H 5 - C = O + MgBrNH 2
|
C6H5
EBD_7504
HINTS & SOLUTIONS (MATHEMATICS – Subject-wise Tests)
Speed Test-96
1. (d) Q f(x) = 2 min (f(x) – g(x), 0)
If 0 < (f(x) – g(x)) -1 æ a 2 - a1 ö -1 æ a 3 - a 2 ö
= tan ç 1 + a a ÷ + tan ç 1 + a a ÷ + ....
Þ g(x) < f(x) and è 1 2ø è 2 3ø
|f(x) – g(x)| = f(x) – g(x)
\ f(x) = 0 = (f(x) – g(x) – (f (x))– g(x)) æ a - an -1 ö
+ tan -1 ç n ÷
= (f(x) – g(x)) – |f(x) – g(x)| è 1 + a n -1 a n ø
= f(x) – g(x) – |g(x) – f(x)|
= (tan–1 a2 – tan–1 a1) (tan–1 a3 – tan–1 a2) + ....
5 5 sin x - cos x + (tan–1 an – tan–1 an–1)
2. (a) Given that sin x - cos x =
sin x cos x æ a -a ö
= tan–1 an – tan–1 a1 = tan–1 ç n 1 ÷
æ sin5 x - cos5 x ö è 1 + a1a n ø
sinx cos x ç ÷ =1
ç sin x - cos x ÷
è ø -1 æ ( n - 1) d ö
sin x cos x {sin 4 x + sin3 x + cos x + sin 2 x cos2 x = tan ç 1 + a a ÷
è 1 n ø
+ sin x cos3 x + cos4 x} = 1
sin x cos x {(sin 4 x + cos4 x) + sin2 x cos2 x é æ d ö -1 æ d ö
+sin x cos x (sin2 x + cos2 x)} = 1 \ tan ê tan -1 ç ÷ + tan ç 1 + a a ÷ + ...
ë è 1 + a1a 2 ø è 2 3ø
sin x cos x {(sin x + cos2 x)2 – sin2 x cos2 x
2
+ sin x cos x} = 1 æ d ö ù ( n - 1) d
+ tan -1 ç ÷ ú=
1 é 1 1 ù è 1 + a n -1 a n ø û 1 + a1 a n
Þ sin 2 x ê1 - sin 2 2x + sin 2x ú = 1
2 ë 4 2 û 6. (c) Let the equation of the circle be x2 + y2 + 2gx + 2fy + c = 0.
sin 2 x (sin 2 2x–2 sin 2x – 4) = – 8 This passes through (–3, 4) and (5, 4). Therefore
sin 3 2x – 2 sin2 2x – 4 sin 2x + 8 = 0 –6 g + 8 f + c + 25 = 0 ...(i)
(sin 2 x – 2)2 (sin 2x + 2) = 0 and, 10 g + 8 f + c + 41 = 0 ....(ii)
Þ sin 2x = ± 2 which is impossible. Subtracting (ii) from (i), we get g = –1
3. (a) Given A + B + C = p Since the centre (–g, –f) lies on 4y = x + 7
Þ tan A + tan B + tan C = tan A tan B tan C \ – 4f = – g + 7
Now, A.M ³ G.M. Þ f=–2 [ Q g = – 1]
So, centre of the circle is at (1, 2)
Þ tan A + tan B + tan C ³ (tan A tan B tan C)1/ 3
3
D C
æ tan A tan B tan C ö 1/ 3
Þ ç ÷ ³ (tan A tan B tan C)
è 3 ø G
2/3
Þ (tan A tan B tan C) ³3
2/3
æ1ö 1 1
Þ ç ÷ ³3Þ ³ 33 / 2 Þ K £ A M B
èKø K 3 3 (–3, 4) (5, 4)
4. (a)Q | ak | < 3, 1 £ k £ n Now, AD = 2 GM
\ | a1 | < 3, | a2 | < 3, ..., | an | < 3 = 2 (length of the ^ from G on AB whose eqn. is y = 4)
Q 1 + a1z + a2z2 + ... + anzn = 0 2- 4
Þ | a1z + a2z2 + ... + anzn | = | – 1| \ AD = 2 =4
1
Þ 1 = | a1z + a2z2 + ... + anzn |
Þ 1£ | a1| | z | + | a2 | | z |2 + ... + | an | | z |n Now, AB = (5 + 3)2 + (4 - 4)2
Þ 1< 3 (| z | + | z |2 + ... + |z |n) < 3 (| z | + | z |2 + ... ¥ )
3| z | 1 = 82 + 02 = 8
Þ1< Þ 1- | z |< 3 | z |Þ| z |> . So, Area of rectangle = 8 × 4 = 32 sq. units
1- | z | 4
7. (d) The coordinates of the focus of the parabola y2 = 4 ax
5. (b) We have, are (a, 0). The line x – y – a = 0 passes through this
æ d ö -1 æ d ö point. Therefore it is a focal chord of the parabola.
tan -1 ç ÷ + tan ç 1 + a a ÷ + .... Hence, the tangents intersect at right angle.
è 1 + a1a 2 ø è 2 3ø
8. (c) Required probability = Prob of right club and good
æ d ö shot or prob. of wrong club and good shot.
+ tan -1
S-MT-2 SOLUTIONS

9. (c) Let there are x families


d Also, ( 3 + 1) 2 m + ( 3 - 1) 2 m
a+b+c+d =x
a + c = 0.25 x, b + c = 0.15 x P m 2m m -1
C = 2[3 + C 2 3 + ....] is an integer
d = 0.65x , c = 2000 c
\ a + b + 2c + d = 1.05x a b Þ Required integer
x + c = 1.05x = ( 3 + 1) 2 m + ( 3 - 1) 2 m
\ 0.05x = c = 2000 Þ x = 40,000
So a = 8000, b = 4000 d = 26000 = 2 m (2 + 3 ) m + 2 m (2 - 3 ) m
c =2000 Þ 5% families own both a car and a phone
a + b + c = 14000 = 2 m [2(2 m + m C 2 2 m- 2 . 3 + m C 4 . 2 m - 4 32 + ....)]
Þ 35% families own either a car or a phone = 2m + 1 × integer
10. (a) Let P º (x1, y1 ) and Q º ( x 2 , y 2 ) (c) Let I + f = (7 + 4 3 ) m 0 £ f < 1
Let the equation of given circle be x2 + y2 = a2
The equation of chord of contact of tangent drawn from the Consider, F = (7 – 4 3 )m 0 £ F < 1
point P (x1, y1) to the given circle is
xx1 + yy1 = a2 \ I + f + F = 2(7 m + m C 2 7 m- 2 (4 3 ) 2 + .....]
Since it passes through Q (x2, y2) = 2k, where k is an integer
\ x2x1 + y2y1 = a2 ...(a) Þ f + F = 2k - I is an integer
Now, l1 = x12 + y12 - a 2 , l 2 = x 22 + y 22 - a 2 Q 0£ f + F< 2Þ f + F =1
Þ I = 2k - 1 , an odd integer..
and PQ = ( x 2 - x1 ) 2 + ( y 2 - y1 ) 2 Thus I cannot be a multiple of 2.

= (x 2
1 )( )
+ y12 + x 22 + y 22 - 2( x1x 2 + y1 y 2 ) (d) Let F = (6 6 - 14) 2 n +1
Then, R - F = (6 6 + 14) 2 n +1 - (6 6 - 14) 2 n +1
= (x 2
1 + y12 )+ (x 2
2 + y 22 ) - 2a 2 [Using (a)]
2[ 2 n +1C1 (6 6 ) 2n (14) + ....] = an even integer
= (x 2
1+ )( )
y12 - a 2 + x 22 + y 22 - a 2 = l12 + l 22 Þ [R ] + f - F = an even integer
n n n
11. (c) ( x + C 0 )(x + 3. C1 )(x + 5 . C 2 ).....(x + (2n + 1) . nCn) Also 0 £ f < 1 and 0 < F < 1
n +1 n n n n
+ x { C 0 + 3 . C1 + 5 . C 2 + ..... + ( 2n + 1) . C n } + ....
n \ -1 < f - F < 1 and f – F is integer
=x
\f–F=0 Þ f =F
Coeff. of xn = n C 0 + 3 . n C1 + 5. n C 2 + ..... + (2 n + 1) . n C n
2 n +1
= 1 + ( n C1 + 2 . n C1 ) + ( n C 2 + 4 . n C 2 ) + .... So, Rf = RF = (6 6 + 14) (6 6 - 14) 2 n +1 = 20 2 n +1
13. (a) Let x be a real root of the equation
+ ( n C n + 2n . n C n )
z 2 + (a + ib)z + c + id = 0, a , b, c, d,Î R - {0}
n n n n n
= (1 + C1 + .... + C n ) + 2( C1 + 2 C 2 + .... + n . C n )
Then x 2 + ( a + ib ) x + c + id = 0
é n(n - 1) n (n - 1)(n - 2) ù
= 2 n + 2ên + 2 . + 3. + ... + n .1ú Þ x 2 + ax + c = 0, bx + d = 0
ë 2! 3! û
n n -1 n -1 n -1
Þ (- d / b ) 2 + a (- d / b ) + c = 0
= 2 + 2n[1 + C1 + C 2 + .... + C n -1 ]
n n -1 Þ d 2 - adb + b 2 c = 0 Þ d 2 + b 2 c = abd .
= 2 + 2n . 2 = 2 n (1 + n ) = (n + 1) . 2 n
y
12. (c) (a) ( 2 + 1) 6 = I + F, where I is integer and 0 £ F < 1 14. (b)
A
M(h, k)
and ( 2 - 1) 6 = G , where 0 < G < 1
x
\ I + F + G = ( 2 + 1) 6 + ( 2 - 1) 6
6 3 6 2 6 6
= 2[ C 0 2 + C 2 2 + C 4 2 + C 6 ] = 198
Now 0 < F + G < 2
But F + G = 198 – I is an integer Let A be (a cos a, a sin a) and M be (h, k) then
\ F + G = 1 Þ I = 198 - 1 = 197 h = a cos a + a and k = a sin a
Eliminating a , we get
(b) ( 3 + 1) 2m = [( 3 + 1) 2 ] m = ( 4 + 2 3 ) m = 2 m ( 2 + 3 ) m
(h - a ) 2 + k 2 = a 2 Þ h 2 - 2ah + k 2 = 0
Now, ( 3 - 1) 2m <
y 2 - 2ax = 0
EBD_7504
Mathematics S-MT-3

tn 3 n (I) = 29 + 23 = 52
15. (a) t n = Sn - Sn -1 = 3t n - 3t n -1 Þ = n (F) = 100 – 52 = 48
t n -1 2
n(M È D) = n(M) + n(D) - n(M Ç D)
3 24 = 23 + 4 - n(M Ç D)
Hence the series is geometric with common ratio .
2
\ n(M Ç D) = 3
Also, S1 = 3t 1 - 2 Þ t 1 = 3t1 - 2 Þ t1 = 1
\ n ( W Ç D) = 4 - 3 = 1
n
æ3ö é cos n θ sin n θ ù
n ç ÷ -1 ìïæ 3 ö n üï An = ê
a (r - 1) è 2 ø 19. (a) ú
Now, Sn = = = 2 íç ÷ - 1ý ë - sin n θ cos n θ û
r -1 3 ïîè 2 ø ïþ
-1
2 é cos nq sin nq ù
\S1 + S2 + S3 + ...... + Sn 1 n ê n n ú
A =ê ú
n ê - sin nq cos nq ú
é 3 ì 3 2 ü ù ëê n n ûú
êìí - 1üý + ïíæç ö÷ - 1ïý + ú
êî 2 þ ïè 2 ø ú But – 1 £ cos nq £ 1 and – 1 £ sin nq £ 1
î ï
þ
= 2ê ú sin nq cos nq
ê ìïæ 3 ö3 üï ìïæ 3 ö n üïú lim = 0, lim =0
ê n ®¥ n n ®¥ n
íç ÷ - 1ý + ..... + íç ÷ - 1ýú
ê ïîè 2 ø ïþ ïîè 2 ø ú
ë þïû lim
1 n é0 0ù
A =ê ú
n ®¥ n ë0 0û
éìï 3 æ 3 ö 2 æ 3 ö 3 æ 3 ö üï ù
n
2 4 6
= 2 êí + ç ÷ + ç ÷ + ........... + ç ÷ ý - n ú 20. (c) sin x + sin x + sin x + ....¥
êï 2 è 2 ø è 2 ø è 2 ø ïþ úû
ëî sin 2 x sin 2 x
= = = tan 2 x
é3ì 3 ù 2 2
n ü 1 - sin x cos x
ê ïíæç ö÷ - 1ïý ú
ê 2 ïîè 2 ø ú 4 6 2
= 2ê
ïþ é ì 3 n ü ù æ 3n ö \ exp{(sin x + sin x + sin x + ...¥) log e 2}
- nú ê3ïíæç ö÷ - 1ïý - n ú = 6 ç
ê 3 ú =2 - 1÷ - 2 n
-1 ê ïè 2 ø ïþ úû ç 2n ÷ 2 2
ê 2 ú ë î è ø = e tan x.log 2
= 2tan x

ëê ûú And the roots of x 2 - 9 x + 8 = 0 are 1 and 8


16. (a) 2
2 tan x
\ 2 tan x
= 1 = 2 0 or 2 = 8 = 23
p q p Ù q p Ú q ~ (p Ú q) (p Ù q) Ù ~ (p Ú q)
\ tan 2 x = 0 or tan 2 x = 3
T T T T F F
T F F T F F or tan x = 0 or tan x = ± 3
F T F T F F p p
\ x= is the only value of x such that 0 < x <
F F F F T F 3 2
cos x 1 1 3 -1
\ (p Ù q) Ù (~ (p Ú q)) is a contradiction. \ = = =
cos x + sin x 1 + tan x 1 + 3 2
17. (b) Corrected Sx = 40 × 200 – 50 + 40 = 7990
\ Corrected x = 7990 / 200 = 39.95 r
21. (a) Let Tr =
Incorrect Sx 2 = n[s 2 + x 2 ] = 200[15 2 + 40 2 ] 1+ r + r4 2

=365000 1 2r 1ì 1 1 ü
Corrected Sx2 = 365000 – 2500 + 1600 = 364100 Tr = = í 2 - 2 ý
2 2
2 (r + 1) - r 2 2 î r - r + 1 r + r + 1þ
364100
\ Corrected s = – (39.95)2 =
1
(a r - a r +1 ) where a r =
1
200 2 (r - 1)r + 1
= (1820.5 –1596) = 224.5 = 14.98. n
1
18. (a) \ å Tr = 2 {(a1 - a 2 ) + (a 2 - a 3 ) + ... + (a n - a n+1)}
r =1
I M F
1 1ì 1 ü
23 = (a1 - a n +1 ) = 2 í1 - (n + 1)n + 1ý
4–x 2 î þ
x n
29 1 ü 1
1- ý=
(n + 1)n + 1 þ 2
S-MT-4 SOLUTIONS

22. (a) f (x) is continuous at x = 0 if 1 ln 3 1 1 3


lim f ( x ) = f ( 0) = lim f ( x ) 2I=
2 òln2 1 dt =
2
(ln3 - ln 2) = ln
2 2
x ®0 - x® 0+
1 3
ÞI= ln
Now lim f ( x ) = lim (1 + ax )1/ x = e a 4 2
x ® 0- x ®0-
x2 y2 x2 y2
25. (c) We have, + = 1 and
= 1 both are +
lim f ( x ) = lim
( x + c)1/3 - 1 a2 b2 b2 a 2
x®0 +
1/2
x ®0+ ( x + 1) - 1
ellipse with centre (0, 0), vertex (a, 0), (– a, 0) and
(0, b), (0, – b)
The above limit is a finite non-zero number only if c = 1
The curves intersect at ( ± a, ± a )
[If c ¹ 1 then the denominator is zero but numerator is non-
zero at x = 0, so the limit will not be a finite number] ab
Thus the right hand limit becomes where, a =
a + b2
2
1/3
lim
( x + 1) - 1
Y-axis
1/2
x®0+ ( x + 1) - 1
Thus for the continuity of f (x) at x = 0, we have
2 2 2
ea = b = Þ b = and a = log e and c = 1
b
3 3 3
a (a, 0)
b a
X-axis
23. (c) Let cost C = av + O b
v
According to given question,
b
30 a + = 75 … (i)
30
b The area of shaded region
40a + = 65 … (ii)
40 a
On solving (i) and (ii), we get b a2
òa a 2 - x 2 dx -
1 2
0
a= and b = 1800
2 a
b é 2 2 2 -1 x ù a2
dC b = x a - x + a sin -
Now, C = av +
b
Þ =a- 2 2a êë a úû 0 2
v dv v
2
b é 2 2 2 -1 a ù a
dC b b = a a - a + a sin -
=0Þ a- 2 =0 Þ v = = 3600 2a êë a úû 2
dv v a
Þ v = 60 kmph b ab a2
= × . +
2a a 2 + b 2 a 2 + b 2
1 l n3 2 x sin x 2
24. (a) I =
2 ò ln 2
sin x 2 + sin(ln6 - x 2 )
dx
ab -1 b a 2 b2 ab
tan -1
b
sin - =
Let x2 = t Þ 2x dx = dt 2 2
a +b 2 2
2 a +b 2 2
( a
)
Also, when x = ln 2 , t = ln2
b
when x = ln3 , t = ln3 Required area = 8I = 4ab tan –1 sq. units
a
1 l n3 sin t dt 26. (a) Consider the diff. equation
\I=
2 ò ln 2 sin t + sin(ln 6 - t ) ...(a)
dy
(x2 – yx2) + y 2 + xy 2 = 0
b b dx
Using ò a f (x) dx = ò a f (a + b - x)dx Þ x2 (1 – y)
dy
+ y 2 (1 + x ) = 0
We get dx
1 ln3 sin(ln6 - t ) Þ x2 (1 – y) dy + y2 (1 + x) dx = 0
I= ò
2 ln 2 sin t + sin(ln6 - t )
dt ...(b) 1- y
dy +
1+ x
dx = 0
Adding values of I
EBD_7504
Mathematics S-MT-5

æ 1 1ö æ 1 1ö Þ 180m 2 - 210mn + 60n 2 = 0


Þ çè y 2 - y ÷ø dy + çè x 2 + x ÷ø dx = 0
or 6m 2 - 7 mn + 2n 2 = 0
Integrate both sides, we get Note that it, being quadratic in m, n, gives two sets of
1 1 values of m, n, and hence gives the d.r.s. of two lines.
òy dy – ò dy + ò x -2 dx + ò dx = 0
-2
Now, factorising it, we get
y x
-1 1 6 m2 - 3mn - 4 mn + 2n2 = 0
Þ - log y - + log x = c
y x or (2m - n ) (3m - 2n ) = 0
1 1 æ xö 1 1 Þ either 2m - n = 0, or 3m - 2n = 0
Þ log x – log y = + + c Þ log ç ÷ = + + c Taking 2m – n = 0 we get 2m = n.
x y è yø x y
Also putting m = n/2 in l = 5m – 3n, we get
27. (b) Let | p | = | q | = | r | = k l = (5n/2) – 3n Þ l = – n/2 Þ n = – 2l

Let p̂, q̂, r̂ be unit vectors along p, q, r respectively. l m n


Thus, we get, –2l = 2m = n or = =
-1 1 2
Clearly p̂, q̂, r̂ are mutually perpendicular vecotrs, so any Þ d.r.s. of one line are –1, 1, 2.
vector x can be weitten as a1p̂ + a 2 q̂ + a 3r̂ . Hence, the d,c,s. of one line are
é -1 1 2 ù é 1 -1 - 2 ù
\ p ´ {( x - q ) ´ p} = ( p . p) ( x - q ) - { p.( x - q )} p ê , , ú or ê , , ú
ë 6 6 6û ë 6 6 6û
= k 2 ( x - q ) - ( p. x ) p [Q p . q = 0] Taking 3m – 2n = 0, we get
= k 2 ( x - q ) - kp̂.( a1p̂ + a 2 q̂ + a 3 r̂ ) kp̂ = k 2 ( x - q - a1p̂) 2n
3m = 2n or m = .
3
Similarly, q ´ {( x - r ) ´ q} = k 2 ( x - r - a 2 q̂ )
Putting this value in l = 5m – 3n, we obtain
and r ´ {( x - p) ´ r} = k 2 ( x - p - a 3 r̂ ) 2n n
l = 5´ - 3n = or n = 3l
According to the given condition 3 3
k 2 ( x - q - a1p̂ + x - r - a 2 q̂ + x - p - a 3 r̂ ) = 0 3m l m n
Thus 3l = =nÞ = =
2 1 2 3
Þ k 2 {3 x - ( p + q + r ) - (a1p̂ + a 2 q̂ + a 3 r̂ )} = 0 Þ the d.r.s of the second line are 1, 2, 3; and hence
Þ k 2 [ 2 x - ( p + q + r )] = 0 é 1 2 3 ù
d.c.s. of second line are ê , , ú
1 ë 14 14 14 û
Þ x= ( p + q + r ) [Q k ¹ 0]
2 é -1 -2 -3 ù
r r r or ê , , ú
28. (a) Since, a, b and c are non-coplanar ë 14 14 14 û
r r r r r r 1 1 1
\ b ´ c , c ´ a, a ´ b are also non-coplanar.. 30. (d) Here p = . q = 1 – p = 1 - =
So, any vector can be expressed as a linear combination 2 2 2
of these vectors. n = 6, N = 64.
r r r r r r r r 6- r 6
Let, r = l (b ´ c ) + m (c ´ a ) + g (a ´ b ) æ1ö æ1ö æ1ö
Then p(r ) = n C r p r q n - r = 6 C r ç ÷ .ç ÷ = 6Cr ç ÷
rr è 2ø è 2ø è 2ø
rr rrr a. r
\ a.r = l [a b c ] + m (0) + g (0)1 Þ l = r r r
[a b c ] 1 6
\ f (r ) = Np(r ) = 64. 6 C r . = Cr
rr rr 64
b .r c .r
Similarly, m = r r r & g = r r r 6

r r
[a b c ] [a b c ] Now å p(r) = p(3) + p(4) + p(5) + p(6)
If r = a 3
r r r r r r r 1 1
Then l (b ´ c ) + m (c ´ a ) + g (a ´ b ) = a = ( 6 C 3 + 6 C 4 + 6 C 5 + 6C 6 ) = (26 - 6 C 0 - 6 C1 - 6 C 2 )
rr r r rr r r rr r r
6
2 26
and (a.a ) (b ´ c ) + (b .a )(c ´ a ) + (c.a) (a ´ b ) 1 42 21
r rrr = (64 - 1 - 6 - 15) 6
= =
= a [a b c ] 2 64 32
29. (a) From the first relation, l = 5m – 3n. Putting this value 6
21
of l in second relation \ f (r ) =N å p(r) = 64. 32 = 42
7(5m - 3n ) 2 + 5m
S-MT-6 SOLUTIONS

Speed Test-97

1. (d) We have, Ai =
x - ai
, i = 1, 2, ...., n and
3. (b) We have, å x1 = sin 2 b, , å x1x 2 = cos 2b ,
x - ai
a1 < a2 < a3 < .... < an – 1 < an. Let x be in the left
å x1x2 x 3 = cos b and x1 x2 x3 x4 = – sin b.
\ tan–1 x1 + tan–1 x2 + tan–1 x3 + tan–1 x4
neighbourhood of am. Then x – ai < 0 for i = m, m + 1, ... , n
and x – ai > 0 for i = 1, 2, ...., m – 1. Therefore. -1 æ
= tan çç
å x1 -å x1 x 2 x3 ö÷
÷
Ai =
x - ai
= -1, for i = m, m + 1, ..., n è 1 - å x1 x 2 - å x1 x 2 x 3 x 4 ø
- ( x - ai )
æ sin 2b - cos b ö
= tan -1 ç ÷
x - ai è 1 - cos 2b - sin b ø
and, A i = = 1, for i = 1, 2, ...., m – 1.
x - ai
æ (2sin b - 1) cos b ö
Similarly, if x is in the right neighbourhood of am. Then = tan -1 ç ÷
x – ai < 0 for i = m + 1,..., n and x – ai > 0 for i = 1, 2, ....., m. è sin b(2sin b - 1) ø
x - ai æ æp öö
\ Ai = = -1, for i = m + 1, ... n = tan -1 ( cot b ) = tan -1 ç tan ç - b ÷ ÷
-(x - a i ) è è2 øø
x - ai p
and, Ai = = 1 for i = 1, 2, ... m. = -b
x - ai 2
n - m +1 x
lim ( A1A 2 ....A n ) = ( -1)
Now, x ®
4. (b) Im = ò
1
( log x )m dx
a- m
x
é( log x ) m . x ù - x m ( log x ) m -1 . 1 x dx
û1 ò1
n -m
and lim+ ( A1A 2....A n ) = -1
( ) = ë x
x ®a m
= (log x)m . x – m lm – 1.
lim ( A1A 2 ....A n ) \ Im = k – l Im – 1
Here x ® am does not exist.
Þ k – l Im – 1 = x (log x)m – m Im – 1
2. (a) The required mean is
Þ k = x (log x)m, l = m.
n n ( n -1) n -2 2
0.qn +1. qn-1p + 2. q p + ... + n.pn
1/n
éæ 1 ö æ 22 öæ 32 ö æ n 2 ö ù
X= 1 2! 5. (c) Let A = lim êç1 + 2 ÷ ç1 + 2 ÷ç1 + 2 ÷ ... ç1 + 2 ÷ ú
n n ( n -1) n -2 2 n ®¥ ëêè n øè n øè n ø è n ø ûú
qn + qn -1p + q p + ... + pn
1 2! æ
n
1 r2 ö
0. n C 0 q n p 0 + 1. n C1q n -1p + ... + n . n C n q 0 p n
Then, log A = lim
n ®¥
å n log çè1 + n 2 ÷ø
r =1
= n n 0 n n -1 1 n n -n n
C 0 q p + C1q p + ... + C n q p 1 2
ò0 log (1 + x ) dx = éë x log (1 + x ) ùû 0 - 2ò0 1 + x 2 dx
1 2 2 1 x
=
n n
n
å r. n Cr q n -r pr år. r n-1Cr-1 qn-r p.pr-1
r =0 r=1 1 é pù p
= n = n log 2 - 2 ëé x - tan -1 x ûù 0 = log 2 - 2 ê1 - ú = log 2 + - 2
å n
Cr q n -r
p r
å n
Cr q n -r r
p ë 4û 2
r =0 r=0 p
log 2 + - 2 2
æ n \ A=e 2 = ep / 2 .
( ) ( )ö 2
np ç å n-1Cr -1 pr -1q n-1 - r -1 ÷ e
ç ÷
è r =1 ø dy y ( x - y In y )
= n 6. (a) =
dx x ( x In x - y )
å n
Cr q n -r
pr
Þ x2 In xdy – xydy = xy dx – y2 In y dx
r =0

n -1 On dividing by x2y2, then


np ( q + p )
= In x 1 1 In y
( q + p )n Þ 2
dy - dy = dx - 2 dx
y xy xy x
= np [Q q + p = 1].
EBD_7504
Mathematics S-MT-7

é æ 1 ö 1 ù æ æ 1 ö 1 ö 5 x
Þ ê In x ç - 2 dy ÷ + dx ú + ç In y ç - 2 dx ÷ + dy ÷ = 0 Þ period of cos = 5π
ë è y ø xy û è è x ø xy ø 5

æ In x ö æ In y ö Thus required period = LCM of 2π & 5π = 10π


Þ dç ÷ + dç ÷=0 10. (d) Distance between (b2, b) and (1, 0) is given by
è y ø è x ø
On integrating, we get d = (b 2 - 1)2 + b 2
In x In y Let p = d2 = (b2 – 1)2 + b2
+ =c
y x dp
\ = 2(b 2 - 1).2b + 2b
x In x + y In y db
or =c
xy d2p
and = 12b 2 - 2
7. (a) Q y = 3x = x tan 60° db2
x-0 y-0 dp 1
\ = =r At = 0, b = 0, ±
cos 60° sin 60° db 2
æ r r 3ö 1 d2p
\ ( x, y ) = ç , ÷ lies on x3 + y3 + 3xy + 5x2 + 3y2 + 4x At b = ± , >0
è2 2 ø 2 db 2
+ 5y – 1 = 0 3
æ 1 ö 1 2
r 3 3 3r3 3 3r 2 5r 2 9r 2 5 3r \ ab = b3 = ç ÷ = =
4
\ + + + + + 2r + -1 = 0 è 2ø 2 2
8 8 4 4 4 2
11. (a) |z1| = |z2| = |z3| = 1 (given)
æ 3 3 + 1 ö 3 æ 14 + 3 3 ö 2 æ 4 + 5 3 ö Now, |z1| = 1 Þ |z1|2 = 1 Þ z1 z1 = 1
r +ç r +ç r -1 = 0
Þ çç 8 ÷÷ ç 4 ÷÷ ç 2 ÷÷
è ø è ø è ø Similarly, z2 z2 = 1, z3 z3 = 1
\ OA . OB. OC
1 8 ( 3 3 - 1)
8 Now, 1 + 1 + 1 = 1 Þ | z1 + z2 + z3 |= 1
= r1 r2 r3 = = = z1 z2 z3
æ 3 3 + 1 ö ( 3 3 + 1) 26
ç ÷ Þ | z1 + z2 + z3 |= 1 Þ |z1 + z2 + z3 | = 1
è 8 ø
12. (a) Ai + 1 A2
4
= ( 3 3 - 1) 2p
13
n A1
3 1 O
8. (d) Probability of showing even number in a throw = =
6 2
\ Required probability
2n 3 2n - 2 2i
2n +1 1æ1ö 2n +1 æ1ö æ1ö ÐA1OA i +1 = p
= C1 . ç ÷ + C3 . ç ÷ ç ÷
2è2ø è2ø è2ø n
2n +1 ip ip
2n +1 æ1ö \ A1A i +1 = 2OA1 sin = 2 sin
+... + C 2n +1 ç ÷ n n
è2ø
n -1
p 2p (n - 1) p ù
=
æ1ö
ç ÷
2n +1
éë 2n +1 C1 + 2n +1
C3 + ... + 2n +1 C 2n +1 ùû
\ å | A1Ai+1 |2 = 4 éêësin 2 n + sin 2 n
+ .... + sin 2
n úû
è2ø i =1

1 1 é 2p 4p 2(n - 1)p ù
´ 22n +1-1 = = 2 ê1 - cos + 1 - cos + ... + 1 - cos ú
= 2n +1 ë n n n û
2 2
9. (b) Period of sinx = 2π Þ period of sin 3 x = 2π éæ 2p 4p ö ù
= 2(n - 1) - 2êç1 + cos + cos + ....÷ - 1ú
x ëè n n ø û
period of sin 3 x = π Þ period of sin3 = 2π
2 = 2n - 2 - 2[0 - 1] = 2n
é 2p 4p 2( n - 1) p ù
period of cos5 x = 2π Þ period of cos5 x = π ê\ 1 + cos n + cos n + ... + cos = 0 n th roots of unity ú
ë 2 û
S-MT-8 SOLUTIONS

13. (c) Given, b -a


From (a), b = a +(4 –1) d Þ d =
n
C r + n C r -1 + n +1C r -1 + n + 2 C r -1 + .... + 2n C r -1 3

=
2 n +1
C 2a + b a + 2b
r 2 -132 \ A1 = a + d = and A 2 = a + 2d =
3 3
Þ n +1C r + n +1C r -1 + ..... + 2n C r -1 = 2n +1C
r 2 -132 \ A1 + A 2 = a + b ...(d)
..... ..... ..... ..... ..... ..... ..... ..... ..... ..... ..... ..... From (b), G1G2 = ab ...(5)
Þ 2 n C r + 2 n C r -1 = 2 n +1C 2 1 1 1 1
r -132
From (c), + = +
H1 H 2 a b
Þ 2 n +1C r = 2 n +1C
r 2 -132
H1 + H 2 a + b A1 + A 2 A1 + A 2 G1G 2
Þ r 2 - r - 132 = 0 Þ = = \ =
H1H 2 ab G1G 2 H1 + H 2 H1H 2
Þ (r - 12) (r + 11) = 0 Þ r =12 Þ n ³12
ì x, x ³ 0
So, minimum value of n = 12. 17. (b) Since | x | = í ,
î - x, x < 0
n -1 n -1
14. (d) We have, S = å k +2 P2 = 2! å k +2 C 2 therefore the equations of two lines are
y = 3x + 2, x ³ 0 and y = - 3x + 2, x < 0
k =0 k =0
Clearly y-axis the only bisector of the angle between these
= (2) [ 2 C 2 + 3C 2 + 4 C 2 + .....+ n +1C 2 ] two lines. There are two points P and Q on these lines at a
distance of 5 units from A. Clearly M is the foot of the
But 2 C 2 + 3 C 2 + 4 C 2 + .... + n +1C 2 perpendicular from P and Q on y-axis (bisector).

= ( 3 C3 + 3C 2 ) + 4 C 2 + .... + n +1C 2 AM = AP cos 30° =


5 3
.
2
= ( 4 C3 + 4 C 2 ) + 5 C 2 + .... + n +1C 2
æ 5 3 ö÷
ç
= 5 C3 + 5 C 2 + .... + n +1C 2 Hence, coordinates of M are ç 0, 2 + 2 ÷
è ø
n +2 1 Y
= C 3 = n (n + 1)(n + 2)
6
Q P
M
Þ 3S = n (n + 1)(n + 2)
Þ n divides 3S, (n + 1) divides 3S and (n + 2) divides 3S.
y = 3x + 2
15. (a) Let the first set of number be a – d, a, a + d. y = - 3x + 2
Then a – d + a + a + d = 15 Þ a = 5 A (0,2)
The second set of numbers will be
b – (d – 1), b, b + (d – 1). Again, O X
b - (d - 1) + b + b + (d - 1) = 15 Þ b = 5
Hence, the sets of numbers are
5 – d, 5, 5 + d and 6 – d, 5, 4 + d.
Further, from the given condition x2 y2
18. (d) Equation of the normal to the hyperbola - =1
(5 - d)5(5 + d) 7 25 - d 2 1 a2 b2
= Þ = at the point (a sec a, b tan a) is given by,
(6 - d) 5(4 + d) 8 24 + 2d - d 2 8 ax cos a + by cot a = a2 + b2
Þ d 2 + 14d - 32 = 0 Þ d = 2,-16
ïì ax cos q + by cot q = a + b
2 2
\ The two sets are 3, 5, 7 and 4, 5, 6 or Normals at q, f are í
2 2
21, 5, –11 and 22, 5, –12. ïî ax cos f + by cot f = a + b
3 -11 3 11 p
\ Ratio of their smallest term is
or i.e. or where f = – q and these passes through (h, k)
4 - 12 4 12 2
16. (a) Let the given numbers be a and b Then,
\ ah cos q + bk cot q = a2 + b2
a, A1, A2, b are in A.P. ...(a)
a, G1, G2, b are in G.P. ...(b) Þ ah sin q + bk tan q = a2 + b2
a, H1, H2, b are in H.P. ...(c) Eliminating h,
EBD_7504
Mathematics S-MT-9

bk (cot q sin q – tan q cos q ) (v) log x {x} ³ 0 Þ 1 > f ( x ) ³ 0 ,


= (a2 + b2) (sin q – cos q ) so 1 > x ³ 0 logx f(x) is positive x Î[0, 1)
or k = – (a2 + b2)/b
19. (d) The truth tables of p ® q and ~ p Ú q are given below: Þ x Î(0, 1)

p q ~p p ® q ~p Ú q 23. (d) Given (A + B) 2 = A 2 + B 2 + 2AB


T T F T T Þ (A + B)(A + B) = A 2 + B 2 + 2AB
T F F F F Þ A 2 + AB + BA + B 2 = A 2 + B 2 + 2 AB Þ BA = AB
F T T T T
éa 1 ù é1 - 1ù é1 - 1ù éa 1 ù
F F T T T Þ ê úê ú=ê úê ú
ëb - 1û ë2 1 û ë2 1 û ëb - 1û
Clearly, truth tables of p ® q and ~ p Ú q are same.
So, p ® q is logically equivalent to ~ p Ú q. éa + 2 - a + 1ù é a - b 1 + 1ù
Hence, option (a) is correct. Þ ê ú=ê ú
ëb - 2 - b - 1û ë2a + b 2 - 1û
If the truth value of p, q, r are T, F, T respectively, then
the truth values of p Ú q and q Ú r are each equal to T. éa + 2 - a + 1ù é a - b 2ù
Therefore, the truth value of (p Ú q) Ù (q Ú r) is T. Þ êb - 2 - b - 1ú = ê2a + b 1ú
ë û ë û
Hence, option (b) is correct.
The corresponding elements of equal matrices are equal.
We have, ~ (p Ú q Ú r) @ ~ p Ù ~ q Ù ~ r
So, option, (c) is correct. a + 2 = a - b, -a + 1 = 2 Þ a = -1
If p is true and q is false, then p Ú q is true. b - 2 = 2a + b, - b - 1 = 1 Þ b = -2
Consequently, Þ a = – 1, b = –2
~ (p Ú q) is false and hence p Ù ~ (p Ú q) is false. 24. (b) Put x2 + x + 1 = y, the equation reduces to
Hence, option (d) is wrong.
( y + 1) 2 - (a - 3)( y + 1) y + (a - 4) y 2 = 0
20. (c) Let A1..... An the event that the letter is placed at right
envelope. 1
Þ (a - 5) y - 1 = 0 Þ y =
\ 1 – P (A1 È A2 È A3 È ..... È An) = 1 – [S P (Ai ) – a -5
S P (Ai Ç Ak ) + S P (Ai Ç Aj Ç Ak ) ...... + (–1)n–1
1 1 2 3 3
P (A1 Ç A2 ... Ç An)] \ x 2 + x +1 = 2
. Now x + x + 1 = ( x + ) + ³
a -5 2 4 4
(n – 1)!
Here, P (Ai) = 1 3 4 - 3a + 15 3a - 19
n! \ ³ Þ ³0Þ £0
a -5 4 4(a - 5) a -5
(n – r)!
P (A1 Ç A2 Ç A3 Ç ....... Ç Ar) = 19 19
n! Þ (a - 5)(3a - 19) £ 0, a ¹ 5 Þ 5 < a £ Q6 < <7
3 3
Þ P(A1 Ç A 2 Ç A 3 Ç ......... Ç A n )
æ 19 ù
é1 1 1 1ù \ only integral value of a such that a Î ç 5, ú is a = 6.
= 1 – ê – + ......(–1) n -1 ú è 3û
ë 1! 2! 3! n!û
25. (c) As we know,
1 1 1 1 A function f (x) is said to be continuous at a point
= – + – .......... + (–1) n
2! 3! 4! n! x = a iff lim f (x) = f (a), otherwise discontinuous.
x ®a
21. (b) A = [ x : x Î R, -1 < x < 1] Thus f (x) is continuous at x = a iff
B = [ x : x Î R : x - 1 £ -1or x - 1 ³ 1] lim f (x) = lim f (x) = f (a)
x ®a - x ®a +
= [ x : x Î R : x £ 0 or x ³ 2]
ì 1/ x
Since, f (x) = í5 , x < 0 and l Î R
\ A È B = R - D , where D = [ x : x Î R,1 £ x < 2] îl[x], x ³ 0
22. (d) f (x) = cos(sin x ) + log x { x} RHL at x = 0
lim f (x) = lim l [x] = lim l [h] = 0
Domain cos(sin x) ³ 0 {x} > 0, x > 0, x ¹ 1, x ®0 + x ® 0+ h ®0
logx {x} ³ 0 LHL at x = 0
(i) cos(sin x) ³ 0 for all x, x Î R [–1, 1] lim f (x) = lim 51/ x = lim 5
- 1/ h
= 5¥ = 0
x ®0 - x ®0- h®0
(ii) {x} > 0, x Ï Integer
and f (0) = l [0] = 0.
(iii) x > 0, x Î(0, ¥) \ f (x) is continuous whatever l may be
(iv) x ¹ 1
S-MT-10 SOLUTIONS

26. (b) Let s = 2t3 – 6t2 + at + 5 Required Area


ds = area of shaded region ABC
Velocity = = 6t2 – 12t + a at time ‘t’ ...(i)
dt 5
= ò [ y(line) - y(parabola) ] dx
2 1
d s
Now, acceleration = = 12t -12
dt 2 5
= ò éë 4a - 2a x - 1 ùû dx
When acceleration is zero 1
i.e., 12t – 12 = 0 we have t = 1
5
ds é 2 ù
and =–3 = ê 4ax - 2a ´ (x - 1)3 2 ú
dt ë 3 û1
\ from eqn (i), we have
– 3 = 6(a)2 – 12(a) + a æ 32a ö 16
Þ – 3 = 6 – 12 + a = ç 20a - ÷ - (4a - 0) = a sq.unit
è 3 ø 3
Þ–3+6=a
Þa=3 29. (a) Let the required vector be
27. (a) r
L v = xiˆ + yjˆ + zkˆ , then
P r
( )
v. ˆi + ˆj - 3kˆ = 0 Þ x + y - 3z = 0 ...(i)
r
O N S ( )
v. iˆ + 3jˆ - 2kˆ = 5 Þ x + 3y - 2z = 5 ...(ii)
r

L¢ ( )
and v. 2iˆ + ˆj + 4kˆ = 8 Þ 2x + y + 4z = 8 ...(iii)

Let P be ( 9t2, 18t) then ON = 9t2 Subtracting (ii) from (i),we have
Area of trapezium – 2y – z = –5 Þ 2y + z = 5 ...(iv)
1 1 Multiply (ii) by 2 and subtracting (iii) from it, we obtain
A= (PP '+ LL ')NS = (36t + 36)(9 - 9t 2 ) 5y – 8z = 2 ...(v)
2 2
Multiply (iv) by 8 and adding (v) to it, we have
= 162 (1 + t 2 )(1 - t) 21y = 42 Þ y = 2 ...(v)
dA Substituting y = 2 in(iv), we get
= 162[2(1 + t )(1 - t ) - (1 + t ) 2 ] = 162(1 + t )(1 - 3t )
dt 2× 2+ z= 5Þz=5–4 =1
Substituting these values in (i),we get
1
A is maximum when t = x+ 2–3= 0Þx=3–2=1
3
Hence, the required vector is
æ1ö
2 r
\ For maximum area ON = 9ç ÷ = 1 v = xiˆ + yjˆ + zkˆ = ˆi + 2jˆ + kˆ
è3ø
2 2
28. (a) Given curve : y = 4a (x – 1) 30. (d) Let N = 2 n +1
C0 + 2 n +1
C1 + ....... + 2 n +1
Cn
Lines : x = 1 and y = 4a
Now, (y – 0)2 = 4a2 (x – 1) Multiplying by 2 on both side
This is a parabola with vertex A (1, 0).
2N = 2[ 2 n +1 C0 + 2 n +1
C1 + ....... + 2 n +1
Cn ]
x=1
a) = ( 2 n +1 C 0 + 2 n +1
C 2 n +1 ) + ( 2n +1C1 + 2n +1C 2n )
4
(1,
C B y = 4a + ....+ ( 2 n +1 C n + 2 n +1
C 2 n +1 )
(5, 4a)
= 2 n +1
C0 + 2 n +1
C1 + ........... + 2 n +1C 2n +1 = 22n + 1
O , if a > 0
(0, 0) A (1, 0) 2N = 22n.2
\ N = 22n
EBD_7504
Speed Test-98

1. (d) It passes through a fixed point (3, 4)


æ 1ö
Slope of line joining (3, 4) and (1, –2) is –6/–2 = 3 6. (d) Let the given expression be ç x i , ÷ , i = 1, 2, 3, 4.
\ Slope of required line = –1/3 è xi ø
1 Now, equation of circle is given as
Equ. is y – 4 = - (x - 3) or x + 3y – 15 = 0
3 x2 + y2 + 2gx + 2fy + c = 0
log(1 + ax) - log(1 - bx) æ 1ö
2. (b) Given f (x) = Since, ç x, ÷ lies on the circle
x è xø

0 æ 1ö 1
Now, At x = 0, it is in form. \ ç x, ÷ satisfies the equation of circle. \Put x = x, y =
0 è xø x
\ By applying L’ Hopital Rule.
1 1
1 1 \ x2 + 2 + 2gx + 2 f +c=0
.a - .( -b) x x
We get: lt. f (x) = lt. 1 + ax 1 - bx = a + b.
1 Þ x4 + 2gx3 + 2fx + cx2 + 1 = 0
x ®0 x ®0
and also, the above equation is of fourth degree
3. (b) 2 2 equation in x having x1, x2, x3 and x4 as its roots.
2(x + iy) = x + y + 2i
Therefore x1 . x2 . x3 . x4
2x = x 2 + y2 and 2y = 2 i.e., y = 1 Constant term 1
= Product of root = 4
= =1
coff .of x 1
1
4x2 = x2 + 1 i.e., 3x2 = 1 i.e., x = ±
3 q2 q2
p dq tan q dq
1 1 3
7. (d) q1 + q2 =
2
\ I= ò æp ö
= ò 1 + tan q
x= (Q x ³ 0) \ z = +i= +i q1 1 + tan ç - q÷ q1
3 3 3 è2 ø

2 y z 2 y z q2
dq
4. (b) 0 = x 2 z = x -2 2- y 0 Also I = ò 1 + tan q
q1
x y 2 x-2 0 2-z
q2
1002p 501p
2 y z \ 2I = ò dq = q2 - q1 = 2008
ÞI=
2008
q1
x-2 2-y 2-z
= (x - 2) (2 - y) (2 - z) 1 1 0 Hence, K = 2008.
8. (a) Suppose the line lx + my + n = 0 cuts the ellipse at
1 0 1
P (a cos q, b sin q) and Q (a cos (p/2 + q), b sin (p/2 + q)).
Then these two point lie on the line
la cos q + mb sin q + n = 0 – la sin q + mb cos q + n = 0
2 y z 2 y z
Þ 0= - - Þ + + =0 la cos q + mb sin q = – n ...(i)
x -2 2- y 2-z 2- x 2- y 2-z
la sin q + mb cos q = – n ...(ii)
5. (b) Let A be the event such that sum is ` 20 or more
\ P (1) = 1 – P (Total value is < 20) Square and add the equations (i) and (ii)
(la cos q + mb sin q)2 + (–la sin q + mb cos q)2 = n2 + n2
6 l2a2(cos2q + sin2q) +m2b2 (sin2q + cos2q) = n 2 +n2
C2 - 2C2 14 1 1
= 1- 8
= 1- = 1- = Þl2a2 + m2b2 = 2n2
C2 28 2 2
S-MT-12 SOLUTIONS

9. (a)
n
Ck
=
1
Þ
n! (k + 1)!(n - k - 1)! 1
=
{ } {
= x esin x - ò esin x dx – esin x sec x - ò esin x sec x cos x dx }
n 2
C k +1 k! (n - k)! n! 2 = x esin x – esin x sec x + C

k +1 1 æ 1 ö
13. (b) sin -1 ç + cot -1 3 ...(1)
or
n-k 2
= Þ 2k + 2 = n – k è 5 ÷ø
n – 3k = 2 .......... (1) æ 1 ö
We have, sin -1 ç -1
÷ = cot 2 5
n
Ck +1 2 è 5ø 1
Similarly, n
= \ from equation (1), we have
Ck + 2 3
-1 1 1 2
cos–1 2 + cot–1 3 = tan + tan -1
n! (k + 2)!(n - k - 2)! 2 2 3
. =
(k + 1)! (n - k - 1)! n! 3
æ 1 1 ö
+ æ 5/6 ö
k+2 2 -1 ç2 3 ÷ tan -1 = tan -1 1 = p
= Þ 3k + 6 = 2n – 2k – 2 = tan ç 1 1÷ = ç 6 -1÷
n - k -1 3 ç1- . ÷ çè ÷ 4
è 2 3ø 6 ø
2n – 5k = 8 ............. (2)
From (1) and (2): n = 14 and k = 4 14. (a) For real roots, D ³ 0
\ n + k = 18 (k – 2)2 – 4 (k2+ 3k + 5) ³ 0
10. (a) Let the observations be x1, x2, x3, x4, x5 and x6, so Þ (k2 + 4 – 4k) – 4k2 – 12k – 20 ³ 0
6 Þ – 3k2 – 16k –16 ³ 0 Þ 3k2 + 16k + 16 £ 0
å xi 6 6
æ 4ö
their mean x = i =1
= 8 Þ å x i = 8 ´ 6 Þ å x i = 48 Þ çè k + ÷ø (k + 4) £ 0
6 i =1 i =1 3
On multiplying each observation by 3, we get the new Now E = a 2 + b2 ; E = (a + b) 2 - 2ab
observations as 3x1, 3x2, 3x3, 3x4, 3x5 and 3x6.
E = (k – 2)2 – 2 (k2 + 3k + 5) = –k2 – 10k – 6
6 E = – (k2 + 10k + 6) = – [(k + 5)2 – 19] = 19 – (k + 5)2
3å x i \ Emin occurs when k = – 4/3
3 ´ 48
Now, their mean = x = i=1 Þx= = 24 121 171 - 121 50
6 6 \ Emin = 19 - = =
9 9 9
Variance of new observations
Emax occurs when k = – 4 Þ Emax = 19 – 1 = 18
6 6
20! 20! p
å (3x i - 24) 2 32 å (x i - 8) 2 15. (a)
p!q!r!
(2x)p (- y)q (z)r =
p!q!r!
2 (- 1)q x p yq z r
i =1 i =1
= =
6 6 p + q + r = 20, q = 0
p + r = 20 (p is even and r is odd).
9
= ´ Variance of old observations = 9 × 42 = 144 even + odd = even (never possible)
1 Coefficient of such power never occur
Thus, standard deviation of new observations \ coefficient is zero
= Variance = 144 = 12 16. (a)
11. (d) We know that ~ (p ® q) @ p Ù ~q Starting with 1 23456789
\ ~ (p ® (q Ù r)) @ p Ù (~ (q Ù r)) @ p Ù (~q Ú ~r))
= 8C4 = 70
[By De'Morgan's laws]
Starting with 2 34 56789
æ x cos3 x - sin x ö
ò
sin x
12. (a) e çç ÷÷ dx = 7C4 = 35
è cos2 x ø
Total = 105
(105)th number 26789
òe òe
sin x sin x
= x cos x dx – tan x sec x dx

ò x d (e )–òe
sin x sin x
= d (sec x)
EBD_7504
Mathematics S-MT-13

22. (c) Let S = 1+ 2.2 + 3.22 + 4.23 +...+100.299 .....(1)


(tan x)3/ 2 [1 - (cos x)3/ 2 ] Þ 2S = 2+2.22 +3.23 +....+ 99.299 +100.2100 .....(2)
17. (b) lim .
x ®0 x 3/ 2 .x 2 Subtracting (2) from (1), we get
– S = ( 1+ 2+ 22 + 23 +.....+ 299) – 100.2100
1 - cos3 x 1
lim .
= 1× x ®0 x2 1 + (cos x)3/2 2100 - 1
Þ S = 100.2100 – = 100.2100 – 2100 + 1
2 -1
1 1 3
= . (1 + cos x + cos 2 x) = = 1 + 99.2100
2 2 4
é -1 2 2 ù é -1 2 2 ù
cos 3x + 3cos x ê 2 -1 2 ú ê 2 -1 2 ú
18. (d) cos3x. sin 2x = .sin 2x
4 23. (b) Here let A = k ê ú \ AT = k ê ú
êë 2 2 -1úû êë 2 2 -1úû
1 3
= (sin 5x – sin x) + (sin 3x + sin x) Since A is orthogonal \ AAT = I
8 8
1 3 1 é -1 2 2 ù é -1 2 2 ù
= sin x + sin 3x + sin 5x. ê ú ê 2 -1 2 ú
4 8 8 \ AAT = ê 2 -1 2 ú ê ú
1 3 1 ëê 2 2 -1ûú ëê 2 2 -1ûú
\ n = 5, a1 = , a2 = 0 , a3 = , a4 = 0, a5 =
4 8 8
é 1 + 4 + 4 -2 - 2 + 4 -2 + 4 - 2 ù é9 0 0 ù
ê ú ê0 9 0 ú
ˆ ˆ ˆ - (a.b)c
19. (a) (a.c)b ˆˆ ˆ=
1 ˆ 1
b+ cˆ = k2 ê - 2 - 2 + 4 4 + 1 + 4 4 - 2 - 2 ú = k2 ê ú = 9k2I
2 2 ëê -2 + 4 - 2 4 - 2 - 2 4 + 4 + 1 ûú ëê0 0 9 úû

1 1 1 1
ˆˆ=
\ a.c an d â.bˆ = - Þ angle between Now, 9k2 I = I Þ k2 = Þk= ±
2 2 9 3

p 3p é -1 æ 1+ x ö ù
and angle between â and bˆ =
aˆ and cˆ = 24. (a) Let y = ê cos ç sin + xx ú
4 4 2 ø ÷
ëê è ûú
20. (c) From the given equation,
tan q + tan(p / 3) tan q + tan(2p / 3) dy -1 1+ x 1 2
tanq + + =3 = . cos . . + xx + xx log x
1 - tan q tan(p / 3) 1 - tan q tan(2p / 3) dx 1+ x 2 4 (1 + x)
1 - sin 2
2
tan q + 3 tan q - 3
or tan q + + =3
1 - 3 tan q 1 + 3 tan q æ dy ö -1 1
So, ç ÷ = .cos1. + 1
è dx ø x =1 1 - sin 2 1 4
3 tan q - tan 3 q
or =3
1 - 3 tan 2 q æ dy ö -1 1 1 3
Þ ç ÷ = .cos1. + 1 = 1 - = .
or tan3q = 1 = tan(p/4) or 3q = np + (p/4) è dx ø x =1 cos1 4 4 4
or q = (4n + 1) (p/12), where n ÎI
21. (a) Given that f (x) = 2x + sin x, x Î R æ pö æ pö
25. (a) Let f (x) = sin ç x + ÷ + cos ç x + ÷
Þ f ' (x) = 2 + cos x è 6ø è 6ø
But – 1 £ cos x £ 1 We knows that; for f (x) to be minimum or maximum:
Þ 1 £ 2 + cos x £ 3 Þ 1 £ 2 + cos x £ 3 f ' (x) = 0
\ f ' (x) > 0, " x Î R æ pö æ pö
Thus f ' (x) = cos ç x + ÷ - sin ç x + ÷
Þ f (x) is strictly increasing and hence one-one è 6 ø è 6ø
Also as x ® ¥ , f (x) ® ¥ and x ® – ¥ , f (x) ® – ¥
\ Range of f (x) = R = domain of f (x) Þ f (x) is onto. æ pö æ pö
and f '' (x) = - sin ç x + ÷ - cos ç x + ÷
Thus, f (x) is one-one and onto. è 6 ø è 6ø
Now for maximum, f ' (x) = 0
S-MT-14 SOLUTIONS

ép æ p öù sin 2 x
æ pö æ pö (c) Let I1 = ò sin -1 t dt
Þ cos ç x + ÷ = sin ç x + ÷ = cos ê - ç x + ÷ ú 29.
0
è 6ø è 6ø ë2 è 6 øû
2
p p p p p p p Put t = sin u Þ dt = 2sin u cos u du
Þ x+ = - x - Þ 2x = - = Þ x =
6 2 6 2 3 6 12 x

æpö p p
Þ dt = sin 2udu \I1 = 0 u sin 2u du ò
f '' ç ÷ = - sin - cos = - 2 < 0
è 12 ø 4 4
cos 2 x
Let I2 = ò0 cos -1 t dt
p
Thus f (x) has the maximum value at x = .
12 Put t = cos 2 v Þ dt = -2 cos v sin vdv
dx 1 dy dy dt dy Þ dt = - sin 2 v dv
26. (c) x = tan–1 t Þ = = . = (1 + t 2 )
dt 1 + t 2 dx dt dx dt x x
........... (1) ò
\I 2 = p v ( - sin 2 v )dv = - p v sin 2 vdv ò
2 2
d2y d é dy é dy d2 y ù
2 ù dt = 2t + (1+ t 2 ) 2 ú (1+ t 2 ) x
= ê
dx 2 dt ë dt
(1 + t ) . ê
úû dx ê dt
ë dt ûú
=- ò p
u sin 2 udu [change of variable]
2
........... (2)
Hence the given differential equation x x
\ I = I 1 + I2 = ò0 ò
u sin 2 udu - p u sin 2 udu
d2 y dy 2
2
+ xy + sec 2 x > 0 , becomes
dx dx
p
2 x x
é dy d yù 2
(1 + t 2 ) ê 2t + (1 + t 2 ) 2 ú
ëê dt dt ûú
ò òp
= u sin 2udu + u sin 2udu - u sin 2udu òp
0
2 2
é dy -1 ù
+ y tan t ê (1 + t 2 ) ú + (1 + t 2 ) = 0
ë dt û p
2
Cancelling (1 + t2) throughout we get p
2
d y dy
ò
= u sin 2udu =
4
[Integrate by parts]
0
(1 + t 2 ) + (2t + y tan -1 t) = -1 Þ k = – 1
dt 2 dt
x - 6 y +1 z + 3
27. (d) f2 (x) = f (f (x)) = f (x) = x 30. (d) Given that = = is the equation of
-1 0 4
f3 (x) = f (f2 (x)) = f (x) = x Þ x3 – 25x2 + 175x – 375 = 0
line and the equation of plane is x + y – z = 3
(x – 5) (x2 – 20x + 75) = 0
(x – 5)2 (x – 15 ) = 0 Þ x = 5, 15 x - 6 y +1 z + 3
Assume, = = = l (say)
28. (c) To determine ' c ' in Rolle’s theorem, f ¢(c) = 0 -1 0 4
Here Þ x = 6 – l, y = – 1 and z = 4l – 3
Since, the given line meets the plane,
æ 1ö \ By putting the value of x, y, z in the equation of
f ¢( x ) = ( x 2 + 3x)e -(1/ 2) x . ç - ÷ + (2 x + 3)e- (1/ 2) x
è 2ø the plane, we get
(6 – l) – 1– (4l – 3) = 3
ì 1 ü
î 2 þ 2
{
= e-(1/ 2) x í- (x2 + 3x) + 2x + 3ý = - 1 e -( x / 2) x 2 - x - 6 } Þ 6 – l – 1 – 4l + 3 = 3
Þ 8 – 5l = 3
Þ – 5l = – 5
\ f ¢(c) = 0 Þ c 2 - c - 6 = 0 Þ c = 3, -2, But Þ l=1
c = 3 Ï [-3,0]. \ The co-ordinates of the points
= (6 – 1, – 1, 4 – 3)
= (5, – 1, 1)
EBD_7504
Speed Test-99
æ 1 3ö Similarly, LMVT in [1, 2] gives, some b Î (1, 2) such that
1. (d) Centre of the circle is ç ,- ÷ .
è 2 2ø g(2) - g(1)
= g ¢ ( b) .......... (2)
Its distance from the line x + y – 1 = 0 is 2 2 -1
Let the required line be mx – y = 0 Eq. (1) + Eq. (2)
g ¢(a ) + g ¢ (b) = g(2) - g(0) ; but g' (x) = f (x4) . 4x3
m 3 {
+ zero
\ 2 2 = 2 Þ m = 1, –1/7
16
m2 + 1 \ 4 éëa 3f (a 4 ) + b3f (b4 ) ùû = ò f (t) dt
0
\ The lines are x – y = 0, x + 7y = 0 7. (c) Given a, b, c are in A.P.
2. (b) Number of persons whose can speak Bengali only \ 2b = a + c ....(1)
= n (B – A) = n (B) – n (A Ç B) = 400 – 150 = 250
3. (b) Let f (x) = sin x + x cos x x +1 x + 2 x + a
Consider Now, x + 2 x+3 x+b [Applying R2 ®2 R2]
x
x +3 x + 4 x +c
g (x) = ò (sin t + t cos t) dt = t sin t]0x = x sin x
0
g (x) = x sin x which is differentiable x +1 x + 2 x+a
1
Now, g (0) = 0 and g (p) = 0, using Rolles Theorem = 2x + 4 2x + 6 2x + 2b
$ atleast one c Î (0, p) such that g '(c) = 0 2
i.e. c cos c + sin c = 0 for atleast one c Î (0, p) x+3 x+4 x+c
4. (a) We know that log x is defined for x > 0 and
log | x | is defined for all x Î R – {0} x +1 x + 2 x+a
Also |log x | > 0 1
= 2x + 4 2x + 6 2x + (a + c)
\ Required area which is symmetrical in four quadrants 2
x+3 x+4 x+c
Y
[ using equation (1) ]
[Applying R2 ® R2 – (R1 + R3) ]
x +1 x + 2 x + a
1 1
= 0 0 0 = ×0 = 0
X' X 2 2
–1 1 x+3 x +4 x+c
8. (d) Case I : x Î [–1, 0]
Y' 1+ x2
x+ = -2x Þ x2 + 6x + 1 = 0
1 1 2
= 4 ò0 | log x | dx = -4 ò0 log x dx Þ x = 2 2 - 3 Þ |10a | = [| 20 2 - 30 |] = 30 - 20 2
( Q at (0, 1), log x < 0)
Case II : x Î [0, 1]
1
= – 4 [x log x – x ]0 = – 4 (–1 ) = 4 sq. unit.
5. (a) Number of digits are 9 1 + x2
x+ = 2x Þ 1 + x2 = 2x Þ x = 1 Þ | 10a | = 10
Select 2 places for the digit 1 and 2 in 9C2 ways 2
from the remaining 7 places select any two places for 3 and 4 in
7C ways and from the remaining 5 places select any two for 5 and | 10a | = 10, | 20 2 - 30 |
2
6 in 5C2 ways Þ [ | 10a | ] = 1, 10
Now, the remaining 3 digits can be filled in 3! ways
\ Total ways = 9C2 . 7C2 . 5C2 . 3! sin 2 y 1 + cos y sin y
9. (d) 1- + -
9! 7! 5! 1 + cos y sin y 1 - cos y
= . . .3! = 9.7!
2!.7! 2!.5! 2!.3!
16 1 + cos y - sin 2 y 1 - cos2 y - sin 2 y
= + = cos y
6. (b) Let I = ò f (t) dt 1 + cos y sin y (1 - cos y)
0 10. (c) x2 – 2x cos q + 1 = 0,
x4
2 cos q ± 4 cos 2 q - 4
Consider g(x) = ò f (t) dt Þ g(0) = 0 \ x=
2
= cos q ± i sin q
0
LMVT for g in [0, 1] gives, some a Î (0, 1) such that Let x = cos q + i sin q \ x2n – 2xn cos nq + 1
q + i sin nq) cos nq + 1
g(1) - g(0) q – 2 sin nq cos nq)
= g ¢ (a )
1- 0
S-MT-16 SOLUTIONS

8 A Bx + C 1 -3 dy dz 1 dz z 1
11. (a) Let = + ........ (i) Put =z\ = Þ + = 3 cos x
(x + 2)(x + 4)2 x + 2 x2 + 4 y 3
y 4 dx dx 3 dx x x
Then, 8= A (x2 + 4) + (Bx + C) (x + 2) ........(ii)
Putting x + 2 = 0 i.e. x = – 2 in (ii), we get 8 = 8A Þ A = 1 dz 3 3
+ . z = 3 cos x
Putting x = 0 and 1 respectively in (ii), we get dx x x
8 = 4A + 2C and 8 = 5A + 3B + 3C This is linear differential equation in z.
Solving these equation, we obtain A =1, C = 2, B = – 1
\ I.F. = e ò
Pdx 3 3
Substituting the values of A, B and C in (i) , we obtain = ò dx = e3log x = e3log x = x3
8 1 -x + 2 x
e
= + \ Solution is given by
(x + 2)(x + 4)2 x + 2 x2 + 4
8 1 -x + 2 z.x 3 = ò 3 cos x dx + c = 3sin x +c
\ ò (x + 2)(x 2 + 4) dx = ò
(x + 2)
dx + ò 2 dx
x +4 x3
Put value of z Þ = 3sin x + c
1 x 1 y3
= ò (x + 2) dx - ò x 2 + 4 dx + 2ò x 2 + 4 dx Given boundary condition is x = 0, y = 1
\ 0=0+c (Q sin 0 = 0)
1 1 1 x
= log | x + 2 | – ò dt + 2. tan -1 + C, (where t = x2 + 4) \ c = 0 \ x3 = 3 y3 sin x is the solution.
2 t 2 2 15. (a) The question can be solved by taking the four different
1 x choices given in the question.
= log | x + 2 | – log t + tan–1 2 + C 1- x
2 Now, Let us consider f (x) =
1 x 1+ x
= log | x + 2 | – log (x2 + 4) + tan–1 2 + C 1- x
2 Let f (x) = y \ y=
r 1+ x
12. (b) n = 7i + 2jˆ - kˆ is normal to plane
ˆ
1- y
(Assuming n = aiˆ + bjˆ + ckˆ and using P(8,2,–12)
Þ y + xy = 1 – x Þ x =
1+ y
r uuur r uuur r uuur n 1- x
n.AB = 0, n.BC = 0, n.AC = 0 ) Thus f (x) = is the inverse of itself.
P = (8, 2, – 12) 1+ x
16. (c) Given circles are x2 + y2 + 4x – 6y + 9 = 0
uuur and x2 + y2 – 5x + 4y – 2 = 0
AP = 6iˆ + 3jˆ - 16kˆ \ locus of centres is 9x – 10y + 11 = 0
\ Distance 17. (c) Given series is
uuur r A(2,–1,4) 1 1 1 1
AP.n 42 + 6 + 16 64 64 64 6 32 6 + + + + .....,
d= r = = = = = 1 1 + 2 1 + 2 + 3 1+ 2 + 3 + 4
|n| 49 + 4 + 1 54 3 6 18 9
13. (a) 1 2 é1 1 ù
r r r r r r r r r tn = = =2 ê -
V = A ´ ëé (A.B)A - (A.A)B ûù .C 1 + 2 + 3 + .....n n(n + 1) ë n n + 1 úû
ær r r r r r r rö r r r r r éæ 1 1 ö æ1 1ö æ1 1ö ù
\ Sum = S = 2 ê ç - ÷ + ç - ÷ + ç - ÷ + .......ú = 2
= çA ´ (A.B)A - (A.A)A ´ B ÷ .C = – | A |2 [A B C]
1424 3 ëè 1 2 ø è 2 3 ø è 3 4 ø û
ç ÷
è zero ø 18. (c) It is in 0/0 form, so using Hospital rule, we have
r 2
Now, | A | = 4 + 9 + 36 = 49 1
cos x -
2 3 6 Limit = lim 1- x (0/0 form)
r r r x®0 2x
[A B C] = 1 1 -2 1
- sin x -
1 2 1
= 2 (1 + 4) – 1 (3 – 12) + 1 (–6 – 6)
(1 - x ) 2
= lim = – 1/2
= 10 + 9 – 12 = 7 x®0 2
r 2
r r r
\ - | A | [A B C] = 49 ´ 7 = 343 6R
14. (b) Given differential equation is 19. (a) Box 5B
dy 4W
x2 y – x3 = y 4 cos x [Bernoulli's form]
dx P(E) = P(R R B W or B B R W or W W R B)
dy n (E) = 6C2 · 5C1 · 4C1 + 5C2 · 6C1 · 4C1 + 4C2 · 6C1 · 5C1
Þ x3 - x 2 y = -y 4 cos x n (S) = 15C4
dx
Dividing throughout by – x3 y4, we get
720 × 4! 48
-1 dy 1 1 1 \ P (E) = =
+ . = cos x 15 ×14 ×13 × 12 91
y 4 dx y3 x x 3 20 (c) If rth term is independent of x, then by the formula
Þ r – 1 = 9 Þ r = 10
E B D _
Mathematics S-MT-17

27. (c) A: blood result says positive about the disease


1 1 1
21. (c) Given: f(x) = \ f (a) = and f(b) = 1
x a b B1: Person suffers from the disease \ P(B1) =
100
1
Also f '(x) = – 99
x2 B2 : person does not suffer \ P(B2) =
100
Now, according to the given theorem
99 1
1 1 -1 a - b -(b - a) P(A/B1) = , P(A/B2) =
– = (b – a) Þ = 100 100
b a x 2 ab x2
P(B1 ).P(A / B1 )
1 1 P (B1 / A) =
Þ = 2 Þ x= ab . P(B1 ).P(A / B1 ) + P(B2 ).P(A / B 2 )
ab x
22. (a) Given : f (x) = x4 – 62 x2 + ax + 9 1 99
×
100 100 99 1
f '(x) = 4x3 – 124 x + a \ f '(x) = 0 at x = 1 = = = = 50%
1 99 99 1 2(99) 2
Þ 4 (1)3 – 124 (1) + a = 0 Þ 4 – 124 + a = 0 Þ a = 120 × + ×
100 100 100 100
Thus, f(x) attains its maximum value when a = 120.
23. (b) Since ~(p Ú q) º (~p Ù ~q) and ~ (p Ù q) º (~p Ú ~ q) 5x + 1
28. (a) Suppose that t = , so y = f (t)
So option (a) and (d) are not true. 10x 2 – 3
(p ®q) º p Ù ~q, so option (c) is not true.
p ® q º ~q ® ~p
dy dt
\ = f '(t). [Since f¢(x) = cos x]
dx dx
24. (a) Domain = {x ; 2x – 3x ³ 0} = {x ; (2/3)x ³ 1} = x Î(–¥, 0]
æ 1ö æ 1 ö æ 1ö æ 1 ö dy æ 5x + 1 ö d æ 5x + 1 ö
çè 1 + ÷ø çè1 + 2 ÷ø + çè 2 + w ÷ø çè 2 + 2 ÷ø = cos ç
è 10x 2 – 3 ÷ø dx çè 10x 2 – 3 ÷ø
25. (b) .
w w w dx
æ 1ö æ 1 ö æ 1ö æ 1 ö 29. (d)
+ ç3 + ÷ çè 3 + 2 ÷ø + .... + çè n + w ÷ø çè n + 2 ÷ø
è wø w w (a) Tr (kA) = k (a11 + a22 + a33) = kTr(A)
æ 1öæ 1 ö (b) Tr (A + B) = a11 + b11 + a22 + b22 + a33 + b33
Consider ç r + ÷ r+
è w ø çè w2 ÷ø = Tr (A) + Tr (B)
(c) Tr (I3) = 1 + 1 + 1 = 3
= r 2 + (w + w 2 ) r + 1 = (r 2 - r + 1)
2 2
n (d) Tr (A2) = S a11 + S a12 ¹ (a11 + a 22 + a 33 ) 2
n(n + 1)(2n + 1) n(n + 1)
= å (r - r + 1) =
2
- +n
r =1 6 2 p /2
30. (a) an = ò (1 - sin t) n sin 2t dt
n n n(n 2 + 2) 0
= [2n2 + 3n + 1 – 3n – 3 + 6] = (2n2 + 4) =
6 6 3 Let 1 – sin t = u Þ – cos t dt = du
1 æ1 1 ö
= 2 u n (1 - u) du = 2 ç u n du - u n +1du ÷
ò çò ò
26. (d) Let the equation of tangent is y = mx + a 2 m 2 + b 2
÷
Foci º (± ae, 0), vertices º (± a, 0), C º (0, 0)
0 è0 0 ø

æ 1 1 ö
mae + a 2 m 2 + b2 - mae + a 2 m 2 + b 2 =2ç - ÷
\ s= , s¢ = è n +1 n + 2 ø
1 + m2 1 + m2
an æ 1 1 ö
Hence, = 2ç - ÷
n è n(n + 1) n(n + 2) ø
ma + a 2 m 2 + b 2 - ma + a 2 m 2 + b 2
a= , a¢ =
n
an æ æ1 1 öö 1 æ 1 1 ö
1 + m2 1 + m2
, lim å n
= 2 çåç -
n n + 1
÷÷ - åç -
2 n n
÷
+2ø
n ®¥ 1 è è øø è

m2 a 2e2 æ n
æ1 1 öö n
æ1 1 ö
2
a m +b 2 2
ss¢ - c 2 - 2
= 2ç
ç å çè n - n + 1 ÷ø ÷÷ - å çè n - n + 2 ÷ø
c= = 1 + m = e2 è 1 ø 1
\
1 + m2 aa ¢ - c 2 m2a 2 é 1 1 1ö æ1 1ö ù 3 1
+ ç - ÷ + .....ú = 2 - =
è3 5ø û 2 2
HINTS & SOLUTIONS (FULL TESTS)
Speed Test-100

PHYSICS 1 N
\ t= log 0
1. (c) For total internal reflection on face AC l N
q > critical angle (C) 2.303 ´ T1/2 N
and sinq ³ sinC = ´ log10 0
0.693 N
1
sin q ³
wm g dN 0
4 N 20
mw But dt = 0 = = 10
sin q ³ Þ sin q ³ 3 dN N 2
mg 3
dt
2
8
\ sin q ³ . 2.303 ´ 5730
9 \ t= ´1
2. (a) As we know, 0.693
-GMm = 19039 years
Gravitational potential energy = 5. (a) The forces acting on the block are shown. Since the
r block is not moving forward for the maximum force F
and orbital velocity, v0 = GM / R + h applied, therefore
1 GMm 1 GM GMm F cos 60° = f = µN... (i) (Horizontal Direction)
Ef = mv02 - = m -
2 3R 2 3R 3R Note: For maximum force F, the frictional force is the
GMm æ 1 ö - GMm limiting friction = µN]
= ç - 1÷ =
3R è 2 ø 6R and F sin 60° + mg = N... (ii)
-GMm From (i) and (ii)
Ei = +K
R
N
Ei = E f
5GMm
Therefore minimum required energy, K = F cos 60o
6R
3. (a) Given, R1 = 100 W, r' = r/2, R2 = ? mg 60
o
f
rl
Resistance of wire, R =
A
Q Area × length = volume F
rV F sin 60o
Hence, R = 2
A F cos 60° = µ [F sin 60° + mg]
Since, r ® constant, V ® constant
m mg
1 Þ F=
Rµ 2 cos 60° - m sin 60°
A
1 1
or R µ Q A = pr2 ´ 3 ´ 10
r4 2 3 5
= = = 20 N
R2 1 1 3 1
= 16 Þ R2 = 16 ´100 = 1600 W - ´
R1 2 2 3 2 4
dN 0 6. (b) If student will use angular momentum = mvr.
4. (c) Given: = 20 decays/min He/she may conclude answer (a) as r is decreasing
dt
angular momentum must decrease hence (a) is
dN incorrect.
= 2 decays/min
dt m
v
T1/2 = 5730 years
q
As we know, mg r
-lt
N = N 0e
/////////////////////////////////////////
O
N d
log 0 = l t
N
EBD_7504
S-FT-2 SOLUTIONS

But the magnitude of angular momentum of particle Dividing eqn (i) by eqn (ii),
about O = mvd
Since speed v of particle increases, its angular 1 3
+
momentum about O increases. 2 2+ 3
tan q = 2 =
Magnitude of torque of gravitational force about 1 1 1 - 2
O = mgd Þ constant -
2 2
Moment of inertia of particle about O = mr2
Hence MI of particle about O decreases. 11. (d) The thermal resistance is given by
v sin q x 4x x 2 x 3x
Angular velocity of particle about O = + = + =
r KA 2KA KA KA KA
Q v and sin q increases and r decreases. dQ DT (T2 - T1 ) KA
\ angular velocity of particle about O increases. \ = =
dt 3x 3x
7. (d) At equilibrium, weight of the given block is balanced KA
by force due to surface tension, i.e.,
2L. S = W 1 ì A(T2 - T1 ) K ü
= í ý
3î x þ
W 1.5 ´ 10 -2 N
or S = = = 0.025 Nm -1 1
2L 2 ´ 0.3 m \f =
3
1æUö v v v
8. (a) As, P = ç ÷ 12. (b) n = , n1 = , n2 =
3èVø 0
2l 2 ( l / 2 - Dl) 2 ( l / 2 + Dl)
U Beat frequency = n1 – n 2
But = KT 4
V é 1 1 ù
1 Þ vê -
ë l - 2Dl l + 2 Dl úû
4
So, P = KT
3
(l + 2Dl) - (l - 2Dl) ù
or
uRT 1
= KT 4 [As PV = u RT] = v éê ú
V 3 ë l 2 - 4 Dl 2 û
4 4D l 8 Dlv 8Dln 0
pR 3T 3 = constant =v = =
3 l2 - 4Dl 2 l 2l l
1 13. (c) Wavelength for which maximum obtained at the hole
Therefore, Tµ
R has the maximum intensity on passing. So,
I e ne eAve 7 7 v v 5 nlD
9. (c) = Þ = ´ e Þ e = x=
I h nh eAvh 4 5 vh vh 4 d
10. (a) For particle C,
xd 1 ´ 10-3 ´ 0.5 ´ 10-3
According to law of conservation of linear momentum, l= =
verticle component, nD n ´ 50 ´ 10-2
2 mv' sin q = mv sin 60° + mv sin 45°
1 ´ 10-6 1000nm
= =
mv mv 3 ...... (i) n n
2mv 'sin q = +
2 2 n = 1, l = 1000 nm ® Not in the given range
Horizontal component, n = 2, l = 500nm
2 mv' cos q = mv sin 60° – mv cos 45° I C q
mv mv 14. (b) I
2mv'cos q = - ...... (ii) I
2 2
L
Y A I
Y' I
B I
v sin 60°

As it can be easily seen by the direction of I that Q is


30° v sin 45° decreasing thus, energy of capacitor is decreasing and

60° 45° hence, energy of inductance is increasing or æç 1 LI 2 ö÷


X X' è2 ø
v cos 60° – v cos 45° gives that I is increasing.
For particle A For particle B
Full Test S-FT-3

1 2 18. (a) The current voltage relation of diode is


15. (d) Distance from A to B = S = ft 1
2 I = (e1000 V /T - 1) mA (given)
Distance from B to C = (ft1 ) t When, I = 5mA, e1000 V /T = 6mA
2 2
u (ft 1 )
Distance from C to D = = 1000
2a 2( f / 2) Also, dI = (e1000 V /T

T
= ft 12 = 2S (By exponential function)
A f B C f /2 D 1000
= (6 mA) ´ ´ (0.01) = 0.2 mA
300
t1 t 2t 1 19. (c) As insect moves along a diameter, the effective mass
and hence the M.I. first decreases then increases so
15 S from principle of conservation of angular momentum,
Þ S + f t1t + 2S = 15 S
angular speed, first increases then decreases.
Þ f t1t = 12 S ............. (i)
R 1R 2
1 2 20. (b) Req = ( Parallel combination);
f t1 = S ............ (ii) R1 + R 2
2
RNet = Reg+R3(Series combination)
t
Dividing (i) by (ii), we get t1 = RNet of the circuit
6
2 5 ´ 15 125 75 5
f t2 + + =5 W
Þ S = f æç ö÷ =
1 t = =
5 + 15 100 20 4
2 è 6ø 72
16. (a) Capacitors 2mF and 2mF are parallel, their equivalent = R1= 5 W
4 mF I1 A
6mF and 12 mF are in series, their equivalent = 4 mF R 2= 15 W 4A R 3= 1.25 W
Now 4mF (2 an d 2 mF) and 8mF in series 4 – I1
4A
3
= λF
8
And 4mF (12 & 6 mF) and 4mF in parallel
= 4 +4 = 8mF E
We know that I = = 20 = 4 A
1 8 R eq 5
8mF in series with 1mF = ∗1 Þ λF
8 9 Potential difference across R1 and R2 are same
8 8 32 (Parallel combination) I1R1 = (4 – I1)R2
Now Ceq = ∗ <
9 3 9 Þ 5I1 = (4 – I1) ´ 15 Þ I1 = 12–3I1 Þ I1 = 3A
1 1 9 32 Thus reading of ammeter = 3A
With C, C < C ∗ 32 < 1 Þ C < 23 μF Voltage across 1.25W = I ´ R = 4 ´ 1.25 = 5V (Reading
eq
of voltmeter)
17. (c) From question, 21. (a) Optical source frequency
Horizontal velocity (initial),
c
f = = 3 ´ 108 /(800 ´ 10-9 ) = 3.8 ´ 1014 Hz
40 l
ux = = 20m/s
2 Bandwidth of channel (1% of above) = 3.8 × 10 12Hz
1 2 Number of channels = (Total bandwidth of channel) /
Vertical velocity (initial), 50 = uy t + gt (Bandwidth needed per channel)
2
Number of channels for audio signal
1
Þ uy × 2 + (–10) ×4 = (3.8 ´ 1012 ) /(8 ´ 103 ) ~ 4.8 ´ 108
2
or, 50 = 2uy – 20 22. (a) Let initial temperature and volume be T0 and V0.
Since the process is adiabatic, the first temperature
70
or, uy = = 35m / s and volume is TV g -1 = T V g -1 (g = 5/3 for
2 0 0
u y 35 7 monoatomic gas)
\ tan q = = =
u x 20 4 2/3
7 = 4T0
Þ Angle q = tan–1
4
EBD_7504
S-FT-4 SOLUTIONS

Q When the magnet is cut into three pieces the pole


23. (c) Flux going in pyramid = 2e strength will remain the same and
0
Which is divided equally among all 4 faces 2
1 æ m öæ l ö I
Q M.I. (I¢) = ç ÷ç ÷ ´ 3 =
12 è 3 ø è 3 ø 9
\ Flux through one face = 8e
0 We have, Magnetic moment (M)
24. (d) From the figure it is clear that liquid 1 floats on liquid 2. = Pole strength (m) × l
The lighter liquid floats over heavier liquid. Therefore \ New magnetic moment,
we can conclude that r1 < r2
Also r3 < r2 otherwise the ball would have sink to the æ lö
M ' = m ´ ç ÷ ´ 3 = ml = M
bottom of the jar. è 3ø
Also r3 > r1 otherwise the ball would have floated in
liquid 1. From the above discussion we conclude that 2 T
\T'= s. =
r1 < r3 < r2. 9 3
p 30. (b) By Newton’s law of cooling
25. (b) A = 0.1m, m = 0.1 kg, KEmax = 18 × 10–3 J, f =
4 q1 - q2 é q + q2 ù
= -K ê 1 - q0 ú
36 ´ 10-3 k 3.6 t ë 2 û
k= = 3.6 ; w = = = 6 rad / s
(0.1) 2 m 0.1 where q0 is the temperature of surrounding.
æ pö Now, hot water cools from 60°C to 50°C in 10 minutes,
\ Eqn. y = 0.1 sin ç 6t + ÷
è 4ø 60 - 50 é 60 + 50 ù
= -K ê - q0 ú ...(i)
26. (a) First, the length of wire goes on increasing ie., area 10 ë 2 û
decreases and finally at breaking stress the wire breaks. Again, it cools from 50°C to 42°C in next 10 minutes.
27. (a) Force acting on conductor B due to conductor A is 50 - 42 é 50 + 42 ù
given by relation = -K ê - q0 ú ...(ii)
m 0 I1 I 2 l 10 ë 2 û
F= Dividing equations (i) by (ii) we get
2 pr
l-length of conductor B 1 55 - q0
r-distance between two conductors =
0.8 46 - q0
4p´ 10-7 ´ 10 ´ 2 ´ 2 10 55 - q 0
\F= = 8 × 10–5 N =
2 ´ p´ 0.1 8 46 - q 0
28. (a) O r 460 - 10q0 = 440 - 8q 0
2q0 = 20
F1 a q0 = 10° C
R y Bowl
CHEMISTRY
A h
æ 1 1 ö
mg cos a mg sin a 31. (c) n = RcZ2 ç 2 - 2 ÷
çn ÷
mg è 1 n2 ø

The insect crawls up the bowl upto a certain height h æ1 1 ö


only till the component of its weight along the bowl is n1 = RcZ2 ç 2 - 2 ÷ = RcZ2
è1 ¥ ø
balanced by limiting frictional force.
For limiting condition at point A æ1 1 ö 3RcZ
2
R = mg cosa ...(i) n 2 = RcZ2 ç 2 - 2 ÷ =
F1 = mg sina ...(ii) è1 2 ø 4
Dividing eq. (ii) by (i) 2
F æ 1 1 ö RcZ
tan a =
1
= 1 = m [ As F1 = m R ] n 3 = RcZ2 ç 2 - 2 ÷= 4
cot a R è2 ¥ ø
1é 1 ù \ n1 - n2 = n3
Þ tan a = m = êQm = ( Given ) ú
3ë 3 û 32. (a)
\ cot a = 3
b 3° b 3°
I I a
29. (b) T = 2p = 2p
M ´B MB
b + ¾¾® (2 H)
1 N N
where I = ml 2
12
Full Test S-FT-5

33. (c) ICl,


2 Þ 2 bp + 3lp (– CH2 – CH = CH – CH2 – CH – CH2 –)n
(thus, sp3d hybridisation) = linear Butadiene -
Styrene copolymer
BrF2∗ Þ 2 bp + 2lp (SBR or BUNA - S)
(thus, sp3 hybridisation) = pyramidal 39. (b) In electrolysis of NaCl when Pt electrode is taken, then
H2 liberated at cathode, while with Hg cathode it forms
ClF4, Þ 4 bp + 2lp
sodium amalgam because more voltage is required to
(thus, sp3d2 hybridisation) = square planar reduce H+ at Hg than at Pt.
AlCl,
4 Þ 4 bp + 0lp
2+ 1
40. (b) No. of A = ´ 8 = 1
8
(thus sp3 hybridisation) = tetrahedral
34. (c) The correct formula of inorganic benzene is B3N3H6 so 3+ 1
No. of B = ´ 4 = 2
(d) is incorrect statement. 2
OH 2- 1 1
| No. of O = 8 ´ ´ 6 ´ = 4
Boric acid (H3BO3 or B - OH ) is a Lewis 8 2
| (AB2O4)
OH 41. (d)
acid so (a) is incorrect statement. (a) For tetrahedral d6 ion,
The coordination number exhibited by beryllium is 4
and not 6 so statement (b) is incorrect.
Both BeCl2 and AlCl3 exhibit bridged structures in solid
state so (c) is correct statement. 4 unpaired electrons
35. (b) H2 (g) + CO2 (g) ƒ CO (g) + H2O (g) (b) For [Co(NH3)6]3+,
At eqm 0.25–x 0.25–x x x
2
x
Kp = 0.16 =
(0.25 - x) 2 0 unpaired electrons
x (c) For square planar d7 ion,
Þ 0.4 = Þ 0.1 - 0.4x = x
0.25 - x
x = 0.0714
0.0714
Mole% of CO (g) = ´ 100 = 14.28
0.50 1 unpaired electrons
(d) B.M. = n(n + 2) , n = unpaired electrons
Anhydrous HI
36. (a) ¾¾¾¾¾® (I)
O S N2 I 5.92 B.M. = n(n + 2)
HO
n = 5 unpaired electrons
Conc. HI P° - P n2
(II) 42. (d) =
S N2 I P° n1 + n2
HO
37. (d) One mole of a substance contains the number of 640 - 600 2.5 m
=
molecules which is independent of pressure. 640 39 78

38. (b) nCH 2 = CH - CH = CH 2


640 ´ 78 ´ 2.5
1, 3-Butadiene m= = 80
39 ´ 40
CH = CH2 43. (a) (a) SiC Þ Covalent carbide
(b) Be2C + 4H2O ¾¾ ®
Na, Heat
+n ¾¾¾¾¾® 2Be(OH)2 + CH4 ­
Polymerisation
(c) CaC2 + 2H2O ¾¾
®
Styrene
HC º CH ­ + Ca(OH)2
(d) Mg C + 2H O ¾¾
®
2Mg(OH)2 + H3C – C º CH ­
EBD_7504
S-FT-6 SOLUTIONS

Conc. HI
44. (a) CH3CH2O CH(CH3)2 ¾¾¾® CH3CH2I + (CH3)2CHI
(A) (B) (C)

NaOH NaOH

CH3CH2OH (CH3)2CHOH

[O] [O]

CH3COOH (CH3)2CO

hence the IUPAC name of compound is


2 1
CH3 — CH 2 — O — CH — CH3
½
3CH3
2 -ethoxy propane

P X 47. (a) The compounds of the type M(AA)2B2 exhibit both


45. (c) PNH3 = PH 2S = = geometrical and optical isomerism
2 2
æ Xö
2 dMnO4–
DG° = - RT ln K p = - RT ln ç ÷ 48. (b) Given - = 4.56 × 10–3 Ms–1
è 2ø dt
From the reaction given,
æ Xö
= -2RT ln ç ÷ = – 2RT (ln X – ln 2)
è 2ø 1 dMnO 4 – 4.56 ´ 10-3
- = Ms -1
46. (c) 2 dt 2
(a) CdS ¯ + (NH 4 ) 2 S ¾¾
® CdS ¯
Yellow Yellow 1 dMnO 4– 1 dI 2
- =
(NH ) S
¾¾¾¾¾
4 2 2 ® CdS ¯ 2 dt 5 dt
Insoluble
5 dMnO4– dI2
(b) SnS2 ¯ + (NH 4 ) 2 S ¾¾
® CdS ¯ \ - =
Yellow 2 dt dt
(NH 4 )2 S2 On substituting the given value
¾¾¾¾¾
® (NH 4 )2 SnS3
Soluble
dI2 4.56 ´ 10-3 ´ 5
(c) Cd 2+
+ (NH 4 ) 2 S ¾¾
® CdS ¯ \ = = 1.14 × 10–2 M/s
Yellow
dt 2
(NH ) S Partial
¾¾¾¾¾
4 2 2 ® CdS ¯ 49. (d) XeF6 + H 2 O ¾¾¾¾
® XeOF4 + 2HF
Insoluble
2+ Partial
(d) Sn + (NH 4 ) 2 S ¾¾
® SnS ¯ XeF6 + 2H 2 O ¾¾¾¾
® XeO2 F2 + 4HF
Brown
Complete
(NH 4 ) 2 S2
¾¾¾¾¾
® (NH 4 ) 2 SnS3 XeF6 + 3H 2 O ¾¾¾¾¾
® XeO3 + 6HF
(Amm. thiostannate)
Soluble
Full Test S-FT-7

50. (b)

COOK
aq. KOH COOH COOK
Me – CH = CH2 + CHCl3 ¾¾¾® CH3 CH – CH3 |
CH3 – CH – CH2 ¾¾® CH3 – CH – CH3
CCl 2

Cl Cl HO OH O OH
C C
aq. KOH OH
CH3 – CH – CH2 ¾¾¾® CH3 – CH – CH2 ¾¾¾®
– H2O
CH3 – CH – CH2

51. (a) 2N 2 O5 (CCl4 ) ¾¾


® 2N 2O 4 (CCl4 ) + O 2 (g) 54. (d) 3CH4 + 2O3 ¾® 3CH2= O + 3H2O
Formaldehyde
t=0 a mol 0
x CH 2 = CHCH = O CH 3 COONO 2
t = 500 min. a–x mol Acrolein Peroxyacetyl nitrate (PAN)
2
a 55. (b)
t=¥ 0 mol
2
a = 100 – +
x = 90 CH = O CO2 K
Conc. KOH
¾¾¾¾®
Cannizzaro
1 a 2.303 CH = O
k= ln = CH2OH
t a-x 500
52. (a) The order of basic character of the transition metal
monoxide is TiO > VO > CrO > FeO because basic (A) (B)
character of oxides decreases with increase in atomic 56. (d) (a) The concentration of sodium thiosulphate
number. solution should always be less than the
53. (d) NH4ClO4 + HNO3 (dilute) ¾¾® concentration of the potassium iodide solution.
(b) Freshly prepared starch solution should be used
NH4NO3 + HClO4
(c) Experiments should be performed with the fresh
(X)
solutions of H2O2 and KI.
NH4NO3 ¾¾¾ heat
® N2O + 2H2O
(X) (Y)

+
+
Ring 1, 2 – CH3
H+ expansion shift +
57. (d)
(3° Carbocation stable)

q = +5, w = –8J
A ¾¾¾¾¾¾® B Therefore, repulsion between both C – O bonds
58. (d)
will be greater than the repulsion between C – H
DU AB = q + w = +5 + (- 8) = - 3 bond and C – OH bond or between C – H and C
= O bond. Repulsion between C – H and C = O
q = – 3, DU BA = + 3
bond is greater than that of between C – H and
DU BA = q + w C– OH bond, because double bond character in
Þ 3 = -3 + w Þ w = + 6 J C– OH bond will always be less than C = O.
(work done on the system). 60. (c) m = E.wt ´ Q ;
59. (a) 96500
m ´ 96500 22.2 ´ 96500
O O \ E. wt = = = 60.3
Q 2 ´ 5 ´ 60 ´ 60
y x y x At wt. 177
C C Oxidation state = = =3
H + H Eq. wt . 60.3
H z O H z O
(more stable structure)
D
B
E
S-FT-8 SOLUTIONS

MATHEMATICS represents a parabola with vertex (0,2) axis as


y–axis and concavity downwards. Both the curves are
æ 1 3ö plotted in the figure and the required area is shown by the
61. (d) Centre of the circle is çè , - ÷ø . shaded region.
2 2
Its distance from the line x + y – 1 = 0 is 2 The points A and C are the points of intersection of y 2 = x 2
Let the required line be mx – y = 0 with x 2 + y - 2 = 0 .
m 3 Solving the two equations, we get
+
\ 2 2 = 2 Þ m = 1, –1/7
y 2 + y - 2 = 0 [putting value of x2 = y2]
m2 + 1
Þ ( y + 2)( y - 1) = 0
\ The lines are x – y = 0, x + 7y = 0 giving y = –2 and 1, but y = –2 is discarded as the required
62. (a) Since a 1 , a 2 and b 1 , b 2 are the roots of area is above the x-axis.
ax2 + bx + c = 0 and px2 + qx + r = 0 respectively, \y = 1 Þ x = ± 1
therefore
The points A and C are respectively (–1, 1) and (1, 1) now
-b c due to symmetry
a1 + a2 = , a1a2 = . ..(1)
a a Area of the bounded region OABCO
-q
[ ]
r 1
and b1 + b2 = , b1b2 = ...(2) = 2 × Area OBCO = 2 × ò ( 2 - x 2 ) - x dx
p p
0
Since the given system of equations has a non-trivial solution
[Since y = 2 –x2 is the upper curve and y = x is the lower
a1 a 2 curve]
\ = 0 i.e. a1b2 – a2b1 = 0
b1 b 2 1
é x3 x 2 ù é 1 1ù 7
a1 a 2 a1 + a 2 a1a 2 = 2ê2x - - ú = 2 ê2 - - ú =
or = = = êë 3 2 úû ë 3 2û 3
0
b1 b 2 b1 + b 2 b1b 2
65. (a) Number of digits are 9
pb pc b 2 ac Select 2 places for the digit 1 and 2 in 9C2 ways from
Þ = Þ = the remaining 7 places select any two places for 3 and
qa ra q 2 pr 4 in 7C2 ways and from the remaining 5 places select
63. (b) Suppose, there are two points x1 and x2 in (a, b) such any two for 5 and 6 in 5C2 ways
that f '(x1) = f '(x2) = 0. By Rolle's theorem applied to f ' Now, the remaining 3 digits can be filled in 3! ways
on [x1, x2], there must be a c Î ( x1, x2 ) such that f ''(c) \ Total ways = 9C2 . 7C2 . 5C2 . 3!
= 0. Th is contradicts the given condition 9! 7! 5!
= . . .3! = 9.7!
f ''( x ) < 0, " x Î (a, b). Hence, our assumption is 2!.7! 2!.5! 2!.3!
16
wrong. Therefore, there can be atmost one point in
(a, b) at which f '(x) is zero.
66. (b) I= ò f (t) dt
64. (b) 0

y x4

y = –x B (0, 2) y=x
Consider g(x) = ò f (t) dt Þ g(0) = 0
0
LMVT for g in [0, 1] gives, some a Î (0, 1) such that
(–1, 1) A C (1, 1) g(1) - g(0)
= g ¢(a ) ..... (1)
1- 0
x
O Similarly, LMVT in [1, 2] gives, some
g(2) - g(1)
b Î (1, 2) such that = g ¢ (b ) .... (2)
We first draw the given curves 2 -1
Eq. (1) + Eq. (2)
The first curve x 2 - y 2 = 0 Þ y = ± x represents a pair of
g ¢(a ) + g ¢ (b) = g(2) - g(0) ;
straight lines with slopes 1 and –1 passing through origin. {
zero
The second curve but g' (x) = f (x4) . 4x3
Þ x 2 + y - 2 = 0 Þ x 2 = -y + 2 Þ x 2 = -( y - 2) 16
\ 4 éëa 3f (a 4 ) + b3f (b4 ) ùû = ò f (t) dt
0
Full Test S-FT-9

1 1
3u 2 2u 3 1 2 3

2 3
Þ ò ò
2 x dx > 2 x dx Þ I1 > I 2 .
3v 2v 1 =0 0 0
67. (a)
3w 2
2w 3
1 Also " 1 < x < 2
2 2
R1 ® R1 – R2 and R2 ® R2 – R3 x2 3

u+v 2
u + v + vu 2
0
2
x <x Þ 2 3
ò ò
dx < 2 x dx Þ I3 < I 4
1 1
Þ v+w 2
v + w + vw 0 = 0 2 r
72. (b) n = 7iˆ + 2jˆ - kˆ is normal to plane
2
w w3 1 (Assuming n = aiˆ + bjˆ + ckˆ and using
R1 ® R1 – R2 r uuur r uuur r uuur
n.AB = 0, n.BC = 0, n.AC = 0 )
1 u+w+v 0 P = (8, 2, – 12)
Þ v+w v2 + w 2 + vw 0 = 0 P(8,2,–12)
n
w2 w3 1
Þ (v2 + w2 + vw) – (v + w) [v + w + u] = 0
Þ uv + vw + wu = 0
68. (d) x Î [–1, 0]
A(2,–1,4)
1+ x2 uuur
x+ = - 2x
2 AP = 6iˆ + 3jˆ - 16kˆ
x2 + 6x + 1 = 0 \ Distance
uuur r
x = 2 2 - 3 Þ |10a | = [| 20 2 - 30 |] = 30 - 20 2 AP.n 42 + 6 + 16 64 64 64 6 32 6
d= r = = = = =
x Î [0, 1] |n| 49 + 4 + 1 54 3 6 18 9
1 + x2 73. (a)
x+ = 2x r r r r r r r r r
2 V = A ´ éë (A.B)A - (A.A)B ùû .C
1 + x2 = 2x Þ x = 1 Þ | 10a | = 10
æ r r r r r r r rö r r r r r
| 10a | = 10, | 20 2 - 30 | = çA1
´(A.B)A- (A.A)A´B÷.C = – | A |2 [A B C]
4 2 43
ç ÷
Þ [ | 10a | ] = 1, 10 è zero ø
69. (d) Each point (x, y) has an image in line y = 0 as (x, –y). So, r 2
replacing y by –y in the given equation, we get the Now, | A | = 4 + 9 + 36 = 49
image as
2 3 6
ax2 – 2hxy + by2 = 0. r r r
70. (c) x2 – 2x cos q + 1 = 0, [A B C] = 1 1 -2
=7
2
1 2 1
2 cos q ± 4 cos q - 4 r 2 r r r
x= = cos q ± i sin q
2 \ - | A | [A B C] = 49 ´ 7 = 343
Let x = cos q + i sin q
3p /4 3p /4
\ x2n – 2xn cos nq + 1
= cos 2nq + i sin 2nq – 2 (cos nq + i sin nq) 74. (a) I= ò (sin x + cos x) dx + ò x{ (sin
I
x - cos x) dx
14243
0 0 II
cos nq + 1
= cos 2nq + 1 – 2 cos2 nq + 3p /4
i (sin 2nq – 2 sin nq cos nq) = ò (sin x + cos x) dx + x(- cos x - sin x) |30p /4
144424443
=0+i0=0 0 zero
1 1 3p / 4
2 3
71. (b) ò
I1 = 2 x dx , I 2 = 2 x dx , ò + ò (sin x + cos x) dx
0 0 0
2 2 3p / 4
2 3
ò
I3 = 2 x dx, I 4 = 2 x dx ò " 0 < x < 1, x 2 > x3 =2 ò
0
(sin x + cos x) dx = 2 ( 2 + 1)
1 1
EBD_7504
S-FT-10 SOLUTIONS

75. (a) A divides C1C2 externally in the ratio 1 : 3.


z 2 n - 1 2i sin nq
\ = = i tan nq
1 3 z 2n + 1 2 cos nq
A(–3,0) ? Taking negative sign, we get
C1(–1,0) B C2(3,0)
z 2 n - 1 -2i sin nq
= = –i tan nq
\ coordinates of A are (–3, 0) z 2n + 1 2 cos nq
We have sin q = 1/2 \ q = 30° z 2n - 1
Þ = | ± i tan nq| = |tan nq|
Area = 3 × 3 tan 30° = 3 3 z 2n + 1
76. (c) Given circles are x2 + y2 + 4x – 6y + 9 = 0 81. (a) Two curves cuts at right angle if product of their slopes
and x2 + y2 – 5x + 4y – 2 = 0 is –1.
\ locus of centres is Two given curves are
9x – 10y + 11 = 0 x3 – 3xy2 + 2 = 0 ..... (i)
77. (b) If n is odd, the required sum is and 3x2y – y3 – 2 = 0 ...... (ii)
12 + 2.22 + 32 + 2.4 2 + ...... + 2.( n - 1) 2 + n 2 Differentiate equ. (i),
2 2
dy 3(x - y )
(n - 1)(n - 1 + 1) 2 Þ m1 = =
= + n2 [Q (n–1) is even dx 6xy
2
Differentiate equ. (ii),
\ using given formula for the sum of (n–1) terms.]
dy -2xy
2
æ n - 1 ö 2 n (n + 1) Þ m2 = =
=ç + 1÷ n = dx (x 2 - y2 )
è 2 ø 2
(x 2 - y 2 ) –2xy
78. (b) \ m1 ´ m 2 = ´
N
2
P(at , 2at)
2xy (x 2 - y 2 )
R(h,k) Þ m1 ´ m 2 = -1
O S(a,0)
82. (d)
q
T : ty = x + at2 ..... (1) 3q
Line perpendicular to (1) through (a, 0)
tx + y = ta ..... (2)
2
Equation of OP : y - x = 0 ...... (3) Let one angle be q then other = 3q
t
From equations (2) and (3) eliminating t we get locus Clearly q + 3q = 180 Þ q = 45o
2x2 + y2 – 2ax = 0 \ Angle between the diameters represented by
6R combined equation
79. (a) Box 5B ax 2 + 2 ( a + b ) xy + by 2 = 0 is 45o
4W
2
P(E) = P(R R B W or B B R W or W W R B) \ Using tan q = 2 h - ab
n (E) = 6C2 · 5C1 · 4C1 + 5C2 · 6C1 · 4C1 + 4C2 · 6C1 · a+b
5C
1 2 a 2 + b2 + ab
Þ1=
n (S) = 15C4 a+b

\ P (E) =
720 × 4!
=
48
15 ×14 ×13 × 12 91
(
Þ ( a + b) = 4 a 2 + b2 + ab
2
)
1 Þ 3a 2 + 3b 2 + 2 ab = 0
80. (a) z + = 2 cos q 83. (a)
z
Þ z2 – 2 cos qz + 1 = 0 p q p Ù q p Ú q ~ (p Ú q) (p Ù q) Ù ~ (p Ú q)
T T T T F F
2cos q ± 4 cos 2 q - 4
Þ z= T F F T F F
2
F T F T F F
= cos q + i sin q
Taking positive sign, F F F F T F

is a contradiction.
Full Test S-FT-11

æ 3q ö æ q ö P(B1 ).P(A / B1 )
2sin ç ÷ cos ç ÷ P (B1 / A) =
sin q+ sin 2q P(B1).P(A / B1 ) + P(B2 ).P(A / B2 )
= è 2 ø è 2 ø = tan æ 3q ö
84. (c) ç ÷
cos q+ cos2q æ 3q ö æ q ö è2ø 1 99
2cos ç ÷ cos ç ÷ ×
è 2 ø è2ø 100 100 99 1
= = = = 50%
1 99 99 1 2 × 99 2
2p × + ×
Hence period = 100 100 100 100
3
p
85. (c) Since, 1, w , w2 , ................. wn -1 are the n roots of 88. (a) Since, Ð APB = Ð AQB = so y = mx + 8 intersect the
2
unity
circle whose diameter is AB.
Consider x n = 1 Equation of circleis x2 + y2 = 16, CD < 4
\ ( x n - 1) = ( x - 1) ( x - w ) ( x - w 2 ) Q
......( x - w n -1 ) D
n
x -1
Þ ( x - w ) ( x - w 2 )......( x - w n -1 ) = P A
x -1
C
Taking lim on both side B
x ®1
n
lim ( x - w ) ( x - w 2 )......( x - w n -1 ) = lim x - 1
x ®1 x ®1 x - 1
8
Þ < 4 Þ 1 + m2 > 4
2
Þ (1 - w )(1 - w 2 )......(1 - w n -1) = n 1+ m
86. (b) The equation of the tangent at Þ m Î (-¥, - 3) È ( 3, ¥)
æ 16 ö If the line passing throw the point A (–4, 0), B (4, 0)
çç 4 cos q, sin q ÷÷ to the ellipse
11 p
è ø then Ð APB = Ð AQB = does not formed.
2
16x2 + 11y2 = 256 is
\ m¹ ±2
æ 16 ö 89. (d) (a) Tr (kA) = k (a11 + a22 + a33) = kTr(A)
16x (4 cos q) + 11y çç sin q ÷÷ = 256
è 11 ø (b) Tr (A + B) = a11 + b11 + a22 + b22 + a33 + b33 = Tr (A)
+ Tr (B)
or 4x cos q + 11y sin q = 16
This touches the circle (x – 1)2 + y2 = 42 if (c) Tr (I3) = 1 + 1 + 1 = 3
(d) Tr(A2 )
4 cos q - 16
=4 2 2
16 cos 2 q + 11sin 2 q = S a11 + S a12 ¹ (a11 + a 22 + a 33 ) 2
Þ (cos q – 4)2 p /2

= 16cos2q + 11sin2q 90. (a) an = ò (1 - sin t) n sin 2t dt


Þ 15cos2q + 11sin2q + 8cos q – 16 = 0 0
Let 1 – sin t = u Þ – cos t dt = du
Þ 4 cos2q + 8 cos q – 5 = 0
Þ (2 cosq – 1) (2 cos q + 5) = 0
1 æ1 1 ö
= 2ò u n (1 - u) du = 2 ç ò u n du - ò u n +1du ÷ = 2 æç 1 - 1 ö÷
è0 ø è n +1 n + 2ø
1 p 0 0
Þ cos q =
Þq= ±
2 3 an æ 1 1 ö
Hence, =2ç -
87. (c) A: blood result says positive about the disease n è n(n + 1) n(n + 2) ø÷
B1: Person suffers from the disease
1 n
an æ æ1 1 ö 1 æ1 1 öö
\ P(B1) =
100
lim
n ®¥
å n
= 2 ç åç -
è è n n + 1
÷ - åç -
ø 2 è n n
÷
+ 2 ø ÷ø
1
99 æ n
1 öö n æ 1
= 2 ç å æç -
B2 : person does not suffer \ P(B2) = 1 1 ö
ç è n n + 1 ÷ø ÷÷ å çè n n + 2 ÷ø
100 - -
è 1 ø 1
99 1
P(A/B1) = , P(A/B2) = éæ 1ö æ1 1ö æ 1 1ö ù 3 1
100 100 = 2(1) - êç 1 - ÷ + ç - ÷ + ç - ÷ + .....ú = 2 - =
ëè 3 ø è 2 4 ø è 3 5 ø û 2 2
EBD_7504
S-FT-12 SOLUTIONS

Speed Test-101

PHYSICS q
= 2 2
2p r Î0
1 k The direction of E is towards negative y-axis.
1. (c) Frequency of spring (f ) = = 1 Hz
2p m r q
\ E=- 2 2 ˆj
k 2p r Î0
2
Þ 4p =
m = 1 kg
m 3. (c) The magnetic field at a point on the axis of a circular loop
at a distance x from centre is,
If block of mass m = 1 kg is attached then, m 0i a 2
k = 4p2 B=
2( x 2 + a 2 )3 / 2
Now, identical springs are attached in parallel with mass
m = 8 kg. Hence, m i
B' = 0
keq = 2k 2a
1 k´2 1 B.( x 2 + a2 )3/ 2
F= = Hz \ B' =
2p m 2 a3
54(53 )
Put x = 4 & a = 3 Þ B ' = = 250 µT
k 3´ 3´ 3
8 kg
u 2 sin 2 q
4. (c) H= or H µ u 2
2. (c) Let us consider a differential element dl. charge on this 2g
element.
j DH Du
= 2. = 2(+ 2%) = 4% increased
H u
+
+

+ dl
+

1 é 1 1 ù
+ dq
+
5. (c) Wave number = RZ 2 ê - ú
+ dE cos q q
+ l êë n1 n2 úû
2
i
O 1
dE Þlµ
Z2
dE sin q
æ q ö q By question n = 1 and n 1 = 2
dq = ç ÷ dl = (rd q) (Qdl = rd q)
è pr ø pr Then, l1 = l2 = 4l3 = 9l4
æqö 6. (d) From conservation of mechanical energy
= ç ÷dq
è pø 1 1 æ v2 ö æ 3v2 ö
Electric field at O due to dq is mv 2 + I ç 2 ÷ = mg ç ÷
2 2 èR ø è 4g ø
1 dq 1 q
dE = . = . dq mR 2
4p Î0 r 2 4 p Î0 pr 2 after solving I = Which is for disc.
2
The component dE cos q will be counter balanced by 7. (c) mg – T = ma
another element on left portion. Hence resultant field T 360
at O is the resultant of the component dE sinq only. \ a= g- = 10 - = 4m / s 2
p
m 60
q
\ E = ò dE sin q = ò 2 2
sin qd q T
0 4p r Î0
q
= [ -cos q]p0 a
®

2 2 T
4p r Î0
q
= (+1 + 1)
4p2 r 2 Î0 mg
Full Test S-FT-13

1 T T
8. (a) X= 2
ò Pinst ´ dt ò sin wtdt
a w 2 - bw + c 0 (wBA) 2 0 1 ( wBA) 2
Pavg = = =
T R T
At resonance X becomes maximum and aw 2 - bw + c 2 R
ò dt ò dt
becomes minimum. 0 0
\ Differential above term and equating it to zero.
é pr 2 ù
b (w B pr 2 ) 2 êA = ú
\ 2aw - b = 0 Þ w = . \ Pavg =
2a 8R ëê 2 ûú
(200 - T1 ) (T - T )
9. (a) Q = KA = (2K)A 1 2 1 q
L L 15. (b) Electric Intensity = ´
4 p Î0 r 2
1
=(1.5K) A (T2-18) ×
L 1 1
or qmax = r2E = × 2.5 × 2.5 × 3 × 106
200 - T1= 2T1 – 2T2 = 1.5 T2- 27, 9 ´ 10 9
9 ´10 9
Solving T1 = 116ºC and T2= 74ºC –3
or qmax = 2 × 10 coulomb
10. (d) 4i 1 = 5i 2
16. (b) 20 cm
5
\ i1 = i2
4 u0 v0 ue
i1 = current through 4 W
i2 = current through Ammeter
5 ´ 4 20
Total R = =
9 9
10 ´ 9 9 5 9
I= = Þ i 2 + i 2 = Þ i 2 = 2A objective
20 2 4 2
11. (a) When positive terminal connected to A then diode Ve=25 cm Eye piece
2
D1 is forward biased, current, I = = 0.4 A Final image distance L = v0 + ue = 20
5
When positive terminal connected to B then diode 1 1 1
for objective = +
2 f0 v0 u 0
D2 is forward biased, current, I = = 0.2 A
10 1 1 1
12. (b) Let pole strength = m for eye piece =- +
fe 25 u e
l
\ M = m ´ l when l' = 1 1 1
2 or + =
ml M 5 25 u e
M' = m ´ l' = = = 40 units
2 2 25
13. (d) Mass per unit length of the wire = r \ue = cm \ v o = 20 - u e
6
Mass of L length, M = rL
and since the wire of length L is bent in a form of 25 95
v0 = 20 - =
L 6 6
circular loop therefore 2pR = L Þ R =
2p 1 1 95 89 95
3 \ = 1- =1- = - \u0 = -
2 u0 v0 6 95
Moment of inertia of loop about given axis = MR 89
2
3 æ Lö
2
3rL3 1 ì hc ü
= rL ç ÷ =
17. (a) V= í - fý
2 è 2p ø eîl þ
8p2
r r
14. (b) f = B. A ; f = BA cos wt Put l1 = 3 ´10 -7 m, V1 = 1.85 ,
df wBA l 2 = 4 ´ 10 -7 m, V2 = 0.82
e=- = wBA sin wt ; i = sin wt
dt R
in the above equation respectively
2
æ wBA ö
Pinst = i2 R = ç ´ R sin 2 wt hc
è R ÷ø f = 1.216 ´ 10 -19 J = Þ l 0 = 5451Å
l0
D
B
E
S-FT-14 SOLUTIONS

18. (b) Let V be volume of the load, and let its relative density 21. (c) Given: Amplitude of electric field,
be r then E0 = 4 V/m
Absolute permitivity,
FL VrgL e0 = 8.8 × 10–12 C2/N-m2
Y= = . ......(1)
Al a Al a Average energy density uE = ?
Applying formula,
when the load is immersed in liquid, the net weight =
weight - upthrust 1
Average energy density uE = e0 E 2
4
F' L (Vrg - V ´ l ´ g )L
\Y = = ......(2) 1
Al w Al w Þ uE = ´ 8.8 ´ 10 -12 ´ (4) 2
4
= 35.2 × 10–12 J/m3
r r -1
Equating (1) and (2), we get = 2 æ Df ö p
la lw 22. (c) 2I0 = 4I0 cos ç ÷ here, Df =
è 2 ø 2
la 2p l
which gives r = But, Df = Dx so, Dx =
la - l w l 4
dy l
Distance travelled on pitch scale = ....(i)
19. (b) Pitch = D 4
Number of rotation
lD
=b ....(ii)
2mm d
= = 0.5 mm Multiplying equation (i) and (ii) we get,
4
b
Least count = y=
4
Pitch 23. (d) Internal energy is a state function hence in a cyclic
Number of division on circular scale process change in internal energy is zero.
24. (d) From the law of conservation of momentum we know
0.5mm that,
= = 0.01 mm
50 m1u1 + m2u2 + .... = m1v1 + m2v2 + ....
20. (d) From the graph it is clear that the amplitude is 1 cm Given m1 = m, m2 = 2m and m3 = 3m
and the time period is 8 second. Therefore the and u1 = 3u, u2 = 2u and u3 = u
equation for the S.H.M. is ®
Let the velocity when they stick = v
æ 2p ö æ 2p ö Then, according to question,
x = a sin ç ÷ ´ t = 1sin ç ÷ t
èT ø è 8ø Y
p 2m, 2u
x = sin t
4
The velocity (v) of the particle at any instant of time ‘t’ is j sin 60°
j
dx d é æ p ö ù p æ pö 60°
v= = sin ç ÷ t = cos ç ÷ t m, 3u X
dt dt êë è 4 ø úû 4 è 4ø P 60° i i cos 60°
The acceleration of the particle is (–j) sin 60°
2
d2x æ pö æ pö –3m, u
= - ç ÷ sin ç ÷ t
dt 2 è 4ø è 4ø
()
m × 3u î + 2m × 2u
4
At t =
3
s we get
( -ˆi cos 60° - ˆjsin 60°) + 3 m × u
( -ˆi cos 60° + ˆjsin 60°) = (m + 2m + 3m) v
2 ®
d2x æ pö p 4 -p2 p
= - ç ÷ sin ´ = sin
dt 2 è 4ø 4 3 16 3
ˆ æ ö ˆ
Þ 3muiˆ - 4mu i - 4mu ç 3 ˆj÷ - 3mu i
- 3p 2 2 è 2 ø 2
= cm / s 2
32 æ 3 ˆö ®
+ 3mu çç j ÷÷ = 6m v
è 2 ø
Full Test S-FT-15

® 28. (d) Here, R = 4 kW = 4 × 103 W


3 3 ˆ
Þ muiˆ - muiˆ - muj = 6m v Vi = 60 V
2 2
Zener voltage Vz = 10 V
1 3 ˆ ®
RL = 2 kW = 2 × 103 W
Þ - muiˆ - muj = 6m v
2 2
VZ 10
® Load current, IL = = = 5 mA
Þ v=
u ˆ
12
(
- i - 3jˆ ) RL 2 ´ 103
25. (d) Gravitational force provides the necessary centripetal Vi - VZ
force. Current through R, I =
R
mv 2 GmM GM
\ = also g = 2 60 - 10 50
( R + x ) ( R + x )2 R = = = 12.5 mA
3
4 ´ 10 4 ´ 103
2 2
mv æ GM ö R n! Fom circuit diagram,
\ = mç
( R + x) è R 2 ÷ø ( R + x )2 r !( n - r ) ! I = IZ + IL
mv2 R2 Þ 12.5 = IZ + 5
\ = mg Þ IZ = 12.5 – 5 = 7.5 mA
( R + x) ( R + x) 2 29. (c) 9.25 days is equal to three half-lives for Tl-201. The
1/ 2
gR 2æ gR 2 ö fraction remaining is then :
\ v2 = Þ v=ç ÷ 1/2 × 1/2 × 1/2 = 1/8. Thus 1/8 of 80 mCi remains.
R+ x è R + xø
1 H
26. (b) According to Wien’s law l m µ and from the figure A 0.9 km B I
T
30. (d) ENGINE L
(lm)1 < (lm)3 < (lm)2 therefore T1 > T3 > T2.
C L
27. (c) Y
Let after 5 sec engine at point C
A(Ö2,Ö2) AB BC
® t= +
r1 330 330
0.9 ´ 1000 BC
5= +
O X 330 330
(0,0) ®
r2 B (2,0) \ BC = 750 m
The distance of point A ( )
2, 2 from the origin, Distance travelled by engine in 5 sec
= 900 m – 750 m = 150 m
ur Therefore velocity of engine
2 2
OA = | r1 | = ( 2) + ( 2) = 4 = 2 units. 150 m
= = 30 m/s .
The distance of point B(2, 0) from the origin, 5sec
uur 2 2 CHEMISTRY
OB = | r2 | = (2) + (0) = 2 units.
31. (d) When mixture of two liquids boil at a lower temperature
1 Q
Now, potential at A, VA = . than either of them, it shows positive deviation from
4p Î0 (OA) Raoult’s law.
1 Q 32. (c) The conformers are interconvertible.
Potential at B, VB = . 33. (c) The correct order of acidic strength of the given species
4p Î0 (OB)
is :
\ Potential difference between the points A and B is
given by HSO 3 F > H 3O + > HSO 4– > HCO 3–
(iv) (ii) (iii) (i)
1 Q 1 Q
VA – VB = . - . (i) < (iii) < (ii) < (iv)
4p Î0 OA 4p Î0 OB
It corresponds to choice (c) which is correct answer.
Q æ 1 1 ö Q æ 1 1ö
= çè - ÷ø = çè - ÷ø
4p Î0 OA OB 4p Î0 2 2
Q
= ´
4p Î0
EBD_7504
S-FT-16 SOLUTIONS

42. (a) AgX + 2 Na2S2O3 ® Na3[Ag(S2O3)2] + NaX


1
34. (a) H 2 (g) + O2 (g) ® H 2 O(g) ΔH = – 249 thiosulphate Sodium argento (soluble complex)
2
Q x + 2 (–2) = –3
Let the bond enthalpy of O - H is x. Then
DH = SB.E. of reactant - S B.E.of product x = +1

1 K f ´ w ´ 1000
–249 = 433 + ´ 492 - 2x 43. (a) Molar mass of solute =
2 DTf ´ W
Þ x = 464 kJ mol-1. Given : Kf = 1.86, w = 1.25 g, W = 20 g,
35. (b) KE = hv – hv0 DTf = 273 – 271.9 = 1.1K
hv1 – hv0 = 2(hv2 – hv0) Therefore, molar mass of solute
v0 = 2 v2 – v1
= 2 (2.0 × 1016) – (3.2 × 1016) 1.86 ´ 1.25 ´ 1000
= = 105.7
= 8 × 1015 s–1 = 8 × 1015 Hz 1.1 ´ 20
+4
2 ´ 63.5 44. (c) H 2SO 3 ( aq ) + Sn 4+ ( aq ) + H 2 O ( l ) ¾¾
®
36. (c) Percentage by mass of copper in malachite = +6
221
Sn 2+ ( aq ) + HSO4- ( aq ) + 3H +
= 57.5%
Hence H2 SO 3 is the reducing agent because it
37. (c) Solar energy is not responsible for green house effect
undergoes oxidation.
instead it is a source of energy for the plants and
animals. 45. (a) 2C2 H5 OH ∗ 2Na ¾¾

38. (c) CH3OCH2Cl is a 1° halide but still hydrolysis takes Ethanol
(A) 2C2 H5ONa ∗ H 2 ←
+ Sod. ethoxide
place through SN1 mechanism because CH 3O - C H 2 (B)
conc. H2SO4
is stabilized by resonance 2C2 H5OH ¾¾¾¾¾¾ ® C2 H 5 OC2 H5
- H 2O
+ + Diethyl ether
C H3O CH 2Cl « CH 3– O – CH2 « CH3– O = CH2 Thus, (A) is ethanol and (B) is sodium ethoxide.
39. (b) It is an example of Diel’s-Alder reaction in which a 46. (b) Claisen self condensation is given by esters which
dienophile (maleic anhydride, here) reacts with a contain at least one hydrogen on an alpha carbon atom
conjugated diene to form cyclic adduct. Here although (e.g., CH3COOC2H5). Compound, C6H5COOC2H5 will
structures (i), (iii) and (iv) have conjugated system of not give this reaction. All remaining compounds will
double bonds, hence theoretically all the three can give this reaction.
undergo Diel’s-Alder reaction, but structure (iii) does 47. (b) The step involved is a precipitation step. Increasing
not undergo this reaction because this will lead to larger the Cl– concentration will reduce the concentration of
ring (9-membered) which is unstable. Ag+ in solution. Remember Ksp = [Ag+ ] [Cl–] and the
source of the Cl – is irrelevant. Thus increased
CH — CO
O concentration of Cl – must result in decreased
+
CH — CO concentration of Ag+ in order to maintain the solubility
product constant. Addition of Ag 2SO4 would probably
6 be counterproductive, since additional Ag+ is being
5 7 added, and probably all will not be recovered.
48. (b) 2F2 + 4KOH ® 4KF + O 2 + 2H 2 O for 1 mole of F2 the
8
4
3 9
molar ratio.
2
CH — CH
1 F2 KOH KF O2 H2O
1
CO CO 1 2 2 1
2
O
HOCl
40. (b) Ag2S + 2NaCN Na2S + 2AgCN 49. (d) CH 2 < CH 2 ¾¾¾↑ ClCH 2 CH 2 OH
AgCN + NaCN ¾
¾® Na[ Ag ( CN ) 2 ] aq. NaHCO
¾¾ ¾¾ ¾
3 ↑ HOCH CH OH
2 2
1 50. (b) All alkali metals dissolve in liquid ammonia giving deep
41. (a) M.P. µ (HCl having more covalent character)
f blue solution.
M.P. µ Lattice energy (Ionic compd.)
LiF is ionic having highest Lattice energy.
Full Test S-FT-17

51. (c) N2Cl 4 has two pyramidal nitrogen atoms with sp3 59. (b) Both egg yolk and mustard contain large quantity of
hybridisation in each. This yields a non-planar sulphur (as its compounds) which react with Ag.
molecule. PCl5 involves sp3d hybridisation. SbF6– and 2Ag + S ® Ag2S (Black colour)
SiF62– both involve sp3d 2 hybridisation. 60. (c) In (c), sulphate ion is present outside the coordination
52. (a) According to van der Waal's equation for one mole of sphere so it can form white ppt of BaSO4 with BaCl2
gas (aq).
æ a ö
çççP ∗ 2 ÷÷÷ (V , b) < RT MATHEMATICS
è V ø
61. (a) The probability of A winning in a game
a
at very high pressure P == 6 1
V2 = P (A) = =
12 2
a The probability of B winning in a game = P (B)
So, is negligible.
V2 4 1
= =
P (V – b) = RT 12 3
PV – Pb = RT \ Reqd. probability = P (A Ç B Ç A) + P (B Ç A Ç B)
on dividing RT on both sides. = P (A).P(B).P(A) + P (B).P(A).P(B)
Pb 1 1 1 1 1 1 5
[ Z < 1∗ compressibility factor.. = × × + × × =
RT 2 3 2 3 2 3 36
53. (d) Since in NaCl type of structure 4 formula units form a 62. (b) Period of sinx = 2π
cell.
Þ period of sin 3 x = 2π
58.5 g of NaCl = 6.023 × 1023 atoms
6.023 ´ 10 23 period of sin 3 x = π
1 g of NaCl = atoms
58.5
4 atoms constitute 1 unit cell 3 x
Þ period of sin = 2π
2
6.023 ´ 1023
\ atoms constitute period of cos5 x = 2π
58.5
6.023 ´ 1023 Þ period of cos5 x = π
=
58.5 ´ 4 x
5
= 2.57 × 1021 unit cells. Þ period of cos = 5π
o 5
54. (d) Given : E Mg 2+ / Mg = -2.36V Thus required period
Mg 2+ (aq) + 2e- ® Mg(s) = LCM of 2π & 5π = 10π
63. (c) We have a + b = – 3 and ab = a/2
0.059 [Mg 2+ ]
E 2+ = Eo 2+ + log a b a2 + b 2
Mg / Mg Mg / Mg 2 [Mg] Now + <2 Þ <2
b a ab
Given [Mg2+] = 0.01 M; [Mg] = 1
0.059 (a + b)2 – 2ab 9–a 9–a
Hence, E Mg 2∗/Mg < ,2.36 ∗ log(0.01) Þ <2 Þ <2 Þ <1
2 ab a/2 a
< ,2.42V
9–a 9 – 2a 2a – 9
55. (c) For electrophilic substitution reaction, the order of Þ –1<0 Þ <0 Þ >0
reactivity among the given compounds is as follows : a a a
C6H5OCH3 > benzene > C6H5NO2 9
Þ a < 0 or a >
56. (b) KI ∗ AgNO 3 (slight excess) ¾ ¾ ↑ 2
AgI ∗ KNO3 64. (a) We have,
∗ ,; 4 cos x (2 - 3 sin 2 x ) + (cos 2x + 1) = 0
AgNO3 ¾¾ ↑ Ag ∗ NO3
AgI(s) ∗ Ag ∗ ¾¾ ↑ [AgI]Ag ∗ Þ 4 cos x (3 cos 2 x - 1) + 2 cos 2 x = 0
57. (b) Saffron is an artificial edible colour.
58. (c) Hydroxylamine and hydrazine, both do not have
Þ 2 cos x (6 cos 2 x + cos x - 2) = 0
carbon, hence NaCN will not be formed in Lassaigne’s Þ 2 cos x (3 cos x + 2)(2 cos x - 1) = 0
extract leading to negative test for nitrogen.
EBD_7504
S-FT-18 SOLUTIONS

Þ either cos x = 0 which gives x = p / 2 or cos x = -2 / 3 x å n 2 - n < {[12 x] + [22 x] + [32 x] + …


Which gives no value of x for which 0 £ x £ p / 2
+ [ n2 x]} £ x å n 2
or cos x = 1 / 2 , which gives x = p / 3
Dividing each term by n3,
So, the required difference = p / 2 - p / 3 = p / 6
æ 1 öæ 1ö
y x ç1 + ÷ç 2 + ÷
dt è n øè nø 1
65. (a) x = ò we get ,
6
-
n2
0 1+ t2
1 dy æ 1 öæ 1ö
Þ 1= . xç1 + ÷ç 2 + ÷
1+ y 2 dx [12 x ] + [2 2 x] + ..... + [ n 2 x] è n øè nø
< £
é y ( x)
dI(x) n3 6
êQ If I( x) = ò f (t ) dt , then = f {y ( x)}. Let n ® ¥, we get,
ê dx
ë f ( x )
x [12 x] + [2 2 x] + ....+ [n 2 x] x
(2) - 0 < Lt £

{ d
dx } {
y ( x ) - f {f( x)} .
d
dx } ù
f( x ) ú
û
6 n®¥ n3 3

[12 x ] + [2 2 x ] + ..... + [n 2 x ] x
Þ lim =
dy n ®¥ 3 3
= 1 + y2 n
dx 69. (d) m
(1 - y ) (1 + y ) n

2
d y 1 dy y = [1 - m C1 y + m C2 y 2 - ......] [1 + n C1 y + n C2 y 2 + .....]
Þ = = .2y .. 1+ y2 = y
2 dx
dx 2 1+ y 2
1+ y 2
= 1 + (n - m) y + ìí m(m - 1) + n(n - 1) - mn üý y 2 + .....
î 2 2 þ
66. (b) | a1 x + a2x + … + anx | £ | a1 | | x |+| a2 | | x | 2
2 n
By comparing coefficients with the given expression, we
+ | a3 | | x | 3 + ……. + | an | | x | n get
£ 2 [| x | + | x |2 + ….. + | x |n ] [Q a n < 2] m2 + n 2 - m - n - 2mn
\ a1 = n - m = 10 and a2 = = 10
2| x| 2| x| 2 1 2
= [1- | x |n ] < <
1- | x | 1- | x | 3 1- | x | So, n – m = 10 and (m - n) 2 - (m + n) = 20
1 3 Þ m + n = 80 \ m = 35, n = 45
Therefore, | a1x + ….. + anxn | < 2. . =1
3 2 s n
\ | a 1x + ….. + anxn | < 1 for all n . 70. (a) å å n
Cs s Cr
–1 < a1x + ….. + anxn < 1 for all n r= 0 s =1
r£s
Þ 1 + a1x + a2x2 + … + anxn > 0 for all n
Þ 1 + a1x + .. + a nxn ¹ 0 for all n n

-
p
p
= å nC s
s =1
( sC 0
+ s C1 + s C2 + ... + s Cs )
67. (a) Since, e 2 <q<
2 n n
å nCs 2s = å n Cs 2s – n C0 2 0 =(1 + 2)
p = n – 1 = 3n – 1
- p
\ log e < log q < log
2 s =1 s=0
2
71. (c) Number of ways of making 3 sets of 10 balls having 2,
p p p
i.e., – < log q < log <1 < 10!
2 2 2 3 and 5 balls and further they can be given to
2!.3!.5!
é p p pù
êQ 2 < e \ log 2 < log e = 1 and 1 < 2 ú 3 persons one each in 3! ways.
ë û
10!
p p Total number of ways = ´ 3!
\ – < log q < \ cos (log q) > 0 2!3!5!
2 2
But 0 < cos q < 1, \ log ( cos q) < log 1 = 0 æ dy ö
i.e., log (cos q) < 0. 72. (d) Given, sin 2 x çè - tan x ÷ø - y = 0
dx
Hence, cos (log q) > log ( cos q)
68. (b) Since 12 x – 1 < [12 x] £ 12 . x dy y
or, = + tan x
22 x – 1 < [22 x] £ 22 x dx sin 2 x
32 x – 1 < [32 x] £ 32 x ……………… dy
n2 x – 1 < [n 2 x] £ n2 x or, - y cosec2 x = tan x ...(1)
Adding all terms, we get, dx

- cosec2 x
Full Test S-FT-19

or, I.F = e
1
- log|tan x|
2 = e
log ( tan x ) -1 1 m n
Þ (l + m + n) 1 n l =0
1 1 l m
= = cot x
tan x
Now, general solution of eq. (1) is written as Þ (l + m + n )( mn + nl + lm - l 2 - m 2 - n 2 ) = 0
Þ l + m + n = 0; l2 + m 2 + n 2 - lm - mn - nl = 0
y (I. F.) = ò Q(I.F.) dx + c \ l2 + m2 + n2 = lm +mn + nl
3 4 2 3 4 2
\ y cot x = ò tan x . cot x dx + c
76. (c) 5 8 2 = 2 4 0
\ y cot x = ò 1.dx + c x y 2 x -5 y -8 0

\ y cot x = x + c (R 2 ® R 2 - R1 , R 3 ® R 3 - R 2 )
73. (b) From cosine and sine formula, we have = 2 (2y – 16 – 4x + 20 ) = 2 (2y – 4x + 4 )
\ given determinant = 0 Þ 2y – 4x + 4 = 0
b2 + c2 - a 2 sin A sin B sin C Þ 2x – y – 2 = 0 which represents st. line.
cos A = and = = = k. 77. (b) Centre of the given circle is (1,2) and its radius =
2bc a b c
sinB 1 + 4 + 20 = 5 . Since the radii of the two circles are
Given, in any DABC, cos A = equal, therefore these will touch externally and the point
2sin C
of contact will lie mid - way between the two centres . If
From the above given formula, we have
(h,k) is the centre of the circle, then
sin B = bk, sin C = ck
h +1 k+2
bk b = 5, =5 \ h = 9, k = 8
\ cos A = = 2 2
2 ck 2 c
Put the value of cos A in the formula, which gives \ its equation is ( x –9)2 + ( y – 8)2 = 25
i.e., x2 + y2 – 18x – 16y + 120 = 0
b b2 + c 2 - a 2
= Þ c2 = a 2 Þ c = a x2 y 2
2c 2bc 78. (a) Ellipse is + =1
16 3
74. (d) Operate C1+C2 +C3 ,
we get, Now, equation of normal at (2, 3/2) is
16 x 3 y
x + 1 + w + w2 w w2 - = 16 - 3
2 3/ 2
x + 1 + w + w2 x + w2 1 =0 13
Þ 8x – 2y = 13 Þ y = 4 x -
2
x + 1 + w + w2 1 x+w
13
Let y = 4 x - touches a parabola
2
x w w2 y2 = 4ax.
Þ x x + w2 1 =0 We know, a straight line y = mx + c touches a parabola
y2 = 4ax if a – mc = 0
x 1 x+w
æ 13ö
\ a - ( 4) ç - ÷ = 0 Þ a = – 26
[Q 1 + w + w2 = 0] è 2ø
Hence, required equation of parabola is
1 w w2 y2 = 4 (– 26)x = – 104 x
Þ x 1 x+w
2
1 =0Þx=0 79. (a) I study or I fail = p Ú q
1 1 x+w Now, ~ (p Ú q) º~ p Ù (~ q)
Hence, negation of 'I study or I fail' is I do not study
and I do not fail.
l m n
æ 3p ö
75. (d) Lines are concurrent if m n l =0 80. (b) sin 4 ç - a ÷ = cos 4 a,
è 2 ø
n l m
sin 4 ( 3p + a) = sin4 a,
l+ m+n m n æp ö
Þ l+ m+n n l = 0 (C1 ® C1 + C2 + C3 ) sin 6 ç + a ÷ = cos6 a, sin 6 (5p – a) = sin 6 a
è2 ø
l+ m+n l m \ given quantity
6a+ sin6 a]
3 sin2 acos2a] = 1
EBD_7504
S-FT-20 SOLUTIONS

æ pö p 1
81. (b) sin x – cos x = 1 Þ sin ç x - ÷ = sin Þx = x+ . Let î be unit vector along x-axis and
è 4ø 4 x
p p p p let ĵ be a unit vector along y-axis. Then
– = np + (–1)n Þ x = np + (–1)n + uuur uuur
4 4 4 4 OP = x1ˆi + y1ˆj, OQ = x 2ˆi + y 2ˆj
r r r r r r
82. (c) [A - B B - C C - A] uuur
r r r r r r Since, OP × ˆi = 1 and OQ.î = -1
= [(A - B) ´ (B - C)]. (C - A) \ x1 = 1 and x2 = –1 Þ y1 = 2 and y2 = –2 ;
r r r r r r r r r
= [A ´ B - A ´ C - 0 + B ´ C].(C - A) \ OP = î + 2 ĵ; OQ = - î - 2 ĵ
r r r r r r
= [A ´ B].C - [B ´ C].A \ 2OP + 3OQ = 2i + 4 j - 3i - 6 j = -i - 2 j ;
rrr r rr
= [A BC] - [A BC] = 0 (Q [ABC] = [BCA]) Hence, 2OP + 3OQ = 1 + 4 = 5
r r
83. (d) Vector A = ˆi + ˆj + k,
ˆ B = 4iˆ + 3jˆ + 4kˆ
88. (b) y = 4 meets the parabola y2 = x at A, if 16 = x
r \ A is (16, 4)
And C = ˆi + aˆj + bkˆ are linearly independent, if
Reqd. area = Area of rect. OMAC – Area OMA
1 1 1
4 3 4 =0
1 a b
Operate C3®C3 – C1 ; C2®C2 – C1
1 0 0
we get, 4 -1 0 =0 16
16 3/ 2
1 a -1 b -1 = 4 ´ 16 – ò x dx = 64 -
x
.
3
Þ–(b–1)=0Þb=1 0
2
r 0
Again, | C | = 1 + a 2 + b 2 = 3
2 128 64
Þ 1 + a2 + b2 = 3 Þ 1 + a2 + 1 = 3 = 64 - (4) 3 = 64 - = sq.units
3 3 3
\ a2 = 1 Þ a = ± 1
Hence, a = ± 1 , b = 1 3p / 2 5p / 6 p
89. (d) ò [ 2 sin x]dx = ò [ 2 sin x]dx + ò [ 2 sin x ]dx
84. (c) Lt [f (x ) + g ( x ) + h (x )] p/2 p/2 5p / 6
x ®3
7p / 6 3p / 2
é 2 x -3 2(2 x + 1) ù + ò [2 sin x ]dx + ò [ 2 sin x ]dx
= Lt ê + - ú
x ®3 ë x - 3 x + 4 x 2 + x - 12 û p 7p / 6

é 2 x + 8 + x 2 - 6x + 9 - 4 x - 2 ù 5p / 6 p 7p /6 3p / 2
= Lt ê
x ®3ê
ë x 2 + x - 12
ú
úû
= ò 1 dx + ò 0 dx + ò (-1) dx + ò (-2) dx
p/2 5p / 6 p 7p / 6
x 2 - 8x + 15 (x - 3)( x - 5) æ 5p p ö æ 7 p ö æ 7 p 3p ö p
= Lt = Lt =ç - + 0 + çp - ÷ + 2ç - ÷=-
x ®3 2
x + x - 12 x ®3 ( x + 4)( x - 3) è 6 2 ÷ø è 6 ø è 6 2 ø 2
90. (a) L f ¢(1)
x -5 3-5 2
= Lt = =- f ( x ) - f (1) ax 2 + b - a - b
x ®3 x + 4 3+ 4 7 = Lt = Lt
85. (d) We know that every quadratic equation has exactly x ®1 x -1 x ®1 x -1
two roots which are either, both real or both are imagi- a ( x 2 - 1)
= Lt = Lt a ( x + 1) = 2a
nary. So, any quadratic equation has neither exactly x ®1 x - 1 x ®1
one real root nor booth roots are always real. Rf ¢(1)
86. (b) We have a theorem that if a square matrix A satisfies f ( x) - f (1) bx 2 + ax + c - a - b
= Lt = Lt
the equation; a0 + a1 x + a2 x 2 + ....... + a n x n = 0, x ®1 x -1 x ®1 x -1
c
where a0 ¹ 0 then A is invertible. = Lt [ b( x + 1) + a ] +
x ®1 x -1
Si nce A, B and C are n × n matrices and
A satisfies the equation x3 + 2x 2 + 3x + 5 = 0 as c
= 2b + a + = 2b + a if c = 0
x -1
A 3 + 2 A 2 + 3A + \2a=2b+a
87. (d) Let P ( x1, y1 ), Q ( holds if a = 2 b, c = 0.

You might also like